Anda di halaman 1dari 1070

2017-18 100 &

op kers
Class 12 T
By E ran culty
-JE Fa r
IIT enior emie .
S fP r es
o titut
Ins

CHEMISTRY
FOR JEE MAIN & ADVANCED
SECOND
EDITION

Exhaustive Theory
(Now Revised)

Formula Sheet
9000+ Problems
based on latest JEE pattern

2500 + 1000 (New) Problems


of previous 35 years of
AIEEE (JEE Main) and IIT-JEE (JEE Adv)

5000+Illustrations and Solved Examples


Detailed Solutions
of all problems available

Plancess Concepts
Topic Covered Tips & Tricks, Facts, Notes, Misconceptions,
Key Take Aways, Problem Solving Tactics
Electrochemistry
PlancEssential
Questions recommended for revision
17. ELECTROCHEMISTRY

1. INTRODUCTION
Electrochemistry is a branch of chemistry which deals with inter-conversion of chemical energies and electrical
energy. We’ll discuss electrolytic reactions (reactions that occur when electricity passes through solutions) as well
as electromagnetic reaction (reactions that produce electric energy). Some examples of electrolytic reactions are
electrolysis, electroplating, electro refining of metals, etc. Some examples of electro genetic reactions are reactions
occurring in cells or batteries.

2. TERMINOLOGIES IN ELECTROCHEMISTY
Some important terms used in Electrochemistry are as follows:
(a) Electrical Conductors: Materials that allow flow of electrons are known as conductors. There are broadly two
types of conductors-Electronic/Metallic and Electrolytic/Solution.

Table 17.1: Difference between Electronic Conductor and Electrolytic Conductors

Electronic Conductors or Metallic Conductors Electrolytic Conductors or Solution Conductors

1. Passage of current by movement of electrons in the Passage of current by ions in molten state or in aqueous solution
metallic lattice, e.g., Cu, Ag, etc. of electrolytes, e.g., NaCl (aq) or NaCl (fused).
2. Passage of current brings in only physical changes. Passage of current brings in physical as well as chemical changes.
3. It generally shows no transfer of matter. It involves transfer of matter in the form of ions.
4. It generally shows an increase in resistance during the It generally shows a decrease in resistance due to decrease in
passage of current due to increase in temperature. viscosity of the medium and degree of hydration of ions with
Thermal motion of metal ions hindering the flow of increase in temperature.
electrons increases with increase in temperature.
5. The conducting power of metals is usually high. The conducting power of electrolytic conductors is relatively low.

(b) Insulators: Those materials which don’t allow the passage of electrons are known as Insulators. For e.g. wood,
wool, plastic, silk, etc.
(c) Electrolytes: The substance that in solution or in the molten state, conducts electric current and is
simultaneously decomposed is called an electrolyte.The extent or degree of dissociation of different electrolytes
in solution is different. Electrolytes can be broadly categorized into two: Strong and Weak Electrolytes.
(d) Strong Electrolytes: Substances which are largely dissociated and form a highly conducting liquid in water
are strong electrolytes, e.g., All salts (except CdBr2, HgCl2), mineral acids like HCl, H2SO4, HNO3, etc. and
bases like NaOH, KOH, etc. are strong electrolytes. The strong electrolytes are almost 100% ionized at
normal dilution.
1 7 . 2 | Electrochemistr y

(e) Weak Electrolytes: Substances which dissociate only to a small extent in aqueous solution forming low
conducting liquid are weak electrolytes, e.g., All organic acids (except sulphonic acids), inorganic acids
like HCN, H3BO3, etc. and bases like NH3, amines, etc. are weak electrolytes.
(f) Electrodes: In order to pass the current through an electrolytic conductor, two rods or plates are always
needed which are connected with the terminals of a battery. These rods/plates are called Electrodes. The
electrode where oxidation reaction takes place is anode and electrode where reduction takes place is cathode.

3. ELECTROLYSIS
The phenomenon in which passage of current through an electrolyte (molten or solution) brings in chemical
changes involving electronation (reduction) as well as de-electronation (oxidation) of ions is known as electrolysis.

3.1 Preferential Discharge Theory


If an electrolytic solution consists of more than two ions and the electrolysis is done, it is observed that all the
ions are not discharged from the electrodes simultaneously but certain ions are liberated from the electrodes in
preference to others. This is explained by preferential discharge theory. It states that if more than one type of
ions are attracted towards a particular electrode, then the one discharged is the ion which requires least energy.
The potential at which the ion is discharged or deposited on the appropriate electrode is termed the discharge
potential or deposition potential. The values of discharge potential are different for different ions.

Table 17.2: Examples of preferential discharge theory

Electrolyte Electrode Cathodic reaction Anodic reaction


Aqueous acidified Pt 2Cl− → Cl2 + 2e−
CuCl2 solution
Cu2+ + 2e− → Cu
Molten PbBr2 Pt 2Br − → Br2 + 2e−
Pb2+ + 2e− → Pb
Sodium chloride Hg 2Cl− → Cl2 + 2e−
solution
2Na+ + 2e− → 2Na
Silver nitrate solution Pt
Ag+ + e− → Ag 1
2OH− → O2 + H2O + 2e−
2
Sodium nitrate Pt
2H+ + 2e− → H2 1
solution 2OH− → O2 + H2O + 2e−
2

Illustration 1: Find the charge in coulomb on 1 g-ion of N3− .  (JEE MAIN)

Sol: First determine charge on one ions of this can be calculated as product of number of electron and charge of
electron. According to Avogadro’s law one g of ion contains 6.02 × 1023 ions. So, charge on one g-ion of N3− can be
calculated by multiplying charge.
Charge on one ions of N3− into Avogadro number.
3− −19
Charge on one ions of N =3 × 1.6 × 10 coulomb One g-ion = 6.02 × 1023 ions

Thus, charge on one g-ion of N3− =3 × 1.6 × 10 −19 × 6.02 × 1023 = 2.89 × 105 coulomb

Illustration 2: Explain the reaction: (a) 2KI + Cl2 → 2KCl + I2 , (b) 2KClO3 + I2 → 2KIO3 + Cl2

Sol: Compound which undergoes oxidation acts as a reducing agent and compound which undergoes reduction
acts as an oxidizing agent.
Chem i str y | 17.3

(a) Cl2 acts as oxidizing agent: 2e− + Cl2 → 2Cl− ; 2I − → I2 + 2e−
(b) I2 acts as reducing agent: 2Cl5+ + 10e− → Cl ; I02 → 2I5+ + 10e−
2

PLANCESS CONCEPTS

Misconception: Electrolysis does not mean breaking up of an ionic compound into ions. An ionic
compound even on dissolution in water furnishes ions.
Note: During electrolysis, oxidation-reduction occurs simultaneously. Oxidation occurs at anode whereas
reduction occurs at cathode.
Nikhil Khandelwal (JEE 2009 AIR 94)

3.2 Faraday’s Law of Electrolysis


The relationship between the quantity of electric charge passed through an electrolyte and the amount of the
substance deposited at the electrodes was presented as the ‘laws of electrolysis’ by Faraday in 1834.

3.2.1 Faraday’s First Law


When an electric current is passed through an electrolyte, the amount of substance deposited is proportional to
the quantity of electric charge passed through the electrolyte.
If W be the mass of the substance deposited by passing Q coulomb of charge, then according to the law, we have
the relation: W ∝ Q
Q = current in amperes × time in seconds = I × t
So, W ∝ I × t or W = Z×I×t
Where Z is a constant, known as electrochemical equivalent and is characteristic of the substance deposited.
When a current of one ampere is passed for one second, i.e., one coulomb (Q = 1), then W = Z.
Definition of electrochemical equivalent: Mass of the substance deposited by one coulomb of charge or one
ampere current for one second.

3.2.2 Faraday’s Second Law


When the same quantity of charge is passed through different electrolytes, then the masses of different substances
deposited at the respective electrodes will be in the ratio of their equivalent masses. Again according to first law,
W= Z × Q
When, Q = 96500 coulomb, W becomes gram equivalent mass (E).
E Z1 E1
Thus, E= Z × 96500 or Z = ; =
96500 Z 2 E2

3.2.3 Faraday’s Law for Gaseous Electrolytic Product


ItVe
For the gases, we use V =
96500
Where, V = Volume of gas evolved at STP at an electrode
Ve = Equivalent volume = Volume of gas evolved at an electrode at STP by 1 faraday charge
1 7 . 4 | Electrochemistr y

Example: A 40.0 amp current flowed through molten iron (III) chloride for 10.0 hours (36,000 s). Determine the
mass of iron and the volume of chlorine gas (measured at 25ºC and 1 atm) that is produced during this time.

Sol:
 1. Write the half-reaction that take place at the anode and at the cathode.
Anode (oxidation): 2Cl− → Cl2 (g) + 2e− Cathode (reduction): Fe3+ + 3e− → Fe(s)
 2. Calculate the number of moles of electrons.

1.44 × 106 C
40.0amps × 36,000s =
1F 1mole e−
1.44 × 106 C × 14.9F ; 14.9F ×
= 14.9mole e−
=
96, 485C 1F

 3. Calculate the moles of iron and of chlorine produced using the number of moles of electrons calculated and
the stoichiometry from the balanced half-reactions. According to the equations, three moles of electrons
produce one mole of iron and 2 moles of electrons produce 1 mole of chlorine gas.

1mole Fe 1mole Cl2


14.9mole
= e− × e− ×
4.97mole Fe; 14.9mole= 7.45mole Cl2

3mole e 2mole e−
 4. Calculate the mass of iron using the molar mass and calculate the volume of chlorine gas using the ideal gas
law (PV = nRT).

55.847 gFe (7.45moleCl2 )(0.0821atm L / mole K)(298K)


=
4.97mole Fe × 278gFe; = 182L Cl2
1mole Fe 1atm

Calculating the Time required


To determine the quantity of time required to produce a known quantity of a substance given the amount of
current that flowed:
(i) Find the quantity of substance produced/consumed in moles.
(ii) Write the balanced half-reaction involved.
(iii) Calculate the number of moles of electrons required.
(iv) Convert the moles of electrons into coulombs.
(v) Calculate the time required.

Example: How long must a 20.0 amp current flow through a solution of ZnSO 4 in order to produce 25.00 g of Zn
metal?
Sol:
(i) Convert the mass of Zn produced into moles using the molar mass of Zn.
1mole Zn
25.00gZn × 0.3823mole Zn
=
65.39gZn

(ii) Write the half-reaction for the production of Zn at the cathode. Zn2+ (aq) + 2e− → Zn(s)
(iii) Calculate the moles of e- required to produce the moles of Zn using the stoichiometry of the balanced half-
reaction. According to the equation, 2 moles of electrons will produce one mole of zinc.
2mole e−
0.3823mole Zn × 0.7646mole e−
=
1mole Zn
Chem i str y | 17.5

(iv) Convert the moles of electrons into coulombs of charge using Faraday’s constant.
1F 96, 485C
0.76mole e− × = 0.7646F; 0.7646F × = 73,770C
1molee − 1F

(v) Calculate the time using the current and the coulombs of charge.


20.0amps
= × t 73,770C;
= t 3,688s or 1.03h

Calculating the Current required


To determine the amount of current necessary to produce a known quantity of substance in a given amount of
time:
(i) Find the quantity of substance produced/or consumed in moles.
(ii) Write the equation for the half-reaction taking place.
(iii) Calculate the number of moles of electrons required.
(iv) Convert the moles of electrons into coulombs of charge.
(v) Calculate the current required.
Example: What amount of current is required to produce 400.0 L of hydrogen gas, measured at STP, from the
electrolysis of water in 1 hour (3600 s)?

Sol:
(i) Calculate the number of moles of H2. (Remember, at STP, 1 mole of any gas occupies 22.4 L)
1mole H2
400.0L H2 × = 17.9mole H2
22.4L H2

(ii) Write the equation for the half-reaction that takes place.
Hydrogen is produced during the reduction of water at the cathode. The equation for this half-reaction is:

4e− + 4H2O(I) → 2H2 (g) + 4OH− (aq)


(iii) Calculate the number of moles of electrons. According to the stoichiometry of the equation, 4 mole of e− are
required to produce 2 moles of hydrogen gas, or 2 moles of e− s for every one mole of hydrogen gas.
2mole e−
17.9mole H2 × 35.8mole e−
=
1mole H2

(iv) Convert the moles of electrons into coulombs of charge.


1F 96, 485C
35.8mole e− × 35.8F; 35.8F ×
= 3.45 106 C

1mole e − 1F

(v) Calculate the current required. I × 3600s =3.45 × 106 C; I =958C / s =958amps

PLANCESS CONCEPTS

As one faraday (96500 coulombs) deposits one gram equivalent of the substance, hence electrochemical
equivalent can be calculated from the equivalent weight,
Eq. wt. of the substance
i.e., Z =
96500
Note: Knowing the weight of the substance deposited (W gram) on passing a definite quantity of electricity
W
(Q coulombs), the equivalent weight of the substance can be calculated, i.e., Eq. wt.
= × 96500
Q
1 7 . 6 | Electrochemistr y

PLANCESS CONCEPTS

Tip: The quantity of electricity actually passed is calculated from the current and time as follows:
Quantity of electricity in columbs = Current amperes × time in seconds
Thus, knowing the quantity of electricity passed, the amount of substance deposited can be calculated.
Faraday’s first law and second law can be combined to give a mathematical relation as follows:-
E Q Q M C×t M
W = ZQ = ×Q = ×E = × = ×
F F F z F z
z = Electrochemical equivalent; Q = Quantity of electricity passed, E = Eq. wt. of the metal,
F = 1 Faraday, M = Atomic mass of the metal; z = Valency of the metal; C = Current passed,
t = Time for which current is passed.
Saurabh Gupta (JEE 2010 AIR 443)

Illustration 3: Electric current of 100 ampere is passed through a molten liquid of sodium chloride for 5 hours.
Calculate the volume of chlorine gas liberated at the electrode at NTP.  (JEE MAIN)

Sol: Here current and time is given so from this first calculate quantity of electricity passed (charge) and from this
calculate the amount of chlorine liberated. Volume of Cl2 liberated at NTP can be determined by multiplying the
amount of chlorine liberated by 22.4 L

The reaction taking place at anode is: 2Cl− → Cl2 + 2e− Q = I × t = 100 × 5 × 60 × 60 coulomb
71.0 g 71.0 g 2×96500 coulomb
1mole

The amount of chlorine liberated by passing 100 × 5 × 60 × 60 coulomb of electric charge


1
= × 100 × 5 × =
60 × 60 9.3264 mole Volume of Cl2 liberated at NTP= 9.3264 × 22.4= 201L
2 × 96500

Illustration 4: How much electric charge is required to oxidize (a) 1 mole of H2O toO2 and
(b) 1 mole of FeO to Fe2O3 ? (JEE MAIN)

Sol: Charge = No of electrons involved in the reaction x faradays constant


So first find out the no of electron reaction involved in the reaction by writing the chemical reaction, balancing it
and then calculate the charge.
1
(a) The oxidation reaction is: H2O → O2 + 2H+ + 2e− ; Q= 2 × F =
2 × 96500 =
193000 coulomb
1mole 2 2mole

1 1
(b) The oxidation reaction is: FeO + H2O → Fe2O3 + H+ + e− ; O= F= 96500 coulomb
2 2

Illustration 5: An aqueous solution of sodium chloride on electrolysis gives H2 (g),Cl2 (g) and NaOH according to
the reaction. 2Cl− (aq.) + 2H2O → 2OH− (aq.) + H2 (g) + Cl2 (g) .
A direct current of 25 ampere with a current efficiency 62% is passed through 20 L of NaCl solution (20% by mass).
Write down the reactions taking place at the anode and cathode. How long will it take to produce 1 kg of Cl2? What
will be the molarity of the solution with respect to hydroxide ion? Assume no loss due to evaporation. 
 (JEE ADVANCED)
Chem i str y | 17.7

Sol: Time can be calculate by using charge and current relationship. Effective current is determined by using
current efficiency. Here it is given that we have to find out the molarity of the solution with respect to hydroxide
Ion. Volume is given. We have to find out the no of moles of oxygen.
This can be achieved by calculating the no of mole of Cl2 present in 1 kg.
Reactions at anode and cathode are: 2Cl− → Cl2 + 2e− (at anode)
1000
2H2O + 2e− → H2 + 2OH− ( at cathode) 1kg of=
Cl2 = 14.08 mole
71.0

Charge to produce one mole of Cl2=2 x 96500


Charge to produce 14.08 mole of Cl2=2x96500x14.08
62
Effective current = × 25.0 =
15.5 ampere
100
Charge 2 × 96500 × 14.08
Time
= = = 175318.7 second
= 48.699 hour
Current 15.5

OH- ions produced = 2 × moles of Cl2 = 2x14.08 = 28.16

Mole 28.16
Molarity
= = = 1.408M
Volume 20

Illustration 6: An acidic solution of Cu2+ salt containing 0.4 g of Cu2+ is electrolyzed until all the copper is deposited.
The electrolysis is continued for seven more minutes with volume of solution kept at 100 mL and the current at 1.2
amp. Calculate the gases evolved at NTP during the entire electrolysis. (JEE ADVANCED)

0.4
Cu2+
Sol: 0.4 g of = = 0.0126g − equivalent
31.75
8
At the same time, the oxygen deposited at anode = × 0.0126g =
0.00315g − mole
32
After the complete deposition of copper, the electrolysis will discharge hydrogen at cathode and oxygen at anode.
The amount of charge passed = 1.2 × 7 × 60= 504 coulomb
1 8
So, Oxygen liberated= × 504= 0.00523g − equivalent = × 0.00523 =
0.001307g − mole
96500 32
1
0.00523g − equivalent =
Hydrogen liberated = × 0.00523 =0.00261g − mole
2
Total gases evolved =(0.00315 + 0.001307 + 0.00261)g
= − mole 0.007067g − mole

NTP 22400 × 0.007067mL = 158.3mL


Volume of gases evolved at=

4. ARRHENIUS THEORY OF ELECTROLYTIC DISSOCIATION


In order to explain the properties of electrolytic solutions, Arrhenius put forth a comprehensive theory. The main
postulates of the theory are:
(a) An electrolyte, when dissolved in water, breaks up into two types of charged particles, one carrying a positive
charge and the other a negative charge. These charged particles are called ions. Positively charged ions are
termed as cations and negatively charged as anions.

A +B− + aq. → A + (aq.) + B− (aq.)


1 7 . 8 | Electrochemistr y

(b) The process of splitting of the molecules into ions of an electrolyte is called ionization. The fraction of
the total number of molecules present in solution as ions is known as degree of ionizations or degree of
dissociation. It is denoted by ‘ α ’

Number of molecules dissociated int o ions


(c) α =
Total number of molecules

(d) Ions present in solution constantly re-unite to form neutral molecules and, thus, there is a state of dynamic
equilibrium between the ionized and non-ionized molecules, i.e. AB  A + + B−
[A + ][B− ]
(e) Applying the law of mass action to the above equilibrium = K . K is known as ionization constant. The
[AB]
electrolytes having high value of K are termed strong electrolytes and those having low value of K as weak
electrolytes.

(f) When an electric current is passed through the electrolytic solution, the positive ions (cations) move towards
cathode and the negative ions (anions) move towards anode and get discharged, i.e., electrolysis occurs. The
ions are discharged always in equivalent amounts, no matter what their relative speeds are.

(g) The electrolytic solution is always neutral in nature as the total charge on one set of ions is always equal to
the total charge on the other set of ions. However, it is not necessary that the number of two sets of ions must
be equal always.

AB  A + + B− (Both ions are equal)

NaCl  Na+ + Cl− (Both ions are equal)

AB2  A2+ + 2B− (Anions are double that of cations)



2+ −
BaCl2  Ba + 2Cl (Anions are double that of cations)

A2B  2A + + B2− (Cations are double that of anions)

Na2SO 4  2Na+ + SO24− (Cations are double that of anions)

(h) The properties of electrolytes in solution are the properties of ions present in solution. For example, acidic
solution always contains H+ ions while basic solution contains OH− ions and characteristic properties of
solutions are those of H+ ions and OH− ions respectively.

Limitations of Arrhenius Theory


(i) You cannot apply Ostwald’s dilutions law which is based on Arrhenius theory to strong electrolytes.
(ii) Strong electrolytes conduct electricity in a fused state, i.e., in the absence of water. This is in contradiction of
Arrhenius theory which states that the presence of solvent is imperative for ionization.
(iii) Arrhenius theory assumes independent existence of ions but fails to account for the factors which influence
the mobility of the ions.

4.1 Factors Affecting Degree of Ionization


(a) Nature of solute: When the ionizable parts of a molecule of a substance are held more by covalent bonding
than by electrovalent bonding, less ions are furnished in solution.
(b) Nature of solvent: The main function of the solvent is to weaken the electrostatic forces of attraction between
the two ions and separate them.
(c) Dilution: The extent of ionization of an electrolyte is inversely proportional to the concentration of its
solution. Thus, degree of ionization increases with the increase of dilution of the solution, i.e., decreasing the
concentration of the solution.
Chem i str y | 17.9

(d) Temperature: The degree of ionization increases with the increase in temperature. This is due to the fact
that at higher temperatures molecular speed is greater than before which overcomes the forces of attraction
between the ions.

5. ELECTRICAL CONDUCTANCE
The conductance is the property of the conductor (metallic as well as electrolytic) which facilitates the flow of
electricity through it. It is equal to the reciprocal of resistance, i.e.
1 1
Conductance
= =  ... (i)
Resistance R
It is expressed in the unit called reciprocal ohm ( ohm−1 or mho) or Siemens.

5.1 Specific Conductance or Conductivity


The resistance of any conductor varies directly with its length (l) and inversely with its cross-sectional area (a),
l l
i.e. R ∝ or R = ρ  …..... (ii)
a a
Where, ρ is called the specific resistance. If l = 1 cm and a = 1cm2 , then R = ρ  ... (iii)
The specific resistance is, thus, defined as the resistance of one centimeter cube of a conductor.
The reciprocal of specific resistance is termed the specific conductance or it is the conductance of one centimeter
cube of a conductor. It is denoted by the symbol κ, Thus,
1
κ= , κ= kappa − The specific conductance ... (iv)
ρ
Specific conductance is also called conductivity.
a. 1 l .1 l l 
From eq. (ii), we=
have ρ R or
= ; κ= × C  = cell constant 
l ρ a R a a 
or Specific conductance = conductance × cell constant

5.2 Equivalent Conductance


Equivalent conductance is defined as the conductance of all the ions produced by one gram-equivalent of an
electrolyte in a given solution. It is denoted by Λ .
In general Λ = κ × V  ... (v)
Where, V is the volume in mL containing 1 g-equivalent of the electrolyte.
In case the concentration of the solution is c g-equivalent per liter, then the volume containing
1 g-equivalent of the electrolyte will be 1000/c.
1000
So, equivalent conductance Λ = κ ×  ... (vi)
c
1000
Λ = κ× ; where, N = normality. The unit of equivalent conductance is ohm−1 cm2 eq−1 .
N

5.3 Molar Conductance


The molar conductance is defined as the conductance of all the ions produced by ionization of 1 g-mole of an
electrolyte when present in V ml of solution. It is denoted by µ.
Molar conductance µ = κ × V  ... (vii)
1 7 . 1 0 | Electrochemistr y

Where, V is the volume in mL containing 1 g-mole of the electrolyte. If c is the concentration of the solution in
1000
g-mole per liter, then µ = κ × Its unit is ohm−1 cm2 mol−1 .
c
Molar conductance Molecular mass
Equivalent conductance = ; where, n =
n Equivalent mass

Illustration 7: 1.0 N solution of a salt surrounding two platinum electrodes 2.1 cm apart and 4.2 sq. cm in area was
found to offer a resistance of 50 ohm. Calculate the equivalent conductivity of the solution. (JEE MAIN)

Sol: As Equivalent conductivity = κ × V


In order to find equivalent conductivity we have to calculate specific conductance.
l 1
Specific conductance ( κ ) is given as κ = .
a R
l 1
Given, l = 2.1 cm, a = 4.2 sq. cm, R = 50 ohm. Specific conductance, κ = .
a R
2.1 1
Or =
κ × = 0.01 ohm−1 cm−1 ; Equivalent conductivity = κ × V
4.2 50
V = The volume containing 1 g-equivalent = 1000 mL
= 0.01 × 1000 = 10 ohm−1 cm2 eq−1
So, Equivalent conductivity

Illustration 8: The specific conductivity of 0.02M KCl solution at 25ºC is 2.768 × 10−3 ohm−1 cm−1 . The resistance
of this solution at 25ºC when measured with a particular cell was 250.2 ohm. The resistance of 0.01 M CuSO 4
solution at 25ºC measured with the same cell was 8331 ohm. Calculate the molar conductivity of the copper
sulphate solution. (JEE ADVANCED)
1000
Sol: Molar conductivity is given
= by Sp. cond. × so first we have to calculate specific conductivity of the
C
solution. Sp.conductivity is given as a product of cell constant and conductance. Now cell constant is not provided;
we can calculate it from the conductance and Sp.conductivity of KCl solution.
Sp. cond. of KCl 2.768 × 10−3
Cell constant = = = 2.768 × 10−3 × 250.2
Conductance of KCl 1 / 250.2
For 0.01 M CuSO4 solution
1
Sp. Conductivity = Cell constant × Conductance = 2.768 × 10−3 × 250.2 ×
8331
1000 2.768 × 10−3 × 250.2 1000
= Sp. cond.
Molar conductance = × = × 8.312 ohm−1 cm2 mol−1
C 8331 1 / 100

6. KOHLRAUSCH’S LAW
At infinite dilution, when dissociation is complete, each ion makes a definite contribution towards molar conductance
of the electrolyte, irrespective of the nature of the ion with which it is associated and the value of molar conductions
of its constituent ions, i.e., Λ = λ + + λ − λC and λa are called the ionic conductance of cation and anion at infinite
dilution respectively. The ionic conductance are proportional to their ionic mobilities. Thus, at infinite dilution,
λC = kuC and λa = kua, where, uC and ua are ionic mobilities of cation and anion respectively at infinite dilution. The
value of k is equal to 96500 C, i.e., one Faraday.
Thus, assuming that increase in equivalent conductance with dilution is due to increase in the degree of dissociation
of the electrolyte, it is evident that the electrolyte achieves the degree of dissociation as unity when it is completely
ionized at infinite dilution. Therefore, at any other dilution, the equivalent conductance is proportional to the
degree of dissociation. Thus,
Chem i str y | 17.11

Λ Equivalent conductance at a given concentration


Degree of dissociation =
α =
Λ∞ Equivalent conductance at inf inite dilution

Ionic Mobility, µ: It is the distance travelled by an ions per second under a potential gradient of 1 volt per meter.
 1. For an, µ = λ º /F
 2. Ionic mobility of an ion depends on its charge, size, viscosity of solvent, temperature, etc.
 3. For aqueous solution, greater the charge or smaller the size of gaseous ion, greater will be the size of aqueous
ion. When such a big ion moves in solution, it experiences greater resistance by the size of solvent particles.
This results in a decrease in its conductance as well as ionic mobility. Following are the increasing order of
ionic mobilities of some ions:

Li+ < Na+ < K + < Rb+ < Cs+ ; F− < Cl− < Br − < I − ; Al3+ < Mg2+ < Na+
 4. The size of gaseous H+ ion is smallest among all the ions and hence its ionic mobility should be minimum
but among all the ions, it is maximum. The ion with second highest ionic mobility is OH–. The very high ionic
mobilities of these ions are due to interchange of hydrogen bonds and covalent bonds, by which migration of
charge occurs without any large displacement in the ions (Grotthus mechanism).

Applications of Kohlrausch’s Law:


0 0
(a) Determining Λm of a weak electrolyte: In order to calculate Λm of a weak electrolyte say CH3COOH, we
0
determine experimentally Λm values of the following three strong electrolytes:
(i) A strong electrolyte containing same cation as in the test electrolyte, say HCl
(ii) A strong electrolyte containing same anion as in the test electrolyte, say CH3COONa
(iii) A strong electrolyte containing same anion of (a) and cation of (b) i.e. NaCl.
0 0 0 0 0
Λm of CH3COOH is then given as: Λm (CH3COOH) = Λm (HCl) + Λm (CH3COONa) − Λm (NaCl)
0
Proof: Λm (HCl) = λH0 + λ  ... (i)
Cl−
0
Λm (CH3COONa) = λ0 +λ  ... (ii)
CH3COO− Na+

0
Λm (NaCl) = λ0 + λ0 −  ... (iii)
Na+ Cl

Adding equation (I) and equation (II) and subtracting (III) from them:
0 0 0
Λ(HCl) + Λ(CH − Λ(NaCl) = λ0 + + λ0 = Λ0(CH
3COONa) (H ) (CH3COO− ) 3COOH)

No. of molecules ionised Λm


(b) Determination
= of degree of dissociation (α): α =
Total number of molecules dissolved Λ0
m

0 1000 κ
(c) Determination of solubility of sparingly soluble salt: Λm = ,
C
Where C is the molarity of solution and hence the solubility.
0 0
(d) Determination of ionic product of water: From Kohlrausch’s law, we determine Λm of H2O where Λm is the
molar conductance of water at infinite dilution when one mole of water is completely ionized to give one mole
0
of H+ and one mole of OH− ions i.e. Λm (H2O) = λ0 + + λ0
H OH−

κ × 1000
Again using the following Λm = , where C=molar concentration i.e. mole L−1 or mole dm−3
C
1 7 . 1 2 | Electrochemistr y

κ
⇒ Λm = , where C = concentration in mole m−3
C
0 0 κ κ
Assuming that Λm differs very little from Λm ; Λm = ⇒C=
C Λm0

Specific conductance ( κ ) of pure water is determined experimentally. Thereafter, molar concentration of


dissociated water is determined using the above equation. K w is then calculated as: K w = C2

7. THEORY OF WEAK ELECTROLYTES


(i) Electrolytes that are not completely ionized when dissolved in a polar medium like water are called weak
electrolytes. There exists equilibrium between ions and unionized molecules. AB  A + + B−
(ii) The Concept of chemical equilibrium and law of mass action can be applied to ionic equilibrium also.
AB  A⁺ + B⁻
t = 0 C 0 0
[A + ][B− ] Cα × Cα Cα 2
teq. C – Cα Cα; K
Cα = = ; K=  ... (i)
[AB] C(1 − α ) 1−α

For weak electrolytes, α << 1 ∴ (1 − α ) ≈ 1


K
Thus, equation (i) can be written as: K= Cα2 ⇒ α =  ... (ii)
C
From eq. (ii), it is clear that on dilution concentration decreases, as a result of which degree of ionization 'α'
increases. Both equivalent and molar conductance increase when at a high degree of ionization.
ΛCe Λm
C
(iii) Degree of ionization can be calculated as: =
α =  ... (iii)
Λe∞ Λm

ΛCe , Λm
C
= Equivalent and molar conductance at concentration ‘C’

Λe∞ , Λm

= Equivalent and molar conductance at infinite dilution. 2
 ΛC 
C ×  ∞e 
Λ  C( ΛCe )2
Substituting the values of ' α ' from eq. (iii) in eq. (i),=
we get K =  e  ... (iv)
ΛCe Λe∞ ( Λe∞ − ΛeC )
C 2
C( Λm ) 1 −
Similarly K =  Λe∞ ... (v)
∞ ∞ C 
Λm ( Λm − Λm )

Equations (iv) and (v) are called Ostwald equation.

Illustration 9: A decinormal solution of NaCl has specific conductivity equal to 0.0092. If ionic conductance of Na+
and Cl− ions at the same temperature are 43.0 and 65.0 ohm−1 respectively, calculate the degree of dissociation of
NaCl solution.  (JEE MAIN)

Sol: Degree of dissociation is calculated as equivalent conductance at a particular dilution divided by equivalent
conductance at infinite dilution.so first we have to calculate Λ ∞
Λ v . Λ ∞ is determined by summing up ionic conductance of each ion. Λ v is product of specific conductance and
dilution.
Equivalent conductance of N/10 NaCl solution
=Λ v Sp.conductivity × dilution = 0.0092 × 10,000 = 92 ohm−1 ;

Λ∞ = λ +λ = 43.0 + 65.0 = 108 ohm−1


Na+ Cl−
Chem i str y | 17.13

Λv 92
Degree of dissociation, =
α = = 0.85
Λ ∞ 108

Illustration 10: At 18ºC, the conductivities at infinite dilution of NH4 Cl,NaOH and NaCl are 129.8, 217.4 and 108.9
mho respectively. If the equivalent conductivity of N/100 solution of NH4 OH is 9.93 mho, calculate the degree of
dissociation of NH4 OH at this dilution. (JEE ADVANCED)

Sol: Degree of dissociation is calculated as equivalent conductance at a particular dilution divided by equivalent
conductance at infinite dilution. Λ v is given. Λ ∞ is determined by summing up ionic conductance of each ion
Λ ∞NH =λ +λ = 129.8  ... (i)
4 Cl NH+
4 Cl−
Λ ∞NaOH = λ +λ = 217.4  ... (ii)
Na+ OH−

Adding eqs. (i) and (ii) and subtracting eq. (iii),


Λ +λ +λ +λ −λ −λ =λ +λ = 129.8 + 217.4 − 108.9
NH+
4 Cl− Na+ OH− Na+ Cl− NH+
4 OH−

Λv 9.93
Λ ∞NH 238.3 mho . Degree of dissociation, =
= α = = 0.04167 or 4.17% dissociated.
4 OH Λ ∞ 238.3

8. ELECTROCHEMICAL CELLS

8.1 Electrolytic Cells


Electrolytic cells are devices in which electrolysis (chemical reaction involving oxidation and reduction) is carried
out by using electricity or in which conversion of electrical energy into chemical energy is done.

8.2 Galvanic Cells


This is a device where a redox reaction is used to convert chemical energy into electrical energy, i.e., electricity is
obtained with the help of oxidation and reduction reaction. The chemical reaction responsible for production of
electricity takes place in two separate compartments. Each compartment consists of a suitable electrolyte solution
and a metallic conductor. The metallic conductor acts as an electrode and the compartments containing the
electrode and the electrolyte solution are called half-cells. When the two compartments are connected by a salt
bridge and electrodes are joined by a wire through the galvanometer, the electricity begins to flow. This is the
simple form of voltaic cell.

8.3 Daniel Cell

Figure 17.1: Representation of a daniel cell


1 7 . 1 4 | Electrochemistr y

Oxidation half reaction, Zn(s) → Zn2+ (aq.) + 2e−

Reduction half reaction, Cu2+ (aq.) + 2e− → Cu(s)

Net reaction Zn(s) + Cu2+ (aq.) → Zn2+ (aq.) + Cu(s)

Salt Bridge: A Salt bridge is usually an inverted U-tube filled with a concentrated solution of inert electrolytes. An
inert electrolyte is one whose ions are neither involved in any electrochemical change nor do they react chemically
with the electrolytes in the two half-cells.
Significance of salt bridge: The following are the functions of the salt bridge:
(i) It connects the solutions of two half-cells and completes the cell circuit.
(ii) It prevents transference or diffusion of the solutions from one half-cell to the other.
(iii) It keeps the solutions in two half-cells electrically neutral.
(iv) It prevents liquid-liquid junction-potential, i.e., the potential difference which arises between two solutions
when in contact with each other.
A broken vertical line or two parallel vertical lines in a cell reaction indicates the salt bridge.
Zn | Zn2+ || Cu2+ | Cu

Electrode Potential: A metal placed in a solution of its ions obtains either a positive or negative charge with respect
to the solution. On account of this, a definite potential is developed between the metal and the solution. This potential
difference is called electrode potential. It depends on the nature of electrode, concentration of ions and temperature.

Oxidation Potential: It is the tendency of an electrode to get oxidized, i.e., to lose electrons.
M → Mn+ + ne−
Reduction potential: It is the tendency of an electrode to get reduced, i.e., to accept electrons.
Mn+ + ne− → M
Standard Electrode Potential: The potential difference developed between metal electrode and the solution of its
ions of unit molarity (1 M) at 25ºC (278 K) is called standard electrode potential.

The magnitude of potential depends on the following factors:


(i) Nature of the electrode,
(ii) Concentration of the ions in solution,
(iii) Temperature.

Standard Electrode Potential: While dipping an electrode in a solution in order to compare the electrode
potentials of different electrodes, it is essential to first specify the ion concentration in the solution as well as the
temperature of the half cell. The potential difference developed between metal electrode and the solution of its
ions of unit molarity (1 M) at 25ºC (298 K) is called standard electrode potential. Standard oxidation potential =
-Standard reduction potential
EMF of a cell: It is the difference in the potential across left and right electrodes due to which electrons flow from
anode to cathode.
Standard EMF: The EMF values of an electrode under standard conditions (1 atm, 298 K) and the unit concentrations

of its ions is called as standard EMF and is denoted by Ecell
º º º º º º
Ecell Ecathode
= − Eanode or Ecell Eright
= electrode − Eleft electrode
Chem i str y | 17.15

8.4 Reference Electrode (Standard Hydrogen Electrode, SHE or NHE)


The potential of an individual half-cell cannot be measured but the difference in the potential of two half-cells
can be measured experimentally. It is therefore, necessary to couple the electrodes with another electrode whose
potential is known. This electrode is termed as reference electrode like standard hydrogen electrode (SHE). Which
is standard electrode potential considered zero.

H2(g)
(at 1 atm)

H3 O+(aq.) Pt black
(1 M) electrode

Figure 17.2: Representation of reference electrode

The hydrogen electrode thus obtained forms one of two half-cells of a voltaic cell. A voltaic cell is created when this
half-cell is connected with any other half-cell. The hydrogen electrode can act as cathode or anode with respect to
other electrode.
SHE half reaction Electrode potential
+ −
H2 → 2H + 2e 0.0 V (Anode)
2H+ + 2e− → H2 0.0 V (Cathode)

8.5 Some other Reference Electrodes


(i) Calomel Electrode: Pt wire

To salt

bridge
Saturated
KCl solution

Hg2Cl2 and
Hg paste

Mercury

Figure 17.3: Representation of calomel electrode

1 
→ Hg + Cl
Reaction when electrode act as cathode: Hg2Cl2 + e− ←
2
(ii) Silver-silver chloride electrode: This is another widely used reference electrode. It is reversible and stable
and can be combined with cells containing chlorides without inserting liquid junction.
Silver chloride is deposited electrolytically on a silver or platinum wire and it is then immersed in a solution
containing chloride ions. Its standard electrode potential with respect to the standard hydrogen electrode is
0.2224 V at 298 K. The electrode is represented as: Ag | AgCl | Cl–
The electrode reaction is: AgCl + e− → Ag + Cl−
1 7 . 1 6 | Electrochemistr y

PLANCESS CONCEPTS

In MnO −4 + 8H+ + 5e → Mn2+ + 4H2O , oxidant should be taken as [MnO −4 ][H+ ]8 ,i.e., all ions
concentration present with oxidant should be reported accordingly in Nernst half-cell potential. Similarly
for reductant, all ions present with reductant should be considered.
Aman Gour (JEE 2012 AIR 230)

8.6 Nernst Equation

Nernst Equation: Suppose, for example, that we reduce the concentration of Zn2+ in the Zn/Cu cell from its unit-
activity value of around 0.5 M to a much smaller value:

Zn(s) Zn2+ (0.001M) || Cu2+ || Cu(s)

This will reduce the value of Q for the cell reaction Zn(s) + Cu2+ → Zn2+ + Cu(s)

The free energy change ∆G more negative than ∆G0, so than E would be more positive than Eº.
The relation between the actual cell potential E and the standard potential Eº is developed in the following way.
According to cell potential and Gibb’s free energy change: ∆G0 = –nEoF; ∆G = –nEF
These expressions can then be substituted into the relation ∆G = ∆G0 + RTIn Q
RT
Which gives –nEF = –nEoF + RTIn Q . Which can be rearranged to E= E − lnQ
nF
This is Nernst Equation, which relates the cell potential to the standard potential Eº. If Q is unity then at 25ºC
0.059
Nernst Equation will be E= Eº − logQ
n
And for a general electrochemical reaction of the type aA+bBcC+dD
RT RT [C]c [D]d
Nernst equation can be written as: Ecell = 
E(cell) − lnQ = 
E(cell) − ln
nF nF [A]a [B]b

PLANCESS CONCEPTS

• The overall reaction and ∆Gº for each cell is same.


º
• Ecell and 'n' values are different for each cell.
º
• Ecell × n is same for each cell.
º
• ∆Gº depends on cell reaction and Ecell depends upon making up of a cell.

B Rajiv Reddy (JEE 2010 AIR 11)


Chem i str y | 17.17

8.7 Electrochemical Series

Characteristics of electrochemical series


(a) Negative sign of SRP (standard reduction potential) indicates that an electrode when joined with SHE acts as
anode and oxidation occurs on this electrode. Similarly positive sign of SRP indicates that an electrode when
joined with SHE acts as cathode and reduction occurs on this electrode.
(b) In the series, those substances that are stronger reducing agents than hydrogen are placed above it.
(c) The substances which are stronger oxidizing agents than H+ ion are placed below hydrogen in the series.
(d) The activity decreases from top to bottom and the metals on top are called active metals.

Application of electrochemical series


(i) Reactivity of metals
•• Alkali metals and alkaline earth metals having high –ve values of SRP which are chemically active react
with cold water, evolve hydrogen and readily dissolve in acids.
•• Metals like Fe, Pb, Sn, Ni, Co etc. do not react with cold water but react with steam to evolve hydrogen.
•• Metals Li, Be, Cu, Ag and Au which lie below hydrogen are less reactive and do not evolve hydrogen from
water.
(ii) Electropositive character of metals: Electropositive character of metals decreases from top to bottom.
(iii) Displacement reactions: The metal having low SRP will displace the metal from its salt’s solution which has
higher value of SRP.
(iv) Reducing power of metals: Reducing nature decreases from top to bottom in the electrochemical series.
(v) Oxidizing nature of non-metals: Oxidizing nature increases from top to bottom in the electrochemical
series.
(vi) Thermal stability of metallic oxides: The thermal stability of the metal oxide decreases from top to bottom.
(vii) Products of electrolysis: The ion which is a stronger oxidizing agent is discharged first at cathode.
K + ,Ca2+ ,Na+ ,Mg+2 , Al+3 , Zn+2 ,Fe+2 ,H+ ,Cu+2 , Ag+ , Au+3 increasing order of deposition.
(viii) Corrosion of metals: Corrosion is defined as the deterioration of a substance because of its reaction with its
environment. The corrosion tendency decreases from top to bottom.
(ix) Extraction of metals: Ag and Au extracted by cyanide process.
1 7 . 1 8 | Electrochemistr y

Table 17.3: Reduction potential of different ions

-
Reaction (Oxidized form + ne ) → Reduced form Eº/V
2.87
F2 (g) + 2e− → 2F−
1.81
Co3+ + e− → Co2+
→ 2H2O 1.78
H2O2 + 2H+ + 2e−
→ Mn2+ + 4H2O 1.51
MnO −4 + 8H+ + 5e−
1.40
Au3+ + 3e− → Au(s)
1.36
Cl2 (g) + 2e− → 2Cl−
→ 2Cr3+ + 7H2O 1.33
Cr2O27− + 14H+ + 6e−
→ 2H2O 1.23
O2 (g) + 4H+ + 4e−
2+
→ Mn + 2H2O 1.23
MnO2 (s) + 4H+ + 2e−
1.09
Br2 + 2e− → 2Br −
Increasing strength of oxidizing agent

Increasing strength of reducing agent


→ NO(g) + 2H2O 0.97
NO3− + 4H+ + 3e−
→ Hg22+ 0.92
2Hg2+ + 2e−
0.80
Ag+ + e− → Ag(s)
0.77
Fe3+ + e− → Fe2+
→ H2O2 0.68
O2 (g) + 2H+ 2e−
0.54
I2 + 2e− → 2I −
0.52
Cu+ + e− → Cu(s)
0.34
Cu2+ + 2e− → Cu(s)
0.22
AgCl(s) + e− → Ag(s) + Cl−
0.10
AgBr(s) + e− → Ag(s) + Br −

→ H2 (g) 0.00
2H+ + 2e-
-0.13
Pb2+ + 2e− → Pb(s)
-0.14
Sn2+ + 2e− → Sn(s)
-0.25
Ni2+ + 2e− → Ni(s)
Chem i str y | 17.19

-
Reaction (Oxidized form + ne ) → Reduced form Eº/V

→ Fe(s) -0.44
Fe2+ + 2e−
→ Cr(s) -0.74
Cr3+ + 3e−
Increasing strength of oxidizing agent

Increasing strength of reducing agent


→ Zn(s) -0.76
Zn2+ + 2e−
2H2O + 2e− → H2 (g) + 2OH− (aq) -0.83

Al3+ + 3e− → Al(s) -1.66

Mg2+ + 2e− → Mg(s) -2.36

Na+ + e− → Na(s) -2.71

Ca2+ + 2e− → Ca(s) -2.87

K + + e− → K(s) -2.93

Li+ + e− → Li(s) -3.05

Two important parameters that can be determined from a cell potential are the equilibrium constant for the cell
reaction and the free energy change for the cell reaction.
º
1. Determining the equilibrium constant from Ecell
º
2. Determining the standard state free energy change from Ecell

3. Determining the non-standard free energy change


º
Determining the Equilibrium Constant from Ecell
To calculate the equilibrium constant for an electrochemical cell we need to know:
1. The standard state potential for a cell
2. The half-reactions involved
º RT
The Nernst equation is used in calculating the equilibrium constant. Ecell = lnQ
nF
At equilibrium Q = K, Substituting in K for Q and the values for R, T and F we get:

º 0.0257 0.0592
=Ecell = lnK logK
n n

Example: Find the value of the equilibrium constant at 25ºC for the cell reaction for the following electrochemical
cell: Cu | Cu2+ (1M) || Ag+ (1M) | Ag

Sol: (i) Write the equations for the cell half-reactions, calculate the standard cell potential and determine the
number of electrons transferred.
º
2Ag+ (aq) + 2e− → 2Ag(s) Ereduction = +0.799 V

º
Cu(s) → Cu2+ (aq) + 2e− Eoxidation = −0.518 V

º
2Ag+ (aq) + Al(s) → 2Ag(s) + Cu2+ (aq) Ecell = +0.281 V
1 7 . 2 0 | Electrochemistr y

n = 2 moles of electrons
(ii) Substitute into the above equations and solve for K.
0.0592 0.0592
º
Ecell = logK ; 0.281 = K 109.49
logK ; logK = 9.49 ; = = 3.1 × 109
n 2
Note: values for the equilibrium constant for electrochemical cell reactions are sometimes very large.
º
Determining the Standard State Free Energy Change from Ecell
To determine the standard state free energy change for a cell reaction
º
1. Determine the Ecell
2. Determine the number of moles of electrons transferred in the reaction.
º
3. Solve for ∆G using the equation ∆G =
−nFEcell

∆Gº =standard state free energy change ( joules); n = number of moles of electrons transferred
º
F = Faraday’s constant (96,485 C/mole e− ); Ecell = standard state cell potential (volts or joules/C)

Example: Find the value of the equilibrium constant at 25ºC for the cell reaction for the following electrochemical
cell: Cu | Cu2+ (1M) || Ag+ (1M)Ag
º
(The solution for the determination of the Ecell and the number of moles of electrons, n, are shown in the example
in the previous section.)
º º
1. Determine the Ecell . Ecell = +0.281 volts
2. Determine the number of moles of electrons transferred. n = 2 moles of e−
3. Substitute into the equation and solve.
∆G =−(2mole− )(96, 485C / mole− )0.281 J / C; ∆G =−54,200 J or − 54.2kJ

Determining the Non-Standard State Free Energy Change


To determine the non-standard state free energy change:
º
1. Calculate the standard cell potential, Ecell
2. Determine the number of moles of electrons transferred, n
3. Calculate the reaction quotient, Q
4. Calculate the non-standard cell potential, Ecell, using the Nernst equation
5. Calculate the non-standard free energy change using the equation: ∆G =−nFEcell

Example: Calculate the free energy change for the following electrochemical cell.

Zn(s) | Zn2+ (1.50M) || Cu2+ (0.25M) | Cu(s)


º
1. Calculate Ecell .
º
Zn(s) → Zn2+ (aq) + 2e− Eoxidiation = +0.762 volts

º
Cu2+ (aq) + 2e− → Cu(s) Ereduction = +0.339 volts

º
Zn(s) + Cu2+ (aq) → Zn2+ (aq) + Cu(s) Ecell = +1.101 volts

2. Determine “n”. n = 2 moles of electrons


Chem i str y | 17.21

[Zn2+ ] 1.50
3. Calculate Q;
= Q = = 6.0
[Cu2+ ] 0.25
0.0257
4. Calculate Ecell ; Ecell = 1.101 volts − ln6 = 1.078 volt
2
5. Calculate ∆G.
∆G =−nFEcell =−(2mole e− )(96, 485C / mole e− )(1.078 volts) ; ∆G =−208,000 joules or − 208kJ

8.8 Corrosion
The weakening and deterioration of a substance because of its reaction with its environment is called as corrosion.
This is also defined as the process by which metals have the tendency to go back to their combined state, i.e.,
reverse of extraction of metals.
Water drop

O2 O2

OH- 2+ OH-
Fe
Rust Rust
Cathode Anode Cathode

Iron
e- e-

Figure 17.4 : Process of corrosion

Oxidation: Fe(s) → Fe2+ (aq.) + 2e−

Reduction: O2 + 4H+ (aq.) + 4e− → 2H2O(l )

Atmospheric: 4Fe2+ + O2 + 4H2O(l ) → 2Fe2O3 (s) + 8H+ (aq.)

Oxidation: Fe2O3 + xH2O → Fe2O3 . xH2O


Rust

8.9 Dry Cell


In this cell, once the chemicals have been consumed, further reaction is not possible. It cannot be regenerated by
reversing the current flow through the cell using an external direct current source of electrical energy.
As the cell operates, the zinc is oxidized to Zn2+ ; Zn → Zn2+ + 2e− (Anode reaction)
The electrons are utilized at carbon rod (cathode) as the ammonium ions are reduced.
2NH+4 + 2e− → 2NH3 + H2 (Cathode reaction)

The cell reaction is Zn + 2NH+4 → Zn2+ + 2NH3 + H2

Hydrogen is oxidized by MnO2 in the cell. 2MnO2 + H2 → 2MnO(OH)

Ammonia produced at cathode combines with zinc ions to form complex ion.

Zn2+ + 4NH3 →[Zn(NH3 )4 ]2+ ; Ecell is 1.6 volt.


1 7 . 2 2 | Electrochemistr y

PLANCESS CONCEPTS

• T
 hese are called dry cells but are not actually dry. These contain moist paste and operate only as long
as the paste in it remains wet.
• In alkaline cells, e.m.f. is independent of concentration of alkali because redox reaction does not
involve [OH⁻].
• Alkaline cells retards corrosion as corrosion is favored more in H+ ions.
• Alkaline cells show more efficient ion transport because of alkaline electrolyte and thus give rise to
more stable current and voltage.
Mredul Sharda JEE Advanced 2013 AIR

8.10 Fuel Cell


Fuel cells are another means to convert chemical energy to electrical energy. . The main disadvantage of a primary
cell is that it can deliver current for a short period only. This is because the quantity of oxidizing agent and
reducing agent is limited. But energy can be obtained indefinitely from a fuel cell as long as the outside supply of
fuel is maintained. One of the examples is the hydrogen-oxygen fuel cell. The cell consists of three compartments
separated by a porous electrode. Hydrogen gas is introduced into one compartment and oxygen gas is fed into
another compartment. These gases then diffuse slowly through the electrodes and react with electrodes that are
made of porous carbon and the electrolyte is a resin containing concentrated aqueous sodium hydroxide solution.
Hydrogen is oxidized at anode and oxygen is reduced at cathode. The overall cell reaction produces water. The
reactions which occur are:

Anode: [H2 (g) + 2OH− (aq.) → 2H2O(l ) + 2e− ] × 2

Cathode: O2 (g) + 2H2O(l ) + 4e− → 4OH− (aq.)

Overall: 2H2 (g) + O2 (g) → 2H2O(l )

These type of cells are used in space-crafts. Fuel cells are efficient and pollution free.
Thermodynamic efficiency of fuel cells is the ratio of the electrical free energy to the enthalpy of the reaction.
∆G
η=
∆H

8.11 Concentration Cell


(a) Electrode concentration cells: In these cells, the potential difference is developed between two like electrodes
at different concentrations dipped in the same solution of the electrolyte. For example, two hydrogen
electrodes at different gas pressures in the same solution of hydrogen ions constitute a cell of this type.

Pt,H2 (Pr essure p1 ) H2 (Pressure p2 )Pt


[H+ ]
Anode Cathode
If p1 > p2 , oxidation occurs at LHS electrode and reduction occurs at RHS electrode.

0.0591 (p )
Ecell = log 1 at 25º C
2 (p2 )

In the amalgam cells, two amalgams of the same metal at two different concentrations are immersed in the
same electrolytic solution. M(HgC1 ) | M n+ | Zn(HgC2 )
Chem i str y | 17.23

0.0591 C
The e.m.f of the cell is given by the expression Ecell = log 1 at 25º C
n C2
(b) Electrolyte concentration cells: In these cells, electrodes are identical but these are immersed in a solution
of the same electrolyte of different concentrations. The source of electrical energy in the cell is the tendency
of the electrolyte to diffuse from a solution of higher concentration to that of lower concentration. With the
expiry of time, the two concentrations tend to become equal. Thus, at the beginning, the e.m.f of the cell is at
its maximum and it gradually falls to zero. Such a cell is represented in the following manner: ( C2 is greater
than C1 ).

Zn | Zn2+ (C1 ) || Zn2+ (C2 ) | Zn


M | Mn+ (C1 ) || Mn+ (C2 ) | M or
Anode Cathode
0.0591 C
The e.m.f of the cell is given by the following expression: Ecell = log 2(RHS) at 25ºC
n C1(LHS)
The concentration cells are used to determine the solubility of sparingly soluble salts, valency of the cation of the
electrolyte and transition point of the two allotropic forms of a metal used as electrodes, etc.

Example: Find the standard cell potential for an electrochemical cell with the following cell reaction.
Zn(s) + Cu2+ (aq) → Zn2+ (aq) + Cu(s)

Sol:
(i) Write the half-reactions for each process. Zn(s) → Zn2+ (aq) + 2e− ; Cu2+ (aq) + 2e− → Cu(s)
º
(ii) Look up the standard potentials for the reduction half-reaction. Ereduction of Cu2+ = +0.339 V
(iii) Look up the standard reduction potential for the reverse of the oxidation reaction and change the sign.
º
Ereduction of Zn2+ = −0.762 V,Eoxidation
º
of Zn = −( −0.762 V) = +0.762 V
(iv) Add the cell potentials together to get the overall standard cell potential.
º º
Oxidation: Zn(s) → Zn2+ (aq) + 2e− Eoxidation = −Ereduction = −( −0.762 V) = +0.762 V
º
Reduction: Cu2+ (aq) + 2e− → Cu(s) Ereduction = +0.339 V
º
Overall: Zn(s) + Cu2+ (aq) → Zn2+ (aq) + Cu(s) Ecell = +1.101 V

Example: Predict the cell potential for the following reaction when the pressure of the oxygen gas is 2.50 atm, the
hydrogen ion concentration is 0.10 M and the bromide ion concentration is 0.25 M.
O2 (g) + 4H+ (aq) + 4Br − (aq) → 2H2O(l ) + 2Br2 (l )
º
Sol: (i) Calculate the standard cell potential for the reaction, Ecell, using the tabled values:
º º
Oxidation: 4Br (aq) → 2Br2 (l ) + 4e Eoxidation = −Ereduction = −( +1.077 V) = −1.077 V
− −

+ − º
Reduction: O2 (g) + 4H (aq) + 4e → 2H2O(l ) Ereduction = +1.229 V

Overall: O2 (g) + 4H+ (aq) + 4Br − (aq) → 2H2O(l ) + 2Br2 (l ) Eº = +0.152 V


cell

(ii) Determine the new cell potential resulting from the changed conditions.
(iii) Calculate the value for the reaction quotient, Q. (Note: We calculate Q using molar concentrations for solutions
and pressures for gases. Water and bromine are both liquids, therefore they are not included in the calculation of Q.)
1 7 . 2 4 | Electrochemistr y

1 1
Q = ; Q Q 1.02 × 106
;=
+ 4 − 4
PO [H ] [Br ] (2.50atm)(0.10 M)4 (0.25M)4
2

(iv) Calculate the number of moles of electrons transferred in the balanced equation, n.
n = 4 moles of electrons
(v) Substitute values into the Nernst equation and solve for the non-standard cell potential, Ecell .

+0.152 V − (0.0257 / 4)ln(1.02 × 106 ),Ecell =


Ecell = 0.063V

Illustration 11: Reaction → 2Ag + Cd2+ . The standard electrode potentials for Ag+ → Ag and Cd2+ → Cd couples
are 0.80 volt and −0.40 volt respectively
(i) What is the standard potential E for this reaction?
(ii) For the electrochemical cell, in which this reaction takes place which electrode is negative electrode?
 (JEE MAIN)
Sol: First write down the two half-cell. Standard potential E for the cell is given by standard potential of reducing
electrode+ standard potential of oxidising electrode. The electrode having less electrode potential act as negative
electrode.

(i) The half reactions are: 2Ag+ + 2e− → 2Ag


Re duction
(Cathode)

Eº = 0.80 volt (Reduction potential); Cd → Cd2+ + 2e−


Ag+ /Ag Oxidation
( Anode )

Eº = −0.40 volt (Reduction potential) or Eº = +0.40 volt


Cd2 + /Cd Cd/Cd2 +

Eº = Eº + Eº = 0.40 + 0.80 = 1.20 volt


Cd/Cd2 + Ag+ /Ag

Illustration 12: The standard oxidation potential of zinc is 0.76 volt and of silver is − 0.80 volt. Calculate the e.m.f
of the cell:
Zn | Zn(NO3 )2 || AgNO3 | Ag At 25ºC. (JEE MAIN)
0.25M 0.1M

Sol: First calculate the standard potential for reaction which is calculated as
º º º
Ecell
= Eoxidation − Ereduction
º
After calculating Ecell e.m.f of the cell can be easily calculated using following equation

º 0.0591 [Products]
E=
cell Ecell − log
n [Reactants]
º
The cell reaction is Zn + 2Ag+ → 2Ag + Zn2+ ; Eoxidation of Zn = 0.76 volt

º º º º
Ereduction of Ag = 0.80 volt ; Ecell = Eoxidation of Zn + Ereduction of Ag = 0.76 + 0.80 = 1.56 volt

º 0.0591 [Products]
We know that, E=
cell Ecell − log
n [Reactants]

º 0.0591 0.25 0.0591


= Ecell − log 1.56 −
= × 1.3979
= (1.56 − 0.0413) volt = 1.5187 volt .
n 0.1 × 0.1 2
Chem i str y | 17.25

Illustration 13: Calculate the e.m.f of the cell. Mg(s) | Mg2+ (0.2M) || Ag+ (1 × 10 −3 ) | Ag

Eº +0.8 volt, Eº 2+
= −2.37 volt
=
Ag+ /Ag Mg /Mg

What will be the effect on e.m.f if concentration of Mg2+ ion is decreased to 0.1 M?  (JEE MAIN)

Sol: First calculate the standard potential for reaction which is calculated as
º º º
Ecell
= Eoxidation − Ereduction
º
After calculating Ecell e.m.f of the cell can be easily calculated using following equation

º 0.0591 [Products]
E=
cell Ecell − log
n [Reactants]
º º
Ecell
= ECathode − EºAnode
= 0.80 − ( −2.37)
= 3.17 volt
º 0.0591 Mg2+
Cell reaction, Mg + 2Ag+ → 2Ag + Mg2+ ; E=
cell Ecell − log
n [Ag+ ]2
0.0591 0.2
= 3.17 − log 3.0134 volt when Mg2+ = 0.1M
3.17 − 0.1566 =
=
2 [1 × 10−3 ]2

º 0.0591 0.1
E=
cell Ecell − log = (3.17 − 0.1477) volt = 3.0223 volt.
2 (1 × 10−3 )2

Illustrations 14: To find the standard potential of M3+ / M electrode, the following cell is constituted:
Pt | M | M3+ (0.0018mol−1L) || Ag+ (0.01mol−1L) Ag
The e.m.f of this cell is found to be 0.42 volt. Calculate the standard potential of the half reaction
M3+ + 3e− → M. Eº = 0.80 volt.  (JEE MAIN)
Ag+ /Ag

Sol: Here e.m.f is given we have to calculate standard potential of anode. So first we have to calculate standard
potential of the cell and subtract it from the provided standard potential of cathode.
Standard potential of cell can be determined using following expression, Nernst equation

º 0.0591 [M3+ ]
E=
cell Ecell − log
3 [Ag+ ]3
The cell reaction is M + 3Ag+ → 3Ag + M3+

º 0.0591 [M3+ ]
Applying Nernst equation, E=
cell Ecell − log
3 [Ag+ ]3

º 0.0591 (0.0018) º º
0.42 =
Ecell − log Ecell
= − 0.064 ; Ecell =(0.42 + 0.064) =0.484 volt
3 (0.01)3

º º
Ecell
= ECathode − EºAnode or =
EºAnode ECathode
º º
− ECell =(0.80 − 0.484) =0.32 volt .
1 7 . 2 6 | Electrochemistr y

PROBLEM-SOLVING TACTICS

(a) Related to electrolysis: Electrolysis comprises of passing an electric current through either a molten salt or
an ionic solution. Thus the ions are “forced” to undergo either oxidation (at the anode) or reduction (at the
cathode). Most electrolysis problems are really stoichiometry problems with the addition of some amount of
electric current. The quantities of substances produced or consumed by the electrolysis process is dependent
upon the following:
(i) Electric current measured in amperes or amps
(ii) Time measured in seconds
(iii) The number of electrons required to produce or consume 1 mole of the substance

(b) To calculate amps, time, coulombs, faradays and moles of electrons:


Three equations related these quantities:
(i) Amperes × time = Coulombs
(ii) 96,485 coulombs = 1 Faraday
(iii) 1 Faraday = 1 mole of electrons
The through process for interconverting amperes and moles of electrons is:
Amps and time Coulombs Faradays Moles of electrons
Use of these equations are illustrated in the following sections.

(c) To calculate the quantity of substance produced or consumed: To determine the quantity of substance
either produced or consumed during electrolysis, given the time a known current flowed:
(i) Write the balanced half-reactions involved.
(ii) Calculate the number of moles of electrons that were transferred.
(iii) Calculate the number of moles of substance that was produced/consumed at the electrode.
(iv) Convert the moles of substance to desired units of measure.

(d) Determination of standard cell potentials: A cell’s standard state potential is the potential of the cell under
standard state conditions, and it is approximated with concentrations of 1 mole per liter (1 M) and pressures
of 1 atmosphere at 25ºC.
(i) To calculate the standard cell potential for a reaction.
(ii) Write the oxidation and reduction half-reactions for the cell.
º
(iii) Look up the reduction potential, Ereduction, for the reduction half-reaction in a table of reduction potentials.
(iv) Look up the reduction potential for the reverse of the oxidation half-reaction and reverse the sign to
º º
obtain the oxidation potential. For the oxidation half-reaction, Eoxidation = −Ereduction .
(v) Add the potentials of the half-cells to get the overall standard cell potential.
º º º
Ecell −Ereduction
= + Eoxidation

(e) For determining non-standard state cell potentials: To determine the cell potential when the conditions
are other than standard state (concentrations not 1 molar and/or pressures not 1 atmosphere):
(i) Determine the standard state cell potential.
(ii) Determine the new cell potential resulting from the changed conditions.
(iii) Determine Q, the reaction quotient.
Chem i str y | 17.27

(iv) Determine n, the number of electrons transferred in the reaction “n”.


(v) Determine Ecell, the cell potential at the non-standard state conditions using the Nernst equation.
º
E=
cell Ecell − (RT / nF)lnQ
º
Ecell = cell potential at non-standard state conditions; Ecell = standard state cell potential
R = constant (8.31 J/mole K); T = absolute temperature (Kelvin scale)
F = Faraday’s constant (96,485 C/mole e− )
n = Number of moles of electrons transferred in the balanced equation for the reaction occurring in the cell;
[C]c [D]d
Q = Reaction quotient for the reaction. aA + bB → cC + dD, Q =
[A]a [B]b
If the temperature of the cell remains at 25ºC, the equation simplifies to:
º º
E=
cell Ecell − (0.0257 / n)lnQ or in terms of log10 ; E=
cell Ecell − (0.0592 / n)logQ

POINTS TO REMEMBER

S.No. Description
1 Electrolyte Any substance which dissolves in water to form a solution that will conduct an electric
current (ionic substances).Electrolytes may be classified as strong (NaCl, HCl, NaOH) or
weak (NH4OH, CH3COOH, HF). Solutions that do not conduct electricity at all are called
non-electrolytes.
2 Strong and Weak Strong electrolyte - Solutions in which the substance dissolved (solute) is present
Electrolytes entirely as ions.
Weak electrolyte - A solute that yields a relatively low concentration of ions in solution.
3 Dissociation The separation of ions that occurs when an ionic substance dissolves: CaCl2(s) + H2O →
Ca+2(aq) + 2Cl-(aq).
4 Electrochemical Cells A system of electrodes and electrolytes in which a spontaneous or
non-spontaneous redox reaction occurs.
5 Components of a. Electrode: An electrical conductor (metal strip) used to establish contact with a non-
Electrochemical cells metallic part of the circuit (usually an electrolyte).
b. Anode: The electrode at which oxidation occurs.
c. Cathode: The electrode at which reduction occurs.
d. Electrolyte: A liquid, paste, or gel that serves to conduct charge by moving ions in
the cell.
e. Half-cell: A single electrode immersed in a solution of its ions.
f. Salt bridge: A device (porous disk or bridge i.e. U-tube containing inert electrolytic
solution, KCl, NH4NO3, etc.) placed between the cells which maintains electrical
neutrality by allowing ions to migrate between the cells.
g. External circuit: The part of the cell where charge is conducted as a current of moving
electrons.
h. Standard Electrode Reduction Potential E : The measurement, in volts, of the
tendency for a half reaction to occur as a reduction half reaction.
1 7 . 2 8 | Electrochemistr y

S.No. Description
6 Voltaic/Galvanic Cells Redox reactions are spontaneous and chemical energy is transformed into electrical
energy. The cell potential E is positive and the anode is the negative electrode. i.e.
batteries
Zn(s) Zn+2(1M) Cu+2(1M) Cu(s)
anode  cathode
7 Electrolytic Cells Cell in which an external electric current is required to drive a non-spontaneous redox
reaction. The cell potential (Eo) is negative and the anode is the positive electrode. i.e.
electrolysis, electroplating, etc.
Cu(s) Cu+2(1M) Cu+2(1M) Cu(s)
anode  cathode
8 Quick Comparison of Type of redox reaction cell Galvanic/Voltaic Electrolytic
Electrolytic Cells potential (Eºcell) Electron
Spontaneous Non-spontaneous
flow
(Eºcell is positive) (Eºcell is negative)
Site of oxidation
Creates one Requires one
Site of reduction
Anode Anode
Positive electrode
Cathode Cathode
Negative electrode
Cathode Anode
Flow of electrons
Anode Cathode
Anode to cathode Anode to cathode
(negative to positive) (positive to negative)
Batteries Electrolysis, electroplating

9 Faraday’s First law of The amount of electrolyte discharged at an electrode is directly proportional to the
Electrolysis quantity of electricity passed:
W ∝ Q where, = I. t
I = Current strength in ampere
T = time in seconds
⇒ W = ZQ = Zit
Z is a constant called electrochemical equivalence (ECE)
10 Electrochemical It is the amount of an electrolyte discharged on passing one coulomb of electricity.
Equivalent
11 Faraday’s Constant It is the charge possessed by 1.0 mole of electrons and it is equal to 96500 coulombs
(approx.). In terms of faraday’s constant the number of gram equivalent of electrolyte
discharged at an electrode is equal to the number of faraday’s passed.

 Q 
⇒ W=E   where, E = Equivalent weight
 96500 

12 Faraday’s Second Law Second Law: If same quantity of electricity is passed through different cells connected
in series, same number of gram equivalent of electrolytes are discharged at each of the
electrodes:
W1 E1
⇒ =
W2 E2

Where, W1 and W2 are the weights of electrolytes discharged at two different electrodes
in two different cells connected in series and E1 and E2 are their respective equivalent
weights.
Chem i str y | 17.29

S.No. Description
13 Nernst Equation
 2.303 RT [Reduced form]
Ehalf–cell = E half–cell – log
nF [Oxidised form]

At 298 K, the Nernst equation can be written as,

Ehalf–cell = E half–cell – 0.0591log [Reduced form]


n [Oxidised form]

Solved Examples

JEE Main/Boards Pt(H2 ) H+


Ag+ Ag Eo
cell = 0.7991 V
1bar =
a 1=
a 1
Example 1: Zn and iron can replace Cu in a solution but
Pt and Au cannot. Why?
Sol: Since platinum electrode has zero reduction
Sol: Both Zn and iron have more EOP than Cu, whereas potential the standard electrode potential will be equal
Pt and Au have less EOP to the standard electrode potential of the cell.
Ecell = EOP 
+ ERP
Example 2: Which of the following metals cannot H/H+ Ag+ /Ag
be obtained by the electrolysis of their aqueous salt  
solution and why? Or 0.7991=0 + ERP ∴ ERP = 0.7991 V
Ag+ /Ag Ag+ /Ag
Al, Na, Cu, Ag.
Example 5: Standard reduction potential of the Ag+/Ag
Sol: Al and Na cannot be obtained because they have
electrode at 298 K is 0.799 V. Given that for AgI, Ksp = 8.7
higher EOP than H and thus, reduction of Cu2+ and Ag+
× 10–17, evaluate the potential of the Ag+/Ag electrode in
will give Cu and Ag.
a saturated solution of AgI. Also calculate the standard
reduction potential of I– / AgI/Ag electrode.
Example 3: Calculate the no. of electron lost or gained
during electrolysis of 2 g Cl– from NaCl (aq) to give Cl2 Sol: Here solubility product is given from this calculate
at anode. the concentration of silver ions. Now substituting this
Sol: First calculate the equivalent of Cl- used during the value in Nernst equation determine E +
Ag /Ag
reaction and on multiplying it with avogadro number
will give us the no of electron lost during the reaction As we have found out E , EI/AgI/Ag can find out by
Ag+ /Ag
∵ 2Cl– → Cl2 + 2e⁻
using the value of solubility product.
Eq. of Cl– used = 2/35.5
∵ 1 eq. of an element involves 1 faraday charge or N E = E +(0.059/1) log [Ag+]  … (i)
Ag+ /Ag Ag+ /Ag
electrons
∴ (2/35.5) eq. of an element involves Also, K sp = [Ag+ ][I – ]
AgI
N× 2 6.023 × 1023 × 2 ∵ [Ag]+ = [I–] (for a saturated solution)
= electrons =
35.5 35.5
= 3.4 × 1022 electrons ∴ [Ag+] = (K sp=) (8.7 × 10 –17 )
AgI

Example 4: Evaluate the E + and E 2+ from the = 9.32 × 10–19 … (ii)


Ag /Ag Zn /Zn
given values:
∴ By Eq. (i),
1 7 . 3 0 | Electrochemistr y

E calculate the concentration of hydrogen ion. From the


Ag+ /Ag
= 0.799 + (0.059/1) log (9.32 × 10–9)
concentration one can easily find out the pH as using
the following expression
= 0.799 – 0.474 = 0.32 V
Also, EI/AgI/Ag = E + (0.059/1) log K sp pH= –logH+
Ag+ /Ag AgI
Ni → Ni2+ + 2e⁻; EOP = 0.236V
= 0.799 + (0.059/1) log [8.7 × 10 ] –17


2H+ + 2e⁻ → H2; ERP =0
= 0.799 – 0.948 = – 0.149 V
∴ ECell = EOP
 
+ ERP = 0.236 + 0.0 = 0.236 V
Ni H
Example 6: The reduction potential diagram for Cu in
acid solution is: 0.059 [H+ ]2
∴ Ecell = Ecell + log10
2 [Ni2+ ]
2+ +0.15 V + +0.50 V
Cu Cu Cu 0.059
0 = 0.236 + log10 [H+]2
o 2
E = X volt
or –logH+ = 4

∴ pH = 4
Calculate X. Does Cu+ disproportionate in solution?

Example 8: A current of 3 ampere was passed for 2


Sol: Given;
hour through a solution of CuSO4.3g of Cu2+ ions were
Cu2+ + e⁻ → Cu+ ; E1 = 0.15V discharged at cathode. Calculate current efficiency. (At
wt. of Cu = 63.5)
−∆G1 = 1 × E1 × F  ... (i)
Sol: Find out current in ampere and from the calculated
Cu+ + e⁻ → Cu ; E2 = 0.5 V
current passed, determine the current efficiency.
−∆G2 = 1 × E2 × F  … (ii) Current efficiency =
Cu + 2e⁻ → Cu ;
2+
E3 =? Current passed actually
× 100
−∆G3 = 2 × E3 × F  Total current passed experimentally
… (iii)
∵ wCu = E. i. t / 96500
Adding Eqs. (i) and (ii)

Cu2+ + 2e⁻ → Cu ; – ( ∆G1 + ∆G2 )  ... (iv) 63.5 × i × 2 × 60 × 60


∴3= or i = 1.266 ampere
2 × 96500
i.e., −∆G3 = −( ∆G1 + ∆G2 )
Current efficiency
2× E3 × F = [1 × 0.15 × F + 1 × 0.5 × F] Current passed actually
= × 100
∴ E3 = 0.325V Total current passed experimentally

= (1.266/3)×100 = 42.2%
Example 7: The standard oxidation potential of Ni/Ni 2+

electrode is 0.236 V. If this is combined with a hydrogen Example 9: An ammeter and copper voltmeter are
electrode in acid solution, at what pH of the solution connected in series in an electric circuit through which a
will the measured e.m.f. be zero at 25ºC? constant direct current flows. The ammeter shows 0.525
(Assume [Ni2+] = 1 M) ampere. If 0.6354 g of Cu is deposited in one hour, what
is percentage error of ammeter? (At. Wt. of Cu = 63.54)
Sol: Here we are provided with standard oxidation
potential of nickel electrode and we all know that Sol: In order to find out the error first from the given
reduction potential of hydrogen electrode is always Weight, time and other term calculate the actual
zero so first the standard electrode potential of the cell current flow. The difference between the two will give
is equal the oxidation potential of nickel electrode. Here us the error shown by the ammeter. As we are asked
it we are also with e.m.f of the cell (zero) so substitute to calculate percentage error divide the error by actual
the values of different term in Nernst equation and current flow
Chem i str y | 17.31

Current flown = 0.525 ampere as shown by ammeter ∴ wNaOH = {710[71/2]} × 40 = 800 g


Actual current flown
And wH = {710/[71/2]} × 1 = 20g
w 2
(I) = × 96500
E×t ∴ VH = wRT / mp
2
0.6354 × 96500 = [20 × 0.0821 × 300] / [2 × 1] = 246.3 liter
(∵ t = 60 × 60 sec)
(63.54 / 2) × 60 × 60
∴ i = 0.536 ampere Example 2: 50 mL 0.1 M CuSO4 solution is electrolyzed
Thus, error in (I) = 0.536 – 0.525 = 0.011 using Pt electrodes with a current of 0.965 ampere for
a period of 1 minute. Assuming that volume of solution
0.011 × 100
∴ % error in ammeter = = 2.05% does not change during electrolysis, calculate [Cu2+],
0.536 [H+] and [SO24− ] after electrolysis. What will be the
concentration of each species, if current is passed using
Example 10: 3 ampere current was passed through Cu electrodes?
an aqueous solution of unknown salt of Pd for 1 hour
2.977 g of Pdn+ was deposited at cathode. Find n. (At. Sol: Meq. Of CuSO4 in solution = Meq. Of Cu2+
Wt. of Pd = 106.4)
= 50 × 0.1 × 2 = 10 (∵ Meq. Of N× V in mL)
Sol: For the reduction: Pd n+
+ ne → Pd
- The redox changes are:

Eq. of Pd or (w/E) = (I × t)/96500 Cu2+ + 2e → Cu (at cathode)

2.977 3 × 1 × 60 × 60 2H2O → 4H + O2 + 4e
+
(at anode)
or =
106.4 / n 96500 ∵ w/E = i.t/96500
And Eq. of Cu2+ lost = Equivalent of H+ formed
∴ n = 4 (an integer).
i. t. 0.965 × 1 × 60
= = = 6 × 10–4
96500 96500
JEE Advanced/Boards Or Meq. Of Cu2+ lost = 0.6

Example 1: Calculate the quantity of electricity that will ∴ Meq. of Cu2+ or Meq. of CuSO4 left in solution
be required to liberate 710g of Cl2 gas by electrolyzing a = 10 – 0.6 = 9.4
conc. solution of NaCl. What weight of NaOH and what
N Meq. of Cu2+
volume of H2 at 27ºC and 1 atm pressure is obtained ∴[Cu2+]= Cu2 +
=
during this process? 2 2 × Volume of solution (in mL)
9.4
= = 0.094 M
Sol: Quantity of electricity can be calculated using 2 × 50
Simple relationship [H+]= (N / 1) = (0.6/50) = 0.012 M
H+
E×i× t [SO24− ] = 0.1M
W= (∵ Q = i × t)
96500
(∵ SO2–
4 is not involved in redox changes)
Similarly calculate wNaOH and wH from the value of
2
wH volume can be calculated using ideal gas equation. Also, if Cu electrodes are used, Cu2+ ions are discharged
2
at cathode and Cu2+ are formed at anode and thus
∵ 2Cl– → Cl2 + 2e; 2H+ + 2e → H2 no changes in molarity of CuSO4 solution. Anode
Cu → Cu2+ + 2e-
∴ Eq. wt. of Cl2 = M. Wt./2
Cathode Cu2+ + 2e → Cu
E×i× t E×Q
Now w = = (∵ Q = i × t)
96500 96500 Example 3: Two students use same stock solution
w 710 ZnSO4 and a solution of CuSO4. The e.m.f., of one cell is
∴ Q= × 96500 = × 96500
E 71 / 2 0.03 V higher than the other. The conc. of CuSO4 in the
cell with higher e.m.f. value is 0.5 M. Find out the conc.
= 1.93 × 106 coulomb
of CuSO4 in the other cell.
Also Eq. of NaOH formed  2.303RT 
= Eq. of H+ discharged= Eq. of Cl2 formed  = 0.06 
 F 
1 7 . 3 2 | Electrochemistr y

Sol: First write down the half-cell reaction and nernst 1 1


equation for both the half cell. By comparing both the ∴ pH1 = pK a − log C
2 1 2
equation, value of C2 can be calculated.
1 1
pH2
Similarly,= pK a − log C
Cell I: Zn | ZnSO4 || CuSO4 | Cu 2 2 2
0.060 [Cu2+ ] (∵ C are same) By Eqs. (i) and (ii), Ecell
(C2 = 0.5 M); Ecell = Ecell + log
2 [Zn2+ ] 1 1 
0.060 [C ] = EOP 
+ ERP + 0.059  pK a − pK a 
= Eocell + log 2  ... (i) H/H+ H/H+ 2 2 2 1

2 [C1 ]
= 0 + 0.059 × 5[5 – 3]/2 = + 0.059 volt
Cell II: Zn | ZnSO 4 || CuSO 4 | Cu
C1 C'2
Example 5: Calculate the minimum weight of NaOH
required to added in RHS to consume all the H+ present
' 0.06 [C'2 ]
E=
cell Ecell + log  ... (ii) in RHS of cell of e.m.f. +0.701 V at 25ºC before its use.
2 [C1 ] Also report the e.m.f. of cell after addition of NaOH.
0.06 [C ]
By Eqs. (i) and (ii) Ecell − E'cell = log 2 Zn Zn HCl Pt(H2 ) E
2+
2 [C' ] 2 + = +0.760V
2
0.1M 1 litre 1 atm Zn|Zn
0.06 0.5
0.03 = log or C'2 = 0.05M
2 C' 2
Sol: For given cell ∵ EOP > EOP
Zn/Zn2 + H/H+
Example 4: Two weak acid solutions HA1 and HA2 each
∴ Redox changes will be: Zn → Zn2+ + 2e
with the same concentration and having pKa values 3
and 5 are placed in contact with hydrogen electrode
2H+ + 2e →
= H2, Ecell EOP + ERP
(1 atm at 25ºC)) and are interconnected through a salt Zn/Zn2 + H+ /H

bridge. Find e.m.f. of cell.


0.059
= EOP − log10 [Zn2+ ]
Sol: First write down the nernst equation for two half- Zn/Zn2 + 2
cell as both solutions are acid we have to find out the
concentration of hydrogen ion we can express it in  0.059 [H+ ]2
+ ERP + log10
the form of pH. As an acid undergoes dissociation, by H+ /H 2 (PH )
substituting pH in terms of degree of dissociation and 2

concentration in nernst equation, e.m.f of the cell can 0.059 [H+ ]2


be easily determined. E=
cell Ecell + log
2 [Zn2+ ](PH )
2
Consider the cell
Pt H2(1 atm) | HA2 || HA1 | (H2 ) (1atm) Pt
0.059 [H+ ]2
0.701 = 0.760 + log10
2 [Zn2+ ](PH )
2
0.059
At L.H.S.=
E + EOP − log[H+ ]2
H/H H/H+ 1 0.059 [H+ ]2
0.701 = 0.760 + log10
∵ –log H+ = pH 2 [0.1] × 1
[H+] = 0.0316 mole liter–1
E + EOP
∴ = + 0.059(pH)2 ….. (i)
H/H H/H+  Since, H+ must be used by NaOH
0.059
At R.H.S.=
E + 
ERP + log[H+ ]1 ∴ Meq. of NaOH = Meq. of [H+]
H /H H+ /H 1
(w/40) × 1000 = 0.0316 × 1000 (∵ V = 1 liter)

E +
= ERP − 0.059(pH)1  ... (ii)
H /H H+ /H ∴ w = 1.265 g
For acid HA1 HA1 

 + −
 H + Al After addition of NaOH to cathode solution [H+]
becomes 10–7 since both acid and base are neutralized
[H+] = C α = Ka . C completely. Thus, new e.m.f. of cell,
Chem i str y | 17.33

 0.059 [H+ ]2 Example 7: Ecell values for


E=
cell E=
cell log10
2 (0.1) Fe → Fe2+ + 2e- and Fe → Fe3+ + 3e- are 0.440 and 0.036
0.059 (10−7 )2 V respectively:
= 0.760 + log10 Ecell = 0.3765 V
2 0.1 (a) Design and point out the number of cells showing
the overall reaction Fe + 2Fe3+→ 3Fe2+
Example 6: For the galvanic cell: (b) Also calculate Ecell and ∆G values for each cell. Also
comment on the result.
AgCl(s) KCl KBr AgBr(s)
Ag Ag
0.2 M 0.001 M Sol: Standard free energy can be calculated using the
equation
Calculate the e.m.f generated and assign correct
polarity to each electrode for a spontaneous process ∆G =−n EF
after taking an account of cell reaction at 25ºC.
Fe → Fe2+ + 2e- ; – ∆G1 = +0.440 × 2 × F
−10 −13
[Give, K sp = 2.8 × 10 , K sp = 3.3 × 10 ]
AgCl AgBr Fe3+ + 3e- → Fe ; – ∆G2 = – 0.036 × 3 × F
Sol: Write down the half-cell equation for both the cell.
∴ Fe3+ + e- → Fe2+ ; – ∆G3 = – ( ∆G2 + ∆G1 )
Here we are provided with solubility product of the two
solution. From the value of solubility product first find −n EF = [2 × 0.440 – 3 × 0.036] × F (n = 1)
out the concentration of silver ion in each solution. Now
substitute this value in Nernst equation and calculate Eº = + 0.772 V
the e.m.f.
Cell no. I: Fe | Fe+2 || Fe3+ | Fe
Ecell EOP
= + ERP ; L.H.S. R.H.S. 3Fe → 3Fe2+ + 6e⁻
Ag Ag

2Fe3+ + 6e⁻ → 2Fe


0.059
= EOP – log [Ag+]L.H.S.
Ag 1 Redox Fe + 2Fe3+ → 3Fe2+

0.059 Ecell = EOP 


+ERP =0.440–0.036=0.404V

+ ERP + log[Ag+ ]R.H.S. Fe|Fe2 + Fe3+ |Fe
Ag 1
0.059 [Ag+ ]R.H.S. Also, ∆G = –6 ×0.404 × F = – 2.424 F
Ecell = log  …. (i)
1 [Ag+ ] Cell no. II: Fe | Fe2+ || Fe3+, Fe2+ | Pt
L.H.S.

For L.H.S. Ksp of AgCl = 2.8 × 10–10 Fe → Fe2+ + 2e-

or [Ag+][Cl–] = 2.8 × 10–10 2Fe3+ + 2e- → 2Fe2+


2.8 × 10−10 2.8 × 10−10 Redox Fe + 2Fe3+ → 3Fe2+
or [Ag+] = = =1.4×10–9 M
[Cl ] − 0.2

= Ecell 
EOP 
+ ERP
For R.H.S. Ksp of AgBr = 3.3 × 10–13 Fe/Fe2+ Fe3+ /Fe2 +

or [Ag+][Br–] = 3.3 × 10–13


Example 8: Oxidizing power of Cr2O27− is more in acidic
or [Ag+] = (3.3 × 10–13) / 0.001 = 3.3 × 10–10 M medium than in alkaline medium.
∴ By Eq. (i), 
Sol: ERP o
in acidic medium is more than ERP in alkaline
0.059 3.3 × 10−10 medium.
Ecell = log = –0.037V
1 1.4 × 10−9 Acid: Cr2O27− + 14H+ + 6e⁻ → 2Cr3+ + 7H2O ;
o
Thus, to get cell reaction (i.e. Ecell, Ecell = +ve) polarity
ERP = +1.33V
of cell
Alkali: Cr2O27− + H2O → 2CrO24− + 2H+

CrO24− + 4H2O + 3e− → Cr(OH)3 + 5OH− ; ERP = −0.13V
1 7 . 3 4 | Electrochemistr y

Example 9: Color of KI solution containing starch turns º


After calculating Ecell e.m.f of the cell can be easily
blue when Cl2 water is added. Explain. calculated using following equation
Sol: Chlorine placed below iodine in electrochemical 0.0591 [Products]
º
series having lesser EOP than iodine and thus, shows E=
cell Ecell − log
n [Reactants]
reduction whereas I– undergoes oxidation. The I2 so
formed gets absorbed in starch to give blue color. ECell ECathode
= 
− EAnode = 0.80–(–2.37)= 3.17 volt
2I– → I2 + 2e- Cell reaction, Mg + 2Ag+ → 2Ag + Mg2+
Cl2 + 2e- → 2Cl– 0.0591 Mg2+

E=
cell Ecell − log
Cl2 + 2I– → I2 + 2Cl– n [Ag+ ]2
0.0591 0.2
= 3.17 – log
Example 10: Calculate the e.m.f of the cell. 2 [1 × 10−3 ]2
Mg(s) | Mg2+(0.2 M) || Ag+ (1 × 10–3) | Ag = 3.17 – 0.1566 = 3.0134 volt
Eo = + 0.8 volt, Eo = – 2.37 volt When Mg2+ = 0.1 M
Ag+ |Ag Mg2 + /Mg
0.0591 0.1
What will be the effect on e.m.f if concentration of Mg2+ E=
cell Ecell − log
ion is decreased to 0.1 M? 2 (1 × 10−3 )2
= (3.17 – 0.1477) volt= 3.0223 volt
Sol: First calculate the standard potential for reaction
º º º
which is calculated =as Ecell Eoxidation − Ereduction

JEE Main/Boards

Exercise 1 Q.4 In the button cells widely used in watches and other
devices the following reaction takes place:
Q.1 Depict the galvanic cell in which the reaction Zn(s) Zn(s) +Ag2O(s) + H2O(l) → Zn2+(aq) + 2Ag(s) + 2OH–(aq)
+ 2Ag+ (aq) → Zn2+ (aq) + 2Ag(s) takes place. Further
show: Determine ∆rG’ and E’ for the reaction.

(i) Which of the electrode is negatively charged?


Q.5 The conductivity of 0.20 M solution of KCl at 298 K
(ii) Individual reaction at each electrode. is 0.0248 S cm–1. Calculate it molar conductivity.

Q.2 Electrolytic conductivity of 0.20 mole L–1 solution of Q.6 How much electricity in terms of faraday is required
KCl at 298 K is 2.48 × 10–2 ohm–1cm–1. Calculate its molar to produce
conductivity.
(i) 20.0 g of Ca from molten CaCl2?

Q.3 Write the Nernst equation and e.m.f of the following (ii) 40.0 g of Al from molten Al2O3?
cells at 298K:
Q.7 How much electricity is required in coulomb for the
(i) Mg(s)|Mg2+ (0.001M)||Cu2+(0.0001 M)|Cu(s)
oxidation of:
(ii) Fe(s)|Fe2+(0.001M)||H+(1M)|H2(g)(1bar)|Pt(s) (i) 1 mole of H2O to O2?
(iii) Sn(s)|Sn (0.050 M)||H (0.020 M)|H2(g)(1 bar)|Pt(s)
2+ +
(ii) 1 mole of FeO to Fe2O3?
(iv) Pt(s)|Br2(l)|Br–(0.010 M) || H+(0.030 M)|H2(g)
(1 bar)|Pt(s)
Chem i str y | 17.35

Q.8 The resistance of a conductivity cell containing Q.16 The standard electrode potentials of Ag+|Ag,
0.001 M KCl solution at 298 K is 1500 Ω. What is the cell Cl2|Cl– and O3|O2 electrodes are +0.80V, +1.36V and
constant if conductivity of 0.001 M KCl solution at 298 +2.07 V, respectively. Using these information, answer
K is 0.146 × 10–3 S cm–1? the following:
(i) Which of the following is strongest oxidizing agent:
Q.9 Conductivity of 0.00241 M acetic acid is Ag+, Cl2, and O3?
7.896 × 10–5 S cm–1. Calculate its molar conductivity.
o
If Λ ∞ for acetic acid is 390.5 S cm2 mole–1, what is its (ii) Which of the following is the strongest oxidizing
dissociation constant? agent: Ag, Ag+, O3, and O2?

(iii) Which of the following is the strongest reducing


Q.10 A solution of Ni(NO3)2 is electrolyzed between agent: Ag, Cl–, and O2
platinum electrodes using a current of 5 amperes for
20 minutes. What mass of Ni deposited at the cathode? (iv) Which of the following is the strongest reducing
agent: Ag, Ag+, O3, and O2
Q.11 The conductivity of sodium chloride at 298 K has
been determined at different concentrations and the Q.17 The following reactions represent the reduction of
results are given below: IO– ion into I–ion in acidic and basic medium.

Concentration M IO3− + 6H+ + 6e− → I – + 3H2O ; Eº = + 0.907 V


0.001 0.010 0.020 0.050 0.100 IO3− + 3H2O + 6e− → I – + 6OH− ; Eº = + 0.260 V
102 × k/S m–1 [Acid Medium]

1.237 11.85 23.15 55.53 106.74
Calculate Λ ∞ for all concentrations and draw a plot Q.18 Can Fe3+ oxidize Br– to Br under standard
o
between Λ ∞ and S. Find the values of Λ ∞ conditions?
Given: E = + 0.77 V, E = + 1.09 V
Fe3+ |Fe2 + Br2 | Br −
Q.12 Three electrolytic cells A, B, C containing solution
of ZnSO4, AgNO3 and CuSO4, respectively are connected
in series. A steady current of 1.5 amperes was passed Q.19 Calculate the electrode potential of the following
through them until 1.45 g of silver deposited at the electrodes at 25ºC:
cathode of cell B. How long did the current flow? What (i) Cu2+ (0.001 M) | Cu ; E = + 0.34 V
mass of copper and zinc were deposited? Cu3+ | Cu

(ii) Zn | Zn2+ (0.02 M) ; E = –0.76 V


Zn2 + | Zn
Q.13 Using the standard electrode potentials of
electrochemical series, predict if the reaction between (iii) AgCl (stand. sol) | Ag ; E + = +0.80 V, Ksp of
Ag | Ag
the following is feasible: AgCl = 2 × 10–10.

(i) Fe3+(aq) and I– (aq) (ii) Ag+(aq) and Cu(s) (iv) Pt | Cl2 (0.5 atm) | Cl– (0.02 M); E

= 1.36 V
Cl 2| Cl–
(iii) Fe (aq) an Br (aq)
3+ –
(iv) Ag(s) and Fe (aq)
3+

(v) HCl (pH = 4.4) | H2(2 atm) | Pt


(iv) Br2(aq) and Fe2+(aq).

Q.20 Calculate the reduction potential at 25ºC for Fe3+/


Q.14 The standard electrode potentials of Cu2+|Cu+ an Fe2+ electrode if the concentration of Fe2+ ion is five
Cu+|Cu electrodes are +0.18 V and +0.50 V, respectively. times that of Fe3+ ion.
Calculate the standard potential of Cu2+| Cu electrode.
E = 0.77 volt.
Fe3+ | Fe2 +

Q.15 Arrange Zn, Pb and Al in the increasing order of


their reducing power under standard conditions. Q.21 Calculate electrode potential for the half-cell
Pt|H2 (1 atm) | 0.357M – CH3COOH. Ka for CH3COOH =
Given: E = –0.76 V, = –0.13 V, E = –1.66 V 1.74 × 10–5.
Zn2 + |Zn Al2 + |Al]

Q.22 The standard reduction potential for Cu2+| Cu is +


1 7 . 3 6 | Electrochemistr y

0.34 V. Calculate the reduction potential at pH = 13 for Q.2 One mole of electron passes through each of the
the above couple. Ksp of Cu(OH)2 is 1.0 × 10–10. solution of AgNO3, CuSO4 and AlCl3 when Ag, Cu and
Al are deposited at cathode. The molar ratio of Ag, Cu
Q.23 Calculate and Al deposited are:
(A) 1: 1: 1 (B) 6: 3: 2 (C) 6: 3: 1 (D) 1: 3: 6
(i) E ; E = 0.80 V, Ksp of AgCl = 2 × 10–10
Cl + | AgCl | Ag Ag+ | Ag

(ii) E ; E = + 0.80 V, Q.3 Salts of A (atomic weight = 7), B (atomic weight =


Cl– (0.004M)AgCl | Ag Ag+ | Ag
27) and C (atomic weight = 48) were electrolyzed under
Ksp of AgCl = 2 × 10–10 identical conditions using the same quantity of electricity.
It was found that when 2.1 g of A was deposited, the
weights of B and C deposited were 2.7 and 7.2 g. The
Q.24 Calculate the e.m.f of the cell in which the
valencies of A, B and C respectively are:
following reaction takes place:
Ni(s) + 2Ag+(0.002 M) → Ni2+ (0.160 M) + 2Ag(s) (A) 3, 1 and 2 (B) 1, 3 and 2

Given that E cell = 1.05 V (C) 3, 1 and 3 (D) 2, 3 and 2

Q.4 The density of Cu is 8.94 g cm–3. The quantity of


Q.25 The molar conductivity of 0.025 mole methanoic
electricity needed to plate an area 10 cm × 10 cm to
acid is 46.1 S cm2 mole–1. Calculate its degree of
a thickness of 10–2 cm using CuSO4 solution would be
dissociation and dissociation constant.
Given Λ0 (H+) = 349.6 S cm2 mol/L and (A) 13586 C (B) 27172 C
λ°(HCOO⁻) = 54.6 S cm mol/L. 2
(C) 40758 (D) 20348 C

Q.26 Suggest a list of metals that are extracted Q.5 During electrolysis of an aqueous solution of
electrolytically. sodium sulphate, 2.4L of oxygen at STP was liberated
at anode. The volume of hydrogen at STP, liberated at
Q.27 Represent the cell in which following reaction cathode would be
takes place: (A) 1.2 L (B) 2.4 L (C) 2.6 L (D) 4.8 L
Mg(s) + 2Ag+ (0.0001 M) → Mg2+(0.130 M) + 2Ag(s)
calculate its Ecell if Eºcell = 3.17V. Q.6 When an aqueous solution of lithium chloride is
electrolyzed using graphite electrodes
Q.28 Calculate the reduction potential of the following (A) Cl2 is liberated at the anode
electrodes: (B) Li is deposited at the cathode
(a) Pt2 H2 (4 atm) | H2SO4 (0.01 M) (C) As the current flows, pH of the solution around the
cathode remains constant
(b) Pt2 H2 (1 atm) | HCl (0.2 M)
(D) As the current flows, pH of the solution around the
(c) Calculate the potential of hydrogen electrode in cathode decreases.
contact with a solution whose
(i) pH = 5 (ii) pOH = 4 Q.7 A standard hydrogen electrons has zero electrode
potential because
Q.29 Calculate the equivalent conductivity of 1M H2SO4 (A) Hydrogen is easier to oxidize
solution, if its conductivity is 26 × 10–2 ohm–1 cm–1.
(B) This electrode potential is assumed to be zero
(C) Hydrogen atom has only one electron
Exercise 2 (D) Hydrogen is the lightest element.
Single Correct Choice Type
Q.8 If the pressure of H2 gas is increased from 1 atm to
Q.1 One gm. metal M+2 was discharged by the passage of 100 atm keeping H+ concentration constant at 1 M, the
1.81 × 1022 electrons. What is the atomic weight of metal? change in reduction potential of hydrogen half-cell at
25ºC will be
(A) 33.35 (B) 133.4 (C) 66.7 (D) 55
(A) 0.059 V (B) 0.59 V (C) 0.0295 V (D) 0.118 V
Chem i str y | 17.37

Q.9 The equilibrium constant for the reaction Sr(s) + Q.16 The dissociation constant of n-butyric acid is
Mg+2 (aq) → Sr+2 (aq) + Mg(s) is 2.69 × 1012 at 25ºC 1.6 × 10–5 and the molar conductivity at infinite dilution
is 380 × 10–4 S m2 mol–1. The specific conductance of
The E for a cell made up of the Sr/Sr+2 and Mg+2/Mg
the 0.01 M acid solution is
half-cells
(A) 1.52 × 10 S m–1 (B) 1.52 × 10–2 S m–1
(A) 0.3667 V (B) 0.7346 V
(C) 1.52 × 10–3 S m–1 (D) None of these
(C) 0.1836 V (D) 0.1349 V

Q.17 Consider the reaction of extraction of gold from


Q.10 A silver wire dipped in 0.1 M HCl solution
its ore
saturated with AgCl develop a potential of –0.25 V. If
Au+2CN–(aq.)+ 1 O2(g)+ 1 H2O→Au (CN)2 + OH–

E = –0.799 V, the Ksp of AgCl in pure water will be:
Ag/ Ag+ 4 2

(A) 2.95 × 10–11 (B) 5.1 × 10–11 Use the following data to calculate ∆Gº for the reaction,
Kf{Au(CN)2} = X
(C) 3.95 × 10–11 (D) 1.95 × 10–11
O2 + 2H2O + 4e– → 4OH– ; E° = + 0.41 volt
Q.11 During electrolysis of an aqueous solution Au3+ + 3e– → Au ; E° = + 1.5 volt
of CuSO4 using copper electrodes, if 2.5 g of Cu is Au + 2e → Au ; E°= + 1.4 volt
3+ – +

deposited at cathode, then at anode


(A) –RT ln X + 1.29 F (B) –RTln X –2.11 F
(A) 890 ml of Cl2 at STP is liberated
(B) 445 ml of O2 at STP is liberated (C) –RTln 1 + 2.11 F (D) –RTln X –1.29 F
X
(C) 2.5 g of copper is deposited
Q.18 Consider the following Galvanic cell
(D) A decrease of 2.5 g of mass takes place
voltmeter
Q.12 The cost at 5 paise / KWH of operating an electric H2(g) Cl2(g)
motor for 8 hours which takes 15 amp at 110 V is KNO3

(A) Rs 66 (B) 66 paise HCl HCl


Pt(s)
(C) 37 paise (D) Rs. 6.60

Anode Cathode
Q.13 The resistance of 0.5 M solution of an electrolyte
in a cell was found to be 50Ω. If the electrodes in the By what value the voltage change when concentration
cell are 2.2 cm apart and have an area of 4.4 cm2 then of ions in anodic and cathodic compartments both
the molar conductivity (in S m2 mole–1) of the solution is increases by factor of 10 at 298 K
(A) 0.2 (B) 0.02 (C) 0.002 (D) None of these
(A) 0.0591 (B) –0.0591 (C) –0.1182 (D) 0

Q.14 Equivalent conductance of 0.1 M HA (weak acid)


Q.19 For the cell
solution is 10 S cm2 equivalent–1 and that at infinite
dilution is 200 S cm2 equivalent–1 Hence pH of HA Pt||H2 (0.4 atm) | H+ (pH=1) || H+ (pH =2) | H2 (0.1 atm) | Pt
solution is
The measured potential at 25ºC is
(A) 1.3 (B) 1.7 (C) 2.3 (D) 3.7
(A) –0.1 V (B) –0.5 (C) –0.041 (D) None of these

Q.15 If x is specific resistance of the electrolyte solution


Q.20 For the fuel cell reaction:
and y is the molarity of the solution, then Λm is given by

y 2H2 (g) + O2 (g) → 2H2O (l); ∆ fH298 (H2O,(l)= –285.5 KJ/mole
1000x
(A) (B) 1000 
y x What is ∆S298 for the given fuel cell reaction?
1000 xy Given O2 (g) + 4H+ (aq) + 4e– → 2H2O (l) E = 1.23 V
(C) (D)
xy 1000
(A) –0.322 J/K (B) –0.635kJ/K
(C) 3.51 kJ/K (D) –0.322 kJ/K
1 7 . 3 8 | Electrochemistr y

Q.21 The standard reduction potentials of Cu2+/Cu Q.6 The highest electrical conductivity of the following
and Cu2+/Cu+ are 0.337 and 0.153 V respectively. The aqueous solution is of (2005)
standard electrode potential of Cu+/Cu half-cell is:
(A) 0.1 M acetic acid
(A) 0.184 V (B) 0.827 V (B) 0.1 M chloroacetic acid
(C) 0.521 V (D) 0.490 V (C) 0.1 M fluoroacetic acid
(D) 0.1 M difluoroacetic acid
Previous Years' Questions
Q.7 The equivalent conductance’s of two strong
electrolytes at infinite dilution in H2O (where ions move
Q.1 When the sample of copper with zinc impurity is to
freely through a solution) at 25ºC are given below: A t
be purified by electrolysis, the appropriate electrodes
25ºC are given below: (2007)
are (2002)
Λ CH = 91.0 S cm2 / equivalent
Cathode Anode 3COONa

(a) Pure zinc Pure copper ΛHCl = 426.2 S cm2/equivalent
What additional information/quantity one needs to
(b) Impure sample Pure copper
calculate Λ° of an aqueous solution of acetic acid
(c) Impure zinc Impure sample
(A) Λ° of NaCl
(d) Pure copper Impure sample
(B) Λ° of CH3COOK
o +
(C) The limiting equivalent conductance of H+ ( Λ H )
Q.2 In the electrolytic cell, flow of electrons is from
 (2003) (D) Λ  of chloroacetic acid (Cl/CH2COOH)

(A) Cathode to anode in solution


 
Q.8 The molar conductivities λNaOAc and λHCl at infinity
(B) Cathode to anode through external supply
dilution in water at 25ºC are 91.0 and 426.2 S cm2/mole
(C) Cathode to anode through internal supply 
respectively. To calculate λHOAc , the additional value
(D) Anode to cathode through internal supply required is (2006)
  
(A) λH (B) λKCl (C) λNaOH (D) λNaCl
Q.3 During the electrolysis of a solution of AgNO3, 9650 2O
coulombs of charge pass through the electroplating
both, the mass of silver deposited in the cathode will Q.9 Resistance of a conductivity cell filled with a solution
be  (2003) of an electrolyte of concentration 0.1 M is 100 Ω. The
conductivity of this solution is 1.29 S m–1. Resistance of
(A) 1.08 g (B) 10.8 g (C) 21.6 g (D) 108 g the same cell when filled with 0.2 M of the same solution
is 520 Ω. The molar conductivity of 0.02 M solution of
Q.4 During the process of electrolytic refining of copper, the electrolyte will be (2006)
some metals present as impurity settle as ‘anode mud.’
(A) 12.4 × 10–4 S m2 mole–1
These are  (2005)
(B) 1240 × 10–4 S m2 mole–1
(A) Sn and Ag (B) Pb and Zn
(C) 1.24 × 10–4 S m2 mol–1
(C) Ag and Au (D) Fe and Ni
(D) 1.24 × 10–4 Sm2 mole–1

Q.5 Conductivity (unit Siemen’s) is directly proportional


Q.10 Saturated solution of KNO3 is used to make ‘salt-
to area of the vessel and the concentration of the
bridge’ because (2002)
solution in it and is inversely proportional to the
length of the vessel then the unit of the constant of (A) Velocity of K+ is greater than that of NO3–
proportionality is (2002) (B) Velocity of NO3– is greater than that of K+
(A) S m mole –1
(B) S m mole
2 –1 (C) Velocities of both K+ and NO3– are nearly the same
(C) S–2 m2 mole (D) S2 m2 mole–2 (D) KNO3 is highly soluble in water
Chem i str y | 17.39

Q.11 The reduction potential of hydrogen half-cell will


3
be negative if (2011) CH3OH( ) + O2 (g) → CO2 (g) + 2H2O( ) at 298K
2
(A) P(H ) = 1 atm and [H+] = 2.0 M standard Gibb’s energies of formation for CH3OH(l),
2
(B) P(H ) = 1 atm and [H+] = 1.0 M H2O(ll) and CO2 (g) are -166.2, -237.2 and 394.4 kJ mol1
2
respectively. If standard enthalpy of combustion of
(C) P(H ) = 2 atm and [H+] = 1.0 M methanol is -726kJ mol-1, efficiency of the fuel cell will
2
(D) P(H ) = 2 atm and [H+] = 2.0 M be (2009)
2
(A) 80 % (B) 87% (C) 90% (D) 97%

Q.12 The cell, Zn|Zn (1M) ||Cu (1M) |Cu ( Ecell = 1.10
2+ 2+

V) was allowed to be completely discharged at 298 K. Q.18 Given:



EFe = 
−0.036V, EFe =
−0.439V .
3+ 2+
The relative concentration of Zn2+ to /Fe /Fe
The value of standard electron potential for the change,
 Zn2+ 
3+
Cu  is (2007) + e − →Fe 2+ (aq) will be: 
2+
 2+ Fe(aq) (2009)
 [Cu ] 
(A) -0.072 V (B) 0.385 V (C) 0.770 V (D) -0.270
(A) Antilog (24.08) (B) 37.3
(C) 1037.3 (D) 9.65 × 104 Q.19 The reduction potential of hydrogen half-cell will
be negative if:  (2011)
Q.13 The standard reduction potential for Fe2+/ Fe and (A) p(H2 ) = 1 atm and [H+ ] =1.0 M
Sn2+/Sn electrodes are –0.44 and –0.14 volt respectively,
For the given cell reaction Fe2+ + Sn → Fe + Sn2+, the (B) p (H2) =2 atm and [H+ ] =1.0 M
standard EMF is (1990) (C) p (H2) = 2 atm and [H+ ] = 2.0 M
(A) +0.30 V (B) –0.58 V (C) +0.58 V (D) –0.30 V (D) p (H2) =1 atm and [H+ ] = 2.0 M

Q.14 Given E −0.036V, E 2+


= −0.439V
=
Fe3+ /Fe Fe /Fe Q.20 The standard reduction potentials for Zn2+/ Zn,
The value of standard electrode potential for the Ni2+/ Ni, and Fe2+/ Fe are –0.76, –0.23 and –0.44 V
change, Fe3(aq) + e− → Fe2+ (aq) will be →
+
respectively. The reaction X + Y2+ X2+ + Y will be
(A) –0.072 V (B) 0.385 V spontaneous when: (2012)
(C) 0.770 V (D) –0.270 V (A) X = Ni, Y = Fe (B) X = Ni, Y = Zn
(C) X = Fe, Y = Zn (D) X = Zn, Y = Ni
Q.15 For the redox reaction:
Zn(s) + Cu2+ (0.1M) → Zn2+ (1M) + Cu(2) taking place in Q.21 Given: E°Cr3+ /Cr3+ =
−0.74V; E° −
MnO /Mn2 +
1.51 V
=
4

a cell, Ecell is 1.10 volt. Ecell for the cell will be
E° 2 − 3+
= 1.33
= V; E° − 1.36 V
Cr2 O7 /Cr Cl/Cl
 RT 
 2.303 = 0.0591   (2003) Based on the data given above, strongest oxidising
 F 
agent will be: (2013)
(A) 2.14 volt (B) 1.80 volt

(C) 1.07 volt (D) 0.82 volt (A) Cl (B) Cr3+ (C) Mn2+ (D) MnO 4

  Q.22 The equivalent conductance of NaCl at


Q.16 Given E
Cr 3 + /Cr
=
−0.72V, EFe 2+
/Fe
=
−0.42V .
concentration C and at infinite dilution are λC and λ∞,
The potential for the cell Cr|Cr3+ (0.1M)||Fe2+(0.01M)|Fe
is (2008) respectively. The correct relationship between λC and λ∞,
is given as (Where the constant B is positive)  (2014)
(A) 0.26 V (B) 0.399 v (C) -0.339 V (D) -0.26 V
(A) λC = λ∞ + (B)C (B) λC = λ∞ - (B)C
Q.17. In a fuel cell methanol is used as fuel and oxygen
(C) λC = λ∞ - (B) C (D) λC = λ∞ + (B) C
gas is used as an oxidizer. The reaction is
1 7 . 4 0 | Electrochemistr y

Q.23 Given below are the half-cell reactions Q.24 Two Faraday of electricity is passed through a
 (2014) solution of CuSO4. The mass of copper deposited at the
cathode is: (at. mass of Cu = 63.5 amu) 
Mn2+ + 2e– → Mn; E° = — 1.18 V  (2015)
(Mn + e → Mn ); E° = + 1.51 V
3+ – 2+
(A) 0 g (B) 63.5 g (C) 2 g (D) 127 g
The E° for 3 Mn → Mn + 2Mn will be
2+ 3+

(A) –2.69 V; the reaction will not occur


(B) –2.69 V; the reaction will occur
(C) –0.33 V; the reaction will not occur
(D) –0.33 V; the reaction will occur

JEE Advanced/Boards

Exercise 1 Q.6 The Edison storage cell is represented as Fe(s) |


FeO(s) | KOH (aq) | Ni2O3(s) |Ni(s) the half-cell reaction are
Q.1 The standard reduction potential values, E° (Bi3+|Bi) Ni2O3(s) + H2O (i) + 2e– l 2NiO(s) + 2OH–,
and E° (Cu2+|Cu) are 0.226 V and 0.344 V respectively.
A mixture of salts of bismuth and copper at unit E° = +0.40 V
concentration each is electrolyzed at 25ºC. To what
FeO(s) + H2O (l) + 2e– l Fe(s) + 2OH–,
value can [Cu2+] be brought down before bismuth starts
to deposit, in electrolysis. E° = –0.87 V

Q.2 The cell Pt, H2 (1 atm) | H+ (pH = x) || Normal calomel (i) What is the cell reaction?
electrode has an EMF of 0.67 V at 25°C. Calculate the pH (ii) What is the cell e.m.f.? How does it depend on the
of the solution. The oxidation potential of the calomel concentration of KOH?
electrode on hydrogen scale is –0.28 V.
(iii) What is the maximum amount of electrical energy
Q.3 Voltage of the cell Pt, H2 (1 atm) |HOCN (1.3 × 10–3 that can be obtained from one mole of Ni2O3?
M) ||Ag+ (0.8 M)| Ag(s) is 0.982 V. Calculate the Ka for
HOCN. Neglect [H+] because of oxidation of H2 (g). Q.7 The standard reduction potential for Cu2+/Cu is
0.34 V. Calculate the reduction potential at pH = 14 for
Ag+ + e → Ag(s) = 0.8 V.
the above couple. Ksp of Cu (OH) 2 is 1 × 10–19.
Q.4 Calculate the potential of an indicator electrode
Q.8 Determine the degree of hydrolysis and hydrolysis
versus the standard hydrogen electrode, which
constant of aniline hydrochloride in M/32 solution of
originally contained 0.1 M MnO4– and 0.8 M H+ and
salt at 298 K from the following cell data at 298 K.
which was treated with 90% of the Fe2+ necessary to
reduce all the MnO4– to Mn+2. Pt|H2 (1 atm) | H+ (1M) ||M/32 C6H5NH3Cl|H2 (1 atm) | Pt;
Ecell = –0.188 V
MnO4– + 8H+ + 5e → Mn2+ + 4H2O,
E° = 1.51 V Q.9 The e.m.f of the cell, Pt|H2(1 atm), | H+ (0.1 M, 30 ml)
|| Ag+(0.8 M)|Ag is 0.9 V. Calculate the e.m.f when 40 ml
Q.5 Calculate the e.m.f of the cell of 0.05 M NaOH is added.

Pt, H2 (1.0 atm) | CH3COOH (0.1 M) || NH3 (aq, 0.01M) | Q.10 The e.m.f of the cell Ag | AgI | KI (0.05 M) || AgNO3
H2 (1.0 atm), Pt (0.05 M) | Ag is 0.788 V. Calculate the solubility product
Ka (CH3COOH) = 1.8 × 10–5, of AgI.
Kb (NH3) = 1.8 × 10–5.
Chem i str y | 17.41

Q.11 Consider the cell Q.20 Estimate the cell potential of a daniel cell having
1 M Zn++ & originally having 1 M Cu++ after sufficient
Ag | AgBr(s) Br– || AgCl(s), Ag | Cl– at 25ºC. The solubility
NH3 has been added to the cathode compartment
product constants of AgBr & AgCl are respectively 5 ×
to make NH3 concentration 2M. Kf for [Cu (NH3)4]2+
10–13 & 1 × 10–10. For what ratio of the concentration of
= 1. × 1012, E° for the reaction,
Br– and Cl– ions would the e.m.f of the cell be zero?
Zn + Cu2+ → Zn2+ + Cu is 1.1 V.
Q.12 The pKsp of Agl is 16.07. If the E° value for Ag+|Ag
is 0.7991 V. Find the E° for the half-cell reaction AgI (s)
Q.21 The normal oxidation potential of Zn referred to
+ e– → Ag + I–
SHE is 0.76 and that of Cu is –0.34 V at 25ºC. When
excess of Zn is added to CuSO4, Zn displaces Cu2+ till
Q.13 For the galvanic cell: Ag AgCl(s) | KCl (0.2 M) || KBr
equilibrium is reached. What is the ratio of Zn2+ to Cu2+
(0.001 M)| AgBr(s) | Ag, Calculate the EMF generated
ions at equilibrium?
and assign correct polarity to each electrode for a
spontaneous process after taking into account the cell
Q.22 Kd for complete dissociation of [Ag(NH3)2]+ into
reaction at 25ºC.
Ag+ and 2NH3 is 6 × 10–8. Calculate E for the following
[Ksp (AgCl)] –2.8 × 10–10; Ksp (AgBr) – 3.3 × 10–13] half reaction; Ag(NH3)2+ + e– → Ag + 2NH3
Ag+ + e– → Ag, Eº = 0.799 V
Q.14 Given, E° = –0.268 V for the Cl–|PbCl2|Pb couple
and –0.126 V for the Pb2+|Pb couple, determine Ksp for Q.23 The overall formation constant for the reaction of
PbCl2 a 25ºC? 6 mole of CN– with cobalt (II) is 1 × 1019. The standard
reduction potential for the reaction
Q.15 Calculate the voltage, E, of the cell at 25ºC [Co(CN]6]3– e– → Co(CN)64– is –0.83 V. Calculate the
Mn(s) | Mn (OH2) (s) |Mn2+ (xM), OH– (1.00 × 10–4M) || formation constant of [Co(CN)6]3– Given Co3++ e– →
Cu2+ (0.675 M) | Cu(s) given that Ksp = 1.9 × 10–13 for Mn Co2+ ; E° =1.82 V
(OH) 2(s) E (Mn2+/Mn) = –1.18 V
Q.24 Calculate E° for the following reactions at 298 K,
Q.16 Calculate the voltage, E, of the cell Ag(NH3)2+ + e– → Ag + 2NH3
Ag(s) | AgIO3(s) | Ag+(x M), HIO3 (0.300 M) || Zn2+
Ag(CN)2– e– → Ag + 2CN–
(0.175 M) | Zn(s) if Ksp = 3.02 × 10–8 for AgIO3(s) and Ka
= 0.162 for HIO3. Given: E – 0.7991 V,
Ag+ |Ag

Q.17 The voltage of the cell Kins [Ag(NH3)2+] = 6.02 × 10–8 and
Pb(s) || PbSO4(s) | NaHSO4 (0.600 M) || Pb2+ (2.50 × Kins[Ag(CN)2–] = 1.995 ×10–19
10–5M) | Pb(s) is E = +0.061 V. Calculate K2 = [H+] [SO42–
]/ [HSO4–], the dissociation constant for HSO4–. Given Q.25 Calculate the equilibrium constant for the reaction:
Pb(s) + SO42–(aq) → PbSO4(s) + 2e– (E° = 0.356) E° (Pb2+/
3Sn(s) + 2Cr2O72– + 28H+ → 3Sn4+ + 4Cr3+ + 14H2O
Pb) = –0.126 V
E° for Sn/Sn2+ = 0.136 V E° for Sn2+/Sn4+ = – 0.154 V
Q.18 The voltage of the cell E° for Cr2O72–/Cr3+ = 1.33 V
Zn(s) | Zn(CN)42–(0.450M), CN–(2.65 × 10–3M) || Zn2+
(3.84 × 10–4M) | Zn(s) is E = + 0.099 V. Calculate the Q.26 Calculate the equilibrium concentration of all ions
constant Kf for in an ideal solution prepared by mixing 25.00 mL of
0.100 M Ti+ with 25.00 mL of 0.200M Co3+.
Zn2+ + 4CN– → Zn(CN)42–, the only Zn2+ + CN⁻
E° (Ti+/Ti3+) =–1.25 V; E° (Co3+/Co2+) = 1.84 V
complexation reaction of importance.
Q.27 Same quantity of electricity is being used to
Q.19 Given the standard reduction potentials Ti+ + e– → liberate iodine (at anode) and a metal x (cathode).
Ti, E° = –0.34 V and Ti3+ + 2e– →Ti+, E° = 1.25 V. Examine The mass of x deposited is 0.617 g and the iodine is
the spontaneity of the reaction, 3Ti+ → 2Ti + Ti3+. Also completely reduced by 46.3 cc of 0.124 M sodium
find E° for this disproportionation. thiosulphate. Find the equivalent mass of x.
1 7 . 4 2 | Electrochemistr y

Q.28 In a fuel cell, H2 & O2 react to produce electricity. battery. Note that the water is produced by the reaction
In the process, H2 gas is oxidized at the anode & O2 at as H2SO4 is used up. Overall reaction is
the cathode. If 67.2 liter of H2 at STP react in 15 minutes,
Pb(s) + PbO2(s) + 2H2SO4 (l) → 2PbSO4(s) + 2H2O (l)
what is the average current produced? If the entire
current is used for electrode deposition of Cu from Cu Q.36 100ml CuSO4 (aq) was electrolyzed using inert
(II) solution, how many grams of Cu will be deposited? electrodes by passing 0.965 A till the pH of the
Anode: H2 + 2OH– → 2H2O + 2 e– resulting solution was 1. The solution after electrolysis
Cathode: O2 + 2H2O + 4e– → 4OH– was neutralized, treated with excess KI and titrated
with 0.04 M Na2S2O3. Volume of Na2S2O3 required was
Q.29 One of the methods of preparation of per 35 ml. Assuming no volume change during electrolysis,
disulphuric acid, H2S2O8, involve electrolytic oxidation calculate:
of H2S O4 at anode (2H2S O4 → H2S2O8 + 2H+ + 2e–) (a) Duration of electrolysis if current efficiency is 80%
with oxygen and hydrogen as by-products. In such (b) Initial concentration (M) of CuSO4.
an electrolysis, 9.722 L of H2 and 2.35 L of O2 were
generated at STP. What is the weight of H2S2O8 formed? Q.37 An external current source giving a current of 0.5
A was joined with Daniel cell and removed after 10 hrs.
Q.30 During the discharge of a lead storage battery Before passing the current the LHE and RHE contained
the density of sulphuric acid fell from 1.294 to 1.139 g. 1L each of 1M Zn2+ and Cu2+ respectively. Find the EMF
mL–1 H2SO4 of density 1.294 g mL–1 is 39% and that of supplied by the Daniel cell after removal of the external
density 1.139 g mL–1 is 20% by weight. The battery holds current source. E of Zn2+/Zn and Cu2+/Cu at 25ºC is
3.5 L of acid and the volume practically remains constant
– 0.76 and + 0.34 V respectively.
during the discharge. Calculate the number of ampere
hours for which the battery must have been used. The
Q.38 Determine at 298 for cell
discharging reactions are:
Pt | Q, QH2, H+ || 1M KCl | Hg2Cl2(s) | Hg (l) | Pt
Pb + SO42– → PbSO4 + 2e– (anode)
(a) Its e.m.f when pH = 5.0
PbO2 + 4H+ + SO42– + 2e– →PbSO4+ 2H2O (cathode)
(b) The pH when Ecell = 0
Q.31 A current of 3 amp was passed for 2 hour through (c) The positive electrode when pH = 7.5
a solution of CuSO4, 3g of Cu2+ ions were deposited as
Given E° RP (RHS) = 0.28, E° RP (LHS) = 0.699
Cu at cathode. Calculate percentage current efficiency
of the process.
Q.39 At 25ºC, ∆Hf (H2O, l) = –56700 cal/mole and
Q.32 An acidic solution of Cu salt containing 0.4 g of
2+ energy of ionization of H2O (l) = 19050 Cal/mol. What
Cu2+ is electrolyzed until all the copper is deposited. The will be the reversible EMF at 25ºC of the cell?
electrolysis is continued for seven more minutes with Pt | H2 (g) (1 atm) | H+ || OH– | O2 (g) (1 atm) | Pt, if at
the volume of solution kept at 100 ml and the current 26ºC the e.m.f increase by 0.001158 V.
at 1.2 amp. Calculate the volume of gases evolved at
NTP during the entire electrolysis. Q.40 Calculate the cell potential of a cell having
reaction: Ag2S + 2e– →2Ag + S2– in a solution buffered
Q.33 In the refining of silver by electrolytic method at pH = 3 and which is also saturated with 0.1 M H2S.
what will be the weight of 100 gm. Ag anode if
For H2S: K1 = 10–8 and K2 = 1.1 × 10–13, Ksp (Ag2S) = 2 ×
5 ampere current is passed for 2 hours? Purity of silver
is 95% by weight. 10–49, E = −0.8 .
Ag+ /Ag

Q.34 Dal Lake has water 8.2 × 1012 liter approximately. Q.41The equivalent conductance of 0.10N solution
A power reactor produces electricity at the rate of of MgCl2 is 97.1 mho cm2 equi–1 at 25ºC. A cell with
1.5 × 106 coulomb per second at an appropriate voltage. electrode that are 1.5 cm2 in surface area and 0.5 cm
How many years would it take to electrolyze the lake? apart is filled with 0.1 N MgCl2 solution. How much
current will flow when potential difference between the
Q.35 A lead storage cell is discharged which causes the
electrodes is 5 volt?
H2SO4 electrolyte to change from a concentration of
34.6% by weight (density 1.261 g ml–1 at 25ºC) to 27% Q.42 A dilute aqueous solution of KCl was placed
by weight. The original volume of electrolyte is one between two electrodes 10 cm apart, across which
liter. Calculate the total charge released at anode of the a potential of 6 volt was applied. How far would the
Chem i str y | 17.43

K+ ion move in 2 hours at 25ºC? Ionic conductance of K+ Q.2 The charge required for the oxidation of one mole
ion at infinite dilution at 25ºC is 73.52 ohm–1 cm2 mole–1? Mn3O4 into MnO42– in presence of alkaline medium is
(A) 5 × 96500C (B) 96500 C
Q.43 When a solution of specific conductance
1.342 ohm–1 meter–1 was placed in a conductivity cell (C) 10 × 96500 C (D) 2 × 96500 C
with parallel electrodes, the resistance was found to be
170.5 ohm. Area of electrodes is 1.86×10–4m2. Calculate Q.3 A solution of sodium sulphate in water is
separation of electrodes. electrolyzed using inert electrodes. The products at the
cathode and anode are respectively.
Q.44 The specific conductance at 25ºC of a saturated
solution of SrSO4 is 1.482 × 10–4 ohm–1 cm–1 while that (A) H2 , O2 (B) O2 , H2 (C) O2, Na (D) None
of water used is 1.5 × 10–6 mho cm–1. Determine at
25ºC the solubility in gm. per liter of SrSO4 in water. Q.4 A hydrogen electrode X was placed in a buffer
Molar ionic conductance of Sr2+ and SO42– ions at solution of sodium acetate and acetic acid in the ratio
infinite dilution are 59.46 and 79.8 ohm–1 cm2 mole–1 a: b and another hydrogen electrode Y was placed in
respectively. [Sr = 87.6, S =32, O = 16] a buffer solution of sodium acetate and acetic acid in
the ratio b: a. If reduction potential values for two cells
Q.45 Calculate the solubility and solubility product are found to be E1 and E2 respectively w.r.t. standard
of Co2 [Fe(CN)6] in water at 25ºC from the following hydrogen electrode, the pKa value of the acid can be
data: Conductivity of a saturated solution of given as
Co2[Fe(CN)6] is 2.06 × 10–6Ω–1cm–1 and that of water
used 4.1 × 10–7Ω–1cm–1.The ionic molar conductivities E1 + E2 E2 − E1 E1 + E2 E1 − E2
(A) (B) (C) − (D)
of Co2+ and Fe(CN)64– are 86.0 Ω–1cm2 mole–1 and 0.118 0.118 0.118 0.118
444.0 Ω–1cm–1 mole–1.
Q.5 The conductivity of a saturated solution of Ag3PO4
Q.46 A sample of water from a large swimming pool has
is 9 × 10–6 S m–1 and its equivalent conductivity is
a resistance of 9200 Ω at 25ºC when placed in a certain
1.50 × 10–4 S m–2 equivalent–1. The Ksp of Ag3PO4 is:
conductance cell. When filled with 0.02 M KCl solution,
the cell has a resistance of 85 Ω at 25ºC. 500 gm. of (A) 4.32 × 10–18 (B) 1.8 × 10–9
NaCl were dissolved in the pool, which was thoroughly (C) 8.64 × 10–13 (D) None of these
stirred. A sample of this solution gave a resistance of
7600 Ω. Calculate the volume of water in the pool.
Q.6 Equal volumes of 0.015 M CH3COOH & 0.015 M
Given: Molar conductance of NaCl at that concentration
NaOH are mixed together. What would be molar
is 126.5 Ω–1cm–1mol–1 and molar conductivity of KCl at
conductivity of mixture if conductivity of CH3COONa is
0.02 M is 138 Ω–1 cm2 mole–1.
6.3 × 10–4 S cm–1?
(A) 8.4 S cm2 mole–1 (B) 84 S cm2 mole–1
Exercise 2 (C) 4.2 S cm2 mole–1 (D) 42 S cm2 mole–1
Single Correct Choice Type
Multiple Correct Choice Type
Q.1 An aqueous solution containing one mole per liter
each of Cu(NO3)2 AgNO3, Hg2(NO3)2, Mg(NO3)2 is being Q.7 Pick out the correct statements among the
electrolyzed by using inert electrodes. The value of following from inspection of standard reduction
standard potentials are potentials (Assume standard state conditions).

E − 0.80V, E − 0.79V, Cl2 (aq.) + 2e → 2Cl–(aq.) E = +1.36 volt


Ag+ |Ag Hg22+ |Hg Cl2 /Cl–

E − 0.34V and E −2.3V


= Br2 (aq.) + 2e → 2Br–(aq.) E = + 1.09 volt
Cu2+|Cu Mg2+|Mg Br2 /Br –

With increasing voltage, the sequence of deposition of I2(s) + 2e → 2I–(aq.) E = + 0.54 volt
I 2 /l–
metals on the cathode will be
(A) Ag, Hg, Cu, Mg (B) Mg, Cu, Hg, Ag S2O28− (aq.) + 2e → 2SO24− (aq.)
(C) Ag, Hg, Cu (D) Cu, Hg, Ag E = + 2.00 volt
S2O32− /SO24−
1 7 . 4 4 | Electrochemistr y

2− Assertion Reasoning Type


(A) Cl2 can oxidize SO 4 from solution
(A) 
Statement-I is true, statement-2 is true and
(B) Cl2 can oxidize Br– and I– from aqueous solution statement-II is correct explanation for statement-I
(C) S2O8
2−
can oxidize Cl–, Br– and I– from aqueous (B) 
Statement-I is true, statement-II is true and
statement-II is NOT the correct explanation for
solution
statement-I.
2−
(D) S2O8 is added slowly, Br– can be reduce in presence (C) Statement-I is true, statement-II is false
of Cl– (D) Statement-I is false, statement-II is true

Q.8 Which of the following arrangement will produce Q.12 Statement-I: The voltage of mercury cell remains
oxygen at anode during electrolysis? constant for long period of time.
(A) Dilute H2SO4 solution with Cu electrodes. Statement-II: It because net cell reaction does not
(B) Dilute H2SO4 solution with inert electrodes. involve active species.

(C) Fused NaOH with inert electrodes.


Q.13 Statement-I: We can add the electrode potential
(D) Dilute NaCl solution with inert electrodes. in order to get electrode potential of net reaction.
Statement-II: Electrode potential is an intensive
Q.9 If 270.0 of water is electrolyzed during an property.
experiment performed by Miss Abhilasha with 75%
current efficiency then Q.14 Statement-I: The SRP of three metallic ions A+,
(A) 168 L of O2 (g) will be evolved at anode at 1 atm & B2+ C3+ are –0.3, –0.5, 0.8 volt respectively, so oxidizing
273 K power of ions is C3+ > A+ > B2+.
(B) Total 504 L gases will be produced at 1 atm & 273 K Statement-II: Higher the SRP, higher the oxidizing
power
(C) 336 L of H2(g) will be evolved at anode at 1 atm &
273 K
Comprehension type
(D) 45 F electricity will be consumed
Paragraph 1: A sample of water from a large swimming
pool has a resistance of 10000 Ω at 25ºC when placed
Q.10 During discharging of lead storage battery, which
in a certain conductance cell. When filled with 0.02 M
of the following is/are true?
KCl solution, the cell has a resistance of 100 Ω at 25ºC.
(A) H2SO4 is produced 585 gm. of NaCl were dissolved in the pool, which was
(B) H2O is consumed thoroughly stirred. A sample of this solution gave a
resistance of 8000 Ω.
(C) PbSO4 is formed at both electrodes
[Given: Molar conductance of NaCl at that concentration
(D) Density of electrolytic solution decreases is 125 Ω–1cm2 mole–1 and molar conductivity of KCl at
0.02 M is 200 Ω–1 cm2 mole–1.]
Q.11 The EMF of the following cell is 0.22 volt.
Q.15 Cell constant (in cm–1) of conductance cell is:
Ag(s) | AgCl(s) |KCl (1M) |H+ (1M) |H2 (g) (1atm); Pt(s)
(A) 4 (B) 0.4 (C) 4 × 10–2 (D) 4 × 10–5
Which of the following will decreases the EMF of cell
(A) Increasing pressure of H2 (g) from 1 atm to 2 atm
Q.16 Conductivity (Scm–1) of H2O is:
(B) Increasing Cl concentration in Anodic compartment

(A) 4 × 10–2 (B) 4 × 10–3
(C) Increasing H concentration in cathodic compartment
+
(C) 4 × 10–5 (D) None of these
(D) 
Decreasing KCl concentration in Anodic
compartment. Q.17 Volume (in Liters) of water in the pool is:
(A) 1.25 × 105 (B) 1250
(C) 12500 (D) None of these
Chem i str y | 17.45

Paragraph 2: Copper reduces NO3– into NO and NO2 Q.3 The half-cell reaction for rusting of iron are:
depending upon conc. Of HNO3 in solution. Assuming 1
2H+ + 2e– + O → H2O (l); E° = + 1.23 V
[Cu2+] = 0.1 M2 and PNO = PNO = 10–3 atm and using 2 2
2
given data answer the following questions:
Fe2+ + 2e– → Fe(s), E = –0.44 V
E  = + 0.34 volt
Cu2 + /Cu ∆Gº (in kJ) for the reaction is  (2005)

E = + 0.96 volt (A) –76 (B) –322 (C) –122 (D) –176
NO3− /NO

E − = +0.79 volt
NO3 /NO2 Q.4 Electrolysis of dilute aqueous NaCl solution was

RT carried out by passing 10 mA current. The time required
At 298 K (2.303) = 0.06 volt to liberate 0.01 mole of H2 gas at the cathode is 
F
(1 F = 96500 C mole–1) (2008)
Q.18 ECell for reduction of NO3– → NO by Cu(s), when (A) 9.65 × 104 s (B) 19.3 × 104 s
[HNO3] = 1 M is [At T = 298]
(C) 28.95 × 104 (D) 38.6 × 104 s
(A) ~0.61 (B) ~0.71 (C) ~ 0.51 (D) ~0.81

Q.5 AgNO3 (aqueous) was added to an aqueous KCl


Q.19 At what HNO3 concentration thermodynamic solution gradually and the conductivity of the solution
tendency for reduction of NO3– into NO and NO2 by was measured. The plot of conductance (Λ) versus the
copper is same? volume of AgNO3 is (2011)
(A) 101.23 M (B) 100.56 M (C) 100.66 M (D) 100.12 M

Match the Columns V V

Q.20

Column I Column II Volume Volume


(P) (Q)
(Electrolysis product using
inert electrode)
V V
(A) Dilute solution of HCl (p) O2 evolved at anode
(B) Dilute solution of NaCl (q) H2 evolved at cathode
(C) Concentrated Solution (r) Cl2 evolved at anode Volume Volume
of NaCl (R) (S)

(D) AgNO3 solution (s) Ag deposition at cathode (A) (P) (B) (Q) (C) (R) (D) (S)

Q.6 Consider the following cell reaction,


Previous Years' Questions 2Fe(s) + O2 (g) + 4H+ (aq) →
2Fe2+ (aq) + 2H2O (l), E = 1.67 V
Q.1 In the electrolytic cell, flow of electrons is from
 (2003) At [Fe2+] = 10–3 M, P (O2) = 0.1 atm and pH = 3, the cell
(A) Cathode to anode is solution potential at 25ºC is  (2011)
(B) Cathode to anode through external supply (A) 1.47 V (B) 1.77 V (C) 1.87 V (D) 1.57 V
(C) Cathode to anode through internal supply
(D) Anode to cathode through internal supply Q.7 For the reduction of NO3– ion in an aqueous
solution E° is + 0.96 V. Values of E° for some metal ion
are given below
Q.2 Zn|Zn2+ (a = 0.1 M) || Fe2+ (a = 0.01 M) | Fe.
V2+ (aq) + 2e– → V E° = –1.19 V
The e.m.f of the above cell is 0.2905 V. Equilibrium
constant for the cell reaction is  (2004) Fe3+ (aq) + 3e– → Fe E° = –0.04 V
(A) 100.32/0.059 (B) 100.32/0.0295 Au3+ (aq) +3e– → Au E° = + 1.40 V
(C) 100.26/0.0295 (D) 100.32/0.295
1 7 . 4 6 | Electrochemistr y

Hg2+ (aq) +2e– → Hg E° = + 0.86 V A 4.0 M aqueous solution of NaCl is prepared and 500
mL of this solution is electrolyzed. This lead to the
The pair (s) of metals that is (are) oxidized by NO3– in
evolution of chlorine gas at one of the electrodes
aqueous solution is (are)  (2009)
(Atomic mass: Na = 23, Hg = 200, 1 F = 96500C). (2007)
(A) V and Hg (B) Hg and Fe
(C) Fe and Au (D) Fe and V Q.11 The total number of moles of chlorine gas evolved is
(A) 0.5 (B) 1.0 (C) 2.0 (D) 3.0
Paragraph 1: Tollen’s reagent is used for the detection
of aldehydes. When a solution of AgNO3 is added to
glucose with NH4OH then gluconic acid is formed. Q.12 If the cathode is a Hg electrode, the maximum
weight (in gram) of amalgam formed this solution is
Ag+ + e– → Ag; E red = 0.80 V
(A) 200 (B) 255 (C) 400 (D) 446
RT F
[Use 2.303 × = 0.0592 and = 38.92 at 298 K]
 F RT (2006) Q.13 The total charge (coulombs) required for complete
electrolysis is

Q.8 2Ag+ + C6H12O6 + H2O → 2Ag(s) C6H12O7 + 2H+ (A) 24125 (B) 48250 (C) 96500 (D) 193000

Find in K of this reaction.


Paragraph 3: Redox reaction play a pivotal role in
(A) 66.13 (B) 58.38 (C) 28.30 (D) 46.29 chemistry and biology. The values of standard redox
potential ( E ) of two half-cell
Q.9 When ammonia is added to the solution, pH is Reactions decide which way the reaction is expected to
raised to 11. Which half-cell reaction is affected by pH proceed. A simple example is a Daniel cell in which zinc
and by how much? goes into solution and copper gets deposited. Given

(A) Eoxi will increase by a factor of 0.65 from Eoxi below are a set of half-cell reaction (acidic medium)

along with their Eº (V with respect to normal hydrogen
(B) Eoxi will decrease by a factor of 0.65 from Eoxi electrode) values. Using this data obtain the correct
(C) Ered will increase by a factor of 0.65 from Ered
 explanations to

(D) Ered will decrease by a factor of 0.65 from Ered


 I2 + 2e– → 2I– E = 0.54
Cl2 + 2e– → 2Cl– E = 1.36
Q.10 Ammonia is always added in this reaction. Which Mn3+ + e– → Mn2+ E = 1.50
of the following must be incorrect?
Fe3+ + e– → Fe2+ E = 0.77
(A) NH3 combines with Ag to form a complex
+
O2 + 4H+ + 4e– → 2H2O E = 1.23 (2007)
(B) Ag (NH3)2 is a stronger oxidizing reagent than Ag
+ +

(C) In absence of NH3 silver salt of gluconic acid is Q.14 Among the following, identify the correct
formed statement
(D) NH3 has no effect on the standard reduction
(A) Chloride ion is oxidized by O2
potential of glucose/gluconic acid electrode
(B) Fe2+ is oxidized by iodine
Paragraph 2: Chemical reactions involve interaction (C) Iodide ion is oxidized by chlorine
of atoms and molecules. A large number of atoms/
(D) Mn2+ is oxidized by chlorine
molecules (approximately 6.023 × 1023) are present
in a few grams of any chemical compound varying
with their atomic/molecular masses. To handle such Q.15 While Fe3+ is stable, Mn3+ is not stable in acid
large numbers conveniently, the mole concept was solution because
introduced. This concept has implications in diverse (A) O2 oxidizes Mn2+ to Mn3+.
areas such as analytical chemistry, biochemistry,
electrochemistry and radiochemistry. The following (B) O2 oxidizes both Mn2+ to Mn3+ and Fe2+ to Fe3+
example illustrates a typical case, involving chemical/ (C) Fe3+ oxidizes H2O to O2
electrochemical reaction, which requires a clear (D) Mn3+ oxidizes H2O to O2
understanding of the mole concept.
Chem i str y | 17.47

Q.16 Sodium fusion extract, obtained from aniline, on Under standard conditions, the number of moles of M+
treatment with iron (II) sulphate and H2SO4 in presence oxidized when one mole of X is converted to Y is
of air gives a Prussian blue precipitate. The blue color is [F = 96500 C mol/L]  (2015)
due to the formation of
(A) Fe4 [Fe (CN) 6]3 (B) Fe3 [Fe (CN) 6]2 Q.20 The molar conductivity of a solution of a weak
(C) Fe4 [Fe (CN) 6]2 (D) Fe3 [Fe (CN) 6]3 acid HX (0.01 M) is 10 times smaller than the molar
conductivity of a solution of a weak acid HY (0.10 M).
If λ ≈ λ 0Y − , the difference in their pKa values, a pKa
0
Paragraph 4: The concentration of potassium ions X−
inside a biological cell is at least twenty times higher (HX) - pKa (HY), is consider degree of ionization of both
than the outside. The resulting potential difference acids to be << 1)  (2015)
across the cell is important in several processes such as
transmission of nerve impulses and maintaining the ion
Q.21 For the following electrochemical cell at 298 K,
balance. A simple model for such a concentration cell
involving a metal M is: Pt(s) | H2(g, 1 bar) | H+ (aq, 1M)|| M4+(aq), M2+ (aq) | Pt(s)
M(s) | M+ (aq; 0.05 molar) || M+ (aq; 1 molar) | M (s)
M2+ (aq) 
For the above electrolytic cell the magnitude of the cell Ecell = 0.092 V when 4+
= 10 x .
potential | Ecell | = 70 mV.  (2010) M (aq) 
Given: E° 4 + 2 + 0.151 RT
Q.17 For the above cell = = V; 2.303 0.059 V
M /M F
(A) Ecell < 0 ; ∆G > 0 (B) Ecell > 0 ; ∆G < 0 The value of x is  (2016)
(C) Ecell < 0 ; ∆Gº > 0 (D) Ecell > 0 ; ∆Gº < 0 (A) -2 (B) -1 (C) 1 (D) 2

Q.18 If the 0.05 molar solution of M+ is replaced by a Q.22 In a galvanic cell, the salt bridge  (2014)
0.0025 molar M+ solution, then the magnitude of the
cell potential would be (A) Does not participate chemically in the cell reaction.

(A) 35 mV (B) 70 mV (C) 140 mV (D) 700 mV (B) Stops the diffusion of ions from one electrode to
another.
Q.19 All the energy released from the reaction 0 –1 (C) Is necessary for the occurrence of the cell reaction.
X → Y. ∆ G° = -193 kJ mol/L
r (D) Ensures mixing of the two electrolytic solutions.
is used for oxidizing M as M → M
+ + 3+
+ 2e ,
-

E° = -0.25V.
1 7 . 4 8 | Electrochemistr y

PlancEssential Questions
JEE Main/Boards JEE Advanced /Boards

Exercise 1 Exercise 1
Q.3 Q.4 Q.8 Q.1 Q.6 Q.12

Q.9 Q.12 Q.16 Q.19 Q.22 Q.30

Q.23 Q.30 Q.35 Q.42 Q.45

Exercise 2 Exercise 2
Q.3 Q.5 Q.9 Q.4 Q.7 Q.16

Q.12 Q.15 Q.18 Q.18

Q.20

Previous Years' Questions


Previous Years' Questions
Q.5 Q.7 Q.17
Q.3 Q.8

Answer Key

JEE Main/Boards Q.21 0.1536 V


Q.22 –0.1615 V
Exercise 1
Q.23 (i) 0.2278 V ; (ii) 0.3692
Q.2 124 ohm cm mole
–1 2 –1
Q.24 0.91 V
Q.4 ∆rG° = -213.04 kJ, E°cell ==++1.104
1.104 V V Q.25 3.67 × 10–4
Q.5 λm = 124 Ω-1cm2mol-1 Q.26 Na, Ca, Mg and Al.
Q.8 0.219cm
-1
Q.27 2.96 V
Q.9 ∧m = 32. 765 cm2 mol–1 , ka = 2. 2 × 10–4 Q.28 (a) –0.118 V; (b) –0.0413 V; (c) (i) –0.0295 V
Q.10 3. 66 gm (ii) –0.59 V

Q.14 E° = 0. 34 V Q.29 1.3 × 102 ohm–1 cm2 equi–1

Q.19 (i) 0.2515 V; (ii) 0.8101 V (iii) 0.226 V (iv) 1.451 V;


(v) –0.268 V
Q.20 0.7288 V
Chem i str y | 17.49

Exercise 2
Single Correct Choice Type
Q.1 C Q.2 B Q.3 B Q.4 B Q.5 D Q.6 D
Q.7 B Q.8 A Q.9 A Q.10 B Q.11 B Q.12 B
Q.13 C Q.14 C Q.15 C Q.16 B Q.17 A Q.18 C
Q.19 C Q.20 D Q.21 C

Previous Years Questions


Q.1 D Q.2 D Q.3 B Q.4 C Q.5 B Q.6 D
Q.7 A Q.8 D Q.9 A Q.10 C Q.11 C Q.12 C
Q.13 D Q.14 C Q.15 C Q.16 A Q.17 D Q.18 C
Q.19 B Q.20 D Q.21 D Q.22 C Q.23 A Q.24 A

JEE Advanced/Boards
Exercise 1
Q.1 [Cu2+] = 10–4 M Q.2 pH = 6.61
Q.3 Ka = 6.74 × 10 4
Q.4 1.39 V
Q.5 – 0.46 V Q.6 (ii) 1.27V (ii) 245.1 kJ
Q.7 Eº = – 0.22 V Q.8 h =2.12 × 10–2, Kh = 1.43 × 10–5 M
Q.9 0.95 V Q.10 Ksp = 1.1 × 10–16
Q.11 [Br–]: [Cl–] = 1: 200 Q.12 Eº = –0.1511 V
Q.13 –0.037 V Q.14 1.536 × 10–5 M3
Q.15 1.66 V Q.16 –1.188V
Q.17 10 –2
Q.18 5.24 × 1016
Q.19 Eº = 1.59 V, non–spontaneous Q.20 Eº = 0.71 V
Q.21 [Zn ]/[Cu ] = 1.941 × 10
2+ 2+ 37
Q.22 0.373 V
Q.23 Kf = 8.227 × 1063 Q.24 0.372 V, –0.307 V
Q.25 K = 10268 Q.26 Ta+ = 5x10–9 ; Co3+ = 10–8
Q.27 Eq. wt. = 107.3 Q.28 643.33 amp, 190.5 g
Q.29 43.456 g Q.30 265 Amp. Hr.
Q.31 42.2 %
Q.32 V (O2) = 99.68 mL, V (H2) = 58.46 mL, Total vol. = 158.1 mL
Q.33 57.5894 gm. Q.34 1.9 million year
Q.35 2.4 × 105 coulomb Q.36 1250 s, 0.064 M
Q.37 1.143 V Q.38 (a) –0.124 V, (b) 7.1 (c) calomel electrode
Q.39 0.4414 V Q.40 –0.167 V
Q.41 0.1456 ampere Q.42 3.29 cm
1 7 . 5 0 | Electrochemistr y

Q.43 4.25 × 10–2 meter Q.44 0.1934 gm. /liter


Q.45 Ksp = 7.682 × 10–17 Q.46 2 × 105dm3

Exercise 2

Single Correct Choice Type

Q.1 C Q.2 C Q.3 A Q.4 C Q.5 A Q.6 B

Multiple Correct Choice Type

Q.7 B, C Q.8 B, C, D Q.9 A, B Q.10 C, D Q.11 D, C

Assertion Reasoning Type

Q.12 A Q.13 D Q.14 A

Comprehension Type

Paragraph - 1 Q.15 B Q.16 C Q.17 A

Paragraph - 2 Q.18 B Q.19 C

Match the Columns


Q.20 A → p, q; B → p, q; C → q, r; D → p, s

Previous Years' Questions


Q.1 C Q.2 B Q.3 B Q.4 B Q.5 D Q.6 D
Q.7 A, B, D Q.8 B Q.9 C Q.10 D Q.11 B Q.12 D
Q.13 D Q.14 C Q.15 D Q.16 A Q.17 B Q.18 C
Q.21 D Q.22 A
Chem i str y | 17.51

Solutions

JEE Main/Boards (iii) Sn + 2H+ → Sn2+ + H2


0. 02 0. 05
Exercise 1 n=2
−0.0591 [Sn2+ ]
Sol 1: Zn(s) | 2+
Zn(aq) || +
Ag(aq) | Ag(s) E = E° – log .PH
2 [H+ ]2 2
(i) Zinc electrode is negatively charged
0.05
E = E° – 0. 03 log
(ii) At anode: 0.04
2+ E = E° – 0. 0029
Zn(s) → Zn(aq) + 2e–
E = 0. 14 – 0. 0029
At cathode
⇒ E = 0. 1371 V
Ag+(aq) + e– → Ag(s)
(iv) Br2 + H2 → 2Br– + 2H+

k × 1000 2.48 × 10−2 × 1000 0.0591


Sol 2: ∧m = = E = E° – log [H+] 2[Br-] 2
M 0.2 2

∧m = 124 Ω-1 cm2 mol–1 = E° – 0. 02955 log (0. 01)2(0. 03)2


E = E° + 0. 208
Sol 3: (i) Reaction: E = 1. 09 + 0. 208
Mg ( s ) + Cu2+ ( aq. ) → Cu ( s ) + Mg2+ ( aq. ) E = 1. 298 V
       0. 0001 M                    0. 001 M
Sol 4: The formula of standard cell potential is
−1
2.303 RT  [Cu2+ ] 
E = E – log   E°cell Eright
= ° °
− Eleft
nF  [Mg2+ ] 
 
Use this link to get all values
0.0591 0.001
E = E – log
2 0.0001
°
E=
cell 0.344 − ( −0.76 )

E = E – 0. 02955 E°cell 0.344 + 0.076 V


=

E = E – 0. 03 E°cell = +1.104 V
= 2. 70 – 0. 03 In balanced reaction there are 2 electron are transferring
= 2. 67 V so that n = 2

(ii) Fe + 2H+ → Fe2+ + H2 Faraday constant, F=96500 C mol-1


  1     0.001 E°cell = +1.104 V
n=2
Use formula
0.0591 [Fe2+ ]PH
E= E – 
log 2
− n FE°cell
∆r G =
2 + 2
[H ]
0.001 × 1 Plug the value we get
E = E° – 0. 03 log
1 Then, =−2 × 96500 C mol−1 × 1.104 V
E = E° + 0. 09 = 0. 44 + 0. 09 = −212304 CV mol−1
⇒ E = 0. 53 V
= −212304 mol−1
1 7 . 5 2 | Electrochemistr y

Sol 11: We use the formula


= −212.304 kJ mol−1 = -213.04 kJ
k × 1000
∧m =
1000 C
Sol 5: ∧m =k×
C
0.0248 × 1000 C/M 0.001 0.010 0. 02 0. 05 0. 100
∧m = = 124 Ω–1cm2mol–1
0.2 102×k/sm–1 12.37 11.85 23. 15 55. 53 106.74

Sol 6: (i) Ca2+ + 2e⁻ → Ca ∧ ºm


12.37 11.85 11.575 11.106 10.674
20 S cm2/mol
Charge needed = 2 × nF = 2 × F = 1 F = 96500 C
40
(ii) Al + 3e⁻ → Al
3+
C 0. 03 0. 1 0. 14 0. 22 0. 31
Charge needed = (3nAl) × 96500C
∧m vs C
40
=3× × 96500 C = 4. 28 × 105 C
27

Sol 7: (i) H2O → O2 12.37

1 11.85
We have, H2O → O + 2H+ + 2e⁻
2 2 11.575
11.105
Charge required = 2 F = 1. 93 × 105 C 10.674

1 1
(ii) Fe2O + H2O → Fe2O3 + H++e– 0.001 0.1 0.02 0.05 0.1
2 2
Charge needed for 1 mole FeO = 1 F = 96500 C We have
∧m = ∧ º m – A C
Sol 8: k = 0. 146 × 10 s cm
–3 –1
At C = 0. 01,
R = 1500 Ω
∧m = 11. 85 = ∧ºm . A×0. 1 …(i)
G* = Rk = 0. 219 cm–1
At C = 0. 02
k 1000 × 7.896 × 10−5
Sol 9: ∧m = 1000 × = ∧m = 11. 575 = ∧ºm . A×0. 14 …(ii)
C 0.00241
∧m = 32. 765 cm2 mol–1 Subtracting
∧m 32.76
q= = 0. 01 A = 0. 275
∧ ºm 390.5
A = 6. 875
α = 0. 084
Cα 2 ∧ºm = 12. 5375 cm2 mol–1
ka = ;k = 2. 2 × 10–4
1−α a (putting A in (i))

Sol 10: Ni2+ + 2e⁻ → Ni


Sol 12: Ag+ + e⁻ → Ag
Charge supplied = 5 × 20 × 60 C = 6000 C
1.45
nAg = = 0. 0134
6000 108
Moles of Ni deposited =
96500 Charge, Q = nAg. F = 0. 0134 × 96500 C
6000
Mass of Ni deposited = × 59 = 3. 66 gm Q = 1295. 6 C
96500
Q 1295.6
t= =
I 1.5
t = 863. 735 s
Zn2+ + 2e⁻ → Zn
Chem i str y | 17.53

nAg ∆E1 = –nFEº = –0. 18 F


Q
Moles of Zn, nZn = =
2F 2 Cu+ + e → Cu,
w Ag 1
Wt. of Zn = nZn × 65 = × × 65 E° = 0. 50 V, n = 1
mAg 2
Wt. of Zn = 0. 43 gm ∆G2 = –nF E° = –0. 5 F

For Cu, Adding


Cu2+ + 2e⁻ → Cu Cu2+ + 2e → Cu,
Q
nCu = = nZn ∆G = ∆G1 + ∆G2, n = 2
2F
Wt. of copper = nCu × 63. 5 = 0. 426 gm –2F E°= –0. 18 F – 0. 5 F

E° = 0. 34 V
1
Sol 13: (i) Fe3(aq)
+ −
+ I(aq) 2+
→ Fe(aq) + I (g)
2 2
Sol 15: Element with maximum oxidation potential will
E = Eº− + Eº = –0. 54 + 0. 77 = +0. 23 > 0
I /I2 Fe3+ /Fe2 + have highest reduction potential.
This is feasible. Now, Eº = 0. 76 V,
Zn/Zn2 +
+
(ii) +
Ag(aq) + Cu(s) → Ag(s) + Cu(aq) Eº = 0. 13 V
Pb/Pb2 +
º º

E =E +E = 0. 8 – 0. 52 = 0. 2870 Eº 2 + = 1. 66 V
Ag+ /Ag Cu/Cu+
Al/Al

This is feasible. \Reducing order: Pb < Zn < Al


1 1
(iii) Fe3(aq)
+ −
+ Br(aq) → Br2(g) + Fe2+(g)
2 2 Sol 16: (i) Since O3 has the highest SRP, it is the strongest
º º oxidising agent.
Eº = E –E = 0. 77 – (1. 09) = –0. 32 < 0
Fe3+ /Fe2 + Br2 /Br −
(ii) As Ag cannot get reduced, O3 is still the strongest
So, this is not feasible oxidised agent.
(iii) Ag is the strongest reducing agent as it has the least
If Fe3+ → Fe, then, E = –0. 036
Fe3+ /Fe negative oxidation potential.
and E° = –1. 126, still not feasible. (iv) Ag is still the strongest reducing agent.
(iv) Ag(s)+Fe3+(aq) → Fe2+(aq) + Ag+(aq)
Sol 17: IO3– will act as better oxidising agent in acidic
E = Eº + Eº = –0. 8 + 0. 77 = –0. 03 < 0 medium due to its larger SRP in acidic medium then
Ag+ /Ag+ Fe3+ /Fe2 +
basic medium.
So, this is not feasible
1
if Fe3+ → Fe, Eº = –0. 03, Sol 18: Fe3+ + Br– → Br + Fe2+
Fe3+ /Fe 2 2

E° = –0. 836, still not feasible. E° = Eº + Eº


Fe3+ /Fe2 + Br − /Br2

1 = 0. 77 – 1. 09 = – 0. 32 < 0
(v) Br2(aq)+Fe2+(aq)→ Br–(aq)+Fe3+(aq)
2
∴ Fe3+ cannot oxidise Br– to Br2.
E° = Eº + Eº = 1. 09 – 0. 77 = 0. 32 > 0
Br2 /Br + Fe2 + /Fe3+
Sol 19: (i) Cu2+ + 2e⁻ → Cu
This reaction is feasible.
0.0591 1 1
E =E° – log = 0. 34 – 0. 0295 log
2 2
[Cu ]+ 0.001
Sol 14: Cu2+ + e → Cu+
E = 0. 2515 V
E° = +0. 18 V, n = 1
1 7 . 5 4 | Electrochemistr y

(ii) Zn → Zn2+ + 2e⁻ Sol 20: Fe3+ + e⁻ → Fe2+


0. 02 Eº = 0. 77 V
Fe3+ /Fe2 +

Eº = – Eº = 0. 76 [Fe2+ ]
Zn/Zn2 + Zn/Zn2 +
E = E – 0. 0591 log
[Fe3+ ]
0.0591 E = 0. 77 – 0. 0591 log 5
E = E – log[Zn2+ ] = 0. 76 – 0. 0295 log (0. 02)
2
( [Fe2+] = 5 × [Fe3+] )
E = 0. 8101 V
E = 0. 7287 V
(iii) AgCl + e⁻ → Ag + Cl–
Ag+ + e⁻ → Ag, E1 = 0. 8, n = 1 Sol 21: CH3COOH  CH3COO– + H+
∆G1 = – nF E1 Cα 2
ka = = Ca2
∆G1 = –0. 8 F 1−α

AgCl Ag+ + Cl–, ka


α=
K s = 2 × 10–10 C
1

∆G2 = –RT ln Ksp [H+] = Cα = Ck a

Adding [H+] = 0.357 × 1.74 × 10−5


AgCl + e → Ag + Cl–
[H+] = 2. 492 × 10–3
∆G = ∆G1 + ∆G2, n = 1 1
H → H+ + e
–F E° = –0. 8 F – 2. 303RT log (2×10 ) –10 2 2
E° = 0
2.303RT
E° = 0. 8 + log (2 × 10–10)
F E = –0. 0591 log[H+] = –0. 0295 log (2. 492 × 10–3)
E° = 0. 8 – 0. 57 E = 0. 1536 V
E° = 0. 226 V
Sol 22: pH = –log[H+]
1
(iv) Cl2 + e⁻ → Cl–
2 13 = –log[H+] ⇒ [H+] = 10–13

Eº = 1. 36 V kw 10 −14 1
Cl2 /Cl− [OH–] = = =
+
[H ] 10 −13 10
[Cl− ]
E = E° – 0. 0591 log Cu(OH)2 Cu + 2OH–
2+
(PCl )1/2
2
0.02 Ksp = [Cu2+] [OH–] 2
= 1. 36 – 0. 0591 log = 1. 36 – 0. 0591(–1. 548)
(0.5)1/2 K sp 1 × 10−19
[Cu2+] = =
E = 1. 451 V [OH− ]2 10−2
[Cu2+] = 10–17 M
1
(v) H + e⁻ → H2
+
2 Cu2+ + 2e → Cu
E° = 0
E° = 0. 34 V, n = 2
pH = – log[H+]
0.0591 1
E = E°– log
[H+] = 10–4. 4 2 [Cu2+ ]
PH21/2 1
E = 0. 0591 log = –0. 0591(log 2 –log[H+] ) = 0. 34 – 0. 0591 log = 0. 34 – 0. 5015
[H+ ] 10−17
= –0. 0591(0. 15 + 4. 4) E = –0. 1615 V
E = –0. 268 V
Chem i str y | 17.55

Sol 27: Mg(s)|Mg2+(aq)||Ag+(aq)|Ag(s)


Sol 23: (i) Ag+ + e → Ag ; E1 = 0. 80 V, n = 1
Mg(s) + 2Ag+ → Mg2+ + 2Ag
∆G1 = –nF E1 = –0. 8F
n = 2, E° = 3. 17 V
AgCl → Ag+ + Cl–
0.0591 [Mg2+ ]
∆G2 = –RT ln ksp = –2. 303 RT log(2×10–10) E = E° – log
2 [Ag+ ]2
Adding,
(0.13)
AgCl + e⁻ → Ag + Cl– = 3. 17 – 0. 0295 log = 3. 17 – 0. 209
(10−4 )2
∆G = –nF E° = ∆G1 + ∆G2, n = 1
E = 2. 96 V
–F E° = –0. 8 F – 2. 303 RT log(2×10–10)
2.303RT Sol 28: (a) H2SO4 → 2H+ + SO42–
E° = 0. 8 + log (2 × 10–10)
F
[H+] = 2[H2SO4] = 0. 02 M
E° = 0. 22 V
1
(ii) E = E° – 0. 0591 log[Cl–] = 0. 22 – 0. 0591 log (4 × 10–3) H → H+ + e⁻
2 2
E = 0. 36 V E° = 0, n = 1
[H+ ] 0.02
Sol 24: Ni(s) + 2Ag+(0. 002 M) E = E° – 0. 0591 log = –0. 0591 log
1/2
PH 4
2
→ Ni2+(0. 160 M) + 2 Ag(s) E = –0. 0591 log(0. 01)
n=2 E = +0. 1182 , Er = –0. 1182
0.0591 [Ni2+ ]
∴E = E1 – log (b) HCl → H+ + Cl–
2 [Ag+ ]2
[H+] = [HCl] = 0. 2
0.16
= 1. 05 – 0. 0295 log = 1. 05 – 0. 135 1
(2 × 10−3 )2 H → H+ + e⁻
2 2
E = 0. 91 V E° = 0, n = 1
0.0591 [H+ ] 0.2
Sol 25 HCOOH  H+ + HCOO– E = E – log = 0 – 0.0591 log
1 P 1/2 1
0 0 0 H2
∧m (HCOOH)= ∧m (HCOO–) + ∧m (H+)
E = 0. 0413 V, Er = –0. 0413 V
= 349. 6 + 54. 6
(c) E = 0, n = 1
0
∧m = 404. 2 S cm mol2 –1
−0.0591
E° = log[H+ ] = 0. 0591 p[H]
∧m 46.1 1
α= =
0
∧m 404.2 (i) pH = 5
α = 0. 114 E = 0. 0591 × 5 = 0. 2955 V
2 2
Cα 0.025(0.114) Er = –0. 2955 V
ka = =
1−α 1 − 0.114
(ii) pOH = 4
ka = 3. 67 × 10–4
pH = 14 – pOH = 10

Sol 26: (i) Potassium E = 0. 0591 × 10 = 0. 591 V

(ii) Sodium Er = –E = –0. 0591 V

(iii) Calcium
Sol 29: n for H2SO4 = 2
(iv) Magnesium
Normality = Molarity × n = 2M
(v) Aluminium
k × 1000 26 × 1000 × 10−2
∧e = = = 130 Scm2 equiv–1
N 2
1 7 . 5 6 | Electrochemistr y

Exercise 2 Sol 7: (B) This is a convention.

Single Correct Choice Type 1


Sol 8: (A) H+ + e– → H
2 2
Sol 1: (C) M+2 + 2e → M E° = 0
For one mole, charge required = 2F −0.0591 (PH )1/2
E= log 2 = –0. 0591 log 10 = –0. 0591 V
If atomic weight is M. 1 [H+ ]
∆E = 0. 0591 V
Then, for M gm, 2F charge
2F Sol 9: (A) We have,
For 1 gm, charge = = 1. 81 × 1022 × 1. 6 × 10–19
M
0.0591
⇒ M = 66. 7 gm E° = log kC
2
0.0591
Sol 2: (B) Moles of Ag = 1 E° = log(2. 69 × 1012) = 0. 3667 V
2
1
Moles of Cu =
2 Sol 10: (B) Ag+ + e⁻ → Ag
1 1
Moles of Al = E = E°– 0. 0591 log
3 [Ag+ ]
1 1 0. 25 = 0. 799 + 0. 059 log [Ag+]
1: : = 6: 3: 2
2 3
[Ag+] = 5. 13 × 10–10
2.1
Sol 3: (B) Moles of A = ksp = [Ag+] [Cl–] = 5. 1 × 10–11
7
2.1 2.7 7.2
Molar ratio = : : = 0. 3: 0. 1: 0. 15 Sol 11: (B) At cathode
7 27 78
Cu2+ + 2e⁻ → Cu
= 3: 1: 1. 5
At anode
= 6: 2: 3
1 1 2H2O → O2 + 4H+ + 4e⁻
= 1: :
3 2 1
Moles nO = nC
\ Valency = 1, 3 and 2
2 2 2
1 2.5
VO = 22400 nO = 22400 × × = 441 mL
Sol 4: (B) Volume of plate = 10×10×10 cm = 1 cm
–2 3 3 2 2 2 63.5
≈ 445 mL O2 is liberated
Mass = 8. 94 g
8.94
Moles = Sol 12: (B) P = VI
63.5
8.94 P = 1. 65 kW
Quantity of electricity = × 2F = 27172 C
63.5
q = 1. 65 × 8 kWh = 13. 2 kWh
1 Cost = 13. 2 × 5 paise = 66 paise
Sol 5: (D) H2O → H2 + O
2 2
2H2O → 2H2 + O2
1  1000
Only water will be oxidised and reduced. Sol 13: (C) ∧m = . .
R a C
VH = 2 × VO = 4. 8 L 1 2.2 1
2 2
= . × = 0. 002 Sm2 mol–1
50 4.4 × 10 −2 0.5
Sol 6: (D) At cathode, water will be reduced,
2H2O + 2e⁻ → H2 + 2OH– (aq) ∧m 10
Sol 14: (C) α = = = 0. 05
0
∧m 200
pH of the solution around cathode will decrease.
Chem i str y | 17.57

[H+] = Cα = 0. 005 Sol 19: (C) At cathode,


pH = –log[H+] = log[200] = 2. 3 1
H+ + e⁻ → H
2 2
1000 1000 1 At anode
Sol 15: (C) ∧m = k × = (k = , C = y)
C xy x 1
H → H+ + e⁻
2 2
ka 1.6 × 10−5 1 1
Sol 16: (B) α = = = 0. 04 H2 + H+ → H2 + H+
C 0.01 2 2
∧m 0. 410–20. → 10–1
α= 10−1.(0.1)1/2
0
∧m E = –0. 0591 log
10−2.(0.4)1/2
⇒ ∧m = 380 × 10–4 × 0. 04 = 1. 52 × 10–3
E = –0. 041 V
1000 × k
∧m =
C × 1000 Sol 20: (D) O2(g) + 4H+(aq) + 4e– → 2H2O()
k = 1. 52 × 10–2 S m–1 E° = 1. 23 V
∆G° = –nFE = –474. 78 kJ
Sol 17: (A) Au+ + 2CN– = Au(CN)2–
∆Hf (O2) = 0
∆G1 = –RT ln X ∆Hr = 2∆Hf(H2O, ) = –2 × 285. 5 kJ = –571 kJ
Au + 2e → Au
3+ – +
∆G = ∆H – T∆S
∆G2 = –nFEº = –2. 8F ∆H − ∆G
∆S = = –0. 322 kJ/k
Au → Au + 3e⁻
3+ T
∆G3 = –nF(–Eº) = 4. 5F
Sol 21: (C) Cu2+ + 2e⁻ → Cu; Eº = 0. 337
1 1
O2 + H2O + e– → OH– –nFE    = –0. 674 F
∆G1 =
4 2
∆G4 = –1F(Eº) = –0. 41F Cu2+ + e Cu+ ; E° = 0. 153
4 Cu+ → Cu2+ + e⁻; E° = –0. 153
Summing, ∆G = ∑ ∆G1 = –RT ln X + 1. 29 F
∆G2 = = + 0. 153 F
–nFE    
i=1
Adding,
Sol 18: (C) At anode Cu+ + e⁻ → Cu
1 ∆G = ∆G1  +∆G2  
H → H+ + e⁻
2 2
–F E° = –0. 674 F + 0. 153 F
At cathode
E° = 0. 521 V
1
Cl + e⁻ → Cl–
2 2
1 1 Previous Years’ Questions
H2 + Cl2 → H+ + Cl–
2 2
Sol 1: (D) Impure metal made anode while pure metal
[H+ ][Cl– ] made cathode.
E = Eº – 0. 0591 log
1/2 1/2
PH .PCl
2 2
Sol 2: (D) In electrolytic cell, cathode acts as source of
[H+ ]2 [Cl– ]2 electrons.
E1–E2 = 0. 0591 log = 0. 0591 × 2 = 0. 1182
[H+ ]1 [Cl– ]1
+ e–
Sol 3: (B) Ag+ 
→ Ag, 96500 C will liberate
∴E2 – E1 = –0. 1182
silver = 108 gm.
9650C will liberate silver = 10.8
1 7 . 5 8 | Electrochemistr y

Sol 4: (C) Gold (Au) and Silver(Ag) are present in the Sol 11: (C) 2H(aq)
+
+ 2e− → H2(g)
anode mud.
0.0591 P(H )
E=
red

Ered − log 2
Sm n (H+ )2
K[A]A C×l
Sol 5 (B) C = ,K= =
1 [A]A mol m-3m2 0.0591 2
Ered= 0 − log
= S m mol . 2 –1 2 (1)2
0.0591
Sol 6: (D) 0.1 M difluoroacetic acid has highest electrical Ered = log2
2
conductivity among the given solutions.
∴ Ered is found to be negative for (c) option
Sol 7: (A) From Kohlraush’s law
0.0591
Λ oCH = Λ oCH o
+ ΛHCl o
− ΛNaCl Sol 12: (C) E=
cell

Ecell − logQ
3COOH 3COONa n
Hence, (A) is the correct answer. [Zn2+ ]
Where, Q =
[Cu2+ ]
Sol 8: (D) According to Kohlrausch’s law, the molar For complete discharge, Ecell = 0
conductivity of an electrolyte at infinite dilution is the
sum of conductivities of its ions. Therefore to obtain 0.0591 [Zn2+ ]
o o o So, Ecell = log
∧HOAc from the conductivity of ∧NaOAc & ∧HCl ,the 2 [Cu2+ ]
o
conductivity value of NaCl i.e. ∧NaCl is also needed.  [Zn ] 
2 +
⇒ 
2+
 = 1037.3.
o o o o  [Cu ] 
ΛNaOAc + ΛHCl − ΛNaCl = ΛHOAc

i.e. Λ o + Λo + Λ o+ + Λo – − (Λo + Λo – ) Sol 13: (D) For the cell reaction, Fe acts as cathode and
Na+ OAc – H Cl Na+ Cl

= Λ o
+ Λ o+ = ΛHOAc
o Sn as anode. Hence,
OAc− H
Ecell =Ecathode − Eanode =−0.44 − ( −0.14) =−0.30V
Sol 9: (A) Cell constant The negative EMF suggests that the reaction goes
Conductivity(l) Conductivity(II) spontaneously in reverse direction.
= =
Conduc tance(l) Conduc tance(II)
129 Sol 14: (C) Fe3+ + 3e– → Fe;
Conductivity (II) = Sm−1
520
∆G1 = –3 × F × E
Fe3+ /Fe
0.2 M = 200 mol m–3.
Molar conductivity of 0.2 M solution Fe+2 + 2e– → Fe; ∆G2 = –2 × F × E
Fe2 + /Fe
129 / 520
= = 12.4 × 10–4 S m2 mol–1. Fe3+ + e– → Fe2+ ; ∆G = ∆G1 –∆G2
200
∆G = 3 × 0.036G – 2 × 0.439 × F
Note: From given data it is not possible to calculate
molar conductivity of 0.02 M solution exactly. However, it = – 1 × E ×F
(Fe3+ /Fe+2 )
is sure that its molar conductivity will be slightly greater
than 12.4 × 10–4 Sm2 mol–1. Assuming linear inverse
E = 2 × 0.439 − 3 × 0.036
variation of molar conductivity with concentration we (Fe3+ /Fe+2 )
can say that = 0.878 – 0.108 = 0.770 V
∴ Molar conductivity of 0.02M solution
0.059 [Zn++ ]
= 1.24 × 10–3 × 10 Sol 15: (C) E = E – log
n [Cu++ ]
= 124 × 10–4 S m2mol–1. 0.059 1
= 1.10 – log
2 0.1
Sol 10: (C) Velocities of both K+ and NO3– are nearly the
= 1.10 – 0.0295 log 10 = 1.07 volt.
same in KNO3 so it is used to make salt-bridge.
Chem i str y | 17.59

Sol 16: (A)


Sol 19: (B) 2H+ + 2e− → H2 ( g)
As E −0.72V and E 2+
= −0.42V
=
Cr3+ /Cr Fe /Fe  
 PH2  2
E= o
E − 0.059 log  (here E is − ve when PH > H+  )
2Cr + 3Fe2+ → 3Fe + 2Cr3+ 2  2  
 H+  
  
 0.0591 (Cr3+ )2
E=
cell Ecell − log −0.0591  2  −0.0591
6 (Fe2+ )3 = log10   = × 0.3010 = negative value
2 1 2
0.0591 (0.1)2 0.0591 (0.1)2
( −0.42 + 0.72) −
= log 0.30 −
= log
6 (0.01) 3 6 (0.01)3
Sol 20: (D)
0.0591 10−2 0.0591
0.30 −
= log 0.30 −
= log10 4 Zn + Fe+2 → Zn+2 + Fe
6 10 −6 6
Ecell=0.2606 V Fe + Ni+2 → Fe2+ + Ni

Zn + Ni2+ → Zn+2 + Ni


Sol 17: (D)
3 All these are spontaneous
CH3OH( ) + O2 (g) → CO2 (g) + 2H2O( ) ∆H = −726kJ mol−1
2
H2O( ) ∆H = −726kJ mol−1 Sol 21: (D)
Higher the SRP, better is oxidising agent.
−166.2 kJ mol−1
Also ∆Gf CH3OH( ) =
Hence, MnO −4 is strongest oxidising agent.
−1
∆Gf H3O( ) −237.2 kJ mol
=

−394.4 kJ mol−1
∆Gf CO2 ( ) = Sol 22: (C)

 ∆G = Σ∆Gf products − Σ∆Gf reac tants According to Debye Huckle onsager equation,

=-394.4-2(237.2) + 166.2 λC = λ ∞ + (A) C

=-702.6 kJ mol-1 Here A = B


∆G ∴ λC = λ ∞ + (B) C
now Efficiency of fuel cell = × 100
∆H
702.6 Sol 23: (A)
= × 100= 97%
726
(1) Mn2+ + 2e⁻ → Mn; E° = –1.18V;
Sol 18: (C) ∆G1 =−2F( −1.18) =2.36F
Fe3+ + 3e− → Fe; E = −0.036 V (2) Mn3+ + e⁻ → Mn2+; E° = +1.51 V;
∴ ∆G1 = −nFE = −3F( −0.036) = +0.108F ∆G1 =−F( −1.51) =−1.51F
Also Fe2+ + 2e− → Fe(aq); E = −0.439 V (1) - 2 × (2)
∴ ∆G1 = −nFE = −2F( −0.439) = 0.878F 3Mn2+ → Mn + 2Mn3+;
To find Eo for Fe3(aq)
+
+ e− → Fe2+ (aq) ∆G3 =∆G1 − 2∆G2
∆G1 =−nFE =−1FE = [2.36 – 2(–1.51)] F

 G= G1 − G2
= (2.36 + 3.02) F
∴ G =0.108F-0.878F = 5.38 F
∴ − FE = +0.108F − 0.878F
But ∆G3 =
12FE
∴ E= 0.878 − 0.108= 0.77v
− 2FE
⇒ 5.38F =
1 7 . 6 0 | Electrochemistr y

⇒ E =
−2.69 V E +E = 0. 982
H2 /H+ Ag+ /Ag
As E° value is negative reaction is non-spontaneous.
E = 0. 188
H2 /H+

Sol 24: (A) Cu2+ + 2e → Cu 1
2F 1mole H → H+ + e⁻
= 63.5g 2 2
0. 188 = 0 – 0. 059 log[H+]

JEE Advanced/Boards [H+] = 6. 6 × 10–4


Cα = 6. 6 × 10–4
Exercise 1 6.6 × 10−4
⇒α= = 0. 5
Sol 1: Cu + 2e → Cu
2+ 1.3 × 10−3

E = 0. 344 Cα 2
ka =
1−α
0.0591 1
E = E – log = 0. 226
2 [Cu2+ ] ka = 6. 74 × 10–4

As after E < 0. 226, Bi will also start to deposit


0.0591 [Mn2+ ]
0.0591 1 Sol 4: E = E – log
0. 344 – log = 0. 226 5 [H+ ]8 [MnO − ] 4
2 [Cu2+ ]
MnO4 + 8H + 5e → Mn + 4H2O
– + - 2+
0. 0295 log [Cu2+] = –0. 118
log [Cu2+] = –4 t = 0 0. 1 0. 8
[Cu2+] = 10–4 M t=t 0. 01 0. 08 0. 09
0.09
E = 1. 51 – 0. 01182 log = 1. 39 V
Sol 2: Reduction potential of calomel electrode = 0. 28 V (0.08)8 (0.01)
0. 67 = 0. 28 – Eº +
H /H2

º
Sol 5: CH3COOH CH3COO– + H+
E = –0. 39
H+ /H2
ka 1.8 × 10−5
1 α= = = 0. 0134
H+ + e → H2 C 0.1
2
[H ] = Cα = 1. 34 × 10–3
+
1
E = E – 0. 059 log

1
[H+ ] H → H+ + e⁻
2 2
1
–0. 39 = – 0. 059 log [H+ ]
[H+ ] E = Eº – 0. 0591 log
H2 /H+ H2 /H+ PH 1/2
−0.039
= log[H+] 2
0.059 E = 0 – 0. 0591 log 1. 34 × 10–3
H2 /H+
pH = –log[H+]
E = 0. 170 V
pH = 6. 61 H2 /H+

At cathode,
Sol 3: Ag+ + e⁻ → Ag
0.0591 1 NH3 + H2O NH4+ + OH–
º
E =E – log
Ag+ /Ag Ag+ /Ag 1 [Ag+ ] kb 1.8 × 10−5
1 α= =
= 0. 8 – 0. 0591 log C 0.01
0.8
α = 0. 042
E = 0. 794
Ag+ /Ag
Chem i str y | 17.61

[OH–] = Cα = 4. 24 × 10–4, 1
E = E°– 0. 0591 log
+
10 −14 [H ]cathode
[H+] = = 2. 35 × 10–11

[OH ] 1
–0. 188 = 0 – 0. 0591 log
1 [H+ ]
H+ + e– → H
2 2 log [H+] = 6. 59 × 10–4
1
E = Eº – 0. 0591 log
H+ /H2 H2 /H+ [H+ ] C6H5NH3Cl C6H5NH2 + H+ + Cl–
1
= 0. 0591 log = –0. 628 V Ch = [H+]
−11
2.35 × 10
6.59 × 10−4
h= = 2. 12 × 10–2
Ecell = E +E = 0. 170 – 0. 628 = 0. 458 V (1 / 32)
H2 /H+ H+ /H2

Cα 2
Sol 6: Fe(s) + 2OH → FeO(s) + H2O()+2e
– Θ kh =
1.α
E° = 0. 87 V kh = 1. 43 × 10–5
Ni2O3(s)+H2O + 2e⁻ → 2NiO(s)+2OH–
E° = +0. 4 V Sol 9: m. equivalents of OH– added = MV

(i) Cell reaction = 40 × 0. 05 = 2 meq.

Fe(s) + Ni2O3(s) → FeO(s) + 2NiO(s) m. equivalents of H+ = MV = 30 × 0. 1 = 3meq.

(ii) E° = 0. 87 + 0. 4 = 1. 27 V ∴ m. equivalents of H+ remaining = 1 meq.


1
It does not depend on [KOH] [H+] = = 0. 014
40 + 30
(iii) Maximum amount of electrical energy
1
H + Ag+ → H+ + Ag
= |–n F E° | 2 2
= |–2 × 96. 5 × 1. 27 kJ| PH = 1 atm,
2
= 245. 1 kJ [H+ ]
∴ E = E° – 0. 0591 log
[Ag+ ]
Sol 7: Cu(OH)2 Cu2+ + 2OH– [H+ ]1 [Ag+ ]2
∴ E2 – E1 = 0. 0591 log .
We have pH = –log[H+] = 14 [H+ ]2 [Ag+ ]1
[H+] = 10–14 0.1
−14
E2 – 0. 9 = 0. 0591 log .1
10 0.014
[OH–] = =1
[H+ ] E2 – 0. 9 = 0. 05
[Cu ] [OH ] = Ksp
2+ – 2
E2 = 0. 95 V
[Cu2+] = 1 × 10–19
Sol 10: E° = 0
Cu2+ + 2e⁻ → Cu
Ag+ + Ag → Ag+ + Ag
0.0591 1 1
E = E° – log = 0. 34 – 0. 0295 log [Ag+] c = 0. 05 M
2 2
[Cu ]+
10−19
−0.0591 [Ag+ ]a
E= log
= 0. 34 – 0. 56 = – 0. 22 V 1 [Ag+ ] c

[Ag+ ]a
1 1 0. 788 = –0. 0591 log
Sol 8: H2 + H+ → H2 + H+ 0.05
2 2
[Ag+ ]a
All conditions are standard except [H+] anode log = –13. 3
0.05
1 7 . 6 2 | Electrochemistr y

[Ag+] = 2. 32 × 10–15 ksp = [Ag+] a[Cl–]

AgI Ag+ + I– 2. 8 × 10–10 = [Ag+] a × 0. 2

ksp = [Ag+] [I–] [Ag+] a = 1. 4 × 10–9

[I–] = [KI] = 0. 05 Ag → Ag+ + e⁻

∴ ksp = 1. 16 × 10–16 At cathode

AgBr Ag+ + Br–


Sol 11: AgBr Ag + Br
+ –

ksp = [Ag+] c [Br–]


ksp = [Ag+] a. [Br] –
3.3 × 10−13
(k sp )AgBr [Ag+] c = = 3. 3 × 10–10
[Ag+] a = 10−3
[Br − ]
Ag + Ag+ + Ag + Ag+
(k sp )AgCl
Similarly, [Ag+] c = Cathode anode
[Cl− ]
Ag+ + Ag → Ag+ + Ag E = 0
[Ag+ ]a 1.4 × 10−9
cathode anode E = –0. 591 log = 0. 0591 log
[Ag+ ]c 3.3 × 10−10
E° = 0
[Ag+ ]a E = –0. 037 V
E = –0. 0591 log =0
[Ag+ ]c For a spontaneous process, electrodes should be
[Ag ] a = [Ag ] c
+ + reversed.

(k sp )AgBr (k sp )AgCl Sol 14: Pb2+ + 2e⁻ → Pb E = –0. 126 V


=
[Br − ] [Cl− ]
Pb → Pb2+ + 2e⁻ E = 0. 126 V
[Br − ] 5 × 10−13 
PbCl2 + 2e⁻ → Pb + 2Cl– E = –0. 268
=
− −10
[Cl ] 1 × 10
PbCl2 → Pb2+ + 2Cl– E = –0. 142 V
[Br − ] 1
= 5 × 10–3 =

[Cl ] 200 ∆G = –Fn E = –RT ln k
nF 
Sol 12: AgI Ag+ + I ln k = E
RT
∆G = –nFE1 nF
log k = E
RT × 2.303
Ag+ + e⁻ → Ag
log k = –4. 8n = 2
∆G = –nFE1
k = 1. 536 × 10–5
AgI + e⁻ → Ag+ + I–
Sol 15: Mn(OH)2 Mn2+ + 2OH–
∆n = –nF E = –nF E1 – RT ln k
Ksp = [Mn2+] [OH–] 2
RT
E =E +
 
lnk
nF 1.9 × 10−13
[Mn2+] =
2.303RT (10−4 )2
E = E1 – pk sp
nF [Mn2+] = 1. 9 × 10–5
Eº = –0. 15
Cell reaction:

Sol 13: At anode Mn + Cu2 → Mn2+ + Cu



AgCl Ag+ + Cl– E = –1. 18 V
Mn2 + /Mn
Chem i str y | 17.63

Eº = 0. 34 V [Pb2+ ]a
Cu2 + /Cu log = –2. 06
2.5 × 10−5
Eºcell = Eº – Eº
Cu2 + /Cu Mn2 + /Mn
[Pb2+] a = 2. 13 × 10–7
Eº = 0. 34 – (–1. 18)
At anode,
Eº = 1. 52 V
0.0591 [Mn2+ ] Pb + SO42– → PbSO4 + 2e⁻ E° = 0. 356
E = Eº – log
2 [Cu2+ ] Pb2+ + 2e⁻ → Pb, E° = –0. 126 V
1.9 × 10−5 Pb2+ + SO42– → PbSO4 E° = 0. 230
= 1. 52 – 0. 0295 log = 1. 52 – (–0. 14)
6.75 × 10−2
PbSO4 → Pb2+ + SO42– E° = –0. 230
E = 1. 66 V
∆G = –nF E° = –RT ln k
Sol 16: HIO3 H + IO3
+
nF
ln k = E°
ka RT
[IO3–] = Cα = C = Ck a
C nF
log k = E
[IO3 ] = 0. 22 M
– 2.303RT
k = 1.60 × 10–8
AgIO3 Ag+ + IO3–
ksp = 1.6 × 10–8 = [Pb2+] [SO4–2]
ksp = [Ag+] [IO3–]
1.6 × 10−8
3.08 × 10−8 [SO42–] =
[Ag+] = 2.13 × 10−7
0.22
[SO42–] = 7. 50 × 10–2
[Ag ] = 1. 37 × 10
+ –7

HSO4– H+ + SO42–
2Ag + Zn → 2Ag + Zn
2+ +

0. 525 0. 075 0. 075


Eº = –0. 80 V
Ag/Ag+
[H+ ][SO24− ]
ka =
Eº = –0. 76 V [HSO −4 ]
Zn2 + /Zn
ka = 9. 46 × 10–3
º º
E cell = EAg/Ag+ + EZn2+ /Zn = –1. 56 V ka ≈ 10–2

0.0591 [Ag+ ]2
E = E – log Sol 18: Cell reaction
2 [Zn2+ ]
Zn + Zn2+ → Zn + Zn2+
(1.37 × 10−7 )2
E = –1. 56 – 0. 0295 log Cathode anode
0.175
E = –1. 56 + 0. 372 E° = 0

E = –1. 188 V −0.0591 [Zn2+ ]a


E= log
2 [Zn2+ ] c

Sol 17: Pb + Pb2+ → Pb + Pb2+ −0.0591 [Zn2+ ]a


0. 099 = log
E = 0 2 3.84 × 10−4
0.0591 [Pb2+ ]a  [Zn2+ ] 
E=0– log +0. 061 log  a  = –3. 356
2 [Pb2+ ]  3.84 × 10−4 
c
 
2+
0.0591 [Pb ]a [Zn2+] a = 1. 69 × 10–7
= log
2 2.5 × 10−3
At anode
1 7 . 6 4 | Electrochemistr y

Zn2+ + 4CN– → Zn(CN)42– Now for Zn + Cu2 → Cu + Zn2+


[Zn(CN)24− ] 0.0591 [Zn2+ ]
kf = E = E – log
[Zn2+ ][CN– ]4 2 [Cu2+ ]
0.450 0.0591 1
= E = 1. 1 – log
1.69 × 10 −7
× (2.65 × 10 )) −3 4 2 1−x
1
kf = 5. 24 × 1016 = 1. 1 – 0. 0295 log
6.25 × 10−14
E = 0. 71 V
Sol 19: Ti+ + e– → Ti E° = –0. 34 V
∆G = –nF E° Sol 21: Zn + Cu2+ → Zn2+ + Cu

Ti3+ + 2e– → Ti+ E° = 1. 25 V we have, E° = 0. 76 – (–0. 34) = 1. 1 V


∆G = –nF E° = –RT ln k
Ti+ → Ti3+ + 2e– E° = –1. 25 V
nF
∆G1 = –nF E° = –2F(–1. 25) log k = E
2.303 × RT
2Ti+ + 2e–→ 2Ti [Zn2+ ]
k=
∆G2 = –2nF E° = –2F(–0. 34) [Cu2+ ]

3Ti+ → 2Ti + Ti3+ [Zn2+ ] 2


log = × 1. 1 = 37. 28
2+
[Cu ] 0.0591
∆G = ∆G1 + ∆G2

⇒ –2FEº = –2FEº1 – 2FEº2 [Zn2+ ]


= 1. 941 × 1037
[Cu2+ ]
Eº = E 1 + E 2

Eº = –0. 34 – 1. 25 Sol 22: Ag(NH3)2 Ag+ + 2NH3


Eº = –1. 59 V k0 = 6 × 10–8

Since, E° < 0, it is non spontaneous reaction. ∆G1 = –RT ln k0

Ag+ + e– → Ag E = 0. 799 V
Sol 20: Cu + 4NH3 → [Cu(NH3)4]
2+ 2+

∆G2 = –nF E = –F × 0. 799


t=0 1 A
t=t 1 – x A – 4x x (n = 1)

We have, A – 4x = 2 Adding,

[Cu(NH3 )4 ]2+ Ag(NH3)2 + e– → Ag + 2NH3


Kf =
[Cu2+ ][NH3 ]4 ∆G = –nF E° = ∆G1+∆G2
x
1 × 10 = 12
–n E° = –RT ln k0 – 0. 799 F
(1.x).24
1−x RT 2.303 RT
= 6. 25 × 10–14 Eº = ln k0 + 0. 799 = log(6 × 10–8) + 0. 799
x F F
1 E° = 0. 373 V
– 1 = 6. 25 × 10–14
x
1 Sol 23: We have,
x=
−14
1 + 6.25 × 10 (i) Co2+ + 6CN– → [Co(CN)6] 4–
6.25 × 10−14 kf = 1 × 1014
1–x= ≈ 6. 25×10–14
−14
1 + 6.25 × 10
Chem i str y | 17.65

∆G1 = –RT ln kf = – 43. 757 RT ∆G'1 = –nF E° = –2F × 0. 136 = –0. 272 F

(ii) [Co(CN)6] 3– + e– → [Co(CN)6] 4– For, 3Sn → 3Sn2+ + 6e⁻

E° = –0. 83 V, n=1 ∆G1 = 3∆G'1 = –0. 816 F

∆G2 = –nF E° = 0. 83 F Sn2+ → Sn4+ + 2e⁻, E° =–0. 154V, n = 2

(iii) Co3+ + e– → Co2+,Eº = 1. 82 V, n=1 ∆G2 = –nF E° = –2 × F × (–0. 154) = 0. 308 F

∆G3 = –nF E° = –1. 82 F For, 3Sn2+ → 3Sn4+ + 6e⁻,

(iii) + (i). (ii) gives, ∆G2 = 3∆G'2 = 0. 924 F

Co3+ + 6CN– → [Co(CN)6] 3– Cr2O72– + 14H+ + 6e⁻ → 2Cr3+ + 7H2O

∆G = ∆G3 + ∆G1 – ∆G2 Eº = 1. 33 V, n = 6

∆G = –RT ln kf = –1. 82F – 43. 757RT – 0. 83 F ∆G'3 = –nFE° = –6F(1. 33) = –7. 98 F

F F For, 2Cr2O72-+28H+ + 12e⁻ → HCr3+ 14H2O


⇒ln kf = × 1. 82 + 43. 757 + 0. 83
RT RT ∆G3 = 2∆G'3
1.82 + 0.83
log kf = + 19 ∆G3 = –15. 96 F
0.059
Now, (i) + (ii) + (iii) gives
kf = 8. 27 × 1063
3Sn(s) + 2Cr2O78– +28H+ → 3Sn4+ + 4Cr+3 + 14H2O
Sol 24: we have, ∆G = –RT ln k = ∆G1 + ∆G2 + ∆G3
(i)Ag + e → Ag E = 0. 7991 V, n=1
+ – 
–RT ln k=–0. 816 F + 0. 924 F –15. 96F
∆G 
1
= –nF E = –0. 7991 F

F
ln k = × 15. 852
RT
(ii) [Ag(NH3)2+] → Ag+ + 2NH3,
15.852
kins = 6. 02 × 10–8 log k =
0.0591
∆G2 = –RT ln kins log k = 268

Adding (i) and (ii), k = 10268

[Ag(NH3)2+] + e– → Ag + 2NH3, n = 1
Sol 26: Ti+ → Ti3+ + 2e⁻; Eº = –1. 25 V
∆G = ∆G1 + ∆G2
Co3+ + e⁻ → Co2+
–F E° = –0. 7991 F – RT ln kins
2Co3+ + 2e⁻ → 2Co2+ Eº = 1. 84 V
2.303RT
E° = 0. 7991 + log kins Adding,
F
= 0. 7991 + 0. 0591 log (6. 02×10–8) Ti+ + 2Co3+ → Ti3+ + 2Co2+

E° = 0. 372 V E° = 0. 59 V

Similarly for [Ag (CN)2–] ∆G = –nF E° = –RT ln k, n = 2

E° = 0. 7991 + 0. 0591 log (1. 995×10–19) 2F 0.59


log k = E = = 20
2.303 RT 0.0295
E° = –0. 307 V
k = 1020

Sol 25: We have, Now, if Ti+ = 0. 1 × 2. 5 = 25

Sn → Sn2+ + 2e, E°= 0. 136 V, n = 2


1 7 . 6 6 | Electrochemistr y

Initial milimoles of Co3+ = 25 × 0. 2 = 5 Charge produced by 3 mole H2=6F

Ti+ + 2Co3+ → Ti3+ + 2Co2+ 6F 6 × 96500


Current, I = =
t 15 × 60
t=0 2. 5 5 0 0
I = 643. 33 A
t = t 2. 5 – x 5–2x x 2x Cu2+ + 2e → Cu
3+ 2+ 2
[Ti ][Co ] number of moles of Cu produced = 3
k= = 1020
[Co3+ ]2 [Ti+ ] wt. = 3 × 63. 5 g
Vfinal = 50 ml wt. = 190. 5 g

x.4x2 .(50)2 .(50)


= 1020 Sol 29: At cathode, only reduction of water takes place
(50).(50)2 (2.5 – x)(5 – 2x)2
2H2O + 2e⁻ → H2(g) + 2OH–(aq)
x3
= 1020
(2.5 − x)(2.5 − x)2 9.722
Moles of H2 produced = =0. 434
22.4
x3
= 1020 Charge needed, Q = 2 × 0. 434 = 0. 868
(2.5 − x)3
At anode, for oxidation of water
x ≈ 2. 5 as k >>>1
2H2O → O2(g) + 4H+ + 4e–
Putting x = 2. 5 2.35
Moles of O2 produced, nO = = 0. 105
(2.5) 3 2 22.4
(2. 5 – x)3 = = 1. 5625 × 10–3
1020 Charge needed, q = 4 × nO = 0. 420
2
2. 5 – x = 2. 5 × 10 –7
Charge utilised for production of H2S2O8
2.5.x
[Ti+] = = 5 × 10–9 = Q. q = 0. 998
50
2(2.5 − x) charge utilised
[Co3+] = = 10–8 Moles of H2S2O8 = = 0. 224
50 2
Weight of H2S2O8 = 0. 224 × 194 = 43. 456 gm
Sol 27: For thiosulphate (S2O3 ), n = 2 2–

Number of equivalent = n. V. M. Sol 30: Initially, mass of H2SO4 solution


= 46. 3 × 0. 124 × 2 × 10–9 = 11. 4824 m. eq. = 1. 294 × 3. 5 × 1000 gm = 4. 529 kg
Equivalents of x = eq. of I– 39
Wt. of H2SO4 = ×4. 529 = 1. 76631 kg
1 100
eq.
=1× of I2 = × eq. of S2O32–
2 2 Moles of H2SO4 = 18. 02

11.4824 × 10−3 [H2SO4] = 5. 15 M


Eq. of x =
2
Finally, Mass H2SO4 solution = 1. 139 × 3. 5 kg
0.617 11.4824 × 10−3
=
M 2 = 3. 9865 kg
M = 107. 3 gm 20
Mass of H2SO4 = × 3. 9865 kg = 0. 7973 kg
100
Sol 28: At anode, Moles of H2SO4 = 8. 136
H2 +2OH → 2H2O + 2e
– –
[H2SO4] = 2. 324 M
67.2
Moles of H2 reacted = =3 ∴ ∆n = 9. 884
22.4 SO24−

Charge produced by 1 mole H2=2F 2F charge consumes 2 moles of [SO42–]


Chem i str y | 17.67

According to net reaction 0.373


\ Moles of Ag oxidised = = 0. 392
Charge required = 9. 884 F 0.95
9.884 × 96500 Mass of Ag oxidised = 0. 392 × 108 = 42. 41 g
Ampere hours = = 265 Amp. hr.
3600 Mass of electrode = 100 – 42. 41 = 57. 58 g

Sol 31: Cu2+ + 2e⁻ → Cu


Sol 34: Charge supplied per second
3
Moles of Cu deposited = = 0. 047 1.5 × 106
63.5 = = 15. 54 F
96500
Charge utilised = 2 × 0. 047 × F = 0. 094 F = 9071 C
Electrolysis:
Charge supplied = 3 × 2 × 3600 = 21600 C
2H2O → 2H2 + O2
Charge utilised 9071 2H2O → O2 + 4H+ + 4e⁻
∴ Efficiency = = = 0. 422
Charge supplied 21600
4 F charges electrolysed 2 moles of H2O.
∴ Efficiency = 42. 2 %
15.54
\ Moles of water electrolysed per second =
2
Sol 32: At cathode, = 7. 77
8.2 × 1012 × 103
Cu2+ + 2e⁻ → Cu Total moles of water =
18
0.4
Moles of Cu deposited = = 6. 3×10–3 8.2 × 1015
63.5 Time required in years = × 86400 × 365
18 × 7.77
Charge supplied = 2 × 6. 3 × 10–3 F = 1215. 74 C = 1. 9 million years
After deposition of copper,
1 Sol 35: Initial mass of electrolyte
H+ + e → H+ (as solution is acidic)
2 = 1. 261 × 1000 = 1261 g
Extra charge supplied = 1. 2 × 7 × 60
34.6
Wt. of H2SO4 = × 1261 = 436. 306
= 504 C = 5. 22 × 10–3 F 100
5.22 × 10−3 436.306
Moles of H2 evolved = = 2. 61 × 10–3 Moles of H2SO4 = = 4. 45
2 98
Volume of H2 evolved = 2. 61 × 10–3 × 22400 = 58. 46 mL Pb(s)+PbO2(s)+2H2SO4() → 2PbSO4(s)+2H2O()
At anode, t=0 4. 45
2H2O → O2 + 4H + 4e
+ –
t=t 4. 45–x x
Total charge,
we have, wt. of H2SO4 = 27
Q = 2 × 6. 3×10–3 × F + 5. 22 × 10–2 F = 1. 78 × 10–2 F
(4.45 − x)98
Q ∴ = 0. 27
Moles of O2 evolved = = 4. 45 × 10–3 12.61 × 436.606 + (4.45 − x)98 + x.18
4
Volume of O2 evolved = 4. 45 × 10–3 × 22400 = 99. 68 mL ⇒ 436. 608 – 98x = 0. 27(1261 – 80x)
Total volume = 158. 14 mL 463. 608 – 98x = 340. 47 –21. 6x
76. 4x = 96. 138
Sol 33: Ag → Ag + e
+ –
x = 1. 258
5 × 2 × 3600 Change in moles of SO42– = x = 1. 258
Total charge supplied = = 0. 373 F
96500
Pb + SO42– → PbSO4 + 2e
Now, since purity of electrode is 95%
Charge produced by 1 mole SO42– = 2F
Here, if charge will oxidise 0. 95 moles of Ag, rather
than 1 mole Charge produced by 1. 258 moles SO42–
= 2 × 1. 258 F = 2. 4 × 105 C
1 7 . 6 8 | Electrochemistr y

Sol 36: At anode, At cathode,


2H2O → O2 + 4H+ + 4e Cu → Cu2+ + 2e⁻
pH = 1 Final, [Cu2+] = 1 + 0. 9326 = 1. 9326
log[H+] = 1 Now, cell reaction,
[H+] = 0. 1 Zn + Cu2+ → Zn2+ + Cu
n = [H+] × v = 0. 1 × 10–1 = 10–2 E° = 1. 1 V, n = 2
H+

[H+] i = 10–7 0.0591 [Zn2+ ]  0.067 


E = E° – log = 1. 1 – 0. 0295 log  
2 2
[Cu ]+
 1.9326 
n = 10–7 × 10–1 = 10–8
H+
= 1. 1 – (0. 0295)(–1. 45)
Moles of H+ produced = 10–2 – 10–8
E = 1. 143 V
≈ 10–2

Charge needed = 10–2 F = 965 C Sol 38: At anode, for quinhydrone electrode,

Q 96.5 × 10 QH2 → Q + 2H+ + 2e⁻


Theoretical time = = = 1000 s
I 0.965 0.0591
E = E – log[H+] 2
Current efficiency = 80% 2

1000 × 5 Eº + 0. 0591 pH = –0. 699 + 0. 0591 pH


Therefore, actual time = s = 1250 s
4 (EºOP = – EºRP)
Now, at cathode
At cathode,
Cu2+(aq) + 2e– → Cu(s)
E = E = 0. 28
Current passed = 10 F –2

10−2 ∴Ecell = 0. 28 – 0. 699 + 0. 0591 pH


∴ Moles of copper reduced = = 5 × 10–3
2 Ecell = –0. 419 + 0. 0591 pH
After passing of current,
Equivalents of copper = equivalents of I2 (a) At pH = 5. 0,

= 2 equivalents of S2O3– Ecell = –0. 419 + (0. 0591) × (5) = –0. 124 V
Equivalents of copper (b) When Ecell = 0,
= 2 × 0. 04 × 35 × 10–3 0.419
pH = = 7. 1
Moles of copper = 31. 4 × 10 –3 0.0591
( n = 2 for Cu2+) (c) At pH = 7. 5

\ Total moles of Cu2+ = 6. 4 × 10–3 Ecell = 0. 0235 V


64 × 10−3 Since, Ecell is positive, reaction takes place in the given
Molarity = = 0. 064 M
100 × 10 −3 direction and cathode (Calomel electrode) is positive
electrode.
5 × 10 × 3600
Sol 37: Current passed = F = 1. 861 F
96500 Sol 39: We have, at anode
At anode
1
Zn2+ + 2e⁻ → Zn H → H+ + e–
Q 2 2
Moles of Zn2+ reduced = = 0. 9326
2 2H2 → 4H+ + 4e⁻
Final, [Zn2+] = 1 – 0. 9326 = 0. 067
E° = 0

E = –0. 0591 log [H+]


Chem i str y | 17.69

At cathode
[H+ ].[HS – ]
= k1
O2 + 2H2O + 4e → 4OH– [H2S]

Overall cell reaction k1 = 10–8, [H+] = 10–3, [H2S] = 0. 1

2H2 + O2 + 2H2O → 4H+ + 4OH– \[HS–] = 10–6M


1 HS– H+ + S2–
we have, H + H2O2 → H2O;∆H = –56. 7 kJ
2 2
10–6 10–3
2H2 + O2 → 2H2O;∆H= –2 × 56. 7 kJ = –113. 4 kJ
[S2− ].[H]+
H2O → H + OH ;∆H = 19. 05 kJ
+ – = k2
[HS – ]
4H2O → 4H+ + 4OH– [H+] = 10–3, [HS–] = 10–6, k2 = 1. 1 × 10–13
∆H = 4 × 19. 05 kJ = 76. 2 kJ [S2− ].10−3
= 1. 1 × 10–13
Adding, 10 −6
[S2–] = 1. 1 × 10–16 M
2H2 + O2 + 2H2O → 4H+ + 4OH–
E = –0. 64 – 0. 0295 log (1. 1 × 10–16)
∆H = –37. 2 kJ
= –0. 64 – 0. 0295(–15. 95)
 ∆E 
∆S = nF   = 4 × 96500×0. 001058 E = –0. 167 V
 ∆T P
∆S = 408. 38 1000
Sol 41: ∧eq = k ×
N
∆G = –nFEº = ∆H – T∆S
97.1 × 0.1
k= = 9. 71 × 10–3 Ω–1 cm–1
 ∆E  ∆H 1000
E° = T   −
 ∆T P ∆F 1
and, k =
E° = 0. 4414 V Ra
 0.5
R= =
Sol 40: Ag2S + 2e⁻ → 2Ag + S2– ka 9.71 × 10−3 × 1.5

Ag2S 2Ag2 + S2– R = 34. 32 Ω


V 5
∆G1 = –RT lnksp I= = Z
R 34.32
Ag+ × e⁻ → Ag; ∆G2 = –FEº, n = 1 I = 0. 1456 A
2Ag+ + 2e⁻ → 2Ag 6
V
Sol 42: E = = = 60 V/m = 0. 6 V/cm
∆G2 = –2FEº L 10 / 100
∧0
Ionic mobility (drift speed) = ∞ ×E
Ag2S + 2e⁻ → 2Ag + S2– F
73.52
∆G = ∆G1 + ∆G2 = –2FEº – RT ln ksp = ×0. 6 cm/s = 4. 57 × 10–4 cm/s
96500
–nFEº = –2FEº – RT ln ksp
Distance in 2 hours = 4. 57 × 10–4 × 2 × 3600 = 3. 29 cm
2.303 RT
E' = E + log ksp 1 
nF Sol 43: R =
k a
E' = –0. 64 V
 = k R a = 1. 342 × 170. 5 × 1. 86 × 10–4 = 0. 0425 m
0.0591
E = –0. 64 – log[S2–]
2  = 4. 25 cm
We have,
H2S H+ + HS–
1 7 . 7 0 | Electrochemistr y

0 0 0 500
Sol 44: ∧m SrSO4 = ∧m Sr2+ + ∧m SO42– nNaCl = = 8. 547
58.5
= 59. 46 + 79. 8 = 139. 26 ohm–1 cm2 mole–1
0.2346
k SrSO kNaCl+H = = 3. 08 × 10–5
0 2O 7600
∧m SrSO4 = 1000 × 4
C
kNaCl = kNaCl+H . kH
−4 2O 2O
1000 × 1.482 × 10
C= = (3. 08 – 2. 55) × 10–5
139.26
C = 1. 064 × 10–3 = 5. 3 × 10–6 Ω–1 cm–1

C = S = 1. 064 × 10–3 mole/L kNaCl × 1000


CNaCl =
∧m
= 1.064 × 10–3 × 183. 6 gm/L
5.3 × 10−6 × 1000
S = 0. 1953 gm/L = = 4. 190 × 10–5 mole/L
126.5
n
0
Sol 45: ∧m (Co2[Fe(CN)6] ) C=
V
0 0
= 2 ∧m (Co2+) + ∧m ([Fe(CN)6] +) n 8.547
V= = (1 L = 1 dm3)
C 4.19 × 10−5
= 2 × 86 + 444 = 616 Ω–1 cm–1 mol–1
V ≈ 2 × 105 dm3.
k Co [Fe(CN)6] = ksolution – kwater
2

= 2. 06 × 10–6 – 0. 41 × 10–6 Exercise 2


= 1. 65 × 10 Ω cm
–6 –1 –1
Single Correct Choice Type
= 1. 65 × 10–6 Ω–1 cm–1
Sol 1: (C) As E > EHg

> E
0 1000 Ag+ /Ag 2 /Hg Cu2 + /Cu
∧m =k×
C
Sol 2: (C) Mn3O4 + 16OH– → 3MnO42– + 8H2O + 10e⁻
k × 1000 1.65 × 10−6 × 1000
C= =
0
∧m 616 ∴ Charge required = 10 F

C = 2. 59 × 10–6 mol/L
Sol 3: (A) Only water will be oxidised and reduced at
S = C = 2. 59 × 10–6 mol/L anode and cathode respectively because SRP of SO42– is
very high (magnitude).
Co2[Fe(CN)6] 2Co2+ + [Fe(CN)6] 4–

ksp = (2s)2. S = 4S3 Sol 4: (C) At X

ksp = 7. 682 × 10–17 [A + ] a


pH = pka + log = pka + log
[HA ] b
0
Sol 46: For KCl, ∧m = 138 Ω–1 cm2 mol–1 1
H+ + e⁻ + H
2 2
1000 E1 = 0 + 0. 0591 log [H+] = –0. 0591 pH
∴ ∧m = k ×
C
E1 a
138 × 0.02 = –pka – log
k= = 2. 76 × 10–3 Ω–1 cm–1 0.0591 b
1000
At Y
G* = Rk
b
= 2. 76 × 10–3 × 85 = 0. 2346 cm–1 pH = pka + log
a
G* 0.2346 E2 = –0. 0591 pH
kwater = =
R 9200
E2 b a
= –pka – log = –pka + log
kwater = 2. 55 × 10–5 Ω–1 cm–1 0.0591 a b
Chem i str y | 17.71

E1 + E2 1
= –2pka H + Ag + Cl– → AgCl = H+
0.0591 2 2
E1 + E2 [H+ ]
E = Eº – 0. 0591 log
pka = – [Cl− ][PH ]1/2
0.118 2

k×t on increasing [H ] or decreasing [Cl–] , E will decrease.


+

Sol 5: (A) ∧eq =


C
9 × 10−6 Assertion Reasoning Type
C= × 10−3 = 6 × 10–5 M
1.5 × 10−4
Sol 12: (A) Cell Reaction:
Ag3PO4 3Ag+ + PO43– Zn(Hg) + HgO(s) → ZnO(s) + Hg()
[Ag+] = 3C = 1. 8 × 10–4 M
Sol 13: (D) E° = E°left – E°right
[PO43–] = 6 × 10–5 M
E° is intensive.
ksp = [Ag+] 3 [PO43–]
Sol 14: (A) These are facts.
ksp = 4. 32 × 10–18

Sol 6: (B) CH3COOH + NaOH → CH3COONa + H2O Comprehension Type

0.015 Paragraph 1
[CH3COONa] = = 0. 0075
2
Sol 15: (B) R = ρG*
1000
∧m = k ×
C for KCl, ∧m = 200 S cm2 mol–1
−4
6.3 × 10 × 1000 C ∧m 0.02 × 200
= = 84 S cm2 mol–1 k= =
0.0075 1000 1000
k = 4 × 10–3
Multiple Correct Choice Type G* = Rk = 100 × 4 × 10–3

Sol 7: (B, C) Eº > Eº > Eº > EºI G* = 0. 4


S2O2 /SO2
4
− Cl2 /Cl− Br2 /Br − 2 /I1
G* 0.4
species with higher E° can oxidise a species with lower E°. Sol 16: (C) kwater = = = 4 × 10–5
R 10 4

Sol 8: (B, C, D) In all case H2O will oxidise at anode to 585


give O2. Sol 17: (A) nNaCl = = 10
58.8
1 G* 0.4
Sol 9: (A, B) H2O → H2 + O kNaCl+H = = = 5 × 10–5
2 2 2O R 8000
270
nH = = 15 kNaCl = kNaCl+H – kH = 10–5
2O 18 2O 2O

VH = 15 × 22. 1 = 33. 6 L kNaCl × 1000


2 ∴CNaCl =
VO = 168 L, evolved at anode ∧m
2 −5
10 × 1000
VT = 504 L = = 8 × 10–5
125
10
Sol 10: (C, D) Refer text CNaCl =
V
10
Sol 11: (D, C) Ag + Cl– → AgCl + e V= = 1. 25 × 105 L
8 × 10−5
Cell reaction
1 7 . 7 2 | Electrochemistr y

Paragraph 2 1 –
Cl → Cl2 + e–
2
Sol 18: (B) NO3– + 4H+ → NO + 2H2O+ 3e(a)
PCl
NO3– + 2H+ + e → NO2 + H2O (b) E = E – 0. 06 log 2
[Cl− ]1/2
For (a), (B) → p, q
0.06 PNO
E=– log SEP of Na+ < SEP of H2O
3 [H ]4 [NO − ]
+
3 \ Water will get reduced to H2 at cathode
For (b),
PNO (C) → q, r
0.06 2
E = E° – log
Cl– will be oxidised as concentrated solution
1 [H+ ]2 [NO − ]
3
(D) → p, s
For (a), [NO3–] =[H+] = 1M, PNO = 10–3
SEP of Ag+ < SEP of H2O
0.06
ER = 0. 96 – log10−3 = 1. 02 ∴ Ag+ will be reduced to deposit Ag at cathode.
3
Cu → Cu2+ + 2e

EL = –0. 34 –
0.06
log[Cu2+ ] Previous Years’ Questions
2
= –0. 34 – 0. 03 log[Cu2+] Sol 1: (C) In electrolytic cell, electrolysis occur at the
cost of electricity :
EL = –0. 34 + 0. 03 log 10–1
At cathode : Mn+ + ne → M (electron gone in solution)
= –0. 31
At anode : Xn− → X + ne– (electron supplied to anode)
E = ER + EL ≈ 0. 71 V Therefore, electron is moving from cathode to anode
via internal circuit.
E for NO3 → NO,
10−3
E1 = 0. 65 – 0. 02 log Sol 2: (B) The cell reaction is :
[H+ ][NO3− ]
10−3 n + Fe2+ → Zn+ + Fe ; Ecell = 0.2905 V
E2 = 0. 48 – 0. 06 log
[H+ ]2 [NO3− ] 0.059 [Zn2+ ]
⇒ E = E – log
2 [Fe2+ ]
Sol 19: (C) E1 = E2 0.059 0.1
⇒ E = 0.2905 + log = 0.32 V
0. 65 – 0. 02(–3 – 5log[HNO3] ) 2 0.01
0.059
= 0. 48 – 0. 06(–3 – 3log[HNO3] ) Also, E = log K
n
0. 65 + 0. 06 + 0. 1 log[HNO3] 2E 0.32
⇒ log K = =
0.059 0.0295
= 0. 48 + 0. 18 + 0. 18 log[HNO3]
⇒ K = (10)0.32/0.0295
0. 08 log[HNO3] = 0. 71 – 0. 66 = 0. 05
5 Sol 3: (B) The net reaction is
log[HNO3] =
8 1
2H+ + O + Fe → H2O + Fe2+ + E = 1.67 V
[HNO3] = 100. 625 ≈ 100. 66 2 2
∆G  = –n E F
Match the Columns 2 × 1.67 × 96500
= kJ
1000
Sol 20 (A) → p, q
= –322.31 kJ
Cl– will not oxidise at low concentration.
Chem i str y | 17.73

Sol 4: (B) 0.01 mol H2 = 0.02 g equivalent Sol 10: (D) NH3 has no effect on the E° of glucose/
gluconic acid electrode.
⇒ Coulombs required = 0.02 × 96500 = 1930 C
⇒ Q = It = 1930 C Paragraph 2
1930 500
⇒t= = 19.3 ×104s Sol 11: (B) Moles of NaCl electrolysed = 4 × = 2.0
10 × 10 −3 1000
⇒ Moles of Cl2 produced = 1.0
Sol 5: (D) As AgNO3 is added to solution, KCl will be
2Cl– → + 2e–
displaced according to following reaction
AgNO3(aq) + KCl (aq) → AgCl(s) + KNO3(aq) Hg
Sol 12: (D) At cathode : Na+ + e– 
→ Na(Hg)
For every mole of KCl displaced from solution, one mole amalgam

of KNO3 comes in solution resulting in almost constant Two moles of Na formed during electrolysis would
conductivity. As the end point is reached, added AgNO3 produce two moles of Na(Hg) amalgam.
remain in solution increasing ionic concentration, hence ⇒ Mass of amalgam =2 × (23 + 200) = 446g
conductivity increases.

Sol 13: (D) Two Faraday of electric charge would be


Sol 6: (D) The half reactions are required for electrolysis of 2.0 moles of NaCl.
Fe(s) → Fe2+ (aq) + 2e– × 2 ⇒ total coulombs = 2 × 96500 = 193000 C
O2(g) + 4H+ → 2Fe2+ (aq) + 2H2O(l)
Paragraph 3
0.059 (10−3 )2
E =E –

log = 1.57 V
4 (10 −3 )4 (0.1) Sol 14: (C) For spontaneous redox reaction: Ecell > 0
For 2I– + Cl2 → 2Cl– + I2
Sol 7: (A, B, D) Metals with Eº value less than 0.96 V will
able to reduce NO3– in aqueous solution. Therefore, metals E° = 1.36 – 0.54 = 0.82 V > 0
V (E°= –1.19 V), i.e. Cl2 will spontaneously oxidize I–.
Fe(E° =–0.04 V), Hg(E° = 0.86 V) will all reduce NO3–
but Au (E° = 1.40V) cannot reduce NO3– in aqueous Sol 15: (D) In other cases Ecell < 0 , they are non-
solution. spontaneous.
For the reaction:
Paragraph 1
(i) 4Fe3+ + 2H2O → 4Fe2+ + 4H+ + O2;
Sol 8: (B) E for 2Ag+ + C6H12O6 + H2O → 2Ag(s) +

E = – 0.46 V
C6H12O7+ 2H+ is 0.75V
(ii) 4Mn3+ + 2H2O → 4Mn2+ + 4H+ + O2;
0.0592 E° = + 0.27 V
Also, E = log K
2
2Eº Sol 16: (A) As evidenced above, reaction (i) is non-
⇒ log K = = 25.33
0.0592 spontaneous, therefore, Fe3+ is stable in acid solution.
⇒ ln K = 2.303 log K = 58.35 However, reaction (ii) is spontaneous Mn3+ oxidises H2O
to O2 and itself reduced to Mn2+ in acidic medium.
Sol 9: (C) On increasing concentration of NH3, the Sodium fusion extract from aniline produces NaCN
concentration of H+ ion decreases, therefore, which reacts with Fe2+ to form [Fe(CN)6 ]4 − . The complex
ion then reacts with Fe3+ give blue precipitate of
0.0592
E=
red

Ered − log[H+ ]2 Prussian blue.
2
0.0592 Fe3+ + [Fe(CN)64– → Fe4 [Fe(CN)6 ]3
=0– × 2log 10 –11 =
0.65 V
2 Pr ussian blue

i.e. Ered increases by 0.65 V.


1 7 . 7 4 | Electrochemistr y

Paragraph 4 2
C  Λm  2
Ka1 0.01  1 
Sol 17: (B) M(s) + M+ (aq, 1 M) → M+ (aq, 0.05M) + M(s) =1 ×  2  = ×  = 0.001
Ka2 C2  Λ 0  0.1  10 
 m2 
2.303 RT 0.05
Ecell = 0 – log >0
F 1 pKa1 − pKa2 =
3
Hence, | Ecell | = Ecell = 0.70 V and ∆G < 0 for spontaneity
of reaction.
Sol 21: (D) Anode: H2 (s) → 2H+ + 2e−
0.0538
Sol 18: (C) Ecell = E – log 0.0025 = 0.139 V Mn+4 + 2e− → Mn+2
1 Cathode:
= 139 V Mn+4 + H2 → Mn+2 + 2H+

  +2   +  2 
Sol 19: X → Y. ∆rG0 = -193 kJ mol-1 0.059  Mn  H  
E= E − log10   
M+ → M3+ + 2e− E0 = −0.25V 2  Mn  PH 
+4
2
 
∆G0 for the this reaction is
∆G0 =−nFE0 =−2 × ( −0.25 ) × 96500 =48250 J / mol
0.092
= 0.151 −
0.059
2
( )
log10 10X

48.25 kJ / mole 0.059


0.092
= 0.151 − X
So the number of moles of M+ oxidized using X → 2
Y will be ⇒ X=
2
193
= = 4moles
48.25 Sol 22: (A) In a galvanic cell, the salt bridge does not
participate chemically in the cell reaction.
Sol 20: HK 

 +
H + X

H+   X − 
Ka =    
HX 

 + −
HY 
H +Y

H+   X − 
Ka =    
HX 

Λm for HX =
Λm
1

Λm for HX =
Λm
2

1
Λm = Λ
1 10 m2

Ka= C ∝2
2
Λ 
Ka= C ×  m1 
1 1  0 
Λ
 m1 
2
Λ 
Ka= C ×  m2 
2 2  0 
Λ
 m2 
2017-18 100 &
op kers
Class 12 T
By E ran culty
-JE Fa r
IIT enior emie .
S fP r es
o titut
Ins

CHEMISTRY
FOR JEE MAIN & ADVANCED
SECOND
EDITION

Exhaustive Theory
(Now Revised)

Formula Sheet
9000+ Problems
based on latest JEE pattern

2500 + 1000 (New) Problems


of previous 35 years of
AIEEE (JEE Main) and IIT-JEE (JEE Adv)

5000+Illustrations and Solved Examples


Detailed Solutions
of all problems available

Plancess Concepts
Topic Covered Tips & Tricks, Facts, Notes, Misconceptions,
Key Take Aways, Problem Solving Tactics
Chemical Kinetics
PlancEssential
Questions recommended for revision
18. CHEMICAL KINETICS

1. INTRODUCTION
Chemical kinetics deals with the study of rate of chemical reactions. Rate of reaction is defined as change in
concentration of reactants or products per unit time. It is influenced by various factors such as nature of substance,
physical state of substance, temperature, concentration, presence of catalyst, etc.

2. RATE OF REACTIONS
The rate of reaction is expressed in mol L–1 s–1.
Change in the concentration of reactants or(Products)
Rate of reaction =
Time

2.1 Average Rate of Reaction


Average rate of reaction is defined as change in the concentration of reactants (or products) during large interval
of time. If ∆C is the change in the concentration of reactants and products in Dt time, then
Change in the concentration of reactants or(Products) ∆C C − C1
Average rate = = ± ± 2
=
Time ∆t t2 − t1
The graph shows the progress of reaction with respect to time.
Concentration

Product

Reactant

Time
Concentration vs Time
Figure 18.1: Concentration vs Time

Unit of concentration gram mole / Litre


Unit of average rate of reaction = = = g mol L–1 sec–1
Unit of time second
1 8 . 2 | Chemical Kinetics

PLANCESS CONCEPTS

For gases, pressure is replaced by concentration. Thus, unit will be atm sec–1
n
PV = nRT or P =   RT or P = CRT
V
∆[P] ∆[C]
= ⋅ RT or Rate in [atm/sec] = Rate in [molarity/sec] ×RT
∆t ∆t
Vaibhav Krishnan (JEE 2009, AIR 22)

2.2 Expression of Rate of Reaction


For the following reaction
n1A + n2B → m1C + m2D

1 ∆[A] 1 ∆[B] 1 ∆[C] 1 ∆[D]


rate of reaction = – –
= =
+ =
+
n1 ∆t n2 ∆t m1 ∆t m2 ∆t

For the reaction 2H2O2 → 2H2O + O2

∆[O2 ] 1 ∆[H2O2 ]
The rate of reaction = =–
∆t 2 ∆t

2.3 Instantaneous Rate of Reaction


Instantaneous rate of reaction is defined as the average reaction rate during a very
short interval of time or rate of reaction at a particular time.
∆c dc
Instantaneous rate = ± Lim
Concentration

=
∆t →0 ∆t dt
B
Note: For the reaction n1A + n2B → m1C + m2D

1 d[A] 1 d[B] 1 d[C] 1 d[D]


Instantaneous rate of reaction = – –
= =
+ =
+
n1 dT n2 dT m1 dT m2 dT
Time
Figure 18.2: Instantaneous Plot

PLANCESS CONCEPTS

Rate of reaction and rate of change of concentration of a reactant (or product) are two different terms,
except when stoichiometric coefficient involved is unity. On the other, rate of reaction is equal to the
rate of change of concentration of a reactant (or a product) divided by the corresponding stoichiometric
coefficient.
Nikhil Khandelwal (JEE 2009, AIR 94)

2.4 Reaction Life Time


It is defined as the time taken to complete 98% of the reaction. The shorter the life time, the faster the reaction. Half-
life period is defined as either the time taken to complete half of the reaction or during which the concentration of
the reactant is reduced to one half of its initial value. It is denoted by t1/2.
Chem i str y | 18.3

Illustration 1: For the reaction R → P, the concentration of a reactant changes from 0.03 M to 0.02 M in 25 min.
Calculate the average rate of reaction using units of time both in minutes and seconds.  (JEE MAIN)

∆[R ] [R] – [R]1 0.02M – 0.030M –0.01M


Sol: Average rate = – =– 2 =– =–
∆t t2 – t1 25minutes 25minutes

0.01M
= 4 × 10–4 mol L–1 min–1 = = 6.66 × 10–6 mol L–1 sec–1.
25 × 60s

1
Illustration 2: Decomposition of N2O5 is expressed by the equation, N2O5 → 2NO2 + O2
2
If, during a certain time interval, the rate of decomposition of N2O5 is 1.8 × 10 mol L min–1. Calculate the rates of
–3 –1

formation of NO2 and O2 during the same interval. (JEE MAIN)

Sol: Decomposition of N2O5 has the following rate expression:

∆[N2O5 ] 1 ∆[NO2 ] ∆[O2 ]


–= = 2
∆t 2 ∆t ∆t
∆[NO2 ] ∆[N2O5 ]
So, =2 = 2 × 1.8 × 10–3 = 3.6 × 10–3 mol L–1 min–1
∆t ∆t
∆[O2 ] 1 ∆[N2O5 ] 1
and
= = × 1.8 × 10–3 = 9× 10–4 mol L–1 min–1
∆t 2 ∆t 2
∆[N2O5 ]
(Rate is always positive and hence – is taken positive.)
∆t

Illustration 3: During the analysis of the products, even when a portion of the mixture is removed, the rate of
reaction in the remaining mixture is not affected. Explain.  (JEE ADVANCED)

Sol: The rate of reaction depends on concentration, which remains constant even if a portion of the mixture is
removed, i.e. amount of substance is independent of the concentration and hence remains unaffected.

Illustration 4: For the reaction A + B → C + D, the rate law equation = K1 [A] + K2 [A] [B], where K1 and K2 represent
two different constants and the products are formed by two different mechanisms. Explain the relative magnitudes
of the rates of two individual mechanisms?  (JEE ADVANCED)

Sol: The rate of formation of products for both the reaction mechanisms must be of the same order of magnitude.
Different magnitude reflects only the rate of reaction of the one which is faster among the two mechanisms. Also,
it involves rate of formation of product from A as well as product from A and B.

3. RATE CONSTANT
Consider a simple reaction A → B
dx dx
At a particular instant, if CA is the molar concentration or acitve mass of A, then ∝ CA or = kCA
dt dt
where k is a proportionality constant, called velocity constant or rate constant or specific reaction rate.
dx
At a fixed temperature, if CA = 1, then rate = = k …(i)
dt
Thus, rate constant can be defined as rate of reaction at unit concentration of the reactants. Let us consider a
general reaction aA + bB  Product
 dx 
Rate =   ∝ [A]a [B]b =k [A]a [B]b when [A] = [B] = 1 mol/L, then rate = k
 dt 
1 8 . 4 | Chemical Kinetics

PLANCESS CONCEPTS

For a particular reaction, rate constant is independent of the concentration of the reactants and
dependent only on temperature.
Neeraj Toshniwal (JEE 2009, AIR 21)

3.1 Units of Rate Constant


For reactions of different order, unit for rate constant is as follows:
n−1
 1 
Unit of rate constant =   × time–1
 unit of concentration 
n−1 n−1
 1   litre 
=  × sec = 
–1
 × sec–1 where, n = order of reaction.
 mol / litre   mol 

PLANCESS CONCEPTS

When order of reaction is not mentioned explicitly, then it can be determined using the unit of rate
constant. For example, if the unit of rate constant is M-1sec-1, then order = 1-(-1)=2.
Aman Gour (JEE 2012, AIR 230)

3.2 Rate of Reaction and Reaction Rate Constant


Table 18.1: Difference between rate of reaction and reaction rate constant

S.No. Rate of reaction Reaction rate constant


1. Speed at which reactants converts into products. Proportionality constant.

2. Measured as the rate of decrease in concentration of Equal to the rate of reaction when the concentration of
reactants or the rate of increase in concentration of each of the reactants is unity.
products with time.

3. Depends upon the initial concentration of reactants. Independent of the initial concentration of the reactants
and has a constant value at fixed temperature.

3.3 Factors Affecting Rate of Reaction


(a) Concentration: According to the law of mass action the greater the concentration of the reactants, the more
rapidly the reaction proceeds.
(b) Pressure (Gaseous reaction): Increasing the pressure, decreases the volume and simultaneously increases
the concentration. Therefore the rate of reaction increases.
(c) Temperature: Increase in temperature increases the reaction rate. It has been observed that rate either
k
doubles or triples for every 10°C rise in temperature. Temperature coefficient of reaction rate, T +10 ≈ 2 or 3,
kT
where k T +10 and kT are rate constants at two temperature, differing by 10°C.
(d) Nature of the reactants: The rate depends upon specific bonds of the reactants involved in the reaction and
hence on the nature of reactants.
Chem i str y | 18.5

(e) Surface area of the reactants: In heterogeneous reactions, if the reactants are in more powdered form, then
velocity is greater [as more active centres are available].
(f) Catalyst: It has an immense effect on the rate of reaction. Positive catalyst increases the rate of reaction by
decreasing the activation energy, while negative catalyst decreases the rate of reaction by increasing the
activation energy. Catalyst mainly affects the activation energy of reaction and hence the rate constant and
rate of reaction changes.
(g) Intensity of radiation: The rate of photochemical reactions generally increases with increase in intensity of
radiation.

Illustration 5: The rate law of a chemical reaction 2NO + O2 → 2NO2 is K[NO]2 [O2 ].If the volume of reaction vessel
is reduced to 1/4th of its original value, then what will be the change in rate of reaction? (JEE MAIN)

Sol: For, 2NO + O2 → 2NO2 Rate = K[NO]2 [O2]


Let us take a moles of NO and b moles of O2 to start a reaction at any time in a vessel with a capacity of V L.
2
 a  b 
r1 = K      …(i)
V V
V
if volume of vessel is reduced to ,then the rate for same number of NO and O2 moles.
4
2 2
 a   b   a  b 
r2 = K     = 64K      …(ii)
V / 4 V / 4 V V
r
Dividing equation (ii) by (i), 2 = 64 i.e. r2 is 64 times of r1
r1

Ilustration 6: Dinitrogen pentoxide decomposes as follows: (JEE ADVANCED)

1 −d[N2O5 ] d[NO2 ] d[O2 ]


N2O5 → 2NO2 + O2 If = K1[N2O5], = K2 [N2O5] and = 2 = K3 [N2O5]
2 dt dt dt
Derive a relation between K1, K2 and K3.

Sol: For the given reaction

d[N2O5 ] 1 d[NO2 ] d[O2 ]


= =2 Putting given values
dt 2 dt dt
1
or K1[N2O5] = K [N O ] = 2K3[N2O5] or 2K1 = K2 = 4K3
2 2 2 5

3.4 Rate Law


(a) It may not depend upon the concentration of each reactant or product of the reaction.
m n
Suppose, mA + nB → Product R ∝  A  B  .

(b) Rate of chemical reaction is directly proportional to the concentration of reactants.


(c) The rate law represents the experimentally observed rate of reaction, which depends upon the slowest step
of the reaction.
(d) Rate law cannot be deduced from the equation for a given reaction. It can be found out by experiments only.
(e) The rate law may not bear a simple relationship for the stoichiometric equation.
(f) It may not depend upon the concentration of species, which do not appear in the equation for the overall
reaction.
1 8 . 6 | Chemical Kinetics

4. ORDER OF REACTION
The order of any reaction may be defined as the sum of the powers to which the concentration terms are raised in
order to determine the rate of reaction.
For the reaction: mA+nB → product
The experimental data suggests that
Rate =k[A]p[B]q; then the order with respect to A = p and the order with respect to B = q and the total order of the
reaction = p + q.
Reactions having order equal to 0 are zero-order reactions.
Reactions having order equal to 1, are Ist order reactions.
Reactions having order equal to 2 are IInd order reactions.
Reactions having order equal to 3 are IIIrd order reactions

PLANCESS CONCEPTS

•• Order can be zero, fractional or integer.


•• Stoichiometric coefficients m and n of the reactants are always not equal to orders p and q.

B Rajiv Reddy (JEE 2012, AIR 11)

5. MOLECULARITY OF A REACTION
It is defined as the number of molecules, atom or radicals that participate in the reaction. Depending on the
number of participating molecules, the reaction can be unimolecular, bimocelular, trimolecular, etc.
Participating molecules Molecularity
One molecule unimolecular, 1
Two molecule bimolecular, 2
Three molecule trimolecular, 3

PLANCESS CONCEPTS

•• Molecularity of a reaction is moslty between 1 to 3, but never zero. It is rare that molecularity exceeds 3.
•• Misconception: Order and molecularity are one and the same. But order may or may not be equal to
molecularity of the reaction.
Rohit Kumar (JEE 2012, AIR 79)

Table 18.2: Differences between order and molecularity

S. No. Molecularity Order of reaction


1. It is the total number of reacting molecules It is the sum of powers of molar concentrations of the
(atoms or ions) leading to chemical change. reacting molecules in the rate equation of the reaction.
2. It is always a whole number. It may be a whole number, zero, fractional, positive or negative.
Chem i str y | 18.7

3. It is a theoretical concept. It is experimentally determined.


4. It is meaningful only for simple reactions or It is meant for the reaction and not for its individual steps.
individual steps of a complex reaction. It is
meaningless for overall complex reaction.

Illustration 7: Write the order of the following reactions? (JEE MAIN)


Reaction Exp. rate equ.
H2 + Cl2 → 2HCl r = k[H2]0[Cl2]0
H2 + Br2 → 2HBr r = k[H2][Br2]1/2

H2 + I2 → 2HI r = k[H2][I2]

Sol: Order is the number to which the concentration of the reactants/products are raised corresponding mostly to
the coefficients.

Reaction Exp. rate equ. Order


H2 + Cl2 → 2HCl r = k[H2]0[Cl2]0 0
H2 + Br2 → 2HBr r = k[H2][Br2]1/2 3/2
H2 + I2 → 2HI r = k[H2][I2] 2

Illustration 8: For the reaction 2NO + Cl2 → 2NOCl At 300 K, following data are obtained:  (JEE ADVANCED)

Initial Concentration
Expt. No. Initial Rate
[NO] [Cl2]
1. 0.010 0.010 1.2 × 10–4
2. 0.010 0.020 2.4 × 10–4
3. 0.020 0.020 9.6 × 10–4

What is the rate law for the reaction and the order of the reaction? Also calculate the specific rate constant.

Sol: Let the rate law for the reaction be


Rate = k [NO]x [Cl2]y
From expt. (1), 1.2 × 10–4 =k [0.010]x [0.010]y  ….(i)
From expt. (2), 2.4 × 10–4 =k [0.010]x [0.020]y  ….(ii)
Dividing Equation (ii) by Equation (i),
2.4 × 10−4 [0.020]y
= or 2 = (2)y y = 1
−4 y
1.2 × 10 [0.010]
From expt. (2), 2.4 × 10–4 = k [0.010]x [0.020]y
From expt. (3), 9.6 × 10–4 =k [0.020]x [0.020]y
9.6 × 10−4 [0.020]x
Dividing Equation (iii) by Equation (ii), = or 4 = 2x, x = 2
2.4 × 10−4 [0.010]x
Order of reaction = x + y = 2 + 1 = 3
Rate law for the reaction is
Rate = k[NO]2 [Cl2]
1 8 . 8 | Chemical Kinetics

Considering Equation (i) again, 1.2 × 10–4 = k[0.010]2[0.010]


1.2 × 10−4
k= = 1.2 × 102 mol–2 L2 s–1
[0.010]3

6. REACTION OF VARIOUS ORDERS

6.1 Zero-Order Reactions


Rate of the reaction proportional to zero power of the concentration of reactants is
known as zero-order reaction. Consider the reaction
d[R]

Concentration of R
-K =Slope
R → P Rate = =k[R]0
dt
As any quantity raised to power zero is unity
d[R]
Rate = = k × 1 ; d[R] = – k dt
dt
Integrating on both sides [R] = – k t + I ....(i)
O
Time(t)

where I is the constant of integration. Figure 18.3: Zero order plot

At t = 0, the concentration of the reactant R = [R]0, where [R]0 is initial concentration of the reactant.
Substituting in equation (i)
[R]0 =– k × 0 + I [Rl0]=I
Substituting the value of I in the equation (i)
[R] = – kt + [R]0 ...(ii)
Equation of a straight line, y = mx + c. We get a straight line with slope = –k and intercept equal to [R]0, when [R]
is plotted against t.
[R ] – [R]
Simplifying equation (ii), we get the rate constant k = k = 0 ...(iii)
t
Zero-order reactions occur only under special conditions and hence they are uncommon. Some enzyme catalyzed
reactions and reactions which occur on metal surfaces are a few examples of zero-order reactions. Another example
of zero-order reaction is decomposition of gaseous ammonia on a hot platinum surface at high pressure.
1130K
2NH3(g) 
Pt catalyst
→ N2(g) + 3H2(g) ; Rate = k[NH3]0 = k

Here, platinum acts as a catalyst. The metal surface reacting with gas molecules under high pressure gets saturated.
So, a further change in reaction conditions does not alter the amount of ammonia on the surface of the catalyst
making rate of the reaction independent of its concentration. The thermal decomposition of HI on gold surface is
another example of zero-order reaction.

[R]0 [R]
(A) Unit of rate constant: k = k mol L−1 sec−1
t
Unit of rate of reaction and unit of rate constant are the same.

(B) Half-life period (t1/2): It is the time taken to complete half of the reaction.

R0 R0
At t = t1/2 ; R= t1/2 = or t1/2 ∝ R0
2 2K

It is directly proportional to the initial concentration of the reactant.


Chem i str y | 18.9

6.2 First-Order Reactions


Rate of the reaction is proportional to the first power of the concentration of the reactant. For example, R → P
d[R] d[R]
Rate = – = k [R] or – = –kdt
dt [R]
Integrating the above equation, we get ln [R] = – kt +I ...(iv)
I is the integration constant ; hence its value can be determined easily.
When t = 0, R= [R]0, where [R]0 is the initial concentration of the reactant.
Therefore, equation (iv) can be written as ln [R]0 = – k × 0 + I = I
Substituting I in equation (iv)
ln [R] = – kt + ln[R]0  ...(v)
[R] 1 [R]
Rearranging this equation ln = – kt or k = ln 0  ...(vi)
[R]0 t [R]
At time t1, from equation (iv) ln[R]1 = – kt1 + ln[R]0  …(vii)
At time t2, ln[R]2 = – kt2 + ln[R]0  ...(viii)
where [R]1 and [R]2 are the concentrations of the reactants at time t1 and t2 respectively. Subtracting (viii) from (vii)
[R]
ln[R]1 – ln[R]2 = – kt1 – (–kt2) ln 1 = k(t2 – t1)
[R]2
1 [R]1
k= ln  …(ix)
(t2 − t1 ) [R]2
[R]
Equation (v) can also be written as ln 1 = –kt
[R]2
Taking antilog on both sides [R] = [R]0 e–kt
When ln [R] is plotted against t ,we obtain a straight line with slope = –k and intercept equal to ln [R]0.
The first-order rate equation (vi) can also be written in the form

2.303 [R]
k= log 0 ...(x)
t [R] 
[R]0 kt
log =
[R] 2.303
If we plot a graph between log [R]0/[R] and t, the slope = k/2.303
Hydrogenation of ethene is an example of first-order reaction.
C2H4(g) + H2(g) → C2 H6(g)
Rate = k [C2H4]
All natural and artificial radioactive decay of unstable nuclei takes place by first-order kinetics.

In[R]
log ([R]0/[R])

K = -Slope
In[R]

Slope = k/2.303

O t O Time
Fig.: A plot between In[R] and t Fig.: Plot of log [R]0/[R] vs time
for a first-order reaction for a first-order reaction

Figure 18.4: First order plot


1 8 . 1 0 | Chemical Kinetics

Examples:
226
(a) 88 Ra →24 He + 86
222
Rn ; Rate = k[Ra]

(b) Decomposition of N2O5 and N2O are few more examples of first-order reactions.
2.303 [R]
(i) Unit of rate constant: K = log 0 = sec–1
t [R]
(ii) Half-life period (t1/2):

R0 2.303 R0 2.303 0.693


At t = t1/2, R = , K= log = × log 2 =
2 t1/2 R0 / 2 t1/2 t1/2
0.693
t1/2 =
K

6.3 Pseudo-Order Reactions


Reactions whose actual order is different from that expected using rate law expression are called pseudo-order
reactions, e.g
RCl + H2O → ROH + HCl
Expected rate law:
Rate = k [RCl][H2O]; expected order = 1+1 = 2
Actual rate law: Rate = k’ [RCl] Actual order = 1
Water is taken in excess; therefore, its concentration may be taken constant. The reaction is, therefore, pseudo first
order. Similarly, the acid catalyzed hydrolysis of ester, viz.,
RCOOR’ + H2O → RCOOH + R’OH
follows first order kinetics: Rate = k [RCOOR’]
It is also a pseudo first-order reaction.

6.4 Some Examples of First-Order Reactions and Their Rate Constants


(a) For Gas Phase Reaction
Let us consider a typical first-order gas phase reaction
A(g) → B(g) + C(g)
Let pi be the initial pressure of A and pt the total pressure at time ‘t’. Integrated rate equation is as follows:
Total pressure pt = pA + pB + pC (pressure units)
pA,, pB and pC are the partial pressures of A, B and C, respectively
Decreasing x atm pressure of A at time t to form one mole of B and C each.. The increase in pressure of B and C
will also be x atm each.
A(g) → B(g) + C(g)
At time, t = 0 pi atm 0 atm 0 atm
At time t (pi – x) atm x atm x atm
Where pi is the initial pressure at time t = 0.
Pt = (pi – x) + x + x = pi + x
x = (pt – pi), where pA= pi – x = pi – (pt – p i ) = 2pi – pt
Chem i str y | 18.11

 2.303   pi 
k=    log   ...(xi)
 t  pA 
2.303 pi
= log
t 2pi – pt

(b) Pseudo-Unimolecular Reaction:


Inversion of cane sugar
H +
C12 H22O11 + H2O → C6 H12O6 + C6 H12O6
Sucrose Glucose
Fructose
The progress of the reaction can be studied using a polarimeter. Cane sugar and glucose are dextrorotatory while
fructose is laevorotatory. Let q0 is polarimetric reading at t = 0; qt is polarimetric reading after time t and θ∞ is
polarimetric reading after infinite time.
2.303 θ − θ∞
k= log 0
t θt − θ∞

(c) (i) Decomposition of H2O2


Pt 1
H2O2 → H2O + O
2 2
The progress of the reaction can be studied by two methods. (1)Volume of oxygen gas at different intervals of
times is measured and or (2) definite amount of reaction mixture is titrated with standard KMnO4 at different
intervals of time. If V0 represent the initial volume of KMnO4 and Vt final volume of KMnO4 at any time t
Then, R0 ∝ V0 ,R ∝ Vt
2.303 V
Therefore, K =– log 0
t Vt

(ii) Decomposition of Ammonium Nitrite


NH4NO2 → N2 + 2H2O
If Vt is the volume of N2 evolved at any time t and V∞ is the volume of N2 evolved when the decomposition is
complete. Then,
2.303 V∞
K= log
t V∞ − Vt

(d) Hydrolysis of Ethyl Acetate (Ester)


H +
CH3COOC2H5 + H2O → CH3COOH + C2H5OH

Kinetics of this reaction is studied by titrating a definite volume of the reaction mixture with standard alkali solution.
If V0 ,Vt and V∞ are volumes of standard alkali needed to neutralize a definite amount of x, will be proportional to
Vt – V0 and a, will be proportional to V∞ – V0
2.303 V − V0
Hence, K = log ∞
t V∞ − Vt

(e) Oxide Layer Formation


1 τmax
K= ln where tmax is the thickness of oxide layer after ∞ times and τ is the thickness of oxide layer at
t τmax − τ
time ‘t’.
1 a+x
(f) Bacterial Growth: K = ln
t a
1 8 . 1 2 | Chemical Kinetics

6.5 Second-Order Reaction


A reaction is said to be of second order if its reaction rate involves two different concentrations.
The kinetics of second-order reactions are given as follows:
(a) When concentrations of both reactants are same or two molecules of the same reactant are involved in the
change, i.e.
dx
A + B → Products or 2A → Products; = k(a – x)2,
dt
1 x
k= ⋅
t a(a − x)
Where a is the initial concentration of the reactant or reactants and x is the concentration of the reactant
changed in time t.
(b) When the initial concentrations of the two reactants are different, i.e.,
A + B → Products
Initial conc. a   b
dx 2.303 b(a − x)
= k(a – x)(b – x) ; log10
dt t(a − b) a(b − x)
(a - x)and (b - x) are the concentrations of A and B, respectively, after time interval, t.

Characteristics of the second-order reactions


(a) The value of k (velocity constant) depends on the unit of concentration.
The unit of k is expressed as (mol/litre)–1 time–1 or litre mol–1 time–1.
1
When ‘t’ is plotted against the resultant slope is a straight line.
(a − x)
1 1
Rearranging equation for k. t = .

Time

k(a − x) ka
1
The slope of the line is , where ‘k’ can be evaluated.
k
1/(a-x)
1 0.5a 1
(b) Half-life period (t1/2) = ⋅ = Figure 18.5 Second order plot
k a × 0.5a ka
Thus, half-life period is inversely proportional to initial concentration.
(c) Second-order reaction confirms to the first order when one of the reactants is present in large excess.
2.303 b(a − x)
Consider, k = log . If a >>> b, then (a – x) ≈ a and (a – b) ≈ a
t(a − b) 10 a(b − x)
2.303 ba 2.303 b
Hence, k= log10 or ka = k’ = log10
ta a(b − x) t (b − x)
here since ‘a’ being very large is a constant after the change. Thus, the reaction follows first-order kinetics with
respect to the reactant taken relatively in small amount.

Examples of second-order reactions


(a) Hydrolysis of ester by an alkali (saponification).
CH3COOC2H5 + NaOH → CH3COONa + C2H5OH
(b) The decomposition of NO2 into NO and O2. 2NO2 → 2NO + O2
(c) Conversion of ozone into oxygen at 100°C. 2O3 → 3O2
(d) Thermal decomposition of chlorine monoxide. 2Cl2O → 2Cl2 + O2
Chem i str y | 18.13

6.6 Third-Order Reaction


Reaction is said to be of third order if its reaction rate involves three different concentrations. When the concentration
of all the three reactants is same or three molecules of the same reactant are involved, the rate expression is given as:
dx
3A → Products; A + B + C → Products; = k(a – x)3
dt
1 x(2a − x)
Therefore k= ⋅
t 2a2 (a − x)2

Characteristics of third-order reactions

1 0.5a(2a − 0.5a) 1 0.5a × 1.5a 3


•• Half-life period = ⋅ = =
k 2
2a (0.5a) 2 k 2a × 0.5a × 0.5a 2a2k
2

Thus, half-life is inversely proportional to the square of initial concentration.


•• The change in the unit of concentration changes the numerical value of k.
•• The unit of k is expressed in (mol/litre)–2 time–1 or L2 mol–2 sec–1.

Examples of third-order reactions


•• Reaction between nitric oxide and oxygen. 2NO + O2 → 2NO2

•• Reaction between nitric oxide and chlorine. 2NO + Cl2 → 2NOCl

Examples
For a zero-order reaction, as shown in the following figure, the plot of [A] versus time is a straight line with
k = -ve slope. Other graphs are curved for a zero-order reaction.
[A] vs time In [A] vs time 1/[A] vs time
0.20 150
-2.0
0.15 In[A] 100
[A] k = - slope -3.0 1
mol A
L
-4.0 50
0.05

0.00 -5.0 0
20 40 60 0 20 40 60 20 40 60
Time (sec) Time (sec) Time (sec)
Figure 18.6: Plot of [A] versus time

For a first-order reaction, as shown in the following figure, the plot of the logarithm of [A] versus time is a straight
line with k = - slope of the line. Other graphs are curved for a first-order reaction.
[A] vs time In[A] vs time 1/[A] vs time
0.20
-2.0
k = - slope 60
0.15
In[A]
[A] 40
mol -3.0
L 1
[A]
0.05 -4.0 20

0.00 -5.0 0
5 10 15 20 0 5 10 15 20 5 10 15 20
Time sec. Time sec Time sec.

Figure 18.7: Plot of the logarithm of [A] versus time


1 8 . 1 4 | Chemical Kinetics

For a second-order reaction, as shown in the following figure, the plot of 1/[A] versus time is a straight line with
k =-ve slope of the line. Other graphs are curved for a second-order reaction.

[A] vs time In[A] vs time 1/[A] vs time


0.20 50
-2.0
0.15 40
In[A]
k = - slope
[A]
mol -3.0 1
L [A]
0.05 -4.0
10

0.00 -5.0 0
100 200 300 0 100 200 300 100 200 300
Time sec. Time sec Time sec.

Figure 18.8: Plot of 1/[A] versus time

6.7 Parallel Reactions

k1 B
[A]0 [B] k1
A ln = (k1 + k2)t =
[A]t [A] k 2
k2 C
k1 [A]0 k 2 [A]0
[A] = [A0] e–kt [B] = (1 – e–kt) [C] = (1 – e–kt)
k1 + k 2 k1 + k 2
Variation of concentration A, B and C with time may be graphically represented as,

Examples:

OH

NO2 B
k1
OH (Main)
C
Conc.
HNO3 OH

A
k2 Time
(Side)

NO2
Figure 18.9: Parallel reactions

6.8 Consecutive Reactions


k k
A → B 
1 2→ C
k1 [A]0  −k1t
– e 2 
−k1t −k t
[A]t = [A0] e ; [B]t =  e
[k 2 − k1 ]  
k2
 k  k2 −k1
[C]t = [A0] – ([A]t + [B]t) ; [B]max = [A0] .  1 
 k2 
1 K
tmax = ln 1
K1 − K 2 K 2
Chem i str y | 18.15

Conc.
[C]

[B]
[A]

Time

Figure 18.10: Consecutive reactions

Examples: (i) Decomposition of ethylene oxide:


k
(CH2)2O → CH3CHO
1

k
CH3CHO 
2→ CO + CH4

6.9 Reversible Reactions


1 k

A ← →B

k 2
Initial state (t = 0) a 0
Conc. at time (t) a – x x
2.303  x  xeq K
Equilibrium conc. (a – xe) xe ; k1 + k2 = log10  e  = f
t x
 e − x  a − x eq K b

Illustration 9: If the time required to decompose SO2Cl2 to half of its initial amount is 60 minutes and the
decomposition is a first-order reaction, then calculate the rate constant of the reaction. (JEE MAIN)
0.693
Sol: For a first-order reaction. k =
t1/2
0.693 0.693 0.693
= = = 1.925 × 10−4 s–1
t1/2 60 minutes 60 × 60 seconds

Illustration 10: Consider a first-order reactions, if it takes 5 minutes for the initial concentration of 0.6 mol L–1 to
become 0.4 mol L–1 Then how long will it take for the initial concentration to become 0.3 mol L–1? (JEE MAIN)

2.303 [A]
Sol: For a first-order reaction k = log 0
k [A]t

We have, [A]0 = 0.6 mol L–1 [A]t = 0.4 mol L–1 t = 5 min

2.303 0.6 2.303 2.303 × 0.1761


So, k = × log =log 1.5 min–1 = min–1 = 8.1 × 10–2 min–1
5min 0.4 5 5

2.303 [A] 2.303 0.6 2.303


For, [A]t = 0.3 mol L–1 ; t = log 0 = log = log 2 = 8.5min.
k [A]t 8.1 × 10 −2 0.3 8.1 × 10−2

Illustration 11: The following were obtained during the first order thermal decomposition of N2O5 (g) at constant
volume:
2N2O5(g) → 2N2O4(g) + O2(g)
1 8 . 1 6 | Chemical Kinetics

S.No. Time (sec) Total Pressure (atm)


1. 0 0.5
2. 100 0.512

Calculate the rate constant. (JEE ADVANCED)

Sol: Let the pressure of N2O5(g) decrease by 2x atm when two moles of N2O5 decompose to give two moles of N2O4
(g) and one mole of O2(g). Therefore pressure of N2O4(g) increases by 2x atm and that of O2(g) increases by x atm.

2N2O5(g) → 2N2O4(g) + O2(g)


At t = 0 0.5 atm 0 atm 0 atm
At time t (0.5 × 2x) atm 2x atm x atm
Pt = PN2O5 + PN2O4 + PO2 = (0.5 × 2x) + 2x + x = 0.5 + x
x = pt – 0.5 = 0.5 – 2(pt – 0.5)= 1.5 – 2pt
At t = 100 s; pt = 0.512 atm
PN2O5 = 1.5 – 2 x 0.512 = 0.476 atm
2.303 P 2.303 0.5atm 2.303
Using equation (11) k = log i = log = × 0.0216 = 4.98 × 10–4 s–1
t PA 100s 0.476 atm 100s

Illustration 12: In a second-order reaction, in which both the reactants have same concentration, consider 20% of
the reaction is completed in 500 sec. What is the time required for 60% completion? (JEE ADVANCED)

Sol: The equation for second order with both the reactants having same concentration is
1 x
k= ⋅ If a = 100 , x = 20, t = 500 sec.
t a(a − x)
1 20
So, k = × When a = 100, x = 60, t = ?
500 100 × (100 – 20)

1 60
t= ⋅
k 100 × 40
Substituting the value of k,
500 × 100 × 80 60
t= × or t = 3000 sec
20 100 × 40

7. THE REACTION MECHANISM

7.1 Elementary Reactions


Elementary reaction are single step reactions in which order and molecularity are always the same. In another
words, rate law and the law of mass action have the same expression.

7.2 Complex Reactions


These are multi-step reactions in which order and molecularity may or may not be same.
(a) In these types of reactions, intermediates are formed during the reaction which is different from the reactants
and the products formed.
(b) Each step involved in this reaction is an elementary reaction.
Chem i str y | 18.17

(c) The overall rate of reaction will be equal to the rate of slowest step. Hence the slowest step is called the rate
determining step (RDS) of reaction. A reaction can have more than one RDS.
(d) The overall molecularity of reaction and the molecularity of the RDS are the same . However, it has no
significance.
(e) The order of reaction and the overall order of reaction may not be the same. It depends on the concentrations
involved in the rate law expression of RDS. If it is not overall order, it may be determined by equilibrium
approach or by steady state approximation.
For a complex reaction, order is not equal to molecularity.

7.3 Reaction Mechanism


The steps involved in a reaction and determining which step is the slowest or rate determining is called mechanism.
A series of steps (known as elementary processes) leading to the formation of products or by which the overall
chemical reaction occurs is called the reaction mechanism.
For e.g.
The reaction between H2 and I2 to form hydrogen iodide was originally postulated as a simple one-step reaction.
H2 + I2 = 2HI
  Rate = k[H2][I2]
But, the formation of HI has been explained on the basis of the following mechanism:
I2 → 2I (fast)  ….(i)
H2 + I → H2I (fast)  ...(ii)
H2I + I → 2HI (slow)  ...(iii)

Overall reaction: H2 + I2 → 2HI

8. DETERMINATION OF RATE LAW FOR A COMPLEX REACTION

8.1 Equilibrium Approach


The concentration of intermediate can be determined from equilibrium constant of the reaction involved.
For example, let the mechanism of reaction A + 2B → C + D is
1 k
Step I: 
A + B ← → I

k 2

k
Step II: I + B 
2→ C+D
Overall rate of reaction, r = rate of step II = k2[I][B]  ....(i)
But this cannot be considerd as correct because the overall rate of reaction should be in terms of concentrations
of A and B (reactants). The concentration of I can be related with the concentrations of A and B with the help of
first equilibrium.
k1 [I] k1
For step I, equilibrium constant, Keq = = \ [I] = [A][B]
k −1 [A][B] k −1
k
Putting this value in equation (i), r = k2 k1 [A][B].[B] = k[A][B]2, where, k = 1
k −1 k −1

Hence, the overall rate of reaction is 1 + 2 = 3


1 8 . 1 8 | Chemical Kinetics

8.2 Steady State Approach


In this method, we assume that the intermediates formed are so reactive that after some time from initiation of
reaction (called induction period), the net rate of their formation becomes zero. They react with the same rate of
their formation. For example, let the mechanism of reaction
A +2B → C + D is
1 k
Step I: 
A + B ← → I

k 2
k
Step II: I + B 
2→ C+D

d[C]
The rate of reaction may be given as r = + = k2 [I] [B] …(ii)
dt
d[I]
Now, from steady state approximation on the intermediate, I – =0
dt
k1 [A][B]
or k1 [A][B] – k–1[I] – k2[I][B] = 0 or, [I] =
k −1 + k 2 [B]
k1 [A][B] k1k 2 [A][B]2
Substituting this value in equation (ii), r = k2 · ·[B] =
k −1 + k 2 [B] k −1 + k 2 [B]

PLANCESS CONCEPTS

•• When the intermediate is less stable, better results are obtained by steady state approximation, when the
intermediate is more stable, better results are obtained by eqilibrium approach. More stable intermediate
reacts very less and hence, the concentration of intermediate at any time remains nearly equal to its
equilibrium concentration.
•• When conditions of equilibrium approach are applied on the result obtained from steady state
approximation, the same rate law expression is obtained. For the above reaction, step I is faster than step
II. As step I is at equilibrium,
r1 = r–1 >> r2 or,k–1[I]>>k2[I][B] or, k–1>>k2[B]
therefore, k–1+k2[B] ≈ k–1
Now,the rate expression obtained from steady state is
k 2k1 [A][B]2
r= ≈ [A][B]2
k −1 + k 2 [B]
Which is exactly same expression obtained from equilibrium approach.

Aishwarya Karnawat (JEE 2012, AIR 839)

Illustration 13: The reaction 2NO + Br2 → 2NOBr obeys the following mechanism:
fast
Step I: 
NO + Br2 ← → NOBr
 2

Slow
Step II: NOBr2  → 2NOBr

Suggest the rate expression. (JEE MAIN)

Sol: The slowest step is the Rate determining step.


Chem i str y | 18.19

Step II is the step that determines the rate of the reaction and hence, r = k[NOBr2][NO] …(i)
However, NOBr2 is an intermediate and thus its concentration is determined step I.
[NOBr2 ]
For step I, equilibrium constant keq = \ [NOBr2] = keq[NO] [Br2]  …(ii)
[NO][Br2 ]
Thus, by equations (i) and (ii), r = k. keq[NO]2[Br2] or, r = k’[NO]2 [Br2]
where k’ = k. keq

Illustration 14: The following mechanisms are proposed for the reaction
CO + NO2 → CO2 + NO at low temperature:
(a) 2NO2 → N2O4 (fast) N2O4 + 2CO → 2CO2 + 3NO (slow)
(b) 2NO2 → NO3 + NO (slow) NO3 + CO → NO2 + CO2 (fast)
d[CO2 ]
Which of the above mechanism are consistent with the observed rate law: + = k[NO2]2 (JEE ADVANCED)
dt

Sol: For mechanism (a), r = rate of step II = k[N2O4][CO]2  ....(i)


[N2O 4 ]
Now, from step I: keq = or, [N2O4] = keq[NO2]2
2
[NO2 ]
Hence from (i). r = k.keq [NO2]2 [CO]2, but it is not the given rate law
Hence, the mechanism is not consistent with the rate law.
For mechanism (b), r = rate of step, I = k[NO2]2, which is the given rate law.

9. COLLISION THEORY OF REACTION RATE/ARRHENIUS THEORY


Postulates:
•• Collision of reactant molecules leads to chemical reaction. The number of collisions taking place per unit time
per unit volume of the reaction mixture is known as collision frequency (Z). The value of collision frequency is
very high (of the order of 1025 to 1028) in case of binary collisions.
•• Not every collision brings a chemical change. Only effective collisions bring about a chemical change to form
products. The effective collisions are less when compared to the total number of collisions. Collisions that
do not form a product are ineffective elastic collisions, i.e. molecules just collide and disperse in different
directions with different velocities. For effective collision, the following two barriers are to be cleared.

9.1 Energy Barrier


The minimum energy required by colliding molecules for the chemical reaction to take place is known as threshold
energy.
Fraction of molecules

Fraction of molecules
capable of bringing
effective collisions

Energy E

Figure 18.11: Threshold Plot


1 8 . 2 0 | Chemical Kinetics

The minimum amount of energy required by reactant molecules to participate in a reaction is called activation
energy.
Activation energy = threshold energy – average kinetic energy of reacting molecules.
Threshold energy = initial potential energy of reactant molecules + activation energy.
Collision of high energy molecules that overcomes the forces of repulsion and forms an unstable molecule cluster
is called an activated complex. Its life span is very less. Thus, the activated complex breaks to either form reactants
again or form new substances, i.e. products. The activation energy (Ea) depends upon the nature of chemical bonds
undergoing rupture and is independent of enthalpies of reactants and products.
The following graphs show the energy changes during exothermic and endothermic reactions versus the progress
of the reaction.
Activated Activated
Complex Complex

Ea
Ea

E C + D Products

Energy
Energy

Reactants Reactants
A+B E A+B

Products
C+D
Progress of the reaction Progress of the reaction
(Exothermic) (Endothermic)

Figure 18.12: Energy profile diagram for exothermic and endothermic reaction

(a) Every reaction, whether exothermic or endothermic, needs to overcome an energy barrier for reactants to get
converted to products.
(b) Lower the activation energy, higher will be the fraction of effective collisions leading to faster reaction and
vice versa.
(c) Activation energy Ea = E(activated complex) – E(ground state)
∆H = activation energy of forward reaction – activation energy of backward reaction.
(i) When Ea,f < Ea,b ; ∆E = –v and, ∆H = –ve
Thus, when the activation energy for the forward reaction is less than that for the backward reaction,
energy is released.
(ii) When Ea,f > Ea,b ; ∆E = + ve and, ∆H = + ve
Thus, when the activation energy for the forward reaction is more than that for the backward reaction,
energy is absorbed.

9.2 Orientation Factor


•• To form products reactants should have a proper orientation in addition to sufficient energy.
•• Rate of reaction is directly proportional to the number of effective collisions.
dx
Rate = – = collision frequency × fraction of effective collisions
dt
= Z × f
Chem i str y | 18.21

A A
A A A A

B B
B B
B B
Reactants Products
Figure 18.13: Collision theory

9.3 Temperature Dependance of Rate Constant


The temperature dependence of the rate of a chemical reaction can be accuratly explained by Arrhenius Equation
k = Ae-Ea/RT ...(i)
where, k=Arrhenius factor, Ea=activation energy, R=gas constant, T=temperature
According to the Arrhenius equation, for most reactions, the rate constant increases with increase in temperature. :
The parameters A and Ea for a given reaction are collectively called Arrhenius parameters. Value of Ea is determined
1
from the graph for and ln k is determined experimentally. Value of A inturn is calculated once Ea is known.
T
In the Arrhenius equation (i), the factor e–Ea/RT corresponds to the fraction of molecules with kinetic energy greater
than Ea.
Ea
Taking natural logarithm of both sides of equation (i), we get ln k = ln A –  ....(ii)
RT
The plot of ln k versus 1/T gives a straight line according to the equation (ii) as shown in the below figure.

Intercept = ln

In k Slope = -Ea/R
A A
log log
R R

0 1/T 1
T
A plot between In k vs 1/T T

Figure 18.14: Plot of ln k versus 1/T

Thus, it has been found from Arrhenius equation (i) that increase in temperature or decrease in activation energy
results in an increase in the rate of the reaction and an exponential increase in the rate constant.
Ea
In the plot, slope = – and intercept = ln A. So we can calculate Ea and A using these values.
R
Ea
At temperature T1, equation (ii) becomes ln k1 = – + ln A  ….(iii)
RT1
Ea
At temperature T2, equation (ii) becomes ln k2 = – + ln A  ….(iv)
RT2
(since A is constant for a given reaction)
k1 and k2 are the values of rate constants at temperatures T1 and T2 respectively. Subtracting equation (iii) from (iv),
we obtain

Ea Ea k2 Ea  1 1  k2 Ea  1 1 
ln k2 – ln k1 = – ; ln =  –  ; log =  – 
RT1 RT2 k1 R  T1 T2  k1 2.303R  T1 T2 
1 8 . 2 2 | Chemical Kinetics

Maxwell’s–Boltzmann Distribution Curve:


The peak of the curve corresponds to the most probable kinetic energy, i.e. kinetic energy of maximum fraction of
molecules. Number of molecules decreases with energies higher or lower than this value. When the temperature
is raised, the maximum of the molecules moves to the higher energy value (figure) and the curve broadens out,
i.e. spreads to the right such that there is a greater proportion of molecules with much higher energies. The area
under the curve must be constant since total probability should always be unity at any given time. We can mark
the position of Ea on Maxwell Boltzmann distribution curve.

t
Fraction of molecules

(t + 10) Energy of
activation
This area shows fractional
This area shows of additional molecules
fraction of which react at (t + 10)
molecules
reacting at t

Kinetic energy
Figure 18.15: Distribution curve showing temperature dependence of rate of a reaction

Increase in the temperature of the substance increases the fraction of molecules, which collide with molecules with
energies greater than Ea. It is clear from the diagram that in the curve at (T +10), the area showing the fraction of molecules
having energy equal to or greater than activation energy gets doubled leading to doubling the rate of a reaction.

9.4 Effect of Catalyst


A catalyst is a substance which alters the rate of a reaction without undergoing any permanent chemical change.
For example, MnO2 catalyses the following reaction so as to increase its rate considerably
2KClO3 
MnO
2 → 2KCl + 3O
2

Reaction path
without catalyst Energy of
activation
Energy of
Potential Energy

without
activation
Reaction catalyst
with
Reactants path with catalyst
catalyst

Products

Reaction coordinate

Figure 18.16: Effect of catalyst on activation energy

Catalytic Mechanism: The mechanism of the catalyst can be explained by intermediate complex theory. According
to this theory a catalyst reacts with the reactant to form temporary bonds resulting in an intermediate complex.
This has a transitory existence and decomposes to yield products and the catalyst.
It is believed that the catalyst provides an alternate pathway or reaction mechanism by reducing the activation
energy between reactants and products and hence lowering the potential energy barrier.
−Ea /RT
According to Arrhenius equation, k = Ae , the lower the value of activation energy, the faster will be the rate
of a reaction.
Chem i str y | 18.23

PLANCESS CONCEPTS

For catalysts
•• Even a small amount of the catalyst has the ability to catalyze a large amount of reactants.
•• Gibbs energy, ∆G, of a reaction is independent of the catalyst.
•• It has an effect on spontaneous reactions but does not catalyse non-spontaneous reactions.
•• Catalyst does not change the equilibrium constant of a reaction, rather it helps to attain equilibrium
faster, i.e. it catalyses the forward as well as the backward reaction to the same extent so that the
equilibrium state remains same and is reached earlier.

For collision theory


•• Activation energy and proper orientation of the molecules together determine the criteria for an
effective collision and hence the rate of a chemical reaction.
•• It considers atoms/molecules to be hard spheres and ignores their structural aspect.
•• Comparing the equation (vi) with Arrhenius equation, we can say that A is related to collision
frequency.
Saurabh Chaterjee (JEE Advanced 2013, AIR)

Note: In the following reaction formation of methanol from bromo-ethane depends upon the orientation of
reactant molecules. Only proper orientation of reactant molecules lead to bond formation, else it simply bounce
back and no products are formed.
CH3 +OH– → CH3OH + Br –

H + -
Improper -
H C Br OH Products
Orientation
H + - H
-
H C Br OH
H H H
Proper - + -
-
Orientation HO C Br OH C H + Br

H H H
Intermediate

Steric Factor: For effective collisions, the probability or steric factor (P) is introduced. It helps the molecules for a
−E /RT
proper oriented i.e., Rate = PZAB e a

Illustration 15: What will be the effect of temperature on rate constant? (JEE MAIN)

Sol: Rate constant of a reaction is nearly doubled with rise in temperature by 10°C. The dependence of the rate
−Ea /RT
constant on temperature is given by Arrhenius equation, k = Ae , where A is called frequency factor and Ea is
the activation energy of the reaction.

Illustration 16: How is the value of activation energy calculated from the rate constants at two different
temperatures. If the value of activation energy is 50 kJ/mol then show that by increasing the temperature from 300
K to 310 K, rate constant becomes nearly double. (JEE MAIN)
Ea 1
Sol: We know that Arrhenius equation can be written as log k = log A –
2.303R T
1 8 . 2 4 | Chemical Kinetics

If k1 and k2 are the rate constants at two different temperature T1 and T2 then Arrhenius equation at both the
temperatures can written as
Ea 1 Ea 1
log k1 = log A – ; log k2 = log A –
2.303R T1 2.303R T2
Ea 1 1  k Ea  1 1 
Subtracting equation (i) from (ii), we get log k2 – log k1 =  −  or =log 2  − 
2.303R  T1 T2  k1 2.303R  T1 T2 

From the values of k1 and k2 at temperature T1 and T2, Ea can be calculated.


Given T1 = 300K, T2 = 310 K, Ea = 50 kJ or 50,000 J
k 50000  1 1  50000  310 − 300 
Put these values =
in eq. (iii), log 2  −  =  
k1 2.303 × 8.314  300 310  2.303 × 8.314  310 × 300 
k2
or log = 0.2808 = Antilog 0.2808 = 1.91 ≈ 2
k1
It is clear that by increasing the temperature from 300 K to 310 K doubles the rate constant.

Illustration 17: Even an exothermic reaction has activation energy. Why? (JEE ADVANCED)
Sol: A reaction can either be endothermic or exothermic, but it possesses a minimum energy level for the molecules
to cross the energy barrier for the reaction to take place. Also
Threshold energy = ΣER + Ea and ∆H = ΣEP – ΣER (∆H may be + ve or – ve)
For example, burning of carbon is highly exothermic process, yet it does not start of its own. The activation energy
of combustion of carbon is quite high and thus, in spite of exothermic nature, the reaction starts only when flame
is applied to the heap of carbon contents.

Illustration 18: A first-order reaction is 50% complete in 30 min at 27°C and in 10 min at 47°C. Calculate the rate
constants, and the energy of activation of the reaction in kJ/mol. (JEE ADVANCED)
Sol: Time for the completion of 50% reaction means t1/2.
It means t1/2 of the reaction at 27°C is 30 min and at 47°C is 10 min.
0.693 0.693 0.693
We know that k = or k= = 0.0231 min–1(at 27°C) and k = = 0.0693 min–1(at 47°C)
t1/2 30 10

k Ea  1 1 
We know that
= log 2  − 
k1 2.303R  T1 T2 

k2 = 0.0693 min–1, k1 = 0.0231 min–1, T1 = 27 + 273 = 300 K, T2 = 47 + 273 = 320 K,



R = 8.314 JK–1 mol–1, E = ? a

0.0693 Ea  1 1 
log =  – 
0.0231 2.303 × 8.314  300 320 

Ea  320 − 300 
or log 3 =  
2.303 × 8.314  300 × 320 

Ea  20 
0.4771 =  
2.303 × 8.314  300 × 320 

 0.4771 × 2.303 × 8.314 × 300 × 320 


or Ea =   = 43848.49 J mol = 43.85 kJ mol
–1 –1

 20 
Chem i str y | 18.25

10. METHODS OF DETERMINATION OF ORDER OF REACTION

10.1 Method of Integration (Hit and Trial Method)


The most simple method is the one in which the quantities a, x and t are determined and substituted in the kinetic
equations of various orders. The equation which gives the most constant value for the specific rate constant (k) for
a series of time intervals is the one corresponding to the order of reaction. If all the reactants are at the same molar
concentrations, the kinetic equations are:
2.303 a
k= log10 , for first-order reactions;
t (a − x)

1 1 1
k=  −  , for second-order reactions;
t  (a − x) a 

1  1 1
k=  −  , for third-order reactions;
2t  (a − x)2
a2 

10.2 Graphical Method


A graphical method based on the respective rate laws can also be used to determine the order of reaction.
If the plot of log (a – x) versus ‘ t’ is a straight line, the reaction follows first order.
1
If the plot of versus ‘ t’ is a straight line, the reaction follows second order.
(a − x)
1
If the plot of versus ‘t’ is a straight line, the reaction follows third order.
(a − x)2
1
In general, for a reaction of nth order, a graph of versus ‘t ‘ must be a straight line.
(a − x)n−1

10.3 Half-Life Method


1
A general expression for the half-life, (t1/2 ), is given by t1/2 ∝
n−1
a

Zero order 1st order 2nd order 3rd order
t1/2

t1/2

t1/2

t1/2

2
Conc. Conc. 1/a 1/a
1 −n
Figure 18.17: Plots of half-lives versus concentration (t1/2 ∝ a )

This relation can be used to determine order of reaction ‘n’.

10.4 Van’t Hoff Differential Method


Relationship between velocity V of nth order reaction and concentration of reactants C is given by
log(V1 / V2 )
Van’t Hoff, n =
log(C1 / C2 )
Where, C1 and C2 are two different concentrations, while V1 and V2 are their velocities.
1 8 . 2 6 | Chemical Kinetics

10.5 Ostwald Isolation Method


This method is used to find out the order of complex reactions. If nA, nB and nC molecules of substance A, B and C,
respectively are present in a reaction, then nA + nB + nC will be the order of reaction:
When B and C are in excess, the order of reaction will be nA.
When A and B are in excess, the order of reaction will be nC.
When A and C are in excess, the order of reaction will be nB.

PROBLEM-SOLVING TACTICS

(a) To determine the average rate for change in concentration over a time period.
Average rate of reaction is the change in concentration of reactants (or products) during large interval of time.
[reactant at t2 ]–[reactant at t1 ]
Rate = –
t2 –t1

Square brackets denote concentration.


Negative sign in the above equation is used to make the rate of reaction positive. Minus sign can be ignored
when calculating average rates from products.

∆[reactant ] ∆[product ]
Rate = – or rate=
∆t ∆t
0.20
(b) To determine instantaneous rate from a plot of concentration
versus time: Instantaneous rate at time t is determined as follows: 0.15 Rate = -Slope =
-2
1.06 x 10 M/sec
(i) Calculate the negative slope of the curve between concentration mol
of a reactant and time at t. 1

0.10
(ii) Calculate the slope of the curve between concentration of a
product and time at t.
(c) Problem-solving tactics to determine the initial rate from a plot 0.05
of concentration versus time: The initial rate of a reaction is the
instantaneous rate at the start of the reaction (i.e., when t = 0). 0.00
Initial rate is equal to the negative slope of the curve between 5 10 15 20 25
reactant concentration and time at t = 0. Time, sec.

Figure 18.18: Plot of concentration


(d) To determining the order of a reaction from its rate law
versus time
Rate law of a reaction is used to determine:
•• The order of the reaction with respect to one or more reactants.
•• The overall order of the reaction.
rate = k[A]n[B]m[c]p
For the rate law: Order with respect to A = n; order with respect to B = m, order with respect to C = p
Reaction or overall order = n + m + p
Note: The stoichiometric coefficient in the balanced equation for a chemical reaction is usually different
from the order.
Chem i str y | 18.27

Reaction Experimental Rate Law Order

NO2 + CO → NO+ CO2 rate = k[NO2]2 2nd order reaction 2nd order in NO2 0 order in CO

CH3CHO → CH4+ CO rate = k[CH3CHO]2 2nd order reaction 2nd order in CH3CHO

(e) To determining rate laws from graphs of concentration versus time (integrated rate laws).
To determine the rate law for a reaction for different concentration (or the values of some function of
concentration) versus time, make three graphs.
[A] versus t (linear for a zero-order reaction)
ln [A] versus t (linear for a 1st order reaction)
1 / [A] versus t (linear for a 2nd order reaction)
The graph that is linear indicates the order of the reaction with respect to A. Then, you can choose the correct
rate equation:

For a zero-order reaction, Rate = k (k = -ve slope of line)

For a first order reaction, rate = k[A] (k = -ve slope of line)

For a second order reaction, rate = k[A]2 (k = slope of line)

(f) Regarding questions of half lives:


For a zero-order reaction A → products, rate = k , t½ = [A0] / 2k
For a first-order reaction A → products, rate = k[A], t½ = 0.693 / k
For a second-order reaction 2A → products or A + B → products (when [A] = [B]), rate = k[A]2 , t½ = 1 / k [A0]
Graphical Relations and half-lives: Differences in half-lives for reactions of different orders (0, 1st, or 2nd) are
determined by plotting graphs between reactant [A] and time (t). Change in time changes the concentration
of a reactant to half.
(i) For a zero-order reaction, half-life decreases with decreasing concentration.
(ii) For a 1st order reaction, half-life is constant.
(iii) For a second-order reaction, half-life increases with decreasing concentration.

[A] vs time for a [A] vs time for a [A] vs time for a


Zero order reaction First order reaction Second order reaction
0.20 0.20 0.20
Half life decreases Length of half life Length of half life
with decreasing is constant increases with decreasing
0.15 concentration 0.15 0.15 concentration
mol mol
[A],M
L st st L st
1 half life 1 1
0.10 rd
0.10 half life 0.10 half
3
nd half life nd life
2 2
0.05 half life 0.05 half life 3rd 0.05
rd
3
half life half life
0.00 0.00 0.00
20 40 60 80 0 5 10 15 20 25 100 200 300 400
Time sec. Time (sec) Time (sec)
Figure 18.19: Plot of concentration versus change in time

For a first-order reaction: For a second-order reaction:


A → products, rate = k A → products, rate = k[A] 2A → products/A + B → products
(when [A] = [B], rate = k[A]2
1 8 . 2 8 | Chemical Kinetics

POINTS TO REMEMBER

Topic Formula
Average rate of
reaction
Change in the concentration of reactants or(Products) ∆C C − C1
Average rate = = ± ± 2
=
Time ∆t t2 − t1

Instantaneous
∆C dC
rate of reaction Instantaneous rate = L im =
∆t → 0 ∆t dt

For the reaction n1A + n2B → m1C + m2D


1 d[A] 1 d[B] 1 d[C]
= – = +
n1 dT n2 dT m1 dT
1 d[A] 1 d[B] 1 d[C] 1 d[D]
Instantaneous Rate of reaction = – = – = + =
= +
n1 dT n2 dT m1 dT m2 dT
1 d[D]
= +
m2 dT
Factors affecting •• Nature of reactants and products
rate of reaction •• Effect of temperature
•• Concentration of Reactants
•• Presence of catalyst
•• Effect of Sunlight
Law of mass The rate at which a substance is directly proportional to its active mass and the rate at which a reaction
action and rate proceeds is proportional to the product of the active masses of the reacting substances
constant
 dx   dx 
For a reaction, aA + bB → product rate =   ∝ [A] [B] ;   = k[A] [B]
a b a b

 dt   dt 
where k is rate constant or velocity constant.
dx
When [A] = [B] = 1 mol/L, then =k
dt

Unit of rate n−1


constant  litre 
Unit of rate constant =   × sec–1
 mol 
When n = order of reaction.

Various types of Type of Integrated rate equation Unite of rate t1/2 Half-life t3/4 period
reactions reaction constant period
Zero-order Concentration
d[A] A0
reaction – = k0[A]0 time–1
dt 2k
dx
Differential form =k
dt
Chem i str y | 18.29

First-order Time–1
reaction k1= 2.303 a 0.693 0.693 1.382
log10 2× =
t (a − x) K1 K1 k1

dx
Differential form
dt
= k(a - x)

Second-order Mole–1 litre


2.303 b(a − x)
reaction k2 = log time–1
t(a − b) a(b − x) 1 3
dx K 2a K 2a
Differential form
dt
= k(a – x)2
Third-order Litre2 mole–2 _____
1(2a − x) 1 3
reaction k3 = time–1
t2a2 (a − x)2 2 K 2a
dx
Differential form
dt
= k(a – x)3
Relationship Zero Order First Order Second Order Third Order
between Rate
of reaction and
rate constant
r r r r

0 1 2 3
(conc) (conc) (conc) (conc)
1/(a - x)

2
(conc)

log[A]

1/(a -x)

t t t t

t1/2 t1/2 t1/2 t1/2

0 1
(conc) (conc) 1/a 1/a

Half-Life
1
t1/2 ∝ where n = order of reaction
n−1
a

Arrhenius
k Ea  1 1 
theory log 2
=  − 
k1 2.303R  T1 T2 
−Ea /RT
k = Ae where k1 and k2 are rate constant at temperatures T1 and T2, respectively (T2 > T1).
1 8 . 3 0 | Chemical Kinetics

Solved Examples

JEE Main/Boards Example 3: In a reaction 2N2O5 → 4NO2 + O2, the rate


can be expressed as
Example 1: The half-life period of a first-order reaction d[N2O5 ]
is 30 min. Calulate the specific reaction rate of the (i) – = k1[N2O5]
dt
reaction. What fraction of the reactant remains after 70
d[NO2 ]
min? (ii) = k2[N2O5]
dt
0.6932 0.6932
Sol: k1 = = = 0.0231 min–1. d[O2 ]
t1 30 (iii) = k3[N2O5]
dt
2

Let the reaction be A → Product How are k1, k2 and k3 related?

Initial concentration a Sol: The rate law reaction is rate =


Concentration after (a – x) x
1 d[N2O5 ] 1 d[NO2 ] d[O2 ]
70 min − = = = k N2O5 
2 dt 4 dt dt
∴ Fraction of the reaction remained unreacted
d[N2O5 ]
(a − x) 2.303 a ∴– = 2k[N2O5] = k1[N2O5]
= Now, k1 = log or 0.0231 dt
a . t a−x d[NO2 ]
2.303 a = 4k[N2O5] = k2[N2O5]
= log dt
70 a−x
d[O2 ]
a 0.0231 × 70 = k [N2O5] = k3[N2O5]
log = = 0.7021 dt
a−x 2.303
a k1 = 2k
Taking antilog, we get = 5.036  k k
a−x ∴ k 2 = 4k or k = 1 = 2 = k
k = k 2 4
a−x 1  3
∴ = ≈ 0.2
a 5.036
or 2k1 = k2 = 4k3.

Example 2: For the reaction 2NO + Cl2 → 2NOCl,


when the concentration of both reactants is doubled Example 4: The reaction 2N2O5 → 4NO2 + O2 forms
the rate increased by a factor of 8, but when only the NO2 at the rate of 0.0072 mol L-1s-1 after a certain time.
concentration of Cl2 concentration is doubled, the rate
(a) What is the rate of change of [O2] at this time?
doubles. What is the order of the reaction with respect
to NO and Cl2? (b) What is the rate of change of [N2O5] at this time?
(c) What is the rate of reaction at this time ?
Sol: Rate = k [NO]m [Cl2]n
Let the concentrations of NO and Cl2 be x and y, Sol: The rate of the reaction is expressed as
respectively.
1 d[N2O5 ] 1 d[NO2 ] d[O2 ]
rate = – = + = +
R1 = kxm yn and, R2 = k (2x)m (2y)n 2 dt 4 dt dt
R2 and given that
∴ = 2m + n = 8 = 23. (given)
R1
d[NO2 ]
∴ m + n = 3 Again, R3 = k(x)m (2y)n =0.0072 molL-1s-1.
dt
R3
= k xmyn. 2n. ∴ = 2n = 2 (given) (a) Rate of appearance of
R1
∴ n = 1; m = 3 – 1 = 2. 1
O2 = × rate of appearance of NO2
4
Chem i str y | 18.31

d[O2 ] 1 d[NO2 ] 1 At zero time, no CH3COOH is formed. Hence,alkali used


= × = × 0.0072 = 0.0018 mole/L/s. at zero time is only for HCl. Thus we have
dt 4 dt 4
1 Time(min) : 0 20 119 ∞
(b) Rate of disappearance of N2O5 = × rate of
appearance of NO2 2 Vol. of alkali

(mL)used 
d[N2O5 ] 1 d[NO2 ] 1  : 19.24 20.73 26.6 42.03
– = × = × 0.0072 against 
dt 2 dt 2
CH3COOH 
= – 0.0036 mole /L/s.
−19.24 −19.24 −19.24 −19.24
1 d[NO2 ]
(c) Rate of reaction = × = 0= 1.49 = 7.36 = 22.79
4 dt (x) (x) (a)
1
= × 0.0072 = 0.0018 (mole/L)s-1.
4 For t – 20;

Example 5: The activation energy for the reaction 2.303 22.79


k1= log =0.0033 min–1
O3 (g) + NO (g) → NO2 (g) + O2 (g) is 9.6 kJ/mol. Prepare 20 22.79 – 1.49
an activation energy plot it ∆H° for this reaction is -200 For t = 119 ;
kJ/mole. What is the energy of activation for the reverse 2.303 22.79
reaction? k 1= log =0.0032 min–1
119 22.79 – 7.36
Sol: Energy of activation for reverse reaction The constancy of k1 shows that the reaction is of first
= 9.6 + 200 kJ = 209.6 kJ. order.

Example 7: For the reaction


k k
[Cr[H 2 O] 4 Cl 2 ] + (aq) → [Cr(H 2 O) 5 Cl 2 ] 2+ (aq) 
1 2→
9.6kJ
E [Cr(H2O)6]3+(aq)
k1 = 1.76 × 10–3 s–1 and k2 = 5.8 × 10–5 s–1 for the initial
Energy

200kJ concentration of [Cr(H2O)5Cl2]+ is 0.0174 mol/L at 0°C.


Calculate the value of t at which the concentration of
[Cr(H2O)5Cl2]2+ is maximum.
Reaction coordinate
2.303(logk1 – logk 2 )
Sol: We have, t =
Example 6: 1 mL of methyl acetate was added to 20 mL k1 – k 2
of 0.5 N HCl and 2mL of the mixture was withdrawn from
time tc during the progress of hydrolysis of the ester and 2.303(log1.76 × 10−3 – log5.8 × 10−5 )
= = 2005 seconds.
titrated with a solution of alkali. The amount of alkali 1.76 × 10 −3 − 5.8 × 10 −5
needed for titration at various intervals is given below:

Time: 0 20 119 ∞ (min) Example. 8: The complexation of Fe2+ with the chelating
agent dipyridyl has been studied kinetically in both
Alkali used: 19.24 20.73 26.6 42.03 (mL)
forward and reverse directions.
Establish that the reaction is of first order. Fe2+ + 3 dipy → Fe (dipy)32+

Sol: Rate (forward) = (1.45 × 1013) [Fe2+][dipy]3and rate


HCl
CH3COOCH3 + H2O 
[H]
→ CH3COOH + CH3OH  (reverse) = (1.22 × 10–4 [Fe (dipy)32+ ].
Ini.Conc. a Find the stability constant for the complex.
Conc. after t (a-x) x
Sol: At dynamic equilibrium, Rate of formation of
HCl acts as a catalyst. The alkali used against HCl is
complex = Rate of its decomposition
subtracted from the total alkali used (given in the data)
to get the volume of alkali used only against CH3COOH. (1.45 × 1013) [Fe2+][dipy]3 = (1.22 × 10–4 [Fe (dipy)32+ ]
1 8 . 3 2 | Chemical Kinetics

= 1.25 × 104 × 1.987 × 10–3 kcal mol–1 = 24.84 kcal mol–1


[Fe(dipy)32+ ] 1.45 × 1013
ka = = = 1.19 × 1017. (b) ln K = 14.34 – [(1.25 × 104)/500]
[Fe2+ ][dipy]3 1.22 × 10−4
(∵ T = 500 K)

Example 9: Bicyclohexane was found to undergo two ∴ K = 2.35 × 10–5 sec–1


parallel first order rearrangements. At 730 K, the first 0.693
order rate constant for the formation of cyclohexene (c) K = (for first order)
256 × 60
was measured as 1.26 × 10–4 sec-1, and for the formation
of methyl cyclopentane the rate constant was 3.8 × 0.693 1.25 × 10 4
∴ log = 14.34 –
10–5sec–1. What is the percentage distribution of the 256 × 60 T
rearrangement products? ∴ T = 513 K

Sol:
Example 2: The specific reaction rate of a first-order
k1
Cyclohexene reaction is 0.02 sec–1. The initial concentration of the
Bicyclohexane reactant is 2 molL–1. Calculate (a) initial rate, and (b) rate
after 60 sec.
k2 Methylcyclopentane
Sol: For a first-order reaction:
k1
Percentage of cyclohexene = × 100 Rate of a reaction = k × molar concentration of the
k1 + k 2
1.26 × 10 −4 reactant.
= × 100 = 77%
1.26 × 10−4 + 3.8 × 10−5 (a) ∴ Initial rate = k × initial concentration

∴ Percentage of methyl cyclopentane = 23%. = 0.02 × 2 = 0.04 mol/L–1sec–1.


2.303
(b) Concentration of the reactant after 60 sec k =
Example 10: A first-order reaction has a specific 2 60
reaction rate of 10–2 sec–1. How much time will it take log = 0.02.
concentration after 60 s
for 10 g of its reactant to be reduced to 2.5 g?
∴ Concentration of the reactant after 60 sec = 0.60 M.
Sol: Rate constant, k = 10–2 sec–1
∴ Rate after 60 sec = k × concentration of the reactant
Initial reactant conc. [A]0= 10 g after 60 sec.
Final reactant conc. [A]t = 2.5 g = 0.02 × 0.60 = 0.012 molL–1sec–1
Time required, t = ?
Example 3: The rate constant is numerically the same
For a first-order reaction.
for first, second and third order reaction, the unit of
t = 2.303 × 102log 4s = 230.3 × 0.6020s = 138.6s. concentration being in moles per litre. Which reaction
should be the fastest and is this true for all ranges of
concentrations ?
JEE Advanced/Boards
Sol: Suppose R1, R2 and R3 are the rates of reactions of
Example 1: The rate constant of a reaction is given by: ln first, second and third order, respectively, and k is the
K(sec–1) = 14.34 – (1.25 × 104)/T. Calculate (a) the energy rate constant, which is the same for the three reactions,
of activation and (b) the rate constant at 500 K. (c) At ∴ R1 = k[A]1 ; R2 = k[A]2 ; R3 = k[A]3
what temperature will its half-life period be 256 min?
[A] being the concentration of the reactant A in moles
Sol: Given: per litre.

ln K = 14.34 – [(1.25 × 104)/T] ….(i) Now if, [A] = 1, R1 = R2 = R3 ;

Arrhenius equation, ln K = ln A–(Ea/RT)..(ii) [A] < 1, R1 > R2 > R3 ; and

(a) Comparing equations (i) and (ii), [A] > 1, R1< R2 < R3.

(Ea/R) = 1.25 × 104 ∴ Ea = 1.25 × 104R


Example 4: For a reaction at 800°C
Chem i str y | 18.33

2NO + 2H2 → N2 + 2H2O, the following data were At ∞ time, i.e. when the reaction is complete, the whole
obtained: of C6H5N2Cl converts into N2. Hence volume of N2 at ∞
time corresponds to the initial concentration ‘a’ while
[NO]×10–4 mole/L [H2]×10–3 mole/L d[NO]/dt×10–4
volumes of N2 at different time intervals correspond to x
mole/L min
as shown above. substituting the given data in equation
(i) 1.5 7.0 4.4
of first-order reaction, we get the following results.
(ii) 1.5 3.5 2.2
(iii) 1.5 2.0 0.24 For t = 20 min
What is the order of this reaction with respect to NO 2.303 162
k1 = log = 0.0032 min–1
and H2 ? 20 162 − 10
For t = 50 min,
Sol: From the data (i) and (ii), we see that when the
concentration of H2 is halved, the rate is also halved at 2.303 162
k1 = log = 0.0033 min–1
constant concentration of NO. Hence the reaction is of 50 162 − 25
first order with respect to H2 Let us now consider the For t = 70 min,
data (ii) and (iii) to determine the order with respect to
2.303 162
NO as [H2] is constant. k1 = log = 0.0032 min–1
70 162 − 33
The rate law of the above reaction is
The consistency of k1 shows that the decomposition of
1 d[NO] C6H5N2Cl is a first-order reaction.
rate = – × = k[NO]m[H2]1
2 dt
Example 6: From the following reaction scheme, write
where, m is the order with respect to NO
the rate law for the disappearance of A, B and C.
d[NO] k
Or – = 2k[NO]m[H2] (1) A + B →C + D
1
dt k
(2) C + D 
2→A + B
Substituting data (ii) and (iii), we get k
(3) B + C →E + D
3
2.2 × 10 = 2k(1.5 × 10 )m. (2 × 10 ) 
–4 –4 –3
...(i)
0.24 × 10–4 = 2k(0.5 × 10–4)m. (2 × 10–3)  …(ii) Sol.: The reaction A is removed in step 1 and produced
in step 2
Dividing (i) by (ii),
d[A]
2.2 (1.5 × 10−4 )m 220 ∴– = k1[A] [B] – k2[C][D] Similarly
= = 3m or = 3m dt
0.24 (0.5 × 10−4 )m 24
d[B]
– = k1[A][B] + k3[B][C] – k2[C][D] and
Taking log, log 220 – log 24 = m log 3 dt
2.4324 – 1.3802 = m × 0.4771 d[C]
– = k2[C][D] + k3[B][C] – k1[A][B].
dt
0.9622
Or 0.9622 = 0.4771 m or m = = 2.
0.4771
Example 7: At a certain temperature, the half change
Hence the reaction is of second and first order with period for the catalytic decomposition of ammonia
respect to NO and H, respectively. were found as follows:
Pressure (Pascals): 6667 13333 26666
Example 5: From the following data for the de-
composition of diazobenzene chloride, show that the Half life period in hours: 3.52 1.92 1.0
reaction is of first order Calculate the order of reaction.
Time 20 50 70 ∞ (min) n−1
(t1/2 )1 a 
Vol.of N2 10 25 33 162 (mL) Sol: =  2 
(t1/2 )2  a1 
Sol: C6H5N2Cl → C6H5Cl + N2
where, n is order of reaction
Initial concentration    a n−1
3.52  13333 
From the given data, = 
After time t   (a – x) x 1.92  6667 
(a ∝ initial pressure) = (2)n –1
1 8 . 3 4 | Chemical Kinetics

3.52 The rate constant and activation energy for reaction


log = (n – 1) log 2 = 0.3010 × (n – 1) (i) are 2.79 × 10–3 min–1 at 237°C and 12.0 kcal mol–1
1.92
respectively. These values for reaction (ii) are 1.52 × 10–4
0.2632 = 0.3010 × (n – 1) ; n = 1.87 ≈ 2 min–1 at 237°C and 24.5 kcal mol–1 respectively. Find out
Similar calculations are made between first and third the temperature at which equimolar quantities of H2O,
observations, n comes equal to 1.908 (≈ 2). CO, CO2 and H2 are formed (R = 2 cal).
Thus, the reaction is of second order. −Ea /RT
Sol: For reaction (a) K1 = A1e
Example 8: The optical rotation of can sugar in 0.5 N 12×103

acid at 25°C at various time intervals are given below: or 2.79 × 10–3 = A1e RT  ……(i)
−Ea /RT
Time (min): 0 1435 11360 ∞ For reaction (b): K2 = A2e
24.5×103
Rotation(°): 34.50° 31.10° 13.98° – 10.77° –
1.52 × 10–4 = A2e RT  …(ii)
Show that the reaction is of first order.
Sol: 2.79 × 10−3 2.79 × 10−3
By equation (i) A1 = =
Lactic acid –
12×103 7.8 × 10−6
C12H22O11 
→ C6H12O6 + C6H12O6 e 2×510

+ H2O Glucose Fructose = 3.58 ×10 2

Sucrose (excess) Dextro Laevo 1.52 × 10−4 1.52 × 10−4


By equation (ii) A2 = =
rotatory rotatory 24.5×103 − 11
– 3.7 × 10
Since in this reaction dextro form clianges to laevo
e 2×510
= 4.11 ×106
form, the optical rotation decreases with the progress
of the reaction. Thus change in rotation is proportional If rate constants of two reactions are same, the reactions
to the amount of sugar remaind after different time will give equimolar mixture of products. Let these be
intervals. We now have, same at temperature T, i.e.
−Ea /RT −Ea /RT
Time 0 1435 11360 ∞ A1e 1 = A2e 2

(min)
Ea −Ea [24.5 −12.0]×103
2 1 A2 A2
Change in 34.50 – 31.10 – 13.98 – – 10.77– ∴ e RT = or e RT =
rotation(°) (–10.77) (–10.77) (-10.77) (–10.77) A1 A1
= 45.27 =41.87 =24.75 =0 4.11 × 106
(a – x)
= = 1.15 × 104
(a) (a – x) 3.58 × 102
Substituting the data in equation, 103 12.5 × 103
Or 9.35 \T =
× 12.5 = = 668 K
for t = 1435 min RT 9.35 × 2
2.303 45.27 ∴ T = 668 K = 395°C
k1 = log = 5.442 × 10–5
1435 41.87
and for t = 1136 min Example 10: For the reversible reaction in equilibrium:
k
2.303 45.27 
A ←
1
→ B. The values of K and K are 2 × 10–3 mol L–1
k1 = log = 5.311 × 10–5 k
 1 2
1435 41.87 2
sec–1 and 3 × 10–3 mol L–1 sec–1 respectively. If we add 0.5
The values of k are fairly constant and so the reaction mol of B in the equilibrium mixture, initially containing
is of first order . 2 mol of A. Calculate the time taken for concentration
of B to become equal to 3/4 of the concentration of A
Example 9: The catalytic decomposition of formic acid at initial equilibrium. The volume of mixture is 1 L and
may take place in two ways: remains constant.
(i) HCOOH → H2O + CO  1
→ B
k
Sol: A ← 
k
(ii) HCOOH → H2 + CO2 2

t=0 2 mol L –1
0
Chem i str y | 18.35

At eq. (2 – X) mol L–1 X or – 2.303 log (0.66 – X) = K2 . t + C


K1 = 2 × 10–3 mole L–1 sec–1 (zero order)
at t = 0, X = 0 ∴ C = – 2.303 log 0.66
K2 = 3 × 10 –3sec–1 (Ist order)
0.66
dX ∴ K2 . t = 2.303 log
= K1[A]0 – K2[B]1 0.66 – X
dt
2.303 0.66
dX ∴t= log = 89.24 sec
At equilibrium =0 3 × 10−3 0.66 – 0.155
dt
(a) Partial pressure becomes half of initial in every 100
K1 2 × 10−3
∴ 0=K1–K2[X]eq ; \[X]eq = = min, therefore, order = 1.
K2 3 × 10−3
800
= 0.66 mol L–1 = Kc (b) k × 100 = ln
400

A ←→B
 k × 100 =ln 2
Initial eq. 1.34    0.66 ⇒ k = 6.93 × 10–3 min–1
Moles added 1.34    0.66+0.5= 1.16 (c) For 75% reaction; time required
At eq. at time t (1.34 +X) (1.16 –X) = 2 × half-life = 200 min
(The reaction will obey back ward direction on addition (d) 2X(g) → 3Y(g) + 2Z(g)
of B) 3
800 – x x
3 3 2
∵ [B] = [A]eq = × 1.34 = 1.005 3
4 4 Total pressure = 800 + x
2
(1.16 – X) = 1.005; ∴ X = 0.155 Also 800 – x = 700 ⇒ x = 100
3
⇒ Total pressure = 800 + × 100 = 950 mm
dX 2
Now, = K1 – K2[X] = 0.66K2 –K2 X
dt
dX
= K2[0.66 – X] ; ∴ = K2 . dt
(0.66 – X)

JEE Main/Boards

Exercise 1 Q.3 The decomposition of a compound P at temperature


T according to the equation
Q.1 A solution of A is mixed with an equal volume of 2P(g) → 4Q(g) + R(g) + S(l)
a solution of B containing the same number of moles,
is the first order reaction. After 30 minutes from the
and the reaction A + B → C occurs. At the end of 1h,
start of decomposition in a closed vessel the total
A is 75% reacted. How much of A will be left unreacted
pressure developed is found to be 317 mm Hg and
at the end of 2h if the reaction is (a) first order in A and
after a long period of time the total pressure observed
zero order in B; (b) first order in both A and B; and (c)
to be 617 mm Hg. Calculate the total pressure of the
zero order in both A and B?
vessel after 75 minute, if volume of liquid S is supposed
to be negligible ,also calculate the tiny fraction t1/8.
Q.2 The reaction CH3–CH2–NO2+ OH– → CH3–CH–NO2-
+H2O obeys the rate law for pseudo first order kinetic Given: Vapor pressure of S () at temperature T = 32.5
in the presence of a large excess of hydroxide ion . If mm Hg.
1% of nitro ethane undergoes reaction in half minute
when the reaction concentration is 0.002 M, What is the Q.4 A certain reactant Bn+ is getting converted to
pseudo first order rate constant? B(n + 4)+ in solution. The rate constant of this reaction is
measured by titrating a volume of the solution with a
1 8 . 3 6 | Chemical Kinetics

reducing reagent which only reacts with Bn+ and B(n+4)+. Q.11 Two reactions
In this process, it converts Bn+ to B(n – 2)+ and B(n +4)+ to
(i) A → products
B(n – 1)+. At t = 0 the volume of the reagent consumed is
25 ml and at t = 10 min, the volume used up is 32 ml. (ii) B → products,
Calculate the rate constant of the conversion of Bn+ to follow first order kinetics. The rate of the reaction (i)
B(n + 4)+ assuming it to be a first order reaction. is doubled when the temperature is raised from 300
K to 310 K. The half-life for this reaction at 310 K is 30
Q.5 Decomposition of H2O2 is a first order reaction. A minutes. At the same temperature B decomposes twice
solution of H2O2 labelled as 20 volumes was left open as fast as A. If the energy of activation for the reaction
due to this, some H2O2 decomposed. To determine is half of that of reaction, calculate the rate constant of
the new volume strength after 6 hours 10 mL of this the reaction (ii) at 300 K
solution was diluted to 100 mL .10mL of this diluted
solution was titrated against 25mL of 0.025 M KMnO4 Q.12 A certain organic compound A decomposes by
solution under acidic conditions. Calculate the rate two parallel first order mechanism
constant for decomposition of H2O2.
k1 B
Q.6 A metal slowly forms an oxide film which completely A
protects the metal when the film thickness is 3.956 k2 C
thousand times of an inch. If the film thickness is 1.281
thou. in 6 weeks, how much longer will it be before it If k1: k2 = 1: 9 and k1 = 1.3 × 10–5s–1.
is 2.481 thou.? The rate of film formation follows first Calculate the concentration ratio of C to A, if an
order kinetics. experiment is started with only A and allowed to run
for one hour.
Q.7 An optically active compound A upon acid catalyzed
k
hydrolysis yield two optically active compound B and C + 
Q.13 The reaction cis – Cr(en)2 (OH)2 ←
1
→ trans–

by pseudo first order kinetics. The observed rotation of k 2
the mixture after 20 min was 5° while after completion Cr(en)2 (OH)+ is first order in both directions. At 25°C
of the reaction it was – 20°. If optical rotation per mole 2

of A, B & C are 60°,40° & – 80°. Calculate half-life of the the equilibrium constant is 0.16 and the rate constant
reaction. k1 is 3.3 × 10–4 s–1. In an experiment starting with the
pure cis form, how long would it take for half the
Q.8 A vessel contains dimethyl ether at a pressure of equilibrium amount of the Trans isomer to be formed?
0.4atm. Dimethyl ether decomposes as CH3OCH3(g)
→ CH4(g) + CO(g) + H2(g). The rate constant of Q.14 For a reversible first-order reaction
k1
k1
decomposition is 4.78 × 10–3 min–1. Calculate the ratio k2

of initial rate of diffusion to rate of diffusion after 4.5 = 10–2 s–1 and [B]eq /[A]eq = 4. If [A]0 = 0.01 mole L–1 and
hours of initiation of decomposition. [B]0 = 0, what will be the concentration of B after 30 s?

Q.9 At room temperature (20°C) orange juice gets Q.15 For the system A ( g) → B ( g) , ∆H for the forward
spoilt in about 64 hours. In a refrigerator at 3°C juice reaction is –33kJ/mol (Note. ∆H = ∆E in this case).
can be stored three times as long before it gets spoilt.
Estimate (a) the activation energy of the reaction that Show that equilibrium constant
causes the spoiling of juice (b) How long should it take [B]
K= = 5.572 × 105 at 300 K. If the activation energies
for juice to get spoilt at 40°C? [A]
Er and Eb are in the ratio 20: 31. Calculate Er and Eb at this
Q.10 A first order reaction, A → B, requires activation temperature .Assume that the pre-exponential factor is
energy of 70 kJ mol–1.When a 20% solution of A was the same for the forward and backward reactions
kept at 25°C for 20 minutes, 25% decomposition took
place. What will be the percent decomposition in the Q.16 The complex [Co(NH3)5F]2+ reacts with water
same time in a 30% solution maintained at 40°C ? according to the equation
Assume that activation energy remains constant in this
range of temperature. [Co(NH3)5F]2+ + H2O → [Co(NH3)5(H2O)]3+ + F–
Chem i str y | 18.37

The rate of the reaction = rate const. x[complex]a x[H+]b. The initial pressure in a container of capacity V litres is
The reaction is acid catalysed i.e. [H+] does not change 1 atm. Pressure at time t = 10 sec is 1.4 atm and after
during the reaction. infinite time it becomes 1.5 atmosphere. Find the rate
constant k1 and k2 for the appropriate reaction.
Thus rate = k[Complex]a where k’ = k[H+]b calculate ‘a’
and ‘b’ given the following data at 25°C.
Q.21 A first order reaction takes 69.3 minutes for 50%
[Complex]M [H+ ]M T1/2hr T3/ 4hr completion. How much time will be needed for 80%
0.1 0.01 1 2 completion?

0.2 0.02 0.5 1


Q.22 The specific rate constant for a reaction increases
k
by a factor 4 of the temperature is changed from 27°C
Q.17 For the two parallel reactions A → B and
1
to 47°C. Find the activation energy for the reaction
k
2
A  → C , show that the activation energy E’ for the
disappearance of A is given in terms of activation Q.23 The reaction 2A+ B + C → D + 2 E is of first order
k E + k 2E2 with respect to A and of second order with respect to B
energies E1 and E2 for the two paths by E’ = 1 1 and is of zero order with respect to C
k1 + k 2
(i) Write down the rate law for the reaction
Q.18 For the mechanism
(ii) What will be the effect of doubling concentration of
k

A + B ←
1 k3
→ C ; C  →D
 A, B and C.
k

2

(a) Derive the rate law using the steady-state Q.24 A first order reaction is 50% completed in 30 min
approximation to eliminate the concentration of C. at 27°C and in 10 minutes at 47°C. Calculate the rate
(b) Assuming that k3 << k2, express the pre-exponential constant at 27°C and the energy- of activation of the
factor A and Ea for the apparent second order rate reaction in kJ per mole
constant in terms of A1, A2 and A3 and Ea , Ea and Ea
1 2 3
for the three steps. Q.25 The optical rotation of sucrose in 0.5 M HCl at 35°
C at different time intervals are given below. Show that
Q.19 The reaction of formation of phosgene from CO the reaction follows first order kinetics
and Cl2 is CO + Cl2 → COCl2 Time (min) 0 10 20 30 40 ∞
The proposed mechanism is Rotation +32.4 +28.8 +25.5 +22.4 +19.6 –11.1
k1 (degrees)

(i) Cl2 ← →
 2Cl (fast equilibrium)
k −1
k

(ii) Cl + CO ←
2 →
 COCl (fast equilibrium) Q.26 t1/2 of a reaction is halved as the initial concentration
k −2
of the reaction is doubled, find out the order of the
(iii) COCl + Cl2 
k3
→ COCl2 +Cl (slow)
 reaction.

Show that the above mechanism leads to the following Q.27 The rate constant of a reaction is 1.5 × 10–7 sec–1
d[COCl2 ] at 50°C and 4.5 × 10–7 sec–1 at 100°C. Evaluate the
rate law = K[CO][Cl2]3/2.
dt Arrhenius parameters A and Ea.
1/2
k  k  Q.28 A substance reacts according to the law of first
Where K = k 3 ⋅ 2  1  .
k − 2k −1 order reaction the velocity constant of the reaction is
1.0 × 10–2 sec–1. If initial conc. of the substance is 1.0 M
Q.20 For the following first order gaseous reaction (a) Find out the initial rate
k1 (b) Find out the rate after 1 min.
2B(g
A(g)
Q.29 What will be initial rate of a reaction if its rate
k2
C(g) constant is 10–3 min–1 and concentration of reactant is
0.2 mol dm–3. How much of reactant will be converted
into products in 200 minutes.
1 8 . 3 8 | Chemical Kinetics

Q.30 A first order reaction is 20% complete in 10 Q.5 t1/4 can be taken as the time taken for the
minutes. Calculate concentration of a reactant to drop to 3/4 of its value.
If the rate constant for a first order reaction is k, the t3/4
(a) Specific rate constant of the reaction and
can be written as
(b) Time taken for the reaction to go to 75 % completion
(A) 0.69/k (B) 0.75/k (C) 0.10/k (D) 0.29/k

Exercise 2 Q.6 A reaction was found to be second order with


respect to the concentration of carbon monoxide. If
Single Correct Choice Type the concentration of carbon monoxide is doubled, with
everything else kept the same, the rate of reaction will
Q.1 For the reaction A + B → C; starting with different
initial concentration of A and B, initial rate of reaction (A) Double (B) Remain unchanged
were determined graphically in four experiments.
(C) Triple (D) Increase by a factor of 4
S. No. [A]0/M (Initial [B]0/M (Initial rate/ (M sec )
–1

conc.) conc.) Q.7 For a hypothetical reaction,


1 1.6×10–3 5×10–2 10–3
A + 3B → P ∆H = –2x kJ/mole of A
2 3.2×10–3 5×10–2 4×10–3
3 1.6×10–3 10–1 2×10–3 and M → 2Q + R ∆H = –x kJ/mole of M
4 3.2×10–3 10–1 8×10–3 If these reactions are carried simultaneously in a
reactor such that temperature is not changing. If rate of
Rate law for reaction from above data is
disappearance of B is y M sec–1 then rate of formation
(A) r = k[A]2W2 (B) r = k[A]2[B] (in M sec–1) of Q is:
(C) r = k[ A] [B]2 (D) r = k[A][B] 2 3 4 3
(A) y (B) y (C) y (D) y
3 2 3 4
Q.2 The rate law for a reaction between the substances
A and B is given by rate = k [A]n [B]m On doubling the Q.8 Gaseous reaction A → B + C follows first order
concentration of A and halving the concentration of kinetics. Concentration of A changes from 1M to 0.25
B, the ratio of the new rate to the earlier rate of the M in 138.6 minutes. Find the rate of reaction when
reaction will be as concentration of A is 0.1 M.
(A) 2(n – m) (B) 1/2m + n (C) (m + n) (D) (n – m) (A) 2 × 10–3M min–1 (B) 10–3M min–1
(C) 10–4M min–1 (D) 5 × 10–4M min–1
Q.3 In a first order reaction, the concentration of the
reactant, decreases from 0.8 to 0.4 M in 15 minutes. The
time taken for the concentration to change from 0.1 M Q.9 The initial rate of zero-order reaction of the
to 0.025 M is gaseous reaction A(g) → 2B(g) is 10–2 M min–1. If the
initial concentration of A is 0.1M. What would be the
(A) 30 minutes (B) 15 minutes concentration of B after 60 sec?
(C) 7.5 minutes (D) 60 minutes
(A) 0.09 M (B) 0.01 M

Q.4 The rate equation for the reaction (C) 0.02 M (D) 0.002 M

2A + B → C is found to be: rate=k [A] [B] The correct


Q.10 Consider the following first order competing
statement in relation to this reaction is
reactions:
(A) Unit of k must be s–1 k k
X  → A + B and Y 
1 → C + D if 50% of the reaction
2

(B) t1/2 is a constant of X was completed when 96% of the reaction of Y was
(C) Rate of formation of C is twice the rate of completed the ratio of their rate constants (k2/k1) is
disappearance of A (A) 4.06 (B) 0.215 (3) 1.1 (D) 4.65
(D) Value of k is independent of the initial concentration
of A and B
Chem i str y | 18.39

Q.11 At certain temperature, the half-life period for Q.16 The reaction A (g) → B(g) + 2C (g) is a first order
the thermal decomposition of a gaseous substance reaction with rate constant 3.465 × 10–6s–1.Starting with
depends on the initial partial pressure of the substance 0.1 mole of A in 2 litre vessel, find the concentration of
as follows A after 200 sec, when the reaction is allowed to take
place at constant pressure and temperature
P(mm Hg) 500 250
t1/2(in min) 235 950 (A) 0.05 M (B) 0.025 M

Find the order of reaction (C) 0.0125 M (D) None of these


[Given log(23.5) = 1.37; log (95)= 1.97; log 2 = 0.30]
Q.17 In respect of the equation k = A exp (– Ea / RT),
(A) 1 (B) 2 (C) 2.5 (D) 3 which one of the following statements is correct?
(A) R is Rydberg’s constant
Q.12 Consider the reaction: A → B + C
(B) k is equilibrium constant
Initial concentration of A is 1M. 20 minutes time is
required for completion of 20% reaction. (C) A is adsorption factor
d[B] (D) Ea is the energy of activation
If =k[A], then half life (t1/2) is
dt
(A) 55.44 min (B) 50 min Q.18 Rate of a reaction can be expressed by Arrhenius
(C) 62.13 min (D) None of these equation as k = Ae–E/RT In this equation, E represents
(A) The fraction of molecules with energy greater than
Q.13 If decomposition reaction A(g) → B (g) follows the activation energy of the reaction
first order kinetics then the graph of rate of formation (B) The energy above which all the colliding molecules
(R) of B against time t will be will react
(A) R (B) R (C) The energy below which colliding molecules will not
react
(D) The total energy of the reacting molecules at a
temperature T.
t t

(C) R (D) R
Q.19 The rate constant, the activation energy and
the Arrhenius parameter (A) of a chemical reaction at
25°C are 3.0 × 10–4 s–1, 104.4 kJmol–1 and 6.0 × 104 S–1
t
respectively. The value of the rare constant at T → ∞
t
(A) 2.0 × 10–18s–1 (B) 6.0 × 1014s–1
(C) Infinity (D) 3.6 × 1030s–1
Q.14 The rate constant for the forward reaction A(g) →
2B(g) is 1.5 × 10–3 s–1 at 100 K. If 10–5 moles of A and 100
moles of B are present in a 10 litre vessel at equilibrium Q.20 A first order reaction is 50% completed in 20
then rate constant for the backward reaction at this minutes at 27°C and in 5 min at 47°C. The energy of
temperature is activation of the reaction is
(A) 1.50 × 104L mol–1 s–1 (B) 1.5 × 1011L mol–1 s–1 (A) 43.85kJ/mol (B) 55.14kJ/mol
(C) 1.5 × 1010L mol–1 s–1 (D) 1.5 × 10–11L mol–1 s–1 (C) 11.97kJ/mol (D) 6.65kJ/mol

Q.15 Reaction A + B → C + D follows following rate law: Q.21 For the first order reaction A → B + C, carried out
rate = k[A]1/2[B]1/2. Starting with initial conc. of 1 M of A at 27°C if 3.8 × 10–16 % of the reaction molecules exists
and B each, what is the time taken for concentration of in the activated state, the Ea(activation energy) of the
A to become 0.25 M. reaction is [log 3.8 = 0.58]
Given: k = 2.303 × 10–3 sec–1 (A) 12 kJ/mole (B) 831.4kJ/mole
(A) 300sec. (B) 600 sec. (C) 900 sec. (D) 1200 sec. (C) 100 kJ/mole (D) 88.57kJ/mole
1 8 . 4 0 | Chemical Kinetics

Q.22 In a reaction carried out at 400 K , 0.0001% of the Q.6 The half-life period of a first order chemical reaction
total number of collisions are effective. The energy of is 6.93 minutes. The time required for the completion of
activation of the reaction is 99% of the chemical reaction will be (log 2 = 0.301)
(2009)
(A) Zero (B) 7.37k cal/mol
(A) 230.3 minutes (B) 23.03 minutes
(C) 9.212 k cal/mol (D) 11.05k cal/mol
(C) 46.06 minutes (D) 460.6 minutes

Previous Years’ Questions Q.7 A reaction involving two different reactants  (2005)

1 (A) Can never be a second order reaction


Q.1 For a reaction A → 2B, rate of disappearance
2 (B) Can never be a unimolecular reaction
of ‘A’ is related to the rate of appearance of B’ by the
(C) Can never be a bimolecular reaction
expression  (2008)
(D) Can never be a first order reaction
d[A] 1 d[B] d[A] d[B]
(A) – = (B) – =
dt 4 dt dt dt
Q.8 A reaction was found to be second order with
d[A] d[B] d[A] 1 d[B] respect to the concentration of carbon monoxide. If
(C) – =4 (D) – =
dt dt dt 2 dt the concentration of carbon monoxide is doubled with
everything else kept the same, the rate of reaction will
Q.2 The rate of a chemical reaction doubles for every  (2006)
10° C rise of temperature. If the temperature is raised (A) Remain unchanged (B) Triple
by 50° C, the rate of the reaction increases by about (C) Increase by a factor of 4 (D) Double
 (2011)
(A) 10 times (B) 24 times Q.9 For the reaction A + 2B → C, rate is given by R = [A]
[B]2 then the order of the reaction is  (2002)
(C) 32 times (D) 64 times
(A) 3 (B) 6 (C) 5 (D) 7
Q.3 In a first order reaction the concentration of
reactant decreases from 800 mol/dm3 to 50 mol/dm3 Q.10The following mechanism has been proposed for
in 2 × 102 sec. The rate constant of reaction in sec–1 is the reaction of NO with Br2 to from NOBr: NO(g) +
 (2003) 
Br2 (g) ← → NOBr (g) NOBr (g) + NO(g) → 2NOBr
 2 2
(A) 2 × 104 (B) 3.45 × 10–5 (g) If the second step is the rate determining step, the
order of the reaction with respect to NO(g) is (2006)
(C) 1.386 × 10–2 (D) 2 ×10–4
(A) 1 (B) 0 (C) 3 (D) 2
Q.4 Consider the reaction, 2 A + B → Products When
concentration of B alone was doubled, the half-life did Q.11 Consider the reaction Cl2(aq) + H2S(aq) → S(s) + 2H+(aq)
not change. When the concentration of A alone was + 2Cl–(aq) The rate equation for this reaction is rate =
doubled, the rate increased by two times. The unit of
k[Cl2][H2S] Which of these mechanisms is/are consistent
rate constant for this reaction is  (2007)
with this rate equation
(A) L mol–1 s–1 (B) No unit
A. Cl2 + H2S → H+ +Cl– + Cl– + HS– (slow)
(C) mol L s
–1 –1
(D) s–1
Cl++ HS– → H+ + Cl– + S (fast)
Q.5 The rate constant for the reaction, 
B. H2S ←→ H+ + HS– (fast equilibrium)

2N2O5 → 4NO2 + O2 is 3 ×10–5 sec–1. If the rate is Cl2 + HS– → 2Cl– + H+ + S (slow)  (2010)
2.40 × 10–5mol litre–1 sec–1.
Then the concentration of N2O5 (in mol litre–1) is(2001) (A) A only (B) B only

(A) 1.4 (B) 1.2 (C) 0.04 (D) 0 (C) Both A and B (D) Neither A nor B
Chem i str y | 18.41

Q.12 The time for half life period of a certain reaction Q.18 The time for half-life period of a certain reaction A
A → Products is 1 hour. When the initial concentration → products is 1 hour. When the initial Concentration
of the reactant ‘A’ is 2.0 mol L–1, how much time does it of the reactant ‘A’, is 2.0 mol/L, how much time does
take for its concentration to come for 0.50 to 0.25 mol it take for its concentration to come from 0.50 to
L–1 if it is a zero order reaction  (2010) 0.25 mol/L if it is a zero order reaction? (2010)
(A) 1 h (B) 4h (C) 0.5h (D) 0.25 h (A) 4 h (B) 0.5 h (C) 0.25 h (D) 1 h

Q.13 The energies of activation for forward and reverse Q.19 Consider the reaction: Cl2(aq) + H2S(aq) →
reactions for A2 + B2 → 2AB are 180kJ mol–1 and S(s) + 2H+(aq) + 2Cl– (aq) The rate equation for this
200kJ mol–1 respectively. The presence of catalyst lowers reaction is rate = k [Cl2] [H2S]
the activation energy of both (forward and reverse)
Which of these mechanisms is/are consistent with this
reactions by 100 kJ mol–1. The enthalpy change of the
rate equation? (2010)
reaction (A2 + B2 → 2AB) in the presence of catalyst will
be (in kJ mol–1) (2007) (A) Cl2 + H2 → H+ + Cl– + Cl+ + HS–(slow)
(A) 300 (B) -120 (C) 280 (D)- 20 Cl+ + HS– → H+ + Cl– + S (fast)
(B) H2S ⇔ H+ + HS– (fast equilibrium)
Q.14 Consider an endothermic reaction X → Y with
Cl2 + HS– → 2Cl– + H+ + S (slow)
the activation energies Eb and Ef for the backward and
forward reactions, respectively, in general (2005) (A) B only (B) Both A and B
(A) Eb < Ef (C) Neither A nor B (D) A only
(B) Eb >Ef
Q.20 The rate of a chemical reaction doubles for every
(C) Eb =Ef 10oC rise of temperature. If the temperature is raised by
(D) There is no definite relation between Eb and Ef 50oC, the rate of the reaction increases by about:
 (2011)
Q.15 If ‘I’ is the intensity of absorbed light and C is the (A) 24 times (B) 32 times
concentration of AB for the photochemical process
(C) 64 times (D) 10 times
AB + hυ → AB*, the rate of formation of AB* is directly
proportional to  (2001)
Q.21 For a first order reaction, (A) → products, the
(A) C (B) I (C) I2 (D) C.I concentration of A changes from 0.1 M to 0.025 M in 40
minutes. The rate of reaction when the concentration of
1
Q.16 For a reaction A → 2B , rate of disappearance A is 0.01 M is: (2012)
2
of ‘A’ is related to the rate of appearance of ‘B’ by the (A) 1.73 x 10–5 M/ min (B) 3.47 x 10–4 M/min
expression (2008)
(C) 3.47 x 10–5 M/min (D) 1.73 x 10–4 M/min
d[A] 1 d[B] d[A] 1 d[B]
(A) − = (B) − =
dt 2 dt dt 4 dt Q.22 The rate of a reaction doubles when its temperature
d[A] d[B] d[A] d[B] changes from 300 K to 310 K. Activation energy of
(C) − = (D) − 4
= such a reaction will be : (R = 8.314 JK-1 mol-1 and log
dt dt dt dt
2 = 0.301) (2013)

Q.17 The half-life period of a first order chemical (A) 53.6 kJ mol-1 (B) 48.6 kJ mol-1
reaction is 6.93 minutes. The time required for the (C) 58.5 kJ mol-1 (D) 60.5 kJ mol-1
completion of 99% of the chemical reaction will be
(log 2 = 0.301) (2009)
Q.23 . For the non-stoichiometry reaction 2A + B → C
(A) 230.3 minutes (B) 23.03 minutes + D, the following kinetic data were obtained in three
(C) 46.06 minutes (D) 460.6 minutes separate experiments, all at 298 K. (2014)
1 8 . 4 2 | Chemical Kinetics

Initial Initial Initial rate of Q.24 Decomposition of H2O2 follows a first order
Concentration (A) Concentration (A) formation (C) reaction. In fifty minutes the concentration of
(mol L-1s-1) H2O2 decreases from 0.5 to 0.125 M in one such
decomposition. When the concentration of H2O2
0.1 M 0.1 M 1.2 × 10-3
reaches 0.05 M, the rate of formation of O2 will be:
0.1 M 0.2 M 1.2 × 10-3 (2016)
0.2 M 0.1 M 2.4 × 10-3
(A) 6.93 × 10–4 mol min–1 (B) 2.66 L min–1at STP
The rate law for the formation of C is
(C) 1.34 × 10–2 mol min–1 (D) 6.93 × 10–2 mol min–1
dC dC
(A) = k[A][B] (B) = k[A]2 [B]
dt dt
dC dC
(C) = k[A][B]2 (D) = k[A]
dt dt

JEE Advanced/Boards

Exercise 1 Q.5 At 27°C it was observed during a reaction of


hydrogenation that the pressure of hydrogen gas
Q.1 Ammonia and oxygen reacts at higher temperatures decreases from 2 atmosphere to 1.1 atmosphere in 75
as 4NH3(g) + 5O2(g)→ 4NO(g) + 6H2O(g) minutes. Calculate the rate of reaction (in M sec–1) and
rate of reaction in term of pressure.
In an experiment, the concentration of NO increases by
1.08 × 10–2 mol litre–1 in 3 seconds. Calculate
Q.6 For the elementary reaction 2A + B2 → 2AB.
(i) Rate of reaction (ii) Rate of disappearance of Calculate how much the rate of reaction will change if
ammonia (iii) Rate of formation of water the volume of the vessel is reduced to one third of its
original volume?
Q.2 In the following reaction 2H2O2 → 2H2O + O rate of
formation of O2 is 3.6 M min–1 Q.7 For the reaction 3BrO– → BrO–3 +2Br– in an alkaline
(i) What is rate of formation of H2O? aqueous solution the value of the second order
(in BrO–) rate constant at 80°C in the rate law for
(ii) What is rate of disappearance of H2O2?
∆[BrO − ]
– was found to be 0.056L mol–1 s–1. What is the
∆t
Q.3 In a catalytic experiment involving the Haber rate constant when the rate law is written for
process N2 + 3H2 → 2NH3, the rate of reaction was
∆[BrO3− ] ∆[Br − ]
measured as (a) (b) ?
∆t ∆t
∆[NH3 ]
Rate = = 2 × 10 mol L s .
–4 –1 –1
∆t Q.8 Dinitropentoxide decomposes as follows:
If there were no side reactions, what was the rate of 1
N2O5(g) → 2NO2(g) + O2(g)
reaction expressed in terms of (a) N2, (b) H2? 2
Given that – d [N2O5]/dt = k1[N2O5]
Q.4 The reaction 2A + B + C → D + E is found to be d[NO2]/dt = k2[N2O5] ; d[O2]/dt = k3[N2O5]
first order in A second order in B and zero order in C. (1)
What is the relation between k1, k2 and k3?
Give the rate law for the reaction in the form of
differential equation.
Q.9 Suppose that the Sun consist entirely of hydrogen
(ii) What is the effect in rate of increasing concentrations atom and releases the energy by the nuclear reaction.
of A, B, and C two times?
Chem i str y | 18.43

4 Q.19 A first order reaction has a rate constant is


411 H →2 He with 26 MeV of energy released. If the
1.5 × 10–3 sec–1. How long will 5.0 g of this reactant take
total output power of the Sun is assumed to remain to reduce to 1.25 g?
constant at 3.9 × 1026 W find the time it will take to burn
all the hydrogen. Take the mass of the Sun as 1.7 × 1030 kg.
Q.20 A drug is known to be ineffective after it has
decomposed 30%. The original concentration of a
Q.10 The reaction A(g) + 2B(g) → C(g) + D(g) is an sample was 500 units/ml. When analyzed 20 months
elementary process. In an experiment, the initial partial later, the concentration was found to be 420units/ml.
pressure of A & B are PA = 0.6 and PB = 0.8 atm. Calculate Assuming that decomposition is of first order, what will
the ratio of rate of reaction relative to initial rate when be the expiry time of the drug?
PC becomes 0.2 atm.

Q.21 A viral preparation was inactivated in a chemical


Q.11 In the given reaction, A → B rate constant is 1.2 × bath. The inactivation process was found to be first
10–2 M s–1. What is concentration of B after 10 and 20 order in virus concentration. At the beginning of the
min, if we start with 10 M of A. experiment 2.0% of the virus was found to be inactivated
per minute. Evaluate k for inactivation process.
Q.12 For the following data for the zero order reaction
A → products. Calculate the value of k. Q.22 If a reaction A → Products, the concentrations of
Time(min) [A] reactant A are C0, aC0, a2C0, a3C0 .........after time interval
0, t, 2t, 3t,........... where a is a constant. Given 0 < a < 1.
0.0 0.10 M Show that the reaction is of first order. Also calculate
1.0 0.09 M the relation between k,a and t.
2.0 0.08 M
Q.23 The reaction SO2Cl2(g) → SO2(g) + Cl2(g) is a first
order gas reaction with k = 2.2 × 10–5 sec–1 at 320°C.
Q.13 The rate constant for a zero order reaction is
What % of SO2Cl2 is decomposed on heating this gas
2 × 10–2 mol/L sec–1, if the concentration of the reactant
for 90 min?
after 25 sec is 0.25 M, calculate the initial concentration.

Q.24 Two substances A(t1/2 = 5 mins) and B(t1/2 = 15 mins)


Q.14 A drop of solution (volume 0.10 ml) contains
follow first order kinetics are taken in such a way that
6 × 10–6 mole of H+, if the rate constant of disappearance
initially [A]= 4 [B]. Calculate the time after which the
of H+ is 1 × 107 mole liter–1 sec–1. How long would it take
concentration of both the substance will be equal.
for H+ in drop to disappear?

Q.25 At 800° C the rate of reaction


Q.15 A certain substance A is mixed with an equimolar
quantity of substance B. At the end of an hour A is 75% 2NO + H2 → N2 + H2O
reacted Calculate the time when A is 10% unreacted. Changes with the concentration of NO and H2 are
(Given: order of reaction is zero)
[NO] in M [H2] in M
1 d[NO]
− in M sec–1
Q.16 A first order reaction is 75% completed in 72 min. 2 dt
How much time will it take for (i) 50% completion
(i) 1.5×10–4 4 × 10–3 4.4 × 10–4
(ii) 87.5% completion.
(ii) 1.5×10–4 2 × 10–3 2.2 × 10–4
Q.17 A first order reaction is 20% complete in 10 min. (iii) 3.0×10–4 2 × 10–3 8.8 × 10–4
Calculate (i) the specific rate constant. (ii) the time taken
for the reactions to go to 75% completion. (a) What is the order of this reaction?
(b) What is the rate equation for the reaction?
Q.18 Show that in case of unimolecular reaction, the (c) What is the rate when
time required for 99.9% of the reaction to take place in
ten times that required for half of the reaction. [H2] = 1.5 × 1–3 M and [NO] = 1.1 × 1–3M?
1 8 . 4 4 | Chemical Kinetics

Q.26 The data below are for the reaction if NO and Cl2 Q.32 A → B + C
to form NOCl at 295 K
Time 0 t
Concentration of Concentration Initial Rate
Volume of reagent V1 V2
Cl2 [M] of NO (M s–1)
The reagent reacts with A, B and C. Find k.
0.05 0.05 1×10–3
0.15 0.05 3×10–3
[Assuming n-factor of A B & C are same]

0.05 0.15 9×10–3


Q.33 A → 2B + 3C
(a) What is the order w. r.t. NO and Cl2 in the reaction.
Time t ∞
(b) Write the rate expression
Volume of reagent V2 V3
(c) Calculate the rate constant
The reagent reacts with A, B and C.
(d) D
 etermine the reaction rate when concentration of
Cl2 and NO are 0.2 M & 0.4 M respectively. Find k. [Assuming n-factor of A B & C are same]

Q.27 The catalytic decomposition of N2O by gold at 900 Q.34 S → G + F


°C and at an initial pressure of 200 mm is 50% complete
Time t ∞
in 53 minutes and 73% complete in 100 minutes
Rotation of Glucose & Fructose r1 r∞
(i) What is the order of the reaction?
(ii) Calculate the velocity constant Find k.
(iii) How much of H2O will decompose in 100 min. at the
3
same temperature but at initial pressure of 600 mm? Q.35 The reaction AsH3(g) → As(s) + H (g) was
2 2
followed at constant volume at 310°C by measuring
Q.28 The pressure of a gas decomposing at the surface the gas pressure at intervals. Show from the following
of a solid catalyst has been measured at different times figures that reaction is of first order.
and the results are given below
Time (in hrs) 0 5 7.5 10
t (sec) 0 100 200 300
Total pressure (in min) 758 827 856 882
Pa(Pascal) 4×103 3.5×103 3×103 2.5×103

Determine the order of reaction, its rate constant. Q.36 The thermal decomposition of dimethyl ether as
measured by finding the increase in pressure of the
Q.29 The half -life period of decomposition of a reaction.
compound is 50 minutes. If the initial concentration is (CH3)2O(g) → CH4(g)+H2(g)+CO(g) at 500°C is as follow:
halved, the half life period is reduced to 25  minutes.
What is the order of reaction? Time (sec.) 390 1195 3155 ∞

Pressure increase 96 250 467 619


Q.30 In this case we have A → B + C (mm Hg)

Time t ∞ The initial pressure of ether was 312 mm Hg. Write the
rate equation for this reaction and determine the rate
Total pressure of A + B +C P2 P3
constant.
Find k.
Q.37 From the following data show that decomposition
Q.31 A → B + C of H2O2 in aqueous media is first order.

Time t ∞ Time in (minutes) 0 10 20

Total pressure of ( B +C) P2 P3 Volume (in c.c. of KMnO4) 22.8 13.3 8.25

Find k.
Chem i str y | 18.45

Q.38 The following data were obtained in experiment Q.43 The reaction A(aq) → B (aq) + C (aq) is monitored
on inversion of cane sugar by measuring optical rotation of reaction mixture
as different time interval .The species A, B and C are
Time 0 60 120 1805 360 ∞
optically active with specific rotations 20°, 30° and – 40°
(minute)
respectively. Starting with pure A if the value of optical
Angle of +13.1 +11.6 +10.2 +9.0 +5.87 –3.8 rotation was found to be 2.5° after 6.93 minutes and
rotation optical rotation was –5° after infinite time. Find the rate
(degree) constant for first order conversion of A into B and C.

Show that the reaction is of first order. After what time


would you expect a zero reading in polarimeter? Q.44 For a reaction k1 y
x
Q.39 At 100°C the gaseous reaction A → 2B + C was k2 zt
observed to be of first order. On starting with pure
A it is found that at the end of 10 minutes the total [x]t
Calculate value of ratio, at any given instant.
pressure of system is 176 mm Hg and after a long time [y] + [z]
270 mm Hg. From these data find (a) initial pressure
of A (b) the pressure of A at the end of 10 minutes B
k1
(c) the specific rate of reaction(d) the half-life period of Q.45 A
the reaction?
k2 C
[C]
Q.40 The decomposition of N2O5 according to the k1 = x hr–1; k1: k2 = 1: 10. Calculate after one hour
[A]
equation 2N2O5 (g) → 4 NO2(g) + O2(g) is a first order
reaction. After 30 min, from start of decomposition in a from the start of the reaction. Assuming only A was
closed vessel the total pressure developed is found to present in the beginning.
be 284.5 mm Hg. On complete decomposition, the total
pressure is 584.5 mm Hg. Calculate the rate constant of
Q.46 How much time would be required for B to reach
the reaction. k
maximum concentration for the reaction A →B
1
k
Q.41 A definite volume of H2O2 undergoing spontaneous
2
 → C.
decomposition required 22.8 c.c. of standard ln2 ln2
Given k1 = k =
permanganate solution for titration. After 10 and 20 4 2 2
minutes respectively the volumes of permanganate
required were 13.8 and 8.25 c.c. Q.47 For first order parallel reaction k1 and k2 are 8 and
(a) Find order of reaction. How may the result be 2 min–1 respectively at 300 K. If the activation energies
explained? for the formation of B and C are respectively 20 and
28.314 kJ/ mol respectively find the temperature at
(b) Calculate the time required for the decomposition
which B and C will be obtained in molar ratio of 2:1.
to be half completed.
k1 B
(c) Calculate the fraction of H2O2 decomposed after
A
25 minutes.
k2 C

Q.42 Hydrogen peroxide solution was stored in a mild [Given: ln4 = 1.4]
steel vessel. It was found, however, that the hydrogen
peroxide decomposed on the wall of the vessel Q.48 In gaseous reactions important for understanding
(a first order reaction) .An experiment with 100 ml of a the upper atmosphere, H2O and O react bimolecularly
solution gave 10.31 ml oxygen (corrected to 1 atm & to form two OH radicals. ∆H for This reaction is 72 kJ
273 K) after 5.1 days. Find how long the peroxide can at 500 K and Ea = 77 kJ mol–1, then calculate Ea for the
be stored before the loss of 20.00 ml oxygen occurs bimolecular recombination of 2OH radicals to form
(per 100 ml solution) under similar storage conditions H2O and O at 500 K.
if complete decomposition of the H2O2sample gave
46.34 mL oxygen.
1 8 . 4 6 | Chemical Kinetics

Q.49 The energy of activation of a first order reaction Q.58 Deduce rate law expressions for the conversion of
is 104.5 kJ mole–1 and pre - exponential factor (A) is H2 and I2 to HI at 400°C corresponding to each of the
5 × 1011 sec–1. At what temperature, will the reaction following mechanisms:
have a half-life of 1 minute?
(a) H2 + I2 → 2HI (one step)

Q.50 The specific rate constant for a reaction increases (b) I2 → 2I


by a factor of 4. If the temperature is changed from 27°C 2I + H2 → 2HI (slow)
to 47°C. Find the activation energy for the reaction.
(c) I2 → 2I
Q.51 The energy of activation and specific rate I + H2 → IH2
constant for a first order reaction at 25°C are 100 kJ/
IH2 + I → 2HI (slow)
mole and 3.46 × 10–3 sec–1 respectively. Determine the
temperature at which half life of the reaction is 2 hours. (d) Can the observed rate law expression
rate = k[H2][I2] distinguish among these mechanisms?
Q.52 A catalyst lowers the activation energy for a
certain reaction from 75 kJ to 25 kJ mol–1. What will be (e) If it is known that ultraviolet light causes the reaction
the effect on the rate of reaction at 25°C after things of H2 and I2 to proceed at 200°C with the same rate law
being equal? expression, which of these mechanisms becomes most
improbable?
Q.53 Given that the temperature coefficient for the
saponification of ethyl acetate by NaOH is 1.75.
Calculate activation energy for the saponification of
Exercise 2
ethyl acetate.
Single Correct Choice Type

Q.54 At 380°C the half-life period for the first order


Q.1 The rate of a reaction is expressed in different ways
decomposition of H2O2 is 360 min. The energy of
as follows:
activation of the reaction is 200 kJ mol–1. Calculate the
time required for 75% decomposition at 450°C. 1  d[C]  1  d[D]  1  d[A]   d[B] 
+   =–  = +  =– 
2  dt  3  dt  4  dt   dt 
Q.55 The reaction 2NO + Br2 → 2NOBr, is supposed to The reaction is:
follow the following mechanism
fast
(A) 4A + B → 2C + 3D
(i) NO + Br2 
  NOBr2

(B) B + 3D → 4A + 2C
slow
(ii) NOBr2 + NO  → 2NOBr
(C) A + B → C + D
Suggest the rate law expression.
(D) B + D → A +C

Q.56 For the reaction 2H2 + 2NO → N2 + 2H2O, the


Q.2 For the reaction system
following mechanism has been suggested 2NO → N2O2
equilibrium constant K1 (fast) 2NO (g) + O2 (g) → 2NO2(g) volume is suddenly
k2 reduced to half of its value by increasing the pressure
N2O2 + H2 
→ N2O + H2O (slow)
k
on it. If the reaction is first order with respect to O2 and
N2O + H2 → H2 + H2O (fast)
3
second order with respect to NO the reaction rate will
Establish the rate law for given reaction. (A) Increase to four times of its initial value
(B) Diminish to one-fourth of its initial value
Q.57 Reaction between NO and O2 to form NO2 is
2NO + O2 → 2NO2 follows the following mechanism (C) Diminish to one-eight of its initial value

K (D) Increase to eight times of its initial value


NO + NO  N2O2 (in rapid equilibrium)
1 

k −1

N2O2 + O2  Q.3 The energies of activation for forward and reverse


→ 2NO2 (slow)
k2

reactions for A2 + B2 
  2AB are 180 kJ mol–1 and

1  d[NO2 ]  200 kJmol respectively. The presence of catalyst lower
–1
Show that the rate of reaction is given by  
= K [NO]2[O2] 2  dt  the activation energy of both (forward and reverse)
Chem i str y | 18.47

reactions by 100 kJ mol–1. The magnitude of enthalpy Q.7 For the first order decomposition of SO2Cl2(g).
change of the reaction (A2 + B2 → 2AB) in the presence
SO2Cl2(g) → SO2(g) + Cl2(g) a graph of log (a0 – x) vs t is
of catalyst will be (in kJ mol–1).
shown in figure. What is the rate constant (sec–1)?
(A) 300 (B) 120 (C) 20 (D) –20
Time (min)
(0, 0) 2 4 6 8 10
Q.4 Which graph represents zero order reaction
-1

log(a0 - x)
[A(g) → B(g)]
-2
(A) (B)
d[B] -3
[B]
dt

(A) 0.2 (B) 4.6 × 10–1


t t
(C) 7.7 × 10–3 (D) 1.15 × 10–2
(C) (D)
Q.8 Decomposition of H2O2 is a first order reaction.
t1/2 t1/4
A solution of H2O2 labelled as “16.8 V” was left open
due to this, some H2O2 decomposed. To determine the
[A]0 [A]0 new volume strength after 2.303 hours. 20 mL of this
solution was diluted to 100 mL. 25 mL of this diluted
Q.5 Consider the reaction A → B, graph between solution was titrated against 37.5 mL of 0.02 M KMnO4
half-life (t1/2) and initial concentration (a) of the reaction is solution under acidic conditions [Given: STP is 1 atm
and 273 K].
The rate constant (in hr–1) for decomposition of H2O2 is:
t1/2 (A) 0.15 (B) 0.30 (C) 0.60 (D) 1.3

a Q.9 The variation of concentration of A with time in


two experiments starting with two different initial
d[A] concentration of A is given in the following graph. The
Hence graph between – and time will be
dt reaction is represented as A (aq) → B(aq).
What is the rate of reaction (M/min) when concentration
(A) (B)
of A in aqueous solution was 1.8 M?
d[A] d[A]
dt dt
Concentration (M)

1.5
t
1.2
1
(C) (D)
0.8 Experiment-1
d[A] d[A] 0.6 Experiment-2
dt dt 5 10 15 20
time (min)
t t (A) 0.08 M min–1 (B) 0.036 M min–1

Q.6 The reactions of higher order are rare because (C) 0.13 M min–1 (D) 1 M min–1

(A) Many body collisions involve very high activation


Q.10 The following mechanism has been proposed for
energy
the exothermic catalyzed complex reaction.
(B) Many body collisions have a very low probability fast k k
A+B 

 IAB → AB+I 
1 2→ P+A
(C)Many body collisions are not energetically favoured
If k1 is much smaller than k2. The most suitable
(D) Many body collisions can take place only in the qualitative plot of potential energy (PE.) versus reaction
gaseous phase. coordinate for the above reaction.
1 8 . 4 8 | Chemical Kinetics

(A)
(1) (2) (3) (4)
(A) ∆E for Ea for ∆Eoverall Ea for
P.E.

E+S→ES ES→EP for S®P EP→E+P


A+B A+P
IAB (B) Ea for ∆E for Ea for ∆Eoverall
AB+I
E+S→ES E+S→ES ES→EP for S®P
reaction coordinate
(B) (C) Ea for Ea for ∆Eoverall ∆E for
ES→EP EP→E+P for S®P EP→E+P
(D) Ea for Ea for Ea for ∆Eoverall
E+S→ES ES→EP EP→E+P for S®P
P.E.

AB+I A+P
A+B
IAB (E) ∆E for ∆Eoverall ∆E for Eafor
E+S→ES for S®P EP→E+P EP→E+P
reaction coordinate

(C)
Multiple Correct Choice Type

Q.13 SO3 gas is entering the environment at a constant


P.E.

rate of 6.93 × 10–5 gm/L/day due to the emission of


A+B AB+I
A+P
polluting gases from thermal power plant but at the
IAB
same time it is decomposing & following first order
reaction coordinate kinetics with half-life of 100 days.
Based on above information select the true statement (s).
(D)
(A) Concentration of SO3 in Kota is 1.25 × 10–5 M
(Assume SO3 present in air reaches steady state)
P. E.

A+B AB+I (B) If 103 L of air is passed through 1L pure water


IAB A+P (assuming all SO3 to be dissolved in it) & resulting
reaction coordinate solution is titrated against 1 N NaOH solution, 15 ml is
required to reach end point
Q.11 The following mechanism has been proposed for
(C) An industry is manufacturing H2SO4 at the rate of
the reaction of NO with Br2 to form NOBr
980 kg per day with the use of SO3 in air it should use
NO(g) + Br2(g) 
 NOBr2(g)

 8 × 105 litre air/day
NOBr2(g) + NO(g) → 2NOBr(g) (D) If SO3 emission is stopped then after 1000 days its
concentrations will reduce to ≈ 1.2 × 10–3 M.
If the second step is the rate determining step, the
order of the reaction with respect to NO(g) is
Q.14 For the reaction A → B. The rate law expression is
(A) 2 (B) 1 (C) 0 (D) 3
d[A]
– = k[A]1/2. If initial concentration of [A] is [A]0, then
dt
Q.12 Choose the correct set of indentifications.
2
(A) The integrated rate expression is k= (A1/2 − A1/2 )
t 0
Potential energy

(B) The graph of A vs t will be


K
(4) (C) The half-life period t1/2 =
(1)
(2) (3)
2[A]1/2
0
(D) The time taken for 75% completion of reaction
reaction coordinate
[A]0
t3/4 =
k
t1/2

a
Chem i str y | 18.49

2
k1=6.93 x 10 min
-1
B Q.19 Which of the following is/are correct statement?

Q.15 Consider the reaction, A (A) Stoichiometry of a reaction tells about the order of
C the elementary reaction.
k2=13.85 x 10 min-1
(B) For a zero-order reaction, rate and the rate constant
A, B and C all are optically active compound. If optical are identical
rotation per unit concentration of A, B and C are 60°, (C) A zero-order reaction is controlled by factors other
–72°, 42° and initial concentration of A is 2 M then than concentration of reactants.
select write statements(s).
(D) A zero-order reaction is always elementary reaction
(A) Solution will be optically active and dextro after
very long time
Assertion Reasoning Type
(B) Solution will be optically active and laevo after very
long time (A) S
 tatement-I is true, statement-II is true and
statement-II is correct explanation for statement-I
(C) Half life of reaction is 15 min
(B) Statement-I is true, statement-II is true and
(D) After 75% conversion of A in to B and C angle of statement-II is NOT the correct explanation for
rotation of solution will be 36° statement-I
(C) Statement-I is true, statement-II is false.
Q.16 Which of the following statement is incorrect?
(D) Statement-I is false, statement-II is true.
(A) The order of reaction is the sum of powers of all the
concentration terms in the rate equation.
Q.20 Statement-I: A fractional order reaction must be
(B) The order of reaction with respect to one reactant
a complex reaction.
is the ratio of the change of logarithm of the rate of
the reaction to the change in the logarithm of the Statement-II: Fractional order of RDS equals to overall
concentration of the particular reactant, keeping the order of a complex reaction.
concentrations of all other reactants constant.
Q.21 Statement-I: Temperature coefficient of a one
(C) Orders of reactions cannot be fractional.
step reaction may be negative
(D) The order of a reaction can only be determined
Statement-II: The rate of reaction having negative
from the stoichiometric equation for the reaction.
order with respect to a reactant decreases with the
increase in concentration of the reactant.
Q.17 Select incorrect statement(s)
(A) Unit of pre-exponential factor (A) for second order Q.22 Statement-I: The overall rate of a reversible
reaction is mol L–1 s–1 reaction may decrease with the increase in temperature.

(B) A zero-order reaction must be a complex reaction Statement-II: When the activation energy of forward
reaction is less than that of backward reaction, then the
(C) Molecularity is defined only for RDS in a complex increase in the rate of backward reaction is more than
reaction that of forward reaction on increasing the temperature
(D) Decay constant (λ) of radioactive substance is
affected by temperature Q.23 Statement-I: In a reversible endothermic reaction
Ea of forward reaction is higher than that of backward
E reaction.
1
Q.18 In a consecutive reaction system A → B
E Statement-II: The threshold energy of forward reaction
→ 2 C when E1 is much greater than E2, the yield of is more than that of backward reaction.
B increase with
(A) Increase in temperature Q.24 Statement-I: The time of completion of reaction
of type A → product
(B) Decreases in temperature
(order < 1) may be determined.
(C) Increase in initial concentration of A
Statement-II: Reactions with order ≥ 1 are either too
(D) Decrease in initial concentration of A slow or too fast and hence the time of completion
cannot be determined
1 8 . 5 0 | Chemical Kinetics

Q.25 Statement-I: A catalyst provides an alternative Q.30 What will be the concentration of A at time t, if
path to the reaction in which conversion of reaction n1 = 1 and n2 = 2 ?
into products takes place quickly  e−kt 
(A) [A0] .e–kt (B) [A0 ]  
Statement-II: The catalyst forms an activated complex  2 − e−kt 
 
of lower potential energy with the reactants by which
 e−kt 
more number of molecules are able to cross the barrier (C) [A0 ]   (D) [A0](1 – 2 . e–kt)
per unit of time.  1 − e−kt 
 

Paragraph 3
Comprehension Type
For the reaction sequential reaction
Paragraph 1 k k
k1 B A → B 
1 2→ C
For a hypothetical elementary reaction A
the concentration of A, B & C at anytime ‘t’ is given by
k2 C
k1 1 k1 [A]0 −k1t −k 2t
where = . Initially only 2 moles of A is present. [A]1 = [A]0e–kt ; [B]1 = [e −e ]
k2 2 (k 2 – k1 )
[C]1 = [A0] – ([A]1 + [B]1)
Q.26 The total number of moles of A, B & C at the end
of 50% reaction are Q.31 The time at which concentration of B is maximum is
(A) 2 (B) 3 (C) 4 (D) 5 k1 k
1
(A) (B) ln 1
k 2 − k1 k 2 − k1 k 2
Q.27 Number of moles of B are
(A) 2 (B) 1 (C) 0.666 (D) 0.33 1 k k2
(C) ln 1 (D)
k1 − k 2 k 2 k 2 − k1
Paragraph 2
The gaseous reaction: n1 A(g) → n2B (g) is first order Q.32 Select the correct option if k1 = 1000s–1 and
with respect to A. The true rate constant of reaction is k2 = 20s–1.
k. The reaction is studied at a constant pressure and
temperature. Initially, the moles of A were ‘a’ and no B (A) [C]1 (B) [C]1

were present.
Conc.

Conc.

[B]1
[A]1 [B]1 [A]1
Q.28 How many moles of A are present at time, t? time time
−n1kt
(A) a ⋅ e-kt (B) a ⋅ e
(C) [C]1 (D) [C]1
−n2kt −n2kt
(C) a ⋅ e (D) a(1 − e )
Conc.
Conc.

[B]1

Q.29 If the initial volume of system were v 0° then the [B]1 [A]1 [A]1
volume of system after time t will be time time

n1 v 0
(A)
n2 Paragraph 4

n2 v 0 Oxidation of metals is generally a slow electrochemical


(B) reaction involving many steps. These steps involve
n1
electron transfer reactions. A particular type of oxidation
n  involve overall first order kinetics with respect to fraction
n  −n kt 
(C) v 0  2 +  1 − 2  ⋅ e 1  of unoxidised metal (1 – ƒ) surface thickness relative to
 n1  n1   maximum thickness (T) of oxidised surface, when metal
surface is exposed to air for considerable period of time
n  n  −n kt 
(D) v 0  2 +  2 – 1  ⋅ e 2  Rate law: = k(1 – ƒ), where f = x/T,
 n1  n1  
Chem i str y | 18.51

x = thickness of oxide film at 200 hrs Match the Columns


0 t
time ‘t' and Q.37 For the reaction of type A(g) → 2B(g)
T = thickness of oxide film at Column I contains four entries and column II contains
t=∞ four entries. Entry of column I are to be matched with
A graph of ln( 1 – ƒ) vs t is
-3 only one entry of column II
shown inthe adjacent figure. In(1-)
Column I Column II
Q.33 The time taken for thickness to grow 50% of T is
(p)
d[B] −d[A]
(A) 23.1 hrs (B) 46.2hrs (A) vs
dt dt
(C) 100 hrs (D) 92.4 hrs for first order

Q.34 The exponential variation of ‘f' with t(hrs) is given by


(A) [1 – e–3t/200] (B) e–3t/200–1
(B) [A] vs t for first (q)
(C) e–3t/200 (D) e3t/200
order

Paragraph 5
A reaction is said to be first order if it's rate is
proportional to the concentration of reactant. Let us
consider a reaction
A(g) B(g) + C(g) (C) [B] vs t for first (r)
order
At t = 0 a 0 0
At time t a-x x x
dx
The rate of reaction is govern by the expression
dt
= k(a – x) and integrated rate equation for a given
reaction is represented as (D) [A] vs t for zero (s)
 a  order
1
k= ln  a − x  where a = initial concentration and
t  
(a – x) = concentration of A after time t.

Q.35 Thermal decomposition of compound X is a first


order reaction .If 75% of X is decomposed in 100 min.
How long will it take for 90% of the compound to Q.38 Column-I and column-II. Entry of column-I are
decompose? Given: log 2 = 0.30 to be matched with one or more than one entries of
(A) 190 min (B) 176.66 min column-II and vice versa.

(C) 166.66 min (D) 156.66 min Column-I (Graphs Column-II (Co-ordinates)
reaction A → Products)
(A) (p) ln [A] (y-axis), t(x-axis)
Q.36 Consider a reaction A(g) 3B(g) + 2C(g) with
(order = 1)
rate constant 1.386 × 10–2 min–1. Starting with 2 moles
of A in 12.5 litre vessel initially, if reaction is allowed to
takes place at constant pressure & at 298 K then find
the concentration of B after 100 min.
(A) 0.04 M (B) 0.36 M
(C) 0.09 M (D) None of these
1 8 . 5 2 | Chemical Kinetics

Q.4 Which one of the following statements is incorrect


(B) (q) t1/2(y-axis), [A0] (x-axis)
about order of reaction?  (2005)
(order = 1)
(A) Order of reaction is determined experimentally
(B) Order of reaction is equal to sum of the power of
concentration terms in differential rate law
(C) It is not affected with stoichiometric coefficient of
(C) (r) r(y-axis), t(x-axis)(order > 0) the reactants
(D) Order cannot be fractional

Q.5 Plots showing the variation of the rate constant


(k) with temperature (T) are given below. The plot that
follows Arrhenius equation is  (2010)
(A) (B)
(D) (s) r(y-axis), t(x-axis)(order = 0)
(t) t 1/2(y-axis), [A0] (x-axis) (order K K
> 1)

1 T T
(u)  (y-axis), t(x-axis)
[A] (C) (D)
(order = 2)
K K
(v) r (y-axis), [A](x-axis)(order = 1)

Previous Years’ Questions T T


Comprehension (Q.6), (Q. 7), (Q.8)
Q.1 The rate constant of a reaction depends on(1981) Carbon-14 is used to determine the age of organic
(A) Temperature material. The procedure is based on the formation of
14
C by neutron capture in the upper atmosphere.
(B) Initial concentration of the reactants
14
14
(C) Time of reaction 7 N +0 n1 →6 C +1 p1
(D) Extent of reaction 14
C is absorbed by living organisms during
photosynthesis. The 14C content is constant in living
Q.2 A catalyst is a substance which  (1983) organism once the plant or animal dies, the uptake
(A) Increases the equilibrium concentration of the of carbon dioxide by it ceases and the level of 14C in
product the dead being, falls due to the decay which C–14
14
14
(B) Changes the equilibrium constant of the reaction undergoes 6 C →7 N + β–
(C) Shortens the time to reach equilibrium The half-life period of 14C is 5770 yr.
(D) Supplies energy to the reaction The decay constant (λ) can be calculated by using the
following formula
Q.3 If I is the intensity of absorbed light and C is the 0.693
λ=
concentration of AB for the photochemical process. t1/2
AB + hv → AB*, the rate of formation of AB* is directly The comparison of the β– activity of the dead matter
proportional to  (2001) with that of the carbon still in circulation enables
measurement of the period of the isolation of the
(A) C (B) I (C) I2 (D) C.I
material from the living cycle. The method however,
ceases to be accurate over periods longer than 30,000
yr. The proportion of 14C to 12C in living matter is
1: 1012.  (2006)
Chem i str y | 18.53

Q.6 Which of the following option is correct? Q.11 A first order gas reaction has A’ = 1.5 × 10–6 per
second at 200° C. If the reaction is allowed to run for 10
(A) In living organisms, circulation of 14C from
h, what percentage of the initial concentration would
atmosphere is high so the carbon content is
have change in the product? What is the half-life of this
constant in organism
reaction? (1987)
(B) Carbon dating can be used to find out the age of
earth crust and rocks
Q.12 An experiment requires minimum beta activity
(C) Radioactive absorption due to cosmic radiation is produced at the rate of 346 beta particles per minute.
equal to the rate of radioactive decay, hence the The half-life period of 42Mo99 which is a beta emitter is
carbon content remains constant in living organisms 66.6 h. Find the minimum amount of 42Mo99 required to
carry out the experiment in 6.909 h.  (1989)
(D) Carbon dating cannot be used to determine
concentration of 14C in dead beings.
Q.13 The gas phase decomposition of dimethyl ether
follows first order kinetics
Q.7 What should be the age of fossil for meaningful
determination of its age? CH3–O–CH3(g) → CH4(g)+ H2(g)+CO(g)
(A) 6 yr The reaction is carried out in a constant volume
container at 500°C and has a half-life of 14.5 min.
(B) 6000 yr
Initially only dimethyl ether is present at a pressure of
(C) 60,000 yr 0.40 atm. What is the total pressure of the system after
(D) It can be used to calculate any age 12 min? Assume ideal gas behaviour. (1993)

Q.8 A nuclear explosion has taken place leading to Q.14 (a) The rate constant of a reaction is 1.5 × 107 s–1 at
increase in concentration of C14 in nearby areas. C14 50° C and 4.5 × 107 s–1 at 100°C. Evaluate the Arrhenius
concentration is C1 in nearby areas and C2 in areas far parameters A and Ea.  (1998)
1
away. If the age of the fossil is determined to be T1 and (b) For the reaction, N2O5 (g) → 2NO2 (g) + O2 (g)
T2 at the places respectively then 2
calculate the mole fraction N2O5(g) decomposed at a
(A) The age of fossil will increase at the place where constant volume and temperature, if the initial pressure
1 C is 600 mm Hg and the pressure at any time is 960 mm
explosion has taken place and T1 – T2 = ln 1
λ C2 Hg. Assume ideal gas behaviour.
(B) The age of fossil will decrease at the place where
1 C Q.15 2X(g) → 3Y(g) + 2Z(g)
explosion has taken place and T1 – T2 = ln 1
λ C2
Time (in min) 0 100 200
(C) The age of fossil will be determined to be the same
Partial pressure of X (in mm of Hg) 800 400 200
T1 C1
(D) =
T2 C2 Assuming ideal gas condition. Calculate (2005)
(A) Order of reaction
Q.9 The concentration of R in the reaction
(B) Rate constant
R → P was measured as a function of time and the
(C) Time taken for 75% completion of reaction.
following data obtained:
(D) Total pressure when px = 700 mm
[R] (molar) 1.0 0.75 0.40 0.10
t (min) 0.0 0.05 0.12 0.18 Q.16 Consider a reaction aG + bH → Products.
When concentration of both the reactants G and H is
The order of the reaction is  (2010)
doubled, the rate increases by eight times. However,
when concentration of G is doubled keeping the
Q.10 Radioactive decay is a first order process. Radioactive concentration of H fixed, the rate is doubled. The
carbon in wood sample decays with a half-life of 5770 yr. overall order of the reaction is  (2007)
What is the rate constant (in yr–1) for the decay? What
fraction would remain after 11540 yr?  (1984) (A) 0 (B) 1 (C) 2 (D) 3
1 8 . 5 4 | Chemical Kinetics

Q.17 Under the same reaction conditions, initial


concentration of 1.386 mol dm-3 of a substance [Q]0
becomes half in 40 seconds and 20 seconds through [Q]
first order and zero order kinetics, respectively. Ratio 1
 k1 
  of the rate constants for first order (k1) and zero
 k0 
order (k0) of the reactions is  (2008) Time

(A) 0.5 mol-1 dm3 (B) 1.0 mol dm-3 (A) 2 (B) 3 (C) 0 (D) 1
(C) 1.5 mol dm-3 (D) 2.0 mol-1 dm3
Q.20 For the elementary reaction M → N, the rate
of disappearance of M increases by a factor of 8 upon
Q.18 An organic compound undergoes first-order
doubling the concentration of M. The order of the
decomposition. The time taken for its decomposition
reaction with respect to M is  (2014)
to 1/8 and 1/10 of its initial concentration are t1/8 and
[t ] (A) 4 (B) 3 (C) 2 (D) 1
t1/10 respectively. What is the value of 1/8 × 10? (take
[t1/10 ]
log102=0.3) (2013) Q.21 According to the Arrhenius equation,  (2015)
(A) A high activation energy usually implies a fast
Q.19 In the reaction, P + Q → R + S the time reaction
taken for 75% reaction of P is twice the time taken for (B) Rate constant increases with increase in temperature.
50% reaction of P. The concentration of Q varies with This is due to a greater number of collisions whose
reaction time as shown in the figure. The overall order energy exceeds the activation energy
of the reaction is  (2013)
(C) Higher the magnitude of activation energy, stronger
is the temperature dependence of the rate constant
(D) The pre-exponential factor is a measure of the rate
at which collisions occur, irrespective of their energy.

PlancEssential Questions
JEE Main/Boards JEE Advanced/Boards

Exercise 1 Exercise 1
Q.1 Q.4 Q.7 Q.9 Q.7 Q.9 Q.20 Q.36

Q.12 Q.19 Q.24 Q.43 Q.58(e)

Exercise 2 Exercise 2
Q.2 Q.10 Q.16 Q.8 Q.12 Q.28 Q.29
Q.33 Q.34 Q.37

Previous Years’ Questions


Previous Years’ Questions
Q.9 Q.14 Q.15
Q.12 Q.15
Chem i str y | 18.55

Answer Key

JEE Main/Boards d(D) k1k 3 (A)(B)


Q.18 (a) =
dt k 2 + k3
Exercise 1 A1 A3
(b) Ea = Ea1 + Ea3 – Ea2 A =
Q.1 (a) 6.25 ; (b) 14.3; (c) 0 % A2
Q.2 2 × 10–2 min–1 Q.20 0.0805

Q.3 P1 = 379.55 mmHg, t7/n = 399.96 min Q.21 161 minutes

Q.4 0.0207 min–1 Q.22 55.33 kJ mole–1

Q.5 k = 0.022 hr–1 Q.23 Rate = K(A][13]2 rate will became 8 times

Q.6 15.13 week Q.24 K27 = 3.85 × 10–4 sec–1

Q.7 20 min K47 = 11.55 × 10–4 sec–1.

Q.8 0.26: 1 E = 43.78 kJ/mol

Q.9 (a) 43.46 kJ mol–1 (b) 20.47 hour Q.25 It is first order kinetics with

Q.10 % decomposition = 67.21 % k = 8.64 × 10–1

Q.11 k = 0 0327 min–1 Q.26 2

Q.12 0.537 Q.27 Ea = 2.2 × 104

Q.13 4.83 mins



A = 5.42 × 1010

Q.14 0.0025 m Q.28 (a) = 1 × 10–2 mol l–1 s–1

Q.15 Er=6 × 104 J; Eb = 9.3 × 104 J (b) = 5.495 × 10–3 mol–1 l–1 s–1

Q.16 a = b= 1 Q.29 Rate = 2 × 10–4 mol dm–3 min–1 x = 18.12%


E1k1 + E2k 2 Q.30 k = 0.02231 min–1 t = 62.07 min
Q.17 E=
k1 + k 2

Exercise 2

Single Correct Choice Type

Q.1 B Q.2 A Q.3 A Q.4 D Q.5 D Q.6 D


Q.7 C Q.8 B Q.9 C Q.10 D Q.11 D Q.12 C
Q.13 C Q.14 D Q.15 B Q.16 A Q.17 D Q.18 C
Q.19 B Q.20 B Q.21 C Q.22 D

Previous Years' Questions


Q.1 A Q.2 C Q.3 C Q.4 A Q.5 D Q.6 C
Q.7 B Q.8 C Q.9 A Q.10 D Q.11 A Q.12 D
Q.13 D Q.14 A Q.15 B Q.16 B Q.17 C Q.18 C
Q.19 D Q.20 B Q.21 B Q.22 A Q.23 D Q.24 A
1 8 . 5 6 | Chemical Kinetics

JEE Advanced/Boards Q.24 15 min

Q.25 (a) Third order, (b) r = k[NO]2[H2],


Exercise 1
(c) 8.85 ×10–3 M sec–1
1 d[NO]
Q.1 (i) r= = 9 × 10–4 mol litre–1 sec–1 Q.26 (a) Order w.r.t. NO = 2 and w.r.t. Cl2 = 1,
4 dt
(b) r = K[NO]2[Cl2],
(ii) 36 × 10–4 mol litre–1 sec–1
(c) K = 8 L2 mol–2s–1,
(iii) 54 × 10–4 mol litre–1 sec–1
(d) Rate = 0.256 mole L–1 s–1
Q.2 (i) 7.2 mol litre min ,
–1 –1

Q.27 (i) First order


(ii) 7.2 mol litre–1 min–1
(ii) k = 1.308 × 10–2 min–1
Q.3 (a) 1 × 10–4 mol L–1 s–1, (iii) 73%
(b) 3 × 10 mol L s
–4 –1 –1

dx Q.28 (i) Zero order, (ii) K = 5 Pa/s


Q.4 (i) = k [A][B]2
dt
Q.29 Zero order
(ii) Rate increases by 8 times
1 P3
Q.5 7.23 × 10–6 Ms–1, 0.012 atm min–1 Q.30 k = ln
t 2(P3 − P2 )
Q.6 Rate increases by 27 times
1 P3
Q.31 k = ln
Q.7 (a) 0.019 mol L s ,
–1 –1 t (P3 − P2 )

(b) 0.037 mol L–1 s–1 1 V1


Q.32 k = ln
t (2V1 − V2 )
Q.8 2k1= k2 = 4k3
1 4V3
Q.33 k = ln
Q.9 2.72 × 10 sec
18
t 5(V3 − V2 )

Q.10 1/6 1 r
Q.34 k = ln ∞
t (r∞ − r1 )
Q.11 (i) 7.2 M, (ii) 10 M
Q.35 First order
Q.12 K= 0.01 M min –1
Q.36 (i) r = K[(CH3)2O]2 0.000428 sec–1
Q.13 0.75 M Q.37 First order
Q.14 6 × 10–9 sec Q.38 966 min

Q.15 1.2 hr Q.39 (a) 90 mm,


(b) 47 mm,
Q.16 (i) 36 min (ii) 108 min
(c) 6.49 × 10–2 per minutes,
Q.17 (i) 0.0223 min , (ii) 62.17 min
–1
(d) 10.677 min.
Q.19 924.362 sec Q.40 k1 =2.605 × 10–3 min–1

Q.20 Expiry time = 41 months Q.41 (a) first order,


(b) 13.75 minutes,
Q.21 3.3 × 10–4 s–1
(c) 0.716 0.180 atm, 47.69 sec
2.303 1
Q.22 k = log
t a Q.42 11.45 days
Q.23 11.2% Q.43 0.1 min–1
Chem i str y | 18.57

1 Q.51 306 K
Q.44
e
(K1 +K2 )
t −1 Q.52 Rate of reaction increases 5.81 × 108 times

[C] 10 11x Q.53 10.757 k cal mol–1


Q.45 = (e – 1)
[A] 11 Q.54 t=20.4 minutes 56. 2000 K
Q.46 t = 4 min Q.55 r = K’ [NO]2 [Br2]
Q.47 0379.75 K Q.56 k = k'2
Q.48 5 kJ mol–1 Q.57 r = K[NO]2 [H2], where K=k2 × K1/k-1
Q.49 349.1 K Q.58 (d) No, (e) mechanism (a) is incorrect
Q.50 55.33 kJ mole –1

Exercise 2

Single Correct Choice Type

Q.1 B Q.2 D Q.3 C Q.4 D Q.5 C Q.6 B

Q.7 C Q.8 C Q.9 A Q.10 A Q.11 A Q.12 B

Multiple Correct Choice Type

Q.13 A, D Q.14 A,B,D Q.15 A, D Q.16 C, D Q.17 A, C, D Q.18 A, C

Q.19 A, B, C

Comprehension Type

Q.20 C Q.21 D Q.22 A Q.23 C Q.24 C Q.25 A

Q.26 B Q.27 C Q.28 B Q.29 D Q.30 B Q.31 C

Q.32 C Q.33 B Q.34 A Q.35 C Q.36 B

Match the Columns

Q.37 A → s; B → r; C → p; D → q Q.38 A → p; B → q, s; C → r, t; D → v

Previous Years’ Questions

Q.1 A Q.2 C Q.3 D Q.4 D Q.5 A Q.6 A

Q.7 B Q.8 A Q.9 0 Q.10 0.25 Q.11 128.33h

Q.12 3.56x10–16 Q.13 0.75 atm Q.14 4.15x103 s–1, 0.375

Q.15 (a) 1 (b) 693x10-3 min-1 (c) 200 min (d) 950 mm Q.16 D

Q.17 A Q.18 9 Q.19 D Q.20 C Q.21 B, C, D


1 8 . 5 8 | Chemical Kinetics

Solutions

JEE Main/Boards 3
⇒k=
[A0 ]
Exercise 1
Now, at time t = 2 hours,
Sol 1: (a) A + B → C −1 1 3 −6
+ = –2 × =
We have, first order in A and 0 order in B. [A] [A0 ] [A0 ] [A0 ]
−d[A] −d[A] 1 7 [A] 1
Rate = = k[A] 1[B] 0 = = – k[A] ⇒ = ⇒ =
dt dt [A] [A0 ] [A0 ] 7
For first order reaction, we know
100
A left is % or 14. 3 %
[A] = [A0] e–kt 7
We have, at t = 1 hour
(c) If the reaction is 0 order
75% of A reacted
d[A]
1 Rate = =k
[A] = A0 – 0. 75; A = A0 dt
4 [A] t
1 ⇒ ∫ d[A] = ∫ k dt
[A ] = [A0] e–kt ⇒ k = ln 4
4 0 [A0 ] 0

Now t = 2 hour, ⇒ [A] – [A0] = –kt


[A] 1
= e–2×ln 4 = ⇒ [A] = [A0]-kt
[A0 ] 16
Now, at time t = 1 hour
[A] = 6. 25 % of A0 [A0 ]
[A] =
∴6. 25% of A will be left unreacted. 4
[A0 ]
– [A0] = –k
(b) Here, reaction is first order in both A and B 4
3
d[A] ⇒k= [A ]
∴Rate = = k[A] [B] 4 0
dt
We see that, if t or [A] = 0
Since the initial concentration of A and B is the same
and they have the same coefficient, their concentration [A0 ] 4
t= = hours < 2 hours
will remain same at any time. k 3
−d[A] The reactions will be finished before 2 hours and 6 % of
= k[A] 2 A will be left after 2 hours.
dt
[A] t −
d[A]
⇒ ∫ = ∫ −kdt Sol 2: CH3CH2NO2 + OH– → CH3 CHNO2 + H2O
2
[A0 ] [A] 0 (A)
Since this is a first order reaction
1 1 [A] = [A0] e–kt
⇒− + = –k(t – 0) = kt
[A] [A0 ] 1
Now, we have at time t = minute,
2
[A0 ]
At time t = 1 hour, [A] = [A] = 0. 99[A0]
4
4 1 (1 % of nitroethane reacted)
∴− + = –k
[A0 ] [A0 ] ∴0. 99[A0] = [A0] e–k. 1/2
Chem i str y | 18.59

⇒ P0 – 2x = 158. 22
⇒ −k = loge(0. 99)
2 233.8 − 158.22
−k ⇒x= = 37. 78
⇒ = 0. 01 min. 2
2
Now, total pressure at t = 75 min
⇒ k = 0. 02 min–1
Pt = P0 + 3x + 32. 5
k = 2 × 10–2 min–1
= 233. 8 + (3 × 37. 78) + 32. 5

Sol 3: Here, since the volume of liquid is negligible and Pt = 379. 55 mm Hg


as the liquid begins to form, pressure due to liquid will
7
Time fraction, t7/8 is time at which th of reaction gets
always be equal to 32. 5 mm Hg. Now, let initial pressure 8
P
of P(g) be P0 mm Hg, then according to stoichiometry completed, ie. P∆ = 0
8
P0 2.303
2P(g) → 4Q(g) + R(g) + S() Using rate law, = P0e –kt ⇒ t = log 8
8 R
at time t = 399. 96 min.
t = 0 min P0 0 0 0
Sol 4: We have, the reaction as,
t = 30 min P0 – 2x 4x x 32. 5
Bn+ → B(n-2)+ + 4e

t = ∞ 0 2P0 P0 32. 5
At time t = 0 V0 0
2
Now, we are given, Pt = 317 mm Hg at t=30 min. total time
∴P0 – 2x + 4x + x + 32. 5 = 317 t = 10 min   V0 – x x
⇒ P0 + 3x = 284. 5  …….(i) Now, for titration we have
At time, t = ∞ (very long time), Pt = 617 mm Hg Bn+ + 2e → B(n–2)+
P0 From this, we see that for volume V0 of Bn+, 2V volume
∴2P0 + + 32. 5 = 617
2 of titrant is used.
5P0 at time t = 0, volume of titrant used = 25 mL
⇒ = 584. 5
2
⇒ 2V0 = 25 mL
⇒ P0 = 233. 8 mm Hg
⇒ V0 =12. 5 mL
Putting this in (i), we get,
For titration of B(n–1)+, we have
3x = 284. 5 – 233. 8
B(n+4)+ + 5e → B(n–1)-
⇒ x = 16. 9
i.e. for volume V of Bn+ + B(n+4), 5V volume of titrant is
Now, since this is first order reaction, we have used.
at t = 30 min a = 233.8 x = 16.9 ∴ At time t = 10 min,
2.303 P 2.303 233.8 Volume of titrant used = 32 mL
k= log 0 = log
t Pp 30 233.8 − (2 × 16.9) ⇒ 2(V0 – x) + 5x = 32
⇒ k = 5.2 × 10–3 min–1
⇒ 2V0 + 3x = 32
Now, at t = 75 min, we have
⇒3x = 32 – 25
2.303 P
k= log 0 7
t Pp ⇒x=
3
⇒ Pt = P0 × 10–kt/2. 303 Now, since this is a first order reaction, rate law is
–5.2×10 –3 ×75 2.303 V 2.303 12.5
= 233. 8 × 10 2.303 k= log 0 = log
t V n+ 10 12.5 − x
B
Pt = 158. 22 m Hg
1 8 . 6 0 | Chemical Kinetics

2.303 12.5 2.303 3.956


= log ; k = 0. 0207 min–1 k= log
10 7 6 3.956 – 1.281
12.5 −
3 k = 0. 065
Sol 5: We have initially, volume strength = 20 2.481
Now, when, τ = inch, we have
According to reaction 1000
2.303 3.956
1 0. 065= log
H2O2 → H2O + O t 3.956 – 2.481
2 2
1 2.303
1 mole (34g) of H2O2 gives moles of O2 i. e. 11. 21 ⇒t= × 0.49
at STP 2 0.065
t = 15. 13 weeks
Now, volume strength is the volume of O2 given by unit
volume of H2O2 t = 15. 13 weeks
Let normality of H2O2 = N longer = 9. 13 weeks
weight
Then, N = Sol 7: We have,
Eq. weight
H +
⇒ Weight = 17 N (equivalent weight = 17) A →B + C

Now, 34 g of H2O2 gives 11. 21 of O2 Let initially x moles of A be present


H +
11.2 × 17N A →B + C
\ 17 N g of H2O2 gives of O2
34 At t = 0 x 0 0
11.2 × 17N At t = ∞ 0 x x
Volume strength = = 5. 6 N
34
We know optical rotation of B and C are 40º and –80º
Now, initially volume strength = 20 respectively at completion,
20
⇒ N0 = = 3. 75 N total optical rotation
5.6
⇒ 40x – 80x = -10
At time, t = 6 hours, let normality be N.
⇒ x = 0. 5
Since, 10 mL of solution be diluted to 100 mL,
Now, optical rotation at t = 0
N.10
Now normality = = 0. 1 N
100 ⇒ 60 × 0. 5 = 30º
Now, since this is first order reaction, according to rate Using first order rate law,
law, as N is proportional to number of moles of H2O2 2.303 θ − θ∞
k= log 0
2.303 N t θt − θ∞
k= log 0 = 2.303 log 3.57 ; k = 0. 022 hr–1
t N 6 3.125 q0 = initial optical rotation

Sol 6: We have for oxide film formation. 0.693 0.693 1


or t1/2 = = +
k 2.303 θ0 − θ∞
2.303 τ log
k= log max θt − θ∞
6 τmax − τ
Putting values
Where, tmax = maximum thickness of oxide film
(at t → ∞) 0.693 20
t1/2 = +
τ = thickness at time t 2.303 (30 − 1 − 20)
log
(5 − 1 − 20)
We are given
since θ = 5º
3.956
tmax = inch 0.3 × 20
1000 at t = 20 min =
log2
1.281 t1/2 = 20 min
τ= inch, at t = 6 weeks
1000
Chem i str y | 18.61

Sol 8: Since, volume and temperature is constant, we +(log3 × 2.303 × 8.314 × 293 × 276)
∴ EA =
take P ∝ n 17
CH3OCH3(g)→CH4(g)+CO(g)+H2(g) EA = 43. 46 kJ/mol
at (b) When T2 = (273 + 40) K = 313K,
t=0 0. 4 0 0 0 k2 43.460  1 1 
We have, log =  − 
t = 4.5hr. 0. 4–x x x x k1 303 × 8.314  293 313 
We are given k = 4. 78 × 10 , since unit is –3 ⇒ k2 = 3312 k1

min–1. It is a first order reaction and 1


Further, as k ∝
t
PA = P0e–kt
t2 k1 6.4
= 0.4 × e(–4. 78 × 10
–3 × 4. 5 × 60)
∴ = ⇒ t2 =
t1 k2 3.12
⇒ PA = 0.11 t2 = 20. 47 hours
0. 4 – x = 0. 11
⇒ x = 0. 29 Sol 10: We have, according to Arrhenius equation

Now, at t = 4. 5hr, mean molecular mass, M2 k2 EA 1 1


log =  − 
k1 2.303R  t1 t2 
(P0 − x)MCH + x.MCH + MCO + x + M0 + xMH
3OCH3 Here,
= 4 2
P+x+x+x+x
EA = 70 ks mol–1
0.11 × 46 × 0.29 × 16 + 0.29 × 28 + 0.29 × 2
= = 18. 77 t1 = (273 + 25) K = 298 K
0.11 + 0.29 + 0.29 + 0.29
t2 = (273 + 40) K = 313 K
at t = 0, molecular mass
Putting values
M1 = MCH = 46 k2
3OCH3 70000  1 1 
log =  − 
Pt = 0. 4 – x + 3x = 0. 4 × 2x = 0. 98 k1 2.303 × 8.314  298 313 
According to Graham’s law ⇒ k2 = 3. 87 k1
P Now, equation
Rate of diffusion ∝
M A→B
D0 P0 M2 18.77 0.4 According to rate law
= = ×
Dt Pt M1 46 0.98
[A] = [A0] e–kt
D0 At T = 298 K, k = k1, … t = 20 min,
= 0. 26
Dt 3[A0 ]
[A] =
4
Sol 9: (a) According to Arrhenius equation, we have
3 −k .20
∴ = e 1
k2 EA  1 1  4
log =  − 
k1 2.303  T1 T2  ⇒ k1 = 0. 014 min–1
Here, we have ∴ k2 = 3. 87 k1 = 0. 055 min–1
T1 = (273 + 20)K = 293 K Now, at T = 313 K, t = 20 min,
T2 = (273 + 3)K = 276 K k = 0. 055 min–1
k1 [A] = [A0] e–0. 055 × 20
k2 =
3
Putting values [A] = 0. 3279 [A0]
∴ 67. 21 % of A got decomposed.
1 EA  1 1 
log =  − 
3 2.303 × 8.314  293 276 
1 8 . 6 2 | Chemical Kinetics

Sol 11: We have, according to Arrhenius equation d[C]


We have, = k2[A]
k2 EA 1 1  dt
log =  − 
k1 2.303R  T1 T2  d[C] −(k +k )t
⇒ = k2[A0] e 1 2
dt
For, A → products
[C] t
If k at 310 K is taken to be k −(k1 +k 2 )t
⇒ ∫ d[C] = k 2 [A0 ]∫ e dt
k 0 0
Then, k at 300 K is
2 k 2 [A0 ] −(k1 +k 2 )t
[C] = (1 − e )
Putting this in equation, we get k1 + k 2
EA  1 1  k [A ](1 − e 1 2 )
−(k +k )t
log 2 =  −  [C]
2.303 × 8.314  300 310  = 2 0
[A] −(k +k )t
(k1 + k 2 ).[A0 ]e 1 2
log2 × 2.303 × 8.314 × 310 × 300
⇒ EA = k2 (k1 +k 2 )t
10 = (e − 1)
k1 + k 2
EA = 53. 67 kJ/mol
Now we are also given at T = 310 K, Now, we have k1 = 1. 3 × 10–5 s–1,

t1/2 = 30 min k2 = 9k1 = 1. 17 × 10–7 s–1

0.693 Time, t = 1 hour = 3600 s


∴ k at 310 K = = 0. 0231 min–1
t1/2 Putting values,
At 310 K, for (ii) B → products [C] 9 1.3×10−4 ×3600
= (e − 1)
k = 211 for (i) [A] 10

= 0. 0462 min–1 [C]


= 0. 537
[A]
Further, for (ii)
(EA ) f k
EA = = 26. 802 k J/mol Sol 13: As Cis-(en)2(OH)2+ 

k b
2
trans-Cr(en)2(OH)2+
Therefore, using Arrhenius equation, we get
We have,
k 268.02  1 1  kf
log =  − 
0.0769 2.303 × 8.314  310 300  Eq. constant = = 0. 16
kb
k = 0. 0327 min–1
and kf = k1 = 3. 3 × 10–1 s–1
kt
Sol 12: We are given, ⇒ kb = = 2. 0625 × 10–4 s–1
0.16
k1 B
Now using formula for eq. reactions.
A
 xeq 
k2 C ln   = (kf + kb) t
 xeq .x 
 
Here, xeq
We have to find t for which x =
k1 1 q
= , k = 1. 3 × 10–5 s–1  
k2 9  x 
eq
Now, both are first order reactions ln   = (3. 3 × 10–4 + 2. 0625 × 10–3)t
 xeq 
d[A]  xeq − 
∴ = –(k1 + k2)[A]  2 
dt
ln 2
⇒ [A] = [A0] e
−(k1 +k 2 )t ⇒t= s
2.3925 × 10−5
Now, for A → C,
t = 289. 71 s ⇒ t = 4. 83 min
Chem i str y | 18.63

1 k

Sol 14: We have, A  B
k 2

k1 [B]eq

Energy →
Also, we know, keq = = Ef Eb
k2 [A]eq
k1
⇒ = 4, k1 = 10–2 s–1
k2 A B ∆H

10−2 Reaction coordinate →


⇒ k2 = = 2. 5 × 10–3 s–1
4 We see that
Now at eq. , [A] = [A0] – xeq
Eb = Ef + |∆H|
[B] = xeq
20Eb
⇒ But Ef =
xeq 31
Then, =4
− xeq
[A]e 20Eb 11Eb
∴Eb = + 33 ⇒ = 33
We are given [A] 0 = 0. 01 mole L–1 31 31
⇒ Eb = 63 kJ/mol
xeq
=1 Ef = Eb – 33
0.01 xeq
5xeq = 0 + 60. 07 Ef = 30 kJ/mol
xeq = 0. 008 mole L–1
Now, using equilibrium first order formula, Sol 16: We have rate k′[complex] a, k′ = k[H+] b

 xeq  Let’s assume that this is Pseudo first order reaction, that is
ln   = (k1 + k2)t a=1
 xeq − x 
  0.693
Then, we have t1/2 = ,
At t = 30 s, k′
1.386
 0.008  t3/4 =
ln   = (12. 5 × 10 ) × 30 = – 0. 375
–3 k′
 0.008 − x 
We note that, here t3/4 = 2. t1/2
x = 0. 0025 mole L–1
which is true for given data also

Sol 15: We have, Now, when, [H+] = 0. 01

 B(g)
A(g) 

 0.693 0.693 0.693
t1/2 = 1 = = ⇒ (0. 01)b =
k′ + b
k[H ] k
For forward reaction, we have
∆H = ∆E = –33 kJ/mol When [H+] = 0. 02
0.693 1.386
We know, (0. 02)b = =
k.t1/2 k
−∆Hº
ln k = (since ∆S ≈ 0) These both will be satisfied for b = 1.
RT
with T = 300 K, ∆H = 33 kJ/mol ∴ Taking a = b = 1, all the given data can be verified
and there are no contradictions
we get, k = 5. 57 × 105
∴a=b=1
Now, we are given
Ef 20 k
= Sol 17: A 
1
→B
Eb 31
2k
Now, we have for this transformation A  →C
−d[A]
We have, k for disappearance of A, = (k1 + k2)t
dt
∴ k = k1 + k2
1 8 . 6 4 | Chemical Kinetics

Using Arrhenius equation, Sol 19 We have,


−E1 −E2
−E
Rate, R = k3[COCl] [Cl2]
Ae RT = A1e RT + A2e RT
(∵ (iii) is slowest step (RDS))
A = Pre exponential constant Also, from equilibrium of (ii),
1 k2 [COCl]
Differentiating with respect to , we get =
T k −2 [CO][Cl]
−E −E −E
−AE RT −A1E1 RT1 −A2E2 RT2
e = e + e And from equilibrium of (i),
R R R
k1 [Cl]2
⇒ Ek = E1k1 + E2k2 =
k −1 [Cl2 ]
E1k1 + E2k 2 k 
1/2
⇒E= [Cl] =  1  ([Cl2 ])1/2
k
 k −1 
E1k1 + E2k 2
⇒E= Putting these values back in rate formula,
k1 + k 2
we get
1
+
Sol 18: (a) We have, k2 k  2
Rate = k 3 [CO]  1  ([Cl2 ])1/2 [Cl]
k
k −2  k −1 
1
A + B 
 C
k 2
k1 d[COCl2 ] k 2 k1
C  
→ D = k3 [CO][Cl2 ]3/2 = k[CO] [Cl2] 3/2
dt k −2 k −1
Since second step is RDS,
1/2
Rate = k3[C] k 2  k1 
k = k3 .  
d[C] k −2  k 2 
Also, = –k3[C] + k2[A] [B] – k2[C]
dt
d[C] k [A][B] k1
Now, at steady state, = 0 ⇒ [C] = 1 2B(g)
dt k3 + k 2
Sol 20: A(g)
k1k 3 [A][B] k2
∴ Rate = C(g)
k3 + k 2
We have,
(b) If k2 >> k3, then k2 + k3 ~
_ k2
−d[A]
Rate = = –(k1 + k2)[A]
k1k 3 dt
Then, Rate = [A] [B]
k2 [A] = [A]0 e–kt ; k = k1 + k2
Using Arrhenius equation, 1 d[B]
Further, Rate = = k1[A]
−EA 2 dt
k = A e RT
Putting value of [A] and integrating
Ea Ea
1 3 −(Ea +Ea −Ea )
A1 e RT A3 e RT 1 3 2 2k1 [A1 ]
A1 A3
k= = e RT [B] = (1 − e−kt )
Ea
2 A2 k1 + k 2
A2 e RT k 2 [A]0
−Ea Similarly, [C] = (1 − e−kt )
Comparing with k = Ae RT k1 + k 2

A1 A3 Now, we are given [A]0 = 1 atm


A=
A2 At t → ∞, Pt = 1. 5 atm
Ea = Ea + Ea − Ea Putting values and taking t = ∞, we get
1 3 2
Chem i str y | 18.65

2k1 k2 Sol 23: 2A + B + C → D + 2E


+ = 1. 5
k1 + k 2 k1 + k 2 (i) Since, this is first order w. r. t. A, second order w. r. t.
B, and 0 order w. r. t. C.
⇒ 2k1 + k2 = 1. 5 k1 + 1. 5k2
Rate law = k[A] [B]2[C]0
k
⇒ k1 = k2 = Rate,R = k[A] [B]2
2
Now, taken at t = 10s, Pt = 1. 4 atm (ii) Now rate, R′ = k[2A] [2B]2 = 8k[A] [B]2 = 8k
k[A]0 ∴ Rate becomes 8 times
∴ [A]0e–k. 10 = (1 − e−k.10 )
k
k[A]0 Sol 24: We know, for first order reaction
(1 − e−k.10 ) = 1. 7
2k 2.303 [A]
k= log 0
[A]0 = 1, 30 [A]t
1 e−10k At T = 27ºC = 300 K, at t = 30 min, 50% of reaction gets
∴ e–10R + 1 – e–10R + – = 1. 4
2 2 completed, i. e. [A] t = 0. 5[A]0
⇒ e–10R = 0. 2 2.303
k= log2
⇒ k = 0. 16 30
k = 0. 0231 min–1 = 3.85 × 10–4 s–1
k
⇒ k 1 = k2 = = 0. 08
2 At T = 47ºC=320 K, time taken for 50% completion is
10 min.
Sol 21: For 50% completion, time taken 2.303
∴k= log2
0.693 10
t1/2 =
k k = 0. 0693 min–1 = 11.55 × 10–4 s–1
0.693 Now, by Arrhenius equation
⇒k= = 0. 01 min–1
69.3
k2 EA  1 1 
Now, for 80%, [A] = 0. 2[A0] ln =  − 
k1 2.303  T1 T2 
Using [A] = [A]0e–RT ⇒ 0. 2 = e–RT
ln5 At T = 300 K, k = 0. 0231 min–1
⇒t=
0.01 T = 320 K, k = 0. 0693 min–1
t = 161 min Putting values

0.0693 EA  1 1 
Sol 22: We have, from Arrhenius equation, log =  − 
0.0231 2.303 × 8.314  300 320 
k2 Ea 1 1 
log =  −  EA = 43. 78 kJ/mol
k1 2.303R  T1 T2 
k2
Now, = 4, T1 = 27ºC = 300 K Sol 25: Firstly, we assume that reaction is first order and
k1
see if the data is consistent.
T2 = 47ºC = 320 K
For first order reaction, we have
Putting values, we find 2.303 θ − θ∞
k= log 0
EA  1 1  t θt − θ∞
log 4 =  − 
2.303 × 8.314  300 320  At time, t = 10 min,

2log2 × 320 × 300 × 2.303 × 8.314 2.303 32.4 − ( −11.1)


⇒ EA = k= log
20 t 28.8 − ( −11.1)
EA = 55. 33 kJ mol–1 k = 8. 14 × 10–3 min–1
At time, t = 20 min
1 8 . 6 6 | Chemical Kinetics

2.303 (32.4 + 11.1) Sol 28: Rate = k[A]


k= log
t 25.5 − ( −11) (i) Initial rate = k[A] 0
k = 8. 64 × 10–3 min–1 = 1. 8 × 10–2 × 0. 1
At time, t = 30 min = 1 × 10–3 mol sec–1 L–1
2.303 32.4 − (11.1) (ii) After 60 sec (1 min),
k= log = 8. 70 × 10–3 min–1
t 22.4 − ( −11) Rate = k[A] 1
At t = 40 min, = k[A]0 e–k×60
2.303 32.4 − (11.1) = 5. 495 × 10–3 mol sec–1 L–1
k= log = 8. 71 × 10–3 min–1
40 19.6 − ( −11.1)
We see that all values of k are pretty close to each other, Sol 29: Since, unit of rate constant is min–1, it is first
order reaction.
∴ The reaction is first order.
Initial rate = k [A]0 = 10–3 × 0. 2 = 2×10–4 mol min–1 dm–3
1 After 200 minutes,
Sol 26: We know, for nth order reaction t1/2 ∝
[A]n0−1 −3 ×200
[A] = [A] 0 e−10 = 0. 8187[A] 0
Here as [A]0 is doubled, t1/2 gets halved,
∴ 18. 12 % of reactant converted into product.
1
∴ t1/2 ∝
[A]0
Sol 30: (i) At time, t = 10 min, [A] = 0. 8 [A]0
Or n – 1 = 1
∴ 0. 8[A]0 = [A]0 e–10k
⇒n=2
⇒ k = 0. 022 min–1
Hence, second order reaction.
(ii) When 75% reaction is completed,
Sol 27: We have, by Arrhenius equation 1
[A] = [A]0,
4
k2 Ea 1 1
log =  −  1
k1 2.303k  T1 T2  ∴ [A]0 = [A]0 e–0. 022t
4
Now, given data, ⇒ t = 63. 01 min.
at t = 50ºC = 323 K,
k = 1. 5 × 10+7 sec–1.
Exercise 2
at T = 100ºC = 373 K,
k = 4. 5 × 107 sec–1 Sol 1: (B) From the table, we see that when (A) is
doubled, rate get becomes 4 times, keeping (B) constant
Filling the data,
∴ Rate ∝ (A) 2
Ea
 1 1 
log 3 =  −  Also, when (B) is doubled, keeping (A) doubled.
2.303 × 8.314  323 373 
−Ea Rate becomes 2 times
Ea = A e RT ∴ Rate ∝ (B)
∴ Rate = k(A)2 (B)
Ea = 2. 2 × 107 J mol–1
−Ea
Now,k = A e RT Sol 2: (A) Rate, ri = k(A) n[B] m
m
+Ea B  k 2n
⇒ A = ke RT rf = k[2A] n     = [A] n [B] m
2.2×107
2
  2 m

7e 8.214 ×323 rf
= 1. 5 × 10 = 2(n – m)
ri
A = 5. 42 × 1010 s–1
Chem i str y | 18.67

Sol 3: (A) We have t1/2 = 15 min (0. 8 M → 0. 4M) Rate = k[A]


Now, for 0. 1 M to 0. 025 M, = 0. 01 × 0. 1 M min–1
t = 2 × t1/2 = 30 min = 10–3 M min–1
t t
(0. 1 
1/2
→ 0. 5 
1/2
→ 0. 025)
Sol 9: (C) For zero order reaction, rate is always
constant.
Sol 4: (D) Only statement (D) is correct as k is constant
at a given temperature. i. e. rate = 10–2 M min–1

(A) and (B) are correct for first order reaction. (C) is A(g) → 2B(g)
wrong as t=0 0. 1
d[C] 1 d[A]  d[A]  t=1 0. 1–x   2x
= −  not 2 
dt 2 dt  dt 
1 d[B]
= 10–2
2 dt
Sol 5: (D) For a first order reaction,
[B] = 0. 02 t
[A] = [A]0e–kt
At t = 60 sec = 1 minute
3
When, [A] = [A]0 , t = t3/4
4 [B] = 0.02 M
3 −k.t
⇒ = e 3/ 4
4 Sol 10: (D) Let time be t
0.29 Then, for X →A + B
1 k
⇒ t3/4 =
k
50% reaction gets completed at time t
Sol 6: (D) rate ∝ [O] 2 1 −k t
⇒ [X]0 = [X]0 e 1
2
∴ When [(O)] is doubled, rate becomes 4 times.
ln2
k1 =
t
Sol 7: (C) A + 3B → P
k
For Y 
2→C + D
∆H = -2x kJ/mole of A  …(i)
96 % of reaction gets completed at time t.
M → 2Q + R
∴ 0. 04[Y] 0 = [Y]
−k 2 t
e
∆H = -x kJ/mole of M  …(ii) 0
ln25
We see that, if these reactions are carried simultaneously k2 =
t
energy released from (i) and energy absorbed is (ii)
counter each other further, ∆H = 2OH, rate of second k2 ln25
= = 4. 65
reaction will be double of that of (i) k1 ln2
−1 d[B] y
Now, rate of first reaction= = Sol 11: (D) We know, for nth order reaction
3 dt 3
1
2y 1 d[Q] d[Q] 4 ∝
Rate of second reaction= = ⇒ = y P0n−1
3 2 dt dt 3 n−1
(t1/2 )2  P0 
⇒ = 1 
Sol 8: (B) We have t = 138. 6 min for (t1/2 )1  P0 
 2 
1M → 0. 25 M Putting values,
2 × 0.693
i.e., 2t1/2 = 138. 6 = 950  500 
n−1
k =  
t t 235  250 
(1 
1/2
→ 0. 5 
1/2
→ 0. 25)
log 4.04
⇒ k = 0. 01 ⇒n–1= = 2; ⇒ n = 3
log2
When [A] = 0. 1,
1 8 . 6 8 | Chemical Kinetics

Sol 12: (C) We have first order reaction Sol 16: (A) We have for first order,

2.303 [A] (A) = (A) 0e–kt


k= log 0
t [A]t 0.1
(A)0 = mol L–1 = 0. 05 mol L–1
at t = 20 min, (A) t = 0.8(A)0 2
k = 3. 465 × 10–6 s–1, t = 200 s
(Since 20% reaction completed)
−6 ×200
(A) = 0. 05 e−3.465×10
2.303 5
⇒k= log ≈ 0. 05 M
20 4
k = 1. 11 × 10–2 min–1
Sol 17: (D) R is universal gas constant
0.693
t1/2 = = 62. 13 minutes k is rate constant
k
A is pre exponential factor
Sol 13: (C) Rate = k(A) = k(A) 0e–kt
Ea is activation energy
It follows an exponential decay.
Sol 18: (C) This is the correct definition.
Sol 14: (D) A(g) → 2B(g)
−E0 /RT
  100 2  Sol 19 (B) k = A e ;As T → ∞,
2    −E0 /RT
B   V   10 4 1 k = A = 6 × 1014 s–1(∵ e → 1)
We have, keq = =   = . = 108
[A]  10
−5
 10 −5 10
 V  Sol 20: (B) We have, from Arrhenius equation
 
k 1.5 × 10 −3 k2 EA 1 1
108 = f = log =  − 
kb kb k1 2.303R  T1 T2 
⇒ kb = 1. 5 × 10–11 L mol–1 s–1 Here, at T1 = 27ºC = 300 K

Sol 15: (B) A + B → C + D 0.693


k1 = min–1
20
t=0 1 1 0 0
t=1 1-x 1–x x x At T2 = 47ºC = 320 K

We have, 0.693
k2 = min–1
−d[A] 5
Rate = = k(A) 1/2(B) 1/2
dt Putting these values,
−d[A] EA  1 1 
⇒ = k(1 – x)1/2 (1 – x)1/2 log 4 =  − 
dt 2.303 × 8.314  300 320 
−d[A] ⇒EA = 55. 14 kJ/mol
⇒ = k(1 – x)
dt
This can be written as, Sol 21: (C) We have, 27ºC = 300 K, 3. 8 × 10–16 of
−d[A] reactant molecules exist in the activated region.
= k(A)
dt i. e, N = 3. 8 × 10–16 N0

⇒ (A) = (A) 0e–kt Now, N = N0 e−Ea /RT

when (A) = 0. 25(A)0 Ea = RT loge N0 = 8. 314 × 300 loge


N
2 n2 2 × 2.303log2 N0
t= =
k 2.303 × 10−3 3.8 × 10−16 N0
t = 600 s
Ea = 100 kJ/mol
Chem i str y | 18.69

Sol 22: (D) Since, at 400 K, 0. 0001% (10–4%) of collisions 0.693 2.303  100 
are effective only 10–4% of molecules have their energy = log  
t1/2 t  100 – 99 
above activation energy.
N0 0.693 2.303 × 2
Ea = 2. 303 RT log =
N 6.93 t
N0
= 2. 303 × 8. 314 × 400 log So, t = 46.06 min.
10−6 N0
= 45. 944 k J/mol = 11. 05 k cal/mol Sol 7: (B) Because two molecules are taking part in
elementary reaction.

Previous Years’ Questions Sol 8: (C) Since –


dx
∝ [CO]2 so on doubling the
dt
1 d[A] 1 d[B] d[A] 1 d[B] concentration of CO, the rate of reaction will increase
Sol 1: (A) A → 2B ; – 2 = ;– = by 4 times.
2 dt 2 dt dt 4 dt

∆T 50 Sol 9: (A) R = k(A)(B)2 order of reaction =2+1= 3


Rate at 50°C T
Sol 2: (C) = (2) 1 = (2)10 = 25 = 32 times
Rate atT1 °C
Sol 10: (D) The differential rate law for,
NO + Br2 → NOBr2 would be
2.303 a
Sol 3: (C) k = log10 ;
t a−x dc
– =k[NOBr2][NO]
dt
t = 2 × 102, a = 800, a – x = 50 [NOBr2 ]
We have, Kc =
2.303 800 2.303 [NO][Br2 ]
k= log10 = log10 16
2 × 10 2 50 2 × 102 Provided the first reaction attains equilibrium rapidly.
dc
2.303 4 2.303 ∴– = k × kc × [NO]2[Br2] = k[NO]2[Br2]
= log10 2 = × 4 × 0.301 = 1.38 × 10 s
–2 –1
dt
2 × 102 2 × 102
∴The order of the reaction with respect to NO(g) is 2.

Sol 4: (A) 2A + B → Product Sol 11: (A) For A


When conc. of B is doubled, the half-life did not Rate = k[Cl2][H2S] (By slow step)
change, hence reaction is of first order w.r.t. B. When
concentration of A is doubled, reaction rate is doubled, For B
hence reaction is of first order w.r.t. A, Rate = k[Cl2][HS–]
Hence, over all order of reaction is 1 +1 = 2
[H+ ][HS − ]
keq =
So, unit of rate constant mol lit s . –1 –1
[H2S]
(According to equilibrium)
Sol 5: (D) Rate= k(N2O5)
k eq [H2S]
hence 2.4 × 10–5 Rate = k[Cl2]
[H+ ]
= 3.0 × 10 (N2O5) or (N2O5)
–5

= 0.8 mol l–1 [H2S]


Rate = k.keq[Cl2]
[H+ ]
Sol 6 (C) In first order reaction for x% completion
a
Sol 12: (D) t1/2 = (for zero order reaction)
2.303  100  2k 0
k= log  
t  100 – x%  a 2
k0 = = =1
2 × t1/2 2 × 1
1 8 . 7 0 | Chemical Kinetics

A0 – [A]t 0.50 – 0.25 Sol 19: (D) Rate equation is to be derived w.r.t slow
k0 = ⇒ k0 = =1
t t Step ∴ from mechanism (A)
t = 0.25 hr. Rate = k[Cl2] [H2S]

Sol 13: (D) So, ∆HReaction = Ef – Eb = 80 – 100 = –20. Sol 20: (B)
Temperature coefficient µ =2;
Sol 14: (A) For endothermic reaction ∆H = +ve ∆T
k
Then from equation ∆H = EaF.R – EaB.R ; EB.R < EF.R . µ 10 =2 ;
k1
50
k2
Sol 15: (B) In photochemical reaction, the rate of µ 10 = 25 = 32 =
k1
formation of product is directly proportional to the
intensity of absorbed light. Therefore 32k1 = k2

Sol 16: (B) 2.303 0.1


Sol 21: (B) k = log ;
1 40 0.025
A → 2B 0.693
2 k=
20
d[A] d[B]
− =
+ 0.693
dt 2dt For a F.O.R., rate = × 10−2 = 3.47 × 10−4 M / min.
20
d[A] 1 d[B]
− =
dt 4 dt
K −Ea  1 1 
Sol 22: (A)
= log 2  − 
Sol 17: (C) K1 2.030R  T2 T1 
K2
0.6932 0.6932 = 2;=
T2 310K T1 300K
=
=
 λ = min−1 K1
t1/2 6.93
−E  1 1 
[A ] ⇒ log2
=  − 
2.303 2.303 × 8.134  310 300 
Also t= log 
λ [A]
=⇒ Ea 53598.6
= J / mol 53.6 KJ / mol
[A ] = initial concentration (amount)
[A] = final concentration (amount)
Sol 23: (D)
2.303 × 6.93 100
∴ t= log = 46.06 minutes
0.6932 1 −1 d[A] d[B] d[C] d[D]
Rate of Reaction = =
− = =
2 dt dt dt dt
Sol 18: (C) Let rate of Reaction = k(A)x(B)y
x d[C]
For a zero order reaction k = …. (i) Or, = k[A]x [B]y
t dt
Where x = amount decomposed Now from table,
k = zero order rate constant 1.2 × 10–3 = k [0.1]x[0.1]y  .....(i)
for a zero order reaction 1.2 × 10–3 = k [0.1]x[0.2]y  .....(ii)
[A]0 2.4 × 10–3 = k [0.2]x[0.1]y  .....(iii)
k=  …. (ii)
2t1/2
Dividing equation (i) by (ii)
Since [A0]=2M, t1/2 = 1hr; k=1 1.2 × 10−3 k[0.1]x [0.1]y
⇒ =
∴ from equation (i) 1.2 × 10−3 k[0.1]x [0.2]y
0.25 y
=t = 0.25 hr 1 
1 ⇒ 1=
 
2
Chem i str y | 18.71

⇒ y=0 +1.08
= × 10 −2 = 3. 6 × 10–3 mol L–1
3
Now dividing equation (i) by (iii)
d[H2O] 6 d[NO]
−3 x y (iii) =
1.2 × 10 k[0.1] [0.1] dt 4 dt
⇒ =
−3
1.2 × 10 k[0.2]x [0.1]y 3 1.08 × 10 −2
= × = 5. 4 × 10–3 mol L–1
1 y 2 3
1  1 
⇒   = 
2 2 Sol 2: (a) 2H2O2 → 2H2O + O2
⇒ xx==
1 −1 d[H2O2 ] −1 d[H2O] d[O2 ]
We know, = =
2 dt 2 dt dt
d[C]
Hence = k[A]1 [B]0
dt d[H2O] d[O2 ]
=2 = 2 × 3. 6 M min–1
dt dt
Sol 24: (A) In 50 minutes, concentration of H2O2 d[H2O]
= 7. 2 M min–1
1 dt
becomes of initial.
4
−d[H2O2 ] 2 d[H2O] −d[H2O2 ]
⇒ 2 × t1/2 = 50 minutes (b) = = = 7. 2 M min–1
dt 2 dt dt
⇒ t1/2 =
25 minutes
Sol 3: N2 + 3H2 → 2NH3
0.693
⇒ K= per minutes
25 d[N2 ] −1 d[H2 ] 1 d[NH3 ]
(a) = =
0.693 dt 3 dt 2 dt
rH = × 0.05 = 1.386 × 10−3
2O2 25 −d[N2 ] 1 d[NH3 ]
=
dt 2 dt
2H2O2 → 2H2O + O2
1
= × 2 × 10–4 mol L–1s–1 = 10–4 mol L–1 s–1
1 2
rO = ×r
2 2 H2O2 −d[H2 ] 3 d[NH3 ]
(b) = ×
dt 2 dt
rO
= 0.693 × 10−3
2 3
= ×2 × 10–4 mol L–1 s–1 = 3 × 10–4 mol L–1 s–1
6.93 × 10−4 mol / minute × litre
rO = 2
2

Sol 4: (i) Reaction of first order in A, second order in B


JEE Advanced/Boards and 0 order w. r. t. C.
dx
= k [A] 1[B] 2[C] 0, k = [D]t
Exercise 1 dt
dx
⇒ = k[A] [B] 2
Sol 1: 4NH3(g) + 5O2(g) → 4NO(g) + 6H2O(g) dt
−1 d[NH3 ] −1 d[O2 ] (ii) If [A], [B], [C] are all doubled.
Rate of reaction = = =
4 dt 5 dt rf = k[2A] [2B] 2 = 8k[A] [B] 2 = 8r
1 d[NO] 1 d[H2O] Rate becomes 8 times
=
4 dt 6 dt
1 d[NO] 1 1.08 × 10−2 Sol 5: We have, by ideal gas equation
(i) Rate of reaction = =
4 dt 4 3 n
PV = nRT ⇒ P = RT
= 9 × 10–4 mol L–1 V
P = CRT
−d[NH3 ]
(ii) = +4 d[NO] P
dt 4 dt C=
RT
1 8 . 7 2 | Chemical Kinetics

∆C 1 ∆P k2
Rate of reaction = = ⇒ k1 = = 2k3 ⇒ 2k1 = k2 = 4k3
∆t RT ∆T 2
We have, R = 0. 0921 L atm/mol. k
1 4
T = 27ºC = 300 K Sol 9: 4 1 H → 2 He ; ∆H = 2. 6 MeV

ΔP = (2 – 1. 1) atm = 0. 9 atm It means 4 atoms of H, eject 26 MeV energy.

ΔT = 75 × 605 = 4500 s Energy released by 1 mole of


1 0.9 NA
Rate of reaction = × Ms–1 H= = 26 × 106 × 1. 6 × 10–14 J = 6. 26 × 1011 J
0.0921 × 300 75 × 60 4
= 7. 23 × 10–6 Ms–1 Power output of sun = 3. 9×1026 W = 3. 9 × 1026 J/s
Rate of reaction in terms of pressure i.e. it radiates 3. 9 × 1026 J in one second.
∆P 0.9 Number of moles of H required per second
= = atm min–1 = 0.012 atm min–1
∆t 75
3.9 × 1026
= = 6. 23 × 1014
6.26 × 1011
Sol 6: 2A + B2 → 2AB
1.7 × 1030
Since, this is an elementary reaction, Total moles of H on sun = = 1. 7 × 1033
10 −3
rate, ri = k[A] 2[B2] 1.7 × 1033
Time taken = = 2. 72 × 1018 s
If volume of vessel reduced to one-third, [A] and [B2] 6.23 × 1014
becomes 3 times each.
∴ rf = k[3A] 2[3B2] = 27k (A) 2[B2] = 27ri Sol 10: A(g) + 2B(g) → C(g) + D(g)

Hence, rate becomes 27 times. t = 0 0. 6 0. 8 0 0


t 0. 6–x 0. 8–2x x x
Sol 7: 3BrO → BrO3 + 2Br
– -
Since this is elementary reaction
We have, Rate, r = k(B) 2(A)
−1 ∆[BrO] ∆[BrO3− ] Now,ri = k(0. 6)(0. 8)2 = 0. 381 k
Rate = =
3 ∆t ∆t
WhenPi = x – 0. 2 atm
1 ∆[Br] −∆[BrO − ]
= = k[BrO–] +2= = 3k[BrO–] 2 PA = 0. 6 – x = 0. 4 atm
2 ∆t ∆t
We have, 3k = 0.056 PB = 0. 8 – 2x = 0. 8 – 0. 4 = 0. 4 atm

k = 0.0186 mol–1 s–1 rf = k(0. 4)(0. 4)2 = 0.064 k


rf 0.064 1
∆[BrO3− ] = =
(a) = k[BrO–] 2 = 0. 0186[BrO–] 2
∆t ri 0.384 6
Hence rate constant = 0. 0186 L mol–1s–1
Sol 11: A → B
∆[Br − ]
(b) = 2k[BrO–] 2 = 0. 037[BrO–] 2 k = 1. 2 × 10–2 Ms–1
∆t
Rate constant = 0. 037 Lmol–1s–1 Since unit of rate constant is Ms–1, it is zero order
reaction
1
Sol 8: We have, N2O5(g) → 2NO2(g) + O2(g) Rate law [B] = kt
2
−d[N2O5 ] 1 d[NO ] At t = 10 min
⇒ = 2
dt 2 dt [B] = 1. 2 × 10–2 × 10 × 60 M
1 d[O2 ] [B] = 7. 2 M
= = +k[N2O5]
1 dt
At t = 20 min
2
k
Hence, k1 = k, k2 = 2k, k3 = [B] = 1. 2 × 10–2 × 20 × 60 M = 14. 4 M
2
Chem i str y | 18.73

But, [A] 0 = 10 M and [B] cannot be greater than [A0] [A]0. When A is 10% unreacted,
∴ [B] = 14.4 M implies that reaction has reached 1
[A] = [A ]
completion, i. e. 10 0
1
[B] max = 10 M ∴ [A]0 = [A]0 – kt
10
[B] = 10 M
9 3
⇒ [A] 0 = [A]0t
Sol 12: For zero order reaction, 10 4

[A] = [A]0 – kt ⇒ t = 1. 2 hr
At t = 0, [A] = [A]0 = 0. 1 M
At t = 1 min, [A] = 0. 09 M Sol 16: (i) We know, for first order reaction
0. 09 = 0.1 – k [A] = [A]0e–kt
⇒ k = 0. 01 M min–1 k=
1 [A]0
ln
t [A]
At t = 2 min, [A] = 0. 1 – 0. 01 × 2
1
At t = 72 min, 75% reaction completed, i. e. [A] = [A]0
= 0. 08 M, which is the value given. 4
1
∴ k = 0.01 M min–1 ∴k = ln 4 = 0. 019 min–1
72
When reaction is 50% complete,
Sol 13: For zero order reaction
ln2
[A] = [A]0 – kt t = t1/2 = = 36 min
k
We are given,
(ii) When reaction is 87. 5% complete, [A] = 0. 125[A]0,
k = 2 × 10–2 mol L–1sec–1
1 1
∴t= ln
and at t = 25 sec, (A) = 0. 25 M k 0.125
[A]0 = [A] + kt t = 108 min
= 0. 25 + 2 × 10–2 × 25 = 0. 75 M
Sol 17: We have, for first order reaction
Sol 14: It is a zero order reaction (∵ unit of rate constant 1 [A]0
k= ln
is M sec–1) t [A]t
6 × 10−6 (i) At t = 10 min, reaction is 20% complete, i. e. [A] = 0.
[H+]0 = = 6 × 10–2 M
0.1 × 10−3 8[A]0
k = 107 mol L–1 sec–1 1 [A]0
k= ln
[H+ ]
10 0.8[A]0
Time taken to disappear =
k k = 0. 0223 min–1
6 × 10 −2
= sec = 6 × 10 sec
–9

107 (ii) When reaction is 75% complete,


1
Sol 15: Since it is a zero order reaction [A] = [A]0
4
[A] = [A]0 – kt
1 [A]0 1 [A]0
1 t= ln = ln
When t = 1 hour, [A] = [A]0 k [A]t 0.0223 1
4 .[A]0
4
(∵ 75% reacted) t = 62. 17 min.
1
∴ [A]0 = [A]0 – k Sol 18: Order will be equal to 1.
4
3 1 [A]0
⇒ k = [A]0 M hr–1 t= ln
4 k [A]t
1 8 . 7 4 | Chemical Kinetics

When reaction is 99. 9 % complete, k = 0. 02 min–1


[A]t = 10–3 [A] 0 k = 3. 3 × 10–4 sec–1.
1 [A]0 3 6.909
t1 = ln = ln10 =
k 10−3 [A] k k Sol 22: If we assume the reaction to be of first order,
0
[A]0 then, let [A]t = α and [A]0 = C0.
When reaction is 50% completed, [A]t =
2 a = C0e–kt
1 [A]0 ln2 0.693
∴t2 = ln = = At time
k 0.5[A]0 k k
t = 0, α = C0
We see that t2 ≈ 10t1
t = t, α = C0e–k = aC0, a = e–k
t = 2t, α = C0e–2k = a2C0, a = e–k
Sol 19: We have, first order reaction
t = 3t, α = C0e–3k = a3C0, a = e–k
N = N0e–kt
Therefore, taking the reaction to be of first order, we
here, k = 1. 5 × 10–3 sec–1
found the concentration of A to be of the form given in
N ∝ 1. 25 (wt. ∝ molar) question. Therefore, the reaction is of first order.
N∝5
−1.5×10−3 t Sol 23: For first order reaction,
∴1. 25 = 5e
1 5 [SO2Cl2] = [SO2Cl2]0 e–kt
⇒t= ln
1.5 × 10−3 1.25 k = 2. 2 × 10–5 sec–1,
t = 924. 3625 t = 90 min = 5400 sec
[SO2Cl2 ] −5400×2.2×10−5
∴ = e = 0. 888
Sol 20: We have, [SO2Cl2 ]0
[N]0 = 500 11. 2 % of [SO2Cl2] decomposed.
At t = 20 months, [N]t = 420,
ln2
Using first order to rate law, Sol 24: For A, k1 = ln2 =
t1/2 5
1 [N]0 Initial concentration = a0
k= ln
t [N]t Let [A]t = a

k = 8. 717 × 10–3 month–1 Then, a = e0 e−k1t


ln2
The drug will be ineffective (will expire) when it has For B, k2 = ln2 =
t1/2 1.5
decomposed 30%, i. e. [N] = 0. 7 [N] 0.
Initial concentration
1 [N]0 1 [N]0
∴ t = log = log [A]0 a0
t [N]t 8.717 × 10−3 0.7[N]0 [B]0 = =
4 4
t ≈ 41 months [B]4 = b
a0 −k 2 t
Sol 21: For first order reaction Then, b = e
4
1 [A]0
k = log When a = b,
t [A]t
a0 −k 2 t
We are given, 2% of virus gets inactivated every minute a0 e−k1t = e
4
at the beginning, i. e. , at t = 1 min, [A]t = 0. 98[A]0
4= e 1 2
(k −k )t

1 [A]0
k = log ln 4
1 0.98[A]0 ⇒t=
k1 − k 2
Chem i str y | 18.75

ln 4 2ln2 2 × 5 × 15 ln2 0.693


= = = ∴k= = = 0. 013 min–1
ln2 ln2  1 1  10 t 53
− ln2  − 
5 15  5 15  (i) We see that both values of k are same, this is first
order reaction.
t = 15 min.
[N2O] −2 ×100
(ii) = e–kt = e−1.3×10 = 0. 27
Sol 25: From the data, we observe that, when [H2] [N2O]0
is halved, keeping [NO] constant, reaction rate gets
Thus 73% of N2O will decompose.
halved. So, order w. r. t H2 is 1.
When [NO] is doubled, keeping [H2] constant, reaction Sol 28: As in previous question, we assume order = 0
rate becomes 4 times so, order w. r. t. NO is 2. and calculate k at different time.
Rate = k[NO]2 [H2] At t = 0 sec, P = P0 = 4 × 103 Pa
(a) Order = 2 + 1 = 3 At t = 100 sec, P = 3. 5 × 103 Pa
(b) When, [NO] = 1. 5 × 10 M, –4
P0 − P (4 − 3.5) × 103
We have, k1 = = Pa/s
[H2] = 4 × 10–3 M, t 100
k1 = 5 Pa/s
Rate = 4. 4 × 10–4 M sec–1
At t = 200 sec, P = 2 × 103 Pa
4.4 × 10−4
∴k = (4 − 3) × 103
(1.5 × 10−4 )2 × (4 × 10−3 )2 k = k2 = = 5 Pa/s
200
When [NO] = 1. 1 × 10–3 M, At t = 300 sec, P = 2. 5 × 103 Pa
[H2] = 1. 5 × 10 M –3
(4.2.5) × 103
k3 = = 5 Pa
(c) Rate = 4. 88 × 106 × (1. 1 × 10–3)2 × (1. 5 × 10–3) 300
Rate = 8. 85 × 10–3 M sec–1 We see that k1 = k2 = k3 = 5 Pa/s is same for all. Hence
this is zero order reaction, rate constant = 5 Pa/s
Sol 26: From given data, we observe that, when [Cl2] is
tripled, rate becomes 3 times, so, order w. r. t. Cl2 is 1. Sol 29: We know, for nth order reaction
1
When [NO] is tripled, rate becomes 9 times, so, order t1/2 ∝
w. r. t [NO] is 2. [A]n0−1

(a) Order w. r. t. NO = 2 We have, at [A]0, (t1/2)1 = t = 50 min


Cl = 1 [A]0
at , ( t1/2 )2 = 25 min
(b) Rate = k[NO] [Cl2]
2
2
n−1
(c) When [Cl2] = 0. 05 M, [NO] = 0. 05 M,   [A]0  
   
Rate = 10–3 50  2   1
∴ =   =
25 [A]0 2 −1
n
rate 10−3  
k= = = 8 M–2 sec–1  
2 2
[NO] [Cl2 ] (0.05) (0.05)
1
(d) If [Cl2] = 0. 2 M, [NO] = 0. 4 M, 2= ⇒ 2n–1 = 2–1
n−1
2
Rate = k[NO] 2 [Cl2] = 8 × (0. 4)2 (0. 2) n – 1 = –1
Rate = 0. 256 mol sec L –1 –1
⇒n=0

Sol 27: Let us assume that this is first order reaction.


i. e. [A] = [A]0e–kt. Using this, we will calculate k at both
times. If they are same, then our assumption is correct.
At t = 53 min, 50% reaction completed, i. e. [A] = 0.
5[A]0
1 8 . 7 6 | Chemical Kinetics

Sol 30: (n0 − x)a + xa + xa (n0 + x)a


Vn = V2 = =
A → B+ C N N
t=0 P0 0 0 NV2
⇒ n0 + x =
t P0 − x x x a
t→∞ 0 P0 P0 NV0 N
x= − n0 =
(V − V1 )
a a 2
We have, at time t = t1
1 n 1 n
k = ln 0 = ln 0
P1 = Pt = P0 – x + x + x t nt t n0 − x
P2 = P0 + x ⇒ x = P2 – P0 1 NV1
= ln
At time, t → ∞, t  NV N 
a  1 −  (V2 − V1 )
Pt = P3 = 2P0  a a
P3 1 V1
⇒ P0 = k= ln
2 t 2V1 − V2
Taking first order reaction,
1 (PA )0 1 P 1 P0 Sol 33: A → 2B + 3C
k= ln = ln 0 = ln
t (PA )t t P0 − x t P0 − P2 + P0 Time, t = 0 a
1 P0 1 P3 Time, t a – x 2x   3x
k= ln = ln
t 2P0 − P2 3 2(P3 − P2 ) Time, t → ∞ 0   2a   3a
Let n-factor of A, B and C be n and normality of reagent
Sol 31: Here, we have, at time t = N then at time, t
P2 N. V2 = (a – x)n + 2xn + 3xn
PB + PC = P2 = 2x ⇒ x =
2
NV2 = n(a + 4x)
at time t → ∞, PB + P0 = P3 = 2P0
NV2
P3 a + 4x =
⇒ P0 = n
2
at time, t → ∞
1 (P ) 1 P3 1 P
k = ln A 0 = ln = ln 3 NV3 = 2a. n + 3a. n
t (PA )t t  P3 P2  t P3 − P2
 −  ⇒ a = NV3 = 5an
2 2
NV3
a=
5n
Sol 32: A → B + C
NV2 NV3
Then 4x = –
time n 5n
t=0 n0 0 0 V3 
N
x=  V2 − 
t n0 – x x x 4n  5 
t→∞ 0 n0 n0
1 a 1 NV3
n0 − a k= ln = ln
At time, t = 0, Vr = V1 = t a−x t  NV NV2 NV3 
N 5n  3 − + 
 5n 4n 20n 
NV1
⇒ n0 = V3 4V3
a 1 1
= ln = ln
(n-factor of A, B, C = a) t 5 t 5(V3 − V2 )
V3 − V + 4V3
4 2
(using NV1 = n0a)
1 4 V3
Normality of reagent = N k= ln
t 5 ( V3 − V2 )
At time, t = t,
Chem i str y | 18.77

Sol 34: S→G+F x


⇒ = 856 – 758
Time t = 0 a 0 0 2
Time, t a–x x x x = 196 mm

Time t → ∞ 0 a a 1 758
k= ln
7.5 758 − 196
Let specific rotation of Glucose and Fructose be q1 and
q2 respectively k = 0. 039 hr–1
Then, at time t x
t = 10 hr, Pt = 889 = P0 +
2
xq1 + xq2 = rt
x = 2(883 – 758)
rt
x= x = 248 mm
θ1 + θ2
1 758
At time, t → ∞ k= ln
10 758 − 248
aq1 + aq2 = r∞
k = 0. 039 hr–1
r∞
a=
θ1 + θ2 Thus, all 3 values of k are approximately the same,
hence first order reaction.
2.303 a 1 a
k= log = log
t a−x t a−x Sol 36 : (CH3)2O(g)→CH4(g)+H2(g)+CO(g)
1 r∞ Time t = 0 P0 0 0 0
k= log
t  r rt 
(θ1 + θ2 )  ∞ −  Time, t P0 – x x x x
 θ1 + θ2 θ1 + θ2  t→∞ 0 P0 P0 P0
1 r∞
k= log
t r∞ − r1 Total pressure, at time t,
Pt = P0 – x + x + x + x = P0 + 2x
3
Sol 35: As H3(g) → As(s) + H2(g) Increase in pressure = (P0 + 2x) – P0 = 2x
2
t = 0P0 We will assume that this is a first order reaction and
3 check the consistency of given data.
t = 1P0 – x x
2
At t = 0, P0 = 312 mm Hg
Since, as a solid, it won’t contribute to pressure.
At t = 390 s, pressure increases;
3 x
∴ At time t, Pt = P0 – x + x = P0 + DP = 96 mm Hg = 2x
2 2
Let’s assume reaction is first order and calculate k at x = 48 mm Hg
1 P
different times, using k = ln 0 1 P 1 312
4 P0 − x ∴k= ln 0 = ln
t P0 − x 390 312 − 48
t = 0 P0 = 758 = Pt
k = 4. 28 × 10–4 s
x
t = 5 hrs , Pt = 827 = P0 +
2 At t = 1195 s, DP = 250 = 2x
x
827 = 758 + x = 125 mm Hg
2
1 312
⇒ x = 138 mm k= ln = 4. 28 × 10–4 s
1195 312 − 125
1 758
k= ln
5 758 − 138 At t = 3155 s, DP = 467 = 2x
x = 233. 5 mm Hg
k = 0. 04 hr–1
1 312
x k= ln = 4. 30 × 10–4 sec–1
t = 7. 5 hr, Pt = 856 = P0 + 3155 312 − 233.5
2
1 8 . 7 8 | Chemical Kinetics

At t → ∞,DP = 2P0 = 624 mm Hg Now, since, this is first order reaction.

≈ 619 mm Hg 2.303 (P ) 2.303 90


k= log A 0 = log
Thus, we see that all 3 values of k are approximately the t (PA )+ 10 47
same. Therefore, this is a first order reaction.
k = 6. 49 × 10–2 min–1
So, rate = k[(CH3)2O], k
1 13.1 − ( −3.8)
= 4. 28 × 10–4 sec–1 k1 = 2.303log .
60 11.6 − ( −3.8)
1 = 1. 55 × 10–3 min–1
Sol 37: H2O2(g) → H2O() + O (g)
2 2
At time, t = 120 min, qt = + 10. 2º
Here, Vr ∝ [H2O2]
Assuming this is a first order reaction 2.303 (13.1) − ( −3.8)
k2 = log = 1. 57 × 10–3 min–1
120 (10.2) − ( −3.8)
2.303 [H O ] 1 V
k= log 2 2 0 = log 0
t [H2O2 ]t t Vt At time, t = 180 min, qt = +9. 0º
At time, t = 0, Vr 22. 8 cc = V0
2.303 (13.1) − ( −3.8)
k3 = log = 1. 54 × 10–3 min–1
t = 10,Vr = 13. 3 cc = Vt 180 (9.0) − ( −3.8)
2.303 22.8 At time, t = 360 min, qt = +5. 87º
k1 = log = 5. 4 × 10–2 min–1
10 13.3
2.303 (13.1) − ( −3.8)
t = 20,Vr = 8. 25 cc = Vt k4 = log = 1. 55 × 10–3 min–1
360 (5.87) − ( −3.8)
2.303 22.8
k2 = log = 5. 1 × 10–2 min–1 We see that, k1, k2, k3 and k4 are all same within
20 8.25
experiment value.
k1 and k2 are equal within experimental error.
Therefore, this is a first order reaction
Therefore, first order reaction.
0.693 0.693
Half-life = = = 10. 677 min
k 6.49 × 10−2
Sol 38: For inversion of cane sugar, assuming first order,
(i) (PA)I = 90 mm Hg
1 θ − θ∞
k= log 0 (ii) (PA)f = 47 mm Hg
t θf − θ∞
(iii) k = 6. 49 × 10–2 min–1
At time, t = 0, θ = q0 = + 13. 1
(iv) t1/2 = 10. 67 min
At time, t → ∞,θ = θ∞ = –3. 8
At time, t = 60, θ+ = 11. 6º Sol 40: 2N2O5(g)→4NO2(g)+O2(g)
time, t = 0 P0 0 0
Sol 39: A(g) → 2B(g) + C(g)
time, t = t P0 – 2x 4x x
Time, t = 0 P0 0 0
t→∞ 0 2 P0 P0
Time, t P0 – x 2x x
2
Time, t→∞ 0 2P0 P0 P0
At t → ∞, Pt = 584. 5 = 2P0 +
At, t → ∞,Pt = 3P0 = 270 mm Hg 2
5P0
P0 = 90 mm Hg = (PA) ⇒ = 584. 5
2
At, t = 10 min, Pt = P0 – x + 2x + x = P0 + 2x = 176
⇒ P0 = 233. 8 mm Hg
2x = 176 – 90
At t = 30 min,
x = 43 mm Hg
Pt = 284. 5 = P0 – 2x + 4x + x
(PA)+ = P0 – x = 90 – 43 = 47 mm Hg
⇒ P0 + 3x = 284. 5
Chem i str y | 18.79

3x = 284. 5 – 233. 8 Fraction decomposed


x = 16. 9 mm Hg Vt −25 × 5.05 × 10−2
= 1. 10 = 1 – 0. 284
PN = P0 – 2x V0 2.303
2O5

= 233. 8 – 2 × 16. 9 = 200 mm Hg Fraction decomposed = 0. 716


We have,
1
Sol 42: H2O2 → H2O + O
−1 d[N2O5 ] 2 2
= k[N2O5]
2 dt We are given complete dissociation gives 46. 34 mL O2.
[N2O5 ]t t Also VO ∝ [H2O2]
d[N2O5 ] 2
∫ = −2k ∫ dt
[N2O5 ]0 ( VO is volume of O2 a given sample of H2O2 can give)
[N2O5 ]0 0 2
∴ [H2O2] 0 ∝ 46. 34
[N2O5 ]t
ln = –2kt Also, at t = 5.1 days, VO = 10. 31 mL
[N2O5 ]0 2

Then [H2O2] t ∝ (46. 34 –10. 31)


2.303 [N O ]
k= log 2 5 0
2t [N2O5 ]t 2.303 [H O ]
So, k = log 2 2 0
t [H2O2 ]t
2.303 (PN O )0
k= log 2 5 (∵ PN O ∝ [N2O5]) 2.303 46.34
2t (PN O )t 2 5
k= log
2 5
5.1 46.34 − 10.31
2.303 233.8
= log k = 4. 93 × 10–2 days–1.
2 × 30 200
When 20 mL, O2 lost,
k = 2. 605 × 10–3 min–1.
[H2O2] t ∝ (46. 34 –20. 00)

Sol 41: (a) Assuming first order reaction 2.303 [H2O2 ]0


t= log
V k [H2O2 ]0 − [H2O2 ]t
2.303
k= log 0
t Vt
2.303 46.34
= log
We have,V0 = 22. 8 cc 4.93 × 10 −2 46.34 − 20
At t = 10 min, Vt = 13. 8 cc t = 11. 45 days

2.303 22.8
k1 = log = 5. 02 × 10–2 min–1 Sol 43: A(aq) → B(aq) + C(aq)
10 13.8
t = 0   n0
At t = 20 min, Vt = 8. 25 cc
t = t   n0 – x   x    x
2.303 22.8 t →∞ 0   n0     n0
k2 = log = 2 × 10–2 min–1
20 8.25
At t → ∞,θ∞ = –5º = n0. θB + n0θC
Since, k1 ≈ k2. This is first order reaction.
We have, θA = 20º, θB = 30º,
0.693 0.693
(b) t1/2 = = = 13. 75 min θC = –40º
k 5.05 × 10−2
∴ 30n0 – 40n0 = –5
(c) At t = 25 min,
n0 = 0. 5
2.303 V At time, t = 6. 43 min.
k= log 0
25 Vt
qt = (n0 – x)θA + x(θB + θC) = 2. 5
1 8 . 8 0 | Chemical Kinetics

20n0 + x(30. 40 – 20) = 2. 5 k1 [x]0 k 2 [x]0


= (1 − e−kt ) + (1 − e−kt )
30x = 20 × 0. 5 – 2. 5 k k

30x = 7. 5  k + k2  (1 −e−kt )
 [x]0 (1 − e ) = [x] 0 e
−kt
=  1
x = 0. 25  k 
For first order conversion, [x]t e−kt 1
∴ = =
2.303 n [y] + [2] 1−e −kt
e
+ (k1 +k 2 )t
−1
k= log 0
t n0 − x
Sol 45: Using results of previous question
2.303 0.5
= log
6.93 0.5 − 0.25 k 2 [A]0
[C]t = (1 − e−kt ) , k = k1 + k2
k = 0. 1 min–1
k

[A]t = [A]0e–kt
Sol 44:
[C]t k2 (1 − e−kt ) 1
= = (e−kt − 1)
k1 B [A]t k1 + k 2 e −kt k1
1+
x k2
k2 C We have, k1 = x hr–1, k2 = 10 k1
= 10 × hr–1, t = 1 hr
(B = y, C = z)
[C]t 1
We have, ∴ = (e(x +10x) − 1)
[A]t 1
1+
d[x] 10
= –(k1 + k2)[x] [C]t
dt 10 11x
= (e – 1)
[A]t 11
[x]t t
d[x]
∫ = −(k1 + k 2 )∫ dt
[x]0
[x] 0 Sol 46: We have,
k k
⇒[x] t = [x]0e–kt, k = k1 + k2 A → B 
1 2→ C,

d[B] ln2 ln2


Now, = k1[x] t k1 = ,k =
dt ln1 2 2
d[B] For [A]
= k1[x]0e–kt
dt d[A]
[B]t t
= –k1[A]
−kt
dt
∫ d[B] = k1[x] 0 ∫e
0 0 ⇒ [A] = [A] 0 e−k1t

−k1 [x]0 t
d[B]
[B]t = e−kt = k1[A] – k2[B]
k 0 dt
d[B]
−k1 [x]0 = k1[A] 0 e−k1t – k2[B]
[B] t = (1 − e −kt
) dt
k1 + k 2
d[B]
+ k2[B] – k1[A]0 e−k1t = 0…(i)
dt
k 2 [x]0
Similarly [C] t = (1 − e−kt ) This is a linear differential equation.
k1 + k 2

[B]t + [C]t = [y] + [x] Solving this, we get


Chem i str y | 18.81

k1 [A]0 −k1t −k 2t  (Ea )2  1 1 


[B] t = [e −e ] k2 = 2 antilog   − 
k 2 − k1
 2.303R  300 T  
d[B]t k1
Now, at [B] max, =0 =2
dt k2
d[B]t
In (i), putting = 0, we get  ((Ea )1 − (Ea )2 )   1 1
dt 4antilog   −  =2
 2.303R   300 T 
k1[A]0 e−k1t = k2[B]
(Ea )1 .(Ea )2  1 1 1
 −  = log
k 2k1 [A]0 −k1t −k 2t 2.303R  300 T  2
k1[A]0 e−k1t = [e −e ]
k 2 − k1 (Ea )1 = 20 kJ/mol,

−k1t −k1t −k1t −k 2t


k 2e .k1e = k 2e .k 2e (Ea )2 = 2. 8314 kJ/mol
3
k1 ⇒ (28.314 − 20) × 10  1 − 1  =0. 7
= (k −k )t
e 1 2 8.314  300 T
k2
1 1
 logk1 − logk 2  − = 7 × 10–4
t =  300 T
 .2.303
 k1 − k 2  1
= 2. 63 × 10–3
1 ln2 −4 1 T
t= ln = ln = 4
k1 − k 2 4.ln2 ln2 2 T = 379. 75 K

∴ t = 4 min.
Sol 48: H2O + O → 2OH–, Ea
1

k1 2OH– → H2O + O, Ea
B 2
Sol 47: A
Potential energy →

k2 C
E a2
[B] k1
= Ea1
[C] k2 2OH

[B] k1
We need, =2= H2O+O
[C] k2
Reaction coordinate →
Let temperature be T.
By Arrhenius equation.
Ea = Ea – ∆H = (77. 72) kJ mol–1
2 1
Ea = 5 kJ mol–1
k2 Ea 1 1 2
log =  − 
k1 2.303R  T1 T2 
Sol 49: For t1/2 = 1 min, k = 0.693
t1/2
For A 
k1
→ B, k2 = k1, k1 = 8,
 = 0. 693 min = 1. 155 × 10 s –1 –2

−1
Ea = 20 kJ/mol (E1), T1 = 300, T2 = T Ea  A 
−Ea

Now, k = A e RT
⇒T=  ln 
R k
k1 (Ea )1  1 1
log =  −  A = 5 × 1013 sec–1
8 2.303R  300 T 
Ea = 104. 5 kJmol–1
 (Ea )1  1 1  −1
k1 = 8 antilog   −  3 
T = 104.5 × 10  ln 5 × 1013 
 2.303R  300 T  

8.134  1.155 × 10 −2 
 
For A 
k2
→ C, similarly,
 T = 349. 1 K
1 8 . 8 2 | Chemical Kinetics

Sol 50: From Arrhenius equation k 308 k


Sol 53: Temperature coefficient = = 1. 75
k2 Ea
1 1 k 298 k
log =  − 
k1 2.303R  T1 T2  By Arrhenius equation

For T1 = 27ºC = 300 K and k 308 k Ea


 1 1 
log =  − 
k2 k 298 k 2.303k  298 308 
T2 = 47ºC = 320 K, =4
k1
log1.75 × 2.303 × 8.314 × 298 × 308
Ea  1 1  Ea =
∴ log 4 =  −  10
2.303 × 8.314  300 320 
= 42. 711 kJ mol–1
log 4 × 2.303 × 8.314 × 300 × 320
Ea = Ea = 10. 757 kJ mol–1
20
Ea = 55. 33 kJ/mol
Sol 54: At T1 = 380ºC = 653 K,

0.693 –1 0.693
Sol 51: For t1/2 = 2 hr, k = 0.693 = hr k1 = 0.693 = = 1. 925 × 10–3 min–1
t1/2 2 t1/2 360
= 9. 625 × 10–5 sec–1
T2 = 450º C = 723 K, k2 = k
So, we have, k1 = 3. 46 × 10–5 sec–1
Ea = 200 k J mol–1
k2 = 9. 625 × 10 sec –5 –1
By Arrhenius equation
Ea = 100 kJ mol–1
k2 Ea 1 1
T1 = 25ºC = 298 K log =  − 
k1 2.303R  T1 T2 
T2 = T
k 200 × 103  1 1 
By Arrhenius equation. log =  − 
−3 2.303 × 8.314  653 723 
1.925 × 10
k2 Ea
1 1
log =  −  k
k1 2.303R  T1 T2  log = 1. 548
1.925 × 10 −3
9.625 × 10 −5 100 × 103  1 1 k = 6. 81 × 10–2 min–1
log =  − 
−5 2.303 × 8.314  298 T 
3.46 × 10 2 × 0.693
Time required for 75 % reaction = 2t1/2 =
1 1 6.8 × 10−2
⇒ − = 8. 5 × 10–5 Time = 20. 4 minutes
298 T
T = 306 K Sol 55: We have,

Sol 52: By Arrhenius equation k1


(i) NO + Br2 NOBr2
k = A e−Ea /RT k2
k3
k2 (Ea −Ea )/RT (ii) NOBr2 + NO  
→ 2NOBr
= e 1 2
k1
This is RDS
We have,
Here, we use equilibrium approach, as equilibrium is
Ea = 75 kJ mol–1 achieved fast
1
Ea = 25 kJ mol–1 Rate = k3[NOBr2] [NO]
2
T = 25ºC = 298 K For equilibrium of (i),
k2
∴ = 5. 8 × 108 k1 [NOBr2 ]
k1 =
k2 [NO][Br2 ]
Therefore, rate of reaction increases by 5. 8 × 108 times.
Chem i str y | 18.83

k1 equilibrium constant = k1, fast


[NOBr2] = [NO] [Br]
k2 (ii) 2I + H2 
k2
→ 2HI, RDS

k1k 3 [I]2
Hence, rate = [NO] [Br] = k[NO] [Br2]
2 2 from (i) equilibrium, k1 =
k2 [I2 ]
k1k 3 [I] 2 = k1[I2]
k=
k2 from (ii)Rate = k2[I] 2[H2]
= k2k1[I2] [H2]
Sol 56: (i) 2NO → N2O2
= k[H2] [I2] , k=k2k1
equilibrium constant = k1 (fast)
(c) (i) I2 → 2I
(ii) N2O2 + H2 
k2
→ N2O + H2O

equilibrium constant = k1, fast
RDS
k3 (ii) I + H2 → HI2
(iii) N2O + H2  → N2 + H2O
 k
3
→ 2HI, RDS
(iii) IH2 + I
As equilibrium step is fast, we use equilibrium approach
equilibrium constant = k2, fast
[N2O2 ]
from (i), k1 = [I]2
[NO] 2 from equilibrium (i), k1 =
[I2 ]
[N2O2] = k1[NO] 2 [I] 2 = k1[I2]
Also, from (ii), Rate = k2[N2O2] [H2] from equilibrium (ii),
= k1k2[NO] [H2] 2
[IH2 ]
k2 = [HI] 2
= k[NO] [H2] 2
[I][H2 ]
k = k 2k 3 [IH2] = k2[I] [H2]

1  k since (iii) is RDS,



Sol 57 (i) NO + NO  N2O2 [fast] k −1
Rate = k3[IH2] [I]
(ii) N2O2 + O2 
→ 2NO2 (RDS)
k2
= k3k2[I] 2[H2]
from equilibrium of (i),
= k1k2k3[I2] [H2]
k1 [N2O2 ]
= Rate = k[H2] [I2]
k −1 [NO]2
(d) No, it cannot distinguish between the three mechanisms
k1 as all the 3 molecules gives the same rate law.
[N2O2] = [NO] 2
k −1
(e) Mechanism (a) becomes most improbable as it just
from (ii) involves one step and it is highly unlikely that I2 and H2 will
combine directly at 200ºC without breaking into radicals.
1 d[NO2 ] k 2k1
Rate = = k2[O2] [N2O2] = [NO] 2[O2]
2 dt k −1

1 d[NO2 ]
Exercise 2
Rate = = k[NO]2[O2] ,
2 dt Single Correct Choice Type
k 2k1
k= Sol 1: (B) We know,
k −1 1
Coefficient in rate law =
stiochiometric coefficient
Sol 58: (a) H2 + I2 → 2HI Given rate law
since, this is one step, rate = k[H2] [I2] 1 d[C] −1 d[D] +1 d[A] d[B]
+ = = = −
2 dt 3 dt 3 dt dt
(b) (i) I2 → 2I
Reaction is 3D + B → 2C + 4A
1 8 . 8 4 | Chemical Kinetics

Sol 2: (D) If volume is reduced to half, pressure will get −kt


doubled. log(a0–x) = + log a0
2.303
Now, reaction is first order w. r. t O2 and second order −kt
Slope of log (a0 – x) vs. t graph is
w. r. t N2 2.303
−k
\ Rate = k[O2] [N2] 2 Slope of given graph = –3. 3 × 10–3=
2.303
rf = k[2O2] [2N2] 2 ⇒ k = 7. 7 × 10–3
rf = k[2O2] [2N2] 2 = 8ri
Sol 8: (C) Volume strength = 5. 6 N0 = 16. 8º
Sol 3: (C) We have Now, after 2. 303 hours, let normality be N.
When 20 mL of this solution is diluted, Normality
becomes
Ef N.20 N
Energy →

Eb =
∆Η 100 5
It is diluted against 0. 02 M KMnO4.
Reaction coordinate→ Under acidic conditions, normality of KMnO4 = M × 5
∴ |∆H| = Eb – Ef = (200 – 180) kJ mol–1 = 0. 02 × 5 = 0. 1 N

|∆H| = 20 kJ/mol Now, using N1V1 = N2V2


N
. 25 = 0. 1 × 37. 5
5
Sol 4: (D) For zero order reaction
N = 0. 75 N
d[B]
rate, =k Now volume strength
dt
[B] = kt V = N × 5. 6 = 0. 75 × 5. 6 = 4. 2 V
Now,
[A] = [A]0 – kt
2.303 V
1 k= log 0
At time t = t3/4, [A] = [A0] t Vt
4
t = 2. 303 hours, V0 = 16. 8 V,
3[A]0
⇒t =
4k Vf = 4. 2 V

⇒t3/4 ∝ [A0] 2.303 16.8


k= log
2.303 4.2
Similarly, t1/2 ∝ [A0]
k = 0. 60 hr–1

Sol 5: (C) Here, t1/2 ∝ (A)0, therefore, it is a zero order


Sol 9: (A) For experiment 1,
reaction.
 d[A]  1.5M
So, rate of reaction   will be independent of time.
Conc.(M)→

 dt 

Sol 6: (B) The given option is correct as there is very 0.95M


little probability for more than 3 atoms to collide
simultaneously. 10min
Time(min)→
2.303 a
Sol 7: (C) We have k = log 0
t a0 − x −∆[A] 0.55
Average rate = = = 0. 055 min–1
∆t 10
kt = 2. 303 log a0 – 2. 30 log(a0–x)
For experiment 2,
Chem i str y | 18.85

Conc.(M)→ Sol 12: (B)

EP
1.5M
P

Energy →
0.8M
S+E
ES
5
Time(min)→
Reaction coordinate→
∆A
Average rate, r2 =
∆t This is the plot for enzyme catalysed reaction.
0.2 1 Correct option is (B).
= M min–1 = M min–1
5 25 Multiple correct choice type
r 0.55 [A]0
⇒ 1 = = 1. 375 ≈ 1. 5 = 1
r2 1 / 25 [A]0 Sol 13: (A, D) We have, emission rate of SO3
2
Hence the reaction is first order
= 6. 93 × 10–6 gm/L/day
When (A) = 1. 8 M
6.93 × 10−6
Rate, r = 1. 8 × [x]2 = mol/L/day
80
1.8
= M min–1 = 0. 072 M min–1 = 8. 6625 × 10–8 mol/L/day
25
≈ 0. 08 M min–1 For decomposition of SO3,

0.693 0.693
k= = = 6. 93 × 10–3 day–1
Sol 10: (A) Since k1 << k2 second reaction will have the t1/2 100
most activation energy
d[SO3 ]
Further, since the overall reaction is exothermic, = 8. 6625 × 10–8 – 693×10–3[SO3]
dt
therefore the final energy of products should be less
d[SO3 ]
than the reactants plot which satisfies above 2 is (A). At steady state, =0
dt
k1 8.6625 × 10−8
Sol 11: (A) NO(g) + Br2(g) NOBr2(g) ⇒ [SO3] =
k2 6.93 × 10−3
NOBr2(g) + NO(g) → 2NOBr(g)
3 k [SO3] = 1. 25 × 10–5 M

We use equilibrium approach In 10–3 of air, moles of SO3 = 1. 25 × 10–5×103

Since second step is RDS, = 1. 25×10–2 mol


When dissolved in water (1 L),
Rate, R = –k3[NOBr2] [NO]
[SO3] = 1. 25 × 10–2 M
From equilibrium of (i),
Normality of SO3 = 2M = 2. 5 × 10–2 N
k1 [NOBr2 ]
= N1V1 = N2V2
k2 [NO][Br2 ]
2. 5 × 10–2 × 1000 = 1 × V2
k1
⇒ [NOBr2] = [NO] [Br2] V2 = 25 mL
k2
k 3k1 SO3 + H2O → H2SO2
Therefore, rate = [NO]2 [Br2 ]
k2 Rate of H2SO4 production
Order w. r. t. NO = 2 980 × 103
= 980 kg/day = mol/day = 104 mol/day
98
SO3 required = H2SO4 manufactured = 104 mol/day
1 8 . 8 6 | Chemical Kinetics

number of moles  [A]1/2 


Now V = 2  [A]1/2 − 0 
M  0
2 
 
10 4 k
= = 8 × 108 L/day
1.25 × 10−5
[A]0
If SO3 emission stopped, t3/4 =
k
d[SO]3
= –6. 93 × 10–3[SO3]
dt Sol 15: (A, D) Solution will be optically active and
−6.93×10−3 t dextro after very long time
⇒ [SO3] = [SO3]0 e

[SO3] 0 = [SO3]0 e−6.93×10


−3 t After 75% conversion of A in to B and C angle of rotation
of solution will be 36°
[SO3] 0 = 1. 25 × 10–5 M

t = 1000 days Sol 16: (C, D)


(A) is correct
\[SO3]0 ≈ 1. 2 × 10–8 M
Rate, R = k[A] m[B] n[C]0
−d[A] log R = log k + m log[A] + n log[B]
Sol 14: (A, B, D) = k(A) 1/2
dt If all concentration other than (A) are constant,
[A] t
−d[A] d(logk)
∫ = ∫ k dt Then =m
1/2
[A]0 [A] 0 d(log[A])
[A] t (C) Order of reaction can be fractional
2[A]1/2 = kt
[A]0 0
(D) Order of a reaction can only be determined
kt experimentally
(A) 1/2 = [A]1/2
0 =
2
−1 1/2 1/2) Sol 17: (A, C, D)
k= (A .A
2 0 (A) Unit of A = Unit of k ≠ mol L–1s–1
kt
A = + A0 (B) Order of an elementary must atleast be 1 (atleast 1
2 molecule should be present)
Straight line (C) Molecularity can be defined for any elementary
[A]0 reaction.
At t = t1/2, (A) =
2 (D) Decay constant (λ) of radioactive substance is
independent of temperature.
2 2k
t1/2 =
[A0 ]1/2 Sol 18: (A, C) With increase in [A]0, rate will increase,
[A0 ]1/2 − 0
2 leading to increase in yield of B. A → B has higher Ea,
1/2 which means it has lower k and thus with increase in
 [A]  temperature k1 will increase more than k2, resulting in
2  [A0 ]1/2 1/2
0 2([2A]0 − 
 2 increase in yield of B.
k

2 2k 2[A]1/2 Sol 19: (A, B, C)


= = 0
( 2 − 1)
( 2 − 1)[A]0 k (D) A zero order reaction is always complex, rest all are
correct.
[A]0
At t = t3/4, (A) =
4
2k
\t3/4 =
[A]1/2
[A]1/2
0 −
0
2
Chem i str y | 18.87

Assertion Reasoning Type Sol 22: (A)

Sol 20: (C) Order of an elementary reaction cannot be k1 B


fractional, through the overall order of a reaction can x
be fractional.
k2 C
k1 2B
A (B = y, C = z)

k2 2C We have,
[B] k 1 d[x]
We have, = 1 = = –(k1 + k2)[x]
[C] k2 2 dt
1 [x]t t
For every mole of A consumed, mole used for forming d[x]
3 ∫ = −(k1 + k 2 )∫ dt
B and rest for forming C. [x]
[x]0 0
[A]0 = 2 moles
⇒[x]t = [x]0e–kt, k = k1 + k2
At end of 50% reaction, A0 = 1 mole
1 2 d[B]
Bn = ×2×1 = moles Now, = k1[x]t
3 3 dt
2 4 d[B]
Cn = ×2×1 = moles = k1[x]0e–kt
3 3 dt
[B]t
[Since, for every mole A, 2 moles of B (or C) is produced] t
−kt
∫ d[B] = k1[x]0 ∫e
0 0
Sol 21: (D) A(aq) → B(aq) + C(aq)
−k1 [x]0 t
t=0 n0 [B]t = e−kt
k 0
t = t n0 – x x x
−k1 [x]0
t →∞ 0 n0 n0 [B]t = (1 − e−kt )
k1 + k 2
At t → ∞,θ∞ = –5º = n0.θB + n0θC
k 2 [x]0
Similarly [C]t = (1 − e−kt )
We have, θA = 20º, θB = 30º, k1 + k 2

θC = –40º [B]t + [C]t = [y] + [x]

\ 30n0 – 40n0 = –5 k1 [x]0 k 2 [x]0


= (1 − e−kt ) + (1 − e−kt )
n0 = 0.5 k k
 k + k2  (1 −e−kt )
At time, t = 6.43 min. =  1 −kt
 [x]0 (1 − e ) = [x]0 e
 k 
θt = (n0 – x)θA + x(θB + θC) = 2.5
[x]t e−kt 1
20n0 + x(30.40 – 20) = 2.5 ∴ = =
[y] + [2] 1−e −kt
e
+ (k1 +k 2 )t
−1
30x = 20 × 0.5 – 2.5
Sol 23: (C) Using results of previous question
30x = 7.5

x = 0.25 k 2 [A]0
[C]t = (1 − e−kt ) , k = k1 + k2
k
For first order conversion,
[A]t = [A]0e–kt
2.303 n 2.303 0.5
k= log 0 = log
t n0 − x 6.93 0.5 − 0.25 [C]t k2 (1 − e−kt ) 1
= = (e−kt − 1)
[A]t k1 + k 2 e −kt k1
k = 0.1 min–1 1+
k2
1 8 . 8 8 | Chemical Kinetics

We have, k1 = x hr–1, k2 = 10 k1 Paragraph 2

= 10 × hr–1, t = 1 hr Sol 28: (B) We have,


[C]t 1 n1A(g) → n2B(g)
∴ (e(x +10x) − 1)
=
[A]t 1
1+ time 0 a 0
10
[C]t 10 11x time t a – n1x n2x
= (e – 1)
[A]t 11
−1 d[A]
\Rate = k[A] = = k[A]
n1 dt
Sol 24: (C) For reactions with order ≤ 1, time of at t
completion can be determined as those reactions gets d[A]
⇒ ∫ = −n1k ∫ dt
fully completed in specific amount of time. a
dt 0

For reactions with order > 1, reaction never goes to 100 at


n = –n1kt
% completion and hence time of completion cannot be a
determined. −n1kt
at = a e

Sol 25: (A)


Sol 29: (D) n1A(g) → n2B(g)
Without catalyst t = 0 a 0
−n1kt n2 −n1kt
t=1 ae a(1 − e )
n1
Ea Total moles present
Energy

R
 −n kt n n −n kt 
= a e 1 + 2 − 2 e 1 
P
 n1 n1 

Reaction coordinate n n  −n kt 
= a  2 −  2 − 1  e 1 
n 
 1  n1  
With catalyst
Concentration is to remain same at time t,

total moles present(n) a


∴ = 0
V V0
Ea
n  n  −n kt 
Energy

R V0 n
⇒V = = V0  2 −  2 − 1  e 1 
a0  n1  n1
P
 

Reaction coordinate Sol 30: (B) If n1 = 1, n2 = 2


at time t,

Comprehension Type nA = ae–kt

Paragraph 1 V = V0(2 – (2 – 1)e–kt)

= V0(2 – e–kt)
2 4
Sol 26: (B) Total number of moles = 1+ + = 3 moles  e−kt 
3 3 nA ae−kt
(A) = = = (A)  
2 V V0 (2 − e−kt )
0  2 − e−kt 
Sol 27: (C) Moles of B = = 0. 666 moles.  
3
Chem i str y | 18.89

Paragraph 3 Sol 33: (B) Slope of ln(1 – f) vs f graph


k1 [A]0 −3 –1
Sol 31: (C) (B) = [e
−k1t
−e
−k 2t
] = –k = hr
k 2 − k1 200
3
d[B] k [A] −k t −k t ⇒k = hr–1
= 1 0 ( −k1e 1 + k 2e 2 ) 200
dt k 2 − k1
1
d[B] When f =
When (B) is maximum, =0 2
dt 1
d[B] = 1 – e–kt
=0 2
dt n2 0.693 × 200
−k1t −k 2t
t= = = 46. 2 hrs.
⇒ −k1e + k 2e =0 k 3

−k1t −k 2t
k1 e = k 2e Sol 34: (A) f = 1 – e–kt

k1 f = [1 – e–3t/200]
=
k2
k1 1 k Paragraph 5
(k1 – k2)t = ln ; ⇒t = ln 1
k2 k1 − k 2 k 2 3
Sol 35: (C) For t = 100 min, x = a
4
Sol 32: (C) For k1 = 1000 s–1, k2 = 20 s–1, 1 a n2
\k = ln = min–1
100 3 50
[A]t = [A]0 e–1000t a− a
4
1000 When x = 0. 9 a,
[B]t = [A] [e−1000t − e−20t ]
20 − 1000 0 1 a
k= ln
1 1000 t a − 0.9a
≈ [A]0e–20t, tmax = n
1000 − 20 20
1 50ln10 50 50
t= ln10 = = = ; = 166. 66 min
[B]t will increase till tmax and then decay exponentially k ln2 ln2 0.3

[C]t = [A]0 – ([A]t + [B]t) 2


Sol 36: (B) [A]0 = = 0. 16
= [A]0 – [A0] e–1000t – [A]0e–20t 12.5
We have,
= [A]0(1 – e –1000t
–e –20t
) 1 d[B]
= k[A]
≈ [A]0(1 – e )
–20t 3 dt
A(g) → 3B(g) + 2C(g)
[C]t will approach [A]0 asymptotically.
At time 0 0. 16
At time t 0. 16–x 3x 2x
Paragraph 4
d[A]
df Also, = –k[A]
We have = k(1 – f) dt
dt
[A] = [A]0 e–kt
f f
df −2 ×100
∫1−f = ∫ k dt = 0. 16 e1.386×10
0 0
= 0. 04
f
−ln(1 − f)0f = k f
0 i. e. , 0. 16 – x = 0. 04
–ln(1 – f) = kt x = 0. 12
f = 1 – e–kt (B) = 3x = 0. 36.
1 8 . 9 0 | Chemical Kinetics

Match the Columns We know,

Sol 37: (A) → s 1


t1/2 ∝
[A0 ]n−1
We have,
decreasing graph
1 d[B] −d[A]
=
2 dt dt Matches (C).

Straight line with slope equal to 2. 1


(U) vs t(order = 2)
[A]
(B) → r
we have,
[A] = [A]0e–kt
−d[A] −k
exponential decay =
dt [A]2
(C) → p
[A] t
d[A]
(B) = 2([A]0 – [A]); = 2[A]0 (1 – e–kt) ⇒ ∫ = −k ∫ dt
[A]0 [A]2 0
Exponential graph will approach 2(A)0 asymptotically.
[A]
(D) → q −1 t
⇒ = −k t
[A]0 0
For zero order, [A0 ]

[A] = [A]0 – kt 1 1
⇒ − = –kt
Straight line with slope –k. [A]0 [A]

1 1
Sol 38: We will draw all graphs given in Column-II and ⇒ = kt +
[A] [A]0
then match them with Column-I.
Straight line with positive slope and positive intercept.
(p) ln (A) vs t (order 1)
Matches no option in Column-I.
We have, [A] = [A]0e–kt
(V)r vs [A] (order = 1)
ln[A] = ln[A]0 – kt
r = k[A]
Straight line with negative slope matches (A). Straight line with positive slope.
(q) t1/2 vs (A)0 (order = 1) Matches (D)
0.693 (A) → p
t1/2 =
k (B) → q, s
It is independent of [A]0, straight line parallel to x-axis. (C) → r, t
Matches (B)
(D) → v
(r) r vs t (order > 0)

r = k[A]n, n > 0
Previous Years’ Questions
for order > D, rate decreases with increasing t as (A)
decreases with t Sol 1: (A) The rate constant (k) of all chemical reactions
\decreasing graph (not straight line) matches (C). depends on temperature.

(s) r vs t (order = 0) k = Ae–Ea /RT


where, A = Pre-exponential factor,
For order, r is constant of time, graph is straight line
parallel to x-axis. Ea = Activation energy.

(t) t1/2 vs [A]0 (order > 1)


Sol 2: (C) A catalyst increases the rate of reaction but
by the same factor to both forward and backward
Chem i str y | 18.91

directions. Hence, a catalyst shorten the time required Sol 11: k = 1.5 × 10–6 s–1
to reach the equilibrium.
100
kt = ln
100 – x
Sol 3: (D) Rate will be directly proportional to both
concentration and intensity, i.e, rate of formation of AB 100
⇒ ln
* ∝ C. I. 100 – x
= 1.5 × 10–6 s–1 ×10 × 60 × 60s; = 0.0054
Sol 4: (D) Order of a reaction can take any real value, 100
⇒ = 1.055
i.e, negative, integer, fraction, etc. 100 – x
⇒ x =5.25% reactant is converted into product
Sol 5: (A) According to Arrhenius equation, rate
ln2 0.693
constant increases exponentially with temperature : Half-life = = = 462000 s; = 128.33 h
k = Ae–Ea /RT k 1.5 × 10 −6

Sol 12: The minimum rate of decay required after 6.909


Sol 6: (A) Living plants maintain an equilibrium
h is 346 particles min–1.
between the absorption of C14 (produced due to cosmic
radiation) and the rate of decay of C14 present inside ⇒ Rate = kN
the plant. This gives a constant amount of C14 per gram Rate 346 × 66.6 × 60
of carbon in a living plant. ⇒N= =
k 0.693
= 1.995 × 106 atoms
Sol 7: (B) Fossil whose age is closest to half-life of C–14
(5770 yr) will yield the most accurate age by C–14 dating. N0
⇒ kt = ln
N
N0 ln2 N
Sol 8: (A) λT =ln ⇒ × 6.909 = ln 0 = 0.0715
N 66.6 N
where N0 = Number of C14 in the living matter and
N = Number of C14 in fossil. Due to nuclear explosion, N0
⇒ = 1.074
amount of C14 in the near by area increases. This will N
increase N0 because living plants are still taking C–14
⇒ N0 = 1.074 × N = 1.074 × 1.995 × 106
from atmosphere, during photosynthesis, but N will not
change because fossil will not be doing photosynthesis. = 2.14 × 106 atoms of Mo
⇒ T (age) determined in the area where nuclear ⇒ Mass of Mo required
explosion has occurred will be greater than the same
2.14 × 106
determined in normal area. = × 99 = 3.56 × 10–16g
6.023 × 1023
C1
Also, λT1 =
ln
C Sol 13: CH3–O–CH3(g) → CH4(g)+ H2(g)+CO(g)
C
ln 2
λT2 = At 12 min :0.40 – p p p p
C
C1 Total pressure = 0.4 + 2p
⇒ T1 – T2 = ln
C2
0.40
C = Concentration of C–14 in fossil. Also k × 12 = ln
0.40 – p
ln2
Sol 9: Rate of reaction is constant with time. = × 12 = ln 0.40
14.5 0.40 – p
ln2 0.693 ⇒ p = 0.175
Sol 10: k = = yr–1 = 1.2 × 10–4 yr–1
t1/2 5770 ⇒ Total pressure = 0.4 + 2p
1 ln2 = 0.4 + 2 × 0.175
Also kt = ln = × 11540 = ln 4
f 5770
= 0.75 atm.
1
⇒f= = 0.25
4
1 8 . 9 2 | Chemical Kinetics

k2 Ea  T2 – T1  Sol 16: (D) aG + bH → Product


Sol 14: (A) ln =  
k1 R  T1 T2  rate ∝ [G]a [H]b
a = 1, b = 2
 4.5 × 107  Ea  50 
⇒ ln  =
 7  8.314  323 × 373  0.693 0.693
 1.5 × 10  Sol 17: (A)
= k1 =
t1/2 40
⇒ Ea = 22 kJ
A0 1.386
Ea k0
= =
Also ln k = ln A – 2t1/2 2 × 20
RT
At 50°C :
k1 0.693 40 0.693
22 × 1000 = × = = 0.5 mol−1 litre
ln A = ln (1.5 × 10 ) –7
= 8.33 k0 40 1.386 1.386
8.314 × 323
⇒ A = 4.15 × 103 s–1
2.303log8 2.303 × 3log2
1 Sol=
18: t1/8 =
(b) N2O5 (g) → 2NO2 (g) + O2 (g) k k
2
p 2.303 2.303
600–p 2p =t1/10 = log10
2 k k
3  2.303 × 3log2 
Total pressure = 690 = 600 + p  
2  t1/8   k  × 10 9
⇒ p = 240 mm =
  × 10 =
 t1/10   2.303 
⇒ Partial pressure of N2O5(g)  
 k 
remaining
= 600 – 240 Sol 19: (D) Overall order of reaction can be decided by
= 360 mm the data given t75% = 2t50%
360 ∴ It is a first order reaction with respect to P.
⇒ Mole-fraction = = 0.375
960
From graph [Q] is linearly decreasing with time, i.e.
order of reaction with respect to Q is zero and the rate
Sol 15: (A) Partial pressure becomes half of initial in expression is r = k [P]1[Q]0.
every 100 min, therefore, order = 1.
Hence (D) is correct.
800
(B) k × 100 = ln = ln 2 n
400 r1
1 M
Sol 20: (C) = =   ⇒n= 3
⇒ k = 6.93 × 10–3 min–1 r2 8 2Mn
 
(C) For 75% reaction; time required
Sol 21: (B, C, D) A high activation energy usually
= 2 × half-life implies a slow reaction.
= 200 min
(D) 2X(g) → 3Y(g) + 2Z(g)
3
800–x x   x
2
3
Total pressure = 800 + x
2
Also 800 – x = 700
⇒ x = 100
⇒ Total pressure
3
= 800 + × 100 = 950 mm
2
2017-18 100 &
op kers
Class 12 T
By E ran culty
-JE Fa r
IIT enior emie .
S fP r es
o titut
Ins

CHEMISTRY
FOR JEE MAIN & ADVANCED
SECOND
EDITION

Exhaustive Theory
(Now Revised)

Formula Sheet
9000+ Problems
based on latest JEE pattern

2500 + 1000 (New) Problems


of previous 35 years of
AIEEE (JEE Main) and IIT-JEE (JEE Adv)

5000+Illustrations and Solved Examples


Detailed Solutions
of all problems available

Plancess Concepts
Topic Covered Tips & Tricks, Facts, Notes, Misconceptions,
Key Take Aways, Problem Solving Tactics
Solid State
PlancEssential
Questions recommended for revision
19. S O L I D S TAT E

1. INTRODUCTION
We know that solids are substances which have a definite volume and a definite shape. A solid is a nearly
incompressible state of matter with a well-defined shape, rigidity and volume.

2. CLASSIFICATION OF SOLIDS
Solids can be broadly classified into two types:
 1. Crystalline
 2. Amorphous

Table 19.1: Difference between crystalline and amorphous solids

S.No. Property Crystalline solids Amorphous solids


1. Geometry These have definite regular geometry These do not have definite orderly
which extends throughout the crystals i.e., arrangement.
these solids have long range order.
2. Melting Points These have sharp m.p. These do not have sharp m.p.
3. Symmetry These possess symmetry. These do not possess any symmetry.
4. Heat of Fusion These have definite heat of fusion. These do not have definite heat of fusion.

5. Interfacial angles Crystals are always guarded by planes so These do not possess interfacial angle.
that a definite angle exists between two
planes called interfacial angle.
6. Volume change There is a sudden change in volume when There is no sudden change in volume on
it melts. melting.
7. Isotropy and These are anisotropes i.e., their physical These are isotropic i.e., their physical
Anisotropy properties such as refractive index, properties such as refractive index,
conductivity etc. have different values in conductivity etc. have same value in all the
different directions. directions.
1 9 . 2 | Solid State

PLANCESS CONCEPTS

Glass Transition Temperature


An important property of amorphous polymer is glass transition temperature at which a glassy polymer
transforms into flexible rubber-like mass capable of taking on shapes and forms. This property is used in
the processing of polymers.
Vaibhav Krishnan (JEE 2009 AIR 22)

3. TYPES OF CRYSTALLINE SOLIDS


On the basis of the nature of the constituent particles and the interparticle forces, crystalline solids may be divided
into four classes.

Table 19.2: Characteristics of various types of solids

Characteristics Molecular Solids Covalent Network Metallic Solids Ionic Solids


1. Units that occupy Atoms or molecules Atoms Positive ions in a ‘sea Positive and negative
of electrons’ ions
2. Binding forces Vander Waal’s forces Covalent bonding Metallic bonding Ionic bonding
(weak intermolecular (shared pair of (extreme delocalised (electrostatic attraction
forces) of following electrons) bond) – positively between positive and
two types charged atomic negative ions)
cores surrounded by
(i) London (dispersion)
‘sea’ of delocalised
forces
electrons
(ii) Dipole-dipole
forces
3. Physical Properties (i) Very soft. (i) Very hard (i) Hard or soft (i) Quite hard and brittle

(ii) Low melting points (ii) Very high melting (ii) Moderate high (ii) Fairly high melting
points melting points points
(iii) Non conductors
(iii) Non conductors (iii) Good conductors
4. Example Solid Ne, Solid NH3, Diamond, Cu, Fe, Ag NaCl, KNO3, Na2SO4
ice (Solid H2O), dry ice Carborundum (SiC),
(Solid CO2) quartz (SiO2)

PLANCESS CONCEPTS

Stability of a crystal is reflected in the value of its melting point. A more stable structure would require
higher energy and hence melt at higher temperature.
Nikhil Khandelwal (JEE 2009 AIR 94)
Chem i str y | 19.3

4. SALIENT FEATURES OF CYRSTALS


A crystal possesses the following characteristic features:
 1. Faces: The surfaces are usually planar and arranged on a definite plane (as a result of internal geometry)., The
planes which bind the crystal, are called faces. Faces are of two types:
(i) Like: A crystal having all faces alike, e.g. Fluorspar.
(ii) Unlike: A crystal having all faces not alike, e.g. Galena.
 2. Form: All the faces corresponding to a crystal are said to constitute a form.
 3. Edges: The intersection of two adjacent faces forms an edge.
 4. Interfacial Angle: The angle between the normals to the two intersecting faces is called interfacial angle.
 5. Zone and Zone Axis: The faces of a crystal occur in sets called zones, which meet in parallel edges or would
do so if the planes of the faces are extended. Each zone forms a complete belt around the crystal.
A line passing through the centre of a crystal in a direction parallel to the edge of zone is known as zone axis.
Space Lattice: A three dimensional arrangement of points that shows how the atoms or ions are arranged in space
is called a Space Lattice. Each point is individually called a lattice point.

Space Lattice
z

Lattice Point
c
a x
b
c
Unit Cell a

b  Representation of
y
dimension of unit cell

Figure 19.1: Representation of space lattice and unit cell

Crystal Lattice: A space lattice is called a crystal lattice when the lattice points in the space are replaced by actual
atoms or ions.
Crystal System: The unit cell in a 3-dimension lattice is characterized by the length a, b, c and their angles α, β, γ.
These are known as unit cell parameters.
Bravais Lattices: An arrangement of spheres as given above leads to simple or primitive unit cell, when there are
points only at the corner of the unit lattice. However, certain unit cells have lattice points on other sides also in
addition to the corners. Such unit cell are called non-primitive unit cells. Bravais (1848) showed from geometrical
considerations that there can be only 14 different ways in which similar points (spheres) can be arranged.
These fourteen types of lattice are known as Bravais lattices. They can be divided into seven crystal systems as
shown below:
Table 19.3: Crystal system table

S.No. Crystal System Axial Characteristics Examples


1. Cubic (Most symmetrical) a=b=c NaCl, KCl, CsCl, zinc blende, Cu, Ag,
α = β = γ = 90º diamond
2. Tetragonal a=b≠c White tin, SnO2, TiO2
α = β = γ = 90º
1 9 . 4 | Solid State

3. Orthorhombic a≠b≠c Rhombic sulphur, PbCO3, PbSO4, KNO3


α = β = γ = 90º
4. Monoclinic a≠b≠c Monoclinic sulphur, CaSO4.2H2O ,
α = γ = 90º; β ≠ 90º
Na2SO4.10H2O

5. Rhombohedral or trigonal a=b=c Calcite, quartz, As, Sb, NaNO3, ICl


α = β = γ ≠ 90º
6. Triclinic(Most unsymmetrical) a≠b≠c K2Cr2O7, H3BO3, CuSO4.5H2O
α ≠ β ≠ γ ≠ 90º
7. Hexagonal a=b≠c Graphite
α = β = 90º; γ = 120º

PLANCESS CONCEPTS

The relationship between the plane faces (F), straight edges (e) and interfacial angle (C) for a given
crystal can be written as:
F + C =e + 2
Saurabh Gupta (JEE 2010 AIR 443)

4.1 Laws of Crystallography


The geometric crystallography (the outward spatial arrangement of crystal planes and shape of crystals) is based
on the following three fundamental laws:
(a) The Laws of Constancy of Interfacial Angles: The crystals of a substance can have different shapes
depending upon the number and size of the faces but the angle at which the two adjacent faces intersect
remains always constant. Depending on the number and size of the faces, the crystals of a substance can have
different shapes. However, the angle at which two adjacent faces intersect always remains constant.
(b) Hauy’ s Law of Rationality of Indices: The intercepts of any face of plane of a crystal on suitable crystallographic
axes can be expressed by small multiples of three unit distances a, b, c or some simple integral multiple (m, n,
p) of these unit distances, i.e., ma: nb: pc or fraction of whole numbers.
(c) The Law of Constancy of Symmetry: According to this law, all crystals of the same substance possess the
same elements of symmetry.

5. SYMMETRY IN A CRYSTAL
(a) Centre of Symmetry: Centre of symmetry of a crystal may be defined as an imaginary point within a crystal
such that any line passing through it, intersects the crystal at equal distances in both directions e.g. staggered
form of ethane possesses a centre of symmetry.

Every Crystal Possesses Only One Centre of Symmetry


(b) Plane of Symmetry: It is an imaginary plane passing through the crystal which can divide it into two equal
parts in such a way that one part is an exact mirror image of the other. Thus, they are also referred to as mirror
planes. There are two types of mirror planes (as shown below).
(A) Rectangular mirror planes (B) Diagonal mirror plane
Chem i str y | 19.5

Rectangular mirror plane Diagonal mirror plane

Figure 19.2: Representation of type of plane of symmetry present in a unit cell

5.1 Axis of Symmetry or Axis of Rotation


An axis of symmetry or axis of rotation is an imaginary line, passing through the crystal such that when the crystal is
rotated about this line, it presents the same appearance more than once in one complete rotation. I.e. through 360º.
360º
When the same appearance of crystal is repeated, on rotating through an angle of around an imaginary axis
it is called n-fold axis of symmetry where n– is known as order of axis. n

Two-Fold Axis: If a similar appearance occurs two times in one complete rotation i.e. after a rotation through 180º,
the axis is called two fold axis of symmetry or dyad axis.

Three-Fold Axis: If the original appearance occurs thrice, then it is called three fold axis of symmetry.
Four fold Axis Two fold axis
of symmetry Three fold Axis

(a) (b) (c)


Six fold Axis
of symmetry 1-Rectangular Plane

2-Rectangular 3-Rectangular
Plane Plane in a cube

(d)

(e)
A B
Diagonal plane -1 CBFB
C Diagonal plane -2 ADHE
D
Diagonal plane -3 ABHG
E F Diagonal plane -4 CDEF
Diagonal plane -5 CAFH
Diagonal plane -6 EGDB
G H
(f)

Figure 19.3: Representation of axis of symmetry present in a unit cell

PLANCESS CONCEPTS

A crystal may have a number of planes or axis of symmetry but it possesses only one centre of symmetry.

Neeraj Toshniwal (JEE 2009 AIR 21)


1 9 . 6 | Solid State

5.2 Total Elements of Symmetry in a Cubical Crystal


Total planes of symmetry =3+6=9
Axis of symmetry = 4 (3-fold axis)
  3 (Four fold axis)
  6 (Six fold axis) = 13 Axes of symmetry
Centre of symmetry =1
Hence, a cube has 23 elements of symmetry.

6. UNIT CELL
The smallest repeating pattern in a space lattice is called a unit cell.

6.1 Types of Unit Cell


Unit cells are classified as simple or primitive, body centred, face centred and end centred. This classification
depends on the position of constituent particles in a unit cell.

Simple or Body centred Face centred End centred


Primitive (P) (I) (F) (E)
Figure 19.4: Different types of unit cell

6.2 Number of Atoms in a Unit Cell


(i) A point lying at the corner of a unit cell is shared equally by eight unit cells and therefore, only one-eighth
(1/8) portion of each of such a point belongs to the given unit cell.
(ii) A point present on an edge is distributed among the four unit cells, therefore only one-fourth (1/4) of such a
point belongs to the given unit cell.
(iii) A face-centred point is shared between two unit cells. Therefore one-half (1/2) part lies in each unit cell.
(iv) A body-centred point belongs entirely to one unit cell since it is not shared by any other unit cell. Therefore,
its contribution to the unit cell is one.

Table 19.4: Number of atoms in different type of unit cell

Type of cell Number of atoms Number of atoms Number of atoms in Total


at corners in faces the body of cube

Simple or primitive 1
8× 1
= 0 0 1
cubic 8

Body-centred cubic 1
8× 1
= 0 1×1=1 2
(bcc) 8

Face-centred cubic 1 1
8× 1
= 6× 3
= 0 4
(fcc) 8 2
Chem i str y | 19.7

Illustration 1: A solid has a cubic structure in which X atoms are located at the corners of the cube, Y atoms are at
the cube centres and O atoms are at the edge centres. What is the formula of the compound?  (JEE MAIN)
Sol: X atom is occupying simple cubic structure, y atom occupies body center and O occupies edge centre. So
calculate the no of atoms of X, Y and O present in a unit cell.
Atoms of X are present at all the eight corners of the cube. Therefore, each atom of X at the corner makes
1/8 contribution towards the unit cell.
1
Number of atoms of X per unit cell = 8 × = 1
8
Y atom is present at the body centre, thus contribution of Y towards unit cell = 1 x 1 = 1
O atom is present at each of the edge centre (number of edges of cube = 12)
And each O atom present at edge centre will make 1/4 contribution towards the unit cell.
1
The number of O atoms per unit cell = 12 × =3
4
The formula of the compound is, therefore XYO3.

Illustration 2: A cubic crystalline solid contains P atoms at the corners and Q atoms at the body centres. lf one
atom from the corner is missing, what will be the simplest formula of the resulting solid?  (JEE MAIN)
1 7
Sol: ZP = 7 × =
8 8
ZQ = 1
∴ The simplest formula is P7 Q = P7 Q8 .
8

Illustration 3: A compound alloy of gold and copper crystallizes in a cube lattice in which the gold atoms occupy
the lattice points at the corners of a cube and the copper atoms occupy the centres of each of the cube faces.
Determine the formula of this compound.  (JEE MAIN)

Sol: Determine the number of atoms of Au (Occupying simple primitive structure) present in one unit cell and no.
of atoms of Cu (occupying face center arrangement) present in one unit cell .
One-eighth of each corner atom (Au) and one-half of each face-centred atom (Cu) are contained within the unit
cell of the compound.
1
Thus, number of Au atoms per unit cell = 8 × =1 , and
8
1
Number of Cu atoms per unit cell = 6 × = 3.
2
The formula of the compound is AuCu3.

Illustration 4: A compound formed by elements A and B crystallizes in cubic structure where A atoms are at the corners
of a cube and B atoms are at the body centre of the cube. What is the formula of the compound? (JEE MAIN)

Sol: Determine the number of atoms of A (Occupying Corner of a cube) present in one unit cell and no. of atoms
of Ag (occupying Body center arrangement) present in one unit cell
An atom at the corner of the cube is shared by 8 cubes and hence, contributes only 1/8 to a particular cube. There
are eight corners of a cube.
1
∴ The number of A atoms in the unit cell = 8 × = 1
8
An atom at the body centre of cube belongs only to one unit cell.
∴ Number of atoms of B in the unit cell = 1 × 1 = 1.
Thus, formula is AB.
1 9 . 8 | Solid State

6.3 Coordination Number


The number of nearest neighbours that an atom has in a unit cell is called the coordination number. In ionic crystal,
the number of oppositely charged ions surrounding each ion is called its co-ordination number. The co-ordination
number of a crystal depends upon its structure.
(a) Simple Cubic Structure: It is clear from the Figure that each atom say (A) has 4 nearest neighbours 2, 3, 4 and
one to the left of A of another unit cell in one plane. In addition, it also possesses one atom vertically above of
another unit cell. Thus, in all, A possesses six neighbour atoms and therefore, a simple cubic structure reveals
a co-ordination number of six, e.g., NaCl.
2
A 4

3
Simple cubic
Figure 19.5-a: Diagramatic representation of simple unit cell

(b) Face Centred Cubic Structure (fcc): It is clear, from the figure, that the nearest neighbour of a corner atom
is the face centred atom and thus, each atom say (A) has 4 nearest neighbours 1, 2, 4 and the other is on its
left side from another cell. In addition, it also possesses four face centred atoms in planes below and above
it. Thus, in all, A possesses 12 neighbour atoms and therefore, face centred cubic structure reveals a co-
ordination number of twelve, e.g., Copper.
E F
2
A
B
1
3
G 4
H

D C
Face centred cubic
Figure19.5-b: Diagramatic representation face centred unit cell

(c) Body Centred Cubic Structure (bcc): It is clear from the figure, that the nearest neighbour of a corner atom
is body centred atom. Thus, each atom say (A) has eight body centred atoms of 8 surrounding unit cells
as neighbours in one plane, since A is surrounded by 8 surrounding unit cells. Thus, a body centred cubic
structure reveals a co-ordination number of eight, e.g., CsCl.

Body centred cubic

Figure19.5-c: Diagramatic representation of body centred unit cell


Chem i str y | 19.9

PLANCESS CONCEPTS

Pressure and temperature changes the coordination number of a crystal.


•• High pressure increases the co-ordination number.
•• High temperature decreases the co-ordination number.
Aman Gour (JEE 2012 AIR 230)

6.4 Density of Lattice Matter


It is defined as the ratio of mass per unit cell to the Volume of unit cells.

mass per unit cell


Density of lattice matter =
volume of unit cell
Atomic mass
Mass per unit cell = No. of Atoms × Mass of one of atom = n ×
Avog. No.
n × at. wt.
Density of lattice matter =
Av.no. × volume of unit cell
n × atomic mass (or molar mass)
ρ =
NA × V
n × atomic mass
ρ =
NA × a3
Where ‘a’ is edge length of cube. (in cm)
Where ‘n’ is number of atoms per unit cell; volume of unit cell is to be derived for given crystal system. It is equal
to a3 for cubic crystal systems.

Illustration 5: The edge length of a cubic crystal of an element is found (by X-ray diffraction) to be ‘a’ picometres.
Show how you can calculate its density. (JEE MAIN)

Sol: The density of the unit cell is the same as the density of the substance.
Edge length of unit cell = a pm = a × 10–10 cm
Volume of the unit cell = a3 × 10–30 cm3
n×M
Density of the unit cell = gcm−3
3 −30
a × NA × 10

Illustration 6: Chromium metal crystallizes with a body centred cubic lattice. The edge length of the unit
cell is found to be 287 pm. Calculate the atomic radius. What would be the density of chromium in g/cm3?
(At. wt. of Cr = 51.99) (JEE MAIN)

Sol: From the given value of edge length first determine the atomic radius for a bcc crystal lattice and density can
be determined using the following formula,

n × at. wt. n × at. wt.


Density = =
V × Av. no. a3 × Av. no.

3a 3
For bcc lattice, r = = × 287 =124.27 pm
4 4
n × at. wt. n × at. wt.
Now, Density = =
V × Av. no. a3 × Av. no.
1 9 . 1 0 | Solid State

n = 2 for bcc; a = 287 pm = 287 × 10–10 cm

2 × 51.99
∴ ρ= = 7.31 g / cm3
−10 3 23
(287 × 10 ) × (6.023 × 10 )

Illustration 7: An element (density 6.8 g cm–3) occurs in bcc structure with unit cell edge of 290 pm. Calculate the
number of atoms present in 200 g of the element.  (JEE ADVANCED)

Sol: First determine the molecular mass of the element by using following equation,
n×M
Density =
Av.no. × a3
And from molecular mass determine the number of atoms present in 200 g of the element.
Let M be the molecular mass of the element.
Volume of the unit cell = a3 = (290 × 10–10 cm)3 = 24.4 × 10–24
In a body centred cubic (bcc) structure n = 2.
n×M
Density =
Av.no. × a3

2×M
6.8 =
23
6.023 × 10 × 24.4 × 10−24

6.8 × 6.023 × 1023 × 24.4 × 10−24


M= = 50 g
2

∴ 50 g of the element contain 6.023 × 1023 atoms

6.023 × 1023 × 200


200 g of the element would contains = = 24.09 × 1023 atoms
50

6.5 Nearest Neighbouring Distance in Case of Identical Spheres


Nearest neighbouring distance is the distance between the centres of any two touching spheres.
(a) In case of SCC (Simple Cubic Lattice)
In a simple cubic lattice the edge length (a) = Nearest neighbouring distance
a = d = 2r
a
⇒ r =
2

d
a
Figure19.6-a: Nearest neighbouring distance in case of SCC

(b) Body Centred


2
(AB)
= (AC)2 + (BC)2
⇒ (Body Diagonal)2 = (Face Diagonal)2 + (edge)2
= 2(edge)2 + (edge)2 = 3(edge)2
Chem i str y | 19.11

⇒ (Body Diagonal)2 = 3(edge)2 = 3a2


∴ body diagonal = 3 a
Also Body Diagonal = r + 2r + r = 4r

3
⇒ 24r = 3a ⇒ r = a
4

3
d = 2r ⇒ d= a = nearest neighbouring distance
2

A B
r
r
r
d r

C D

Figure19.6-b: Nearest neighbouring distance in case of BCC

(c) Face Centred Cubic Lattice


Here (Face diagonal)2 = (edge)2 + (edge)2
∴ ( face diagonal) = a2 + a2 = 2a2
2

∴ face diagonal =
2a
∴ 4r = 2a

B
r
r
r
C
r
A
d est ing
ar r u
ne bo ce
g h an
i t
ne dis

Figure 19.6-c-Nearest neighbouring distance in case of FCC

6.6 Packing Fraction

Packing Fraction in Case of Identical Spheres


Packing fraction is the ratio of volume occupied by sphere to the total volume of the cube
4 3
πr × z
Volume occupied by sphere 3

= Packing Fraction =
Total volume of cube a3
where a = edge of cube, z = number of sphere per unit cell
1 9 . 1 2 | Solid State

Packing fraction of SCC


3
4 a
π  × z
a 3 2 4π π
r= , z = 1; =
PF = =
2 a3 8×3 6 '

π
⇒ PF = =0.524
6 π
⇒ % volume occupied by sphere = × 100 =
52.4%
6

⇒ % Free space = 100 - 52.4 = 47.6%

Packing fraction for BCC


3
4  3 
π a × 2
3 3  4  4 π×3 3 ×2 3π
r= a , z = 2; PF = = × = 0.68
=
4 a3 3 64 8

⇒ % volume occupied = 100 × 0.68 =


68%
% Free space = 100 – 68 = 32%

Packing Fraction for FCC


3
4  2 
π a × 4
2 3  4  4 π×2 2 × 4 2π
r= a , z = 4 PF = = × = 0.7407
=
4 a3 3 64 6


⇒ PF = = 0.740
6
% volume occupied = 100 × 74 =
74 %
% Free space = 100 – 74 = 26 %

PLANCESS CONCEPTS

The packing density data reveals that close packing of atoms in cubic structure follow the order,
FCC > BCC > SCC, i.e., more closely packed atoms are in FCC structure.
B Rajiv Reddy (JEE 2012 AIR 11)

Table 19.5: Summary of Various Characteristics of Different Cubic Systems

S. No. Cubic Nuber of Coordi- Atomic Packing Density (ρ) Nearest


effective nation radius (r) efficiency neighbour
(g cm )–3
atoms per Number distance
unit cell (Zeff) (CN) (d)

Z eff × Mw
π ρ= a
Simple a PE= × 100
1. 1 6 r= 6 NA × a3 × 10−30
cube (sc) 2 = 52.4% 1 × Mw
=
NA × a3 × 10−30
Chem i str y | 19.13

Body 3
3a PE
= π × 100 2 × Mw 3
2. centred 2 8 r= 8 ρ= a
cubic (bcc) 4 = 68% 3
NA × a × 10 −30 2

Face 2 4 × Mw
3. centred 4 12 a PE
= π × 100 ρ= a
r= 6
cubic (fcc) 2 2 NA × a3 × 10−30 2
= 74.04%

Illustration 8: The density of solid Argon is 1.65 g/mL at –233ºC. If the Argon atom is assumed to be sphere of
radius 1.54 × 10–8 cm. What percentage of solid Argon is apparently empty space? (Ar = 40)  (JEE ADVANCED)

Sol: First determine the no. of atoms of Ar present in 1.65 g from this calculate the total volume occupied by all
atoms of Ar. we can determine % empty space by subtracting the volume of solid Ar taken and volume occupied
by Ar,
4
Volume of one atom of Ar = πr3
3
1.65
Also, number of atoms in 1.65 g or one cm3 = × 6.023 × 1023 =
N
40
4 3 4 22 1.65
∴ Total volume of all atoms of Ar in solid state = πr N = × × (1.54 × 10−8 )3 × × 6.023 × 1023 =
0.380 cm3
3 3 7 40
As the volume of solid argon taken = 1 cm3

[1 − 0.380]
∴ % empty space = × 100 =
62%
1

7. CLOSED PACKING OF CONSTITUENTS


(a) Close Packing in One Dimension: Here, spheres are arranged in a row, touching each other, in a one-
dimensional close packed structure as shown below,

Figure 19.7-a: Close


Closepacking
packingofofspheres in one dimension
spheres
in one dimension
(b) Close Packing in Two Dimensions: Two-dimensional close packed structure can be produced by stacking
the rows of close packed spheres. This can be done in two different ways as shown in figures (a) and (b).
(i) Square close packing
(ii) Hexagonal close packing

Figure 19.7-b: Close packing of spheres in two dimension


1 9 . 1 4 | Solid State

(c) Close Packing in Three Dimensions:


(i) Hexagonal Close Packing (HCP):.While forming the first layer, minimum space is wasted by utilizing the
maximum space possible. In every second row the particles occupy the depressions (also called voids)
between the particles of the first row. In the third row, the particles are vertically aligned with those in the
first row giving AB AB AB ... type of arrangement. This structure has hexagonal symmetry and is known
as hexagonal close packing (HCP). In HCP, the coordination number is 12 and only 26% space is free. A
single unit cell has 4 atoms.
G fold axis
A
B

B
A

(a) (b) (c)


Figure 19.7-c: Hexagonal close packing of spheres in three dimension

(ii) Cubic Close Packing (CCP): If we start with hexagonal layer of spheres and second layer of spheres
is arranged by placing the spheres over the voids of the first layer, half of these holes can be filled by
these spheres. Presume that spheres in the third layer are arranged to cover octahedral holes. When
organized this way, the third layer does not resemble either the first or second layer; but the fourth layer
is similar to the first layer, fifth layer to second, sixth to third and so on giving pattern ABCABCABC ... .
This arrangement has cubic symmetry and is known as cubic closed packed (CCP) arrangement. This is
also called face-centred cubic (FCC) arrangement.
The free space available in this packing is 26% and coordination number is 12.

C Three fold axis


B

(a) (b) (c)

Figure 19.7-d: Cubic close packing of spheres in three dimension

(iii) Body Centred Close Packing (BCP): In B.C.P. arrangement, 8 particles are present at the corners and 1
particle is present at the body centre. The coordination number of central sphere is 8. The efficiency of
this type of packing is less, only 68% space is occupied and 32% remains unoccupied. A single unit cell
has 2 atoms.
Chem i str y | 19.15

PLANCESS CONCEPTS

•• In two dimensional hexagonal close packing, there is less free space between sphere than two dimensional
square close packing. Thus, hexagonal close packing is denser than square close packing.
•• In hexagonal close packing about 60.4% of available space is occupied by spheres whereas square close
packing occupies only 52.4% of the space by spheres.
•• If only one layer of spheres is to be packed, the spheres arrange themselves in hexagonal close packing.
•• In both CCP and HCP, the co-ordination number of spheres remain twelve.

Rohit Kumar (JEE 2012 AIR 79)

Illustration 9: A hexagonal close-packed structure and a cubic close-packed structure for a given element would
be expected to have the same density. Explain.  (JEE MAIN)

Sol: In both the structures, the fraction of the total volume occupied is 0.74. The two structures have the same
co-ordination number of 12.

8. VOIDS OR HOLES
The voids/holes are the empty spaces in between closed packed spheres.

T T T T T T T
o o o o o o

T T T T T T
T T T T T T
o o o o o o

T T T T T T T

Figure 19.8-a: Representation of a Void

Types of Voids:
(a) Octahedral Voids: This void is surrounded by six spheres and formed by a combination of two triangular
voids of the first and second layer. There is one octahedral void per atom in a crystal. The radius ratio
(r void/rsphere) is 0.414.

Tetrahedral Tetrahedral
hole hole
Tetrahedron

Octahedral
hole Octahedral
hole
Octahedron
(a) (b) (c)

Figure 19.8-b: Tetrahedral and Octahedral Voids (a) top view, (b) exploded side view, and (c) Geometrical shape of the void
1 9 . 1 6 | Solid State

(b) Tetrahedral Voids: These voids are surrounded by four spheres, which lie at the vertices of a regular
tetrahedron. There are 2 tetrahedral voids per atom in a crystal and the radius ratio is 0.225.
(c) Trigonal Voids: The void enclosed by three spheres in contact is called a trigonal void. There are 8 trigonal
voids per atom in a crystal and the radius ratio is 0.155.

FigureTrigonal
19.8-c: Trigonal
void void
Locating Tetrahedral and Octahedral Voids: Both octahedral and tetrahedral voids are present in all closed
packed structures. In a CCP pattern, there is one octahedral void at the centre of body and 12 octahedral voids in
each of the 12 edges of the cube. Each void on the edge is shared by four other unit cells.

PLANCESS CONCEPTS

The co-ordination number of a tetrahedral void is four. The co-ordination number of an octahedral void is six.

Krishan Mittal (JEE 2012, AIR 199)

Table 19.6: Summary of the Main Characteristics and Examples of Some Simple Ionic Solids

Crystal Structure Brief description Examples Coordination Number of Coordination


number formula unit/ number
units cell
Type AB: Rock salt It has fcc arrangement Halides of Li, Each Na+ is General 6: 6
(NaCl) type in which Cl– ion occupy Rb, AgF, AgBr, surrounded by formula A4B4
the corners and face NH4Cl, NH4Br, 6 Cl–. Each Cl– is or AB
centres of a cube while NH4I surrounded by 6
Zeff = 4
Na+ ions are present Na+; occupy all OVs
at the body and edge
centres.
Radius for fcc
(r +
) a
+ r− =
2

Cesium Chloride It has bcc arrangement CsCl, CsBr, CsI, Each Cs+ is General 8: 8
(CsCl) type with Cs+ at the body CsCN, TlCl, TlBr, surrounded by 8 Cl–. formula AB
centre and Cl– ions at TlI and TlCN
Each Cl– is Zeff = 1
the corners of a cube
surrounded by 8 Cs .
+
or vice versa.
Radius for bcc type
(r +
+ r− =)
3a
2
Zinc blende (ZnS) It has ccp arrangement CuCl, CuBr, CuI, Each Zn2+ is General 4: 4
(sphalerite) type in which S2– ions form AgI surrounded by formula A4B4
fcc and each Zn2+ 4 S2–. Each S2– is or AB
ion is surrounded surrounded by 4
Zeff = 4
tetrahedrally by S2– ion Zn2+. Zn2+ are in
vice versa. alternate TVs.
Chem i str y | 19.17

Crystal Structure Brief description Examples Coordination Number of Coordination


number formula unit/ number
units cell
Type AB2 Fluorite It has ccp arrangement BaF2, BaCl2, SrF2, Each Ca2+ is General 8: 4
(CaF2) type in which Ca2+ ions form SrC2, CdF2, PbF2 surrounded by formula A4B4
fcc with each Ca2+ ions 8 F–. Each F– is or AB
surrounded by 8 F– ions surrounded by 4
Zeff = 4
and each F– by 4 Ca2+ Ca2+. F– ions occupy
ions all the 8 TVs.
Type A2B Here O2– form the Na2O, Li2O Each Na+ is General 4: 8
Antifluorite Type ccp arrangement so surrounded by 4 formula A4B4
that each Na+ ion is O2–. Each O2– is or AB
surrounded by four O2– surrounded by
Zeff = 4
ions and each O2– ion 8 Na+. Na+ ions
is surrounded by 8 Na+ occupy all the TVs
ions.

Illustration 10: In a cubic lattice, the closed packed structure of mixed oxides of the lattice is made up of oxide ion;
one eighth of the tetrahedral voids are occupied by divalent ions (A2+) while one half of the octahedral voids are
occupied by trivalent ions (B3+). What is the formula of the oxides?  (JEE ADVANCED)

Sol: Let there be n no. of O2– in the crystal.


∴ Octahedral voids = n
Tetrahedral voids = 2n
1 n 3+ 1 n
A2+ ions = × 2n = , B ions = × n =
8 4 2 2
n n
A2+ : B3=
+
: O2 − =: :n 1:2: 4
4 2
∴ Formula is AB2O4.

9. RADIUS RATIO
The ratio of radius of cation to anion is known as radius ratio of ionic solids, i.e.,
Radius of cation (or r + )
Radius ratio =
Radius of anion (or r − )

Following important conclusions can be drawn from radius ratio of ionic solids.
(a) If the radius ratio (r+/r–) = 0.225, the cation would fit exactly into the tetrahedral voids and have co-ordination
number four.
(b) If the radius ratio (r+/r–) = 0.414, the cation would fit exactly into the octahedral voids and have co-ordination
number six.

Illustration 11: If the radius of the bromide ion is 0.182 nm, how large a cation can fit in each of the tetrahedral
hole? (JEE MAIN)
Radius of cation (or r + )
Sol: Radius ratio =
Radius of anion (or r − )
For tetrahedral voids (r+/r–) = 0.414
1 9 . 1 8 | Solid State

From the given value of radius of the bromide ion determine the radius for cation. Depending upon the size of the
cation predict the answer.
For tetrahedral voids (r+/r–) = 0.414 (for maximum value of r+)
r+ = 0.414 × 0.182 = 7.53 × 10–2 nm
Thus, cation of 7.53 × 10–2 nm will fit exactly in tetrahedral voids.

Illustration 12: In the cubic crystal of CsCl (d = 3.97 g cm-3) the eight corners are occupied by Cl– with a Cs+ at the
centre and vice versa. Calculate the distance between neighbouring Cs+ and Cl– ions. What is the radius ratio of the
two ions? (At. wt. of Cs = 132.91 and Cl = 35.45) (JEE ADVANCED)

Sol: In a unit cell, n = 1 for cubic crystal

n × mol.wt. n × mol.wt.
∴ =
density =
V × Av.No. a3 × Av.No.
1 × 168.36
∴ 3.97 =
a3 × 6.023 × 1023
a = 4.13 × 10–8 cm
a = 4.13 Å
For a cube of side length 4.13 Å.
Diagonal = 3 × 4.13 =
7.15 Å
As it is a bcc with Cs+ at centre (radius r+) and Cl– corners (radius r–) so,
2r+ + 2r– = 7.15 or r+ + r– = 3.57 Å
i.e., distance between neighbouring Cs+ and Cl– = 3.57 Å
Now assume two Cl– ions touching each other
So, Length of unit cell = 2r– = 4.13 Å
∴ r– = 2.06 Å
∴ r+ = 3.57 – 2.06 = 1.51
r+ 1.51
∴ = = 0.73
r − 2.06

10. DEFECTS
Atomic Imperfection: Atomic imperfections are the defects that arise due to irregularity in the arrangement of
atoms or ions. These are of following types:

Point Defects: These are caused by missing or misplaced atoms or ions. These involve:
(a) Vacancy defect: If an atom or ion is missing from the lattice site, the defect is known as vacancy defect.
(b) Impurity defect: If a foreign atom or ion occupies a lattice site as an impurity, the defect is called impurity
defect. If the foreign atom or ion occupies voids in the structure, the defect is called interstitial impurity.

Point Defects: These are caused by a departure from the periodic arrangement in the vicinity of an atom or a
group of atoms. Point defects in crystals may be classified as:
(a) Defects in stoichiometric solids, also called as stoichiometric defects.
(b) Defects in non-stoichiometric solids, also called as non-stoichiometric defects.
(c) Impurity defects.
Chem i str y | 19.19

Defects in Stoichiometric Solids: Stoichiometric solids are those in which the number of positive and negative ions
are exactly in the ratio indicated by their chemical formulae. These are of 2 types (i) Schottky defect, (ii) Frenkel defect
(i) Schottky Defect: A pair of ‘holes’ or vacancies exists in the crystal lattice due to one cation and one anion
missing from the normal lattice sites. As the number of missing cations and anions remain the same, the crystal as
a whole remains the same. This sort of defect occurs in highly ionic compounds with a high coordination number
and where the ions (both cations and anions) are of similar size. Alkali metal halides such as NaCl, KCl, KBr, AgBr
and CsCl show this defect. In NaCl crystal, at room temperature, there will be one Schottky defect per 1016 ions.
Since the presence of large number of Schottky defects in a crystal leads to decrease in the number of ions in the
lattice, this results in lowering the density of the solid markedly.

A+ B+ A+ B+ A+

B+ B+ A+ B+

A+ B+ A+ A+

B+ A+ B+ A+ B+

Figure 19.9-a: Schottky defect in a crystal

(ii) Frenkel Defect: When a ‘hole’ or ‘vacancy’ exists in the crystal lattice because an ion occupies an interstitial
lattice site, it gives rise to Frenkel defect. The crystal remains neutral. The defect occurs more frequently in solids
which have low coordination number and possess ions (cations and anions) of different sizes. Since cations are
generally smaller than anions, it is more common to find the cations occupying the interstitial sites. For example,
in AgBr and ZnS crystals. Ag+ ions and Zn2+ ions are missing from their normal lattice sites and are present in the
interstitial positions. AgBr, AgCl and AgI are special cases which show both Schottky and Frenkel defects.
However, the density of the solid remains the same as the presence of Frenkel defect does not change the number
of ions in the lattice. The defect causes increase in dielectric constant of the crystals as similar charges come closer.

A+ B+ A+ B+ A+
+
A
+
B B+ A+ B+

A+ B+ A+ B+ A+

B+ A+ B+ A+ B+

Figure 19.9-b: Frenkel defect observed in a crystal

Illustration 13: Explain, why ZnO becomes yellow on heating?  (JEE ADVANCED)

Sol: Due to Frenkel defect, when ZnO is heated, it loses oxygen reversibly and turns yellow in colour.
1
ZnO(s) → Zn2+ + O2 + 2e−
2
The Zn2+ ions formed enter the vacant voids in the crystal to form non-stoichiometric solid. The released electrons
are trapped in its neighbourhood. It is therefore, non-stoichiometric ZnO is yellow and shows increased conductivity.

Table 19.7: Difference between frenkel and schottky defects

S.No. Frenkel Defects Schottky Defects


1. The ion occupies an interstitial position than its Both cations and anions are missing from their lattice
actual lattice position. positions in the crystal (vacancy defect).
1 9 . 2 0 | Solid State

2. There is no change in the density of the crystal. The defect decreases the density of the crystal.
3. It occurs in ionic compounds with low It occurs in compounds with high coordination number
coordination number and with a large difference and with ions of similar sizes.
in the size of cations and anions.

Illustration 14: Calculate the concentration of cation vacancies if NaCl is doped with 10–3 mole % of SrCl2.
 (JEE ADVANCED)

Sol: Doping of SrCl2 to NaCl brings in replacement of two Na+ ion by each Sr2+ ion, but each Sr2+ occupies only one
lattice point. This produces one cation vacancy.
Thus, doping of 10–3 moles of SrCl2 in 100 mole NaCl will produce cation vacancies = 10–3 moles.
∴ 100 mole NaCl will have cation vacancies after doping = 10–3 moles.
10−3
∴ 1 mole of NaCl will have cation vacancies = = 10−5
100
∴ Total cationic vacancies = 10–5 × NA = 6.02 × 1018

11. MAGNETIC PROPERTIES OF SOLIDS


On the basis of their magnetic properties, substances can be classified into five categories.

Table 19.8: Magnetic properties of crystals

Sr. Properties Information Magnetic Example Application


No Alignment
1. Diamagnetic Repelled weakly in magnetic Benzene, Insulators
field. Such solids have only NaCl, TiO2,
paired electrons. V2O5, etc.
2. Paramagnetic Have unpaired electrons; O2, VO, CuO, Electronic devices
weakly attracted in magnetic TiO
field. They cannot be
permanently magnetised.
3. Ferromagnetic Also, have unpaired electrons. Fe, Ni, Co, CrO2 is used in audio,
Strongly attracted in magnetic CrO2 video tapes.
field. Such solids can be
permanently magnetised.
On heating to a temperature
called Curie Point, these solids
change to paramagnetic solid.
4. Antiferromagnetic In these solids changes Cr2O3, CoO, Used in the
electrons align themselves Co3O4, Fe2O3, instruments of
in such a way that resultant MnO, MnO2 magnetic susceptibility
magnetic moment is zero. measurement
5. Ferrimagnetic unpaired electrons align Fe3O4 –
themselves in such way
or
that there is a net magnetic
moment.

and so on
Chem i str y | 19.21

Curie Temperature: The temperature at which a ferromagnetic substance loses its ferromagnetism and attains
paramagnetism only is called Curie temperature. For iron, the Curie temperature is 1033 K, for Ni it is 629 K and for
Fe3O4 it is 850 K. Below this temperature, paramagnetic substances behave as ferromagnetic substances.

12. ELECTRICAL PROPERTIES OF SOLIDS


Solids are classified into three groups on the basis of their electrical conductivities:
1. Conductors: These generally include metals. Their conductivity is of the order of 107- 104 ohm–1 m–1.
2. Semiconductors: Those solids which have intermediate conductivities ranging from 10–6 to 104 ohm–2 m–1 are
classified as semiconductors. As the temperature rises there is a rise in conductivity because electrons from the
valence band jump to conduction band.
3. Insulators: These are solids which have very low conductivity values ranging from 10–10 to 10–20 ohm–1 m–1.

Causes of conductance in solids:


In most of the solids, conduction takes place due to migration of electrons under the influence of electric field.
However, in ionic compounds, the movement of the ion is responsible for their conducting behavior. In metals,
conductivity strongly depends upon the number of valence electrons available in an atom.
A band is formed due to closeness of molecular orbitals which are formed from atomic orbital.
If this band is partially filled or it overlaps the unoccupied higher energy conduction band, the electrons can flow
easily under an applied electric field and the solid behaves as conductor.
If the gap between valence band and next higher unoccupied conductions band is large, electrons cannot jump
into it and such a substance behaves as insulator.

Figure 19.10-a: Distinction among metals, insulators and semiconductors

If the gap between the valence band and conduction band is small, some electrons may jump from valence band
to the conduction band. Such a substance show some conductivity and it behaves as a semiconductor [Fig(c)]. An
increase in temperature will increase the electrical conductivity of semiconductors as more electrons can jump
from valence to conduction band. Silicon and germanium show this type of behaviour and are called intrinsic
semiconductors.
Doping: The process of introducing an impurity into semi-conductors to enhance their conductivity is called
doping.
1 9 . 2 2 | Solid State

n-type semiconductor: When silicon or germanium crystal is doped with a group 15 element like P or As, the
dopant atom forms four covalent bonds like a Si or Ge atom but the fifth electron, not used in bonding, becomes
delocalised and contributes its share towards electrical conduction. Thus, silicon or germanium doped with P or As
is called n-type semiconductor, n indicative of negative since it is the electron that conducts electricity.
p-type semiconductor: When silicon or germanium is doped with a group 13 element like B or Al, the dopant
atom forms three covalent bonds like a B or Al atom, but in place of the fourth electron, a hole is created. This hole
moves through the crystal like a positive charge giving rise to electrical conductivity. Thus, Si or Ge doped with B
or Al is called p-type of semiconductor (p stands for positive hole), since it is the positive hole that is responsible
for conduction.
Silicon atom Mobile electron Positive hole
(no electrons)

As B

Perfect crystal n-type p-type


(a) (b)

Figure 19.10-b: Distinction among perfect crytsal, n-type and p-type semiconductor

Diode: When n-type and p-type semiconductors are combined, what results, is known as a diode. These diodes
are used as rectifiers.
Transistors: These are used to detect or amplify radio or audio signals. They consist of pnp or npn Sandwich
semiconductors.
Photodiode: These are diodes which are capable of converting light energy into electrical energy and are used in
solar cells.
Table 19.9: Electrical properties of crystals

Properties Information Dipolar Example Application


Property
1. Piezoelectricity When a crystal of dielectrics is Development of Quartz and –
subjected to mechanical stress, charge in some Rochelle salt
then small magnitude current part of crystal
is produced. It is called direct
Piezoelectric effect.
2. Anti piezoelectricity In some solids, electric field Crystal –
develops mechanical effect. suffers elastic
deformation in
an electric field
3. Ferroelectricity Piezoelectric crystals having BaTiO3, Electromagnetic
permanent dipoles are said to KH2PO4, appliances
possess ferroelectricity Rochelle salt
4. Anti ferroelectricity Piezoelectric crystals with zero PbZrO3, Lead
dipole are said to posses anti zirconate
ferroelectricity.
5. Pyroelectricity Some polar crystals produce – Crystals of Used in fire alarms,
electric impulse on heating. tartaric acid thermostat
Chem i str y | 19.23

13. TYPES OF IONIC STRUCTURES


(a) Rock Salt Structure: (NaCl) Larger atoms form ccp arrangement and smaller atoms fill all octahedral voids.
Na Cl

Na+
Cl-

Rock salt structure


Figure 19.11-a: Arrangement of atoms in rock salt type structure

(b) Zinc Blende (Sphalerite) Structure: (ZnS) Larger atoms form ccp arrangement and smaller atoms fill half of
alternate tetrahedral voids.

S-
2
Zn
+2
Zn

Zinc blende structure

Figure 19.11-b: Arrangement of atoms in Zinc blende type structure

(c) Cesium Halide Structure: (CsCl) Cl– at the corners of cube and Cs+ in the centre.

Cl

Cesium chloride structure

Figure 19.11-c: Arrangement of atoms in cesium chloride type structure

(d) Fluorite Structure: (CaF2) Ca2+ form ccp arrangement and F– fill all tetrahedral voids.

Fluorite structure
Figure 19.11-d: Arrangement of atoms in fluorite type structure
1 9 . 2 4 | Solid State

(e) Antifluorite Structure: (Li2O) O2– ion form ccp and Li+ take all tetrahedral voids.

Figure 19.11-e: Arrangement of atoms in antifluorite type structure

(f) Corundum Structrure: (Al2O3) O2– form hcp and Al3+ fill 2/3 octahedral voids.
(g) Rutile Structure: (TiO2) O2– form hcp while Ti4+ ions occupy half of the octahedral voids.
(h) Pervoskite Structure: (CaTiO3) Ca2+ in the corner of cube, O2– at the face centre and Ti4+ at the centre of cube.

Figure 19.11-f: Arrangement of atoms in pervoskite type structure


Table 19.10: Summary of some characteristic of some closest and non-closest packed structure

S. Formula Type of Type of Radius Packing factor (PF) TV occupied OV occupied Ligancy
Z eff
No. lattice structure

1. AB Rock salt fcc 4 None All by Na+ 6:6


 4 2  4 2
(NaCl)* π  4 × 3 π rA °   4 × 3 π rB°  ions
(rA° + rB° ) = PF =   
2
a2

2. ZnS (Zinc ccp 4 Half by Zn2+ None 4:4


 4 2  4 2
blende) ions (Alternate
3  4 × 3 π rA °   4 × 3 π rB° 
 
(rA° + rB° ) = a PF =  TV)
4 a2

3. CaCl hcp 1 None Only one by 8:8


 4 2  4 2 
2 (rA ° + rB° ) =3a 1× 3 π rA °  1× 3 π rB°  Cs+
PF =   
2
a

4. AB2 CaF2 * ccp 4 All by Na+ ions None 8:4


 4 2  4 2 
3  4 × 3 π rA °  8 × 3 π rB° 
 
(rA° + rB° ) = a PF = 
4 a2

Note: Here radius rule will change. Now r/r lies in 0.225 – 0.414 as anion lies in TV. 4:8

5. A2B Na2O ccp 4 All by Na+ ions None 4:8


 4 2  4 2
(Antifuorite)
3 8 × 3 π rA °   4 × 3 π rB° 
   
(rA° + rB° ) = a PF =
4 a2
4 2
8× πr °
3
=
PF 3 A= =
π 0.34
a2 16
6. Diamond fcp 8 4 2 4 more C – None 4
Cubic (DC) 8× πr ° atoms are
3 3
r= a =
PF 3 A=
2
=
π 0.34 present in
a 16
8 alternate TVs
2
Height c = 4 r
3
Chem i str y | 19.25
S. Formula Type of Type of Radius Packing factor (PF) TV occupied OV occupied Ligancy
Z eff
No. lattice structure

7. Hexagonal hcp 6 2r = 4 - - -
 4 3
closed  6 × πr 
packed 3 π
=
PF  =  = 0.74
(AB AB … 2 3
Height (c) = 4r 24 2 r 3 2
packing) 3
1 9 . 2 6 | Solid State

Base area,  4 3
 6 × πr 
3 π
=
PF  =  = 0.74
3
24 2 r 3 2
A = 6 3 r2
Volume = 24 2 r3

8. AB2O4 Spinel fcc 4 3  4 3   4 3   4 3  -


Volume
= a= 16 2 r3 2−
O 2 × 3 πrAl3+  + 1× 3 πrZn2 +  +  4 × 3 πrO2 −  1 1
PF =       Zn2 + = TV Al3+ = OV
( ZnAl2O4 ) Al3+ = 2 For TV, 8 2
16 2 r 3 2 −
O
Zn3+ = 1 r
2− Zn2 +  3  2
Q =4 = 0.225  r  Al3+ = OV
r =
π  1  rAl3+  + 1  Al3+  + 1 3
O2 −   4  r 2−  
3 2  2  rO2 −   O  
For OV,  
r π 1 3 1 3 
Al3+ =  ( 0.414 ) + =( 0.225) + 1 0.77
= 0.414 3 2 2 4 
r
O2 −

9. A2O3 Correndum Hcp 6  4 3   4 3  6:4


Volume of hcp = 24 2 r 3
6 × 3 πrO2 −  +  4 × 3 πrAl3+  2
( Al2O3 ) r PF =     Al3+ = OV
O2 − = 6 For OV, ⊕ = 0.414 3
rΘ 24 2 r 3 2 −
O
Al3+ = 4
 
3
π  2  rAl3+  
= 1 + 

3 2  3  rO2 −  
 
3.1416  2 3
= 1 + ( 0.414 )  = 0.7756
3 × 1.414  3 
Chem i str y | 19.27

POINTS TO REMEMBER
Radius Ratio: In a given coordination number,
C.N r+/r-
radius ratio has fixed value in the limiting case:
3 0.155
4(Td) 0.225
4(SP) 0.414
6 0.414
8 0.732

Packing fraction [φ] : It is the maximum fractional Typing of unit cells Packing fractions
volume of unit cell which can be occupied by Simple cubic 0.53
atoms or ions. In case of atomic solids of like
atoms, following are the values of packing Body centred cubic 0.68
fractions Fcc 0.74
Hcp 0.74

Relationship between radius and sides of


SC A=2r
unit cells in atomic solids (a = edge length,
r = atomic radius): Bcc
3a = 4r
Fcc
2a = 4r
Hcp a=2r
 2
h = 4 r
 3
 

Bravias lattices: Based on the arrangement of atoms or ions, the seven


crystal systems are further classified into 14 types known as
the bravias lattices.

Primitive cell: A primitive cell is a bravias lattice that homes only one atom
(effectively) in one unit cell.

Density of solid [ρ] : Massof atoms / ions in one unit cell


ρ=
volume of an unit cell

Cubic System : NM  1  −3
ρ=  3  gcm
NA  a 
N=number of atoms per unit cell,
M=molar mass,
NA =Avogadro’s number
a3=volume of unit cell in cm3
1 9 . 2 8 | Solid State

Tetrahedral and Octahedral void (Holes) : (i) Number of tetrahedral voids=2 × effective number of
atoms/unit cell.
(ii) Number of octahedral voids =Number of atoms
(effectively) per unit cell.

B
The interstities below B
is tetrahedral void
x is the central of
octahedral void

Crystal defects : A) Point Defects\Stoichiometric defects


Two types:-
(i) Schottky Defect –Atoms or ions absent from their
normal lattice sites. Observed in NaF, KCl etc.
(ii) Frenkel Defect- Smaller cation leaves its normal lattice,
occupies the interstitial site. Observed in solids with smaller
cations like LiI.
+ - + - + - + - + - + -
- + + - + - + - + - +
+
+ - - + - + - + - + -
- + - + - + - + - + - +
Schottky defect Frenkel defect

B) F-center defect-Alkali metal vapours passed on an


ionic solid lets out electrons which substitute some anions
leading to trapped electrons.

Based on the geometry of unit cells, solids are divided into seven crystal systems as:

System Parameters Interaxial angle

Triclinic a≠b≠c α≠β≠γ

Monoclinic a≠b≠c α = γ = β ≠ 90ο

Orthorhombic a≠b≠c α = β = γ = 90ο

Tetragonal a= b ≠ c α = β = γ = 90ο

Cubic a=b=c α = β = γ = 90ο

Hexagonal a= b ≠ c α = β = 90ο ≠ γ = 120ο

Rhombohedral (or) (Trigonal) a=b=c α = β = γ ≠ 90ο


Chem i str y | 19.29

Solved Examples

JEE Main/Boards = 400 × 10–12 m


(1 pm = 10–12 m)
Example 1: Both diamond and rhombic (solid) sulphur
= 400 × 10–10 cm
are covalent solids, however, the latter has very low
m.pt. in comparison to diamond. Explain. Volume of unit cell
= (400 × 10–10 cm)3
Sol: Value of Boiling point and melting point depends
on intermolecular force of attraction. By taking into = 64 × 10–24 cm3
consideration the forces that help the two structure
Number of atoms (n) in the fcc unit cell = 4
(diamond and sulphur) to hold together, answer the
question. 4 × 60
∴ Density =
Diamond has three dimensional covalent network and 6.023 × 1023 × 64 × 10−24
strong intermolecular forces whereas rhombic sulphur   = 6.23 g cm–3
has one dimensional covalent network with puckered
eight membered ring held together by weak van der
Waals’ forces. Example 4: The figures given below show the location
of atoms is three crystallographic planes in a fcc lattice.
Draw the unit cell for the corresponding structure and
Example 2: Calculate the number (n) of atoms contained identify these planes in your diagram.
within (a) simple cubic cell (b) a body centred cubic cell
(c) a face centred cubic cell.

Sol: (a) The simple cubic unit cell has 8 atoms at eight
corners. Each atom is shared by 8 unit cells.
1 (i) (ii)
∴ n =8 × =1
8
(b) The body centred cubic cell consists of 8 atoms at
the corners and one atom at centre.
 1 (iii)
∴ n = 8 ×  + 1 = 2
 8
Sol: (i) Face plane,
(c) The face centred cubic unit cell consists of 8 atoms at
(ii) Face diagonal plane and
the eight corners and one atom at each of the six faces.
This atom at the face is shared by two unit cells. (iii) Diagonal plane
1  1
∴ n =8× + 6 ×  = 4
8  2 Example 5: Ferric oxide crystallizes in a hexagonal
closed packed array of oxide ions, with two out of every
Example 3: An element having atomic mass 60 has face three octahedral holes occupied by ferric ions. Derive
centred cubic unit cells. The edge length of the unit cell the formula of the ferric oxide.
is 400 pm. Find out density of the element.
Sol: Hexagonal close packing has an array of oxide ions,
Sol: From the edge length determine the volume of unit where two out of three octahedral holes occupied by Fe3+
cell as, Volume =a3 ions has one octahedral void or holes corresponding to
each atom constituting the close packing. In iron oxide
Density can be determined using the following
only 2/3 of octahedral holes are occupied by Fe3+ ions.
expression that relates density with edge length.
It means corresponding to each oxide ion, there are 2/3
4 × Mw
ρ= Fe3+ ions. Thus, molecular formula of iron oxide is Fe2O3.
NA × a3 × 10−30
Edge length of unit cell = 400 pm
1 9 . 3 0 | Solid State

Example 6: The structure of CsCl is different from NaCl, Area (of base) of unit cell in hcp
though both have the similar formula. Explain.
= 6 3r 2 (Follow text)
Sol: CsCl has radius ratio ≥ 0.732 and shows cubic Height of unit cell in hcp
arrangement, whereas radius of NaCl is < 0.732 and has 2
octahedral arrangement. = 4r × (follow text)
3
6 × 4 πr 3
Example 7: Calculate the wavelength of X-rays, which ∴ Packing fraction =
2
produces a different angle 2θ equal to 16.80º for a 3 × 6 3r 2 × 4r ×
3
crystal. Assume first order diffraction with inter particle π
distance in crystal of 0.2 nm. = = 0.74 or 74%
3 2
Sol: using Bragg’s equation, =
nλ 2dsin θ Thus, empty space in hcp =
1 − 0.74 =
0.26 or 26%
Calculate the Wavelength.
Given, n = 1, d = 0.2 × 10–9 m,
JEE Advanced/Boards
16.80
=θ = 8.40º
2 Example 1: Metallic gold crystallizes in the face-
2 × 0.2 × 10 −9 sin8.4 centred cubic lattice. The length of the cubic unit cell
Thus, λ= = 5.84 × 10–11 m is a = 4.070 Å.
1
(a) What is the closest distance between gold atoms?
Example 8: Explain, why solids with F-centres are
(b) How many ‘nearest neighbours’ does each gold
paramagnetic?
atom have at the distance calculated in (a)?
Sol: Solids containing F-centres are paramagnetic (c) What is the density of gold?
because the electrons occupying the holes are unpaired.
(d) Prove that the packing factor for gold, the fraction
of the total volume occupied by the atoms themselves,
Example 9: A fcc lattice cube is formed by atoms A and is 0.74.
B. If atom A is present at the corner of the cube and the
atom B at the faces of the cube. Find out the formula of Sol: (a) In fcc, a = 2 2 r
the compound. a
∴ 2r =
Sol: Contribution of atom ‘A’ at eight corners of the 2
cube 2r is closest distance between two atom.
1 4.070
= × 8 = 1 atom ∴ 2r
= = 2.878 Å.
8 1.414
1
Contribution of atom at each face = atom D
2
E

The atom ‘B’ at six faces of the cube a G B
1 F
= × 6 = 3 atom
2 C A
∴ Formula of the compound = AB3
a
a
Example 10: A metallic element crystallizes into a lattice
containing a sequence of layers of ABABAB ............ Any (b) The problem is to find how many face centers are
packing of spheres leaves out voids in the lattice. What equidistant from a corner atom. Point A in figure may
percentage by volume of this lattice is empty space? be taken as the reference corner atom. In that same
4 3 figure, B is one of the face-center points at the nearest
6× πr distance to A. In plane ABD in the figure, there are
Sol: Packing fraction in hcp = 3
Volume three other points equally close to A: the centers of the
6 × 4 πr 3 squares in the upper right, lower left, and lower right
=
3 × Area × Height quadrants of the plane, measured around A. Plane ACE,
Chem i str y | 19.31

parallel to the plane of the paper, also has points in the Dividing dFCC and dBCC
centers of each of the squares in the four quadrants
around A. Also, plane ACF, perpendicular to the plane dFCC 4 (3)3
= × = 2 × 0.6297 = 1.259
of the paper, has points in the centers of each of the dBCC 2 (3.5)3
squares in the four quadrants around A. Thus there are
12 nearest neighbors in all, the number expected for a Example 3: You are given some marbles with a diameter
close-packed structure. of 10mm. They are to be placed such that their centres
The same result would have been obtained by counting are lying in a square bond by four lines each of length
the nearest neighbours around B, a face-centered point. 40 mm. What will be the arrangements of marbles in
a plane so that maximum number of marbles can be
(c) In fcc, n = 4, M = 197 g mol-1 placed inside the area? Sketch the diagram and derive
a = 4.070 Å = 4.070 × 10-10 cm expression for the number of molecules per unit area.

n×M 4 × 197 Sol: Area of square having spherical marbles in it


density = =
( )
3 3
NA .a 6.022 × 1023 × 4.07 × 10−10 40mm
A B
= 19.4 g cm-3
(d) Since atoms at closest distance are in contact in a
close-packed structure, the closest distance between
a
centres calculated in (a), , must equal the sum of
2
the radii of the two spherical atoms, 2r. Thus, r = a/23/2. = 40 × 10−1 × 40 × 10−1 cm2
From (c), there are 4 gold atoms per unit cell. D
= 16 cm2 A B
4 

mm
= 4  πr3 

10
Then, Volume of 4 gold atoms

mm
3

10

∴ Volume of unit cell = a3 C


( ) 3
3
= 2 2 r CD 10 sin60º
= = 10 = 5 3
2
= 16 2 r 3 The maximum number of spheres of 10 mm diameter in
volume of 4 gold atoms hcp packing can be seen in figure.
∴ Packing fraction =
volume of unit cell Total length converted by spheres
16 = 5 + 4 × CD = 5 + 4 × 10sin60º
π r3
3 π
= = = 0.74
3
= 5 + 4 × 5 3 = 40mm = 4 cm
16 2 r 3 2
Maximum number of spheres (s)
Example 2: A metal crystalizes into two cubic phases, = 14 + 8

face centred cubic (FCC) and body centred cubic (BCC), (Full) (half)
whose unit cell lengths are 3.5 and 3.0 Å, respectively. = 14 + 4 = 18
Calculate the ratio of densities of FCC and BCC. 18
∴ Number of spheres per cm=
2
= 1.125
16
Sol: We know that
Example 4: Using the given data find out the type of
Z × Formula mass of substance
ρ= cubic lattice to which the crystal belong:
N0 × a3
Since for FCC, Z = 4 and for BCC, Z = 2 Fe V Pd Al
a in pm 286 301 388 405
4 × Formula mass of metal
dFCC = ρ in g cm–3 7.86 5.96 12.16 2.70
N0 × (3.5)3
At. mass in 55.85 50.94 106.4 26.98
4 × Formula mass of metal g mol–1
and dBCC =
N0 × (3)3
1 9 . 3 2 | Solid State

n × At.mass Number of LiF molecules per mole


Sol: We know, ρ = 3
N0 × a (i.e., Avogadro constant)
3
ρ × N0 × a 1.201 × 1024
or n = = = 6.01 × 1023
At.mass 2
For Fe:
7.86 × 6.023 × 1023 × (286 × 10−10 )3 Example 6: Calculate the glancing angle on the
n= =2 cube (100) of a rock salt (a = 2.814 Å) corresponding
55.85
to second order diffraction maximum for X-rays of
(i.e., cubic lattice of Fe is body centred) wavelength 0.710 Å.
For V:
Sol: First determine the distance between lattice plane
5.96 × 6.023 × 1023 × (301 × 10−10 )3 a
n= =2 by using the following expression, dhkl =
50.94
h2 + k 2 + l2
(i.e., cubic lattice of V is body centred)
And then θ angle can be calculated using Bragg’s
For Pd: equation
12.16 × 6.023 × 1023 × (388 × 10−10 )3 2d sin θ = nλ  ...(i)
n= =4
106.4
The distance between consecutive lattice planes is
(i.e., cubic lattice of Pd is face centred) defined by Miller indices (h, k, l) in a cubic lattice by
For Al: the relation
a
2.70 × 6.023 × 1023 × (405 × 10−10 )3 dhkl = = 2.814 Å, h=1, k=0, l=0
n= =4 h + k 2 + l2
2
26.98
2.814 Å
(i.e., cubic lattice of Al is face centred) ∴ d100
= = 2.814 Å = 2.814 × 10–8 cm
12 + 02 + 02
Example 5: The density of a particular crystal of LiF is From equation (i)
2.65 g/cc. X-ray analysis shows that Li+ and F– ions are 2 × 2.814 × 10−8 sin θ = 2 × 0.710 × 10−8
arranged in a cubic array at a spacing of 2.01 Å. From
these data calculate the apparent Avogadro constant. 2 × 0.710 × 10−8
sin θ = = 0.2533
[Li = 6.939, F = 18.998 (1Å = 10–8 cm)] 2 × 2.814 × 10−8
sin θ =0.2533
Sol: In this problem, we have to calculate Avogadro
constant, i.e., the number of LiF molecules percent in 1 ∴ θ =sin−1 (0.2533)
mole of LiF weighs 25.937 g or θ =14º 36' 40"
(LiF = 6.939 + 18.998),
wt. of 1 mole Example 7: The edge length of unit cell of a metal
Volume of 1 mole =
wt. per cc (density) having molecular weight 75 g mol–1 is 5 Å which
25.937 crystallizes in body centred cubic lattice. If density is 2
= = 9.78 cc.
2.65 g cm–3, calculate radius of metal atom.
Since this volume is supposed to be a cube, the length n × Mol.wt.
of each edge of the cube Sol: Density =
V × NA
3
= 9.78 2.138 cm.
= 2 × 75
2=
Number of ions present in one edge a × 6.023 × 1023
3

2.138 (n = 2 for bcc)


= = 1.063 × 108
2.01 × 10−8 ∴ a= 5 × 10−8 cm3
Number of ions (Li + F ) present in cube
+ –
3 3
 r= × a= × 5 × 10 −8
= (1.063 × 108 )3 = 1.201 × 1024 4 4
= 2.16 × 10–8 cm = 216 pm
Chem i str y | 19.33

Example 8: A solid compound contains X, Y and Z = a 5.64 × 10−8 cm


atoms in a cubic lattice with X atoms occupying the
corners, Y atoms in the body centred position and Z Molar mass 58.8
Molar
= volume =
atom at the centre of the face of the unit cell. What is Density 2.165
the empirical formula of the compound? 1/3
 58.8 
Edge length
= (a) =  3 cm
(A) XY2Z3 (B) XYZ3  2.165 
(C) X2Y2Z3 (D) X8YZ6 (E) XYZ
Example 11: In a solid AB having the NaCl structure, A
1 atoms occupy the corners of the cubic unit cell. If all the
Sol: (B) Number of atoms of X in a unit cell = × 8 = 1
8 face centred atoms along one of the axis are removed.
Number of atoms of Y in a unit cell = 1 what would be the resultant stoichiometry of solid?
1 Sol: AB has NaCl structure, i.e., 8 corner and 6 face
Number of atoms of Z in a unit cell = ×6 = 3
2 centred atoms. If we remove face centred atoms of one
Empirical formula of the compound = XYZ3 axis, two face centred atoms are removed. Thus, A is at
8 corners and B is at four faces.
Example 9: When an electron in an excited state of Mo 8
∴ Contribution of A = =1
atom falls from L to K-shell, an X-ray is emitted. These 8
X-rays are diffracted at angle of 7.75º by planes with a 4
separation of 2.64 Å. What is the difference in energy Contribution of B = = 2
2
between K-shell and L-shell in Mo, assuming a first
order diffraction? (sin 7.75º = 0.1349) ∴ Stoichiometry of resultant solid is AB2.

Sol: Using Bragg’s equation first determine the


wavelength and then energy can be calculated as
hc
E=
λ
=nλ 2dsin θ
1 × λ = 2 × 2.64 sin7.75º
=
2 × 2.64 × 0.1349
   λ =0.712 Å
Energy difference between K and L-shell of Mo
hc 6.626 × 10−34 × 3 × 108
= = = 2.791 × 10−15 J
λ −10
0.712 × 10

Example 10: The density of crystalline sodium chloride


is 2.165 g cm–3. What is the edge length of the unit cell?
What would be the dimensions of cube containing one
mole of NaCl?
N M
Sol: We know that, ρ = 3  
a  NA 
Where, ρ = density = 2.165 g cm–3
M = molar mass = 58.5
NA = Avogadro’s number = 6.023 × 1023
N = number of formula unit
per unit cell = 4 (for bcc)
N M 
a3 =   = 4  58.5 
 = 1.794 × 10
–22
ρ  NA  2.165  6.023 × 10 
23
1 9 . 3 4 | Solid State

JEE Main/Boards

Exercise 1 Q.10 An element crystallizes in fcc structure. 200 g of


this element has 4.12 × 1024 atoms. The density of A is
Q.1 Explain each of the following with a suitable 7.2 g cm–3. Calculate the edge length of the unit cell.
example:
Q.11 Lithium metal crystal has body-centred cubic
(i) Paramagnetism. structure. Its density is 0.53 g cm–3 and its molecular
(ii) Frenkel defect in crystals. mass is 6.94 g mol–1. Calculate the volume of a unit cell
of lithium metal.=[NA 6.023 × 1023 mol−1 ]
Q.2 State the difference between Schottky and Frenkel
defects. Which of the two changes the density of the Q.12 If NaCl crystals are doped with 2 × 10−3 mol per
solid? cent of SrCl2, calculate the cation vacancies per mole.

Q.3 (a) What is meant by anisotropy? Q.13 An element exists in bcc structure with a cell
edge of 288 pm. If the density of the element is 7.2
(b) Give a sketch of arrangement of particles in a solid g cm–3, what is the atomic mass of the element?
which would show anisotropy. [NA 6.023 × 1023 mol−1 ]
=

Q.4 (a) Explain with the help of figure square close- Q.14 An element A crystallises in fcc structure, 208 g
packing and hexagonal close-packing of particles in a of this element has 4.283 × 1024 atoms. If edge length
solid in two dimensions. of the unit cell of this element is 408 pm, calculate its
(b) What is the coordination number of a particle in the density.
two packings?
Q.15 Copper crystallises in face-centred cubic lattice
Q.5 Some crystal defects are shown in the figures and has a density of 8.930 g cm−3 at 239 K. Calculate
below: the radius of Copper atom. [At. mass of Cu = 63.55 u,
Avogadro’s constant =NA 6.02 × 1023 mol–1]

Q.16 Metallic gold crystallises in a fcc lattice and has a


density of 19.3 g cm–3. Calculate the radius of gold atom.
Indicate the types of defect shown by (a) and (b). [At. mass of gold = 197 u,= NA 6.023 × 1023 mol–1]

Q.6 A compound is formed by two elements M and Q.17 A substance forms face-centred cubic crystals. If
N. The element N forms ccp and atoms of M occupy the edge length of the unit cell is 630 pm and density of
1/3rd of tetrahedral voids. What is the formula of the the substance is 1.984 g cm–3, calculate the molar mass
compound? of substance.

Q.7 An element with molar mass 2.7 × 10−2 kg mol–1 Q.18 In a solid AB, having the NaCl structure, B atoms
forms a cubic unit cell with edge length 405 pm. If its occupy the corners of the cubic unit cell. If all the face
density is 2.7 × 103 kg m–3, what is the nature of the centred atoms along one of the axes are removed, then
cubic unit cell? what is the resulting stoichiometry of the solid?

Q.8 An element occurs in bcc structure with cell edge Q.19 Lithium boron hydride crystallizes in an
300 pm. The density of the element is 5.2 g cm–3. How orthorhombic system with 4 molecules per unit cell. The
many atoms of the element does 200 g of the element unit cell dimensions are a = 6.8 Å, b = 4.4 Å and c = 7.2
contain? Å. If the molar mass is 21.76, calculate density of crystal.

Q.9 The density of chromium metals is 7.2 g cm–3. If the Q.20 The simple cubic lattice consists of eight identical
unit cell has edge length of 289 pm, determine the type of spheres of radius R in contact, placed at the corners of
unit cell.[Atomic mass of Cr = 52 u; NA = 6.02 × 1023 mol–1] a cube, what is the volume of the cubical box that will
just enclose these eight spheres and what fraction of
this volume is actually occupied by the spheres?
Chem i str y | 19.35

Q.21 In an ionic oxide, oxide ions are arranged in hcp Exercise 2


array and positive ion occupy two thirds of octahedral
void. Suggest the simplest formula assuming metal as M. Single Correct Choice Type

Q.22 A compound containing Zn, Al and S crystallises Q.1 A solid has a structure in which W atoms are located
with a closed packed array of sulphide ions. Zinc ions at the corners of a cubic lattice, O atom at the centre
are found in one-eighth of the tetrahedral holes and of the edges and Na atom at centre of the cubic. The
aluminium ions in one half of the octahedral holes. formula for the compound is:
What is the empirical formula of the compound? (A) NaWO2 (B) NaWO3 (C) Na2WO3 (D) NaWO4

Q.23 What is the percent by mass of titanium in rutile, a


Q.2 The coordination number of cation and anion in
mineral that contains Ti and oxygen? If structure can be
Fluorite CaF2 and CsCl are respectively
described as a closest packed array of oxide ions with
titanium ions in a one-half of the octahedral holes? (A) 8: 4 and 6: 3 (B) 6: 3 and 4: 4
What is the oxidation number of titanium? (Ti = 48) (C) 8: 4 and 8: 8 (D) 4: 2 and 2: 4

Q.24 Calculate packing efficiency in ccp structure. How Q.3 The interstitial hole is called tetrahedral because
does it differ from packing efficiency in hcp structure?
(A) It is formed by four spheres.
Q.25 A strong current of trivalent gaseous boron passed (B) Partly same and partly different.
through a germanium crystal decreases the density (C) It is formed by four spheres the centres of which
of the crystal due to part replacement of germanium form a regular tetrahedron.
by boron and due to interstitial vacancies created by (D) None of the above three.
missing Ge atoms. In one such experiment, one gram
of germanium is taken and the boron atoms are found
to be 150 ppm by weight, when the density of the Ge Q.4 The mass of a unit cell of CsCl corresponds to
crystal decreases by 4%. Calculate the percentage of (A) 1 Cs+ and 1 Cl– (B) 1 Cs+ and 6 Cl–
missing vacancies due to germanium, which are filled
(C) 4 Cs+ and 4 Cl– (D) 8 Cs+ and 1 Cl–
up by boron atoms. [Atomic wt. Ge = 72.6, B = 11]

Q.26 What is a semiconductor? Describe two main Q.5 Which one of the following schemes of ordering
types of semiconductors and explain mechanism for closed packed sheets of equal sized spheres do not
their conduction. generates close packed lattice.
(A) ABCABC (B) ABACABAC
Q.27 (a) The electrical conductivity of a metal decreases
with rise in temperature while that of a semiconductor (C) ABBAABBA (D) ABCBCABCBC
increases. Explain.
Q.6 An ionic compound AB has ZnS type structure. If
(b) The ions of NaF and MgO have the same number of
the radius A+ is 22.5 pm, then the ideal radius of B–
electrons and internuclear distances are about the same
would be
[235 pm and 215 pm]. Why are then the melting points
of NaF and MgO so different [992ºC and 2642ºC]? (A) 54.35 pm (B) 100 pm
(c) Why does zinc oxide exhibit enhanced electrical (C) 145.16 pm (D) None of these
conductivity on heating?
Q.7 The tetrahedral voids formed by ccp arrangement
Q.28 Bring out with the help of a sketch the difference of Cl– ions in rock salt structure are
between metals insulators and semiconductors in terms of
energy gap between valence band and conduction band. (A) Occupied by Na+ ions
(B) Occupied by Cl– ions
Q.29 Explain with the help of a figure, how an electron (C) Occupied by either Na+ or Cl– ions
(i) moving in an orbital and (ii) spinning in an atom give
(D) Vacant
magnetic properties.

Q.30 How can you show with the help of a diagram that Q.8 The number of nearest neighbours around each
number of octahedral voids is equal to the number of particle in a face-centred cubic lattice is
atoms in cubic close packed structure? (A) 4 (B) 6 (C) 8 (D) 12
1 9 . 3 6 | Solid State

Q.9 If the anions (A) from hexagonal closest packing Q.5 The packing efficiency of the two-dimensional
and cations (C) occupy only 2/3 octahedral voids in it, square unit cell shown below is (2010)
then the general formula of the compound is
(A) CA (B) CA2 (C) C2A3 (D) C3A2

Q.10 A solid is formed and it has three types of atoms


X, Y, Z. X forms a FCC lattice with Y atoms occupying all l
the tetrahedral voids and Z atoms occupying half the
octahedral voids. The formula of the solid is (A) 39.27% (B) 68.02% (C) 74.05% (D) 78.54%
(A) X2Y4Z (B) XY2Z4 (C) X4Y2Z (D) X4YZ2
Q.6 Statement-I: In any ionic solid (MX) with Schottky
Q.11 The intermetallic compound LiAg crystallizes defects, the number of positive and negative ions are
in cubic lattice in which both lithium and silver have same.
coordination number of eight. The crystal class is Statement-II: Equal number of cation and anion
(A) Simple cubic (B) Body centred cubic vacancies are present. (2001)
(C) Face centred cubic (D) None of these (A) Statement-I is true, statement-II is true, statement-II
is the correct explanation of statement-I.
Q.12 NH4Cl cyrstallizes in a body-centred cubic type (B) Statement I is true, statement-II is true, statement-II
lattice with a unit cell edge length of 387 pm. The distance is not the correct explanation of statement-I.
between the oppositely charged ions in the lattice is (C) Statement-I is true, statement-II is false.

(A) 335.1 pm (B) 83.77 pm (D) Statement-I is false, statement-II is true.


(C) 274.46 pm (D) 137.33 pm
Q.7 Sodium crystallizes in a bcc cubic lattice with the
cell edge, a = 4.29 Å. What is the radius of sodium
Previous Years’ Questions atom? (1994)

Q.1 The coordination number of a metal crystallizing in Q.8 A metallic element crystallizes into a lattice containing
a hexagonal close-packed structure is (1999) a sequence of layers of ABABAB ......... Any packing of
(A) 12 (B) 4 (C) 8 (D) 6 layers leaves out voids in the lattice. What percentage by
volume of this lattice is empty space? (1996)
Q.2 In a solid ‘AB’ having the NaCl structure, ‘A’ atoms
occupy the corners of the cubic unit cell. If all the face- Q.9 Chromium metal crystallizes with a body centred
centred atoms along one of the axes are removed, then cubic lattice. The length of the unit edge is found to be
the resultant stoichiometry of the solid is  (2001) 287 pm. Calculate the atomic radius. What would be
(A) AB2 (B) A2B (C) A4B3 (D) A3B4 the density of chromium in g/cm3? (1997)

Q.3 A substance AxBy crystallizes in a face centred Q.10 A metal crystallizes into two cubic phases, face
cubic (fcc) lattice in which atoms ‘A’ occupy each corner centred cubic (fcc) and body centred cubic (bcc),
of the cube and atom ‘B’ occupy the centres of each whose unit cell lengths are 3.5 and 3.0 A⁰, respectively.
face of the cube. Identify the correct composition of the Calculate the ratio of densities of fcc and bcc.  (1999)
substance AxBy. (2002)
(A) AB3 (B) A4B3 Q.11 The figures given below show the location of
atoms in three crystallographic planes in a fcc lattice.
(C) A3B (D) Composition cannot be specified
Draw the unit cell for the corresponding structures and
identify these planes in your diagram. (2000)
Q.4 Which of the following fcc structure contains
cations in alternate tetrahedral voids? (2005)
(A) NaCl (B) ZnS (C) Na2O (D) CaF2
Chem i str y | 19.37

Q.12 In a compound atoms of element Y from ccp lattice Q.16 In a face centred cubic lattice, atom A occupies the
and those of element X occupy 2/3rd of tetrahedral corner positions and atom B occupies the face centre
voids. The formula of the compound will be  (2008) positions. If one atom of B is missing from one of the face
centred points, the formula of the compound is:  (2011)
(A) X 4 Y3 (B) X 2 Y3 (C) X 2Y (D) X 3 Y4
(A) AB2 (B) A 2B3 (C) A 2B5 (D) A 2B
Q.13 The edge length of a face centered cubic cell of an
ionic substance is 508 pm. If the radius of the cation is Q.17 Lithium forms body centred cubic structure. The
110 pm, the radius of the anion is  (2010) length of the side of its unit cell is 351 pm. Atomic
radius of the lithium will be: (2012)
(A) 288 pm (B) 398 pm (C) 618 pm (D) 144 pm
(A) 75 pm (B) 300 pm (C) 240 pm (D) 152 pm
Q.14 Copper crystallizes in fcc with a unit cell length of
361 pm. What is the radius of copper atom (2009) Q.18 CsCl crystallises in body centred cubic lattice.
If ‘a’ is its edge length then which of the following
(A) 108 pm (B) 127pm (C) 157pm (D) 181pm expressions is correct?  (2014)
3a
(A) rCs+ + rCl− =
3a (B) rCs+ + rCl− =
Q.15 Percentage of free space in cubic close packed 2
structure and in body centred packed structure are 3
respectively  (2010) (C) rCs+ + rCl− = a (D) rCs+ + rCl− =3a
2
(A) 30% and 26% (B) 26% and 32%
(C) 32% and 48% (D) 48% and 26% Q.19 Sodium metal crystallizes in a body centred cubic
lattice with a unit cell edge of 4.29 Å. The radius of
sodium atom is approximately:  (2015)
(A) 1.86 Å (B) 3.22 Å (C) 5.72 Å (D) 0.93 Å

JEE Advanced/Boards

Q.5 Spinel is a important class of oxides consisting of


Exercise 1
two types of metal ions with the oxide ions arranged
Q.1 A cubic solid is made up of two elements A and in ccp pattern. The normal spinel has one-eighth of
B. Atoms B are at the corners of the cube and A at the the tetrahedral holes occupied by one type of metal
body centre. What is the formula of compound? ion and one half of the octahedral hole occupied by
another type of metal ion. Such a spinel is formed by
Q.2 A compound alloy of gold and copper crystallizes Zn2+, Al3+ and O2–, with Zn2+ in the tetrahedral holes.
in a cubic lattice in which gold occupies the lattice Give the formulae of the spinel.
points at the corners of the cube and copper atom
occupies the centres of each of the cube faces. What is 
Q.6 KF crystallizes in the NaCl type structure. If the
the formula of this compound? radius of K+ ions 132 pm and that of F– ion is 135 pm,
what is the shortest K- F distance? What is the edge
Q.3 A cubic solid is made by atoms A forming close pack length of the unit cell? What is the closet K-K distance?
arrangement, B occupying one fourth of tetrahedral
void and C occupying half of the octahedral voids. Q.7 A closed packed structure of uniform spheres has
What is the formula of compound? the edge length of 534 pm. Calculate the radius of
sphere, if it exist in
Q.4 What is the percent by mass of titanium in rutile, a (a) simple cubic lattice
mineral that contain Titanium and oxygen, if structure
can be described as a close packed array of oxide ions, (b) BCC lattice
with titanium in one half of the octahedral holes. What (c) FCC lattice
is the oxidation number of titanium?
1 9 . 3 8 | Solid State

Q.8 Calculate the density of diamond from the fact Q.20 If the length of the body diagonal for CsCl which
that it has face centered cubic structure with two atoms crystallises into a cubic structure with Cl– ions at the
per lattice point and unit cell edge length 3.569 Å. corners and Cs+ ions at the centre of the unit cells is 7 Å
and the radius of the Cs+ ion is 1.69 Å, what is the radii
Q.9 An element crystallizes into a structure which may be of Cl– ion?
described by a cubic type of unit cell having one atom on
each corner of the cube and two atoms on one of its body Q.21 Iron has body centered cubic lattice structure.
diagonals. If the volume of this unit cell is 24 × 10−24 cm3 The edge length of the unit cell is found to be 286 pm.
and density of element is 7.2 g cm–3, calculate the number What is the radius of an iron atom?
of atoms present in 200 g of element.
Q.22 Cesium chloride forms a body centered cubic
Q.10 Silver has an atomic radius of 144 pm and the lattice. Cesium and chloride ions are in contact along
density of silver is 10.6 g cm–3. To which type of cubic the body diagonal of the unit cell. The length of the
crystal, silver belongs? side of the unit cell is 412 pm and Cl– ion has a radius of
181 pm. Calculate the radius of Cs+ ion.
Q.11 AgCl has the same structure as that of NaCl. The
edge length of unit cell of AgCl is found to be 555 Q.23 In a cubic closed packed structure of mixed
pm and the density of AgCl is 5.561 g cm–3. Find the oxides the lattice is made up of oxide ions, one eighth
percentage of sites that are unoccupied. of tetrahedral voids are occupied by divalent ions (A2+)
while one half of the octahedral voids occupied trivalent
Q.12 Xenon crystallises in the face-centred cubic lattice ions (B3+). What is the formula of the oxide?
and the edge of the unit cell is 620 pm. What is the
nearest neighbour distance and what is the radius of Q.24 A solid A+ and B– had NaCl type closed packed
Xenon atom? structure. If the anion has a radius of 250 pm, what
should be the ideal radius of the cation? Can a cation C+
Q.13 The two ions A+ and B– have radii 88 and 200 pm having a radius of 180 pm be slipped into the tetrahedral
respectively. In the closed packed crystal of compound site of the crystal of A+B–? Give reasons for your answer.
AB, predict the co-ordination number of A+.
Q.25 Calculate the value of Avogadro’s number from
Q.14 CsCl has the bcc arrangement and its unit cell edge the following data:
length is 400 pm. Calculate the interionic distance in CsCl. Density of NaCl = 2.165 cm–3
Distance between Na+ and Cl– in
Q.15 Gold crystallizes in a face centered cubic lattice.
If the length of the edge of the unit cell is 407 pm, NaCl = 281 pm.
calculate the density of gold as well as its atomic radius
assuming it to be spherical. Q.26 If the radius of Mg 2+ ion, Cs + ion, O2– ion, S2– ion
and Cl– ion are 0.65 Å, 1.69 Å, 1.40 Å, 1.84 Å, and 1.81
Atomic mass of gold = 197 amu.
Å respectively. Calculate the co-ordination numbers of
the cations in the crystals of MgS, MgO and CsCl.
Q.16 The density of KBr is 2.75 g cm–3. The length of the
edge of the unit cell is 654 pm. Show that KBr has face
Q.27 Iron occurs as bcc as well as fcc unit cell. If the
centered cubic structure.
effective radius of an atom of iron is 124 pm. Compute
(N = 6.023 × 1023 mol–1, At. mass: K = 39, Br= 80) the density of iron in both these structures.

Q.17 An element crystallizes in a structure having FCC Q.28 KCl crystallizes in the same type of lattice as does
unit cell of an edge 200 pm. Calculate the density, if 200 r + r +
g of this element contains 24 × 1023 atoms. NaCl. Given that Na = 0.5 and Na = 0.7 Calculate:
r − r+
Cl K
Q.18 The effective radius of the iron atom is 1.42 Å. It (a) The ratio of the sides of unit cell for KCl to that for
has FCC structure. Calculate its density. (Fe = 56amu) NaCl and
(b) The ratio of densities of NaCl to that for KCl.
Q.19 A crystal of lead(II) sulphide has NaCl structure.
In this crystal the shortest distance between Pb+2 ion Q.29An element A(Atomic weight = 100) having bcc
and S2– ion is 297 pm. What is the length of the edge structure has unit cell edge length 400 pm. Calculate
of the unit cell in lead sulphide? Also calculate the unit the density of A and number of unit cells and number
cell volume. of atoms in 10 gm of A.
Chem i str y | 19.39

Exercise 2 Q.8 In FCC unit cell, what fraction of edge is not covered
by atoms?
Single Correct Choice Type (A) 0.134 (B) 0.24
Q.1 The density of the CaF2 (fluorite structure) is 3.18 g/ (C) 0.293 (D) None of these
cm3. The length of the side of the unit cell is
(A) 253 pm (B) 344 pm Multiple Correct Choice Type
(C) 546 pm (D) 273 pm Q.9 80.0 gm salt of weak base & strong acid XY is
dissolved in water and formed 2 litre of aqueous
Q.2 Which of the following statements is correct in the solution. The pH of the resultant solution was found to
rock-salt structure of an ionic compounds? be 5 at 298 K. If XY forms CsCl type crystal having
(A) Coordination number of cation is four whereas that r + (radius of X + ) = 1.6 Å
X
of anion is six. and r − (radius of Y − ) = 1.864 Å
Y
(B) Coordination number of cation is six whereas that then select write statement(s).
of anion is four.
(Given: Kb(XOH) = 4 × 10–5; NA = 6 × 1023)
(C) Coordination number of each cation and anion is four.
(A) Molar mass of salt is 100 g/mol.
(D) Coordination number of each cation and anion is six.
(B) % Degree of dissociation of salt is 0.25.
Q.3 A compound XY crystallizes in BCC lattice with (C) Edge length of AB is 4 Å.
unit cell edge length of 480 pm. If the radius of Y– is
(D) Density of solid salt XY is 2 in gm/cc.
225 pm, then the radius of X+ is
(A) 127.5 pm (B) 190.68 pm
Q.10 Select correct statement(s)
(C) 225 pm (D) 255 pm
(A) 8 Cs+ ions occupy the second nearest neighbour
location of a Cs+ ion
Q.4 In the closest packing of atoms A (radius: ra), the
(B) Each sphere is surrounded by six voids in two
radius of atom B that can be fitted into tetrahedral
dimensional hexagonal close packed layer
voids is
(C) If the radius of cations and anions are 0.3 Å and 0.4
(A) 0.155 ra (B) 0.225 ra
Å then coordination number of cation in the crystal is 6.
(C) 0.414 ra (D) 0.732 ra
(D) In AgCl, the silver ion is displaced from its lattice
position to an interstitial position such a defect is called
Q.5 In diamond, carbon atom occupies FCC lattice a frenkel defect.
points as well as alternate tetrahedral voids. If edge
length of the unit cell is 356 pm, then radius of carbon
Q.11 Which of the following statements is/are correct?
atom is
(A) The coordination number of each type of ion in CsCl
(A) 77.07 pm (B) 154.14 pm
is 8.
(C) 251.7 pm (D) 89 pm
(B) A metal that crystallises in BCC structure has a
coordination ion number 12.
Q.6 Which of the following will show schottky defect
(C) A unit cell of an ionic cyrstal shares some of its ions
(A) CaF2 (B) ZnS (C) AgCl (D) CsCl with other unit cells.
(D) The length of the unit cell in NaCl is 552 pm.
Q.7 Copper metal crystallizes in FCC lattice. Edge length
= [r + 95pm;
= r − 181pm]
of unit cell is 362 pm. The radius of largest atom that can Na Cl
fit into the voids of copper lattice without disturbing it.
(A) 53 pm (B) 45 pm (C) 93 pm (D) 60 pm
1 9 . 4 0 | Solid State

Q.12 Which of the following statements is/are correct? Q.16 Statement-I: KCl is more likely to show schottky
defect, while LiI is more likely to show Frenkel defect.
(A) In an anti-fluorite structure anions form FCC lattice
and cations occupy all tetrahedral voids. Statement-II: Schottky defect is more likely in ionic
solids in which cations and anions are of comparable
(B) If the radius of cations and anions are 0.2 Å and 0.95
size while Frenkel defect is more likely is which cations
Å the coordination number of cation in the crystal is 4.
and anions have large differences in their ionic sizes.
(C) An atom/ion is transferred from a lattice site to an
(A) Statement-I is true, statement-II is true, statement-II
interstitial position in Frenkel defect.
is the correct explanation of statement-I.
(D) Density of crystal always increases due to
(B) Statement-I is true, statement-II is true, statement-II
substitutional impurity defect.
is not the correct explanation of statement-I.
(C) Statement-I is true, statement-II is false.
Assertion Reasoning Type
(D) Statement-I is false, statement-II is true.
Q.13 Statement-I: Distance between nearest lattice
points in BCC is greater than the same in FCC of the Comprehension Type
atoms of comparable size.
Statement-II: FCC has greater packing efficiency than Paragraph 1: Calcium crystallizes in a cubic unit cell
BCC. with density 3.2 g/cc. Edge-length of the unit cell is 437
picometre (pm).
(A) Statement-I is true, statement-II is true, statement-II
is the correct explanation of statement-I. Q.17 If the metal is melted, density of the molten metal
(B) Statement-I is true, statement-II is true, statement-II was found to be 3 g/cc. What will be the percentage of
is not the correct explanation of statement-I. empty space in the melt?

(C) Statement-I is true, statement-II is false. (A) 31% (B) 36% (C) 28% (D) 49%

(D) Statement-I is false, statement-II is true.


Paragraph 2: Silicon carbide (SiC) and diamond are
covalent solid which crystallize in cubic structures. In
Q.14 Statement-I: Increasing temperature increases
SiC, carbon atoms occupy points of the face centred
the density of point defects.
cubic lattice (FCC positions) and silicon atoms occupy
Statement-II: The process of formation of point defects half of the tetrahedral voids available. In diamonds,
in solids in endothermic and has ∆S > 0. same position of the tetrahedral voids are occupied by
other carbon atoms.
(A) Statement-I is true, statement-II is true,
statement- II is the correct explanation of statement-I. Also the density of SiC and diamond are 3.2 and 3.6
(B) Statement-I is true, statement-II is true, g/cc respectively. Answer the following four questions
statement-II is not the correct explanation of based on the above information: (M: Si = 28)
statement-I.
Q.18 The radius of silicon atom is
(C) Statement-I is true, statement-II is false.
(A) 0.76 Å (B) 1.12 Å (C) 3.54 Å (D) 4.75 Å
(D) Statement-I is false, statement-II is true.

Q.19 Which of the following will not change the density


Q.15 Statement-I: Increase in dielectric constant is of SiC solid?
observed in Frenkel defect.
(A) Substitution of some Si atoms by some carbon
Statement-II: Anions come in interstitial space in case atoms
of Frenkel defect.
(B) Schottky defects
(A) Statement-I is true, statement-II is true, and
statement-II is the correct explanation of statement-I. (C) Interchange of some Si atom by some C atom

(B) Statement-I is true, statement-II is true, and statement- (D) Decreasing the temperature of solid
II is not the correct explanation of statement-I.
(C) Statement-I is true, statement-II is false.
(D) Statement-I is false, statement-II is true.
Chem i str y | 19.41

Paragraph 3: Calcium crystallizes in a cubic unit cell Q.2 The correct statement(s) regarding defects in solids
with density 3.2 g/cc. Edge-length of the unit cell is 437 is(are). (1999)
picometer (pm).
(A) Frenkel defect is usually favoured by a very small
Q.20 The type of unit cell is difference in the sizes of cation and anion.

(A) Simple cubic (B) BCC (B) Frenkel defect is a dislocation defect.

(C) FCC (D) Edge-centred (C) Trapping of an electron in the lattice leads to the
formation of F-centre.
Q.21 The nearest neighbour distance is (D) Schottky defects have no effect on the physical
properties of solids.
(A) 154.5 pm (B) 309 pm (C) 218.5 pm (D) 260 pm
Paragraph 1: In hexagonal systems of crystals, a frequently
Match the Columns encountered arrangement of atoms is described as a
Q.22 hexagonal prism. Here, the top and bottom of the cell
are regular hexagon and three atoms are sandwiched
Column I Column II in between them. A pace-filling model of this structure,
(A) Rock salt structure (p) Co-ordination number of called hexagonal close-packed (hcp), is constituted of a
cation is 4 sphere on a flat surface surrounded in the same plane by
(B) Zinc Blende six identical spheres as closely as possible. Three spheres
3 are then placed over the first layer so that they touch each
(q) d= r+ + r−
4 other and represent the second layer. Each one of these
(C) Fluorite structure (r) Co-ordination number of three spheres touches three spheres of the bottom layer.
cation and anion are same Finally, the second layer is covered with a third layer that is
(s) Distance between two nearest identical to the bottom layer in relative position. Assume
anion is a radius of every sphere to be ‘r’. (2008)
2

Q.3 The number of atoms in one of this hcp unit cell is


Q.23 [Distance in terms of Edge length of cube (a)]
(A) 4 (B) 6 (C) 12 (D) 17
Column I Column II
(A) 0.866 a (p) Shortest distance between Q.4 The volume of this hcp unit cell is
cation and anion in CsCl structure. 26r3
(B) 0.707 a (q) Shortest distance between two (A) 24 2r 3 (B) 16 2r 3 (C) 12 2r 3 (D)
3 3
cations in CaF2 structure
(C) 0.433 a (r) Shortest distance between
Q.5 The empty space in this hcp unit cell is
carbon atom in diamond
(s) Shortest distance between two (A) 74% (B) 47.6% (C) 32% (D) 26%
cation in rock salt structure
6. Match the crystal system/unit cells mentioned in
column I with their characteristic features mentioned in
Previous Years’ Questions column II. (2007)

Q.1 Which of the following statement(s) is(are) correct? Column I Column II


 (1998) (A) Simple cubic and (p) have these cell parameters
face-centred cubic a = b = c and α = β = γ
(A) The coordination number of each type of ion in CsCl
crystal is 8. (B) Cubic and (q) are two crystal system
(B) A metal that crystallizes in bcc structure has a rhombohedral
coordination number of 12.
(C) Cubic and (r) h
 ave only two crystallographic
(C) A unit cell of an ionic crystal shares some of its ions tetragonal angles of 90º
with other unit cells.
(D) Hexagonal and (s) b
 elongs to same crystal
(D) The length of the unit cell in NaCl is 552 pm.
monoclinic system
=(r + 95
= pm; r − 181 pm)
Na Cl
1 9 . 4 2 | Solid State

Q.7 Silver (atomic weight = 108 g mol–1) has a density of this structure, called hexagonal close-packed (HCP),
of 10.5 g cm–3. The number of silver atoms on a surface is constituted of a sphere on a flat surface surrounded
of area 10–12 m2 can be expressed in scientific notation in the same plane by six identical spheres as closely as
as y × 10x. The value of x is. (2010) possible. Three spheres are then placed over the first
layer so that they touch each other and represent the
Q.8 The number of hexagonal faces that are present in second layer. Each one of these three spheres touches
a truncated octahedral is. (2011) three spheres of the bottom layer. Finally, the second
layer is covered with a third layer that is identical to
the bottom layer in relative position. Assumer radius of
Q.9 Marbles of diameter 10 mm are to be put in a square
every sphere to be ‘r’.
area of side 40 mm so that their centers are within this
area. Find the maximum number of marbles per unit
area and deduce an expression for calculating it.(2003) Q.14 The number of atoms on this HCP unit cell is
 (2008)
Q.10 The crystal AB (rock salt structure) has molecular (A) 4 (B) 6 (C) 12 (D) 17
weight 6.023y u. where, y is an arbitrary number in u. If
the minimum distance between cation and anion is y1/3 Q.15 The volume of this HCP unit cell is (2008)
nm and the observed density is 20 kg/m3. Find the (a)
density in kg/m3 and (b) type of defect.  (2004) 64r3
(A) 24 2r 3 (B) 16 2r 3 (C) 12 2r 3 (D)
3 3
Q.11 An element crystallizes in fcc lattice having edge
length 400 pm. Calculate the maximum diameter of Q.16 The empty space in this HCP unit cell is  (2008)
atom which can be placed in interstitial site without
(A) 75% (B) 47.6% (C) 32% (D) 26%
distorting the structure.  (2005)

Q.12 The edge length of unit cell of a metal having Q.17 The correct statement (s) regarding defects in
molecular weight 75 g/mol is 5 Å which crystallizes in cubic solids is (are)  (2009)
lattice. If the density is 2 g/cc then find the radius of metal (A) Frenkel defect is usually favoured by a very small
atom. (NA = 6 x 1023). Give the answer in pm.  (2006) difference in the sizes of cation and anion
(B) Frenkel defect is a dislocation defect
Q.13 Match the crystal system/unit cells mentioned in
column I with their characteristic features mentioned (C) Trapping of an electron in the lattice leads to the
in column II. Indicate your answer by darkening the formation of F-center
appropriate bubbles of the 4 × 4 matrix given in the (D) Schottky defects have no effect on the physical
ORS.  (2007) properties of solids

Column I Column II
Q.18 The coordination number of Al in the crystalline
(A) Simple cubic and (p) Have these cell parameters state of AICl3 is  (2009)
face-centred cubic
a = b = c, α = β = γ = 900
(B) Cubic and (q) Are two crystal systems Q.19 The packing efficiency of the two-dimensional
rhombohedra square unit cell shown below is  (2010)

(C) Cubic and tetragonal (r) Have only two


crystallography
(D) Hexagonal and (s) Belong to same crystal
monoclinic system

Paragraph 2: In hexagonal systems of crystals, a


frequently encountered arrangement of atoms is
described as a hexagonal prism. Here, the top and
bottom of the cell are regular hexagons and three atoms (A) 39.27% (B) 68.02%
are sandwiched in between them. A space-filling model
(C) 74.05% (D) 78.54%
Chem i str y | 19.43

Q.20 A compound MpXq has cubic close packing (ccp) Q.22 If the unit cell of a mineral has cubic close
arrangement of arrangement of X. Its unit cell structure packed (ccp) array of oxygen atoms with m fraction of
is shown below. (2012) octahedral holes occupied by aluminium ions and n
fraction of tetrahedral holes occupied by magnesium
ions, m and n, respectively, are  (2015)

1 1 1 11 11
(A) , (B) 1, (C) (D)
2 8 8 22 48

Q.23 The CORRECT statement(s) for cubic close packed


(ccp) three dimensional structure is(are)  (2016)
(A) MX (B) MX2 (C) M2X (D) M5X14 (A) The number of the nearest neighbours of an atom
present in the topmost layer is 12
Q.21 The arrangement of X” ions around A+ ion in solid (B) The efficiency of atom packing is 74%
AX is given in the figure (not drawn to scale). If the
(C) The number of octahedral and tetrahedral voids per
radius of X − is 250 pm, the radius of A+ is (2013)
atom are 1 and 2, respectively
(D) The unit cell edge length is 2 2 times the radius of
the atom

(A) 104 pm (B) 125 pm (C)183 pm (D) 57 pm

PlancEssential Questions
JEE Main/Boards JEE Advanced/Boards

Exercise 1 Exercise 1
Q.18 Q.25 Q.29 Q.5 Q.11 Q.15 Q.24

Exercise 2 Exercise 2
Q.5 Q.10 Q.7 Q.10 Q.11 Q.22

Previous Years’ Questions Previous Years’ Questions


Q.5 Q.7 Q.10 Q.7 Q.9 Q.12
1 9 . 4 4 | Solid State

Answer Key

JEE Main/Boards
Exercise 1
Q.6 M2N3 Q.7 face-centred cubic

Q.8 2.85 × 1024 atoms Q.9 2

Q.10 299.8 pm Q.11 4.348 × 10−23 cm3

Q.12 1.2046 × 1019 mol−1 Q.13 52 u

Q.14 2.86 g cm–3 Q.15 127.7 pm

Q.16 144.2 pm Q.17 74.70 g mol–1 or 74.70 u

Q.18 A4B3 Q.19 0.6709 g cm–3

Q.20 0.523 Q.21 M4O6 or M2O3

Q.22 ZnAl2S4 Q.23 60%; 4

Q.24 74% Q.25 4.7 %


Q.30 4

Exercise 2

Q.1 B Q.2 C Q.3 C Q.4 A Q.5 C Q.6 B

Q.7 D Q.8 D Q.9 C Q.10 A Q.11 B Q.12 A

Previous Years’ Questions

Q.1 A Q.2 D Q.3 A Q.4 B Q.5 D Q.6 A

Q.7 1.86 Å Q.8 26% Q.9 7.3 g/cc Q.10 432:343 Q.12 A Q.13 D

Q.14 B Q.15 B Q.16 C Q.17 D Q.18 C Q.19 A

JEE Advanced/Boards
Exercise 1
Q.1 A-B Q.2 AuCu3

Q.3 A4B2C2 Q.4 59.95%, +4

Q.5 ZnAl2O4 Q.6 267 pm, 534 pm, 378 pm

Q.7 267 pm, 231.2 pm, 188.8 pm Q.8 3.5 g cm–3

Q.9 3.472 × 1024 atoms Q.10 FCC


Chem i str y | 19.45

Q.11 0.24% Q.12 438.5 pm, 219.25 pm

Q.13 6 Q.14 346.4 pm

Q.15 19.4 g/cm3, 143.9 pm Q. 16 FCC

Q.17 41.67 cm–3 Q.18 5.74 g cm–3

5.94 × 10−8 cm, V =


Q.19 a = 2.096 × 10−22 cm−3 Q.20 1.81 Å

Q.21 123.84 pm Q.22 175.8 pm

Q.23 AB2O4 Q.24 103.4 pm, No

Q.25 6.01 × 1023 Q.26 4, 6, 8

Q.27 7.887 g/cc, 8.59 gm/cm3 Q.28 (a) 1.143, (b) 1.172

Q.29 5.188 gm/cm3, 6.023 ×1022 atoms of A, 3.0115 × 1022 unit cells

Exercise 2

Single Correct Choice Type

Q.1 C Q.2 D Q.3 B Q.4 B Q.5 A Q.6 D

Q.7 A Q.8 C

Multiple Correct Choice Type

Q.9 A, C Q.10 B, D Q.11 A, C, D Q.12 A, C

Assertion Reasoning Type

Q.13 A Q.14 A Q.15 C Q.16 A

Comprehension Type

Q.17 A Q.18 B Q.19 C Q.20 C Q.21 D

Match the Columns

Q.22 A → r, s; B → p, q, r, s; C → q Q.23 A → p; B → q, s; C → r

Previous Years’ Questions

Q.1 A, C, D Q.2 B, C Q.3 B Q.4 A Q.5 D

Q.6 A → p, s; B → p, q; C → q; D → q, r Q.7 7 Q.8 8 Q.9 25

Q.10 (a) 5 kg/m3 (b) Metal excess defect Q.11 117 pm Q.12 217 pm

Q13 A → p, s; B → p, q; C → q; D → q, r Q.14 B Q.15 A Q.16 D

Q.17 B, C Q.18 6 Q.19 D Q.20 B Q.21 A

Q.22 A Q.23 B, C, D
1 9 . 4 6 | Solid State

Solutions

JEE Main/Boards Sol 7: molar mass = 2.7 × 10–2 kg/mol = 27 gm/mol


Let’s say in one cubic unit, there is Z-effective atoms
Exercise 1
(z) x mass of one atom
= density
Sol 1: (i) See theory on page no. 19.20 Volume of cell
(ii) See theory on page no. 19.22 27
(z) × 23
× 10 −3
6 × 10 = 2.7 × 1023
Sol 2: See theory on page no. 19.21 (405 × 10−12 )3

Z=4
Sol 3: (a) When same property is measured in different
direction in solid gives different value is called So cubic unit would be fcc
Anisotropy.
Sol 8: Effective Atom = 2
Sol 4: (a) Square close packing
(a) Square close packing 2 × (mass of one atom)
= 5.200 kgm/m3
(a) Square close packing (300 × 10−12 )3
5.2 × 106 × 27 × 10−30
Mass of one atom = gm
2
= 70.2 ×10–24 gm
200
Hexagonal close packing Atoms in 200 gm = = 2.84 × 1024 atoms
Hexagonal close packing 70.2 × 10 −24
Hexagonal close packing
Sol 9: Like question number 7, we suppose it has
z-effective atoms in one unit cell

(b) number of nearest atoms Z


=
(52 × 1.66 × 10 =) −24

7.2 × 106 gm / m3
(298 × 10 )
3
−12
(b) number of nearest atoms
(b) Coordination number of square close packing is 4. z=2
Coordination number of hexagonal close packing is 6. So the unit would be bcc

Sol 5: (a) Schottky defect Sol 10: 4.12 × 1024 atom = 200 gm
(b) Frenkel defect 200
Mass of one atom = = 4.85 × 10–23 gm
4.12 × 1024
Sol 6: Effective number of atom in Effectively number in fcc = 4
1 1
ccp = fcc = 6 × +8× 4 × (4.85 × 10−23 )
2 8 = 7.2 g/cm3
r3
= 4 = N-atom
r = 300
Tetrahedral void = 8 (in fcc)
8 Sol 11: Zeff. = 2
Number of M-atom =
3
 6.94 
So empirical formula = M8 N4 = M2N3 2× 23 
 6.023 × 10  = 0.53 gm/cm3
3
V

V = 4.348 × 10–23 cm3


Chem i str y | 19.47

Sol 12: One Sr2+ will replace 2 Na+ Na+ at centre of cube = 1
So incoming of one Sr2+ will result into one cation If face centred, that is Cl– is removed, along one axis.
vacancy 1 1
Let’s say 100 moles are total Then Cl– = 4 ×   + 8   = 3
2
  8
2 × 10−3
In one mole, Sr2+ = 6.023 × 1023× Na+ = 3 + 1 = 4
100
Na4Cl3
Cation vacancy = 12.04 × 10+18 = 1.204 × 1019
Similarly A4B3
2 × (atomic mass)
Sol 13: = 7.2×106 gm/m6 4 × (mass of a molecule)
(2.88 × 10−10 )3 Sol 19: Density =
abc
Atomic mass = 8.6 × 10–23 gm  21.76 
4× 23 
Atomic mass ≈ 52 amu  6.023 × 10  gm/m6
=
6.8 × 4.4 × 7.8 × 10−30
Z eff (mass of one atom)
Sol 14: Density = = 0.6709 gm/cm3
r3
 208 
4× 24 
 4.283 × 10  = 2.86 gm/cm3 Sol 20 Let’s say radius of sphere = r
=
(4.08 × 10−10 )3 Side of cube that just encloses the sphere = 4r

 63.55 
4× 23 
Sol 15: 8.930×106 gm/m3 =  6.023 × 10 
a3
4 × 63.55
a3 = ×10–29
8.93 × 6.023
a = 3.6161 Å
a
2 a = 4r (in fcc structure, analyze diagonal of a face)
r = 127.8 pm
 197 
4× 23 
Sol 16: 19.3×106 =  6.023 × 10 
a3
Similarly a = 407.74 pm Volume = (4R)3 = 64 R3
4 3
2 a = 4r (in fcc structure) Volume of sphere = 8 × pR
3
r = 144.15 pm 4
 M  8× π
4× 23  Fraction = 3 = 52.35%
Sol 17: 1.984×106 =  6.023 × 10  64
(6.3 × 10 –10 )3
M = 74.69 gm Sol 21: Analyzing hcp structure
O2– (in hcp array) = 6
Sol 18: In NaCl (fcc structure) Octahedral void (in hcp) = 6
1 2 2
Cl– at centre of face = 6 ×   = 3 M occupies rd of octahedral voids = × 6 = 4
2 3 3
1 M4O6 ⇒ M2O3
Cl– at corners = 8 ×   = 1
8
1 Sol 22: Closest packed array = fcc
Na+ at centre of edge =12×   = 3
4 1
Zinc = × (Number pf tetrahedral void)
8
1 9 . 4 8 | Solid State

1 x = 95.3%
= ×8=1
8 % of vacancy = 4.7%
1
Aluminium = × (Number of octahedral void)
2 Sol 26: Refer theory.
1
= ×4=2
2 Sol 27: (a) In metal on increasing temp, the alignment
S (in fcc) = 4 of electron get disturb due to excitation of electron
due to which conductivity decrease while in case of
ZnAl2S4
semiconductor on increasing temp. electrons absorb
energy and get excited to conduction band hence
Sol 23: O (in fcc) = 4 conductivity increases.
1 1 (b) Due to difference in size of atoms.
Ti = × (Number of Octahedral void) = ×4=2
2 2 (c) On heating ZnO, O2− ion is replaced by e− due
Ti2O4 = TiO2 to which lattice contain free electrons to conduct
48 electricity.
% by mass of Ti = = 60 % 1
48 + 32 ZnO( s ) → Zn2+ + O2 + 2e−
2
Ti oxidation state = + 4
Sol 28: Refer theory
Sol 24: Packing efficiency of hcp
Zeff = 6 Sol 29: Refer theory

a = 2r (side of hexagon) Sol 30: In ccp structure = fcc structure


2 2 Number of octahedral void
b = (height of hexagon) = r
3
= one at center of cube +
1/2
4 1
6 × π(r)3 At very edge centre, there is  
3 4
So packing eff. = = 74%
3 2 2 2 1
6× × (2r) × r octahedral void = 1 + 12   = 4
2 3 4
Packing eff. For fcc Number of effective atom
th
Zeff. = 4 1
= at every corner, there is   part of an atom + at
a=
4
r (side of cubic) 8
nd
2 1
every center of face, there is   part of an atom
4 
4 ×  πr 3  2
So packing eff. = 3  = 74% 1 1
16 2r 3 = 8   +6   = 4
8 2

Sol 25: Let’s suppose weight of Ge crystal is W

di = initial density =
W Exercise 2
V
1 1
W Sol 1: (B) W = 8   = 1,0 = 12   = 3
df = 0.96 df = 0.96 8 4
V Na = 1
Volume is fixed so it implied boron decreases weight
NaWO3
4%
mf = mass final = 0.96 W Sol 2: (C) CaF2 ⇒ Ca2+ (in fcc) + F–
W (at octahedral void)
xW + (1 – x) × 11 = 0.96 w
72.6
CsCl ⇒ BCC ⇒ Cs+ (at center) + Cl–
Chem i str y | 19.49

(at every corner) 2( r +r ) = 670.30 pm


NH+
4 Cl−
Cordination number ⇒ 8 : 4 (CaF2)
8 : 8 (CsCl) r +r = 335.15 pm
NH+
4 Cl−

Sol 3: (C) Correct


Previous Years’ Questions
Sol 4: (A) CsCl ⇒ BCC ⇒ Cs+ (at center) + Cl– (at every
corner) Sol 1: (A) Three consecutive layers of atoms in
hexagonal close packed lattice is shown below:
 1
1Cs+ +  8 ×  Cl– = 1Cs+ + 1Cl–
 8
A
Sol 5: (C) For close packing two same sheets cannot
be together.

Sol 6: (B) ZnS ⇒ S2– (fcc) + Zn2+ (occupies alternate B


tetrahedral void)
r 2+
For occupation of tetrahedral void Zn = 0.225
r 2−
S
r 2− = 100 pm A
S

Sol 7: (D) NaCl ⇒ Na+ (at octahedral voids) + Cl– (fcc Atom X is in contact of 12 like atoms, 6 from layer B and
structure). 3 from top and bottom layers A each.

Sol 8: (D) fcc ⇒ number of nearest neighour around Sol 2: (D) In NaCl, Na+ occupies body centre and edge
each particle = 12 centres while Cl– occupies corners and face centres,
giving four Na+ and four Cl– per unit cell. In the present
Sol 9: (C) A ⇒ hcp effective number of atoms = 6 case, A represent Cl– and B represents Na+. Two face
centres lies on one axis.
Octahedral void = 6
1
2 ⇒ Number of A removed = 2 × =1
C = 6  = 4 2
3
Number of B is removed because it is not present on
C4A6 ⇒ C2A3
face centres.

1 1 ⇒ A remaining = 4 – 1 = 3


Sol 10: (A) Zeff. For X ⇒ 8   + 6   = 4
8 2 B remaining = 4
Zeff. For Y ⇒ 8
⇒ Formula = A3B4
1
Zeff. For Z ⇒ (4) = 2
2 1
Sol 3: (A) In cubic system, a corner contribute th part
X4Y8Z2 ⇒ X2Y4Z 8
1
of atom to one unit cell and a face centre contribute
2
Sol 11: (B) Co-ordination number = 8 = BCC part of atom to one unit cell. Therefore,
1
Number of A per unit cell = ×8 = 1
Sol 12: (A) Oppositely charged ions will be nearest 8
atom. 1
Number of B per unit cell = ×6 = 3
Analyzing body (cubic) diagonal of BCC structure 2
⇒ Formula = AB3
(r + 2r +r )= 3 (387)
NH+
4 Cl− NH+
4
1 9 . 5 0 | Solid State

Sol 4: (B) In ZnS, S2– (sulphide ions) are present at 4 3 1


= 6× πr × = 0.74
fcc position giving four sulphide ions per unit cell. To 3 24 2r3
comply with 1 : 1 stoichiometry, four Zn2+ ions must be
present in four alternate tetrahedral voids out of eight ⇒ Percent empty space
tetrahedral voids present. = 100(1 – 0.74) = 26%
In NaCl, Na ions are present in octahedral voids while
+

in Na2O, Na+ ions are present in all its tetrahedral voids Sol 9: In bcc unit cell 4r = 3a
giving the desired 2 : 1 stoichiometry. In CaF2, Ca2+ ions
occupies fcc positions and all the tetrahedral voids are 3 3
⇒ r(Cr) = a = × 287 pm = 124.3 pm
occupied by fluoride ions. 4 4
NM
Density of solid =
Sol 5: (D) NA .a3
N = Number of atoms per unit cell
M = Molar mass
1 a3 = Volume of cubic unit cell
Contribution of circle from corner of square =
4 NA = Avogadro’s number
⇒ Effective number of circle per square 3
2 × 52g  1 
= ×  = 7.3 g/cm
3
1
= × 4 + 1(at centre) = 2 6.023 × 1023 −8
 2.87 × 10 cm 
4
⇒ Area occupied by circle = 2πr 2 ;
Sol 10: We have, F.C.C unit cell length =3.5 Å,
r = radius.
B.C.C unit cell length= 3.0 Å
Also, diagonal of square 4r = 2 L ,
Now no. of atoms for F.C.C(n1)=4;
where L = side of square.
=3 (3.5 × 10−8 )3
V1=a
⇒ Packing fraction
Now no. of atoms for B.C.C(n1)=2
Area occupied by circle
= a3 (3.0 × 10−8 )3
V2==
Area of square
(n1 × At. wt)
2πr 2 2πr 2 π Density in F.C.C =
= = == 0.7854 (V1 × Av .No)
L2 8r 2 4
(n2 × At. wt)
⇒ % packing efficiency = 78.54% Density in B.C.C =
(V2 × Av .No)

Sol 6: (A) In ionic solid MX (1 : 1 solid), same number of Therefore,


Mn– and Xn– ions are lost in Schottky defect to maintain DF.C.C
electro-neutrality of solid. = (n1 / n2 ) × (V1 / V2 )
DB.C.C
= (4 / 2) × (3.0 × 10−8 )3 / (3.5 × 10−8 )3
Sol 7: In bcc arrangement of atoms: 4r = 3a, atoms on
= 432 : 343
body diagonal remain in contact

3a 3 × 4.29
=
⇒ r = = 1.86 Å Sol 11:
A A

Sol 8: The given arrangement: ABABAB......... represents


hexagonal close-packed unit cell in which there are six
atoms per unit cell. Also, volume of unit cell = 24 2r 3 .
⇒ Packing fraction
Volume occupied by atoms
=
Volume of unit cell
Chem i str y | 19.51

Sol 12: (A) No. of atoms of Y = 4 1


Copper (Cu) =   × 6 = 3
2 2
No. of atoms of X= × 8 AuCu3
3
Formula of compound will be X 4 Y3 
Sol 3: =
A ccp
= fcc
= 4

Sol 13: (D) For an ionic substance in FCC arrangement,
1  Z eff. = effective
= B = (8) 2 
4  number of atoms
2 (r + + r − ) =
edge length =
1
C = (4) 2

2 

2 (110 + r − ) =
508 A4 B 2 C 2

r − = 144 pm
Sol 4: O=2−
= 4  Z = effective
ccp
 eff.
Sol 14: (B) For F.C.C 1 
= Ti = (4) 2  number of atoms
2 
2a = 4r
2a 2 × 361 TiO2
=r = = 127pm
4 4 ↓
+4
Sol 15: (B) Packing fraction of cubic close packing and

body centred packing are 0.74 and 0.68 respectively. Sol 5: O=
2−
ccp
= 4 

1 1  Z = effective
Sol 16: (C) Effective no. of A atoms = ×8 = 1 A = × (8) =1 =Zn2+  eff.
8 8  number of atoms
1 5 1 
Effective no. of B atoms= × 5 (One is missing) = B= (4)= 2= Al3+ 
2 2 2 

Therefore formula is A1B 5 = A2B5 ZnAl2O4


2

Sol 6: Analyzing one edge of FCC (KF) structure


Sol 17: (D) For BCC, 3 a = 4r
2(r++ r–) = a
3 × 351
=r = 152 pm r++ r– = shortest KF – distance = 132 + 135 = 267 pm
4
a = 534 pm
2a
Sol 18: (C) 2rCl− + 2rCs+ =3a Closest K – K distance = = 377.6 pm
2

3a
rCl− + rCs+ = Sol 7: (a) 2r = a
2 r = 267 pm

3 (b) 3 a = 4r
Sol 19: (A)
= R = a 1.86 Å
4 r = 231.22 pm
(c) FCC
JEE Advanced/Boards 2 a = 4r
R = 188.8 pm
Exercise 1
 12 
8 23 
Sol 1: AB = (CsCl type structure) 6.023 × 10 
Sol 8: density = 
(3.569 × 10−10 )3
1
Sol 2: Gold (Au) = 8 × =1 (Zeff. = 8 for fcc of diamond) = 3.5 gm/cm2
8
1 9 . 5 2 | Solid State

Sol 9: Mass of unit cell = 7.2 × 24 × 10-24 Mass of unit cell (theoretical)
= 17.28 × 10-23 gm  108 + 35.5 
=4×   = 95.301 × 10 gm
–23

Number of unit cell in 200 gm  6.023 × 1023 


95.30 − 95.06
200 % of sites = ×100 = 0.24 %
= = 11.57×1023 95.30
17.28 × 10−23
1 2a
Number of atoms in one unit cell = 8 ×   +2 = 3 Sol 12: Nearest height distance = = 2r
8 2
So, Number of atoms in 200 gm of solid = 3 × 11.57 × 1023 = 438.40 pm

= 3.472 × 1024 atom r = 219.2 pm


r
A+ 88
Sol 10: rAg = 144 pm Sol 13: = = 0.44 > 0.414
r− 200
B
 108 
Z eff. ×  23  So it will occupy octahedral void
Density =  6.023 × 10  = 1.6 × 106
a3 So coordination number would be = 6
3
a
= 16.91×10–30 Sol 14: 3 a = 2(r+ + r–)
Z eff.
3 × 400
⇒ a = (Z eff. × 16.91 × 106 )1/3 pm (r+ + r–) =
2
Case I: Simple cubic r+ + r– = 346.41 pm

Zeff. = 1
Sol 15: a = 407 pm
a = 2r
 197 
Which does not satisfies 4× 23 
Density =  6.02 × 10  = 19.40 gm/cm3
Case II : BCC (4.07 × 10−10 )3
Zeff. = 2 In fcc, 2 a = 4r
a = 323.38 pm
R = 143.9 pm
3 a=4r
 39 + 80 
r = 140 pm Z eff.  23 
Sol 16: 2.75 ×106 =  6.023 × 10 
Which does not satisfy (6.54 × 10−10 )3
Case III : FCC Zeff ~ 4
Zeff. = 4 FCC structure
a = 407.44 pm
mass 200
2 a = 4r Sol 17: Density = =
volume volume of 200gm
r = 144 pm
4 atoms then volume = (2.00 × 10–10)3 m3
So, the structure will be FCC
8 × 10 –30
24×1023 atom then volume = ×24×1023 m3
Sol 11: a = 555 pm 4
200 × 4
NaCl in FCC = Na4Cl4 Density = gm/m3 = 41.67 gm/m3
28 × 10−30 × 24 × 1023
Mass of unit cell
5.561 × 106 =
(5.55 × 10−10 )3
Sol 18: 2 a = 4r
95 × 10–23 = mass of unit cell
a = 2 2 r = 4.016 Å
95.06 × 10–23 = mass of unit cell
Chem i str y | 19.53

 56  Because for tetrahedral voids


4× 23 
 6.023 × 10  gm/m6 = 5.74 gm/m6 r+
Density = 0.225 < < 0.414
10−30 × (4.016)3 r−

Mass of unit cell


Sol 19: ( r + r 2− ) = 297 pm Sol 25: Density =
Pb2 + S
Volume of unit cell
2( r +r )=a
PbS ⇒ NaCl type structure Na+ Cl−

Analyzing edge in Rock Salt Structure a = 2 × 281 = 562 pm

a = 2( r + r 2− ) = 594 pm 4 × (mass of a molecule)


Pb2 + S
2.165×106 =
(5.62 × 10−10 )3
V = a3 = 209.5 × 10–30 m3
Mass of a molecule

Sol 20: 3 a = 2 (r +r )=7Å 23 + 35.5


Cs+ Cl–
= = 96.07 × 10–24
Avogadro's number
3 = 2 (1.69 + r )=7Å NA = Avogadro’s number = 6.089×1023
Cl–

r = 1.81 Å
Cl–
Sol 26: MgS
Sol 21: 3 a = 4rFe r+ 0.65
= = 0.35
r− 1.84
rFe = 123.8 pm
r
0.225 < + < 0.414 (Tetrahedral void)
r−
Sol 22: 3 a = 2 (r +r )
Cs+ Cl– Mg2+ will occupy tetrahedral void (alternate)
3 (412) = 2 ( r + 181 ) So the coordination number = 4
Cs+

r = 175.8 pm MgO
Cs+
r+ 0.65
= = 0.464
Sol 23: ccp = fcc r− 1.40
r
Tetrahedral void = 8 ; 0.414 < + < 0.732
r−
Octahedral void = 4
1 1 So Mg2+ will occupy octahedral void
Zeff. = effective number of atoms = 4 = 8×   + 6  
8 2 Coordination number = 4
1
A = (8) = 1
2+
CsCl
8
1 r+ 1.69
B3+ = (4) = 2 = = 0.933
2 r− 1.81
r
O2– (fcc) = 4 0.732 < + < 1
r−
AB2O4 So Cs+ will occupy center position in BCC structure
Coordination number = 8
Sol 24: In NaCl structure, Na+ occupies tetrahedral
r
voids, so radius of cation + = ( 2 – 1) Sol 27: BCC
r−
r+ = 103.5 pm r = 124 pm
r+ 3 a = 4r
It r+ = 180 pm then = 0.72
r− a = 286.3 pm
No, it can not be slipped into tetrahedral void
1 9 . 5 4 | Solid State

 56  Exercise 2
2× 23 
density =  6.02 × 10  = 7.924 gm/cm3
Single Correct Choice Type
(2.863 × 10−10 )3
FCC Sol 1: (C) CaF2 = Ca2+ (in fcc structure) + F–

r = 124 pm (at tetrahedral void)

2 a = 4r = 4 + 8 = 12 atoms = 4 molecules

a = 350.72 pm  78 
4× 23 
3.18 × 106 =  6.023 × 10  ⇒ a = 546 pm
 56 
4× 23  a3
density =  6.023 × 10  = 8.620 gm/cm3
(3.5072 × 10−10 )3
Sol 2: (D)
r (a) False each has co-ordination number = 6
Na+
Sol 28: r =
K+ 0.7 (b) False
r r 5
K+ Na+ (c) False
= = = 0.714
r (0.7)r 7
Cl− Cl− (d) True
5  Both should have equal co-ordination number
(a) Sides of KCl = 2( r +r ) = 2  + 1 r −
K+ Cl−
7  Cl
Sides of NaCl = 2( r +r ) = 2 (1.5) r Sol 3: (B) Analyzing cubic diagonal in three dimension
Na+ Cl− Cl−
 r – + 2r + + r − 
12 8  y x y 
= 480 pm
Ratio = = = 1.143  
7 × 1.5 7 3
 
MNaCl r + + 225 = 240 3
4× x
d1 (rNaCl )3
(b) Ratio density = = r + = 190 pm
d2 MKCl x
4× 3
(rKCl )
3 Sol 4: (B) Closest packing ⇒ fcc
MNaCl  rKCl 
=  
MKCl  rNaCl  2ra 2ra
3
58.5  8 
= = 1.1725
74.5  7  2ra
o
109 o
Sol 29: BCC structure 180-109
2
Effective number of atoms = 2
 100  2ra ra + rb
2× 23 
=
 6.023 × 10  = 5.188 gm/cm3 sin109º sin(35.5º )
Density =
(4 × 10 −10 )3 rb = 0.225ra
10
Volume of 10 gm = = 1.927 cm3 Sol 5: (A) Analyzing unit of tetrahedral void
5.188
= 1.927 × 10 cm –6 3

Volume of one unit = (4 × 10–10)3 = 64 × 10–30 cm3

1.927 × 10−6
Number of unit cells = = 0.301 × 1023
64 × 10−30
Number of atoms = 6.023 × 1022 atoms
Chem i str y | 19.55

In the shown fig. of four carbon atoms, there is one rA


more carbon atom which is in tetrahedral void of the rCu = 0.414
group. It is in touch with all other carbon atoms. None
⇒ rA = 53 pm
of the 4 carbon-atoms touches each other.
Two other
Sol 8: (C) Face of FCC unit cell:
carbons

R
2r
4r
2r
Carbon of 109
o

tetrahedral void 4r
Edge length = =2 2 r
R 2r 2
=
sin109º  180 – 109º  Covered length = 2r
sin  
 2  2r
R = 3.256 rc Fraction cover = = 70%
2 2r
Face of cubic
Fraction not covered = 100 – 70.7 = 29.3%

Multiple Correct Choice Type

Sol 9: (A, C) XY has CsCl type structure


2R
So 2( r + + r − ) = 3a
x y
a
A = 4Å
2 a = 2R
Let’s suppose molecular weight of XY = M
a
R= 80
2 So[XY] = =C
2×M
356
3.256 rc = X+ + H2O → XOH + [H+]
2
rc = 77.0 pm c
c(1–α) cα cα
Sol 6: (D) Schottky defect appears in molecules which Kw [XOH][H ] +

have same size of ions & have high co-ordination. Thus, =


Kb [X + ]
CsCl will show this defect.
[H+] = 10–5
Sol 7: (A) Edge length = a = 362 pm
10−14 cα
= ×10–5
In fcc unit, analyzing face diagonal 4 × 10 −5 c(1 − α )
4r = 2 a 1
α ~ ×10–4
4
radius of copper
[H+] = cα = 10–5
356
r= = 128 pm 10−5
2 2 C=
fcc has two types of void (1 / 4) × 10 −4

⇒ tetrahedral, octahedral C = 0.4


80
Octahedral void will allow for largest radius of atom = 0.4
without any disturbance 2M
M = 100 gm/mole
1 9 . 5 6 | Solid State

 100  Sol 14: (A) Statement-II ∆G = ∆H – TDS


1× 23 
Density =  6.02 × 10  = 2.6 gm/cm3 ∆H for formation of point defect > 0
(4 × 10 −10 )3 DS > 0
So hence increasing T leads to decrease in G
Sol 10: (B, D)
So increasing temperature, we will get more point
(A) Wrong. Analyzing CsCl structure defect.
(B) Voids on center of edge so six void Statement-II is explaining statement-I
(C) Co-ordination will not be affected.
(D) Correct Sol 15: (C) Statement-II is wrong. Because cations
comes in interstitial space because of small size.
Sol 11: (A, C, D)
Sol 16: (A) Both are correct, statement-II is explaining
(A) Correct statement-I
(B) No, co-ordination in BCC structure = 8
(C) Yes Comprehension Type
r + 95
(D) Na = = 0.524 > 0.414 Sol 17: (A) In fcc packing efficiency = 74%
r − 181
Cl Let’s say total volume of the solid = V
So length of edge = 2( r +r ) Mass = (3.2) V
Na+ Cl−
= 552 pm Now, after melting, mass would be same.
(3.2)V
Sol 12: (A, C) 3=
Vf
(A) Correct
Vf = 1.066 V
r+ 0.2 (0.74)V
(B) = = 0.210 < 0.225 So, final packing efficiency = = 69.3%
r− 0.95 (1.066V)
r+ Hence empty space ~ 31%
> 0.155
r−
It will occupy trigonal void Sol 18: (B) SiC ⇒ C = fcc

Co-ordination number = 3 Si = at half of tetrahedral voids

(C) Correct Molecular weight = 28

(D) No, It depends on other atom that is coming Diamond ⇒ C = fcc


C = at tetrahedral void
Assertion Reasoning Type For SiC
 28 + 12 
Sol 13: (A) Statement-II correct (Z eff. )  
106 × 3.2 =  6.023 × 1023 
Statement-I BCC ⇒ 4r = 3a a3
4r Zeff. = 4
a=
3 a = 436.23 pm
FCC ⇒ 2 a = 4r
For diamond
4r
a=  12 
2 8× 23 
106 × 3.6 =  6.023 × 10 
BCC > FCC (distance between nearest lattice points) a3

b = 353.76 pm
Chem i str y | 19.57

r = 154.5 pm

2r
R Distance between nearest neighour = 309 pm

o Match the Columns


109

R 2rc Sol 22: (A → r, s; B → p, q, r, s; C → q)


=
sin109º  180 − 109  (A) Rock salt structure
sin  
 2 
Co-ordination number = 6
R = 3.256 rC
A = 2 (r+ + r–)
2R = 2b
Co-ordination number of cation and anions are same
rC = 76.81 pm
2a a
Analysis by ratio of densities Distance between nearest anion = =
2 2
d1 3.2 M /V (B) Zinc blende structure
= = sic
d2 3.6 MC / V Co-ordination number of cation = 4
2

3a
8 (2rC )3 (28 + 12) = r+ + r–
= × 4
9 (rSi + rC )3 24
Co-ordination number of cation and anions are same
rSi
= 1.46 (C) Fluorite structure
rC
(p) Incorrect
rSi = 1.12Å
(q) Correct
Sol 19: (C) (r) Incorrect
(A) It will change density (s) Incorrect
(B) Changes always
Sol 23: A → p; B → q, s ; C → r
(C) It will not change as the mass & the volume both
are constant. Solve matrix reverse

(D) Decreasing temperature → Decreases distance 3a


(p) Shortest distance = (r+ + r–) =
→ increases density 2
a
2 × (Atomic mass) (q) = Shortest distance between two cations =
Sol 20: (C) Density = 2
Volume
(r) 0.433a = 2r
 40 
Z× 23  2a
 6.023 × 10  (s) = = 0.707 a
3.2 × 106 = 2
(4.37 × 10−10 )3
Z~4
Previous Years’ Questions
FCC unit
Sol 1: (A, C, D) The unit cell of CsCl has bcc arrangement
Sol 21: (D) Face of ions in which each ion has eight oppositely charged
ions around it in the nearest neighbours as shown below
:

Cs+
Cl-

4r = 2 (437 pm)
Unit cell of CsCl
1 9 . 5 8 | Solid State

(B) In bcc, coordination number of atom is 8. 2 2


=
⇒ PR = RS r
(C) In a unit cell, a corner is shared in eight unit cells 3 3
and a face centre is shared between two adjacent unit
cells. In right angle triangle PQR,

(D) In NaCl unit cell; 2(r +r )=


a PQ
= QR 2 − PR 2
Na+ Cl−
⇒ a = 2(95 + 181) = 552 pm 2
=2 r
3
2
Sol 2: (B, C) (A) Incorrect statement. A small difference ⇒ Height of hexagon
= 2PQ
= 4 r
3
in sizes of cation and anion favour Schottky defect while
⇒ Volume = Area of base × height
Frenkel defect is favoured by large difference in sizes of
cation and anion. 3 2
= 6 (2r)2 × 4 = r 24 2 r 3
(B) Correct statement. In Frenkel defect, the smaller 4 3
atom or ion gets dislocated from its normal lattice
positions and occupies the interstitial space. Sol 5: (D) Packing fraction
(C) In F-centre defect, some anions leave the lattice Volume occupied by atoms
=
and the vacant sites hold the electrons trapped in it Volume of unit cell
maintaining the overall electro-neutrality of solid.
4 3 1
(D) Incorrect Statement. In Schottky defect, some of = 6× πr × = 0.74
3 24 2 r3
the atoms or ions remaining absent from their normal
lattice points without distorting the original unit cell ⇒ Fraction of empty space
dimension. This lowers the density of solid. 1 − 0.74 =
= 0.26 = 26%

Sol 3: (B) Sol 6: (A → p,s; B → p,q) (A) Simple cubic and face
centred cubic both have cell parameters a = b = c and
α = β = γ = 90º. Also both of them belongs to same,
A cubic, crystal system.

B (B) Both cubic and rhombohedral crystal systems have


Q their cell parameters a = b = c and α = β = γ but they
M belongs to different crystal systems.
C P
(C) Cubic and tetragonal are two different type of
R S N crystal systems having different cell parameters.
An hcp unit cell
(D) Hexagonal and monoclinic are two different crystal
1 systems and both have two of their crystallographic
Contribution of atoms from corner =
6 angle of 90º.
1
Contribution from face centre = Sol 7: Ag crystallises in FCC unit cell with 4 atoms per
2
unit cell.
⇒ Total number of atoms per unit cell
4 × 108
ρ= = 10.5 g cm–3
1 1 6.023 × 1023 × a3
= 12 × + 2× + 3 = 6
6 2
⇒ a3 (Volume of unit cell)
Sol 4: (A) In close packed arrangement, side of the = 6.83 × 10−23 cm3
base = 2r
⇒ a= 4 × 10−8 cm = 4 × 10−10 m
⇒ RS = r
⇒ Surface area of unit cell
Also MNR is equilateral triangle, ∠PRS =
30º 2
= a= 1.6 × 10−19 m2
RS 3
In triangle PRS, cos30º
= = ⇒ Number of unit cells on 10–12 m2 surface
PR 2
Chem i str y | 19.59

10−12 (b) If x mm is the side of square and d is diameter of


= = 6.25 × 106 marble then maximum number of marbles on square
1.6 × 10−19
are with centres within square area can be known by
 There are two atoms (effectively) on one face of unit the following general formula :
cell 2
x 
Number of atoms on 10 –12
m surface = 2 × number of
2 N  + 1
=
d 
unit cell = 1.25 × 107.
⇒ x=7 Sol 10: (a) In rock-salt like crystal AB, there are four AB
units per unit cell. Therefore, density (d) is
Sol 8: The truncated octahedron is the 14-faced
4 × 6.023y
Archimedean solid, with 14 total faces : 6 squares and 8 d=
regular hexagons. The truncated octahedron is formed 6.023 × 1023 × 8y × 10−27
by removing the six right square pyramids one from [ a = 2y1/3 mm = 2y1/3 × 10–9 m]
each point of a regular octahedron as :
= 5 × 103 g/m3 = 5 kg/m3

(b) Since, observed density is greater than expected,


theoretical density, there must be some excess metal
occupying interstitial spaces. This type of defect is
known as metal excess defect.

Sol 11: In a cubic crystal system, there are two types of


voids known as octahedral and tetrahedral voids. If r1 is
the radius of void and r2 is the radius of atom creating
these voids then
Truncated octahedron
 r1  r 
  = 0.414 and  1  = 0.225
 r2 octa  r2 tetra
The above radius ratio values indicate that octahedral
void has larger radius hence for maximum diameter of
atom to be present in interstitial space:
r1 = 0.414 r2
Truncated octahedron unfolded
in two dimension Also in fcc,
4r2 = 2 a
Sol 9: 10 mm
⇒ Diameter required
a
(2r
= 1 ) (2r2 ) × 0.414
= × 0.414
2
400 × 0.414
= = 117 pm
2

40 mm Sol 12: From the given information, the number of


atoms per unit cell and therefore, type of unit cell can
(a) Side of square = 40 mm be known as
Distance of marble = 10 mm NM ρNA a3
ρ= ⇒ N=
NA a3 M
Number of marbles spheres along an edge of square
with their centres within the square = 5 (shown in 2 × 6 × 10 × (5 × 10−8 cm)3
23
= = 2 (bcc)
diagram) 75
Maximum number of marbles per unit are = 5 × 5 = 25 ⇒ In bcc,
1 9 . 6 0 | Solid State

4r = 3a Sol 17: (B, C) Conceptual Fact.

3 Sol 18: Coordination number of Al is 6.. It exists in ccp


⇒ r= a
4 lattice with 6 coordinate layer structure.
3
= 5 × 10−10 m
4 Sol 19: (D)
= 2.17 × 10−10 m = 217pm

Sol 13: A → p,s ; B → p,q; C → q; D → q,r

Crystals class Axial distances Angles


Cubic
a= b= c α = β = γ = 90

Tetragonal (
a = 2 2r )
a= b ≠ c α = β = γ = 90
2 × πr 2 2πr 2 π
Packing Efficiency
= = =
( ) 4
2
Orthorhombic
2 2r 8r 2
a≠b≠c α = β = γ = 90

Hexagonal
a= b ≠ c α = β = 90 1 1
Sol 20: (B) X = 8 × 6 × = 4
γ =90  8 2
1
M = 4 × +1= 2
Trigonal and 4
a= b= c α = β = γ ≠ 90
rhombohedral
So, unit cell formula of the compound is M2X4 and the
Monoclinic empirical formula of the compound is MX2
a≠b≠c α = β = 90
γ ≠ 90 Sol 21: (A) According to the given A + is present in
Triclinic the octahedral void of X − . The limiting raduius in
a≠b≠c α ≠ β ≠ γ ≠ 90 octahedral void is related to the radius of sphere as

r void = 0.414rsphere
Sol 14: (B)
rA+ = 0.414rX‒ =0.414 × 250pm =103.5 = 104pm

Sol 22: (A) In ccp lattice:


Number of O atoms → 4
Number of Octahedral voids → 4
Number of tatrahedral voids → 4
1 1
Total effective number of atoms = 12 × + 2 × + 3 = 6 Number of Al3+ = 4 × m
6 2
Number of Mg2+ = 8 × m
2
Sol 15: (A) Height of unit cell = 4r Due to charge neutrality
3
3 −( −
44( −
4 22) )+
2 ++4m
4m +( +
4m( + 33) )+
3 ++8n
8n +( +
8n( + 22) )=
2 0
00
=
=
( 2r )
2
Base area = 6 ×
4 1
11 = 1 11

=

=∴
=m
mm andn
and
andn
andn
==
Volume = height × base area = 24 2r 3 2
22 8
88

Sol 16: (D) Packing fraction = 74%


Sol 23: (B, C, D) Conceptual facts.
Empty space = 26%
2017-18 100 &
op kers
Class 12 T
By E ran culty
-JE Fa r
IIT enior emie .
S fP r es
o titut
Ins

CHEMISTRY
FOR JEE MAIN & ADVANCED
SECOND
EDITION

Exhaustive Theory
(Now Revised)

Formula Sheet
9000+ Problems
based on latest JEE pattern

2500 + 1000 (New) Problems


of previous 35 years of
AIEEE (JEE Main) and IIT-JEE (JEE Adv)

5000+Illustrations and Solved Examples


Detailed Solutions
of all problems available

Plancess Concepts
Topic Covered Tips & Tricks, Facts, Notes, Misconceptions,
Key Take Aways, Problem Solving Tactics
Solutions and Colligative
Properties PlancEssential
Questions recommended for revision
20. SOLUTIONS AND
C O L L I G AT I V E
PROPERTIES
1. INTRODUCTION
(a) Solutions are homogenous mixtures of two or more substances.
(b) We call what is being dissolved in the solution as a “solute”: for, it is the active ‘ingredient’ of that particular
solution.
(c) In the same way, in what we are dissolving the solute is termed as a “solvent”: Typically, there is more solvent
than any solute.
(d) Further, solutes and solvents can be technically in any state of matter.
Solutions that contain relatively high concentration of a solute are called concentrated solutions, whereas solutions
that contain relatively low concentration of a solute are called dilute solutions.
Let us now discuss saturated, unsaturated and supersaturated solutions.
Generally by saturated solution we mean a solution, which contains as much solute as it can hold particularly at a
given temperature. Similarly, a solution, which contains comparatively less solute than the quantity of solute it can
actually hold is called a unsaturated solution. Further, a solution is called a supersaturated solution, if it contains
comparatively more solute than the quantity it can actually hold at a particular temperature.
Binary solutions are such solutions that hold only two components.
We now proceed to discuss various types of solutions.

Table 20.1: Types of Solutions

S. No. Solute Solvent Type of solution Example


SOLID SOLUTIONS (solid solvent)
1. Solid Solid Solid in solid Alloy (brass, German silver, bronze, 22 carat gold, etc.)

2. Liquid Solid Liquid in solid Hydrated salts, amalgam of Hg with Na

3. Gas Solid Gas in solid Dissolved gases in minerals or H2 in Pd


LIQUID SOLUTIONS (liquid solvent)
4. Solid Liquid Solid in solid Salt or glucose or sugar or urea solution in water

5. Liquid Liquid Liquid in solid Methanol or ethanol in water

6. Gas Liquid Gas in solid Aerated drinks, O2 in water


2 0 . 2 | Solutions and Colligative Properties

S. No. Solute Solvent Type of solution Example


GASEOUS SOLUTIONS (gaseous solvent)
7. Solid Gas Solid in Gas Iodine vapors in air, camphor in N2 gas

8. Liquid Gas Liquid in Gas Humidity air, chloroform mixed with N2 gas

9. Gas Gas Gas in Gas Air (O2 + N2)

Out of the various types of solutions listed in the table given above, the most significant types of solutions are
those which are in liquid phase, i.e., liquid solutions. We therefore confine ourselves to the study of solutions of
solids, liquids or gases in liquids.

2. METHODS FOR EXPRESSING CONCENTRATION OF SOLUTIONS

Mass of solute
1. Mass percentage: % (w/w) Mass percentage of solute = × 100
Mass of solution

Mass of solute
2. Percent mass by volume: % (w/v) = × 100
Volume of solution

Volume of solute
3. Volume percentage: Percent of solute by volume = × 100
Volume of solution

Mass of solute
4. Percentage mass by volume: Percent of solute mass by volume = × 100
Volume of solution
5. Strength of concentration:
Mass of solute in gram Mass of solute in gram
Concentration of solution = = × 1000
Volume of the solution in litres Volume of the solution in mL

Mass of solute
6. Parts per million (ppm): = × 106
Mass of solution

7. Mole Fraction: Let us assume that n moles of solute (A) and N moles of solvent (B) are present in a solution.
n N
Then, mole fraction of the solute = = XA, whereas mole fraction of the solvent = = XB
N+n N+n
Hence, in a binary solution, XA + XB = 1.
Mole fraction is, therefore, independent of temperature of the solution.

Number of moles of solute


8. Molality: Molality (m) =
Number of kilograms of the solvent

Number of moles of solute


9. Molarity: Molarity (M) =
Number of litres of solution

Molarity × Number of liters of solution = Number of mole of solute

Number of gram equivalents of solute


10. Normality: Normality (N) =
Number of litres of the solution

Or Normality × Number of liters of the solution = Number of gram equivalents of the solute.
Chem i str y | 20.3

PLANCESS CONCEPTS

XB × 1000
Relation between molality and mole fraction =m
(1 − XB )mA

Molality is the most convenient form of representing concentration of a solution as it is independent of
temperature.
1 ρ m
Relation between molality and molarity = − B
m M 1000
where, ρ = density of solution; m = molality; M = molarity and mB = molar mass of a solute.
xB × 1000 × d
Relation between molarity and mole fraction M =
x AmA + xBmB
Relation between normality and strength
wB
Normality × Equivalent mass = = Strength of the solution in g/L
V
Relation between normality and molarity
Normality = n × Molarity
If volume V1 of a solution of normality N1 is mixed with volume V2 of another non-reacting solution of
normality N2, then the normality N3 of the final solution can be calculated as follows:
N1V1 + N2V2 = N3(V1 + V2) or N3 = (N1V1 + N2V2) / (V1 + V2)
Similarly, if molarities are used, then M3 = (M1V1 + M2V2) / (V1 + V2)
T.P. Varun (JEE 2012, AIR 64)

Illustration 1: The density of a solution containing 13% by mass of sulfuric acid is 1.09 g/mL. Calculate its molarity.
 (JEE MAIN)
Sol: We are provided with strength of solution and density hence for this numerical following formula can be
applied
In solving such problems, the following formula can be applied:
%strength of soln. × density of soln. × 10 13 × 1.09 × 10
Molarity = ;M= = 1.445 M
Mol. mass 98

Illustration 2: The density of a 3M sodium thiosulphate solution (Na2S2O3) is 1.25 g/mL. Calculate (i) the percentage
by mass of sodium thiosulphate, (ii) the mole fraction of sodium thiosulphate and (iii) molalities of Na+ and S2O3
ions.  (JEE ADVANCED)

Sol: From the given data determine the strength of the acid. In order to determine the mole fraction we have
to calculate the no of moles of acid and water present in the solution. From the number of moles calculate the
molalities of different ions.
x × d × 10 x × 1.25 × 10
M= ⇒3= ∴ x = 37.92
mA 158
474
No. of moles of Na2S2=
O3 = 3 ; mass of water = (1250 – 474) = 776g
158
776
No. of moles of water
= = 43.1
18
2 0 . 4 | Solutions and Colligative Properties

3 3
Mole fraction of Na2S=
2 O3 = = 0.065
43.1 + 3 46.1
No. of moles of Na+ ion = 2 × No. of moles of Na2S2O3 = 2 × 3 = 6

No. of moles of Na+ ions 6


Molality of Na+ ion = = × 1000 =7.73 m
Mass of water in kg 776

No. of moles of S2O32– ions = No. of moles of Na2S2O3


3
Molality of S2O32– ions = × 1000 =
3.86 m
776

3. SOLUBILITY
Solubility is defined as the maximum amount of solute that we can dissolve in any given solvent (yielding a
saturated solution). It not only depends on the nature of solute and solvent but also on temperature and pressure.

3.1 Solubility of a Solid in Liquid


Suppose that a solid solute is added continuously to a liquid solvent; then, the solute continues dissolving and
consequently the concentration of the solution increases. We call this process as dissolution. However, ultimately, a
state is reached where no more solute dissolves at the given temperature. This is basically because of the fact that
from the solution, the solute particles keep on colliding on the surface of solid solute particles and ultimately get
separated out of the solution. This process is known as crystallization. Thereafter, no more solute particles dissolve
because the rate of dissolution equalizes the rate of crystallization, i.e., a dynamic equilibrium is attained.
Solute + Solvent → Solution
Further, the solution at this stage is said to be a saturated solution. However, an unsaturated solution is one in
which more solute particles dissolve at the same temperature. Thus, the concentration of such a saturated solution
is called its ‘solubility.’ Now, as per our discussion so far in this regard, we define solubility as follows:
The solubility of a solid in a liquid at any temperature is defined as the maximum amount of the solid
(solute) in grams which can dissolve in 100g of the liquid (solvent) to form the saturated solution at that
particular temperature.
Factors affecting solubility of a solid in a liquid
(a) Nature of Solute and Solvent: Generally, a solid dissolves in a liquid which is chemically similar to it. We
can say this conveniently as ‘like dissolves like.’ In this regard, ionic compounds usually get dissolved in polar
solvents, whereas covalent compounds are soluble in non-polar solvents.

Explanation: For ionic compounds that are being dissolved in polar solvents, the solubility is due to the fact that
there are strong electrostatic forces of attraction between the ions of the crystal and the polar solvent molecules
with the negative ions being attracted with the positive poles of the solvent molecule and positive ions by negative
poles. However, in case of water molecule, the situation can be represented as

- + -
+ +
O
+ 104.5o +
Or
+
-+ + - + -+ +- +
H H + + + +
+ - + -+ -
+ - -
+ +
Water molecule - - +
+
- +
(polar molecule) + + + - +
+ - +
- +
- + -
+ + +
+ + -

Figure 20.1: Dissociation of an ionic compound, like Na+Cl‒, in a polar solvent (H2O)
Chem i str y | 20.5

Thus, the water molecules pull apart the ions of the crystal and the electrostatic forces of attraction between the
ions of the crystal are hence cut off. Further, we observe that the ions are surrounded by the water molecules which
act as a sheath (or envelope) around the ions and hence prevent the possible recombination of the ions. Such ions
that are thus moving freely in the solution are said to be hydrated. It is to be noted here that whereas energy is
required for the splitting of the ionic compound into ions (called lattice energy), energy is released when the ions
get hydrated (called hydration energy). It follows then that a substance dissolves if its hydration energy is greater
than its lattice energy, i.e., Dhyd H > Dlattice H.
It may be further mentioned here that whereas water is the best polar solvent (having the highest dielectric constant),
liquid ammonia, liquid hydrogen sulfide and liquid sulfur dioxide are also good solvents for ionic compounds.
In the case of non-polar compounds being dissolved in non-polar solvents, the solubility principle is due to similar
solute–solute, solute–solvent and solvent–solvent interactions.

(b) Temperature: The solubility of solutes is generally dependent on temperature. For example, when a solid
dissolves in a liquid, a change in the physical state of the solid, analogous to melting, takes place. However,
heat is required to break the bonds holding the molecules in the solid together. On the contrary, heat is
liberated during the formation of new solute–solvent bonds.

PLANCESS CONCEPTS

Solute + Solvent → Solution. ∆Hsolution = ±x


If ∆Hsolution < 0, i.e., (–ve), then the dissolution is exothermic. In this case, as the temperature increases,
solubility decreases (Le Chatelier’s principle).
If ∆Hsolution > 0, i.e., (+ve), then there is endothermic dissolution. In this case, increase in temperature
increases the solubility (Le Chatelier’s principle).
Rohit Kumar (JEE 2012, AIR 79)

3.2 Solubility of a Gas in a Liquid


The solubility of any gas in a particular liquid is the volume of the gas in cc (converted to STP) that can dissolve in
unit volume (1 cc) of the liquid to form the saturated solution at the temperature of the experiment and under a
pressure of one atmosphere. This method helps us to express the concentration as an absorption coefficient of the
gas and is usually represent by α.
Further, solubility of a gas in a liquid at a particular temperature is also expressed in terms of molarity (moles of
the gas dissolved per liter of the solvent to form the saturated solution, i.e., in terms of mol L–1) or in terms of mole
fraction (xA) of the gas.

(a) Nature of the gas and solvent: It is to be noted here that gases like H2, O2, N2, etc. dissolve in water only
to a small extent, whereas gases such as CO2, HCl and NH3 are highly soluble. However, the greater solubility
pattern of the latter gases is mainly due to their reaction with the solvent or chemical similarity.

(b) Effect of Temperature: The solubility nature of gases decreases with an increase in temperature as dissolution
is an exothermic process, i.e., it is accompanied by evolution of heat. Thus
Gas + Solvent → Solution + Heat
By applying Le Chatelier’s principle, it is clearly evident that increase of temperature would shift the equilibrium
in the backward direction, i.e., the solubility would decrease.
Exceptions – The solubility of some sparingly soluble gases, such as hydrogen and inert gases, increases
slightly with increase of temperature especially in the case of non-aqueous solvents such as hydrocarbons,
alcohols and acetone.
2 0 . 6 | Solutions and Colligative Properties

(c) Effect of Pressure (Henry’s law): As the pressure increases, the solubility also increases. We explain this
concept using the following example. As an example
of a solution of a gas in a liquid, consider a system Gaseous w1
as shown in Fig. 2 (a). In this figure, the lower part phase at
is the solution, whereas the upper part is gaseous pressure P,
w1 w2
at pressure p and temperature T. Now, let us temperature T
suppose that the system is in dynamic equilibrium,
i.e., rate of gaseous particles entering and leaving Solution
the solution is the same, which means that rate of phase
dissolution = rate of evaporation. Now, let us a b
increase the pressure over the system as shown in Figure 20.2: Demonstrating the effect pressure
Fig. 2 (b). Consequently, the gas gets compressed on solubility of a gas in liquid
to a smaller volume. Due to this effect, the total
number of gaseous particles per unit volume tends to increase. Because of this, the total number of gaseous
particles striking at the surface of the solution and hence entering into it also increases. The process is
continuous till a new equilibrium is reestablished. Thus, we learn that upon increase of the pressure of the gas
above the solution, the solubility also increases.
Quantitatively, this fact was also explained by Henry’s law which states that the solubility of a gas in a liquid is
directly proportional to the pressure of the gas over the solution at a definite temperature.
m ∝ P or m = Kp
In this regard, Dalton also concluded independently that, if a mixture of gases is simultaneously is in equilibrium
with the liquid at a particular temperature, then the solubility of any gas in the mixture is directly proportional
to the partial pressure of that gas in the mixture.
By expressing solubility in terms of mole fraction of the gas in the solution, for a gas A, Henry’s law can be
written as XA = K′PA
1 1
However, this can also be written as: PA = X A or PA = KH X A where, KH = is called Henry’s constant.
K' K'
Limitations of Henry’s law
Henry’s law is applicable only when
The pressure of the gas is not too high and temperature is not too low.
The gas should not undergo any chemical change.
The gas should not undergo association or dissociation in the solution.

Illustration 3: Why sealed soda water bottle upon opening shows the evolution of gas with effervescences? 
 (JEE MAIN)

Sol: CO2 gas is dissolved at high pressure in soda water bottle. On opening the seal, pressure becomes equal to
atmospheric pressure and the dissolved gas comes out from solution with effervescences because solubility of gas
decreases with decrease in pressure.

Illustration 4: NH3 is one of the few gases that do not obey Henry’s law. Suggest a reason. (JEE ADVANCED)

Sol: The solubility of NH3 in water also involves H-bonding and thus, solubility of NH3 in water depends on pressure
as well as tendency to show H-bonding.
Chem i str y | 20.7

PLANCESS CONCEPTS

Calculation of solubility of a gas at a particular pressure from the known solubility at some other pressure
(but at the same temperature).
From Henry’s law, we know that m = Kp. Hence, if m1 is the solubility of a gas at pressure P1 and m2 is its
m1 P1
solubility at pressure P2, then = .
m2 P2
Thus, by knowing m1 at P1, we can easily calculate m2 at P2. Further, the equation, m = KP also suggests
that the plot of pressure P vs. solubility m will be a straight line passing through the origin with slope = K.
However, the greater the value of K, the greater is the solubility. For example, O2 is more soluble than N2
at the same temperature and pressure as shown in the figure provided.

Aishwarya Karnawat (JEE 2012, AIR 839)

3.3 Solutions of Liquids in Liquids


Consider the case wherein one liquid dissolves in another. Here, the molecules of the solvent are forced to move
apart so as to accommodate the solute molecules. In the same way, the molecules in the solute must also be
separated so that they can assume their positions in the mixture. However, it is to be noted here that in both these
processes, energy is required. Further, as the molecules of both the solute and the solvent are brought together,
there is release of energy due to the attractive forces between them. Moreover, when the molecules of both the
solute and the solvent are strongly attracted to each other, more energy is released in the final step. There is a
possibility of three cases under such circumstances. Either the overall dissolution process results in evolution of
heat or absorption of heat, or energy is released in the final step of the reaction and is the same as the energy
absorbed in the first two phases, i.e., net change is zero.
Examples
1. Benzene and carbon tetrachloride No evolution or absorption of heat
2. Acetone and water Evolution of heat
3. Ethyl alcohol and water Absorption of heat
However, we must be aware of the fact that a liquid may or may not be soluble in another liquid. Therefore, purely
based upon the relative solubility of a liquid in another, the following three cases are possible:
Miscible liquids form three types of solution, which can be ideal or non-ideal solutions.

Liquids-Liquid System

1. Liquids that are 2. Liquids that are 3. Liquids that are


completely miscible. partially miscible. completely immiscible.
Examples: Examples: Examples:
Benzene and toluene, Ether and water; Benzene and water;
Ethyl alcohol and phenol and water; Carbon tetra-chloride
water, carbon tetra- Nicotine and water. and water; Benzene
chloride and benzene. and alcohol.

Flowchart 20.1: Types of liquid-liquid system

Miscible liquids form three types of solutions, which can be ideal or non-ideal solutions.
2 0 . 8 | Solutions and Colligative Properties

4. IDEAL SOLUTION AND RAOULT’S LAW

4.1 Ideal Solution


By an ideal solution, we mean the one that obeys Raoult’s law (we discuss this law hereunder) for a wide range of
concentrations and at a specified temperature. Some examples in this regard include
(1) Mixture of methanol and ethanol;
(2) Mixture of n-hexane and n-heptane and
(3) Mixture of benzene and toluene.

Characteristics
(a) Volume change on mixing should be zero, =
i.e., ∆Vmix 0; VSolvent
= + Vsolute Vsolution
(b) Heat change on mixing should be zero, i.e., ∆Hmix =
0 (heat is neither absorbed nor evolved)
(c) Solute must not undergo association or disociation in solution
(d) There should be no chemical reaction between the solute and solvent

Things to remember
(a) If FA − A is the force of attraction between molecules of A and FB–B is that of molecules of B, then A and B will
form an ideal solution only if, F= A −B F=
A−A FB −B
.
(b) The solution of liquids A and B will be ideal if A and B have similar structures. Further, both methanol and
ethanol have the same functional group and almost same polarity and therefore, form ideal solutions.

4.2 Non-Ideal Solution


A solution which does not obey Raoult’s law is called a non-ideal solution.
For a non-ideal solution.
(a) Raoult’s law is not obeyed, i.e., PA ≠ PA0 x A and PB ≠ PB0 xB .
(b) ∆Hmix ≠ 0 and
(c) ∆V ≠ 0
mix

4.3 Raoult’s Law


This law states that the partial pressure of any volatile constituent of a solution at a constant temperature is equal
to the vapour pressure of pure constituent multiplied by the mole fraction of that constituent in the solution.
Let us assume that a mixture is prepared containing nA and nB moles of liquid A and B, respectively. Further, let PA
and PB be the partial pressures of A and B, respectively and PA0 and PB0 the vapour pressure pure state.
Thus, according to Raoult’s law,
nA
=PA = P0 mole fraction of =
A × PA0 X APA0
nA + nB A
nB
and
= PB = P0 mole fraction of =B × PB0 XBPB0
nA + nB B
If the total pressure be P, then
nA nB
P PA + PB =
= PAo + PBo Thus, =P X APA0 + XBPB0
nA + nB nA + nB
.
Chem i str y | 20.9

PLANCESS CONCEPTS

This law is an important governing factor in deciding whether a solution is ideal or nonideal. Solutions
obeying Raoult’s law over a wide range of concentrations are ideal; otherwise, they are nonideal.
A non-ideal solution can either show positive or negative deviation from Raoult’s law.

Nikhil Khandelwal (JEE 2009, AIR 94)

4.4 Ideal and Non-Ideal Solutions


Table 20.2: Differences between ideal and non-ideal solutions

Ideal solutions Non-ideal solutions


Positive deviation from Raoult’s law Negative deviation from Raoult’s law
1. Obey Raoult’s law at every range of Do not obey Raoult’s law Do not obey Raoult’s law
concentration

2. ∆Hmix = 0; neither heat is evolved ∆Hmix > 0; endothermic dissolution; ∆Hmix < 0; exothermic dissolution;
nor absorbed during dissolution heat is absorbed heat is evolved

3. ∆Vmix = 0; total volume of solution


is equal to sum of volumes of the ∆Vmix > 0; volume is increased after ∆Vmix < 0; volume is decreased
components dissolution during dissolution

4. P = PA + PB = PA0 X A + PB0 XB
0 PA > PA0 X A ; PB > PB0 XB PA < PA0 X A ; PB < PB0 XB
i.e.,
= PA P=
A X A ; PB PB0 XB
∴ PA + PB > PA0 X A + PB0 XB ∴ PA + PB < PA0 X A + PB0 XB
5. A – A, A – B, B – B interactions
should be same, i.e., ‘A’ and ‘B’ are A – B attractive force should be weaker A – B attractive force should be greater
identical in shape, size and character than A – A and B – B attractive forces. than A – A and B – B attractive forces.
‘A’ and ‘B’ have different shape, size ‘A’ and ‘B’ have different shape, size
6. Escaping tendency of ‘A’ and ‘B’
and character. and character.
should be the same in pure liquid and
in the solution ‘A’ and ‘B’ escape easily showing Escaping tendency of both
higher vapor pressure than the components ‘A’ and ‘B’ is lowered
expected value showing lower vapor pressure than
Examples
expected ideally
Dilute solutions; benzene + toluene
Examples Examples

Acetone + ethanol; acetone + CS2 Acetone + aniline; acetone+


chloroform

4.5 Relation Between Dalton’s Law and Raoult’s Law


In this regard, we can calculate the composition of the vapour in equilibrium with the solution by applying Dalton’s
law of partial pressures. Let us assume that the mole fractions of vapours A and B be YA and YB respectively.
,
Further, let PA and PB be the partial pressures of vapours A and B, respectively and the total pressure be P.
PA = YAP  …(i)

PB = YBP  …(ii)

PA = X APA0 …(iii)

PB = XBPB0  …(iv)

2 0 . 1 0 | Solutions and Colligative Properties

Now, by equating (i) and (iii), we obtain


X APA0 PA
YAP = X APA0 or
= YA =
P P
XBPB0 PB
Similarly, by equating (ii) and (iv), we obtain
= YB = .
P P
Thus, it is clear to us that in the case of an ideal solution the vapor phase is richer with more volatile component,
i.e., the one having relatively greater vapor pressure.

Illustration 5: At 300 K, the vapor pressure of an ideal solution containing one mole of A and 3 moles of B is
550 mm of Hg. At the same temperature, if one mole B is added to this solution, the vapor pressure of solution
increases by 10 mm of Hg. Calculate the VP of A and B in their pure state. (JEE MAIN)

∵ PM0 PA0 ·X A + PB0 ·XB (from Raoult’s law)


Sol: =

 1  0 3 
PA0 × 
Case I: 550 =  + PB   … (i)
1 + 3  1 + 3 
 1  0 4 
PA0 × 
Case II: On addition of one mole of B in liquid mixture 560 =  + PB   … (ii)
1 + 4  1 + 4 
Solving eqs. (i) and (ii) PA0 = 400 mm, PB0 = 600 mm

Illustration 6: The vapor pressure of ethanol and methanol are 44.5 mm and 88.7 mm Hg, respectively. An ideal
solution is formed at the same temperature by mixing 60 g of ethanol with 40 g of methanol. Calculate the total
vapour presure of the solution and the mole fraction of methanol in the vapor. (JEE ADVANCED)

Sol: Total vapour pressure is a sum of of partail preesure of A and B and partial pressure is given by product of
mole fraction and vapour pressure in pure form of A and B. So first we have to find out the mole fraction of A and
B and than partial pressure of A and B.
Mole fraction of a compound in the vapour is given by Partial pressure of that compound divided by total vapour
pressure of the solution.
60
Mol. mass of ethyl alcohol = C2H5OH = 46; No. of moles of methyl alcohol = = 1.304
46
40
Mol. mass of methyl alcohol = CH3OH = 32; No. of moles of methyl alcohol = = 1.25
32
1.304
‘XA’, mole fraction of ethyl alcohol = = 0.5107
1.304 + 1.25
1.25
‘XB’, mole fraction of methyl alcohol = = 0.4893
1.304 + 1.25
Partial pressure of ethyl alcohol = X A ·PA0 = 0.5107 × 44.5 = 22.73 mm Hg
Partial pressure of methyl alcohol = XB ·PB0 = 0.4893 × 88.7 = 43.40 mm Hg
Total vapour pressure of solution = 22.763 + 43.40 = 66.13 mm Hg
Partial pressure of CH3OH 43.40
Mole fraction of methyl alcohol in the vapour = = = 0.6563
Total vapour pressure 66.13

5. THEORY OF FRACTIONAL DISTILLATION


Fractional distillation is the process of separation of one liquid from another (binary mixture) having different
boiling points by distillation. However, the information regarding can a particular solution of two liquids can be
separated by distillation or not, is provided by the study of liquid–vapor equilibrium diagrams at constant pressure,
say, e.g., atmospheric pressure. The separation is possible only when the vapor phase has a composition different
from that of the boiling liquid mixture.
Chem i str y | 20.11

5.1 Azeotropes and Azeotropic mixture


Azeotropes are binary mixtures having the same composition both in liquid and vapor phase and boil at a constant
temperature.

Types of Azeotropes
Minimum Boiling Azeotropes: These are the binary mixtures whose boiling point is less than either of their
two components. The non-ideal solutions show a large positive deviation from Raoult’s law for minimum boiling
azeotrope at a specific composition, e.g., a mixture of 94.5% ethyl alcohol and 4.5% water by volume.
Maximum boiling Azeotropes: These are the binary mixtures whose boiling point is more than either of their two
components. The solutions show large negative deviation from Raoult’s law for maximum boiling azeotrope at a
specific composition, e.g., a mixture of 68% HNO3 and 32% H2O by mass.

5.2 Liquid Solutions of Type I (Ideal Solutions)


If a solution of composition x is heated, then
(a) Vapor phase will be richer in B and A will have a composition x1 in
Vapor
the distillate, whereas the residue will become richer in A and let it TA Liquid
have a composition y.

Boiling point
Ty
(b) Now, if this liquid is heated again, the residue will be even more Tx
richer in A and if this process is repeated sufficient number of times, Tz
then pure A will be obtained from the residue. TB

(c) Similarly, if the distillate is recondensed and then heated again, then A 100% y x y1 x1z1 B 100%
the distillate will be richer in B and on continuing the process, pure B 0% Composition A 0%
B will be obtained from the distillate.
Figure 20.3 a: Composition of solution
(d) Thus, we can separate two liquids that form an ideal solution by showing idea behavior
fractional distillation.

5.3 Type II Solutions (Positive Deviations from Raoult’s Law)


If a mixture of composition x is heated, then
(a) Vapor will have a composition x1 and the composition of residual
liquid will shift toward A while the composition of distillate shifting
toward C
A
Boiling point

Va
po

B
(b) On repeating fractional distillation, pure liquid A is obtained as
r
po
r

Liq
Va

a residue while mixture of composition C (minimum boiling


uid
T uid
Liq
azeotropic mixture) is obtained as a distillate. C

(c) However, pure B cannot be obtained.


A 100% x x1 y1 y B 100%
B 0% Composition A 0%
If a mixture of composition y is heated, then
Figure 20.3 b: Composition of solution
(i) Vapor will have composition y1 and residual liquid composition showing positive deviation from raoults law
will shift toward C.
(ii) Ultimately, mixture of composition C is obtained as a distillate and pure liquid B is obtained as A residue
(iii) However, it is not possible to obtain pure liquid A.

5.4 Type III Solutions (Negative Deviations from Raoult’s Law)


When a mixture composition‘a’ is taken, then on boiling
(a) Distillate ‘a’ is obtained, which is richer in component A
2 0 . 1 2 | Solutions and Colligative Properties

(b) Residue hence obtained is ‘b.’ Further, successive distillation D


V
of ‘b’ leads to azeotropic solution D (+ve boiling azeotropic V
mixture). L TB0
B
(c) Successive distillation of ‘a’ will lead to pure (A). TA0 L

When a mixture composition 'c' is taken and boiled, then


a’ a b c c’
(i) Distillate obtained is ‘c’, which is richer in component B and
successive distillation of distillates leads to solution becoming A =1 d e B =1
pure ‘b.’ Mole fraction
Figure 20.3 c: Mole fraction of solution
(ii) The residue of ‘c’ is ‘e.’ Further, successive distillation of
showing negative deviation from raoults law
‘e’ makes the residue richer in ‘D’ which is +ve boiling
azeotropic mixture.

6. COLLIGATIVE PROPERTIES OF DILUTE SOLUTIONS


A dilute solution is one in which the amount of the solute is very small in comparison to the amount of the solvent.
A dilute solution containing non-volatile solute exhibits some special properties which depends only upon the
number of solute particles present in the solution irrespective of their nature. These properties are termed as
colligative properties and are listed hereunder.
1. Lowering in the vapor pressure,
2. Elevation in the boiling point,
3. Depression in the freezing point and osmotic pressure.
4. Osmotic pressure.
Further, dilute solutions also obey Raoult’s law.

PLANCESS CONCEPTS

•• For applying the formulae of colligative properties, the following conditions should be satisfied:
•• The solution should be very dilute.
•• The solute should be nonvolatile.
•• The solute does not dissociate or associate in solution.
Vaibhav Krishnan (JEE 2009, AIR 22)

6.1 Lowering in the Vapor Pressure


When a non-volatile solute is present in a solution, then the vapor pressure of the solution will be less than the
vapor pressure of pure solvent.
The vapor pressure may be lowered due to the reasons listed hereunder.
(a) Percentage surface area occupied by the solvent decreases. Thus, the rate of evaporation and vapor pressure
decreases. The solute molecules occupy the surface, and therefore the percent surface area occupied by the
solvent decreases.
1
(b) According to Graham’s law of evaporation, rate of evaporation ∝
density
Further, when a non-volatile solute is dissolved in a liquid, its density increases. Thus, both rate of evaporation and
vapor pressure are lowered.
Chem i str y | 20.13

Now, let us consider the case in detail. 


Only ‘A’ particles
Let there be a solution of solute B in solvent A.
Since ‘B’ is nonvolatile, it would not go to the vapor phase; hence, the Solvent (A)
vapors above the solution will be only due to volatile solvent ‘A.’ Since
Solute (non volatile) (B)
χ APA0 ⇒ PA < PA0 so vapor pressure is lowered.
PA =
Figure 20.4: Lowering in the Vapor Pressure
For a binary system, χA + χB = 1.
Substituting the value of χA
⇒ PA = PA0 (1 – χB); PA = PA0 – PAoχB
⇒ PA0 χB = PA0 – P = DP (relative lowering in vapor pressure); DP = PA0 χB ⇒ DP ∝ xB
⇒ Lowering in vapor pressure is directly proportional to the mole fraction of a solute. Further, the relative lowering
in vapor pressure is equal to the mole fraction of solute.
∆P PA0 − P nB
= χB , which can also be written as: =
PA0 PA0 nA + nB
or we can rephrase it as ,
P0 − Ps nB
according to Raoult’s law, =
P0 nB + nA
where, P0 = vapor pressure of pure solvent; Ps = vapor pressure of solution;
  n2 = number of moles of solute; and n1 = number of moles of solvent.

PLANCESS CONCEPTS

•• Lowering in vapor pressure is directly proportional to the mole fraction of solute in the solution.
•• Relative lowering in vapor pressure is equal to the mole fraction of solute.
•• Determination of molecular mass from lowering of vapor pressure.

Neeraj Toshniwal (JEE 2009, AIR 21)

Po − Ps nB
We know that, according to Raoult’s law, = .
P o nB + nA
If W2 g of the solute is dissolved in W1g of the solvent and if M2 and M1 are their respective molecular masses, then
w w
we have nB = B and nA = A .
MA MA
By substituting these values in the above expression, we obtain
Po − Ps wB / MB
=  … (i)
P o w A / MA + wB / MB
However, for a dilute solution, n2 can be neglected in comparison with n1 so that Raoult’s law equation becomes
Po − Ps nB Po − Ps wB / MB Po − Ps wB × MA
≡ or ≡ or =
P o nA P o w A / MA P o w A × MB
From expressions (i) and (ii), it is clear that if w2g of the non-volatile solute is dissolved in w1g of the solvent and the
vapor pressure of the pure solvent (Pº) and that of the solution (Ps) are measured experimentally, then by knowing
the molecular mass of the solvent (M1), that of the solute (M2) can be calculated.
2 0 . 1 4 | Solutions and Colligative Properties

Actually, this is not a preferred method because other methods (namely, elevation in boiling point or depression in
freezing point) give results more easily and accurately.

Determination of Relative Lowering in Vapor Pressure


We can determine the relative lowering in vapor pressure by using Ostwald and Walker method. In this method, a
stream of dry air is passed successively through a set of weighed bulbs containing the solution and then through a set
of merged bulbs containing pure solvent and finally through weighed U-shaped tubes containing anhydrous CaCl2.
u2 + Rg u2
<
3g 2g
(a) Let the loss in weight of solution bulbs be w1 gm
(b) Loss in weight of solvent bulbs be w2 gm
(c) Loss in weight of the solution bulbs (w1) ∝ vapor pressure of the solution
(d) Loss in weight of the solvent bulbs (w2) ∝ vapor pressure of the solvent – vapor pressure of the solution.
⇒ Ps ∝ w1 and (P0 – Ps) ∝ w2
Po − Ps w2 Loss in weight of solvent bulbs
⇒ = = = XB
P o w1 + w2 Total loss in weight of soltuion bulbs and solvent bulbs
i.e., mole fraction of the solute.

Vapor Pressure of Liquid and Temperature


Liquid  Vapor ∆Hvap > 0
Basically, vapor pressure of liquid increases with temperature. Here, ∆H is enthalpy of vaporization. We provide a
graphical representation of vapor pressure versus temperature hereunder.
Vapor pressure

log,P

Temperature 1/T

Figure 20.5: Variation of vapor pressure with temperature

Generally, if vapor pressure of a liquid is known at a temperature, then it can be calculated at another temperature
P  ∆Hvap.  1 1 
using Clausius–Clapeyron equation: log10  2  =  − 
 P1  2.303R  T1 T2 
Here, P1 = vapor pressure at temperature T1 and P2 = vapor pressure at temperature T2.

6.2 Elevation in Boiling Point (Ebullioscopy)


We know that the boiling point of a liquid is the temperature at which its vapor pressure is equal to the atmospheric
pressure. Further, the vapour pressure of a liquid is generally lowered upon addition of a non-volatile solute to it.
Hence, the temperature of the solution when its vapor pressure will be equal to atmospheric pressure will be higher
than the temperature of the pure solvent. In other words, we mean that the boiling point of the solvent is elevated
by the addtion of a non-volatile solute. However, the difference in the boiling point of the solution and the boiling
point of the pure solvent is termed as elevation of the boiling point.
Chem i str y | 20.15

Elevation of boiling point, (DT) = Boiling point of the solution – Boiling point of pure solvent
This can be better understood by ploting a graph of vapor pressure vs. temperature for a pure solvent and two
solutions of different concentrations. In such a graph, the curves for solution will always lie below the curves for
pure solvent.
In the graph provided hereunder, P0E represents atmospheric pressure, TB0, T1 and T2 are boiling points for pure
solvent, solution-1 and solution-2, respectively. Similarly, the vapor pressures of solution-1 , solution-2 and pure
solvent at temperatures TB0 are P1, P2 and P0, respectively.
Relationship for elevation in boiling points.

0
P E - Atmospheric Pressure

0 1 atm A D E
P
Vapour
Pressure P TB0=Boiling Point
1
temperature
t
en

1
n-
lv
tio
So

P2 2
n-
lu
So

tio
lu
So

TB0{ T2 T1
TB
Temperature

Figure 20.6: Elevation in Boiling Point

Let us consider the graph of vapor pressure vs. temperature (in Kelvin scale). Here, triangle ADB and ACE are similar.
Hence,
AD AB
=
AE AC
T1 − T0 P0 − P1
= ⇒ T1 – T0 ∝ P0 – P1
0 0
T2 − T P − P2
⇒ DTB ∝ DP. Also, DP ∝ XB ⇒ DTB ∝ XB
nB nB
DTB ∝ ; DTB ∝ [dilute solutions]
nA + nB nA
WB MA WB
DTB ∝ × ⇒ ∆TB =
Kb
MB WA MB × WA
WB
When = 1 mole and WA = mass of solute in kilogram (kg)
MB
⇒ ∆TB = Kb × m m = molality of solution, Kb = ebullioscopic constant
∆TB ∝ Kb [when concentration is kept constant].
Hence, elevation in boiling point depends upon KB, i.e., nature of solvent.
∆TB ∝ m [when solvent taken is same]
Kb is defined as the elevation in boiling point of a solvent when 1-g mole of a non-volatile solute is dissolved in
1000 g of solvent.
RTb2
Kb = 1v = latent heat of vaporization per gram of the solvent.
1000 lv
RTb2 × M0
Also, Kb = where M0 = molecular weight of solvent
1000 × ∆Hv
2 0 . 1 6 | Solutions and Colligative Properties

PLANCESS CONCEPTS

Kb Wsolute Kb WB
=∆TB × 1000 =
∆TB × 1000
Msolute × Wsolvent MB × WA
WB = Wsolute = Mass of solute taken
MB = Msolute = Molecular weight of solute
WA = Wsolvent = Mass of solvent taken in grams
Kb WB
⇒ = Msolute × 1000
∆TB × WA

TB(1) Kb(1)
Also, = when concentration is same but solvent different
∆TB(2) Kb(2)
∆TB(1) m1
= when solvent is same but concentration different
∆TB(2) m2
∆TB(1) MB(2) 1
= ⇒ DTB ∝ when solvent and amount of solute is same
∆TB(2) MB(1) MB(1)

B Rajiv Reddy (JEE 2012, AIR 11)

6.3 Depression in Freezing Point (Cryoscopy)


Freezing point of a substance is defined as the temperature at which the vapor pressure of its liquid is equal to the
vapor pressure of the corresponding solid. This is primarily because the addition of a non-volatile solute always
lowers the vapor pressure of a solvent; therefore, it will be in equilibrium with solid phase at a lower pressure and
hence at a lower temperature. Further, the actual difference between the freezing points of the pure solvent and its
solution is called depression of freezing point. Thus
Depression of freezing point (DT) = Freezing point of the solvent – Freezing point of the solution
Here again,
DTf ∝ DP
DP ∝ molality or DP ∝ XB
nB
DTf ∝ molality DTf ∝ XB ⇒ DTf ∝ for dilute solutions
nA
⇒ DTf = Kf WB when WA is in kilogram ⇒ ∆Tf =
Kfm
MB WA
Kf = molal depression constant or Cryoscopic constant.
Kf is defined as the depression in freezing point of a solvent when 1 g mole of a non-volatile solute is dissolved
RTf2 RTf2 M0
in 1000 g of solvent, i.e., K f = also K f =
1000lf 1000∆Hf
where Tf = freezing point and Tf = latent heat of fusion per gram
RTf × w
Also, ∆Tf
= solute
× 1000
M solute× W solvent

Hence, molecular weight of solute


Kf × W solute
Msolvent
= × 1000
∆Tf × W solvent
Chem i str y | 20.17

Since DTf = Kfm ⇒ DTf ∝ Kf [when concentration is same but different solvents are taken]
DTf ∝ m [when solvent is same but different concentration are taken]

6.4 Osmotic pressure


Semipermeable Membrane: We define a semipermeable membrane as “A membrane which allows the solvent’s
molecules to pass through it but prevents the passage of solute molecules through it.”

Osmosis: The phenomenon of osmosis was first observed by Abbe Nollet. It may be defined as a process in
which pure solvent molecules pass through a semipermeable membrane from a solution of low concentration to
a solution of higher concentration.
(a) Further, exosmosis is the mechanism of outward flow of water from a cell containing an aqueous solution
through a semipermeable membrane, e.g., grapes in NaCl solution.
(b) On the other hand, endosmosis is the mechanism of inward flow of water into a cell containing an aqueous
solution through a semipermeable mebrane, e.g., grapes in water.
Therefore, if the solution of density d rises to height h, then osmotic pressure is expressed as
π = h × d × g, where g is the acceleration due to gravity.
Osmotic pressure may also be defined as the excess pressure which must be applied to a solution in order to
prevent flow of solvent into the solution through the semipermeable membrane.
However, we can define osmotic pressure in several other ways also as listed hereunder.
(a) Osmotic pressure is the excess pressure which must be applied to a given solution in order to increase its
vapour pressure until it becomes equal to that of the solution.
(b) Osmotic pressure is the negative pressure when a solution is separated from the solvent by a semipermeable
membrane.
(c) Osmotic pressure is the hydrostatic pressure produced when a solution is separted from the solvent by a
semipermeable membrane.

Differences between Osmosis and Diffusion


Let us now proceed to understand the differences between the processes of osmosis and diffusion as listed
hereunder.
(a) In the process of diffusion, both the solute as well as the solvent molecules flow in opposite directions while
in the process of osmosis the flow of solvent molecules is unidirectional.
(b) For osmosis to take place, a semipermeable membrane is essential while it is not so for diffusion.

6.4.1 Van’t Hoff Theory of Dilute Solutions


Dilute solutions behave like ideal gases and gas laws obeyed by ideal gases are also obeyed by dilute solutions.
We know that
1
V∝ (Boyle–van’t Hoff law); V ∝ T (Charles–van’t Hoff law)
π
V ∝ n (Avogadro–van’t Hoff law)
n× T nRT
⇒ V ∝ ⇒ V= R = solution constant (0.0821 it atm K–1 mol–1)
π π
n
⇒ π= RT ⇒ π = CRT
V
Hence, at a given temperature, π ∝ C
2 0 . 1 8 | Solutions and Colligative Properties

Pressure–Temperature law (Gay-Lussac–van’t Hoff law)


Concentration remaining same, the osmotic pressure of a dilute solution is directly proportional to its absolute
temperature (T), i.e., π ∝ T
π π
or = constant or = constant
T T

PLANCESS CONCEPTS
n
• pV = nRT or pV = nST or π = ST or π = CST
V
where π = osmotic pressure in atmospheres
n = number of moles of the solute present in V liters of the solution
C = concentration of the solution in moles per liter
T = temperature in K
R or S = 0.08211 atm deg–1 mole–1

w  w  wST
• pV = ST  n =  ; M0 = (M0 = molecular weight of solute)
M0  M0  πV
where w = Wt. of solute in g dissolved in V1 of solution.
m = molecular wt. of the solute; π = hdg
• In isotonic solutions, since osmotic pressure p is equal, their concentration (C) must also be equal,
n1 n2 w1 w2
i.e., = ; =
V1 V2 m1 V1 m2 V2
• If a number of solutes is present in the solutions and p1, p2, p3, etc. are their individual osmotic
pressure, then the total osmotic pressure = p1 + p2 + p3 + ….
T P Varun (JEE 2012, AIR 64)

6.4.2 Reverse Osmosis


Generally, we understand that if a pressure higher than osmotic pressure is applied on a solution, then the solvent
will flow from the solution into the pure solvent through the semipermeable membrane. However, here as the flow
of solvent is in the reverse direction to that observed in the usual osmosis, we call this process as reverse osmosis.
This process is extensively used in the desalination of sea water to obtain pure water.

6.4.3 Isotonic Solutions


A pair of solutions having the same osmotic pressure are known as isosmotic or isotonic solutions. However, if two
such solutions are separated by a semipermeable membrane, then there will be transference of solvent from one
solution to the other. Thus, isosmotic solutions that are separated using a semipermeable membrane are known as
isotonic solutions. Further, isotonic solutios have the same concentration.
For example, 0.85% NaCl solution is found to be isotonic with blood, while 0.9% NaCl solution isotonic with
human RBCs. On the contrary, a solution that is having lower or higher osmotic pressure than the other is said
to be hypotonic or hypertonic, respectively with respect to other solution. (i) When placed in water or hypotonic
solutions, cells swell and burst and this process is known as hemolysis. (ii) When placed in hypertonic solutions,
the fluid from the plant cells comes out and thus the cells contract in size. This process is known as plasmolysis.
Further, when excess of fertilizers (like urea) is applied, then plasmolysis takes place and hence plants dry up (wilt).
Chem i str y | 20.19

Illustration 7: Estimate the boiling point of a solution of 25.0 g of urea (NH2CONH2) plus 25.0 g of thiourea
(NH2CSNH2) in 500 g of chloroform, CHCl3. The boiling point of pure chloroform is 61.2°C, Kb of chloroform =
3.63 km–1.  (JEE MAIN)

Sol: Molality is given by moles of Solute by mass of solvent in Kg. since here two solute are used we have to first
calculate the mole of urea and thiourea used. The sum of the two solute will give us total moles of solute which can
be used to calculat the molality of the solution.
Mass of urea 25.0g
Moles of urea = = = 0.42 mol
Molecular mass of urea 60g / mol
25.0g
Moles of thiourea = = 0.33mol
76g / mol
\ Total moles of solute = 0.42 + 0.33 = 0.75
Moles of solute 0.75mol
Molality, m = = = 1.50 m
Mass of solvent in kg (500g / 1000g)kg

DTb = Tb – Tbo =Kb.m = 3.63 × 1.50 = 5.44 K = 5.445ºC


Tb = 5.445ºC + Tbo = 5.445ºC +61.2ºC = 66.645ºC

Illustration 8: Consider a vertical tube of cross-section area of 1 cm2. The bottom of the tube is closed with a
semipermeable membrane and 1 g of glucose is placed in the tube. The closed end of the tube is immersed in pure
water. What will be the height of the liquid level in the tube at equilibrium? The density of solution may be taken
as 1 g/cm2. What is the osmotic pressure at equilibrium at 25ºC? Assume negligible depth of immersion of tube.
 (JEE ADVANCED)

Sol: Since here we have to calculate the osmotic pressure, we can use the following relation pV = nRT
Volume term is not provided instead cross section area of the tube is given so from this we can calculate the
volume term.
Let height of the tube = h cm
V = (h × 1) cm3 ∵ cross-sectional area = 1 cm2
 h×1  1 134.92
pV = nRT; π ×   = × 0.0821 × 298; π = atm
 1000  180 h

134.92 h×1 9.8


π = h × d × g; = × , 1 atm = 101.325 KPa
h 100 101.325
h = 375 cm = 3.75 m
π = h × d × g = 3.75 × 1 × 9.8 = 36.7 KPa

Illustration 9: A solution containing 0.2563 g of nephthalene (molecular mass = 128) in 50 g of carbon tetrachloride
yields a boiling point elevation of 0.201ºC while a solution of 0.6216 g of an unknown solute in the same mass of
the solvent gives a boiling point elevation of 0.647ºC. Find the molecular mass of the unknown solute.
 (JEE MAIN)
∆Tb × W × m
Sol: From the given dataa first calculate Kb by using following formula Kb = ;
1000 × w
Now by using the value of Kb find out the molecular mass of the unknown solute.
We know that,
∆Tb × W × m 0.201 × 50 × 128
Kb = ; for CCl4, Kb = = 5.019
1000 × w 1000 × 0.2563
2 0 . 2 0 | Solutions and Colligative Properties

1000 × 5.019 × 0.6216


Kb is now used in the second part of the problem; m = = 96.44
0.647 × 50

Illustration 10: In a cold climate, water is frozen causing damage to radiator of a car. Ethylene glycol is used as an
antifreezing agent. Calculate the amount of ethylene glycol to be added to 4 kg of water to prevent it from freezing
at –6ºC (Kf for water = 1.85 K kg mol–1).  (JEE ADVANCED)

Sol: Here we are provided with the value of DT ,W ,m and Kf so we can calculate th weight by using the following
formual:
m × M × ∆Tf m × M × ∆T 62 × 4000 × 6
w= Given, DT = 6ºC, W = 4kg = 4000g, m = 62, Kf = 1.85; w = = = 804.32 g
1000 × K f 1000 × K f 1000 × 1.85

6.5 Abnormal Molar Masses


Association : Association of molecules generally leads to decrease in the number of particles in a solution and
this results in a decrease in the value of colligative property. As we are aware, colligative property is inversely
related to the molecular mass. Therefore, obviously a higher value is obtained for molecular mass. For example,
when ethanoic acid is dissolved in benzene, it undergoes dimerization and shows a molecular mass of 120 (normal
molecular mass is 60).
O --- H – O
2CH3 — COOH H3C – C C — C H3
O – H --- O

Dissociation : In contrast to association, dissociation leads to an increase in the number of solute particles in a
solution and this results in an increase in the value of colligative property. Further, since colligative property is
inversely related to the molecular mass molecular mass of such a substance as calculated from colligative property
will therefore be less than its normal value. For example, we know that KCl is an electrolyte. However, when
it is dissolved in water it dissociates into K+ and Cl– ions and there would be double the number of particles if there
+ −
is complete dissociation. Hence, it is expected to have molecular mass 37.25 g = 74.5/2. KCl  
 K + Cl
To account for the above anomalies, van’t Hoff therefore introduced a factor ‘i’ in the van’t Hoff equation (pV = RT)
of osmotic pressure. Accordingly, the modified equation may thus be written as pV = iRT. The factor ‘i’ is defined
Observed osmotic pressure
by the expression, i =
Normal osmotic pressure
In this regard, the experimental value of i can be obtained by dividing the observed osmotic pressure of the solution
under the study with that of the solution of a normal substance (say, e.g., sucrose) of the same concentration
(molarity) as that of the substance under study.
Now, since osmotic pressure behaves like other colligative properties, the factor i can also be applied to other
colligative properties. Thus, in general,
Actual number of particles Observed colligative property (experimental)
i= ; i=
No. of particle when no ionisation/association Normal colligative property(calculated)

Now, since molecular weight of a solute is inversely proportional to the colligative property the factor ‘i’ may also
be defined in the following way.

Normal molecular wt. (calculated value)


i=
Observed molecular wt.(experimental value)

Evidently, for a normal substance (which neither dissociates nor associates in solution), the factor ‘i’ is always unity.
When the van’t Hoff factor is included, then the colligative properties get modified shown hereunder.
Chem i str y | 20.21

(a) Relative Lowering in Vapor pressure


P0 − Ps
= i × XB where i = van’t Hoff factor
P0
P0 − Ps nB
XB = mole fraction of solute for dilute solution = i× ; nB = number of moles of solute; nA = number of
moles of solvent P0 nA

P0 − Ps WB MA
⇒ i×
= × where WB = amount of solute dissolved; WA = amount of solvent taken;
P 0 MB WA
MB = molecular mass of solute; MA = molecular mass of solvent
Elevation in Boiling point
WB
iKbm ; ∆TB = iKb ×
∆TB = × 1000
MB × WA (gms)
Depression in Freezing Point
WB
iK f m ; ∆Tf = iK f ×
∆Tf = × 1000
MB × WA (gms)
Osmotic pressure
pV = inRT; π = iCRT

6.6 Relationship Between van’t Hoff Factor for Dissociation and Association
(a) Dissociation
Degree of dissociation: We define the degree of dissociation as the number of moles dissociated over the initial
concentration taken
Total number of particles at equilibrium
Van’t Hoff factor :
Initial concentration
Let c moles per liter of an electrolyte Axby be taken and let it dissociate
as xA y + & yBx − . ' α ' be degree of dissociation.
∴ No. of solute particle dissociated = Cα
A B → xA y + + yBx −
x y
Initial conc. C    O   O
Conc. at equilibrium C – C α   x c α y c α
[(c − cα ) + xcα + Ycα] 1 − α + xα +yα
i= ⇒ i= = 1 + [(x + y) – 1] a
c 1
Let x + y = n = number of particles formed after dissociation
i−1
⇒ i = 1 (n – 1) α; α =
n−1

(b) Association
Degree of Association: The degree of association may be defined as the number of molecules associated over
initial concentration.
number of moles associated Total number of moles of particles at equilibrium
α= ; i=
Initial concentration Initial concentration
Let n molecules of an electrolyte A undergo association
nA → An
2 0 . 2 2 | Solutions and Colligative Properties

Let the initial concentration of A = C; the degree of association = a



when C α moles of A associate, then moles of An are formed
n Cα
Cα C − Cα +
Number of moles of equilibrium (C – Cα) ⇒ i= n
n C
α
1−α+
If C = 1, then i = n ; i = 1 +  1 − 1 α ⇒ α = i − 1
 
1 n  1
−1
n

Illustration 11: The freezing point depression of 0.001m Kx[Fe(CN)6] is 7.10 × 10–3 K. Determine the value of x.
Given, Kf = 1.86 K kg mol–1 for water. (JEE MAIN)

Sol: Dx = i × Kf × m; 7.10 × 10–3 = i × 1.86 × 0.001; i = 3.817


i−1 3.817 − 1
α= ;1= ; x = 2.817 ≈ 3
n−1 (x + 1) − 1
∴ Molecular formula of the compound is K3[Fe(CN)6].

Illustration 12: Three particles of a solute, A, associate in benzene to form species A3. Calculate the freezing point
of 0.25 molal solution. The degree of association of solute A is found to be 0.80, the freezing point of benzene and
its cryoscopic constant are 5.5ºC and 5.12 km–1, respectively. (JEE MAIN)

Sol: 3A
→ A3
No. of moles dissolved 3m(1 – α) 0
No. of moles after dissociation mα/3
Total moles present after dissociation
α  α  2α   3 − 2 × 0.8 
= m(1 – α) + m = m 1 − α +  = m 1 −  = 0.25 m   = 0.177 m
3  3   3   3 
DTf = Kfm or Tf0 – Tf = 5.12 km–1 × 0.117 m = 0.6
Tf = Tf0 – 0.6ºC = 5.5ºC – 0.6ºC = 4.9ºC
Chem i str y | 20.23

PROBLEM-SOLVING TACTICS
General Plan for Solving Problems Involving Freezing Point Depression and Boiling Point Elevation

Mass of solute

Convert using the


molar mass of the solute
Amount of solute
in moles
Determine if solute mol particles
m=
is an electrolyte or kg solvent
non-electrolyte

Amount of
Molal concentration
particles
of particles in solution, m
Determine if in solution

Multiply by the molal Multiply by the molal


freezing point boiling point
constant, kf constant, kb

Freezing point Boiling point


depression, Tf elevation, Tb

Add Tf to the normal Add Tb to the normal


freezing point of boiling point of
the solvent the solvent

Freezing point Boiling point


of solution of solution

Figure 20.7

For example, let us consider the table provided hereunder.


Table: List of freezing point depression and boiling point elevation constant for common solvents.

Solvent Normal f.p. (in ºC) kf (in ºC/m) Normal b.p. (in ºC) kb (in ºC/m)
Acetic acid 16.6 –3.90 117.9 3.07
Camphor 178.8 –39.7 207.4 5.61
Ether –116.3 –1.79 34.6 2.02
Naphthalene 80.2 –6.94 217.7 5.80
Phenol 40.9 –7.40 181.8 3.60
Water 0.00 –1.86 100.0 0.51

Sample Problem 1: What is the freezing point of a solution of 210.0 g of glycerol, HOCH2CHOHCH2OH, dissolved
in 350 g of water?

Sol 1: Analyze

(i) What is given in the problem? The formula and mass of solute and the mass of the water used
(ii) What are you asked to find? The freezing point of the solution
2 0 . 2 4 | Solutions and Colligative Properties

Items Data
Identity of solute Glycerol, HOCH2CHOHCH2OH
Particles per mole of solute 1 mol
Identity of solvent Water
Freezing point of solvent 0.00ºC
Mass of solvent 350 g
Mass of solute 210.0 g
Molar mass of solute 92.11 g/mol
Molal concentration of solute particles ?
Molal freezing point constant for water –1.86º C/m
Freezing point depression ?
Freezing point of solution ?

2. Plan
What steps are needed to calculate the freezing point of the solution?
In such cases, use the molar mass of the solute to determine the amount of solute. Then, apply the mass of solvent
to calculate the molality of the solution.
From the molality, now use the molal freeezing constant for water to calculate the number of degrees the freezing
point is lowered. Thereafter, add this negative value ot the normal freezing point.

Freezing point of the glycerol solution


Mass of glycerol in g 6

Multiply by the inverse


Add Tf to the
of the molar mass
normal freezing point
of glycerol
of water

2 5
Amount of glycerol in solution Freezing point elevation, Tf

The solute is a non electrolyte, Multiply by the molal


so the amount of solute freezing point constant
equals the amount Kf of water
of particles in solution
4
Divide the amount
3
of the particles in Molal concentration of
Amount of particles in solution particles in water, m
Solution by the
mass of the solvent
in kilograms
Mass of water in kg

1 kg
Multiply by the conversion factor
1000 g

given
1 kg
gH2O× kg H2O
= Mass of water in g
1000 g
freezing point
1 Calculate depression
molar mass of glycerol above constant freezing po int Calculated
given of H O above
1 mol glycerol 1 −1.86º C 2
g glycerol × × × =∆Tf ; 0.00º C + ∆Tf =tf
92.11 g glycerol kg H2O mol / kg
Chem i str y | 20.25

3. Compute
1kg
350 g H2O × = 0.350 kg H2O
1000g
1 mol glycerol 1 −1.86º C
210.0 g glycerol × × × −12.1º C
=
92.11 g glycerol 0.350kg H2O mol / kg

0.00ºC + (–12.1ºC) = –12.1ºC

4. Evaluate

(i) Are the units correct? Yes, units canceled to give Celsius degrees.
(ii) Is the number of significant figure correct? Yes, three significant figures are correct be-
cause the data had a minimum of three signif-
icant figures.
(iii) Is the answer reasonable? Yes, the calculation can be approximated as

200 ÷ [90 × 3(350 ÷ 1000)] × –2 = –400/30 =


–13, which is close to the calculated value.

Sample Problem 2: What is the boiling point of a solution containing 34.3 g of the ionic compound magnesium
nitrate dissolved in 0.107 kg of water?

Sol:
1. Analyze

(i) What is given in the problem? the formula and mass of solute and the mass of the water used
(ii) What are you asked to find? the freezing point of the solution

Items Data
Identity of solute Magnesium nitrate
Equation for the dissociation of the solute Mg(NO3)2 → Mg2+ + 2NO3–
Amount of ions per mole of solute 3 mol
Identity of solvent Water
Boiling point of solvent 100.0ºC
Mass of solvent 0.107 kg H2O
Mass of solute 34.3 g
Molar mass of solute 148.32 g/mol
Molal concentration of solute particles ?m
Molal boiling point constant for solvent 0.51ºC/m
Boiling point depression ?ºC
Boiling point of solution ?ºC

2. Plan
What steps are needed to calculate the freezing point of the solution?
In such a case, use the molar mass to calculate the amount of solute in moles. Then, multiply the amount of solute
by the number of moles of ions produced per mole of solute. Use the amount of ions with the mass of solvent to
compute the molality of particles in solution. Thereafter, use this effective molality to determine the boiling point
elevation and the boiling point of the solution.
2 0 . 2 6 | Solutions and Colligative Properties

1 6
Mass of Mg(NO3)2 in g Boiling point the of Mg(NO3)2 in g

Add Tb to the


normal boiling point
of water

5
2 Boiling point elevation, Tb
Amount of Mg(NO3)2 in mol
The solute is an electrolyte, Multiply by the molal
so multiply the amount if boiling point constant
solute by number of Kb of water
particles per mole of solute
4
Divide the amount
3 Molal concentration of
of the particles in
Amount of particles in solution particles in water, m
solution by the
mass of the solvent
in kilograms
Mass of water in kg

1
molal boiling-po int
molar mass of Mg(NO3 )2 given cons tant for water
given
1 mol Mg(NO3 )2 3 mol particles 1 0.51º C
g Mg(NO3 )2 × × × × ∆tb
=
148.32 g Mg(NO3 )2 1 mol Mg(NO3 )2 kg H2O mol / kg

boiling po int Calculate


of H2O above
100.0º C + ∆tb =tb

3. Compute

1 mol Mg(NO3 )2 3 mol particles 1 0.51º C


34.3g Mg(NO3 )2 × × × × 3.31º C
=
148.32 g Mg(NO3 )2 1 mol Mg(NO3 )2 0.107kg H2O mol / kg

100. 0ºC + 3.31ºC) = 103.3ºC

4. Evaluate

(i) Are the units correct? Yes, units canceled to give Celsius degrees.
(ii) Is the number of significant figure correct? Yes, the number significant figures is correct because the boiling point of
water was given to one decimal place.
(iii) Is the answer reasonable? Yes, the calculation can be approximated as
[35 × 3)/150 × 5 = (7/10) × 5 = 3.5, which is given to one decimal place.
Chem i str y | 20.27

POINTS TO REMEMBER
P = KHX, where, P = partial pressure of a gas over solution, KH = Henry’s constant,
Henry’s law
X = mole fraction of the gas in solution

0
Raoult’s law P =XA PA + XB PB0 , where, P = total pressure of the solution, XA and XB = mole fraction
0
of substituents A and B. pAo and PB = partial pressure of A and B, respectively

A A X P0
A PB B B X P0 P
YA
Relationship between Dalton’s law= = , YB
= = ,
P P P P
and Raoult’s law
where, YA and YB denotes mole fraction of components A and B in vapor phase

PA0 − P
Lowering of vapor pressure = x B , relative lowering of vapor pressure = mole fraction of solute
PA0

K W
Elevation in boiling point ∆TB = B B × 1000
MB × WA

RTf × wsolute
Depression in freezing point =∆Tf × 1000
Msolute × Wsolvent

Osmotic pressure Osmotic pressure, π =CRT

Normal molecular wt.(calculated value)


van’t Hoff factor, i i=
Observed molecular wt.(experimental value)

van’t Hoff factor in case of i −1


For association, α =
association n−1

i −1
van’t Hoff factor in case of For dissociation, α =
1
dissociation −1
n

Solved Examples

JEE Main/Boards Example 2: The osmotic pressure of a solution of an


organic substance containing 18 g in one litre of solution
Example 1: One litre of sea water weighs 1030 g and at 293 K is 2.414 × 105 Nm–2. Find the molecular mass of
contain about 6 × 10–3 g of dissolved O2. Calculate the the substance if S = 8.3 JK–1 per mol.
concentration of dissolved oxygen in ppm. w w
Sol: Applying the equation, PV = · ST or m = · ST
m PV
Sol: Mass of O2 in mg = 6 × 10–3 g × 103 mg/g = 6 mg
Given, P = 2.414 × 105 Nm–2,
ppm of O2 in 1030 g sea water
V = 1.0 liter = 1 × 10–3 m3,
Mass of O2 in mg
= × 106 S = 8.3 JK–1 per mol,
Mass of sea water in mg
6 w = 18 g and T = 293 K
= × 106 = 5.8 ppm
(1030 × 1000)mg
2 0 . 2 8 | Solutions and Colligative Properties

18 Example 6: Why does the use of the pressure cooker


m= × 8.3 × 293 = 181.33
5
2.414 × 10 × 1 × 10 −3 reduces cooking time?

Sol: At higher pressure over the liquid (due to weight


Example 3: What is the molality of 40.0 wt% ethylene of the pressure cooker lid) the liquid boils at higher
glycol solution used for automobile antifreeze ? (molar temperaure and cooking occurs faster.
mass of ethylene glycol = 62 g mol–1).

Sol:100 g ethylene glycol solution has Example 7: One liter of a sample of hard water
contains 1 mg of CaCl2 and 1 mg of MgCl2. Find the
= 40 g ethylene glycol total hardness in terms of parts of CaCO3 per 106 parts
60 g water (solvent) has = 40 g ethylene glycol of water by mass.
Or 0.060 kg water has Sol: CaCl2 → CaCO3
40 111 g 100 g
= mole ethylene glycol
62
 100 
mole of solute 40 / 62 \ 1 mg of CaCl2 is equivalent to  × 1  mg CaCO3
∴ Molality = = = 10.75 molal  111 
kg of solvent 0.060
MgCl2 → CaCO3
Example 4: To 500 cm3 of water 3.0 × 103 kg of acetic 95 100
acid is added. If 23% of acetic acid is dissociated, what  1
will be the depression in freezing point ? Kf and density \1 mg of MgCl2 is equivalent to =  2 ×  = 0.82 mg
 3
of water are 1.86 K kg–1 and 0.997 g cm–3, respectively.
 100 100 
Total mass of CaCO3 =  +  mg
Sol: Mass of solute = 3 × 10–3 kg = 3 g  111 95 
Mass of solvent = 500 × 0.997 = 498.5 g = 1.9535 mg

α = 23% = 0.23, Given volume = 1 liter, mass

i = 1 – α + nα= 1 – 0.23 + 2 × 0.23 = 1.23 = 1000 g = 106 mg

DTf = i(molality × Kf ) Thus, 1.9535 mg of CaCO3 is present in 106 mg of H2O.

 3 × 1000  Hardness = 1.9535 ppm


⇒ 1.23 ×   × 1.86 ⇒DTf = 0.229
 60 × 498.5 
Example 8: Vapor pressure of mixtures of C6H6 and
C7H8 at 50ºC are given by PM = 179 XB + 92, respectively
Example 5: Concentration of NaOH solution is 3.0
where XB is the mole fraction of C6H6. Calculate
molal and its denisty is 1.110 g/mL. What is the molarity
of the solution? (a) Vapor pressure of pure liquids.
w 1000 (b) Vapor pressure of liquid mixture obtained by mixing
Sol: Molarity = ×
M V 936g C6H6 and 736 g toluene.
[V = volume of the solution] (c) If the vapors are removed and condensed into liquid
and again brought to the temperture of 50ºC, what
3 mol. of NaOH = 120 g of NaOH
would be the mole fraction of C6H6 in vapor state?
Weight of the solution = [120 + 1000] g
Sol: Given, PM = 179XB+ 92
= 1120 g
For pure C6H6,
1.110 g solution has the volume = 1 c.c.
1 XB, i.e., mole fraction of benzene = 1
∴ 1120 g solution has the volume = × 1120
1.110 \PB0 = 179 + 92 = 271 mm
 w  1000
Hence, molarity =   ×
M  V (a) For pure C7H8; XB = 0
1000 \PT0 = 179 × 0 + 92 = 92 mm

= × 1.110 =
2.9732M
1120
Chem i str y | 20.29

936 Example 10: x g of non-electrolytic compound (molar


78 12 mass = 200) is dissolved in 1.0 liter of 0.05 M NaCl
(b)
= XB = = 0.60
936 736 12 + 8 solution. The osmotic pressure of this solution is found
+ to be 4.92 atm at 27ºC. Calculate the value of ‘x.’ Assume
78 92
complete dissociation of NaCl and ideal behavior of
736 this solution.
92 8
XT
= = = 0.40
936 736 12 + 8 Sol: (i) For NaCl; p1 = i(CRT)
+
78 92
p1 = 2 × 0.05 × 0.0821 × 300 ; ⇒ p1 = 2.463 atm
PM = 179 × 0.6 + 92 = 107.4 + 92 = 199.4 mm For unknown compound
(c) Mole fraction of C6H6 in vapor phase of initial mixture x
p2 = CRT ⇒ p2 = × 0.821 × 300 ;
X′B = P′B / PM 200 × 1

∴ X′B = [271 × 0.6] / 199.4 = 0.815 ⇒ p2 = 0.1231 × atm

Similarly, X′T = [92 × 0.4] / 199.4 = 0.185 Total osmotic pressure π = p1 + p2

These fractions of vapors are taken out and condensed 4.92 = 2.463 + 0.1231x; x = 19.959 g
into liquid. The liquid is again brought to 50ºC to get
again vapour–liquid equilibrium.
JEE Advanced/Boards
∴ Mole fraction of components in vapor phase of initial
mixture = Mole fraction of components in liquid phase Example 1: Two liquids A and B form ideal solution.
of II mixture. At 300 K, the vapor pressure of a solution containing 1
\PM = PB0 · X′B + PT0 · X′T mole of A and 3 moles of B is 550 mm of Hg. At the same
temperature, if one more mole of B is added to this
= 271 × 0.815 + 92 × 0.185 mm
solution, the vapor pressure of the solution increases
= 220.865 + 17.02 = 237.885 mm by 10 mm of Hg. Determine the vapor pressure of A
and B in their pure state.
Example 9: The solution of a non-volatile solute in
Sol: Let the VP of pure A and B are PA0 and PB0,
water freezes at –0.30ºC. The vapor pressure of pure
respectively.
water at 298 K is 23.51 mm Hg and Kf of water is 1.86
degree/molal. Calculate the vapor pressure of this Total VP of solution (1 mole A + 3 moles B)
solution at 298 K. 1 0 3 0
PM = XAPA0 + XBPB0 ⇒ 550 = P + P
4 A 4 B
Sol: DTf = molality × Kf; 0.30 = molality × 1.86
or PA0 + 3PB0 = 2200 ... (i)
0.30 10 5 n × 1000
molality = = = ⇒ Total VP of solution (1 mole A + 4 moles B)
1.86 62 31 W(gm)
1 0 4 1 0 1 0 4 0
n = moles of solute, W = wt. of solvent PM = P + P ⇒ 560 = P + P
5 A 5 5 B 5 A 5 B
M = mol. wt. solvent, N = mole of solvent
or PA0 + 4PB0 = 2800 … (ii)
n 5
= Solving (i) and (ii)
N × 18 31 × 1000
PA0 = 400 mm of Hg PB0 = 600 mm of Hg
n 5 × 18 9

= =
N 31 × 1000 3100
Example 2: Two liquids A and B form an ideal solution
N at temperature T. When the total vapor pressure
N 0
∵ PS = P ⇒ n P above the solution is 400 torr, the mole fraction of A
n+N N 0 in the vapor phase is 0.40 and in the liquid phase 0.75.
1+
n What are the vapor pressure of pure A and pure B at
3100 temperature T?

⇒PS = 9 × 23.51 =
23.442 mm
3100 Sol: Mole fraction of A in vapor phase YA = 0.4 and in
1+ liquid phase XA = 0.75
9
2 0 . 3 0 | Solutions and Colligative Properties

PTotal = 400 torr (iii) To calculate mole fraction of water


Let VP of pure A and B are PA0 and PB0 Pº − Ps nB
By Raoult’s law, = = xB ,
XAPA0 = YAPTotal Pº nA + nB
mole fraction of solute, i.e.,
YAPTotal 0.4 × 400 160
PA0 = = = Pº − Ps
XA 0.75 0.75 xB = = 0.00351
PA0 = 213.33 torr. Pº
∴ Mole fraction of solvent (water),
PTotal = XAPA0 + (1 – XA)PB0
XA = 1 – xB = 1 – 0.00351 = 0.99649
 160  0
400 = 0.75   PB + (1 – 0.75),
 0.75  Example 5: Ethylene dibromide (C2H4Br2) and
PB0= 960 torr 1,2-dibromo propane form a series of ideal solutions
over the whole range of composition. At 85ºC, the
Example 3: Calculate the freezing point of an aqueous vapor pressure of these two liquids are 173 and
solution of nonelectrolyte having an osmotic pressure 127 torr, respectively. What would be the mole fraction
of 0.2 atmosphere at 300 K. of ethylene dibromide in a solution at 85ºC equilibrated
(Kf = 1.86 K kg mol–1, R = 0.821 liter-atm K–1 mol–1) with 1:1 molar mixture in the vapor?
(assume molarity = molality)
Sol: Suppose the mole fraction of ethylene dibromide in
π 2 the liquid phase = x. Then, mole fraction of 1,2-dibromo
Sol: π = CRT, C = = mol lit–1
RT 0.0821 × 300 propane will be = 1 – x.
In dilute solution, the density of water can be taken as Vapor pressure of ethylene dibromide
1.90 gm cm–3. = x × Pº = x × 173 torr = 173 x torr
Hence, molality ≈ molarity Vapor pressure of 1,2-dibromo propane
2 = (1 – x) × 127 torr
DTf = (molalty × Kf )= × 1.86 ; ⇒DTf = 0.151 K
0.0821 × 300
As they have 1:1 molar ratio in the vapor phase,
∵ (Tf )solution = (Tf ) solvent – DTf = (273 – 0.151)
173 x = (1 – x) × 127 or 173 x = 127 – 127x
(Tf )solution = 272.749 K or –0.151ºC
127
or 300x = 127 or x = = 0.423
300
Example 4: Vapor pressure of water at 293 K is
17.51 mm. Lowering of vapor pressure of a sugar Example 6: Vapor pressure of solution containing 6 g
solution is 0.0614 mm. Calculate (i) relative lowering of of a non-volatile solute in 180 g water is 20.0 torr. If
vapor pressure, (ii) vapor pressure of the solution and 1 mol water is further added vapor pressure increases
(iii) mole fraction of water. by 0.02 torr. Calculate vapor pressure of water
and molecular weight of non-volatile solute with
Sol: Here, we are given that
temperature remaining constant.
vapor pressure of water; (P0) = 17.51 mm
Sol: Let molecular wt. of solute = m
Lowering of vapor pressure
and VP of water (solvent) = P0
(P0 – Ps) = 0.0614 mm
Psolution = 20 torr
(i) Relative lowering of vapor pressure 6
moles of solute, n =
Pº − Ps 0.0614mm m
=
= = 0.00351 180
Pº 17.51mm moles of solvent, N = = 10
18
(ii) Vapor pressure of the solution (Ps)
 N  0
Pº – Ps = 0.0614 mm; Pº = 17.51 mm ∴Ps =  P
n+N
∴ 17.51mm – Ps = 0.0614 mm
 
or Ps = 17.51 mm – 0.0614 mm = 17.4468 mm  10   10m  0
⇒ 20=   P0=  P  … (i)
 6 + 10   6 + 10m 
 
m 
Chem i str y | 20.31

If 1 mol water is further added, then moles of solvent N Example 8: Calculate the molarity, molality and mole
= 10 + 1 = 11 mol and VP of solution becomes fraction of ethyl alcohol in a solution of total volume
95 mL prepared by adding 50 mL of ethyl alcohol (density
Ps = 20 + 0.02 = 20.02 torr
= 0.789 mL–1) to 50 mL water (density = 1.00 g mL–1).
N 0
⇒Ps = P
n+N Sol: No. of moles of ethyl alcohol
  Vol. × density 50 × 0.789
 11   11m  = = =0.8576
Mol. mass 49
20.02=  = P 
0
… (ii)
 6 + 11   6 + 11m  No. of moles of water
 
m 
Vol. × density 50 × 1
Now, divide (2) by (1) = = = 2.7777
Mol. mass 18
20.02 11(6 + 10m) No. of moles
= Molarity = × 1000
20 10(6 + 11m) Vol. of sol. in mL
⇒ m= 54 g 0.8576
= × 1000 = 9.027 M
Put this value of m in (i) or (ii) 95
No. of moles of solute
P0 = 22.22 torr Molality = × 1000
Mass of solvent in grams

Example 7: The density of a solution containing 13% 0.8576


= × 1000 = 17.152 m
by mass of sulfuric acid is 1.09 g/mL. Calculate the 50
molarity and normality of the solution. 0.8576 0.8576
Mole fraction = = = 0.236
0.8576 + 2.7777 3.6353
Sol: Volume of 100 g of the solution
100 100 100 1 Example 9: A current of dry air was bubbled through
= = mL = liter = liter
d 1.09 1.09 × 1000 1.09 × 10 a bulb containing 26.66 g of an organic compound in
200 g of water and then through a bulb at the same
Number of moles of H2SO4 in 100 g of the solution
temperature, containing water and finally through a
13 tube containing anhydrous calcium chloride. The loss
=
98 of mass in bulb containing water was 0.087 g and
No. of moles H2SO 4 gain in mass of the calcium chloride tube was 2.036 g.
Molarity =
Volume of soln. in litre Calculate the molecular mass of the organic substance.
13 1.09× 10 P0 − Ps Loss in mass solvent bulb 0.087
= × = 1.445 M Sol: = =
98 1 P0 Gain in mass of CaCl2 tube 2.036
[Note: In solving such numericals, the following formula
Let the molecular mass of the organic substance be m.
can be applied:]
According to Raoult’s law,
%strength of soln. × density of soln. × 10
Molarity = P0 − Ps
Mol. mass w /m
=
Similarly, normality P0 w W
+
%strength of soln. × density of soln. × 10 m M
=
Eq. mass
26.66
We know that, 0.087 m 26.66
= = ; m = 53.75
Normality = Molarity × n 2.036 26.66 200 200
+ 26.66 + m
m 18 18
= 1.445 × 2
 Mol. mass 98 
n
= = = 2 = 2.89 N Example 10: 2.0 g of benzoic acid dissolved in 25.0 g of
 Eq. mass 49  benzene shows a depression in freezing point equal to
1.62 K. Molal depression constant (Kf ) of benzene is 4.9
K kg mol–1. What is the percentage association of the
acid if it forms dimer in the solution?
2 0 . 3 2 | Solutions and Colligative Properties

Sol: Mass of solute (benzoic acid), w2 = 2.0 g 122


= = 0.504
Mass of solvent (benzene), w1 = 25.0 g 242

Observed DTf = 1.62 K If α is the degree of association of benzoic acid, then


we have
Kf = 4.9 K kg mol–1


2C6H5COOH 
(C6H5COOH)2

Observed molar mass of benzoic acid
1000 × K f × w2 Initial moles 1 0
M2 =
∆Tf × w1 After association 1 – a α/2

1000g kg−1 × 4.9 K kg mol−1 × 2.0g ∴ The total number of moles after association
= = 242 g mol–1
1.62K × 25.0g α
=1–α+ = 0.504
Calculated molar mass of benzoic acid 2
α = (1 – 0.504) × 2 = 0.496 × 2 = 0.992
(C6H5COOH) = 72 + 5 + 12 + 32 + 1 = 122 g mol–1
Calculated mol. mass
Or percentage association = 99.2%
van’t Hoff factor, i =
Observed mol. mass

JEE Main/Boards

Exercise 1 Q.8 A solution of a non-volatile solute in water freezes


at –30.0. The vapour pressure of pure water at 298 K
Q.1 How many grams of H2SO4 are required to prepare is 23.51 mm Hg and Kf for water is 1.86 degree/mol.
250 mL of 0.2 molar solution? Calculate the vapour pressure of this solution at 298 K.

Q.2 A solution contains 23g of ethanol and 72g of Q.9 The vapour pressure of pure benzene at 25ºC is
water. Calculate the mole fraction of ethanol and water. 639.7 mm Hg and the vapour pressure of a solution of
non–volatile solute in benzene at the same temperature
Q.3 2.82 g glucose (mo. mass = 180) are dissolved in is 631.9 mm Hg.
30g of H2O. Calculate the (i) Molality of the solution (ii) Calculate the mole fraction of the solute and the
Mole fraction of glucose and water. molality of the solution. (C = 12, H = 1)

Q.4 Prove that the relative lowering of vapour pressure Q.10 (i) What is osmotic pressure and how is it reached
is equal to the mole fraction of non–volatile solute in to the molecular mass of a non–volatile substance?
the solution.
(ii) What advantage the osmotic pressure method
Q.5 What is the relation between elevation of boiling has over the elevation of boiling point method for
point and osmosis pressure? determining molecular masses?

Q.6 One litre aqueous solution of sucrose (molar mass Q.11 State Raoults’ law for solutions of nonvolatile
= 342 g mol–1) weighing 1015 g is found to record an solute in volatile solvents. Derive a mathematical
osmotic pressure of 4.85 atm at 293 K. What is the expression for this law?
molality of the sucrose solution?
Q.12 What is the molarity of a 13% solution (by weight)
Q.7 A solution containing 12.5 g of a non-electrolyte of sulphuric acid? Its density is 1.020 g cm–1 (Atomic
substance in 175 g of water gave a boiling point masses : H = 1, O = 16, S = 32 amu)
elevation of 0.70 K. Calculate the molar mass of the
substance. (Elevation constant for water is Kb = 0.52 K Q.13 What is the molar concentration of solute particles in
kg mol–1) the human blood if the osmotic pressure is 72 atm at the
body temperature of 37ºC? (R = 0.0821 L atm K–1 mol–1)
Chem i str y | 20.33

Q.14 Calculate the molar concentration of urea solution Q.24 Two elements A and B form compound having
if it exerts an osmotic pressure of 2.45 atm at 300K. (R = formula AB2 and AB4. When dissolved in 20g of
0.0821 L atm mol–1 K–1) benzene (C6H6) 1g of AB2 lowers the freezing point by
2.3 K whereas 1.0 g of AB4 lowers by 1.3K. The molar
Q.15 To 500 cm2 of water, 3.0 × 10–5 kg of acetic acid depression constant for benzene is 5.1 K kg mol–1.
is added. If 23% of acetic acid is dissociated, what will Calculate atomic masses of A and B.
be the depression in freezing point? Kf and density of
water are 1.86 K kg–1/mol and 0.997 g cm–1 respectively. Q.25 19.5 g of CH2FCOOH is dissolved in 500g water.
The depression in freezing point of water observed is
Q.16 How many mL of 0.1M HCl are required to react 1.0ºC. Calculate the van’t Hoff factor and dissociation
completely with 1g mixture of Na2CO3 and NaHCO3 constant of fluoroacetic acid.
containing equimolar amount of two?
Q.26 Vapour pressure of pure acetone and chloroform
Q.17 The boiling point of benzene is 353.23K. When at 328K are 741.8 mm Hg and 632.8 mm Hg respectively.
1.80g of a non–volatile solute was dissolved in 90g Assuming that they form ideal solution over the entire
of benzene, the boiling point is raised to 354.11 K. range of composition plot Ptotal , Pchloroform and pacetone as a
Calculate the molar mass of the solute. Kb for benzene function of xacetone. The experimental data observed for
is 2.53K kg mol–1. different compositon of mixture as:

Q.18 1.0 g of a non-electrolyte solute dissolved in 50g 100 × 0 11.8 23.4 36.0 50.8 58.2 64.5 72.1
of benzene lowered the freezing point of benzene Xacetone
by 0.40K. The freezing point depression constant of Pacetone 0 549 110.1 202.4 322.7 405.9 454.1 521.1
benzene is 5.12 K kg mol–1. Find the molecular mass of Pchloroform 632.8 548.1 469.4 359.7 257.7 193.6 161.2 120.7
the solute.
Plot this data also on the same graph paper. Indicate
Q.19 Calculate the mass of a non–volatile solute (mol whether it has positive deviation or negative deviation
mass = 40) which should be dissolved in 114 g octane from the ideal solution.

to reduce its vapour pressure to 80%.
Q.27 (a) Derive the relationship between relative
Q.20 What is the relation between elevation in boiling lowering of vapour pressure and mole fraction of the
point and molar heat of vapourisation? volatile liquid.

Q.21 (a) The outer shells of two eggs are removed. One (b) (i) Benzoic acid completely dissolves in benzene.
of the egg is placed in pure water and other is placed What will be the vapour pressure of a solution
in saturated solution of sodium chloride. What will be containing 61g of benzoic acid per 500g benzene
observed and why? when the vapour pressure of the pure benzene of the
temperature of experiment is 66.6 torr?
(b) A decimolar solution of potassium ferrocyanide.
K4[Fe(CN)4] is 50% dissociated at 300K. Calculate (ii) What would have been the vapour pressure in the
osmotic pressure of the solution. [R = 8.314 JK–1 mol–1] absence of dissolution?
(iii) Derive a relationship between mole–fraction and
Q.22 Why do calculations based on lowering in vapour pressure of a component of an ideal solution in
freezing points of a solutions sometime give abnormal phase and vapour phase.
molar mass value for solutes? How may the equation
for the related colligative property be modified for Q.28 (a) Which solution has higher concentration:
substance exhibiting abnormal molar mass value? A 1 molar or 1 molal solution of the same solute. Give
solute contains 7.45g KCl per litre of solution. It has reasons.
an osmotic pressure of 4.68 atm at 300K. Calculate the
degree of dissociation for KCl in this solution. (b) 0.5 g KCl was dissolved in 100g of water and the
solution originally at 20ºC, froze at –0.24ºC. Calculate the
Q.23 0.6 mL of acetic acid (CH3COOH) having density percentage ionization of salt. KCl per 1000g water = 1.86.
1.06 g mL–1, in dissolved in 1 litre of water. The
depression in freezing point observed for this strength Q.29 How will you explain Henry’s law in terms of Le
of acid was 0.0205ºC. Calculate van’t Hoff factor and Chatelier’s principle and Kinetic theory of gases? or
dissociation constant of acid. State Henry’s law for the solubility of gases in liquid.
Explain it in terms of Le Chatelier’s theorem.
2 0 . 3 4 | Solutions and Colligative Properties

Exercise 2 Q.8 1.0 molal aqueous solution of an electrolyte A2B3 is


60% ionized. The boiling point of the solution at 1 atm
Single Correct Choice Type is Kb(H O) = 0.52 K kg mol–1
2
(A) 274.76 K (B) 377 K (C) 376.4 K (D) 374.76 K
Q.1 Mole fraction of A vapours above the solution in
mixture of A and B (XA = 0.4) will be [Given: PA0 = 100
Q.9 The van’t Hoff factor for a dilute aqueous solution
mm Hg and PB0 = 200 mm Hg]
of glucose is
(A) 0.4 (B) 0.8
(A) Zero (B) 1.0 (C) 1.5 (D) 2.0
(C) 0.25 (D) None of these
Q.10 The correct relationship between the boiling
Q.2 The exact mathematical expression of Raoult’s law is points of very dilute solution of AlCl3 (T1K) and CaCl2
Po − Ps Po − Ps N (T2K) having the same molar concentration is
(A) (B) =
P o P o n (A) T1 = T2 (B) T1 > T2 (C) T2 < T1 (D) T2 ≤ T1

Po − Ps n Po − Ps Q.11 A 0.001 molal solution of a complex [MA3] in water


(C) = (D) = n×N
Ps N Po has the freezing point of –0.0054ºC. Assuming 100%
ionization of the complex salt and Kf for H2O = 1.86
Q.3 A mixture contains 1 mole of volatile liquid km–1, what the correct representation for the complex
A (PA0 = 100 mm Hg) and 3 moles of volatile liquid B (A) [MA8] (B) [MA7]A (C) [MA6]A2 (D) [MA5]A3
(PB0 = 80 mm Hg). If solution behaves ideally, the total
pressure of the distillate is Q.12 The vapour pressure of a solution of a non–volatile
(A) 85 mm Hg (B) 85.88 mm H electrolyte B in a solvent A is 95% of the vapour pressure
of the solvent at the same temperature. If the molecular
(C) 90 mm Hg (D) 92 mm Hg
weight of the solvent is 0.3 times the molecular weight
of solute, the weight ratio of the solvent and solute are
Q.4 Assuming each salt to be 90% dissociated, which of
(A) 0.15 (B) 5.7 (C) 0.2 (D) 4.0
the following will have highest boiling point?
(A) Decimolar Al2(SO4)3 Q.13 The vapour pressure of a saturated solution of
(B) Decimolar BaCl2 sparingly soluble salt (XCl3) was 17.20 mm Hg at 27ºC.
If the vapour pressure of pure H2O is 17.25 mm Hg at
(C) Decimolar Na2SO4 300K, what is the solubility of sparingly soluble salt XCl3
(D) A solution obtained by mixing equal volumes of (B) in mole/litre
and (C) (A) 4.04 × 10–2 (B) 8.08 × 10–2
(C) 2.02 × 10–2 (D) 4.04 × 10–3
Q.5 The vapour pressure of a solvent decreased by
10 mm of Hg when a non–volatile solute was added Q.14 At 300K, the vapour pressure of an ideal solution
to the solvent. The mole fraction of solute in solution containing 3 mole of A and 2 mole of B is 600 torr. At
is 0.2, what would be mole fraction of the solvent if the same temperature, if 1.5 mole of A and 0.5 mole of
decrease in vapour pressure is 20 mm of Hg. C (non–volatile) are added to this solution the vapour
(A) 0.2 (B) 0.4 (C) 0.6 (D) 0.8 pressure of solution increases by 30 torr. What is the
value of PB0?
Q.6 Elevation of boiling point of 1 molar aqueous (A) 940 (B) 405 (C) 90 (D) None of these
glucose solution (density = 1.2 g/mL) is
(A) Kb (B) 1.20 Kb (C) 1.02 Kb (D) 0.98 Kb Q.15 The freezing point depression of a 0.1 M aq.
solution of weak acid (HX) is –0.20ºC. What is the value
of equilibrium constant for the reaction?
Q.7 What will be the molecular weight of CaCl2
determined in its aq. solution experimentally from HX(aq) 1H+ (aq) + X– (aq) [Given: Kf for water = 1.8 kg
depression of freezing point? mol–1 K and Molality = Molarity]

(A) 111 (B) <111 (C) >111 (D) Data insufficient (A) 1.45 × 10–4 (B) 1.35 × 10–3
(C) 1.21 × 10–2 (D) 1.35 × 10–4
Chem i str y | 20.35

Q.16 What is the following plots represents an ideal Q.4 25 mL of a solution of barium hydroxide on titration
binary mixture? with a 0.1 molar solution of hydrochloric acid gave
litre value of 35 mL. The molality of barium hydroxide
(A) Plot of Ptotal v/s 1/XB is linear (XB = mole fraction of
solution was (2003)
‘B’ in liquid phase)
(A) 0.07 (B) 0.14 (C) 0.28 (D) 0.35
(B) Plot of Ptotal v/s YA is linear (YB = mole fraction of ‘A’
in vapour phase)
Q.5 Which of the following moles of expressing
1
(C) Plot of v/s YA is linear concentration is independent of temperature? (1988)
Ptotal
(A) Molarity (B) Molality
1
(D) Plot of v/s YB is non–linear
Ptotal (C) Formality (D) Normality

Q.17 Two liquids A and B form an ideal solution. What Q.6 Two solutions of a substance (non-electrolyte) are
is the vapour pressure of solution containing 2 moles mixed in the following manner 480 mL of 1.5M first
of A and 3 moles of at 300K? Vapour pressure of pure solution + 520 mL of 1.2 M second solution. What is
liquid A (PA0) = 100 torr, Vapour pressure of pure liquid the molarity of the final mixture  (2005)
B (PB0) = 300 torr/
(A) 1.20 M (B) 1.50 M
(A) 200 torr (B) 140 torr
(C) 1.344 M (D) 2.70 M
(C) 180 torr (D) None of these
Q.7 Increasing the temperature of an aqueous solution
Q.18 The van’t Hoff factor for 1.0 M Ba(NO3)2 solution is will cause  (1993)
2.74. The degree of dissociation is
(A) Decrease in molality
(A) 91.3% (B) 87% (C) 100% (D) 74%
(B) Decrease in molarity
(C) Decrease in mole fraction
Previous Years’ Questions (D) Decrease in %w/w

Q.1 The Henry’s law constant for the solubility of N2 gas


in water at 298 K in 1.0 × 105 atm. The mole fraction Q.8 6.02 × 1020 molecules of urea are present in 100 mL of
of N2 in air is 0.8. The number of moles of N2 from its solution. The concentration of urea solution is  (2004)
air dissolved in 10 moles of water at 298 K and 5 atm (A) 0.02 M (B) 0.01 M (C) 0.001 M (D) 0.1 M
pressure is  (2009)
[Avogadro constant, NA = 6.02 × 10 mol ] 23 –1

(A) 4.0 × 10–4 (B) 4.0 × 10–5


Q.9 To neutralize completely 20 mL of 0.1 M aqueous
(C) 5.0 × 10–4 (D) 4.0 × 10–6
solution of phosphorus acid (H3PO3), the volume of
0.1M aqueous KOH solution required is  (2004)
Q.2 The normality of 0.3 M phosphorus acid (H3PO3) is
 (1999) (A) 40 mL (B) 20 mL (C) 10 mL (D) 60 mL
(A) 0.1 (B) 0.9 (C) 0.3 (D) 0.6
Q.10 A 5.2 molal aqueous solution of methyl alcohol,
Q.3 With increase of temperature, which of these CH3OH, is supplied. What is the mole fraction of methyl
changes? (2002) alcohol in the solution? (2011)

(A) Molality (A) 0.100 (B) 0.190 (C) 0.086 (D) 0.050

(B) Weight fraction of solute


Q.11 The density of solution prepared by dissolving
(C) Fraction of solute present in water 120 g of urea (mol mass = 60 u) in 1000g water is 1.15
(D) Mole fraction g/mL. The molarity of this solution is  (2012)
(A) 0.50 M (B) 1.78 M (C) 1.02 M (D) 2.05 M
2 0 . 3 6 | Solutions and Colligative Properties

Q.12 Equimolar solutions in the same solvent have Q.18 Value of gas constant R is  (2002)
 (2005)
(A) 0.082 litre atm (B) 0.987 cal mol–1K–1
(A) Same boiling point but different freezing point
(C) 8.3 J mol–1 K–1 (D) 83 erg mol–1 K–1
(B) Same freezing point but different boiling point
(C) Same boiling and same freezing point Q.19 During depression of freezing point in a solution
the following are in equilibrium  (2003)
(D) Different boiling and different freezing point
(A) Liquid solvent, solid solvent
Q.13 Two liquids X and Y form an ideal solution. At (B) Liquid solvent, solid solute
300K, vapour pressure of the solution containing 1
(C) Liquid solute, solid solute
mol of X and 3 moles of Y is 550 mm Hg. At the same
temperature, if 1 mol of Y is further added to this (D) Liquid solute, solid solvent
solution, vapour pressure of the solution increases by
10 mm Hg. Vapour pressure (in mm Hg) of X and Y in Q.20 The degree of dissociation (α) of a weak electrolyte,
their pure states will be respectively (2009) AxBy is related to van’t Hoff factor (i) by the expression
(A) 200 and 300 (B) 300 and 400 (2011)
i −1 i −1
(C) 400 and 600 (D) 500 and 600 (A) α = (B) α =
(x + y − 1) x + y +1

Q.14 The vapour pressure of water at 20ºC is 17.5 mm x + y −1 x + y +1


(C) α = (D) α =
Hg. If 18g of glucose (C6H12O6) is added to 178.2 g i −1 i −1
of water at 20ºC the vapour pressure of the resulting
solution will be  (2005) Q.21 At 80°C , the vapour pressure of pure liquid ‘A’
(A) 15.750 mm Hg (B) 16.500 mm Hg is 520 mm Hg and that of pure liquid ‘B’ is 1000 mm
Hg. If a mixture solution of ‘A’ and ‘B’ boils at 80°C
(C) 17.325 mm Hg (D) 17.675 mm Hg and 1 atm pressure, the amount of ‘A’ in the mixture is
(1 atm = 760 mm Hg) (2008)
Q.15 A mixture of ethyl alcohol and propyl alcohol
(A) 52 mol percent (B) 34 mol percent
has a vapour pressure of 290 mm at 300K. The vapour
pressure of propyl alcohol is 200 mm. If the mole (C) 48 mol percent (D) 50 mol percent
fraction of ethyl alcohol is 0.6, its vapour pressure (in
mm) at the same temperature will be  (2007) Q.22 If sodium sulphate is considered to be completely
(A) 350 (B) 300 (C) 700 (D) 360 dissociated into cations and anions in aqueous solution,
the change in freezing point of water ( ∆ Tf ) , when 0.01
mol of sodium sulphate is dissolved in 1 kg of water, is
Q.16 Which of the following liquid pair shows a positive
(Kf = 1.86 K kg mol-1)  (2010)
deviation from Raoult’s law  (2004)
(A) 0.0372 K (B) 0.0558 K
(A) Water–nitric acid
(C) 0.0744 K (D) 0.0186 K
(B) Benzene–methanol
(C) Water–hydrochloric acid Q.23 Ethylene glycol is used as an antifreeze in a cold
(D) Acetone–chloroform climate. Mass of ethylene glycol which should be added
to 4 kg of water to prevent it from freezing at −6°C will
be: [Kf for water = 1.86 K kg mol-1, and molar mass of
Q.17 A 5.25% solution of a substance is isotonic with
ethylene glycol = 62g mol-1]
1.5% solution of urea (molar mass = 60g mol–1) in the
same solvent. If the densities of both the solutions are (A) 204.30g (B) 400.00 g
assumed to be equal to 1.0 g cm–3, molar mass of the
(C) 304.60 g (D) 804.32g
substance will be (2007)
(A) 90.0 g mol–1 (B) 115.0 g mol–1
(C) 105.0 g mol–1 (D) 210.0 g mol–1
Chem i str y | 20.37

Q.24 Kf for water is 1.86 K kg mol-1. If your automobile (C) 0.125 M Na3PO 4( aq) has the highest osmotic
radiator holds 1.0 kg of water, how many grams of pressure.
ethylene glycol (C2H6O2) must you add to get the freezing
(D) 0.500 M C2H5OH( aq) has the highest osmotic
point of the solution lowered to ‒2.8°C? (2012)
pressure.
(A) 72g (B) 93g (C) 39g (D) 27g
Q.26 The vapour pressure of acetone at 20°C is 185 torr.
Q.25 Consider separate solutions of 0.500 M When 1.2 g of a non-volatile substance was dissolved in
100 g of acetone a 20°C, its vapour pressure was 183 torr.
C2H5OH( aq) , 0 . 1 0 0 M Mg3 (PO 4 ) 0.250 M KBr( aq)
2( aq) The molar mass (g mol-1) of the substance is: (2014)
and 0.125 M Na3PO 4( aq) at 25°C . Which statement
(A) 32 (B) 64 (C) 128 (D) 488
is true about these solutions, assuming all salts to be
strong electrolytes? (2014)
Q.27 18 g glucose ( C6H12O6 ) is added to 178.2 g water.
(A) They all have the same osmotic pressure. The vapor pressure of water (in torr) for this aqueous
solution is.  (2016)
(B) 0.100 M Mg3 (PO 4 ) has the highest osmotic
2( aq)
pressure. (A) 76.0 (B) 752.4 (C) 759.0 (D) 7.6

JEE Advanced/Boards

Exercise 1 Q.7 A solution containing 3.24 of a nonvolatile


nonelectrolyte and 200g of water boils at 100.130ºC at
Q.1 At 25ºC, the vapour pressure of methyl alcohol 1atm. What is the molecular weight of the solute? (Kb
is 96.0 torr. What is the mole fraction of CH3OH in a for water 0.513ºC/m).
solution in which the (partial) vapour pressure of CH3OH
is 23.0 torr at 25ºC? Q.8 The molecular weight of an organic compound is
58.0 g/mol. Compute the boiling point of a solution
Q.2 The vapour pressure of pure liquid solvent A is 0.80 containing 24.0 g of the solute and 600g of water, when
atm. When a non–volatile substance B is added to the the barometric pressure is such that pure water boils at
solvent its vapour pressure drops to 0.60 atm. What is 99.725ºC.
the mole fraction of components B in the solution?
Q.9 An aqueous solution of a non–volatile solute boils
Q.3 The vapour pressure of pure water at 26ºC is 25.21 at 100.17ºC. At what temperature will this solution
torr. What is the vapour pressure of a solution which freeze? [Kf for water 1.86ºC/m]
contains 20.0 glucose, C4H12O6, in 70g water?
Q.10 Find the freezing point of a glucose solution
Q.4 The vapour pressure of pure water at 25ºC is 23.76 whose osmotic pressure at 25ºC is found to be 30 atm.
torr. The vapour pressure of a solution containing 5.40 Kf(water) = 1.86 kg/mol K.
g of a non–volatile substance in 90.0g water is 23.32
torr. Compute the molecular weight of the solute. Q.11 The vapour pressure of an aqueous solution of
glucose is 750 mm Hg at 373K. Calculate molality and
Q.5 What weight of the non–volatile solute, urea needs mole fraction of solute.
to be dissolved in 100g of water, in order to decrease
the vapour pressure of water by 25%? What will be the Q.12 The vapour pressure of pure benzene at 25ºC is
molality of the solution? 639.7 mm of Hg and the vapour pressure of a solution
of a solute in C6H6 at the same temperature is 631.7mm
Q.6 When 10.6 g of a non–volatile substance is dissolved of Hg. Calculate molality of solution.
in 740g of ether, its boiling point is raised 0.284ºC. What
is the molecular weight of the substance? Molal boiling Q.13 The vapour pressure of pure benzene at a certain
point constant for ether is 2.11ºC kg/mol. temperature is 640 mm of Hg. A nonvolatile non-
2 0 . 3 8 | Solutions and Colligative Properties

electrolyte solid weighing 2.175 g is added to 39.0 of raised to 25ºC, when the osmotic pressure is found to
benzene. The vapour pressure of the solution is 600 mm be 105.3 mm. Determine extent of dilution.
of Hg. What is molecular weight of solid substance?
Q.23 The freezing point of a solution containing 0.2g
Q.14 The vapour pressure of water is 17.54 mm Hg at of acetic acid in 20.0 g of benzene is lowered by 0.45ºC.
293 K. Calculate vapour pressure of 0.5 molal solution Calculate the degree of association of acetic acid in
of a solute in it. benzene.

Q.15 Benzene and toluene form two ideal solutions A Q.24 0.85% aqueous solution of NaNO3 is apparently
and B at 313K. Solution A (total pressure PA) contains 90% dissociated at 27ºC. Calculate its osmotic pressure.
equal moles of toluene and benzene. Solution B contain (R = 0.082 l atm K–1 mol–1).
equal masses of both (total pressure PB). The vapour
pressure of benzene and toluene are 160 and 60 mm Q.25 A 1.2% solution (w/v) of NaCl is isotonic with 7.2%
Hg respectively at 313K. Calculate the value of PA/PB. solution (w/v) of glucose. Calculate degree of ionization
and Van’t Hoff factor of NaCl.
Q.16 Sea water is found to contain 5.85% NaCl and
9.50% MgCl by weight of solution. Calculate its normal Q.26 Calculate the boiling point of a solution containing
boiling point assuming 80% ionization for NaCl and 0.61g of benzoic acid in 50g of carbon disulphide
50% ionization of MgCl2 [K2(H2O) = 0.51 kg mol–1 K]. assuming 84% dimerization of the acid. The boiling
point and Kb of CS2 are 46.2ºC and 2.3 K kg mol–1,
Q.17 The latent heat of fusion of ice is 80 calories per respectively.
gram at 0ºC. What is the freezing point of a solution
of KCl in water containing 7.45 grams of solute in 500 Q.27 At 25ºC, 1 mol of A having a vapour pressure of
grams of water, assuming that the salt is dissociated to 100 torr and 1 mol of B having a vapour pressure of 300
the external of 95%? torr were mixed. The vapour at equilibrium is removed,
condensed and the condensate is heated back to
Q.18 A complex is represented as CoCl3 · xNH3. Its 0.1 25ºC. The vapours now formed are again removed,
molal solution in aq. solution shows DTf = 0.558ºC. Kf recondensed and analysed. What in the mole fraction
for H2O is 1.86 K mol–1 kg. Assuming 100% ionization of A in this condensate?
of complex and coordination no. of Co is six, calculate
formula of complex. Q.28 Phenol associates in benzene to a certain extent to
form a dimer. A solution containing 20 × 10–3 kg phenol
Q.19 The molar volume of liquid benzene (density in 1kg of benzene has its freezing point depressed
= 0.877 g mL–1) increases by a factor of 2750 as it by 0.69K. Calculate the fraction of phenol that has
vaporizes at 20ºC and that of liquid toluene (density dimerised K for benzene = 5.12 kg mol–1 K.
= 0.867 mL–1) increases by a factor of 7720 at 20ºC.
Solution of benzene and toluene has a vapour pressure Q.29 30 mL of CH3OH (d = 0.7980 gm cm–3) and 70 mL
of 46.0 torr. Find the mole fraction of benzene in vapour of H2O (d = 0.9984 gm cm–3) are mixed at 25ºC to form
above the solution. a solution of density 0.9575 gm cm–3. Calculate the
freezing point of the solution. Kf (H2O) is 1.86 kg mol–1
Q.20 At 100ºC, benzene and toluene have vapour K. Also calculate its molarity.
pressure of 1375 and 558 torr respectively. Assuming
these two form an ideal binary solution, calculate the
composition of the solution that boils at 1 atm and Exercise 2
100ºC. What is the composition of vapour issuing at
these conditions? Single Correct Choice Type

Q.21 At 300K, the solution of glucose in water of Q.1 For an ideal binary liquid solution with PA0 > PB0,
concentration 0.01 M and 0.001M are separated which relation between XA (mole fraction of A in liquid
by semipermeable membrane. Pressure needs to phase) and YA (mole fraction of A in vapour phase) is
be applied on which solution, to prevent osmosis? correct?
Calculate the magnitude of this applied pressure.
(A) YA < YB (B) XA > XB
Q.22 At 10ºC, the osmotic pressure of urea solution is YA XA YA XA
500mm. The solution is diluted and the temperature is (C) > (D) <
YB XB YB XB
Chem i str y | 20.39

Q.2 Which of the following aqueous solution will show v2 (10)2


maximum vapour pressure at 300K? (B) tan=
θ = = 1
Rg (10)(10)
(A) 1 M NaCl (B) 1 M CaCl2
(C) θ= tan−1 (1)= 45°
(C) 1 M AlCl3 (D) 1 M C12H22O11
dv
(D) =a
dt
Q.3 At a given temperature, total vapour pressure in
Torr of a mixture of volatile components A and B is
Q.7 FeCl3 on reaction with K4[Fe(CN)6] in aqueous solution
given by PTotal = 120 – 75XB hence, vapour pressure of
given blue colour. These are separated by a semipermeable
pure A and B respectively (in Torr) are
membrane AB as shown. Due to osmosis there is
(A) 120, 75 (B) 120, 95 0.1M 0.01M
(C) 120, 45 (D) 75, 45 K4Fe(CN) 6 FeCl 3

Side x Side y
Q.4 A liquid mixture having
composition corresponding to z 5PM
point z in the figure shown in
subjected to distillation at (A) Blue colour formation in side X
Temp
constant pressure. Which of (B) Blue colour formation in side Y
the following statement is (C) Blue colour formation in both of the sides X and Y
correct about the process xA=0 mole xA=1
yA=0 fraction yA=1 (D) No blue colour formation
(A) The composition of
distillate differs from the mixture Multiple Correct Choice Type
(B) The boiling point goes on changing
Q.8 Acetone and carbon disulphide form binary liquid
(C) The mixture has highest vapour pressure than for solution showing positive deviation from Raoult’s law.
any other composition The normal boiling point (Tb) of pure acetone is less
(D) Composition of an azeotrope alters on changing than that of pure CS2. Pick out the incorrect statement
the external pressure among the following
(A) Boiling temperature of mixture is always less than
boiling temperature of acetone
Q.5 The following graph represents variation of boiling
point with composition of liquid and vapour of binary (B) Boiling temperature of Azetropic mixture is always
liquid mixture. The graph is plotted at constant pressure. less than boiling temperature of acetone
Which of the following statement(s) is incorrect. Here (C) When a small amount of CS2 (less volatile component)
X and Y stands for mole fraction in liquid and vapour is added to excess of acetone boiling point of resulting
phase respectively mixture increases
(D) A mixture of CS2 and CH3COCH3 can be completely
2L separated by simple fractional distillation.
⇒t=
ω2R
(A) Xbenzene = 0.5 and Ytoluene = 0. 2 Q.9 Which of the following would be equal to zero
(B) Xtoluene = 0.3 and Ybenzene = 0. 6 when the liquid pairs form an ideal solution
(C) Xbenzene = 0.3 and Ytoluene = 0. 4 (A) ∆H (B) ∆S (C) ∆G (D) ∆V
(D) Xbenzene = 0.7 and Ytoluene < 0. 3
Q.10 For a non–volatile solute
Q.6 Which of the following represent correctly the (A) The vapour pressure of solute is zero
changes in thermodynamic properties during the (B) Vapour pressure of solution = Vapour pressure of
formation of 1 mol of an ideal binary solution pure solvent
+
(C) Vapour pressure of solution = Vapour pressure of
∆Gmin
T∆Smin solvent in solution
(A) –1
J mol 0
(D) All of the above
– ∆Hmin
Mole fraction
2 0 . 4 0 | Solutions and Colligative Properties

Q.11 A difference between diffusion and osmosis is Q.17 Which of the following is/are true?
(A) A semipermeable membrane is required for osmosis (A) For the same solution, elevation in boiling point =
while diffusion requires no semipermeable membrane depression in freezing point
(B) In osmosis movement of molecules is only in one (B) The Van’t Hoff factor for a dilute solution of BaCl2.
direction whereas in diffusion movement is on both sides
(C) The elevation in boiling point is due to increase in
(C) In osmosis only the solvent moves while in diffusion
vapour pressure.
both solute and solvent move
(D) None of the above (D) The depression in freezing point is due to decrease
in vapour pressure.
Q.12 Which of the following statement is/are correct?
Q.18 Which pairs of liquids on mixing is/are expected
(A) The freezing point of water is depressed by the
addition of glucose to show no net volume change and no heat effect?
(B) The degree of dissociation of a weak electrolyte (A) Acetone and ethanol
decrease as its concentration decreases (B) Chloro benzene and bromo benzene
(C) Energy is released when a substance dissolves
in water provided that the hydration energy of the (C) Chloroform and benzene
substance is more than its lattice energy (D) n–Butyl chloride and n–butyl bromide
(D) If two liquids that form an ideal solution are mixed,
the change in entropy is positive Assertion Reasoning Type

Q.13 Which of the following gases are highly soluble Q.19 Statement-I: Addition of ethylene glycol (non–
in water volatile) to water lowers the freezing point of water
(A) HCl (B) SO2 (C) NH3 (D) H2 hence used as antifreezing
Statement-II: Heat must be removed from the water to
Q.14 Which of the following would be equal to zero make it freeze.
when the liquid pairs form an ideal solution.
(A) Statement-I is true, statement-II is true and
(A) ∆H (B) DS (C) ∆G (D) DV statement-II is correct explanation for statement-I
(B) Statement-I is true, statement-II is true and
Q.15 Two miscible liquids A and B having vapour
statement-II is NOT the correct explanation for
pressure in pure state PA0 and PB0 are mixed in mole
statement-I
reaction XA and XB to get a mixture having total vapour
pressure of mixture PM. Which of the following relation (C) Statement-I is true, statement-II is false.
are correct? (D) Statement-I is false, statement-II is true
PM − PB0 X A(  ) PM
(A) X A = (B) =
PA 0 − PB0 X'A(V ) PA 0 Q.20 Statement-I: The freezing of water is an
endothermic process.
X A(  ) PM
(C) = (D) All of these Statement-II: Heat must be removed from the water to
X'A(V) PB0
make it freeze.
(A) Statement-I is true, statement-II is true and
Q.16 Which relations are not correct for aqueous dilute
statement-II is correct explanation for statement-I.
solution of K3PO4 if its degree of dissociation is α?
(B) Statement-I is true, statement-II is true and
∆P Molality × 18 × (1 + 3α ) statement-II is NOT the correct explanation for
(A) =
Pº 1000 statement-I.
∆P πobs × 18 × (1 + 3α ) (C) Statement-I is true, statement-II is false.
(B) =
Pº RT × 1000 (D) Statement-I is false, statement-II is true
∆P ∆Tf obs × 18
(C) =
Pº K f × 1000
(D) Mw of K2PO4 = Mwobs × (1 + 3α)
Chem i str y | 20.41

Comprehension Type Match the Columns


The various relations between colligative properties Q.26 PA = partial pressure of components A in liquid
and molecular mass are applicable only to solutions of mixture. PA0 = vapour pressure of A, χA = mole fraction
non–electrolytes and in dilute solutions. Out of various of A in liquid
colligative properties osmotic pressure measurement is
especially suitable for the determination of molecular Column I Column II
masses of proteins whereas freezing point depression (A) C2H5OH + H2O (p) Azeotropic mixture
and other colligative properties are too small. Certain
(B) C2H2Br + C2H5I (q) Obeys Raoult’s law
solutes which dissociate or associate in solution affect
the colligative property and hence molecular mass (C) PA = XAPA0 (r) Non–ideal solution with
determination also. In case of association observed positive deviation
molecular mass is more than normal the correction
(D) H2O + H2SO4 (s) Non–ideal solution with
factor i < 1. For dissociation of solute i > 1 and observed
negative deviation
molecular mass is less than the normal molecular mass
(t) Ideal solution
Q.21 Which of the following 0.1 m aqueous solution
will have the lowest freezing point?
Q.27 Match the following
(A) Al2(SO4)3 (B) C6H12O6
Column I Column II
(C) KI (D) C12H22O11
(A) Urea, glucose, fructose (p) 1 : 0.8 : 1

Q.22 Correction factor for 0.1M ideal solution is (B) NaCl, MgCl2.K2SO4 (q) 1 : 2 : 1

(A) 0.1 (B) 1 (C) 0.01 (D) 2 > 1 (C) Al2(SO4)3, Na3PO4, K3r[Fe(CN)6] (r) 1 : 1 : 1

(D) Glucose, NaCl, CaCl2 (s) 2 : 3 : 3


Q.23 The Van’t Hoff factor for 0.1M Ba(NO3)2 solution is
2.74. The degree of dissociation is
(A) 91.3% (B) 87% (C) 100% (D) 74% Q.28

Column I Column II
Q.24 Which one of the following statements gives
below concerning properties of a solution describe a (A) ppm (p) Van’t Hoff factor
colligative effect? (B) DTb (q) Molal depression
(A) Boiling point of pure water increase by addition of (C) Kf (r) Elevation in boiling point
EtOH
(B) Vapour pressure of pure water decrease by addition (D) i
Mass of solute
of HNO3 (s) × 106
Mass of solution
(C) Vapour pressure of pure benzene decrease by
addition of naphthalene
(D) Boiling point of pure benzene increase by addition
of toluene Previous Years’ Questions

Q.25 After adding a solute, freezing point of water Q.1 An azeotropic solution of two liquids has boiling
decreases to –0.186ºC. What is the value of DT? point lower than either of them when it (1981)
(kb = 0.521; kf = 1.86) (A) Shows negative deviation from Raoult’s law
(A) 0.521 (B) 0.0521 (B) Shows no deviation from Raoult’s law
(C) 1.86 (D) 0.0186 (C) Shows positive deviation from Raoult’s law
(D) Is saturated
2 0 . 4 2 | Solutions and Colligative Properties

Q.2 Which of the following 0.1M aqueous solution will In answering the following questions, consider the
have the lowest freezing point? (1989) solution to be ideal dilute solution and solute to be
non–volatile and non–dissociative. (2008)
(A) Potassium sulphate (B) Sodium chloride
(C) Urea (D) Glucose
Q.5 The freezing point of the solution M is
(A) 268.7 K (B) 268.5 K
Q.3 During depression of freezing point in a solution
the following are in equilibrium (2003) (C) 234.2 K (D) 150.9 K
(A) Liquid solvent, solid solvent
Q.6 The vapour pressure of the solution M is
(B) Liquid solvent, solid solute
(A) 39.3 mm Hg (B) 36.0 mm Hg
(C) Liquid solute, solid solute
(C) 29.6 mm Hg (D) 28.8 mm Hg
(D) Liquid solute, solid solvent

Q.4 The freezing point (in ºC) of solution containing Q.7 Water is added to the solution M such that the
0.1 g of K3[Fe(CN6)] (mol. wt. 329) in 100 g of water mole fraction of water in the solution becomes 0.9. The
(Kf = 1.86 K kg mol–1) is (2011) boiling point of this solution is
(A) 380.4 K (B) 376.2 K
(A) –2.3 × 10–2 (B) –5.7 × 10–2
(C) 375.5 K (D) 354.7 K
(C) –5.7 × 10–3 (D) –1.2 × 10–2

Q.8 Following statement is true only under some


Paragraph 3
specific conditions. Write the condition, for it “Two
Properties such as boiling point, freezing point and volatile and miscible liquids can be separated by
vapour pressure of a pure solvent change when solute fractional distillation into pure components”.
molecules are added to get homogeneous solution.
These are called colligative properties. Applications of Q.9 An organic compound (CxH2yOy) was burnt with
colligative properties are very useful in day-to-day life. twice the amount of oxygen needed for complete
One of its examples is the use of ethylene glycol and combustion to CO2 and H2O. The hot gases when
water mixture as anti–freezing liquid in the radiator of cooled to 0ºC and 1 atm pressure, measured 2.24 L.
automobiles. The water collected during cooling weigh 0.9g. The
vapour pressure of pure water at 20ºC is 17.5 mm Hg
A solution M is prepared by mixing ethanol and water.
and is lowered by 0.104 mm when 50g of the organic
The mole fraction of ethanol in the mixture is 0.9.
compound are dissolved in 1000g of water. Give the
Given, freezing point depression constant of water molecular formula of the organic compound. (1983)
(kfwater) = 1.86 K kg mol–1
freezing point depression constant of ethanol (kfethanol) Q.10 The vapour pressure of ethanol and methanol are
= 2.0 K kg mol–1 44.5 and 88.7 mm Hg respectively. An ideal solution
Boiling point elevation constant of water (kbwater) is formed at the same temperature by the mixing 60g
= 0.52 K kg mol–1 of ethanol with 40g of methanol. Calculate the total
vapour pressure of the solution and the mole fraction
Boiling point elevation constant of water (kfethanol)
of methanol in the vapour.  (1986)
= 1.2 K kg mol–1
Standard freezing point of water = 273 K
Q.11 The vapour pressure of pure benzene at a certain
Standard freezing point of ethanol = 155.7 K temperature is 640 mm Hg. A non–volatile, non–
Standard boiling point of water = 373 K electrolyte solid weighing 2.175 g is added to 39.0 g of
benzene. The vapour pressure of solution is 600 mm Hg.
Standard boiling point of ethanol = 351.5 K
What is the molecular weight of the solid substance?
Vapour pressure of pure water = 32.8 mm Hg  (1990)
Vapour pressure of pure ethanol = 40 mm Hg
Molecular weight of water = 18 g mol–1
Molecular weight of ethanol = 46 g mol–1
Chem i str y | 20.43

Q.12 The degree of dissociation of Ca (NO3)2 in a dilute These are called colligative properties. Applications of
aqueous solution, containing 7.0g of the salt per 100g colligative properties are very useful in day-to- day life.
of water at 100ºC is 70%. If the vapour–pressure of One of its examples is the use of ethylene glycol and
water at 100ºC is 760 mm, calculate the vapour pressure water mixture as anti-freezing liquid in the radiator of
of the solution. (1991) automobiles.
A solution M is prepared by mixing ethanol and water.
Q.13 Nitrobenzene is formed as the major product The mole fraction of ethanol in the mixture is 0.9.
along with a minor product in the reaction of benzene
Given: Freezing point depression constant of water
with a hot mixture of nitric acid and sulphuric acid. The
minor product consists of carbon : 42.86%, hydrogen : (K ) = 1.86 K kg mol Freezing point depression
water
f
−1

2.40%, nitrogen : 16.67% and oxygen : 38.07%, constant of ethanol (K ) = 2.0 K kg mol Boiling
ethanol
f
−1

(i) Calculate the empirical formula of the minor product. point elevation constant of water (K )
water
b
(ii) When 5.5g of the minor product is dissolved in 45g = 0.52 K kg mol−1
of benzene, the boiling point of the solution is 1.84ºC
higher than that of pure benzene. Calculate the molar (
Boiling point elevation constant of ethanol Kbethanol )
= 1.2 K kg mol−1
mass of the minor product then determine its molecular
and structural formula. (Molal boiling point elevation In answering the following questions, consider the
constant of benzene is 2.53 K kg mol–1). (1999) solutions to be ideal dilute solutions and solutes to be
non-volatile and non- dissociative. (2008)
Q.14 The vapour pressure of two miscible liquids (A)
and (B) are 300 and 500 mm of Hg respectively. In a flask
Q.17 The freezing point of the solution M is
10 moles of (A) is mixed with 12 moles of (B). However,
as soon as (B) is added, (A) starts polymerizing into a (A) 268.7 K (B) 268.5 K
completely insoluble solid. The polymerization follows (C) 234.2 K (D) 150.9 K
first order kinetic. After 100min, 0.525 mole of a solute is
dissolved which arrests the polymerization completely.
The final vapour pressure of the solution is 400 mm of Q.18 The vapour pressure of the solution M
Hg. Estimate the rate constant of the polymerization (A) 39.3 mm Hg (B) 36.0 mm Hg
and ideal behavior for the final solution.  (2001)
(C) 29.5 mm Hg (D) 28.8 mm Hg

Q.15 Consider the three solvents of identical molar


Q.19 Water is added to the solution M such that the
masses. Match their boiling point with their kb values 
fraction of water in the solution becomes 0.9. The
 (2003)
boiling point of this solution is
Solvents Boiling point kb values (A) 380.4 K (B) 376.2 K
X 100ºC 0.92 (C) 375.5 K (D) 354.7 K
Y 27ºC 0.63
Q.20 The Henry’s law constant for the solubility of N2
Z 283ºC 0.53
gas in water at 298 K is 1.0 × 105 atm. The mole fraction
of N2 in air is 0.8. The number of moles of N2 from air
dissolved in 10 moles of water at 298 K and 5 atm
Q.16 When 20 g of naphthoic acid (C11H8O2) is dissolved
pressure is (2009)
in 50 g of benzene (Kf = 1.72 K kg mol−1), a freezing
point depression of 2 K is observed. The van’t Hoff (A) 4.0 × 10−4 (B) 4.0 × 10−5
factor (i ) is (2007)
(C) 5.0 × 10−4 (D) 4.0 × 10−6
(A) 0.5 (B) 1 (C) 2 (D) 3
Q.21 The freezing point (in oC) of a solution containing
Paragraph for Question Nos. 17 to 19 0.1 g of K3[Fe(CN)6 (Mol. Wt. 329) in 100 g of water
(Kf = 1.86 K kg mol-1) is (2011)
Properties such as boiling point, freezing point and
vapor pressure of a pure solvent change when solute (A) -2.3 × 10-2 (B) -5.7 × 10-2
molecules are added to get homogeneous solution.
(C) -5.7 × 10-3 (D) -1.2 × 10-2
2 0 . 4 4 | Solutions and Colligative Properties

Q.22 For a dilute solution containing 2.5 g of a non- Q.24The mole fraction of a solute in a solution is 0.1.
volatile non- electrolyte solute in 100 g of water, the At 298 K, molarity of this solution is the same as its
elevation in boiling point at 1atm pressure is 2°C. molality. Density of this solution at 298 K is 2.0 g cm-3.
Assuming concentration of solute is much lower than The ratio of the molecular weights of the solute and
the concentration of solvent, the vapor pressure (mm  MWsolute 
of Hg) of the solution is (take Kb = 0.76 K kg mol-1) solvent   , is (2016)
 (2012)  MWsolvent 

(A) 724 (B) 740 (C) 736 (D) 718 Q.25 Mixture(s) showing positive deviation from
Raoults law at 35οC is (are) (2016)
Q.23 If the freezing point of a 0.01 molal aqueous (A) Carbon tetrachloride + methanol
solution of a cobalt (III) chloride-ammonia complex
(which behaves as a strong electrolyte) is – 0.05580C, (B) Carbon disulphide + acetone
the number of chloride(s) in the coordination sphere of (C) Benzene + toluene
the complex is [Kf of water = 1.86 K kg mol–1]
(D) Phenol + aniline

PlancEssential Questions
JEE Main/Boards JEE Advanced/Boards

Exercise 1 Exercise 1
Q.2 Q.4 Q.9 Q.6 Q.10 Q.16
Q.14 Q.29 Q.20 Q.21 Q.25

Exercise 2 Exercise 2
Q.2 Q.4 Q.14 Q.6 Q.7 Q.12
Q.17 Q.15 Q.17 Q.25
Q.28
Previous Years' Questions
Q.6 Q.9 Q.11 Previous Years' Questions
Q.16 Q.17 Q.1 Q.13
Chem i str y | 20.45

Answer Key

JEE Main/Boards
Exercise 1
Q.1 4.9 gm Q.2 0.89 Q.3 (i) 0.52m, (ii) 0.99
Q.6 0.2121 m Q.7 53.06g mol–1 Q.8 23.44 mm Hg
Q.9 0.988; 0.156 m Q.121.35 mol/litre; 1.52 mol/kg Q.13 0.283 mol/L
Q.14 0.099 mol L–1 Q.15 0.23 K
Q.16 158.9 mL of 0.1 M HCl Q.17 57.5 g mol–1 Q.18 256 g mol–1
Q.19 10 g Q.21 (b) 7.389 atm
Q.22 Degree of dissociation = 90% Q.23 18.6 × 10–4
Q.24 Atomic mass of A = 25.59; B = 42.64 Q.25 1.0753; 30.68 × 10–4 Q.28 (b) 92%

Exercise 2
Single Correct Choice Type

Q.1 C Q.2 C Q.3 A Q.4 A Q.5 C Q.6 D


Q.7 B Q.8 D Q.9 B Q.10 B Q.11 C Q.12 B
Q.13 A Q.14 C Q.15 B Q.16 C Q.17 D Q.18 B

Previous Years’ Questions


Q.1 A Q.2 D Q.3 C Q.4 A Q.5 B Q.6 C
Q.7 B Q.8 B Q.9 A Q.10 C Q.11 D Q.12 C
Q.13 C Q.14 C Q.15 A Q.16 B Q.17 D Q.18 C
Q.19 A Q.20 A Q.21 D Q.22 B Q.23 D Q.24 B
Q.25 A Q.26 B Q.27 B

JEE Advanced/Boards
Exercise 1
Q.1 0.24 Q.2 0.25 Q.3 24.5 torr
Q.4 57.24 g/mol Q.5 111.1 g, 18.52 molal Q.6 106 g/molal
Q.7 64.0 g/mol Q.8 100.079ºC Q.9 –0.62ºC
Q.10 T = -2.28ºC Q.11 0.741m, 0.013 Q.12 0.162 m
Q.13. 65.25 Q.14 17.38 Q.15 0.964
Q.16 Tb = 101.9ºC Q.17 Tf = –0.73ºC Q.18 [Co(NH3)Cl]Cl2
Q.19 0.73 Q.20 xB = 0.2472, yb = 0.4473
2 0 . 4 6 | Solutions and Colligative Properties

Q.21 P = 0.2217 atm should be applied Q.22 (Vtotal = 5.Voriginal)


Q.23 94.5% Q.24 4.64 atm Q.25 0.95; 1.95
Q.26 46.33ºC Q.27 x = 0.1 Q.28 a = 0.7333
Q.29 –19.91ºC, 7.63M

Exercise 2
Single Correct Choice Type

Q.1 C Q.2 D Q.3 C Q.4 D Q.5 B Q.6 C

Q.7 D

Multiple Correct Choice Type

Q.8 A, C, D Q.9 A, D Q.10 A, C Q.11 A, B, C Q.12 A, C, D Q.13 A, B, C

Q.14 A, D Q.15 A, B Q.16 A, C, D Q.17 B, D Q.18 B, D

Assertion Reasoning Type

Q.19 C Q.20 D

Comprehension Type

Q.21 A Q.22 B Q.23 B Q.24 C Q.25 B

Match the Columns

Q.26 A → p, r; B → q, t; C → q, t; D → p, s Q.27 A → r; B → s; C → p; D → q

Q.28 A → s; B → r; C → q; D → p

Previous Years’ Questions

Q.1 C Q.2 A Q.3 A Q.4 A Q.5 D Q.6 A

Q.7 B Q.8 Condition Q.9 C5H10O11 Q.10 0.657 Q.11 65.25 Q.12 746.32

Q.13 168 Q.14 1.005 × 10–4 min–1 Q.15 Subjective Q.16 A Q.17 D

Q.18 B Q.19 B Q.20 A Q.21 A Q.22 A Q.23 3

Q.24 9 Q.25 A,B


Chem i str y | 20.47

Solutions

JEE Main/Boards Ws
Where δs density
= = of solute
V
Exercise 1 RTδs

= π ∆T0
kb
Sol 1: We have
Molecular mass of H2SO 4 = 98g Sol 6: We have,
Let weight be w π =CR T
w π 4.85
Then moles n = ⇒C= = = 0.201 m
98 RT 0.0821 × 293
n Weight of solvent of solute
= 0.201 ×=
1 0.201
Now = 0.2
V
w × 1000 Weight of solvent = 1015 − 68.95g =
946.04 g
= 0.2
98 × 250 0.201
∴ Molality = × 1000
946.04
w = 4.9 g
Molality = 0.2121m
Sol 2: Molecular wt of ethanol = 46 gm
Sol 7: We have
23
Moles of ethanol n= = 0.5 ∆ Tb 0.7
1
46 ∆ Tb =
kb m ⇒ m= = ;m
;m= =1.346m
1.346m
kb 0.52
Molecular mass of water = 18 gm n
m=
72 ws
Moles of water, n=
2 = 4
18 n=
1.346× | 75 |1000; n =
0.235
n1 0.5
=Xe = = 0.11 w w 12.5
n1 + n2 4 + 0.5 n= ⇒M= =
m n 0.235
n2 4
Xw
= = = 0.89
n1 + n2 4 + 0.5 M = 53.6 g mol-1

2.82
Sol 3: Moles of glucose,=
n1 = 0.0152 Sol 8: We have,
180
n1 × 1000 0.0156 Tf 0.30=
∆= C kf m
(i) Molality = = = ×1000= 0.52
30 30 0.3
m
= = 0.161m
30 1.86
(ii) moles of water, n=
2 = 1.66
18 Now we have,
0.0152 P0 − P
=Xg ≈ 0.01
0.0152 + 1.66 = Xa
P0
Xw =
1 − Xg =0.99 n n
m= ⇒ ws =
ws m

Sol 5: ∆ Tb =
kb m ws n
n
=s =
Ms Ms
π = CR T
n n 1 1
π CR T n RT Xa
= = = =
∴ = = RT. ws = .δ ns + n n 1 1
∆ Tb kbm Vkb .n kb s +n 1+ 1+
mMs mMs 18 × 0.161
1000
2 0 . 4 8 | Solutions and Colligative Properties

X a 2.88 × 10−3
= 1 1000
n
Volume of sol = = = 980.39 cm3 = 980.39ml
s 1.020
Now, P0 = 23.51mmHg
1326
23.51 − P Molarity == ×1000= 1.35M
= 2.88 × 10−3 980.39
23.51
n 1.326 × 1000
⇒ P=
23.44mmHg Molality
== = = 1.52m
Ws 870

P0 − P Sol 13: We have,


Sol 9: X a =
P0
π =CR T
639.7 − 631.9
= 1.22 × 10 −2 π 7.2
639.7 ∴C= =
RT 0.0821 × 310
Also, from previous question.
1
=(
T 37
= 0
C 310K )
=Xa = , Ms 78 g
1 C = 0.283M
1+
mMs

1 1 Sol 14: π =CR T


1+ =
mM s X a
π 2.45
C
= = ⇒C=
0.099M
1 1 1 − Xa R T 0.0821 × 300
= − 1=
mM s X a Xa
Sol 15: We have,
Xa
mMs =
1 − Xa 

CH3 COOH 
 CH3 COO + H
− +

Xa n= 2, α= 0.23
m=
(1 − X a ).Ms
i = 1 + (n − 1) α =
1 + 0.23 =
1.23
By substituting the values,
3 × 10−3
Now, n
= = 0.05
We get m = 0.156m 60 × 10−3
Mass of water = Vs
Sol 10: (i) Osmotic pressure is the excess pressure
which must be applied to a given solution in order to = 500 × 0.997
increase it’s vapour pressure until it becomes equal to
= 498.5gm
that of the solution.
0.05 × 1000
wR T ∴m=
M0 = ; w = weight of solute 498.5
πV
m = 0.1
π = osmotic pressure
∴ ∆ Tf =
i m k f = 1.23 × 0.1 × 1.86
(ii) Osmotic pressure has a relatively large magnitude
and is easy to measure compared to ∆ Tb . ∆ Tf =
0.23k

Sol 12: For every 1 kg of solution, Sol 16: Let moles of Na2CO3 and NaHCO3 be n each
and wt. of NaHCO3 be x then,
13
wt.of H2SO 4 = × 1000 =130 g x 1−x n
100 = ⇒ x=
0.446 g
130 84 104 V
moles=
,n = 1.326
98 x
Then, =
n = 5.32 × 10 −3
Wt. of solvent = 870g 84
Now, moles of HCl = moles of
NaHCO3 + 2(molesof Na2Co3 )
Chem i str y | 20.49

V × 0.1 π =iCR T
⇒ =5.32 × 10 −3 (1 + 2) = V = 158.9ml
1000
= 3 × 0.1 × 0.0821 × 300 = 7.389 atm

Sol 17: we have,


Sol 22: For reasons and equations, refer text
kB wB
=M × 1000 KCl K + + Cl−
∆ TB × w A
n=2
2.53 × 180 × 1000
= = M = 57.5g mol-1 i = 1 + (n − 1) α = 1 + α
(354.11 − 353.23) × 90
∴ π =iCR T
kf × w 5.12 × 1 π
Sol 18: M= B
× 1000= × 1000 i=
∆ Tf × w A 0.4 × 50 CR T
4.68 × 74.5
1+ α=
M = 256 g mol−1 7.45 × 0.0821 × 300
1 + α = 1.9
Sol: 19: We have, P = 0.8P0
α = 0.9
P −P
∴ X A= 0 = 0.2 Degree of dissociation = 90%
P0

M Sol 23: CH3COOH CH3COO − + H+


0.2 =
40
M 114 1 − α α    α
+ α2
40 114 ka =
1−α
M M
⇒ + 0.2 = n=2
200 40

⇒M=
10 g i= 1 + α
∆ Tf =
ik f m
Sol 20: ∆ Tb =
kb m
Weight of acetic acid = 1.06 × 0.6 =
0.636 g
2
R Tb
But, kb = 0.636
1000 × ∆ Hv =m = 0.0106
60
RT 2 1
∴ Tb = b .m
1000 × ∆ Hv 0.0205
=i = 1.04
1 1.86 × 0.0106
or ∆ Tb α
∆ Hv α =0.04

Sol 21: (a) For egg shell placed in pure water, Cα2 1.06 × 0.6 (0.04)2
ka
= = ×
endosmosis will take place and shell will swell. 1−α 60 (1 − 0.04)
For Egg shell placed in saturated NaCl solution, k a 18.6 × 10−6
=
exosmosis will occur and shell will shrink.
4−
(b) K 4 [Fe(CN)6 ] 

 +
 4K + [Fe(CN)6 ] Sol 24: We have,
n=5 k f × wa
=M × 1000
α =0.5 ∆ Tf × ws
Let atomic mass of A and B be a and b respectively.
⇒ i = 1 + (n − 1) α
5.1 × 1
⇒ 1 + 4α ∴ a=
+ 2b × 1000
= 110.87
2.3 × 20
⇒ 3
2 0 . 5 0 | Solutions and Colligative Properties

5.1 × 1  1 1
a+=
4b × 1000
= 196.15 1 − 1 −  =
1.3 × 20  2 2
∴ 2b =
85.28
P0 − P 1 61
b = 42.64 = i X= .
P0 2 122
a = 25.60 61 500
+
122 78
Sol 25: k f = 1.86 66.6 − P
= 0.036
66.6
19.5 × 1000
[CH2=
FCOOH].C = 0.5M
78 × 500 P = 64.20 torr
19.5
=m = 0.5m P0 − P
78 (ii) = 0.072 ; P = 61.78 torr
500 P0
1000 (iii) Refer Text
− +
CH2 FCOOH CH2 FCOO + H
C − Cα Cα Cα Sol 28: (a) For 1 molal

n=2 n n
=i ≈
ws qV
i= 1 + α
n
∆ Tf =
i kf m For 1 molar = 1 =
V
1 For density, p > 1 : 1 molal will have more concentration.
=i = 1.075
1.86 × 0.5
(b) KCl 

 +
 K + Cl

α =0.075
n=2
C α2 i= 1 + α
ka
= = 3.06 × 10−3
1−α
0.5
Sol 26: PT= Pa= Pc = 741.8 X a + 632.8 X c = 74.5
m = 0.067
100
= 741.8 X a + 632.8(1 − X a ) 1000
PT 632.8 + 109 X a
=
∆ Tf =
ik f m
PC 632.8 − 632.8 X a
=
0.24
Pa = 632.8 X a =i = 1.92
1.86 × 0.067
Plotting this and given that a
⇒ α =0.92
92% ionisation
741.8
632.8
Sol 29: As, we increase the pressure over a gas, the
number of gaseous particles per unit volume increase
Xa=0 Xa=1 and also the rate at which the gases are striking
the surface of solution to enter it. According to Le-
It will show negative deviation Chatelier’s principle, the gas will try to return to its
previous state and its solubility will increase, until a new
Sol 27: equilibrium is reached resulting in an increase in the
pressure of the gas above the solution and hence its
(a) Refer text
solubility increase.
 1
(b) (i)i =1 −  1 −  α
 n
Chem i str y | 20.51

Exercise 2 Weight of glucose = 180 g


Weight of solvent = 1200-180=1020g
Single Correct Choice Type
1
∴ molality = = 0.98m
Sol 1: (C) We have, 1.02

P = PA + PB = X A PA 0 + XB PB0 ∴ ∆ Tb= Kb m
= 0.98Kb
= 0.4 × 100 + 0.6 × 200
Normal molecular wt
= 40 + 120 = 160 mm Hg Sol 7: (B) i
= >1
Observed molecular wt
PA = 40mmHg
⇒ Observed molecular wt < normal molecular wt.
PA = P.YA
40
YA
= = 0.25 Sol 8: (D) A2 B3 2A + + 3B−
160
α =0.6
P0 − P n
Sol 2: (C) Raoult’s Law = = n=5
P0 n+N
n+N P i = 1 + (n − 1) α = 3.4
= 0
n P0 − P ∆Tb = i Kb m
N P = 3.4 × 0.52 × 1
1+ =0
n P0 − P = 1.768K
N P −P +P P −P n Tb 373 + 1.768 = 374.768 K
=
= 0 0 ⇒ 0 =
n P0 − P P N
Sol 9: (B) Since, glucose does not dissociate,
Sol 3: (A)
= P X A PA + XB PB 0 0 n= 0⇒i= 1

1 3
= .100 + .80 = 25 + 60 = 85 mm Hg Sol 10: (B) ∆ Tb =
iK b m
1+3 1+3
For v equimolar solution are equimolal also very dilute.

Sol 4: (A) ∆ Tb =
ikb m ∴ ∆ Tb ∝ i

Tb will be highest for solute for which ∆ Tb is highest or iAlCl > iCaCl
3 2
i is highest.
i α(as iα
n, nACn,
 n AlCl = 2)
4 ,n CaCl =
3 3 2
For same degree of dissociation, i α n , n is largest for
∆ Tb AlCl3 > ∆ Tb CaCl2
Al2 (SO 4 )3 (n = 5)
T1 > T2
P0 − P
Sol 5: (C) = Xa
P0 Sol 11: (C) ∆ Tf =
iK f m
∆P
= Xa
P0 ∆ Tf 0.0054
i=
= = = 2.903
Xa Kf m 1.86 × 0.001
∆ P1 1
=
∆ P2 Xa α =1
2

10 0.2 ∴ 1 + (n − 1) =
α 2.9 ≈ 3
= ⇒ X a = 0.4
20 X a 2 n–1=2
2
∴ XS =1 − 0.4 =0.6 n=3
∴ Out of the given options, only (C) is possible
Sol 6: (D) For every 1L solution, moles of glucose = 1,
weight of soln = 1.2 × 1000 = 1.2 kg
2 0 . 5 2 | Solutions and Colligative Properties

Sol 12: (B) We have 3 0 2 0


600
= P + P
P = 0.95 P0 5 A 5 B

P0 − P 3PA0 + 2PB0 =
3000
= X=
a 0.05
P0 9PA0 + 6PB0 =
9000  … (i)
wa After adding C,
= 0.05
Ma P PA + PB
=
w a wb
+ 3 + 1.5 2
Ma Mb =630 PA 0 + PB0
3 + 2 + 1.5 + 0.5 3 + 2 + 1.5 + 0.5
w a 1 w a 1 wb 9 0 4 0
= + 630
= P + P
Ma 20 Ma 20 Mb 14 A 14 B
19 w A 1 wb 9PA 0 + 4PB0 =
8820  … (ii)
=
20 Ma 20 Mb
(i)- (ii) → 2PB0 = 180
wb Mb
= 19.
wA Ma PB0 = 90 torr

19 × 0.3 =
5.7
Sol 15: (B) HX H+ + X −
Sol 13: (A) XCl3 → X + 3Cl n=2
n= 4 i =1 + (n − 1) α = 1 + α
α =1 m
= M
= 0.1
i =1+3 =5 ∆ Tf =
i kf m
P0 − P ∆ Tf 0.2
XA = =i = = 1.11
P0 k f m 1.8 × 0.1
17.25 − 17.2
XA = α =0.11
17.25 × 4
n C α2 0.1 × (0.11)2
X A 7.24 × 10−4 =
= ∴ k a= = = 1.35 × 10−3
n+N 1−α (1 − 0.11)

For n + N =, n 7.24 × 10−4


1=
Sol 16: (C) PT vs XB is linear
N = 0.999
PT = PA 0 ⋅ X A + PBo ⋅ XB
N.18
mw = PB = YB PT
1000
N.18 PB0 XB YB (PA 0 X A 0 + PB0 XB0 )
=
Vw = L
1000
1 1 1
n ⇒ = . Y Linear = Y
∴ [A] = PT PB B PA A
Vw
X A .1000
Sol 17: (D)
= P PA0 X A + PB0 XB
(1 − X A ).18
100.2 3
= mole/L
[A] 4.04 × 10−2 Mole /L = + 300 = 40 + 90 = 130 torr
5 5

Sol 14: (C) Initially Sol 18: (B) Ba(NO3 )2 → Ba2+ + 2NO3−
P = PA + PB = PA0 . X A + PB0 XB n=3
i = 1 + (n − 1) α
Chem i str y | 20.53

i = 1 + 2α 0.1 × 2 × 20
∴ V2 = = 40 mL
0.1 × 1
2.74 =1 + 2 α
1.74 5.2
=
α = 0.87
= 87% Sol 10: (C) Xmethyl alcohol = = 0.086
2 1000
5.2 +
18

Previous Years’ Questions moles of sulphur


Sol 11: (D) Molarity =
volume of sol.(l )
Sol 1: (A) PN= KH × XN
2 2 120 × 1.15
1 = × 1000 = 2.05 M
XN = × 0.8 × 5 = 4 × 10 −5 60 × 1120
2
105
in 10 mole, solubility is 4.0 × 10–4 Sol 12: (C) Equimolar solutions in the same solvent
have same boiling and same freezing point.
Sol 2: (D) Basicity of H3PO3 is 2.
Sol 13: (C) As
= PT Px0 . X x + Py0 . X y
Hence 0.3 M H3PO3 = 0.6 N.
1 3
550 Px0 .
= + Py0 .
Sol 3: (C) With increase of temperature, fraction of 4 4
solute present in water changes.
2200 
Px0 + 3Py0 = …. (i)

Sol 4: (A) For HCl M = N = 0.1 On adding 1 mole of Y (P) be comes 560 so
N1V1 = N2V2; 25 × N1 = 0.1 × 35 1 4
560 Px0 .
= + Py0 .
0.1 × 35 5 5
N1 = ;
25 2800 
Px0 + 4Py0 = …. (ii)
0.1 × 35 From eq. (i) and (ii)
∴ M = = 0.07
25 × 2
Px0 = 400 mm Hg
Sol 5: (B) Mobility do not depends on temp. as it do Py0 = 600 mm Hg
not depends on volume.

P0 − Ps n
Sol 6: (C) M1V1 + M2V2 =M3V3; Sol 14: (C) =
P 0 n+N
1.5 × 480 + 1.2 × 520 = M × 1000 18 178.2
=n = 0.1N
= = 9.9 = 0.1 N
720 + 624 180 18
M= = 1.344 M
1000
175 − P0 0.1 175 − P0 1
= = ;
17.5 0.1 + 9.9 17.5 100
Sol 7: (B) An increase in temperature increases the
volume of the solution and thus decreases its molarity. 1750 – 100P = 17.5; 1732.5 = 100p
s s

0.02 × 10 Ps = 17.325 mm Hg
Sol 8: (B) Mole urea = = 10–3 moles
23
6.02 × 10
Sol 15: (A) Let the vapour pressue of pure ethyl alcohol
10−3
Conc. of solution (in molarity) = × 1000 = 0.01 M be P,
100
According to Raoult’s law
Sol 9: (A) H3PO3 is a dibasic acid 290 = 200 × 0.4 + P × 0.6
N1V1(acid) = N2V2(base) 290 − 80
P= = 350 mm Hg
0.1 × 2 × 20 = 0.1 × 1 × V2 0.6
2 0 . 5 4 | Solutions and Colligative Properties

Sol 16: (B) In solution showing positive type of w 2 × 1000


deviation, the partial pressure of each component of Sol 23: (D) ∆Tf = K f × m = K f ×
w1 × m2
solution is greater than the vapour pressure as expected
according to Raoult’s law. w1 and w2 = wt of solvent and solute respecting
In solution of methanol and benzene, methanol m2 = mw of solute
molecules are held together due to hydrogen bonding w 2 × 1000
as shown below, ∆Tf = 0° − ( −6° ) = 6 = 1.86 ×
w1 × m2
On adding benzene, the benzene molecules get in Therefore w 2 = 800 g
between the molecule of methanol thus breaking the
hydrogen bonds. As the resulting solution has weak
Sol 24: (B) ∆Tf =
K f .m
intermolecular attraction, the escaping tendency of
alcohol and benzene molecule from the solution wt 1000
increases. Consequently, the vapour pressure of the 2.8 = 1.86 × ×
62 1000
solution is greater than the vapour pressure as expected
from Raoult’s law. Wt = 93 g

Sol 17: (D) Solution with the same osmotic pressure Sol 25: (A) π =i CRT
are isotonic.
πC =1× 0.500 × R × T = 0.5 RT
2H5OH
Let the molar mass of the substance be M
πMg (PO ) = 5 × 0.100 × R × T = 0.5 RT
p1 = C1RT = C2RT = p2 3 4 2

So, C1 = C2 πKBr = 2 × 0.250 × R × T = 0.5 RT

As density of the solutions are same πNa PO =4 × 0.125 × RT =0.5 RT


3 4

5.25 15 5.25 × 60
So, = ; M = = 210 g mol-1 P0 − PS
M 60 1.5 n
Sol 26: (B) =
PS N
Sol 18: (C) Value of R = 8.314 J mol-1 K-1
185 − 183 1.2 / M
=
183 100 / 58
Sol 19 (A) During depression of freezing point in a
solution liquid solvent, solid solvent are iin equilibrium. M ≈ 64 g / mol

Sol 20: (A) AxBy → xAy+ + yBx– 18


Sol 27: (B) Moles of glucose
= = 0.1
1– α xα yα 180
178.2
i = 1 – α + xα + ya Moles of water
= = 9.9
18
i = 1 + α(x + y – 1) ⇒ nTotal =
10
i −1
α= ∆P 0.1
(x + y − 1) ⇒ =
P° 10

Sol 21: (D)=


PT PAo X A + PBo XB ∆P 0.01 P°
⇒=
= 0.01× 760 = 7.6 torr
760= 520X A + PBo (1 − X A ) ⇒ X A =
0.5
PS 760 − 7.6 = 752.4 torr
=
Thus, mole % of A = 50 %

Sol 22: (B) Vant Hoff’s factor (i) for Na2SO 4 = 3

0.01
∴ ∆ Tf = ( i) k f m 3 × 1.80 ×
= 0.0558 K
=
k 1
Chem i str y | 20.55

JEE Advanced/Boards P0 − 0.75P0


=
w × 18
0.75P0 60 × 100
Exercise 1 60 × 100
w=
3 × 18
Sol 1: PCH = PA 0 ⋅ X A = 23
3OH
w = 111.11 gm
23
X
= A = 0.24 111.11
96 m=
60
P0 − P 100
Sol 2: = XB 1000
P0
0.8 − 0.6 m = 18.52 mol
= XB
0.8
XB = 0.25 Sol 6: ∆ Tb =
K bm

2.11 × 10.6 × 1000


20 0.284 =
Sol 3: X g
= = 0.027 M × 740
180
20 70 2.11 × 10.6 × 1000
+ m=
180 18 740 × 0.284
P0 − P m = 106.42g
= Xg
P0

25.21 − P Sol 7: ∆Tb =


Kb M
= 0.027
P0 0.513 × 3.24 × 1000
0.130 =
P = 24.51 torr M × 200
0.513 × 3.24 × 1000
lc =
P0 − P 0.13 × 200
Sol 4: = Xa
P0 lc = 64 gm

23.76 − 23.32 5.4 Sol 8: ∆ Tb =


Kb M
=
23.76 M
5.4 90 24
+ 0.513 ×
M 18 58
5.4 600 / 1000
⇒ = 0.018
M 0.3530 C
∆ Tb =
5.4
+5 =Tb 99.725 + 0.353
M
5.4 0.0972 100.0780 C
∆ Tb =
⇒ = 0.09 +
M M
Sol 9: ∆ Tb =
Kb M
⇒ M = 57.24 gm
∆ Tf = KfM
Sol 5: P = 0.75P0 Kf 0.86
∆ Tf= . ∆ Tb = × 0.17 = 0.62ºC
Kb 0.513
P0 − P nu
= ∴ Tf = − 0.620 C
P0 nu + nw

P0 − P nu
= Sol 10: π = CR T
P0 nw
30
C=
0.0821 × 298
2 0 . 5 6 | Solutions and Colligative Properties

C = 1.226mole / L Sol 15: For A,


M≈C 1
X=
b X=
a
∆ Tf= K f M= K f C = 1.86 × 1.226 2
160 + 60
∆ Tf =2.280 C PA =Xb Pbo + X a Pbo =
2
∴ Tf = − 2.280 C PA = 110mmHg
For B, Lets mass of each be m.
Sol 11: P0 = 760mm
m m
P0 − P 760 − 750 nb
= = , na
= 78 92
P0 760 m
∴ Xb = 0.541
=
1 78
= Xa
76 m m
+
78 92
X a = 0.013
0 o
P
=B Xb Pb + (1 − Xb )P+
For, nA + nS =1 ⇒ nA =X a =0.013
PB = 114.11atm
nS =1 − nA =0.987
PA 110
MS = 17.766 g =
PB 114.11
na 0.013 × 1000 PA
m
∴= = = 0.964
Ms 17.766 PB

m = 0.741 Molal
Sol 16: =
∆ Tb Kb (i1m1 + i2m2 )

Sol 12: Refer to solution 9, Exercise I For NaCl



NaCl ←→ Na+ + Cl−

P0 − P P0 − P nA n= 2 , α
= 0.8
Sol 13: We have, = Xa = =
P0 P0 nS
i = 1 + (n − 1)α
640 − 600 2.175 × 78 i = 1 + 0.8
=
600 M.39
i = 1.8
M = 65.25gm 5.84
=m = 1
na 58.5
Sol 14: =
m 0.5
= 100
ws
1000
For 1 kg water, na = 0.5
For MgCl2
1000
nw = → Mg+2 + 2Cl−
18 MgCl2 ←
na 0.5
∴ Xa =
= n= 3, α= 0.5
na + nw 1000
0.5 + i = 1 + 2 × 0.5 = 2
18
−3
X a 8.92 × 10
= 9.5
=m = 1
95
P0 − P
= Xa 100
P0
1000
P = P0 (1.X a ) = 17.54 × 0.99 ∆ Tb 0.51(1.8 × 1 + 2 × 1)
=
P = 17.38mmHg 1.9380 C
∆ Tb =
Tb = 101.9380 C
Chem i str y | 20.57

Sol 17: K Cl K + + Cl− Sol 21: Pressure must be applied on 0.01 M glucose
solution to prevent osmosis.
n= 2, α= 0.95
Excess pressure = π1 − π2
i = 1 + (n − 1) α
(C1 − C2 )RT
i= 1 + 0.95
(0.01 − 0.001) × 0.0821 × 300 = 0.2217 atm
=i 1.95 α
RT f 2
Kf = Sol 22:
1000 × ∆Hf
πb =CR T
∆ Hf 80 cal
= = / gm 334.72 J / gm
π1 C1 T1
=
8.314 × (273) 2 π2 C2 T2
=Kf = 1.851
1000 × 334.72 π2C1 T1
C2 = (T1 = 10ºC = 283k)
7.45 π1 t2
=M = 0.2
74.5
C2 105.3 × 283
500 = (T2 = 25ºC = 298k)
C1 500 × 298
1000
C2
iK f m = 1.95 × 1.851 × 0.2 = 0.73ºC
∆ Tf = = 0.2
C1
Tf = ‒ 0.73ºC
C2 = 0.2C1
Sol 18: Let n particles be formed after dissociation V2 = 5 V1
Taking α =1
Sol 23: ∆ Tf =
i Kf m
i =1 + (n − 1) =n
0.2
∆ Tf =
i K f .m 0.45 =
i × 5.12 ×
60
0.558 20
n= i= = 3
1.86 × 0.1 1000
Formula [Co(NH3 )5 Cl] − Cl2  1
i =0.527 =1 −  1 −  α
 n
Sol 19: Refer Exercise II, Comprehension I
n=2
1
Sol 20: At Pt = 1 i =1− α
2
=Pt PB0 XB + PT 0 X T
= 2(1 − i)
α
760= 1375 XB + 558(1 − XB )
α =0.945
760
= 1375 XB + 558 − 558 XB ∴ 94.51 association
202 = 817XB
XB = 0.247 Sol 24: NaNO3 Na+ + NO3−

X T = 0.753 n= 2, α= 0.9

For Vapour i = 1 + (n − 1)α = 1.9


0.85
PB = YB .Pt π =i CR T = 1.9 × × 10 × 0.082 × 300
85
PB × B 1375 (T = 27ºC = 300k)
YB
= = = 0.447
Pt 760 π = 4.64 atm
2 0 . 5 8 | Solutions and Colligative Properties

Sol 25: π1 =π2 PA 100 × 0.25


Y=
A =
iC1 R T = C2 R T (for glucose PT 250

α = 0 ⇒ i= 1 ) YA = 0.1

For NaCl, i = 1 + (n − 1)α = 1 + α YB = 0.9


1.2 × 10 7.2 Mole fraction of A in final condensate = 0.1
(1 + α ) = × 10
58.5 180
1 + α =1.95 Sol 28: For phenol, n = 2

α =0.95  1 α
i = 1 1 −  α = 1 −
 n 2
i = 1.95
Now,

Sol 26: 2C6H5 (C6H5 COOH)2 ∆ Tf =


i Rf m

n= 2, α = 0.84  α 20 α
0.69 = 1 −  5.12 × ⇒ 1− =0.63
 2 94 2
 1 α
i =1 −  1 −  α = i = = 0.58 α
 n 2 = 0.366
2
∆ Tb =
i Kb M α =0.7333
0.61
0.58 × 2.3 × Sol 29:
122
50 Mass of CH3 OH= V × ρ = 30 × 0.7980 gm = 23.94 gm
1000 Moles of CH3 OH,n = 0.748
0.13340 C
∆ Tb =
mass of water = 70 × 0.9984
0 0
Tb 46.2 + 0.1334 C
= n 0.748
m
= = × 1000
Tb = 46.3334 C 0 0
mS 69.888

m = 10.70
Sol 27: For first distillate
k f m = 10.7 × 1.86
∆ Tf =
X A= XB= 0.5
19.910 C
∆ Tf =
PA 0 + PB0 100 + 300 Tf = −19.910 C
=PT =
2 2
Mass of final solution = 23.94 × 69.888 = 93.828 gm
PT = 200 Torr Mass 93.828
Volume = = = 98 ml
PA = X A PA 0 = 50 torr Density 0.9575
n 0.748
PA = YA PT There for Molarity = = ×1000 = 7.63 M
V 98
PA 50 Molarity = 7.63 M
Y=
A = = 0.25
PT 200
YB = 0.75
Exercise 2
For second distillate
Single Correct Choice Type
X A= YA= 0.25, XB= YB= 0.75
Sol 1: (C)  PA 0 > PB0
PT0 =100 × 0.25 + 300 × 0.75
YA will be more than X A
PT = 250 Torr
(nA )L (nA )V
Now, PA =YA PT i.e <
(nB )L (nB )V
Chem i str y | 20.59

XA YA YA Multiple Correct Choice Type


< ⇒
XB YB XA
Sol 8: (A, C, D) Only (B) is wrong.
YA XA
>
YB XB Sol 9: (A, D) For ideal solution.
∆H = 0 , ∆ V = 0
Sol 2: (D) Relative lowering will be least in C12H22O11
which will not dissociate; i = 1
Sol 10: (A, C) Both are correct facts.
For rest, i > 1 and show greater decrease.

Sol 11: (A, B, C) All are correct.


Sol 3: (C) =
P PA + PB

X A PA + XB PB0 Sol 12: (A, C, D) Degree of dissociation increases as c


decrease
PA0 (1 − XB ) + XB PB0
 R 
PA0 XP (P0 − P0 )
P =− α= a 
A B  c 
 
PA0 = 120
For ideal soln., ∆ G =∆ H − T∆ S
PA0 − PB0 =
75
∆ G =− T∆ S ( H = 0)
PB0 = 45
−∆ G
S
∆= > 0 ( ∆ G < 0 )
T
Sol 4: (D) A liquid mixture having composition
corresponding to point z in the figure shown in subjected
Sol 13: (A, B, C) Due to polarity and H-bonding.
to distillation at constant pressure ,composition of an
azeotrope alters on changing the external pressure
Sol 14: (A, D) For ideal solution.

Sol 5: (B) From the graph ∆H = 0 , ∆ V = 0


When XBenzene = 0.5 YToluene = 0.8
Sol 15: (A, B)
= PM X A PA 0 + XB PB0
When YToluene = 0.3 YBenzene = 0.6
XBenzene = 0.3, YToluene = 0.9 PM= X A PA 0 + PB0 − X A PB0

XBenzene = 0.7, YToluene >0.3 PM − PB0


= XA
PA 0 − PB0
Sol 6: (C) ∆ Hmix = 0
PA = PT X A(V )
∆G =∆H − T ∆ S
PA X A(  ) .PA
0
∆G=−T ∆S
X'A(V= =
G=-TS )
PT PT

TS X A(  ) PT
=
O X 1
A(V )
PA 0
G

Sol 16: (A, C, D) K 3 PO 4 → 3K + PO 4


X + 3−

Sol 7: (D) Through a semipermeable membrane, only n=4


solvent can flow and not the solute. i =1 + (n − 1) α = 1 + 3 α
∆P
= iX A
P0
2 0 . 6 0 | Solutions and Colligative Properties

n And i α n
m=
ws
n is maximum for Al2 (SO 4 )3
1000
For 1 kg solved, N =
18
Sol 22: (B) Correction factor is 1
18n
m=
1000 Sol 23: (B) Ba(NO3 )2 Ba2+ + 2NO3−
1000m n=3
n=
18
i = 1 + (n − 1) α
n 1000m
=X =
n+N  1000m 1000  i − 1 2.74 − 1
18  +  =α =
 18 18  n−1 3 −1
m
X= α =0.87
m+1
8.7% dissociation
∆P i n i n
= ≈
P0 N + n N Sol 24: (C) Colligative effects involve properties whose
magnitude depends upon number of particles.
∆ P (1 + 3α )m.18
=
P0 1000 ∆ Tb kb
Sol 25: (B) =
∆ Tf kf
∆ P ∆ Tf 18
= . 0.186 × 0.521
P0 K f 1000 =∆ Tf = 0.0521
1.86
M
1+ 3 α = w
Mw obs Sol 26: A → p, r
This mixture will be show positive deviation due to
Sol 17: (B, D) Vant-hoff factor for a dil. sol. of BaCl2 is H-bonding between C2H5OH and H2O
1 + 3 −1 = 3 (D) is correct
B→ q, t
∆ Tb is also due to lowering in vapour pressure.
They will form ideal solution as
FA= F= FB −B
Sol 18: (B, D) Acetone and ethanol will show negative −A A −B

deviation due to H-bonding between them. No strong interactions


Chloroform and benzene will show positive deviation C → q, t
because of intramolecular H-bonding in chloroform.
This is Raoult’s law for ideal solution
D → p, s
Assertion Reasoning Type
Very strong intramolecular H-bonding in H2SO 4 leads
Sol 19: (C) The added substance once must be non - to negative deviation.
volatile.

Sol 27: A → r
Sol 20: (D) As heat must be removed from the water to
make it freeze, it is exothermic process. No dissociation in any of these
∴ i = 1 for all.
Comprehension Type B→s
Sol 21: (A) ∆ Tf =
i kf m Strong electrolytes, α = 1

∆ Tf α i 1 + (2 − 1) : 1 + (3 − 1) : 1 + (3 − 1) = 2 : 3 : 3
Chem i str y | 20.61

C→p Sol 6: (A) Vapour pressure = p(H2O) + p(ethanol)


1 + (5 − 1) : 1 + (4 − 1) : 1 + (5 − 1) = 5 : 4 : 5 = 1 : 0.8 : 1 = 32.8 × 0.1 + 40 × 0.9
D→q = 3.28 + 36 = 39.28 mm
1 : 1 + (2 − 1) : 1 + (3 − 1)
Sol 7: (B) New ethanol is solute.
1:2:3
0.1
Molality of solute = × 1000 = 6.17
Sol 28: A → s 0.9 × 18
⇒ DTb = 6.17 × 0.52 = 3.20
Definition of ppm
⇒ Tb = 373 +3.2 = 376.2 K
B→r
C→ q  ∆T 
Sol 8: Lim  f  = k f (Cryoscopic constant)
D→ p m→0 m
 

Sol 9: From lowering of vapour pressure information:


Previous Years’ Questions 0.104 n2
= x= 2
17.5 n1 + n2
Sol 1: (C) In case of positive deviation from Raoult’s law, n1
the observed vapour pressure is greater than the ideal ⇒ + 1 = 168.27
n2
vapour pressure and boiling of azeotrope becomes
lower than either of pure liquid. n1
⇒ = 167.27
n2
Sol 2: (A) K2SO4: i = 3;NaCl: i = 2 1000 M
⇒ × = 167.27
Urea i = 1; Glucose: i = 1 18 50

Greater the value of ‘i’, greater the lowering in freezing ⇒ M = 150g/mol


point, lower will be the freezing temperature, if molarity, Also, the combustion reaction is:
in all cases are same. Therefore, K2SO4 solution has the
lowest freezing point. CxH2yOy + xO2 → xCO2 + yH2O

∴ 18y g of H2O is produced from 1.0 mole of compound.


Sol 3: (A) During freezing, liquid solvent solidify and
0.9 1
solid solvent remains in equilibrium with liquid solvent. \ 0.9 g of H2O will be produced from = mol
18y 20y
x
Sol 4: (A) Van’t Hoff factor (i) = 4 {3K+ + [Fe(CN)6]}3– ⇒ At the end, moles of O2 left =
20y
0.1 1000 1 x
Molality = × =⇒ –DTf = iKf.m
329 100 329 Moles of CO2 formed =
20y
1 2x 2.24
= 4 × 1.86 × = 2.3 × 10–2
329 ⇒ Total moles of gases at STP = =
20y 22.4
⇒ Tf = –2.3 × 10–2 ºC ⇒x=y
(As % freezing point of water is 0ºC)
⇒ Molar mass; 150 = 12x + 2x + 16x = 30x
150
Sol 5: (D) In the given solution ‘M’, H2O is solute ⇒x= =5
30
Therefore, molality of H2O
⇒ Formula = C5H10O5
0.1
= × 1000 = 2.4 60
0.9 × 46 Sol 10: Moles of ethanol = = 1.3
46
⇒ –DTf = kfethanol × 2.4 = 2 × 2.4 = 4.8 40
Moles of methanol = = 1.25
⇒ Tf = 155.7 –4.8= 150.9 K 32
2 0 . 6 2 | Solutions and Colligative Properties

1.3 Empirical formula weight (84) is half of molar mass,


Mole fraction of ethanol = = 0.51
1.3 + 1.25 molecular formula is C6H4N2O4 – a dinitro benzene.

Vapour pressure of solution = pethanol + pmethanol NO2

= 0.51 × 44.5 + 0.49 × 88.7= 66.16 mm


Mole fraction of methanol in vapour phase
pmethanol 43.463 NO2
= = = 0.657 (C6H4N2O4)
Total vapour pressure 66.16

Sol 11: According to Raoult’s law: Sol 14: Let after 100 min, x moles of A are remaining
unpolymerized moles of B = 12.
p = p0x1
Mole of non–volatile solute = 0.525
 n1 
⇒ 600 = 640   x
 n1 + n2  Mole fraction of A =
n x + 12 + 0.525
64 1
⇒ 2 = –1= 12
n1 60 15 Mole fraction of B =
x + 12 + 0.525
39 1
⇒ n2 = × = 0.033
78 15  x   12 
400 =   × 300 +   × 500
2.175  x + 12 + 0.525   x + 12 + 0.525 
⇒ = 0.033 ⇒ M = 65.25
M x = 9.9
Moles of A polymerized in 100 min = 10 – 9.9 = 0.10
Sol 12: Ca(NO3)2 → Ca2+ + 2NO3–
1 10 1 10
1 – α α 2a k= ln = ln min–1 = 1.005 × 10–4 min–1.
t 9.9 100 9.9
i = 1 + 2α, where, α = 0. 7
Sol 15: Higher the value of kb of a solvent suggest that
⇒ i = 1 + 2 × 0.7 = 2.4
there is larger polarity of solvent molecules, which in
n1 turn implies higher boiling point due to dipole–dipole
⇒ Mole fraction of solvent =
n1 + in2 interaction.
 100  Therefore, the correct order of kb values of the three
 
=  18  = 0.982
given solvents is:
 100  7
  + 2.4 × Solvents Boiling point kb
 18  164
X 100ºC 0.63
⇒ p = p0x1
Y 27ºC 0.53
= 760 × 0.982 (VP of H2O at 100ºC = 760 mm of Hg)
Z 283ºC 0.92
= 746.32 mm
Sol 16: (A)
Sol 13: (i) Empirical formula determination
∆Tf = K f × molality × i
Element C H N O 20 1000
2 = 1.72 × × ×i
Weight % 42.86 2.40 16.67 38.07 172 50
Moles 3.57 2.40 1.19 2.38 i = 0.5
Simplest ratio 3 2 1 2 Hence (A) is correct.

⇒ Empirical formula = C3H2NO2


Sol 17: (D) ∆Tf = Kf × m
5.5 1000
(ii) DTb = 1.84 = 2.53 × × 0.1
M 45 2× × 1000 =
4.83K
0.9 × 46
⇒ M = 168
Chem i str y | 20.63

Freezing point of solution M = 155.7 – 4.83 = 150.87 K Sol 23:


≈ 150.9 K
∆Tf =i × K f × m
0.0558 =
i × 1.86 × 0.01
Sol 18: (B) P = 0.9 × 40 = 36 mm Hg
i=3

0.1 ∴ Complex is [Co(NH3)5Cl]Cl2


Sol 19: (B) ∆Tb = Kb × m =0.52 × × 1000 =3.2 K
0.9 × 18
Tb = 373 + 3.2 = 376.2 K X solute 0.1 1
Tb = 373 + 3.2 = 376.2 K Sol 24: = =
X solvent 0.9 9

Sol 20: (A) P = KHXN Wsolute Msolvent 1


2 = =  … (i)
Wsolvent Msolute 9
0.8 × 5 =1× 105 × XN
2
Wsolute + Wsolvent = Wsolution = density × volume
XN = 4 × 10 −5
(in 10 moles of water)
2

nN Wsolute + Wsolvent = 2 × V  … (ii)


⇒ 4 × 10−5 = 2
nN + 10
2
Molarity = molality
−5
nN × 5 × 10 + 4 × 10 −4 =nN nsolute nsolute
2 2
=
4 10 −4
⇒ nN =× Vsolvent Wsolvent
2
Wsolute + Wsolvent
Wsolvent = Vsolution =
Sol 21: (A) 2
⇒ 2Wsolvent =Wsolute + Wsolvent
+ 3−
K 3 [Fe(CN)6 ] → 3K + [Fe(CN)6 ]
i=4 ⇒ Wsolute = Wsolvent ... (iii)
m 1000 0.1 1000
∆Tf =×
Kf i × × 1.86 × 4 ×
= × 2.3 × 10−2
= Using eq. (i) and (iii), we get
M W 329 100
−2.3 × 10−2
Tf = Msolute
=9
Msolvent
Sol 22: (A)
Sol 25: (A, B) Conceptual fact
=∆Tb 2o C;=ma 2.5g
msolvent 100
= = g Kb 0.76K kgmol−1
Psolution = ?

∆Tb = Kb × m

2
2= 0.76 × m ∴ m=
0.76

P0 − P 760 − P 2
= m × MM × 10−3 ∴ = × 18 × 10 −3
P P 0.76
36 796
760 − P
= P ∴ 760
= P
760 760

 796 
∴ P 760  =
=  torr 725.6 torr ≈ 724 torr
 760 
2017-18 100 &
op kers
Class 12 T
By E ran culty
-JE Fa r
IIT enior emie .
S fP r es
o titut
Ins

CHEMISTRY
FOR JEE MAIN & ADVANCED
SECOND
EDITION

Exhaustive Theory
(Now Revised)

Formula Sheet
9000+ Problems
based on latest JEE pattern

2500 + 1000 (New) Problems


of previous 35 years of
AIEEE (JEE Main) and IIT-JEE (JEE Adv)

5000+Illustrations and Solved Examples


Detailed Solutions
of all problems available

Plancess Concepts
Topic Covered Tips & Tricks, Facts, Notes, Misconceptions,
Key Take Aways, Problem Solving Tactics
Surface Chemistry
PlancEssential
Questions recommended for revision
21. S U R FA C E C H E M I S T R Y

THE COLLOIDAL STATE

1. INTRODUCTION
The foundation of Colloidal Chemistry was laid down by an English Scientist, Thomas Graham, in 1861. He classified
the soluble substances into categories depending on their rates of diffusion through animal and vegetable
membranes or parchment paper. He observed that certain substances diffuse freely through the membrane
whereas others do not diffuse. The former types of substances are crystalline in nature like sugar, common salt etc
while the latter were termed as colloids. Thus,
Crystalloids: The ones having higher rates of diffusion.
Colloids: The one having slower rate of diffusion.
However, this classification has now been rejected because some substances, like NaCl, diffuse fast in water but slow
in alcohol whereas, some other substances like RCOONa diffuse slowly in water and rapidly in alcohol. Therefore, a
new term colloidal state was preferred.

Characteristics of Colloidal State


It is a particular state and not a class of compounds.
It is heterogeneous state, i.e., number of phases >= 2
It must have 2 phases: DISPERSED PHASE & DISPERSION MEDIUM
The one phase dispersed in other is known as dispersed phase (or D.P.) or internal phase or discontinuous phase
whereas, the other in which dispersions are made is known as dispersion medium (or D.M.) or external phase or
continuous phase.

Sol Particles
It is an intermediate state between true solution state and suspension state, i.e., particle size of dispersed phase
usually called sol particles lies between true solution state and suspension state.
True solution Colloidal state Suspension
Solute + Solvent D.P. + D.M. Precipitate + Medium
Size Solute 1m µ or 10 Å D.P. – 1m µ to 200 m µ Precipitate – 200 m µ onwards
2 1 . 2 | Surface Chemistr y

Characteristics of Sol particles:


(a) They have enormous surface area/gram as a result of their small size which gives rise to certain special
properties of colloidal state.
(b) They are not detectable even under microscope.
(c) They do not settle down even under the influence of gravity.
(d) They usually carry charge: positive/negative

Illustration 1: What are the physical states of dispersed phase and dispersion medium of froth? (JEE MAIN)

Sol: Froth is a Mass of small bubbles in liquid that is caused by agitation.


Dispersed phase : Gas
Dispersion medium : Liquid

Illustration 2: Why medicines are more effective in colloidal state? (JEE ADVANCED)

Sol: Sol particles of a colloidal state has larger surface area and thus, shows more effective adsorption. So if the
medicine is in colloidal state it will get adsorbed more effectively and will give better results.

2. CLASSIFICATION OF COLLOIDAL DISPERSIONS


(a) Classification based on the nature of D.M.

(i) D.M. is water : Hydrosols


(ii) D.M. is alcohol : Alcohols
(iii) D.M. is benzene : Benzosols
(iv) D.M. is air : Aerosols

(b) Classification based on the nature of charge


If dispersed phase particles carry +ve charge, it is +ve charge; it is +ve Sol If dispersed phase particles carry
–ve charge; it is –ve Sol
(c) Classification based on the physical state

Table 21.1: Classification based on the physical state of D.P. and D.M.

Internal phase or External phase or Colloidal Name Examples


Dispersed phase dispersion medium

S S Solid sols Alloys, ruby glass, gems or precious stones, marbles,


optical and vision glasses

L Sols Muddy water, gold sol, sols of protein, starch, agar


and gelatin in water

G Aerosols of solids Smoke, storm

L S Gels Cheese, jams, jellies, curd, plants, fruits, vegetables,


cementation
Chem i str y | 21.3

Internal phase or External phase or Colloidal Name Examples


Dispersed phase dispersion medium

L Emulsions Butter, milk, blood, cosmetic products, e.g., shampoo,


creams, emulsified oils, polish and medicines.

G Aerosols of liquids Fog, clouds, mist

G S Solid foams Pumice stone, styrene foam, rubber foam, porous


pot, rubber pillows and mattresses

L Foam or froths Froths, soap suds, air bubble

G Homogeneous Thus, not colloids


system

(d) Classification based upon interaction forces between the dispersed phase and dispersion medium:
(i) If strong interactive force exist between the dispersed phase and dispersion medium then such colloids
are called Lyophilic colloids.
(ii) If no interactive forces exist between the dispersed phase and dispersion medium. Such colloids are
called lyophobic colloids.

Table 21.2: Essential Differences between Lyophobic and Lyophilic Sols

Property Lyophobic Sols Lyophilic Sols

1. Surface Tension Surface tension is usually the same as that Surface tension is generally lower than that
of the medium (i.e. the liquid in which the of the medium (i.e. the liquid in which the
particles are dispersed) particles are dispersed)

2. Viscosity Viscosity is about the same as that of the Viscosity is much higher than that of the
medium medium

3. Visibility The particles, though invisible, can be readily The particles cannot be readily detected even
detected under an ultra-microscope. under an ultra-microscope.

4. Migration in an The particles migrate either towards anode The particles migrate in either direction or not
electric field or towards cathode in an electric field. at all in an electric field.

5. Action of electrolytes The addition of small quantities of The addition of small quantities of electrolytes
electrolytes can cause precipitation has little effect. Much larger quantities are
(coagulation). needed to cause precipitation.

6. Reversibility These are irreversible These are reversible.

7. Hydration The particles are not hydrated to a large The particles are extensively hydrated. This
extent is due to the presence of a number of polar
groups in the molecules of lyophilic colloids as,
for example, in polysaccharides, proteins, etc.
2 1 . 4 | Surface Chemistr y

PLANCESS CONCEPTS

•• Stability of lyophilic sols (reversible) is due to a thin layer of dispersion medium round dispersed phase
particles whereas, the stability of lyophobic sols (irreversible) is due to the presence of charge on dispersed
phase particles.
•• If water is used as dispersion medium, lyophilic sols are called hydrophilic and lyophobic sols as
hydrophobic sols.
•• Inorganic sols, e.g. AgI, SnO2, Fe(OH)3, As2S3, CdS, S, in water etc are usually hydrophobic whereas organic
sols, e.g. protein, gelatin, agar-agar, polymers, etc. in water are hydrophilic.

Vaibhav Krishnan (JEE 2009, AIR 22)

Illustration 3: Lyophilic sols are more stable than lyophobic colloids. Why? (JEE MAIN)

Sol: The lyophilic system contains highly solvated colloidal entities.

3. PREPARATION OF COLLODIAL SOLUTIONS


The primary consideration in the preparation of colloidal solution is that the dispersed particles should be
within the size range of 1nm-100 nm. The lyophilic sols can be readily prepared since colloidal materials such
as starch, gelatin, acacia, etc, when added to water swell up and spontaneously break into particles of matter
of colloidal range. The lyophobic sols, however, require special techniques for their preparation. The methods
consist either in
 1. Breaking down the coarser aggregates into particles of colloidal size
 2. Grouping molecules into larger aggregates of colloidal size. The methods belonging to these two categories
are known as dispersion and condensation methods, respectively.

(a) Mechanical Dispersion: By breaking up suspension particles into colloidal size. The goal is achieved by:

(i) Grinding: Freshly precipitated mass + medium  Grind → sols


in grinder
(ii) Peptization: Certain freshly precipitates, such as silver chloride, ferric hydroxide, aluminum hydroxide,
can be converted into colloidal solutions by the addition of a small amount of a suitable electrolyte. An
electrolyte having an ion in common with the material to be dispersed is required for sol formation.
The peptization action is due to the preferential adsorption of one of the ions of the electrolyte by the
particles of the material. As would be illustrated a little later in this chapter, as a result of the preferential
adsorption of the ion which is more closely related chemically to the precipitate, the particles acquire a
positive or a negative charge depending upon the charge on the ion adsorbed. Because of the presence
of the same type of charge, the particles of the precipitate are pushed apart. The precipitate thus gets
dispersed resulting in the formation of a stable sol.
(iii) Bredig arc method: Two metal electrodes are immersed in a liquid medium. Both of them are connected
to a high tension battery by which an electric arc is produced between the electrodes. High temperature
of the arc vaporizes some of the metal, which condenses on cooling to form sols. The heat generated
during process is removed by putting system under ice cold water. The method is generally used for
preparation of metal sols in water. A little amount of KOH in water prevents growth of sol particles and
stabilizes the dispersion. Au sol in water, i.e., Purple of Cassius is obtained by this method.
(iv) Condensation Methods: Colloidal stems can be obtained by various chemical reactions such as double
decomposition, oxidation, reduction, hydrolysis etc.
Chem i str y | 21.5

(b) Double Decomposition: A sol of arsenious sulphide is prepared by passing H2S gas through a dilute solution
of arsenious oxide and removing the excess H2S by boiling
(i) NaCl + AgNO3 → AgCl + NaNO3
(ii) As2O3 + 3H2S → As2S3 + As2S3 + 3H2O

(c) Oxidation: A colloidal sulphur sol is obtained by the oxidation of an aqueous solution of hydrogen sulphide
with air or sulphur dioxide.
(i) H2S + 2HNO3 → 2NO2 + S + 2H2O
(ii) 2H2S + O2 → 2S + 2H2O
(iii) 2H2S + SO2 → 3S + 2H2O

(d) Reduction: Sols of metals such as silver, copper, gold and platinum are obtained by reducing the aqueous
solutions of their salts by non-electrolytes such as formaldehyde, tannin, phenyl hydrazine, carbon monoxide
and phosphorus. Zsigmondy prepared the gold hydrosol by reducing potassium aurate with formaldehyde.
In the reaction, chlorauric acid, H[AuCl4 ].4H2O first formed, is made to react with potassium carbonate in an
aqueous solution to yield potassium aurate:
(i) 2H[AuCl4] + 5K2CO3 → 2KAuO2 + 5CO2 + 8KCl + H2O
The resulting solution is heated and a dilute solution of formaldehyde is added drop-wise when reduction
occurs according to the reaction:

(ii) 2KAuO2 + 2CH2O + K2CO3 → 2Au (sol) + 3HCO2K + KHCO3 + H2O

(e) Hydrolysis: Colloidal sols of heavy metals are obtained by the hydrolysis of the solutions of their salts. Thus,
when a small amount of ferric chloride is added to boiling water, a red-brown sol of ferric hydroxide is
obtained:
(i) FeCl3 + 3H2O → Fe(OH)3 + 3HCl
(ii) This is because FeCl3 → Fe3+ + 3Cl−
+
(iii) And Fe3+ + H2O¯ → Fe(OH)3 + 3H
Sol

Boiling promotes the reactions because HCl formed is removed with water vapors from the system. In this reaction,
iron oxychloride, FeOCl, formed as a result of incomplete hydrolysis of FeCl3, is believed to act as the stabilizer:
(i) FeCl3 + H2O → FeOCl + 2HCl
Thus, the structure of the Fe(OH)3 sol can be expressed by any of the following formulas depending upon
which substance acts as a stabilizer:
1. (m [Fe(OH)3]nFeO+.(n – x)¯Cl¯)*Cl¯

2. (m [Fe(OH)3]nFe+.3(n – x)Cl¯)3*Cl¯

3. (m [Fe(OH)3]nH+.(n – x)Cl¯)*Cl¯

(f) Exchange of Solvents: Sols can also be obtained by exchange of solvents. For instance, when a concentrated
solution of sulphur in alcohol is poured in a large amount of boiling water, the alcohol evaporates leaving
behind sulphur particles which form nuclei that rapidly grow into a colloidal solution.

Illustration 4: On passing H2S in cold, dilute solution during group II qualitative analysis of basic radicals sometimes
yellow turbid solution is obtained instead of precipitate. Explain. (JEE ADVANCED)

Sol: It is because of the formation of sol of As2S3 or CdS.


As2O3 + 3H2S → As2S3 + 3H2O
sol
2 1 . 6 | Surface Chemistr y

Illustration 5: Describe a chemical method each for the preparation of sols of sulphur and platinum in water.
 (JEE ADVANCED)

Sol: By bubbling H2S gas in cold solution of an oxidizing agent like HNO3 SO2 or Br2

(a) H2S + 2HNO3 → 2H2O + 2NO2 + S


sol

(b) H2S + Br2 → 2HBr + S


sol

(c) 2H2S + SO2 → 2H2O + 3S


sol

Platinum sol can be prepared by reducing its salt solution by using suitable reducing agent like formaldehyde,
SnCl2, hydrazine, carbon monoxide etc.

PtCl2 + 2SnCl2 → Pt + 2SnCl4


sol

4. PURIFICATION OF SOLS
Sols so obtained are contaminated with two types of impurities and need purification:

(a) Insoluble impurities: These are removed by simple filtration of impure sols. Impurities are retained on filter
paper and sols are filtered.

(b) Soluble impurities: Special methods are required to separate these impurities.
(i) Dialysis: The separation of soluble impurities from sols on the basis of their different rates of diffusion
through parchment membrane (known as dialyser) is known as dialysis. If impurities are of electrolytic
nature electro dialysis is preferred. The ions of electrolyte move faster than sol particles towards opposite
electrodes and thus removed from sol.
(ii) Ultra filtration: Ultra filters used for separation of soluble impurities from sols are specially prepared
filters of uniform pore sizes obtained by dipping filters in colloid ion solution (4% nitrocellulose in
alcohol + ether)
(iii) Ultra centrifuge: Based on sedimentation technique.

PLANCESS CONCEPTS

The process of dialysis finds applications in the purification of blood by artificial machine when kidney
fails.
Nikhil Khandelwal (JEE 2009, AIR 94)

5. PROPERTIES OF COLLOIDAL SOLUTIONS


(a) Physical properties
(i) Heterogeneity: Sols are heterogeneous in nature.
(ii) Diffusibility: Sol particles have slow rate of diffusion through parchment membrane.
(iii) Filterability: Sol particles are passed through ordinary filter papers.
(iv) Non-Settling nature: Sol particles do not settle down due to gravity.
Chem i str y | 21.7

(v) Viscosity and surface tension: Viscosity and surface tensions of sols are almost similar to those of pure
solvent in case of lyophobic colloidal solutions, viscosity is higher than the solvent and surface tension
being lesser than the solvent used as dispersed medium for sols.
(vi) Visibility and size of the particles: It is not possible even by the help of a most powerful microscope to
see colloidal particles because clear image formation of a particle smaller in size than the wavelength of
light used is not possible. However, zones of scattered light can be seen by a microscope and sometimes
with naked eyes also.
(vii) Surface area and adsorption: The surface area of the particles in colloidal state is appreciably larger
than compared with an equal mass of the matter in coarse grained size. It is due to the larger surface
area, sol particles show high tendency for adsorption. The property has got application in medicinal
chemistry, where drugs are sold in form of sols.
(viii) Shape of sol particles and color: The sol particles possess different shapes and size. Due to their different
size and shape, they acquire different colors, because the nature of light absorbed and transmitted out
depends upon size and shape of particle. A larger particle will absorb larger wavelength and will transmit
shorter ones. Therefore, as the size of particles increases, the color approaches to violet one. That is
why red gold sol on slow coagulation shows a color change from red to blue. Also, red sol particles are
spherical and blue sol particles are disc like in gold Sol

(b) Colligative Properties: Like true solutions, colloidal solutions also exhibit colligative properties such as
osmotic pressure, elevation in b. pt., depression in freezing point, lowering in vapor pressure.

(c) Optical Properties vis-a-vis Tyndall effect


(i) All colloidal solutions are capable of scattering of light or opalescence.
(ii) A beam of converging rays on passing through a colloidal solution, scattering of light by sol particles in
all the directions, gives rise to a bright glowing cone (fig.1) when looking at it sideways. This is known
as Tyndall effect.
Dark

Water Illumination

Dark

Illumination

Sol Illumination
(Scattered light)
Illumination
(Scattered light)

Figure 21.1: Scattering of light-Tyndall Effect

Some examples of Tyndall effects are:


•• Blue color of sky and sea water.
•• Visibility of tails of comets.
•• Twinkling of stars.
•• Visibility of sharp ray of sunlight entering through a slit in dark room.
•• Visibility of projector path and circus light.
2 1 . 8 | Surface Chemistr y

PLANCESS CONCEPTS

The scattering of light depends upon wavelength of light used: Scattering α(1 / λ 4 ) : that is why scattering
of blue-violet light is maximum.
Saurabh Gupta (JEE 2010, AIR 443)

(d) The Brownian movement: Robert Brown noticed the irregular or chaotic motion of (pollen grains) particles
suspended in water. This was later on named as Brownian motion. Later on Zsigmondy after the discovery of
ultramicroscope (based on Tyndall effect) noticed the similar irregular motion of the images of sol particles
(figure) and concluded that colloidal particles move much more vigorously than small particles.

Figure 21.2: Brownian movement - zig-zag motion of a colloidal particle.

Applications of Brownian motion are


(i) For an explanation for the stability of colloidal solutions.
(ii) For an experimental set up to determine Avogadro’s number
(iii) For an explanation of kinetic theory of matter.

PLANCESS CONCEPTS

Brownian motion is not observed in suspension, however spontaneous movement of solute particles in
solvent (i.e., true solution) also show Brownian motion.
Neeraj Toshniwal (JEE 2009, AIR 21)

6. ELECTRICAL PROPERTIES
The charge:
(a) Charge on sol particles is a fundamental factor for their existence, i.e., no charge on dispersed phase particles,
no stability to colloidal solutions.
(b) It is the charge on sol particles, which keeps them apart and prevents them from coming closer to each other
to coagulate.
(c) Sol particles either carry +ve or –ve charge.
(d) Metal sols are –ve; blood is –ve, smoke is –vely charged.
Chem i str y | 21.9

6.1 Origin of Charge


(a) Frictional electricity; Due to friction between D.P. and D.M.
(b) Due to electron capture by sol particles.
(c) Nature of dispersed phase: The nature of dispersed phase also contributes for the charge on sol particles.

(a) RCOONa → RCOO¯ + Na+ RCOO¯ ions undergo


R COO¯ aggregation to from
sol particles and thus,
Hydrophobic Hydrophilic they carry -ve charge.

(b) RNH3 Cl → RNH3+ + Cl¯ RNH3+ ions undergo


R aggregation to form
NH3+
sol particles and thus,
Hydrophobic Hydrophilic they carry +ve charge.

(d) Preferential ion adsorption theory: According to this theory sol particles possesses the tendency for
adsorption of common ion present in solution and thereby, acquiring their charge, e.g.
Addition of KI drop by drop to dilute, cold solution of AgNO3 gives rise to +ve sol of AgI, since, AgI particles
adsorb Ag+ ions present in excess in solution.

KI + AgNO3 → AgI + KNO3


Excess
Ag+
[AgI]  Ag+, NO3¯, K+
Similarly, addition of AgNO3, drop by drop to dilute, cold solution of KI gives rise to –ve sol of AgI, since, AgI
particles adsorb I¯ ions present in excess in solution.

AgNO3 + KI → AgI + KNO3


Excess

 AgI   I¯, NO3¯ , K +

6.2 Distribution of Charge

Figure 21.3: Distribution of charge


2 1 . 1 0 | Surface Chemistr y

(a) Helmholtz gave electrical double layer concept for distribution of charge on sol particles.
(b) One layer is made up of dispersed phase carrying either +ve or –ve charge adhered on it. This is fixed layer.
(c) Other layer is made up of dispersion medium carrying +ve as well as –ve charges distributed in it. This is
movable layer.
(d) Two layers are separated by an imaginary plane known as hydrodynamic plane of shear.
(e) The net charge on fixed layer is just equal and opposite to that on movable layer.
(f) The potential difference set-up across the surface of separation of two oppositely charged layers just in
contact with each other is known as electro kinetic potential or zeta potential.

7. SOME OTHER IMPORTANT PHENOMENA

7.1 Cataphoresis or Electrophoresis


(a) Reuss observed the migration of colloidal particles under the influence of electrical field and the phenomenon
was named as cataphoresis or electrophoresis.
(b) Sol particles carry charge and thus, move towards opposite electrodes.
(c) The migration velocity of sol particles is almost equal to the velocity of ions and is of the order of 10–5 cm
sec–1 under a potential gradient of 1 volt cm–1.
(d) The phenomenon has found applications in:
(i) Determining nature of charge on sol particles.
(ii) Determining electrokinetic potential.
(iii) Coagulation

7.2 Electro-Osmosis
The movement of the dispersion medium under the influence of electrical field when the dispersed phase particles
are prevented from moving is known as electro-osmosis.

7.3 Coagulation or Flocculation


When an electrolyte is added to colloidal solution, the particles of the solution take up oppositively charged ions
and thus, get neutralized. The neutral particles then comes closer and get accumulated to form bigger particles
which settle down. Hence coagulations is defined as a process which involves precipitation of a colloidal solution
by addition of excess of electrolyte.

Coagulation Value
Coagulation value is the minimum amount of electrolyte required to coagulate a definite amount of sol.
E.g. For +ve sol -ve ion of electrolyte is effective ion:
Coagulation value: KNO3 > K2SO4 > K3PO4 > K4Fe(CN)6

Coagulation power: NO3¯ < SO42– < PO43– < Fe(CN)64–

For –ve sol +ve ion of electrolyte is effective ion:

Coagulation value: KNO3 > Ba(NO3)2 > Al(NO3)3 > Th(NO3)4

Coagulation power: K+ < Ba2+ < Al3+ < Th4+


Chem i str y | 21.11

Illustration 6: A negatively charged yellow sol of As2S3 on mixing with a +vely charged Fe(OH)3 red sol in equivalent
amount give rise to colorless solution. Why? (JEE MAIN)

Sol: This is because the charges on sol particles neutralize to cause mutual coagulation and thus dispersed particles
of As2S3 and Fe(OH)3 are settled down leaving water, which is colorless.

7.4 Lyotropic Series


Hofmeister studied the efficacy of various ions on coagulation of lyophobic sols and the results obtained were
known as Lyotropic series of Hofmeister series
For –ve sol: Mg2+ > Ca2+ > Ba2+ > Na+ > K+
For +ve sol: SO42– > Cl¯ > NO3¯ > ClO3¯ > I¯

7.5 Isoelectric Point


Addition of electrolyte to sol particles, neutralizes their charge and thus, electrokinetic potential approaches to
zero. At this point sol particles do not carry charge and are either coagulated or just to coagulate; however, they
become inert towards cataphoresis. This condition is known as isoelectric point, at which the colloidal particles do
not carry charge and possess minimum stability.

7.6 Protective Action of Lyophobic Colloids and Gold Number


We know that lyophobic sols (like metal sols) are unstable and are easily coagulated by addition of electrolytes.
However, it is observed that if a lyophilic colloid is added to lyophobic one, the latter is not coagulated easily by
the addition of electrolytes. The process is called protection and the lyophilic sols used for protection are known
as protective colloids. The protective action is generally so strong that the resulting sol is reversible i.e. it can be
evaporated to dryness and then peptized simply by shaking with water to get the Sol Protargol and Argyrol used
as eye drops are protected forms of colloidal silver.
Zsigmondy introduced the term gold number to measure the protective powers of different colloids. Gold
number is number of milligrams of dry colloids which when added to 10 mL of a standard red gold sol (0.0055%
AU) will just prevent its coagulation (indicated by change of color from red to blue) on addition of 1 mL of 10%
sodium chloride solution rapidly to it. Smaller the value of the gold number of a protective colloid, the greater
is its protective action.

Illustration 7: What is electrostatic precipitation of carbon? (JEE ADVANCED)

Sol: The method is used to control air and soil pollution. The apparatus is known as Cottrell’s precipitator. Smoke
formed in industries contains carbon particles which are negatively charged. Before it comes out from the chimney,
it is passed through a chamber containing plates at high tension, i.e., high electrode potential. The carbon particle
gets attracted towards anode and loses their charge at anode and get precipitated.

Illustration 8: Gold number for different lyophilic (A, B, C, D) are in the order A > B > C > D. Which one is more
powerful protective colloid and why? (JEE MAIN)

Sol: Smaller the value of the gold number of a protective colloid, the greater is its protective action.
Thus D is most powerful protective colloid.

Illustration 9: The coagulation of 100 mL of colloidal solution of gold is completely prevented by 25g of starch to
it before adding 10 mL of 10% NaCl solution. Find out the gold no. of starch. (JEE ADVANCED)

Sol: The values given cannot be altered as this is experimental fact. Thus, such type of calculations should not be
made. In order to find out gold no. we have to use 10 mL gold solution of given sample and must observe the
coagulation by adding 1 mL of 10% NaCl taking different amount of protective colloid each time.
2 1 . 1 2 | Surface Chemistr y

8. EMULSIONS
(a) A dispersion of liquid droplets in another liquid, the two liquids should be immiscible.
(b) Emulsions are of two types; normally one of the two constituents of emulsions is water.
(i) Water in oil or w/o: D.P. is water; D.M. is oil, e.g., butter, cream
(ii) Oil in water or o/w/: D.P. is oil; D.M. is water, e.g., milk.
(c) The interconversion of phases in emulsion is known as phase inversion, e.g. creaming of milk (milk: fat in
water, cream: water in fat.)

Emulsifying agents: Emulsions, like other colloidal systems are unstable and are stabilized by the addition of a
third component known as emulsifying agent or emulgents or emulsifiers. In the absence of an emulsifying agent
the dispersed droplets coalesce together and the two liquids are separated into separate layers.
Commonly used emulsifiers are surfactants, polymers and metal oxides and hydroxides in finely divided states.

9. GELS
Liquid dispersions in solid are known as gels, e.g. curd, cheese, etc. Liquid rich systems are known as jellies. Gels
too are unstable and stabilized by gelling agent, e.g. gelatin.
Some remarkable properties of gels are:
(a) Syneresis or weeping of gels: Spontaneous outcome of internal liquid without disturbing gel structure.
(b) Imbibition or swelling: Gels on keeping in contact with their dispersed phase takes in considerable amount
of it and swells up.
(c) Thixotropy or melting by touch: Outcome of internal liquid on applying shear to gels and thus, gel structure
is also disturbed.
(d) Periodic precipitation: Precipitation reactions, if carried out in gel medium, layers or rings of precipitates are
formed at definite intervals in order of geometrical progression.

10. ASSOCIATED COLLOIDS

10.1 Micelles
The substances which at low concentration in a medium behave as normal strong electrolytes but at higher
concentration exhibit colloidal properties due to the formation of aggregated particles are called associated
colloids. The aggregated particles thus formed are called micelles.
Polar head, COO-
COO-
COO-

COO-
Non-polar tail, R
water
COO- COO-

COO-
COO-

COO- COO-
COO-

Aggregation of RCOO-
ions to form anionic micelle

Figure 21.4: Micelle Formation


Chem i str y | 21.13

Their formation takes place above a particular temperature called Kraft temperature (TK) and above a particular
concentration called Critical Micelle concentration (CMC). The minimum concentration of surfactant at which
micelle formation starts is Critical Micelle Concentration (CMC).

PLANCESS CONCEPTS

Lesser is CMC of surfactant, more is its surface activity and detergency.


The micellisation nature depends upon the hydrophilic lipophilic balance (HLB) in a surfactant molecule.
More is the hydrophobic character in hydrocarbon tail lesser is the CMC of surfactant molecule in
aqueous solution, i.e., CMC of CH3 ( CH2 ) SO 4Na is lower than CMC of CH3 ( CH2 ) SO 4Na when n1
n1 n2
> n2 in aqueous solution.
In other words longer is hydrocarbon chain lesser is its CMC. The surfactant molecules with linear
hydrophobic chain possess the lower CMC than the corresponding surfactant molecules with branched
chain in aqueous solution.
Aman Gour (JEE 2012, AIR 230)

Multimolecular Colloids: A multimolecular colloid consists of aggregated atoms or molecules as colloidal particles
of diameter less than 1 nm, e.g. gold, sulphur Sol

Macromolecular Colloids: In a macromolecular colloids, the dispersed phase particles are themselves
macromolecules (polymers), e.g., protein Sol

PLANCESS CONCEPTS

•• Macromolecular colloids partly dissolves to form homogeneous solution and partly in colloidal nature
(heterogeneous).
•• Macromolecular colloids possess higher viscous nature due to high degree of solution.
•• Macromolecular colloidal solution show larger deviations from ideal nature because of larger size and
shape.
Nitin Chandrol (JEE 2012, AIR 134)

Illustration 10: What is the difference between multimolecular and macromolecular colloids? Give one example of
each. How are association colloids different from these two types of colloids?  (JEE MAIN)

Sol:

Multimolecular Colloids Macromolecular colloids


Multimolecular colloids are dispersed particles made Macromolecular colloids are molecularly dissolved solutions of a
of aggregates of many molecules. polymer (due to large size of polymer molecular, it takes colloidal
state)
These are lyophobic colloids. They are lyophilic colloids.
E.g. As2S3 Sol E.g. Starch solution

Association colloids: These are also lyophilic colloids but are formed by the aggregation of soap or detergent
molecules in solution above CMC (these are micellar colloids).
2 1 . 1 4 | Surface Chemistr y

11. SURFACTANTS AND DETERGENTS


Surfactants: Substances which possess surface activity i.e., the property to lower the surface tensions of liquids or
the tendency to increase surface area. Surfactants are classified into three categories:
(a) Anionic surfactants: The anionic part of surfactant molecule possesses hydrophobic and hydrophilic particles
and thus give rise to anionic micelles, e.g., sodium palmitate (C15H31COONa), sodium locate (C17H33COONa), sodium
dodecyl sulphate (C12H25SO4Na), etc.

(b) Cationic surfactants: The cationic part of surfactant molecule possesses hydrophilic and hydrophobic moieties
and thus give rise to cationic micelles, e.g. octadecyl ammonium chloride (C18H37NH3+Cl¯), cetyl trimethyl ammonium
chloride [C16H33(CH3)3N+Cl¯], cetyl pyridinium chloride

NH Cl-
+
C16H33

(c) Non-ionic surfactant: The whole molecule of non-ionic surfactant possess non-dissociation nature and due
to the presence of hydrophilic and hydrophobic moities in non-ionic surfactant undergoes micelle formation, e.g.
poly oxy ethylene glycol derivatives, e.g., C8H2n+1 (OCH2CH2)m·OH.

Detergents: Substances which possess surface activity as well as detergency (i.e., cleansing action).

PLANCESS CONCEPTS

All detergents are surfactants but all surfactants are not detergents.
Rohit Kumar (JEE 2012, AIR 79)

Soaps: Metal salts (preferably alkali metals) of higher fatty acids. These belong to anionic class of detergents.

Illustration 11: Why hard water consumes more soap? (JEE MAIN)

Sol: Soap are metal salt of fatty acids, on dissolution in water it gives RCOO¯ ions, which undergoes to micelle
formation, RCOONa →RCOO¯ + Na+. This micelle formation is responsible for cleaning action. Hard water contains
ions like Ca+2, Mg+2. Hence in hard water RCOO¯ ions are used up by Ca2+ or Mg2+ ions to form insoluble (RCOO)2Ca
or (RCOO)2Mg and thus micelle formation starts only when whole of Ca2+ and Mg2+ ions are precipitated out by
soap. Thus it consumes more soap.

Illustration 12: Which type of bonding occurs between greasy material and soap during detergency? (JEE MAIN)

Sol: Van der Waal’s bonding.

Saponification: The action of alkalis over fats or oil is known as saponification.

R’, R’’ and R’’’ may be same or different.


Chem i str y | 21.15

ADSORPTION AND CATALYSIS

1. INTRODUCTION
There are many properties of substances, particularly of solids and liquids which depend upon the nature of the
surface. The branch of chemistry which deals with the nature of surfaces and changes occurring on the surfaces is
called surface chemistry. Adsorption of solid or on solution surfaces is an important surface effect which is useful
to understand many physical and chemical properties of the substances.
(a) Adsorption: The phenomenon of attracting and retaining the molecules of a substances on the surface of a
liquid or solid leading to a higher concentration on the surface in comparison to the bulk is called adsorption.
Examples are (i) vapor on silica gel, (ii) H2O2NH3 on activated charcoal.
(b) Absorption: It is the phenomenon in which a substance is uniformly distributed throughout the bulk, i.e. it is
the penetration of the substance through the surface into the bulk of the solid.
Example: Water vapors are absorbed by anhydrous CaCl2.
(i) Adsorbent: The solid substance on the surface of which adsorption occurs is known as adsorbent.
(ii) Adsorbate: The substances that get adsorbed on the solid surface due to intermolecular attraction are
called adsorbate.
(c) Sorption: In some cases, both absorption and adsorption occur together and are not distinguishable. In
such cases, the substance gets uniformly distributed into the bulk of the solid but at the same time, its
concentration is higher at the surface than in the bulk. Such a phenomenon is called sorption.

2. TYPES OF ADSORPTION
Depending upon the nature of forces which hold the molecules of the adsorbate on the surface of the adsorbent,
adsorption is classified into two types: Physical adsorption and Chemical adsorption.
(a) Physical adsorption: When the particles of the adsorbate are held to the surface of the adsorbent by physical
forces, e.g. Van der Waals forces, the adsorption is called physical adsorption or physisorption.
(b) Chemical adsorption: When the molecules of the adsorbate are held to the surface of the adsorbent by the
chemical forces, the adsorption is called chemical adsorption or chemisorption.

Table 21.3: Differences between physical adsorption and chemical adsorption

Physical Adsorption Chemical Adsorption


The forces between the adsorbate molecules and the The forces between the adsorbate molecules and the
adsorbent are weak Van der Waals forces. adsorbent are strong chemical forces.
Low heat of adsorption of the order of 20-40kJ mol–1. High heat of adsorption of the order of 200-400 kJ mol–1.
Usually occurs at low temperature and decreases It occurs at high temperature.
with increasing temperature.

It is reversible. It is irreversible.
The extent of adsorption depends upon the ease of There is no correlation between extent of adsorption
liquefication of the gas. and the ease of liquefication of gas.

It is not specific in nature i.e., all gases are adsorbed It is highly specific in nature and occurs only when there
on the surface of a solid to same extent. is bond formation between adsorbent and adsorbate
molecules.
2 1 . 1 6 | Surface Chemistr y

The state of adsorbate is same as in the bulk from State of adsorbate molecules may be different.
that in the bulk.
It forms multimolecular layers. It forms mono-molecular layer.
Rate of adsorption increases with increase in Rate of adsorption usually remains almost same and
pressure of adsorbate. does not change appreciably with change in pressure.

Illustration 13: Why are all adsorption exothermic? (JEE ADVANCED)

Sol: Adsorption process (be it physical or chemical) involves attractions between the molecules of adsorbate and
adsorbent and thus, energy is given out.

3. ADSORPTION OF GASES ON SOLIDS


Gases are adsorbed on the finely divided metals such as Ni, Pt, Pd, Fe, etc. The extent of adsorption of a gas on a
solid surface is affected by the following factors:
(a) Nature of the gas (b) Nature of adsorbent (c) Effect of pressure
(d) Effect of temperature (e) Activation of adsorbent.

(a) Nature of the gas: The higher the critical temperature, the more easily a gas is liquefied and hence more
readily it will be adsorbed. For example.

Table 21.4: Critical temperature of some gases

Gas H2 N2 CO CH4 CO2 NH3 SO2


Critical temp. (K) 33 126 134 190 304 406 430
Amount of gas as
4.5 8.0 9.3 16.2 48 180 380
adsorbed in mL

(b) Nature of adsorbent: Activated charcoal can adsorb which are easily liquefied. Many poisonous gases are
adsorbed by charcoal.

(c) Effect of pressure: The extent of adsorption of a gas per unit mass of adsorbent depend upon the pressure
of the gas. The variation of extent of adsorption (expressed as x/m where x is the mass of the adsorbate and
m is the mass of the adsorbent) and the pressure is plotted. A graph between the amount of adsorption and
gas pressure keeping the temperature constant is called an adsorption isotherm.

Adsorption Isotherms Adsorption Isotherms


adsorbed per gram

o
-78 C
Amount of gas

o
-29 C x/cm
o
0C

O P PS
Pressure (1) (2)

Figure 21.5: Effect of pressure on rate of adsorption


Chem i str y | 21.17

It is clear from the graph-2 that extent of adsorption (x/m) increases with pressure and becomes maximum
corresponding to pressure Ps. called equilibrium pressure. Since adsorption is a reversible process, desorption also
takes place simultaneously. At this pressure (Ps) the amount of gas adsorbed becomes equal to the amount of gas
desorbed so that the extent of adsorption becomes constant even though the pressure is increased. This state is
saturation state and Ps is called saturation pressure.

Freundlich Adsorption Isotherm: The variation on extent or adsorption (x/m) when pressure (p) was given
mathematically by Freundlich. The following observations can be easily made:
(i) At low pressure, the graph is almost straight line which indicates the x/m is directly proportional to the
x x
pressure. This may be expressed as: ∝ p or =kp
m m
(ii) At high pressure, the graph becomes almost constant which mean that x/m becomes independent of pressure.

This may be expressed


= as:
x
m
x
Cons tant or ∝ p0
m
(
p0 = 1 )
x 0
Or = pk
= k As p0 = 1 (pressure raised to the power zero = 1)
m
(iii) This, in the intermediate range of pressure, x/m will depend upon the power of pressure which lies between 0
x x
and 1 i.e., fractional power of pressure. This may be expressed as ∝ p1/n ; = k p1/n
m m
Calculation of k and n of adsorption isotherm:
The constant k and n can be determined as explained below:
x
Taking logarithms on both sides of equation = kp1/n
m
x 1
log= logk + logp
m n log x slope=
1
m n

 Intercept = log k
O log p
(3)
Figure 21.6: Plot of log (x/m) vs log p

Thus, if we plot a graph-3 between log (x/m) and log p, a straight line will be obtained. The slope of the line is equal
to 1/n and the intercept is equal to log k.

Illustration 14: How much surface area of cube of edge length 1cm increases if it is broken into cube of edge
length 1 × 10–3 cm? (JEE ADVANCED)

Sol: First calculate the surface area of one cube of edge length of (1x10-3) now by calculating the volume of one
cube, find out the number of cubes formed by a cube of 1 cm now by using volume and no of cubes calculate the
new surface area.
Here It is given that the cube has an edge length if 1 cm.
So the surface are of the cube (by mathematical expression) = 6a2
Here a = 1
The surface area of cube of edge length 1 cm is 6 cm2.
If edge length is reduced to 1 × 10–3 cm,
2 1 . 1 8 | Surface Chemistr y

The surface area of one cube becomes 6 × 10–6 cm2


[n × volume of 1 cube (1 × 10–3)3 = Volume of 1 cube (1)3]
1
∴ Number of cubes = = 109
1 ×10 −9

Number of cubes of edge length 1 × 10–3 cm formed by a cube of 1cm = 109


Thus total new surface area becomes
= 6 × 10–6 × 109 = 6 × 103 cm2
Or increase in surface area is 103 times.

Illustration 15: 1.30 liter of O2 gas 1 atm and 300 K is exposed to a solid surface of 3g in a container. After complete
x
adsorption the pressure of O2 is reduced to 0.7 atm. Calculate the value of .  (JEE MAIN)
m
Sol: By using ideal gas equation calculate x/m
PVm 1 ×1.5 × 32
w
= O
= = 1.948g
2
RT 0.0821 × 300

wO Left after= 0.7 ×1.5 × 32


adsorption = 1.364 g
2
0.0821 × 300

∴ xO adsorbed =
1.948 – 1.364 = 0.584 g ;
2

x 0.584
= = 0.194
m 3
Langmuir Adsorption Isotherm: Consider a solid surface of definite area in contact with gaseous molecules.
Let fraction of the surface is occupied by the gaseous molecules to show adsorption at equilibrium, then rate of
desorption ∝ θ = Kd θ (where Kd is desorption constant)
Rate of absorption ∝ P(1 – θ) = KaP(1 – θ ) (Where Ka is adsorption constant)

At equilibrium, Kd θ = KaP(1 – θ )

Ka
·P
K a ·P Kd
Or θ
= =  ... (i)
K a ·P + K d K a
·P + 1
Kd
K1 P  Ka 
θ =  where K1   … (ii)
1 − K1  K d 
x x
Also the amount of gas adsorbed per unit mass of adsorbent i.e. is directly proportional to θ i.e. θ K2 · θ
∝=
m m
x  K1P  K1K 2P
Or
= K=
2    … (iii)
m  1 + K1P  1 + K1P
x aP
=  ... (iv)
m 1 + bP
a
(Where a = K1K2 and b = )
K2
x aP
The Langmuir adsorption isotherm is thus, represented as =
m 1 + bP
(Where a and b are two Langmuir parameters)
Chem i str y | 21.19

At very high pressure, the above isotherm acquires the limiting form:
x a
= (K1P >>> 1 bP >>> 1)  ... (v)
m b
And at very low pressure, it is reduced to
x
= aP (K1P <<< 1bP <<< 1)  ... (vi)
m
The parameters a and b, can be determined by transforming the eq. (iv)
m 1 + bP b 1
= = +  … (vii)
x aP a aP
A plot of m/x against 1/P would give a straight line with slope and intercept equal to 1/a and b/a, respectively.
The Langmuir isotherm, is preferred over the Freundlich isotherm when a monolayer is formed. A plot of x/m versus
P is shown in Fig. 21.7. At low pressure, x/m increases linearly with P. At high pressure x/m becomes (eq. v) constant
i.e., the surface is fully covered. Now change in pressure has no effect and non-further adsorption takes place.
Amount of gas adsorbed

x/m

Pressure, P

Figure 21.7: Plot of (x/m) vs pressure 1

PLANCESS CONCEPTS

x
The ∝ surface covered ∝ θ . Also it increases with pressure and exothermic. At high pressure,
m
x a
= = Constant, i.e. achieves the condition of equilibrium when rate of adsorption and rate of
m b
desorption becomes constant. Thus ∆G = 0.
Krishan Mittal (JEE 2012, AIR 199)

x ∆H°− T∆S °
−∆G°  
x − 
∆G° = – 2.303 RT log or
= 10 =
2.303RT
10 
2.303RT 

m m

x  2.303R
+ ∆S °
  2.303RT
−∆H°
 x  ∆RS °   −∆RTH° 
K eq.
= = 10  × 10  ; K eq.
= = e  × e 
m     m    

Illustration 16: The chemisorptions of H2 on an activated surface becomes 40% faster if temperature is raised from
500 K to 1000K. Calculate the activation energy of chemisorptions. (JEE ADVANCED)

Sol: Here comparative rates are given hence by using the following equation calculate the activation energy.
r2 Ea [T2 − T1 ]
Formula to be used = 2.303log =
r1 R T1 T2
2 1 . 2 0 | Surface Chemistr y

Rate of chemisorption at 500K = a


a
Rate of chemisorption at 1000K = 50 × +a=1.5a
100
r2 Ea [T2 − T1 ]
Now, 2.303log = ;
r1 R T1 T2

1.5a Ea  1000 − 500 


2.303log =  
a 8.314  1000 ×500 
Ea = 3371 J = 3 kJ

Competing adsorption: Adsorption is competitive for different adsorbates. A strong adsorbate adsorbs more
efficiently than weak adsorbate. Even a strong adsorbate can adsorb by displacing pre-adsorbed weak adsorbate.
This is called competing adsorption or preferential adsorption.

Illustration 17: Different concentrations of aqueous solution placed with charcoal which adsorbs a part of solute
from solution as reported below at equilibrium.
Conc. of solution × 102 2.0 4.0
x
0.185 0.290
m
Calculate the values of log k and n. (JEE MAIN)
X 1
log
Sol: By using Freundlich equation = logC + logk calculate the values of n and k
m n
1
log0.185= log2 ×10−2 + logk  … (i)
n
1
log0.290= log 4 ×10−2 + logk  … (ii)
n
1
By Eq. (i) – 0.7328 = × [– 6989] + log k
n
1
By Eqs. (ii) – 0.5376 = × [– 1.3979] + log k
n
By Eqs. (i) and (ii) n = 0.64, k = 83.5 and log k= 1.92

(d) Effect of temperature

Adsorption isobar for


physisorption

x/m

Temperature

Figure 21.8: Effect of temperature on physisorption


Chem i str y | 21.21

Maximum
adsorption

e
Adsorption isobar for

as
re
Chemisrption

inc
x/m

p.
m
m
te
x/
th
wi
Temperature
Figure 21.9: Effect of temperature on chemisorption

At low temperature, x/m is small. As temperature is increased, the molecules of the adsorbate gain energy and
become equal to activation energy so that proper bonds are formed with the adsorbent molecules.
Temperature

Chemisorption

Pressure
Figure 21.10: Effect of pressure on Chemisorption

(e) Activation of adsorbent: Activation of adsorbent means increasing the adsorbing power of the adsorbent. It
is very necessary to increase the rate of adsorption. This can be done by the following methods.
(i) Metallic adsorbents are activated by mechanical rubbing or by subjecting them to some chemical
reactions.
(ii) To increase the adsorbing power of adsorbents, they are sub-divided into smaller pieces. As a result, the
surface area of imbalanced forces increases and therefore, the adsorbing power increases.

Breaking New Imbalance Force New Imbalance Force


x + x x + x
x x x x
x x x x x + x x + x

x x x x x + x x + x
x x x x
x + x x + x

Figure 21.11: Adsorption power

(iii) Some adsorbents are activated by strong heating in contact with superheated steam. For example,
charcoal is activated by subjecting it in to the action of superheated steam.

Applications of Adsorption: Some of the important applications of adsorption are given below:
(a) In gas masks: Activated charcoal is generally used in gas masks to adsorb poisonous and toxic gases from
air. These masks are commonly used by the miners because there are poisonous gases like CO, CH4 etc. in the
atmosphere in the coal mines. Therefore, these masks help to purify the air for breathing.
2 1 . 2 2 | Surface Chemistr y

(b) In dyeing cloth: Mordants such as alums are used in dyeing cloth. They adsorb the dye particles which,
otherwise, do not stick to the cloth.
(c) In dehumidizers: Silica gel is commonly used to adsorb humidity or moisture from air. This is necessary for
storage of delicate instruments which might otherwise be damaged by moisture.
(d) Removal of coloring matter: Many substances such as sugar, juice and vegetable oils are colored due to the
presence of impurities. They can be recolored by placing in contact with adsorbents like activated charcoal
or fuller’s earth. This method is commonly used in the manufacture of sugar. The colored sugar solution is
treated with animal charcoal or activated charcoal.
(e) Heterogeneous catalysis: The phenomenon of adsorption is useful in the heterogeneous catalysis. The
metals such as Fe, Ni, Pt, Pd, etc are used in the manufacturing process such as Contact process, Haber
process and the hydrogenation of oils. Its use is based upon the phenomenon of adsorption.
(f) In ion-exchange resins: The organic polymers containing groups like – COOH, – SO3H etc possess the
property of selective adsorption of ions from solution. These are quite useful in the softening of water.
(g) In adsorption indicators: Many adsorption indicators are being used in volumetric analysis, e.g. dyes such as
eosin and fluorescein are used as adsorption indicators.
(h) In qualitative analysis: Certain qualitative tests such as the lake test for the confirmation of Al3+ ions are
based upon adsorption i.e. Al (OH)3 has the capacity to adsorb the color of blue litmus from the solution.
(i) Production of high vacuum: The adsorption of air in liquid air helps to create high vacuum in a vessel. This
process is used in high vacuum instruments as Dewar flask for storage of liquid air or liquid hydrogen.

4. CATALYSIS
Berzelius used the term catalyst for the first time for the substances which accelerate the rate of chemical reaction.
Now, the term catalyst has been used for the foreign substances which influence the rate of reaction and the
phenomenon is known as catalysis. Usually, two terms are used for catalysis.

(a) Positive catalysis: The phenomenon in which presence of catalyst accelerates the rate of reaction.
(b) Negative catalysis: The phenomenon in which presence of catalyst retards the rate of reaction. Such
substances are also known as inhibitors or negative catalyst.

PLANCESS CONCEPTS

Substances that accelerates a reaction are called catalysts whereas those which decreases the rate of a
reaction are called inhibitors.
T P Varun (JEE 2012, AIR 64)

4.1 Characteristics of Catalysts


The catalyst is generally specific in nature. Different catalysts for the same reactants may form different products.
For example
Cu CO2(g) + H2 (g) (Dehydrogenation)
HCOOH(l)
HCOOH()
Al2 O3
CO(g) + H2 O(g) (Dehydration)

(a) A catalyst can never initiate a chemical reaction. It simply influences the rate of reaction. However, combination
of H2 and Cl2 takes place only when moisture (catalyst) is present.
Chem i str y | 21.23

(b) A small quantity of catalyst is sufficient to influence the rate of reaction, e.g. 1 g-atom of Pt (i.e. 195 g) is
sufficient to catalyze decomposition of 108 liter H2O2.
(c) A catalyst does not influence the equilibrium constant or reaction. It simply helps in attaining equilibrium
earlier. It alters the rate of forward and backward reactions equally.

Illustration 18: What role a catalyst plays in establishing the equilibrium in reversible process?(JEE ADVANCED)

Sol: Catalyst is a substance that increases the rate of forward reaction thus increases concentration of products
which in turn favors backward reaction faster and ultimately helps in attaining the equilibrium earlier.

4.2 Types of Catalysis


(a) Homogenous Catalysis: In this, both the catalysts and the reactants are in the same phase.
Example,
(i) Catalytic oxidation of SO2 into SO3 in the presence of NO as catalyst in the lead chamber process. In this
case all the reactant and the catalyst are in the gaseous phase.
NO(g)
2SO2 (g) + O2 (g) → 2SO3 (g)

(ii) Hydrolysis of ethyl acetate in the presence of acid


H (aq)
+

CH3COOC2H5 (l) + HOH (l) → CH3COOH (l) + C2H5OH (l)

(b) Heterogeneous Catalysis: In this, both the catalysts and the reactant are in different phase.
Example: Some common examples of heterogeneous catalysis are as follows.
(i) Oxidation of HCl into Cl2 by Deason’s process in the presence of CuCl2.
CuCl2 (s)
4HCl (g) + O2 (g)  → 2Cl2 (g) + 2H2O (g)
450°C
(ii) Manufacture of NH3 from N2 and H2 by Haber’s process using finely divided iron as catalyst.
Fe(s)

N2 (g) + 3H2 (g)  2NH3 (l)
+Mo (Promoter)

(iii) Oxidation of sulphur dioxide (SO2) into sulphur trioxide (SO3) in the presence of vanadium pentoxide as
catalyst.
V O (s)
2 5 

2SO2 (g) + O2 (g)  SO3 (g) (Contact Process)

Important Characteristic of Heterogeneous Catalyst


The two important aspects of heterogeneous catalysis are activity and selectivity.
(a) Activity: It implies the ability of catalyst to accelerate a chemical reaction. The degree of acceleration can be
sometimes as high as 1010. For example, the reaction between H2 and O2 gases in the presence of platinum
catalyst to form water occur with an explosive violence, whereas a mixture of pure H2 and O2 gases can be
stored indefinitely without any reaction.

(b) Selectivity: Selectivity of a catalyst is its ability to direct the reaction in such a way so as to yield particular
products excluding others. For example.
2 1 . 2 4 | Surface Chemistr y

(i) n-Heptane selectivity give toluene in the presence of platinum catalyst.

(ii) Acetylene on reaction with H2 in the presence of platinum catalyst gives ethane while in the presence of
Lindlar’s catalyst (Pd supported over BaSO4 partially deactivated by sulphur or quinoline) gives ethene
as the main product.
Pt
CH3-CH3

H-C C-H+H2
Lindlar’s catalyst
CH2=CH2

(iii) Enzyme catalysis: Enzymes are complex nitrogenous organic compounds present in living begins inside
the human body like digestion therefore enzymes are also called biocatalyst.
Example:
Uresse
NH2CONH2 + H2O  → 2NH3 + CO2
Urea
Zymase
C6H12O6 + H2O  → 2C2H5OH + 2CO2
Invertase
C12H22O11 + H2O → C6H12O6 + C6H12O6
Gulcose Fructose

PLANCESS CONCEPTS

Substances that accelerates a reaction are called catalysts whereas those which decreases the rate of a
reaction are called inhibitors.
T P Varun (JEE 2012, AIR 64)

For Example:
Ni
CH4(g) + H2O(g)
Cu / ZnO.Cr2O2
CO(g) + H2(g) CH3OH(g)
Cu
HCHO(g)

Note: Thus action of a catalyst is highly specific or selective in nature i.e., a substance can act as a catalyst for a
particular reaction and not for all the reactions.

5. THEORIES OF CATALYSIS
(a) Intermediate compound formation theory: According to this theory catalyst combines with one or more of
the reactants to give an intermediate product which either decomposes or reacts with other reactants to give
product and regenerates the catalyst.
2 MnO
(i) Catalytic decomposition of KClO3: 2KClO3 → 2KCl + 3O2
Chem i str y | 21.25

The mechanism of this reaction according to intermediate compound formation theory is given below:
2KClO3 + 2MnO2 → 2KMnO 4 + Cl2 + O2
1 −int ermediate

2KMnO4 → K 2MnO 4 + MnO2 + O2


II −int ermediate

K2MnO4 + Cl2 → 2KCl + MnO2 + O2

Illustration 19: Which acts as catalyst for the destruction of ozone layer in presence of Freon or chlorofluoro
carbons? (JEE MAIN)
UVrays
O3  → O2 + O

Sol: The destruction of ozone takes place by a free radical mechanism. Presence of chlorofluoro carbon forms free
radicals which catalyzes the decomposition of ozone.

hυ → CClF + Cl
CCl2F2  2
• •
Cl + O → O2 + ClO
• •
ClO+ O → Cl + O

(b) Adsorption Theory: According to this theory gas molecules are adsorbed on the surface of catalyst giving
rise to higher concentration of reacting species at the surface which results in faster rate of reaction.

(c) The Modern Theory: It is a combination of previous two theories.


(i) The catalyst enters into loose chemical or physical combination (i.e. chemisorptions or physic adsorption)
with reactants to form and adsorbed activated complex, having its energy level somewhat lower than the
complex which would have been formed in absence of catalyst.
Thus, the catalyst provides new pathway involving lower threshold energy level due to large number
of effective collisions occurring in the presence of a catalyst in comparison to effective collisions at the
same temperature in absence of a catalyst. Hence, the presence of a catalyst makes the reaction to go
faster. Fig. 21.12, shows threshold energy level ET and activation energy EA, in absence of a catalyst. These
values are higher than the corresponding threshold energy level ET’ and activation energy EA’ in presence
of a catalyst, ER and EP represent the average energies of reactants and products. The difference (EP – ER)
gives the value of ∆H.

Uncatalysed activated complex


ET
Energy
berrier
Chemical potential energy

Catalysed activated complex


ET
EA

EA

ER
Initial Reactants
state(A+B) H of reaction
Product
EP
Final state (C+D)

Reaction sequence

Figure 21.12: Catalytic Reaction Pathway


2 1 . 2 6 | Surface Chemistr y

(ii) It is thus, evident that catalyst lowers the threshold energy barrier (ET to E’ T) and thereby lowers the energy
of activation of reaction, threshold energy level is the minimum energy level which the reactant molecules
must possess in order to show a chemical change whereas energy of activation is the additional, i.e., the
minimum amount of energy provided to reactant molecules to form activated complex. It is thus evident
that catalyst provides another path way for a reaction which has lower values of threshold energy level.

6. SOME OTHER TYPES OF CATALYSIS


(a) Negative Catalysis or Inhibitors: The phenomenon in which presence of foreign substance retards the rate
of the reaction.e.g.
(i) Acetanilide or glycerin or H3PO4 acts as negative catalyst for decomposition of H2O2.
(ii) Aldehydes and alcohols act as negative catalyst for oxidation of Na2SO4.

(b) Autocatalysis: The phenomenon in which either of the product formed during reaction acts as catalyst for
the reaction.e.g.
(i) The color of KMnO4 disappears slowly on treating it with oxalic acid but becomes faster after sometime
due to formation of Mn2+ ions which acts as autocatalyst for the reaction.

2KMnO4 + 5H2C2O4 + 3H2SO4 → K2SO4 + 2MnSO4 + 8H2O + 10CO2

(c) Acid-Base Catalysis: The phenomenon in which a reaction is catalysed by acid and base simultaneously.
(i) An acid donates proton to substrate or reactant to form an intermediate, which then loses proton to a
base to give product.

(d) Mutarotation of glucose: The phenomenon in which optical activity of a freshly prepared α-glucose solution
in water rapidly decreases with time till equilibrium is reached between α and β form and similarly, rotation of
β-glucose in solution increases with time.
H+
HG  Base
Acid → HGH →
+
+
HG
α− glucose Intermediate −H β− glucose

(i) The mechanism of specific catalysis by H3O+ or OH¯ involves formation of an ionized intermediate, which
then yields products.
(ii) Ester hydrolysis in aqueous solution is catalysed only by H3O+ and OH¯ and other acids or base (Lewis
acid or Lewis base which do not furnish H3O+ or OH¯) are ineffective.

PLANCESS CONCEPTS

The acid hydrolysis of an ester is reversible while alkaline hydrolysis of ester (also called saponification)
is irreversible.
Saurabh Chatterjee (JEE 2013, AIR)

(e) Enzyme Catalysis: The phenomenon in which reactions are catalysed by enzymes.
Enzymes are complex, biological, and nitrogenous, macromolecules (i.e. proteins) derived from living
organisms and thus, phenomenon is also named as biological or biochemical catalysis.
Characteristics of enzyme catalysis
Chem i str y | 21.27

(i) Enzyme catalysed reactions are highly specific, i.e., one enzyme for one reaction.
(ii) Enzyme catalysed reactions are normally hydrolytic in nature.
(iii) Enzyme catalysed reactions take place with evolution of gases.
(iv) The rate of reaction depends upon enzyme concentration.

PLANCESS CONCEPTS

Enzyme catalysed reactions are highly susceptible to pH of medium. Favorable range of pH is 5 to 7. The
optimum temperature for enzyme catalysed reaction is nearly 30°C. Enzymes being colloidal in nature
and thus, their action is ruined by electrolytes.
Mredul Sharda (JEE 2013, AIR)

(f) Induced Catalysis: The phenomenon in which one reaction influences the rate of other reaction which does
not occur under ordinary condition. e.g. Sodium arsenite solution is not oxidized by air. However, if air is passed
through a mixture of solution of sodium arsenite and sodium sulphite, both of them undergo simultaneous
oxidation. Oxidation of Na2SO3 induces the oxidation of Na3AsO3.

(g) Shape Selective Catalysis by Zeolites: Zeolites are microporous alumino-silicates of the general formula
Mn[(AlO2)x(SiO2)y].mH2O
Where n is the valency of cation Mn+. They may be considered as open structures of silica in which a fraction
x/(x + y) of the tetrahedral sites have been substituted by aluminum. The net negative charge on the
aluminosilicate framework is neutralized by exchangeable cations M of valency n. Zeolites have high porosity
due to the presence of one, two or three dimensional network of interconnected channels and cavities of
molecular dimensions. Other elements such as Ba, Mg, B, Ga and P can take the place of Si and Al in the
zeolitic frame work. Shape selectivity of a catalyst depend upon the pore structure of the catalyst. The pore
size of zeolites generally varies between 260 pm and 740 pm. Depending on the size of the reactant and
product molecules when compound to the size of cages or pores of the zeolite, reactions proceed in a specific
manner.

PROBLEM-SOLVING TACTICS
When free energy change is given and extent of adsorbent has to find out, use the following equation.
x
G° = – 2.303 RT log or
m
It can be written as in terms of enthalpy and Entropy.
−∆G°  ∆H°− T∆S ° 
x −
2.303RT 

= 10
= 2.303RT
10 
m
x  ∆S °   −∆H° 
x  2.303R
+ ∆S °
  2.303RT
−∆H°
 K eq.
= = e R  × e RT 
K eq.
= = 10  × 10 ; m    
m    
K eq1 ∆H (T2 − T1 )
Also we can have ln = x
K eq2 R T1 T2
r2 Ea [T2 − T1 ]
When rate of adsorption or desorption is given use the following equation, 2.303log =
r1 R T1 T2
2 1 . 2 8 | Surface Chemistr y

POINTS TO REMEMBER

COLLOIDS

Characteristics of Colloidal State


•• It is a particular state, Heterogeneous in nature, number of phases = 2. i.e Dispersed phase and Dispersion
medium.
•• The one phase dispersed in other is known as dispersed phase (or D.P.) or internal phase or discontinuous
phase whereas, the other in which dispersions are made is known as dispersion medium (D.M) or external
phase or continuous phase.
Preparation of Colloid

1) Mechanical Double Oxidation Reduction Solvent Hydrolysis


Dispersion Decomposition Exchange

i) Grinding

ii) Peptization

iii) Bredig arc


Method

iv) Condensation
Method.

Properties of Colloids

Colligative
property Optical properties
Colloidal solution Scattering of light
exhibit properties like Colloidal particles
osmotic pressure, elevation scatters light in all
in boiling point, depression Properties of direction this is
in freezing point. colloidal know as Tyndall effect.
solution

Electrical Brownian Movement


property Movement of colloidal
Origin of charge particle in zigzag manner
Preferential ion
adsorption theory
Physical Properties

(Heterogeneity) (Diffusibility) (Filter ability) (Non setting) (Viscosity) (Visibility) (Surface


Heterogenous Show rate of Not Do not Viscosity is Very small, area &
nature diffusion filterable settle down higher & Surface can not be adsorption)
due to gravity Tension is lesser seen with They have
than the solvent the help of large surface
used as dispersed microscope. area.
medium for sols.
Chem i str y | 21.29

Important Phenomena:

Electrophoresis

Movement of Important Gold Number


colloidal particles Phenomena
under the No. of milligrams
influence of
of dry colloids
electric filed.
which when added
to 10 mL of a
standard red gold
so it will just prevent
its coagulation on addition
Electro-
osmosis of 1 mL of 10% NaCl rapidly
to it.
Coagulation Lytropic Iso-electronic
Movement of Or Points
Series
dispersion Flocculation
medium Condition at which
under the Process which
the colloidal
influence of electric involes
particle do not
field when the precipitation
carry charge and
dispersed of a colloidal
possess minimum
phase particles solution by
are prevented. stability.
addition of
excess of
electrolyte.

ADSORPTION

Absorption and Adsorption process:

Absorbent Adsorbent

Adsorbate
Absorbate

Absorption Adsorption

Physical Adsorption and Chemical Adsorption


2 1 . 3 0 | Surface Chemistr y

Freundlich Adsorption Isotherm


Extent of adsorption (x/m) increases with pressure.
 1. At low pressure extent of pressure is directly proportional to pressure.
x x
∝ p or =kp
m m

 2. At high pressure extent of adsorption is independent of pressure.


x x
= cons tant or ∝ p0
m m

 3. In the intermediate range of pressure, x/m will depend upon the power of pressure which lies between 0 and
1 i.e., fractional power of pressure.
x x
This may be expressed as ∝ p1/n ; = k p1/n Calculation of k and n of adsorption isotherm:
m m
x 1
log
= logk + logp
m n

log x 1
slope=
m n

 Intercept = log k
O log p
(3)
Langmuir Adsorption Isotherm

K1 P  Ka 
θ =  where K1 
1 − K1  K d 

θ= Fraction of area occupied.


Ka= Rate of adsorption
Kb= Rate of desorption.
The Langmuir adsorption isotherm is thus, represented as
x aP
= (Where a and b are two Langmuir parameters)
m 1 + bP
At very high pressure, the above isotherm acquires the limiting form:
x a
= (K1P >>> 1 bP >>> 1)  ... (v)
m b
And at very low pressure, it is reduced to
x
= aP (K1P <<< 1∴ bP <<< 1)
m
Soap and Detergent:
•• Saponification of fats and oil yields soap and glycerol as a byproduct.
•• Detergents are surface active agents having cleaning properties they are also known as surfactants (Surface
active agents).This cleaning properties is due to their characteristic structure.
Chem i str y | 21.31

•• They have polar head group and long hydrocarbon chain tail which is apolar.
•• The concentration at which the surfactants molecule aggregates and micelle formation takes place is known
as critical micellar concentration.
•• Though their hydrocarbon chain they interact with the dirt particle (say oil or grease)
•• Forming and emulsion and imparting cleansing property.

Micelle Formation:

Hydrocarbon chain
Polar head group

ionic or polar head Micelle of detergent


(surfactant) molecules
non-polar (hydrocarbon) tail

Detergent (surfactant) molecule

CATALYSIS
Type of Catalysis

Catalysis

Homogenous Heterogenous
Catalysis Catalysis
Catalyst & Both the
reactants are catalyst & the
in the same reactants are in
phase. different phases.

Theories of Catalysis
 1. Intermediate Compound Formation theory.
 2. Adsorption Theory.
 3. The Modern Theory.
2 1 . 3 2 | Surface Chemistr y

Other Catalysis
Chem i str y | 21.33

Solved Examples

JEE Main/Boards Example 7: A catalyst:


(A) Alter the state of equilibrium
Example1: What are the physical states of dispersed
(B) Decreases the activation energy
phase and dispersion medium of froth?
(C) Increases collision frequency
Sol: Froth is a small mass of bubbles in liquid.
(D) Increases the average kinetic energy of reacting
Dispersed phase : Gas species.
Dispersion medium : Liquid
Sol: (B) Catalyst shows exothermic adsorption of reactant
Example 2: Define the term ‘Tyndall effect’. molecules and thus energy of activation is lowered.

Sol: The scattering of light by colloidal particles is x


Example 8: Plots of log   vs. log C showing a
known as Tyndall effect. m
straight line parallel to X-axis reveals that:
Example 3: What is meant by the term peptization?
1
(A) n = 0 (B) =0
Sol: The process of conversion of a freshly prepared n
precipitate into a colloidal solution by adding a suitable (C) C = 0 (D) log C = constant
electrolyte is called peptization.
x
Sol: (B) = K ·C1/n
m
Example 4: Explain the following terms with suitable
example (i) Alcohol (ii) Serosol and (iii) Hydrosol.
Example 9: Which characteristic is not correct for
Sol: (i) Alcohol: It is a colloidal dispersion having alcohol physical adsorption?
as the dispersion medium e.g. collodion (a colloidal sol (A) Adsorption is spontaneous
of cellulose nitre in ethyl alcohol.)
(B) Both enthalpy and entropy change of adsorption
(ii) Serosol: It is a colloidal dispersion of a liquid in a are negative
gas e.g. fog.
(C) Adsorption on solid is reversible
(iii) Hydrosol: It is a colloidal sol of a solid in water as
the dispersion medium e.g. starch sol or gold sol. (D) Adsorption increases with increase in temperature

Sol: (D) Physical adsorption is non-directional, reversible,


Example 5: The movement of sol particles under an
multilayers exothermic process where adsorbent are
applied electric field is called:
held by physical forces such as van der waal forces.
(A) Electro deposition (B) Electrodialysis
(C) Electro-osmosis (D) Electrophoresiss Example 10: The minimum energy level necessary to
permit a reaction to occur is:
Sol: (D) Colloidal Particles carry charge (positive or
(A) Internal energy (B) Threshold energy
negative) and thus on application of electric field they
move towards opposite electrodes and the phenomenon (C) Activation energy (D) Free energy
is known as cataphores is or electrophoresis.
Sol: (B) The minimum energy required to permit a
Example 6: At CMC, the surfactant molecules reaction to occur is called as Activation Energy.
undergoes:
(A) Association (B) Aggregation
(C) Micelle formation (D) All of these

Sol: (D) All are same processes.


2 1 . 3 4 | Surface Chemistr y

JEE Advanced/Boards A part of fat is thus used and rest fat is removed by the
soap so formed.
Example 1: A negatively charged yellow sol of As2S3
on mixing with a +vely charged Fe(OH) 3 red sol in Example 6: Which of the following is not correct about
equivalent amount give rise to colorless solution. Why? colloids?
(A) The [H+] at which colloids particles do not carry
Sol: Mutual coagulation leaves behind colorless Liquid
charge is called isoelectric point of colloid
which is water. The negative charge on As2S3 and positive
charge on Fe(OH) 3 neutralizes each other causes mutual (B) At isoelectric point of colloidal state possess
coagulation and thus dispersed particles of As2S3 and minimum stability
Fe(OH) 3 are settled down leaving water, which is colorless.
(C) At isoelectric point sol particles possess no
electrophoretic motion
Example 2: 0.1 M AlCl3 solution is more effective than
(D) Silicic acid sol have minimum stability at its
0.1 M NaCl solution in coagulating As2S3 sol while 0.1 M
isoelectric point.
AlCl3 is less effective than 0.1 M Na3PO4 solution in
coagulating ferric oxide
Sol: (D) It is an exception and the silicic acid sol possess
maximum stability.
Sol: As2S3 is a negatively charged sol and thus
effective ions are Al3+ and Na+ whereas Fe2O3 is a
positively charged sol and the effective ions and Cl¯ Example 7: Why a finely divided substance is more
and PO43–. More is the valence of effective ion, more is effective as an adsorbent?
its coagulating power. Thus, Al3+ is more effective than
Na+ and PO43- is more effective than Cl¯ Sol: Finely divided substances have very large specific
surface area and therefore provide more sites for
adsorption.
Example 3: Action of soap is due to emulsification and
micelle formation. Comment.
Example 8: Show that if a cube of 1 cm3 is broken into
Sol: Soap molecule has water soluble head and oil smaller cubes such that each side is divided just in the
soluble tail. The soap molecules get adsorbed on the middle. How many cubes will be formed? Also calculate
dirty (oily) surface through the tail and try to emulsify the total surface area of such cubes.
it. Soap micelles do solubilize the insoluble dirt/greasy
material. Sol: First calculate the surface area of one cube of edge
length of (0.5 cm) now by calculating the volume of one
cube find out the number of cubes formed by a cube
Example 4: The average molecular weight of colloidal
of 1 cm now by using volume and no of cubes calculate
particles is determined by:
the new surface area.
(A) Tyndall effect
Surface area of cube = 6a2
(B) Osmotic pressure measurement
a=1
(C) Victor Meyer’s method
Total surface area of cube of 1cm2 = 6 × 12 = 6 cm2
(D) None of the above
Total surface are of one cube of 1/2 cm edge
Sol: (B) The osmotic pressure measurements gives 1 1
average molecular weight of colloidal particles. =6× × =1.5
2 2
No of cubes x Volume of 1 cube of ½ edge length (1/2) 3cm
Example 5: Acid deposits on pans can be removed by = Volume of 1 cube of 1 cm edge length (1) 3
boiling pans with washing soda. Explain.
n x 0.125 = 1
Sol: The basic reaction behind this process is n = 1/0.125
saponification.
n=8
Na2CO3 (washing soda) on hydrolysis forms NaOH
which on boiling with fat give rise to saponification. Let each side be halved, than total number of cubes
1
RCOOR + NaOH → RCOONa + ROH formed of edge length of each cube 2 cm are 8.
Chem i str y | 21.35

1 1 Example 10: What role does adsorption play in


Thus surface area of 8 cubes = 8 × 6 × × = 12 cm2.
2 2 heterogeneous catalysis?

Example 9: Why it is advantageous to use a catalyst for Sol: Heterogeneous catalysis generally employs a solid
a reaction having endothermic nature? adsorbent and the reactants are mostly gases. The
reactions occurs at the surface of the catalyst where the
Sol: For an endothermic reaction (∆H = +ve) and thus reactant molecules (adsorbates) get chemisorbed. Due
heat is required to get better yield which raises the cost to adsorption, chances of effective collisions in reactant
appreciably. In presence of catalyst, less heat is required molecule increases.
to start a reaction.

JEE Main/Boards

Exercise 1 Q.16 Account for the following:


(i) Ferric hydroxide sol is positively charged.
Q.1 Why is ferric chloride preferred over Potassium
(ii) The extent of physical adsorption decreases with
chloride in case of a cut leading to Bleeding?
rise in temperature.
Q.2 What causes Brownian movement in a colloidal (iii) A delta is formed at the point where the river enters
solution? the sea.

Q.3 Which has a higher enthalpy of adsorption, Q.17 Explain the following terms:
physisorption or chemisorptions?
(i) Electro-dialysis
Q.4 Mention two ways by which lyophilic colloids can (ii) Phases of a colloidal solution
be coagulated.
Q.18 What is meant by ‘shape selective’ catalysis?
Q.5 What is an emulsion?
Q.19 What is adsorption? How does adsorption of a
Q.6 What is physical adsorption? gas on a solid surface very with (a) temperature and (b)
pressure? Illustrate with the help of appropriate graphs.
Q.7 What is meant by chemical adsorption?
Q.20 Explain what is observed when
Q.8 What is desorption?
(i) An electrolyte is added to ferric hydroxide Sol
Q.9 How is adsorption of a gas related to its critical (ii) An emulsion is subjected to centrifugation.
temperature?
(iii) Direct current is passed through a colloidal Sol
Q.10 Why is a colloidal sol stable?
Q.21 How do size of particles of adsorbent, pressure of
Q.11 Write two difference between sols and emulsions. gas and prevailing temperature influence the extent of
adsorption of a gas on a solid?
Q.12 Define ultrafiltration?
Q.22 (a) In which of the following does adsorption take
Q.13 Why do colloidal solutions exhibit Tyndall effect? place and why?
(i) Silica gel placed in the atmosphere saturated with water.
Q.14 Why artificial rain can be caused by throwing
common salt on the clouds? (ii) Anhydrous CaCl2, placed in the atmosphere saturated
with water.
Q.15 What happens when an electric field is applied to
(b) How does BF3 act as a catalyst in industrial process?
a colloidal dispersion?
Give an example of shape-selective catalysis.
2 1 . 3 6 | Surface Chemistr y

Q.23 (i) What are micelles? How do they differ from Q.2 Which of the following factors affects the adsorption
ordinary colloidal particles? Give two examples of of a gas on solid?
micelles forming substances.
(A) Tc (critical temp.) (B) Temperature of gas
(ii) State Hardy-Schulze rule.
(C) Pressure of gas (D) All of them
Q.24 Describe the following types of colloids, giving an
example for each: Q.3 The volume of gases NH3, CO2 and CH4 adsorbed
by one gram of charcoal at 298 K are in
(i) Multimolecular colloids
(A) CH4 > CO2 > NH3 (B) NH3 > CH4 > CO2
(ii) Macromolecular colloids
(C) NH3 > CO2 > CH4 (D) CO2 > NH3 > CH4
Q.25 Of physisorption and chemisorption which type
of adsorption has a higher enthalpy of adsorption? Q.4 Platinum is not used as a catalyst in the
Q.26 Explain what is observed when (A) Oxidation of CH3OH to HCHO
(i) KCl, an electrolyte, is added to hydrated ferric oxide sol (B) Oxidation of SO2 to SO3
(ii) An electric current is passed through a colloidal (C) Combination of H2 and I2 to form HI
solution. (D) Synthesis of NH3 from N2
(iii) A beam of light is passed through a colloidal
solution. Q.5 Which type of metals form effective catalysts?
Q.27 Describe the following: (A) Alkali metals
(i) Tyndall effect (B) Transition metals
(ii) Shape-selective catalysis (C) Alkaline earth metals
(D) Radioactive metals
Q.28 Coagulations of lyophobic sols can be carried out
by?
Q.6 The heat of physisorption lie in the range of
Q.29 Describe a conspicuous change observed when (A) 1 – 10 kJ mol–1 (B) 20 to 40 kJ mol–1
(i) A solution of NaCl is added to a sol of hydrated ferric (C) 40 to 200 kJ mol–1 (D) 200 to 400 kJ mol–1
oxide.
(ii) A beam of light is passed through a solution of NaCl Q.7 Adsorption is multilayer in case of
and then through a sol
(A) Physical adsorption (B) Chemisorptions
Q.30 What is meant by coagulation of a colloidal (C) In both (D) None of these
solution? Describe briefly any three methods by which
coagulation of lyophobic sols can be carried out.
Q.8 Reversible adsorption is
(A) Chemical adsorption
Exercise 2 (B) Physical adsorption
Single Correct Choice Type (C) Both (A) and (B)

Q.1 Which gas will be adsorbed on a solid to greater (D) None of these
extent?
(A) A gas having non polar molecule Q.9 Which of the following is not a gel?

(B) A gas having highest critical temperature (A) Cheese (B) Jellies

(C) A gas having lowest critical temperature (C) Curd (D) Milk

(D) A gas having highest critical pressure


Chem i str y | 21.37

Q.10 The amount of gas adsorbed physically on Q.16 The Tyndall effect associated with colloidal
charcoal increases with increase of particles is due to
(A) Temperature and pressure (A) Presence of electrical charges
(B) Temperature and decreases of pressure (B) Scattering of light
(C) Pressure and decreases of temperature (C) Absorption of light
(D) None of these (D) Reflection of light

Q.11 An emulsion is a colloidal system of Q.17 Which one of the following is not applicable to
chemisorptions?
(A) Two solids
(A) Its heat of adsorption is high
(B) Two liquids
(B) It takes place at high temperature
(C) One gas and one solid
(C) It is reversible
(D) One gas and one liquid
(D) It forms mono-molecular layers
Q.12 Which of the following is a lyophobic colloid?
Q.18 In the colloidal state the particle size ranges
(A) Gelatin (B) Sulphur
(A) Below 1 nm
(C) Starch (D) Gum
(B) Between 1 m to 100 nm
Q.13 The nature of bonding forces in adsorption (C) More than 100 nm
(A) Purely physical such as Van Dar Waal’s forces (D) None of the above
(B) Purely chemical
Q.19 All colloids
(C) Both chemical and physical always
(A) Are suspensions of one phase in another
(D) None of these
(B) Are two-phase systems
Q.14 Which can adsorb larger volume of hydrogen gas? (C) Contain only water-soluble particles
(A) Colloidal solution of palladium (D) Are true solutions
(B) Finely divided nickel
Q.20 Which is an example of auto-catalysis?
(C) Finely divided platinum
(A) Decomposition of KCIO3 + MnO2 mixture
(D) Colloidal Fe(OH)3
(B) The decomposition of nitroglycerine
Q.15 Which statement is correct? (C) Breakdown of 6C14
(A) A catalyst increases the rate of a reaction by (D) Hydrogenation of vegetable oils using catalyst
decreasing the rate of backward reaction
(B) The reaction is fast if the activation energy of a Q.21 Colloids can be purified by
reaction is low
(A) Condensation (B) Peptization
(C) The activation energy of a forward reaction can
(C) Coagulation (D) Dialysis
never be smaller than that of the back ward reaction
(D) Reaction rate increases with temperature because Q.22 Milk is an example of
the activation energy decreases at high temperature
(A) Emulsion (B) Suspension
(C) Foam (D) Sol
2 1 . 3 8 | Surface Chemistr y

Q.23 Colloidal particles in a sol can be coagulated by Q.30 The minimum concentration of an electrolyte
required to cause coagulation of a sol is called
(A) Heating
(A) Flocculation value (B) Gold number
(B) Adding an electrolyte
(C) Protective value (D) None of these
(C) Adding oppositely charged sol
(D) Any of the above methods
Q.31 Smoke precipitator works on the principle of
(A) Distribution law
Q.24 Enzymes are
(B) Neutralization of charge on colloids
(A) Substances made by chemists to activate washing
powder (C) Le-Chaterlier’s principle
(B) Very active vegetable catalysts (D) Addition of electrolytes
(C) Catalysts found in organisms
Q.32 The disperse phase in colloidal iron (III) hydroxide
(D) Synthetic catalysts
and colloidal gold is positively and negatively charged
respectively. Which of the following is not correct?
Q.25 Fog is a colloidal system of
(A) Magnesium chloride solution coagulates the gold
(A) Gas in liquid (B) Liquid in gas more readily than iron (III) hydroxide Sol
(C) Gas in gas (D) Gas in solid (B) Sodium sulphate solution cause coagulation in both
sols
Q.26 Given below are a few electrolytes, indicate which (C) Mixing of the sols has no effect
one among them will bring about the coagulation of a
gold sol quickest and in the least of molar concentration? (D) Coagulation in both sols can be brought about by
electrophoresis.
(A) NaCl (B) MgSO4
(C) Al2(SO4)3 (D) K4[Fe(CN)6]
Previous Years’ Questions
Q.27 When a lyophobic colloidal solution is observed,
Q.1 Which of the following statements is incorrect
we can see
regarding physisorption  (2009)
(A) Light scattered by colloidal particle
(A) It occur because of Van der Waals forces
(B) Size of the colloidal particle
(B) More easily liquefiable gases are adsorbed readily
(C) Shape of the colloidal particle
(C) Under high pressure it results into multi molecular
(D) Relative size of the colloidal particle layer on adsorbent surface.
(D) Enthalpy of adsorption (∆H-adsorption) is known
Q.28 The process which is catalysed by one of the and positive
products formed during the reaction is known
(A) Auto-catalysis (B) Anti-catalysis Q.2 Which one of the following characteristics is not
(C) Negative catalysis (D) Acid catalysis correct for physical adsorption  (2003)
(A) Adsorption on solid is reversible
Q.29 Colloidal solutions are classified on the basis of (B) Adsorption increases with increase in temperature
(A) Molecular size (C) Adsorption is spontaneous
(B) Organic or inorganic (D) Both enthalpy and entropy of adsorption are
(C) Surface tension value negative

(D) pH value
Chem i str y | 21.39

Q.3 The equation for Freundlich adsorption isotherm is Q.7 Gold numbers of protective colloids A, B, C and D
(2012) are 0.50, 0.01, 0.10 and 0.005, respectively. The correct
x order of their protective powers is  (2008)
(A) = kp1/n (B) x = mkp1/n
m (A) D < A < C < B (B) C < B < D < A
x (C) A < C < B < D (D) B < D < A < C
(C) = kp−n (D) All of these
m

Q.4 Which one of the following is not a colloid? (1992) Q.8 Which of the following statements is incorrect
regarding physisorption?  (2009)
(A) Milk (B) Blood
(A) It occurs because of Vander Waal’s forces.
(C) Solution of urea (D) Ice cream
(B) More easily liquefiable gases are adsorbed readily.

Q.5 Choose the correct reason(s) for the stability of the (C) Under high pressure it results into multi molecular
lyophobic colloidal particles (2012) layer on adsorbent surface.

(A) Preferential adsorption of


(I) ions of their surface from
(D) Enthalpy of adsorption
positive.
( ∆Hadsorption ) is low and
the solution
Amount of gas adsorbed

(B) Preferential adsorption of solvent on their surface


P constant
Q.9 According to Freundlich adsorption isotherm,
from the solution
which of the following is correct?  (2012)
(C) Attraction between different particles having
x
opposite charge on their surface (A) ∝ P0
m
(D) Potential difference between the fixed layer and the
x
diffused layer of opposite charges around
T
the colloidal (B) ∝ P1
particles
(II) m
x
Amount of gas adsorbed

P constant
(C) ∝ P1/n
Q.6 The given graphs/data I, II, III and IV represent m
general trends observed for different physisorption
(D) All the above are correct for different ranges of
and chemisorptions processes under mild conditions
pressure
of temperature and pressure. Which of the following
choice(s) about I, II, III and IV is (are) correct. (2012)
T
Q.10 The coagulating power of electrolytes having ions
(I)
Na+ , Al3+ and Ba for arsenic sulphide sol increases in
2+
(III)
the order: 
Amount of gas adsorbed

(2013)
Amount of gas adsorbed

P constant

200K (A) Al3+ < Ba2+ < Na+ (B) Na+ < Ba2+ < Al3+
250K
(C) Ba2+ < Na+ < Al3+ (D) Al3+ < Na+ < Ba2+

Q.11 3 g of activated charocoal was added to 50 mL of


T acetic acid solution (0.06N) in a flask. After an hour it
P
(II)
(IV)
was filtered and the strength of the fitrate was found
to be 0.042 N. The amount of acetic acid adsorbed (per
Amount of gas adsorbed

P constant
gram of charcoal) is:  (2015)
Potential Energy

(A) 18 mg (B) 36 mg (C) 42 mg (D) 54 mg


Eads
O
Distance of molecule
from the surface
Hads= 150 k/mol
-1 Q.12 For a linear plot of log(x/m) versus log p in a
Freundlich adsorption isotherm, which of the following
T statements is correct? (k and n are constants)  (2016)
(III)
(A) I is Physisorption and II is chemisorption (A) 1/n appears as the intercept
Amount of gas adsorbed

(B) I is Physisorption200K
and III is chemisorption (B) Only 1/n appears as the slope.

(C) IV is chemisorption
250K
and II is chemisorption (C) log (1/n) appears as the intercept.

(D) IV is chemisorption and III is chemisorption (D) Both k and 1/n appear in the slope term.

P
2 1 . 4 0 | Surface Chemistr y

JEE Advanced/Boards

Exercise 1 x x
(C) = KP at low pressure and = K at high pressure
m m
Q.1 Give reason why a finely divided substance is more
(D) All of these
effective as an adsorbent.

Q.2 What are enzymes? Write in brief the mechanism Q.2 Emulsifier is an agent which
of enzyme catalysis.
(A) Accelerates the dispersion
Q.3 What are emulsion? What are their different types? (B) Homogenizes an emulsion
Give example of each type.
(C) Stabilizes an emulsion
Q.4 Why are substances like platinum and palladium (D) Aids the flocculation of an emulsion
often used for carrying out electrolysis of aqueous
solutions? Q.3 The process which is catalysed by one of the
products formed during the reaction is known
Q.5 Why are powdered substance more effective
adsorbent that their crystalline forms? (A) Auto-catalysis (B) Anti-catalysis
(C) Negative catalysis (D) Acid catalysis
Q.6 Why is it necessary to remove CO when ammonia
is obtained by Haber’s process?
Q.4 The electrical charge on a colloidal particle is
Q.7 What is the role of desorption in the process of indicated by
catalysis? (A) Brownian movement (B) Electrophoresis
Q.8 Why is ferric chloride preferred over potatassium (C) Ultra microscope (D) Molecular sieves
chloride in case of a cut leading to bleeding?
Q.5 Among the following, the surfactant that which will
Q.9 What is the ‘coagulation’ process? from micelles in aqueous solution at the lower molar
concentration at ambient conditions is
Q.10 Which will be adsorbed more readily on the
surface of charcoal and why-NH3 or CO2? (A) CH3(CH2) 15N+(CH3) 3Br¯
(B) CH3(CH2) 11OSO3¯Na+
Q.11 How is adsorption of a gas related to its critical
temperature? (C) CH3(CH2) 6COO¯Na+
(D) CH3(CH2) 11N+(CH3) 3Br¯
Q.12 What does you mean by activity of catalysis?

Q.13 What happens when gelatin is added to gold sol? Comprehension Type
Paragraph 1: In macromolecular type of colloids, the
dispersed particles are themselves large molecules
Exercise 2
(usually polymers) . Since these molecules have
Single Correct Choice Type dimensions comparable to those of colloidal particles,
their dispersions are called macromolecular colloids.
Q.1 Which equation represents Freundlich adsorption Most lyophilic sols belong to this category. There
isotherm (physical adsorption is basis of this theory)? are certain colloids which behave as normal strong
x electrolytes at low concentrations, but exhibit colloidal
(A) = K(P)1/n Where x is amount of gas adsorbed on properties at higher concentrations due to the
m
formation of aggregated particles. These are known as
mass ‘m’ at pressure P
micelles or associated colloids. Surface active agents
x 1 like soaps and synthetic detergents belong to this class.
(B) log
= logK + logP
m n
Chem i str y | 21.41

Critical micelle concentration (CMC) is the lowest Q.9 Cleansing action of soap occurs because:
concentration at which micelle formation appear. CMC
(A) Oil and grease can be absorbed into the hydrophobic
increases with the total surfactant concentrations. At
centers of soap micelles and washed away
concentration higher than CMC, they from extended
parallel sheets known as lamellar micelles which (B) Oil and grease can be absorbed into hydrophilic
resemble biological membranes. With two molecules centers of soap micelles acid washed away
thick, the individual molecule is perpendicular to the (C) Oil and grease can be absorbed into both hydrophilic
sheets such that hydrophilic groups are on the outside and hydrophobic centers but not washed away
in aqueous solution and on the inside is a non-polar
(D) Cleansing action is not related to micelles
medium.
In concentrated solutions, micelles take the form of Paragraph 2: The protective power of the lyophilic
long cylinders packed in hexagonal arrays and are colloids is expressed in terms of gold number a term
called lytotropic mesomorphs. introduced by Zsigmondy. Gold number is the number
In an aqueous solution (polar medium), the polar group of milligram of the protective colloid which prevent the
points towards the periphery and the hydrophobic coagulation of 10 mL of red gold sol. When 1 mL of a 10
hydrocarbon chain point towards the center forming percent solution of sodium chloride is added to it. Thus,
the core of the micelle. smaller the gold number of lyophilic colloid, the greater
is the protective power.
Micelles from the ionic surfactants can be formed
only above a certain temperature called the Kraft Q.10 On addition of one mL of solution of 10% NaCl to
temperature. They are capable of forming ions. 10 mL of red gold sol in presence of 0.025 g of starch,
Molecules of soaps and detergents consist of lyophilic the coagulation is just prevented. The gold number of
as well as lyophilic parts which associate together to starch is
form micelles. Micelles may contain as many as 100
molecules or more. (A) 0.025 (B) 0.25 (C) 2.5 (D) 25

Q.11 Which of the following statement(s) is/are correct


Q.6 Select incorrect statement(s):
(A) Higher the gold number, more protective power of
(A) Surface active agent like soaps and synthetic
colloid
detergents are micelles
(B) Lower the gold number, more the protective power
(B) Soaps are emulsifying agents
(C) Higher the coagulation value, more the coagulation
(C) C17H35 (hydrocarbon part) and –COO– (carboxylate) power
part of stearate ion (C17H35COO–) both are hydrophobic
(D) Lower the coagulation value, higher the coagulation
(D) All are incorrect statements power

Q.7 Which part of the soap (RCOO–) dissolves grease Q.12 Gold number gives an indication of
and forms micelle?
(A) Protective nature of colloids
(A) R part (called tail of the anion)
(B) Purity of gold in suspension
(B) – COO – part (called head of the anion)
(C) The charge on a colloidal solution of gold
(C) Both (A) and (B)
(D) g-mole of gold per liter
(D) None of these

Paragraph 3: Whenever a mixture of gases is allowed


Q.8 In multimolecular colloidal sols, atoms or molecules to come in contact with a particular adsorbent under
are held together by: the same conditions, the stronger adsorbate is
(A) H-bonding (B) Van der Waals forces adsorbed to greater extent irrespective of its amount
present, e.g. H2O is adsorbed to more extent on silica
(C) Ionic bonding (D) Polar covalent boding gel than N2 and O2. This shows that some adsorbates
are preferentially adsorbed. It is also observed that
preferentially adsorbable adsorbents can displace
a weakly adsorbed substance from the surface of an
adsorbent.
2 1 . 4 2 | Surface Chemistr y

Q.13 Which of the following gases is adsorbed to Previous Years' Questions


maximum extent:
(A) He (B) Ne (C) Ar (D) Xe Q.1 When the temperature is increased, surface tension
of water  (2002)
Q.14 Which of the gas can displace remaining all the (A) Increases (B) Decreases
gases.
(C) Remains constant (D) Shows irregular behavior
(A) O2 (B) N2 (C) CO (D) H2
Q.2 Rate of physisorption increases with (2003)
Q.15 When temperature is increased (A) Decrease in temperature
(A) Extent of adsorption increases (B) Increase in temperature
(B) Extent of adsorption decreases (C) Decrease in pressure
(C) No effect on adsorption (D) Decrease in surface area
(D) Extent of adsorption first decreases, then increases
Q.3 Spontaneous adsorption of a gas on solid surface
Assertion Reasoning Type is an exothermic process because (2004)

These questions consist of two statements each, (A) ∆H increases for system (B) ∆S increases for gas
printed as assertion and reason, while answering these (C) ∆S decreases for gas (D) ∆G increases for gas
questions you are required to choose any one of the
following responses. Q.4 As gas is adsorbed on surface of solid, entropy
(A) If assertion is true but the reason is false decreases, i.e. ∆S < 0. Therefore, for ∆G < 0, ∆H must
be negative. Lyophilic sols are (2005)
(B) If assertion is false but the reason is true
(A) Irreversible sols
(C) If both assertion and reason are true and the reason
(B) Prepared from inorganic compounds
is a correct explanation of assertion
(C) Coagulated adding electrolytes
(D) If both assertion and reason are true but reason is
not a correct explanation of assertion. (D) Self-stabilizing

Q16 Assertion: For adsorption ∆G, ∆H, ∆S all have –ve Q.5 Among the following, the surfactant that will
values form micelles in aqueous solution at the lowest molar
concentration at ambient condition, is (2008)
Reason: Adsorption is a spontaneous exothermic
process in which randomness decreases due to force of (A) CH3(CH2) 15N+(CH3) 3Br¯ (B) CH3(CH2) 11OSO3¯N+
attraction between adsorbent and adsorbate. (C) CH3(CH2) 6COO¯N+ (D) CH3(CH2) 11N+(CH3) 3Br¯

Q.17 Assertion: A gas with higher critical temperature Q.6 Among the electrolytes Na2SO4, CaCl2, Al2(SO4)3
gets adsorbed to more extent than a gas with lower and NH4Cl, and NH4Cl, the most effective coagulating
critical temperature. agent for Sb2S3 sol is (2009)
Reason: The easily liquefiable gases get adsorbed to (A) Na2SO4 (B) CaCl2 (C) Al2(SO4)3 (D) NH4Cl
more extent.
Q.7 The correct statement(s) pertaining to the
Q.18 Assertion: The micelle formed by sodium stearate adsorption of a gas on a solid surface is (are) (2011)
in water has – COO– groups at the surface.
(A) Adsorption is always exothermic
Reason: Surface tension of water is reduced by the
(B) Physisorption many transform into chemisorption at
addition of stearate.
high temperature
(C) Physisorption increases with increasing temperature
Q.19 Assertion: Isoelectric point is pH at which colloidal
but chemisorption decreases with increasing temperature
can move towards either of electrode.
(D) Chemisorption is more exothermic than
Reason: At isoelectric point, colloidal particles become physisorption, however it is very slow due to higher
electrically neutral. energy of activation.
Chem i str y | 21.43

Assertion and Reasoning Type Q.10 When O2 is adsorbed on a metallic surface,


electron transfer occurs from the metal to O2. The true
Read the following questions and answer as per the
statement(s) regarding this adsorption is(are) (2015)
direction given below:
(A) O2 is physisorbed
(A) Statement-I is true: statement-II is a correct
explanation of statement-I. (B) Heat is released
*
(B) Statement-I is true; statement-II is true; (C) Occupancy of k π2p of O2 is increased
statement-II is not the correct explanation of statement-I (D) Bond length of O2 is increased
(C) Statement-I is true; statement-II is false.
(D) Statement-I is false, statement-II is true Q.11 Choose the correct reason(s) for the stability of
the lyophobic colloidal particles.  (2012)
Q.8 Statement-I: Micelles are formed by surfactant (A) Preferential adsorption of ions on their surface from
molecules above the critical micelle concentration (CMC). the solution
Statement-II: The conductivity of a solution having (B) Preferential adsorption of solvent on their surface
surfactant molecules decreases sharply at the CMC. (2007) from the solution
(C) Attraction between different particles having
Q.9 Methylene blue, from its aqueous solution, is opposite charges on their surface
adsorbed on activated charcoal at 250C. For this
(D) Potential difference between the fixed layer and the
process, the correct statement is  (2013)
diffused layer of opposite charges around the colloidal
(A) The adsorption requires activation at 25oC. particles
(B) The adsorption is accompanied by a decrease in
enthalpy.
(C) The adsorption increases with increase of
temperature.
(D) The adsorption is irreversible

PlancEssential Questions
JEE Main/Boards JEE Advanced/Boards

Exercise 1 Exercise 1
Q.3 Q.9 Q.13 Q.17 Q.2 Q.4 Q.13
Q.19 Q.24 Q.31
Exercise 2
Exercise 2 Q.1 Q.5 Q.7 Q.10-12
Q.3 Q.12 Q.15 Q.26
Q.31 Previous Years' Questions
Q.2 Q.6 Q.19
Previous Years' Questions
Q.6
2 1 . 4 4 | Surface Chemistr y

Answer Key

JEE Main/Boards
Exercise 2
Single Correct Choice Type

Q.1 B Q.2 D Q.3 C Q.4 D Q.5 B Q.6 B

Q.7 A Q.8 B Q.9 D Q.10 C Q.11 B Q.12 B

Q.13 B Q.14 A Q.15 B Q.16 B Q.17 C Q.18 B

Q.19 B Q.20 B Q.21 D Q.22 A Q.23 D Q.24 C

Q.25 B Q.26 C Q.27 A Q.28 A Q.29 A Q.30 A

Q.31 B Q.32 C

Previous Years' Querstions


Q.1 D Q.2 B Q.3 D Q.4 C Q.5 A, D Q.6 A, C

Q.7 C Q.8 D Q.9 D Q.10 B Q.11A Q.12 B

JEE Advanced/Boards

Exercise 2
Single Correct Choice Type

Q.1 D Q.2 C Q.3 A Q.4 B Q.5 A

Comprehension Type

Q.6 A, D Q.7 A Q.8 B Q.9 A Q.10 D

Q.11 B, D Q.12 A Q.13 D Q.14 C Q.15 B

Assertion Reasoning Type

Q.16 C Q.17 C Q.18 A Q.19 B

Previous Years’ Questions


Q.1 B Q.2 A Q.3 C Q.4 D Q.5 A Q.6 C

Q.7 A,B,D Q.8 B Q.9 B Q.10 B,C,D Q.11 A, D


Chem i str y | 21.45

Solutions

JEE Main/Boards Sol 12: Ultra filtration is a type of filtration in which


purification is carried out through special filters or
Exercise 1 semipermeable membranes. Generally, they allow
passage of electrolyte but not colloid particles.

Sol 1: As the charge of Fe3+ is high, it’s coagulation Sol 13: The size of particles is greater than the
power will be high. So, it can stop bleeding. wavelength of visible light. So, colloid particles scatter
light which is Tyndall effect.
Sol 2: The Brownian movement is due to the collision
between the particles of colloid & the medium of the Sol 14: Clouds are colloids of water droplet in air.
solution. Adding salt coagulates the water droplets. So they form
bigger drops. So, it comes down as rain.
Sol 3: Chemisorption because it involves breaking &
forming of bonds. Physisorption just depends on weak Sol 15: The oppositely charged go to separate
Vander Waals forces. electrodes and get coagulated there.

Sol 4: We can coagulate lyophilic colloids by Sol 16: (i) Fe+3 is the preferential adsorbate. So, it’s
positively charged Sol
•• adding on electrolyte. (it reacts with colloids)
(ii) Physical adsorption decreases with increase in T due
•• adding a suitable solvent.
to increase in the energy of particles, they escape Van
Sol 5: Emulsion is a mixture of two or more liquids der Waals forces.
which are normally immiscible. Ex. Milk. (iii) It’s due to possible formation of colloid at the point
It’s a colloid of a liquid in another liquid. of mixing.

Sol 6: It’s a kind of adsorption in which adsorbate is Sol 17: (i) Electro-dialysis : It’s the process of transporting
held by weak Vander Waal’s forces to the medium salt ions from one solution to other through ion-
(adsorbent) . exchange membrane under a potential difference.
(ii) (a) Dispersal phase : - It’s the component present in
Sol 7: If there is bond formation (chemical reaction) small proportions and is just like a solute in a solution.
between adsorbate & the medium, then it’s chemical It’s also called the ‘Internal phase’.
adsorption.
(b) Dispersion medium : - It’s generally the component
Sol 8: Desorption is the reverse of adsorption. It’s a present in excess and is just like a solvent in a solution.
process of removing an adsorption from some medium. It’s also called the ‘External phase’.

Sol 9: If TC of a gas is high, then it can be liquified easily, Sol 18: Shape selectivity of a catalyst depends upon
so greater is the extent of adsorption (because of high the pore structure of the catalyst.
Van der Waals force) . If TC is low, the given gas is a bad Depending on the size of the reactant and product
adsorbate. molecules when compared to the size of cages or pores
of the zeolite, reactions proceed in a specific manner.
Sol 10: Colloidal sol is stable because of the like
charged particles. So, they do not form large particles. Sol 19:
Sol is solids dispersed in liquid.
o
Amount -78 C
Sol 11: (i) Sols are solids in liquids, where as emulsions
o
-29 C
are liquids in liquids. o
0C
(ii) Sols are more stable compared to emulsions.
P
2 1 . 4 6 | Surface Chemistr y

Amount
(ii) Hardy-Schulze Rule – Greater is the valency of the
coagulating ion, greater is its power to bring coagulation.
And the effective ions of the electrolyte in bringing
Constant P about coagulation are those which carry opposite
charge to that of colloid particles.
T

Sol 24: (i) Multimolecular colloids: - They are formed by


Sol 20: (i) It leads to coagulation as oppositely charged aggregates of a large number of atoms or molecules.
electrolyte particles get attracted to the sol particles. E.g. Sols of Gold, Sulphur etc.
(ii) It leads to separation of dispersed phase & dispersion (ii) Macromolecular colloids: - They are molecules of
medium. very large size. E.g. Polymers like rubber, starch etc.
(iii) Colloid particles will be accumulated at electrodes.
Sol 25: Chemisorption have higher activation energy
Sol 21: In physisorption, the size is an important factor and enthalpy of adsorption.
because it determines Van der Waals forces.
x/m Sol 26: (i) The sol coagulates due to addition of
electrolyte.
(ii) The positive and negative sol particles deposit at
respective electrodes.
P (iii) We observe Tyndall effect i.e. light gets scattered.

x/m Sol 27: (i) Tyndall effect: - Tyndall scattering, is light


scattering by particles in a colloids or particles is a fine
suspension. Individual particles of colloid reflect and
scatter light of wavelength (in same order as particle sizes).
T (ii) A catalytic reaction which depends upon the active
site and pore structure of catalyst and not on the size of
Physical
reactant and product is known as shape selective catalysis.
x/m

Sol 28: Coagulation of hydrophobic ions can be done


by using method of neutralising ions.

T
Sol 29: (i) It will lead to coagulation as NaCl acts as
Chemical
electrolyte. and FeCl3 comes as precipitate.
(ii) If beam of light is passed through NaCl solution. The
Sol 22: (a) (i) Silica gel – adsorption
intensity decreases as NaCl (aq) is a true solution.
(ii) Anhydrous CaCl2 – Absorption
It does not show Tyndall effect.
(b) Due to the presence of empty orbital in B.
But when light is passed through sol, we observe
Ba, Mg, B, Ga, P in zeolites.
scattering (Tyndall effect).

Sol 23: (i) A micelle is an aggregate of surfactant


Sol 30: Coagulation is the process which involves
molecules dispersed in a liquid colloid. The hydrophobic
precipitation of a colloidal solution by addition of
end dissolves in the grease where as hydrophilic end
excess of electrolyte.
dissolves in H2O.
It can be caused by
Grease
•• Electrophoresis
•• Prolonged dialysis

Hydrophilic
•• Addition of two sols (Mutual precipitation)
Hydrophobic
ends ends
Chem i str y | 21.47

Exercise 2 Sol 14: (A) Colloidal solution of Pd because Pd has


highly affinity towards H and colloid solution will have
Single Correct Choice Type higher Pd surface.

Sol 1: (B) A gas with high critical temperature (TC) can Sol 15: (B) The reaction is fast, if activation energy is
be liquefied easily. So, it’s adsorbed on a solid to a low. Catalyst reduces activation energy.
greater extent.
Activation energy need not be inversely proportion to
Sol 2: (D) Adsorption of a gas depends on temperature.

•• TC of gas
Sol 16: (B) Scattering of light.
•• Temperature
•• Pressure Sol 17: (C) Chemisorption is not reversible. Other
statements are true about chemisorption.
Sol 3: (C) NH3 > CO2 > CH4 (ease of liquefaction)
Sol 18: (B) In colloids, size of particles is nearly
It’s because of their TC.
wavelength of visible light. i.e. 1 nm – 100 nm range.
TC > TC > TC
NH3 CO2 CH4

Sol 19: (B) Colloids are 2 phase system. They are


Sol 4: (D) Pt is not use in synthesis of NH3. N2, H2 are different from suspensions.
not easily liquifiable.
Sol 20: (B) Decomposition of nitro glycerine is auto-
Sol 5: (B) Transition metals form effective catalyst due catalysis. So it’s used in explosive, TNT.
to high inter-atomic spaces.
Sol 21: (D) Colloids can be purified by dialysis.
Sol 6: (B) The heat of physisorption is quite low. It’s in
the range of 20-40 kJ/mole. Sol 22: (A) Milk–Emulsion. (Oil in water)

Sol 7: (A) Physical adsorption can be multi-layered Sol 23: (D) Adding oppositely charged Sol. Heating
because of Van der Waals forces. and adding an electrolyte can coagulate sols.

Sol 8: (B) Physical adsorption is reversible but not Sol 24: (C) Enzymes are naturally occurring catalysts.
chemical. It’s because chemisorption has high activation (Present in organisms) .
energy.
Sol 25: (B) Fog – Liquid in gas (water vapour in
Sol 9: (D) Gels – Solid in liquid ( jelly type). Milk is an atmosphere)
emulsion, rest all are gels.
Sol 26: (C) Gold sol in negative. So Al3+ will be the best
Sol 10: (C) Increases with P, decreases with T. coagulating agent.

Sol 11: (B) Emulsion is a colloidal system of 2 liquids. Sol 27: (A) We can observe scattering of light due to
larger size of particles.
Sol 12: (B) Sulphur does not dissolve in water. It’s a
lyophobic colloid. Sol 28: (A) Auto-catalysis as we need very less amount
of catalyst but the reaction goes quickly.
Sol 13: (B) Physical – Van der Waals. Chemical –
Chemical bonds. Sol 29: (A) Colloids are classified on the basis of their
particle size.
2 1 . 4 8 | Surface Chemistr y

Sol 30: (A) Flocculation value is the minimum Sol 7: (C) Higher the gold number lesser will be the
concentration of an electrolyte required to cause protective power of colloid.
coagulation.
Sol 8: (D) Enthalpy of adsorption regarding physisorption
Sol 31: (B) Smoke precipitation works on neutralisation is not positive and it is negative.
of charge on colloids. It uses the force of an induced
electrostatic charges. Sol 9: (D)
x
∝ P1 ;
x
∝ P1/n are true at low and
m m
Sol 32: (C) Mixing of sols will lead to coagulation. moderate pressures.
Electrophoresis can coagulate both.
MgCl2 will coagulate gold sol quickly because Mg2+ Sol 10: (B) According to Hardy Schulze rule, greater the
has higher charge than Cl¯. Mg+2 coagulates negatively charge on cation, greater is its coagulating power for
charged Sol negatively charged solution. So, order of coagulating
power: Na+ < Ba2+ < Al3+ .

Previous Years’ Questions Sol 11: (A)


Initial m moles of CH3COOH
= 0.06 × 50
Sol 1: (D) Since adsorption is exothermic process so
∆H of adsorption is always negative. Final m moles of CH3COOH
= 0.042 × 50

Hence, mass of CH3COOH adsorbed per gram of


Sol 2: (B) Adsorption increases with increase in
temperature = charcoal
( 0.06 − 0.042 ) × 50 × 10−3 × 60 × 103
= 18 mg
3
x
Sol 3: (D) = kp or x = m.kp
1/n 1/n
m Sol 12: (B) According to the Freundlich adsorption
or x / m = kp -n isotherm
All of these equation represent Freundlich adsorption x
= kP1/n
isotherm. m
x 1
log= log K + log P
m n
Sol 4: (C) Solution of urea is not a colloid.

Sol 5: (A, D)
(A) Due to preferential adsorption of common ion JEE Advanced/Boards
(B) Incorrect
(C) Incorrect (due to repulsion, not due to attraction) Exercise 1
(D) The layer of oppositely charged particles around any
Sol 1: Finely divided substance has more surface area
colloidal particles will decrease the potential energy of
than normal crystals. So, adsorption will be more
system.
effective.
Sol 6: (A, C) In Physisorption, on increasing temperature
at constant pressure, adsorption decreases while in Sol 2: Enzyme:-They are complex organic compounds
chemical adsorption on increasing temperature due present in living beings inside the human body (or
to requirement of activation energy adsorption will other organisms) and act as catalyst in many processes.
increases at same pressure. So, I is physisorption while They are also called Bio-Catalyst. Its mechanism is
II is chemisorption. similar to other catalysis. By providing an alternative
III is physical adsorption as on increasing temperature, reaction route, the enzyme reduces the energy required
extent of adsorption is decreasing. IV is representing to reach the highest energy transition state of the
enthalpy charge (which is high) during chemical reaction.
adsorption (due to bond formation) So, is valid for
chemical adsorption. So, answer is (A) and (C).
Chem i str y | 21.49

Sol 3: Emulsion are colloids of liquid in liquid. Exercise 2


•• Oil in water type – Ex. Milk
Single Correct Choice Type
•• Water in oil type – Ex. Butter.
x 1
x 1
Sol 1: (D) = k (P ) n ⇒ log   = log P + log K.
Sol 4: There are 2 main reasons: m m n
•• They are chemically inert. At high P
x
•• They are good catalyst due to high inter atomic ≈K
spaces. Their affinity towards hydrogen is very high. m
At low P
Sol 5: Powdered substances have very high surface x
area compared to crystals. So, adsorption on powdered ≈ KP'
m
substance is effective.
Sol 2: (C) Emulsifier is a substance used to stabilise
Sol 6: CO acts as poison in Haber’s process. Poison is a emulsions.
negative catalyst i.e. it reduces the rate of reaction.
Sol 3: (A) Auto-catalysis as we need very less amount
Sol 7: The products after reaction must be desorbed of catalyst but the reaction goes quickly.
from the catalyst so that further reaction can take place.
Sol 4: (B) Electrophoresis depends on electric charge.
Sol 8: Fe+3 has higher coagulating power compared Rest, all methods depends on size of particles.
to K+, So, it can stop bleeding by coagulating blood
around the wound. Sol 5: (A) To form micelle at very low concentration,
it must be highly insoluble in water. CH3(CH2)
Sol 9: It’s the process of setting down of colloid 11
OSO3¯,CH3(CH2) 6COO¯has charged ends. N+ in other
particles. 2 structure is crowded, so it’s interaction with H2O will
be less. So CH3(CH2) 15 N+ (CH3) 3 Br¯ will need minimum
Sol 10: NH3 has a higher TC than CO2. So it is easily conc. as its H.C. length is high.
liquefiable.
Comprehension Type
∴ NH3 will be adsorbed more readily.
Sol 6: (A, D) COO¯ is hydrophilic part. Soaps are
Sol 11: More is the TC of a gas, more is the easiness to
emulsifying agents.
liquefy the gas. So, more is the extent of adsorption.
Sol 7: (A) R part is hydrocarbon part. It dissolves in
Sol 12: Activity is the ability of a catalyst to drive the grease and forms micelle.
reaction in forward direction.
It depends on many factors like surface area, Sol 8: (B) In multi-molecular colloids, atoms or
temperature etc. molecules are held together by weak Van der Waals
forces.
Sol 13: Gelatin has a low gold number, so even adding
Sol 9: (A) Cleansing action of soap is due to attraction
small amounts of gelatin will stabilise it. It will act as a
between hydrophobic part of soap & grease (oil) .
lyophilic colloid.

Sol 10: (D) 0.025 g – 25 mg


∴ Gold number = 25

Sol 11: (B, D) Lower the gold number, more is the


protective power as lesser the amount of substance
required to prevent coagulation. Similarly, lower is
coagulation value, higher is coagulation power.
2 1 . 5 0 | Surface Chemistr y

Sol 12: (A) Protective nature of colloids. Sol 6: (C) Sb2S3 is a negative (anionic) Sol According
to Hardy Schulze rule, greater the valency of cationic
Sol 13: (D) Xe will be adsorbed to maximum extent coagulating agent, higher its coagulating power.
because of its high TC compared to other elements in Therefore, Al2(SO4)3 will be the most effective
it’s group. coagulating agent in the present case.

Sol 14: (C) CO. As its the most easily liquefiable gas.
Sol 7: (A, B, D) (A) In the process of adsorption, a bond
Sol 15: (B) Adsorption is inversely proportional to is formed between adsorbate and adsorbent, hence
temperature. always exothermic.
(B) Physisorption required very low activation energy
Assertion Reasoning Type while chemisorption require high activation energy.
Therefore, a physisorption may transform into
Sol 16: (C) In adsorption, ∆H is –ve (adsorption chemisorption but only at high temperature.
is exothermic) ∆S is also negative as freedom of (C) It is wrong statement as at higher temperature,
adsorbent decrease. physically adsorbed substance starts desorbing.

Sol 17: (C) If TC of a gas is high, its easily liquefiable. So, (D) In physical adsorption, van der Waal’s force hold
it can be adsorbed easily. the adsorbate and adsorbent together which is a
weak electrostatic attraction. In chemisorption, strong
Sol 18: (A) Adding detergents reduces the surface chemical bond binds the adsorbate to the adsorbent.
tension of water. COO¯ interacts with polar H2O Therefore, chemisorption is more exothermic than
molecules. physical adsorption.

Sol 19: (B) Assertion is false, isoelectric point is the pH Sol 8: (B) Both statements are independently correct
at which particle is electrically neutral (may be Zwitter but statement-II does not explain statement-I. Critical
ions). A is false. micelle concentration is the minimum concentration of
surfactant at which micelle formation commences first.
At critical micelle concentration, several molecules of
Previous Years’ Questions surfactant coalesce together to form one single micelle
molecule. This decreases the apparent number of
Sol 1: (B) As temperature increases, surface tension of molecule suddenly lowering conductivity sharply.
liquid decreases.
Sol 9: (B) Adsorption of methylene blue on activated
charcoal is physical adsorption hence it is characterised
Sol 2: (A) It is an exothermic process. According to Le- by decrease in enthalpy.
Chatelier’s principle, lowering temperature drive the
process in forward direction. Sol 10: (B, C, D) Adsorption of O2 on metal surface is
exothermic. During electron transfer from metal to O2
*
Sol 3: (C) ∆G = ∆H- TDS electron occupies π2p orbital of O2.
Due to electron transfer to O2 the bond order of O2
Sol 4: (D) Lyophilic sols are reversible, not easily decreases hence bond length increases.
coagulated because it is self-stabilizing.
Sol 11: (A, D) Lyophobic colloids are stable due to
preferential adsorption of ions on their surface from
Sol 5: (A) Larger the hydrophobic fragment of
solution and potential difference between the fixed
surfactant, easier will be the micellization, smaller
layer and the diffused layer of opposite charges around
the critical micelle concentration. Therefore, CH3(CH2)
the colloidal particles that makes lyophobic sol stable.
15
N+(CH3) 3Br¯ will have the lowest critical micelle
concentration.
2017-18 100 &
op kers
Class 12 T
By E ran culty
-JE Fa r
IIT enior emie .
S fP r es
o titut
Ins

CHEMISTRY
FOR JEE MAIN & ADVANCED
SECOND
EDITION

Exhaustive Theory
(Now Revised)

Formula Sheet
9000+ Problems
based on latest JEE pattern

2500 + 1000 (New) Problems


of previous 35 years of
AIEEE (JEE Main) and IIT-JEE (JEE Adv)

5000+Illustrations and Solved Examples


Detailed Solutions
of all problems available

Plancess Concepts
Topic Covered Tips & Tricks, Facts, Notes, Misconceptions,
Key Take Aways, Problem Solving Tactics
Alcohols, Phenols
and Ethers PlancEssential
Questions recommended for revision
22. ALCOHOLS, PHENOLS AND
ETHERS

ALCOHOLS

1. INTRODUCTION
(a) These are the organic compounds in which –OH
group is directly attached with carbon.
(b) These are hydroxyl derivatives of alkanes, mono
alkyl derivatives of water.
(c) Their general formula is CnHn+1OH or CnH2n+2O.

1.1 Classification of Alcohols


Mono, Di, Tri or Polyhydric Compounds
Alcohols and phenols may be classified as mono-, di-,
tri- or polyhydric compounds depending on whether
they contain one, two, three or many hydroxyl groups
respectively in their structures as given below:

1.1.1 Compounds Containing Csp3 – OH Bond


In this class of alcohols, the –OH group is attached to an sp3 hybridised carbon atom of an alkyl group. They are
further classified
(a) Primary, secondary and tertiary alcohols:
In these types of alcohols, the –OH group is -CH₂-OH CH- OH - C- OH
attached primary secondary and tertiary carbon
atom, respectively as depicted below. o o o
Primary (1 ) Secondary (2 ) Tertiary (3 )

(b) Allylic alcohols: In these


- - - -

alcohols, the –OH group H -C-


- - -

is attached to an sp3 CH₂-


-CH-CH₂-OH CH₂-
-CH-C-OH CH₂-CH-C-OH
hybridised carbon next to
the carbon-carbon double -C- -C-
bond, i.e. to an allylic
carbon. For example
o
o
Primary (1 ) Secondary (2o) Tertiary (3 )
2 2 . 2 | Alcohols, Phenols and Ethers

(c) Benzylic alcohols: In these alcohols,


the –OH group is attached to an sp3 H C
–hybridized carbon atom next to an
CH2OH C OH C OH
aromatic ring.
For example Allylic and benzylic C C
alcohols may be primary, secondary
or tertiary
Primary Secondary Tertiary

1.1.2 Compounds Containing Csp2–OH Bond


OH
These alcohols contain –OH group bonded to a carbon- carbon double i.e., OH CH₃
to a vinylic carbon or to an aryl carbon. These alcohols are also known as
vinylic alcohols
Vinylic alcohol : CH2=CH-OH

2. PREPARATION OF ALCOHOLS

2.1 From Alkenes

H+
C=C +H₂O C-C
-
-

H OH
H-
CH₃CH=CH₂ + H₂O CH₃-CH-CH₃

2.1.1 By Acid Catalyzed Hydration


Alkenes react with water in the presence of acid as catalyst to form alcohol. In case of unsymmetrical alkenes, the
addition reaction takes place in accordance with Markonikov’s rule.
Mechanism: The mechanism of the reaction involves the following three steps:
+ +
Step 1 H2O + H H3O
H H
+ +
C=C +H -O -H C C + H2 O

Step 2 H
+
H H O H
+
C C + H2 O C C

Step 3 H H H OH
+ +
C C O H + H2 O C C + H3O

2.1.2 By Hydroboration-Oxidation
Diborane (BH3)2 reacts with alkenes to give trialkyl boranes as addition product. This is oxidized to alcohols in the
presence of aq. sodium hydroxide (NaOH) and peroxide.
Chem i str y | 22.3

Note: This is the addition of water at double bond according to Anti-Markonikov Rule.

H3C-CH=CH2+ (H-BH2)3 CH3-CH-CH2


H BH2
CH3-CH=CH2
CH3-CH=CH2
(CH3-CH2-CH2)3B (CH3-CH2-CH2)2BH
H2O 2H2O2, OH

CH3-CH2-CH2-OH + B(OH)3
Proapn-1-ol

Mechanism of hydroboration – deboration

CH₃-CH- B₂H₆-THF/H₂O₂/OH- CH₃-CH₂-CH₂-OH


-CH₂

+ H- transfer
+ CH₃-CH-CH₂ CH-CH-CH₂ CH₃-CH₂-CH₂
-

-
:

+ - O. - BH₃ BH₂
.- BH₃
These steps are repeated thrice to form (CH3— CH2— CH2)3B and then

R .. .. R R
.. O
H O .. H -H
+ ..
R B R B O O H R B .. ..O
O H
-
R R R -OH

R B OR
R

OR
|
With H2O2, finally RO − B − OR is formed by above mentioned method.

OR
|
3NaOH
RO − B − OR  → Na3BO3 + 3ROH

2.1.3 Oxymercuration Demercuriation


Involves an electrophilic attack on the double bond by the positively charged mercury species. The product is a
mercurinium ion, an organometallic cation containing a three-membered ring.
With mercuric acetate, the product is 3-methyl-2-butanol (Markonikov’s addition with no rearrangement,
oxymercuration-demercuration reaction)

Hg(OCOCH3)2 NaBH4
(CH3)2 CHCH = CH2 (CH3)2 CHCH-CH2 (CH3)2CHCHCH3
3-Methylbut-1-ene
OH HgOCOCH3 OH
Not-isolated 3-Methyl-1-butanol
2 2 . 4 | Alcohols, Phenols and Ethers

OAc
Hg+
C C Hg(OAc) C C
mercurinium ion

Mercuration commonly takes place in a solution containing water and an organic solvent to dissolve the alkene.
Attack on the mercurinium ion by water gives (after deprotonation) an organomercurial alcohol.

Hg(OAc)
Hg(OAc) Hg(OAc)

- - -

- -

: - -

- -
-H+
-C C- -C C- -C C-
-

H-+O: :OH
:

H₂O:
:

H₂O: Organomercurial alcohol


H

Hg(OAc) H
- -
- -

- -
- -
- -
-C-C- + NaBH₄ + 4OH -C-C- +NaB(OH)₄ + 4Hg + 4OAc

OH OH
Organomercurial alcohol alcohol

The second step is demercuration, to form the alcohol. Sodium borohydride (NaBH4, a reducing agent replaces the
mercuric acetate with hydrogen.)

2.2 From Carbonyl Compounds

2.2.1 By Reduction of Carbonyl Compounds


LiAlH₄ / Na + C₂H₅OH
R-CHO + 2H R- CH-H
-
=

O OH
1o alcohol

LiAlH₄ / Na + C₂H₅OH
R-C-R + 2H R-CH-R
-
=

O OH
2o alcohol
2.3 From Acid Derivatives

2.3.1 By Reduction of Acid and its Derivatives

LiAlH₄
R - C - OH + 4H R-CH₂-OH
=

O
LiAlH₄
R - C - X + 4H R-CH₂-OH + HX
=

O
LiAlH₄
R - C - OR’ + 4H R-CH₂-OH + R’OH
=

O
LiAlH₄
RCOOCOR + 8H 2 RCH₂OH + H₂O
Chem i str y | 22.5

2.4 From Grignard Reagents

2.4.1 Reaction with Oxirane

δ- +δ +

: :
R : MgX + H₂C -CH₂ : : R-CH₂-CH₂-O MgX

O
H₂O+

R-CH₂-CH₂-OH
Primary alcohol

2.4.2 Reaction with Carbonyl Compounds


- -

- i) ether
R : Mg - X +- C = O R-C-O-H + MgX₂
+
ii) H₃O , X-
- -

- - +
R : MgX +-C = O R-C-O MgX
- + + -
- -

- -

-
R - C - O MgX + H - O - H R- C - O-H + O-H + MgX₂
-

H H

2.4.3 Reaction with Acetaldehyde


CH₃ CH₃
CH₃ +
H₃O OH
CH₃-CH₂-MgBr + H O Ether H₃C H₃C
H OMgBr
butan-2-ol

2.4.4 Reaction with Ketone

CH₃ CH₃
CH₃ H₃C
H₃C H₃O OH
CH₃-CH₂-CH₂-MgBr + CH₃ O Ether
H₃C OMgBr
CH₃
2-methylpentan-2-ol

2.5 By Fermentation
Fermentation is a low decomposition of complex organic compounds into simpler compound in the presence of
suitable micro-organisms which are the source of biochemical catalyst known as yeast.

(C6H10 O5 )n → CH3CH2CH2CH2OH+ CH3COCH3


Starch n−Butylalcohol
2 2 . 6 | Alcohols, Phenols and Ethers

3. PHYSICAL PROPERTIES OF ALCOHOLS


(a) The lower alcohols are liquids while higher having more than 12 carbon atoms are solids. They are colourless,
neutral substance with characteristic sweet, alcoholic odour and burning taste.
(b) The lower alcohols are readily soluble in water and the solubility decreases with the increase in molecular
weight.
The solubility of alcohols in water can be explained due to the formation δ+ δ- δ+ δ- δ+ δ-
of hydrogen bond between the highly polarized –OH groups present H-O H-O H-O
both in alcohols and water.
H R H
However, in higher alcohols, the hydrocarbon character (alkyl chain)
increases, showing a steric hindrance. Hence, the solubility in water decreases.
When the ratio of C:OH is more than 4, alcohols have little solubility in water.
(c) Boiling points of alcohols are much higher than those of the corresponding δ+ δ- δ+ δ- δ+ δ-
alkanes. It is due to the intermolecular hydrogen bonding present H-O H-O H-O
between the hydroxyl groups of the two molecules of an alcohol with the
result several molecules are associated to form a large molecule. R R R

Among the isomeric alcohols, b.p. and m.p. show the following trend.
Primary > Secondary > Tertiary
This is because of the fact that in secondary and tertiary alcohols, the alkyl part (hydrogen character) outweighs
the –OH group due to branching.
(d) Lower alcohols form solid addition compounds with anhydrous metallic salts like CaCl2 and MgCl2, viz., CaCl2,
4C2H5OH and MgCl2.6C2H5OH
By analogy to water of crystallization, these alcohols molecules are referred to as alcohols of crystallization.
For this reason, alcohols cannot be dried over anhydrous calcium chloride.

PLANCESS CONCEPTS

Preparation of alcohols:
•• Key takeaway - Hydration and oxymercuration-demercuration gives Markonikov’s product but
hydroboration-oxidation gives Anti-markonikov’s product.
•• Misconception - Hydroboration follows Markonikov’s rule but in this case, the electron deficient
species is Boron and not Hydrogen.
•• Note - On replacing water with carboxylic acid in hydroboration-oxidation, the product obtained is
alkane instead of alcohol.
•• Note - Tertiary alcohols cannot be obtained by reduction of carbonyl compounds.
•• Fact - If we use NaOH as a reductant in reduction of carbonyl compounds to alcohols, the process is
known as Darzen’s process.
•• Tips and tricks - In conversion of oxirane to alcohols using Grignard’s reagent, the alkyl part adds to
the carbon with less steric hindrance as it proceeds via SN2 mechanism.
Physical properties of alcohols:
Alcohols generally have high boiling point because of hydrogen bonding.

Vaibhav Krishnan (JEE 2009, AIR 22)


Chem i str y | 22.7

Illustration 1: Write the IUPAC names, as their names by Carbinol system, and classify them as 1º, 2º, 3º, allylic,
vinylic, benzylic, and propargylic of the following compound. (JEE MAIN)

Me Me
(A) Me (B) OH
Me Ph
OH Me

Me
(C)
(D) Et₃C OH
OH

Sol:

S.No. Structure IUPAC name Carbinol system name Type of alcohol


1
6 4 2 Me n-Butyl isopropyl
a. 7 3 2-Methyl heptan-3-ol 2º
Me 5 Me carbinol
OH
Me1 2
OH Dimethyl phenyl
b. 2-Phenyl propan-2-ol 3º
Ph carbinol
Me
3

3 1
2 Me Methyl vinyl
4
c. But-3-en-2-ol 2º allylic
carbinol
OH
5 4
3 OH
d. 3-Ethyl pentan-3-ol Triethyl carbinol 3º
2
Me
1

Illustration 2: (a) Write the structure of all isomeric alcohols of molecular formula C5H12O and give their IUPAC,
common and carbinol names. Indicate each as 1º, 2º and 3º and also their stereoisomers, if any-
(b) Write the structures and names of all the cyclic and stereoisomers of C4H7OH.  (JEE MAIN)

Sol: (a) (i)

4 5 24 2
OH OH
5 OH OH 4 5 4
OH 4 5 4
5 Me4 Me2 OH 5 2 2 5 3
2 1 2
3
1
Me 3 13 1 5 Me4 Me2 + Me + Me 5 Me4 3Me2 1 Me Me
3 1 Me 3
+ Me 13 1 Me Me
(I) (I) 3 1
OH OH
(I) [(II) + [(II)
(III)]+ (III)]
[(II) + (III)] OH
(O.A) (O.A) (IV) (IV)
(O.A) (IV)
IUPAC Pentan-1-ol (±)-Pentan-2-ol Pentan-3-ol
Common n-Amyl alcohol — —
Carbinol n-Butyl carbinol Methyl propyl carbinol Diethyl carbinol
Type 1º 2º 2º
2 2 . 8 | Alcohols, Phenols and Ethers

(ii) Write the four atoms in a straight chain and put Me and (–OH) at different positions.
OH
OH 2 4 Me
3 1
Me
14
2 Me Me
 3
3
Me Me Me
1
4 2 2
OH
3
Me
1  3 Me
4Me Me 2 2 Me
Me OH 1
[(V)Me
+ VI] (O.A) Me OH Me3 Me
1
[(V) + VI] (O.A) OH (VII) Me (VIII)
(VII) (VIII)
IUPAC (±)-3-Methyl butan-2-ol 2-Methyl
butan-2-ol

Common –– t-Pentyl alcohol


Carbinol Isopropyl methyl carbinol Dimethylethyl carbinol

Type 2º 3º

(iii) Write the three C atoms in a straight chain and put two Me and (–OH) at different positions.

3 1
Me
Me 2 Me
4 2
Me 3 OH
Me Me Me 1
) OH Me
(VII) (VIII)
IUPAC 2,2-Dimethyl propan-1-ol
Common Neopentyl alcohol
Carbinol t-Butyl carbinol
Type 1º

Hence, total isomers including stereoisomers of C5H12O are 8.

Me
Me
MeMe2 22 Me
Me
MeMe2 22 2 1 11 1HH
HH
2 1 11 1OH
OH
OHOH
OH
OH
OHOH HHH H OH
OH
OHOH
OH
OH
OHOH HHH H HHH H
Me
Me
MeMe 3 33 3 3 33 3
(I)(I)
(I) (I) (III
(III+(III
(III ++IV)+
IV)IV)
IV)
Name (III
(III
(III+(III
++IV)
IV)
+
IV)IV)
(II)
(II)(II)
(II) O.A
O.A
O.A O.A O.A
O.A
O.A O.A
(±)
(±)
(±)
or
(±)
or
or
racemate
racemate
or
racemate
racemate (±)
(±)
(±)(±)
oror
or or racemate
racemate
racemate
racemate

Cyclopropyl 1-Methyl (±) or r-cis-2-Methyl (±) or r-trans-2-Methyl


methanol cyclopropan-1-ol cyclopropan-1-ol
cyclopropanol

OH
(cyclobutanol) Hence, the total isomers including stereoisomers of C4H7OH are 7.

(VII)
Chem i str y | 22.9

Illustration 3: Cyclobutyl ethene


(A)

+
Dil. H2SO4
(B)
 Number of
isomeric products
including stereo-
isomers

 (JEE ADVANCED)

4 5
3
Sol: + Me
H Ring
expansion
2 1
4+
(A)
H OH H OH H₂O+ 5 Me
-H 2
 H  Me 1
2 C+
o

 
Me + H
+
(trans) (cis) Me
3 C+
o
Optically active Optically active
() or racemate () or racemate -H + H₂O
III and IV [ and I]
Me
OH
(V)
The total number of isomeric products including stereoisomers is 5.

Illustration 4: Synthesize the following:


(a) Butene to butanol and butan-2-ol
(b) 1-Chloro butane to pentanol and pentan-2-ol (JEE MAIN)

? 3 1
Sol: (a) Me
4
Me OH (Butan-1-ol) (I)
2
Butene
OH
4
Me (Butan-2-ol) (II)
3
2
1
Me

Hydroborato oxidation proceeds with Anti-Markovnikov addition, so it would give (I), while acid-catalysed hydration
and mercuration – demercuration reaction proceed with Markovnikov addition, so it would give (II).

Synthesis:
a. Anti-Mark
Dil H₂SO₄ Me
Me or Me
(i)B₂H₆ + THF
Me - Me OH Hg(OAc)₂/ -
OH
Mark H₂O+NaBH₄+OH
(ii)H₂O₂/OH (I) add HBr
Anti-Mark HBr + ROOR (II)
Me Me Aq. NaOH
Aq. KOH
Me Br
Br
3 1 4 2
4 5 OH
b. Me 2
Cl Me 3
1-Chlorobutane (Pentan-1-ol)
(I)
OH

Me Me
(Pentan-2-ol)
(II)
2 2 . 1 0 | Alcohols, Phenols and Ethers

The 4C-atom chain has to be increased to 5C-chain by a G.R. With CH2=O(HCHO)


3 1
4 Mg/ether
Me Cl Me MgCl
2 

R (i) CH₂ O
4 2
5 OH
Me (ii) H₃O +
3 1
(I)
4 2 1 4 2
Conc. H₂SO₄ 5 3 5 3
(I) Me Me + Me
1
-H₂O Pent-2-one Pent-1-one
(Major) (Major)

(i) B₂H₆/THF
-
(ii) H₂H₂/OH
OH
4
5
Me 3
2 Me
1
(II)

4. CHEMICAL PROPERTIES OF ALCOHOLS

4.1 Reaction with Active Metals-Acidic Character


Alcohols are weakly acidic in nature due to which when they react with group one alkali metals they liberate
hydrogen gas and form alkoxides.
2R – O – H + 2Na → 2R – O– Na+ + H2↑
The acidic order of alcohols is MeOH > 1º > 2º > 3º. This acidic nature of alcohol is due to the presence of polar
O-H bond.

4.2 Esterification/Reaction with Carboxylic Acid


Reaction of alcohol with carboxylic acid in presence of sulphuric acid gives an ester. In this reaction sulphuric acid
react as protonating agent as well as dehydrating agent.

2 4 conc.H SO
R – O – H + H – C – R → R – C – O – R + H2O
|| ||
O O

Mechanism:
+ -
H2SO4 H + HSO4

+ ROH
RCO H+H R C O H R C R O C R + RCOR
H2 O -H
O O H O H O O

Note : The above reduction is laboratory method of ester preparation.


Chem i str y | 22.11

4.3 Reaction with Acid Derivatives


When alcohols are treated with acid derivatives , hydrogen of hydroxyl group is substituted by acyl group.

conc.H2SO4
R O H+X C R R O C R + HX

O O
conc.H2SO4
R O H +R C O C R R O C R

O O O

4.4 Reaction with Isocyanic Acid

ROH + H − N = C → H − N = C − OR → H2N − C − OR
|| | ||
O OH O
Amino ester

4.5 Reaction with Ethylene Oxide


- + ROH
R O H + CH₂ CH₂ CH₂ CH₂ CH₂ CH₂
-H₂O
O OR OH OR OR

1,2-dialkoxy ethane

4.6 Reaction with Diazomethane

R O H + CH2 N2 R O CH3
(ether)

4.7 Reaction with H2SO4


140oC
CH3 CH2 OH + H2SO4 (excess) CH3 CH2 O CH2 CH3

Mechanism:
+
H₂SO₄ H + HSO₄-
.. + ..
CH₃ CH₂ O + H CH₃ CH₂ O+ H
..

H H
-H₂O
.. CH₃ CH₂ OH
CH₃ CH₂ O CH₂ CH₃ CH₃ +CH₂
..

H
(protonated ether)
+
-H
..
CH₃ CH₂ ..
O CH₂ CH₃
o
160 C
(ii) CH3 CH2 OH + H2SO4 CH2 = CH2
(excess)
H + HSO-4
+
H2SO4
2 2 . 1 2 | Alcohols, Phenols and Ethers

160oC
CH3 CH2 OH + H2SO4 CH2 = CH2
Mechanism:
(excess)
H + HSO-4
+
H2SO4

+ .. H
+

CH3 CH2 OH + H CH3 CH2 O H CH3 CH2 CH2 = CH2


-H2O
H

4.8 Action of Halogen Acids


Alcohol react with HX to give RX. Reactivity order of ROH is 1°>2°>3°. Hence primary alcohols react in presence of
catalyst (If X is Cl Luca’s reagent and if X is Br small amount of H2SO4), but secondary and tertiary alcohols can react
in absence of catalyst. However, when alcohol react with HI/Red P they reduced in hydrocarbon.

The reactivity of halogens is in the order: HI > HBr > HCl

SN1 reaction with the Lucas reagent (fast)


CH3 CH3 CH3 CH3
ZnCl2 ZnCl2
+ + Cl
H CH C 3O H H CHC 3O H H C CH H CH
C 3 Cl
ZnCl2 +
ZnCl2 3
+ CH3 Cl
H CHC 3O H H C CH3O H H C H CCH3Cl
HO ZnCl2 CH3
CH3 CH3 CH3
SN2 reaction with Lucas reagent is slow: HO ZnCl2
CH3CH2CH2 CH2CH2CH3
ZnCl2 -
- C O
+ +
ClCH3CH2HCH2 Cl CH2CHO
C
2CH3
ZnCl2
ZnCl
H 2 -
- C O
+
Cl C O +
ZnCl2
Cl HH H H H H
H TransitionHstate
H H
Transition state
CH2CH2CH3 ZnCl2
Cl C + O
CH2CH2CH3 ZnCl
H 2
Cl C + O
H H H
H H

4.9 Action of Thionyl Chloride


Alcohols react with thionyl chloride to form alkyl halide and reaction is called diarzon process.

C2H5OH + SOCl2 → C2H5Cl + HCl + SO2

Meachanism

Cl Cl O O
R O S =O R O S O R O S R O S + HCl
+ +
H Cl H Cl H Cl Cl

Thionyl chloride Cl-


Chlorosulfite ester

O O
(fast) O
R S=O R+ S=O R S=O
Cl Cl Cl
Chlorosulfite ester Ion pair
Chem i str y | 22.13

This mechanism resembles the SN1, except that the nucleophile is delivered to the carbocation by the leaving
group, giving retension of configuration as shown in this following example. (Under different conditions, retension
of configuration may not be observed).
H H
OH SOCl₂ Cl
C C
CH₃(CH₂)₄CH₂ CH₃ CH₃(CH₂)₄CH₂ CH₃
(R)2-octanol dioxane (R)-2-chloroocatane
(solvent) (84%)

4.10 Action of Phosphorus Halides (PX5 and PX3)


Phosphorous halide react with alcohols to form corresponding haloalkanes.

For Example: C2H5OH + PCl5 → C2H5Cl + HCl + POCl3

Mechanism:
3 R- OH + PCl₅ P (OR)₃Cl₂+3HCl
P(OR)₂Cl₂ + ROH P (OR)₃Cl+HCl
Cl
(OR)₃P
(OR)₃P = O + RCl
: :

O
R
O

(RO₃)P = O + 3HCl P + 3ROH


Cl Cl Cl

4.11 Action of Ammonia


When vapours of ammonia with alcohol passed over heated alumina mixture of primary, secondary and tertiary
amines is formed.

Al2O3 ROH ROH


ROH + NH3 RNH2 R2NH R3N
Al2O3 Al2O3
0 0 0
1 amine 2 amine 3 amine

The ease of dehydration of alcohols is in the order Tertiary > Secondary > Primary

4.12 Dehydration
Alcohols undergo dehydration (removal of a molecule of water) to form alkenes on treating with acid e.g.,
concentrated H2SO4 or H3PO4 or catalysts such as anhydrous zinc chloride or alumina

+
H
C C C = C + H2 O
Heat
H OH
OH
85%H2PO4
CH3CH CH3 CH3 CH = CH2 + H2O
440 K

CH3 CH2
20%H2PO4
CH3 C OH CH3 C CH3 + H2O
358 K
CH3
2 2 . 1 4 | Alcohols, Phenols and Ethers

Mechanism of dehydration

H H H H H
Fast
Step 1 : H C C O H + H+ H C C O+ H
H H H H
Ethanol Protonated alcohol
(Ethyl oxonium ion)

H H H H H H
Fast
Step 2 : H C C O+ H H C C+ + O H
H H H H
Protonated alcohol Carbocation

H H H H
Step 3 : H C C+ C=C + H+
H H H H
Ethene

The acid used in step 1 is released in step 3. To drive the equilibrium to the right, ethane is removed as it is formed.
The relative ease of dehydration, i.e., 3º > 2º > 1º, of alcohols follows the order of stability of carbonium ions.

(a) With heated alumina (Al2O3): When vapours of an alcohol are passed over heated alumina, different products
are obtained at different temperatures as given below:
(i) At 513 – 523 K (240º – 250º C), intermolecular dehydration takes places to form ethers e.g.,
Al2O3
2CH3CH2OH CH3CH2 O CH2CH3 + H2O
513-523 K
Ethyl alcohol Diethyl ether

(ii) At 633 K (360ºC), intermolecular dehydration takes place to form alkenes, e.g.,
Al2O3
CH3CH2OH CH2 = CH2 + H2O
633K
Ethanol Ethene

4.13 Oxidation of Alcohols


(a) Oxidation: Oxidation of alcohols involves the formation of carbon-oxygen double bond with cleavage of O–H
and C–H bond.
H C O H C = O +H2

These are also called dehydrogenation reactions since it involves loss of hydrogen from the alcohol molecule. The oxidation
of alcohols can be carried out with a variety of reagents such as neutral, acidic or alkaline KMnO4, acidified K2Cr2O7 or dil.
HNO3. The ease of oxidations and nature of the products, however, depends upon the type of alcohol used.
(i) Primary Alcohols are easily oxidized first to aldehydes and then to acids, both containing the same
number of carbon atoms as the original alcohol.
H O
[O] [O]
RCH2OH R C=O R C OH
o
Oxidation Oxidation
1 Alcohol Aldehyde Carboxylic acid

CH3CH2OH+[O] K2Cr2O7 + DilH2SO4 CH CHO [O] CH3 COOH


3
-H2O
Ethyl alcohol Acetaldehyde Acetic acid
H O
[O] [O]
RCH2OH R C=O R C OH
o
Oxidation Oxidation Chem i str y | 22.15
1 Alcohol Aldehyde Carboxylic acid

E.g., CH3CH2OH+[O] K2Cr2O7 + DilH2SO4 CH3CHO


[O] CH3 COOH
-H2O
Ethyl alcohol Acetaldehyde Acetic acid

The oxidation can, however, be stopped at the aldehyde stage if Cr(VI) reagent such as Collin’s reagent
(CrO3.2C5H5N, chromium trioxide-pyridine complex), Corey’s reagent or pyridinimum chlorochromate
(PCC, CrO3.C5H5N.HCl or C5H5NH + CrO3Cl–) pyridinimum dichromate [PDC, (C5H5NH)22+ Cr2O72–] in
anhydrous medium (i.e., CH2Cl2) are used as the oxidizing agents.

O
C5H5NH CrO3Cl- (PCC)
+
RCH2OH R C H
CH2Cl2
o
1 Alcohol Aldehyde

(ii) Secondary Alcohols are easily oxidized to ketones with the same number of carbon atoms. However,
ketones resist further oxidation but in some conditions, they are oxidized to carboxylic acids containing
lesser number of carbon atoms than the original alcohol.
CH₃ K₂Cr₂O₄/H₂SO₄ CH₃ K₂Cr₂O₄/H₂SO₄
CHOH C O CH₃COOH + CO₂ + H₂O
-H₂O
CH₃ CH₃
Acetic acid
Isopropyl alcohol Acetone

OH O
[O] [O]
CH₃ CH CH₂CH₂CH₃ CH₃ C CH₂CH₂CH₃ CH₂COOH HOOCCH₂CH₃
Pentane-2-ol Pentane-2-one Acetic acid Propionic acid

This oxidation be stopped at the ketone stage by using chromic anhydride (CrO3)
CrO₃/C₅H₅N
R CH R’ R CH R’
CH₂/Cl₂
OH O
Ketone
(iii) Tertiary Alcohols are resistance to oxidation in neutral or alkaline KMnO4 solution but are readily
oxidized in acidic solution (K2Cr2O7/H2SO4 or KMnO4/H2SO4) to a mixture of a ketone, and an acid each
containing lesser number of carbon atoms than the original alcohol. The oxidation presumably occurs
via alkenes formed through dehydration of alcohols under acidic conditions. For example
CH₃ CH₃
H+ [O] CH₃ [O]
CH₃ C OH -H₂O CH₃ C CH₂ CH₃ C O + [HCOOH] CO₂ + H₂O
CH₃
tert-Butyl alcohol 2-Methylprop-1-ene Acetone Formic acid

CH₃ CH₃
H + [O] CH₃
CH₃ C CH₂ CH₃-H₂O CH₃ C CH CH₃ C O + CH₃COOH
CH₃
OH
2-Methylbutan-2-ol 2-Methylbut-2-ene Acetone Acetic acid

4.14 Oppenauer Oxidation


The aluminium-catalyzed hydride shift from the α-carbon of an alcohol component to the carbonyl carbon of a
second component, which proceeds over a six-membered transition state, is named Meerwein-Ponndorf-Verley-
Reduction (MPV) or Oppenauer Oxidation (OPP) depending on the isolated product. If aldehydes or ketones are
the desired products, the reaction is viewed as the Oppenauer Oxidation.
2 2 . 1 6 | Alcohols, Phenols and Ethers

Non-enolizable ketones with a relatively low reduction potential, such as benzophenone, can serve as the carbonyl
component used as the hydride acceptor in this oxidation.

RO OR
OH O Al(OR)3 Al O OH
O O
1 2
+ 3 4 1 1 2
+ 3 4
R R R R R H 4
R R R R
2 3
R R R OPP MPV

Action of heated copper: Different classes of alcohols give different products when their vapors are passed over
heated copper at 573 K (300º C)
(a) Primary alcohols undergo dehydrogenation to give aldehydes.
Cu/573K
CH3CH2OH CH3 CHO +H2
Ethanol Ethanal
Ethyl alcohol Acetaldehyde

(b) Secondary alcohols also undergo dehydrogenation to give ketones.


H
CH3 Cu/573K
CH3
C C = O +H2
CH3 OH CH3
Propan-2-ol Propanone
(Isopropyl alcohol) (Acetone)

(c) Tertiary alcohols, however, undergo dehydration to form alkenes.



CH3 CH3
Cu/573K
CH3 C OH CH3 C = CH2 +H2O
CH3
2-Methylpropan-2-ol 2-Methylpropene
(tert-Butyl alcohol)

4.15 Pinacol-Pinacolone Rearrangement Reaction


When pinacols (mostly ditertiary alcohols) are treated with mineral acids, acid chlorides, ZnCl2 or other electrophilic
reagent, they rearrange to form ketones called pinacolones with the elimination of H2O.

Mechanism:

Me Me Me Me
H
Me Me Me Me
Step 1
OH OH OH2 OH
Slow R.D.S
Step 2

Me Me shift Me Me
Me Me Step 3
Me Me
Me O H OH
3oC
-H Step 4

Me Me
Me
Me O
Chem i str y | 22.17

4.16 Dihydric Alcohols


Ethylene glycol or ethane-1, 2-diol
(a) Preparation:
(i) 3CH2 = CH2 + (alkaline) KMnO4 + 4H2O
3HOH2C-CH2OH+2MnO2+2KOH

O2/Ag
(ii) CH2 = CH2 O (Epoxy ethane) or
575 K
H2O/473K
(Ethylene epoxide) HOH2C-CH2OH
Hydrolysis

Ethylene glycol undergoes extensive intermolecular H-bonding . As a result, dihydric alcohols are highly
associated and have high b.p., high viscosity, and are highly soluble in H2O.
(b) Reactions:
PCl5 or HCl, 433K
(i) HOCH2 CH2 OH CH2Cl CH2Cl
or SOCl2

2Hl
(ii) HOCH2 CH2 OH -H O [l-CH2-CH2-l]
2

CH2=CH2+l2
H2SO4
(iii) HOCH2 CH2OH+HNO3 CH2ONO2+2H2O

CH2ONO2
Ethylene dinitrate

CH3COOH/H2SO4
(iv) HOCH2 CH2OH CH2O COCH3

CH2OCOCH3
Glycol diacetate

(c) Oxidation: Ethylene glycol on oxidation with conc. HNO3 mainly gives glycolic acid and oxalic acid. The other
oxidation products such as glyoxal and glyoxalic acid are also formed in small quantities because they are
more readily oxidized than glycol itself.

HOCH2 CH2OH
[O]
OCH CH2OH HOOC CH2OH
Glycoaldehyde Glycollic acid
[O] [O]
[O] [O]
OHC CHO HOOC CHO (COOH)2
Glyoxal Glyoxalic acid Oxalic acid

(d) Dehydration:
(i) HOCH2 CH2 OH
773K
O + H2 O

(ii) HOCH2 CH2 OH Anhd.ZnCl2 [CH2 = CHOH]


-H2O
Vinyl alcohol
Tautomerise
CH3 CHO
Conc.H3SO4
(iii) With conc. H2SO4: HO CH2 CH2 O H
distill
+ 2H2O
H O CH2 CH2 OH
Dioxane
2 2 . 1 8 | Alcohols, Phenols and Ethers

(iv) Conc.H3PO4
HO CH2 CH2OH distill

H O CH2 CH2 OH
CH2 CH2OH
O
CH2 CH2OH
Diethylene glycol

Trihydric Alcohols; Glyerol or Glycerine 1, 2, 3-Propanetriaol


(a) Preparation:
(i) CH3CH = CH2
Cl2773K
Cl CH2 CH = CH2
-HCl

aq. KOH or aq. Na2CO3


HO CH2 CH = CH2
423 K, 1-2 atm
Allyl alcohol
HOCl
HO CH2 CHCl CH2OH
+ -
Cl OH

aq. NaOH
-NaCl HOCH2 CHOH CH2OH

(b) Properties: Due in the presence of three (–OH) groups, it undergoes extensive intermolecular H-bonding and
thus it has high boiling point viscosity and is highly soluble in H2O.
(c) Reaction: When glycerol is treated with a small amount of HI or Pl3 allyl iodide is formed.
l2
3Hl (-3H2O) [lCH2 CHl CH2l] CH2 = CH CH2l
HOCH2 CHOH CH2OH 1,2,3-Triiodopropane (glycerol tri-iodide) Allyliodide
(Unstable)
When large moment of HI is used, the main product is isopropyl iodide.
+Hl
CH2 = CH CH2l [lCH3 CHl CH2l]
Allyliodide
-l2

+Hl
CH3 CHl CH3 CH3 CH = CH2

(d) Nitration:
Conc.HNO3
+Conc. H2SO4
HOCH2 CHOH CH2OH O2NOCH2 CHONO2 CH2ONO2
(283-298 K)
(Glyceryl trinitrate) (Nitroglycerine)

A mixture of glycerol trinitrate and glyceryl dinitrate absorbed on Kieselguhr is called dynamite discovered by
Alfred Noble.

(e) Dehydration with KHSO4 or conc. H2SO4:


KHSO4, 473-508K
HOCH2 CHOH CH2OH
-2H2O

Unstable
CH2 = CH CHO [CH2 = C = CHOH]
Tautomerisation
Chem i str y | 22.19

(f) Oxidation:
[O] [O] [O]
HOCH2 CHOH CH2OH OHC CHOH CH2OH HOOC CH2OH
[O] Glyceraldehyde Glyceric acid
[O]
OH CH2 CO CH2OH HOOC CO COOH HOOC CHOH COOH
Dihydroxy acetone Mesoxalicacid Tartonic acid

(i) With dil. HNO3, a mixture of glyceric and tartaric acid is obtained.
(ii) With conc. HNO3 mainly glyceric acid is obtained.
(iii) With bismuth nitrate, only mesoxalic acid is obtained.
(iv) Mild oxidizing agent, such as Br2 water, sodium hypobromite (Br2/NaOH) and fenton’s regagent (H2O2 +
FeSO4) give a mixture of glyceraldehyde and dihydroxy acetone. This mixture is called glycerose.

(g) Reaction with HIO4: HOCH2 CHOH CH2OH + 2HlO4


HOCH2 CHOH CH2OH + 2HlO4

2HCHO + HCOOH + 2HlO + H O

2HCHO + HCOOH + 2HlO3 +3H2O 2
HOCH2 CHOH CH2OH + 6[O]
(h) With acidic KMnO4: HOCH2 CHOH CH2OH + 6[O]
HOOC COOH (oxalic acid) + CO2 +3H2O
HOOC COOH (oxalic acid) + CO2 +3H 2O

(i) Reaction with oxalic acid: When oxalic acid is heated with glycerol at 383 K, it forms glycerol mono-
oxalate which loses a molecule of CO2 to give glycerol mono- formate which in turn on hydrolysis gives
formic acid.
HOCH2 CHOH CH2 OH + HO OCH COOH
 383 K
CH2 OOC COO H
-CO2 -H2O
COOH
CH2OH
HOH
HOH2C CHOH CH2 OOCH
OH H
HCOOH + HOH2C CHOH CH2OH
(ii) At 230º C (503 K), oxalic acid reacts with glycerol to form glycerol dioxalate which loses two molecules
of CO2 to give allyl alcohol.

5. D
 ISTINCTION BETWEEN PRIMARY, SECONDARY AND TERTIARY
ALCOHOLS
(a) Lucas test: This test is based on the difference in the three types of alcohols (having δ or less carbon towards
Lucas reagent (a mixture of conc. Hydrochloric acid and anhydrous zinc chloride)

ROH + HCl
ZnCl2
RCl + H2O

Since alkyl halides are insoluble, their formation is indicated by the appearance of a turbidity in the reaction
mixture. The order of reactivity is tertiary >secondary >primary, the tertiary alcohols produce turbidity
immediately, the secondary alcohols give turbidity within 5 – 10 minutes, and the primary alcohols do not
give turbidity at all, at room temperature.
2 2 . 2 0 | Alcohols, Phenols and Ethers

(b) Catalytic dehydrogenation (action of reduced copper at 300°). Discussed earlier,


(i) Primary alcohols form aldehydes
(ii) Secondary alcohols form ketones.
(iii) Tertiary alcohols form olefins.

(c) Victor Meyer test: This test is based on the different behaviour of primary, secondary and tertiary nitroalkanes
towards nitrous acid. The test involves the following steps.
(i) Alcohols is treated with concentrated hydroiodic acid or red phosphorus and iodine to form the
corresponding alkyliodide.
(ii) Alkyl iodide is reacted with silver nitrite to form the corresponding nitroalkane.
(iii) The nitroalkane is treated with nitrous acid (NaNO2 + HCl) followed by treatment with alkali (NaOH or
KOH). Upon such treatment different alcohols give different colours.
•• Primary alcohols produce a blood red colour
•• Secondary alcohols produce a blue colour
•• Tertiary alcohols produce no colour.

Primary Secondary Tertiary
CH₃CH₂OH (CH₃)₂CHOH (CH₃)₂COH
P + l₂ P + l₂ P + l₂

CH₃CH₂l (CH₃)₂CHl (CH₃)₃Cl


AgNO₂ AgNO₂ AgNO₂

CH₃CH₂NO₂ (CH₃)₂CHNO₂ (CH₃)₃CNO₂


HONO HONO HONO

CH₃CNO₂ (CH₃)₂CNO₂ No reaction


(Colour less)
NOH NO
Nitrolic acid (Pseudonitril)
NaOH NaOH

CH₃CNO₂ NO reaction
(Blue colour)
NONa
Sod. salt of nitrolic acid
(Red colour)

Illustration 5: Give the structure of the major organic product when 3-ethylpent-2-ene is treated with Hg(OAc)2 ,
H2O, NaBH4. (JEE MAIN)

Sol:
CH2CH3 CH2CH3 CH2CH3
Hg(OAc)2/H2O NaBH
CH3CH2 C = CH CH3 CH3CH2 C CH 4
CH3 Reduction CH3CH2 C CH2CH3
(Mark addition)
3-Ethylpent-2-ene
OH HgOAc OH
3-Ethylpentan-3-ol
Chem i str y | 22.21

Absolute alcohol:

Wash-Absolute alcohol

Wash [10-15% EtOH]

Distillation

Raw sprit [90% EtOH]

Fractional distillation

o
o
20-21 C 78 C

Mainly CH3CHO Rectified spirit [95.5% EtOH] Fuel oil

Excess benzene Azeotropic distillation [Glycerol + CH3COCH3


65oC + Higher alcohol]

C6H6+C2H2OH + H2O
(74.1%)(18.5%) (7.4) (Ternary mixture)

68oC

C6H6+EtOH
(Binary mixture)
(80%) (20%)

78.3oC

Absolute alcohol
[100% EtOH]

Flowchart 22.1 Preparation of absolute alcohol


2 2 . 2 2 | Alcohols, Phenols and Ethers

PLANCESS CONCEPTS

Chemical properties of alcohols:


(a) Rate of reaction of alcohols with carbonyl compounds depends on two factors:
(i) Leaving group ability of the substituent: Better the leaving group, faster the reaction.
(ii) Bulkiness of the alkyl part of alcohol: Bulkier the alkyl part, slower is the reaction because of steric
hinderance.
(b) Only alkyl methyl ether can be prepared by reaction of alcohol with Diazomethane.
(c) In reaction of alcohols with excess of sulphuric acid at lower temperatures, we obtain ethers. But, as
the temperature increases, alkenes become the favourable product. Also, in case of secondary and
tertiary alcohols, alkene is the predominant product due to ease of elimination.
(d) In reaction of alcohols with SOCl2 it proceeds via SNi mechanism. Thus, the configuration in case of
chiral carbon is retained. But, if pyridine is used as a solvent, the reaction proceeds via SN2 mechanism
with inversion of configuration.
(e) In reaction of alcohols with PCl5 and PCl3 proceeds via SN2 mechanism.
(f) Weak oxidizing agents like PCC, PDC etc oxidize 1⁰ alcohols to aldehydes while strong reagent
oxidizes 1⁰ alcohols to carboxylic acids. All these oxidizing agents oxidize 2⁰ alcohols to ketones but
3⁰ alcohols are not affected.
(g) In pinacol-pinacolone rearrangement, With unsymmetrical glycols, the product obtained is
determined mainly by the OH that is lost as H2O to give more stable carbocation and, thereafter, by
the better migrating group.
(i) The order of migratory aptitudes is Ar > > H > R.
(ii) The migratory order in aryl: Ar containing more e--donating (or more e- rich) migrates. For
example,

MeO Me Ph- Cl

p-Anisyl p-Tolyl p-Chlorophenyl


(iii) The phenyl group is more e- rich than (Me) group, therefore, (Ph) group migrates in preference
of (Me) group
(iv) The migrating group should be trans (anti) to the leaving (–H) group.
(v) The (–OH) group will be lost from the C atom which would leave the most stable carbocation.
(vi) The rate determining step (R.D.S. and slow) is the formation of stable carbocation, i.e., conversion
in step 2 to step 3.
T P Varun (JEE 2012, AIR 64)

Illustration 6: Arrange the following compounds in the decreasing order of their b.p s’ and solubility in H2O.
 (JEE MAIN)
(a) (I) Methanol (II) Ethanol (III) Propan-1-ol
(IV) Butan-1-ol (V) Butan-2-ol (VI) Pentan-1-ol
(b) (I) Pentanol (II) n-Butane (III) Pentanal
(IV) Ethoxy ethane
Chem i str y | 22.23

(c) (I) Pentane (II) Pentane – 1, 2, 3-triol


(III) Butanol

Sol: (c) B.P. order: VI > IV > V > III > II > I
Solubility order: I > II > III > V > IV > VI
Explanation: All of the mare alcohol so all have H-bonding. As the molecular mass and surface area increases, the
B.P. increases and solubility decreases.
Out of (IV) and (V), there is branching in (V) and has less surface are than (IV), So the boiling point of (IV) > (V), but
solubility of (V) > (IV)
(b) B.P.order : I > III > IV > II
Solubility order: I > III > IV > II
In (I), there is H-bonding, in (II) (aldehyde), dipole-dipole interaction, in (III) (ether), slightly polar due to EN of O
and in (IV) (alkane), Van der Waals interaction (non-polar)
(c) B.P. Order: II > III > I
Solubility Order: II > III > I
In (II), there (–OH) groups, more H-Bonding; in (II), one (–OH) group, less H-bonding; in (I) (alkane), Van der Waals
interaction

Illustration 7: Explain the following: (JEE MAIN)


(a) Which has higher B.P.?
(i) Phenol (ii) Benzenethiol
(b) Which has higher melting point?
(i) Hydroquinone (ii) Catechol
(c) Explain the less solubility and lower b.p. of :
(i) o-Nitrophenol (ii) o-Hydroxy benzaldehyde
(iii) o-hydroxybenzoic acid (salicylic acid) compared with their p-and m-isomers.

Sol: (a) Although the molecular mass of benzenethiol (Ph – SH) is higher, phenol has high boiling point. It is
because there is no H-bonding in PhSH.

OH (II) because of the


(b) Hydroquinone HO OH (I) has high M.P. than catechol
OH
Symmetrical packing of p- is its crystal
lattice which requires more energy for its
O H OH
melting. O
N C C
O O
(c) In ortho-isomers of (I), (II) and (III), H
intramolecular H-bonding (chelation) O O H O H
occurs which inhibits the intermolecular
o-Nitophenol o-Hydroxybenzaldehyde o-Hydroxybenzoic acid
attraction between these molecules and
(Salicylaldehyde) (Salicylic acid)
thus, lowers the b.p. and also reduces
H-bonding of these molecules with H2O thereby, decreases water solubility. Intermolecular chelation does not
occur in p –and m-isomers.
2 2 . 2 4 | Alcohols, Phenols and Ethers

Illustration 8: Synthesize the following: (JEE MAIN)


(a) Benzene to (4-chorophenyl)propan-1-ol)
(b) Ethyne to butanol
(c) Propane to allyl alcohol
(d) Propane to propanol and propan-2-ol

Synthesis:
(a) 3 2 1 2
OH
CH₂CH₂CH₂OH 3
1

or (I)

Cl Me Cl Me

CH₃Cl + AlCl₃ Cl₂ + Fe


F.C. alkylation

(o-,p-directing)
Cl
CH₂ MgCl CH₂ Cl (Major)
(i)
O Mg NBS
+ Reacts at
(ii) H₃O Allylic
benzylic
halide
Cl R Cl
CH₂CH₂CH₂OH

(I)

Cl

3 1
(b)
4
HC CH Me 2
OH 1-Butanol (I)
Ethyne(A)
NaNH₂ - CH₃CH₂Br
(A) HC C
1 mol
H₂ + Pd + BaSO₄
(Lindlar’s catalyst)
(C C)  (C C)
HC C CH₂CH₃
(i)BH₃/THF
H₂C CH CH₂CH₃ -
(ii)H₂O₂/OH
Anti-Mark

HOCH₂CH₂CH₂CH₃
(I)
Chem i str y | 22.25

(c) MeCH₂Me H₂C CH CH₂ OH


Propane (A) Allyl alcohol (I)
Cl
Cl₂/hv alc. KOH
(A) Me Me Me CH CH₂
-HCl
Allylic NBS
aq. NaOH
(I) BrCH₂ CH CH₂

OH (Propan-1-ol) (I)
(d) MeCH₂Me Me
OH
(Propane)
Me Me (Propane-2-ol) (II)
as in 1. B₂H₆/THF
(A) Me CH CH₂ - (I)
(f)
2. H₂O₂/OH
dil. H₂SO₄ Anti-Mark

Mark. add
(II)

Illustration 9: Complete the following: (JEE MAIN)

H
3 1
4
(A) Me O
2
But-2-cn-l-al
(a) (b) (c) (d) (e) (f)
LiAlD₄/ LiAlD₄/ NaBD₄/ NaBD₄/ D₂/Pt in D₂/Pt in
H₃O D₂O H₂O D₂O aprotic H₂O
solvent

I II III IV V VI
Me

(B) with reagents (a) to (f)


O
Mechanism:
D form LiAlD 4 and NaBD 4 isadded to C of (C = O)group 
 
and solvent gives H or D to O atom to form OH or OD,e.g.,

(i) (ii)

H AlH₃Li - H OH D AlD₃Li - H OH
R₂C O or R₂C O - R₂C O or R₂C O -
H BH₃Na -OH D BD₃Na -OH
H R₂C OH D R₂CD OH
R₂C OD
D OH
D
(iii) LAH and NaBH4 do not reduce (C = C) bond whereas catalytic hydrogenation reduces (C = C) bond to (C – C)
bond
2 2 . 2 6 | Alcohols, Phenols and Ethers

(iv) LAH and catalytic hydrogenation reduce epoxide but NaBH4 does not

H D D
Reagent H
Reagent in (b) and (d) in (c) OD H₂O
Me OH Me
in (a) and (c) Syn add. of
D
D₂ at (C C ) (V) D
(I) or (III)
H D
DH
Reagent
Me OD in (f)
Me OH
D (VI) D
(II) or (IV)

The D of ROD rapidly exchanges for the H of H2O Mechanism in (B).

Me -
H from LiAlH₄ or NaBH₄ H
-
Me H₂O Me
O D from LiAlH₄ or NaBH₄ CH₂ CH₂ H
- -
Nu attack at less hindered site by O D₂O OH
Sn² mechanism
Me
H₂/catalyst CH₂ H
(Or)
OD
D₂/catalyst Me D H₂O Me
Me H CH₂ CH₂D
Me -
CH₂ CH₂ D O OH
D₂O
OH OD Me
CH₂ D
OD

Reaction:
Reagent in Me
(B) CH₂D (VII)
(a)
OH
in (b) Me
CH₂D (VIII)
OD
in (c) and (d)
(B) No reaction

in (c) Me
(B) CH₂ D
OD H₂O
in (f) Me
CH₂ D
OH
The D of ROD rapidly exchanges for H of H₂O
Chem i str y | 22.27

POINTS TO REMEMBER

+
H /H₂O
( i) Hydroboration

( ( I) Oxirane
ii) Carbonyl
( R Mg X



ii) Oxymercuration Compds
C=C Grignard
Demercuration
Reagent
Alkenes

Preparation
R-OH
H₂
Reduction
Reduction
Fermentation i) R-C-OH


C=O
O Acid &
Organic
ii) R-C-X its
Carbonyl Compounds
Derivatives
Compounds O
iii) R-C-OR’

O
Reactions of Alcohols

o o NH₃ anhy.ZnCl₂
1 / 2 / 3o Amine R-Cl
Al₂O₃
H
+ CH₂N₂
C=C R-O-CH₃
[O ] SOCl₂
Aldehyde/Carboxylic acid R-OH
R-OH RCl
Pinacol Pinacolone PCl₅
RCl
Rearrangement
H₂SO₄(ex.)
Aldehyde/Ketone R-O-R



CH₂ CH₂
 
CH₂ CH₂



OR OR R O Na
O Na

R-OH  
R C H


  
NH₂ C O R
 
H N C
 
R C X R  C O R






O
O O O
R   R
C O


O
2 2 . 2 8 | Alcohols, Phenols and Ethers

Alcohol
GMP GR
HX or PX3 or PX5
HO (1) RX
(1) Alkene or Kl + H3PO4 or SOCl2 or SO2Cl2
aq. NaOH or aq. KOH RedP/Hl
(2) RX (2) RH
or aq. K2CO3 or moist Ag2O
dil. H2SO4 NH3 o o o
1 ,2 ,3 amines
(3) R-O-X (3)
dil. H2SO4 H2S
(4) ROOOR (4) R - SH Thiol
RCOOH Tho2
o HNO2 Na
(5) 1 amine (5) RONa
H2S
Exception-Methyl amine gives
CH3MgX
CH3-O-CH3 or ether (6) CH4
NaH R’ OR
(6) Aldehyde or ketone Darzon reduction ald. R’-CHO
(1o alc.) (2o alc.) (7) C Acetal
dry HCl
(7) Acid or Na/EtOH H OR
Acid derivative Bouveault Blanc reduction
o
(1 alc.)
o
(2 alc.) R-OH R’ OR
Ket one R’COR’
(8) C Ketal
dry HCl
(8) HCHO or Ald. or ketone RMgX R OR
(1o alc.) (2o alc.) (3o alc.) H2O R’COZ
(9) R’COOR ester (Z=OH, Cl, OCOCH3)
H2SO4
(10) ROSO2OH(Alkyl hydrogen sulphate)
O2
(9) RMgX
H2O (11) HNO3 RONO2(Alkyl nitrate)

H2O CH2 PhSO2Cl


(12) RSO2Ph(Alkyl benzene sulphonate)
CHCH
(10) CH3MgBr O (13) H2C-CH(OR)2 Acetal
H3O+ CH2N2
(14) R-O-CH3 Ether
O

Fermentation H2C CH2


(10) Sugar (15) RO-CH2-CH2-OH

CH2=C=O
(16) ROCOOH3 Ether

Formation of EtOH by fermention Dehydrofon


(17) Alkene
Crystallization
(1) Cane sugar Molasses Catalytic dehydrogenation
Sucrose (18) Aldehyde or ketone
Invertase Zymase 1 or2o alcohol, Cu or ZnO, 300oC
o

Invert sugar EtOH


Hydrolysis Fermentation
Exception -3o alc  Alkene
o [O] [O]
(2) Grain Starch
Diastase (19)1 alc. Aldehyde Acid (same no. of C-atom)
Maltose
HOH
o [O] [O]
Maltase Zymase (20)2 alc. Ketone
Glucose EtOH o [O] [O]
Hydrolysis Fermentation (21)2 alc. Ketone
o [O]
(22)3 alc.
-
(23)3o alc. OH,CrO4 No reaction (No. green colour)
(orange)
Chem i str y | 22.29

PHENOLS

1. INTRODUCTION
When OH group is attached at benzene ring,the compound is known as phenol
OH

Nomenclature of Phenols

OH OH OH OH
CH₃ H₃C CH₃

CH₃
2-Methyl phenol 3-Methyl phenol 2-6 Dimethyl phenol
CH₃
(o-Cresol) (o-Cresol)
2-Methyl phenol
(o-Cresol)
Some dihydric and trihydric phenols are given below:

OH OH OH
OH

OH
1,2-Benzenediol 1,3-Benzenediol
OH
(Catechol) (Resorcinol)
1,4-Benzediol
(Quinol)

OH OH OH
OH

OH OH HO OH
1,2,3-Benzenediol OH
(Pyragallol)
1,3,4-Benzenetriol 1,3,5-Benzenetriol

2. METHODS OF PREPRATION OF PHENOLS

2.1 From Haloarenes


- +
Cl ONa OH

123K HCl
+ NaOH 300 atm
2 2 . 3 0 | Alcohols, Phenols and Ethers

2.2 From Benzenesulphonic Acid


SO3H OH

Oleum (i) NaOH


(ii) H+

2.3 From Diazonium Salts


+ -
NH2 N2Cl NH2

(I) NaNO2 H2 O
+ N2 + HCl
(ii) +HCl Warm

Aniline Benzene diazonium


chloride

When diazonium salts react with water vapour it gives phenol.

2.4 From Cumene


When cumene (isopropylbenzene) is oxidized in the presence of air and acid, it gives phenol and acetone.
CH3 CH3

CH3 CH CH3 C O O H OH

Oleum +
H
+ CH3COCH3
H2O
Cumene Cumene
hydroperoxide

3. PHYSICAL PROPERTIES OF PHENOLS


(a) Pure phenols are generally colorless solids or liquids. The light colour usually associated with phenols is due
to its oxidations by air in presence of light.
(b) Phenols, generally are insoluble in water; but phenol itself, and polyhydric phenols are fairly soluble in water
which is believed to be due to the formation of hydrogen bond with water.
(c) Due to intermolecular hydrogen bonding, phenols usually have relatively high boiling points than the
corresponding hydrocarbons aryl halides and alcohols. For example, phenol (mol. Wt. 94) boils at 182ºC while
toluene (mol. Wt. 92) boils at 110ºC.
Higher b.p. than alcohols is due to higher polarity of the O-H bond and consequently stronger intermolecular
hydrogen bonding in phenols than in alcohols. Appreciable solubilities of the phenol and polyhydric phenols
in water is also due to strong hydrogen bonding between phenols and water molecules.
Chem i str y | 22.31

O
Ar Ar H
O O
O H Ar O H O
H H H N
Intermolecular hydrogen Hydrogen bonding between o-Nitro phenol (Intermolecular
bonding phenols phenols and water molecules H-bonding possible due to
close mass of NO2 and -OH
groups)

Phenols containing groups like-NO2 or –COOH in the ortho position to the –OH group can also form
intermolecular hydrogen bonds (e.g. o-nitro phenol) which is responsible for their lower boiling points and less
solubility in water than the corresponding meta or para isomer. Due to possibility of intermolecular hydrogen
bonding (also known as chelatom) in the ortho isomer, intermolecular hydrogen bonding is not possible and
hence the ortho isomer can neither get associated nor can from hydrogen bonding with water with the results
it has a low b p. and less solubility in water than the meta and para isomers which can associate (union of two
or more molecules of the same speoins) as well as can form hydrogen bonding with water.

O H O O H
OH O
H
N

N
N
O O O H
O O H H
O
p-Nitro phenol (1 molecules) Hydrogen bonding
(intermolecular H-bonding) is not between p-nitro and
possible due large distance between-NO2 water
and -OH occups hence intermolecular
H-bonding is possible.

(d) They possess characteristic colour. They are highly toxic in nature and possess antiseptic properties. They may
produce wounds on skin.
(i) Phenol exists as resonance hybrid of the following structures.
+ + +
OH OH OH OH
- -
Mirror Image of I

-
I II III IV
Due to resonance oxygen atom of the –OH group acquires & positive charge (see structures III to V) and
hence attract electron pair of the O–H bond leading to the release of hydrogen atom as proton.

O H O

+ H+

Phenol Phenoxide ion


2 2 . 3 2 | Alcohols, Phenols and Ethers

Since resonance is not possible in alcohols (due to absence of conjugation of the lone pair of electron of
oxygen with a double bound), the hydrogen atom is more firmly linked to the oxygen atom and hence
alcohols are neutral in nature.
(ii) Once the phenoxide ion is formed, is stabilizes itself by resonance, actually phenol acid ion is more stable
than the parent phenol.
O O O O
- -
Mirror Image of V

-
V VI VII VIII

Comparison of acidity of phenols and carbonic acid


Relative acidity of the various common compounds.
RCOOH > H2CO3 > C6H5OH > HOH > ROH
Carboxylic acid Carbonic acid Phenol Water Alcohols.

4. CHEMICAL PROPERTIES OF PHENOLS

4.1 Nitration
(a) When phenol react with dilute nitric acid at low temperature (290 K), give a mixture of ortho and para nitro
phenols. OH OH OH
NO₂
Dilute HNO₃
+

o-Nitrophenol NO₂
p-Nitrophenol

(b) When phenols react with concentrated nitric acid, it gives 2, 4, 6-trinitrophenol.

OH
OH
O₂N NO₂
Conc.HNO₃

NO₂
2,4,6-Trinitrophenol
(Picric acid)

4.2 Halogenation
(a) When the reactions carried out in solvents of low polarity such as CHCl3 or CS2 and at low temperature,
monobromophenols are formed.
OH OH OH
Br
Br₂ in CS₂
273 K +
Minor Br
Major
Chem i str y | 22.33

(b) When phenol is treated with bromine water 2, 4, 6-tribromophenol is formed as white precipitate.
OH
OH
Br Br

+ 3Br₂

Br
2,4,6-Trinitrophenol

4.3 Kolbe’s Reaction


OH ONa OH
(i) CO2 COOH
NaOH
+
(ii) H

2-Hydroxybenzoic acid
(Salicylic acid)

Mechanism of Reaction

O H
H O OH
+- O H - O OH
Na O C O
tautomerisation + O
O O Na+ H3O
Na+
O Salicyclic acid
Sodium salicylate

4.4 Reimer-Tiemann Reaction


On treating phenol with chloroform in the presence of sodium hydroxide, a–CHO group is introduced at ortho
position of benzene ring. This reaction is known as Reimer – Tiemann reaction. The intermediate substituted benzal
chloride is hydrolyzed in the presence of alkali to produce salicyladehyde.

- +
- +
OH O Na O Na OH
CHO + CHO
CHCl3 + aq. NaOH NaOH H

Intermediate Salicylaldehyde

The mechanism of the Reimer – Tiemann reaction is believed to involve the formation of dichloromethylene.
NaOH + CHCl3 → :CCl2 + NaCl+ H2O

O Cl O Cl O
O O
Cl H Cl
CCl2 H H2O +H
+ Cl OH- O
-
OH ;H O Cl H2O
2
2 2 . 3 4 | Alcohols, Phenols and Ethers

Phenols with blocked p-positions give cyclohexadienones containing the dichloromethyl group.
OH OH O

NaOH O
CHCl3

CH3 CH3 H3C CH3

In the Reimer-Tiemann reaction, the o-isomer predominates, but if one of the o-position is occupied the aldehyde
group tend to go to the p-positions; e.g. guaiacol forms vanillin
OH OH
O O
CH3 NaOH CH3
CHCl3

O
4.5 Libermann’s Reaction
When phenol is treated with sodium dissolved in conc. Sulphuric acid a red colouration appears which changes to
blue on adding aqueous NaOH. This reaction is called Libermann’s reaction.

2NaNO2 + H2SO4 Na2SO4 + 2HNO2


Nitrous acid

HO H + H - O - N=O HO N=O O= = N - OH
-H2O
p-nitrosophenol

H- -OH
NaOH - +
O= =N HO O= =N ONa
-H2O -H2O
Blue
Indophenol (Red)

4.6 Reaction of Phenol with Zinc Dust


When phenol is heated with zinc dust, it gives benzene.  OH

+ Zn + ZnO

4.7 Oxidation
Oxidation of phenol with chromic acid produces a conjugate diketone known OH O
as benzoquinone. In the presence of an oxidizing agent, phenols are slowly
Na2Cr2O7
oxidized to dark coloured moisture containing benzoquinone.
H2SO4

O
Benzoquinone
Chem i str y | 22.35

5. DISTINCTION BETWEEN ALCOHOL AND PHENOLS


(a) Phenols turns blue litmus red but alcohols do not.
(b) Phenols neutralize base, while alcohols do not.
OH ONa

+ NaOH + H2O

R-OH + NaOH No reaction

(c) Phenols give violet colour with FeCl3. while alcohols do not.

 
OH OH

3 + FeCl3 Fe + 3HCl

Violet
R-OH + FeCl3 No reaction

Illustration 1: Identify the major products in the following reactions:


NO2 OH
CH3
HNO3 Bromine
(I) A (II) B
H2SO4 Water
O

CH3
Sol: (I) The nitrating mixture gives the attachment of the nitro group on the ortho position. The presence of
methoxy group is an electron-donating groupwhich makes the ortho position more electron-rich enabling the
attachment of the electron-withdrawing NO2 group.
(II) Bromine is an electrophile and the presence of electron donating groups i.e. –OH and CH3 make the ortho and
the para positions available for the attachment.

CH3 Br
O OH
A= O2N B=
Br CH3
NO2
2 2 . 3 6 | Alcohols, Phenols and Ethers

PLANCESS CONCEPTS

•• Phenols although colourless turn reddish due to atmospheric oxidation.


•• Phenols and alcohol have high boiling point due to intermoelcular hydrogen bonding
•• Out of three isomeric nitrophenols, only ortho isomer is steam volatile and has lesser solubility and
lower boiling point than meta and para. Ortho cannot form H-bond with water and in ortho there
exist intramolecualr H- Bonding.
•• Phenols are stronger acids than alcohols but weaker than carboxylic acid and carbonic acid.
•• Phenols are stronger acids than alcohols because the phenoxide ion formed after the release of
proton is stabilised by resonance where as alkoxide ion does not.
•• TEST OF PHENOL– Phenols give violet colour with neutral FeCl3. Depending upon the nature of Phenol,
colour varies from violet to blue green or even red.
•• Preparation of phenol from cumene proceeds via peroxide radical mechanism.

Saurabh Chaterjee JEE Advanced 2013, AIR

POINTS TO REMEMBER

MgBr SO3Na
O2 NaOH;Fusion
H2O OH H+/H2O
+
OH N2Cl-
COOH Phenol
NaOH + CaO Preparation Warm H2O

H3O+ 2NaOH
300oC
Cl
HCl
H3C - CH- CH3 1/2 O2
O2
O
O Raschig’s
H process
H3C - CH- CH3
Cumene
Riemer Tiemann Reaction
ONa
343 K
OH Kolbe
CHCl3 H3CO NaOH
NaOOC Schmidt NaOH CH4Cl Na ONa
CO2; NaOH CO CHO -H2
473 K Miscellanous O Cleavage
O
CO CH3COCl of O-H
4-100 atm Phenolphthalein H3C-C
OH OH OH Conc NaOH OH
HOH2C indicator NaOH
H2SO4 ONa
HCHO OH -H2O
HCl +HCN O
H2SO4
AlCl3 Et-C C2H5COCl
CH2OH Schotten
NaCl
Bakelite Baumann
CHO
Reaction
Gatteramann
Reaction
Reaction
OH OH of
H3COCO OH
+ OH OH NH3
CH3COCl Br2 +
Br ZnCl2 Zn
OCOCH3 Br 573K Cleavage
OH OH OH
NO2 NO2 HOS3 Br OH Br of C-O
HNO3 Substitution Br2 Cl
+ OH
H2SO4 OH PCl2
NO2 SO3H Br P2S5
F.C. H2SO4
OH OH OH OH HS -P2O5
H3 C +
+ SO3H
SO3H
CH3
Chem i str y | 22.37
2 2 . 3 8 | Alcohols, Phenols and Ethers

ETHERS

1. INTRODUCTION
The ethers are those compounds that have a C–O–C in their structure where, importantly, each C can only be part
of an alkyl or an aryl group – i.e. R–O–R’. The electronegative oxygen, flanked as it is by two electron pushing alkyl
groups, has very little tendency to participate in any reaction. This lack of reactivity is also attributed to the two alkyl
groups enveloping the oxygen, shielding it from reagents. The ether molecule appears to have an outer unreactive
alkyl shield or sphere with the “reactive” oxygen sitting in the centre.
 
Without any hydrogens directly attached to the oxygen, the molecules are H H
not capable of forming H-bonds. The consequence of this is that the melting
and boiling points are lower than the corresponding alcohols. Compatibility /   
solubility with water is also affected; though the smallest ether is miscible with R C O C R
water, any increase in the size of the alkyl chain drastically lowers the ether’s
solubility in water and soon forms immiscible mixtures.
H H
 

2. METHODS OF PREPARATION OF ETHERS


(a) Williamson’ Synthesis: Heating of alkyl halide with sodium or potassium alkoxide gives ether. This is a good
method for preparation of simple as well as mixed either.
R
R − X + Na − O − R ' → R − O − R '+ NaX

This method is not applicable to tert alkyl halides because the alkoxide ions being both powerful nucleophiles
and bases could bring dehydrogenation of the tertiary alkyl halides to form alkenes.
R - ONa R - O- + Na+

R O Na + R’ X R O R + Nax

R’ O Na R O R
R X+
Ar O Na R O Ar
Aryl Ether

aq. NaOH
- OH + CH3 - CH2 - Br - O - CH2 - CH3

The reactivity of primary (1º) alkyl halide is in the order CH3- > CH3 – CH2- > CH3 – CH2 – CH2- and the
tendency of the alkyl halide to undergo elimination is 3º > 2º > 1º. Hence for better yield the alkyl halide
should be primary of the alkoxide should be secondary or tertiary.

C2H5Br + NaO - C - C2H5 - O - C + NaBr

(b) By Heating excess of alcohols with conc. H2SO4 e.g.,


conc. H2SO4
C2H5 - OH + HO - C2H5 C2H5 - O - C2H5 + H2O
o
Ethanol (2 molecules) 140 C Diethyl ether

Recall that 2° and 3° alcohols under the above conditions give alkenes as the main product. Moreover, this
method is limited only for the preparation of simple ethers.
Chem i str y | 22.39

(c) By heating alkyl halide with dry silver oxide (only for simple ethers)

C2H5l + Ag2O + lC2H5 C2H5OC2H5 + 2Agl

Remember that reaction of alkyl halides with moist silver oxides (Ag2O + H2O = AgOH) gives alcohols.
(d) By the use of diazomethane to form methyl ethers.
BF3
n-C7H15OH + CH2N2 n-C7H15OCH3 + N2
Methyl n-heptyl ether
BF3
n-C7H15OH + CH2N2 C6H5OCH3
Anisole

3. PHYSICAL PROPERTIES OF ETHERS


(d) Due to absence of intermolecular H-bonding, B.P of ether is much lower than isomeric alcohols.
(e) Ethers are slightly polar with some net dipole. (e.g. 1.18 D for diethyl ether.)This is due to a bend structure with
bond angle of 1100 which causes because of repulsion between bulky alkyl groups.

4. CHEMICAL PROPERTIES OF ETHERS


Ethers are less reactive than compounds containing other functional group. They do not react with active metals
like Na, strong base like NaOH, reducing or oxidizing agents.

4.1. Formation of Peroxides


On standing in contact with air, ethers are overrated into unstable peroxides (R2O → O) which are highly explosive
even in low concentrations. Hence ether is always purified before distillation. Purification (removal of peroxide) can
be done by washing ether with a solution of ferrous salt (which reduces peroxide to alcohols) or by distillation with
conc. H2SO4 (which oxidizes peroxides).
The presence of peroxides in ether is indicated by formation of red colour when ether is shaken with an aqueous
solution of ferrous ammonium sulphate and potassium thiocyanate. The peroxide oxidizes Fe2+ to Fe3 which reacts
with thiocyanate ion to given red colour of ferric thiocyanate
CNS −
Peroxide + Fe2+ → Fe3+  → Fe (CNS)3

Red

However, the formation of peroxide is prevented by adding a little Cu2O to it.

4.2 Basic Nature


Owing to the presence of unshared electron pairs on oxygen, ethers are basic, Hence they dissolve in strong acids
(e.g., HCl, conc. H2SO4 ) at low temperature to form oxonium salts.

(C2H5 )2 O + H2SO 4 → [(C2H5 )2 OH]− HSO −4


Diethyl ether Diethyloxonium
hydrogen sulphate

On account of this property ether is removed from ethyl bromide by shaking with conc. H2SO4. The oxonium salts
are stable only at low temperature and in a strongly acidic medium. On dilution, they decompose to give back the
original ether and acid.
2 2 . 4 0 | Alcohols, Phenols and Ethers

Ether also form coordination complexes with Lewis acids like BF3, AlCl3 RMgX, etc.
R2O R
R2O + BF3 R2O BF3 (b) R2O + RMgX Mg
R2O X
It is for this reason that ethers are used as solvent for Grignard reactions.

4.3 Action of Dilute H2SO4 (Hydrolysis)


dil. H2SO4 heat
C2H5-O-C2H5 2C2H5-OH
Pressure

4.4 Action of Concentration H2SO4


Heat
C2H5-O-C2H5+H2SO4(conc.) C2H5OH + C2H5HSO4

4.5 Action of Conc. HI or HBr.


(i) C2H5-O-C2H5+HI(cold) C2H5-OH + C2H5+I
(ii) C6H5-O-C2H5+HI C6H5OH + C2H5I

Mechanism of reaction: SN2 and SN1 mechanisms for the cleavage of ethers. SN2 cleavage occurs at a faster rate
with HI than with HCl.

Step 1 : R O R’ + Hl + -
O +I
R R’
base1 acid2 acid1 base2

H
+
- O
Step 2 for SN2 I + R R’ slow o
Rl + HOR’ (R is 1 )

Step 3 for SN2 R+ + I-


o
Rl (R is 3 )

(a) The transfer of H+ to ROR’ in step 1 is greater with HI, which is a stronger acid, than with HCl Furthermore, in
step 2, I, being a better nucleophile than Cl+, reacts at a faster rate.
Chem i str y | 22.41

PLANCESS CONCEPTS

•• Boiling point of ethers is lower than alcohol due to absence of hydrogen bonding.

+
HCl (conc) R-O-R
-
H Cl oxonium salts

BF3 R
O BF3
R
R
RMgX R R
O Mg O
R R
X
HI (excess)
2RI +H2O

HI
ROR R-OH +R - I

dil. H2SO4
2ROH

PCl5
2RCl+ POCl3

R’COCl
R’ -COOR

Cl. light
First - hydrogen gets halogenated

•• In reaction with HI, if cold and dilute HI solution is treated with ether, alcohols are formed while in hot
and concentrated HI, alkyl halides are formed.
•• The reaction mechanism in case of HI depends on the substrate. If the substrates attached to oxygen
are 10 or 20 ,then the mechanism is SN2 but if the substrate is 30 or the carbocation is very stable then
the mechanism is SN1.
Nikhil Khandelwal (JEE 2009, AIR 94)

Illustration 1: How are the ethers distinguished from alcohols? (JEE MAIN)

Sol: (i) All alcohols give CH4 (methane gas) when reacted with MeMgBr.

CH₃O H + Me MgBr CH₄ + CH₃OMgBr


o
1 alcohol
Me Me
O H + Me MgBr CH₄ + OMgBr
Me Me
o
2 alcohol

Me Me
O H + Me MgBr CH₄ + OMgBr
Me Me
Me Me
3o alcohol
2 2 . 4 2 | Alcohols, Phenols and Ethers

(ii) K2Cr2O7 in acid has bright orange colour. When it oxidizes 1º or 2º alcohol, it is reduced to blue green due to
the formation of Cr3+.

2-
MeCH2OH + Cr2O7 + H
o
1 alcohol Orange
colour
( (
3+
Me COOH + Cr + H2O
Acetic acid (blue-green)
Me - + Me
OH + Cr₂O₇² + H +
O + Cr³ + H₂O
Me o
Me
2 alcohol

(iii) All alcohols evolve H2 gas on addition of sodium (Na).


(iv) Dry ethers give negative test with all the reagents (a, b and c).

Illustration 2: Complete the following reaction:  (JEE MAIN)


Ph
H+
Ph O Ph
Ph
Sol:

Ph
H+ Ph
Ph O Ph HO +
Ph
Ph Ph
SE reaction (More stable3oC+ )
o

at p-position (Acts as electrophile)


of phenol
Ph
HO Ph
Ph
Mainly para isomer, no ortho isomer due to
steric hindrance of bulkyl Ph₃C gp.

Illustration 3: There are two paths for the preparation of phenyl-2, 4-dinitro phenyl either (C). Which path is
feasible and why? (JEE ADVANCED)

Sol:
Br Br
NO₂ PhONa
Dinitration
Path I: Ac₂O+N₂O₅

(A)
NO₂
(B) NO₂
Br
O NO₂

Path II: +PhONa (C)


(D)
(A)
O Dinitration
(E) Ac₂O+N₂O₅
Chem i str y | 22.43

a. Path I is feasible. ArSN reaction (Williamson’s synthesis) of nucleophile PhOΘ with (B) is feasible. Also, Br of (B)
is activated by the two EWG (–NO2) groups.
b. Path II is not feasible. ArSN reaction of the nucleophile PhOΘ with (A) is not feasible because no activating
group is present in (A).
c. Dinitration of (E) does not give (C) but it gives because the first nitro
group is deactivating so that second nitro group enters the other ring
at p-position.
( O₂N O NO₂ (
Illustration 4: Complete the following: (JEE ADVANCED)

C₂H₅OH O
a. A B
H+ H+
O

+
H₂O/H O
b. C D
O H+

CH₃NH₂ O
c. E F
O H+ H+

Sol:
a.

b.

c.

Illustration 5: Complete the following reaction: (JEE MAIN)


OH
(i)NaOH
D D (i)NaOH
(Major) (B) (ii)CO₂
(ii)CO₂
(iii)D + (iii)H +

(A) (Major)

OH O-
(i)NaOH
D D (i)NaOH
(Major) (B) (ii)CO₂
(ii)CO₂
(iii)D + (iii)H +
2 2 . 4 4 | Alcohols, Phenols and Ethers
(A) (Major)
Sol:
OH O-
O
D D D D
NaOH D D

OD O
- O
D COO D
-
C O C O

H+ D+ O
OD
OD
D COOD
D COOH

(B)
(A)
Last image is (A) not (D)

Illustration 6: Complete the following reactions:  (JEE MAIN)


OH OH
D D D D
CHClBrI CBr₂l₂
(B) (Major) (ii) - (D) (Major)
(i) EtO
-
OH

D D
(A) (C)

OH
D CCl₂BrI + OH
-
CHClBrI (iv) (E) (G)
(iii) - (F) H₃O
+
t-BuO

(E) NH₂

Sol: Cl
(a) EtO
-
-
H C Br EtOH + CClBrl
(Base) I breaks
Acidic
CClBrl
Chloro bromo
carbene

O O
D D D D
-
CClBr CClBr
-
e-deficient
acts as electrophile
D D
(SE reaction)
-
OH O
D CD O D CDClBr
(i)+H +
(ii)+H₃O +

D D
(o-isomer, major)
Chem i str y | 22.45

(b)
a. CBr₂l₂
4NaOH -2H₂O C O
a. CBr₂l₂ 2NaBr + 2Nal + C(OH)₄
4NaOH -2H₂O C O
2NaBr + 2Nal + C(OH)₄ O
O
This produces CO2 ,the reaction is Kolbe reaction.
O O
O
b. O O
b. D DO D D
D D D
SE reaction D -
++
CC O O COO COO
- SE reaction -
-

DD
(C) OD
(C) OD OD OD
-
DD COOH D D COO COO-
COOH H₃O +
H₃O +

(D)
(D) D D
(o-Isomer D
is major) D
(o-Isomer is major)
Cl
- -
Me₃CO H C BrCl Me₃COH+ CClBrI
c. (Base) - I -
Me₃CO H C Br Me₃COH+ -ICClBrI
-
c. (Base) Acidic CBrCl
I
-
+ − -I
Acidic CBrCl
( − N ≡ C −)
(c) Carbene also converts (– NH2) group to (Carbylamine reaction) and also adds to (C = C) bond of
cyclopentane ring and undergoes Reimer-Tiemann reaction at o-position w.r.t. (–OH) group in benzene ring.

OH OH Bond breaks
4
D O C 5
CBrCl 3

D 6
Br
1 2
+ -
NH₂ N C Cl
(E)
OH
4
O CD 5
3 Ring ecpansion
Weak C-Br
6
1
2 Cl bond breaks
+ - -HBr

( (
N C
Numbering in accordance
with problem
understanding
(F)

(d) CCl₂BrI 4NaOH


2NaCl + NaBr + Nal
-2H₂O
+C(OH)₄ CO₂

OH OH
O
D HOOC
+ C O

NH₂ NH₂
(E) (G)
Ph Me
2 2 . 4 6 | Alcohols, Phenols and Ethers Ph H
HNO₂
(B)
-H+
(C)
(A)
OH NH₂
Illustration 7: Complete the following reactions: (JEE ADVANCED)
O
Ph Me
HNO₂ -H+ CH₃NO₂ LAH HNO₂
Ph H (B) (C) (B) (B) (C) (D)
(A) EtO - -H
+
OH NH₂ (Aldol type)
(A)
O
Sol: CH₃NO₂ LAH HNO₂
(B) (B) (C) (D)
(A) EtO - +
-H
Ph(AldolMe
type) Ph Me
(A) HNO₂
Ph H Ph H
Ph Me Ph Me
HNO₂ -N₂
Ph OH NH₂H Ph OH NH N
-N₂
OH(A)NH₂ OH N (B)
N
(A) (B)
Ph Ph Me
+ Ph migrates
Ph Ph Me PhPh Me+ H
+ Ph migrates
HPh O H Me Ph OH + H
H O H OH
+
-H
+
-H
Ph Ph
Ph Ph
O Me
H Me
O H
(C)
(C)
-  -
EtO- H CH₂ NO₂ EtOH + CH₂NO₂
-
(B) EtO H

CH₂
-H NO₂
atoms are EtOH + CH₂NO₂
-H atoms
acidic due to are
-lacidic
effectdue
of NO₂
to
-l effect of NO₂


O
HO CH₂NO₂ HO CH₂NH₂
-
CH₂NO₂ [H]
LAH
(A) (B) (C) HNO₂

+ 1 +
HO CH₂ HO CH₂ N N
Ring 7
-N₂
6 2
expansion
5 3
4

O
H O
+
7 1 + 7 1
2 -H 2
6 3 6 3
5 4 5 4
(D)
(cycloheptanone)
Chem i str y | 22.47

POINTS TO REMEMBER
R’ONa moist
R-X
Preparation Ag₂O

conc. R-O-R’ BF₃


2R-OH
H₂SO₄ CH₂N₂

dil. air
2R-OH Peroxide
H₂SO₄ Reactions
+
conc. R-O-R’ H
R-OH+RHSO₄ Oxonium salts
H₂SO₄ Low
HI/ temp.
conc.
HBr
R-OH + R’-I/R’-Br
2-
Ag CO3
(A) Me3C Br AgBr + Me3C 2-
Ag CO3
(A) Me3C Br AgBr + Me3C O
Me3C -CO2C O
Me3C O Me3C O C O
Solved Examples
Me C 3
Me3C O
-CO2C
Me3C O C O
Me3C O CMe3
Me3C (A)
O CMe3
(A)
JEE Main/Boards H
(B)b. Me OH Me + Me CH = O
-H
H 2O
(B) Me OH Me + Me CH = O
Example 1: Complete the following reactions: -H2O
Cl
Me CH OMe Me CH OMe
Ag CO
2 3 → (A)
a. 2Me3C− Br 
∆ Me CH OMe
Cl
Me CH OMe
Cl
b. 2MeOH + MeCH =
O + HCl (g) → (B) ( Chloroether
Cl )
 Chloroethyl
( Chloroether) ether
methyl
c. MeOH + H2C =O + HCl (g) → (C)
 Chloroethyl methyl ether
(C) CH =O MeOH CH OMe ( Chlorether dimethyl ether)
Sol: A = Me3 C – O – Me3 (Di-t-butyl ether)
2 2
+ HCl
(C)c. CH =O MeOH CH OMe ( Chlorether dimethyl ether)
2 2
+ HCl Cl
Ag+ reacts with Br− leaving Me3CΘ ,which reacts with
Cl
CO32– to give Me3C – OCO2Θ . The latter loses CO2 leaving
Me3COO which reacts with Me3C⊕ to give the product.
Due to steric hindrance, the yield is less.
Example 2: Complete the following reactions:
Mechanism:
a. C6H6 + MeCOCl
AlCl3 HNO3 NaBH
4 H SO
B C D 2 4 E
2- AlCl3 H2SO
HNO 4 NaBH4 H SO
(A)a. Me3C
Ag CO3 Benzene
C6H (A)
6 + MeCOCl B 3
C D 2 4 E
Br AgBr + Me3C H2SO4 
Benzene (A)
O NaBH4 H2SO4 HNO 3
(B) F G H
4 H2SO
b. (B) NaBH4
Me3C -CO2C H2SO HNO 3
4

Me3C O Me3C O C O F G H
 H2SO4

Me3C O CMe3
(A)

H
(B) Me OH Me + Me CH = O
-H2O

Cl
Me CH OMe Me CH OMe

Cl
( Chloroether)
 Chloroethyl methyl ether
2 2 . 4 8 | Alcohols, Phenols and Ethers

Sol: \ DTf = i Kf × M ; 3x × 0.1 = 0.3x K


(b) Ph3C– OH + H2SO4 reacts with cold, conc. H2SO4 as
O
follows:
AlCl3 Me
a. C6H6 + MeCOCl
F.C. Nitration 1. Ph3+COH + H2SO4 Ph3C + H2O + HSO4
(A) acylation (B) m-directing
Acetophenone 2. H2SO4 + H2O H3O + HSO4
OH O
H2SO4,  Ph3COH + H2SO4 Ph3C + H3O + 2HSO4
3 Me NaBH4 Me
-H2O 2
1 Number of moles of particles formed per mole solute
NO2 NO2
(D)
(i) (Van’t Hoff factor) = 4
(C)
NO2
m-Nitroacetophenone (The reaction produces stable Ph3C⊕ ion due to resonance
(E) stabilization, and Ph3C⊕ ion, and Ph3C⊕ persists in the
solution.)
m-Nitrostyrene
\ DTf = i Kf × M ; = 4x × 0.2 = 0.8x K
OH

b. (B)
NaBH4 Me H2SO4,  Example 4: Complete the following reactions:
-H2O

(F) (G) H
B
Styrene a. HO
o-and p- (A)
Nitration
Directing Me
b. + BF3
OH (C)
Me
(A) Me
O2N (B)
(H)
(Major)
p-Nitrostyrene
Sol:

a. B 

Example 3: (a) Calculate the depression in freezing


O Me
point (DTf ) of 0.1 m solution of ROH in cold conc.
H2SO4. Kf = K kg mol–1. Mechanism:
(b) Calculate the DTf of 0.2 m soln of Ph3 C – OH in cold
conc. H2SO4 . Kf= K kg mol–1. 1,2H
+H
Shift
Sol: (a) ROH reacts with cold conc. H2SO4 as follows: OH HO Me H O Me
+H

1. ROH + H2SO4 ROH2 + HSO4

ROSO2 OH + H2O
O (B) Me
2. H2SO4 + H2O H3O + HSO4
ROH + 2H2SO4 ROSO2OH + H3O + HSO4
Me
Me
C
Number of moles of particles formed per mole of solute b.
Me
(i) (van’t Hoff factor) = 3 The reaction does not produce
R⊕ , because R⊕ ion or even R3C⊕) ion is not stable
enough to persist.
Chem i str y | 22.49

Mechanism: Sol: (a) One Du in (A) and unreactivity with cold alk.
KMnO4 (Baeyer’s reagent) suggest (A) to be a ring
Me Me compound. (A) is optically active, suggesting a trans
OH + BF3 O BF3 expoxide.
Me Me
Me Me - HOBF3
H
Me Me H Me
(B)
1,2-Me + Me Me (or) H Me
Me Me CH2 H H Me H
shift (or)
Me Me H O H Me O H
(A) Me
Me cis-Butene-2-oxide
O trans-Butene-2-oxide
O
Me
Me Achiral (plane of symmetry) trans-Butene-2-oxide
cis-Butene-2-oxide Chiral (A)
Achiral (plane of symmetry) Chiral (A)
Me OH
H Me H OH Me
(C)
H Me H H Me
Me H Me H
H O
O H
Example 5: (a) Write the reaction of EtOH with (i) KNH2 Me H Me
Me O
(ii) aq. KOH (iii) Potassium ethynide. O
(A) H
(A) Me
(b) Complete the following reaction: H OH H trans compound and
H OH trans (anti)
trans additions
compound and
and
OH H OH
product is meso
trans (anti) additions and
H Me OH
Ph3CBF4 product is meso
Me Me (B) Meso-2,3- Dihydroxy
Me butane
OH
(A) (Achiral)
Meso-2,3- (B) butane
Dihydroxy
Ph3CBF4
Me Me (B) (Achiral) (B)
Sol: a.(A) I. K NH2 + EtOH NH3 + EtO K
(b) Zero DU is (C) and (D) suggests that both are saturated
a.
I. K NH2 + EtOH NH3 + EtO Kcompounds; (C) can be either diol or containing one
II. K OH + EtOH H2O + EtO K (OH) and one (OMe) group since only one mole MeI
II. K OH + EtOH H2O + EtO K reacts with (C) (five C atoms) to give (D) (six C atoms).
III. HC  C K + EtOH HC  CH + EtOCompound
K (C) contains one (OH) and one (OMe) given
III. HC  C K + EtOH HC  CH + EtO K at adjacent positions to make (C) chiral.
OH OH
OH OH 1
Me Me
b. Me Me Me (CH2)4 Me 2
b. Me H Me Me (CH2)4 Me H OH Mel + Na H OH Plane of
H CPh3
CPh3 Williamson3 symmetry
O H H H OMeOMe
O H RX  1o
4
Me RONa  2 o
Me
Me Me(CH2)4 Me (CH2)4 Me Meso-2,3-Dimethoxy butan
O O
(C) (Chiral) Achiral (D)
+ BF4 + BF
+Ph +Ph3CH(3-Methoxy butan-2-ol)
4 3CH
HBF
HBF +
4 4 +
MeMe (CH(CH
2 4 2)4
) Me Me

Example 6: Identify the following compounds: Another possibility is;


Me
Cold alk. KMnO4 Me Na
a. C4H8O (A) No reaction OMe
2 O
Chiral HO * OMe
1 3
C4H10O2 (B) H Me - l
2-Methyl-3-methoxy o Williamson
H3O Achiral 1 RX
propane-1-ol
(C2) (Chiral) Me
b. Na + Mel 2
C5H12O2 (C) C6H14O2 (D) MeO OMe
Chiral Achiral 1 3
H
Achiral (D1)
2-Methyl 1,3-dimethoxy
propane
2 2 . 5 0 | Alcohols, Phenols and Ethers

Example 7: Give the stereochemcial product of Example 8: Convert benzene to the following
following reactions: compounds:
OH a. SOCl2/Py O Me
Cl B Me
a. b.
b. SOCl2
O2N NO2 OMe
C
(B)
c. PBr3 (A)
H
D
Me Me
TsCl Sol: Williamson’s synthesis of (I) and (II) would take
cis-4-Isoprophy d. E place since ArSN is feasible in (I) because (X) is (I) is
Cyclohexanol activated ty two (–NO2) groups. Synthesis (I) from
(A) e. TsCl followed by Br benzene and then react with (II) to obtain the product.
F
O X
Sol: ROH with SOCl2 gives RCl (with retention
Me
a. + NaO
Me
configuration) but with SOCl2/pyridine, RCl is found O2N NO2 O2N NO2
with inversion of configuration (SN1 reaction) (A)
(I) (II)
O X
Me
H Cl H a.
+ NaO
Me
Cl H Br O2N NO2 O2N NO2
(I) (II)
a. b. c.
(A)

Williamson’s synthesis of (I) and (II) would take place


H H H
since ArSN is feasible in (I) because (X) is (I) is activated
Me Me Me
Me Me Me ty two (–NO2) groups. Synthesis (I) from benzene and
(B) (C) (D) then react with (II) to obtain the product.
trans-(Inversion) cis-(Retention) trans-(Inversion) (b)
(C-O) bond breaks Cl Cl

d. cis-Tosylate, no change in configuration because NO2


Cl2/Fe Dinitration
none of the (C–O) bonds breaks.
HNO3 + H2SO4
O
Me NO2
R O H + Cl S Me O
ONa
O Me
NO2
p-Toluene sulphonyl
(A)
chloride
O - Ts (TsCl)
Cl O NO2

R O S Me Me
Ph
O (Path I) + Me
H OMe X Alkoxide 1
o

Williamson’s
Me Me (or) RX is 2 o
(B) synthesis
(E) (R O Ts) Me
Ph
(cis-Tosylate) + MeX
Williamson’s o
ONa 1 RX
feasible o
2 Alkoxide
H
e. Me
Ph
Br -H Alkoxy
(Path II) + MeOH
TsCl Br +H mercuration-
(A) (E) OMe
demercuration
(C O) bond (B) Me
feasible Ph
breaks (Inversion) H + MeOH
not feasible
Me Me
(F)
trans
Chem i str y | 22.51

Example 9: Complete the reaction:  Example 10:


OH
OH
-
CH3O/anhyd. CH3OH
- + C2H5I
C2H5O/ anhyd. C2H5OH
+ C2H5l
(B)
(B)
(A) NO2
(A)
Sol: C2H5O- acts as a base. It abstracts H+ from phenol i. P-NO2-C6H4-OC2H2
to form PhO- ion. ii. P-NO2-C6H4-O-NO2-P
C2H5O is a stronger nucleophile than PhO . Hence, the
- - iii. C2H5-O-CH3
product is obtained by path II. iv. P-NO2-C2H5-I
(acidic character: PhOH > C2H5OH)
Sol: (iii) CH3O– acts as a base. It abstracts H⊕ from
(Basic and nucleophilic character : PhO- < C2H5O-) p-nitrophenol to form p-NO2 – C6H4O–

CH3
CH3OΘ is a stronger nucleophile than
CH3
S N2 p–NO2 – C6H4OΘ , hence the product is obtained by path II.
PhO CH2 I CH2 O Ph
Path I (Basic and nucleophilic character : p–NO2 – C6H4HΘ <
CH3OΘ)
C2H5O SN2 Path II CH3
S N2
p-NO2 C6H4O CH2 I
C2H5 O C2H5 Path I
Path II
CH3O S N2

CH3 CH2 OCH3

p-NO2 C6H4 O CH2 CH3

JEE Advanced/Boards
Example 1: Complete the following reactions:
Me

Me
Fuming KMnO4/H 1. Sn+ HCl Taut H2O/ 
(B) (C) (D) (E) (F) (G)
HNO3 [O] -CO2
Fuming KMnO4/H 1.
2. Sn+
OH HCl Taut H2O/ 
Toluene (B) (C) (D) (E) (F) (G)
HNO3 [O] -CO2
(A) 2. OH
Toluene
Sol:
(A)
Me Me COOH COOH
Me Fuming O2N Me NO2
KMnO4/H O2N COOHNO21. Sn+HCl H2N COOHNH2
HNO3 O2N [O] Taut
NO2
Fuming KMnO4/H O2N NO2 2. OH HN
1. Sn+HCl 2
NH2
(A) Taut
HNO3 NO2 [O] NO2 NH2
T.N.T 2. OH
(A) 2,4,6-Trinitro 2,4,6-Triamino
NO2
(2,4,6-Trinitro) NO2acid
benzoic NH2acid
benzoic
toluene
T.N.T (C) (D)
(B) 2,4,6-Trinitro 2,4,6-Triamino
(2,4,6-Trinitro) benzoic acid benzoic acid
toluene COOH COOH
(C) (D)
(B) HO OH O O  
O COOHO HN COOH NH
Taut 
HO OH O O   HO OH
-CO2 O O HN2 NH2
Taut  O
OH O H2O OH
-CO2 O NH H22
Phloroglucinol O
Triamino benzoic acid
OH O (F) H2
(G) O NH
Phloroglucinol (E)
(F) Triamino benzoic acid
(G) (E)
2 2 . 5 2 | Alcohols, Phenols and Ethers

Example 2: Complete the following reactions: Example 3: Complete the following reactions:
(a) OH
OH (B)
CHO a.
PhOH PhOH EtMgBr/H3 O
CHO (B) (C) (C)
PhOH H PhOH H O b. 2 mol of Brady’s reagent
(B) (C)
Salicyaldehyde H H (D)
c. 1 mol of HBr/CHBR3
Salicyaldehyde (E)
o-Hydroxy benzaldehyde
(A)
o-Hydroxy benzaldehyde
d. Butadiene ( (
(F)
O
(A) p-Benzoquinone e. (i) (1,3-Cyclohexadien) (ii) heat
(b) H2Ni KMnO4/H+ 1. NH3
PhOH (B) (C) (D) (O) (G)
H2Ni
(A)
20 atm KMnO4/H + 1. NH3 2. 
o
PhOH 200(B)C (C) (D)
H22. 
Sol: The reaction of quinones is that of α,β-unsaturated
20 atm /Ni P2O5
(A) o (F) (E)
200 C
H2/Ni P2O5 ketone.
(F) (E)
+
Sol:
Et Mg Br/H3O+ OH
(A) O O Et Mg Br/H3O O OH
(A) O O 1,2-addition O Et
Et
(a)
1,2-addition
OH OH (B)
CH=O CH OH (B)
H NO2
OH NO2
(A) OH
(B) O O + H2 N NH NO2
(B) O O + H2 N NH NO2
H HO CH
-H2O 2,4-DNP (Brady’s reagents)
2,4-DNP (Brady’s reagents)
OH (B) NO NO
OH NO22 NO22
OH HN N N NH NO2
O2N N NH NO2
HO CH O2N HN N
OH (C)
(C)

HO CH O OH
O OH

1,4-Addition
(C) + H Br 1,4-Addition
(C) + H Br Br
(C) Br
OH H
H
O O
O O
(b) OH
OH
OH OH

H2/Ni KMnO4/H COOH Br


Br
20 atm [O] COOH OH
OH
200oC
(A) (B) (C) (d) It is an example of Diels-Alder reaction.
Cyclohexanol Adipic acid

O O O
2 1 H
3 CN P2O5 C N H2 1. NH3
Diels-
2.  +
4 CN -2H2O C N H2 Alder
6
5
Butadiene H
(E) O O O
(Diene) (Dienophile)
Hexane-1,6-dinitrile (D)
Adipic amide OH
2
1 NH2 Taut.
H2/Ni 3

[H] 4
6 NH2
5
(F) OH
Hexamethylene (E)
diamine O O
1
H
Diels- 2 Taut.
Alder 3
Taut.

Chem i str y | 22.53

OH
(e) (E) (c) Reverse reaction occurs.
O O
− _
1
H PhOH
 + HCO3 → PhO
 +H 2 CO3
Taut. pK =10 Weaker C

Diels- 2 a B SA
Alder 3

2 − 
2 −  _ _ − −
(d) 

4
H PhOH + CO
PhOH + CO
3  
3  PhO

 + HCO
PhO + HCO
3 3
O O 

Weaker CB CB
Weaker
OH OH pKapK
=10.3
a =10.3
1 WA WA
Heat 2
CH2 CH2 + C2-C2 bond 3 Example 6: How will you synthesize the following
C3-C4 breaks 4 alcohol using grignard reagent
OH and aromatisation OH
occurs Ph
(A) OH (B)
Me Ph OH
Me
Example 4: Distinguish between the following pairs: OH
Ph Ph
OH OH (C) OH (D)
(A) (I) Me and (II) Me Me

(B) (I) OH and (II) Cl Sol:


Me
(C) (I) Ph OH and (II) Ph O a 1. Ether 
Ph 2. H3O Me
(A) OH b Ph MgBr + O
Sol: (a) (II) is unsaturated alcohols (allyl alcohol). When Me Path (a) Me
R
Br2/CCl4 solution is added to it, orange colour of Br2/CCl4 Me or
Ph
disappears. However, (I) (propyl alcohol) does not react 3o alcohol Me MgBr + O
Path (b)
with Br2/CCl4 and orange colour persists. R
Me
1. Ether 
(b) (I) (cyclopentanol) dissolves in conc. H2SO4 and 2. H3O
forms one layer, while (II) (cyclopentyl chloride) does not
dissolve in conc. H2SO4 and two distinct layers appear. (B) Ph OH
Path (a)
(c) (I) (benzyl alcohol) (1º ROH) is oxidized by acid
1. Ether 
Cr2O72– and orange colour of Cr2O72– changes to green 2. H3O
(Cr3+), whereas (II) (benzyl methyl ether) does not react. (OR) MgBr + CH2 O
Ph
Ph OH
Example 5: Explain which of the following reactions R
will occur. (b) 1o alcohol Ph MgBr +
Path (b) O
R
a. RCOOH + HCO3− →

 
pKa =5 pKa =10.3
Ph Same Ph
b.RCOOH + CO32− → (C) OH MgBr + CH2 O
Me condition Me

c.PhOH
 + HCO3 → a as above R
pKa =10 o
1 alcohol
d.PhOH + CO32− →
OH
(D) Ph
Sol: (a) The reaction is ; b
Path (a)
a Me
RCOOH + HCO3− → RCOO



+ H2O + CO2

   2o alcohol Ph MgBr + Me CH O
Weaker conjugate
pKa =5 pKa =10.3
base(CB )
WA (pKa =6.4) R R’
Me - MgBr + Ph CH O
(b) Reaction occurs Path (b)
R R’
RCOOH + CO32− → RCOO


 + HCO
 3


(WC ) pKa =10.3
B
WA
2 2 . 5 4 | Alcohols, Phenols and Ethers

Example 7: +
n mol of (C) Complete
n mol of the
(F) following reaction
(G) (Polymer)

H H2/Ni
PhOH + 3CH2 = O (C) + (D)
O O , pressure O O
(A) (B) -H2O
n HO C (CH2)4 C OH+n H HN (CH2)6 HN H C (CH2)4 C NH (CH2)6 NH
Adipic acid Repeating unit (mer) Nylon 66

Sol: (C) (F) Liner polyamide


(G)
(b)

O H
3CH2 3CH2 OH
CH2OH CH2OH
3CH2 OH H2/Ni
HO HO CH2OH HO CH2OH
(B) , pressure [H]
(A) HOH2C (C) CH2OH

Example 8: Give the products of the pinacol rearrangement of the following glycols in acids.
Me Me H H Me H
Me Me H H Me H
(A) Ph Me Me Ph (B) Ph H H Me (C) Me Me H H
(A) (B) Ph Me (C) Me
(A) Ph Ph (B)PhPh (C) H
OH OH Ph OH OH Me Me OH OH H
OH OHOH OH OH OHOH OH OH OH OH OH

Sol:
Me Me Me Me Me
Me Me
Me Me H Me Me Me
Me -H+
+ Me
(A) a. Ph + Me Me
Me Me
Ph PhH Ph -H Ph
-H
(A) (a)
(A) a. Pha. Ph H
Ph Ph Ph Ph Ph PhPhmigrates Ph Ph
OH OH OH Ph migrates
Ph migrates
Ph O
OH OH OH OH OH OH Ph OPh O

H H H H
H
HH H H H
H H
(b)b. Ph HH
b. Ph
b. Ph
Me
Me H Ph
Ph Me
Me
(B) (B) Me Ph Me
(B) OHOH OHOH OH
OH
OH OH OH
Benzyl
Benzyl CC
(More Benzyl
Stable)
(More Stable) C
(I) (More Stable)
HH (I)
H
H
H migrates
migrates
H migrates
H H HH
H MeH
Ph Ph Me H
H Me
Ph Me Me
OH Ph H Me
OH o Ph
o 2 C (II)
2 C (II)OH H Ph O
Less stable 2than
o (I) H O
Less stable thanC(I)(II) H O
Less stable than (I)

Me H Me H
c. Me H H Me H
Me Me H H Me
c. H MeH H
Me H
c. H Me H
(C) OH
Me OH H Me OH H
(C) OH OH 2OH
o
C
(C) OH OH OH
2o(I)
C
(I) 2oC
(I)
H migrates
Me H H migrates
OH Ph
o
2 C (II)
H O
Less stable than (I) Chem i str y | 22.55

Me H Me H
(c)
c. Me H
H
Me H

(C) OH OH OH
2oC
(I)

H migrates
Me H
Me Me Me H
OH Ph
1oC (II) H O
Less stable than (I) (Aldehyde)

JEE Main/Boards

Exercise 1 Q.4 How will you prepare the following:

Q.1 Give IUPAC substitutive names for the following (1) 3-phenyl but-1-ene to 2-phenyl butan-2-ol
alcohols:

(a) CH3 CHCH3 CHCH2 OH (b) CH3CHCH3 CHCH3 (2) CH2 to cyclopentyl methanol
| | | |
CH3 CH3 OH C 6H5
Q.5 Arrange the following compounds in the decreasing
(c) CH3CHCH2 CH = CH2 order of their boiling points and solubility in H2O.
|
a. (I) Methanol (II) Ethanol
OH
(III) Propan-1-ol (IV) Butane-1-ol
Q.2 How will you convert ethanol into the following (V) Butane-2-ol (VI) Pentan-1-ol
compounds?
b. (I) Pentanol (II) n-Butane
(i) Butane-1, 3-diol (ii) But-2-enal (iii)
But-2-enoic acid (III) Pentanal (IV) Ethoxy ethane
c. (I) Pentane (II) Pentane-1, 2, 3-triol
Q.3 Write all the stereoisomers of 2-isoproyl-5-methyl (III) Butanol
cyclohexanol and give the decreasing order of their
stabilities.
Q.6 Explain the less solubility and lower boiling point of:

Me (I) o-Nitrophenol
Me
(I) O (II) C2H5 O C2H5 O
THF (Tetraphydrofuran)
(III)
Me (II) o-Hydroxy
Me benzaldehyde
Diethyl ether
(III) o-Hydroxybenzoic
Diisopropyl ether acid (salicyclic acid) compared
with their p-and m-isomers.
Me Me
H5 O C2H5 (III) O
Me Me
iethyl ether
Diisopropyl ether
2 2 . 5 6 | Alcohols, Phenols and Ethers

Q.7 Which isomer (o, m, or p) of hydroxy acetophenone formed. Determine the mass percentage of acetic acid
is steam volatile? and phenol in the given solution.

Q.8 Q.16 Find the structure of (A), C10H10 O2 , a sweet-


 Number of 
 
Cyclobutyl ethene
Dil.H2SO 4
 → (B)  isomeric products  smelling liquid that has the following properties. It
 including stereoisomers 
 
does not dissolve in NaOH or give a colour with FeCl3 ;
it adds one equivalent of H2 on catalytic hydrogenation.
Reductive ozonolysis affords H2C = O and C9H8 O3
Q.9 Give the decreasing order of Lewis basicities of the
(B) that gives a positive Tollens test. Oxidation of
following:
(A) with KMnO 4 gives an acid (C) (MW=166) which
gives no colour with FeCl3 . When (C) is refluxed with
Q.10 Explain the formation of B and C, optically pure concentrated HI, H2C = O and 3,4-dihydroxybenzoic

 
different isomers from (A) with little racemisation. acid are isolated and identified.
Me
1 CH3OH/H
Exercise 2
Me 2 (B + C)
4 3
O
(S-) 2- Methyl-1,2-butene oxide Q.1 An organic compound (A) with molecular formula
(A)
C7H8O dissolves in NaOH and gives characteristic colour
with FeCl3. On treatment with Br2, it gives a tribromo
Q.11 Show how will you synthesize product C7H5OBr3. The compound is:

(a) 1-phenylethanol from a suitable alkene, (A) p-Hydroxybenzene

(b) Cyclohexylemethanol using an alkyl halide by an SN2 (B) 2-Methoxy-2-phenyl propane


reaction, (C) m-Cresol
(c) Pentan-1-ol using a suitable alkyl halide? (D) p-Cresol

Q.12 Preparation of ethers by acid dehydration of Q.2 Which of the following paths is/are feasible for the
secondary or tertiary alcohols is not a suitable method. preparation of ether (A)?
Give reason.
(A) Path I : ONa + X
Q.13 Compound (D), an isomer of (A) in Problem 4,
Θ
reacts with BH3 . THF and then H2O2 / OH to give chiral (B) Path II : OH + Conc. H2SO4
HO Low temp. O
(E). Oxidation of (E) with KMnO 4 or acid dichromate
affords a chiral carboxylic acid, (F). Ozonolysis of (D) (A)
after reduction with Zn gives the same compound (G) 1. Hg(OCOCF3)2 Divinyl ether
(C) Path III : CH2=CH2 2. HO-CH=CH2
obtained by oxidation of 2-methyl pentan-3-ol with 3. NaBH4
KMnO 4 . Identify (D), (E), (F), and (G).
ClCH2CH2OH 1. Conc.H2SO4
Q.14 An organic compound (A) ( C8H8 O3 ) was insoluble (D) Path IV :
+ HOCH2CH2Cl at 413 K
in water, dilute HCl, and NaHCO3 . It was soluble in 2. NaNH2
NaOH. A solution of (A) in dilute NaOH was boiled and
steam distilled and distillate was reacted with NaOH to Q.3 Which of the following statement is correct?
give a yellow precipitate was reacted with NaOH to give i. Glycerol on reaction with oxalic acid at 110°C (383 K)
a yellow precipitate. The alkaline residue is acidified to and followed by heating and hydrolysis gives formic acid
give a solid (B) ( C7H6 O3 ) . (B) dissolved in aqueous and glycerol.
NaHCO3 with the evolution of gas. Identify (A) and (B).
ii. Glycerol on reaction with oxalic acid at 230°C (503 K)
and followed by heating gives allyl alcohol.
Q.15 Neutralisation of 30 gm of a mixture of acetic acid
and phenol solutions required 100 ml of 2M sodium iii. Glycerol on oxidation with dil.HNO3 gives a mixture
hydroxide solution. When the same mixture was of glyceric and tartonic acid.
treated with bromine water, 33.1 gm of precipitate was
Chem i str y | 22.57

iv. Glycerol on oxidation with conc. HNO3 gives glycerol Q.9 Reaction involving anti addition is:
acid.
H+ /H O
(A) CH
= 2 →
CH2 
(A) i and ii (B) i and iii 2

HX
(C) iii and iv (D) i, ii, iii, iv (B) CH
=3CH CH2  →
Hg(OAc) /H O
(C) CH3= 2 2 →
CH CH2 
Q.4 In the reaction NaBH 4
alc HBr CH3ONa B H /THF
CH3CHCH3 
KOH
→ A 
Peroxide
→ B → C (D) CH
=2
2 6
CH2  −

| H2O2 /OH

Br
H2O/H+ +
? Product/(s)
H2O/H will be:
(A) Diethyl ether (B) 1-Methoxypropane Q.10 O ?? Product/(s) will be:
Product/(s) will be:
O
(C) Isopropyl alcohol (D) Propylene glycol.
(A)
(A)OH OH
Q.5 The compound which is not isomeric with diethyl (B)
OH
OH
ether is: (B) OH + OH +
(A) n-propyl methyl ether (B) 2-methyl propan-2-ol OH OH
OH
(C) + OH
(C) +
(C) Butanone (D) Butan-1-ol
OH
(D) OH
(D)
Q.6 Phenol reacts with bromine water is CS2 at low
temperature to give:
CH2OH
(A) o-Bromophenol (B) o-and p-Bromophenol
(C) p-Bromophenol (D) 2,4, 6-Tribromphenol Q.11 +H2SO4
170oC
A
(conc.)

Q.7 In the following compounds: What is the major product A?

OH OH OH OH CH2 CH3

(A) (B)
NO2
CH3 NO2 CH3
(I) (II) (III) (IV) (C) (D)
The order of acidity is
(A) (III) > (IV) > (I) > (II) (B) (I) > (IV) > (III) > (II)
Q.12
(C) (II) > (I) > (III) > (IV) (D) (IV) > (III) > (I) > (II)
MnO2 CH2MgBr
OH x A+B
Q.83CH=CH
CH The reaction ofOH
CH3with
CH=CHHBr gives OH with HBr gives
OH

(A) CH3CHBrCH2 OH
(A) CH (A) OMgBr (B) O & CH4
3CHBrCH2
OH
CH3
OH OMgBr
(B) CH3CHBr OH
(B) CH3CHBr OH
(C) OMgBr (D)
CH3
(C) CH3CHBrCH2 Br
(C) CH Br CH3OMgBr
3CHBrCH2

(D) CH3CH2CHBr Br Br
(D) CH3CH2CHBr
2 2 . 5 8 | Alcohols, Phenols and Ethers

(I) CH3-O-CH3 Q.17 Reaction of CH2 CH2 with RMgX leads to


Q.13 Cl
Mg/dry ether
Br ether
Mg/dry A -O-CH3+
(1 mole) (I) CH3(ii) H3O formation of O
Cl Br A (I) CH -O-CH
(1 mole)
Mg/dry ether (ii) H33O+ 3
Cl Br A
What is B? (1 mole) (ii) H O+
3 (A) RCH2CH2OH (B) RCHOHCH3
OH
OH R
(A) Cl C CH3
OH (C) RCHOHR (D) CHCH2OH
(A) Cl C CH3
CH R
(A) Cl C CH 3
CH3 3
OHCH3 OH Q.18 Ethyl chloride is converted into diethyl ether by
OH OH
(B) CH3
OH
C
OH
C CH3
(A) Perkin’s reaction (B) Grignard reaction
(B) CH3 C C CH3
CH3 CH3
(B) CH3 C C CH3 (C) Wurtz synthesis (D) Williamson’s synthesis
CH3 OH CH 3
OH
(C) CH
CH C CH3 Br
Q.19
3
3
(C) CH3 C
OH
CH3 Br (i) Mg Br2
(C) CH3 C Br CH2=CH CH2 Br (ii) HCHO A B
CH3 CCl2
(D) CH3 (iii) H2O KOH
(D) Pd alc.KOH
(D)
E 
D C
Product (E) is

Q.14 When 3, 3-dimethyl 2-butanol is heated with (A) (B)


H2SO4 the major product obtained is

: :
O O
(A) 2, 3-dimethyl 2-butene HO
CH2 CH CH2 CH2 OH
(B) cis and trans isomers of 2, 3-dimethyl 2-butene (C) (D)
OH OH
(C) 2, 3-dimethyl 1-butene HO O

(D) 3, 3-dimethyl 1-butene

Q.15 Consider the following reaction


Q.20 CH3 CH CH2
Phenol O PCl5 Mg O
CH2OH A B C
CH2Cl Alkaline KMnO 4
ether
 →X  → Y  → Z
Zn dust Anhydrous AlCl3 H2O/H+ Product (C) is
The product Z is
(A) Benzene (B) Toluene (A) (B)
O O O
(C) Benzaldehyde (D) Benzoic acid OH

(C) (D) OH
Q.16 In Reimer-Tiemann reaction, the intermediate
O
which does not form is OH O
-+
O ONa

(A)
CHCl2
(B)
H
Previous Years Questions
CCl2

Q.1 Hydrogen bonding is maximum in (1987)


O
(A) Ethanol (B) Diethyl ether
H
+
(C) CCl2 (D) All of these (C) Ethyl chloride (D) Triethyl amine
Chem i str y | 22.59

Q.2 Hydrogen bonding is maximum in (1987) Q.8 Read the following question and answer as per the
direction given below:
(A) Ethanol (B) Diethyl ether
(C) Ethyl chloride (D) Triethyl amine (A) Statement-I is true ; statement-II is true; statement-II
is a correct explanation of statement-I.
Q.3 In CH3CH2OH, the bond that undergoes heterolytic (B) Statement is true; statement-II s true; statement-I is
cleavage most readily is (1988) not the correct explanation of statement-I.
(A) C – C (B) C – O (C) C – H (D) O – H (C) Statement-I is true; statement-II is false.
(D) Statement-I is false ; statement-II is true.
Q.4 The products of combustin of an aliphatic thiol
(RSH) at 298 K are (1992) Statement-I: Solubility of n-alcohol is water decreases
with increase in molecular weight.
(A) CO2(g), H2O(g) and SO2(g)
Statement-II: The relative proportion of the hydrocarbon
(B) CO2(g), H2O(l) and SO2(g) part in alcohols increases with increasing molecular
(C) CO2(l), H2O(l) and SO2(g) weight which permit enhanced hydrogen bonding with
water (1988)
(D) CO2(g), H2O(l) and SO2(l)

Q.9 The yield of a ketone when a secondary alcohol


Q.5 Which one of the following will most readily be
is oxidized is more than the yield of aldehyde when a
dehydrated in acidic condition?  (2000)
primary alcohol is oxidized. (1983)
O OH OH
(A) (B) Q.10 Sodium ethoxide is prepared by reacting ethanol
with aqueous sodium hydroxide.  (1985)
O O

(C) (D) Q.11 A liquid was mixed with ethanol and a drop of
concentrated H2SO 4 was added. A compound with a
OH OH
fruity smell was formed. The liquid was: (2009)
(A) CH3OH (B) HCHO
Q.6 Compound ‘A’ (molecular formula C3H8O) is
treated with acidified potassium dichromate to form (C) CH3COCH3 (D) CH3COOH
a product ‘B’ (molecular formula C3H6O) ‘B’ forms a
shining silver mirror on warming with ammonical silver Q.12 The major product obtained on interaction of
nitrate. ‘B’ when treated with an aqueous solution of phenol with sodium hydroxide and carbon dioxide is:
H2NCONHNH2 and sodium acetate gives a product ‘C’.  (2009)
Identify the structure of ‘C’  (2002)
(A) Benzoic acid (B) Salicylaldehyde
(A) CH
CH333CH
CH222CH
CH =
= NNHCONH
(A) NNHCONH222 (C) Salicylic acid (D) Phthalic acid
(B)
(B) H3C - C = NNHCONH2
H33C - C = NNHCONH22
CH
CH333 Q.13 The main product of the following reaction is
C6H5CH2CH ( OH) CH ( CH3 ) 
(C) H C - C = NCONHNH conc. H SO
2 4 →? 
(C) H333C - C = NCONHNH222 (2010)
2
CH3
CH33
H5 C 6 H C6H5CH2 CH3
(D) CH
(D) CH333CH
CH222OH
OH +
+ NCONHNH
NCONHNH222 (A) C=C (B) C=C
H CH(CH3)2 H CH3
Q.7 How many structures of F is possible? (2003)
CH3 C5H6 CH(CH3)2 H5C6CH2CH2
+
H Br2/CCl4 (C) C=C (D) C = CH2
[F] C4H8Br8
/H2O H3 C


H H
H3C OH 5 such products
are possible
(A) 2 (B) 5 (C) 6 (D) 3
2 2 . 6 0 | Alcohols, Phenols and Ethers

Q.14 Ortho–Nitrophenol is less soluble in water than Q.16 Arrange the following compounds in order of
p– and m– Nitrophenols because: (2012) decreasing acidity: (2013)
(A) o–Nitrophenol is more volatile in steam than those OH OH OH OH
of m – and p–isomers
(B) o–Nitrophenol shows Intramolecular H–bonding ; ; ; ;

(C) o–Nitrophenol shows Intermolecular H–bonding


Cl CH3 NO2 OCH3
(D) Melting point of o–Nitrophenol is lower than those
(I) (II) (III) (IV)
of m–and p–isomers.
(A) II > IV > I > III (B) I > II > III > IV
Q.15 Iodoform can be prepared from all except: (2012) (C) III > I > II > IV (D) IV > III > I > II
(A) Ethyl methyl ketone
(B) Isopropyl alcohol Q.17 An unknown alochol is treated with the “Lucas
reagent” to determine whether the alcohol is primary,
(C) 3–Methyl – 2– butanone secondary or tertiary. Which alcohol reacts fastest and
(D) Isobutyl alcohol by what mechanism: (2013)
(A) Secondary alcohol by SN1
(B) Tertiary alcohol by SN1
(C) Secondary alcohol by SN2
(D) Tertiary alcohol by SN2

JEE Advanced/Boards

Exercise 1 (i) Provide a complete mechanism for the formation of


the major product
Q.1 Calculate the depression in freezing point ( ∆Tf ) of (ii) Briefly explain the choice of major product.
0.1 m solution of ROH in cold conc. H2SO4. Kf = x K kg
mol-1
Q.4
3,3,6,6-Tertramethyl-1-1,4-cyclohexadiene (A)
Q.2 0.218 gm of the acetyl derivative of a polyhdric
alcohol (molecular mass = 92) requires 0.168 gm of (a) (i) Excess of (b) (i) Excess of
KOH for hydrolysis. Calculate the number of (-oH) B2H6+THF Hg(OAC)2+H2O
groups in the alcohol. (ii) H2O2/OH
(ii) NaBH4/OH

Mixture of isomeric Mixture of isomeric


Q.3 Consider the following reaction:
C10H20O2 C10H20O2

   
OH HOof isomers
No. No. of isomers
(B) excluding (C) excluding
H2SO4 enantiomers enantiomers
H2O HO
(A) (B) What are(C)
the numerical values of (B) and (C)?
OH HO
Q.5
O
OH
HO
(A) (B) (C) H+ / (i) O3 NaOH
X Y
(ii)Zn/CH3COOH
Chem i str y | 22.61

Q.6 Compound X (molecular formula, C5H8O) does OH


not react appreciably with Lucas reagent at room |
temperature but gives a precipitate with ammoniacal Q.11 20 If Me − CH− CCl3 is treated with alkaline NaN3
silver nitrate with excess of MeMgBr, 0.42 g of X and followed by reduction with H2 / Pd it gives an
gives 224 mL of CH4 at STP. Treatment of X with H2 in
presence of Pt catalyst followed by boiling with excess  Me − CH − COOH 
 
HI, gives n-pentane. Suggest structure for X and write α -amino acid alanine  |  . Explain the
the equation involved. reaction.  NH2 
 

Q.7 An organic compound (A) gives positive Liebermann Q.12 Convert


reaction and on treatment with CHCl3 / KOH followed
by hydrolysis gives (B) and (C). Compound (B) gives O O
colour with Schiff’s reagent but not (C), which is steam 1 2
3
4 Br 1 2 4 6

volatile. (C) on treatment with LiAIH4 gives (D), C7H8O2, H3C 5 H3C 3 5 OH
which on oxidation gives (E). Compound (E) reacts with
( CH3CO ) O/ CH3COOH to give a pain reliever (F). Give
2 Q.13 Assign the structure of (B), the principal organic
the structures of (A) to (F) with proper reasoning. product of the following reaction:
I Ph
Q.8 Two isomeric compound, (A) and (B), have the same Ag -

-
Me OMe (B)
formula C11H13OCl . Both are unsaturated, yield the Heat
same compound (C) on catalytic hydrogenation, and Ph OH
(A)
produce 4-chloro-3-ethoxybenzoic acid on vigorous
oxidation. (A) exists in geometrical isomers, (D) and
(E), but not (B). give structures of (A) to (E) with proper Q.14 When a mixture of t-butyl alcohol and ethyl
reasoning. alcohol is heated with conc. H2SO 4 , a single ether
product is obtained. Identify the product giving proper
reasons.
Q.9 15 Nitrobenzene is formed as the major product
along with a minor product in the reaction of benzene
with a hot mixture of nitric acid and sulphuric acid. The Q.15 Identify the major products (B) to (H).
minor product consists of carbon: 42.86%, hydrogen:
2.40%, nitrogen: 16.67%, and oxygen: 38.07%. OH -
(i) Calculate the empirical formula of the minor product. (i) H -
-
a. (i)OH Anhyd. AlCl2
(B) (C) (D)
(ii) When 5.5 gm of the minor product is dissolved in (ii) ClCH2COO - (ii) SOCl2
(A)
45 gm of benzene, the boiling point of the solution is
1.84°C higher than that of pure benzene. Calculate the b. O + HN O (G)
D2/Ni
(H)

molar mass of the minor product and determine its
molecular and structural formula.
(Molal boiling point elevation constant of benzene is Q.16 When pent-4-en-1-ol is treated with aqueous
2.53 K kg mol-1.) Br2 / OH , a cyclic bromo ether is formed rather than the
expected bromohydrin. Propose a suitable mechanism
Q.10 Identify A, B and C in the following reaction. for the above.

Exercise 2
CH3COCl
B
i. NaOH
C6H5OH A
ii. CO2, 130o, 6 atm
Br3/Fe
iii. H3O+ C Single Correct Choice Type

Q.1 Select the correct statement.


(A) Solvolysis of (CH3)2C=CH-CH2Cl in ethanol is over
6000 times than alkyl chloride ( 25°C )
(B) CH3-CH=CH-CH2-OH when reacts with HBr give a
mixture of 1-bromo-2-butene and 3-bromo 1-butene
2 2 . 6 2 | Alcohols, Phenols and Ethers

(C) When solution of 3-buten 2-ol in aqueous sulphuric Which of the following statements is/are correct about
acid is allowed to stand for one week, it was found to the above reaction?
contain both 3-buten 2-ol and 2-buten-1-ol
(A) The compounds (B) and (C), respectively, are:
(D) All of these
O2N I+I CH2 OH
Q.2
(B) The compound (D) and (E), respectively, are:
MeO CH

CH OH OH + I CH2 OH
xHIO4
CH OH O What is the maximum value if (x) ?
(C) The compound (B) and (C), respectively, are:
CH OH

HC NO2 OH + I CH2 I
CH2 OH
(D) The compounds (E) and (F), respectively, are:

(A) 1 (B) 2 (C) 3 (D) 4


I+I CH2 I

Q.3 Esterification (shown below) is a reaction converting


a carboxylic acid to its ester. It involves only the carbonyl Q.6 Phenols are generally not changed with
carbon. Esterification of (-) lactic acid with methanol NaBH4/ H3O⊕ 1, 3-and 1, 4-benzenediols and 1, 3,
yields (+) methyl lactate. Assuming that there are no 5-benzenetriols are unchanged under these conditions.
side reactions, what is true about this reaction? However, 1, 3, 5- benzenetriol (phloroglucinol) gives a
OH OH high yield of product (B).
CH3OH
OH OCH3 OH
HCl
O O NaBH4/H3O
(-) (+) (B)
HO OH
(A) An SN 2 process has occurred, inverting the absolute Phloroglucinol
configuration of the chiral center.
OH OH
(B) An SN 1 reaction at the chiral center has inverted
the optical rotation.
(A) (B)
(C) A diastereomer has been produced; diastereomers OH O OH
have different physical properties including optical OH
rotation
(D) Optical rotation is not directly related to absolute (C)
configuration, so the change in sign of rotation is OH
merely a coincidence

OH Q.7 Diethyl ether on heating with conc. HI gives two


moles of
Q.4 C2H5O /anhyd. C2H5OH 
+ C2H5I (A) Ethanol (B) Iodoform
(B)
(A) (C) Ethyl iodide (D) Methyl iodide
(A) PhOC2H5 (B) PH – O – Ph
Q.8 An industrial method of preparation of methanol is
(C) Phl (D) C2H5OC2H5
(A) catalytic reduction of carbon monoxide in presence
of ZnO-Cr2O3
Q.5
(B) by reacting methane with stem at 900°C with nickel
Excess
(I) O2N O CH2 OMe HI
B+C catalyst
A (C) by reducing formaldehyde with LiAlH4
(II) O CH2
Excess
OMe HI E+F (D) by reacting formaldehyde with aqueous sodium
D hydroxide solution
Chem i str y | 22.63

Q.9 Which one of the following will most readily be Me


dehydrated in acidic condition? Me
OH Br
O OH OH (A)
(A) (B)

Me
O O
OH Me
(C) (D) (B)
Br
OH OH
Me

Q.10 OH Me
(C) Br
C3H7 − OH + Er3O +BF4− → C3H7 − O − Et + EtOEt

Which of the following statements is wrong? Me Me


(A) The nucleophile in the reaction is C3H7OH. OH Br
(D)
(B) The nucleophile in the reaction is Bf4
(C) The leaving group is Et2O.
(D) SN2 reaction occurs Comprehension Type
Reimer-Tiemann reaction introduces an aldehyde group
Q.11 Which of the best method for the conversation of on to the aromatic ring of phenol, ortho to the hydroxyl
(A) pantan-3-ol to 3-bromopentane (B)? group. This reaction involves electrophilic aromatic
substitution. It is a general method for the synthesis of
(A) Me Me + Pbr3 (B) substituted salicyaldehydes as depicted below:
OH
OH ONa OH
Br - CHO CHO
(B) (A) + TsCl Me Me aq. HCl
SN 2
[I]
OTs Intermediate
(TsCl-Tosyl chloride,p-Me-C6H5SO2Cl)
CH3 CH3 CH3
(I) (II) (III)
(B) + Me SO3

(C) (A) HBr (B)


(Tosylate ion)
Q.14 Which one of the following reagents is used in the
above reaction?
(D) Both (A) and (B)
(a) aq. NaOH + CH3Cl (b) aq.NaOH + CH2Cl2
(c) aq.NaOH+CHCl3 (d) aq.NaOH+CCl4
Q.12 In Zeisel’s method for the determination of
methoxyl groups, a sample of 2.68 gm of a compound
(A) gave 14.08 gm of AgI. If the molecular weight of Q.15 The electrophile in this reaction is:
compound (A) is 134, the number of (-OCH3) group(s)
(A) :CHCl (B) ⊕
CHCl2 (C) :CCl2 (D) ᛫CCl3
in the compound (A) is:
(A) 1 (B) 2 (C) 3 (D) 4 Q.16 The structure of the intermediate (I) is:

Q.13 ONa ONa

(i) CH3CHO HBr CH2Cl CHCl2


MgBr
(A) (B) (A) (B)
(ii) H O+
3

CH3 CH3

ONa ONa
2 2 . 6 4 | Alcohols, Phenols and Ethers

Assertion Reasoning Type Q.18 Assertion: 2, 6-Dimethyl-4-nitrophenol (I) is more


acidic than 3, 5-dimethyl-4-nitrophenol (II).
Q.17 Assertion:
OH Reason: It is due to steric inhibition of the resonance of
(-NO2) group with two (Me) groups in (II).
3Cl2/Fe
(B)
Q.19 Assertion: Diphenyl ether (I) on dinitration gives

 
(A) the product (II).
Cl
Dinitration
The product (B) is Cl OH O
Fuming HNO3

Cl O2N

O NO2
Reason (R): Phenol cannot be chlorinated because the
ring is susceptible to oxidation by Cl2.
Reason: The ring with first nitro group is deactivated
by e withdrawing NO2 group, so the second nitro group
enters the other ring.

Match the Columns

Q.20 Match the reactions of column I with the Mechanism of column II.

Column I Column II
Reactions Mechanism
(A) OH OH (p) Carbocation intermediate
CHO
Me3CO
+ CHBrClI

(B) Me HCl + ZnCl2 Me Me (q) Bromochloro carbine


OH
Me Me intermediate
Cl
(C) Me Me (r) SE reaction
PCl3
OH or Cl
Me Me
SOCl2

(D) Excess (s) Rearrangement of carbocation


MeO O CH2 intermediate
HI

HO OH+lCH2

(E) Me (t) SN1 mechanism


BF
MeO + OH or3
Me
HF

Me
MeO Me
Me
(F) 4 OH (u) No rearrangement
Me 3 2 1 Dil. Me
Me Me Me
H2SO4 Me Me
Chem i str y | 22.65

Previous Years’ Questions Paragraph 1: A tertiary alcohol H upon acid catalysed


dehydration gives product I. Ozonolysis of I leads to
Q.1 When phenyl magnesium bromide reacts with tert- compounds J and K. Compound J upon reaction with
butanol, which of the following is formed? (2005) KOH gives benzyl alcohol and a compound L, whereas
K on reaction with KOH gives only M.  (2008)
(A) Tert-butyl methyl ether (B) Benzene
O
(C) Tert-butyl benzene (D) Phenol H3 C Ph
M=
Ph H
Q.2 The best method to prepare cyclohexene from
cyclohexanol is by using (2005)
(A) conc. HCl + ZnCl2 (B) conc. H3PO4 Q.7 Compound H is formed by the reaction of
(C) HBr (D) conc. HCl O O
; PhMgBr ; PhCH2MgBr
(A) Ph CH3 (B) Ph CH3
Q.3 (I) 1, 2-dihydroxy benzene
O O
O O
(II) 1, 3-dihydroxy benzene ; PhMgBr O
PhMgBr
; PhCH
CH 2 CH2
(A) Ph (B) 2MgBr (B) ;; PhCH2MgBr
CH3 (A) Ph
(C) Ph CH33; PhCH2MgBr (D)
CH Ph CH3
(III) 1, 4-dihydroxy benzene Ph H Ph H Ph MgBr

(IV) Hydroxy benzene O CH


O CH2 CH2 CH2
O O2
; Ph
; CH
PhMgBr
2 MgBr ; ;
Ph CH MgBr ;
(C) Ph (D)
The increasing order of (A) boiling
Ph points
H of above
CH3
(C)
(B) Ph
Ph
Ph
HH
CH3
; PhCH
Ph 2 2MgBr (D) Ph
MgBr H Ph MgBr
mentioned alcohols is (2006)
O CH2 CH2
(A) I < II < III < IV (B) I < II < IV < III
; PhCH2MgBr ;
(C) (D) Ph
(C) IV < I < II < III (D) IV < IIPh< I < III H H Ph MgBr

Q.4 The major product of the following reaction is Q.8 The structure of compound I is
 (2011)
RCH2OH Ph CH3 H3C Ph
H (anhydrous) (A) (B)
O
H Ph H Ph
(A) A hemiacetal (B) An acetal
Ph CH3 H3C CH3
(C) An ether (D) An ester
(C) (D)
H CH3Ph H
Q.5 The products of reaction of alcoholic silver nitrate Ph
with ethyl bromide are  (1998)
(A) Ethane (B) Ethene Q.9 The structures of compound J, K and L respectively,
(C) Nitroethane (D) Ethyl alcohol are
(A) PhCOCH3, PhCH2COCH3 and PhCH2COO– K+
Q.6 The following ether, when treated with HI produces (B) PhCHO, PhCH2CHO and PhCOO– K+
 (1999)
O CH2 + HI (C) PhCOCH3, PhCH2CHO and CH3COO–K+
(D) PhCHO, PhCOCH3 and PhCOO–K+

(A) CH2I (B) CH2OH Q.10 Give reasons for the following in one or two
sentences. “Acid catalysed dehydration of t-butanol is
faster than that of n-butanol. (1998)

(C) I (D) OH
2 2 . 6 6 | Alcohols, Phenols and Ethers

Q.11 Write the structures of the products:  (1998) Q.15 The correct statement(s) about O3 is (are) (2013)
HI,Excess
(CH3 )2 CHOCH3 → Product (A) O–O bond lengths are equal
Heat
(B) Thermal decomposition of O3 is endothermic

Q.12 Explain briefly the formation of products giving (C) O3 is diamagnetic in nature
the structures of the intermediates.  (1999) (D) O3 has a bent structure

HCl
(i)
OH Cl Q.16 The major product(s) of the following reaction is
(are) (2013)
+ CH2Cl + etc. OH

CH3 CH3 aqueous Br2(3.0 equivalents)

(ii)
HCl
OH Cl
SO3H
OH OH OH
Q.13 Cyclobutylbromide on treatment with magnesium Br Br Br Br
in dry ether forms an organometallic compound (A). The
organometallic reacts with ethanol to give an alcohol (B) Br Br Br Br
after mild acidification. Prolonged treatment of alcohol
(B) with an equivalent amount of HBr gives 1–bromo-1- SO3H Br Br
methylcyclopentane (C). Write the structures of (A), (B) P Q R
and explain how (C) is obtained from (B).  (2001)
OH OH OH OH
Br Br Br Br Br
Q.14. In the following reaction, the product(s) formed
is (are) (2013)
Br Br Br Br Br
OH
SO3H Br Br SO3H
CHCl3
? P Q R S
OH-
(A) P (B) Q (C) R (D) S
CH3
OH O O Q.17 The reactivityO of compound Z with different
OHC CHO halogens under appropriate
CHOconditions is given below:
 (2014)
Mono halo substituted
H3C CHCl2 H3C OH
CHCl
CH3 2 CH3 derivative when X2 = I2

P Q R X2S
O O
Di halo substituted
CHO
C(CH3)3 derivative when X2 = Br2
Z

Tri halo substituted


CHCl2 H3C CHCl2 CH3 derivative when X2 = Cl2
R S
The observed pattern of electrophilic substitution can
(A) P(major) (B) Q(minor) be explained by
(C) R(minor) (D) S(major) (A) The steric effect of the halogen
(B) The steric effect of the tert-butyl group
(C) The electronic effect of the phenolic group
(D) The electronic effect of the tert-butyl group
Chem i str y | 22.67

Q.18 The acidic hydrolysis of ether (X) shown below is


fastest when  (2014) Q.22 The major compound Y is (2015)

CH3 CH3
(A) (B)

acid CH3
OR OH+ ROH CH2

CH3
(C) CH3
(D)

(A) One phenyl group is replaced by a methyl group. Q.23 The correct statement(s) about the following
reaction sequence is(are) (2016)
(B) One phenyl group is replaced by a para-
methoxyphenyl group. Cumene ( C9H12 ) 
i) O 3 → Q (major ) + R (minor ) Q 
2 → P CHCl /NaOH NaOH
+
ii)H3 O PhCH
2
(C) Two phenyl groups are replaced by two para-
methoxyphenyl groups. Cumene ( C9H12 ) 
i) O2

CHCl3 /NaOH
→ P  → Q (major ) + R (minor ) Q 
NaOH
→S 
+ ii)H3 O PhCH Br 2
(D) No structural change is made to X.
(A) R is steam volatile
Q.19 The number of resonance structures for N is (B) Q gives dark violet coloration with 1% aqueous
 (2015) FeCl3 solution
OH (C) S gives yellow precipitate with 2,
NaOH
N 4-dinitrophenylhydrazine
(D) S gives dark violet coloration with 1% aqueous FeCl3
Q.20 The number of hydroxyl group(s) in Q is (2015) solution

H+ aqueous dilute. KMnO4 (excess)


H P Q
heat o
0C

HO
H3C CH3

Paragraph 2: In the following reactions

Pd-BaSO4 i) B2H4
C8H6 C8H8 X
H2 ii) H2O4, NaOH, H2O
H2O
HgSO4, H2SO4

i) EtMgBr, H2O
C8H8O Y
ii) H+ , heat

Q.21 Compound X is (2015)


O OH

CH3 CH3
(A) (B)
O
OH
(C) CHO
(D)
2 2 . 6 8 | Alcohols, Phenols and Ethers

PlancEssential Questions
JEE Main/Boards JEE Advanced/Boards
Exercise 1 Exercise 1
Q.1 Q.5 Q.8 Q.1 Q.8 Q.11
Q.10 Q.15

Exercise 2 Exercise 2
Q.2 Q.3 Q.7 Q.2 Q.5 Q.7
Q.14 Q.15 Q.16 Q.10 Q.12 Q.15
Q.18

Previous Years’ Questions Previous Years’ Questions


Q.3 Q.4 Q.6
Q.3 Q.6 Q.8
Q.12

Answer Key

JEE Main/Boards
Exercise 2
Single Correct Choice Type

Q.1 C Q.2 D Q.3 D Q.4 B Q.5 B Q.6 B


Q.7 D Q.8 B Q.9 C Q.10 A Q.11 C Q.12 B
Q.13 A Q.14 A Q.15 D Q.16 D Q.17 A Q.18 D
Q.19 B Q.20 C

Previous Years’ Questions


Q.1 A Q.2 A Q.3 D Q.4 B Q.5 A Q.6 A
Q.7 D Q.8 C Q.9 F Q.10 F Q.11 D Q.12 D
Q.13 A Q.14 B Q.15 D Q.16 C Q.17 B
Chem i str y | 22.69

JEE Advanced/Boards
Exercise 2
Single Correct Choice Type

Q.1 D Q.2 B Q.3 D Q.4 A Q.5 A Q.6 C


Q.7 C Q.8 A Q.9 A Q.10 B Q.11 B Q.12 C
Q.13 B

Comprehension Type
Q.14 C Q.15 C Q.16 B

Assertion Reasoning Type


Q.17 D Q.18 A Q.19 D

Match the Columns


Q.20 A → q, r; B → p, s; C → u; D → p, t; E → p, r, s; F → p, s

Previous Years’ Questions


Q.1 B Q.2 B Q.3 C Q.4 A Q.5 C, E Q.6 A, D
Q.7 B Q.8 A Q.9 D Q.14 B,D Q.15 A, C, D Q.16 B
Q.17 A, B, C Q.18 C Q.19 9 Q.20 D Q.21 C Q.22 D
Q.23 B, C

Solutions
(A) CH 3 C HCH2 C HCH2OH
CH3 CH3
2,4-Dimethylpentan-1-ol

JEE Main/Boards 1 2 3 4 5
(B) CH3 CHCH2 CHCHCH

Exercise 1 OH C6H5
4-Phenyl-2-pentanol
(or 4-phenylpentan-ol)
Sol 1: The longest chain to which the hydroxyl group
is attached gives us the base name. The ending is ol. 1 2
(C) CH3 CHCH2 CH=CH2
3 4 5

We then number the longest chain from the end that


gives the carbon bearing the hydroxyl group the lower OH
number. Thus, the names, in both of the accepted 4-Penten-2-ol
IUPAC formats, are (or pent-4-en-2-ol)

(A) CH3 C HCH2 C HCH2OH


CH3 CH3
2,4-Dimethylpentan-1-ol

1 2 3 4 5
(B) CH3 CHCH2 CHCHCH
OH C6H5
4-Phenyl-2-pentanol
(or 4-phenylpentan-ol)
2 2 . 7 0 | Alcohols, Phenols and Ethers

Sol 2:
H H H OH Me H Me OH
(i) 5 1O
6 OH H OH H
Me Me H H
CH3 C
3 H2 Me
CH CH2 CH CH3 Me Me Me
2
4
Me Me Me Me
H OH OH(II) H H H
(I) (III) (IV)
H

 
O

     
OHe.iDil.NaOH
Pre CH3 C OH
CH2a.i
C HPre OHe.i Pre OH  a.
CH3 C H
Mee  OHMee Mea i Pr  e
H2/Pd
Me  a
Two (e) Two (e)
All equatorial (most stable) One (e)
Three (e) CH3 CH CH2bulky,
Me is CH2soOH
in Me is in (a)-position,
e-position; more stable least stable
OH so less stable
O
(ii) O OH
CH3 C H H C C C CH3 Me
CH3 C H H C C C CH3 --
1
2

-
O H H Me

-
O H H -

-
3
O H O Me 5
4
ODil.NaOH
CH C CH2H C H O
(Chiral C aloms at C-1,
CH3 C H
 Dil.NaOH 3 C-2 and C-5)
CH3 C H CH3 C CH2 C H

Stability order is: I > II > III> IV
H O
HCl
CH3 C C HC HO Sol 4:
(1)
HCl
CH3 H C C C H
Ph - -

-
Ph 2
O H H O
3 1 H- Me
1,2 H
(iii) Me
4
Me -
shift

-
Dil.NaOH H H 2C
O

CH3 C H CH3 C CH2 C H


 Ph 3
2
1 H-
Ph -
-
1,2 H
-
4 Me
OH shift
Me Ph Me - H₂O Ph
-
O
H H 2C
Me - Me
H-
-
H O Me Me
Ph OH H₂O Ph OH
HCl Me3O benzylic C -

-
Me -
CH3 C C C H H-
-

Me Me
OH OH
3 benzylic C -
-
O

H
(2)

 
O
-

 
CrO3/H2O/Acetone B₂H₆/THF
CH₂
CH3 CH CH C OH CH₂ - H₂O₂/OH
CH₂B₂H₆/THF H₂O₂/OHB OH
CH₂ B OH
H H
Cyclopentyl Cyclopentyl
Sol 3: There are three chiral C atoms and there are four methanol methanol
diastereomers, each with a pair of enantiomers. Thus
total stereoisomers are 8. Sol 5: a. Boiling point order: VI > IV > V > III > II > I
H H H OH
Solubility
Me H order: I > II > IIIMe
> V > IVOH> VI
5 1
H Explanation: All of them ate alcohols, so all have
6 OH H OH H
Me H
3 2 Me Me Me H-bonding. As the Me molecular mas and surfaceMe area
increases, the boiling point increases andMesolubility
4
Me Me Me
H H
(I) (II) H decreases.
(III) (IV) H

 
Out of (IV) and (V), there is branching in (V) and has less

     
OHe.i Pre OHa.i Pre e.i area
Pr OH  a.
surface
OH than
e (IV), so the boiling point of (IV) > (V),
i Pr  e
Mee Mee butsolubility
Me a of (V) > (IV).
Me  a
All equatorial (most stable)
Two (e) b. Boiling point order: I > III > IVOne
Two (e) > II(e)
Me is bulky, so in Me is in (a)-position,
Three (e)
e-position; more stable Solubility order: I > III > IV > IIleast stable
so less stable
Chem i str y | 22.71

In (I), there is H-bonding in (II) (aldehyde), dipole- Sol 8:


dipole interaction, in (III) (ether), slightly polar due to -5

-
3 4
EN of O, and in (IV) (alkane), van der waals interaction Me
H-

-
(non-polar). Ring
expansion
c. Boiling point order: II > III > I 2 1
-

-
(A) 4

Solubility order: II > III > I


H OH H OH H₂O 3 Me
In (II), three (-OH) groups, more H-bonding; in (II), one
- 5

-
H 2

(-OH) group, less H-bonding; in (I) (alkane), van der * H * Me 1


2 C-

-
O

Waals interaction. * *
H
Me +

Sol 6: In ortho-isomers of (I), (II), and III, intramolecular


-

-
Me
(trans) (cis)
H-bonding (chelation) occurs which inhibits the
3O C -

-
intermolecular attraction between these molecules
Optically active Optically active

H - H₂O
( ) or racemate ( ) or racemate
* *

-
and thus lowers the boiling point and also reduces
III and IV I and II
H-bonding of these molecules with H2O, thereby, Me
decreases water solubility. Intramolecular chelation
does not occur on p-and m-isomers. OH
(V)

O- H The total numberH of isomeric products including


OH
O
O stereoisomers isO5.
C O C O C CH₃
N
-
-

OSol 9: Greater the steric hindrance in the ether molecule


H H H
O
O encountered in the formation of the coordinate bond,
o-Nitophenol o-Hydroxybenzaldehyde o-Hydroxybenzoicweaker is the Lewis acetophenone
o-Hydroxy basicity. In (i), R groups (the side
(Salicyaldehyde) acid
of the ring) are ‘tied back’ leaving a very exposed O
(Salicylic acid)
atom free to serve as basic site. In other words, more
compact the molecule (due to ring), more easily O atom
H OH HO can donate its LP e ’s to the Lewis acid, and therefore,
O stronger the Lewis base.
C O C O C CH₃
H H Sol 10:
O O
o-Hydroxybenzoic Me Me -
o-Hydroxy acetophenone -
-
nzaldehyde SN1
-

- H
-

H
ehyde) acid (A)
Et MeOH Et
(Salicylic acid) -
-

O OH
-

 
-

3O C
Attack from
Sol 7: Chelated o-isomers have a minimum attraction H
backside

 
with H2O, and they are steam volatile or steam distills. Attack at MeOH
Steam volatile or steam distills are the compounds
less substituted SN2 Frontside attack
C atom. is blocked by
which are mixed with boiling H2O but not dissolved. On (CH₂OH) gp. Me
Me OMe
passing steam to such boiling mixture, steam carries * MeO *2 Et
the compound with it. 4
Et OH
2 1
1
OH
(C)
(B) (R)-2-Methyl-2-methoxy
(S)-2-Methyl-1-methoxy butan-2-ol butan-1-ol (optically pure
(optically pure compounds) compound)

(B) is obtained by SN2 by the attack of nucleophile


 .. 
 MeOH  at less substituted C, without changing the
 .. 
 
(2 alcohol) (2 carbocation )
CH3
CH3 CH3
CH3-CHOH +
CH3 CH+
H
CH3 CH O CH CH3 CH3 CH=CH2
2 2 . 7 2 | Alcohols, Phenols and Ethers 2-Propoxy-2-propane CH3 Propene
Isopropyl
carbocation

configuration or groups priorities and the product is CH3


H+
CH3 CH3 CH3
+ + -H+
CH3 C OH CH3 C OH2 -H2O
CH3 C CH3 C CH3
S-stereoisomer. CH3 CH3 CH3 2-Methylprop-1-ene
t-Butyl
(C) is obtained by SN1 ring opening to give stable Carbocation

 .. 
3° C⊕ .The nucleophile  MeOH  attacks from the
 ..  Sol 13: Proceed reserve from the oxidation of
 
backside because front side attack is blocked by the 2-methylpentan-3-ol.
(-CH2OH) group.
Me Me
1 2 4 [O] 2 4
3 3 5
Sol 11:
5
Me KMnO4 Me Me
Me 1
CH=CH OH OH
(A) dil.H2SO4 2-Methylpentan-3-ol 2-Methylpentan-3-one
+HOH Markovnikov
rule
The possible structure of (D) is:
Ethenyl benzene CH CH3
OH
Me
1-Phenylethanol
(or) Et 2 1 1.BH3/THF
Me Me CH2
CH2Br Me 3 -
2.H2O2/OH
 CH2 4
Anti-Mark.
(B) +NaOH S 2 hydrolysis Me
N add.
2-Ethyl-3-methyl butene Et 2
1
Cyclohexyl methyl
bromide CH2OH (D) OH
O3/Red. Me 3
+NaBr 4
Et Me
Cyclohexylmethanol O (Chiral)
Me + CH2 O (E)
 Methanal 2-Ethyl-3-methyl
(C) CH3CH2CH2CH2CH2Br + NaOH Me
SN2 hydrolysis butan-1-ol
1-Bromopentane
Et 1
CH3CH2CH2CH2CH2OH + NaBr 2 COOH
Pentan-1-ol *
Me 3
[O]
Me KMnO4
Sol 12: Acid –catalyzed dehydration of 1° alcohols (Chiral)
(F)
to ethers takes place by SN 2 reaction Involving 2-Ethyl-3-methyl butanoic acid
nucleophilic attack by the alcohol molecule on the
protonated alcohol molecule as.
Sol 14:
However, under these conditions, 2° alcohols give
alkenes rather than ethers. This is because of the
i. DU in ( A ) =
( 2nC + 2) − nH = ( 8 × 2 + 2) − 8 = 5°
stearic hindrance, nucleophilic attack by the alcohol on 2 2
the protonated alcohol molecule does not take place. ii. Five DU and ( C : H ≈ 1:1 ) suggest benzene ring
Instead of this, the protonated 2° and 3° alcohols lose a (4 DU) and 1 DU has to be accounted.
molecule of water to form stable 2° and 3° carbocations.
These carbocations then prefer to lose a proton to form iii. It does not contain (-COOH) group, since it is not
alkenes rather than undergoing nucleophilic attack by soluble in NaHCO3.
alcohol molecule to form ethers. iv. It is soluble in NaOH, which suggests phenolic (OH)
group.
+
CH3 CH OH H CH3 CH +OH2 CH3 CH+
CH3 CH3 CH3
v. (A) contains three O atoms, which suggests an ester
Propan-2-ol Isopropyl carbocation group (-COOR) and one phenolic (OH) group.
(2o alcohol) (2o carbocation )
CH3
The presence of an ester group is also indicated by the
reaction of (B) with NaHCO3.
CH3 CH3
CH3-CHOH H+
CH3 CH O CH CH3 CH3 CH+ CH3 CH=CH2
2-Propoxy-2-propane CH3 Propene

(A)
Isopropyl
NaOH Steam NaOH
carbocation 
Boiled
→ Residue 
distilled
→ Distillate  
CH3
H+
CH3 CH3 CH3 ( −COOR )  suggests ortho 
 compound 
+ + -H+  
CH3 C OH CH3 C OH2 -H2O
CH3 C CH3 C CH3
CH3 2-Methylprop-1-ene
CH3 CH3
t-Butyl
Carbocation
Chem i str y | 22.73

H⊕ Acid + 0.1 mol = 0.2 mol


 Dissolves
 Steam
distilled
NaOH
→ Distillate  → Yellow ppt.  → (B )( −COOH group )  
 in NaHCO 3  = 0.2 – 0.1 = 0.1 mol
suggests ortho 
compound H⊕   Dissolves 
∴ Acid
w ppt.  → (B )( −COOH group )  
 in NaHCO3  = 0.1 Eq.
1 Eq. of CH3COOH = 60 gm
vi. Yellow precipitate with NaOH is a characteristic test
0.1 Eq. of CH3COOH = 6 gm
for methyl salicylate.
Weight of acid = 6 gm
OH OH
Weight of phenol = 9.4 gm
COOMe COONa
NaOH 6
+ CH3OH Mass percentage of acid = × 100 =
20 %
30
(A) 9.4
Methyl salicylate
Sodium salicylate
Mass percentage of phenol = × 100 = 31.3 %
(Yellow ppt.) 30
(oil of wintergreen)
-
-
used as a flavoring H
agent Sol 16: i. Six DU in (A) and (C : H ≈ 1 : 1) suggest
OH OH benzene ring (4 DU).
COONa ii. (A) does not contain phenolic group since it does not
NaHCO3
+ 2CO2 (gas) +H2O dissolve in NaOH and does not colour with FeCl3.
iii. (A) reacts with 1 Eq. of H2 , which suggests one (C = C)
(B)
bond. Ozonolysis also suggests one (C = C) bond. It also
Solid. soluble in
counts one more DU.
NaHCO3, Salicylic acid
iv. Remaining two oxygen atoms must be present in
Sol 15: fused ring (which is conformed by the formation of 3,
4-dihydroxybenzoic acid with HI) (Acetal ring).
OH OH
Br Br Reactions:

+ 3Br2/H2O CH2 CH CH2 CH2 CH2 CH3

1 Mol of H2
Br
Mw = C6H6O Mw = C6H3OBr3 O O
= 331 gm O O
= 94 gm
(A) O2/Zn
(A) (B) CH2 CH O

331 gm of (B) is obtained from 94 gm of (A).


[O] KMnO4
+CH2 O
94 HI COOH
33.1 gm (B) is obtained from = × 33.1 O
331 O
(B) Positive Tollens test
= 9.4 gm of phenol COOH
1 O
Weight of phenol = 9.4 gm
6 2
3
+ CH2=O O
5 (C) (Mw. = 166)
9.4 OH
= = 0.1 mol 4
94 OH
ii. NaOH will react with both CH3COOH and phenol. 3,4-Dihydroxybenzoic acid

Total molar equivalent of NaOH = 100 × 2


= 200 mEq
200
= = 0.2 Eq. of NaOH
1000
= 0.2 mol of NaOH
Acid + Phenol = 0.2 mol
2 2 . 7 4 | Alcohols, Phenols and Ethers

Exercise 2 Sol 9: (C) Oxymercuration demercuration is anti


Markovnikov addition of water molecule to alkenes.
Single Correct Choice Type
Sol.10: (A) Ethers on hydrolysis gives alcohol.
Sol 1: (C) Four DU in A and (C: H ≈ 1: 1) suggest benzene
ring with one extra C atom. Reactivity with NaOH and
FeCl3 suggest (A) to be a phenol. The formation of a Sol 11: (C)
tribromo product suggests that o-positions are vacant. CH2OH CH2 OH2
+
Hence. (A) is m-cresol. H+
+H2SO4

Sol 2: (D) The ether preparation follows the following -H2O

steps- CH 2
+
+ ring
1. Protonation expantion
-H+
2. Nucleophilic substitution(SN2)
3. Deprotonation by the base and release of HCl by
shifting of bonds due to the presence of a good leaving
group to give stability.
Sol 12: (B)

Sol 3: (D) All of the given statements regarding glycerol MnO2 CH3MgBr
are correct. OH O O +CH4

OH OH OMgBr
Sol.4: (B)
alc. KOH
CH3 − CHBr − CH3 → CH3= HBr
CH CH2  Sol −13:
→ CH3CH CH(A)
CH3ONa
Peroxide 2 2Br → CH3CH2 CH2 OCH3
1 −Methoxypropane
Mg/dry
CH ONa
Cl Br Cl MgBr
HBr 3
H2 
Peroxide
→ CH3CH2 − CH2Br → CH3CH2CH2OCH3 1 mole
1 −Methoxypropane (A) CH3COCH3

Sol 5: (B) Ethers on exposure to sunlight slowly react OH


with oxygen from air to form peroxide. These peroxide
are unstable and decompose on distillation resulting Cl C CH3
violent reaction.
CH3

Sol 6: (B) Sol 14: (A)


OH OH OH CH3 CH3
CH3 H+ CH3
Br CH3 C CH CHH 3 -H O CH3
+ C CH CH3
Br2inCS2
+ CH3 C CH CH3 2
CH3 C CH CH3
293K CH3 OH -H2O CH3
CH3 OH CH3 1:2 methyl shift
1:2 methyl shift
o-Bromphenol Br
p-Bromopheno +
CH3 CH3
CH3 + C CH CH3 CH C = CCH
-H+
3 3
CH3 C CH CH3 C = C
Sol 7: (D) Phenol is more acidic than cresol but CH3 CH3 -H+ CH 3 CH 3
CH3 CH3
less acidic than nitrophenol. P-nitrophenol is more CH3 CH3
acidic than m-nitrophenol. Thus, the correct order is
p-nitrophenol > m-nitrophenol > phenol > cresol.

Sol 8: (B) The addition of a proton at β -carbon gives


a carbocation (I) which is resonance stabilized because
of electron donating effect of –OH group. The addition
of Br − ion to the carbocation gives the main product.
Chem i str y | 22.75

Sol 15: (D) Sol 20: (C)

OH CH3 PCl3
Mg/ether

Zn dust Anhy.AlCl3 O CH3OH O CH3Cl
+ CH3Cl
-ZnO +HCl
x + CH3 CH CH2
Alkaline +
H3O/H O (CH2)2 CH CH3
KMnO4 O CH3MgCl :O:
OH
COOH

Sol 16: (D) Previous Years’ Questions


O- O O
Sol 1: (A)
H H
-
- :CCl2
CCl2 CH3 CH3 CH3
-H2O Br-
CH3 C OH + H+ CH3 C+ CH3 C Br
O O
CH3 CH3 CH3
H
- H3O H tautomerism 2-methylpropan-2-ol o
H3O - 3 carbocation
CHCl2 CHCl2
Sol 2: (A) Ethanol is capable in forming intermolecular
O- O- OH H-bonds :
CHO CHO CHCl2
H+ OH- H O H O H

C2H5 O C2H5 C2H5

Sol 17: (A)


Sol 3: (D) OH → O– + H+ (has maximum electronegativity
CH2 CH2 + R Mgx CH2 CH2 R
H3O+
R CH2 CH2 OH difference)

O: OMgx
Sol 4: (B) Thiol, RSH, on combustion produces CO2(g),
SO2(g) and H2O. At 298 K, H2O will be liquid phase.
Sol 18: (D) Heating of alkyl halide with sodium or
potassium alkoxide gives ether. This is a good method Sol 5: (A)
for preparation of simple as well as mixed either known
as Williamson’s synthesis. O OH
+
O
+
O
H + -H
RX + NaOR’  R-O-R’ + NaX -H₂O
I conjugated
Sol 19: (B) O O O
+ +
H -H
OH OH
-H₂O +
CHO
NaOH OH II
+ CHCl3
Reimer Tiemann reaction
Salicyladehye Although both reactions are giving the same product,
carbocation I is more stable than II.

Sol 6: (A) A is an alcohol and its oxidation product gives


Tollen’s test i.e., B must be a aldehyde (CH3CH2CHO)
2 2 . 7 6 | Alcohols, Phenols and Ethers

Sol 7: (D) Sol 14: (B) Ortho–Nitrophenol is less soluble in water


than p– and m– Nitrophenols because o–Nitrophenol
CH3
+
H H shows Intramolecular H–bonding.
H + + C=C
H2O
trans H3 C cis CH3
H3C OH
Sol 15: (D) Iodoform is given by
1) methyl ketones R − CO − CH3
Sol 8: (C) 2) alcohols of the type R − CH ( OH) CH3

R OH Hydrophilic Where R can be hydrogen also

Hydrophobic O
||

Increasing molecular weight increases hydrocarbon (R) H3 C − C − C 2H5 
proportion that lowers the solubility in water. ethyl methyl ketone 

Sol 9: (F) 2º - alcohol on oxidation yields ketone while CH3 
1º alcohol on oxidation produces aldehyde which can
|

further be oxidized to acid.
H3 C − CH − OH  can give iodoform Test
Isopropyl alcohol 
Sol 10: (F) Ethanol is weaker acid than water, not O CH3 
neutralized with NaOH. || | 
H3 C − C − CH − CH3 
3 −methyl 2−bu tanone 
Sol 11: (D) Esterification reaction is involved 
H +
CH3COOH( ) + C2H5OH( ) → CH3
CH3COOC2H5 ( ) + H2O( ) |
H3 C − CH − CH2 − OH → cant give
Isopropyl alcohol
Sol 12: (D)
Kolbe – Schmidt reaction is Sol 16 :(C)
OH ONa ONa ONa

NaOH CO2
COONa
H2O+
COOH OH OH OH OH
6atm, 140oC
Salicylic acid > > >

Sol 13 : (A)
NO2 Cl CH3 OCH3
CH2 CH CH CH3 (-m, -I) (-I) (-I, +HC) (+m)
OH CH3
Electron releasing group decreases and electron
conc. H2SO4 withdrawing group increases acidic strength.

CH2 CH CH CH3 Sol 17: (B) The reaction of alcohol with lucas reagent
- is mostly an SN 1 reaction and the rate of reaction is
-

CH3
directly proportional to the carbocation stability formed
loss of proton
in the reaction, since 3° R − OH forms 3° carbocation
CH3 hence it will react fastest.
CH = CH HC (conjugated system)
CH3
Trans isomers is more stable & main product here
H CH(CH3)2
C=C (trans isomer)
H
Chem i str y | 22.77

JEE Advanced/Boards Therefore, molecular mass of R(OCOCH3)


= [M + (43 - 1)n]
Exercise 1 = (M + 42n)
nKOH
R(OCOCH3 )n → R(OH)n + nCH3COOH
Sol 1: ROH reacts with cold conc. H2SO4 as follows:
Molecular mass of nKOH = 56n
1. ROH + H2SO4 ROH2 + HSO4 0.218 gm of acetyl derivative requires 0.168 gm of KOH
for hydrolysis.
ROSO2 OH + H2O
∴ (M + 42n) gm of aceytyl derivative requires
2. H2SO4 + H2O H3O + HSO4 0.168(M+ 42n)
= = 56n
ROH + 2H2SO4 ROSO2OH + H3O + HSO4 0.218
On solving, we get n = 3
Number of moles of particles formed per mole of
 w(M+ 42n) 
solute (i) (van’t Hoff factor) = 3 (The reaction does not Use the formula: = 56n 
produce R , because R ion or even R3C⊕ ion is not
⊕ ⊕
 W 
 w = Weight of KOH, W = weight of acetyl derivative,
stable enough to persist)  
M = Molecular mass of alcohol; n = Number of
 
∴ ∆T=
f iK f × M (-OH) groups 
= 3 x × 0.1 = 0.3 x K
Sol 3:
Sol 2: Let the formula of alcohol is R(OH)n, and the
formula of its acetyl derivative is R(OCOCH3)n. - H₂O H
H-OH₂ OH₂ +
Molecular mass of R(OCOCH3) = (M + 42n)
OH OH +
Ph Ph + + H₃O
Ph Ph
M is the molecular mass of alcohol. (b) Initial reaction of the alkene with H3O+ can form
Molecular mass of (CH3 – CO -) group = 43 two carbocations. The more stable benzylic tertiary
carbocation (shown in the above mechanism) is formed in
One H atom is replaced by (OH) group of CH3COOH
preference to the less stable primary carbocation. This is
group
the rate determining step, and thus controls the product
distribution. Formation of the more stable carbocation is
the mechanistic basis for Markovnikov’s rule.
Sol 4:
Me Me
3
Ha 4 Ha
2

Ha 1 5 Ha
6
Me Me
-
Note : Ha and Hb atoms are (I) Excess of B ₂H ₆ + THF (ii) H ₂O ₂/OH
abbreviated only for Anti-Mark, addition of Hb and OH
understanding the problem. Addition of Hb and OH syn (Hb comes from B ₂H ₆)

Me Me Me Me Me Me Me Me
HO 3 OH HO 3 Ha HO 3 Ha HO 3 Ha
4 4 4 4
Ha 2
Ha Ha 2 OH Ha 2
Hb Ha 2 Hb
Ha 1 5 Ha Ha 1 5 Hb Ha 1 5 OH Ha 1
5
OH
Hb 6 Hb 6 6 6
Hb Ha Hb Ha Hb Ha
Me Me Me Me Me Me Me Me

      
(C₁) (OH at 2 and 4) (C₂) (OH at 2 and 4) (C₃) (OH at 2 and 5) (C₄) (OH at 2 and 5)


Both -OH in cis but both Both -OH in trans Both -OH in trans
Both -OH in cis
-OH and Hb are in anti and both -OH and Hb and both -OH and Hb
bt both -OH and Hb
position are in anti position are in anti position
are in anti position

 Optically inactive
plane of sysmm.  (Optically active)
(Optically active)
 Optically inactive
centre of symm. 
2 2 . 7 8 | Alcohols, Phenols and Ethers

Thus, the number of isomers excluding enantiomers is 4.

Me Me
3
Ha 4 Ha
2

Ha 1 5 Ha
6
Note : Although first step of this reaction, addition
Me Me
of electrophile AcOHg to (C = C) to form -
(I) Excess of Hg(OAc)2/H2O (ii) NaBH4/OH
mercurinium ion is stereospecific. But second
step, addition of H - from NaBH4 is of no clear Mark, adition of Hb and OH
-

stereospecificity, but it is assumed to be and to (OH) gp. Addition of Hb and OH anti (Hb comes from NaBH4)

Me Me Me Me Me Me Me Me
HO 3 OH HO 3 Ha HO 3 Ha HO 3 Ha
4 4 4 4
Ha 2
Ha Ha 2 OH Ha 2
Hb Ha 2 Hb
Ha 1 5 Ha Ha 1 5 Hb Ha 1 5 OH Ha 1
5
OH
Hb 6 Hb 6 6 6
Hb Ha Hb Ha Hb Ha
Me Me Me Me Me Me Me Me

       
(C₁) (OH at 2 and 4) (C₂) (OH at 2 and 4) (C₃) (OH at 2 and 5) (C₄) (OH at 2 and 5)
Both -OH in cis but both Both -OH in trans Both -OH in cis Both -OH in trans
-OH and Hb are in anti and both -OH and Hb bt both -OH and Hb and both -OH and Hb
position are in anti position are in anti position are in anti position

 Optically inactive
plane of sysmm.  (Optically active) (Optically active)

Optically inactive
centre of symm. 
Thus, the number of isomers excluding enantiomers is 4. Excess of 2 H /Pt
X 
CH MgBr
→ CH4 ; X →
HI excess
n − pentane
3
Note: The products B1 and C1, B2 and C2, B3 and C3, B4
and C4 are same. Above information suggest that X has a terminal triple
If the reaction (a) is carried out with excess of B2D6 + bond and it contain primary –OH group.
THF + H2O2/ OH and (b) is carried out with excess of 3 2 Ag(NH )+
⇒ H − C ≡ C − CH2 − CH2 − CH2OH  →
Hg(OAc)2 + H2O + NaBD4, then in place of Hb, D will
Ag − C ≡ C − CH2CH2CH2OH
come in all product, and the product, and the products
B1 and C1, B2 and C2, B3 and C3, and B4 and C4 would be
different.
Sol 7:

Sol 5: (A) Positive Liebermann’s test (test for phenol)


+ CHCl3+KOH
OH CH₂ B + C (Stem volatileortho)
+
+
H Ring
expansion Positive Schiff’s test (test for (-CHO)group)
X LialH4 [O] Acetylation
(C) (D)(C7H8O2) (E)
O
H Aspirin (pain killer)
O₃
X
Zn/CH₃COOH OH OH OH
O CHO
Y Reimer–
O Tiemann
reaction (C)
NaOH CHO
aldol (B)

Lucas reagent
Sol 6: Compound ' X' 
→ No reaction at OH OH OCOCH3
room temperature.
CH2OH COOH COOH
[O] Acetylation

Ammoniacal
C5H8 O →
AgNO
ppt (D) (E) (F)
3
Aspirin
Chem i str y | 22.79

Sol 8:  2.53 × 55  −1
M=  gm mol = 168 gm mol-1
 1.84 × 0.045 
H2/Ni
(A) and (B) (C)
C11H13OCl
COOH Number of unit of empirical formula in the molecular
(Unsaturated) 1 formula
6 2
[O]
168 gm mol−1
=
3
5 =2
4 OC2H5 84 gm mol−1
Cl
(Nice C atom) Hence, the molecular formula of the minor product
(So two C atoms in the side chain with double bond) is 2 ( C3H2NO2 ) , i.e., C6H4 (NO2 ) . The product is
2
m-dinitrobenzene.
H H H CH3
C C C C
CH3 . H Sol 10:
A
OC2H5 OC2H5 O
Cl Cl
(cis) (D) (trans) (E) OH OH O C CH3
COOH COOH
Steps 1,2,3
CH3COCl
H2/Pi
CH2CH CH2 CH2CH2CH3 (A) (B)
OH
B C Br2 COOH
OC2H5 OC2H5 Fe
Cl Cl
Br (C)
Sol 9: The rations of atoms in the minor products are: Sol 11:
42.86 2.40 16.67 38.07
C :H:N: O :: : : :
12 1 14 16 -
OH Cl O Cl
: : 3.57 : 2.40 : 1.19 : 2.38 OH
:: 3 : 2 : 1 : 2 H3C Cl H3C Cl
H Cl H Cl
Empirical formula of the minor product: C3H2NO2
Molar empirical formula mass of the minor product is O
- H O
N3
(3×12 + 2 × 1 + 1 × 14 + 2 × 16 ) gm −1
mol = 84 gm mol -1 H3C
H
Cl
Cl
H3C
Cl
Let M be the molar mass of the minor product. For 5.5 N3
gm of the minor product dissolved in 45 gm benzene,
the molality of solution is given by H
O
O -
H2O H2/Pd O
55 gm / M H3C
m= OH H3C
0.045 kg N3 - NH
-

3
Substituting this in the expression of elevation of
boiling point, we get
∆Ta =
K bm

(  55 gm / M 
1.84 K = 2.53 K kg mol−1 
 0.045 kg 
 )
2 2 . 8 0 | Alcohols, Phenols and Ethers

Sol 12:
O (glycol)
OH Protection of (C-O) O
Br HO Br
H3C -

-
H O CH3
Mg(C2H5)2O
O
HCHO/H -
4 6 O

-
1 2
H3C 3 5 OH MgBr
6-Hydroxy hexan-2-one O CH3

Sol 13: It is an intramolecular SN2 -type reaction that proceeds through an intermediate epoxide.

Ph Ph
I Ph Ph OH - O- -
Ag -

-
-

Me OMe Me - - OMe Me OH - ClCH3COO


OMe

-
-Agl
Ph OH Ph OH O
(A) (A) -

-
O (i) H O COO -
(ii) SOCl2

Ph Ph HPh
(B)
Ph Ph Ph O
-H -
AlCl3
-

Me C
OMe - OMe Intramole
Me cular OMe
ShiftF.C. reaction -
-

-
O Me O Me OH
(More stable diphenyl C- )

-
(B) O

(D) O

Sol 14: t-Butylalcohol on heating in the presence conc. -H3O


Taut
H2SO 4 forms a stable 3° carbocation which then reacts (b) O OH+ H N O
with C2H5OH (nucleophile) to give the product. (E) (F)
H H
CH3 (E) D2/Ni
H3C O N O N O
Syn-add.
CH3 CH3 D
(H) (G)
H B
Sol 16:
Sol 15: (A)
OH - O- - 1 3 5 Br2
-

O-
ClCH3COO
-
: :

OH HO
:
2 4 -
OH
(A) (Pent-4-en-1-ol) Br -
-
-

COO -
-

O (i) H O (A)
(ii) SOCl2
Br
H (B)
AlCl3 O O
C
Intramolecular
F.C. reaction
O
Exercise 2
(D) O
Single Correct Choice Type
Taut -H3O
O OH+ H N O
Sol 1: (D) All of the above given statements are correct.
H H (E) (F)
(E)
N O
D2/Ni
N O
Sol 2: (B) vicinal diol sites are only two, thus only 2
Syn-add. equivalents of HIO4 will be consumed
D (G)
H B (H)
Chem i str y | 22.81

Sol 3: (D) Refer mechanism of esterification in the Sol 9: (A) Although both reactions are giving the same
theory. product, carbocation I is more stable than II.

O OH O O
Sol 4: (A) iv. C2H5O‒ acts as a base. It abstracts H⊕ H
+
+ -H
+
from phenol to form PhO‒ ion. C2H5O‒ is a stronger -H₂O
nucleophile than PhO‒. Hence the product is obtained I conjugated
by path II. O
O O
CH3 + +
H -H
S N2 -H₂O
p-NO2 C6H4O CH2 I +
Path I OH II
Path II
CH3O SN2
Sol 10: (B) BF3, being a good lewis acid accepts a pair
CH3 CH2 OCH3 of electrons to give us a good conjugate acid, and not
a nucleophile.
p-NO2 C6H4 O CH2 CH3
(Acidic character: PhOH > C2H5OH) Sol 11: (B) Method (c) would give rearranged product
also. It would give a mixture of 2-bromo and 3-bromo
(Basic and nucleophilic character: PhO‒ < C2H5O‒)
pentane. In methods (a) and (b), no rearrangement
occurs and it gives (B) exclusively. The tosyl group a
Sol 5: (A) 1. Presence of electron-donating or electron- good leaving group, is then easily displaced by reaction
withdrawing group on the respective rings. with Br– in an SN2 reaction.
2. SN2 reaction mechanism is followed in which
protonation is followed by attack of halo group. Sol 12: (C) c. 2.68 gm of (A) gives 14.08 gm of AgI
14.08 × 134
Sol 6: (C) 134 gm of (A) gives = 704 gm of AgI
2.68
704
= mol of AgI
235
= 3 (OMe) groups

O
R O H+HO C CH₃ O
R O C CH₃

Sol 13: (B)

Me
MgBr - OH H -
-

H₃O
+ CH₃ CH O

(A) Me
Sol 7: (C) CH3 − CH2 − O − CH2 − CH3 + HI → 2C2H5 I
- 1
-

Me Me 3 2
-
-

Me - 2 Ring
ZnO −Cr2O3 Br - 3
-

Sol 8: (A) CO + H2 → CH3OH Br expansion


heat 4 5 4 5
(B)
2 2 . 8 2 | Alcohols, Phenols and Ethers

Comprehension Type: Previous Years’ Questions


Sol 14 to 16: (C, C, B) Refer Reimer-Tiemann reaction
from the theory part. Sol 1: (B) C6H5MeBr+(CH3)3 COH → C6H6 +
Mg[(CH3)3CO]Br
Assertion Reasoning Type
Sol 2: (B)
Sol 17: (D) Phenols can be chlorinated.Moreover,
presence of –OH on the benzene ring, is an electron-
donating group which makes the attachment of the Cl
electrophile on the o- and p-position possible.
Concentrated H3PO4 solution does not involve any
substitution product while with others, substitution
Sol 18: (A) self-explanatory. Remember, -NO2 is an
products are also formed.
electron-withdrawing and –CH3 is an electron-donating
group.
Sol 3: (C) All dihydroxy benzene will have higher boiling
points, then monohydroxy benzene. Also among
Sol 19: (D) Electron-withdrawing nature of –NO2 and
dihydroxy benzenes, 1,2-di-hydroxy benzene has
electron-donating nature of –O-Ar makes the reaction
lowest boiling point due to intra-molecular H-bonding.
possible.

Match the Columns

Sol 20: A → q, r; B → p, s; C → u; D → p, t; E → p,
r, s; F → p, s
A. Reimer-Tiemann reaction proceeds by (CBrCl)
(bromochlorocarbene), which acts as an electrophile.
So, it is an SE reaction.
B. The reaction proceeds by the formation of carbocation
with rearrangement. Sol 4:
C. No reaction proceeds by the formation of carbocation
with rearrangement.
D.

a. Me O O CH₂
HI SN1

-
-

b. HO OH+ CH₂
excess -
-

Stable benzyl C
HI
-
I

No reaction, ArSN reaction


does not occur unless ring is
ICH₂ Sol 5: (C, E)
activated by EWG(e.g.NO₂) ••
CH3CH2Br
= + O N – O – → CH3CH2NO2 + CH3CH2ONO
ambident
E. It is Friedel-Crafts alkylation which proceeds by the nucleophile
nitroethane ethyl nitrite
formation of a carbocation followed by rearrangement.
So, it is an SE reaction.
F. It is hydration of alkene and proceeds by the formation
of a carbocation with rearrangement.
Chem i str y | 22.83

Sol 6: (A, D) Sol 9: (D)

Sol 10: Acid catalysed dehydration proceeds via


carbocation intermediate. Also, greater the stability of
reactive intermediate, faster the reaction :

Phenol does not react further with HI.

Paragraph 1: Compound J must be benzaldehyde


because it one treatment with KOH undergoing
Cannizaro’s reaction producing benzyl alcohol and
potassium-benzoate (L).
KOH
C6H5 – CHO  → C6H5 – CH2OH+ C6H5COOK(L) n-butanol forms less stable (1º) carbocation.
J benzyl alcohol

Sol 11:
Also M is aldol condensation product formed from
acetophenone

Sol 12:

⇒I=

Sol 13:

Sol 7: (B)

Sol 8: (A)
I=
2 2 . 8 4 | Alcohols, Phenols and Ethers

Sol:14 (B, D) Sol 17: (A, B, C)


OH OH OH OH

CHO I
I2
CHCl3
+ Rxn (i)
OH
CMe3
OH OH
H3C CHCl2
CH3 (Minor) CH3 (Major) Br
Br2
OH O Rxn (ii)
CMe3 CMe3
CCl2
+ CCl2 Br
H
OH
Cl Cl
CH3 CH3
Cl2
O OH Rxn (iii)
CHCl2 CHO CMe3
OH
Br

CH3 CH3 (Major) Sol 18: (C) When two phenyl groups are replaced by
O O O two para methoxy group, carbocation formed will be
more stable.
H2O

Sol 19: (9)


:CCl2 H3C CCl2 H3C CHCl2
CH3
(Minor) HO
NaOH
N

Sol 15: (A, C, D) N is


O-

1.21A -
O- O O
o
117
-
1 2 3
O O- O

- -
Sol16: (B) 5 6
4
OH OH O O O-

Br Br
Br2(3equivalence) 7 8 9

Sol 20: (D)

SO3H Br
H+
H H

+
HO

(P)
aqueous KMnO4
o
0C

OH

HO OH
HO
(Q)
Chem i str y | 22.85

Sol 21 (C) and 22 (D):

CCH CH=CH2
Pd/BaSO4

H2

(1) B2H6
HgSO4, H2SO4, H2O (2) H2O2, NaOH, H2O
O
CH2 CH2 OH
C CH3

(X)
(1) EtMgBr
(2) H2O

OH CH3
H+ /heat
Ph C CH3 Ph C = CH CH3
(Y)
Et

Sol 23: (B,C)

CH3
CH3 C H OH
O
(i) O3
(ii) H3O
+ CH3 C CH3

(P)
OH OH OH
CHO
CHCl3+NaOH
+

(Q)
CHO
Steam Volatile (R)
OH O- O CH2 Ph
CHO CHO
NiOH Ph-CH2Br CHO

(Q) (S)
2017-18 100 &
op kers
Class 12 T
By E ran culty
-JE Fa r
IIT enior emie .
S fP r es
o titut
Ins

CHEMISTRY
FOR JEE MAIN & ADVANCED
SECOND
EDITION

Exhaustive Theory
(Now Revised)

Formula Sheet
9000+ Problems
based on latest JEE pattern

2500 + 1000 (New) Problems


of previous 35 years of
AIEEE (JEE Main) and IIT-JEE (JEE Adv)

5000+Illustrations and Solved Examples


Detailed Solutions
of all problems available

Plancess Concepts
Topic Covered Tips & Tricks, Facts, Notes, Misconceptions,
Key Take Aways, Problem Solving Tactics
Aldehydes and Ketones
PlancEssential
Questions recommended for revision
23. ALDEHYDES AND KETONES

CARBONYL COMPOUNDS

1. INTRODUCTION

Carbonyl compounds have a general formula CnH2nO and contain a >C=O group which is present in aldehydes
R
C=O as well as in C=O ketones. They are constituents of fabrics, plastics and drugs. These are also used
H R
as reagents and solvents.

1.1 Structure of Carbonyl Compounds


In carbonyl group both the carbon and oxygen atoms are in sp2 hybridised state. One of the sp2 hybrid orbital of
one carbon atom overlaps with one of the sp2 hybrid orbital of oxygen atom forming C–O σ-bond. The remaining
two sp2 hybrid orbitals of C atom overlap with either sp3 orbital of C-atoms (as in ketone) or one with sp3 orbital
carbon and other with s orbital of hydrogen (as in aldehyde forming) 2 more σ-bonds. On the other hand each of
two sp3 hybrid orbitals of ‘oxygen’ atom contains a lone pair of electrons. Unhybrid orbitals present at the carbon
and oxygen atom form π- bond by sideways overlapping. The structure can be represented as:

2 1 1 1 1
C* = 1s 2s 2px 2py 1
2pz Unhybrid orbital O = 1s2 2s2 2p 2 2p 1 2pz Unhybrid orbital
x y

2 2
sp hybridization sp hybridization
2 2
3sp hybrid orbital 3sp hybrid orbital

unhybrid orbitals

2 2
sp 2 sp
2
sp sp
2 2
sp sp

Figure 23.1: Structure of carbonyl compounds


2 3 . 2 | Aldehydes and Ketones

Thus C = O group contains one σ-bond and one π-bond as



C  O:

:
H
1.2 Bonding in Aldehydes and Ketones
The carbonyl carbon atom is sp2 hybridized. The unhybridized p-orbital form a π-bond with a p-orbital of oxygen.
The double bond between carbons and oxygen is shorter, stronger and polarized.

120
o C O

R
o
120

Figure 23.2: Bonding of carbonyl compounds

Length Energy

Ketone C =O bond 1.23 Å 178 kcal /mol (745 kJ/mol)

Alkene C = C bond 1.34 Å 146 kcal/mol (611 kJ / mol)

The double bond of the carbonyl group has a large dipole moment because oxygen is more electronegative than
carbon.
R R
: :

: :

 R
C=O
R
C=O
:

major minor

1.3 Comparison of C=O Bond with C=C Bond


(a) Both atoms in both the cases are in the sp2 hybridised state.
(b) Both the cases contain one σ-bond and one π-bond.
The difference between C=O and C=C is because of O-atom in carbonyl group is more electronegative than carbon
+ -
as a result polarity is developed as -> C = O

Thus, the double bond of the carbonyl group has a large dipole moment. This polarity conforms that there is
nucleophilic addition reaction in the carbonyl compound on other hand in alkene (C = C) there is electrophilic
addition reaction

2. METHOD OF PREPARATION OF ALDEHYDES AND KETONES


2.1 From Alcohols

2.1.1 By Oxidation of Alcohols


1° Alcohol on oxidation using PCC gives an aldehyde. 2° alcohol gives a ketone on oxidation by Na2Cr2O7
[O]
RCH2OH  → R − CH= O
Chem i str y | 23.3

PLANCESS CONCEPTS

•• If acidified K2Cr2O7 or KMnO4 is used then aldehyde further oxidise to give acid.

•• PCC (Pyridinium chloro-chromate in CH2Cl2) and Collin’s reagent (CrO3 Pyridine) are used to get
aldehyde from 1° alcohol. These reagents do not attack at double bond.
Saurabh Gupta (JEE 2010, AIR 443)

Illustration 1: In the following reaction when we mix Dichromate with Ethanol, the colour of the solution changes
from orange to green. Give reason. CH3 − CH2 − OH + Cr2O27− + H+  (JEE ADVANCED)

Sol: Dichromate is an oxidizing agent and during the reaction itself undergoes the reduction process and the green
colour is due to the formation of reduced chromate Cr3+
Dichromate is a good oxidizing agent, it oxidizes the primary alcohol to Acid and itself gets reduced to Chromium
ion (Cr+3).The colour of dichromate is Orange and that of Chromium is green. This is shown by the following
equation:

CH3 − CH2 − OH + Cr2O27− (Orange) + H+ → CH3 − COOH + Cr 3+ (green)

Thus this method is not useful in preparation of aldehydes and ketones.

Illustration 2: Complete the following reaction:  (JEE MAIN)

OH

PCC

Sol: As the alcohol is a 2° alcohol, and PCC is a mild oxidizing agent. So, we get a ketone as a product.

OH O

PCC

2.1.2 By Dehydrogenation of Alcohols


Dehydrogenation means removal of hydrogen and the reagent used is heated copper.
Cu/300°C
1° alcohol (RCH2OH)  → Aldehyde (R − CH = O)
−H2

Cu/300°C
2° alcohol (R 2CHOH)  → Ketone (R 2C = O)
-H2

Cu/300°C
3° alcohol  → Alkene
−H2O
2 3 . 4 | Aldehydes and Ketones

PLANCESS CONCEPTS

2° alcohol can also be oxidised to ketone by aluminium t-butoxide. During the reaction, 2° alcohol is first
refluxed with reagent [(CH3)2CO]3 Al, followed by acetone.

3R 2CHOH + [(CH2 )3 CO]3 Al →(R 2CHO)3 Al + 3(CH3 )3 C − OH

CH CH3
C=O 3R2C = O + CO Al
(R2CHO)3Al + 3 CH CH3
Ketone 3

Neeraj Toshniwal (JEE 2009, AIR 21)

Illustration 3: Complete the following reaction.  (JEE MAIN)


CH3
Cu/300°C
CH3– C– OH
CH3

Sol: It is an example of dehydrogenation of alcohols. The product formed will be an alkene.


CH3 CH3
Cu/300°C
CH3– C– OH CH3– C=CH2
–H2O
CH3

PLANCESS CONCEPTS

As the reaction proceeds at high temperature, so a thermodynamically stable product would be favored
in this case.
Aman Gour (JEE 2012, AIR 230)

2.2 From Hydrocarbons

2.2.1 Hydration of Alkynes


It is the addition of water in the presence of a heavy metal ion. Acetylene on hydration gives aldehyde while any
higher alkyne gives ketone.
Hg++ /H SO
2 4 → CH − CH = O
H − C ≡ C − H  3
H2O

Hg++ /H2SO 4
R − C ≡ C − H  → R − C − CH3
H2O
||
O
For example, In the case shown below, by varying the Alkyl (–R) group, the product also varies accordingly.
Hg++ /H SO
2 4 → R − C − CH
R − C ≡ C − H  3
||
O
(A) (B)
Chem i str y | 23.5

R A B
H H-C≡C-H
O=C–CH3
H

CH3 CH3–C–CH3
CH3–C≡C–H
O

Cl-CH2-
Cl–CH2–C≡C–H Cl–CH2–CH2–CH = O

In the above reaction, the carbonyl group will be formed on that carbon of the alkyne which is easy to attack by the
nucleophile (water in this case). Thus, a less crowded carbon will favour the formation of a carbonyl group whereas
a more crowded carbon will not favour it. . Therefore, in case (iii), the carbonyl group is formed on that carbon
which is easy to attack

PLANCESS CONCEPTS

•• The preparation of carbonyl compounds from the alkyne depends upon the R part and also presence
of inductive effect of the group attached to R.
•• Carbonyl group will be at the C-atom at which H2O will attack as a nucleophile
B Rajiv Reddy (JEE 2012, AIR 11)

2.2.2 Hydroboration of Alkyne


Hydroboration is a process used to get an aldehyde from a terminal alkyne. Here regents are (i) diborane (B2H6) (ii)
H2O2(OH–)

(i) B H
2 6 → R − CH − CH= O
R − C= C − H  2 In this reaction, Borane (BH3) is the electrophile.
(ii) H2O2 /OH−

BH H O
3 →(R =
3R − C ≡ C − H  2 2
− CH CH)3 B → R–CH=CH–OH R–CH2–CH = O

A higher alkyne except a terminal alkyne will give a ketone during hydroboration.
O
(i)BH
3→
CH3 − CH2 − C ≡ C − CH2 − CH3  CH3–CH2–C–CH2–CH2–CH3
(ii)H2O2 (OH− )

PLANCESS CONCEPTS

For an unsymmetrical alkyne ketone will be corresponding to that carbon atom over which electrophile
BH3 will attack. It depends upon the inductive effect and finally the polarization of π-electrons of C=C
bond. Hydroboration occurs on the anti-Markownikoff position.
O
||
(i) BH3 ⋅ THF
CH3–C≡C––CH2–CH3 CH3–C–CH2–CH2–CH3
(ii) H2O2(OH–)
(+ I of ethyl is more than CH3)
Rohit Kumar JEE 2012, AIR 79
2 3 . 6 | Aldehydes and Ketones

2.2.3 Ozonolysis of Alkenes


Ozonolysis is used to get carbonyl compounds from alkenes. The reaction is -

O
R1 R3
C
R1 R3 O3 C Zn-H2O R1 R3
R2 R4 C=O + C=O + ZnO
C=C
R2 R4 R2 R4
O O
Mono-ozonide

Note: That the carbon-carbon double bond is broken and we are forming a carbon-oxygen double bond on each
of the two carbons that originally composed the alkene. The second step in ozonolysis is called the “workup”. There
are two different types of “workup”, and the most common is referred to as the “reductive workup”. In this step,
we add a reducing agent (commonly zinc metal or dimethyl sulfide) that decomposes the intermediate formed at
the end of the ozonolysis reaction (which is called an “ozonide” ). The third oxygen of ozone is now attached to
what used to be our reducing agent (which may be either zinc oxide (ZnO) or dimethyl sulfoxide (DMSO). Using
a “reductive workup” preserves all other aspects of the molecule save the double bond. So if we start with, say, a
trisubstituted alkene, as in the example, we will end up with a ketone and an aldehyde. [What happens if the alkene
carbon is attached to two hydrogens? It becomes formaldehyde, which is then further converted to carbon dioxide]
note that the H
CH3 CH3 is preserved
H (i) O3 O H
O
H3C (ii) (CH3)2 S H3C
CH3 CH3
Ketone Aldehyde

“Reductive workup” merely cleaves the C=C bond and replaces with oxygen
Note: That although (CH3)2S is written as the reductant here, it’s essentially CH3 CH3
interchangeable with Zn for our purposes. An interesting consequence of (i) O 3

ozonolysis is that if the alkene is within a ring, you end up with a chain (ii) (CH3)2 S O
containing two carbonyls: Cyclic alkenes becomes chains -If your molecule H
has multiple alkenes, then you will end up with more than two fragments. H O
For many years, ozonolysis was used as a method for determining the
structures of unknown molecules. By “stitching” together the fragments and analysing them, it is then possible to
deduce what the original structure was.
[This was particularly important in the case of unsaturated molecules known as terpenes]. Here’s one example:
Molecules with multiple alkenes are cleaved into fragments:

H H H
H
(i) O3 O O
(ii) (CH3)2 S O
O H

This isn’t the end of the story with ozonolysis. There’s a second type of workup that can be used, which is referred
to as oxidative workup. Instead of using Zn or S (CH3)2, if we use the oxidant hydrogen peroxide [H2O2], any
aldehydes that form will be oxidized to give carboxylic acids. Like in the example below – notice that the C-H bond
is oxidized to C-OH [but all the other hydrogens remain intact].
“Oxidative workup” oxidized sp2 hybridized C–H bonds to C–OH as well as cleaving C = C
Chem i str y | 23.7

note that H is
CH3 CH3 replaced with OH
H (i) O3 O OH
H3C (ii) H2O2 H3C O
CH3 CH3
Ketone Aldehyde
Typical oxidant used for “oxidative workup” is H2O2; this oxidizes any aldehydes to carboxylic acids

PLANCESS CONCEPTS
•• This method is used because the double bond in olefin or exact structure of hydrocarbon can be
determined by knowing ozonolysis product i.e. by placing double bond at the place of two carbonyl
oxygen groups of two carbonyl compounds
•• Among the molecules of carbonyl compounds produced:
•• If there’s a single molecule containing two carbonyl groups, then hydrocarbon will be alkadiene.
•• If all the three molecules contain two carbonyl groups, then hydrocarbon will be cycloalkatriene.
•• An alternative to using ozone for the oxidative workup is to use the reagent KMnO4, especially in the
presence of hot acid; this will lead to the same result.
Krishan Mittal (JEE 2012, AIR 199)

Illustration 4: Which hydrocarbon on ozonolysis gives a mixture of acetone, acetaldehyde and methyl glyoxal?
(a) 2, 3-dimethylhexa-2-4-diene (b) 2, 4-dimethylhex-2,4-diene (JEE ADVANCED)

Sol:
(a) CH3 O O
| ozonolysis | ||
CH3–C= C–CH=CH–CH3 CH3–C=O+CH3–C–CH=O+CH3–CH=O
|
CH3 (A)

2, 3-dimethylhexa-2-4-diene

(b) CH3 O CH3


| ozonolysis
|| |
CH3–CH= C–CH=CH–CH3 CH2–CH=O+CH3–C–CH=O+CH3–C=O
|
(B) CH3

2, 4-dimethylhex-2,4-diene
Answer will be both isomeric structures (A) and (B)

Illustration 5: Which hydrocarbon on ozonolysis gives 3 moles of glyoxal (JEE MAIN)

Sol: As for formation of 3 moles of glyoxal we require a compound having six carbon atoms and three site of
unsaturation so that ozonolysis can occur.so it appears to be an aromatic compound having six carbon backbone.
I.e. Benzene
Benzene on ozonolysis gives 3 moles of glyoxal.
ozone
Benzene  → Benzenetriozonide
2 3 . 8 | Aldehydes and Ketones

CH
O=CH
CH CH ozonolysis CH=O CH=O
+ CH=O
CH CH CH=O
CH O=CH

2.2.4 Wacker Process


Alkenes can directly be oxidised to corresponding aldehydes or ketones by treating them with a solution of PdCl2
containing a catalytic amount of CuCl2 in the presence of air or O2. Apart from ethane, any higher alkene will give
a ketone.
CuCl2
CH2=CH2+H2O+PdCl2 CH3–CH=O+Pd +2HCl
air or O2
O
CuCl2 ||
R–CH=CH2+H2O+PdCl2 R–C–CH3+Pd+2HCl
air or O2

PLANCESS CONCEPTS

During the reaction PdCl2 is reduced to Pd and CuCl2 is reduced to Cu(I)


T P Varun (JEE 2012, AIR 64)

2.3 From Grignard’s Reagent


(a) Hydrogen cyanide on treating with Grignard reagent followed by double decomposition with water gives an
aldehyde via aldimine
H2O/H
+
H2O/H
+
H–C=O+NH3
H–C≡N+R–MgBr →H–C=NMgBr H–C=NH
| | |
R R R
Aldimine Aldehyde

Alkylcyanide by using above process gives ketone via ketimine



H2O/H+ H2O/H+
R'–C≡N+R–MgBr →R'–C=NMgBr R'–C=NH R'–C=O+NH3
| | |
R R R
Ketimine Ketone

(b) Alkylformate with Grignard reagent gives 2° alcohol via aldehyde while alkyl alkanoate under similar condition
gives 3° alcohol via ketone

O OMgBr
|| | H3O+ (i) R–MgBr
H–C–OC2H5 +R–MgBr → H–C–OC2H5 R–CH=O R–CH–OH
| –MgBr (OC2H5) (ii) H2O/H+ |
Ethylmethanoate R Me

R 222alcohol
O OMgBr
|| | H3O+ (i) R–MgBr |
R'–C–OC2H5 +R–MgBr → R'–C–OC2H5 R'–C=O R–C–OH
Br | (ii) H2O/H+ |
| –MgBr
R OC2H5 R R 3° alcohol
Chem i str y | 23.9

Illustration 6: Complete the following reaction:


(i)CH MgBr(excess)
2 4 → B  H SO
3
C6H5CH2CO2CH3 → A  (JEE MAIN)
H2O ∆

Sol: First step is attack of Grignard reagent to form an alcohol, as the starting compound is an ester we end up
getting a tertiary alcohol which on treatment with Acid gives an alkene.

CH3 CH3
| |
A - C6H5– CH2 –C–CH3 B - C6H5– CH=C–CH3
|
OH

2.4 From Miscellaneous Groups

2.4.1 On Aqueous Alkali Hydrolysis of Gem Di-Halides


Terminal gem dihalides will give an aldehyde while a non-terminal will give ketones as follows

aq. KOH OH
R-CHCl2 R-CH R-CH=O
OH -H2O Aldehyde

Cl
aq. KOH
R C R’ R - C - R’ R - C - R’
-H2O
Cl HO OH O ketone

2.4.2 By Dry Distillation of Calcium Salts of Acid


O
∆ ||
(RCOO)2 Ca R–C–R+CaCO3
Ketone
O
On the dry distillation of calcium salt of acid with the R C R
calcium salt of formic acid we get a mixture of aldehyde, O A mixture
ketone and formaldehyde  of these three
(RCOO)2Ca+(HCOO)2Ca H C R
O
H C H

PLANCESS CONCEPTS
In this reaction, the yield is generally poor due to side reactions viz. formation of formaldehyde and
acetone from calciumformate and calcium acetate respectively.
Aishwarya Karnawat (JEE 2012, AIR 839)

2.4.3 On Passing Vapours of Fatty Acids Over Manganous Oxide at 300°C


O
On passing the mixture of vapours of the fatty acid with
MnO/300oC
formic acid we get a mixture of aldehyde, ketone and 2 RCOOH R C R + CO2 + H2O
formaldehyde. (Vap)
2 3 . 1 0 | Aldehydes and Ketones

O
MnO / 300oC
RCOOH + HCOOH R C R
O A mixture
Vapour H C R of these three
O
H C H


Illustration 7: Complete the following reaction: (CH3COO)2Ca  (JEE MAIN)

Sol: This method is commonly used for the preparation of a ketone.


O

||
(CH3COO)2Ca CH3–C–CH3
Calcium Acetate

Acetonr
2.5 Methods Used for the Preparation of Aldehydes Only

2.5.1 Rosenmund’s Reaction


Here, acid chlorides are reduced to aldehyde O
O
With H2 in boiling xylene using palladium as a Pd-BaSO4
R C Cl+H2 R C + HCl
Boiling Xylene
catalyst supported on barium sulphate. H

PLANCESS CONCEPTS

•• Pd catalyst is poisoned by BaSO4 to stop further reduction of aldehyde to alcohol.


•• Formaldehyde (HCHO) can’t be obtained by this method because HCOCl is unstable at room
temperature.
•• On reacting acid chloride with dialkyl cadmium, we can obtain ketone.
Saurabh Chaterjee (JEE Advanced 2013, AIR)

2.5.2 Stephen’s Reduction

2 SnCl /HCl 2 → R − CH H O
R − C ≡ N 
→ R − CH= NHHCl  = O + NH4 Cl
Aldimine hydrochloride

2.5.3 Oxo Process


The Oxo process is also called carbonylation, here the alkene with water gas at high temperature and pressure in
the pressure of cobalt carbonyl catalyst to give aldehyde.
CO +H / ∆ ,Pr essure
2
R − CH ≡ CH2 → R − CH − CH3 + R − CH2 − CH = O
[CO(CO)4 ]2
|
CH = O

2.5.4 Reimer-Teimann Reaction


By this, a phenolic aldehyde is prepared
By this method phenolic aldehyde is prepared
Chem i str y | 23.11

OH OH
CHO
CHCl3 / KOH

Phenol Salicylic aldehyde

2.6 Methods used for the Preparation of Ketones


(a) Using alkanoyl chloride and Grignard reagent
O O
|| ||
R–C–Cl+R'–MgCl → R – C–R' +MgCl2

(b) Using alkanoic anhydride and Grignard reagent


O O O
Br
R C O C R+R’ MgBr R C R’ +Mg
O C R
O
(c) Using alkanoyl chloride and dialkyl cadmium
O O O O
|| || || ||
R–C–Cl+R'2Cd → R–C–R' +R'–Cd–Cl2 2R–C–Cl+R'2Cd → 2R–C–R' + CdCl2
(Alkyl cadmium chloride) (cadmium chloride)

(d) By acylation or benzoylation of aromatic hydrocarbon (Friedel-Craft Reaction)

Dry Dry
→ C6H5COCH3 + HCl ; C6H6 + C6H5COCl 
C6H6 + CH3COCl  → C6H5COC6H5 + HCl
AlCl3 (Acetophenone) AlCl3 (Benzophenone)

(e) By acid hydrolysis followed by heating of β-Ketoester

O O O O
|| || H2O/H
+ || ∆
||
CH3–C–CH2–C–OC2H5 CH3–C–CH2–COOH CH3–C–CH3+CO2
β α β-ketoacid

β-ketoester

Note: It is β-ketoacid which decarboxylate more readily as it proceeds via six membered cyclic transition-state.
H H
O O O O OH O
CH3 C C=O CH3 C C=O -CO2
CH3 C = CH2 CH3 C CH3
CH2 CH2 Acetone
 Transition
state

2.7 Pinacol – Pinacolone Rearrangement


Pinacole is obtained when 2 moles of acetone are heated with divalent active metal magnesium followed by
treating with water
2 3 . 1 2 | Aldehydes and Ketones

O O CH3 CH3 CH3 CH3


CH3 C Mg/ H2O
+ CH3 C CH3 CH3 C C CH3 CH3 C C CH3
CH3 O O OH OH
Mg
Pinacole
Pinacole undergoes rearrangement in acidic media to give pinacolone

CH3 O
H
(CH3)2C C(CH3)2 CH3 C C CH3
-H2O
OH OH CH3
Pinacole Pinacolone

3. PHYSICAL PROPERTIES OF CARBONYL COMPOUNDS


(a) Physical state: Methanal is a pungent smelling gas. Ethanal is a volatile liquid, with a boiling point of 294 K.
Other aldehydes and ketones containing up to 11 carbon atoms are colourless liquids while higher members
are solids.
(b) Smell: With the exception of lower aldehydes which have unpleasant odours, aldehydes and ketones generally
have a pleasant smell. As the size of the molecule increases, the odour becomes less pungent and more
fragrant. In fact, many naturally occurring aldehyde and ketones have been used in the blending of perfumed
and flavourings agents.
(c) Solubility: Aldehydes and ketones up to 4 C-atoms are miscible with water. This is due to the presence of
hydrogen bonding between the polar carbonyl group and water molecules as shown below:
-
- - 
+ + + +
  O 
C O H H O=C

With the increase in the size of alkyl group, the solubility decreases. All aldehydes and ketones are, however,
soluble in organic solvents such as ether, alcohol, etc. The ketones are good solvents themselves.
(d) Boiling points: The boiling points of aldehydes and ketones are higher than those of non-polar compounds
(hydrocarbons) or weakly polar compounds (such as ethers) of comparable molecular masses. However,
their b.p.s’ are lower than those of corresponding alcohols or carboxylic acids. This is because all aldehydes
and ketones are polar compounds having sufficient intermolecular dipole-dipole interaction between the
opposite ends of C = O dipoles.
However, these dipole-dipole interactions are weaker than the intermolecular hydrogen bonding in alcohols
and carboxylic acids. Therefore, b.p.s’ of aldehydes and ketones are relatively lower than the alcohols and
carboxylic acids of comparable molecular masses.
Among the carbonyl compounds, ketones have a slightly higher boiling points than the isomeric aldehydes.
This is due to the presence of two electron releasing groups around the carbonyl carbon, which makes them
more polar.
CH3 CH3
:

C=O
:
C=O
:
H CH3
Acetadehyde Acetone
( = 2.52 D, bpt = 322 K) ( = 2.88 D, bpt = 329 K)

(e) Density: Density of aldehydes and ketones is less than that of water
Chem i str y | 23.13

4. CHEMICAL REACTIONS OF CARBONYL COMPOUNDS


4.1 Nucleophilic Addition Reactions
Addition of a nucleophile and a proton across the (C = O) double bond. The reactivity of the carbonyl group arises
from the electronegativity of the oxygen atom and the resulting polarization of the carbon-oxygen double bond.
The electrophilic carbonyl carbon atom is sp2 hybridized and flat, leaving it relatively unhindered and open to
attack from either face of the double bond.
H R R
C=O> C=O> C=O>
H H R

Illustration 8: Why are aldehydes more reactive than ketones? (JEE ADVANCED)

Sol: The factors which influence the reactivity of ketone and aldehyde are
(i) Inductive effect (ii) Steric factor
(i) As alkyl group is electron releasing,+ I effect of alkyl group decreases the amount of charge on C+ (C+ – O –).
in ketones.
(ii) Steric effect (crowding of bulky group) also causes the less reactivity of carbonyl group of ketone.

4.1.1 Reaction with Alcohols


Carbonyl compounds react with alcohols in the presence of dry HCl gas to give acetal (if aldehyde) and ketal if
ketone via formation of unstable hemiacetal and hemiketal respectively.
R R R
H R’OH
C = O + R’OH C OH C OR’ + H2O
HCl
H H H
OR’ OR’
Hemiacetal acetal

PLANCESS CONCEPTS

•• Acetal is formed to protect aldehyde (as a functional group) and ketal to protect ketone for a long
time. Acetal has a functional group ‘Ether’.
•• Acetal can be decomposed to original Aldehyde by dilute acids.
•• On treating with ethylene glycol we get cyclic acetal or ketal (1, 3-dioxolane)
CH2 OH  O CH2
C=O+ C
CH2 OH Para toluene sulphonic acid (PTS) O CH2
Mechanism
HO CH2  O CH2
C=O+ C
HO CH2 -H2O O CH2

•• Acetal formation is found to be more favourable than ketal formation if both the carbonyl groups are
present within the molecule.
Mredul Sharda (JEE Advanced 2013, AIR)
2 3 . 1 4 | Aldehydes and Ketones

4.1.2 Addition of HCN


It Is a Base Catalyzed Addition

:
B
C = O + HCN C OH
CN
Cyanohydrin
:

B
H C N BH + CN

BH
C = O + CN C O : C OH
-B
CN CN

PLANCESS CONCEPTS

•• Addition of HCN over aldehyde gives cyanohydrin and cyanohydrin on acid hydrolysis gives α-hydroxy
acid.
•• Cyanohydrin on treating with NH3 () followed by acid hydrolysis gives α-amino acid
•• In case of ketone cyanohydrin formation is reversible due to bulky group of ketone which hinders the
formation.
Vaibhav Krishnan (JEE 2009, AIR 22)

4.1.3 Addition of Sodium Bisulphite (NaHSO3)


This addition is used to isolate carbonyl compounds from the mixture as we get a salt.

C = O + NaHSO3 C ONa C OH
SO3H SO3Na
(salt)
The salt on acidification gives carbonyl compounds again
H2O
C OH C OH C=O
-H2O
SO3Na OH
4.1.4 Addition of Water
Aldehydes or ketones react with water to form gem-diols. Water is a poor nucleophile and therefore adds relatively
slowly to the carbonyl group, but the rate of reaction can be increased by an acid catalyst. Reaction is reversible so
removal of water gives back the corresponding carbonyl compound.
Chem i str y | 23.15

Mechanism:

O O H OH
+H
R C H+ H R C H R C H
-H

: :
O
H H

OH OH
+H
R C H R C H
-H OH
O
H H

4.2 Addition Elimination Reactions


Certain compounds related to ammonia add to the carbonyl group to form derivatives that are important chiefly
for the characterization and identification of aldehydes and ketones, the product contains a carbon nitrogen double
bond resulting from the elimination of a molecule of water from the initial addition products.

+
+ : NH2OH H C NHO C=NOH + H2O
C
oxime
O OH

+ : NH2NHC6H5 H C NHNHC6H5 C=NNHC6H5 + H2O


C
Phenyl hydrazone
O OH

+
H
+ : NH2NHCONH2 C NHNHCONH2 C=NOH + H2O
C
OH Semi carbazone
O
Reaction with ammonia derivatives (H2N–Z)
This reaction is a nucleophilic addition followed by water elimination.

C= O+H2 N-Z C=N-Z


-H2O

C O-
:

C OH 
C=O + H2N-Z C=N-Z
-H2O
H N H N H
Z Z
This reaction is carried out in slightly acidic media which will generate a nucleophilic centre for weak base ammonia
derivatives. On using strong acidic media lone pair of electrons present at N-atom of ammonia derivatives will
accept a proton forming protonated ammonia derivatives which cannot act as a nucleophile for a carbonyl carbon.
> CO + H2N − Z → > =
C N− Z
2 3 . 1 6 | Aldehydes and Ketones


Z
–OH Hydroxylamine Oxime
–NH Hydrazine Hydrazone
–NH–C6H5 Phenylhydrazine Phenylhydrazone

–NH– –NO2 2, 4-dinitrophenylhydrazine 2, 4-dinitrophenylhydrazone


NO2 (Brandy's reagent) or 2, 4-DNP (Solid orange precipitate)

O
||
–NH–C–NH2 Semicarbazide Semicabazone

4.3 Beckmann’s Rearrangement in Oxime


O O
R' + || ||
H or
C=N–OH R–C–NHR + R'–C–NHR (If R’ is bulkier than R)
R dry HCl
(E+Z)
Mechanism:
H H
O OH O
R’ R’ Tautomerism
:

H H2O
: -H
: :

C = N OH C = N OH2 R C=N R’ R C=N R’ R C=NR’ R C NHR’


R R -H2O

PLANCESS CONCEPTS

•• Brady’s reagent is used to distinguish carbonyl compounds from the mixture


•• Oxime undergoes Beckmann rearrangement to give its isomer amide.
•• In this reaction, the group which is anti –OH group migrates.
C6H5 O O
Rearrangement
C = N OH CH3 C NH C6H5 + C6H5 C NH CH3
CH3

C6H5 OH O
Rearrangement
C=N C6H5 C NH CH3
CH3

(-CH3 is anti to -OH)

C6H5 O
Rearrangement
C=N CH3 C NH C6H5
CH3 OH

(-C6H5 is anti to -OH)

Nikhil Khandelwal (JEE 2009, AIR 94)


Chem i str y | 23.17

4.4 Aldol Condensation


It is condensation between two moles of carbonyl compounds among which at least one must have α-hydrogen
atom in dilute basic media to get α, β-unsaturated aldehyde / ketone via the formation of β-hydroxy aldehyde /
ketone.
β α β α
Base ∆
2CH − CH =O  → CH3 − C H − CH − CH =O → CH − CH = CH − CH =O
3 | 2 −H2O 3
α , β − Unsaturatedaldehyde
OH

Mechanism:
:

B
CH3 CH = O CH2 CH = O + BH

BH
CH3 CH = O + CH2 CH = O CH3 CH CH2 CH = O CH3 CH CH2 CH = O

:
Nucleophile -B
O OH
-hydroxy aldehyde

From β-hydroxy aldehyde/ ketone, water is eliminated on using either acidic or basic media.-
:

B (basic media)
CH3 CH CH2 CH = O CH3 CH CH CH = O
: OH -BH
OH
:

(Acidic
H -OH
media)
CH3 CH CH2 CH = O CH3 CH CH CH = O CH3 CH CH CH = O
-H2O
OH2 H
-H
Now try to get carbonyl compounds from α,β-unsaturated carbonyl compounds as - keep ‘H’ at α-position and
–OH at β-position of α, β- unsaturated carbonyl compound to get β-hydroxy carbonyl compound.
O O
 
C=C C R C C C R
 
OH H
,-unsaturated -hydroxy carbonyl
carbonyl compound compound

Now break α and β carbon shown below to get carbonyl compound.


O O
  Base
C C C R C = O + H2C C R
O HH

These two carbonyl compounds can be obtained on the ozonolysis of hydrocarbon

C=C=CH2
R

4.4.1 Cross Aldol Condensation

On using two type of carbonyl compounds both having α-hydrogen atoms we get a mixture of four condensed
products because two types of carbonyl compounds will give two type of carbanions which will be nucleophile for
2 3 . 1 8 | Aldehydes and Ketones

itself and the other molecule.


On using formaldehyde and acetaldehyde during the crossed aldol, all the α-hydrogen atom of acetaldehyde
are replaced one by one by the hydroxymethyl group because of the smaller size of formaldehyde to give
trihydroxymethylacetaldehyde which undergoes crossed Cannizaro’s reaction with formaldehyde to give
tetrahydroxymethyl methane and formate ion as a final product

OH CH2=O/OH
CH2 = O + CH3 CH = O CH2 CH2 CH = O (CH2OH)2CH CH = O

OH
H CH2=O/OH

H C=O/OH
(CH2OH)4 C + HCOO- Na
+
(CH2OH)3 C CH=O

Illustration 9: Show how cinnamaldehyde is prepared by crossed aldol condensation? (JEE MAIN)

OH −
Sol: C6H5CHO + CH3CHO 
( − H O)
→ C6H5CH
= CH − CHO + H2O
2
Cinnamaldehyde

4.4.2 Intramolecular Aldol Condensation


If two carbonyl groups with α-hydrogen atoms are present within the same molecule, then we get cyclic α,
β-unsaturated aldehyde / ketones via the formation of cyclic β-hydroxy aldehyde/ketone in presence of basic media

O O
O O O O
:

B 
H C 
-H2O
-BH HO
CH3 
Cyclic -hydroxy-
ketone

By knowing the product we can get the reactant as in case of intermolecular aldol condensation: Aldol condensation
also takes place in acidic media too -

H
:

CH3 CH=O CH3 CH=O H


:
(i)
: :

H CH2 CH=O H CH2=CH OH


(ii)

-H
: :

CH2= CH OH CH3 CH=O H CH3 CH CH2 CH=O H CH3 CH CH2 CH=O


OH OH H

CH3 CH=CH CH=O CH3 CH CH CH=O CH3 CH CH CH=O


  -H2O
,-unsaturated aldehyde -H H OH2 H

4.5 Cannizaro Reaction


Carbonyl compounds without α-hydrogen atoms undergo disproportionation or redox reactions in strong basic
Chem i str y | 23.19

media.
The reactions are intermolecular Cannizzaro reactions
H O
NaOH
(i)
(i) 2 H C=O H C ONa + CH3OH

NaOH
(ii)
(ii) 2 C6H5CH = O C6H5COONa + C6H5CH2OH
Sodiumbenzoate Benzylalcohol

CH=O NaOH COONa CH2OH


(iii)
(iii) 2 +

Cl Cl Cl


Mechanism: By this mechanism, it is clear that acid is corresponding to that carbonyl compound over which OH
is going easily as a nucleophile

H C =O
H H
H
OH
C = O + CH3 O-
:

H C =O H C O H
Hydride ion Transfer
(A) O H OH
(I)
H C = O + CH3OH
O-
Note: It is observed that a hydride ion transfer from (I) to Carbonyl compound (B) is a rate determining step.

4.5.1 Crossed Cannizaro Reaction


On using two types of carbonyl compounds without α-hydrogen atom, the acid will be corresponding to that

aldehyde over which OH will approach without any hindrance.

H O
OH
(i) H C = O + C6H5 CH=O H C + C6H5CH2OH
(A) (B) O-
H O
OH
(ii) (CH2OH)3C CH=O + H C=O (CH2OH)3C CH2 OH + H C
(B)
(A) O-

In case (i) , OH will easily go to (A) and in case (ii) it will go to (B) hence acid will be formate ion in both the cases.

4.5.2 Intramolecular Cannizaro Reaction


Here two carbonyl groups (without α-hydrogen atom) are present within the same molecule.

OH
=CH O  → CH2OH
| |
CH = O COO −
Glycolate ion
2 3 . 2 0 | Aldehydes and Ketones

Mechanism:
H
H C=O OH H C = O H shift H C=O
H C=O H C=O
C=O
OH
OH
CH2OH

COO- (Glycolate ion)

4.5.3 Benzil – Benzilic Acid Rearrangement

C6H5

C6H5 C=O C6H5 C OH


OH
C6H5 C=O COOH

(i) NaOH
(ii) acidification

Other example is

(i) OH O
C C (ii) acidification C OH
O O O
O O COOH
Furil Furilic acid
Mechanism:

OH
(i) OH
C C C C
O O O O
O O O O
Furil

O
acidification
C OH H
+ C C OH
O O
COOH O O
Furilic acid Furilic acid

4.6 Perkin Reaction


When an aromatic aldehyde like benzaldehyde is treated with an anhydride in the presence of the sodium salt of
an acid from which an anhydride is derived we get α, β-unsaturated acid e.g.
Chem i str y | 23.21

 
CH=O CH=CH COOH
(CH3CO)2/CH3COONa
Acetic anhydride Sodium acetate
(Acts as base) Cinnamic acid
Benzaldehyde

Mechanism:

:
B
CH3COOCOCH3 CH2COOCOCH3
-BH

C6H5 CH = O + CH2COOCOCH3 C6H5 CH CH2COOCOCH3


O-
BH

+
 H2O/H
C6H5CH=CH COOH C6H5CH CH3COOH C6H5 CH CH2COOCOCH3
  -H2O (-CH3COOH)
,  unsaturated acid OH OH
-hydroxy acid

PLANCESS CONCEPTS

By knowing α, β-unsaturated acid we can get an idea about the anhydride used in the Perkin reaction.
This can be done by keeping ‘H’ at α and –OH at β-carbon atom followed by breaking α, β carbon as
given below. By this we can know about acid and it will be anhydride of this only.
Saurabh Gupta (JEE 2010, AIR 443)

NaOC H in absolute C H OH
Illustration 10: Complete the reaction; C5H5CHO + CH3 − COOC2H5 
2 5 2 5 → (D)  (JEE MAIN)
heat

Sol: It is an example of Perkin reaction.


The product D is: (D) C6H5 –CH = CHCOOC2H5

4.7 Knoevenagel Reaction


It is the preparation of α, β-unsaturated acid with a carbonyl compound using a malonic ester in the presence of
a pyridine base
 
CH=O CH=CH COOH
CH3(COOC2H5)

Pyridine
Benzaldehyde Cinnamic acid

Mechanism:
:

B
CH2(COOC2H5)2 CH(COOC2H5)2 + BH
Nucleophile

C6H5 CH=O CH(COOC2H5)2 BH


C6H5 CH CH(COOC2H5)2 C6H5 CH CH(COOC2H5)2
:

-B
O- OH

H2O/H-
Nucleophile

C H CH=O CH(COOC BH
2 63 . 25 2 | Aldehydes and2HKetones
5)2 C6H5 CH CH(COOC2H5)2 C6H5 CH CH(COOC2H5)2

:
-B
O- OH

H2O/H-

COOH
   
C6H5 CH=CH COOH C6H5 CH=CH2 COOH C6H5 CH CH
-H2O CO2
OH OH COOH

4.8 Reformatsky Reaction


When a carbonyl compound and a halogenated ester are heated with zinc followed by treating it with water we
get β-hydroxy ester.
(i) Zn/
C = O + Br CH COOC2H5 C CH COOC2H5
(ii) H2O

R OH R
-halogenated ester -halogenated ester

This reaction can be represented as -


Zn/
Br CH COOC2H5 Br Zn CH COOC2H5

R R

R
H2O
C = O + Br Zn CH COOC2H5 C CH COOC2H5 C CH COOC2H5

R OZnBr OH R
-hydroxyester

4.9 Wittig Reaction


The Wittig Reaction is used in order to get an alkene from a carbonyl compound using phosphourus ylide via the
formation of cyclic structure betaine.

C=O C P(Ph)3 C O C=C + O = P(Ph)3
Alkene
C P(Ph)3
Betaine

PLANCESS CONCEPTS

Phosphourus ylides are prepared from alkylhalide and triphenylphosphine in the presence of base like
sodium ethoxide as -

Base
:

R CH2 Br + (C6H5)3 P R CH2 P (C6H5)3 R CH P(Ph)3 or R CH=P(Ph)3


-Br -BH Phosphorous ylide

Neeraj Toshniwal (JEE 2009, AIR 21)


Chem i str y | 23.23

Illustration 11: Complete the following reaction:  (JEE MAIN)



+ Ph3P = CH2

O
Sol: The above reaction is the Wittig reaction.

+ Ph3P = CH2 CH2

4.10 Benzoin Condensation


During this reaction, benzoin is obtained when an ethanolic solution of benzaldehyde is heated with a strong alkali
like potassium cyanide or sodium cyanide.
Θ
2C6H5–CH=O CN C6H5–CH–C–C6H5
| ||
OH O
(Benzoin)

Reaction Mechanism:

H
CN
C6H5 CH=O C6H5 C O- C6H5 C OH
CN CN
Carbanion (Nucleophile)

CN CN
C6H5 CH=O C C6H5 C6H5 CH C C6H5 C6H5 CH C C6H5
OH O- O H OH O

4.11 Baeyer-Villiger Oxidation


It is preparation of an ester from a ketone using peracid-
R R’’COOOH
C=O R C OR’ + R” COOH
peracid
R’
O
Mechanism:
O
R H R’’ C O18 O
: :

C=O R C = OH R C OH
:
R’
R’ R’

O
O O18 O O C R’’
18
R’ C OR+ R’’ C O + H R C O H
R’
2 3 . 2 4 | Aldehydes and Ketones

4.12 Haloform Reaction


Acetaldehyde and methylalkyl ketones react rapidly with halogens (Cl2, Br2, or I2) in the presence of an alkali to give
a haloform and acid salt.

O O
Br2/NaOH
R C CH3 R C ONa + CHBr3 (Bromoform)

In this reaction –CH3 of CH3 –C– group is converted into a haloform, as it contains an acidic hydrogen atom and the
rest of alkyl methyl ketone gives an acid salt having a carbon atom corresponding to alkyl ketone.
Preparation of haloform from methylketone involves two steps
O O
Br2
(a) Halogenetion R C CH3 R C CBr3 (Halogenation)

O O
(b) Alkali hydrolysis R C CBr3
NaOH
CHBr3 + R C ONA (Alkaline hydrolysis)

4.13 Clemmensen Reduction


Zn-Hg/HCl
Used to get alkane from carbonyl compounds. C=O CH2
3 3
sp sp
Mechanism:
++
H C=O H C OH Zn Zn + 2e 2H
C=O C OH
3
sp
2+
H CH Zn  Zn + 2e CH CH OH2
CH2 -H2O
3
sp

4.14 Wolf Kishner Reduction


Used to get alkane from carbonyl compounds

NH2 NH2 / KOH


C=O CH2
Using high boiling
3 3
sp solvent (ethylene glycol) sp

Mechanism:
:

-H2O B
C = O + H2 N NH2 C=N NH2 C=N NH C N=
2
-BH (A)
sp
:

BH B BH
CH2 CH CH N = N CH N = NH
-N2
:
:

-B -BH -B
2
sp
Chem i str y | 23.25

4.15 Addition of Grignard Reagent Over Carbonyl Compounds

It gives alcohol
H2O/H
C=O+R MgBr C OMgBr C OH + MgBr OH

R R
(i) When formaldehyde is treated with Grignard reagent
H H H
| | H O / H⊕ |
2
H − C= O + R − MgBr → H − C− OMgBr  → R '− C
| |
R R
1° alcohol
Followed by acid hydrolysis, a primary alcohol is obtained
(ii) When an aldehyde except formaldehyde is treated with Grignard reagent followed by hydrolysis, we get 2°
alcohol
H H H
H2O/H
Br
R’ C=O + R MgBr R’ C OMgBr R’ C OH + Mg
OH
R R
o
2 alcohol

This 2° alcohol is also obtained as -


H H
| (1) R'MgBr |
R'–C=O (2) H2O/H+
R–C– OH
|
R'
2° alcohol
(iii) When a ketone is treated with Grignard reagent followed by acid hydrolysis, it gives 3° alcohol

R'' R''
| |
R–C=O
(1) R'MgBr
R'–C– OH
(2) H2O/H+
|
R
3° alcohol

This 3° alcohol is also obtained by using the following two methods

R R'' R R
| (1) R'' MgBr | | (1) R'' MgBr |
R'–C=O R'–C– OH R''–C=O R''–C– OH
(2) H2O/H⊕
| (2) H2O/H⊕ |
R R'
3° alcohol 3° alcohol

Note: 2° alcohol on oxidation gives a ketone


2 3 . 2 6 | Aldehydes and Ketones

4.16 Reduction of Carbonyl Compounds


(a) Reduction to alcohols
R R OH
H2/Ni or Pt or Pd
C=O C
H LiAlH4 or NaBH4 H H

O O OH
H
C C C
Ni-H2 Ni-H2
H H H
e.g. H (Raney nickel) H H

(90%)
O OH
|| |
(1) NaBH4
CH3CH2CH2CCH3 CH3CH2CH2CHCH3
(2) HΘ, H2O

(b) Reduction to pinacols


CH3 CH3 CH3 CH3
| | (i) Mg | |
CH3 – C + C – CH3 CH3–C–––C–CH3
|| || (ii) H2O | |
O O OH OH

C6H5 C6H5 C6H5 CH3


| | (i) Mg | |
e.g. C6H5–C + C– CH3 C6H5–C––––C–CH3
|| || (ii) H2O | |
O O OH OH

4.17 Reaction with PCl5


Cl
Carbonyl compounds give geminal dihalides C = O + PCl5 C + POCl3
Cl
(a) CH3CH=O+PCl5 → CH3–CHCl2+POCl3

CH3 CH3
| |
(b) CH3–C=O + PCl5 CH3–C–Cl+POCl3
|
Cl

4.18 Other Reactions


(a) NH3
Hexamethylene tetramine
(Urotropine)

Evaporated
CH2 = O -CH2OCH2OC2O-
to dryness (Paraformaldehyde)
(Formaldehyde)
Kept at room O
temp for a longtime CH2
CH2

O O

CH2
Trioxane
(Metaformaldehyde)
Chem i str y | 23.27

(b) NH2
NH3
CH3 CH OH (Acetaldehyde ammonia)

Conc.H2SO4
CH3 CH = O Cyclic trimer (Paradehyde)
(few drops)
(Acetaldehyde)
Dry HCl
Cyclic tetramer (Metaldehyde)

(c) NH2 O
NH2
CH3 C CH2 C CH3 (Diacetone amine)

CH3 CH3
H2SO4 / 
CH3 C=O Mesitylene
(few drops)
(Acetone) CH3 O
CH3 O
Dry HCl
CH3 C=CH C CH=C CH3 + CH3 C=CH C CH3
(Phorone)
CH3
(Mesityloxide)

(d) O
O C CH3
OH/H2O (CH3CO2)O
C6H5 CH O C6H5 CH O C CH3 C6H5 CH3
(-CH3COOH) CrO2
(Benzadehyde or the bitter
almond) CH3 O
(i)CrO2Cl2/CCl4
NH3 (ii)H2O
(i)HCN+HCl/AlCl3
(ii)H2O
C6H5 CH=N CH N=CH C6H5 CO+HCl/AlCl3
C6H5 O
Hydrobenzamide H2/Pd-BaSO4
Boiling xylene
C6H5 C Cl

C6H5NH2 C6H5 CN =N C6H5


Schiff base

(e) Cl2
C6H5 C CH2 Cl
O O
Phenacyl chloride (Lacrymater)
C6H5 C CH3
Acetophenone CH2 CH3
CH3COCl / AlCl3 Zn Hg / HCl
2 3 . 2 8 | Aldehydes and Ketones

(f) O
C6H5 C C6H5
(Benzophenone)

C6H5COCl / AlCl3 COCl2 / AlCl3

excess

4.19 Some Important Reagents used for the Identification of Aldehydes


(a) Tollen’s reagent: It is ammoniacal silver nitrate solution, prepared by the addition of ammonium hydroxide
to AgNO3 solution. During the reaction, first Ag2O is formed which is dissolved in ammonium hydroxide to
give Tollen’s reagent
2AgNO3 + 2NH4 OH → Ag2O + NH4NO3 + H2O

Ag2O + 4NH4 OH → 2[Ag(NH3 )2 ]OH + 3H2O

Tollen’s reagent is a weak oxidizing agent. It gives Ag mirror test with an aldehyde.

= O + 2Ag(NH3 )2⊕ + 3OH− + → RCOO − + 2Ag + 2NH3 + 2H2O


R − CH

R − CH
= O + 2Ag2O → R − COOH + 2Ag(Silver)

(b) Fehling’s solution: It is an alkaline solution of cupric ion complexed with sodium potassium tartarate. Two
solutions are kept by naming Fehling solution (I) (CuSO4 solution) and Fehling solution (II) (Alkaline solution
of sodium potassium tartarate). When these two solutions are mixed we get deep blue coloured solution.

CuSO 4 + 2NaOH → Cu(OH)2 + Na2SO 4

O–CH–COONa
Cu(OH)2 + HO – CH –COONa → Cu |
| O–CH–COOK
HO–CH–COOK (Blue coloured compound)
(Roschelle salt)

Equal volume of both the solutions are heated with aldehyde to give red brown precipitate of cuprous oxide
(Cu2O) which confirms the presence of aldehyde.

R − CHO + 2CuO → RCOOH + Cu2O(Redppt)


blue

RCHO + 2Cu2 + +3OH− → RCOO − + 2Cu⊕ + 2H2O


(Re d ppt)


(c) Benedict solution: It is solution of CuSO4, Sodium citrate and sodium carbonate. It also consists of two
solutions. Solution (I) is alkaline solution of sodium citrate and solution (II) is CuSO4 solution.
CuSO4 + 2NaOH → Cu (OH)2 + Na2SO4
CH2COONa CH2COONa
| |
Cu(OH)2+HO–C–COONa HO–Cu–O–C–COONa
| |
CH2–COONa CH2–COONa

(Blue colour)

Aldehyde gives positive test with Benedict solution

= O + 2Cu2+ + 3OH− → RCOO − + 2Cu+ + 2H2O


RCH
(Blue) ( Re d ppt.)


Chem i str y | 23.29

(d) Schiff’s Reagent: It is a dilute solution of p-rosaniline hydrochloride. Its red colour has been discharged by
passing SO2. Aldehyde restores red colour when treated with Schiff’s reagent (Magenta solution in H2SO3).

5. USES OF ALDEHYDES AND KETONES


In the Chemical industry, aldehydes and ketones are used as solvents, and reagents for the synthesis of products.
Formaldehyde is known as formalin (40%) solution used to preserve biological specimens and to prepare Bakelite,
urea-formaldehyde glues and other polymeric products. Acetaldehyde is used primarily as a starting material in
the manufacture of acetic acid, ethyl acetate, vinyl acetate, polymers and drugs. Benzaldehyde is used in perfumery
and in dye industries. Many aldehydes and ketones, e.g. butyraldehyde, vanillin, acetophenone, camphor, etc. are
well known for their odours and flavours.

PROBLEM-SOLVING TACTICS
Problem 1: Identify the reagents to prepare the compound below using a witting reaction

Sol: Begin by focusing on the carbon atom of the double bond. One carbon atom must be a carbonyl group, while
the other must be the Wittig reagent. This gives two potential routes to us to explore.
Step 1: Using a retrosynthetic analysis, determine the two possible sets of reactants that could be used to form
the C=C bond

H H
Ph3P O
O Ph3P
+ +

(Method 1) (Method 2)

Let’s compare these two methods, by focusing on the Wittig reagent in each case. Wittig reagent is prepared by
an SN2 process, and we therefore must consider steric factors during its preparation. Method 1 requires the use of
a secondary alkyl halide.
X Ph3P
(1) PPh3
(2) BuLi
o
2 Alkyl halide

But method 2- requires the use of a primary alkyl halide


H

X (1) PPh3 Ph3P


(2) BuLi
o
1 Alkyl halide

Step 2: Consider how you would make each possible Wittig reagent, and determine which method involves the
substituted alkyl halide
2 3 . 3 0 | Aldehydes and Ketones

Method 2 is likely to be efficient, because a primary alkyl halide will undergo SN2 more rapidly than a secondary
alkyl halide. Therefore, the following would be the preferred synthesis.

Ph2
O P
Ph2
Ph2

Problem 2: Propose an efficient synthesis for the following

Sol: Always begin a synthesis problem by asking the following two questions.

Step 1: Inspect whether there is a change in the carbon skeleton and /or a change in the identity or location of the
functional group.
1. Is there any change in the carbon skeleton? Yes. The product has two additional carbon atoms.
2. Is there any change in the functional groups? No. Both the starting material and the products have a double
bond in the exact same location. If we destroy the double bond in the process of adding the two carbon atoms we
will need to make sure that we do so in such a way that we can restore the double bond. Now, let’s consider how
we might install the additional two-carbon atoms. The following C–C bond is the one that needs to be made. In
this chapter, we have seen a C–C bond-forming reaction, let’s consider each one as possible.

Step 2: When there is a change in the carbon skeleton, consider all of the C–C bond-forming reactions and all of
the C-C bonds. Breaking the reaction that you have formed so far.
We can immediately rule out cyanohydrin formation, as that process installs only one carbon atom, not two. So,
let’s consider forming the C–C bond with either a Grignard reaction or a Wittig reaction. A Gignard reagent won’t
attack a C–C double bond into a functional group that can be attacked by a Grignard reagent, such as a carbonyl
group.
HO
H O
< H
(1) EtMgBr
(2) H2O

This reaction can indeed be used to form the crucial C–C bond. To use this method of C–C bond formation, we must
first form the necessary aldehyde, then perform the Grignard reaction, and then finally restore the double bond in
its proper location. This can be accomplished with the following reagents.
Chem i str y | 23.31

(1) BH3 THF


(2) H2O2 NaOH HO
H O
H

PCC (1) EtMgBr (1) TsCl, py


(2) H2O (2) NaOEt, heat

C-C bond forming reaction

This provides us with a four-step procedure, and this answer is certainly reasonable. Let’s now explore the possibility
of proposing a synthesis with a Wittig reaction. Recall that a Wittig reaction can be used to form a C=C bond, so
we focus on the formation of this bond

This bond can be formed if we start with a ketone and use the following Wittig reagent. PPh3

To use this reaction we must first form the necessary ketone from the starting alkene.

This can be accomplished with ozonolysis. This gives a two-step procedure for accomplishing the desired
transformation: Ozonolysis followed by a Wittig reaction. This approach is different from our first answer. In this
approach, we are not attaching a two-carbon chain, but rather, we are first expelling a carbon atom and then
attaching a three-carbon chain
In short, there are two plausible methods have been discovered. Both methods are correct answers to this problem,
however the method employing the Wittig reaction is likely to be more efficient, because it requires fewer steps.
1. BH3 . THF
2. H2O2 NaOH
3. PCC
4. EtMgBr
5. H2O
6. TsCl, py
7. NaOEt, heat

1. O3
2. DMS
3. Ph2P

O
Problem 3: Propose a mechanism for the following transformation: O O
+
H3O
+
HO OH

Sol: This is a hydrolysis reaction in which a cyclic acetal is opened to form a ketone. We therefore expect the
mechanism to be the reverse of acetal formation. Begin by considering all of the intermediates in acetal formation
2 3 . 3 2 | Aldehydes and Ketones

: :
OH OH OH OH
: : :

H H H
: O O O
:

:
: :

: :
HO O HO O H O O

: :

: :
O O O O

:
: :

:
H H

:
Ketone First Hemiacetal Acetal
intermediate

Step: 1 Draw all intermediates for acetal formation in reverse order


We simply draw all of these intermediates in reverse order so that the first intermediate above (Highlighted)
becomes the last intermediate of the hydrolysis mechanism.

: :
OH

:
OH OH
: :
OH
:

H H
O O O HO O HO O O
:

:
:
: :

: :
H H O O:
: :

O O
:

: O
:
:

:
H

:
Acetal Hemiacetal First Ketone
intermediate

Hydrolysis of the acetal must involve these intermediates, in the order shown above. If any of the intermediates
has a negative charge, then a mistake has been made. With the intermediates placed in the correct order, the final
step is to draw the reagents and curved arrows that show how each intermediate is transformed into the next
intermediate. Begin with the acetal, and work forward until reaching the ketone. Make sure to use only the reagents
that are provided, and obey the master rule for proton transfer. For example this problem indicates that H3O+ is
available. This means that H3O+ should be used for protonating, and H2O should be used for deprotonating. Do
not use hydroxide ions, as they are not present in sufficient quantity under acid-catalyzed conditions, Application
of these rules gives the following answer?
:

:
OH : OH : OH
:

H
:

H O HO
:

O O: O:
::
: :

O O O O
:

:
:

:
O H O H O
:
:
:

:
:
:

H H H H H H

Acetal Hemiacetal

H
O H
H
:

H
O O HO
:
:

:O
:

O H
:
:
:

H H
HO OH +

Ketone

Step: 2 Draw the curved arrows and necessary reagents for each step of the mechanism
Notice the use of equilibrium arrows, because the process is governed by an equilibrium, as noted in the previous
section.
Chem i str y | 23.33

POINTS TO REMEMBER

Nucleophilic Addition Reactions:


R R
C = O or C=O
H R
Aldehyde Ketone

with 2 mol. of HCN with with


ROH NaHSO3 water

R OR R R R OH
C C OH C OH C
H/R OR H/R H/R H/R OH
CN SO3Na
Acetate Cyanohydrin Adduct Diol

Reagents used to identify Aldehyde and Ketone:

No. Reagent Reactant Product


1. Tollen’s Reagent: (Ammoniacal O O
solution of AgNO3) || ||
R − C −H→ R − C − O −+ Ag
⊕ Θ
 Ag (NH )  OH
Silver mirror

 3 2
α , β -Unsaturated Aldehyde.
Also known as silver mirror Test. R CH =CH − CHO → R − CH = CH − COOΘ + Ag mirror

α -hydroxy ketone
R − C H − C− R → R − C + C − R + Ag mirror
| || || ||
OH O O O

Ketone No reaction
2. Fehling’s solution: Ammoniacal O RCOO − + Cu2O
solution of CuSO 4 + Rochelle || (Red ppt)
salt. i.e. – sodium potassium R − C− H →
tartarate
α , β -Unsaturated aldehyde R − CH = CH − COOΘ + Cu2O
R − CH = CH − CHO →
α - hydroxyl ketone
R − C − C − R + Cu2O
R − C H − C−R || ||
|| ||
O O
O O
Ketone No red ppt.

3. Schiff’s reagent Containing Schiff’s reagent + Aldehyde Red colour of dye is restored.
rosaniline hydrochloride in H2O,
whose red colours is decolurised
with SO2.
2 3 . 3 4 | Aldehydes and Ketones

No. Reagent Reactant Product


4. Benedicts’s solution: (Ammoniacal O RCOO − + Cu2O
solution of CuSO4 + sodium citrate) || (Red ppt)
2− R − C− H →
[Cu(NH3)4 ]2 + (OH )2
α , β -Unsaturated aldehyde
R − CH = CH − CHO → R − CH = CH − COO − + Cu2O
(Red ppt)
α -hydroxy ketone
R − C H − C− R → R − C − C − R + Cu2O
| || || || (Red ppt)
OH O O O

Ketone No reaction

(No red ppt)


5. Haloform or Indoform or Aldehyde or ketone containing 3 O
Hypohalite oxidation α H atom (methyl ketone) ||
O CH3 − C − O − + CHI3
NaOH + x2 (yellow)
||
Or KOH x2 = I2 CH3 − C − H → O
||
Or O CH3 − C − O − + CHI3
|| (yellow)
Ca(OH)2 CH3 − C − CH3 →

Important Reaction Flow Chart:

Amide Alkene  Unsaturated


Carbonyl Compound
ion
Ba arra or

(Witting t
Witting sa
Re H

ck ng HC

en
m em l

Reagent) d
Reactions on OH
an
+

l C l Na O
n’ ent


C-P(Ph)3 o
s

d i
Al 1) D /-H
2


2)
Important
Benzoin Condensation
Benzoin Reactions of Aldehyde
CN Can
& Ketone nizz
aro
tion
c NaO reacti
i n rea O Re H on
k )
Per H 3CO 2 el fo
(C ag Re rm
n n 1) a c at
e
ev ctio H5
)2 Zn tion sky Alcohol + Salt of Acid
 Unsaturated o a C 2 ,H
Kn Re O
O ine 2O
Acid (2 C rid
CH Py
-Hydroxy
 Unsaturated
ester.
Acid
Chem i str y | 23.35

Solved Examples

JEE Main/Boards
Example 1: Identify A to E in the following reactions:

COOH

Conc.HNO₃ SOCl₂ (i)NaBH₄


A B C
+Conc. H₂SO₄  (ii)H₃O+

SOCl₂

H₂-Pd/BaSO4
D E
+ S or quinoline

Sol: The First step is nitration, the reaction with SOCl2 forms an acid chloride derivative, which on Rosenmund
reduction forms a formyl group whereas the acid chloride derivative on reduction with NaBH4 gives an alcohol.
COOH COOH COCl

Conc. HNO₃ SOCl₂


+Conc. H₂SO₄  -SO₂, -HCl
(Nitration)
NO₂ NO₂
Benzoic acid m-Nitro- m-Nitrobenzoyl
benzoic acid (A) chloride (B)

-SO₂ SOCl₂ (i) NaBH₄


-HCl
(ii) H₃O+

COOH CHO CH₂OH

H₂-PdBaSO₄
+ S or quinoline
(Rosenmund NO₂
Benzoyl reduction) Benzaldehydro (E) m-Nitrobenzoyl
chloride (D) alcohol, (C)

Example 2: Identify A to E in the following series of reactions:

273-283 K H₃O+
CH₃+CrO₃+(CH₃CO)₂O A B

Conc. NaOH
KMnO₄,KOH
Heat COONa
H₃O+
D E C+

Sol: Toluene with KMnO4/KOH undergoes oxidation to form Benzoic acid. On the other side, toluene with chromium
oxide and acetyl acetate gives benzylidene acetate which on treatment with an acid gives aldehyde which on
reaction with a strong base undergoes the Cannizzaro reaction.
2 3 . 3 6 | Aldehydes and Ketones

OCOCH₃
CH₃ CH
273-283 K OCOCH₃
+CrO₃+(CH₃CO)₂O
Toluene Benzylidene
diacetate (A)
KMrO₄,KOH, 
(Oxidation) H₃O+

COOK CHO

Pot. benzoate (D) Benzaldehyde (B)

Cone. NaOH
H₃O+
(Cannizzaro reaction)

CH₂OH COONa
COOH

Benzoic acid (E) Benzyl alcohol (C)

Example 3: Complete each synthesis by filling the missing starting materials, reagents or products (X, Y and Z).
NaOH
(a) C6H5CHO + CH3CH2CHO  →X
Y
(b) CH3CH2CH2CH2OH → CH3CH2CH2COOH
Z
(c) CH3 (CH2 )3 COOC2H5 → CH3 (CH2 )9 CHO

Sol: (a) The reaction is Claisen-Schmidt condensation.

O OH CH3 CH3
NaOH | | 3| 2
C6H5–C–H + CH3CH2CHO C6H5–CH–CH–CHO –H2O C6H5–CH=C–CHO
(Claisen Schmidt
Benzaldehyde Propanal condensation) 2-methyl-3-phenylprop-
2-en-1-al (X)

(b) Oxidation of alcohol to acid thus the reagent used must be KMnO4
(i) Alk.KMnO
4 → CH CH CH COOH
CH3CH2CH2OH  3 2 2
Butan−1−ol (ii) H+ ,H2O Butanoic acid

(i) DIBAL −H
(c) CH3 (CH2 )9 COOC2H5  → CH3 (CH2 )9 CHO
Ethyl undecanoate (ii) H2O Undecanal

Thus, Y = (i) Alk. KMnO4, (ii) H+, H2O


And Z = (i) DiBAL-H, (iii) H2O
Chem i str y | 23.37

Example 4: (a) Identity A, B and C in the following reaction


dil.H SO
dil.NaOH
2 4 → A → heat
HC ≡ CH  B  →C
HgSO 4

(b) Give reasons: (i) p-Nitrobenozoic acid has higher Ka value than benzoic acid
(ii) Acetone is highly soluble but benzophenone is not

Sol: (a) The first step is conversion of the alkyne to an unsaturated alcohol, which then tautomerises to form an
aldehyde. When this aldehyde is treated with dil NaOH and undergoes self-condensation, reaction known as aldol
which form β- hydroxyl carbonyl compound On heating it gives α,β unsaturated carbonyl compound.

2 4→ dil.H SO Tautomerises dil.NaOH


HC ≡ CH  =[H2C CHOH]  → CH3CHO →
Acetylene (Addional of H2O) Vinylalcohol Acetaldehyde(A) (Aldolcondensation)

heat
CH3 − CHOH − CH2 − CHO  → CH3 − =
CH CH − CHO
β−Hydroxybutyraldehyde)(B) Dehydration( −H2O) But −2 −en−1 −al(C)
(Analdol)

Sol: (b) (i) Higher the K a stronger is the acid.


p-nitrobenzoic acid is a stronger acid than benzoic acid. This is due to the following two reasons:
I. Due to -I-and - R-effect of the –NO2 group, the electron density in the O–H bond decreases. As a result, the O–H
bond becomes weak and hence p-nitrobenzoic acid more easily loses a proton than benzoic acid.
II. Due to -I and R-effect of the NO2 group, dispersal of the-ve charge occurs and hence
p-nitrobenzoate ion becomes more stable than the benzoate ion.
O O
O + -O +
N C O H N + C O H
-O -O

O + O -O + O O
N C - -O N C -O C -O
-O O
p-Nitrobenzoate ion (more stable) Benzoate ion (less stable)

(ii) This is because, the carbonyl group of Acetone easily forms Hydrogen bonds with water and hence acetone is
highly soluble in water.
However, in Benzophenone, the phenyl groups are larger and hence C = O group cannot form hydrogen bonds
with water due to steric crowding. Benzophenone is thus insoluble in water.

Example 5: Show how each of the following compounds could be converted to benzoic acid
(i) Ethylbenzene (ii) Acetophenone (iii) Bromobenzene (iv) Phenylethene (Styrene)

Sol: First step is oxidation with KMnO4 and KOH to for Potassium Benzoate followed by treatment. with acid.
(a) This can be achieved by treating it with KMnO4-KOH followed by treatment with acid.

CH₂CH₃ COO-K+ COOH

KMnO₄-KOH H₃O+

Ethylbenzene Pot. benzoate Benzoic acid
2 3 . 3 8 | Aldehydes and Ketones

(b) COCH₃ COO-K+ COCH

KMnO₄-KOH H₃O+

Acetophenone Pot. benzoate Benzoic acid
COOH₃ COONa COOH
I₂-NaOH
(lodoform reaction) H₃O+
Alternatively,
-CH₃
Acetophenone Sod. benzoate Benzoic acid

(c) First of all we have to prepare the Grignard reagent. It is prepared by treating Bromobenzene with Mg in dry
ether. Now this Grignard reagent is treated with dry ice followed by hydrolysis to form Benzoic acid.
O

Br MgBr C
Mg O=C=O OMgBr H₃O+
Dry ether (dry ice) -Mg(OH)Br

Bromobenzene Phenylmag O
bromide
C
OH

Benzoic acid

(d) This can be achieved by following the same process of KMnO4-KOH and acid.

CH=CH₂ COO-K+
KMnO₄-KOH H₃O+
+ HCOO-K+
 Pot. formate
Phenylethene Pot. benzoate
or styrene
COOH

+ HCOOH
Methanoic acid
Benzoic acid

Example 6: An organic compound A (molecular formula C8H16O2) was hydrolysed with dilute sulphuric acid to give
a carboxylic acid (B) and an alcohol (C) Oxidation of (C) with chromic acid produced (B). (C) on dehydration gives
but-1-ene. Write equations for the reactions involved.

Sol: (i) Since the organic compound (A) on hydrolysis with dil. H2SO4 gives carboxylic acid (B) and the alcohol (C)
therefore, (A) must be an ester. Further, since the oxidation of (C) with chromic acid produces the acid (B) therefore
both the carboxylic acid (B) and the alcohol (C) must contain the same number of carbon atoms.
(ii) Since the ester (A) contains eight carbon atoms, therefore both the carboxylic acid (B) and the alcohol (C) must
contain four carbon atoms each.
(iii) (C) on dehydration gives but-1-ene, therefore, (C) must be a straight chain alcohol. i.e. butan-1-ol.
(iv) If (C) is butan-1-ol, then acid (B) which is gives on oxidation must be butanoic acid and the ester (A) must be
butyl butanoate.
Chem i str y | 23.39

O O
Dil.H₂SO₄
CH₃CH₂CH₂ C OCH₂CH₂CH₂CH₃ CH₃CH₂CH₂ C OH + CH₃CH₂CH₂CH₂OH
Hydrolysis
Butyl butanoate (A) Butanoic acid (B) Butan-1-ol (C)
(M.F. = C₈H1₆O₂)
O
CrO₃H₂SO₄ Dehydration
CH₃CH₂CH₂ C OH CH₃CH₂CH₂CH₂OH CH₃CH₂CH=CH₂
Oxidation (-H₂O)
Butanoic acid (B) Butan-1-ol (C) But-1-ene

Example 7: How will you prepare the following compounds from benzene? You may use any inorganic reagent and
any organic reagent having not more than one carbon atom.
(a) Methyl benzoate (b) m-Nitrobenzoic acid (c) p-Nitrobenzoic acid
(d) phenyl acetic acid (e) p-Nitrobenzaldehyde.

Sol:
Br MgBr COOH

Br₂/FeBr₃ Mg/dry ether (i) Dry ice


(a) (Nuclear (Grignard (ii)H₃O+
bromination) reaction)
Bromo- Phenylmag.
benzene bromide
COOCH₃
CH₃OH (excess),
Conc. H₂SO₄, 
(Esterification)

COOH COOH
Conc. HNO₃
As in (i) above + Conc. H₂SO₄, 
(b)
(Nitration)
NO₂
Benzene Benzoic acid m-Nitrobenzoic acid

CH₃ CH₃ CH₃


Conc. HNO₃
CH₃Cl/Anhyd.AlCl₃ + Conc. H₂SO₄, 
NO₂
(c) +
(F.C.alkylation) (Nitration)

Benzene Toluene o-Nitrotoluene


NO2 (solid)
p-Nitrotoluene (major product)
(solid)
(major product)

CH₃ COOH

Separate by (I) KMnO₄/KOH,

filtration (ii) Dil.H₂SO₄

NO2 NO₂
p-Nitrotoluene p-Nitrobenzoic acid
2 3 . 4 0 | Aldehydes and Ketones

CH₃ CH₂Br
NBS , hv
(d) As in (iii) above or Br₂,  and hv Alc. KCN,
(side chain, (Nucleophilic
bromination) substitution)
Benzene Toluene

CH₂CN CH₂COOH
+
H /H₂O
(Hydroliysis)

Benzyl cyanide Phenylacetic acid

CH₃ CHO
(i) CrO₂Cl₂/CS₂
As in (iii) or CrO₂/(CH₃CO)₂O
(e) above (ii) H₃O+

Benzene
NO₂ NO₂
p-Nitrotoluene p-Nitrobenzaldehyde

Example 8: An aromatic compound ‘A’ (Molecular formula C8H8O) gives positive 2, 4-DNP test. It gives a yellow
precipitate of compound ‘B’ on treatment with iodine and sodium hydroxide sol. Compound ‘A’ does not give
Tollen’s or Fehling’s test. On drastic oxidation with potassium permanganate is forms a carboxylic acid ‘C’ (Molecular
formula C7H6O2), which is also formed along with the yellow compound in the above reaction, Identify A, B and C
and all the reaction involved.

Sol: (i) From the given data we can see that aromatic compound ‘A’ (MFC8H8O) gives positive 2, 4-DNP test, thus it
must be an aldehyde or a ketone.

(ii) As compound ‘A’ does not give Tollen’s test or Fehling’s test, thus ‘A’ must be a ketone.
(iii) Compound ‘A’ on treatment with I2/NaOH, gives yellow precipitate of compound ‘B’ therefore, compound ‘B’
must be iodoform and the ketone A’ must be a methyl Ketone (Haloform reaction)
(iv) Since methyl ketone ‘A’ on drastic oxidation with KMnO4 gives a carboxylic acid ‘C’ (MFC7H6O2), therefore, ‘C’
must be benzoic acid and compound ‘A’ must be acetophonenone (C6H5COCH3).

Example 9: When liquid ‘A’ is treated a freshly prepared ammonical silver nitrate sol, it gives bright silver mirror.
The liquid forms a white crystalline solid on treatment with sodium hydrogensulphite. Liquid ‘B’ also forms a white
crystalline solid with sodium hydrogensulphite but it does not give test with ammoniacal silver nitrate. Which of
the two liquids is the aldehyde? Write the chemical equations of these reactions as well.
Chem i str y | 23.41

Sol: (i) As we can see, liquid ‘A’ forms a white crystalline solid on treatment with NaHSO3, thus it may be an aldehyde
or a methyl ketone. And also liquid ‘A’ forms a bright silver mirror on treatment with a freshly prepared ammoniacal
Sol of AgNO3. Therefore, liquid ‘A’ is an aldehyde. (Aldehyde gives positive silver mirror test)
(ii) On the other side, liquid ‘B’ forms a white crystalline solid on treatment with NaHSO3, it may be an aldehyde or
a methyl ketone. But liquid ‘B’ shows negative test with ammoniacal AgNO3 solution therefore, liquid ‘B’ must be
a methyl ketone.
Chemical equations for the reactions discussed are
R R SO₃Na
C=O + NaHSO₃ C
H H OH
Liquid ‘A’ White NaHSO₃ adduct

R R SO₃Na
C=O + NaHSO₃ C
CH₃ CH₃ OH
Liquid ‘B’ White NaHSO₃ adduct

+ - warm
RCOO- + 2 Ag


RCHO + 2 [Ag(NH₃)₂] + 3 OH + 4 NH₃ + 2 H₂O
Liquid ‘A’ Tollens reagent Carboxylate ion Silver
mirror

Tollens reagent
RCOCH₃ No silver mirror
Liquid ‘B’

Example 10: Give reasons for the following:


(a) Oximes are more acidic than hydroxylamines
(b) Iodoform is obtained by the reaction of acetone with hypoiodite but not with the iodide ion.
(c) Oxidation of toluene to benzaldehyde with CrO3 is carried out in presence of acetic anhydride and not in
presence of H2SO4.

Sol: (a) In case of oxime, on proton loss it gives a conjugate base which is stabilized by resonance but the conjugate
base of NH2OH is not.

C=N O H -H
+
C=N O-
-
C N O
Oxime Conjugate base (Stabilized by resonance)

+
-H
NH₂ O H NH₂ O (Not Stabilized by resonance)

Hydroxylamine Conjugate base

(b) During the course of reaction, I+ is required which is supplied by IO– ion but not by I– ion thus iodide ion is not
used. The reaction is as shown below:

CH₃ CO CH₂ H + O I CH₃COCH₂ + H O I


Acetone

+ 
CH₃COCH₂ + I OH CH₃COCH₂I+OH
+
(Source of I )

+IO +IO OH
CH₃COCH₂I CH₃COCHI₂ CH₃COCl₃ CH₃l + CH₃COO
OH OH
1, 1, 1, -Tri-iodoacetone Iodoform Acetate ion
2 3 . 4 2 | Aldehydes and Ketones

(c) This is because CrO3/H2SO4 is a powerful oxidizing agent. During the oxidation of toluene with CrO3/H2SO4 the
intermediate benzaldehyde formed readily undergoes oxidation to form benzoic acid due to the presence of H2O
in H2SO4.
CrO₃/H₂SO₄ H₂O CrO₃/H₂SO₄
C₆H₅ CH₃ C₆H₅ CHO C₆H₅CH(OH)₂ C₆H₅COOH

Toluene Benzaldehyde Benzaldehyde hydrate Benzoic acid

However, with CrO3 in (CH3CO)2 O, due to the absence of H2O as soon as benzaldehyde is formed, it reacts with
acetic anhydride to form benzylidene diacetate which does not undergo further oxidation. In this way, oxidation of
benzaldehyde to benzoic acid is prevented. The gem-diacetate this formed upon subsequent hydrolysis with alkali
or acid gives benzaldehyde.
CrO₃-(CH₃CO)₂O H₃O+
C₆H₅CH₃ C₆H₅CH(OCOCH₃)₂ C₆H₅CHO
273-283 K (Hydrolysis)
Toluene Benzylidene diacetate Benzaldehyde
(gem-diacetate)

JEE Advanced/Boards
Example 1: Complete the following reaction by identifying (A), (B) and (C)
Pd/BaSO
(a) A + H2 (g) 
4 →(CH ) CH − CHO
3 2

CH3
|
(b) CH3– C— C– CH3 +NaOl → B + C
| ||
CH3 O

Sol: (a) This is a well-known Rosenmund reduction reaction. The catalyst used is palladium poisoned with BaSO4
supported on carbon.
The reagent is specific and acts only upon acid chloride group. Since acid chlorides on Rosenmund reduction give
aldehydes, therefore, compound (A) must be an acid chloride, (CH3)2CH–COCl

CH₃ CH₃
Pd/BaSO₄
CH₃ CH COCl + H₂ (g) (Rosenmund reaction) CH₃ CH CHO + HCl
2-Methylpropanoyl 2-Methylpropanal
chloride (A)

(b) This is an example of an Iodoform reaction. A Ketone containing methyl group undergoes this reaction. This method
is used to identify methyl ketone specifically. Yellow precipitate of Iodoform confirms the presence of a methyl group.
Methyl ketone on oxidation with NaOI (I2 + NaOH) gives iodoform and the sodium salt of the acid. Thus,
CH₃ CH₃
NaOl
CH₃ C C CH₃ CH₃ C COONa + CHI₃
(Iodoform reaction)
CH₃ O CH₃
3,3-Dimethylbutan-2-one Sod. 2, 2-dimethylpropanoate (B) Iodoform (C)

Example 2: Suggest a sequence of reactions to carry out the following transformation, but one intermediate must
be an alkene.
N(CH₃)₂
O
OH from
Chem i str y | 23.43

Sol: Here we can apply new approach for the following transformation. i.e Retrosynthesis. This is achieved by
transforming a target molecule into simpler precursor structures without assumptions regarding starting materials. 
There are many alkene reactions, so this is really not a significant restriction. The starting compound ketone
suggests that a Wittig route might be possible
Retro synthesis:
1. First locate the Bond which has been broken to form the end product.
2. Presence of two alkyl group and OH outside the Cyclohexane shows that it must be an Epoxy group.
3. Nucleophilic addition of (CH3)2NH open ups the ring.
4. Now we all know Epoxide is formed from a double bond so there must be a double bond outside the cyclohexane
ring. (We have satisfied the demand of the question that transformation should contains one alkene intermediate.)
5. As our starting material is a carbonyl group we can easily convert it into an alkene by a well-known wittig
reaction.
6. The Retrosynthetic approach for the above transformation can be written in form of reaction as follows:
Retrosynthesis:

(CH₃₃))₂₂N
(CH N
Nucleophilic addition
Nucleophilic addition
HN(CH
HN(CH₃₃))₂₂ Epoxidation
Epoxidation Witting
Witting
O
O
OH
OH O
O

Forward direction:
(CH₃₃))₂₂N
(CH N
+
+ -- CH
Ph₃P
Ph₃P CH₂
CH₂ CH₃₃CO
CO₃₃H
H (CH₃₃))₂₂NH
(CH NH
O
O CH₃OH
CH₃OH
O
O OH
OH

Example 3: Explain:

(a) α-Halocarbonyl compounds, even the 3° types like


( R
R
Cl (
CHO , do not undergo SN1 reaction.

(b) Which is more reactive: (i) PrCl or (ii) MeCOCH2Cl with Nal/acetone?

(c) Acetals and ketal are regenerated back to carbonyl, compounds with H3O⊕ but not with OH .
(d) Cyclohexanone is more reactive than cyclopentanonone with HCN.

Sol: (a) SN1 reaction involves formation of Carbocation, although it contains adjacent two alkyl group the carbocation
formed is unstable this is because of the presence of positive charges on adjacent C atoms.(Electron withdrawing
effect of chlorine makes two alkyl group slightly electron deficient)
(b) Both are Primary alkyl halide and undergo SN2 (Substitution nucleophilic bimolecular) reaction. Orde of reactivity
is (ii) >> (i).
The high reactivity of (ii) in SN2 reaction is due to the stabilizing effect produced in the transition state of ketone
by the overlap of adjacent π-bond with the p-orbital of the sp2-hybridised C atom. This type of overlap is also
responsible for the high reactivity of halides in SN2 reaction.

SN2 -
Cl I + Cl
i. Me Finkelstein reaction (F.R.) Me
-
I

O O
SN2 -
C Cl I + Cl
SN2 -
Cl I + Cl
i. Me Finkelstein reaction (F.R.) Me
2 3 . 4 4 | Aldehydes
I
- and Ketones

O O
SN2 -
C Cl I + Cl
CH₂ Me
ii. Me Finkelstein reaction (F.R.)

-
I

O
Me C l

C H
( Orbital diagram of the
intermediate transition state
of the ketone
(
H
Cl 

Me Me Me
C O  C O C O
l- CH₂ Cl l -
CH₂ Cl -
l -
CH₂ Cl -
Intermediate transition state

O
l + Cl
-
Me

(c) The formation of hemiacetal or half hydrate takes place in acid only. OH- is a weaker base than OR- It’s not
possible in base because a weaker base (OH− ) cannot displace stronger base (RO − ) . But the formation of acetal
from hemiacetal can take place by both acid and base. So the reverse process of hemiacetal to (C=O) compound
can occur only with acid but not with base.
(d) (i) Cyclopentanone suffers from ring strain. In Cyclopentanone, the hybridization of C atom of (C=O) group
changes from sp2 → sp3, and the adjacent hydrogen atoms are in eclipsed position which increases the strain due
to eclipsing interaction.
(ii) In cyclohexanone, the hybridization of C atom of (C=O) group changes from sp2 → sp3, the ring is free from strain
and after nucleophilic addition, all the adjacent H and OH atoms are in staggered position hence destabilization
effect is lost as there no eclipsing interaction.
H H H H
HCN OH
O sp²  sp³ CN
H H H H
sp² sp³
All adjacent H atoms are in eclipsed
position : more strained and high energy.

H H
H HCN
H
H H
sp²  sp³ CN
2
O H 1
H sp³
sp²
OH

( ((
Initially (C=O) is in
eclipsed position with the
adjacent equatorial H atoms
All adjacent H atoms and (OH)
are in staggered position (
(No eclipsing interaction
(

( (
High energy eclipsing (More stable)
interaction
(Less stable)
Chem i str y | 23.45

Example 4: Complete the following reaction


Me
(a) CH₃COOH  (C) (b)
H₂O 
(B) O (B) (C)
Me 700OC EtOH
(A) (D) (A) CH₃ COOH
PrOH
(D)
Ph
SO₂Cl₂
(c) O (B)
Me
(A) (C)
NBS

Sol: The formation of (B) in both (a) and (b) is industrial method for the preparation of ketone .
(a) O -
-
-
 H OH
H₂C C -H₂O
H₂C C O
Ketene
H OH (B) CH₃ C O
- -
-

(A) PrO H
Acetic acid OH
CH₃ C O (C)
Acetic acid
Opr
Propylacetate
(D)
(b) H₃C - -
-

 EtO H
C O 700 C
O CH₂ C O
H H₂C Ketene
(A) (B) CH₃ C O
Acetone - -
-

CH₃COO H OEt
O O (C)
Ethylacetate
CH₃ C O (or) CH₃ C O C CH₃
OCOCH₃
(D) (Acetic anhydride)
(Commercial method for the preparation
of acetic anhydride)

(c) Halogenation occurs at the α-position of the (C=O) group, via free radical mechanism. NBS (N-Bromosuccinimide)
is commonly used brominating agent.

Ph Ph
SO₂Cl₂
 O  O
Me Me Cl
H
H -Chlorobutan-2-one
Butan-2-one
(A) (B)
Ph
NBS  O
Me
Br
-Bromobutan-2-one
(C)

Example 5: Complete the following reactions:


(a) HO Br (i) DIBAL-H
Product (B)
(ii) H₃O -
-

(A) ll
O (i) LiAlH (O- tBu)₃
Product (C)
(ii) H₃O -
-

(i) H₂+Pd+ BaSO₄


(ii) H₃O -
-

Product (D)
2 3 . 4 6 | Aldehydes and Ketones

(b) Give the major products of the following


O
ll
1 1. LAH
2 (B)
-
l

-
2. H₃O
4 3
(C)
1. Sia₂BH
Me Me -

-
Me 2. H₃O
4-r-Butylcyclo
hexanone
(D)
H₂/Pr

Sol: (a) After looking at the question you will notice that all the three reagents, namely, DIBAL-H, LiAlH (O-t-But)3,
and (H2+Pd +BaSO4) (Rosenmund reduction ) are reducing agents.
Thus they will reduce the acid halides to aldehydes.
The products (B), (C), and (D) are same Hydroxyl aldehydes than undergo intramolecular reaction to give cyclic
hemiacetal. On reduction with all three reagents give (I) Hydroxyaldehyde anion, which undergoes ring closure. A
chiral center is formed during the ring closure reaction and as a result, two enantiomer forms are obtained and the
product is a racemic mixture. (Optically inactive)

[H] CH O -

-
H₃O
(A) All three -
reagents
O
(1) OH HO
- -

-
H H
O O
( - )or racemate
-

(b) Bulky-1-butyl group always occupies less hindered equatorial in cyclohexane.


(i) Sia2BH is a bulky group. Thus it approaches (C=O) from the less hindered equatorial side, pushing (OH) to axial
position which result in the formation of less stable cis-isomer.
(ii) Less bulky LAH approaches (C=O) from the more hindered axial side, pushing (OH) to equatorial position.
Resulting in the more stable trans-isomer.
(iii) With H2+Pt : Cis-alcohol is formed in major amount, because H2 is adsorbed on the surface of a finely divided
catalyst, and H approaches from the less hindered equatorial side and gives cis-alcohol as in (i)
Sia₂BH
H H
LAH
-
-

H₃ O
Me₃C 1 Me₃C OH
2
3 O
(A) H
LAH (B)
Sia₂BH
H - (trans-Alcohol)
-

H₃O
(trans-4-t-Butylcylohexanol)

Me₃C H (OH) and (t-Bu) gps, are in trans




trans-position (OH) in up ( ) and




(t-Bu) in down ( ) position


cis-Alcohol OH (more stable)
(C)
(cis-r--Butylcylohexanol)
Both (OH) and (t-Bu) gps, are in


cis position: both down ( ) position.


(Less stable)
Chem i str y | 23.47

Example 6: Complete the following reactions:


(a) OTs Ph₃P (excess) 2 eq. of PhLi 2 eq. of
(B) (C) MeCHO
OTs 

(D) + (E)
(b) I

Ac₂O + AlCl₃ 2 EtOH/HCl 1. Mg


(B) (C)
2. D₃O
-

-
(A) H₃O
(E) (D)
(c) Et O
-
O AlCl₃ NH₂NH₂/OH
+ (C) (D)
+H -
-

(i) EtMgl
O (G) (F) (E)
(ii) H₃O -
(A) -H₂O -
(B) O
Sc at 500 C
(Succinic anhydride)

(H)
(d) Br 1. Ph₂S NaH 1. NaOH
Br 1. Ph₂S (B) NaH (C) 1. NaOH (D)
I 2. AgBF₄ (B) (C) 2. Acetone (D)
I 2. AgBF₄ 2. Acetone
(A)
(A)
Me I
(e) Me I + Ph₂S PhLi PhCHO
Me PhLi (C) PhCHO (D) + (E)
Me + Ph₂S (C) (D) + (E)
(A) (B)
(A) (B)

Sol:
(a) - - -
-
-

PPh₃
( ( - 2 eq. of PPh₃
( -(
-

(B)  - Li
-

- PPh₃ OTs PhLi


-

2 2
- PPh₃

2MeCH=O PPh₃ O HC Me

PPh₃ O CH Me
CH Me (C)
2Ph₃P O+
CH Me
(E)
(D)
(b) I I
O O
AlCl₃
+ Me O Me F.C. acylation
Acetic anhydride at para
(A) (Ac₂O)
O Me
(B)
D D I
H₃O -
-

Me EtOH + HCl
D₂O Ketal formation
+ Me O Me F.C. acylation
Acetic anhydride at para
(A) (Ac₂O)
2 3 . 4 8 | Aldehydes and Ketones Me
O
(B)
D D I
H₃O -

-
Me EtOH + HCl
D₂O Ketal formation

O Me EtO Me EtO Me
OEt EtO
(p-Deuterio-
acetophenone) (D) (C)
or
(p-d-acetophenone)
(E)
(c) First step is Friedal craft acylation that takes place at para position followed by Wolf kishner reduction. Further
steps involved are Grignard reaction and aromatization.
Et Et
O
AlCl3
O
F.C. acylation OH
(A) at para O
O
O
(B)
(C)

Et
NH2NH2 + OH
HF Wolff-Kishner
HO
-H2O (C=O)  CH2
H O
O
Et (D) Et

EtMgl H
O Et
H2O -H2O
OH

Et Et
7
6 8
o
Sc at 500 C
5 Et Et
1
Aromatisation
4 2
3
(H) (G)
(1.7 Diethyl napthalene)

(d) It is a Wittig type reaction and proceeds via-sulphur ylide. Dihalide compound is treated with Ph2S in AgBF4
Followed by the treatment with a strong base to form Witting reagent. Now this Wittig reagent when treated with
KOH forms sulphurylide which reacts with carbonyl compound to form a Spiro compound.
Chem i str y | 23.49

Br Ph₂S Br -
- BF₄ + Agl

-
l AgBF₄ SPh₃
(A) I - is better leaving

( gp. than Br - ;
AgBF₄ - helps
in removal of I -
( (B)

- - -
-
-
-

Br
- Na NaH (H acts as base)
-

- SPh₂

- KOH
- Br
-

SPh₂
(C)
( Intermediate carbanion displaces
Br -and forms 4-membered ring. (
- --
-

SPh₂ Me₂C O - SPh₂


Me₂C O
-
(Sulphur ylide)

Ph₂S +

Me O
Me
(D)
(Spiro compound) (Spiranyl epoxide)

(e) It is a Wittig type reaction and proceeds via sulphur ylide, the product formed is a substituted epoxide ring.

-
-

Me I Me SPh₂
Me + Ph₂S Me
(A) (B)
PhLi

-
Me SPh₂
Me -
PhCH=O
(C)
-
-

SN2 Me SPh₂
-Ph₂S
Me CH Ph
-
Me O
Me Ph Me
-Ph₂S + Me
O O Ph
(E) (D)
2 3 . 5 0 | Aldehydes and Ketones

Example 7: Complete the following reactions:


O
CHO -
Me (i) OH
(a)
Give the number
+Me
(ii) H₃O -
of products

-
CHO (B)
(A)

(b)
Cl

H₃O -

-
KCN (i) NaOEt/EtOH
(B) (C)
-
(ii) PhCHO/OH
(i) SOCl₂
(A) (E) (D)
(ii) MeNH₂
(I)
(I) Moist
Moist Ag
Ag22O
(c) OHC COOH
O
(B)
(B)
OHC COOH
(ii)
(ii) Br
Br22/CCl
/CCl22D
D
(A)
(A)

COOH
COOH (i)
(i) Ba
Ba (OH)₂
(OH)₂
(d) (B)
(B)
COOH
COOH (ii)
(ii) Dry
Dry distillation
distillation
(A)
(A)

Sol: (a) In this reaction, 4 products are possible.


First of all it can undergo intermolecular Cannizzaro reactions. Other is it can undergo aldol condensation here 2
products are formed. And third is Claisen Schmidt reaction (Reaction between A and B)
(i) Intramolecular Cannizzaro reaction of (A):

CHO - COO -
(i) OH H₃O -
-

(ii) H₃O -
-

CHO CH₂OH
(not -H) O O

OH
O
OH
(I)

(ii) Aldol condensation of (B)


(1)
O
-
Me OH
Me O H₂ CH  

Me 
Me Me
Me O
(II)
Chem i str y | 23.51

(2) Me
Me -
 OH  Me
O H Me
Me  Me 

Me H Me O
O
(III)
1 Me
CH O H 5 -
(iii)
2
6 OH
H O
7
CH Seven C atom
3 O H₂ HC
4 side ring
1
2 3 Me
6
5 O
4 7
(IV)
( Numbering in accordance
with problem understanding (
(b) (i) First step is nucleophilic substitution.
(ii) Second step is hydrolysis, thus acid is formed(CN→COOH)
(iii) Third step is esterification using ethanol (COOH→COOC2H5)(protection of acid group)
(iv) Fourth step is Knoevenagel Condensation, product obtained is hydrolysized to obtain acid group.(Deprotection)
(v) Reaction with SOCl2 produces acid chloride group which on treatment with methyl amine gives the product.

CH2C N COOH

KCN
Hydrolysis in
basic medium


(B) Esterification with EtOH


O HO H O
H
Ph CH OEt  OEt
H
H₃O -
-

PhCH O
- OH
(C)
PhCH (-H acidic)
CO OH
Ph CH C SOCl₂
Ph O
(D)
C Cl H- HN Me
Ph CH C
Ph O
Ph
NH Me
H Ph
(E)
2 3 . 5 2 | Aldehydes and Ketones

(c) Moist Ag2O gives AgOH which acts as an oxidising agent and oxidises (–CHO) group to (–COOAg); the (COOH)
group also change to (–COOAg), which then undergoes Hunsdiecker reaction.

(d) First step is a simple acid base neutralization reaction forming a salt of Ba which on heating gives off BaCO3 and
bicyclic ketone as a final product.

2 O 2 2

( (
11
(A) Ba(OH)₂ 1
O 
3
1 10
1
3 Numbering in
Ba accordance with
O -BaCO₃ 7 4
4 6 7 4 9 7 naming
Seven C atom 5 6 O 6 5 O
5 O 8

( (
side ring
Numbering in (Bicyelo-[5.4.0] undecan-4-one)
accordance with (B)
problem understanding

Example 8: Complete the following reaction:

CHO

H₂/Pd HCN Conc. HI or HBr Ac₂O + AcONa


(E) (D) (B) (C)
(Vanillin) or BBr₃/H₂O
OMe
OH
(A)

Sol:

OH O
OH CH H₂ HC COO COMe
NH₂ H C N Perkin reaction
H₂/Pt HCN (A) Hl
[(C NCH₂NH₂]  
OH OH CH CH COOH
(E) OH
OH
Noradrenaline (B)
(an adrenal hormone) (D)
OH
OH (C)
Caffinic acid (found in coffee beans)
Chem i str y | 23.53

NH2OH,HCl H⊕
Example 9: (A) (C8H8O) (B) + (C)
Rearrangement

(B,C,D, E all are isomers of C8H9NO) (D) + (E)


Boiled with CH3COCl
(D) Alc.KOH
An oil (F) (C6H7N) (D)
(i) Alkali
(E) (ii) H⊕
(G) (C7H6O2) A white solid

Sol: Degree of unsaturation = 5° (aromatic and ketone), i.e. Beckmann rearrangement


Ph
NH2OH.HCl
C=O
CH3 (A)

Ph OH Ph
C=N + C=N
CH3 ( c) CH3 OH
(B)
B.R. B.R.
- -
-

H H
Ph C NH CH3 CH3 C NH Ph
O O
CH3COCl
Alkali, H -
-

Alc. KOH

PhCOOH + CH3NH2 CH3COOH + PhNH2


(G) (A white solid) (F) (An oil)

Example 10: Complete the following:


(a) 2PhCHO + NH2NH2 →
(b) Reduction of PhCHO with the following
(i) Tollens reagent (ii) Zn/HCl (iii) LAH (iv) Clemmensen’s reduction (Zn (Hg)/conc. HCl)

(v) Wolff-Kishner reduction (NH2NH2 + Glycol + OH ) vi. Two moles of PhCHO with Zn+HCl

Sol: (a) CH O + H₂ N.N H₂+O CH CH N N CH

Benzylidenazine (A)

(b) (i) with Tollen’s regent, it will form PhCOOH


(ii) Zn/HCl is a reducing agent so it will reduce the aldehyde in to primary alcohol, i.e. PhCH2OH
(iii) LAH is also a reducing agent thus it will form the same product, i.e. PhCH2OH
(iv) and (v) Wolf kishner reduction and Clemensen’s reduction-It will reduce the carbonyl group (CHO → CH2)
Thus hydrocarbon will be formed. i.e PhCH3
(vi) A pinacol will be formed.

Zn/ HCl
2 CHO + 2H CH CH

OH OH
(Pinacol) (Hydrobenzoin)
2 3 . 5 4 | Aldehydes and Ketones

JEE Main/Boards

Exercise 1 Q.8 Write the names of the reagents and equations in


the conversion of
Q.1 Explain Knovengeal Reaction with mechanism? (i) Phenol to salicylaldehyde.
(ii) Anisole to p -methoxyacetophenone.
Q.2 Which of the following compounds would undergo
aldol condensation, which the Cannizzaro reaction and Q.9 Write one chemical reaction each to exemplify the
which neither? Write the structures of the expected following
products of aldol condensation and Cannizzaro
reaction. (i) Rosenmund reduction (ii) Tollens’ reagent

(i) Methanal (ii) 2-Methylpentanal


Q.10 Explain Pinacole-Pinacolone Rearrangement?
(iii) Benzaldehyde (iv) Benzophenone
(v) Cyclohexanone (vi) 1-Phenylpropanone Q.11 Write reactions for obtaining
(vii) Phenylacetaldehyde (i) Acetone from acetic acid.
(viii) Butan-1-ol (ii) Benzene from toluene.
(ix) 2,2-Dimethylbutanal
Q.12 a) How will you obtain an aldehyde by using
Q.3 How will you convert ethanal into the following following process
compounds? (i) Dehydrogenation (ii) Catalytic hydrogenation?
(i) Butane-1, 3-diol (ii) But-2-enal b) (i) Why do aldehydes behave like polar compounds?
(iii) But-2-enoic acid (ii) Why do aldehydes have lower boiling point than
corresponding alcohols?
Q.4 Write structural formulas and names of four
possible aldol condensation products from propanal Q.13 Explain witting reaction with mechanism?
and butanal. In each case, indicate whichaldehyde acts
as nucleophile and which as electrophile.
Q.14 Convert:
(i) Acetaldehyde to Acetone
Q.5 An organic compound with the molecular formula
C9H10O forms 2, 4-DNP derivative, reduces Tollens’ (ii) Acetone to Acetylene
reagent and undergoes Cannizzaro reaction. On
vigorous oxidation, it gives 1, 2-benzenedicarboxylic Q.15 An organic compound (A) (molecular formula
acid. Identity the compound. C8H16O2) was hydrolysed with dilute sulphuric acid to
give a carboxylic acid (B) and an alcohol (C). Oxidation of
Q.6 Describe the following reactions (C) with chromic acid produced (B). (C) on dehydration
gives but-I-ene. Write equations for the reactions
(i) Cannizzaro’s reaction
involved.
(ii) Cross aldol reaction
Q.16 Give simple chemical tests to distinguish between
Q.7 Give chemical tests to distinguish between the following pairs of compounds.
(i) Acetaldehyde and Benzaldehyde (i) Propanol and Propanone
(ii) Propanone and propanol. (ii) Acetophenone and Benzophenone
(iii) Phenol and Benzoic acid
Chem i str y | 23.55

Q.17 How will you bring about the following conversions (vi) CH3—CH—C—CH3
in not more than two steps? | ||
OH O
(i) Propanone to Propene
O
(ii) Ethanol to 3-Hydroxybutanal (vii) (viii) O OMe

(iii) Benzaldehyde to Benzophenone

Q.18 Give possible explaination for each of the


following:
Q.23 Which of the amino group in semi carbazide will
(i) Cyclohexanone forms cyanohydrin in good yield but react with Ph-CH = O carbonyl group?
2, 2, 6-trimethylcyclohexanone does not. O
(ii) There are two -NH; groups in semicarbazide. ||
H2N–C–NH–NH2
However, only one is involved in the formation of (1) (2) (3)
semicarbazones.
(iii) During the preparation of esters from a carboxylic acid Q.24 How many organic products are formed in good
and an alcohol in the presence of an acid catalyst, the water amount in above reaction?
or the ester should be removed as soon as it is formed.
Q.25 How many molecules of MeMgCl will be consumed
Q.19 An organic compound contains 69.77% carbon, for per molecule of phosgene
11.63% hydrogen and rest oxygen. The molecular mass Cl C Cl
of the compound is 86. It does not reduce Tollens’
reagent but forms an addition compound with sodium
hydrogensulphite and give positive iodoform test. On
  O

vigorous oxidation it gives ethanoic and propanoic


acid. Write the possible structure of the compound.
Exercise 2
Q.20 Write the difference between aldol condensation
and cannizzaro reaction. Single Correct Choice Type

O
Q.21 A compound with molecular formula C8H18O4
does not give litmus test and does not give color C CH3
with 2, 4-DNP. It reacts with excess MeCOCl to give a
Q.1 (A) NaOH reactant (A) is
compound whose vapour density is 131. Compound A
contains how many hydroxy groups? 
III

Q.22 Which of the following compounds will give (A) O O


Fehling’s test positive?
|| ||
CH3 –C–(CH2)5–C–CH3
CHO CHO
(i) (ii) (B) O O
|| ||
CH3 –C–(CH2)4–C–H

(C) O O
(iii) OH || ||
H –C–(CH2)5–C–H
CH2OH
O
(D) O
OH ||
OH
OH CH3 –C–(CH2)4–CH2–OH

(iv) HCOOH (v) CH3–C–C–CH3


|| ||
O O
2 3 . 5 6 | Aldehydes and Ketones

Q.2 O Q.4 O O
||
CH3 –C–CH2–CH3
CH2N2 || ||
CH3–C–CH2–CH2–C–OC2H5

A(Major)
CH3CO 3H
B(Major) H O
| ||
Product B is CH3–C–CH2–CH2–C–OC2H5
|
(A) O OH
||
CH3–O–C–CH2–CH2–CH3 The conversion is carried out by using which of the
following
(B) O
(A) NaBH4 (B) LiAlH4 (C) Pd/H2 (D) Na-EtOH
||
CH3–C–O–CH2–CH2–CH3
Q.5 Which of the following compounds can undergo
(C) O
aldol condensation.
||
CH3–CH2–C–O–CH2–CH3 (A) Me3C–CHO (B) PhCHO
(D) O (C) MeCHO (D) HCHO
|| O
CH3–C–O–CH2–CH3
Q.6
Ba(OH)2
(X)
Q.3 The product Octalone is obtained by Michael 

addition followed by aldol condensation of reactants R Major product (X) is


and S in presence of a base. S gives positive iodoform
O O
test.
O
(A) (B)
B
R+S

Octalone O O

R and S respectively.
(C) (D)
(A) O

Q.7 O
+ CH3 CH=O || NH2–CH3
CH3–C–CH3 ∆
(X) Major
(B) O
H3C
Major product (X) is
+ CH2 = CH C H
NH=CH2 NH
O | ||
O (A) CH3–CH–CH3 (B) CH3–C–CH3
(C)
+ CH2 = CH C CH3
N–CH3 NH–CH3
O || |
(C) CH3–C–CH3 (D) CH3–CH–CH3
O
(D)
+ CH3= CH2 C CH3
Q.8 O
O

CH3O OCH3 CH3O OCH3


Chem i str y | 23.57

Conversion can be achieved by (C) CH2–OH (D) CHO


| |
(A) Clemmenson reduction HO–CH2–C–CH2OH HO–CH2–C–CH2–CH2–OH
| |
(B) Wolf-Kishner reduction CHO CH2–OH
(C) Pd/BaSO4
Q.12 Principal product of following reaction is isolated
(D) Sn/HCl in form of CH2=C=O+H2S →?

Q.9 BuC  CH
LiNH2 (i) PhCHO MnO2 (A) SH (B) O
(ii) H2O | ||
CH2=C CH3C–SH
|
A B C
SH
D
 (C) CH3C–OH (D) OH
|| |
O CH2=C–SH
Compound D of the above reaction is -
Bu O
Q.13 ,
(A) (B)
N2H4 KOH/
COOH (X) (Y) + N2,
OH
Bu OH
COPh Bu

(D) the structures of (X) and (Y) are-


(B)
C-Ph OH
(A) and
O
Q.10 O
NHH2
EtONa
+ HCO2 Et (X)
(B) and
Identify unknown (X) in
O O O

(A) C O Et
(B) (C) and
NH2
CHO

O O
N
CH OH (D) and
(C) (D)
CO2Et

Q.11 OH O
Q.14 O O
| || HO⊕ || || KOH
CH3–CH–CH2–C–H (Retro aldol)
CH3–C–CH2–CH2–CH2–CH2–C–CH3 ∆

Na2CO3 Possible products are-


(A), 3HCHO+A (B)
40°C 82%
O
CH3
Product (B) of above reaction is -
(A) (B)
(A) CH2–OH (B) CH2–OH
| | CH3 C CH3
HO–CH2–C–CH2OH HO–CH2–C–CHO O
| |
CH2–OH CH2–OH
O O
CH3
(C) (D)
(A) (A) (B) (B)
CH CH C CHC3 CH3
2 3 . 5 8 | Aldehydes
3 and Ketones
3
O O

O O O O (C) 2.2, 2-Trichloropropanol


CH3 CH3 (D) Chloroform
(C) (C) (D) (D)
CH3 CH3
CH3 CH3 Q.4 In Cannizzaro reaction given below

OH
Q.15 The given reaction can not be performed by the 2PhCHO → PhCH2OH + PhCO2− the slowest step
is (2009)
use of which of the following reagents?
(A) The attack of at the carboxyl group
(B) The transfer of hydride to the carboxylic group
CHO COOH
O O (C) The abstraction of proton from the carboxylic group
(A) KMnO4/H2SO4 (B) K2Cr2O7/H2SO4 (D) The deprotonation of PhCH2OH
(C) Ag2O/NaOH (D) LiAlH4
Q.5 A mixture of benzaldehyde and formaldehyde on
heating with aqueous NaOH solution gives  (2001)
Q.16 Citral can be converted into geraniol by the use of
(A) Benzyl alcohol and sodium formate
CH3 CH3
(B) Sodium benzoate and methyl alcohol
CHO CH2OH
(C) Sodium benzoate and sodium formate
(D) Benzyl alcohol and methyl alcohol
H3C CH3
Q.6 Which one of the following undergoes reaction
Which reagent with 50% sodium hydroxide solution to give the
corresponding alcohol and acid  (2007)
(A) H2/Pd–C (B) LiAlH4
(A) Butanal (B) Benzaldehyde
(C) H2/Pd-BaSO4–CaCO3 (D) NaBH4
(C) Phenol (D) Benzoic acid

Q.7 The increasing order of the rate of HCN addition to


Previous Years’ Questions
compounds A-D is  (2006)
Q.1 Which of the following on heating with aqueous (a) HCHO (b) CH3COCH3 (c) PhCOCH3 (d) PhCOPh
KOH produces acetaldehyde  (2009)
(A) a < b < c < d (B) d < b < c < a
(A) CH3COCl (B) CH3CH2Cl
(C) d < c < b < a (D) c < d < b < a
(C) CH2ClCH2Cl (D) CH3CHCl2
Q.8 The pair of compounds in which both the compounds
Q.2 Ozonolysis of an organic compound gives give positive test with Tollen’s reagent is  (2004)
formaldehyde as one of the products. This confirms the
(A) Glucose and Sucrose
presence of  (2011)
(B) Fructose and Sucrose
(A) Two ethylenic double bonds
(C) Acetophenone and Hexanal
(B) A vinyl group
(D) Glucose and fructose
(C) An isopropyl group
(D) An acetylenic triple bond Q.9 In the given transformation which the following is
the most appropriate reagent  (2012)
Q.3 Trichloroacetaldehyde was subjected Cannizzaro’s CH=CHCOCH3
reaction by using NaOH. The mixture of the products Reagent
contains sodium trichloroacetate ion and another
HO
compound. The other compound is  (2001)
CH=CHCH2CH3
(A) 2.2, 2-Trichloroethanol
(B) Trichloromethanol HO
(C)

O
Chem i str y | 23.59
O


(A) NH2NH2 OH (B) Zn–Hg/HCl CH2OH HOH2C

(C) Na, Liq, NH3 (D) NaBH4


(D)

Q.10 Compound ‘A’ (molecular formula C3H5O) is treated


with acidified potassium dichromate to form a product CH2OH HOH2C
‘B’ (molecular formula C3H6O.) ‘B’ forms a shining silver
mirror on warming with ammonical silver nitrate. ‘B’ Q.12 The product of acid hydrolysis of P and Q can be
when treated with an aqueous Sol: of H2NCONHNH2. distinguished by  (2003)
HCl and sodium acetate gives a product ‘C’. Identify the
OCOCH3 H3C
structure of ‘C  (2002)
P=H2C Q=
(A) CH3CH2CH = NNHCONH2
CH3 OCOCH3
(B) CH CH = NNHCONH
3 2
| (A) Lucas Reagent (B) 2, 4-DNP
CH3
(C) Fehling’s solution (D) NaHSO3

(C) CH3CH = NCONHNH2


| Q.13 On vigorous oxidation by permanganate solution
CH3 (CH3)2C= CH –CH2CH2CH3 gives  (2002)
(D) CH3CH2CH–NCONHNH2 OH OH
OH OH
(A) CH C CH CH2CH3
(A) CH33 C CH CH2CH3
Q.11 CHO OHC CH
CH33
(i) NaOH/100oC
CH3
(ii) H+/H2O CH3
(B) CHCO2H + CH3COOH
(B) CH CHCO2H + CH3COOH
CHO OHC CH33

Major product is.  (2003) CH3


CH3
(C) CHOH + CH3CH2CH2OH
(C) CH CHOH + CH3CH2CH2OH
COOH HOOC CH33

CH3
(A) CH3
(D) C=O + CH3CH2COOH
(D) CH C=O + CH3CH2COOH
CH33
COOH HOOC

Q.14 Silver Mirror test is given by which one of the


OH HOOC following compounds (2011)
(A) Acetaldehyde (B) Acetone
(B)
(C) Formaldehyde (D) Benzophenone
COOH OH
Q.15 In the following sequence of reactions, the alkene
O affords the compound ‘B’
O O H O
CH 3 → A 
2 → B.
= 3CH CHCH3  Zn
(C) The compound B is (2008)
(A) CH3CH2CHO (B) CH3COCH3
O
O (C) CH3CH2COCH3 (D) CH3CHO

CH2OH HOH2C

(D)
2 3 . 6 0 | Aldehydes and Ketones

Q.16 Which of the following on heating with aqueous Q.18 Which compound would give 5-keto
KOH, produces acetaldehyde? (2009) -2-methylhexanal upon ozonolysis. (2015)
(A) CH3COCl (B) CH3CH2Cl
CH3 CH3 CH3 CH3
(C) CH2ClCH2Cl (D) CH3CHCl2 CH3 H3C
(A) (B) (C) (D)

Q.17 In Cannizzaro reaction given below,


CH3 CH3 CH3

aqu.KOH
CH3CHCl2  → CH3CH(OH)2 → CH3CHO Q.19 Ozonolysis of an organic compound gives
−H O 2
formaldehyde as one of the product. This confirms the
the slowest step is:
presence of: (2011)
(−)
: OH
2PhCHO → PhCH2OH + PhCO2( − ) (2009) (A) A vinyl group

(B) An isopropyl group
(A) The attack of OH at the carboxyl group
(C) An acetylenic triple bond
(B) The transfer of hydride to the carbonyl group
(D) Two ethylenic double bonds
(C) The abstraction of proton from the carboxylic group
(D) The decomposition of CH2OH

JEE Advanced/Boards

Exercise 1 CCH
+
(i) HgSO4, H3O
Complete the following equations and identify the Q.6 ?
(ii) NH2OH
products A, B, C, D, E, F, G, H etc in the following
reactions
O
SOCl
Q.1 o − HOOC − C6H4 − CH2 − C6H5  →
Q.7 (i) MeMgBr
?
anhydrousZn − Hg
(ii) aq HCl
G  →H I OMe
AlCl3 HCl

CH3
CH3
Q.2 HBr
[E]
Perroxide Q.8 CH3 C CH2Br C2H5OH

CH3

H CrO CrO
Q.3 [J] ←
2 3 → CH CHO
4  [I] 
3 H2
H2O (Pyridine)
Q.9 CH3
Lindlar Catalyst
?

OH NaHSO
Q.4 Acetone (2mol.)  4
→[K] →[L]
F
O

Q.5 (i) Br2 (I equivalent) Q.10


NaOCH3
CH3 
(ii) NaBH4

NO2
Chem i str y | 23.61

O moist 2 5
2 4 → B  H SO C H OH
Q.22 C2H5 I → A  →C
Ag2O 140°C
Q.11 N
Br2/Fe(aq)
X excess
Q.23 CH3COOH → ClCH2COOH  →Y
NH3
O
HgSO
Q.24 [A] or [B] + H2O 
(i) dil. alkali 4→
Q.12
(ii) Heat H2SO 4
O NaOI
CH3CH2 COCH3  → [C] + [D]
Boil.alkali
Q.13 H3C − CH2 − CHCl2 → ?
[A] C6H5 [C]
Q.25 CH4 CH2Br [B]
AlCl 3(anhy.)

KOH CHCl2
Q.14 H3CO CHO+HCHO CHCl2
| +
H2O,H sod. acetate (anhyd.)
[D] [E]
H2O.H [D] sod. acetate (anhyd.) [E]
+ acetic anhydride

Q.15 Br3/CCl4
A
Na/NH2 acetic anhydride
[F]
CH=NOH
HgSO4/H2SO4
B C
CH=NOH
C
NH2NHCONH2
D
|
NaOD/D2DO (excess)
D E
Q.26 Explain giving reasons
(i) Solubility of carbonyl compounds decreases with the
Q.16 COO
HNO3/H2SO4
? increase in their molecular masses.
(mononitration)
(ii) Sodium bisulphite is used for the purification of
aldehydes and ketones.
OH
(iii) Oxidation of toluene with chromium trioxide to
(CH3CO2)O
Q.17 ? benzaldehyde is carried out in presence of acetic
CH3COONa
CHO
anhydride.
(iv) Although aldehydes are easily oxidisable yet
propanal can conveniently be prepared by the oxidation
NaOH of propanol by acidic dichromate.
Q.18 A’ B’ CH=CH.CHO
(v) O part of the O group does not react with
|| ||
−C − −C − OH
Q.19 CH3COOH + NH3 → hydroxylamine hydrochloride.
heat C
A  → B  → CH3NH2
Q.27 Give Reason

(O) Cl (i) Me3CCH2COOH is more acidic than Me3SiCH2COOH.


Q.20 CH3CH2OH  2→
→ A 
(ii) The K2 for fumaric acid is greater than for maleic
B →
dil.NaOH acid.
CHCl3
(iii) Carbon oxygen bond lengths in formic acid are 1.23
P and Br2 Å and 1.36 Å and both the carbon oxygen bonds in
Q.21 CH3CH2 COOH  →A
sodium formate have the same value ie. 1.27 Å.

(i)alc.KOH(excess)
→B (iv) The reaction CH3COOC2H5 + H2O → CH3COOH +
C2H5OH is slow in the beginning but fast subsequently)
2 3 . 6 2 | Aldehydes and Ketones

(v) Although both > C = O and > C = C < groupings (B) Me


have double bond, they exhibit different types of |
addition reaction. Me2C = CH–C–CHO
|
Q.28 An organic compound (A) C9H12O was subjected Me
to a series of tests in the laboratory. It was found that
this compound. (C) Me2CH–CH2–OH
(i) Rotates the plane of polarised light. (D) Me2CH–COOH
(ii) Evolves hydrogen with sodium.
(iii) Reacts with I2 and NaOH to produce a pale yellow Q.2 CH2 Cl
solid compound. O

(iv) Does not react with Br2/CCl4. Ph3P


(X)
Ph-Li
(Y)
H-C-H
(Z)
(v) Reacts with hot KMnO4 to form compound (B)
C7H6O2 which also be synthesised by the reaction of
benzene and carbonyl chloride followed by hydrolysis. End product (Z) in above reaction

(vi) Loss optical activity as a result of formation of CH= CH2


compound (C) on being heated with HI and P.
(vii) Reacts Lucas reagent in about 5 min. Give structure (A) (B)
of A and C with proper reasoning and draw Fischer
projections for (A). Give reactions for the steps wherever
possible. CH2 CH3

Q.29 When 0.0088 g of a compound (A) was dissolved (C) (D)


in 0.5 g of camphor, the melting point of camphor
was lowered by 8°C. Analysis of (A) gave 68.18% C
and 13.63% H. Compound (A) showed the following Q.3 If 3-hexanone is reacted with NaBH4 followed by
reactions: hydrolysis with D2O the product will be
(a) It reacted with acetyl chloride and evolved hydrogen
(A) CH3CH2CH(OD)CH2CH2CH3
with sodium.
(b) When reacted with HCl + ZnCl2, a dense oily layer (B) CH3CH2D(OH)CH2CH2CH3
separated out immediately. Compound (A) was passed (C) CH3CH2CH(OH)CH2CH2CH3
over Al2O3 at 350°C to give compound (B) which on
ozonolysis gives (C) and (D) which gave positive test (D) CH3CH2CD(OD)CH2CH2CH3
with carbonyl reagents but only (C) gave a positive test
with Fehling Sol: and resinous substance with NaOH. CHO
ldentify (A), (B), (C) and (D) with proper reasoning. Kf for Q.4 CH2
14 ZnHg
A
camphor = 40 K kg mol–1. HCl
CHO
Final major product of this reaction is
Exercise 2
14 14
(A) CH3 CH2 CH3 (B) CH3 CH2CH3
Single Correct Choice Type
14 14
NaOH (C) CH3 CH2 − CHO (D) CH3 − CH2 − CHO
Q.1 Me2CH − CHO 
→ Major products of this

reaction is -
Q.5 SH
OH Me OH
| | dry HCl
(A) Me2CH –CH–C–CHO + Me2C=O A is?
| OH
Me
Chem i str y | 23.63

SH
SH OH
H3O
SH
SH Q.8 (A) + (B)
(A)
(A) O
OH
OH (A) + (B) formed can be distinguished by -
CMe2
CMe
(A) Iodoform (B) Fehling
2

S
(C) NaHSO3 (D) 2, 4-DNP
S Me
Me
(B)
(B) NaBH CN
O Me
Me Q.9 (X) + (Y) 
2 → C6H5CH2NHCH2CH3
O methanol
OH
OH
(A) C6H5CH2OH + C2H5NH2
SH
SH
Me (B) C6H6 + CH3 + NH CH2 CH3
O
O Me
((C))
C
O
O Me (C) C6H5 CHO+CH3 + NH CH2 CH3
Me
(D) C6H5CHO+C2H5NH2
CMe
CMe22
Me
(D)
OH
OH Me Q.10 OH OH
(D) H+
OH
OH Me
NaBH4 H+ (C)
Me (A) (B)

Q.6 Compound A (molecular formula C2H8O) is treated (Major). Product C is
with acidified potassium dichromate to form a product
B (molecular formula C3H6O). B forms a shining silver (B)
mirror on warming with ammoniacal silver nitrate, B (A)
when treated with an aqueous sol of NH2NHCONH2
and sodium acetate gives a product C.ldentify the
structure of C. (C) (D)

(A) CH3CH2CH=NNHCONH2
(B) CH3C = NHHCONH2 O
| Q.11
1
CH3 2
3
(C) CH3C = NCONHNH2
O
| Me
CH3
above compound is hydrated maximum at which
(D) CH2CH2CH=NCONHNH2
position?
(A) 1 (B) 2 (C) 3 (D) Equal
Q.7 When cyclohexanone is treated with Na2CO3 sol,
we getO O
O O Q.12 Et – C – Me is prepared as one of the products by
OH ||
(A) OH (B) O
(A) (B) OH dry distillation of calcium salt of which o the following
OH acids:
OH
OH OH (A) Ethanoic acid and methonoic acid
COOH
(C)
OH
(D) COOH (B) Propanoic acid and methonoic acid
(D) COOH
(C) COOH (C) Propanoic acid and ethanoic acid
(D) None of these
2 3 . 6 4 | Aldehydes and Ketones

Q.13 Reagents A & B are Q.17 Major product of this reaction is


OH Cl
O
O
A> O MeOK
O MeOH
O OH
H
O B>
OH OMe
O COOCH3
OH
(A) (B)
(A) H2/Pd & LiAlH4 (B) LiAlH4 & NaBH4
(C) NaBH4 & LiAlH4 (D) LiAlH4 & H2/Pd H
OMe Cl OMe
Q.14 The reagent used to distinguish ethanol & O
OH
acetone is
(C) (D)
(A) Schiff’s reagent (B) Fehling’s sol.
(C) Ceric ammonium nitrate (D) Iodine with NaOH
H H

Q.15 Which one of the following compounds is the


Q.18 In the given reaction
best candidate for being prepared by an efficient mixed
aldol addition reaction? OH OH
OH O CH2 CH CH CH CH

(A) CCH2 CH O O O O
C C
CH3
H3C CH3 H3C CH3

O (i) H3O
(ii) NaIO4(aq)
(B) CCHCH3
Product will be -
CH2OH (A) 1 mole HCOOH and 1 mole HCHO
(B) 2 mole HCOOH and 1 mole HCHO
O
(C) 2 mole HCHO and 1 mole HCOOH
(C) CH2CCHCH3
(D) 2 mole HCHO and 4 mole HCOOH
HO C CH3
Q.19 In the given reaction
CH3
Br O
| || NH2–NH2/alc.KOH
O O
CH3–CH2–CH–C–CH3 X

(D) CCH2CCH3
[X] will be -
O Br
|| |
(A) CH3–CH=CH–C–CH3 (B) CH3–CH–CH2–CH2–CH3
Q.16 PhCHO and HCHO will behave differently with
which of the following reagents
(C) CH3–CH=CH–CH2–CH3 (D) CH3–CH2–CH2–CHO
(A) Tollen’s Reagent (B) Fehling’s sol.
(C) Schiffs reagent (D) NaBH4
Chem i str y | 23.65

Multiple Correct Choice Type H3O


Q.22 (A)++(B)(B)
(A) formed
formed
Q.20 Compound (X) C4H8O, which gives 2,
O
4-Dinitrophenyl hydrazine derivative (orange or red or
yellow colour) and negative haloform test. cannot be differentiated by
O (A) Iodoform (B) Fehling
||
(A) CH3–CH=CH–C–CH3 (B) CH3–CH–CHO (C) NaHSO3 (D) 2, 4-DNP
|
CH3 Comprehension Type
OH
(C) (D) CH3–CH2–CH2–CHO O
O
58%
(CH3)3C C CH3 (CH3)3C C CH2 Br
Q.21 Which of the following reaction is not representing (a)
major product - (b) 54%

OH + O
H O OH
(A) C=N 
CH3 C NH CH3
:

CH3 68%
(CH3)3C C CH2 (CH3)3C C CH2 Br

H
14 OH 14
Ph-Li CH3
C=C CC
(B) Q.23 Suggest a reagent appropriate step (a) the
Cl
synthesis -
CH3 (A) HO– / Br2 (1 mole) (B) H+ / Br2 (1 mole)
O (C) Both (A) and (B) (D) None of these
Br2
(C) Ph C NH2 Ph NH2
KOH
Q.24 Yield of each step as actually carried out in laboratory
Hoffman Brommamide
is given each arrow. What is over all yield of the reaction ?

O :NH
(A) 60% (B) 21% (C) 40% (D) 68%

HN3
(D)
H2SO4

Q.25 Match the column

Column I Column II
O

(A) (A) HCN LiAlH4 NaNO2 (p) Formation of six member ring takes place
(A) (B) (C)
traces of KOH HCl

(B) NH2OH H+ LiAlH4


(B) (A)

(B) (C) (q) Final product is Ketone

O O
OH-
(C) CH3 C CH2 CH2 CH2 C (A)
(C) 
(r) Final product will give positive Tollen’s test

Ph OH-
(D) (A)
(D) CH3
 (s) Final product formed will react with 2, 4-DNP,
OH OH (2, 4-Di-nitrophenyl hydrazine)
2 3 . 6 6 | Aldehydes and Ketones

Q.26 Match the column

Column I Column II

Me conc.H2SO4
(A) (A) C=N 
Product (p) Carbene formation is involved
Et OH

(B) (B) MCPB A LiAlH NaNO2 (q) Nitrene formation is involved


Products (A) (B)
HCl
(C)

(C) (C) CH2 = CH2 + NH3



Product (r) Carbocation formation is involved

CHCl3
(D) (D) Product (s) Final product is a cyclic compound
KOH/excess

(t) Azonium ion formation is involved

Previous Years’ Questions Q.3 What is X?  (2005)


CHO
Q.1 A mixture of benzaldehyde and formaldehyde on
+ (X)
heating with aqueous NaOH Solution gives  (2001)
MeO
COOH
(A) Benzyl alcohol and sodium formate
(D) Sodium benzoate and methyl alcohol CH3COONa
(C) Sodium benzoate and sodium formate
MeO
(D) Benzyl alcohol and methyl alcohol
(A) CH3COOH (B) BrCH2COOH
(C) (CH3CO)2O (D) HOC–COOH
Q.2 The order of reactivity of phenyl magnesium
bromide with following compounds is  (2004)
Q.4 Butan-2-one can be converted to propanoic acid
O O O by which of the following? (2006)
(A) NaOH, NaI/H+ (B) Fehling Sol:
H3C CH3 H3C H Ph Ph
(C) NaOH, I2/H+ (D) Tollen’s reagent
(I) (II) (III)

(A) II > III > I Q.5 The smallest ketone and its homologue are reacted
with NH2OH to form oxime  (2006)
(B) I > III > II
(A) Two different oximes are formed
(C) II > I > III
(B) Three different oximes are formed
(D) All the above react with the same rate
(C) Two oximes are optically active
(D) All oximes.are optically active

Q.6 Cyclohexene on ozonolysis following reaction with


zinc dust and water gives compound. Compound E on
further treatment with aqueous KOH yields compound
F. Compound F is - (2007)
Chem i str y | 23.67

Q.10 The structure of the product Q and R, respectively,


CHO
(A) (B) CHO are
O

H
(C) CHO (D) (A) CH3 COCH3
COOH
H3C CH3 H3C CH3
Q.7 Among the following compounds, which will react
with acetone to give a product containing C=N– ? O
 (2014)
H
(A) C6H5NH2 (B) (CH3)3N (B) CHO
(C) C6H5NHC6H5 (D)C6H5NHNH2 H3C CH3 H3C CH3

Q.8 A new carbon bond formation is possible is  (1998) O


(A) Cannizzaro’s reaction
H
(B) Friedel-Craft’s reaction (C) CHO

(C) Clemmensen’s reduction Me Et Me Et


(D) Reimer-Tiemann reaction
O
Me
Paragraph 1
CH3
A corbonyl compound P, which gives positive iodoform
(D) CHO
test, undergoes reaction with MeMgBr followed by Me
dehydration to give an olelfin Q. Ozonolysis of Q Me Et
leads to a dicarbonyl compound R, which undergoes Q.11 The structure of the product S, is-
intramolecular aldol reaction to give predominantly S.
O
1.MeMgBr 3 2 O /Zn H O OH  − (2009)
P  → Q → R S
2.H+ ⋅H2O heat (A) (B)
3.H2SO 4 /Heat
O
Me Me
Q.9 The structure of the carbonyl compound P, is Me

O O

(A) Me
(C) (D)
O CH3
CH3
Me Me
(B)
O CH3
Paragraph 2
Two alipbatic aldehydes P and Q react in the presence of
(C) aqueous K2CO3 to give compound R, which upon treatment
with HCN provides compound S. On acidification and
O C2H5 heating. S gives the product shown below:  (2010)
O
H3C OH
(D)
H3C
C2H5 O O
O

H3C C
2 3 . 6 8 | Aldehydes and Ketones H
(B) C
H3C
H2C
Q.12 The compounds P and Q respectively are Q.14 The compound S is -
CN

CH3 CH3 CH3 CN


O
CH H H3C CH CH CH
(A) H C
H3C (A) H3C CH H (C) H3C CH H
C C
and
O O H2C H2C
CN OH
CH3

CH O CN
H H H
(B)
H3C C C C H3C CH
H3C OH
and H (D) C
(B) C H3C
O O H3C
H2C H2C
H3C CH2 H H3C H CN
CN
(C) CH C and C
CH3 CN
CH3 O O
Q.15 Match CHcompounds/ions
CH in column I with their
properties
(C) H3C / reactions
CH inHcolumn II. (2007)
H3C CH2
H H H ColumnHI2C Column II
CH C
(D) C and (A) C6H5CHO (p) OH gives precipitate with
CH3 O O 2,4-dinitrophenylhydrazine
CN
(B) CH3C≡CH (q) gives precipitate with AgNO3

Q.13 The compound R is - (C) CN


H3C– (r) is a nucleophile
CH
(D) C OH
(D) IH3C

(s) is involved in cyanohydrin formation
O
H2C
H3C C Q.16 In the scheme
CN given below, the total number of
H
(A)
H3C
C intramolecular aldol condensation products formed
H2C from ‘Y’ is:  (2010)
OH
1. O3 1. NaOH(aq)
Y
O 2. Zn,H2O 2. heat

H3C C
(B) C H Paragraph for Questions 17 and 18
H3C
CH In the following reactions sequence, the compound J is
H3C OH an intermediate  (2012)

3 (CH CO) O
2 2 (i) H ,Pd/C
CH3
O I →
CH COONa
J 
(ii)SOCl /Anhy.AlCl
→K
3 2 3
CH C
(C) H3C CH H J(C9H8O2)gives effervescences on treatment with
NaHCO3 and positive Baeyer’s test.
H2C

OH
Q.17 The compound K is
CH3
O
CH (A) (A)
(B) (B) (C) (C) (D) (D
C
(D) H3C CH H O O O O
O O
CH (A) (B) (C) (D) O
(A) (B) (C) (D) O
H2C OH
O OO O
O O
?
OH-

CH3 Chem i str y | 23.69

Q.18 The compound OH


I is O OH OH
OHC CHO CHO
H
O H OH O CH3 H
H
CH3 H3C CHCl2 H3C CHCl2
CH3
(A) (B) (C) (D)
P Q R S

(A) P(major) (B) Q(minor)


H
OH O CH3 H (C) R(minor) (D) S(major)
H

Q.22 After completion of the reactions (I and II), the


(C) (D)
organic compound(s) in the reaction mixtures is(are)
 (2013)

Q.19 The number of aldol reaction(s) that occurs in the O


given transformation is  (2012)
Reaction I : H C Br3(1.0 mol)
OH OH 3 CH3 aqueous/NaOH

conc.aq. NaOH (1.0 mol)


CH3CHO+4HCHO O
O
HO OH
Br3(1.0 mol) Reaction II : Br3(1.0 mol)
Reaction I : H C CH3 H3C CH3
3 aqueous/NaOH CH3COOH
(A) 1 (B) 2 (C) 3 (D) 4 (1.0 mol)
(1.0 mol)
O O O O O
Q.20 The major product H in the given reaction
sequenceReaction
is  II :
(2012)
Br3(1.0 mol)
H3C CH3 H3C CH2Br H3C CBr3 Br3C CBr3 BrH3C CH
CH3COOH
95%
95% H
H22SO P Q R S
CH CH 95% HCN
2SO
SO44
2 CO CH3 4 G H
CN
CH333
H3 CH2 CO CH CHCN
2 COGCH3
(1.0 Gmol)
H Heat H
Heat Heat
O O O O O

(A) CH3 CH=C (A)


(A) CH
CH33 CH=C
COOH CH=C COOH
COOH CNCH
(B)
(B) CH3 CH=C (B) CH33 CH=C
CH=C CNCN
H3C CH3 CH2Br H3C CBr3 Br C CBr
CH BrH3C CH2Br H3C ONa CHBr3
CH3 CH3 CH3 3 CH333
P Q R S T U
OH OH
OH
(A) Reaction I: P and Reaction II : P
CH
CH33 CH22 C
(C)COOH CH C COOH (D) CHNH
COCH 3 CH=C CO NH
CO NH22
C) CH3 CH2 C(C) COOH (D) CH3 CH=C (D) 3 CH=C
(B) Reaction I: U, acetone and Reaction II:Q, acetone
2

CH CH CH
CH3
CH CH3
(C) Reaction I : T, U, acetone and Reaction II : P
3 3 3 3

(D) Reaction I : R, acetone and Reaction II : S, acetone


Q.21 In the following reaction, the product(s) formed is
OH
(are)  (2013)
OH CHCl3 Q.23 The most suitable reagent for the conversion of
OH-
CHCl 3
? R-CH2-OH → R-CHO is:  (2014)
?
OH- (A) KMnO4
CH3
(B) K2Cr2O7
CH3
(C) CrO3
OH O OH OH
OHC OH CHO O OH
(D) PCC (Pyridinium
OH
chlorochromate)
CHO
OHC CHO CHO

CH3 H3C CHCl2 H3C CHCl2


CH3
CH H3C CHCl2 H3C CHCl2
P 3 Q R CH
S 3
P Q R S
2 3 . 7 0 | Aldehydes and Ketones

Q.24 Among the following, the number of reaction(s) Q.27 Reagents which can be used to bring about the
that produce(s) benzaldehyde is (2015) following transformation is  (2016)

CO, HCl
(A) Anhydrous AlCl3/CuCl O O O O
C O C O

H OH
CHCl2
O O
H2O COOH COOH
(B) o
100 C
(A) LiAlH4 in (C2H5)2O
COCl (B) BH3 in THF
(C) NaBH4 in C2H5OH
H2
(C)
Pd-BaSO4
(D) Raney Ni/H2 in THF
CO2Me
DIBAL-H Q.28 (2016)
(D) o
Toluene, -78 C
H2O OO

i) i)HCHO
HCHO(excess)/NaOH, heat
(excess)/NaOH, heat
Q.25 The major product of the following reaction is ii) HCHO/H+ +
(catalytic amount)
 (2015) ii) HCHO/H (catalytic amount)
O
O O O O O OH
i. KOH, H2O O O O O O OH
CH3 ii. H+ , heat i) HCHO (excess)/NaOH, heat
(A) (B)
i) HCHO (excess)/NaOH, heat
(A) (B)
O ii) HCHO/H+ (catalytic amount)
ii) HCHO/H+ (catalytic amount)
O OH
CH3 CH3 O O
HO O OH
O O
O O O O O HOO O OH
O O (D) O O OH
(C)
(A) i) HCHO (excess)/NaOH, heat (D)
(C)
(A) (B) (B)
(A)
ii) HCHO/H+ (catalytic amount) (B) OH
O OH
CH3 O OH
O O O OH
O HO O O
CH3 O O HO O O OH
(C) (D) (D)
(C) (D)
(C)
(A) (B)
OH
OH
Q.26 Positive test is observed for  O O (2016) O OH
HO
H O
H O CHO (D)
(C) CHO
(A) H (B)
(A) H H (B)
H OH
H
H

OH O
OH O
(C) Ph (D) Ph Ph
(C) Ph Ph (D) Ph Ph
Ph
O
O
Chem i str y | 23.71

PlancEssential Questions
JEE Main/Boards JEE Advanced/Boards
Exercise 1 Exercise 1
Q.1 Q.3 Q.9 Q.11 Q.12 Q.15
Q.10 Q.16 Q.22 Q.25 Q.26 (4) Q.27 (3)
Q.28 (5-i)

Exercise 2 Exercise 2
Q.3 Q.8 Q.12 Q.2 Q.6 Q.8
Q.15 Q.9 Q.17 Q.18
Q.25

Previous Years’ Questions Previous Years’ Questions


Q.2 Q.7 Q.8 Q.9 Q.14
Q.12

Answer Key

JEE Main/Boards
Exercise 2

Single Correct Choice Type

Q.1 B Q.2 B Q.3 C Q.4 A Q.5 C Q.6 A


Q.7 C Q.8 B Q.9 D Q.10 C Q.11 C Q.12 C
Q.13 B Q.14 B Q.15 D Q.16 D

Previous Years’ Questions


Q.1 D Q.2 B Q.3 A Q.4 B Q.5 A Q.6 B
Q.7 C Q.8 D Q.9 A Q.10 A Q.11 B Q.12 C
Q.13 D Q.14 A, C Q.15 D Q.16 D Q.17 A Q.18 B
Q.19 A
2 3 . 7 2 | Aldehydes and Ketones

JEE Advanced/Boards
Exercise 2

Single Correct Choice Type

Q.1 C Q.2 C Q.3 A Q.4 B Q.5 B Q.6 A Q.7 C


Q.8 A Q.9 D Q.10 C Q.11 B Q.12 C Q.13 C Q.14 C
Q.15 B Q.16 B Q.17 B Q.18 D Q.19 C

Multiple Correct Choice Type


Q.20 B, D Q.21 A, B, D Q.22 B, C, D

Comprehension Type
Q.23 C Q.24 B

Match the Columns


Q.25 A → p, q, s; B → p; C→ p, q, s; D → p, q, s; Q.26 A → r; B → r, s; C → q, s; D → p

Previous Years’ Questions


Q.1 A Q.2 C Q.3 C Q.4 C Q.5 B Q.6 A Q.7 A, D
Q.8 B, D Q.9 B Q.10 A Q.11 B Q.12 B Q.13 A Q.14 D
Q.15 A → p, q, s; B → q, r; C → q, r, s; D → q, r Q.16 1 Q.17 C Q.18 A Q.19 C
Q.20 B Q.21 B, D Q.22 C Q.23 D Q.24 A, B, C, D Q.25 A Q.26 A, B, C
Q.27 C Q.28 A

Solutions

JEE Main/Boards CN
or CH2
CN
CH2
COOH CN
Exercise 1
Mechanism
Sol 1: Knovengeal reaction COOH COOH
CH2 
CH + H
It is the condensation of any carbonyl compounds with COOH COOH
compounds containing in active methylene compound O OH
in the presence of pyridine ||  COOH | COOH
H–C–CH H–C–CH
CHO CH=CH–COOH COOH COOH

COOH
Pyridine
2 + CH2   OH
H2 O,  CO2
COOH | COOH
H–C–CH CH=CH–CH–COOH
COOH COOH
Instead of CH2 we can also use (–H2O)
COOH (–CO2)
Chem i str y | 23.73

OH (vii) 2Ph–CH2–CH=O 


Dil. NaOH
→ Ph–CH–CH=O
CN Pyridine | ∆
|
CH3–C–H + CH2 CH–C–CH–CN
|| COOH | | CH–CH2–Ph
O H COOH |
OH
(viii) 2CH3–CH2–CH2–CH2–OH →
OH
| –H2O It will not undergo any reaction
CH3–C–CH–CN CH3–CH=CH–CN
–CO2
| | CH3 O
H COOH | ||
(ix) 2H3C – H2C – C – CH 
Conc. NaOH


|
Sol 2: (i) CH2 = O it will undergo cannizzaro reaction : CH3

Conc. NaOH CH3


2CH2= O → HCOONa + CH3OH |

CH3 – CH2 – C – COONa +
(ii) Aldol condensation |
O CH3
|| CH3
Dil. NaOH
H–C–CH–CH2–CH2–CH3  → |
| ∆

CH3
CH3 – CH2 – C – CH2 – OH
|
OH CH3 CH3
| |
H – C – CH – CH2 – CH2 – CH3 Sol 3:
O
O
|| ||
H–C–C–CH2–CH2–CH3 (i) CH3 – C – H  → CH2 – CH2 – CH – CH3
| | |
CH3 OH OH
O
Cannizzaro reaction ||
Dil. NaOH
(iii) Benzaldehyde CH3 – C – H  →

O O
H O
|| ||
| || H2/Pd
C–H C–ONa CH2–OH CH3 – C – CH2 – C – H CH3–CH–CH2–CH2–OH
| |
Conc. NaOH OH OH
2   +
O


||
(iv) Benzophenone (ii) CH3 – C – H → H – C – C = C – CH3
O || | |
|| O H H
C
O H O
|| | ||
Dil. NaOH
CH3 – C – H  → CH3 – C – CH2 – C – H
It will not undergo aldol or cannizzaro reaction ∆
|
O OH
O Aldol OH H O
| ||
HCl
(v) 2 Dil NaOH
   → CH3 – C = C – C – H

|
H
2 3 . 7 4 | Aldehydes and Ketones

O H O Sol 5: C9H10O → gives +ve tollen's test


|| | ||
⇒ It is a aldehyde → undergoes cannizzaro
(iii) CH3 – C – H → CH3–C–CH2–C–H
Dil. NaOH

| ⇒ α hydrogens not present → vigorous oxidation →
OH
COOH
H O COOH
| ||
HCl
 → CH3 – C = C – C – H
|
H ⇒ C9H10O has two groups attached at neighbouring
O positions
||
CrO3/H2O/Acetone ⇒ 1 group is aldehyde with no α-hydrogen
CH3–CH=CH–C–OH
⇒ 2nd group is ethyl group
CHO
Sol 4: CH3–CH2–CHO CH2–CH3
Aldol
+  → So C9H10O is
CH3–CH2–CH2–CHO

(1) CH3–CH–CHO Sol 6: (i) Refer page-23.15 Section-6 in the sheet


|
CH–CH2–CH3 (ii) When the condensation is between two different
| carbonyl compounds, it is called crossed aldol condensation.
OH For example.
Θ
CH3–CH2–CHO is nucleophile OH
(a) R3C – CHO + CH3CHO  →
CH3–CH2–CHO is electrophile
R3C – CH(OH) – CH2 – CHO crossed product
O
(2) CH3–CH2–CH2–CH–CH–CH2–CH3 ||
| | OH–
(b) CH3CHO + CH3C – CH3
OH CHO
CH3 – CH(OH) – CH2 – COCH3
CH3–CH2–CH2–CHO is nucleophile
4-Hydroxypentan-2- one (crossed product)
CH3–CH2–CH2–CHO is electrophile
OH–
(c) C6H5CHO + CH3COCH3
(3) CH3–CH2–CH–CH–CH2–CH3
| | C6H5 – CH = CH – CO – CH3
OH CHO
Sol 7: (i) Fehling’s Test, (ii) 2, 4 DNP Test
CH3–CH2–CHO is electrophile
CH3–CH2–CH2–CHO is nucleophile OH OH O

CHO Sol 8: (i) CHCl3 H


3KOH
|
(4) CH3–CH2–CH2–CH–CH–CH3 Reimer – Tiemann reaction
|
OH (ii) OCH3 OCH3

CH3–CH2–CH2–CHO is electrophile
CH3–CH2–CHO is nucleophile
O

Friedel
Friedel Craft Acylation
– Crafts Acylation
Chem i str y | 23.75

Sol 9: (i) Rosenmund Reduction Sol 12: (a)


O O Catalytic
CH3COOCH3 + 2H2 CH3CH2OH + CH3OH
H2 hydrogenation
CH3 Cl Pd-BaSO4 CH3 H
Dehydrogenation
(ii) Tollen’s Reagent
CH3CHO + 2[Ag(NH3)2]+ + 3OH– → RCOO– + 2Ag +
2H2O + 4NH3 CH3CHO + HCHO

(b) (i) Due to the presence of 0 group with is


C =O
Sol 10: The pinacol-pinacolone rearrangement is a polar
method for converting a 1, 2- dial to a carbonyl compound
in organic chemistry. This 1, 2-rearrangement takes place (ii) Aldehydes have lower boiling points as they are not
under acidic conditions. The name of the reaction comes associated with intermolecular
from the rearrangement of pinacole to pinacolone.
H-bonding whereas alcohols are associated with
H H+ H H + H H intermolecular H-bonding. Aldehydes have lower B. P.
O O O O
Sol 13: Refer theory.
H3C CH3 CH3 CH3

CH3 CH3 CH3 CH3 Sol 14: (i)

pinacol LiAH4 SO2Cl2


CH3CHO CH3 CH2 OH CH3 CH2 Cl
H2O
H H
O O Alcoholic
H3C H3C KCN
CH3 CH3
H3C H3C hydrolysis
CH3 CH3
O LiAH4
H3C CH3 CH=CH2 dehydrate CH3 CH2 CH2 Cl CH3 CH2 COOH
H2O

H3C CH3
CH3
H2O
pinacolone

CrO2
CH3 CH CH3 CH3 CH CH3
Sol 11: (i)
OH O
LiAlH4 SO2Cl2
CH3COOH CH 3–CH2OH CH3–CH2–Cl
H2O (ii)
Alcoholic
bleach H3O+
KCN CH3–C–CH3 CH3COO–+CHCl3 CH3COOH
NaOH
||
dehydrate LiAlH4 O
CH3 –CH=CH2 CH3 –CH2 – CH2 – OH CH3 –CH2 –COOH
H2O
Br2 H2SO4 LiAlH4
H2O BrCH2–CH2Br CH2=CH2 CH3CH2OH
 ether
CrO3
CH3–CH–CH3 CH3–C–CH3 alc. KOH
H+/H2O
| ||
OH O
acetylene
(ii) Friedel – Craft reaction
CH Cl+FeCl
Benzene 
3 3→ Toluene + HCl + FeCl3
2 3 . 7 6 | Aldehydes and Ketones

H SO (ii) Semicarbazide has two –NH2 groups. The attachment


2 4 →B + C
Sol 15: A C8H16O2  of the two –NH2 groups on the
hydrolysis

C = O is different one –NH2 is directly attached to


C 
Chromic acid
→B C = O while the other –NH2 is not directly attached out
dehydration
attached through –NH. Therefore due to resonance
C  → but-1-ene the electron density on this –NH2 group decrease and
therefore cannot act as a nucleophile. The lone pair of
⇒ C = CH3CH2CH2CH2OH
electrons in –NH2 which is attached through –NH is
⇒ B → CH3CH2CH2CH2COOH not involved in resonance and therefore is available for
nucleophilic attack on the C = O.
⇒ A → CH3–CH2CH2 COOCH2CH2CH2CH3
(iii) The reaction between carboxylic acid and alcohol
is a reversible reaction. The ester and the water which
Sol 16: (i) 2-4 DNP test is formed as a product of can again react to form
Only carbonyl compounds give positive test carboxylic acid and alcohol so for a reaction to proced
in the forward direction in a reversible reaction, one
(ii) Acetophenone - It give iodoform test of the product should be removed. That is the reason
Benzophenone-It doesn’t give Iodoform test during the preparation of esters form a carboxylic acid
and an alcohol in the presence of an acid catalyst, the
(iii) Phenol - It give violet colour with FeCl3 test.
water on the ester should be removed as soon as it is
Benzoic acid-It doesn’t give violet colour with FeCl3 formed.

Sol 17: (i) Sol 19: Calculation of molecular formula :-


LiAlH4 H2SO4 % of C = 69. 77
CH3–C–CH3 CH3–CH–CH3 CH3CH=CH2
|| | % of H = 11.63
O OH % of O = 100 – (69. 77 + 11.63) = 18. 6%
Ratio of C = 69. 77 / 12 = 5. 88
(ii) CH3CH2OH Reduction
Cu Ratio of H = 11.63 / 1 = 11 63
Ethanol
Ratio of O = 18.6 / 16 = 1. 16
CH3CHO+NaOH → CH3–CH–CH2CHO smallest ratio
|
C = 5. 88/1. 16 = 5
OH
3-Hydroxybutanal H = 11.63/1. 16 = 10
O = 1. 16/1. 16 = 1
(iii) CHO CHOHC6H5 Empirical formula = C5H10O1

H+/H2O Mol mass = 86


+ C6H5–MgBr
n = molecular mass/Empirical formula mass = 86/86 = 1
COC6H5 MF = C5H10O
Reductions Since the compound forms on addition compounds
+ CH2Cl2
C6H5NH+Cl-/CrO3 with sodium hydrogensulphite, the said compound
should be ketone as aldehyde.
Since the compound does not reduce Tollen’s reagent,
Sol 18: (i) 2, 2, 6-trimethyl cyclohexanone has three therefore it is a ketone. Since the compound gives
methyl group at the alpha position. There is a lot positive iodoforms test, the compound should be
of steric hindrance in this molecule. Due to steric methyl ketone. On vigorous oxidation it gives ethanoic
hindrance the CN ion do not attack the molecule. These and propanoic acid therefore the given compound
is not steric hindrance in cyclohexanone. Therefore its should be pentan-2-one.
forms cyanohydrin in good percentage.
Chem i str y | 23.77

The structure is Sol 25:


O
|| Cl–C–Cl
MeMgCl
Me–C–Cl
MeMgCl
Me–C–Me
CH3–C–CH2–CH2–CH3 || || ||
O O O
Mg
Sol 20: Aldol takes place in the presence of α-hydrogen
and cannizzaro takes place in the absence of
MeMgCl
α-hydrogen.

Sol 21: A does not give litmus test ⇒ A does not Me


contain –COOH group. |
Me–C–Me
A does not give 2, 4-DNP test ⇒ A does not contain |
C = O group OH

A + MeCOCl → B So 3 moles of MeMgCl required for 1 mole Cl–C–Cl


262 ||
Initial mass = 178 O

Final mass = 262 difference is 84 O Exercise 2


||
Now A has lost hydrogen and gained (Me–C–) group Single Correct Choice Type
so change in mass corespnding to
Sol 1: (B)
1(–OH) group 43 – 1 = 42
84
So no of hydroxy groups = =2
42

Sol 22: (i) (3) (4) will give positive fehling test as they
are aldehydes (6) will give positive Fehling test as it has
α-hydroxy ketone.

Sol 23: 3 nitrogen will give reaction with C=O


because it has maximum electron density. The other
O
||
two nitrogens are in conjugation with (–C–) group so
they are loss electron density.

Sol 24: 2 products are formed in good amount 1 with Sol 2: (B)
1st nitrogen O O
H O ||
CH2N2
||
| || CH3–C–CH2–CH3 CH3–C–CH2–CH2–CH3
Ph–C=N–C–NH–NH2 (A)
(1) (2) (3)
CH3CO3H
1 with 3rd nitrogen

H O O
| || ||
Ph–C=N–NH–C–NH2 CH3–C–O–CH2–CH2–CH3
(3) (2) (1)
(Bayer villiger)
2 3 . 7 8 | Aldehydes and Ketones

Sol 3: (C) Sol 7: (C)

O O CH2=CH–C–CH3 O O
|| H || || NH2–CH3
B
  O CH3–C–CH3 CH3–C–CH3
, –H
NH–CH3
O O
||


OH2
H2O
CH3–C–CH3 CH3–C–CH3
O
||
OO O N–CH3 NH–CH3
O O O
|| ||
B

Sol 8: (B) O

H2O H2N NH2


+ N2

O H3C O OCH3 H3C O OCH3

Octane Wolf-kishner
Walf kisher --reduction
reduction

In clemmension reduction HCl will break ether bonds.


Sol 4: (A) It is carried out using NaBH4 as NaBH4 will
not reduce ester and reduces only carbonyl part.
Sol 9: (D)
LiNH2 (i)HCHO
Sol 5: (C) MeCHO can undergo aldel condensation as Bu–CCH Bu–CCH Bu–CC–C–Ph
(ii)H2O
its has more than 1 α hydrogen present.
OH

Sol 6: (A)
O MnO2
O O

Bu Bu
Ba(OH)2 
Bu–CC–C–Ph
 
C–Ph C–Ph O
O O
O O
O
O O
Sol 10: (C) H O
O EtoNa  H–C–OEt
O
H H–C–OEt
H2O,  EtoNa 

O
O
O O
C–H O C–H
O O
C–H OH C–H

OH
Chem i str y | 23.79

Sol 11: (C) Sol 15: (D) KMnO4 / H2SO4 ; K2Cr2O7 / H2SO4 ; Ag2O /
NaOH will oxidise – CHO → –COOH but LiAlH4 is a strong
O OH O
reducing reagent so it will reduce –CHO.
Aldol
2CH3–C–H CH3–CH–CH2–C–H
(A) Sol 16: (D) H2 / Pd–C will reduce –C= C and –CHO. H2/
Pd – BaSO4 – CaCO3 will reduce only –C ≡ C– LiAlH4 and
NaBH4 will reduce –CHO to –CH2OH.
O CH2–OH
Aldol
CH3–C–H + 3HCHO HOH2C–CH2–OH Previous Years’ Questions
CHO
Sol 1: (D)
Sol 12: (C) CH2 = C = O + H2S
Cl OH
KOH
CH3–CH CH3–CH
H+ 
HS Cl

OH
CH2=C=O CH2=C=OH CH2=C–OH

HS –H2O
CH3CHO

H O H
CH2–C–OH Sol 2: (B) CH2 = C O3
H–C +O=C
R H R
S Presence of one vinyl group gives formaldehyde as one
Sol 13: (B) of the product in ozonolysis
O NNH2
Sol 3: (A) The cannizzaro product of given reaction
N2H4 KOH/s yields 2, 2, 2-trichloroethanol.
OH Cl Cl Cl OH
| O | O | |
OH Cl–C–C
NaOH
Cl–C–C + Cl–C–CH2
| H | O– |
(x) (Y) Cl Cl Cl

Sol 4: (B)
Wolf kishner reaction  O
O O ||
|| fast | Ph–C–H
Sol 14: (B) Ph–C–H+OH Ph–C–H R.D.S
|
Br2 OH
CH3–C–CH2–CH2–CH2–CH2–C–CH3
KOH O O
|| + ||
O O Ph–C–OH+Ph –CH2O
H
Ph–C–O+Ph –CH2–OH
exchange
Intramolecular aldol condensation will take place.
O
Sol 5: (A) Crossed aldol reaction gives benzyl alcohol
CH3
and sodium formate.
+
NaOH(aq)
C–CH3 CH3 C6H5CHO+ HCHO  →
Bezaldehyde Formaldehyde

O C6H5CH2OH+ HCOONa
Benzylalcohol Sod. formate
CHO CHO

OH/100°C
2 3 . 8 0 | Aldehydes and Ketones
Intra molecular
Cannizzaro reaction

Sol 6: (B) Benzaldehyde will undergo Cannizzaro’s COO– CH2OH


reaction on treatment with 50% NaOH to produce
benzyl alcohol and benzoic acid as it does not contain +
H /H2O
α-hydrogen

CHO CH2OH CHO


| | | CHO OOC

50% NaOH COOH CH2OH


+

Benzaldehyde Benzyl alcohol Sodium benzoate

Sol 7: (C) It is a nucleophilic addition whose reactivity CH2OH COOH


depends upon the electrophilic character of carbonyl
carbon and steric hindrance only, So. the ease of the
reaction would be HCHO > CH3COCH3 > PhCOCH3 >
PhCOPh Sol 12: (C)

OH
Sol 8: (D) Tollen’s reagent oxidizes the compound +
H2O/H O
having aldehyde group like glucose and also oxidizes P H2C H3C—
α-hydroxy ketone having –COCH2OH group as in CH3 CH3
fructose - H2O/H
+
Q H3C
H3C—
Sol 9: (A) OH
CHO
CH=CHCOCH3

Sol 13: (D) CH3



NH2NH2/OH
KMnO4
HO
Wolf-kishner Reduction C= CHCH2CH2CH3
CH3
CH3
CH=CH–CH2CH3 C= O+HOOCCH2CH3
CH3

HO
Sol 14: (A, C)
–OH group and alkene are acid sensitive groups so O +
O
clemensen reduction can not be used. CH3–C
(Ag(NH3)2)
CH3–C +Ag 
H O
(O)
Sol 10: (A) CH3 CH2 CH2OH  → CH3 CH2 − CHO O
(A) (B) O
|| (Ag(NH3)2)

H–C–H H–C +Ag 


Sol 11: (B) CHO CHO O

CHO CHO

OH/100°C
Intra molecular
Cannizzaro reaction

COO– CH2OH

+
H /H2O
Chem i str y | 23.81

Sol 15: (D) CH3 CH3


O HBr
Sol 2: Peroxide
H3C CH CH CH3 CH3 H3C CH CH CH CH3 Br
(A)
O O HBr/Peroxide → Anti-Markownikoff’s Addition.
H2O/Zn

O Sol 3:
H2CrO4 CrO3
C [J] [I] CH3–CHO
H3 C H H2O (Pyridine)
(B)
Sol 16: (D)
Strong oxidizing Weak oxidising agent
aqu.KOH agent
CH3CHCl2  → CH3CH(OH)2 →
−H O
CH3CHO
2

J → CH3COOH I → CH3 CH2–OH


Sol 17: (A) In Cannizzaro reaction given below, the
slowest step is the attack of OH at the carboxyl group Θ
OHΘ HSO4
Sol 4: Acetone (2 mol) [K] [L]
Sol 18 : (B) The first reaction. Aldol condensation.
O OH
CH3
CH3
C dil OH C O
CH3 C CH2 CH2 CH CHO ∴ 2 CH3 CH3 CH3
CH3 O CH3 CH3  C
CH3
(5-keto-2-methyl hexanal)
(5-keto-2-methyl hexanal)
Thus, K is

Sol 19: (A) Vinyl group ( CH=2 CH − ) on ozonolysis


OH
|
O
||
give formaldehyde CH3 – C – CH2 – C – CH3
|
CH3
JEE Advanced/Boards

Exercise 1
HSO4 (H )

OH2 H O
Sol 1: Chlorination
| | ||
O CH3 – C – CH – C – CH3
|| |
COOH C–Cl
CH3
CH2–Ph CH2–Ph (a) O
SOCl2
||
 L is CH3 – C = CH – C – CH3
Anhydrous
AlCl3 CH3

O
||
Sol 5:
Zn–Hg O O
HCl || ||
(H) C – CH3 C–CH2–Br
Br2 (1 eq,)
2 3 . 8 2 | Aldehydes and Ketones

As only 1 equivalent is used, Br will be added to the Sol 9:


alkyl group otherwise the phenyl ring also would have CH3
be substituted. [H2/indlar’s catalyst
is addition of H2]
[NaBH4 → weak reducing agent. ] H
H
O OH
|| |
C – CH2Br CH2–CH2–Br Sol 10:
NaBH4
F F OCH3 OCH3

Na OCH3
HgSO4/H ⊕  –F
Sol 6: R–C≡CH R–C–CHO
O NO2 NO2 NO2
||
CCH CH2–C–H
HgSO4/H This is ArSN
Aromatic Substitution
O R
R–C + NH2OH  C=N
| R’ OH2 Sol 11: O
R’ ||
C
Fe/Br2
Ph–CH2 N 
CH2–CHO
C=N
 + NH2–OH H OH
This ring is This is
deactivated activated
(geometrical isomer)

O
Sol 7: O ||
||
(i) Me MgBr N  Br
(ii) aq. HCl
OMe
Sol 12:
Me MgBr [Gignards Reagent] O
O
|| || 
dil alkali

|| ||
Me OMg Br Me OH
O O
HCl
O
|| O
OMe OMe ||

Sol 8:
Et–OH
 O O

Br
H
O O
(Carbocation
Rearrangement)



(Elimination)  OH
H
Chem i str y | 23.83

Sol 13: R D
D2O
R–CC–H C=C
Cl OH NaOD
Boil/alkali :OD H
CH3–CH2–CH CH3–CH2–CH D
Cl OH
R C H
C
two OH groups on same C, || D
unstable hence dehydration O

D2O

CH3–CH2–CHO
D

D R
R R D C H
Sol 14: C D D2O C
C D C D || D
KOH || DO
CH3O CHO + HCHO O OD 

(Cannizaro Reaction)
Sol 16:
O O
MeO CH2OH + HCOOH || .. HNO3/H2SO4
||
CO CO NO2
(mono)
Here, alcohol is formed on the phenyl ring
Note :- as the (MeO –) group. Favours +ve charge Deactivated Activated
(stabilizes) O O
|| .. ||

CO
HNO3/H2SO4
CO NO2
(mono)
Sol 15:
Br
Deactivated Activated
Sol 17:
Br2/CCl4 Br NaNH2
OH
(trans addition) |
OH COOH
O ( NH2 - Nuclophile) (CH3CO)2O

|| NaOD/D2O
CH3COONa
CHO
HgSO4/H2SO4 (excess)
This is a very standard example of Perkin’s condensation
O
|| {If possible remember it}
NH2 – C – NH–NH2
O Sol 18:
|| D
NH NaOH
NH NH2 A+B C=CCHO
D 
D | |
O H H

Mechanism with NaOD/D2O (excess) {Aldol condensation}


CHO
+ CH3CHO

(Desired product)
2 3 . 8 4 | Aldehydes and Ketones

Sol 19: Sol 23:


O red P/Cl2
|| CH3–COOH ClCH2–COOH
(1 eq.)
CH3–COOH + NH3  CH3–C–NH2
 (excess) NH3
(HVZ reaction)
Br2/KOH
CH3–CH2–NH2 CH3–CN O
Step 1 is : Nucleophilic substitution of –OH by –NH2 ||
CH2–C–NH2
Step 2 is : Clearly dehydration |
Step 3: Hoffman Bromanide Reaction (through the NH2
intermediate R–N=C=O)
( NH2 is a nucleophile)
Sol 20: CH3–CH2–OH
(O)
CH3–CH=O
Sol 24:
(i) Cl2
HCOOH
(ii) dil NaOH O O
+ CHCl3 (A) 
HgSO4 NaOI
[Chloroform reaction]
H2SO4
OH
Iodoform → presence of α-hydrogen (B) 
+ CHI3
Br
Sol 21: |
P and Br2
CH3–CH2–COOH CH3–CH–COOH
Br
| (Iodoform Reaction)
P and Br2
CH3–CH2–COOH CH3–CH–COOH
Br
| Sol 25:
Br
OH H
| CH3
alc. KOH Br2/Hv
CH3–C–COOH
OH CH4 CH3 Br
| (A) AlCl3 (anhyd) (C)
alc. KOH
CH3–C–COOH
OH (B) Cl2/hv
| (2 eq.)
OH
CHO OH Cl
dehydration CH CH
OH H2O/H Cl
dehydration
CH3–C–COOH (D)
||
CH3–C–COOH
O
CHO COOH
|| CH3COONa (anhy.)
red P/Br2 → Hell – Volhard Zelinski
O Reaction (HVZ)
[α - bromination]
(CH3CO)2O
[Perkin]
Moist H2SO4 C2H5OH C2H5OH
Sol I AgCO2H5CI 2Moist
C2H522: H5OHC2H5OHCHH22=CH
SO4
2 2=CH2
CH
2 Ag2O 140°C 140°C (D)
(A) (A) (B) (B)
..
.. .. NH2OH N
C2H5–OH C
2H5–OH
  
CH2 = CHCH
2 2 = CH2 CH2 – CHCH
2 –2 O – C22H
– CH – 5O – C2H5 (F) H OH
| |
H H (Mechanism)

C2H5–O–C H55–O–C2H5
C22H
∴ (C) → Et – O – Et
Chem i str y | 23.85

Perkins O
|| ..
CHO O (iii) In formic acid H–C–O–H
.. (I)
O O
| O O
C O || ||
 C C
CH3 CH3 O CH3 O–
|
+ H–C=O–H (II)
COOH CH3COO
Structure (I) is more stable than structure (II)
OH O
COOH
| || So C=O bond length will be less than C–O in (I)
H O
+ ()
But in sodium formate resonance is equivalent
O O–
|| |
Sol 26: (i) Higher carbonyl compounds aare insoluble H–C–O H–C=O
in water due to more covalent character.
(iv) Because in the first step cleavage of ester is done
(ii) Bisulphites of O are soluble in water which is diffcult to do.
(v) Because in addition reaction, electron donating
∴ Carbonyl compounds form solid additive products
with NaHSO3 which are separated out. The solid tendency of C=O and C=C are different
bisulphites of carbonyl compounds on hydrolysis by dil.
acid regenerate original carbonyl compound. C=C is better than C=O
(iii) Oxidation of toluene with chromium trioxide to
benzaldehyde is carried out in presence of acetic
anhydride because it converts benzaldehyde into its Sol 28: (A) C9H12O
diacetate so preventing its further oxidation to acid form. (i) Optically active
(iv) This is because aldehyde is more volatile than (ii) acidic hydrogen –OH
corresponding acid or alcohol.
O
(v) In a carboxylic acid, there is a pair of electrons on
(iii) –C–CH3 group
the oxygen in conjugation with carbonyl oxygen. This
prevents the creation of a positive centre at carboxyl
carbon. Therefore nucleophilic addition of phenyl (iv) C==C do not present
hydrazine to give phenyl hydrazine is not possible.
hot H+
O (v) (A) (B) ←
|| KMnO 4 C7H6O2

Sol 27: (i) OH is more acidic than O


O ||
|| + Cl – C – Cl
Si
OH because Si has empty orbital so can do
backbonding and by this decreases the acidity.
P
(ii) After first ionisation maleic acid ion is stabilised by (vi) (A) →
HI
(e)
Optically
hydrogen bonding but there is no such stabilisation in inactive compound
fumeric acid.
O (vii) 2 alcohol
O
C O– OH
O– C
H H
C O C O H CH3
||
O O (A)
(A) →
Mallic fumaric
acid
2 3 . 8 6 | Aldehydes and Ketones

H Sol 3: (A)
Sol 29: × 40 = 8 H
−4
5 × 10
NaBH4
moles
DT = kg m O
H–
O-
mass in kg
0.088 H
= 10−6
M D2O

mass = 88 OD
68.18
carbon atom = ≈ 12 Sol 4: (B)
12
O OH+ OH
oxygen = 1

Al2 O3
(A)  O3
 (B)   (C) + (D) C–H C–H C–H

350C  802 H O 14 H 14 14
CH2 CH2 CH2
–OH
Group C–H C–H C–H
Present
O O O
C5H12O
Zn
fehling
test
OH OH
H
OH  
OH C C–H
14
HCl 14 14
CH3–C–C–H CH2 C–H : CH2
(D) O
O H O O– C–H
||
(A) (B) (C) CH3–CH By O
Further Reduction
14
CH3–CH2–CH3
Exercise 2
Single Correct Choice Type Sol 5: (B)

SH S
NaOH
Sol 1: (C) 2 MeCH–CHO MeCH–CH2– OH OH
∆ dry
OH + MeCH–COONa HCl
O
Cannizzaro reaction OH OH

Sol 2: (C)
S S
CH2–Cl
O
PPh3   Ph–Li   O OH
Ph3P–CH2Cl Ph3P–CH
OH –OH OH

Ph
H+
O Ph3P=CH Sol 6: (A) CH3–CH2–CH2OH CH3–CH2–CHO
K2Cr2O7
+ (A) (B)
H–C–H
+ Ph3–P=O + LiBr
B forms a shining silver mirron on wesming
Chem i str y | 23.87

NH2 NH CONH2 Sol 11: (B) Hydration will occur at position 2 as carbon
CH3–CH2–CHO at positive 2 has maximum δ+ charge due to –I effect of
(B) other carbonyl groups.
CH3CH2CH=N NH CO NH2
Sol 12: (C) CH3–CH2–COOH + CH3–COOH
Ca
O O CH3–CH2–C–CH3
OH distillation
Na2 CO3 O
Sol 7: (C)
Sol 13: (C) NaBH4 is weak reducing agent. So it will
aldol condensation will occur as Na2CO3 is a base. reduce only the ketone but LiAlH4 is strong reducing so
it will reduce both ester and ketone.
Sol 8: (A) O
H O
H3O Sol 14: (C) CH3–CH2–OH & CH3–C–CH3
O H
LAN oxidises alcohol to aldehyde or ketone so only
CH3–CH2–OH will give positive test.
It will not
Give iodoform Sol 15: (B)
test O
O
–C–CH2–CH3 + CH2=O
+
H O

It will give positive –C–CH–CH3


iodoform test
CH2–OH

Sol 9: (D)
best
N–C2H5 NH–C2H5
most reactive carbonion C=O
CHO C–H CH2
group

H O
+ C2H5NH2

–C–CH–CH3

Sol 10: (C)


H2O
+ Sol 16: (B) Aromatic aldehydes do not give positive
OH OH
OH test with Fehling’s solution.
OH
 
Sol 17: (B)
+
H
O

H O C
 MeO

OH OH COOMe
+
H NaBH4

2 3 . 8 8 | Aldehydes and Ketones

Sol 18: (D) O


OH OH Br2
(C) Ph–C–NH2 Ph–NH2
NaIO4 KOH
CH2–CH–CH–CH–CH–CH2 2HCHO + 4HCOOH

OH OH OH OH Hoffmann
Brommamide
Sol 19: (D)
O NH
Br O
N2H4/KOH HN3
CH3–CH2–CH–C–CH3 (D)
 H2SO4

Br
Sol 22: (B, C, D)
CH3–CH2–CH–CH2–CH3
(A) H

KOH/ O

(B) CH–CH3
CH3–CH=CH–CH2–CH3
O
So only Iodoform can distinguish between them.
Multiple Correct Choice Type

Sol 20: (B, D) Aldehydes gives 2, 4-DNP test so (x) Comprehension Type

should have aldehyde x gives negative Iodoform ⇒ Sol 23: (C)

–C–CH3 group is absent so possible structure of (x) are O O


OH 
(CH3)3–C–CH2 (CH3)3–C–CH2
O
Br–Br
CH3–CH–CHO & CH3–CH2–CH2–CHO H OH
O
CH3
(CH3)3–C–CH2Br
Sol 21: (A, B, D)
O OH
O
(A) OH
H+ (CH3)3–C–CH3 (CH3)3–C=CH2
C=N C–NH–CH3

Br–Br
CH3
OH
(B)
14 H R–C–CH2Br + HBr
Ph–Li
C=C 

Cl

CH3 O
14
CC CH R–C–CH2–Br
Chem i str y | 23.89

Sol 24: (B) Overall yield = 0. 58 × 0. 54 × 0. 68 Sol 26: A → r; B → r, s; C → q, s; D → p


Overall yield = 0. 21 = 21% O
Me Conc. H2SO4
(A) C=N Me–NH–C–Et
Sol 25: A → p, q, s; B → p; C → p, q, s; D → p, q, s Et 
OH

(A) O
O
O OH
MCPBA O
CN (B)
HCN LiAlH4


traces of KOH +
+ N3H N3
(C) +
H HN3 +
-H
OH
CH2–NH2
NaNO2 (D) CHI3
Carbene
HCl KOH (excess)
formation

O OH OH OH
⊕ ⊕ Previous Years’ Questions

O
||
Sol 1: (A) CHO + H – C–H
O N–OH
NaOH
–CH2OH+H–COONa
H2O
+
NH2OH H
(B)
 This is an example of cross Cannizzaro reaction in which
formaldehyde is always oxidized.
OH
O Sol 2: (C) The reactivity of carbonyl compound towards
CH2
nucleophilic addition of Grignard’s reagent depends on
NH LAH NH extent of steric hindrance at α-carbon. Greater the steric
hindrance smaller the reactivity. Hence. reactivity order is

O CH3CHO>CHl3–CO–CH3>Ph –CO–Ph
OH– II I III
(C) CH3–C–CH2–CH2–CH2–C–H 

O O Sol 3: (C) X is (CH3CO)2 O and it is an example of Perkins


reaction.
(D) Ph H Ph CH3 NaOH
CH3   CH3 I2/H2O
Sol 4: (C) H3C ||
OH OH OH O
+
H
CHI3 + CH3CH2COONa CH3CH2COOH

Ph
Ph
CH3 
O CH3
OH
2 3 . 9 0 | Aldehydes and Ketones

Sol 5: H Comprehension 1 (Questions 9 to 11)


H2C = O + H2N–OH C=N–OH (Single)

H Sol: Q.9 B, Q.10 A and Q.11 B


O
|| CH3
AlCl 3
CH3–C–H+H2NOH + CH3Cl

CH3 OH H3C (two)


C=N + C=N Friedel -Crafts alkylation
H H OH
CH3
CH3MgBr

Sol 6:
O O CH3
COOH || H P
CH3
Zn–H2O H || +
H /H2O
O CH3
CH3
KOH Q
H2O (gives possible iodoform test)

intramolecular aldol condension CHO


CH3
reaction O3
Q
Zn-H2O
Sol 7: O
H3 C CH3
H3C H3C
R
O + C6H5NH2 C=N–C6H5
H3C H3C

H3C OH
aldol
C=N–C6H5 + C6H5NHNH2 O
H3C S
H3C OH
|
C=N–NHC6H5
H3C + CHCl3 + NaOH
OH
Sol 8: In both Friedel-Crafts reaction and Reimer- |
CHO
Tiemann’s reaction new carbon-carbon bond is formed. H+
CH3
AlCl 3
+ CH3Cl Reimer-Tiemann's reaction

All those carbonyl compounds containing α-H to sp2


Friedel -Crafts alkylation carbon show keto-enol tautomerism.

Comprehension 2 (Questions 12 to 14)

Sol: Q.12 B Q.13 A and Q.14 D


The given product an ester, obtained by condensation
of a hydroxy acid obtained through hydrolysis of a
cyanohydrin –
Chem i str y | 23.91

CH3 OH S is obtained by nucleophile addition of HCN on R,


hence R is
CH3 —C—CH
+
H3 C OH CH3 O CH3 OH
H
CH2 C=O
 H3 C
H3C —C—CH—H+HCN H3C —C—CH—CN
HO
OH O O CH2OH CH2OH
R S
Acid above is obtained by acid hydrolysis of cyanohydrin
S as R is obtained by treatment of P and Q aqueous K2CO3
CH3 OH CH3 OH Through aldol condensation reaction as
+
H
H3C —C—CH—CN H3C —C—CH—COOH CH3 O CH3
H2O

OH
CH2OH CH2OH CH3–CH–CHO+H–C–H OHC–C–CH2OH
S
P+Q CH3
R

Sol 15: A → p, q; B → q, r; C → q, r, s; D → q, r
NO2 O2N
(A)

PhCHO+O2N NH NH2 PhHC=N NH NO2


(ppt.)
NH3
PhCHO+Ag2O PhCOO- + Ag
(white ppt.)
CN
KCH
PhCHO Ph C O-

H
Ammonical AgNO
(B) CH3C ≡ CH 
3 → CH C ≡ C − Ag+
3
White ppt

CN
KCH
(C) PhCHO Ph C O-

(D) AgNO3 + CN− → AgCN ↓


AgNO3 + I − → AgI ↓

Sol 16: (1)


O

I) O3 O NaOH
O 
II) Zn3/H2O
2 3 . 9 2 | Aldehydes and Ketones

Sol 17 and 18: (C) and (A)

CHO CH=CH-COOH CH2-CH2-COOH


(CH3CO)2O H2-Pd/C
CH3COONa
I J

CH2 CH2 C Cl
SOCl2 AlCl3
O

K O

Sol 19: (C)

OH

O CH2
(i) OH- (i) OH-
H3C C H HO CH2 CH2 CH=O HO CH2 C CH=O
(ii) H-CH-O (ii) H-CH-O
(aldol) (aldol) H O
-
(i) OH (ii) H-C-H
(aldol)
OH
O OH OH
CH2
H C O + Conc. NaOH+HCHO
HO CH2 C CH=O
Crossed Cannizzaro reaction
HO OH CH2

OH

Sol 20: (B)

CN
CN- 95% H2SO4
HCN 
O OH CN

(G)

Sol 21: (B, D)


OH O OH
CHO
CHCl3
OH

CH3 H3C CHCl2 CH3 (Mnjor)


(Minor)

CHCl3+OH : CCl2+H2O+Cl-
Chem i str y | 23.93

OH O-

+ OH + H2O

CH3 CH3

O- O O- OH
CCl2 CHCl2 CHO
+ :CCl2 OH
H

CH3 CH3 CH3 CH3(Major)

O- O O

H2O

CH3 :CCl2 H3C CCl2 H3C CHCl2(Minor)

Sol 22: (C) Solve as per law of limiting reagent.

OH OH
O +
C -H+ C
C + H+ H3C CH3 H2C CH3
H3C CH3
O
Br2
Br-+BrCH2 C CH3

Sol 23: (D) PCC (Pyridinium chlorochromate) is a mild oxidising agent. It oxidises alcohol to aldehyde.

Sol 24: (A, B, C, D)


CHO
CO, HCl
(I) Anhydrous AlCl3/CuCl

CHCl2 CHO
H2O
(II) o
100 C

COCl CHO
H2
(III) Pd-BaSO4

CO2Me CHO
DIBAL-H
(IV)
Toluene, -78oC
H2O
2 3 . 9 4 | Aldehydes and Ketones

Sol 25: (A)

O O
OH
CH2 CH
CH3 CH3
O O

H2O

CH3 CH3
OH
O O
H+ , H2O/

Sol 26: (A, B, C)

CHO OH
Ph
CH2 = CH CHO, and Ph
O
Gives positive test with Tollen’s reagent.

Sol 27: (C) NaBH4 in C2H5OH selectively reduces aldehydic group.

Sol 28 : (A)
CH3
O CH3
CH3 C CH
O C CH
C CH CH3 CH3
CH3 O CH2 OH O

CH3 CH2OH
CH3 C CH
C CH CH3
O O CH2 O
CH3
CH2OH
2017-18 100 &
op kers
Class 12 T
By E ran culty
-JE Fa r
IIT enior emie .
S fP r es
o titut
Ins

CHEMISTRY
FOR JEE MAIN & ADVANCED
SECOND
EDITION

Exhaustive Theory
(Now Revised)

Formula Sheet
9000+ Problems
based on latest JEE pattern

2500 + 1000 (New) Problems


of previous 35 years of
AIEEE (JEE Main) and IIT-JEE (JEE Adv)

5000+Illustrations and Solved Examples


Detailed Solutions
of all problems available

Plancess Concepts
Topic Covered Tips & Tricks, Facts, Notes, Misconceptions,
Key Take Aways, Problem Solving Tactics
Carboxylic Acid
and Derivatives PlancEssential
Questions recommended for revision
24. CARBOXYLIC ACID AND
DERIVATIVES

1. INTRODUCTION
Compounds containing the carboxyl group are distinctly O O
acidic and are called carboxylic acids. R C O C R Anhydride

R C  N Nitrile
O
R C OH
R C X Acyl halide
On simple O
They have general formula CnH2nO2 acidic / basic
hydrolysis R C NH2 Amide
Carboxylic acid derivatives are compounds with functional O
groups that can be converted to carboxylic acids by a R C O R’ Ester
simple acidic or basic hydrolysis. O
Flowchart 24.1: Derivatives of carboxylic acid

2. CLASSIFICATION OF CARBOXYLIC ACIDS

Dicarboxylic Aromatic
acids carboxylic acids
- Structure contain two R = Aromatic ring
COOH grps.

Classification
of
Carboxylic acids
R-COOH

Hydroxy acids Amino acids


- R will contain - R = Contains NH2 group
(-OH) groups

Unsaturated
carboxylic acid
- R will contain double/
Triple bond

Flowchart 24.2: Classification of carboxylic acid


2 4 . 2 | Carboxylic Acid and Derivatives

3. NOMENCLATURE OF CARBOXYLIC ACIDS


Name Alkane Alkanoic acid
The IUPAC system of nomenclature assigns a characteristic suffix to these
classes. The ending “e” is removed from the name of the parent chain and Given the  1 location
is replaced –”oic” acid.
Position in numbering.

  O

   OH

Adjacent to C
O

Table 24.1: IUPAC name of simple carboxylic acid

Formula Common Name Source IUPAC Name Melting Point Boiling Point

HCOOH Formic acid Ants (L. Formica) Methanoic acid 8.4 ºC 101 ºC

CH3COOH Acetic acid Vinegar (L. Acetum) Ethanoic acid 16.6 ºC 118 ºC

CH3CH2COOH Propionic acid Milk (Gk. Protus prion) Propanoic acid -20.8 ºC 141 ºC

CH3(CH2)2COOH Butyric acid Butter (L. Butyrum) Butanoic acid -5.5 ºC 164 ºC

CH3(CH2)3COOH Valeric acid Valerian root Pentanoic acid -34.5 ºC 186 ºC

CH3(CH2)4COOH Caproic acid Goats (L. Caper) Hexanoic acid -4.0 ºC 205 ºC

CH3(CH2)5COOH Enanthic acid Vines (Gk. Oenanthe) Heptanoic acid -7.5 ºC 223 ºC

CH3(CH2)6COOH Caprylic acid Goats (L. Caper) Octanoic acid 16.3 ºC 239 ºC

CH3(CH2)7COOH Pelargonic acid Pelargonium (an herb) Nonanoic acid 12.0 ºC 253 ºC

CH3(CH2)8COOH Capric acid Goats (L. Caper) Decanoic acid 31.0 ºC 219 ºC

Example:

O O

OH CH3CH2C
OH
Butanoic acid Propanoic acid
(Butyric Acid)
(Propionic Acid)

O O

ClCH2CH2C
OH
OH
3-Chloropropanoic acid
2-Methylpentanoic acid (-Chloropropionic acid)
(-Methylvaleric acid)
Chem i str y | 24.3

Table 24.2: IUPAC naming of substituted carboxylic acid

(1) Naming carboxyl - Carboxyl group added to a ring. Example


groups attached to
- Suffix “ carboxylic acid” added to O
a ring
name of the cyclic compound
C
OH

Cyclopentanecarboxylic acid
O

C
OH

Br
Cis-2-Bromocyclohexanecarboxylic acid

(2) Naming - Carboxyl group attached to a cation O


carboxylates Acetate
Sodium
C
H3 C O Na
Cation “-ic acid”
Sodium ethanoate
replaced by
(Sodium Acetate)
“-ate” O

H3C C
CH2 O K

Potassium Propanoate
(Potassium propionate)

(3) Naming the acids i) Hydroxyl substituent –OH O


containing other
‘’ from hydroxyl is removed
functional groups OH
(-COOH = highest OH OH
priority )
2, 4- Dihydroxybutanoic acid

ii) Aldehyde / ketone substituent Name as “ OXO-”


O
O O
or
H OH

Aldehyde / Ketone O
2- Oxobutanoic acid

iii) Amino substituent – NH2 O

OH
NH2
2-Aminobutanoic acid
2 4 . 4 | Carboxylic Acid and Derivatives

iv) Alkene substituent O

OH
Butanoic acid
O
cis/trans or E/Z nomenclature for
the alkene needs to be included if OH
necessary. 3-Butenoic acid

Position
O

OH
Trans-3-pentenoic acid

OH

(E) -2-Methyl-2-butenoic
acid
(4) Di-carboxylic acids - Carboxyl groups occupy the ends of O O
the chain
OH
HO
Propanedioic acid

4.PHYSICAL PROPERTIES OF ACIDS AND ACID DERIVATIVES


(a) Physical appearance and odour
C1 – C3 = Liquid = Colourless & pungent smelling
C4 – C6 = Liquid (oily) = Colourless & unpleasant smell
C > C7 = waxy solids= Colourless
(b) Boiling oints: Refer the following Flow-chart for a better understanding.
(L.H.S.=Left Hand Side)
Reasons

Similar molecular weight Gives a stable hydrogen bonded


dimer
Alcohols O H O
O R C C R
O H O
R C OH
Aldehyde Possess a higher
b.p as compared
to L.H.S.
In alcohols, OH bond is less
Ketone
polar than OH of COOH due
to the presence of
C = O group.

Flowchart 24.3: Comparison of B.p of carboxylic acid with other carbonyl compound
Chem i str y | 24.5

(c) Melting Points:


(i) Carboxylic acid with more than 8 carbon atom and a double bond have low melting point due to the
inability to form a stable lattice.
Low m.p.
High m.p.
COOH
CH3 COOH
H 3C

Lie on
Lie on opposite side same side

(ii) Structures with even number of C atoms have a higher melting point as compared to structures having
odd number of C-atoms.
(iii) The high M.Pof primary and secondary amides is due to the strong hydrogen bonding and the presence
of electron-withdrawing and electron-donating group.

e - withdrawing
:

O:
C R/H
:

R N
e - donating
R/H

H-bo
O ndin
g
C 
H O
R N
C  H
H R N
H

(d) Solubility:
(i) Lower carboxylic acid ( C1 – C2  ) are - miscible with water acids
(ii) Instead of dimerization, they form H-bonds with water.
(iii) As the length of chain increases, the solubility decreases.
(iv) The derivatives like acid chlorides and anhydrides react with solvents like H2O & alcohol.

5. METHODS OF PREPARATION OF CARBOXYLIC ACIDS


General reaction of preparation of carboxylic acid

O
RMgX + C=O dry ether
+ R CH2 COOH
H3O
(CO2)
H+ or OH-
Grignard reagent
hydrolysis
carboxylation
R C  N+2H2O
Acid / Alkaline
hydrolysis of
nitriles
2 4 . 6 | Carboxylic Acid and Derivatives

(a) Synthesis of carboxylic acids by the carboxylation of Grignard reagents


:
:O O
- +

:
:
R MgX + C=O C =O

Dry ether

O MgX
R C =O : :

H3 O
OH X
R C= O + Mg
:

:
Carboxylic OH
acid

-RMgX acts as a nucleophile

Example:
1. Mg / diethyl ether
CH C HCH CH → CH C HCH CH
3| 2 3 2. CO 3| 2 3
2
Cl CO H
3. H O+ 2
3
2-Chlorobutane 2-Methylbutanoic acid (78 - 88%)


1.Mg/diethylether
2. CO2
CH3 3. H3O+ CH3
Br CO2H
9-Bromo-10-methylphenanthrene 10-Methylphenanthrene-8-carboxylic acid

(i) CO2
CH3 CH MgBr CH3 CH COOH
(ii) H2O/H+
CH3 CH3
Isobutyric acid (2-methyl propanoic acid)

(b) Synthesis of Carboxylic acids by the hydrolysis of nitriles Mechanism:


Hydrolysis of cyanides (Acid catalysed):
O
Heat
RCN +2H2O + H +
R C OH + NH+4
Nitrile Water Carboxylic Ammonium
acid ion

Mechanism:
(a) Formation of Amide
R-CN H
 
:

R-C=N-H
: :

O
H H
:

R-C=N-H
O
:

H  H
tautomerisation
R C NH2
O
:
Chem i str y | 24.7

(b) Formation of Acid



H H  H
:
:O:

:
R C NH2 O
H
 OH

:
R C NH2
:

:
OH

: :
:OH
H+ transfer 
R C NH3
:
OH

: :
:O

R C + NH4
OH

Example:
O
H2O
CH2Cl NaCN H2SO4
CH2CN CH2COH
DMSO heat

Benzyl chloride Benzyl cyanide (92%) Phenylacetic acid (77%)

PLANCESS CONCEPTS

•• Alkyl cyanides needed for the purpose can easily be prepared from the corresponding alkyl halides
with alcoholic KCN or NaCN.
R – Cl + KCN → R – C ≡ N + KCl
•• This reaction is used to ascend the series having one carbon atom more than the corresponding alkyl
halides which are prepared from alcohol on treating with phosphorus halide.
ROH + PX5 → R – X + POX3 + HX
•• This hydrolysis of alkyl cyanide provides a useful method to get carboxylic acid having one carbon
atom more than the original alkyl halide and alcohols.

By Oxidation of alkyl benzenes – aromatic acids are produced.

CH3 COOH CH2CH3

KMnO4/OH- KMnO4/OH-
+ +
(ii) H /H2O (ii) H /H2O

Toluene Benzoic acid Ethylbenzene


E.g
K2Cr2O7
CH3 CH2 + 6[O] HOOC COOH + 2H2O
H2SO4

p-Xylene Terephthalic acid

Vaibhav Krishnan (JEE 2009, AIR 22)


2 4 . 8 | Carboxylic Acid and Derivatives

6. CHEMICAL REACTIONS

6.1. Acidic Strength



Acidity of carboxylic acids : R C O H

: :
: :
R C O: + H
O O

Exists as

2 Equivalent cannonical strs.

O O
R C R C
O O

O
R C
O
Resonance Hybrid

Table 24.3: Comparative acidity of Carboxylic acid with electron withdrawing group vs Carboxylic acid with electron donating
group

R= Electron withdrawing Group R= Electron Donating Group


⇒ Shows -I effect ⇒ Shows +I effect

⇒ Stabilises anion & increases acidic nature ⇒ Destabilizes anion & decrease acidic nature

⇒ Example ⇒ Example

O O
R C R C
O O

Tips and Tricks


Acidity of acids is compared by comparing stability of conjugate base
Chem i str y | 24.9

6.2 Reactions Involving Removal of Proton From–OH Group

Removal of proton
from -OH Grp.
* Blue litmus turns red

R-COOH

R’MgX
Na2CO3/ ether
NaHCO3
NaOH
Na R’- H + RCOO MgX
RCOO Na + H2O (with Grignard reagent)
+ CO2
Brisk effervescence

RCOO Na + H2O
(with alkalis)
RCOONa + H2
(with metals)

Flowchart 24.4: n Reaction involving displacement of H from -OH group.

Important Point:
A stronger acid displaces a weaker acid from the salt of the weaker acid.
Example: CH3COOH (Strongeracid) + CH3ONa → CH3COONa + CH3 − OH
culsalenacid

Example: CH3COOH (Stronger acid) + NaHCO3 → CH3COONa + H2CO3 (Weaker acid) → H2O +CO2↑ (lab. test of
carboxylic acid)

6.3 Reactions Involving Replacement of –OH Group


Replacement
of
-OH Group
R C OH
P2O5 PCl5
O
R C O C R R C Cl
O O NH3 R’OH; O
Anhydride
R C NH2 R C OR’
O O
Amide Fischer
Esterification

Flowchart 24.5: Reaction Involving removal of OH group.

O O
:

G
R C OH R C G + OH

Strong base so not a basicity must be less


good leaving group than basicity of G‒
2 4 . 1 0 | Carboxylic Acid and Derivatives

(a) Formation of acid chlorides

O
PCl5
R C Cl + POCl3 + HCl
Pyridine
O
R-COOH PCl3
R C Cl + H3PO3
Pyridine
O
SOCl2 R C Cl + SO2 + HCl

O O
Example:
reflux
C + SOCl2 C + SO2 + HCl
OH OH
Benzoic acid Benzoyl chloride

Example: COOH COCl

heat

+ PCl5
O2N NO2 O2N NO2 + POCl3 + HCl

3, 5-Dinitrobenzoic 3, 5-Dinitrobenzoyl
acid chloride

(b) Fisher Esterification: Carboxylic acid react with alcohol to form esters through a condensation reaction
known as esterification.

General Reaction:
O O
H
R C OH + R’ OH R C OR’ + H2O

Specific Examples:

O O
H
CH3 C OH+CH3CH2 OH CH3 C OC2H5+H2O
O O
H
C6H5 C OH+CH3 OH C6H5 C OCH3 +H2O

Mechanism : (Acid catalysed esterification)


(i) A reactive electrophile is generated by addition of a proton or a Lewis acid.
(ii) A tetrahedral intermediate containing two equivalent hydroxyl groups is obtained by the nucleophilic
attack of the alcohol .
(iii) Elimination of these hydroxyl groups after a proton shift (tautomerism) occurs leading to the formation
of water and the ester.
Chem i str y | 24.11

:O: 
:O H


:
R C OH +H O H R C OH
H

:
:O H
R’ OH
R C OH
O Proton

:
R’  H Transfer


:
H O H

:
R C O H
:O R’

:
: :
R C O H
:O R’
:


:
:
+
R C O: + H O H (H3O )
OR’ H

The forward reactions give acid catalysed esterification of an acid while the reverse account for the acid catalysed
hydrolysis of an ester
Acid catalysed ester hydrolysis.
O O

H3 O
R C OR’ + H2O R C OH + R’ OH

For esterification of an acid we can use an excess of the alcohol and removal of water to prevent the reverse
reaction, Hydrolysis of an ester uses a large excess of water and refluxing the ester with dilute aqueous HCl or dilute
aqueous H2SO4 occurs.

(c) Formation of amides:


O O O
  150-200C
R C OH + NH3 R C ONH4 R C NH2 + H2O

In fact, amides cannot be prepared from carboxylic acids and amines unless the ammonium salt is heated
strongly to dehydrate it. This is not usually a good method of preparing amides.

(d) Formation of acid anhydride:


O
O RC
P2O5
2R C OH O+ H2O
RC
O

6.4 Decarboxylation Reactions

DECARBOXYLATION
REACTIONS
O
R C OH
Soda-lime
O H Ele H2 O Kolbe’s
decarboxylation
Na aO ctr electrolysis
oly
Na2CO3+ R- H C sis R R +CO2+H2
2 4 . 1 2 | Carboxylic Acid and Derivatives

(a) Soda-lime decarboxylation:


General reaction:
O
NaOH + CaO
R C OH R H + CO2
Mechanism
NaOH 
R C OH R C ONa
-H2O
O O
OH
CaO 
R C ONa

O


R Na + NaHCO3
Carbanion
RH + Na2CO3
Alkane

(i) The stability of carbanion intermediate decides the rate of reaction .


(ii) Rate of decarboxylation increases with the presence of electron withdrawing group at R-COOH.

Example: CH3
HCOOH CH3 COOH CH3 CH COOH CH3 C COOH
CH3 CH3

(iii) Presence of some functional groups on aliphatic acids enable the decarboxylation.


: :

O
:
: :

R C O O :O :
-CO2
:

CH2 C R C R C
:

: O: CH2 CH2
:

Resonance
stabilized anion
Aliphatic acids that do undergo successful decarboxylation have certain functional groups or double or triple
bonds in the α or β positions.

Table 24.4: Decarboxylation product of substituted carboxylic acid

Acid type Decarboxylation product


(1) Malonic | |
HOOC − C− COOH HOOC − C− H
| |

(2) α-Cyano | |
NC − C− COOH NC − C− H
| |

(3) α-Nitro | |
O2N − C− COOH O2N − C− H
| |

(4) α-Aryl | |
Ar − C− COOH Ar − C− H
| |
Chem i str y | 24.13

Acid type Decarboxylation product


(5) β-Keto | |
− C − C− COOH − C − C− H
|| | || |
O O

(6) α, α, α-Trihalo X3C–COOH X3C–H


(7) β, γ-Unsaturated | |
− C = C− C− COOH − C = C− C− H
| | | | | |

(b) Kolbe’s electrolysis


Electrolysis
2RCOOK + 2HOH  → R – R + 2CO2 + H2 + 2KOH

ANODE CATHODE

R C O: -e R C O:
: :

K +e K+H2O
O O RCOOK RCOO + K
R + CO2
+2e- 
2H2O 2OH + 2H

R
R -R H2

RCOOK + H2O

Electrolysis
Example: 2CH3 – COOK + 2H2O 
→ CH3CH3 +2CO2 + H2 + 2KOH

6.5 HVZ Reaction (Halogenation of Aliphatic Acids and Substituted Acids)


Converts a carboxylic acid possessing an α-hydrogen to an α-halocarboxylic acid when treated with phosphorus
and halogen. It is called as Hell-Volhard-Zelinsky reaction where regioselectivity allows alpha-halogenation only.
Cl ,P Cl ,P Cl ,P
2 → ClCH COOH 
CH3COOH  2
2 → Cl CHCOOH 
2
2 → Cl CCOOH
3

Mechanism
(a) Carbonyl oxygen reacts with phosphorus trihalide to form a P-O bond giving the release of a halide anion.
(b) Attack of halide forms an intermediate to release a rearranged acyl halide, an acid and a phosphine oxide.
(c) Enol tautomer of acyl halide attacks the halogen molecule to form α-halo acyl halide.
(d) On hydrolysis, α-halocarboxylic acid is formed.

RCHCOOH + large excess of NH3 RCHCOOH

Br NH2
An -halogenated acid An -amino acid
H
+

RCHCOOH + NaOH RCHCOONa RCHCOOH

Br OH OH
An -hydroxy acid
+
H
RCH2CHCOOH + KOH (alc) RCH =CHCOO- RCH=CHCOOH
An , -unsaturated
Br acid
2 4 . 1 4 | Carboxylic Acid and Derivatives

7. CARBOXYLIC ACID DERIVATIVES


Functional derivatives of carboxylic acids
-Acid chlorides
-Anhydrides
-Amides
-Esters
The above are compounds in which the replacement of –OH of a carboxyl group is done by –Cl,–COOR , –NH2,or
–OR.

Characteristic reaction of acid derivatives (Nucleophilic acyl substitution):

Nucleophilic acyl substitution


-An addition - elimination mechanism

Nu
R R
Nu
:

: :
C O: C O:
L L
Tetrahedral
carbon - SP3
- L, the substituent is
protonated, to serve
as a better leaving
group. Nu
R
: :

- L = Leaving Group O:
- Nu = Nucleophile L

- Weaker the base, better


will be the leaving group.
R
C=O +:L
Nu
- The Nu should be a
stronger base.
O
L = -X - O- C - R - OR - OH - NH2
[Order of Reactivity]

Examples:

(a) O O O O O
R C Cl + CH3 C O R C Cl R C O C CH3 + Cl
Stronger base
O C CH3
(pka of CH3COOH is 5)
weaker base
O
(pka of HCl = -7)
(b) O O

R C OH + NH2 R C NH2 +OH


Stonger base weaker base
(pka of NH2 is 35) (pka of HOH is 15)
Chem i str y | 24.15

(c) O O O

CH3 C O C CH3 + ROH CH3 C OR + CH3COO


pka = 16 pka = 5
(d) O O

C6H5 C OCH3 + NH3 C6H5 C NH2 + CH3O


pka = 35 pka = 16
(e) O O
C6H5 C NH2 + CH3OH C6H5 C OCH3 + NH2
pka = 16 pka = 35

PLANCESS CONCEPTS

Condition for nucleophilic substitution reaction:

O O

R C L + Nu R C Nu + L

• L must be better leaving group than Nu⊖, i.e., basicity of Nu should be more than that of L.
• Must be a strong enough nucleophile to attack RCOL.
• Carbonyl carbon must be enough electrophilic to react with
Nikhil Khandelwal (JEE 2009, AIR 94)

7.1 Acid Halides


(a) Methods of preparation Acyl halides

By-Products
PCl5 By-Products
PCl5 POCl3+HCl
SOCl2 POCl3+HCl
R - COOH R - COCl SO2 + HCl
SOCl2
Pyridine
R - COOH R - COCl SO
H 2 + HCl
Pyridine 3PO3
PCl3 H PO
O PCl3 O 3 3
CH3 O C OH + SOCl2 CH3 OC Cl + SO2 + HCl
CH3 C acid
E.g Ethanoic OH + SOCl2 CH3 C chloride
Ethanoyl Cl + SO2 + HCl
E.g Ethanoic acid Ethanoyl chloride

O O
C OH + SOCl2 C Cl + SO2 +HCl
Benzoic acid Benzoyl chloride
2 4 . 1 6 | Carboxylic Acid and Derivatives

(b) Chemical Reactions O


O O
R’ C OH
Py
R C O C R’+ HCl
Anhydride
O
R’ OH
R C O R’+ HCl
Py Ester
O
R C Cl NH3/R’NH2 O
R C NH2 + H2O + Cl
Base Amide (Substituted)
H2O O
R C OH + HCl
Acid
Flowchart 24.5: General reactions of Acid halides

(i) Reaction with carboxylic acids


Carboxylic acids with acyl chlorides yield acid anhydrides with the help of a weak organic base, pyridine.
Pyridine acts both as a catalyst and a base, which neutralizes the formed hydrogen chloride.
O O O O
Pyridine
CH3(CH2)5CCl + CH3(CH2)5COH CH3(CH2)5COC(CH2)5CH3
Heptanoyl Heptanoic Heptanoic anhydride
chloride acid (78 - 83%)

(ii) Reaction with alcohols


O O
Pyridine
C6H5CCl + (CH3)3COH C6H5COC(CH3)3

Benzoyl tert-Butyl tert-Butyl


chloride alcohol benzoate (80%)

(iii) Reaction with ammonia and amines


O O
NaOH
C6H5CCl + HN C6H5C-N
H2O

Benzoyl Piperidine N-Benzoylpiperidine


chloride (87-91%)

(iv) Hydrolysis
O O

C6H5CH2CCl + H2O C6H5CH2COH + HCl

Phenylacetyl Water Phenylacetic Hydrogen


chloride acid chloride

(v) Reaction of acid halide with organometallic


• With Grignard reagent
• Reaction with Gilmann reagent
(Grignard’s Reagent)
R’MgX O R’MgX
OMgX
O
R C R’ R C R’
R C Cl
R’ CuLi R’
(Gilmann’sReagent) H2O
O OH
R C R’ R C R’
Ketone
R’
Chem i str y | 24.17

(vi) Reduction of acid halides


• By LiAlH4
• By H2/Pd/BaSO4 (Rosenmund reduction)
O O
H2 LiAlH4
R C H R C Cl R CH2 OH
Pd/BaSO4
Rosenmund
reduction

7.2 Acid Amides

Methods of preparation of acid amide


(a) With ammonia
CH2 CH2
C C
O OH HO O
Succinic acid
O
R C O R’ - R’OH
Ester O
R C NH2
- NH4Cl NH3 Amide
O
R C Cl
Acid chloride
H4
OON 
- RC
O
R C
O O O
R C NH
Succinimide
O (82 - 83%)
Anhydride

Flowchart 24.6: Preparation of amides from acid derivative

(b) With (i) Cyanide (ii) Ammonia salt of acid

General reaction:
H2O2 O

R_ C  N NaOH
R C NH2 R C O NH4
Or
+ HCl O
H2O Ammonia
salt of acid

(i) From ammonia salts of carboxylic acid



CH3COONH4  → CH3CONH2

(ii) From cyanides


dil.H SO
2 4 → CH – CONH
CH3C ≡ N + H2O  3 2
2 4 . 1 8 | Carboxylic Acid and Derivatives

Example: O O

_
_

_
_
_ NH3 _
H C=C CNH
H C=C COCH
2 3 + 2 2 + CH3OH

_
_
CH3 Ammonia CH3 Methyl
Methyl-2-methylpropenoate 2-Methylpropenamide alcohol
(75%)

Amines, which are substituted derivatives of ammonia, react similarly

Example: O O
heat
FCH2COCH2CH3 + NH2 FCH2CNH + CH3CH2OH

Ethyl fluoroacetate Cyclohexylamine N-Cyclohexyl- Ethyl


fluroacetamide (61%) alcohol

Important Chemical Reactions

General reaction:

HOFMANN AMIDE
REARRANGEMENT HYDROLYSIS
General Reaction General Reaction
O H/R’ O
R _ N=C=O
:

NaOH + R C N + H2O R C OH
Isocyanate + H/R’ Acid
Amide
+ H/R’
Hydrolysis Br2
H N :
:

R-NH2 H/R’
Amine Amine

(a) Hoffmann rearrangement: In the Hofmann rearrangement an unsubstituted amide is treated with sodium
hydroxide and bromine to give a primary amine that has one carbon fewer than starting amide

General reaction:
O
Hydrolysis
R_C_ NH2 +NaOH + Br2 R_N C=O R NH2
isocyanate
Mechanism:
The mechanism is divided into 3 parts:-
(i) Formation of Isocyanate intermediate
(ii) Formation of carbamic acid
(iii) Decarboxylation to give Amine
Chem i str y | 24.19

(i) Formation of Isocyanate intermediate


:
:O:

:
O:

: :
H :OH
R N:

:
R N
H H

:
:O

:
R N:
H

Br - Br

:
:O
Br

:
:
R N :O
Br

: :
:OH

: :
H R N

:
:O:
-Br

:
R N

:
Migration of O:
-R to Nitrogen

:
N- R
Isocyanate
(ii) Formation of carbamic acid

:

:O :
:

O:
: :

:O - H
C
C
:

N- R
:

N- R HO
Isocyanate
:

HO - H O:
C
HO NH - R
Carbamic acid

(iii) Decarboxylation to give Amine



:

O:
:

O:
C
:
:

H O NH R C + R NH2
Amine
:

:O
(CO2)
2 4 . 2 0 | Carboxylic Acid and Derivatives

(b) Hydrolysis of amides


Example: O O
H2O/H2SO4 +
CH3CH2CHCNH2 CH3CH2CHCOH + NH4HSO4


Ammonium hydrogen
sulfate
2-Phenylbutanamide 2-Phenylbutanoic
acid (88-90%)

Example: O O
KOH
CH3CNH Br ethanol
CH3CO-K + + H2N Br
water, heat
Potassium
N-(4-Bromophenyl) acetamide acetate p-Bromoaniline (95%)
(p-bromoacetanilide)

7.3 Esters

(a) Methods of Preparation


ESTERS
O
H+
=

R-C-O-R’
R C OH PREPARATION R C OH
=
=

O O
+ +
R’-OH R’-OH

Examples
Examples:
H +
(i) CH3COOH + C2H5OH → CH3COOC2H5 + H2O
Acetic acid

H +
C6H5COOH + CH3OH → C6H5COOCH3 + H2O

Pyridine
(ii) CH3COCl + C2H5OH  → CH3COOC2H5 + HCl

Alcohols react with acyl chlorides by nucleophilic acyl substitution to yield esters. These reactions are typically
performed in the presence of a weak base such as pyridine.
Chem i str y | 24.21

Here conc. H2SO4 is used in the obove reactions which acts as a catalyst & a dehydeating agent.
O O
=

=
RCCl + R’OH + RCOR’ + Cl-
+ +
Acyl Alcohol N N
Ester
chloride H H
Pyridine Pyridinium
chloride

Example: O2N O2N


O O
Pyridine
CCl + (CH3)2CHCH2OH COCH2CH(CH3)2

Isobutyl
O2N alcohol O2N

3, 5-Dinitrobenzoyl Isobutyl
chloride 3, 5-dinitrobenzoate(85%)

NaOH
Example: C6H5COCl + CH3CH2OH   → C6H5COOCH2CH3 + HCl

(b) Chemical Reactions: The acid and base catalysed ester hydrolysis involves the acyl cleavage of the ester
group.
Acy cleavagel
O
R-C- O-R’

AAc2 is Acid catalysed ester hydrolysis by acyl cleavage through bimolecular mechanism.
BAc2 is Base catalysed ester hydrolysis by acyl cleavage through bimolecular mechanism.

Table 24.5: Mechanism of Acid catalysed and Base catalysed ester hydrolysis

Acid Catalysis Acyl Cleavage AAc2  


+H  slow 
:
: :

: :

R C O R’ R C O R’ R C O R’ R C
-R’OH
:O :OH :O H :O
:
:

:
slow 
R C OH2
H2 O
:

:O
:

-H 
R C OH
:

:OH
:

-H R C OH
:O
:

Base Catalysis Acyl Cleavage BAc2


:

:OH
: :

:OH
: :

: :

R C OR’ R C OR’ R C OH R C O
slow
: :

O: :O: :OR’ :O :O
:
:

+R’OH
2 4 . 2 2 | Carboxylic Acid and Derivatives

(i) Acid catalysed hydrolysis of ester (AAc2):


O
H
CH3 C O R + H2O CH3COOH + ROH

The yield of products would be raised by adding excess of water.


O
H
CH3 C OR + H2O (excess) CH3COOH + ROH

(ii) Base-Promoted Hydrolysis of Esters : Saponification (BAc2):


The base catalysed hydrolysis is also known as Saponification.
O O
H2 O
RC OR’ + NaOH RC O-Na+ + R’OH
Ester Sodium Alcohol
carboxylate

The unreactive negatively charged carboxylate ion does not undergo nucleophilic substitution. The
irreversible nature of this reaction,i.e., the base-promoted hydrolysis of an ester is seen over here. The
mechanism for this reaction also involves a nucleophilic addition-elimination at the acyl carbon.

Mechanism:
H3C-CH2 H3C-CH2
H2 O 18
C=O+NaOH C=O+H3C-CH2-O -H
Ethanol containing
O Na
18
H3C-H2C-O18 labelled O

The mechanism is studied with the help of isotopically labelled esters. Ethyl propionate consisting of
labelled 18O in the ether-type oxygen of the ester undergoes hydrolysis with aqueous NaOH wherein the
18
O is observed to be contained in the produced ethanol only.
The result would have been different if it was an alkyl cleavage instead of the acyl one. But attack at the
alkyl is not possible.
Such kind of attack of the nucleophile at the alkyl carbon occurs rarely in the case of carboxylic acid
esters, but such attacks are preferred in case of esters of sulfonic acids (e.g. tosylates and mesylates)

H3C-CH2 H3C-CH2
H2 O
C=O+OH C=O+H3C-CH2-OH

H3C-H2C-O18 O18

O-
18
H3C-CH2-C=O
Cannot be formed
O CH3 O H3 C
H3C S O C H3 C S O + C OH
O H
CH2CH3 O H3CH2C
H
Inversion of
configuration

Alkyl attack is seen in cases of alkyl sulfonates.


Chem i str y | 24.23

7.4 Acid anhydrides


O O
R C O C R’
Anhydride

PREPARATION
CHEMICAL
REACTIONS

From With aromatic


carboxylic compound
acids
With alcohols

From With NH3 & RNH2


acid &
acid halide Hydrolysis

Flowchart 24.6: Preparation and reactions of anhydride

(a) Method of Preparation

(i) From carboxylic acids


O O
R C R C
OH  O
+
P2O5 R C
OH
R C O
O ANHYDRIDE

Example:
O
C
(i) CH2 COOH  CH2
O
CH2 COOH P2O5 CH2
C
O
Succinic
anhydride

O
CH2 COOH O
(ii) CH2

P2O5
CH2 COOH O

(ii) From acid and acid halide


O O
R C R C
OH Pyridine O
+
R’ C
Cl
R’ C O
O
2 4 . 2 4 | Carboxylic Acid and Derivatives

O
O
Example: CH3 C
CH3 C  OH
O +
H3 C C Cl
CH3 C
O
O
 Pyridine
O O
CH3 C C CH3
Cl+NaO

(b) Chemical Reaction


Friedel-Crafts
O O Acylation O O
H2 O ArH
R’ C OH +R C OH dry ether R C Ar + R’ C OH
Acid Ketone
- Acid anhydrides give
two carboxylic acids REACTIONS
- Cyclic anhydrides O
yield dicarboxylic
acids R C
O
R C
O

O 2NH3 O O
or R”OH
R C NHR” R C O R” + R’ C OH
2R”NH2
Amide Ester

Flowchart 24.7: Important reaction of anhydride

(i) Reaction with aromatic compounds (Friedel-Crafts acylation)

F F
Example: O O O
Dry, AlCl3
CH2COCCH3 + OCH3 CH3C OCH3 + CH3COOH
Acid Acetic
anhydride o-Fluoroanisole 3-Fluro-4-methoxyacetophenone acid
(70-80%)
(ii) Reaction with alcohols: Esters are formed when acid anhydrides react with alcohols in presence of
pyridine or a catalytic acid.
The below given example shows the incorporation of only one acyl group in the ester while the other
forms an acetic acid molecule.

Example: O O O
H2SO4
CH3COCCH3 + HOCHCH2CH3 CH3COCHCH2CH3 + CH3COOH
Acid Acetic
CH3 CH3 acid
anhydride
sec-Butyl sec-Butyl
alcohol acetate (60%)
Chem i str y | 24.25

(iii) Reaction with ammonia and amines: Amides are formed when acid anhydride reacts with 2 molar
equivalents of ammonia or amines.
The below given example shows the incorporation of only one acyl group into the amide and the other
forms the amine salt of acetic acid.

O O O
CH3COCCH3 + H2N CH(CH3)2 CH3CNH CH(CH3)2

Acetic p-Isopropylaniline p-Isopropylacetanilide (98%)


anhydride

(iv) Hydrolysis: Carboxylic acids are formed when acid anhydrides react with water. Cyclic anhydrides
hydrolyse to dicarboxylic acids.
O
O
COH
O + H2O
Water COH
O
O
Phthalic anhydride
Phthalic acid

8. HEATING EFFECTS

O
O
R C OH
+ R C
- H2O
R C OH O Anhydride
R C
O
Monocarboxylic acid O

COOH - CO2
CH2 CH3 COOH Acetic acid
COOH
Dicarboxylic acid

Table 24.8: Heating effects on hydroxy acids & ester

(1) a - Hydroxy acid O O O


OH HO  O
or O
O H HO - H2O O O
O O
O
(2) b - Hydroxy acid  O O
 
C H2 CH2 C OH CH2 = CH C OH
-H2O
OH Unstability of 4/8 - membered rings lead to
the formation of a,b - unsaturated acids

(3) g - Hydroxy acid O O




C H2 CH2 CH2 C OH O
-H2O
OH
2 4 . 2 6 | Carboxylic Acid and Derivatives

(4) d - Hydroxy acid O


  O
  
C H2 CH2 CH2 CH2 C OH O
-H2O
OH
(5) Ester 
R C OH
R C O CH2CH2R
O O
+
R C O CH2 CH R R CH = CH2
O H
Intermediate step.

POINTS TO REMEMBER

(a) Summary of Reactions of Carboxylic Acids


Na Metal R-CH2-COONa + 1/2H2
NaOH R-CH2COONa + H2O
Na. metal
based reaction NaHCO3
R-CH2-COONa+CO2 + H2O
NaOH(CaO)
R-CH2-Na2CO3

Grignard CH3MgBr
R-CH2-COOMgBr+ CH4
Reaction

SOCl2
RCH2COCl + SO2
PCl3
R-CH2COCl
Halogenation
Reaction (i)P+X2 (ii)H2O
R-CH-COOH
O
AgOH;Br2 X
R-CH2-C R-CH2-Br + CO2
OH
NH3,
R-CH2C-NH2
Multiple O
P2O5,
Functional R-CH2C-O-C-CH2-R
group O O
R’OH/H2SO4
R-CH2C-OR’
O
(i)SOCl2(ii)CH3N2
R-CH2-CH2-COOH
(iii)Ag2O (iv) H2O
Chain
Lengthening Ca(OH)2 O
Reaction Dry distillation R-CH2-C-CH2-COOH

NaOH,electrolysis
R-CH2-CH2-R
Chem i str y | 24.27

(b) Summary of Reactions of Acid Halides

Reaction of Acid Chloride

H2O
RCOOH + HCl (Hydrolysis)
R’OH RCOOR’ + HCl (Alcoholysis)
2NH3
RCONH2 + NH4Cl (Ammonolysis)
Multiple
R’NH2
Functional RCONHR’ + R’NH+Cl-
3

group C6H5/Anhyd,AlCl3
C6H5COR+HCl(Friedal -Craft reaction)
R2’Cd/Ether
2RCOR’ + CdCl2
LiAlH4/Ether
RCH2OH

RCOOH/Pyridine
(ROC)2O + HCl
O Anhydride
R-C-Cl Formation
RCOONa
(ROC)2O + NaCl

H2/Pd-BaSO4
RCHO + HCl
+S or quinoline
Reduction
Reaction
LiAlH4/Ether
RCH2OH
2 4 . 2 8 | Carboxylic Acid and Derivatives

(c) Summary of Reactions of Amides

Reaction of amide

+
H /H2O
RCOOH + NH4
Acid Forming
Reaction
SOCl2
RC  N + SO2 + HCl

NaOH
RCOONa + NH3
Na-metal
Based Reaction
2Na
RCONHNa + 1/2H2

O Hydrochloride
Conc. HCl
R-C- NH2 Formation RCONH2HCl
Reaction

P2O5
3RC  N + 2N3PO4
Cyanide
Forming
reaction SOCl2
RC  N + SO2 + Hcl

Hoffmann Br2 + 4KOH


Bromamide RNH2 + KCO2 + 2KBr + H2O
reaction

Reduction LiAlH4/dry ether RCH2NH2


reaction 1 Amie
Chem i str y | 24.29

(d) Summary of Reactions of Esters

Reaction of Esters
+
H /H2O
RCOOH + R’OH (Hydrolysis)
Multiple
NaOH/H2O
Functional RCOONa + R’OH (Saponification)
group
+
R”OH/H
RCOR” + R’OH (Trans-esterification)
R”ONa

NH3
RCONH2” + R’OH (Ammonolysis)
Amide
Formation
R’NH2
RCOHNR” + R’OH
O
R-C-OR
H2/Copper chromite
RCH2OH + R’OH
Alcohol or LiAlH4

Forming
Reaction Na/alcohol
RCH2OH + R’OH (Bouveault-Blanc reduction)

R”

Grignard (i) 2R’MgX R-C-OH (In case of esters of formic acid,


Reaction
+
(ii) H /H2O 2 alcohols are combined)
R”
3”Alcohol

Solved Examples

JEE Main/Boards (A) (I) decarboxylates faster than (II) on heating.


(B) Only *CO2 is eliminated on heating of compound(I).
Example 1: Select the correct statement about the
(C) Compound (I) eliminates a mixture of CO2 and *CO2
following compounds I, II, II.
on heating.
COONa COONa (D) The rate of decarboxylation of (II) is faster than (III).
CH2 CH2
*
COOH
*
COONa Sol 1: (A) Nature of functional group also has an
(I) (II) influence on rate of decarboxylation. Presence of
Electron Withdrawing Group-Increases its rate of
CH3 C CH2 COONa decarboxylation.
O
(III)
2 4 . 3 0 | Carboxylic Acid and Derivatives

COONa COONa O O
CH2 CH2
* COOH * COONa O NH
(A) (B)
(I) (II) NH NH
CH3 C CH2 COONa O O
O
O O
(III)
NH NH
CO2 (C) (D)
NH
rate of decarboxylation : III > I > II
No decarboxylation
rate of decarboxylation : III > I > III O O
Example 2: Which of these represents correct reaction ?
Sol 3: (B)
(A) H C O Conc.NaOD
DCOO- +DCH2OD COOH COCl
O

NH2NH2 NH
O SOCl2
NH
COOH COCl
(B) CH3 C H+H C H NaOH
C(CH2OH)4+HCOO O

O O
Example 4: Identify (A), (B), (C) and (D).
P +Br2
(C) CH3 − CH3 − C − OH 
→ CH3 − C H − COOH Mg/dry ether (i) CO2 [O]
|| | C3H5Cl (A) (B) (C) C8H12 (D)
O Br
Saturated (ii)H2O/H+
H H
18 18
(D) CH3 C OH + CH3 CH2 C OH Conc.H2SO4 CH3 CH2 C O C CH3
Sol: First step is preparation of gringnard reagent
C2H5 O O C2H5
P+Br2 Second is reaction of G. R. with CO2 to form an acid
CH3 CH3 C OH CH3 CH COOH

Sol 2: (A, B, C, D) H C O
Conc.NaOD
DCOO- +DCH2OD Cannizzaro
O Br
(A) = Cl; (B) =
reaction MgCl; (C) = COOH;(D) = CH
O
Cannizzaro reaction (A) = Cl; (B) = MgCl; (C) = COOH;(D) = CH=CH

(B) CH3 C H+H C H NaOH C(CH2OH)4 +HCOO-


O O Example 5: Give the reaction of preparation of
(excess) propanoic acid from ethyl alcohol.
Aldol + Cannizzaro reaction
Sol: CH3 CH2OH 5 PCl KCN
(Haloalkane formation)
CH3 CH2 Cl Nucleophilic substitu
P+Br2
(C) CH3 CH3 C OH CH3 CH COOH
PCl KCN
O CH3 CH2OH (Haloalkane
Br 5

formation)
CH3 CH 2 Cl Nucleophilic substitution CH3 CH2 CN
HVZ reaction
H H H2O/H+
18 18 CH3 CH2 COOH
(D) CH3 C OH + CH3 CH2 C OH Conc.H2SO4
CH3 CH2 C O C CH3 hydrolysis
Propanoic acid
C2H5 O O C2H5
(Esterification reaction)
Example 6: Identify (A), (B) and (C).

Example 3:Final product is :


+
KCN H2O/H 
C3H6Cl2 (A) (B) (C) -CO2
2-Methylpropanoic acid
COOH
SOCl2 NH2NH2 2-Methylpropanoic acid

COOH Sol: First step is Nocleophilic substitution (CN-) followed


Phthalic acid by Hydrolysis. (Both Cl is replaced by CN)
Chem i str y | 24.31

It produces diacarboxylic acid which on mono ‒NO2, ‒Cl‒ Electron withdrawing group thus rate of
decarboxylation produces 2-methyl propanoic acid. decarboxylation increases
Cl CN COOH -CH3, -OCH3 Electron donating group and hence rate

KCN H2O/H  decreases.
CH3 C CH3 CH3 C CH3 CH3 C CH3 2-Methylpropanoic acid
Hydrolysis -CO2
Cl CN COOH
Example 3: Identify (A), (B) and (C).
N COOH
Br (1 eqv ) / P
→ ( A )  → (B ) 
→ (C)
2 KCN 2 H O/H'/ ∆
H2O/H  CH3 − CH2 − COOH 
CH3 CH3 C CH3 2-Methylpropanoic acid
Hydrolysis -CO2
Br (1 eqv ) / P
N 2
COOHCH3 − CH2 − COOH 
→ A  2
→ B 
→ C ( ) KCN
( ) H O/H'/ ∆
( )

Sol 3: (A) CH3 − C H − COOH (B) CH3 − C H − COOH


JEE Advanced/Boards |
Br
|
CN

Example 1: Predict A, B, C, D and E. (C) CH3–CH(COOH)2

 Mesitylene/AlCl3
Acid(A) B C + CH3COOH Example 4: Write the structures of (A) C3H7NO which
Zn-Hg/Conc. HCl on acid hydrolysis gives acid (B) and amine (C). Acid (B)
(D)
gives (+)ve silver-mirror test.
Sol 1: (A) = CH3COOH;
Sol: Since it gives positive silver mirror Test, It has to be
(B)= CH3 − C − O − C − CH3 an aldehyde (-CHO)
|| ||
O O C3H7NO-CHO=C2H6N,
CH3 Now C2H6N can be either (CH3)2N or CH3CH2-NH group.
COCH3 Thus A can be.
(c) = O O
H3C CH3 A = H C NH C2H5 or H C N CH3

CH3 CH3
CH2CH3 Example 5: Which are correct against property
(D) = mentioned?

H3C CH3 (A) CH3COCl > (CH3CO)2O > CH3COOEt > CH3CONH2
 (Rate of hydrolysis)

Example 2: Find the rate of soda-lime decarboxylation. CH3


(B) CH3 CH2 COOH > CH2 CH COOH > CH3 COOH (Rate of est
COOH COOH COOH COOH COOH
CH3 CH3

(Rate of esterification)
OH OH OH
NO2 Cl CH3 OCH3
(C) > > (Rate of este
I II III IV V
ON2
Sol 2: Rate of soda-lime decarboxylation. I > II > III > (Rate of esterification)
IV > V O O
Presence of Electron withdrawing group Increases the (D) CH3 C COOH > CH3 C CH2 COOH > Ph CH2 COOH (Rate of deca
rate of decarboxylation.
(Rate of decarboxylation)
Presence of Eelectron donating group. decreases the
rate of decarboxylation. Sol 5: (A, B) Self explanatory
2 4 . 3 2 | Carboxylic Acid and Derivatives

Example 6: Match the product of column II with the reaction of column I.

Column I Column II
(A) (p) Ester with O18

HOOC COOH
18
OH
(B) (q) A β-diketone with –18OH group
18 
COOH
OH O
(C) 18 (r) A cyclic anhydride with –18OH group
OH
COOH


(D) O (s) A cyclic ester without O18


18 OH /
HOOC O
O

Sol: A → r; B → s; C → p; D → q

Self explanatory

JEE Main/Boards

Exercise 1 Q.4 Esterification does not take place in the presence


of ethyl alcohol and excess of concentrated H2SO4 at
Q.1 Two isomeric carboxylic acids H and I, C9H8O2, 170°C. Explain.
react with H2/Pd giving compounds C9H10O2. H gives
a resolvable product and I gives a non-resolvable Q.5 Why does carboxylic acid functions as bases though
product. Both isomers can be oxidized to C6H5COOH. weak ones?

Give the structure of H and I.


Q.6 Which ketone of the formula C5H10O will yield an
acid on halo form reaction?
Q.2 Identify the products (A), (B), (C) and (D) in the
following sequence: Q.7 Highly branched carboxylic acids are less acidic
than unbranched acids. Why?
LiAlH4 HCl (i) Mg, ether
C15H31COOH (A) (B)(ii) O
Q.8 A carboxylic acid does not form an oxime or phenyl
(C) hydrazone. Why?
KMnO4 + conc. H2SO4
(D)
Q.9 Formic acid reduce Tollen’s reagent. Why?
Q.3 A neutral liquid (Y) has the molecular formula
C6H12O6. On hydrolysis it yields an acid (A) and an alcohol Q.10 The K2 for fumaric acid is greater than maleic acid.
(B). Compound (A) has a neutralization equivalent of Why.
60. Alcohol (B) is not oxidized by acidified KMnO4, but
gives cloudiness immediately with Lucas reagent. What Q.11 Identify the final product in the following
are (Y), (A) and (B) ? sequence of reaction.
Chem i str y | 24.33

O Q.22 Two mole of an ester (A) are condensed in


H2C-CH2 H3O+ presence of sodium ethoxide to give a β-ketoester (B)
CH3 CH2 MgBr X Y and ethanol. On heating in an acidic solution(B) gives
Z
KMnO4 ethanol and β-ketoacid(C). On decarboxylation (C) gives
3-pentanone. Identify (A), (B) and (C) with reactions.
Q.12 What is (Z) in the following sequence of reactions ?

HgSO Q.23 Compound(A)(C6H12O2) on reaction with LiAlH4


HCCH (i)(ii)2NaNH 2
(X) H SO 4 (Y)
2CH3I 2 4 yields two compounds (B) and (C). The compound (B)
(i)NaOH/Br2 on oxidation gave (D) 2 moles of (D) on treatment with
(Z) (ii)H3O- alkali (aqueous) and subsequent heating furnished
(E). The later on catalytic hydrogenation gave (C). The
Q.13 Acetic acid has a molecular weight of 120 in compound (D) was oxidized further to give (F) which
benzene solution why ? was found to be a monobasic acid (m.wt.60.0). Deduce
structures of (A) to (E).
Q.14 Place the following in the correct order of acidity
Q.24 Compound (A) C5H8 O2 liberated CO2 on reaction
(i) CH≡C–COOH; (ii) CH2=CH–COOH;
with sodium bicarbonate. It exists in two forms neither of
(iii) CH3CH2COOH which is optically active. It yielded compound (B). C5H10O2
on hydrogenation. Compound (B) can be separated into
Q.15 Phenol is a weaker acid than acetic acid why? enantimorphs. Write structures of (A) and (B).

Q.16 Which acid derivative show most vigorous alkaline Q.25 The sodium salt of a carboxylic acid, (A) was
hydrolysis ? produced by passing a gas (B) into aqueous solution of
caustic alkali at an elevated temperature and pressure
(A) on heating in presence of sodium hydroxide
Q.17 59 g of amide obtained from the carboxylic acid followed by treatment with sulphuric acid gave a
RCOOH, on heating with alkali gave 17g of ammonia. dibasic acid (C). A sample of 0.4g of (C) on combustion
What is the formula of acid ? gave 0.08 g of H2O and 0.39 g of CO2. The silver salt of
the acid, weighing 1.0 g, on ignition yielded 0.71 g of
Q.18 Which carboxylic acid (X) of equivalent mass of Ag as residue. Identify (A), (B) and (C).
52g / eq loses CO2 when heated to give an acid (Y) of
equivalent mass of 60g/eq. Q.26 An organic compound (A) on treatment with
acetic acid in presence of sulphuric acid produces an
Q.19 Which of the reagent reacts with C6H5CH2CONH2 ester (B). (A) on mild oxidation gives (C). (C) with 50%
to form C6H5CH2CN. KOH followed by acidification with dilute HCl generates
(A) and (D). (D) with PCl5 followed by reaction with
Q.20 Consider the following ester - ammonia gives (E). (E) on dehydration produces
hydrocyanic acid. Identify (A) to (E).
(i) MeCH2COOH (ii) Me2CHCOOH
(iii) Me3CCOOH (iv) Et3CCOOH Q.27 Acetophenone on reaction with hydroxylamine-
Correct order of the rate of esterification hydrochloride can produce two isomeric oximes. Write
structures of the oximes.

Q.21 An organic compound (A) on treatment with


ethyl alcohol gives a carboxylic acid (B) and compound Q.28 An acidic compound (A), C4H8O loses its optical
(C). Hydrolysis of (C) under acidic conditions gives activity on strong heating yielding (B). C4H6O2 which
(B) and (D). Oxidation of (D) with KMnO4 also gives reacts readily with KMnO4. (B) forms a derivative (C) with
(B). (B) on heating with Ca(OH)2 gives (E) (Molecular SOCl2, which on reaction with (CH3)2NH gives (D). The
formula C3H6O) (E) does not gives Tollen’s test compound (A) on oxidation with dilute chromic acid
and does not reduce Fehling solution but forms 2, gives an unstable compound (E) which decarboxylates
4-dinitrophenylhydrazone. Identify (A) to (E). readily to give (F), C3H6O. The compound (F) gives
a hydrocarbon (G) on treatment with amalgamated
Zn and HCl. Give structures of (A) to (G) with proper
reasoning.
2 4 . 3 4 | Carboxylic Acid and Derivatives

Q.29 An organic acid (A), C5H10O2 reacts with Br2 in the COOH
presence of phosphorus to give (B). Compound (B) Q.4 H3C 
X, X will be
contains an asymmetric carbon atom and yields (C) on OH
dehydrobromination. Compound (C) does not show O
geometric Isomerism and on decarboxylation gives
COOH H3C
an alkene (D) which on ozonolysis gives (E) and (F). (A) H3C (B) O
Compound (E) gives a positive Schiff’s test but (F) does
not. Give structures of (A) to (F) with reasons.
CH3
O
Q.30 An liquid (X) having molecular formula C6 H 12O2 (C) O (D) None
is hydrolysed with water in presence of an acid to give O
a carboxylic acid (Y) and an alcohol (Z). Oxidation of (Z) O CH3
with chromic acid gives (Y). What are (X), (Y) and (Z) ?

O
Exercise 2 || 18
dil.H SO
Q.5 Me3C − C − O− CMe3 
2 4 → Product

Single Correct Choice Type


of this reaction and the mechanism is :
O
Q.1 EtO H3O Zn(Hg)
(X) || 18
N
 HCl
((A)
A ) Me3C − C − OH + Me3C − OH, A 1
AC
CH2 CO2Et O18
||
Product (X) of above reaction is : ((B)
B ) Me3C − C − OH + Me3C − OH, A
AL1
O
(A) (B) || 18

N N ((C)
C ) Me3C − C − OH + Me3C − OH, A 2
AC

CO2H O
|| 18
((D)
D ) Me3C − C − OH + Me3C − OH, A
AL2
(C) (D)
N N
Q.6 Guess the product
CH3
5→? PCl
CH3CH2CONH2 

Q.2 Correct order of reactivity of following acid
(A) CH3CH2–CN (B) CH3CH2COCl
derivatives is
(C) CH3CCl2CONH2 (D) CH3CH2CONHCl
(I) MeCOCl (II) MeCON3
(III) MeCOOCOMe
Q.7 End product due to hydrolysis of (A) and subsequent
(A) I > II > III (B) II > I > III heating is :
(C) I > III > II (D) II > III > I O
CH3O COC2H5
NaOEt H3O+
Q.3 CH2 (COOEt)2 + (CH2 )3 
EtOH
→ I → IIis : CH3O COC2H5
O
O
COOH COOH
(A) (B) (A) O (B) O COH
O
O
COOH CH3O COH
COOH (C) COH (D) O
(C) (D) CH3O COH
COOH COOH
O
Chem i str y | 24.35

O
||
Q.11 Which of the following pair will form same

Q.8 Me − C− O − CH2 + CH2 − NH3 
→ Q ,Q is?
NaOH osazone when it reacts phenyl hydrazine

O CHO CHO
H C OH H C OH
(A) Me C O CH2 CH2 NH2
H C OH HO C H
O and
(A) H C OH H C OH
(B) Me C O CH2 CH2 OH
H C OH H C OH
O O CH2OH CH2OH
(C)
N
D-allose D-glucose

H CHO CHO
(D) MeCOONa + HOCH2CH2NH2 H C OH HO C H
HO C OH HO C H
Q.9 Which of the following give two alcohols when it and
reacts with LiAlH4. (B) H C OH H C OH
O H C OH H C OH
(A) CH3 C O CH3 CH2OH CH2OH

O O D-glucose D-mannose

(B) CH3 C O C CH2 CH3 CHO CHO


H C OH H C OH
O
HO C H H C OH
(C) CH3 CH C O CH2 CH3 and
(C) H C OH HO C H
CH3
H C OH H C OH
(D) All
CH2OH CH2OH
Q.10 In which of the following reaction CO2 gas will be D-glucose D-gulose
evolved.
CHO CHO
CO2H H C OH H C OH
(A) CO2H  H C OH HO C H
and
(D) H C OH HO C H
O O
H C OH H C OH
(B) Ph C CH2 C OH 
CH2OH CH2OH
D-allose D-glucose
CO2H
Q.12
(C) NaOH CO3

O
||
(D) All HCl
Ph − C − O − H + HO18 CH(CH3 )2  →(X)

Major product (X) is :

O O18
|| ||
(A) Ph − C − O18 − (B) Ph − C − O −

O
||
(C) Ph − C − O − (D) Ph–O–
2 4 . 3 6 | Carboxylic Acid and Derivatives

Q.13 O
Q.16 O C CH3
(i) I2 + OH Br2 + H2O
AlCl3
A (ii) H O+ B C
2

‘C’ form white precipitate compound ‘C’ is:


Select incorrect statement OH OH
(A) P can turn blue litmus red
(A) (B)
(B) P can not give effervescence of CO2 with NaOH3. Br Br
(C) It is Dieckmann condensation Br Br
O
(D) Product is a bicylo compound OH
Br O C CH3
Br
(C) (D) Br
CO2Et
 Br
Q.14 EtO
A H3O B
Zn(Hq)
C, C is C is Br
Br
HCl
N
Q.17 Which of the following esters cannot undergo self
CH2 CO2Et Claisen condensation
(A) CH3CH2CH2CH2CO2C2H5
(B) C6H5CO2C2H5
(A) (B)
N N (C) C6H5CH2CO2C2H5

COOH CH2OH (D) CH3CH2CO2C2H5

CO2H
18 conc.H2SO4
Q.18 + OH P
(C) (D) COOH
N N
CH3
18
(A) C O (B) C O
O
P2O5 (i) CH MgBr O O
C NH2 A (ii) H3 O+ B
Q.15 3

(i) I2 + Ca(OH)2 (C) C O (D) C O


18
C (ii) 
O O
Product is:
Q.19 Method to distinguish RNH2 & R2NH
O O (A) NaNO2/HCl
(A) C CH3 (B) C OH (B) Hoffmann’s mustard oil reaction
(C) Hinsberg test

C C CH2CH3 (D) All of the above


(C) (D)
O O
Chem i str y | 24.37

Previous Years’ Questions Q.7 Statement-I: Acetic acid does not undergo
haloform reaction.
Q.1 When propionic acid is treated with aqueous Statement-II: Acetic acid has no alpha hydrogen.
sodium bicarbonate, CO2 is liberated. The C of CO2  (1998)
comes from  (1999)
(A) Methyl group (B) Carboxylic acid group Q.8 Statement-I: p-hydroxybenzoic acid has a lower
boiling point than o-hydroxybenzoic acid.
(C) Methylene group (D) Bicarbonate group
Statement-II: o-hydroxybenzoic acid has intramolecular
hydrogen bonding.  (2007)
Q.2 Benzoyl chloride is prepared from benzoic acid by
 (2000)
Q.9 Hydrolysis of an ester in presence of a dilute acid is
(A) Cl2 ,hν (B) SO2Cl2 (C) SOCl2 (D) Cl2, H2O known as saponification.  (1983)

Q.3 The product of acid hydrolysis of P and Q can be Q.10 The boiling point of propanoic acid is less than
distinguished by  (2003) that of n-butyl alcohol, an alcohol of comparable
molecular weight.  (1991)
OCOCH3 H3C
P = H2C = ,Q=
CH3 Q.11 A liquid was mixed with ethanol and a drop of
OCOCH3
concentrated H2SO4 was added. A compound with a
(A) Lucas reagent (B) 2,4-DNP fruity smell was formed. The liquid was:  (2009)

(C) Fehling’s solution (D) NaHSO3 (A) CH3OH (B) HCHO


(C) CH3COCH3 (D) CH3COOH
CH MgBr
Q.4 Ethyl ester 
3 → P, the product ‘P’ will be 
( excess ) Q.12 Sodium ethoxide has reacted with ethanoyl
 (2003)
chloride. The compound that is produced in the above
H3C CH3 H3C C2H5
(A) (B) reaction is:  (2011)
H3C OH H5C2 OH (A) 2-Butanone (B) Ethyl chloride

H5C2
(C) Ethyl ethanoate (D) Diethyl ether
H5C2 C2H5 C2H5
(C) (D)
H5C2 OH H7C3 OH Q.13 The strongest acid amongst the following
compounds is:  (2011)
(A) HCOOH
Q.5 An enantiomericaily pure acid is treated with
racemic mixture of an alcohol having one chiral carbon. (B) CH3CH2CH(Cl)CO2H
The ester formed will be  (2003) (C) ClCH2CH2CH2COOH
(A) Optically active mixture (D) CH3COOH
(B) Pure enantiomer
(C) Meso compound Q.14 Which of the following reagents may be used to
distinguish between phenol and benzoic acid?  (2011)
(D) Racemic mixture
(A) Tollen’s reagent (B) Molisch reagent
Q.6 Benzamide on treatment with POCl3 gives : (2004) (C) Neutral FeCl3 (D) Aqueous NaOH
(A) Aniline (B) Benzonitrile
Q.15 A compound with molecular mass 180 is acylated
(C) Chlorobenzene (D) Benzyl amine with CH3COCl to get a compound with molecular mass
390. The number of amino groups present per molecule
of the former compound is:  (2013)
(A) 2 (B) 5 (C) 4 (D) 6
COOH COCH3
(A) (B)
2 4 . 3 8 | Carboxylic Acid and Derivatives
COOH3
Q.16 Compound (A), C8H9Br, gives a white precipitate
OH OCOCH3
when warmed with alcoholic AgNO3. Oxidation of (A)
gives an acid (B), C8H6O4. (B) easily forms anhydride on COOCH3
heating. Identify the compound (A). (2013)
CH2Br
(C) (D)
COOH
CH2Br
CH2Br C2H5 CH2Br
(A) CH2Br
CH2Br
(B) C2H5 (C) (D)
(A) (B) (C) (D)
Br Q.19 In the reaction,
CH3
Br CH3
CH3 CH3 CH COOH  LiAIH4 PCl5 Alc.KOH
CH3 CH3 3 → A  → B  → C,
CH2Br CH2Br
Br C2H5 C2H5 CH2Br CH2Br the product C is  (2014)
(B) (C) (C) (D) (D) (A) Acetaldehyde (B) Acetylene
Br Br CH3 CH3
(C) Ethylene (D) Acetyl chloride
CH3 CH3

Q.20 In the following sequence of reactions:


Q.17 An organic compound A upon reacting with NH3
KMnO
4 → A  SICl
2 → B 
2 H /Pd
gives B. On heating B gives C. C in presence of KOH Toluene  BaSO
→C
4
reacts with Br2 to given CH3CH2NH2. A is:  (2013)
the product C is:  (2015)
(A) CH3COOH (B) CH3CH2CH2COOH
(A) C6H5COOH (B) (B) C6H5CH3
(C) CH3 − CH − COOH (D) CH3CH2COOH (C) (C) C6H5CH2OH (D) C6H5CHO
|
CH
3 Q.21 In the Hofmann bromamide degradation reaction,
the number of moles of NaOH and Br2 used per mole of
Q.18 Sodium phenoxide when heated with CO2 under
amine produced are: (2016)
pressure at 125°C yields a product which on acetylation
produces C. (A) Four moles of NaOH and two moles of Br2
ONa 125
o
H+ (B) Two moles of NaOH and two moles of Br2
+ CO2 B C
5 Atm Ac2O
(C) Four moles of NaOH and one mole of Br2

The major product C would be (2014) (D) One mole of NaOH and one mole of Br2

OCOCH3 OH

COOH COCH3
(A) (B)

COOH3

OH OCOCH3

COOCH3
(C) (D)
COOH
Chem i str y | 24.39

JEE Advanced/Boards

Exercise 1 Q.9 An organic compound ‘A’ on treatment with ethyl


alcohol gives a carboxylic acid ‘B’ and compound ‘C’
Q.1 (i) Give the structures of the four optically-active Hydrolysis of ‘C’ under acidic conditions gives ‘B’ and
isomers of C4H8O3 (D through G) that evolve CO2 with ‘D’ Oxidation of ‘D’ with KMnO4 also gives ‘B’ . ‘B’ on
aq. NaHCO3. heating with Ca(OH)2 gives ‘E’(molecular formula
C3H6O). ‘E’ doesnot give Tollen’s test and does not
(ii) Find the structure of (D), the isomer that reacts with reduce Fehling’s solution but forms a 2,4-dinitrophenyl
LiAlH4 to give an achiral product. hydrazine. Identify ‘A. B’ C’ ‘D’ and ‘E’.
(iii) Give chemical reactions to distinguish among (E),
(F) and (G). Q.10 Two moles of an ester (A) are condensed in the
presence of sodium ethoxide to give a β–keto ester
Q.2 Complete the following equation: (B) and ethanol. On heating in an acidic solution (B)
gives ethanol and β–keto acid (C). On decarboxylation
CH3 (C) gives 3–pentanone. Identify (A), (B) and (C) with
HCl Mg CO2 H2O/H+
proper reasoning. Name the reaction involved in the
H3C C =CH2
Peroxide
? Ether ? ? ? conversion of (A) to (B).

Q.3 Give structures of compounds: Q.11 An alkali salt of palmitic acid is known as?
CO2
Acetylene +CH3MgBr (G) (H)
-CH4 Q.12 Acid do not react with sodium bisulphite though
(C3H4O3) (J)
H2O,H2SO4
(C3H2O2)(I)
H2 O O
HgSO4 ||
KMnO4 they have − C − group. Why ?
CH2(COOH)2

Q.4 An ester C6H12O2 was hydrolysed with water an acid Q.13 In the reaction sequence
(A), and an alcohol (B), were obtained. Oxidation of (B) Ca(OH)2 Dry Conc.
with chromic acid produced A. What is the structure of X Y
distillation
Acetone
H2SO4
Z
the original ester? Write equations for all the reactions.
X, Y and Z are ?

Q.5 Complete the following equation: O 


Q.14 CH3CH2COOH →
 
SeO
X, Product X is–
SOCl2 NaN2 D Hydrolysis 2
RCO2H ? ? ? ?
(inert solvent)
Q.15 Which of the reagent attack only the carbonyl
Q.6 Acid halides of formic acid are unstable. Why? group of a fatty acid?

Q.7 What is the product of the following reaction? Q.16 In the sequence

H CHO CH CH CH
(i) Silver oxide in aq. base ||| | 3 | 3
X Y Z
C=C
(ii) H
? CH  → CHO  → COOH  → CH4
H3 C CH3
The reagent X, Y and Z are:

2-Methyl-2-pentenal
Q.17 In the reaction sequence
Q.8 An unsaturated acid (A) of molecular formula C5H6O4
eliminates CO2 easily and gives another unsaturated X
H3O
Y
NH3
Z Br2
CH3NH2
acid (B) of formula C4H6O2. By saturation with H2/Pt (B)
 KOH

gives butanoic acid. Neither (A) nor (B) shows cis-trans X, Y, and Z are?
isomerism. What are (A) and (B)?
2 4 . 4 0 | Carboxylic Acid and Derivatives

Q.18 An acid X react with PCl5 to form a compound (Y). Q.24 An acidic compound (A), C4H8O3 loses its optical
X also react with NaOH to form a compound (Z). Both Y activity on strong heating yielding (B), C4H6O which
and Z react together and from (E), E react with a reagent reacts readily with KMnO4. (B) forms a derivative
(F) to give back compound (Y) what are X, F , Z,E and F? (C) with SOCl2, which on reaction with (CH3)2NH
gives (D). The compound (A) on oxidation with
Q.19 How will you synthesise? dilute chromic acid gives an unstable compound (E)
which decarboxylates readily to give (F), C3H6O. The
(i) Acetyl chloride from methyl chloride compound (F) gives a hydrocarbon (G) on treatment
(ii) Acetamide from ethyl alcohol with amalgamated Zn and HCl. Give structures of (A)
to (G) with proper reasoning.
(iii) Ethyl acetate from acetic acid
Q.25 A pleasant smelling optically active ester
Q.20 Complete the following reaction? (F) has M.W = 186. It does not react with Br2 in
(i) CH O O CCl4 Hydrolysis of (F) gives two optically active
compounds, (G) soluble in NaOH and (H). (H) gives
CH 3
CH33 O
O O
O
O Y
NH
NH
O
a positive iodoform test and on warming with cone.
X + O Y
Y O= NH + Z
X
X++ O O=
O= NH +
+ZZ
CH
O
O
O
NH
NH O H2SO4 gives (I) (Saytzeff-product) with no geometrical
CH 3
O O
CH33 O O isomers. (H) on treatment with benzene sulfonyl
NH
NH 2
NH22 PCl3
chloride gives (J), which on treatment with NaBr gives
(ii)
O=C
O=C
O=C
+
+
+E
E PCl
E
PCl33 F + C2H5OH
FF +
+CC22H
H55OH
OH
optically active (K). When the Ag+ salt of (G) is treated
NH
NH 2 with Br2 racemic (K) is formed. Give structures of (F) to
NH22
(K) and explain your choices.
NH
NH 2
NH22 Urea
(iii) X + O=C Y Z Urea Resin
Urea
X
X++ O=C
O=C Y
Y Z
Z Resin
Resin Q.26 Compound (A), M.F C6H12O2 reduces ammoniacal
NH
NH silver nitrate to metallic silver and loses its optical
2
NH22
activity on strong heating yielding (B), C6H10O which
readily reacts with dilute KMnO4. (A) on oxidation
Q.21 Complete the following equations:
with KMnO4 gives (C) having M.F C6H10O3 which
CN- H2O/H
+
decarboxylates readily on heating to 3–pentanone.
(i) CH3CH2CH2CH2Br ? ?
The compound (A) can be synthesized from a carbonyl
CH3 compound having M.F. C3H6O on treatment with
(ii) H3C C Br +CN-
Cold conc. dilute NaOH. Oxidation of (B) with ammonical silver
? ?
H2SO4
nitrate followed by acidification gives (D). (D) forms
CH3 HOH a derivative (E) with SOCl, which on reaction with
H3CNHCH2CH3 yields (F). Identify (A) to (F) giving
(i)CO2 Mg SOCl2
? (ii) H2 ? Ether ? ? proper reaction sequences. What is the name of the
reaction involved in the conversion of C3H6O to (A)?
Give the IUPAC nomenclature of compounds (A) to (F).
Q.22 Identify the compounds:
Q.27 A solid organic compound (A), C9H6 O2 is insoluble
1, 4-Cyclohexadiene t-BuOK (D)
in dilute NaHCO3. It produces a dibromoderivative
+ CHBr3 (C7H8Br2) (B), C9H6O2Br2 on treatment with Br2/CS2. Prolonged
KMnO4 boiling of (A) with concentrated KOH solution followed
(E) by acidification gives a compound (C), C9H8O3. The
H2
(C7H8Br2O4) compound (C) gives effervescence with aqueous
(F) Ni NaHCO3 Treatment of (C) with equimolar amount of
(C7H10O4)
Me2SO4/NaOH gives (D), C10H10O3. The compound
(D) is identical with the compound prepared from
Q.23 Compound (A) C5H8O2 liberated carbon dioxide on
o-methoxy benzaldehyde by condensation with
reaction with sodium bicarbonate. It exists in two forms
acetic anhydride in the presence of sodium acetate.
neither of which is optically active. It yields compound
Treatment of(C) with alkaline C6H5SO2Cl produces (E)
(B) C5H10O2 on hydrogenation. Compound (B) can be
which on vigorous oxidation with KMnO4 gives (F).
separated into two enantiomorphs. Write the structural
Hydrolysis of(F) gives a steam volatile compound (G)
formulae of (A) and (B) giving reason.
having M.F. C7H6O3. Give the structures of(A) to (G)
giving the proper reaction sequences.
Chem i str y | 24.41

Q.28 A neutral compound (A) C9H16O2on refluxing with CO2Me


dilute alkali followed by acidification yields (B) C5H8O2 (A)
and (C) C4H10O. (B) liberates CO2 from bicarbonate CO2Me
solution. (C) on dehydration yields 2–butene as the CH2 CO2Me
major product. B on treatment with OsO4 followed by (B)
reactive hydrolysis gives (D) C5H10O4 (D) when treated
CH2 CO2Me
with lead tetraacetate furrnishes acetone and (E) C2H2O3.
(E) is acidic and reduces Tollen’s reagent. Identify (A), CO2Me
(B), (C), (D) and (E) and write the reactions involved.
(C)
CO2Me
Q.29 An organic compound A on treatment with
ethyl alcohol gives a carboxylic acid B and compound CH2 CO2Me
C. Hydrolysis of C under acidic conditions gives (D)
B and D. Oxidation of D with KMnO4 also gives B. CH2 CO2Me
The compound B on heating with Ca(OH)2 gives E
(molecular formula C3H6O). E does not give Tollen’s test
and does not reduce Fehling’s solution but forms a 2,4– NH
SOCl2
dinitrophenyihydrazone. Identify A, B, C, D and E. Q.3 4-Pentenoic acid (X) (Y)
(i) LAH
(ii) H3O
Q.30 An aqueous alcoholic solution of acetoacetic (Z) (iii)dil OH-
ester imparts a blue colour with a solution of FeCl3.
Identify final (major) product :
To this solution if bromine solution is added carefully, O
the initial colour disappears and the brown colour of
bromine appears, which fades soon and the solution (A) CH2=CH-CH2-CH2-C-N
after remaining colourless for some time regains the
blue violet colour. Explain.
(B) CH2=CH-CH2-CH2-CH2-N
Exercise 2 OH

Single Correct Choice Type (C) CH2=CH-CH2-CH2-CH-N

Q.1 Find the reagent used to bring about following (D) CH3–CH2–CH2–CH2–CH2–OH
conversions.
H H
OH O O O O
|| ||

OH O Q.4 H2N − C − CH2 − CH2 − C − N3  → P; P is :
H H O
O
(A) ClCOCH2–CH2 COCl O
H N N H
(B) CH3COOCOCH3 (A) (B) H N NH
O
N H N
(C) CH3 COCl O
H
(D) ClCOCOCl O
O
O
NH
(C) (D) NH
C Cl
(i) CH2N2 (4 mole) N O HN
Q.2 (A), O
(ii) Ag2O; /MeOH H
C Cl
O
Product (A) of reaction is ?
2 4 . 4 2 | Carboxylic Acid and Derivatives

O (B) Statement-I is true, Statement-II is true and Statement-


II is NOT the correct explanation for Statement-I.
C Cl
Q.5
NaN3
(P)
Ag2O
 (Q) (C) Statement-I is true, Statement-II is false.
H2 O (D) Statement-I is false, Statement-II is true.
NaNO2
(S) +HCl
(R)
Comprehension Type
Identify (S) major product:
Paragraph 1: Ozonolysis of a compound Agathene
OH O dicarboxylic acid gives following compounds:
(A) (B)
COOH
O
HCHO, CHO &
NH2 C Cl O
(C) (D) COOH
O
On complete reaction by Na-EtOH Agathene
dicarboxylic acid give hydrocarbon C20H38 which have 5
chiral carbon in it.
Multiple Correct Choice Type

Q.6 Which will elimination CO2 only on heating Q.9 The structure of Agathene dicarboxylic acid is:
COOH
(A) Me C CH2 COOH
O (A) CH2
(B) Ph C CH2 SO2H COOH
O
OH COOH
(C) COOH
(B) COOH
CH2
(D) CH2=CH–CH2–COOH

CH COOH
Q.7 Methanoic acid and Ethanoic acid can be
differentiated by : (C) CH2
CH COOH
(A) Fehling test (B) Iodoform test
(C) Schiff’s test (D) NaHCO3 test
CH COOH

Assertion Reasoning Type (D) CH COOH


CH2
COOH
Me Et
Q.10 How many chiral carbon are present in Agathene
Q.8 Statement-I: COOH is optically inactive, it
dicarboxylic acid:
is taken in a glass container and plane polarized light
(PPL) is passed through it after heating it for several (A) 2 (B) 3 (C) 4 (D) 5
minutes. The PPL shows significant optical rotation.
Statement-II: Like β-keto acid, gem dicarboxylic acid Q.11 Total stereoisomers possible for Agathene
eliminates CO2 on heating. dicarboxylic acid are :

(A) Statement-I is true, Statement-II is true and (A) 16 (B) 18 (C) 32 (D) 64
Statement-II is correct explanation for Statement-I.
Chem i str y | 24.43

Q.12 Structure of product formed when Agathene Paragraph 3:


dicarboxylic acid is heated with soda lime is :
O
COOH
H3O KOH
H C O CH=CH A ?
(A) CH2 (B) CH2 2 B
MnO
CH2 CH2 Blue Litmus

KOH
CH2 Red C ?


(C) CH2 (D) CH2


Q.16 Mechanism of formation of A and B is
CH2 CH2
(A) A (B) A
AC2 AC1

(C) A (D) A
AL1 AL2
Paragraph 2:
18 Q.17 Select true statement:
OMe
(A) Both B and C give same name reaction with KOH
(i) O3
A+B (B) Both B and C give iodoform test
(ii) H2O/Zn OH-/
+
H /H2O (C) Both B and C give chiral product with PhMgCl
E followed by NH4Cl
C+D +
H /
(D) Both B and C are redox reaction
F
Q.18 Best method out of the given to prepare B is
Q.13 Product C and D are : O
O O (A) H C Cl
MeMgBr
|| ||
(A) H − C − C − OH + MeOH O
Me2Cd
O O (B) H C Cl
|| || 18
(B) H − C − C − OH + MeOH O
MeMgCl
O O (C) H C OMe
|| || 18
(C) H − C − C − OH + MeOH O
(D) H C OMe Me2Cd
O O
|| || 18
(D) HO − C − C − OH + MeOH

Q.14 Mechanism for hydrolysis of A will be:

(A) A (B) A (C) A (D) A


AC2 AL1 AC1 AL2

Q.15 F is

(A) H − C − CH− C − C H (B) H − C − C − OH


|| | || || || ||
O OH O O O O
O
O
(C) O (D)
C O
O
O
2 4 . 4 4 | Carboxylic Acid and Derivatives

Match the Columns

Q.22 Match reactions given in column I with Names in column II.

Column I Column II
(A) COOEt (p) Knoevenagel reaction

EtONa

COOEt

(B) (q) Perkin reaction


CH2(COOEt)2 + (i) EtOK

(ii) H3O ,

(C) Br (r) Reformatsky reaction


O
(i) Zn
O (ii) H3O

(D) O (s) Dieckmann’s condensation


MeO
MeOK
OEt
O

Q.23

Column I Column II (Product Differentiate By)

(A) (p) By Haloform test


CH3 =
− CH CH2 
Zn ( ) ( )
3→ U + V O

(B) (q) By Fehling test


CH
| 3
→(W) + (X)
O3
CH3−
= C CH − CH3 
Zn

(C) O (r) By aq. NaHCO3


||
H3O⊕ (s) By Tollen Test
Ph − C − O − Ph  → (Y) + (Z)

Q.24

Column I (Reactions) Column II (Types of Reaction)


(A) CH3–CH=CH2 + HCl → (p) Regioselective
(B) CN (q) Stereoselective

+ 

CN

(C) HCl
(r) Stereospecific
D CH2

HCl
(s) Diastereomers
CMe2 CH2
(D) (t) Cyclic addition
H
Chem i str y | 24.45

Q.25

Column I Column II
(A) CH3 C H (i) Al(OEt)3,
Products (p) One of the organic product formed will

(ii) H3O decolourise bromine water
O
(B) COOMe (q) One of the organic product formed will give
(i) MeO brisk effervescence with NaHCO3
C H + CH2  Product(s)
(ii) H3O /
O COOMe

(C) (i) CO2 (r) One of the organic product formed will give
PhMgCl  Products haloform test.
(ii) H
(iii) SOCl2
(iv) MeMgCl (s) One of the organic product formed will give 2,
4 DNP test

Q.26

Column I (Reaction) Column II (Product obtained by reaction)


(A) (i) LAH (p) R’–CH3
R C OR’
(ii) H2O
O
(B) (i) LAH (q) R’–OH
R’ C OH
(ii) H2O
O
(C) LAH (r) R’–CH2–OH
R '− CH2 − Br  →

(D) R’ C H SBH/EtOH (s) R’–H

O
(E) Red P/HI (t) R–CH3
R C OR’
O

Previous Years’ Questions COOCH3 COOH


(A) (B)
Q.1 When benzene sulphonic acid and p–nitrophenol COOH CHO
are treated with NaHCO3, the gases released respectively, CHO COOH
are  (2006) (C) (D)
(A) SO2, NO2 (B) SO2, NO CHO COOH
(C) SO2, CO2 (D) CO2, CO2
Q.3 In the following reaction sequence, the correct
structures of E, F and G are
Q.2 Which of the following reactants on reaction
with conc. NaOH followed by acidification gives the O O
following lactone as the only product? (2006) Heat
[E]NaOH
I
* OH 2
[F] + [G]
O 13
(* implies C labelled carbon)
(* implies 13C labelled carbon) (2008)
O
2 4 . 4 6 | Carboxylic Acid and Derivatives

O O Q.5 How can the conversion of (i) to (ii) be brought


(A) E= F=  about ?
Ph * CH3 Ph * ONa
(A) KBr (B) KBr + CH3ONa
G=CHI3 (C) KBr + KOH (D) Br2 + KOH
O O
(B) E= F=
*  Q.6 Which is the determining step in Hofmann
Ph Ph ONa
CH3 bromamide degradation ?
G=CHI3 (A) Formation of (i) (B) Formation of (ii)
O O (C) Formation of (iii) (D) Formation of (iv)
(C) E= F= 
Ph * Ph ONa
CH3
Q.7 What are the constituent amine formed when the
* mixture of (1) and (2) undergoes Hofmann bromamide
G=CHI3
degradation ?
O O
(D) E= F= 
15
* CONH2 CONH2
Ph CH3 Ph ONa
* D (1) (2)
G=CH3I
Q.4 Reaction of RCONH2 with a mixture of Br2 and KOH
15
gives R–NH2 as the main product. The intermediates (A) NH2, NH2,
involved in this reaction are : (1992)
D D
(A) RCONHBr (B) RNHBr
(C) R–N=C=O (D) RCONBr2
15
(B) NH2, NH2
Comprehension: RCONH2 is converted into RNH2 by
means of Hofmann bromamide degradation. D
15
(C) NH2, NH2
O O
Cl
NH2 NHBr 15
(i) (ii) (D) NHD,
O
C O Q.8 (±) 2-Phenylpropanoic acid on treatement with (+)
N Cl Cl 2-butanol gives (A) and (B). Deduce their structures and
N- Br
: :

also establish stereochemical relation between them.


(iv) (iii)
 (2003)

-+
OM Q.9 Compound A of molecular formula C9H7O2Cl exists
O in keto from and predominantly in enolic form B. On
N Cl NH2 Cl oxidation with KMnO4, A gives m-chlorobenzoic acid.
H Identify A and B. (2003)
(v) (vi)
In this reaction, RCONHBr is formed from which this Q.10 Cl
reaction has derived its name. Electron donating group
at phenyl activates the reaction. Hofmann degradation KCN
(A)
C2H5ONa/EtOH
(B)

reaction is an intramolecular reaction. 
DMF C6H5CHO
(2006)
H3O+/ SOCl2
(C) (D)
heat CH3NH2
Identify A to D.  (2004)
Chem i str y | 24.47

Paragraph 1 (Questions 11 to 12) Q.14 The total number of carboxylic acid groups in the
product P is:  (2013)
In the following sequence, products I, J and L are
formed. K represents
1. Mg/ether
a reagent. O
O
1. NaBH4 2.CO2 K
1. Mg/ether
Hex-3-ynal I J 2.CO Me K +
2. PBr3 1. NaBH
+ 1. H3O ,
Hex-3-ynal3. H3O 4
I 2
J Me Cl O P
2. PBr3
3. H3O+ Cl
2. O3
3. H2O2
O
Me O
Me
H2 OO
Cl L H2
Pd/BaSO4 Cl L
quinoline Pd/BaSO4
O
quinoline
Paragraph 2 (Questions 15 to 16)
O
P and Q are isomers of dicarboxylic acid C4H4O4. Both
Q.11 The structures of compounds J and K respectively decolorize Br2/H2O. On heating, P forms the cyclic
are  (2008) anhydride.

Me COOH
COOH Upon treatment with dilute alkaline KMnO4, P as well as
(A)
(A) Me and
and SOCl
SOCl22 Q could produce one or more than one from S, T and U.
COOH COOH COOH
OH
OH
(B) Me
(B) Me and SOCl
and SOCl22 H OH H OH HO H
O
O H OH HO H H OH
(C) Me
(C) Me and SOCl
and SOCl22
COOH COOH COOH COOH
COOH
S T U
(D) Me COOH
COOH
(D) Me
and
and CH
CH33SO Cl
SO22Cl
Q.15 Compounds formed from P and Q are, respectively
 (2013)
Q.12 The structure of product L is:  (2008)
(A) Optically active S and optically active pair (T, U)

(A) Me CHO (B) Optically inactive S and optically inactive pair (T, U)
(B) Me CHO
(C) Optically active pair (T, U) and optically active S
Me CHO
(A)
CHO
(B) Me (D) Optically inactive pair
CHO (T, U) and optically inactive S
(B) Me CHO
CHO (D) Me CHO
Q.16 In the following reaction sequences V and W are,
CHO (C) Me
CHO respectively (2013)
(B) Me CHO (D) Me CHO
CHO (C) Me 2 H /Ni
(D) Me CHO Q 

→V

CHO (D) Me CHO AlCl3 (anhydrous) 1. Zn-Hg/HCl


+V W
2. H3PO4

Q.13 The carboxyl functional group (–COOH) is present O


in  (2012) CH2OH
(A) Picric acid (B) Barbituric acid (A) O and (B) and

(C) Ascorbic acid (D) Aspirin CH2OH


O W O V W
V
O
CH
O 2OH
and HOH2C
(A) O and (B)
(D) and
(C) O OH
CH and
2
CH2OH
O W O V W
V V W
V O W

O
HOH C
(A) O and (B) and

CH2OH
2 4 . 4 8 | OCarboxylic Acid
W Oand Derivatives
V W
V

CH
O2OH Q.18 Different possible thermal decomposition
(B) and HOH2C pathways for peroxyesters are shown below. Match
(C) CH
O2OH
and
(D) and each pathway from list I with an appropriate structure
V W CH2OH from list II and CH
select
2OH
the correct answer using the code
W O
V given
W below the lists. (2014)
V O W

P R + R’O
HOH2C -CO2
(D) and
Q
CH2OH CH2OH O R + R’O R + X + carbonyl compound
O -CO2
V W R O R’
W R RCO2 + R’O R + X + carbonyl compound
(Peroxyester)
-CO2
Q.17 In the reaction shown below, the major product(s) S
RCO2 + R’O R + R’ O
formed is/are (2014) -CO2

NH2 List I List II


(i) Pathway P (p) O
Acetic anhydride
products(s) O
NH2
CH3NH2 C6H5CH2 O CH3

(ii) Pathway (q) O


O Q
O
H C6H5 O CH3
NH2
N CH3 (iii) Pathway R (r) O
H O CH3
O + CH3COOH O
(A) (B) N CH3 C6H5CH2 CH3
+ CH3COOH CH2C6
NH2
O O
(iv) Pathway S (s) O
O
H NH2 O CH3
H - - C6H5 O
-

NNH2 CH3 NH CH COO


3 3 CH3
N CH3 C6H5
H
+ CH3COOH
O
H Code: NH CH3
(A) O + CH3COOH (B)
H + CH3COOH (i) (ii)
N (iii) CH
(iv)
(C) NNH2 CH (D)
(B) + 3H2O 3
N CH3 O O
(A) p r s q
O
O O O O
O O (B) q s r p
H NH - -
-

2
(C) NH
s 3CH
p3 COO
q r
- -
-

N CH3
NH3CH3COO
H (D) r q p s
+ CH3COOH
O H
(B) N
H CH3
(C)
H + H2O (D) Q.19 The number
N ofCH
hydroxyl
3
group(s) in Q is  (2015)
(D) N
O N O CH3
CH3
O O H+ aqueous dilute KMnO4 (excess)
O O H heat
P 0oc
Q
O O
- -
-

NH3CH3COO HO
H3C CH3

H
(D) N CH3

O O
Chem i str y | 24.49

Q.20 Among the following, the number of reaction(s) Q.22 The correct order of acidity for the following
that produce(s) benzaldehyde is  (2015) compounds is  (2013)
CO2H CO2H CO2H CO2H
CO, HCl
CO, HCl HO OH OH
(A)
(A) Anhydrous,
Anhydrous, AlCl
AlCl3/CuCl
/CuCl 3
OH
CHCl OH
CHCl22
H
H22O
O (I) (II) (III) (IV)
(B)
(B) o
100occ
100
(A) I > II > III > IV (B) III > I > II > IV
COCl
COCl (C) III > IV > II > I (D) I > III > IV > II
H
H22
(C)
(C) Pd-BaSO
Pd-BaSO4 4
Q.23 Reagent(s) which can be used to bring about the
CO
CO22Me
Me
following transformation is(are) (2016)
DIBAl-H
DIBAl-H
(D)
(D) o
Toluene,
Toluene, -78
-78oCC
H2O
H2
O O O O O
C C
Q.21 The major product U in the following reactions is
 (2015) H OH
O O
CH2=CH-CH3,H+ Radical initiator, O2 COOH COOH
T U
High pressure, heat
(A) LiAlH4 in (C2H5)2O (B) BH3 in THF
H
(C) NaBH4 in C2H5OH (D) Raney Ni/H2 in THF
O H3C CH3
O O
(A) (B) Q.24
O The Hcorrect acidity order of the following is
CH3
 (2009)
OH OH COOH COOH
H

O H3C CH3
O O
(A) H (B) O H
H3C CH3 CH
3
O O CH2
(B) O O H
O Cl H CH3
H3
(C) (D) O
CH3 (I) (II) (III) (IV)
H3C CH3
H (A) (III) > (IV) > (II) > (I) (B) (IV) > (III) > (I) > (II)
O
CH2
H3
(B) OO H (C) (III) > (II) > (I) > (IV) (D) (II) > (III) > (IV) > (I)
O O H
(C) CH2 (D) O
CH3
O H Q.25 Amongst
H3C CH3
the following, the total number of
H3
(D) O compounds
N soluble inCOOH
aqueous NaOH is OCH
:  CH(2010) OH
2 3

H3C CH3
CH OH
CH2 N COOH OCH2CH3 2 OH

O H CH2OH
(D) O
H3

NO2 OH CH2CH3 C
CH2CH3
NO2 OH CH2CH3 C
CH2CH3

N
H3 C CH3
N
H3C CH3
CH2OH

2 4 . 5 0 | Carboxylic Acid and Derivatives

HOH
2CH3
CH2CH3
OH COOH

CH2OH CH2CH3

N
CH3

2CH3 Q.26 The compounds


COOH P, Q and S  (2010)
CH2CH3 O

COOH OCH3 C
O

H3C H3C
P P S
were separately subjected to nitration using HNO3/H2SO4 mixture. The major product formed in each case
respectively, is
O

COOH OCH3 C
O
(A)

HO H3C NO2

NO2 NO2

O
COOH OCH3
C
(B) O
HO NO2 H3C
NO2
NO2

O NO2

COOH OCH3 C
O
(C)

HO H3C NO2

NO2 O

COOH OCH3 C
O NO2
(D)
H3C NO2
HO
NO2

Q.27  (2010)

O
(1) NaOH/Br2
H3C C T
O
NH2 (2) C
O
Chem i str y | 24.51

O-
H3C C O- O
H3C C O
O C NH
(A) O C (B) NH
(A) (B) C C
C C
O
O

O
H3C O O
H3C NH C O
H3C NH
(C) H3C NH (D) NH C
(C) C (D)
C
O
O

Q.28 The major product of the following reaction is Q.29 Among the following compounds, the most acidic
 (2011) is  (2011)
O (A) p-nitrophenol
C
(i) KOH (B) p-hydroxybenzoic acid
CH
(C) o-hydroxybenzoic acid
C
(ii) Br CH2Cl
O (D) p-toluic acid

O O
Q.30 The compound that undergoes decarboxylation
C most readily
C under mild condition is  (2011)
(A) N CH2 Br (B) N CH2Cl
COOH COOH
C C
O O CH2COOH O
O (A) O (B)
O
C C
C O O
(A) N CH2 Br (B) N CH2Cl
Br (B) C
C N CH2Cl C
C CH2COOH
COOH
C N
O N
O COOH
(C) (D) O
C O C
(C) (D)
O
O CH2 Br O
O CH2Cl
O
O
C C
CC N Q.31 With reference to the scheme given, which of the
N
Br (C)
(B) CH2Cl (D) given statements about T,U,V and W is correct?(2012)
(D) CN C
CC O
O CH2 Br O CH2Cl
Br O CH2Cl O

O CH3
C LiAlH4
-
-

N CrO /H
3 U (CH3CO)2O
(D) V W
C
(A) T is soluble in hot aq. NaOH
Br O CH2Cl
(B) U is optically active
(C Molecular formula of W is C10H18O4
(D) V gives effervescence on treatment with aq. NaHCO3
2 4 . 5 2 | Carboxylic Acid and Derivatives

PlancEssential Questions
JEE Main/Boards JEE Advanced/Boards
Exercise 1 Exercise 1
Q.2 Q.8 Q.17 Q.2 Q.8 Q.18
Q.21 Q.25 Q.29 Q.22 Q.25 Q.30

Exercise 2 Exercise 2
Q.1 Q.4 Q.7 Q.4 Q.6 Q.9
Q.13 Q.14 Q.18

Previous Years’ Questions Previous Years’ Questions


Q.1 Q.10 Q.12 Q.2 Q.8 Q.14
Q.17 Q.21 Q.21 Q.25 Q.31

Answer Key

JEE Main/Boards

Exercise 2
Q.1 B Q.2 A Q.3 D Q.4 C Q.5 B Q.6 A
Q.7 B Q.8 B Q.9 D Q.10 D Q.11 B Q.12 A
Q.13 B Q.14 C Q.15 C Q.16 C Q.17 B Q.18 B
Q.19 D

Previous Years’ Questions


Q.1 D Q.2 C Q.3 C Q.4 A Q.5 D Q.6 B
Q.7 C Q.8 D Q.9 False Q.10 False Q.11 D Q.12 C
Q.13 B Q.14 C Q.15 C Q.15 C Q.16 D Q.17 D
Q.18 A Q.19 C Q.20 D Q.21 C
Chem i str y | 24.53

JEE Advanced/Boards
Exercise 2
Single Correct Choice Type

Q.1 D Q.2 B Q.3 B Q.4 C Q.5 B

Multiple Correct Choice Type


Q.6 A, C, D Q.7 A, C

Assertion Reasoning Type

Q.8 D

Comprehension Type

Q.9 A Q.10 C Q.11 C Q.12A Q.13 C Q.14 A

Q.15 D Q.16 A Q.17 D Q.18 B

Match the Columns

Q.19 A → s; B → p; C → r; D → s Q.20 A → p; B → q, s; C → r

Q.21 A → p; B → r, t; C → p, s; D → p, q, s Q.22 A → q, r; B → p, q; C → r, s

Q.23 A → q, ; B → r; C → p; D → r; E → s, t

Previous Years’ Questions


Q.1 D Q.2 C Q.3 C Q.4 A, C Q.5 D Q.6 D

Q.7 B Q.11 A Q.12 C Q.13 D Q.14 B Q.15 B

Q.16 A Q.17 A Q.18 A Q.19 D Q.20 A, B, C, D Q.21 B

Q.22 A Q.23 C Q.24 A Q.25 5 Q.26 C Q.27 C

Q.28 A Q.29 C Q.30 B Q.31 A, C, D


2 4 . 5 4 | Carboxylic Acid and Derivatives

Solutions

JEE Main/Boards Sol 5: In the presence of strong acids, the H⊕ is captured


by the carboxylic acid and the following equilibrium is
established:
Exercise 1
O OH
+ -
R C + H2SO4 R C + H2SO4
Sol 1: The uptake of 2H atoms shows the presence
OH OH
of one >C=C< along with C6H5– and –COOH, which
accounts for the 6° unsaturation.. Furthermore H and I
are monosubstituted benzene derivatives. O
||
H is C6H5–C(COOH)=CH2 giving Sol 6: H3C − CH2CH2 C − CH3
H3CCHC6H5COOH with one asymmetric carbon atom.
I is C6H5CH=CHCOOH, giving Sol 7: It is because the carboxylate group
(–COO–) of the branched acid is more shielded from
C6H5CH2CH2COOH with no asymmetric carbon. the solvent molecules, there, it cannot be stabilized
H effectively by salvation.
*
H2 C CH3
C=CH2 Sol 8: It is because carboxylic group does not have true
Pd
COOH
COOH carbonyl group due to resonance.
(H) O O-
+
H2 R C O H R C=OH
C=CHCOOH
Pd
CH2CH2COOH Due to resonance >C=O bond of –COOH develops
(I) partial double bond character and cannot show
reactions with hydroxylamine, phenyl hydrazine, etc.
Sol 2: (A) = C15H31CH2OH,
Sol 9: It is because formic acid combines the properties
(B) = C15H31CH2Cl,
of both an aldehyde an acid.
(C) = C15H31CH2CH2CH2OH, Aldehyde
(D) = C15H31CH2CH2COOH. O
H C
Sol 3: (Y) is an ester because it is hydrolysed to acid and OH
alcohol. Since the alcohol is not oxidized by acidified Hence it has reducing character of aldehydes.
KMnO4 and gives cloudiness at once with Lucas reagent, HCOOH + 2(Ag(NH3)2)+OH– →

hence it is a t-alcohol.
HCOONH4 + 3NH3 + H2O + 2Ag↓
O CH3 O CH3
or HCOOH + Ag2O → CO2 +H2O + 2Ag↓
HOH H3C C OH + H3C C OH
H3C C O C CH3
CN (A) CH3 Sol 10: Both these unsaturated acids have two ionisable
(Y) (B) hydrogens. After the release of first hydrogen, second
hydrogen of maleiate ions is involved in H-bonding,
Sol 4: This is because C2H5OH undergoes dehydration whereas no H-bonding is possible in fumarate ion.
to form C2H4 at 170°C in presence of excess of conc.
H2SO4.
H2SO4(conc.)
CH3 CH2OH o
H2O+CH2=CH2
170 C
Chem i str y | 24.55

 
O O O carbon atoms. Consequently –C ≡ C– and –C ≡ C– are
H C C - -
O C acid strengthening EWG’s (Electron withdrawing group,
O
C OH stabilizes anion, thus strengthens acid) This makes
CH3CH2COOH weaked of all these three acids since
H-Bond
H C C OH H
O –C ≡ C– is more acid strengthens group than –C ≡ C–
O (Fumarate ion group. This makes acid (I) stronger than acid (II)
(Maleiate ion) (H-bond not possible)

Due to the formation of H-bond in maleiate ion more Sol 15:

 
energy is required to remove H⊕ from it than from - -
fumarate ion, in which H⊕ release is easy comparatively. OH OH OH
Thus, K2 for fumaric acid is more than maleic acid.
 - + H3O+
Sol 11:

 
O 
O O O
H3C CH2
CH3 CH2 MgBr R C OH R C OR C  + H3O
+
O
- H3O+
CH3 CH2 CH2 CH2OMgBr
The electron charge in carboxylate ion is more dispersed
(X)
in comparison to phenoxide ion, since there are two
CH3 CH2 CH2 CH2OH KMO4
electro negative oxygen carboxylate ion as compared
(Y) to oxygen atom in phenoxide ion.
CH3 CH2 CH2 COOH
(Z) Butanoic acid
Sol 16: CH3COCl will after least stearic hindrance hence
it hydrolysis will be more vigorous.
Sol 12:
NaNH2 2CH2I Sol 17: Amide = CH3CONH2 Therefore acid is CH3COOH
HCCH NaCCNa

H2SO4 Sol 18: Acid (Y) obtained after decarboxylation must be


CH3 CC CH3 CH3 C=CH CH3
HgSO4 mono carboxylic acid thus molecular weight = Equiva
OH lent weight
O
Tautomaries
CH3 C CH2 CH3 The acid must Be (COOH → 45g/mol) Given mass
(Y) = 60g ; \ = 60 – 45 = 15g/mol
NaOH/Br
Na+O- C CH CH 2 3
Which is definitely due to – CH3
Haloform reaction
O Hence Y is CH3COOH
H3O+
CH3CH2COOH Carboxylic acid (X) has second COOH replacing H of
(Z) CH3COOH
SO (X) is malonic CH2(COOH)2 of molecular mass
Sol 13: 60 + 44 = 104
O H O Since it has two group so its equivalent mass
= 104/2 = 52b/eq.
CH3 C C CH3 (Dimmer)
O H O
Sol 19: Dehydration occur with all the three reagent
Dimerization of acetic acid occur in benzene via
intermolecular H-bonding Hydrogen bond is a special P O
2 5 → C H CH CN + H O
C2H5CH2CONH2  6 5 2 2
type of dipole-attraction.
SOCl
2 → C H CH CN + 2HCl + SO
C6H5CH2CONH2  6 5 2 2
Sol 14: sp hybridized carbon of –C≡C– of acid (I) and POCl Or
3
C6H5CH2CONH2 → C6H5CH2CN + H2O
sp2 hybridized carbon of –C=C– of acid (II) attract the PCl 5
bonded electron more than do the sp3 – hybridzed
2 4 . 5 6 | Carboxylic Acid and Derivatives

Sol 20: As the size of the substituent on α-carbon Sol 26: A = CH3OH (Methanol)
increases, the tetrahedral bonded intermediate become
B = CH3COCH3 (Methyl ethanoate)
more crowded. The greater the crowding the slower is
the reaction. C = HCHO (Methanal)
D = HCOOH (Methanoic acid)
Sol 21: (A) (CH3CO)2O (Acetic anhydride)
E = HCONH2 (Formamide or methanamide)
(B) CH3COOH (Ethanoic acid)
(C) CH3COOC2H5 (Ethyl ethanoate) Sol 27:
(D) C3H5OH (Ethanol) H5C6 H5C6
NH2OH
(E) CH3COCH3 N
H3C H3C OH
Sol 22: (A) CH3CH2COOC2H5 (Ethyl propionate) H5C6 CH5
CH3 and H5C6
(B) CH3CH2CO CHCOOC2H5
| N N
CH
3 OH HO
Ethyl-(3-keto 2-methylpentanoic acid)

(C) CH3 − CH2 − CO − CH − COOH Sol 28:


|
CH OH
3 A=HO OH
3-Keto-2-methylpentanoic acid B=H2C O
CH3 O
CH3
Sol 23: (A) CH3CH2CH2CH2COOCH2CH3
C=H2C Cl D=H2C N
or CH3COOCH2CH2CH2CH3 CH3
O O
(B) C2H5OH
(C) CH3CH2CH2CH2OH E=H2C OH F=H3C CH3
(D) CH3CHO O O O
(E) CH3CH =CHCHO
G=H3C CH3
(F) CH3COOH

Sol 24: (A)

CH3 − C − COOH CH3 − C − COOH Sol 29:


|| ||
CH3 − C − H H − C − CH3 H3C H3C Br

Cis Trans A = H3C O B = H3C O


H HO HO
|
(B) CH3CH2− C *− COOH H3C H3C
|
CH D= CH2
3 C = H3C O
HO H3C
(2-methylbutanoic acid)

H3C
Sol 25: (A) HCOOH
E = HCHO F= O
(B) CO
(C) (COOH)2 H3C
Chem i str y | 24.57

Sol 30: Sol 5: (B) Acid catalyzed alkyl cleavage

O O
X = H3C CH3
Me3 C C O18 CMe3
O
Propyl propionate

H3C
Sol 6: (A)
Y= O
HO PCl
5 → CH CH CN -Reduction
CH3CH2CONH2 
∆ 3 2
Propionic acid

H3C H3C Sol 7: (B)


Z= OH D= CH2
O
Propan-1-ol H3C O
H3CO C O Et H3O
+

O C OH
H3CO C O Et 

Exercise 2 O

Single Correct Choice Type Sol 8: (B)


O O
NaOH
Sol 1: (B) CH3 C OCH2 + CH2 NH3 CH3 C O CH2 CH2 OH

OC2H5 O ZnHg Sol 9: (D) Reduction reactions.


N H3O,  N
HCl
N
Sol 10: (D)
CH2COOC2H5
COOH COOH
COOH COOH
A COOH + CO
A COOH + CO22
Sol 2: (A) MeCOCl > MeCON3 > MeCOOCOMe. 

Consider electronegativity of halogen, azide & ester.
Halogen is on top, since it has the highest O
O
O
O
O
O
electronegativity. B
B Ph C CH2 C OH Ph C CH3 +CO2
Ph C CH2 C OH  Ph C CH3 +CO2

Sol 3: (D) COOH
COOH
COOC2H5 COOH
COOC2H5 H3O C NaOH
C2H5ONa NaOH + Na CO
CH2 + COOC2H5 C COOH CO2
CO2
++ Na
Na22 CO
2
CO33
3
COOC2H5 C2H5OH



COOC2H5 COOC2H5 COOH


C2H5ONa H3O
Sol 11: (B)
2 + COOC2H5 COOH
COOC2H5 C2H5OH

H 1 O H NNHC6H5
Sol 4: (C) H
2
OH NNHC6H5
CH3 HO 3 H 3 C6H5-NHNH2
HO H
O 4
H OH H OH
COOH
H3C  O H5 OH H OH
-2H2O 6 CH2 OH
OH CH2 OH
O
D-(+)-glucose common D-osazone
O CH3
2 4 . 5 8 | Carboxylic Acid and Derivatives
O

O C CH3 OH OH
H NNHC6H5 H 1 O OH
2 I + OH- Br Br
AlCl3 HO H Br2 + H2O
NNHC6H5 2

3 C6H5-NHNH2 HO 3 H HO 3
+

HO H A
B
4 C
H OH H OH
C CH 53 Br
H OH H OH
O
6 CH OH
CH2 OH 2

Sol 17: (B) Since it lacks active methylene componenet


common D-osazone D-(+)-mannose stable anion formation does not take place and thus it
can not undergo self condensation reaction.
Sol 12: (A) Esterification.
Sol 18: (B) Esterification.
O O
|| ||
HCl
Ph − C − O − H + HO18 CH(CH3 )2  Sol) 19: (D) Self-explanatory reactions
→ Ph − C − O18 − CH(CH
∆ 3 2

O O
||
HCl
||
Previous Years’ Questions
Ph − C − O − H + HO18 CH(CH3 )2  → Ph − C − O18 − CH(CH3 )2

Sol 1: (D) CH3–CH2–COOH + NaHCO3 →


Sol 13: (B) Dieckmann condensation product.
*
CH3CH2COONa + H2O + CO2
Sol 14: (C)
O Sol 2: (C) C6H5COOH + SOCl2 → C6H5–COCl
COOC2H5

EtO ZnHg Sol 3: (C)


H3O+ HCl
N
N N
OH O
| ||
H+
CH2 COOC2H5
P 
→ CH=
3− C CH 2 → CH3 − C − CH3
NaOH
 → CHI3 ↓
Sol 15: (C) Yellow
O MgBr H +
P2O5 CH3MgBr Q → CH3 − CH
= CH − OH → CH3CH2OH
C NH2 C  N H O+ C =N Fehling
3
 → Cu2O ↓
CH3 red

C
Sol 4: (A)
C =O
I2 + Ca(OH)2
O (RCOO ) CH3 O OMgBr
|| |
3 CH MgBr
Sol 16: (C) CH3 − C − OC2H5 
excess
→ CH3 − C − CH3
|
O CH
3
O C CH3 OH OH OH CH
|
3
Br Br H2O
AlCl3 I2 + OH- Br2 + H2O → CH3 − C − OH
H3 O
+
|
A
B CH
C
3
C CH3 Br
O
Chem i str y | 24.59

O H O- Sol 13: (B) Electron releasing groups (Alkyl groups) de


-H+ stabilizes conjugate base.
Sol 5: (D) R C OH+ OR’ R C OH
The +I effect of C3H7 is less than - I effect of Cl
OR’
Reaction occur at planar sp2 carbon giving racemic Ka of HC3CH2CH2 C H − COOH is 139 × 10−5
|
mixture of product.
Cl
O Sol 14: (C) Phenol gives violet colored complex
||
Sol 6: (B) C6H5 − C − NH2
POCl3
→C compound with neutral FeCl3, benzoic acid gives pale
 6H5 − CN
dull yellow ppt. with neutral FeCl3
O O
|| ||
Sol 7: (C) Compound with CH3 − C − or CH3–CH(OH)– Sol 15: (C) By reaction with one mole of CH3 − C − Cl
group gives haloform reaction but this reaction is given with one -NH2 group the molecular mass increases with
only by aldehydes, ketones and alcohols, so acetic 42 unit. Since the mass increases by (390-180) = 210
acid does not give haloform reaction. However acetic hence the number of -NH2 groups is 5.
acid has three α-H, therefore, statement-I is true but O O
statement-II is false. || ||
R − NH2 + CH3 − C− Cl 
( − HCl)
→ R − NH − C− CH3
Sol 8: (D) p-hydroxy benzoic acid has higher boiling
point than o-hydroxy benzoic acid because former Sol 16: (D)
prefers intermolecular H-bonding while the later prefer
intramolecular H-bonding. CH2 Br
alcoholic
CH2 OR
+ AgBr
AgNO2
H O CH3 CH3
HOOC O C OH
H O Oxidation
Intermolecular H-bonding OH
O
C COOH
O 
O
O H COOH O
Intermolecular
H-bonding (Phthalic anhydride)

Sol 9: Saponification is hydrolysis of ester in presence Sol 17: (D)


of dilute base rather in presence of dilute acid. O
|| − ⊕
3 → CH CH CO ONH NH
Sol 10: Propanoic acid has higher boiling point than CH3CH2 − C − OH  3 2 4

n-butanol because of more exhaustive H-bonding in (A) (B)


former case. O
||
∆ 2 Br ,KOH
 → CH3 − CH2 − C − NH2 
Hofmann
→ CH3 − CH2 − NH2
Sol 11: (D) Esterification reaction is involved bromamide
(C) reaction
H+
CH3COOH( ) + C2H5OH( ) 

CH3COOC2H5 ( ) + H2O( )

Sol 12: (C)

- -
-

C2H5O Na+ CH3 C Cl  CH3 C O C2H5 Ethyl ethanoate


O O
2 4 . 6 0 | Carboxylic Acid and Derivatives

Sol 18: (A) JEE Advanced/Boards


O- O O
- O=C=O
H O- Exercise 1
C
O Sol 1: (i) The isomers have 1°of unsaturation that must
be due to –COOH, since CO2 is evolved on adding
OH OH NaHCO3. The remaining oxygen may be present as –
COOH COO- OH or –OR.
(CH3CO)2O H+
COOH CH2COOH
* *
(B) H3C-CH H3C-CH
OCOCH3 CH2OH OH
COOH (D) (E)

(C) COOH COOH


* *
H2CH3C-CH H3C-CH
OH OCH3
Sol 19: (C)
(F) (G)
LiAlH4
CH3COOH CH3CH2OH ‘A’ (ii) LiAlH4 converts–COOH to –CH2OH. Only (D) is
reduced to an a chiral product.
PCl5
Not chiral
CH3CH2Cl ‘B’
COOH CH2OH
Alc.KOH
* LiAlH4
H3C-CH
H3C-CH
CH2 =CH2 ‘C’
HO HO (E)
Sol 20: (D)
(D)
CH3 COOH COCl CHO

KMnO4 SOCl2 (iii) The ether (G) differs from (E) and (F) in that it is
H2/Pd
inert
BaSO to oxidation by KMnO4 or CrO3. (E) gives a positive
iodoform test.
4

Toluene (A) (B) (C)


3 COOH

COCl CHO Sol 2:
KMnO4 SOCl2 H2/Pd CH3 CH3
BaSO4 HCl Mg
H3C-C Peroxide H3C-C-Cl Ether
ne (A) (B) (C)
CH2 CH3
Sol 21: (C) Hofmann bromamide degradation reaction
CH3 CH3
O CO2
|| H3C-C-MgCl H3C-C-COOMgCl
R − C − NH2 + Br2 + 4NaOH →
R − NH2 + Na2CO3 + 2NaBr + 2H2O CH3 CH3
1 mole bromine and 4 moles of NaOH are used for per
mole of amine produced. CH3
+
H2O/H
-Mg(OH)Cl H3C-C-COOH
CH3
Chem i str y | 24.61

Sol 3: Sol 8:
 2 HO
H-CC-H+CH MgBr H-C  CMgBr C5H6O4 -CO2 C4H6O2
3
-CH4 (B)
(G)
CO2 + CH3CH2CH2COOH
H-CC.COOMgBr H H-CC-COOH
COOH
(H) (I) 
H2C=CHHC -CO2
HgSO4/H2SO4 Tautomerise
H2O H-C=C-COOH COOH
OH H CH2=CH-HC2COOH 2
H /Ni
COOH
KMnO4 (B) CH CH CH COOH
H-C-CH2-COOH (O)
H2 C 3 2 2

O (J) COOH
Sol 9:
(A) (CH3CO)2O (B) CH3COOH
Sol 4:
(C) CH3COOC2H5 (D) C2H5OH
O
H2O (E) CH3COCH3
CH3CH2–C–OCH2CH2CH3 H+

Sol 10: (A) (C2H5COOC2H5)


CH3CH2COOH + CH3CH2CH2OH
(A) (B) (B) C2H5CO–CH(CH3)COOC2H5

CH3CH2CH2OHCH3 Chromic acid (C) C2H5COCH(CH3)COOH


(O)
(O)
CH2CHO CH3CH2COOH Sol 11: An alkali salt of palmitic acid is known as soap.
(A) The general formula of palmitic acid C15H31COOH.
Which on hydrolysis in presence of alkali give
soap(C15H31COONa) and glycerol as by product.
SOCl2 NaN3
Sol 5: RCO2H RCOCl
D Hydrolysis Sol 12: Acid do not reacts with NaHSO3though they
RCON3 RNCO RNH2
have >C=O group because of resonance stabilization.
The resonance take place as follows.
Sol 6: C≡O bond is very stable due to large ∆Hf of CO; O O O
_

O -C -C -C
so the decomposition reaction H–C–C CO+HCl O O O

is favoured. Formyl chloride is not stable above –60°C.


Ca(OH)
Sol 13: CH3COOH 
2 → (CH COO) Ca
3 2

Sol 7: An extremely mild but selective oxidizing agent (X) (Y)


for aldhydes is silver oxide suspended in aqueous
base. An unsaturated acid is obtained with this reagent Acetic acid Cal. acetate
because the >C=C<remains untouched by this reagent.
CH3
Dry
CH3
H CHO H COOH C=O Conc. H2SO4
distillation
CH3
C=C C=C
Acetone
CH3 CH3
H2CH3C CH3 H2CH3C CH3
(Z) Mesitylene
2 4 . 6 2 | Carboxylic Acid and Derivatives

O  (ii) C2H5OH → CH3 CONH2


Sol 14: CH3CH2COOH →
SeO CH3CO COOH + H2O
2
Ethyl alcohol Acetamide
Propionic acid Pyruvic acid
[O] [O]
CH3OH  → CH3CHO 

K2Cr2O7 /H+
Sol 15: Acid are directly reduced to the corresponding
primary alcohol with powerful reactant like LiAlH4. It Methyl alcohol
attack only on the carbonyl group of a fatty acid. SOCl
CH3COOH 
2 → CH COCl
3
O
|| NH
4 → RCH OH + H O LiAlH 2 → CH CONH Acetamide

R − C − OH + 4H  2 2
3 2

Alkanol (iii) CH3COOH → CH3COOC2H5


Acetic acid Ethyl acetate
Sol 16: LiAlH
CH3COOH → CH3CH2OH
CH → CH3 
 →
||| 2+ Hg + 2−
,H2SO 4 | H /Cr2 O7 Acetic acid
CH CHO CH COOH/H+
3
→ CH3COOC2H5
NaOH
CH3COOH  → CH4

CaO Ethyl acetate

H O+ NH Sol 20: (i)


Sol 17: CH3CN →
3 CH3COOH 
3→
O
Ethane nitrile Ethanoic acid NH-H CH3CH2-O-C
O=C CH2
(X) (Y) NH-H CH3CH2-O-C
CH3COONH4 ∆
→ CH3CONH2 (X) Urea O
O

Ammonium ethanoate Ethanamide NH-C
PCl3
Br2 /KOH C O= CH2+2C2H5OH
→ CH 3
NH2
(Y) NH-C
Amino methane
(Z) Ethanol O
NaOH
Sol 18: CH3COONa ←
 CH3COOH (ii) NH-H
C2H5-O-C=O
O=C +
(Z) (X) C2H5=O-C=O
NH-H
PCl3
 → CH3COCl Urea (E) Diethyl oxalate
(Y)
PCl3 NH-C=O
CH3COCl + CH3COONa → C O= +2C2H5OH
NH-C=O
(Y) (Z) (F) Parabanic acid
(Oxalyl urea)
O O O
|| || ||
CH3 − C − O − C − CH3 2CH3 − C − Cl (iii) NH2 HCl
CH2=O+O=C
(E ) (Y) NH2
Formaldehyde (X) Urea

Sol 19: (i) CH3Cl → CH3COCl CH2 (OH) NH CONH2


CH2=O

Monomethylol urea
Methyl chloride Acetyl chloride
CH2(OH)NHCONH(OH)CH2 Urea
Mg CO2 Resin
CH3Cl 
→ CH3MgCl  → Dimethylol urea
H2O/H+ urea-formaldehyde
Methyl chloride
2→ SOCl
CH3COOH  CH3COCl
Acetyl chloride
Chem i str y | 24.63

Sol 21: (i) Sol 24: (A) CH3CHOHCH2COOH


CN− (B) CH3CH=CHCOOH
CH3CH2CH2CH2Br →
−Br − / 2HOH/H+ (C) CH3CH=CHCOCl
CH3CH2CH2CH2CN 

CH3CH2CH2CH2COOH (D) CH3–CH=HCON(CH3)2
n − Pentanoic acid (E) CH3COCH2COOH
(F) CH3COCH3
(ii) CH3 CH3
CN-
H3C C Br -HCN H3C C 2 4
H SO (G) CH3CH2CH3
MR
CH3 -Br- CH2
Sol 25: Is a saturated monoester with
CH3 CH3
M.W = 186
H3C C HSO4 HOH H3C C OH
CH3 O CH3
CH3 CH3 CH3 O CH3 CH3
CH3
CH3 F = H 3C O
F = H 3C
SOCl2 Mg CH3 O CH3
H3C C Cl CH3 CH3
Ether
CH3 CH3 O
CH3 O H
CH3 CH3 H
(i) CO2 G = H3 C O
H3C C MgCl H3C C COOH G = H3C
(ii) H3O
2
CH3 O
CH3 CH3 CH3
OH
OH CH
Sol 22: CH33
H = H3C
H = H3 C
H CH3
H
Br
CH3
t-BuOK KMnO4
HOOCCH2 Br 2H
C C
2 OSO2C6H5
HOOCCH2
Ni OSO2C6H5
H
Br Br CH3
H J = H3C CH3
E J = H3C
Meso compound CH3
CH3
H
Br
HOOCCH2 Br
CH2 CH3
HOOCCH2 K = H3C CH3
K = H3C
H CH3
F CH3
Meso compound

Sol 23: Sol 26:


CH3 CH3 CH3 COOH CH3
C=C and C=C
O
H COOH H CH3 A = H3 C
CH3
(A) Geometrical isomers CH3
O
CH3 CH3 B = H3C
C2H5 C COOH or HOOC C C2H5 CH3
H H O
C = H3C H
O O
(B)
CH3
O
D = H3C H
O
CH3
Cl
E = H3C
O O
CH3 (B) CH3
O C=CHCO2H
2 4 . 6 4 | Carboxylic Acid and Derivatives H3C
D = H 3C H
O
CH3 (C) CH3CHCH2CH3 CH3CH2CH2CH2OH
Cl
OH Dehydration
E = H3C
O 2-Butene
CH3
CH3
(D) CH3
N C CH CO2H
F = H 3C H3C
O OH OH
CH3
M.F C5H10O4

Sol 27: (E) O=CH CO2H


M.F C2H2O3
A=
O O
Sol 29: The given reaction are as follows.
Br
+
Br C2H5OH H
A B+C B + D
B=
(Carboxylic
O O Acid) KMnO4
CO2H Ca(OH)2
B D C3H6
C= O
(E)
OH
CO2H The compound E must be ketonic compound as it does
not give Tollens test and does not reduce Fehling’s
D= solution but forms a 2, 4-dinitrophenyl-hydrazone.
O Therefore, its structure would be CH3COCH3(acetone).

CH3 Since E is obtained by heating B with Ca(OH)2, the


compound B must be CH3COOH (acetic acid).
CO2H
Since B is obtained by oxidation of D with KMnO4, the
E= compound D must an alcohol with molecular formula
OSO2C6H5 CH3CH2OH(ethanol).
Since B and D are obtained by acid hydrolysis of C,
CO2H the compound C must be an ester CH3COOC2H5(ethyl
F= acetate).
OSO2C6H5 Since the compounds B (acetic acid) and C (ethyl acetate)
are obtained by treating A with ethanol, the compound
CO2H A must be an anhydride (CH3CO)2O (acetic anhydride).
G=
The given reaction are
OH
(A) CH3 C2H5OH
C=CH-C-OCH2CH2CH2CH3 (CH3CO)2O CH3COOH
Sol 28:
CH3 Acetic anhydride Acetic acid (B)
O
(A) + CH3COOC2H5
CH3 or Ethyl acetate (C)
(A) CH 3 +
C=CH-C-OCH2CH2CH
C=CH-C-O-CH-CH 2CH3
H
2CH2CH3 Ca(OH)2
CH
H3C3
O CH
O CH3COCH3 CH3COOH + C2H5OH
3
or Acetone (E) Acetic acid Ethanol
CH3 (B) (D)
(B) CH3 C=CH-C-O-CH-CH CH CH
2 2 3 KMnO4
H3C C=CHCO2H
H3C O CH3

(B) CH3
(C) CH3CHCH 2CH3 CH3CH2CH2CH2OH
C=CHCO2H
Chem i str y | 24.65

Sol 30: Acetoacetic ester shows tautomerism and the Sol 2: (B)
two forms are called as keto and enol forms. O
C -Cl CH2COOMe
O O i) CH3N2 (4 mole)
C2H5 ii) Ag2O; MeoH
H3C O C - Cl CH2COOMe
Keto form OH O O
Amdt - Eistert Reaction
C2H5
O O
H3C O
C -Cl C Cl
Enol form  CH - N=N

:
CH2 - N=N 
 OH  CH2 - N=N
H
 |  C -Cl  C Cl

:
The enol  H3C − C = CH =  gives blue-violet colour O CH - N=N
(2 moles) O 
  H
 
  2CH2 -N=N
with FeCl3solution. When Br2 is added, it reacts at once - 2HCl

with = of the enol form. O



O=C=HC C - CH - N=N
OH O Ag2O
C2H5 +Br2 -N2 
H3C O O=C=HC C CH - N=N
CH3OH O
Br Shifting
HO Me O O C - H2C to - CH
C2H5
H 3C O Me O O C - H2C
Br O
As soon as enol form is consumed, its colouration with
FeCl3 disappears and excess of bromine gives brown Sol 3: (B)
colour. As keto and enol forms are in equilibrium, when SOCl
2→
HOOC − CH2 − CH2 − CH = CH2 
enol form is used, the equilibrium shifts to right hand
side to give more enol form which discharges the ClOC − CH2 − CH2 − CH =
CH2
colour of excess of Br2 and gives blue violet colour with
excess of FeCl3 present in the reaction mixture.

N
O
Exercise 2 H
N - C - CH2 - CH2 - CH = CH2

Single Correct Choice Type


i) LAH
Sol 1: (D) ii) H3O
N - CH2 - CH2 - CH2 - CH = CH2
iii) dilOH
H
OH H
Phosgene O O

COCl2 O O Sol 4: (C)


OH H
H - 2HCl O O
Cl-C O O
 NH
Cl-C H2N - C - CH2 - CH2 - C - N3
O -N2
N O

H
2 4 . 6 6 | Carboxylic Acid and Derivatives

Sol 5: (B) Sol 12: (A)


O O
Cl Na COOH
C C
NaN3 -2CO2
NaOH+
CH2 CH2
COOH CaO CH2
soda
lime

Multiple Correct Choice Type

Sol 6: (A, C, D) Self–explanatory, Rearrangement reactions Paragraph 2 (Questions 13 to 15)

Sol 7: (A, C) Self-explanatory Sol 13: (C); Sol 14: (A); Sol 15: (D)

Comprehension 2 18
Assertion Reasoning Type OMe

Sol 8: (D) The given compound is optically active.


O18
OMe
Comprehension Type O
O3
O O
Paragraph 1 (Questions 9 to 12) O
O O
Sol 9: (A) Abstraction of a -H takes place to given a O
carbanion, from the lower side to give C2OH38 O
Zn/H2O
Sol 10: (C) Agathene Dicarboxylic Acid: H
O O
18 C =O
H C C OMe + 2
2 C =O
AAC
COOH A H B
* *
+
* H /H2O
*
O O
HOOC 18
H C C OH + HO Me
∴ 4 Chiral Carbons (shown by *) C D

Sol 11: (C) No. of Chiral carbons = 4.


Paragraph 3 (Qeustions 16 to 18)

Sol 16: (A) Mechanism of formation of A and B is A AC2


COOH
*
* Sol 17: (D) Both B and C are redox reaction as B involves
* * reduction and C reaction involves oxidation step.
HOOC O O
|| ||
Me2Cd
∴ No. of Optical isomers = 24 = 16 Sol 18: (B) H − C − Cl  →H − C −H

∴ Stereoisomers = No. of optical isomers + No. of


geometrical isomers = 32. Match the Columns

Sol 19: A → s; B → p; C → r; D → s
(A) It is an example of Dieckmann reaction which
involves condensation of two ester.
Chem i str y | 24.67

(B) Condensation between an active methylene Re d P/HI


(E) R − C − OR ' → RCH3 + R 'H
compound and an keto compound is known as ||

Knoevenagel reaction. O

(C) It is an example of reformatsky reaction


Previous Years’ Questions
(D) It is also an example of Dieckmann reaction which
involves condensation of two ester to form a ring
structure. Sol 1: (D)
SO3 SO3Na
Q.20 A → p; B → q, s; C → r. +NaOH
(A) CH3 = ( ) ( )
O
3 → U + V − Haloform rection
− CH CH2 
Zn
+H2O +CO2
(B) Product are ketone and aldehyde which can be
OH
differentiated by Fehling’s and Tollens reagent. ONa
+NaHCO3
(C) Product are acid and alcohol. Acid gives effervescence O2N O2N
with aq. NaHCO3.
+H2O +CO2

Q.21 A → p; B → r, t; C → p, s; D → p, q, s
Sol 2: (C)
(A) Markonikov’s rule-Regioselective CHO COO-
OH-
(B) Example of Diels Alder reaction-Cyclic addition,
stereospecific CHO CH2OH
(C)Addition reaction- Regioselective O

And will form diastereomers. H+


O
(D) Regio as well as stereoselective addition and will
form diastereomers.
Sol 3: (C)
Q.22 A → q, r; B → p, q; C → r, s O O O
heat * I2
(A) Acid will give brisk effervescence with NaHCO3. Ph C CH3 NaOH
Ph OH
Other organic product formed will give haloform test *

(B) Presence of unsaturation will cause decolourisation Ph COONa + CHI3


of Br2 water. And Acid functional group will give F G
effervescence with NaHCO3.
Sol 4; (A, C)
(C) One of the organic product formed will give
haloform test. One of the organic product formed will O O
give 2, 4 DNP
R C NH2 +OH- + Br2 R C NHBr
(A)
Q.23 A → q ; B → r; C → p; D → r; E → s, t
O O
_
(i) LAH
(A) R − C − OR '  → R − CH2OH + R 'OH R C NHBr+OH- R C N Br
|| (ii) H2O
(B)
O
(i) LAH
(B) R '− C − OH → R 'CH2OH R N=C=O + Br-
|| () 2
ii H O
(C)
O
LAH
(C) R '− CH2 − Br 
→ R 'CH3

SBH/EtOH
(D) R '− C − H 
→ R 'CH2OH
||
O
2 4 . 6 8 | Carboxylic Acid and Derivatives

Sol 5: (D) Cl Cl
[O] tautomerism
O O
Cl C NH2 + KOH + Br2 Cl CCOOH
NHBr Cl
O
O (i) O An intermediate (ii) CHO

C NH2 + KOH + Br2 Cl C NHBr O



H O H
(i) An intermediate (ii)
Sol 6: (D) Sol 10:

O CH2Cl CH2CN

Cl C _ Slow KCN C2H5ONa/EtOH


:

_ DMF C6H5CHO
N Br
:

(iii) (A)
Cl N=C=O+Br- OH C6H5 C6H5
H3O+/
C6H5 CH CH C6H5 C=C
(iv) H
CN HOOC
Sol 7: (B) The rate determining step of Hofmann
C6H5 C6H5
bromide reaction is unimolecular rearrangement SOCl2
of bromamide anion (iii) and no cross-products are CH3NH2 C=C
formed when mixture of amides are taken. CH3HN C H
15 O
CONH2 + CONH2

D (i) Sol 11: (A); Sol 12: (C)


1.Mg/Ether
KOH 15 2. CO
2
CH3 − CH2 − C ≡ C − CH2 − CH2Br  →
NH2 + NH2 +
Br2 3.H3O
K
D J → CH3 − CH2 − C ≡ C − CH2 − COCl2

Sol 8: The two stereoisomers of 2-phenyl propanoic


acid in the racemic mixture are : J = CH3 – CH2 −C ≡ C – CH2 –COOH
* K = SOCl2
COOH COOCH(CH3)Et
*
CH3CH(OH)Et
H C Ph H C Ph
CH3 CH3 Sol 13: (D)
(A)
OH HO
* O2N O
COOH COOCH(CH3)Et NO2
*
CH3CH(OH)Et
and Ph C H Ph C H HN NH O O
HO
CH3 CH3
(B) O O
HO OH
Here A and B are diastereomers. NO2
(Picric Acid) (Barbituric Acid) (Ascorbic Acid)
Sol 9: Compound A of molecular formula C9H7O2Cl
exist in keto and predominantly in enolic from B. Hence,
A must be a carbonyl compound which contain α-H.
Enolic forms of B predominates because of presence of
intramolecular H-bonding.
Chem i str y | 24.69

HO
O COOH
O
NH O O
HO O C CH3

O
HO OH

uric Acid) (Ascorbic Acid) (Asprin)

Sol 14: (B)

O O O O O
COOH 
HOOC
O
H3O+ CO2 O2/H2O2 HOOC
COOH
O O O O O

Sol 15: (B)

COOH
H COOH
C KMnO4 H OH
OH-
C H OH
H COOH
P COOH
Soptically inactive
COOH COOH
H COOH
C KMnO4 H OH OH H
OH-
C HO H H OH
HOOC H
Q COOH COOH
T U

Optically inactive pair
2 4 . 7 0 | Carboxylic Acid and Derivatives

Sol 16: (A)


O

Q  O

O
O
O
C
AlCl3 Zn-Hg, HCl
+ O

C C
O HO OH O
O

H3PO4

Sol 17: (A) Only amines undergo acetylation and not acid amides.

O
NH2 NH C CH3
O O
C C
H3N O OH
+ CH3COOH

C NH2 C NH2
O O

Sol 18: (A) (i) - p; (ii) - r; (iii) - s; (iv) - q

O
C6H5 CH2 + CH2+ CH3O
O
H5C6H2C O CH3
O
O C6H5 CH2+CO2+Ph CH2 C CH3
CH3
O CH3
H5C6H2C O CH3
CH2C6H5
Ph CH2+CH3 CO CH3
Chem i str y | 24.71

O O
CH3
O CH3 C6H5 CO2+ CH3 C CH3 Ph+CH3 CO Ph+CH3
C6H5 O -CO2
C6H5
C6H5

O C6H5 CO2+ CH3O


O
C6H5 O CH3 C6H5 +CO2

Sol 19: (D)

+
H
H 
+
HO

(P)
aqueous dilute KMnO4
o
0C

OH
CHO
CO, HCl
(I) Anhydrous AlCl3/CuCl
HO OH
HO CHCl2 CHO
H2O
(II) (Q) o
100 C
Sol 20: (A, B, C, D)

CHO COCl H2
CHO
CO, HCl
(I) (III) Pd-BaSO4
Anhydrous AlCl3/CuCl

CHCl2 CHO CO2Me CHO


H2O DIBAL-H
(II) o
(IV)
100 C Toluene, -78oC
H2O

COCl CHO
H2
(III)
Sol 21: (B) Pd-BaSO4

CH3
CO2Me CHO
- DIBAL-H O2 C CH3
-

(IV) + H3C CH CH3 o


Toluene, -78 C O
H2O O
(U) H
CH
H3C CH3
(D)
2 4 . 7 2 | Carboxylic Acid and Derivatives

Sol 22: (A)


COOH COOH COOH COOH
HO OH OH
> > >
OH
OH

Sol 23: (C) NaBH4 is a mild reducing agent. It selectively reduces aldehydic group.

Sol 24: (A)


OH OH COOH COOH

(I) (III)
Cl CH3
pKa=9.98 (II) pKa=4.17
(IV)
pKa=9.38
pKa=4.37

Decreasing order of acidic strength: III > IV > II > I

Sol 25: (5)

COOH OCH3CH3 OH OH COOH


CH2OH
&

N
H3C CH3

are soluble in aqueous NaOH.

Sol 26: (C)

COOH O2N COOH


HNO3
H2SO4 (-OH group is more activating)
HO HO
(P)

OCH3 OCH3
NO2
HNO3
H2SO4 (-OCH3 group is more activating)

CH3 CH3
NO2
O
O
C
C HNO3
O
O H2SO4
Chem i str y | 24.73

Sol 27: (C)


O
NaOH/Br
H3C C NH2 H3C NH2
Hoffmann brom-amide deg radation
O
C NH2

O
H3C NH C

(T)

Sol 28: (A)


O O

(i) KOH
NH NH CH2 Br
(ii) Br CH2 Cl

O O

Sol 29: (C) Due to ortho effect o-hydroxy benzoic acid is strongest acid and correct order of decreasing Ka is

COOH COOH COOH OH

OH

> > >

CH3 NO2
OH

Sol 30: (B) In decarboxylation, β -carbon acquire δ− charge. Whenever δ− charge is stabilized decarboxylation
becomes simple. In (B) it is stabilized by-m and-o of C=O, which is best amongst the options offered.
O O H
C
O

Sol 31: (A, C, D)


O
O

CH3 T
LiAlH4

OH OAc
COOH
CrO /H - OH (CH CO) O OAc
-

3 3 2
COOH excess
CH3 V CH3 U CH3 W
2017-18 100 &
op kers
Class 12 T
By E ran culty
-JE Fa r
IIT enior emie .
S fP r es
o titut
Ins

CHEMISTRY
FOR JEE MAIN & ADVANCED
SECOND
EDITION

Exhaustive Theory
(Now Revised)

Formula Sheet
9000+ Problems
based on latest JEE pattern

2500 + 1000 (New) Problems


of previous 35 years of
AIEEE (JEE Main) and IIT-JEE (JEE Adv)

5000+Illustrations and Solved Examples


Detailed Solutions
of all problems available

Plancess Concepts
Topic Covered Tips & Tricks, Facts, Notes, Misconceptions,
Key Take Aways, Problem Solving Tactics
Amines and Aromatic
Compounds Containing PlancEssential
Nitrogen Questions recommended for revision
25. AMINES AND AROMATIC
COMPOUNDS CONTAINING
NITROGEN

1. INTRODUCTION
(a) Alkyl derivatives of NH3 are called Amines.
(b) If a hydrogen atom of NH3 is replaced by an alkyl group, then it is called a primary amine and possesses −NH2
(amino) group.
(c) If two hydrogen atoms of NH3 are replaced then it is called a secondary amine and it posseses NH (Imino)
group.
(d) If all hydrogen atoms of NH3 are replaced then it is called a tertiary amine and has a nitrile N group.
(e) N is in sp3 hybridisation and has tetrahedral geometry.
(f) The bond angle increases from ammonia to 3º amines. NH3 (107º ) < RNH2 < R 2NH < R3N

2. STRUCTURE AND BONDING

2 3
sp hybridized sp hybridized

CH3 H = CH3 CH3 =


H 108 CH3
1.47 Å

Figure 25.1: Hybridisation in primary and tertiary amine

(a) Trigonal pyramidal geometry


(b) High negative potential on N
CH3NH2 (CH3)3N

Figure 25.2: Bond Structure of primary and tertiary amine


2 5 . 2 | Amines and Aromatic Compounds Containing Nitrogen

(c) Amine N can be a stereogenic centre.

N H H N
R R
R’ R’

Figure 25.3: Amines as a Pair of enantiomer

(d) Enantiomers rapidly interconvert.

R
R R
N : N : : N
H H
R’ R’ H R’

The two mirror images are interconverted.

Planar transition state


Figure 25.4: Interconversion of Enantiomers

(e) Enantiomers of quaternary ammonium salts:

R” R”
+ +
N N
R R’” R’” R
R’ R’

Figure 25.5: Isomerism shown by quaternary ammonium salt

3. PREPARATION OF AMINES

3.1 Nucleophilic Substitution


(a) Hofmann’s Ammonolysis Reactions:

Direct Nucleophilic Substitution

H
S N2 +
:

NH3
:

+
R X + NH3 R N H R NH2 + NH4
_
H X 1° amine
H
S N2 +
:

H R’NH2
:

+
R X + R’NH2 R N R N H + R’NH3
_
R’ X R’
H
S N2 +

:
NH3

:
+
R X + NH3 R N H R NH2 + NH4 Chem i str y | 25.3
_
H X 1° amine
H
S N2 +

:
H R’NH2

:
+
R X + R’NH2 R N R N H + R’NH3
_
R’ X R’
2° amine

H
S N2 +

:
R’ R’2NH

:
+
R X + R’2NH3 R N R N R’ + R’NH3
_
R’ X R’
3° amine

R’

_
S N2
R _ X + R’3N R_ N
:
+ _ R’ X

_
R’
Drawback:
(i) RX limited to methyl and 1º alkyl halides.
(ii) Complicated by polyalkylation.
(iii) Useful for preparing 1º amines and quaternary ammonium salts.

CH3
_

-
CH3 _ Br + CH3 _ N CH3 _ N
:

+
Br
_
_

CH3 CH3
Quaternary ammonium salt

(b) Gabriel Phthalamide Synthesis:



:

O O O
: :
_ SN2
H_N: _: N: R _ N:
_ R_X
: :

OH: :O :O O
:

Alkylated imide
Phthalimide _
pKa = 10 +X
_
OH H2O
_
CO2
• R = Me or 1º alkyl
R _ NH2 +
_

1° amine CO2
Dicarboxylate
by product

3.2 Reduction of Nitrogen Containing Functional Groups


(a) From Nitro Componds
reduce
R — NO2  → R — NH2
2 5 . 4 | Amines and Aromatic Compounds Containing Nitrogen

Reducing Agents:
(i) H2/Pd-C
(ii) (1) Fe, HCl (2) OH–
(iii) (1) Zn, or Sn, or SnCl2 + HCl (2) OH–

(b) From Nitriles


CH3CH2CH2 − C ≡ N →
1) LiAlH
4 CH3CH2CH2 − CH2NH2
2) H O 2

Nitriles from alkyl halides:

CH3CH2CH2 − Br NaCN
→
S 2
CH3CH2CH2 − C ≡ N
N

(c) From Amides


O
=

[1] LiAlH4
C [2] H2O
RCH2-NH2
R NH2
1° amide 1° amide
O
=

[1] LiAlH4
C [2] H2O
RCH2-N-R’
R NHR’
_

H
2° amide 2° amide
O
=

[1] LiAlH4
C RCH2-N-R’
[2] H2O
_

R NR’2
R’
3° amide 3° amide

3.3 Reductive Amination

Examples: O H NH2
NH3
NaBH3CN
Amphetamine
a powerful stimulant

CH3 CH3
CH3NH2
O NaBH2CN H NCH3
2° amine
Methamphetamine
Retrosynthetic Analysis: OH OH
NHCH3 O
+ H2NCH3
CH3 CH3
2° amine
Methamphetamine
Chem i str y | 25.5

3.4 Rearrangement Reactions


(a) Hofmann’s Bromamide Reaction: O 1. X2, base, H2O
(e.g. NaOH)
C 2. 
R-NH2
R NH2
1° amide 1° amine

X = Cl, Br

Mechanism:
Θ Θ : X _X : Θ
: :

: :
: :
: OH H2O

:
:

O :O: O :O:

:
:

:
:

: :
H X X:
R N R N R N R N

:
:

H H Θ H Θ
H2O
: :

: :
:X: : OH
Θ

: :
:X:

H H2O H H
O

:
:
Θ Θ
O O
:

:O :O :O
O
:
:
::

:
::

C
:
:

H
N N N :O H

:
N
H  H
:

R R
:

R H O
:O H R
:

H
:

Isocyanate
H H H intermediate
:

:O
:O Θ
:

O O:
:
::

N
:

R H N
:

R H
H
Θ CO2 (g) H
:

:O
:

:O O O:
:
::
:

N
N N  R H
R H R H
H H

(b) Curtius Rearrangement: O


=

C  HO
R-NH2
R-N=C=O -CO
2

R N3 -N2 2

Mechanism:

Step 1 Preparation of azides:

O + O O
Na
+ _ +
_
_ NaCl + +
R Cl N=N=N R N=N=N R _ N=N
_ Nl
2 5 . 6 | Amines and Aromatic Compounds Containing Nitrogen

Step 2 Decomposition: O
__ + 
R=N=C=O
R N=N
_ Nl -N2

Reaction with water to the unstable carbamic acid derivative which will undergo spontaneous decarboxylation:

O O
O
H H R H H2N-R
R-N  O R 2R-NH2
N OH N O -CO2
H H
Carbamic acid
Isocyanates are versatile starting materials:
O
O R
R’NH N OR’
C

R-N  O H
-N2 O
R N3
RNH2
R
N NHR
H
(c) Leuckart Thiophenol Reaction:

N2Cl KS OR S OR
+
S S
S OR SH
Base
S
S OR SR


The Leuckart Thiophenol Reaction allows the preparation of thiophenols and corresponding thioethers from
anilines or their corresponding diazonium salts. The first step of the reaction of an aryl diazonium salt with a
potassium alkyl xanthate to give an aryl xanthate, which affords an aryl mercaptan upon the basic hydrolysis
or an aryl thioether upon warming.

(d) Ritter Reaction:


O
H2SO4
+R N
N
H2 O R
H
O
H2SO4
OH + R N
H2 O R N
H

The acid-induced nucleophilic addition of a nitrile to a carbonium ion, followed by the hydrolysis to the
corresponding amide.
Chem i str y | 25.7

Mechanism of the Ritter Reaction: Any substrate capable of generating a stable carbonium ion is a suitable
starting material; primary alcohols do not react under these conditions, with the exception of benzylic alcohols.

H
_
H
+
+
O Ol
H H
-H2O

+
CH2 H
+

The carbonium ion adds to the nitrile nitrogen to give a nitrilium ion intermediate, which undergoes hydrolysis to
the corresponding amide upon the aqueous work-up.

+
+ + N R N R N
_+ R

H H H
_ H _ H
N O
+
-H /+H
+
+N Ol +N O H
+

N
_ O
+ +
H H
R R R R

(e) Lossen’s Rearrangement:


O
2
O R Θ
1 OH 1 H2 O 1
R N R -N=C=O R -NH2 + CO2
H O

Mechanism:
O O HO-H
2 2
O R Θ O R 1 1
1 1
R N R N R CO2 + R -N=C=O
Θ
O OH O
H :OH2
isocyanate
intermediate
H decarboxylation
N O 1
R -NH2 + CO2
1
R H
O
:B

(f) Claisen Schmidt rearrangement:


O
=

RCOOH
HN3
R _ C _ N3 (i) 
RNH2 + CO2
H2SO4 (ii) H2O

Illustration 1: Prepare n-butylamine by Gabriel synthesis. (JEE MAIN)

Sol: This method is mainly used in preparation of Primary amine by use of alkyl halide and Potassium phthalimide.
After the attack of alkyl halide, Potassium phthalimide no longer behaves as a nucleophile. Product is cleaved by
reaction with base or hydrazine, which leads to a stable cyclic product. The by-product of this reaction is an acid.
2 5 . 8 | Amines and Aromatic Compounds Containing Nitrogen

O O

Θ  n-Bu-Br
N_ K N _ Bu-n
-KBr

O O
Θ 
1. aq. OH 2. H3O

COOH
n-BuNH2 +
COOH

Illustration 2: RCH2CH=O + [Intermediate]


H2/Ni
(B) (JEE MAIN)
or
H (A) NaCNBH3
_

N
Piperidine

Sol: Reaction between aldehyde and piperidine proceed via formation of carbinolamine which gets converted to
Iminium ion intermediate. On reduction with Raney Ni, tertiary amine is formed.

RCH2 _ CH=O + H _ N
:

OH
_

RCH2 _ CH2_ N
A carbinolamine

Reduction 
RCH2CH2 _N RCH2 _ CH=N

(B) Iminium ion


Intermediate

(A)

Illustration 3: Complete the following reactions: (JEE ADVANCED)

(A) (A)
+ NH3 (B) (C) (D)
O
Oxirane
(A)


Sol: H3N
:

SN 2 H3N NH3
O Θ H2N OH
O (B)
:

(A)
(Aminoethanol)
(B) is basic and
reacts further
O with (A)

O (HOCH2CH2)2NH or NH
:

OH OH
(HOCH2CH2)3N (C)
(D)
Chem i str y | 25.9

Illustration 4: Complete the following: (JEE ADVANCED)


NO2
NH4SH
(B)

NO2
(A)

Sol: It is an example of region selective reaction. One of the nitro group preferentially gets reduced by NH4 SH
(NH4 )2 S or NH4 SH reduces only one nitro group. (B) is

NH2
B is

NO2

4. PHYSICAL PROPERTIES OF AMINES


(a) Unlike most other organic compounds, amines are far more soluble in water, this is because all amines form
a stronger H–bond with water.
(b) Like ammonia, amines are polar compounds and apart from 3º amines, they can form intermolecular H–
bonds that’s why they have higher b.p.s’.
(c) The boiling points of amines are lesser than of alcohols and acids of comparable mol. weight because H–
bonding in amines is less pronounced in 1º and 2º than that in alcohols and carboxylic acids. This is because
nitrogen is less electronegative than oxygen.
(d) Boiling point of 1º, 2º and 3º amines follow the order.
1º > 2º > 3º amine
(e) Solubility in water follow the order.
1º > 2º > 3º amine
This is all due to H–Bonding.

Basicity of Amines: Due to basic nature, they react with acids to form salts.


R _ NH2 + H _ X R _NH3X Θ (Salt)
:


Ph _ NH2 + HCl R _NH3Cl Θ (Anilinium chloride)
:

Amine salts on treatment with bases such as NaOH regenerate the parent amine.
+ − •• −
RNH3 X − + OH → R NH2 + H2O + X

Relative Basicity of Amines: Compare basicities by using conjugate acid pK a values.


(a) An Amine and Ammonia
H − NH3+ pK a =
9.3 NH3 (weaker base)
CH3CH2 − NH3+ pK a =
10.8 CH3CH2NH2 ( strong base)
2 5 . 1 0 | Amines and Aromatic Compounds Containing Nitrogen

(i) Alkyl amines are stronger bases than NH3


(ii) Electron-donating alkyl group increase the electron density on N.

(b) An Alkylamine and an Arylamine


+
NH3
+
CH3CH3NH3

pKa = 4.6 pKa = 10.8


Aniline is a weaker base that ethylamine. Why?
_ + +
:

NH2 NH2 NH2


:

etc.

_:
••
CH3CH2 NH2 localized lone pair

(i) Electron donors increase basicity.



:

NH2
D D can be -NH2, -OH, -OR, -NHCOR, -R

(ii) Electron withdrawers decrease basicity.



:

NH2
W can be -X, -CHO, -COR, -COOR,
W
-COOH, -CN, -SO2H, -NO2, -NR3

p-nitroaniline Aniline p-toluidine

Increasing Basicity

Figure 25.6: Order of Relative basicity of Amine

Order of Relative basicity of Amine

(c) An Alkylamine and an Amide


_

:
:

:O :O:
Lone pair on N is
C C + resonance delocalized.
:

R NH2 R NH2
Chem i str y | 25.11

(i) Amides are less basic than alkyl amines.


(ii) Preferred site of protonation is on oxygen.
+

:
:O: : OH : OH : OH
C H A C C C +
+ :A-
+

:
:
R NH2 R NH2 R NH2 R NH2

:O: :O:
C H A C +
+ :A-
:

R NH2 R NH3

(d) Heterocyclic Aromatic Amines



+ + :
N N N N
:

H H
H H
pKa = 5.3 pyridine pKa = 0 pyrrole

(i) Pyridine – stronger base – lone pair localized


(ii) Pyrrole – weaker base – lone pair delocalized

(e) Effect of Hybridization


N N N N
H H
H H
pKa = 5.3 pyridine pKa = 11.1 piperidine

(i) Pyridine – weaker base – lone pair in sp2 orbital


(ii) Piperidine – stronger base – lone pair in sp3 orbital

Illustration 5: Give the decreasing order of boiling points for the following:  (JEE MAIN)
Me
I. Et2NH II. Me NH2 III. Et N
Me
Sol: Extent of hydrogen bonding decreases in the order (1º > 2º > 3º amine) thus boiling point also decreases in
the order (1º > 2º > 3º amine)
(II) > (I) > (III) (1º > 2º > 3º amine)

Illustration 6: Arrange the following in the decreasing order of their basic strength:  (JEE MAIN)
a. i. PhNH2 ii. EtNH2 iii. Et2NH iv. NH3
b. i. EtNH2 ii. PhNH2 iii. NH3 iv. PhCH2NH2 v. Et2N

Sol: Extent of hydrogen bonding decreases in the order (1º > 2º > 3º amine) thus boiling point also decreases in
the order (1º > 2º > 3º amine)
2 5 . 1 2 | Amines and Aromatic Compounds Containing Nitrogen

a. Aliphatic 2º amine > Aliphatic 1º amine > NH3: Aromatic amine


(iii) > (ii) > (iv) > (i)
b. (v) > (i) > (iv) > (iii) > (ii)

Illustration 7: Complete the following acid-base reaction and name the products. (JEE ADVANCED)

NH2
(i) Me + HCl (ii) Et3N+ HCl

Sol: (i)

NH3Cl Θ
(i) (Propanammonium chloride)
Me

(ii) Et3N+ Cl− (Triethylammonium chloride)

PLANCESS CONCEPTS

Preparation of amines:
•• reduction of nitro compounds to amines by metals(Fe, Sn, Zn) in dil HCl or SnCl2 or by hydrogen in
presence of Ni, Pt, Pd.
Misconception: LiAlH4 doesn’t reduce the nitro compounds to amines. Instead it reduces it to
R-N=N-R.
•• Amines can also be prepared by the Hoffmann ammonolysis reaction which is a SN2 reaction. NH3
reacts with the alkyl halide to give primary halides which in turn react with more alkyl halides to give
2º and 3 amines and then gives 4º ammonium salts.
Exception: Hoffmann ammonolysis reaction cannot be used to prepare aromatic amines.
•• Amines can also be produced by the reduction of nitriles with LiAlH4 or catalytic hydrogenation. But
note that only primary amines can be produced.
Misconception: Besides NaBH4 does not give primary amines.
•• Amines can also be prepared by the reduction of amides, oximes and azides by LiAlH4 . But again
only primary amines can be produced by this method.
•• Primary amines can also be produced by Gabriel phthalamide synthesis. In this pthalic anhydride
is converted to phthalamide which is then treated with an alkyl halide to give N-alkyl pthalamide.
Finally it is treated with hydrazine to give a primary amine.
Exception: primary aromatic amines cannot be prepared by Gabriel phthalamide synthesis because
aromatic halide doesn’t give a substitution reaction with phthalamide.
•• Primary amines can be prepared by Hoffmann bromamide reaction where the amide is treated with
Br2 in the presence of an alkali to give a primary amine. The intermediate species is nitrene which
undergoes intra rearrangement to give RNH2.
•• Some other methods to prepare primary amines are Lossen rearrangement, Curtius rearrangement,
Claisen-Schmidt rearrangement. All have one thing in common, i.e., they proceed through the
nitrene mechanism.
•• Amines can also be produced by the reductive amination of carbonyl compounds. Carbonyl
compounds give imines which on reduction with H2 in presence of Ni gives an amine. Aldehydes
give primary amines and ketones give secondary amines.
Vaibhav Krishnan (JEE 2009 AIR 22)
Chem i str y | 25.13

5. VARIOUS TESTS FOR AMINES


(a) Hofmann Mustard Oil Reaction (Test for 1º Amines)

S
||
Warm HgCl
2 → 2HCl + HgS + R −=
RNH2 +=
S =
C S  → RNH − C− SH  N =
C S
1º amine Dithioalkyl Alkyl
carbamic acid isothiocyanate

Alkyl isothiocyanates gives a smell like that a mustard oil and this is used as a test for 1º amines.

(b) Carbylamine Reaction (Test for 1º Amine): It consists of heating a mixture of 1º aliphatic or aromatic
amines and chloroform with alcoholic KOH solution. The reaction proceeds via carbene mechanism.
R – NH2 + CHCl3 + 3KOH (Alcoholic)
∆ –
R – N+ ≡ C + 3KCl + 3H2O

Mechanism:
Cl
Θ Θ Θ
OH H C Cl H2O + CCl3 :CCl2 + Cl
Cl

R – NH2 + : CCl2 RN+ ≡ C + 2HCl


Isocyanides have a very offensive smell. So, this reaction is use to distinguish 1º amines.

(c) Hinsberg’s Test (for distinguishing 1º, 2º and 3º amines): Three classes of amines are distinguished by
Hinsberg reagent test. The Hinsberg reagent is a benzene sulphonyl chloride (C6H5SO2Cl).

(d) Reaction of Hinsberg Reagent with Primary Amines: A primary amine forms a precipitate of N-alkyl
benzene sulphonamide with Hinsberg reagent. This precipitate is soluble in alkali.


RNH2 (primary amine) + C6H5SO2Cl (Hinsberg reagent) → R − NH − SO2 − C6H5 
NaOH
→ R − N−Na+ − SO2C6H5 (soluble)

(e) Reaction of Hinsberg Reagent with a Secondary Amine: Secondary amine reacts with the Hinsberg reagent
to form a precipitate N,N-dialkyl benzene sulphonamide. But this precipitate is insoluble in alkalis.
NaOH
R 2NH (sec ondary amine) + C6H5SO2Cl (Hinsberg reagent) → R 2NSO2C6H5 
→ Insoluble (no reaction)
(Precipitate)

(f) Reaction of Hinsberg Reagent with a Tertiary Amine: Tertiary amines do not react with Hinsberg reagent.

R 3N (Tertiary amine) + C6H5SO2Cl (Hinsberg reagent) → No reaction

(g) Reaction with Nitrous acid (For Distinguishing 1º, 2º and 3º amines): Nitrous acid (HNO2 or HONO)
reacts with aliphatic amines in a fashion that provides a useful test for distinguishing, primary, secondary and
tertiary amines.

1º-Amines + HONO (cold acidic solution) → Nitrogen Gas Evolution from a Clear Solution
2º-Amines + HONO (cold acidic solution) → An Insoluble Oil (N-Nitrosamine)
3º-Amines + HONO (cold acidic solution) → A Clear Solution (Ammonium Salt Formation)

Nitrous acid is a Bronsted acid of moderate strength (pKa = 3.3). Because it is insoluble, it is prepared immediately
before use in the following manner.
H2O, 0º
NaNO2 + H2SO4 H—O—N=O + NaHSO4
2 5 . 1 4 | Amines and Aromatic Compounds Containing Nitrogen

Under the acidic conditions of this reaction, all amines undergo reversible salt formation:
1 1
R R
2 2
 Θ
R N: + HX R N H X [X = HSO4 or NO2]
3 3
R R
This happens with 3º-amines, and the salts are usually in water. The reactions of nitrous acid with 1º- and 2º-
aliphatic amines may be explained by considering their behaviour with the nitrosonium cation, NO(+), an electrophilic
species present in acidic nitrous acid solution.

Mechanism of reaction of Nitrous acid with primary amine

H R

HNO2, 0  -H Tautomerism
R_NH Θ
:

:
:
:
2 R N N=O X :N N=O R-N=N-OH
N=O
H H

H
Alcohols  -H2O 
H2 O -N2
R-NN: XΘ

:
:
and R R-N=N-OH2
Alkenes

6. CHEMICAL REACTIONS OF AMINES


6.1 Reaction with Carbonyl Compounds
(a) Imine Formation


:

H  OH H
:

O OH Me H
:

H2N N



OH2
H
 H H
:

N N N
imine
Imines are formed when any primary amine reacts with an aldehyde or ketone under appropriate conditions.
Imine formation requires an acid catalyst, otherwise the reaction is very slow. The acid is needed for the
elimination of water.

(b) Enamine Formation:


H

:

 OH
:

O OH H
H
:

N N


OH2
 H
H
N N
:

N
enamine

Under the appropriate conditions, secondary amines react with carbonyl compounds to form enamines. Like
imine formation, the formation of an enamine also requires an acid catalyst for the removal of water.
Chem i str y | 25.15

6.2 Elimination
(a) Hofmann Elimination: When a primary amine bearing one of more beta hydrogens is treated with methyl
iodide, followed by aqueous silver oxide, followed by heat, the primary amine is converted to an alkene. This
reaction is known as Hofmann elimination, not to be confused with Hofmann rearrangement.
1. 3 eq. CH3I
2. aq. Ag2O
E.g.
CH3CH2NH2  3. ∆
→ CH2 = CH2

The net reaction is 1,2-elimination, hence the name Hofmann Elimination.

H2C CH2 NH2

H
+
H -NH
2

Hofmann elimination occurs in three stages.


+
3 CH3I
CH3CH2NH2 Stage 1 CH3CH2N(CH3)3I-

Stage 2 aq. Ag2O

 +
CH2 = CH2 CH3CH2N(CH3)3 -OH
Stage 3

Mechanism:
Stage 1:
CH3

NH2 + I -
:

CH3CH2 NH2 + CH3 I CH3CH2


+
CH3 CH3
+ +
:

CH3CH2 N H + H2NCH2CH3 CH3CH2 N: + H3NCH2CH3

H H
CH3 CH3
+
CH3CH2 N: + CH3 I CH3CH2 N H + I-

H CH3
CH3 CH3
+ +
:

CH3CH2 N H + H2NCH2CH3 CH3CH2 N: + H3NCH2CH3

CH3 CH3
CH3 CH3
+
CH3CH2 N: + CH3 I CH3CH2 N CH3 I -

CH3 CH3

Stage 2:
Ag2O (s) + H2O AgOH (s)

+
+ CH3CH2N(CH3)3I -
CH3CH2N(CH3)3 -OH + AgI (s) Ag+ + -OH
2 5 . 1 6 | Amines and Aromatic Compounds Containing Nitrogen

Stage 3:
+ +

:
H2C CH2 N(CH3)3 + - : OH

: :

: :
CH2 CH2 N(CH3)3 + H2O

H
CH2 = CH2 + N(CH3)3
Stage 3 is a 1,2-elimination via E1CB mechanism. Hofmann elimination is regioselective. Since the 1,2-elimination
in Stage 3 occurs via E1CB mechanism. Hofmann rule is used to predict the major product.
eg.
1. 3eq. CH3I
2. aq. Ag2O major
3. 
NH2
minor

(b) Cope’s Elimination: Tertiary amine oxides undergo the elimination of a dialkylhydroxylamine when they are
heated. This reaction is called the Cope elimination:
:

:O:-
:

:OH:
RCH CH N+_ CH
2 2 3 RCH=CH2 + :N CH3
150
CH3 An alkene CH3
A tertiary amine N,N-Dimethyl-
oxide hydroxylamine

The Cope elimination is a syn elimination and proceeds through a cyclic transition state:
R
CH-CH2 CH R
3

N R-CH=CH2 + N
H
:

+
: :

-:O CH H-O CH3


: :

Tertiary amine oxides are easily prepared by treating amines with hydrogen peroxide.
The Cope elimination is useful synthetically. Consider the following synthesis of methylenecyclohexane:
H : O:-
:

160C
:

CH2+(CH3)2NOH
N+
CH3
CH3

6.3 Reaction with Acid Halides and Anhydrides


O O
+
C + NH3 C + NH4 Z-
(2 equiv)
R Z R NH2
1 amide
O O
+
C + R’NH2 C + R’NH3 Z-
(2 equiv)
R Z 1 amide R NHR’
2 amide
O O
+
C + R’2NH C + R’2NH2 Z-
(2 equiv)
R Z 2 amide R NR’2
Z=Cl or OCOR 3 amide
Chem i str y | 25.17

6.4 Electrophilic Substitution Reaction for Aniline


Electrophilic substitution: Due to +M effect of –NH2 generate electron density at ortho and para position hence,
aniline active toward electrophilic substitution
(a) Bromination: Aniline reacts with bromine water at room temperature to give a white precipitate of
2,4,6-tribromoaniline.
NH2 NH2
Br Br
Br2/H2O
+ HBr2 + 3HBr

Aniline Br
If we have to prepare a monosubstituted aniline derivative. This can be done by protecting the –NH2 group by
acetylation with an acetic anhydride, then carrying out the desired substitution followed by the hydrolysis of
the substituted amide to the substituted amine.
O O
NH2 H-N-C-CH3 H-N-C-CH3 NH2
1
(CH3CO)2O Br2 2
Pyridine CH3COOH OH- or H+ 3
Aniline N-Phenylethanamide Br Br
(Major) 4-Bromoaniline
The lone pair of electrons on nitrogen of acetanilide interacts with oxygen atom due to resonance as shown
below:
:O: :O:
+
:

N C CH3 N=C CH3

Hence, the lone pair of electrons on nitrogen is less available for donation to the benzene ring by resonance.
Therefore, activating the effect of –NHCOCH3 group is less than that of amino group.
(b) Nitration: Direct nitration of aniline is not possible because in the strongly acidic medium, aniline is protonated
to form the anilinium ion which is meta directing.

NH2 NH2 NH2 NH2


NO2
HNO3 +H2SO4,
+ +
NO2
NO2 47% 2%
51%

However, by protecting the –NH2 group by acetylation reaction with acetic anhydride, the nitration reaction
can be controlled and the p-nitro derivative can be obtained as the major product.

NH2 NHCOCH3 NHCOCH3 NH2


-
OH or H+
(CH3CO)2O HNO3+H2SO4, 288K
Pyridine

NO2 NO2
p-Nitroacetanilide p-Nitroaniline
2 5 . 1 8 | Amines and Aromatic Compounds Containing Nitrogen

(c) Sulphonation: + - +

:
NH2 NH3 HSO4 NH2 NH3

H2SO4 453-473K

Anillinium hydrogensluphate -
SOH SO3
Sulphanilic acid Zwitter ion

PLANCESS CONCEPTS

Chemical reaction of amines:


•• Tips and tricks: Only primary amines react with carbonyl compounds to give imines while secondary
amines react with carbonyl compounds to give enamines.
•• Tips and tricks: In Hoffman elimination, the alkene which is substituted least is the major product while
in Cope’s elimination the alkene which is most substituted is the major product..
•• Tips and tricks: In the Hoffmann elimination, there is anti-elimination while in Cope’s elimination, there
occurs syn elimination.
•• In electrophilic substitution of bromine on aniline, if the medium is CS2 then only 1 Br is substituted while
in aqueous solution, the substitution of bromine occurs 3 times.
•• If bromination is to be done once in aqueous solution then acetylation of bromine is carried out due to
which the ring becomes less activated.
•• In nitration, the major products are para and metanitroaniline while the ortho product is very less. This is
because of the acid base reaction between aniline and nitrous acid due to which some of aniline becomes
meta directing.
Vaibhav Krishnan (JEE 2009 AIR 22)

Illustration 8: Complete the following reactions:  (JEE MAIN)

Me
(a) NH2
NaNO2 + HCl NH2
Me (B) Me HNO2
(b) (D)
Me
(A)
(C)

Sol: Attack of nitrous acid forms a primary carbocation which gets rearranged to more stable secondary carbocation.
Attack of nucleophile (OH-) result in formation of secondary alcohol which on treatment with acid forms alkene.
(a) NH2  
HNO2
Me Me CH2 Me Me
 More stable
(A) 1C
2C 

OH

OH

H Me Me
Me
Since the conditions
(B) are acidic, H2O is
Propene
lost (-H2O).
Chem i str y | 25.19

(b) Me 
Me CH2 1,2-Me
HNO2
Me NH2
Me shift
Me Me
(C) 1C

Me 1 2 3 4  Me
H Me OH
Me 
Me -H O Me Me
Me 2

2-Methyl
Me More stable
3C
OH
but-2-ene
(D)

Illustration 9: Explain:  (JEE ADVANCED)


Br

I. Dehydrohalogenation of
Me Me

Me 

II. Hofmann elimination of


Me NMe3
OH

 −
Sol: E2 elimination of an alkyl halide with base  Et O  gives mainly Saytzeff alkene (i.e., more-substituted alkene)
 
 
Alc.
I. Me Me + Me
KOH
(70%) (Mostly trans) + (30%)

II.  → 3% (cis and trans) + (97%)
 −
E2 elimination of an alkyl halide with the base  Et O  gives mainly a Saytzeff alkene (i.e., more-substituted alkene),
 
 
whereas 4º ammonium salt undergoes Hofmann elimination to give a less-substituted alkene, resulting from a loss
of a more acidic β -H (1º>2º>3º) called Hofmann’s rule.
Thus, the acidity of β –H is more important than the stability of the alkene that is formed.

Illustration 10: Give the products of the following by the application of Hofmann’s exhaustive methylation and
elimination:  (JEE ADVANCED)

Me

(i) (ii) (iii) HN O


N
N
H

Sol: This process involves thermal decomposition of quaternary ammonium hydroxide compounds to yield an
olefin and water is known as Hofmann’s exhaustive methylation reaction. This method has been employed in
identifying site of unsaturation present in the given compound.
2 5 . 2 0 | Amines and Aromatic Compounds Containing Nitrogen

(i) 
Me H  Me Me
MeI H
OH

AgOH 
N N N Me
Me Me
H Me

Me  Me
MeI
 OH
  AqOH
Me N Me
Me

(ii) 6 4
  5
 7 3

  8 2
N 9
  1
(Nona-1,4,8-triene)

O
(iii)   O

 
N
(Divinyl ether)
N

7. DIAZONIUM SALTS
7.1 Introduction
The diazonium salts have the general formula RN2X where R stands for an aryl group and X ion may be Cl, Br, HSO4,
BF3 etc.
Resonance of benzene diazonium ion is
   
:
:

:
:

NN : NN : NN : NN :

7.2 Preparation
273-278 K
PhNH + NaNO2 + HCl


Ph-NN Cl +NaCl+2H2O
Benzene diazonium
chloride (1)
Chem i str y | 25.21

7.3 Chemical Reactions Involving Displacement of Nitrogen


HOH ArOH+H
 Slow  CN
ArN2-X Ar-N2 Ar Ar-CN
-N2
+ X
ArX
X
Boiling
(a) Replacement by ‒OH group: ArN2Cl + HOH  → ArOH + N2 + HCl

(b) Replacement by H: ON reduction with sodium stannite (Na2SnO2) (alkaline SnCl2) or hypophosphorous acid
(H3PO2) or on warming with C2H5OH, ArN2Cl gives benzene.
ArN2Cl+H3PO2+H2O → ArH+N2+2CuCl2
ArN2Cl+Na2SnO2+H2O → ArH+N2+ Na2SnO3+HCl

(c) Sandmeyer reaction (replacement by halogen):


ArN2Cl+Cu2Cl2+HCl → ArCl+N2+2CuCl2
ArN2Br+Cu2Br2+HBr → ArBr+N2+2CuBr2

(d) Gattermann reaction:


Finely divided Cu/HCl
ArN2Cl → ArCl+N2
Finely divided Cu/HBr
ArN2Br → ArBr+N2
Boiling
(e) Iodo compounds: ArN2Cl+KI(Aq.)  → Ar–I +N2+KCl

(f) Balz-Schiemann reaction:



NaNO2+HBF4
Ar-N2BF4 
(Fluoroboric acid)
Ar-NH2 Tetrafluoroborate
273-298K
salt
ArF+BF3+N2

(g) Cyano compounds:


CuCN
ArN2Cl+KCN 
or Cu powder
→ ArCN+N2+KCl
ArN2Cl+KCN 
K [Cu(CN) ]
→ ArCN+N2+KCl
3 4

(h) Nitro compounds:


Cu O
ArN2Cl+HONO →
2 ArNO2+N2+HCl
ArN2Cl+CuNO2 

→ ArNO2+N2+CuCl
⊕⊖ ∆
ArN2BF4+NaNO2 Cu powder Ar–NO2+NaBF4+N2

(i) Gomberg reaction (replacement by an aryl group)

ArN2Cl+PhH+NaOH Ar–Ar+N2+NaCl+H2O
Benzene Diphenyl

2 5 . 2 2 | Amines and Aromatic Compounds Containing Nitrogen

ArN2Cl+PhH+NaOH Ar-Ar+N2+NaCl+H2O
PhH+NaOH Ar-Ar+N 2+NaCl+H2O
Benzene Diphenyl
nzene Diphenyl 
 Br N2Cl+C6H6+NaOH Br + N2+NaCl+H2O
Br N2Cl+C6H6+NaOH Br + N2+NaCl+H2O

+
+ PhN2 Cl + NO2+NaOH NO2+N2+NaCl+H2O
PhN2 Cl + NO2+NaOH NO2+N2+NaCl+H2O

Whatever is the nature of the substituent in the second component, o-and p-substitutions occur and the
reaction takes place by the free radical mechanism.
 OH  
PhN2Cl Ph-N=OH Ph +N2+ OH

Ph +PhNO2+ OH

NO2+H2O

( j) Meerwein Reaction:
(i)I. PhN2Cl+CH2=CH-CN(Acrylonitrile)


(ii) 
II. Ph +CH2=CHCN PhCH2CHCN
2+
Cu
Cl
 Cl
PhCH2CHCN + Cu PhCH2CH-CN

(iii) Addition to α, β-unsaturated acid is accompanied by decarboxylation.


ArN2Cl+PhCH=CH-COOH
Cinammic acid

PhCH=CH-Ar+N2+CO2+HCl
(Stilbene)

(iv) Mechanism:

PhCH=CH-COOH ArCl
-CO2
and Ph-CH-CH-COOH
-HCl Cl Ar
Ph-CH=CH-Ar

Reduction:

Zn+HCl Zn+HCl
Ar–N=N–Cl [Ar–NH.NH2]
[H] [H]
Ar–NH2+ NH3
Chem i str y | 25.23

7.4 Coupling Reactions (Retention of Diazo Group)


 OH, pH 9-10
(a) Ph-NNCl + H OH 273-278K

Ph-N=N OH-HCl
p-Hydroxyazobenzene
(Orange)

 
H ,pH 4-5
(b) Ph-NNCl +H2N

H, 
Rearranges Ph-N=N-NH + HCl

Ph-N=N NH2
p-Amino azobenzene
(Yellow)

(c)
 NaNO2/HCl
NaO3S NH2 273-278K
Sodium salt of sulphanilic acid

N,N-Dimethyl aniline  
NaO3S NNCl
OH
273-278K

 Me
NaO3S N=N N + HCl
Me

(d) With excess of diazonium salts, the bisazo (o- and p-) and the trisazo compounds may be produced.
Ph-N=N-Cl+PhOH

PhN2Cl
OH N=N-Ph

OH N=N-Ph
PhN2Cl
N=N-Ph
Bisazo compound
OH
Ph-N=N N=N-Ph

N=N-Ph
The introduction of a second azo group is facilitated by the presence of an alkyl group in the para-position to
the hydroxyl group or by two (–OH) groups in the m-position.
2 5 . 2 4 | Amines and Aromatic Compounds Containing Nitrogen

(e) Dye test (test for 1º aromatic amine):



Coupling position
OH
273-278 K  OH
Ar-NH2+HONO+HCl Ar-NNCl + N=N
Dil. NaOH

 - Naphthol
1-Phenylazo-2-naphthol
(Orange dye)
Coupling with α -or 1-naphthol takes places at the position shown by an arrow.
OH

(Coupling position)

(f) 
In case, a compound contains both (–OH) and (–NH2) groups, the coupling takes place at o-position of the
amino group in the acid solution.
In the alkaline solution, coupling takes place at o-position to the (–OH) group.
OH OH OH
N=N-Ph
PhN2Cl PhN2Cl
Alkaline Acid
solution solution N=N-Ph
NH2 NH2 NH2

(g) 
In cases where a (–COOH) or (–SO3H) group is present at p-position to (–OH) or to (–NH2). it is an example of
ipso substitution, special case of electrophilic aromatic substitution where the leaving group is not hydrogen.

OH OH OH

PhN2Cl PhN2Cl
pH=9-10 pH=9-10

COOH N=N-Ph SO3H

NH2 NH2 NH2


PhN2Cl PhN2Cl
pH=4-5 pH=4-5

COOH N=N-Ph SO3H


Chem i str y | 25.25

PLANCESS CONCEPTS

•• Diazonium salts of alkanes, alkenes and alkynes are not at all stable at room temperature while benzene
Diazonium salts are stable to some extent due to delocalization of the positive charge in the benzene ring.
•• Whenever you see a nucleophile with benzene Diazonium salt, substitute it in place of N2, as it is a very
good leaving group.
•• Benzene Diazonium salt gives an orange red dye with β -naphthol due to extended conjugation and is
hence a test for it.
•• Being an electron deficient species benzene Diazonium salt shows a coupling reaction with an electron rich
species such as phenol and amines.
Nikhil Khandelwal (JEE 2009 AIR 94)

Illustration 11: Give the decreasing order of the reactivity of the diazonium ion coupling with the phenol. 
 (JEE MAIN)
⊕ ⊕ ⊕ ⊕ ⊕
(I) p-NO2–C6H4 N2
(II) p-Cl–C6H4 N2 (III) C6C5 N2 (IV) p-Me–C6H4 N2 (V) p-MeO–C6H4 N2

Sol: The more Electron withdrawing group in diazonium ion, the faster the coupling is.
The more EWG in diazonium ion, the faster the coupling is.
(I) > (II) > (III) > (IV) > (V) (I) ⇒ (p-NO2, –I, and –R), (II) ⇒ (p-Cl, –I)
(III) ⇒ Standard (IV) ⇒ (p-Me, +I, and H.C.)
(V) ⇒ [p-MeO–, –I, and + R, net ED power of Me–O is greater than ED power of (Me–) group
Illustration 12: Starting from benzene or toluene or aniline and with the aid of the diazonium salt synthesise the
following: p-Nitrobenzene (JEE MAIN)

Sol: We are going to prepare p-Nitrobenzene from Aniline. Since, –NH2 is o- and NH2 NO2
p- directing, introduce one (–NO2) group directly by nitration after protecting
(–NH2) group (direct nitration of aniline would yield mixture and tarry products). (a)
Nitration is done using the nitrating mixture (conc.HNO3+conc.H2SO4).Next step is
deprotection which is done by using an aqueous base. Now we can introduce the
NO2
second (NO2) group via diazonium salt method.

NH2 NHAc NHAc NH2

Ac2O HNO3 OH/H2O


+H2SO4

NO2 NO2
Major

NO2 [NN]Cl

NaNO2 NaNO2
+Cu2O +HCl
0-5
2 5 . 2 6 | Amines and Aromatic Compounds Containing Nitrogen

Illustration 13: Explain why 2,4-dinitrobenzene diazonium ion couples with anisole but Ph N 2⊕ does not. Write the
coupling reaction. (JEE ADVANCED)

Sol: The ring is not sufficiently activated by –OMe group for it does not
NN+H

:
O2N OMe
react with most Ph N 2⊕ However, e -withdrawing (–NO2) groups make
this diazonium ion less stable and thus more reactive than Ph N 2⊕ . NO2 Anisole

The ring is not sufficiently activated by –OMe group for it does not
react with most Ph N 2⊕ . However, e -withdrawing (–NO2) groups make O2N NN OMe
this diazonium ion less stable and thus more reactive than Ph N 2⊕ .
NO2

Illustration 14: Convert the following: (JEE MAIN)

Aniline H2N SO2NH2


Sulphanilamide

O O
NH3
Sol: PhNH2 Ac2O
PhNHAc
HO-SO2-Cl
Cl S NHAc H2 N S NHAc 1. H3O
Chlorosulphonic
2. OH
acid
O O
O
H2N S NH2

POINTS TO REMEMBER

Preparation of amines

Nucleophilic Reduction of nitrogen Rearrangement


Reductive amination
Substitution containing reactions
functional groups

Hoffmann’s Claisen-Schmidt
Ammomolysis From
- NO₂ (nitro)
compounds Lossen
Gabriel
Phthalimide From
Hoffmann
nitrites
bromamide

From
amides Curtius

From
Ritter
others
Chem i str y | 25.27

Solved Examples

JEE Main/Boards HNO2


3 4
OH 5 OH
2 
Example 1: Distinguish between the following pairs: NH2 CH2
1
(I) (II)

a. (PhNH3 )2+ SO24− and H3N SO3 3
4

-H 5
O OH
 ⊕  2
b. Me4NΘClΘ and Me3 NH ClO– 1
  Cyclopentanone
(X)
Sol: First two compound can be differentiated by using
BaCl2 solution as barium will form a ppt with sulphate
ion. Other two compound as can be distinguished by Example 4: + I-N=C=O (VIII)
H2 O
(IX)
treated with NaOH.
a. Add BaCl2 solution. (I) is a sulphate salt and will
give a white precipitate of BaSO4. (II) is a sulphonate Sol: First step will lead to formation of an isocyanate.
(sulphonic acid salt) and will not give any precipitate. On treatment with water it yields primary amine.

b. Add concentrated NaOH and heat the mixture. (II will 


change to a volatile free base Me3N. which has typical + I-N=C=O (I N=C=O)
NH3 odour. Compound (I) will not react.
I I
H2 O
Example 2: Complete the following reactions: NH2 N=C=O
Zn+ aq. NH Cl
(VIII)
4 → (I)
a. PhNO2  (IX) 2-Iodocyclopentyl
isocyanate
Al O /aq. NaOH
b. p-Me–C6H4NO2 
2 3 → (II)
Example 5: Give the reagents in the following reactions:
Sol: a. (I) PhNHOH (N-Phenyl hydroxylamine)

 Me Me Me
b. (II) Me N=N Me
Br Br
(1)? (2)?
O
p-Methyl azoxy benzene (3)?
(4)?
NO2 NO2 Br
Example 3:

2. Sn + HCl;OH
HNO2
OH (X) Sol: 1. Br2 + Fe
NH2 3. HNO2 + 0-5ºC 4. CuBr

Sol: It is an example of pinacole-pinacolone type ring Example 6:


expansion which takes place via carbocation.
Cl
AlCl3 1. HNO3/H2SO4
PhH+Me (C) (D)
2. Sn/HCl
(A) Me
3.OH
(B)
2 5 . 2 8 | Amines and Aromatic Compounds Containing Nitrogen

Sol: First step is Friedal craft alkylation. This step Example 8: Identify compounds (A) to (E) in the
produces a new chiral centre on the product but it is following:
optically inactive. The alkylated product is subjected
Sn/HCl 1. HNO3, 5C
to nitration to introduce nitro group at para position p-NO2C6H4OEt (A)
2. PhOH
(B)
on treatment with reducing agents like Sn+HCl, nitro
group gets reduced to amino group. MeCOCl SnCl2 OH
(E) (D) (C)
Me
*
Et2SO4
Me (C10H13O2N)
Me
HNO3+ Sn+HCl
(C) Ph Me Sol:
H2SO4
OH

NO2 OEt OEt


()or Racemate
Sn/HCl 1. NaNO2, 5C
Me
* Me 2. PhOH

NO2 NH2
(A)
NH2
()or Racemate EtO N=N OH
(D) (B)
OH+Et2SO4
Example 7: Explain the formation of the mixture
PhCH2CHO (I) and PhCOMe (II) when PhCH(OH)
CH2NH2(A) is treated with HNO2. EtO N=N OH
(C)
Sol: Attack of Nitrous acid produces primary carbocation
[H] SnCl2
which gets rearranged to secondary carbocation and
tertiary carbocation. Due to formation of two different
carbocation we get different product. With secondary 2EtO NH2
carbocation we end up getting PhCH2CHO and with
tertiary carbocation we get PhCOMe. (D)

OH MeCOCl
OH
NH2
HNO2 1,2-H
Ph Ph CH2 Shift 2EtO NH COMe
(A) 1,2-Ph
shift
(E)
OH Phenacetin
OH (Analgesic and antiopyretic)
-H Ph CH3
H2 O
CH2-Ph H2O -H
O=HC-H2C-Ph
O
(I)

Ph CH3
Chem i str y | 25.29

Example 9: Convert benzene to o-nitro aniline as the amine on treatment with benzene sulphonyl chloride
only product. its derivative.

Sol: 
(A) Ph-NH2 Cl (B) ⇒ Ph—NH—CH3
+
NO2
NO2 NH2
HNO3/H2SO4 Sn/HCl CH3

CH3COCl Base
O
NH O NH O (C) Ph N S Ph (insoluble in NaOH)
Conc.
H2SO4 CH3 O
CH3 CH3
SO3H

HNO3/H2SO4
O CH3 Example 2: An organic compound (A) of molecular
weight 135 on boiling with NaOH evolves a gas which
NH O NH
Steam gives dense white fumes on bringing a rod dipped
in HCl near it. The alkaline solution thus obtained on
CH3
SO3H NO2 NO2 acidification gives the precipitate of a compound (B),
H 3O having molecular weight 136. Treatment of (A) with
Hydrolysis HNO2 also yields (B), whereas its treatment with Br2/
KOH gives (C). Compound (C) reacts with cold HNO2 to
NH2
give (D) which gives red colour with ceric ammonium
nitrate. On the other hand, (E) an isomer of (A) on boiling
NO2 with dilute HCl gives an acid (F), having molecular
weight 136. On oxidation, followed by heating, (F) gives
an anhydride (G) which condenses with benzene in the
Example 10: m-Me–C6H4NO2 
LAH
→ (III) presence of AlCl3 to give anthraquinone, Give structures
of (A) to (G) with proper reasoning.
Sol: (III) is
Sol:
:
:

Me N=N O
(Anti) NaOH
Ph CH2 C NH2 PhCH2COONa NH3

Me p-Methyl (A)
and
azobenzene Mw=135
HNO2 PhCH2COOH
Me N=N Me
Br2-KOH NH2 -OH (B)
(Syn) Mw=136
Cold
PhCH2NH2 PhCH2OH
IINO2
(C) (D)
JEE Advanced/Boards (Test for alcohol)
(Positive ceric ammonium)
nitrate test
Example 1: The aqueous solution of a nitrogen and
chlorine containing organic compound (A) is acidic O
to litmus. (A) on treatment with aqueous NaOH, it HCl
C NH2 COOH(Mw=136)
gives a compound (B) containing nitrogen, but not
chlorine. Compound (B) on treatment with C6H5SO2Cl CH3 CH3
in the presence of NaOH gives an insoluble product (C) (E) (F)
C13H13NO2S. Give the structures of (A) and (B).
Oxidation
O
Sol: Quaternary ammonium salt on treatment with
aqueous NaOH gives secondary amine. Secondary 
C COOH
H2 O
C O COOH
(G)
O [H] SnCl2
HCl
NH2 and Aromatic Compounds
2 5 . 3 0 | CAmines COOH(Mw=136)
Containing Nitrogen
2EtO NH2
CH3 CH3
(E) (F) (D)
Oxidation MeCOCl
O

C COOH 2EtO NH COMe
H2 O
C O COOH (E)
(G) Phenacetin
O (Analgesic and antipyretic)

O O

AlCl3 Example 4:
+O
F.C. Me Me
H OC Me
OH
O (1)? (2)?
H2O H2SO4
(3)?
Br Br Br Br
O NH2 NH2

Sol:
1. Br2 + Fe
O 2. HNO2, 0-5ºC
Anthraquinone
3. H3PO2

Example 3: Identify compounds (A) to (E) in the following: Example 5:


Me Me
1.HNO2 .5o C
Sn/HCl
P − NO2C6H4 OEt 
→ (A)  → NO2 NH2
2.PhOH (1)? (2)?
Θ
OH → (C) →
2 SnCl
MeCOCl
(B)  (D)  →(E)(C10 H13O2N) Me Me
Et2SO 4
NHAC NHAC
(3)?
OEt OEt
O2N
1. NaNO2, 5C (4)?
Sn/HCl
2. PhOH Me Me
NH2 NH2
NO2 NH2 (5)?
(A) O2N (6)?
O2N Br
(7)?
EtO N=N OH
(B) Me Me
Br Br
OH+Et2SO4

(8)?
Br O2N Br
(9)?
EtO N=N OEt (10)?
(C)
[H] SnCl2
Sol: 1. Sn + HCl 2. Ac2O
O

3. HNO3 + H2SO4 4. OH
2EtO NH2

(D)
MeCOCl
Chem i str y | 25.31

5. Br2 + Fe 6. HNO2, 0-5ºC Sol:


O

i. Compound (A) is insoluble in water and burns with a
7. CuBr 8. Sn + HCl; OH
smoky flame; hence it should be an aromatic compound.
9. HNO2, 0-5ºC 10. H3PO2 ii. It has no specific element such as N, S and halogens
and gives a CO2 gas with NaHCO3 solution; hence it
Example 6: should contain (–COOH) group.
iii. It gives the oil of wintergreen (methyl salicylate) with
Me CH3OH in acidic medium; hence it is salicylic acid.
O2N C Cl OH O OH O
E2NH + (C)
O
(A) (B) OH CH3OH/H OCH3
H2+Ni
(E) Room temp.
(D)
Compound (A) Methyl salicylate
Novocaine
(Local anaeshetic)
iv. Compound (B) is water soluble and burns with a
Sol: non-smoky flame; hence it should be an aliphatic
compound.
v. This compound has less carbon content because its
:

E2NH +
O sodium extract is prepared with sucrose and it gives a
(A) Prussian blue colour with the freshly prepared solution
(B)
of FeSO4 + 2-3 drops of NaOH and a few drops of
Et2N O H Cl C NO2 H2SO4; hence it is a nitrogen-containing compound.
(C) O NaCN+FeSO4 → Na4[Fe(CN)6]+Na2SO4
[O]
Et2N H2C H2C O C NO2
FeSO4  → Fe2(SO4)3
H2+Ni O
3Na4 [Fe(CN)6 ] + 3Fe2 (SO 4 )3 → Fe4 [Fe(CN)6 ]3 + 6Na2SO 4
(Pr ussian blue colour)
Et2N (CH2)2 O C NH2
O
vi. On heating, it gives ammonia gas, which turns red
(D)
litmus blue. Hence, it contains (-CONH2) group.
Example 7: A mixture of two organic compounds is
vii. It also gives the biuret test.
added to cold water. After filtration, water-insoluble
compound (A) burns with a smoky flame and it does ∆
2NH2CONH2  → NH2CONHCONH2 + NH3 ↑
not respond to Lassaigne’s and Beilsteins test. When Biuret
a small amount of this is added to NaHCO3 solution, a Cu2 +
colourless gas is evolved with effervescence. When this NH2CONHCONH2 → Violet colour
compound is heated with CH3OH in acidic medium, it
gives the characteristic smell of the oil of wintergreen. Hence, this compound is urea (NH2CONH2).
Compound (B), which is water soluble, burns with a
non-smoky flame and its sodium extract is prepared Example 8: Give the structural formula of a chiral
with cane sugar. It gives a Prussian blue colour with compound C8H11N (X), which dissolves in dilute HCl and
freshly prepared solution of FeSO4 + 2-3 drops NaOH evolves N2 gas with HNO2.
and with few drops of H2SO4. When a small amount of (2nc + 2) − (nH − nN ) 18 − 10
this compound is heated in a dry test tube, a colourless Sol: D.U. in (X) = = = 4º
2 2
gas is evolved that turns moist red litmus paper i. 4 D.U. is (X) suggests that is contains benzene ring.
blue and a white residue is left. This white residue is
dissolved in water and a drop of CuSO4 is added in the ii. (X) is a 1º amine since it dissolves in HCl and gives N2
basic medium-a violet colour is obtained. Identify the with HNO2.
compounds (A) and (B) with the help of the reactions iii. The remaining C atom and the NH2 must form a
involved. chiral molecule. So, (X) is:
2 5 . 3 2 | Amines and Aromatic Compounds Containing Nitrogen

Me

Ph * NH2

Example 9: Identify compounds (A) through (E) in the


following:

1. OH Zn/HCl
p-NO2C6H4OH 
2. EtBr
→ (A)  → (B)

NaNO /HClPhOH LiAlH


2
→
5ºC
(C) → (D) 
4 → (E)+(F)

(F) dissolves in NaOH.

Sol:
OH ONa OEt OEt OEt OEt

OH EtBr Zn/HCl NaNO2+HCl at 5C


(or) (C)
NaOH


NO2 NO2 NO2 NH2 [NH3]Cl NN-Cl
(A) PhOH
(B) Coupling

OH OEt OEt

LiAlH4
+

NH2 NH2 NN OH


(F) (E)
(D)
(Soluble in NaOH)

JEE Main/Boards

Exercise 1 Q.2 Describe a method for the identification of primary,


secondary and tertiary amines. Also write chemical
Q.1 Arrange the following: equations of the reactions involved.

(i) In decreasing order of the pKb values:


Q.3 Explain Hofmann Bromamide reaction with
C2H5NH2, C6H5NHCH3, (C2H5)NH and C6H5NH2 Mechanism.
(ii) In increasing order of basic strength.
Q.4 Why cannot aromatic primary amines be prepared
C6H5NH2, C6H5N(CH3)2, (C2H5)2NH and CH3NH2
by Gabriel phthalimide synthesis ?
(iii) In increasing order of basic strength:
(a) Aniline, p-nitroaniline and p-toluidine Q.5 Write the reactions of (i) aromatic and (ii) aliphatic
primary amines with nitrous acid.
(b) C6H5NH2, C6H5NHCH3, C6H5CH2NH2,
Chem i str y | 25.33

Q.6 Write one chemical reaction each to illustrate the Q.15 Write the method of formation of benzene
following diazonium chloride
(i) Hofmann Bromamide reaction.
Q.16 Account for the following:
(ii) Gabriel Phthalimide reaction
(i) Diazonium salts of aromatic amines are more stable
than those of aliphatic amines.
Q.7 Assign a reason for the following statements
(ii) Gabriel phthalimide synthesis is preferred for
(a) Alkylamines are stronger bases than arylamines.
synthesizing primary amines.
(b) How would you convert methylamine into
ethylamine?
Q.17 How will you convert:
(i) Ethanoic acid into methanamine
Q.8 Illustrate the following with an example of reaction
in each case: (ii) Hexanenitrile into 1-aminopentane
(i) Sandmeyer reaction (iii) Methanol to ethanoic acid
(ii) Coupling reaction (iv) Ethanamine into methanamine
(v) Ethanoic acid into propanoic acid
Q.9 Write the chemical reaction equations for one
(vi) Methanamine into ethanamine
example each of the following
(vii) Nitromethane into dimethylamine
(i) A coupling reaction
(viii) Propanoic acid into ethanoic acid?
(ii) Hofmann’s bromamide reaction
(iii) Aryl cyanides cannot be formed by the reaction of
Q.18 Write short notes on the following:
aryl halides and sodium cyanide.
(i) Coupling reaction
Q.10 Account for the following: (ii) Ammonolysis
(i) Aniline is weaker base than methylamine. (iii) Acetylation
(ii) Aryl cyanides cannot be formed by the reaction of (iv) Gabriel phthalimide syntheisis
aryl halides and sodium cyanide.
Q.19 Accomplish the following conversions
Q.11 Describe tests to distinguish between: Secondary
(i) Nitobenzene to benzoic acid
amine and tertiary amine.
(ii) Benzene to m-bromophenol
Q.12 Account for the following observations: (iii) Benzoic acid to aniline
(i) pKb for aniline is more than that for methylamine. (iv) Aniline to 2,4,6-tribromofluorobenzene
(ii) Methylamine solution in water reacts with ferric (v) Benzyl chloride to 2-phenylethanamine
chloride solution to give a precipitate of ferric hydroxide.
(vi) Chlorobenzene to p-chloroaniline
(iii) Aniline does not undergo Friedel Crafts reaction.
(vii) Aniline to p-bromoaniline
(viii) Benzamide to toluene
Q.13 State the reactions and reaction conditions for the
following conversion (ix) Aniline to benzyl alcohol.
(i) Benzene diazonium chloride to nitrobenzene.
Q.20 Write the equation of Curtius reaction with
(ii) Aniline to benzene diazonium chloride.
mechanism?
(iii) Ethyl amide to methylamine.
Q.21 Complete the following reactions:
Q.14 Write the physical property of aniline
(i) C6H5NH2 + CHCl3+alc. KOH →
2 5 . 3 4 | Amines and Aromatic Compounds Containing Nitrogen

(ii) C6H5N2Cl + H3PO2 + H2O → Q.2 Examine the following two structures for the
anilinium ion and choose the correct statement from
(iii) C6H5NH2 + H2SO4 (conc.) →
the ones given below.
(iv) C6H5N2Cl + C2H5OH → +
NH3 NH3
(v) C6H5NH2 + Br2(aq) →
+
(vi) C6H5NH2 + (CH3CO)2O →

(A) II is not an acceptable canonical structure, because


Q.22 Give possible explanation for each of the following:
carbonium ions are less stable than ammonium ions
(i) Why are amines less acidic than alcohols of
(B) II is not an acceptable canonical structure, because
comparable molecular masses?
it is non aromatic
(ii) Why do primary amines have higher boiling point
(C) II is not an acceptable canonical structure, because
than tertiary amines?
the nitrogen has 10 valence electrons
(iii) Why are aliphatic amines stronger bases than
(D) II is an acceptable canonical structure.
aromatic amines?

Q.3 The correct order of basic strength in CCl4


Q.23 Write the reaction and conditions for the following
conversions (1) NH3 (2) RNH2 (3) R2NH (4) R3N
(i) Aniline to benzene where R is CH3 group is
(ii) Methylamine to methyl cyanide (A) 3>2>1>4 (B) 2>3>4>1
(iii) Propanenitrile to ethylamine (C) 3>2>4>1 (D) None of these
(iv) m-Bromoaniline to m-bromophenol
Q.4 Place the following in the decreasing order of
(v) Nitrobenzene to 2,4,6-tribromoaniline.
basicity.
(1) Ethylamine (2) 2-aminoethanol
Q.24 Write the method of formation of zwitter ion?
(3) 3-aminopropan-1-ol
Q.25 Explain nitration of aniline? (A) 1>3>2 (B) 1>2>3
(C) 2>1>3 (D) None of these
Q.26 Why aniline does not give Friedel-Crafts reaction?
Q.5 Which of the following will give a positive
Q.27 How will you convert 4-nitrotoluene to carbylamine test?
2-Bromobenzoic acid?
(A) H3CNH2 (B) H3C–NH–CH3
Q.28 Draw the structure of trimethylamine and tell the (C) (CH3)3N (D) C6H5NH2
shape of the molecule. Show the angle between two
methyl groups. Q.6 Isopropylamine can be obtained by
LiAlH
(A) (CH3)2CHO + NH2OH → ? 
4→
Exercise 2
H /Ni
(B) (CH3)2CHO + NH3 → ? 
2 →
Single Correct Choice Type ∆

(C) CHOH + NH3 →


Q.1 When aniline is treated with fuming sulphuric acid
(D) All of these
at 475K, it gives
(A) Sulphanilic acid
Q.7 The basic strength of amines (ethyl) and ammonia
(B) Aniline sulphate in H2O is
(C) o-aminobenzenesulphonic acid (A) NH3>p>s>t (B) P>s>t>NH3
(D) m-aminobenzenesulphonic acid (C) s>p>t>NH3 (D) None of these
Chem i str y | 25.35

Q.8 Which of the following will have highest Kb value? Q.13 Identify compound (A) in the following oxidation
reaction.
(A) (B)
K2Cr2O7 H2SO4
N (A) O O
N
H
(C) (A) NH2 (B) OH
NH2 (D) NH2

Me Cl NH2 NH2

(C) OH (D) All of these


Q.9 The product not obtained in the following reaction,
CH3–NO2+Cl2 + NaOH → is
(A) ClCH2NO2 (B) Cl2CHNO2
(C) Cl3CNO2 (D) CH3NH2 OH

Q.10 A sequential reaction may be performed as


represented below: Q.14 NH2

SO Cl
+phosgene X. Here X is
(a) R–CH2CO2H 
2 2 → R–CH COCl
(1) 2

(A) O (B) N = C = O
NH3

(2)
→ R–CH2CONH2  → R–CH2NH 
(4)
→ NH -C -Cl
(3)

R–CH2OH  → R–CO2H
(5) (C) Cl (D) None of these
- -

The appropriate reagent for step (3) is CH- C -H


(A) NaBr (B) Bromine+alkali (C) HBr (D) P2C5 Cl

Q.11 Which of the following amine form N-nitroso Q.15 Ethylamine undergoes oxidation in the presence
derivative when treated with NaNO2 and HCl? of KMnO4 to give
(A) CH3COOH (B) CH3CH2OH
(A) H3C NH2 (B) NH2
(C) CH3CHO (D) N-oxide
(C) N(CH3)H2 (D) NH(CH3)
Q.16 Baker Mulliken’s test is used to detect the presence

Q.12 The strongest base among the following is (A) –COOH gp (B) –NO2
(C) –OH (D) –NH2
H2NH2N H2NH2N H2NH2N H2NH2N

(A) (A) C =CNH = 2NH2(C) (C) C =CO= O (D)Q.17
= NH (B) (B)C =CNH (D)CH
t-amines
= OH
CH = OHwith different alkyl group has a chiral
H2NH2N H2NH2N H2NH2N nitrogen
H2NH2N atom still it is optically inactive because
(A) Chiral N-atoms cannot rotate plane polarized light
N H2N H2N H2N H2N H2N
(C2) (C)C = OC = (D)
C = NH O (D) (B) The lone pair prevents the rotation of plane polarized
(B)
C = NH CH = CH
OH = OH
2
light
N H 2N H N H2N H N H2N
(C) Both of these
2 2

(D) None of these


2 5 . 3 6 | Amines and Aromatic Compounds Containing Nitrogen

Q.18 In CH3NO2 we can observe


List I List II
(A) H-bonding
(iv) CH3CH(OH)CH3 (s) With Lucas reagent
(B) α-halogenation reaction cloudiness appears after 5
minutes
(C) Tautomerism
(D) All of these (A) (i) - q, (ii) - p, (iii) - s, (iv) - r
(B) (i) - r, (ii) - q, (iii) - p, (iv) - s
Q.19 Match list I (condition of reaction of nitrobenzene) (C) (i) - q, (ii) - r, (iii) - p, (iv) - s
with list II (products formed) and select the correct
answer the codes given below. (D) (i) - s, (ii) - q, (iii) - r, (iv) - p

List I List II Q.2 A primary amine is formed an amide by the


(i) Sn and HCl (p) Hydrazobenzene treatment of bromine and alkali. The primary amine has
 (2004)
(ii) Zn and NH4Cl (q) Azoxybenzene
(A) 1 carbon atom less than amide
(iii) Methanolic NaOMe (r) Phenyl hydroxylamine
(B) 1 carbon atom more than amide
(iv) Zn and KOH (s) Aniline
(C) 1 hydrogen atom less than amide
(A) (i) - q, (ii) - p, (iii) - r, (iv) - s
(D) 1 hydrogen atom more than amide
(B) (i) - s, (ii) - r, (iii) - q, (iv) - p
(C) (i) - p, (ii) - s, (iii) - q, (iv) - r Q.3 Indicate which nitrogen compound amongst the
following would undergo Hofmann’s reaction (i.e.
(D) (i) - p, (ii) - r, (iii) - q, (iv) - s
reaction with Br2 and strong KOH) to furnish the primary
amine (R–NH2)  (1989)
Q.20 The increasing order of basicity of RCN, RCH=NR
and RNH2 is (A) (B)
(A) RCN<RCH=NR<RH2N
(C) (D)
(B) RNH2<RCN<RCH=NR
(C) RCN >RCH=NR <RNH2
(D) None of these Q.4 The order of basic strength among the following
amines in benzene solution is (1991)

Q.21 How many isomeric amines with that formula (A) CH3NH2>(CH3)3N>(CH3)2NH
C7H9N contain a benzene ring? (B) (CH3)2NH>CH3NH2 >(CH3)3N
(A) Two (B) Three (C) Four (D) Five (C) CH3NH2>(CH3)2NH>(CH3)2NH
(D) (CH3)3N>CH3NH2>(CH3)2NH
Previous Years' Questions
Q.5 The refluxing of (CH3)2NCOCH3 with acid gives
Q.1 Match the compounds given in list I with their  (1996)
characteristic reactions given in list II. Select the correct (A) 2CH3NH2 + CH3COOH
option (2010)
(B) 2CH3OH + CH3COOH
List I List II (C) (CH3)2NH + CH3COOH
(i) CH3CH2CH2CH2NH2 (p) Alkaline hydrolysis (D) (CH3)2NCOOH + CH4
(ii) CH3C≡CH (q) With KOH (alcohol) and
CHCl3 produces bad smell
(iii) CH3CH2COOCH3 (r) Gives white ppt. with
ammonical AgNO3
Chem i str y | 25.37

Q.6 Order of basicity of ethyl amines is (1988) Q.13 The electrophile, E+ attacks the benzene ring to
generate the intermediate σ -complex of the following,
(A) Secondary > Primary > Tertiary
which σ-complex is of lowest energy?  (2008)
(B) Primary > Secondary > Tertiary
NO2
(C) Secondary > Tertiary > Primary
H
(D) Tertiary > Primary > Secondary
(A) + (B) + E
Q.7 The following reaction is RX+KOH (solid) → heat

 (1988)
(A) Nucleophilic substitution H E
(B) Electrophilic substitution NO2 NO2

(C) Free radical substitution H

(D) None of these (C) + E (D) + H

E
Read the assertion and reason carefully to mark the
correct option out of the options given below:
Q.14 In the chemical reactions the compounds ‘A’ and
(a) If both assertion and reason are true and the reason
‘B’ respectively are  (2010)
is the correct explanation of assertion.
(b) If both assertion and reason are true but reason is NH2
not the correct explanation of the assertion.
NaNO2 HBF4
(c) If assertion is true but reason is false. A E
HCl, 278 K
(d) If assertion is false but reason is true.

(A) Nitrobenzene and fluorobenzene


Q.8 Assertion: Benzene diazonium chloride does not
give tests for nitrogen. (B) Phenol and benzene

Reason: N2 gas lost during heating  (1999) (C) Benzene diazonium chloride and fluorobenzene
(D) Nitrobenzene and chlorobenzene
Q.9 Assertion: Amines are basic in nature.
Reason: Presence of lone pair of electron on nitrogen Q.15 Which of the following compounds can be
atom. (1999) detected by Molisch’s test?  (2012)
(A) Nitro compounds (B) Sugars
Q.10 Assertion: Alkyl isocyanides in acidified water
(C) Amines (D) Primary alcohols
give alkyl formamides.
Reason: In isocyanides, carbon first acts as a nucleophile Q.16 On heating an aliphatic primary amine with
then as an electrophile.  (2005) chloroform and ethanolic potassium hydroxide, the
organic compound formed is:  (2014)
Q.11 Assertion: Amines are more basic than esters and
(A) An alkanol (B) An alkanediol
ethers.
(C) An alkyl cyanide (D) An alkyl isocyanide
Reason: Nitrogen is less electronegative than oxygen.
It is in better position to accommodate the positive
charge on the proton.  (2007) Q.17 Considering the basic strength of amines in
aqueous solution, which one has the smallest pKb
value?  (2014)
Q.12 Assertion: Nitrobenzene is used as a solvent in
Friedel-Craft’s reaction. (A) (CH3)2NH (B) CH3NH2

Reason: Fusion of nitrobenzene with solid KOH gives a (C) (CH3)3N (D) C6H5NH2
low yield of a mixture of o- and p- nitro phenols.  (2008)
2 5 . 3 8 | Amines and Aromatic Compounds Containing Nitrogen

Q.18 In the reaction  (2015) COOH

NH2
(A) (B) H3C CH3
NaNO2/HCl CuCN/KCN
D  E + N2
o
0-5 C CH3

CH3
the E is: product CN CH3

(C) (D)

CH3

JEE Advanced/Boards

Exercise 1 Q.4 An organic compound (A), C6H4N2O4, is insoluble


in both dilute acid and base and its dipole, moment is
Q.1 Aspartame, an artificial sweetener, is a peptide and zero. Deduce the structure of (A).
has the following structures:
Q.5 Explain the following observations:
NH2 CH2C6H5
(i) Aniline dissolves in aqueous HCl.

HOOC—CH2CH—CONH—CH—COOCH3 (ii) The amino group in ethylamine is basic whereas that
in acetamide it is not basic.
(i) Identify the four functional groups.
(iii) Dimethylamine is a stronger base than
(ii) Write the zwitter ionic structure trimethylamine.
(iii) Write the structures of the amino acids obtained (iv) Sulphanilic acid although has acidic as well as basic
from the hydrolysis of aspartame. group, it is soluble in alkali but insoluble in mineral
(iv) Which of the two amino acids is more hydrophobic? acids.

Q.2 Compound of A(molecular formula C9H11NO gives Q.6 Explain, why?


a positive Tollen’s test and is soluble in dilute HCl. It (i) Glycine exists as H3N+CH2COO– while anthranilic acid,
gives no reaction with benzene sulphonyl chloride or p–NH2–C6H4–COOH does not exist as dipolar ion.
with NaNO2 and HCl at 0ºC. (A), upon oxidation with
(ii) Benzenesulphonic acid is a stronger acid than
KMnO4 gives an acid (B). When (B) is heated with soda-
benzoic acid.
lime, compound (C) is formed which reacts with NaNO2
and HCl at 0–5ºC. What is (A)? (iii) A weakly basic solution favours coupling with
phenol.
Q.3 An organic compound A, when treated with nitrous (iv) It is difficult to prepare pure amines by ammonolysis
acid yields an alcohol B, C4H10O with the evolution of alkyl halides.
of N2. B on careful oxidation yields a substance C of
vapour density 36 which forms oxime; B can react with
Q.7 Explain with reason?
NaHSO3 but does not reduce Fehling solution. Identify
compound A and write the structural formulae of the (i) Although trimethylamine and n-propylamine have
isomeric compounds that behave with HNO2 in the same molecular weight, the former boils at a lower
same manner. temperature (3ºC) than the latter (49ºC).
Chem i str y | 25.39

(ii) Dimethylamine is a stronger base than methylamine


(ii) P2O5 +
H
but trimethylamine is a weaker base than both CONH2 F 
G
dimethylamine and methylamine.
KOH
(iii) Silver chloride dissolves in aqueous solution of (iii) EtNH2 + KCN + Br2  → KBr +H
methylamine. Explain.
(iv) N(CH3)2 + HNO2 I
Q.8 Explain it?
(i) An aqueous solution of ethylamine gives a red (v) 2,4-Dinitroaniline →2 (i) NaNO /HCl, 5ºC
(J)
(ii) anisole
precipitate with ferric chloride. Explain.
NaOH
(ii) Tertiary amines do not undergo acetylation. (vi) C6H6 
Oleum
→ (K)  → (L)
Comment NaOH

heat
→ (M)

(iii) 2,6-Dimethyl-N,N-dimethylaniline, although has
a free p-position, does not undergo coupling with SO3H OH
benezene diazonium chloride. Comment.
(iv) In the following compounds: (vii) I CHCl3/NaOH
N
O

(viii) SO3H
Fuming
O
(i)NaOH fuse
+ P
H2SO4 (ii)H
N N
Et SO HCN, HCl
H N H 2 4 → (Q) →
(ix) Phenol 
NaOH AlCl 3
PhNH.NH2
(I) (II) (III) (R)  → (S)

(x) CH3CONHC6H5 


Br2 , Fe
→T + U
N
(A)
(xi) C6H5N2Cl  → (V) Gattermann reaction
H

(IV) Q.11 Give structures for the compounds (A) to (I):


The order of basicity is I > III > II > IV. Explain. NaNO2/HCl KCN
C8H11N B CuCN C
(A) Hot H2SO4

Q.9 Explain it with reason. Heat to Hot


I G D
(i) tert-Butylamine cannot be prepared by the action of m.p. aq.KMnO4
Cl2, 2 moles
NH3 on tert-butyl bromide. CH3OH
u.v.
at 30C Hot
(ii) Isocyanides are hydrolysed by dilute acids but not Hot KMnO4
by alkalis to form amine and formic acid. H2SO4 Hot NaOH

(iii) How will you explain the acidic nature of 1º and 2º H F acidify
nitroalkanes?
(iv) Aniline does not undergo Friedel Craft’s reaction? Q.12 When 2.25 g of an unknown amine was treated
(v) Although boron trifluoride adds on trimethylamine, with nitrous acid, the evolved nitrogen, corrected to
it does not add on triphenylamine. Comment. S.T.P. measured 560 ml. The alcohol isolated from the
reaction mixture gave a positive iodoform reaction.
What is the structural formula of the unknown amine?
Q.10 Complete the following reactions:

(i) C6H5COOH 


PCl
5 → [C] 
3 → [D]NH Q.13 The aqueous solution of a nitrogen and chlorine
containing organic compound (A) is acidic towards litmus.

P O
2 5 → C H CN 
 → [E]
2  H /Hi
(A) on treatment with aqueous NaOH gives a compound
6 5
(B), containing nitrogen, but not chlorine. Compound (B)
2 5 . 4 0 | Amines and Aromatic Compounds Containing Nitrogen

on treatment with C6H5SO2Cl in the presence of NaOH Q.22 An optically active amine (A) is subjected to
gives an insoluble product (C), C13H13NO2S. Give the exhaustive methylation and Hofmann elimination to
structures of compounds (A) and (B). yield an alkene (B). (B) on ozonolysis gives an equimolar
mixture of formaldehyde and butanal. Deduce the
Q.14 An organic compound (A) composed of C, H and structures of (A) and (B). Is there any structural isomer
O gives characteristic colour with ceric ammonium to (A), if yes draw its structure.
nitrate. Treatment of (A) with PCl5 gives (B), which reacts
with KCN to form (C). the reduction of (C) with warm Q.23 An aromatic compound (a) having molecular formula
Na/C2H5OH products (D), which on heating gives (E) C7H7NO2 dissolves in NaHCO3 to evolve CO2 and when
with evolution of ammonia Pyridine is obtained on reacted with NaNO2/HCl forms (b), C7H6O3. (B) dissolves in
treatment of (E) with nitrobenzene. Give structure of NaHCO3 and gives colour reaction with FeCl3 and can be
compounds (A) to (E) with proper reasoning. prepared by the action of CCl4 and NaOH on phenol. When
(B) is reacted with excess HNO3, it forms (C), C6H3N3O7.
Q.15 One mole of bromo derivative (A) and mole of (C) undergoes acetylation and decomposes NaHCO3 to
NH3 react to give one mole of an organic compound evolve CO2. On reaction with PCl5 (C) is converted to (D),
(B). (B) reacts with CH3I to give (C). Both (B) and (C) react C6H5N3O6Cl which when reacted with water gives back (C).
with HNO2 to give compounds, (D) and (E) respectively. Identify compounds (A) to (D).
(D) on oxidation and subsequent decarboxylation
gives 2-methoxy-2-methyl propane. Give structures of Q.24 Compound (A) having M.F. C8H8O on treatment
compounds (A) to (E) with proper reasoning. with NH2OH.HCl gives (B) and (C). (B) and (C) rearrange
to give (D) and (E), respectively on treatment with acid.
Q.16 What happens when cyclopentanone reacts with Compounds (B), (C), (D) and (E) are all isomers of molecular
formula C8H9NO. When (D) is boiled with alcoholic KOH,
(i) CH3CH2NH2 (1º amine) and oil (F) C6H7N separated out. (F) reacts rapidly with
(ii) (CH3CH2)2NH (2ºamine) CH3COCl to give back (D). On the other hand (E) on boiling
with alkali followed by acidification gives a white solid (G),
C7H6O2. Identify the compounds (A) to (G).
Q.17 Cyclohexyl amine is a stronger base than aniline.
Why?
Q.25 An aromatic compound (A), having M.F
C7H5NO2Cl2 on reduction with Sn/HCl gives (B), which
Q.18 How does the formation of 2º and 3º amines can
on reaction with NaNO2/HCl gives (C). Compound (B)
be avoided during the preparation of 1º amines by
is unable to form a dye with β -naphthol. However, (C)
alkylation?
gives red colour with ceric ammonium nitrate and on
oxidation gives an acid (D), having equivalent weight
Q.19 It is necessary to acetylate aniline first for 191. Decarboxylation of (D) gives (e) which forms a
preparing bromoaniline. Why? single mononitro derivative (F), on nitration. Give the
structures of (A) to (F) with proper reasoning.
Q.20 Dimethyl amine is a stronger base than
methylamine but trimethylamine is a weaker base than Q.26 An organic compound (A) of molecular weight
both dimethyl amine and methylamine. Why? 135, on boiling with NaOH evolves a gas which gives
white dense fumes on bringing a rod dipped in HCl near
Q.21 From analysis and molecular weight it. The alkaline solution thus obtained on acidification
determination, the molecular formula of (A) is C3H7NO. gives the precipitate of a compound (B) having
The compound gave following reactions. molecular weight 136. Treatment of (A) with HNO2 also
yields (B), whereas its treatment with Br2/KOH gives
(i) On hydrolysis, it gives an amine (B) and a carboxylic (C). Compound (C) reacts with cold HNO2 to gives (D),
acid (C) which give red colour with ceric ammonium nitrate.
(ii) Amine (B) reacts with benzene sulphonyl chloride On the other hand, (E) an isomer of (A) on boiling with
and gives a product which is insoluble in aqueous dilute HCl gives an acid(F), having molecular weight
sodium hydroxide solution. 136. On oxidation followed by heating, (F) gives an
anhydride (G), which condenses with benzene in the
(iii) Acid (C) on reaction with Tollen’s reagent gives a presence of anhydrous AlCl3 to give anthraquinone.
silver mirror when are A, B and C. Explain the reactions. Give the structures of (A) to (G) with proper reasoning.
Chem i str y | 25.41

Q.27 An organic compound (A) having M.F C7H9N on Exercise 2


treatment with NaNO2 and HCl at room temperature
forms another compound (B), C7H8O. When (A) or Single Correct Choice Type
(B) is treated with bromine water, they form dibromo
derivatives, When (A) is reacted with chloroform and Q.1 Match the compounds in list I with the appropriate
alkali, it forms (C) having the molecular formula C8H7N. test that will be answered by each one of them in list II
Hydrolysis of (C) followed by reaction with NaNO2 from the combinations shown.
and HCl at low temperature and subsequent reaction
with HCN in the presence of Cu (D), which is isomeric Selects the correct answer using the codes given below
to (C). (D) on hydrolysis followed by oxidation gives the list.
a dibasic acid which on halogenation forms only one
List I List II
monohaloderivative. Identify the compounds (A) to (E).
(i) Propyne (p) Reduces Fehling’s solution

Q.28 An optically active compound(A),C3H7O2N forms (ii) Ethyl benzoate (q) Forms a precipitate with
a hydrochloride but dissolves in water to give a neutral AgNO3+C2H5OH
solution. On heating with soda lime (A) yields (B) C2H7N. (iii) Acetaldehyde (r) Insoluble in water, but dissolves in
Both (A) react with NaNO2 and HCl the former yielding aqueous NaOH upon heating
a compound (C) C3H6O, which on heating is converted
to (D), C6H8O4 while the latter yields (E), C2H6O. Account (iv) Aniline (s) Dissolves in dil. HCl in the cold
for the above reactions and suggest how (A) may be and is reprecipitated by the addition
synthesized. of alkali

(A) (i) - r, (ii) - q, (iii) - p, (iv) - s


Q.29 An optically inactive acid (A), C5H8O5, on being
(B) (i) - q, (ii) - r, (iii) - p, (iv) - s
heated lost CO2 to give an acid (B), C4H8O3 capable
of being resolved. On action of sulphuric acid, B gave (C) (i) - q, (ii) - r, (iii) - s, (iv) - p
an acid C whose ethyl ester gave (D) on the action (D) (i) - p, (ii) - r, (iii) - q, (iv) - s
of hydrogen and platinum. (D) with conc. NH3 gave
E, C4H9OH which with Br2 and KOH solution gave (F),
C3H9N. F with HNO2 gave G (G) on mild oxidation gave Q.2 Activation of benzene ring by –NH2 in aniline can
H. Both A and H gave the iodoform reaction. Elucidate be reduced by treating with
the reaction mechanism and suggest a synthesis of (C). (A) Dilute HCl (B) Ethyl alcohol
(C) Acetic acid (D) Acetyl chloride
Q.30 A neutral compound (A) C8H9OH on treatment
with NaOBr forms an acid soluble substance C7H9N.
On addition of aqueous NaNO2 to a solution of B in Q.3 Dipolar ion structure for amino acid is
dilute HCl at 0-5ºC, an ionic compound (C) C7H7N2Cl is +
obtained. (C) yields a red dye with alkaline β-napththol (A) H2N CH COOH (B) H3− N CH COO

solution. When treated with potassium cuprocyanide R H2N R
(C) yields a neutral substance (D) C8H7N. ON hydrolysis –
(C) H3+ N CH COO (D) None of these
(D) gives E (C8H6O4). (F) on nitration yields two isomeric
mononitro derivatives (G and H) having molecular H2N R
formula C8H5NO6. Write the reactions involved in
different steps. Q.4 –NH2 group shows acidic nature while reacts with
regent.
(A) Na (B) CS2 (C) Br2+NaOH (D) Water

Q.5 Which of the following does not give ethylamine


on reduction
(A) Methyl cyanide (B) Ethyl nitrile
(C) Nitro ethane (D) Acetamide
2 5 . 4 2 | Amines and Aromatic Compounds Containing Nitrogen

Q.6 Aniline is a weaker base than ethyl amine because (A) Carbylamine reaction
(A) Phenyl gp in aniline is a +R gp (B) Hofmann reaction
(B) Ethyl gp in ethyl amine decreases the electron (C) Gabriel phthalimide synthesis
density on nitrogen atom
(D) Cope reaction
(C) The lone pair of electron on nitrogen atom in aniline
is delocalized over aniline. Q.11 The conjugate acid of HO(CH2)3NH2 is
(D) Aniline is less soluble in water than ethylamine + +
(A) H2O(CH3 )3 NH2 (B) HO(CH2 )3 NH3
Q.7 Diazonium coupling reaction with aniline should be − +
(C) O(CH2 )3 NH2 (D) HO(CH2 )3 NH
carried out in
(A) Weakly basic medium
Q.12 Consider the following compounds:
(B) Weakly acidic medium
1. H2C=CHCH2NH2
(C) Strongly basic medium
2. CH3CH2CH2NH2
(D) Strongly acidic medium
3. HC≡CCH2NH2
The increasing order of basicity is
Q.8 For CH3CHO, CH3NO2, CH3COOH
(A) 3<1<2 (B) 3<2<1
(A) All have same chemical property
(C) 2<1<3 (D) None of these
(B) All have one common chemical behaviour
(C) All are basic
Q.13 Reaction of RCONH2 with a mixture of Br2 and
(D) None of these KOH gives RNH2 as the main product. The intermediate
involved in the reaction is
Q.9 Bromine in CS2 reacts with aniline to give (A) Br O NHBr (B) R–NHBr
NH2 NH2
C
Br
(A) (B) O
Br
(C) R C N (D) R–C=N=O
Br
Br
NH2
Br Br Q.14 Amines are highly soluble in:
(C) (D) Both (A) and (B)
(A) Alcohol (B) Diethyl ether
Br (C) Benzene (D) Water

Q.10 The reaction: Q.15 Which of the following reagents can convert
O O benzene diazonium chloride into benzene?
C C
- +
(A) Water
NH KOH :N:K
C C (B) Acid

O O (C) Hypophosphorous acid


n-BuBr
(D) HCl
O
C
(1)aq. NaOH Q.16 The bromination of aniline produces
N Bu-n n-BuNH2 +
C
(2)H3O+
(A) 2-bromoaniline (B) 4-bromoaniline
O (C) 2,4,6-tribromoaniline (D) 2,6-dibromoaniline
COOH
COOH is called
Chem i str y | 25.43

Q.17 The compound, which on reaction with aqueous Diazonium salts also couple with phenols and aromatic
nitrous acid at low temperature produces an oily amines to form coloured azo dyes. The reactivity of
nitrosoamine is diazonium salts towards coupling reactions is favoured
by presence of electron withdrawing groups; the
(A) Methylamine (B) Ethylamine
reactivity of 2,4,6-trinitrobenzenediazonium chloride is
(C) Diethylamine (D) Triethylamine so high that it even couples with reactive hydrocarbons
such as mesitylene.
Q.18 Carbylamine test is performed in alcoholic KOH
by heating a mixture of Q.20 Consider the following ions:
(A) Chloroform and silver powder
(A) Me2N N N
+

(B) Trihalogenated methane and a primary amine


(C) An alkyl halide and a primary amine
(D) An alkyl cyanide and a primary amine. (B) O2N N N
+

Multiple Correct Choice Type


(C) CH3O N N
+

Q.19 When nitrobenzene is treated with Br2 in


presence of FeBr3, the major product formed is
m-bromonitrobenzene. Statements which are related (D) CH3 N N
+

to obtain m-isomer are:


(A) The electron-density on meta carbon is more than The reactivity of these ions towards azo coupling
that on ortho and para positions reactions under similar conditions is
(B) The intermediate carbonium ion formed after initial (A) I<IV<II<III (B) I<III<IV<II
attack of Br+ at the meta position is least destabilized.
(C) III<I<II<IV (D) III<I<IV<II
(C) Loss of aromaticity, when Br+attacks at the ortho
and para positions, and not at meta position.
Q.21 Which of the following diazonium salts when
(D) Easier loss of H to regain aromaticity from the meta
+ boiled with dil. H2SO4 gives the corresponding phenol
position than from the ortho and para positions. most readily?
OMe
Comprehension Type (A) N N
+
(B) MeO N N
+

Arene diazonium salts are more stable than


alkanediazonium salts due to dispersal of the positive
charge on the benzene ring. Obviously electron (C) Me N+N (D) N N
+

donating groups favour diazotisation by retarding the


decomposition of diazonium salts to phenyl cation.
The high reactively of arenediazonium salts is due to
Q.22 Which of the following arylamines undergoes
the excellent leaving ability of the diazo group as N2
diazotisation most readily?
gas. Therefore, diazonium salts undergo a number
of substitution reactions in which the diazo group is
replaced by a monovalent atom/group such as H (by (A) NO2 NH2 (B) Cl NH2

H3PO2 in presence of Cu+ ions, CH3CH2OH, NaBH4 etc),
OH (by boiling in presence of mineral acids), OCH3 (by
heating with CH3OH) Cl (by CuCl/HCl or Cu/HCl), Br (by (C) CH3 NH2 (D) CH3 NH2
CuBr/HBr or Cu/HBr) I (by KI in presence of Cu+ions), F
(by first converting into N2F4 followed by heating), CN
(by first neutralizing with Na2CO3 and then reacting with
KCN/CuCN), NO2 (by first neutralizing with Na2CO3 and
then treating with NaNO2) phenyl or substituted phenyl
(by treating with benzene or substituted benzene in
presence of NaOH) etc.
2 5 . 4 4 | Amines and Aromatic Compounds Containing Nitrogen

Q.23 The product formed when bromobenzene reacts Reason: Chlorobenzene undergoes nucleophilic
with benzenediazonium chloride in presence of NaOH is substitution by elimination-addition mechanism
while 4-nitrochlorobenzene undergoes nucleophilic
(A) Diphenyl
substitution by addition-elimination mechanism.
(B) p-Bromodiphenyl
(C) p,p’-Dibromodiphenyl Q.30 Assertion: 1° amides react with Br2 + NaOH to
give 1º amines with one carbon atom less than the
(D) p-Bromoazobenzene
parent amide.

Assertion Reasoning Type Reason: The reaction occurs through intermediate


formation of acylnitrene.
Each of the questions given below consists of two
statements, an assertion (A) and reason (R). Select the
Q.31 Assertion: Acetamide reacts with Br2 in presence of
number corresponding to the appropriate alternative
methanoic CH3ONa to form methyl N-methylcarbonate.
as follows:
Reason: Methyl isocyanate is formed as an
(A) If both assertion and reason true and reason is the
intermediate which reacts with methanol to form
correct explanation of assertion, then mark (A)
methyl N-methylcarbamate.
(B) If both assertion and reason are true but reason is
not the correct explanation of assertion, then mark (B)
(C) If assertion is true but reason is false, then mark (C) Previous Years Questions
(D) If both assertion and reason false, then mark (D) Q.1 Benzene diazonium chloride on reaction with
phenol in weakly basic medium gives  (1998)
Q.24 Assertion: Benzyl amine is more basic than
(A) Diphenyl ether (B) p-hydroxy azobenzene
aniline.
(C) Chlorobenzene (D) Benzene
Reason: Positive inductive effect of phenyl group
creates high electron density around N atom.
Q.2 (2007)
Q.25 Assertion: White precipitate of silver chloride (CH3)2NH
gets dissolved in NH4OH solution. F NO2 DMF,∆
(A)
Reason: NH3 reacts with AgCl to form a solution (i)Fe/HCl
complex with formula [Ag(NH3)2]Cl. (ii) NaNO2/HCl/0ºC
(iii) H2/Ni
(B) is
Q.26 Assertion: o-nitrophenol is more acidic than
p-nitrophenol. CH3
(A) H2N N (B) H2N NH2
Reason: Nitro group has +M and –I effect. CH3

Q.27 Assertion: 3°amine is proved to be less basic in


(C) O N CH3 (D) O2N NH2
aq. solution 2 N

H2N
Reason: Conjugate acid of 3°amine is poorly solvated
in aq. solution.
Q.3 Amongst the compounds given, the one that would
form a brilliant coloured dye on treatment with NaNO2
Q.28 Assertion: In order to convert R–Cl to pure R–NH, in dil. HCl followed by addition to an alkaline solution
Gabriel-phthalimide synthesis can be used. of β -naphthol is (2011)
Reason: With proper choice for alkyl halides, phthalimide N(CH3)2 NHCH3
synthesis can be used to prepare 1°, 2° and 3° amines. (A) (B)

Q.29 Assertion: 4-Nitrochlorobenzene undergoes NH2


nucleophilic substitution more readily than CH2NH2
chlorobenzene. (C) H3C (D)

Chem i str y | 25.45

Q.4 Match the Columns Q.11 In the following reaction the structure of the
major product ‘X’ is:  (2007)
Match the compounds in column I with their
characteristic test(s)/reaction(s) given in column II.
 (2010) O

Column I Column II N conc.HNO3


X
H conc.H2SO4
(p) Sodium fusion extract
⊕ O

of the compound gives
(A) H2N — NH3 Cl
Prussian blue colour with
O
FeSO4
(A) NO2

(q) Gives positive FeCl3 N
NH2I test H
(B) HO
COOH
O2N
 (r) Gives white precipitate O
(C) HO NH3Cl with AgNO3
(B)
N
H
 (s) Reacts with
(D) O2N NHNH3Br aldehydes to form
NO2 the corresponding O
hydrazone derivative.
(C)
N
H
Q.5 In an acidic medium, ..............behaves as the NO2
strongest base. (nitrobenzene, aniline, phenol) (1981)
O
Q.6 The high melting point and insolubility in organic (D)
solvents of sulphanilic acid are due to its ................ O2N N
structure.  (1994) H

Q.7 State the equation for the preparation of following


Q.12 Among the following, the least stable resonance
compound. Chlorobenzene from aniline (in two steps)
structure is (2007)
 (1982)
+ O
+ + O
Q.8 State the conditions under which the following (A) -
+
N
N
preparation carried out. (A) -
O
“Aniline from benzene” (1983) O
-
-
- O
- + O
+
Q.9 Complete the following with appropriate structures: (B) + N
N
(B) +
 (1986) O
O
-
Base -
NH2 + COCl ? O
-
- + O
+
N
(C) +
(C) + N
Q.10 Write the structure of the major organic product O
O
-
expected from the following reaction. (1992) -
-
-O
CH3 + O
N + HNO2 (D) +
N
(D) +
+ N
CH3
O
O
-
-
2 5 . 4 6 | Amines and Aromatic Compounds Containing Nitrogen

Q.13 Statement-I: Aniline on reaction with NaNO2/HCl (iii) NO2 (r) Scheme III
at 0oC followed by coupling with β- naphthol gives a
(i) red hot iron, 873(ii) fu mingHNO3 ,H2SO 4 , heat
dark blue coloured precipitate. (ii) H S.NH (iv) NaNO ,H SO (v)hydrolysis
2 3 2 2 4
?  → C6H5NO3
Statement-II: The colour of the compound formed in C6H5NO3
the reaction of aniline with NaNO2/HCl at 0oC followed
by coupling with β-naphthol is due to the extended (iv) NO2 (s) Scheme IV
conjugation. (2008) (i) conc . H2SO 4 , 60o C
(ii) conc . HNO3 , conc. H2SO 4 (iii)dil. H2SO 4 . heat
(A) Statement-I-I is True, statement-II is True; ? 
→ C6H5NO 4
statement-II is correct explanation for statement-I CH3 C6H5NO4

(B) Statement-I is True, statement-II is True;


statement-IIis NOT a correct explanation for statement-I Code:
(C) Statement-I is True, statement-II is False (i) (ii) (iii) (iv)
(D) Statement-I is False, statement-II is True (A) p s q r
(B) r p s q
Q.14 Match the compounds in column I with their
(C) r s q p
characteristic test(s)/ reaction(s) given in column II.
Indicate your answer by darkening the appropriate (D) s p r q
bubbles of the 4 × 4 matrix gives in the ORS  (2008)

Column I Column II Q.16 The major product of the reaction is  (2015)

+ −
(p) sodium fusion extract of
(A) H2N − NH3 Cl the compound gives Prussian H3C CO2H
NaNO2, aqueous HCl
blue colour with FeSO4 o
CH3 0C
NH2
 (q) gives positive FeCl3 test
NH2I
(B) HO
COOH (A) H3C NH2 (B) H3C CO2H

 (r) gives white precipitate CH3 OH CH3 OH


(C) HO NH3Cl with AgNO3 (C) H 3C CO2H (D) H3C NH2

CH3 OH CH3 OH
(D) (s) reacts with aldehydes
 to form the corresponding Q.17 In the following reactions, the product S is
O2N NHNH3Br
hydrazone derivative  (2015)
NO2 H3C
i. O3 NH3
R S
ii. ZnH2O
Q.15 Match the four starting materials (P, Q, R, S) given
in list I with the corresponding reaction schemes (I, II,
III, IV) provided in list II and select the correct answer
H3 C H3C N
using the code given below the lists.  (2014) N
(A) (B)
List I List II
(i) H H (p) Scheme I
(i) KMnO 4 , HO− , heat (ii), H2O
(ii) SOCl2 (iv) NH3 N
→ CC77H66N
?  N22OO3 3 N
(C) (D)
(ii) NO2 (q) Scheme II H3C H3C

(i) Sn/HCl (ii) CH3COCl (ii) conc H2SO 4


(ii) NHO3 (v) dil.H2 SO 4 , heat (vi) HO−
? → C C
6HHNNOO
66 62 2 22
Chem i str y | 25.47

Q.18 In the following reactions, the major product W is 

OH  (2015)
NH2
NaNO2, HCl , NaOH
o V W
0C

OH
(A) (B)
N=N OH N=N

HO

OH
(C)
(D)
N=N N=N

Q.19 The product (s) of the following reaction sequence is (are) (2016)

NH2 (i) Acetic anhydride/pyridine


(ii) KBrO3/HBr

(iii) H3O+, heat


(iv) NaNO2/HCl, 273-278 K
(v) Cu/HBr

Br Br Br Br
Br Br Br Br
(A) (B) (C) (D)
Br

Br Br
2 5 . 4 8 | Amines and Aromatic Compounds Containing Nitrogen

Paragraph: Treatment of compound O with KMnO4/H+ gave P, which on heating with ammonia gave Q. The
compound Q on treatment with Br2/NaOH produced R. On strong heating, Q gave S, which on further treatment
with ethyl 2-bromopropanoate in the presence of KOH followed by acidification, gave a compound T

Q.20 The compound R is  (2016)

O O O
NH2 Br NHBr
(A) (B) (C) (D) NBr
NH2 Br NHBr

O O O

Q.21 The compound T is  (2016)


(A) Glycine (B) Alanine (C) Valine (D) Serine

PlancEssential Questions
JEE Main/Boards JEE Advanced/Boards

Exercise 1 Exercise 1
Q.1 (iii) Q.3 Q.12 (iii) Q.2 Q.6 (4) Q.7 (4)

Q.17 (ii, vi) Q.19 (iv) Q.22 (ii) Q.14 Q.23 Q.29

Q.27

Exercise 2 Exercise 2
Q.10 Q.14 Q.19 Q.8 Q.19 Q.20
Q.22 Q.28

Previous Years’ Questions Previous Years’ Questions


Q.1 Q.5 Q.10 Q.2 Q.4
Chem i str y | 25.49

Answer Key

JEE Main/Boards
Exercise 2
Single Correct Choice Type

Q.1 A Q.2 C Q.3 D Q.4 B Q.5 D Q.6 C


Q.7 D Q.8 A Q.9 D Q.10 B Q.11 D Q.12 A
Q.13 D Q.14 B Q.15 D Q.16 B Q.17 D Q.18 D
Q.19 B Q.20 A Q.21 B Q.22 A

Previous Years' Questions


Q.1 C Q.2 A Q.3 C Q.4 B Q.5 C Q.6 C
Q.7 A Q.8 A Q.9 A Q.10 A Q.11 A Q.12 B
Q.13 B Q.14 C Q.15 B Q.16 D Q.17 A Q.18 C

JEE Advanced/Boards
Exercise 2
Single Correct Choice Type

Q.1 B Q.2 D Q.3 C Q.4 C Q.5 B Q.6 C


Q.7 B Q.8 B Q.9 C Q.10 C Q.11 B Q.12 A
Q.13 A Q.14 D Q.15 C Q.16 C Q.17 C Q.18 B

Multiple Correct Choice Type


Q.19 A, B

Comprehension Type
Q.20 B Q.21 A Q.22 C Q.23 B

Assertion Reasoning Type


Q.24 C Q.25 A Q.26 D Q.27 A Q.28 C Q.29 B
Q.30 A Q.31 A

Previous Years Questions


Q.1 B Q.2 A Q.3 C Q.4 A → p; B → q; C → r; D → s Q.11 B
Q.12 A Q.13 D Q.14 A → r, s; B → p, q; C → p, q, r; D → p, s Q.15 C
Q.16 C Q.17 A Q.18 A Q.19 B Q.20 A Q.21 B
2 5 . 5 0 | Amines and Aromatic Compounds Containing Nitrogen

Solutions

JEE Mains/Boards Sol 8: (i) Sandmeyer Reaction is used to synthesize


aryl-halides from aryl amines.
Exercise 1
CH3 CH3 CH3
NH2 NaNO2 N+2Cl- CuCl, HCl Cl
Eg. HCl 60C
NH > 0.5C
Sol 1: (i) NH > NH2 > NH2
CH3 (ii)

(ii) C6H5NH2 < C6H5NHCH3 < C2H5NH2 < (C2H5)2NH


⊕ ⊖
(iii) (a) p-nitro aniline < Aniline < p-toluidine N2 Cl ∶OH

(b) C6H5NH2 < C6H5NHCH3 < C6H5CH2NH2 +

Sol 2: 1º and 2º amines react with acetyl chloride to



form acetyl derivatives. O—H N⚌N OH

R–NH2 + CH3COCl → RNHCOCH3 + HCl


(CH3)2–NH + CH3COCl → (CH3)2N–COCH3 + HCl ⊖

Tertiary amine do not undergo this reaction.


Sol 9: (i) Refer to Sol.8 (ii)
(ii) The reaction involves the conversion of an amide
Sol 3: O
O
into one carbon loss, by action of hypoactive NaOH
Br2
C
H2 O
R NH2 solution +Br2.
R NH2 NaOH R N -CO2
O
Reaction of a primary amide to form primary amine Example: CH3—C—NH2 Br2/KOH
CH3NH2 + CO
with one fewer carbon atom.
(iii) Acetylation - 1º or 2ºC amines react will acetyl
Sol 4: +ve charge or secondary carbon is not stable chloride acetic anhydride to from acetyl derivatives.
Ex: CH3NH2 + CH3COCl → CH3NCOCH3 + HCl
Sol 5: Aliphatic amines
Primary: RNH2 + HNO2 → ROH + H2O + N2 Sol 10: (i) Aniline is a weaker base than methyl
compound
R R
Secondary: NH+HNO2 N-N=O In case of aniline, the lone pair is delocalised on the
R R +H2O ring and hence it is less basic.
+ –
Tertiary:R3N + HNO2 → R3NH + NO2 Cl CN
(ii) + NaCN
Aromatic Amines forms diazonium ions.
Cl– is a stronger base than CN– hence the reaction can
Sol 6: (i) Refer Sol.3 take place, + there is not sufficient + ve on the for
(ii) Refer Gabriel Pthalimide synthesis nucleophilic substitute to take place.

Sol 11: Refer to sol. 2


Sol 7: (a) Lone pair delocalised in aryl amines.
(b) CH3NH2 → CH3OH → CH3I → CH3CN
Sol 12: (i) Aniline is a weaker base as compared to
→ CH3CH2NH2 methyl amine as lone pair of NH2 is delocalised on the
ring. Hence pkB aniline > pkB methyl amine.
Chem i str y | 25.51

(ii) MeNH2 is a good base in water,


O it reacts will FeClO
3
to O
form Fe(OH)3 + MeNH+4 Cl− OH
PCl5
Cl
KCN
CN
LiAlH4 NH2
(iii) Due to NH2 group on aniline, there is extra e– density
NaNO2+ H2SO4
on benzene ring. Hence, nucleophilic substitution does
not take place. O 
Kr2Cr2O7/H2SO4 N2Cl
N2 +,Cl- NO2
OH
Sol 13: (i) HNO2
(vi) CH3NH2
HCl (i)NaNO2/HCl NH2
low FeCl3 (ii)KCN
(iii)LiAlH4
NH2 N=N+Cl-
NH
(ii) NaNO2 (vii) CH3NO2 (i)NH3
(ii)NH2 CH3
HCl
(viii) (i) NH3/H2SO4
O COOH (ii) NaNO2/HCl CH3 COOH
Br2 /KOH (iii) K2Cr2O7/H2SO4
(iii) CH3—C—NH2 → CH3—NH2 + CO

Sol 18: (i) Any reactions which involves two or more


equivalent of reactions for proceeding reaction are
Sol 14: Factual known as coupling reaction

NH2 N2+Cl Exp. Wurtz,Pinacol-pinacolone etc.


(ii) Addition of ammonia → Ammonolysis
NaNO2
Sol 15: HCl O
(iii) Addition of acetyl group CH3—C— acetylation
Sol 16: (i) In aromatic amines, the ⊕ charge is delocalised
on the less electron -ve ring C- due to good resonance. (iv) Gabriel Synthesis:
(ii) Due to the smaller size of 1º amine it easer to attack
and hence. phthalimide synthesis is preferred for any O
N
amine. R NH
N2H4
R Cl O R NH2 +
O
NH
Sol 17:
NK
NH3 /H2SO 4
(i) —COOH  → CH
 3
—NH2 O
Test for 1 amines
(i)H2O/H
+ +
O
(ii) N Br2/4KOH O NH2
R N NH
N2H4
R Cl
(i) CHCl R NH2 +
(iii) CH3OH  O 3 /HCl → CH
O
—COOH
(ii) 3KOH 3 NH
NK
(iv) NH2
(i) NaHO2/HCl
O (ii) H O OH
2 Test for 1 amines
K2Cr2O7/H2SO4

O
HN3/H2SO4
CH3 NH2 + CO2 OH

(v) O O O
OH
PCl5 KCN
CN
LiAlH4 NH2
Cl
NaNO2+ H2SO4

O 
Kr2Cr2O7/H2SO4 N2Cl
2 5 . 5 2 | Amines and Aromatic Compounds Containing Nitrogen

NO2 COOH (ix) NH2


(i) NaNO2/HCl
OH
(ii) KCN
Sol 19: (i) (i) NaNH2/KOH
(iii) LiAlH
(ii) CHCl3/HCl, 
(iv) NaNO2/H2O

(ii)
OSO3H
OH OSO3H
Sol 20: Refer Text for Curtius mechanism.
H2O2/Fe
2+
Oleum Br2/H2O
Br
OH Sol 21: (i) ph—N ≡ C
H2SO4/H2O
(ii)
Br

(iii) (iii) Acid-base reaction ph- NH3 + H2SO −4
COOH COCl CONH2 NH2 (iv) phOEt
SOCl2 NH3 Br2/
NaOH
NH2 NH2
Benzoyl Benzamide Aniline Br
Benzoic acid
chloride (v) +
(iv) (Major) (Minor)
Br
NH2 NH2 F
Br Br (i) (CH CO) O Br Br (vi) Acetophenone
Br Br
Br2/H2O 3 2 AgF
excess (ii) H3PO2 (vii) Nitrobenzene
Br Br Br
Sol 22: (i) Electronegativity of ‘O’ > ‘N’
F
Br Br (ii) More surface area ⇒ more interaction b/w molecular
Br Br AgF
⇒ higher B.P.
(iii) In aromatic amines,they have lone pair of N in
Br
Br conjugation with ring ⇒ less basic
CH2 Cl
NH2
(v) (i) KCN
NH2
(ii) LiAlH4 Sol 23: (i) (i) NaNO2/HCl
(ii) LiAlH4
Cl

Cl (ii) CH3NH2 


(i) NaNO2 /HCl
→ CH3CN
(ii) KCN
(vi)
+
(i) H2O/H
(i) HNO3/H2SO4 (iii)
(ii) LiAlH4
N (ii) Br2/KOH NH2
NH2
Schmidt reaction
(vii)
NH2 OH
NH2 NH2
NH2
(i)H2SO4 (2eq’s)
SO3H SO3H +
(iv) (i) NaNO2/HCl
H /H2O
(ii)Br2/H2O (ii) KOH
Br Br
Br Br
NO2 NH2
CONH2 CH3
Br Br
(viii) . (v) (i)NaNO2/HCl
(i) H2/Ni (ii)NH3
(ii) NaNO2/HCl (iii)Br2/H2O (excess)
(iii)LiAlH4 Br
Chem i str y | 25.53

Sol 24: Intramolecular acid-base reactions of molecule Sol 6: (C) End product has a hydrophobic and a
hydrophilic part.
(like some amine + carboxylic acid in same molecule)

Sol 7: (D) s > t > p > NH3


NH2 NH2
NO2 NO2
Sol 25:
(i)(CH3CO)2O Sol 8: (A)
(ii)HNO3/H2SO4
(iii)H+/H2O
NO2 localised lone pair

:
N
Sol 26: because carbocation intermediate in Friedel
craft alkylation reach with l.p. of N in aniline
Sol 9: (D) CH3NO2 
Cl2
→ CH2ClNO2 

Cl2


NaOH NaOH

COOH CHCl2NO2  → CCl3NO2


Cl2
(i) NH3 NaOH
(ii) LiAlH4
Br
Sol 27: (iii) Br2/H2O CH3NH2 is not formed
(iv) K2Cr2O/H2SO4
NO2
Sol 10: (B) Br2/NaOH
Br
R—CH2CONH2 2 → R—CH2–CH
Sol 28: N due to flipping it is considered as NaOH

Sol 11: (D)


planar molecule.
CH3
NaNO2 N N=O
NHCH3
Exercise 2
HCl

Single Correct Choice Type H2N


SO3H Sol 12: (A) NH
C
H2N
Sol 1: (A) (i) → + H2SO4 + H2 O

NH2 NH2 Sol 13: (D) All of these


K2Cr2O7 is a strong oxidising agent.
H
H H
N Sol 14: (B) O
Cl Cl
C
Sol 2: (C) Not acceptable NH2 N
H

O H

Nitrogen can’t form 5 bonds + C


Cl Cl
Sol 3: (D) Order of basic strength
4 > 3 > 2 > 1 Basic strength ∝ +I effect O Cl
C
Sol 4: (B) 1 > 2 > 3
N=C=O NH

Sol 5: (D) Primary amines gives carbylamine test.


2 5 . 5 4 | Amines and Aromatic Compounds Containing Nitrogen

Sol 15: (D) CH3—CH2—NH2 + KMnO4 → CH3CHO Sol 3: (C) Hofmann degradation of amide
R–CONH2 + Br2 + 4KOH →
Sol 16: (B) Baker-Mulliken’s Test is a test for R – NH2 + 2KBr + K2CO3 + 2H2O
nitrocompounds.
(pr imary amine)

Sol 17: (D) R2


R1 R3 Sol 4: (B) (CH3)2NH>CH3NH2>(CH3)3N
N
N Kb=5.4 ×10–4, 4.5×10–4, 0.6×10–4
R1 R3
R2
flips to Sol 5: (C) (CH3)2NCOCH3 + HCl/H2O →

Due to flipping, tertiary amine is a racemic mixture. (CH3)2NH + CH3COOH

O Sol 6: (C) Secondary>tertiary>primary


Sol 18: (D) CH3 shows H-bonding,
N
O - Sol 7: (A) Because OH– is nucleophile.
α -halogenation as well as tautomerism.
Sol 8: (A) It is true that benzene diazonium chloride
Sol 19: (B) Nitrobenzene on reduction with Sn/HCl does not respond Lassaigne test of nitrogen because
gives aniline. With Zn it gives phenyl hydroxylamine. benzene diazonium chloride loses N2 on slight heat
With methanolic NaOMe nitrobenzene gives and thus it can’t react with sodium metal.
Azoxybenzene. On reaction with Zn and strong base it
gives Hydrazobenzene. Sol 9: (A) Amines are basic due to the presence of a
lone pair of electrons on nitrogen atom. The lone pair
Sol 20: (A) RCN < RCH = NR < RH2N can be easily donated.
tertiary secondary primary

Sol 10: (A) In an isocyanide, first an electrophile and


Sol 21: (B)
then a nucleophile add at the carbon to form a species
CH3 which usually undergoes further transformations.
CH3 CH3 + −
NH2 RN ≡ C+ H2O →=
RN CHOH → RNHCHO
Alkyl Formamide

NH2 , NH2 ,
Sol 11: (A) self-explanatory
Sol 22: (A)
Sol 12: (B) Due to strong deactivation of benzene ring
O OH NH2 NH2 by –NO2 group, nitrobenzene is often used as a solvent
LiAlH4
+ NH2OH in Friedel–Craft’s reaction.

Sol 13: (B) -NO2 is electron withdrawing which will


destabilize σ - complex.
Previous Years’ Questions
Sol 14: (C)
Sol 1: (C) Self- explanatory
+ -
NH2 N2 Cl F

Sol 2: (A) CH3CONH2 +Br2+4KOH 


–2H O
→ NaNO2 HBF4
2 + N2 + Bf3 + HCl
(Acetamide) HCl, 278 K

CH3NH2 +2KBr+2K2CO3 (A) (B)


Benzene diazonium Fluorobenzene
(Methyl amine) chloride
Chem i str y | 25.55

Sol 15: (B) Molisch’s Test: when a drop or two of JEE Advanced /Boards
alcoholic solution of a–naphthol is added to sugar
solution and then conc. H2SO4 is added along the sides
of test tube, formation of violet ring takes place at the Exercise 1
junction of two liquids.
Sol 1: (i) Carboxylic acid, amine, ester
CHCl3 /KOH
Sol 16: (D) R

− CH2 − NH2 
C2H5OH
→R − CH2 − NC NH3 CH2C6H5
(ii) OOC CH2 CH C NH CH COOCH3
Sol 17: (A) Among C6H5NH2, CH3NH2, (CH3)2NH,(CH3)3N.
O
C6H5NH2is least basic due to resonance.
(iii) Aspartic acid and phenylalanine
+ +
NH2 NH2 NH2 O
NH2
- OH
NH2
HOOC CH2 CH COOH
-
+ (iv) Phenylalanine is more hydrophobic because of the
NH2 NH2
benzyl side chain.
-

Sol 2:

Out of (CH3)3N, CH3NH2, (CH3)2NH. (CH3)2NH is most CHO COOH N(CH3)2


basic due to +I effect and hydrogen bonding with H2O. KMnO4 Sada-lime

CH3 +I effect
N(CH3)2
:

N(CH3)2
N

H3C
+I effect
H Sol 3:
:

O HNO2 KMnO4

Hydrogen H H NH2 OH O
bonding (A) (B) (C)
Other isomers are primary amines.

Sol 18: (C)


+ - NO2
NH2 N2Cl CN

NaNO2,/HCl Sol 4:
CuCN/KCN
+ N2
o
0-5 C 
NO2
CH3 CH3 CH3
(D) (E)
Sol 5: (i) + HCl

NH2 NH3Cl

••
(ii) CH3—CH2— NH2 Ethylamine
2 5 . 5 6 | Amines and Aromatic Compounds Containing Nitrogen

O Sol 9: (i) Steric hinderance

CH3 C NH2 Acetamide (ii) Isocyanides are stable in strong basic conditions, but
they are sensitive of acids.
Ethylamine is basic due of the presence of localised O O
lone pair. (iii) CH3 N CH3 N +H


Acetamide is acidic due to the presence of acidic O O


hydrogen.
O
Stable
N CH3 N CH3 CH3 N
(iii) H CH3 O
CH3 CH3
(iv) Aniline forms a salt with AlCl3. Due to this nitrogen
Dimethylamine Trimethylamine atom of aniline acquires a positive charge and hence
acts as a strong deactivating group and doesn’t allow
More steric hinderance in Trimethylamine
the reaction to take place.
SO3- (v) The basicity of nitrogen in trimethylamine is greater.

(iv) It forms zwitter ion


Sol 10:

NH3
 (i) C=C6H5COCl D=C6H5CONH2
E=C6H5CHNH2
Sol 6: (i) Amine group in glycine is more basic. (ii) F=C6H5CN; G=C6H5COOH
(ii) More equivalent resonance structures (iii) H=EtNHBr
(iii) It stabilises the complex. N(CH3)2
(iv) Because of side products
(iv) I=

Sol 7: (i)
NO
Me NO2
CH3 NH n-propylamine forms H-bond.
N (v) J =O2N N N Me
Me Me CH3

(vi) K=C6H5SO3H ; L= C6H5SO3Na


(ii) Steric hinderance in trimethylamine
(iii) Due of formation of a soluble complex salt M= C6H5OH
OH
AgCl + 2CH3NH2 → [Ag(CH3NH2)2]+Cl–

(vii) I = NaOH N =
Sol 8: (i) Ethylamine in aqueous solution reacts with
ferric chloride to precipitate hydrated ferric acid.
SO3H OH
(ii) No N—H bond in tertiary amine.
N (viii) O = P=
SO3H OH

(iii) Steric hinderance of 2 Me groups OEt

present on benzene ring.


(iv) In (I) lone pair is localized, Inductive effect of ‘O’ in (III) (ix) Q = Ph–O–Et R=

In IV the lone pair is delocalised CHO

I > II > III > IV


S = EtO CH=N–NH–Ph
Chem i str y | 25.57

Sol 15:
(x) T = CH3–C–NH- + para isomer
OCH3
O Br
Cl (A) Cl—C—CH2CH2Br

+ N2 CH3
(xi) U = Cu Powder V =
OCH3
⊕ ⊖ (B) H3C—C—CH2CH2NH2
NH2 NH2Cl
Sol 11: (A) (B) CH3
Et Et
OCH3
CN COOH
(C) C—C—CH2CH2NH–CH3
(C) (D)
Et E CH3
COOH COOH OCH3

(E) (F) (D) H3C—C—CH2CH2OH


C—CH3 C—CH3

Cl Cl O CH3
COOH OCH3 CH3
COOH
(G) (H) (E) H3C—C—CH2CH2N—CH3
C—OMe
COOH
O CH3
C
O Sol 16 (i)
(I)
C
CH2CH3
CH2CH3
O O+CH3CH2NH2
O+CH3CH2NH2
N
N
CH2CH3
Tautomerise N CH2CH3
Sol 12: C2H5NH2CH3—CH2—NH2 Tautomerise N
H
H
(ii) CH2CH3
Sol 13: O+(CH3CH2)2NH2 N CH CH3
CH2CH32
 O+(CH3CH2)2NH2 N
CH3 NH2 Cl CH3 NH CH2CH3
H H
NaOH H H
Tautomerise N(CH2CH3)2

Tautomerise N(CH2CH3)2

Sol 14:
Sol 17:
CH2OH CH2Cl CH2 CN CH2 CH2 NH
NH2 NH2
PCl5 KCN Na H
CH2 CH2 CH2 CH2
EtOH
CH2OH CH2Cl CH2 CN CH2 CH2 NH2
Cyclohexylamine Aniline


Lone pair of nitrogen in aniline is delocalized due to


NH resonance.
OH
COOH NO2 NO2
OHHNO3
B = and Aromatic excess
2 5 . 5 8 | Amines Compounds Containing Nitrogen
NO2
Sol 18: Use excess OH
of ammonia to reduce the chances
COOH Cl
of reaction of NO
1º amine with
NO2 alkyl halide
NO2 to formNO2º
2 and NO2 NO2 NO2 NO2
3º amines. CO2 PCl5

Sol 19: Amino groupNObeing


2 activating group
NO2 activates NO2 NO2
bromination of aniline and forms tribromoaniline.
NH OH.HCl
Sol 24: C8H8 O 
2 →B + C
Sol 20: Although inductive effect of alkyl groups is Ketone/
greater in trimethylamine but the alkyl groups crowds → D
B 
acid
the nitrogen atom and reduces its basicity.
acid
C 
→ E
Sol 21: D + alc. KOH → F
CH3 CH3 F + CH3COCl → D
hydrolysis
N CHO NH + HCOOH E alkali → acid/solid C7H6O3 (G)
CH3 CH3 C8H8O → Double bond equivalent
(A) (B) (C) 1
DBE = ((16+2)–8) = 5 → benzene ring and a π
2
Sol 22: NH2 bond outside.
∴ NH2OH/HCl → 2 product (G.I.)
∴ It is a ketone.
NH2 Alkene
Optically active amine ozonolysis
O
Ph
HCHO + CH3CH2CH2CHO
 (A) CH3 NH2OH N
No, there is no structural isomer possible. HCl
CH3 OH
(2 G.I.)
Sol 23: cis and trans

OH O
Ph
COOH COOH Ph
N acid NH CH3
NH2 NaNO2 OH
HCl (D)
(B) (Beckman Rearrangement)

Ph
The compound gives +ve less for NH2 and COOH N acid Ph NH
CH3
OH
O
COOH
(E)
NH2
 A is O
Ph O
OH NH CH3 alc. KOH
Ph N H2 CH3 C OH
COOH NO2 NO2
OH HNO3 O
B= excess CH3 C Cl
NO2
OH COOH Cl O
NO NO2 NO2 NO2 NO2 NO2 NO2 NO2 Ph NH C CH3
(Hydrolysis)
CO2 Ph PClNH
5 alc. KOH
Ph COOH + CH3 NH2
CH3
(White solid)
NO2 NO2 NO2 O NO2
(G)
(E)
Chem i str y | 25.59

Sol 25:
[O y ], ∆
F 
→ G
(A)
Sn/HCl NaNO2/HCl
C7H5NO2Cl2 (B) (C)
{does not form dye +G
with β -napthol} given CAN test AlCl3
B = C7H5NH2Cl2 does not give compound B-does not
form a dye with β -naphthol
nitration (D) (O)
(F) (E) –COOH
(forms single mono- (acid) ∴ A is (C) is a primary alcohol.
nitro derivative) eq.wt = 191
as de-carboxylation gives single derivatives
∴ Cl should be symmetrically oppositely placed.
Cl Cl
Cl Cl Cl C
CH2NO2 CH2NH2 CH2O
CH2NO2 CH2-NH2
(A) (B)
A=

Cl Cl Cl Cl Cl C

Cl
Cl Cl Cl Cl Cl
CH2NO2 CH2-NH2 CH2O COOH
CH2OH A = COOH
(C) (D)
Cl Cl Cl Cl
Cl Cl Cl Cl
NO2
Cl Cl
NO2
(E) (F) Cl Cl

Cl Cl
Sol 27:
NH2 OH
Sol 26: ∴ NH+4
as gas is NH3
(A) (B)
−NH2 + OH
M.W. 135 → B mole IRH
alc. CH3 CH3
→ R—CH2COOH → Mass of R =
A + NaOH 
NC CN
R—CH2—NH2 
HNO2
→ RCOOH (B)
Br /KOH
A →
2 C primary amine R–CH2–NH2 (C) (D)
cold
R–CH2–CONH2 (C)  → (D) Primary amine
 red
→
HNO 2 CH3 CH3
cold CAN

dil HCl COOH


E 
→ F (mol. wt 136)

(E)

COOH
2 5 . 6 0 | Amines and Aromatic Compounds Containing Nitrogen

Sol 28: Degree of unsaturation of A = 2 since A CH2OH


forms hydrochoride, it contain both a basic and acidic
functional group. It’s most likely to be an amino acid. A = H3C—C—CO2H
On decarboxylation, it forms an amine B, Degree of
unsaturated of B = 0 CO2H
∴ B is saturated amine.
CH2OH
NaNO2
∴B 
HCl
→ give C2H5OH (E)
B = H3C—C—CO2H
∴ B is CH3CH2NH2
H
NaNO2
 → C (a hydroxyl acid) Which yeilds cyclic OH
HCl

diester on heating C = H3C—CH—CO2H

D = (CH3)2CHCO2C2H5
∴ Reactions
E = (CH3)2 CHCONH2
NaOH
H3C—CH—COOH 
CaO

→ : CH3 – CH2 – NH2 F = (CH3)2 CHNH2

NH2 G = (CH3)2 CHOH

CH3 – CH2 – NH2  H = CH3COCH3


→ CH3CH2OH
NaNO2
HCl

Sol 30:
NaNO2
H3C—CH—COOH  → H3C—CH—COOH
HCl CONH2 NH2
NH2 (A) (B)
OH
(c) CH3 CH3
O +
N2Cl– CN
C (C) (D)
O CH-CH3
H3C2 CH COOH 
CH3 CH3
-H2O
CH3-HC O
OH C
CO2H COOH
O (E) (F)
P/Br
H3C2—CH—COOH CH3 COOH
CH3–CH2–COOH →
2

Br CO2H CO2H
NH3 (G) (H)
 → H3C2—CH—COOH
CO2H O2N CO2H
NH2

H SO
Sol 29: (A) C5H8O5 → (B) C4H8O3 
2 4→

Exercise 2
H conc. NH3
C 
alkyl ester
2 → D 
→  → E (C4H9OH)
Single Correct Choice Type
Br
C3H9N ←2  (C H OH)
KOH 4 9 Sol 1: (B) Propyne- Forms a precipitate with AgNO3 in
degree of unsaturation = 2 × 5 + 2 – 8 ethanol.

=4–2=2 Ethyl Benzoate -Insoluble in water, but dissolves in


aqueous NaOH upon heating
Chem i str y | 25.61

Acetaldehyde- Reduces Fehling’s solution Br


Aniline- Dissolves in dilute HCl in the cold and is
reprecipitated by the addition of alkali. Sol 9: (C) Br2
Cs2
Br Br
Sol 2: (D) Acetic acid NH2 NH2

Sol 3: (C) H3N+ —CH—COO–


Sol 10: (C) Gabriel-phthalimide synthesis.
R
Sol 4: (C) Sol 11: (B)
H
 
H NH+ HBr+Br HO CH2 CH2 CH2 NH2 HO (CH2)3 NH3
N Br Br
H
Sol 12: (A) 2 > 1 > 3
Sol 5: (B) Ethylnitrile (CH3CN)
Consider inductive effect of primary, secondary and
tertiary law.
Sol 6: (C) The lone pair of electron on nitrogen atom in
aniline is delocalized over aniline. O O
Br-Br
•• Sol 13: (A) R C NH2
KOH
R C NH
Sol 7: (B) R — NH2 behave as a base in weakly acidic
medium Br

Sol 14: (D) Self-explanatory


NH2
+2HCl
+NaNO2
. Sol 15: (C)
-2H2O
-NaCl
 Cl
OH NN

OHH PO
3 2

Cl + N2
 N
N N
HCl N Sol 16: (C)

OH
Br
OH
N
N Br
2 →
N NH2

H O Br Br
HCl 2

NH2
O
O O Sol 17: (C) Secondary amine
CH3 N
Sol 8: (B) CH3 C H CH3 C OH O-
Sol 18: (B) Trihalogenated amine and a primary amine
All three shows acidic behaviour.
2 5 . 6 2 | Amines and Aromatic Compounds Containing Nitrogen

Multiple Correct Choice Type R


Sol 27: (A) Large size less solvate or
Sol 19: (A, B) The electron density on meta carbon N
is more than that on ortho and para positions. The
intermediate carbonium ion formed after initial attack R R
of Br+ at the meta position is least destabilized due to
hence less basic.
no mesomeric effect.

Sol 28: (C) 3º amine cannot be synthesised using


Comprehension Type
Gabriel phthalimide
Sol 20: (B) NO2 → –M effect
Sol 29: (B) Both statements are true but
CH3 → +Z effect
4-nitrochlorobenzene goes nucleoliphilic substitutions
OCH3 → mid +M effect more readily than benzene due to the M-effect of
nitro group 2 +ve charge is created on C attaches to
NMe2 → high +M effect
Cl Molecule

Sol 21: (A) –I effect of –OMe


O
Br2/NaOH
Sol 22: (C) +M effect of –OMe at para position Sol 30: (A) R—C—NH2 R—NH2

Sol 23: (B) The product formed when bromobenzene Sol 31: (A) Self explanatory (refer to theory of reaction)
reacts with benzenediazonium chloride in presence of
NaOH is p-Bromodiphenyl

Assertion Reasoning Type Previous Years’ Questions

Sol 1: (B) C6H5 N+ Cl– + C6H5OH →


2
NH2
N=N OH
Sol 24: (C)
p-hydroxy azobenzene
N
Sol 2: (A)

Sol 25: (A) False lone pair is delocalised of Ph-NH2


whereas its localised for aniline . F NO2
(CH3)2NH CH3
DMF CH3 N NO2
Self Explanatory
(i)Fe/HCl
NO2 NO2 (ii)NaNO2 CH3
(iii)H2/Ni N NH2
OH CH3

Sol 3: (C) As we know, benzenediazonium salt


Sol 26: (D)
forms brilliant coloured dye wit h β -naphthol, the
compound under consideration must be p-toluidine
OH (c) as it is a primary aromatic amine. Primary aromatic
NO2 has –M and –I effect. p-Nitro-phenol more acidic amine, on treatment with NaNO2 in dil. HCl forms the
than ortho nitro phenol as o-nitro phenol intermolecular corresponding diazonium chloride salt.
H-bonding where p-nitro phenol has inter molecular
H-bond.
+
NH2 N2Cl-
NaNO2
H3C HCl H3C Chem i str y | 25.63

NH2
+
N2Cl- Sol 9:
O
NaNO2 Base
HCl NH2+ COCl NH C
H3C H3C

OH
Sol 10:

N N CH3 CH3
 -naphthol N + HNO2 No reaction
CH3
OH
Tertiary amine does not react with nitrous acid.

Sol 4: A → p; B → q; C → r; D → s
Sol 11: (B)
Sodium fusion extract gives Prussian blue colouration,
nitrogen and carbon both present in the compound. O
Phenolic group and salt of carboxylic acid gives FeCl3 conc.HNO3

:
N C
test. Chloride salt gives white precipitate of AgCl on conc.H2SO4
treatment with AgNO3. H
Hydrazone formation occur effectively at pH = 4.5 O
The reaction proceeds in that condition only when H+

:
concentration is just sufficient to activate the following O2N NH C
enolization.
O OH Due to presence of lone pair of electron on+ nitrogen
O
+
C
H
C+ atom, it will activate the ring
(A) and it -
will stabilize
N
intermediate cation at o and p positions.
O
As H+ concentration rises sufficiently, a large number -
Hence (B) is correct
of molecules of hydrazine gets converted into - O
hydrazonium ion which is not nucleophilic and reaction +
N
becomes impossible. Further low concentration of H+ Sol 12: (A) Same charges (B)
are present
+ at nearest
(in the case of 2,4-dinitrophenylhydrazinium bromide) position (Less stable) O
-
is not effective to proceed elimination. O
+ O -
+ +
N (C) + N
Sol 5: Aniline: It is a stronger base than either phenol (A) -

or nitrobenzene. O O
-
-
O -
- O
Sol 6: Zwitter ionic: +
N +
(B) + (D) + N
+ O
- O
H3N SO-3 -
- O
sulphanilic acid +
⊕ ⊝
Sol
(C) 13:
+ (D) C6H5N2 Cl
N gives scarlet red coloured dye

CuCl
+ with β-naphthol O
Sol 7: C6H5NH2 + NaNO2 
HCl
→ C6H5 N2 Cl 

heat
→ -
0ºC
C6H5–Cl -
Sol 14: A → r; s B →
O p, q; C → p, q, r; D → p, s
+
(D) + N
Sol 8: C6H6 + conc.HNO3/conc.H2SO4 → C6H5–NO2
O
Zn−HCl -
 → C6H5–NH2 (aniline)
2 5 . 6 4 | Amines and Aromatic Compounds Containing Nitrogen

Sol 15: (C)


NO2 NH2

red hot Fuming HNO3/H2SO4 H2S/NH3 NaNO2


3CH  CH
iron 873K Selective reduction H2SO4

NO2 NO2
N2 HSO 4-
+
OH

Hydrolysis

NO2 NO2

N2 HSO -4
+
OH

Hydrolysis

NO2 NO2

OH OH OH OH
O2N O2N
+
conc.H2SO4 NO2 dil.H2SO4
60oC 
OH OH OH OH
SO3H SO3H

NO2 NO2 NHCOCH3 NHCOCH3 NHCOCH3

NO2
Sn/HCl CH3COCl conc.H2SO4 HNO3

SO3H SO3H
dil. H2SO4 

NH2 NHCOCH3
NO2 NO2
OH-

NO2 NO2 NO2 NO2 NO2

KMnO4 H3O+ SOCl2 NH3


-OH/

CH3 COO- COOH COCl CONH2


Chem i str y | 25.65

Sol 16: (C)

:
OH
:
OH
H3C H3C C=O
C=O
NaNO2/HCl
CH CH2 CH CH CH2 HC
+
H3C H3C
NH2 NN

H
+
H3C H3C O
CH CH2 CH COOH CH CH2 HC
H2O C=O
H3C H3C
OH

Sol 17: (A)


O O
H3C H3C C H H3C C H
(i) O3 :
NH3
(ii) Zn.H2O +
CH2 C H CH2 CH NH3
O O-

OH O- O
H3C H3C + H3C C H
NH NH2
CH

:
OH CH2 OH CH2 CH NH2
-2H2O OH

OH
H3C
N

(S)

Sol 18: (A)


N=N-Ph
+ -
NH2 N2Cl OH
NaNO2, HCl -Napthol/NaOH
o
0C

(V) (W)
2 5 . 6 6 | Amines and Aromatic Compounds Containing Nitrogen

Sol 19: (B)


O
NH2 NH C CH3
O O
Pyridine
+ CH3 C O C CH3
-CH3COOH

Br2 + Acidic medium


O
Br N2Cl NH2 NH2 C CH3
+
Cu+HBr NaNO2+HCl H3O
273-278 K 

Br Br Br Br

Sol 20 and 21 (A) and (B):


Pr COOH COONH4
KMnO4 NH3
H+
Pr COOH COONH4
(P)

-2H2O

NH2 CONH2
Br2/NaOH

NH2 CONH2
(R) (Q)

Heat
Br
O O O
CH3 CH C O Et
C CH3 C C
O - + KOH
N CH N K N H
-H2O
C COOEt C C

O O O
+
H3O

COOH
+ CH3 CH COOH + Et OH
COOH NH2
(T)
Alanine
2017-18 100 &
op kers
Class 12 T
By E ran culty
-JE Fa r
IIT enior emie .
S fP r es
o titut
Ins

CHEMISTRY
FOR JEE MAIN & ADVANCED
SECOND
EDITION

Exhaustive Theory
(Now Revised)

Formula Sheet
9000+ Problems
based on latest JEE pattern

2500 + 1000 (New) Problems


of previous 35 years of
AIEEE (JEE Main) and IIT-JEE (JEE Adv)

5000+Illustrations and Solved Examples


Detailed Solutions
of all problems available

Plancess Concepts
Topic Covered Tips & Tricks, Facts, Notes, Misconceptions,
Key Take Aways, Problem Solving Tactics
Biomolecules and Polymers
PlancEssential
Questions recommended for revision
26. BIOMOLECULES AND
POLYMERS

1. INTRODUCTION
The study of chemical make-up and structure of living matter and of the chemical changes that takes place within
them is called biochemistry.
The various activities of living organisms are regulated by complex organic molecules, such as carbohydrates,
lipids, proteins and nucleic acids, called biomolecules.

2. CARBOHYDRATES
Carbohydrates are principally plants products and are a part of an extremely large group of naturally occuring
organic compounds. Cane sugar, glucose, starch and so on are a few examples of carbohydrates. The general
formula for carbohydrates is Cz(H2O)y. Carbohydrates are generally hydrates of carbon, which is where the name
was derived. So, carbohydrates on hydrolysis produce polyhydroxy aldehydes or polyhydroxy ketones.

2.1 Classification of Carbohydrates

(a) On the basis of Physical Characteristics

(i) Sugar: Characteristics of sugars are crystalline substances, taste sweet and readily water soluble. Because
of their fixed molecular weight, sugars have sharp melting points. A few examples of sugars are glucose,
fructose, sucrose, lactose, etc.

(ii) Non-Sugars: Amorphous, Tasteless, waster insoluble substances with variable melting points e.g., Starch.

(b) On the basis of Hydrolysis

On the basis of hydrolysis

Monosaccharides Oligosaccharides Polysaccharides


(Mono-one) (Oligo-few) (Poly-many)

Flowchart 26.1: Classification based on Hydrolysis

Monosaccharaides’: A carbohydrate that can be hydrolyzed only once to break down into simpler units of
polyhydroxy aldehyde or ketone is called monosaccharide. These include glucose, mannose, etc.
2 6 . 2 | Biomolecules and Polymers

Monosaccharide classification
 O 
 || 
(i) Based on location of  − C − 
 

Monosaccharide classification 
 
Based on location of C=O
H CH2OH

C=O C=O

H-C-OH HO-C-H

H-C-OH H-C-OH

H-C-OH H-C-OH

CH2OH CH2OH
Aldose Ketone
Aldehyde (-CHO), ketone (C=O)
 O 
 || 
Aldehyde (-CHO), ketone  − C − 
 
 
 
(ii) Based on number of carbon atoms in the chain
H

H C=O

H C=O H-C-OH
H C=O H-C-OH H-C-OH

C=O H-C-OH H-C-OH H-C-OH

H-C-OH H-C-OH H-C-OH H-C-OH

CH2OH CH2OH CH2OH CH2OH


Triose Tetrose Pentose Hexose

Can be either aldose or ketose sugar.

Oligosaccharides: Sugars that on hydrolysis produce two or more molecules of monosaccharides are called
oligosaccharides. These are further classified as di-, tri- or tetrasaccharides, etc.

•• Disaccharides: These are sugars that produce two molecules of the same or different monosaccharides on
hydrolysis. Examples are sucrose, maltose and lactose. An example for disaccharides is sucrose: C12H22O11.
Invertas e
C 12 H 22 O 11 + H 2 O  → C 6 H12 O 6 + C 6H12 O 6
S ucros e or H + G lucos e F ructos e

Maltas e
C 12 H 22 O 11 + H 2 O  →2 C 6 H12 O 6
Maltos e or H + G lucos e
Lactas e
C 12 H 22 O 11 + 2H 2 O  → C 6 H11O 6 + C6H12O6
Lactos e or H + G lucos e Galactose

•• Trisaccharides: Sugars that yield three molecules of the same or different monosaccharides on hydrolysis are
called trisacchardies. An example of trisaccharides is Raffinose C18H32O16
C 18 H 32 O 16 + 2H 2 O → C 6 H12 O 6 + C6H12O6 + C6H12O6
R affinos e G lucos e Galactose Fructose
Chem i str y | 26.3

•• Polysachharides: On hydrolysis polysaccharides yield large number of monosaccharides units. An example of


polysaccharide is starch cellulose.

( C6H10O5 )n + nH2O → nC6H12O6


Strach cellulose Glucose

(c) On basis of test with reagents (like Benedict’s solution, Tollen’s reagent and Fehling’s solution):
(i) Reducing Sugars:
 O 
 || 
•• These have a free aldehyde (-CHO) or ketone  − C −  group.
 
 
 
•• These have the ability to reduce the cupric ions (Cu2+; blue) in Fehling’s or Benedict’s Solution to
cuprous ions (Cu+; reddish) that separates out as cuprous oxide (Cu2O) from the solution.
•• Examples include maltose, lactose, melibiose, gentiobiose, cellobiose, mannotriose, rhamnotriose.

(ii) Non-reducing sugars:


•• A free aldehyde or ketonic group is absent.
•• No cuprous oxide (Cu2O) producing chemical reaction takes place.
•• Examples are sucrose, trehalose, raffinose, gentiarose, melezitose.

Table 26.1: The common oxidizing agents used to test for the presence of a reducing sugar

Oxidizing Reagent Benedict’s Solution Fehling’s Solution Tollen’s Reagent


Composition Copper sulfate in alkaline Copper sulfate in alkaline Silver nitrate in aqueous
citrate tartrate ammonia
Non-reducing sugar Deep blue Deep blue Colorless
Color after reaction with Brick red precipitate Cu2O(s) Brick red precipitate Cu2O(s) Silver mirror forms Ag(s)
reducing sugar
Species being reduced Cu2+ Cu2+ Ag⊕
(the oxidant)
Cu2++e- → Cu+ Cu2+ + e- → Cu+ Ag+ + e‒ → Ag(s)
Species being oxidized Reducing sugar oxidized to Reducing sugar oxidized to Reducing sugar oxidized to
(the reductant) carboxylate carboxylate carboxylate

2.2 Classification of Aldols


In the given chemical formulae, symbols D and L refer to the relative configuration of the (–OH) group at the
penultimate carbon, where glyceraldehyde is taken as standard. D and L refer to the (–OH) groups that lies on right
hand side and left hand, respectively.

1 1 1
CHO 1 CHO
2 2
H OH or 2
HO OH or 2

3
3CH2OH
3 3CH2OH

D(+) Glyceraldehyde L(-) Glyceraldehyde

Pair of C-2, Epimer as well as enantiomers


2 6 . 4 | Biomolecules and Polymers

2.3 Epimers
Epimers are essentially diastereomers that contain multiple chiral centers that are absolutely separate from each
other in configuration at only one chiral center.

Example: CHO CHO


H OH HO H

H OH H OH

H OH H OH

CH2OH CH2OH

1 2

2.4 Anomers
Cyclic monosaccharides or glycosides that are epimers, differing from each other in the configuration of C-1 if they
are aldoses or of C-2 if they are ketoses are called anomers. The epimeric carbon in such compounds is known as
anomeric carbon or anomeric center.
Example 1: α-D-Glucopyranose and β-D-glucopyranose are anomers.
6 6
CH2OH CH2OH
Anomeric 5 O OH
5 O carbon
4 1
4 OH 1 OH

OH OH Anomeric
OH 3 2
3 2 carbon
OH OH
-D-glucopyranose -D-glucopyranose

Example 2: α-D-Fructofuranose and β-D-fructofuranose are anomers.

1 1
6 6 OH
O CH2OH O
HOCH2 HOCH2

5 2 Anomeric 5 2 Anomeric
HO carbon HO carbon
4 3
OH 4 3 CH2OH
OH OH
-D-fructofuranose -D-fructofuranose

Example 3: Methyl α-D-glucopyranoside and methyl β-D-glucopyranoside are anomers.

6
6 CH2OH
CH2OH
Anomeric 5 O OCH3
5 O carbon
4 1
4 OH
OH 1
OH Anomeric
OH OCH3 3
3 2
2 carbon
OH
OH
Merthl -D-glucopyranoside Methyl -D-glucopyranoside
Chem i str y | 26.5

2.5 Mutarotation
Mutarotation is commonly used in carbohydrate chemistry to describe the change in specific rotation of a chiral
compound due to epimerization.
For example, the monosaccharide D-glucose can be found in two cyclic forms, α-D-glucose ([α]D25 = +112) and
β-D-glucose ([α]D25 = +18.7), which are epimers and are available as pure compounds.
CH2OH CH2OH

O O OH

OH OH
OH OH OH

OH OH
-D-glucose -D-glucose

On adding water to one of the cyclic forms of D-glucose, through reversible epimerization it changes to the other
via open-chain form, while the specific rotation of the solution gradually changes, until it reaches the equilibrium
value +52.7°.
CH2OH CHO CH2OH

O H OH O OH
H2O
HO H
OH H2O OH
H OH
OH OH OH
H OH
OH OH
CH2OH

2.6 Representation of Structures of Glucose and Fructose

(a) Open chain structures (Fischer projection):


1
1 CH2OH
CHO
2 2 C=O
H OH
3 3
HO H HO H
4 4 OH
H OH H
5 5 OH
H OH H
6 6
CH2OH CH2OH
D (+) Glucose D (-) Fructose

D symbolizes the comparative configuration of C-5 -OH group with relation to D-glyceraldehyde, which these
are prepared from. Here, (+) and (–) refer to optical rotation. Naturally, D-glucose is (+) or dextrorotatory but
D-fructose is (–) or laevorotatory.
(b) Cyclic structures of glucose: The CHO group of glucose reacts with either C-5 -OH group or with C-4 -OH
group to form stable six- and five-membered cyclic rings, respectively, having hemiacetalic linkage. When
-CHO group reacts with C-6 -OH group, seven-membered ring is formed, or four-membered ring is formed
on reaction with C-3 -OH; both are unstable.
(i) -CHO reaction with C-5 -OH group produces two anomeric glucose: In such a case, a six-membered ring,
pyranose, is formed.
2 6 . 6 | Biomolecules and Polymers

1 1
H C OH HO C H

 2  2
H OH H OH
3 3
 HO H O + Enantiomer  HO H O + Enantiomer
4  H 4 OH
 H OH
 H 5  H 5

6 6
CH2OH CH2OH
(C-1) (OH) on R.H.S. is (C-1) (OH) on L.H.S. is
called -glucose called -glucose
-D (+) Glucopyranose -D (+) Glucopyranose

O
Pyranose refers to six-membered ring, like in pyran , or to δ-linkages, since there is hemiacetalic
linkage is between C-1 and C-5 δ-C atom. δ-C atom is formed next to the functional group -CHO and β next
to the α-C atom and so on.
(ii) Haworth representation: English chemist W. N. Haworth (awarded the Nobel Prize in chemistry for his study
of carbohydrate in 1937) established cyclic structure of glucose. Below, R.H.S. shows the -OH group above
the plane of the ring in Haworth structures while as per Fischer projection, on the L.H.S., it is shown below the
plane of the ring .
6 6
CH2OH CH2OH

H 5 O H H 5 O OH
H H
4 OH H 1 4 OH H 1

OH OH OH 3 2 H
3 2
H H OH
OH
-D (+) Glucopyranose -D (+) Glucopyranose

(iii) Chair-form conformation structures: Haworth projection structures are transformed to the chair conformation.
The -OH groups present below the plane of the ring in Haworth structures remain below the plane in the chair
conformation as well.
H H
6 6
CH2OH O 4 CH2OH O
4
HO 5 HO 5 H
H
H 2 HO H 2 OH
HO H
OH 3 OH
3 1 1

H OH H H

-D (+) Glucopyranose -D (+) Glucopyranose


(Less stable one OH (more stable since all OH
Group is in axial position) Groups is in equatorial Position)

(iv) -CHO reaction with C-4 -OH group produces two anomeric glucose: In this case, five-membered ring (furanose)

is formed. Furanose refers to five-membered ring, like in furan, or O to γ-linkage, as the hemiacetal linkage

is between C-1 and C-4 γ-C atom. Not all carbohydrates exist in equilibrium as six-membered hemiacetal

rings; in several compounds, the ring is five-membered, as in fructose. However, glucose naturally occurs
Chem i str y | 26.7

only in pyranose form (six-membered), and in small extent in furanose form, which is in equilibrium with five-
membered hemiacetal ring,.

1 1
H C OH HO C H

 2  2
H OH H OH
3 3
 HO H O + Enantiomer  HO H O + Enantiomer
4  H 4
 H
5 5
H OH H OH
6 6
CH2OH CH2OH
-D(+) Glucopyranose
(C-1) (OH) on L.H.S.
Haworth representation

6 CH OH 6 CH OH
2 2

6CHOH 6CHOH
O O
H OH
54 1 54 1

HOH H OH HOH HH
2 2
3 3
H OH H OH
-D (+) Glucofuranose -D (+) Glucofuranose

Structures of Glucosides
D-Glucose on reaction with MeOH + HCl gives α- and β- D-glucopyranoside.
CH2OH CH2OH
O O
H H H H OMe
MeOH/HCl
H
D-Glucose +
OH H OH H
HO OMe HO H

H OH H OH
Methyl--D (+) Methyl--D (+)
glucopyranoside glucopyranoside

Methyl glucoside reacts with an excess of Me2SO4/NaOH to produce pentamethyl derivatives. Presence of many
electronegative O atoms in the -OH groups of monosaccharides makes them more acidic than alcohols, and all
of them exert –I effect on the nearby -OH groups. In aqueous NaOH, the -OH groups get converted to alkoxide
ion (RO–) that reacts with Me2SO4 by SN2 mechanism and forms methyl ether. This process is called exhaustive
methylation.
The OMe groups at C-2, C-3, C-4 and C-6 of the pentamethyl derivative are ordinary ether groups. These groups
are stable in diluted aqueous acid, since ether groups are cleaved by heating with concentrated HBr or HI.
The OMe groups at C-1 are formed partly of acetal linkage (it is glucosidic) and hence are different from others.
CH2OMe CH2OMe 1
CHO
H O H O 2
H H3 O
+
H H OMe
OMe H OMe OMe H OH 3
MeO H
MeO MeO 4
H OMe
H OMe H OMe 5
H OH
Pentamethyl 6
derivative CH2OMe
(I)
2 6 . 8 | Biomolecules and Polymers

Hydrolysis of the glycosidic -OMe group occurs when pentamethyl derivatives are treated with dilute aqueous acid
to give 2, 3, 4, 6-tetra-O-methyl-D-glucose. (O in the name refers that the Me groups that are attached to O atoms.)
In the open-chain structure, absence of Me group at C-5 because it was originally a part of the cyclic hemiacetal
linkage of D-glucose.

Representation of Structure of Fructose


1. Haworth representation
1
1 CH2OH
CH2OH2
2 CO
C  O3
HO H
HO 3 H4
4 H OH
H OH
5
H 5H OH OH
6
6 CH OH
CH2OH 2
Open chain
Open chain
Structure
Structure
of D(-) fructose
of D(-) fructose




1 2 1 2
1 2  2C C OH
HOH 1 2 HO C CH OH
HOH2 C C OH3 HO C2CH23OH2 2
HO H O + Enantiorner HO H O + Enantiorner
 HO

3 H4 O + Enantiorner HO 3 H4 O + Enantiorner
H OH H OH
 H 4 OH5 H 4 OH
5
 H 5H OH 5 H OH
 OH 6 H OH6
 6 6
-D(-) Fructopyranose
-D(-) Fructopyranose -D(-) Fructopyranose
(C=O) reacts with C-6 OH -D(-) Fructopyranose
(C=O) reacts with C-6 OH OH group on L.H.S. is 
Six-membered pyranose OH group on L.H.S. is 
Six-membered pyranose
Ring of -linkage
Ring of -linkage
OH group on R.H.S. is 
OH group on R.H.S. is 





H H H H
1 O
1 O H 6 HO 6
H 6 H O 6CH CH2OH OH
2OH
H H HOH
5 H 5 2 OH 2
2OH 2 5 5
OH H OH H
OH HOH OH 3 OH OH HOH CH2OH CH OH
3 1 2
4 34 4 3 41
OH HOH H OH HOH H
-D (-) Fructopyranose
-D (-) Fructopyranose -D (-) Fructopyranose
-D (-) Fructopyranose

2. Chair-form conformational structures

H H
H O H O
HO OH 1 OH
CH2OH HO OH
H 3 H 3
H H 2 H H 2
4 4 1
OH CH2OH
OH OH
-D(-) Fructopyranose -D(-) Fructopyranose
More stable, since Less stable, since
bulky (CH2OH) group bulky (CH2OH) group
on equatorial position on axial position
Chem i str y | 26.9

3. Fructose occurs in nature in furanose form (five-membered cyclic ring) and in small extent in pyranose form
which is in equilibrium with six-membered hemiacetal ring.
 O 
 || 
It also exists in two cyclic forms that are obtained by the reaction of  − C −  group with C-5 -OH group.
 
 
 

1 2 2
HOH2C C OH HO C 1CH2OH
3
 HO H O + Enantiomer HO 3 H O + Enantiomer
4
 H OH H 4 OH
 H 5 5
H
 6 6
CH2OH CH2OH
-D (-) Fructopyranose -D (-) Fructopyranose

Haworth representation

O O
6 1 6
HOH2C O CH2OH HOH2C O OH
6 5 12 65 2
HOH2C CH
2OH HOH2C OH
H H HO OH H H HO CH 2OH
5 2 5 2 1
H H4 HO 3OH H H
4 HO3 CH2OH
OH H OH H 1
3 3
4 (-) Fructofuranose
-D -D4(-) Fructofuranose
H
OH H OH
O O
-D (-) Fructofuranose -D (-) Fructofuranose
HOH2C CH2OH HOH2C OMe
O O
HOH2CH CH2OH HOH2HC OMe
H HO OMe H HO CH2OH

H H H OMe
HO H OH
H H CH2OH
HO
OH
-D (-) Fructofuranose -D (-) Fructofuranose
OH H OH H
-D (-) Fructofuranose -D (-) Fructofuranose

2.7 Glucose

Preparation:

(a) From sucrose (cane sugar): Boiling sucrose with diluted HCl or H2SO4 in alcoholic solution produces glucose
and fructose in equal proportions.
H ⊕
C12H22O11 + H2O → C6H12O6 + C6H12O6
(Sucrose) Glucose Fructose

(b) From starch: Industrially, glucose is manufactured by the hydrolysis of starch by boiling it with dil. H2SO4 at
393 K under pressure.
H ⊕
(C6H12O5 )n + nH2O → nC6H12O6
Starchor Gellolose 393K;2−3bar Glucose
2 6 . 1 0 | Biomolecules and Polymers

PLANCESS CONCEPTS

(a) Carbohydrates: Polyhydroxy aldehydes or ketones with general formula Cx(H2O)y.


(b) Classification based on hydrolysis:
(i) Monosaccharides: Simplest carbohydrates that cannot be hydrolyzed any further.
(ii) Oligosaccharides: On hydrolysis gives two to nine units of monosaccharides.
(iii) Polysaccharides: On hydrolysis gives many units of monosaccharides.

(c) Reducing sugar: Sugars that reduce Fehling’s and Tollen’s solutions.
(d) D-L configuration: Refers to relative configuration of the (–OH) group with respect to glyceraldehydes.
(e) Epimers: Have more than one stereocenter but differ in configuration about only stereocenter.
(f) Anomers: Differ in configuration about the acetal or hemiacetal carbon.
(g) All anomers are epimers but the reverse is not true.
(h) Mutarotation: Change in specific rotation of an optically active compound when dissolved in solution is
called mutarotation.
(i) β-D glucose is more stable compared to α-D glucose, since all –OH groups are in equatorial position.
( j) In solution, β-D glucose and α-glucose are both in equilibrium.
(k) Fructose being a polyhydroxy ketone still gives positive test for Tollen’s, Benedict’s and Fehling’s reagent
and thus is a reducing sugar.
Vaibhav Krishnan (JEE 2009, AIR 22)

Chemical Reactions: Glucose has one aldehyde group, one primary (—CH2OH) group and four secondary
(—CHOH) hydroxyl groups, and gives the following reactions:
(a) Acetylation of glucose with acetic anhydride forms a penta-acetate, proving the presence of five hydroxyl
groups in glucose.
(CH COO) O
3 2 → OHC — (CHOCOCH ) — CH OOCCH
OHC — (CHOH)4 — CH2OH  3 4 2 3

(b) Glucose reacts with hydroxylamine to form monoxime and adds up a molecule of hydrogen cyanide to form
a cyanohydrin.
HOH2C — (CHOH)4 — CHO + HONH2 → HOCH2 — (CHOH)4 — CH =
NOH

Glucose monoxime

HOH2C — (CHOH)4 — CHO + HCN→ HOCH2 — (CHOH)4 — CH(OH)CN


Glucosecyanohydrin

The above reactions validate the occurrence of a carbonyl group in glucose.

(c) Ammonical silver nitrate solution (Tollen’s reagent) is a reducing sugar reducing to metallic silver.
Similarly, Fehling’s solution is reduced to reddish brown cuprous oxide. On the other hand, it itself gets
oxidized to gluconic acid. This proves the presence of an aldehydic group in glucose.

(d) HOCH — (CHOH) — CHO + Ag O → HOCH — (CHOH) — COOH + 2Ag


2 4 2 2 4
Gluconic acid
Chem i str y | 26.11

H O
COOH C COOH

H C OH H C OH H C OH

OH C H Bromine water HO C H Nitric acid HO C H

H C OH H C OH H C OH

H C OH H C OH H C OH

CH2OH CH2OH COOH


Gluconic acid Glucose Glucaric acid

(e) On prolonged heating with HI, glucose forms n-hexane, indicating that all the six carbon atoms in glucose
are linked linearly.
HI
HOCH2 — (CHOH)4 — CHO → H3C — CH2 — CH2 — CH3
n−hexane

(f) D-Glucose reacts with phenyl hydrazine and gives glucose phenylhydrazone, which is soluble.
Excess use of phenylhydrazine produces, a dihydrazone, called osazone.
1 1
CHO CH = NHPh
2 PhNHNH2 2 CH = NHPh
H C OH H C OH
5 -H2O 5 2H2N.NPh
(CHOH)3 (CHOH) 3 PhNH2 + NH3 + C = NHPh
6 6 (CH2OH)3
CH2OH CH2OH
D-Glucose D-Glucose CH2OH
phenylhydrazone D-Glucosazone

H
H H
C=O
C=O H NH2 C=O
H OH HO H H OH
HO H H OH HO H
H OSO3 H OH H OH
H OH CH2OH H OH
CH2OH D - Glucosamine CH3
D - Glucose -4-Sulfate Re 6 - Deoxy -D- Glucose
p lac
wit eO Replace OH with NH3
hs H
ulp
ate H H
OH C=O C=O
C=O H OH H OH
H OH Oxidation of the C - 1 Oxidation of the C - 6
HO H HO H
HO H aldehyde to a H OH position H OH
H OH carboxilic acid H OH H OH
H OH CH2OH COOH
CH2OH D - Glucose D - Glucuronic Acid
D - Gluconic Acid H te (An Aldonic Acid)
(An Aldonic Acid) O ha Reduction of the anomeric carbon
c e sp
a o
H pl Ph
Re ith CH2OH CO2H
C=O w O OH
OH H OH H
H H
HO H HO H OH H
H OH H OH OH H

H OH H OH H OH
H2C-PO4 CH2OH Look! this too
Sorbital can be drawn in a
D- Glucose -6- Phosphate Haworth projection
(An alditol, Polyol)

Lobry de bruyn van ekenstein rearrangement reaction


2 6 . 1 2 | Biomolecules and Polymers

Mechanism
 to a common intermediate enediol, establishing the following
(a) The ketose and aldose tautomerise in OH
equilibrium: H
1 1 1
H C=O H C = ÖH H C = OH
2 2 2
H C OH C ÖH C OH
3 3 3
H C OH H C OH H C OH

Aldose Ketose
(Glucose) (Fructose)

(b) When the aldose reforms from the enediol, H+ can add to C-2 from either face of the (C=C) to give C2
aldohexose epimers.
H C OH H C = OH H C = OH
C OH HO C H + HO C H

Enediol Glucose Mannose


(C2 epimeric aldoses)

2.8 Some Disaccharides


Maltose: (a) Preparation: Fractional hydrolysis of starch by enzyme diastase.
Diastase 6 6
2(C6H10 O5 )n + nH2O  → nC12H22O11 CH2OH CH2OH
Starch Maltose
5 O
H O H H H
5
H H
4 1
OH H 1 4 OH H 
O OH
HO 3 2 -Linkage 3 2
H OH H OH

-D-glucose (Maltose) -D-glucose

(b) Units: Two units of α-D glucose.


(c) Reducing sugar
(d) Linkage: α glycosidic linkage between C1 and C4 carbon atoms.

Sucrose: (a) Preparation: Prepared from sugarcane and beetroot 6


CH2OH
(b) Units: α-D glucose and α-D fructose.
5 O
H H
(c) Non-reducing sugar
4
H 1 -link (Glucose unit)
(d) Linkage: α gylcosidic linkage with reference to glucose and β OH H
gylcosidic linkage to fructose, both linked at C1 carbon. HO 3 2
O
H OH
6
HOH2C O
5 -link
H H HO (Fructose unit)
CH2OH
4 31
HO H
Chem i str y | 26.13

Lactose: 6 6
CH2OH CH2OH 
(a) Preparation: Found in milk.
5 5 HO
O O
(b) Units: β-D glucose and β-D galactose. HO H 1
H H
(c) Reducing sugar. 4
OH
1 O OH
H H
(d) Linkage: β glycosidic linkage. H
-Linkage 4 H
H 3 2 3 2
The laevo rotation of fructose (–92.4°) is more
H OH H OH
than rotation of glucose (+52.5º), so the mixture
is laevorotatory. Sucrose hydrolysis brings about a
change in the sign of rotation, from dextro (+) to
laevo (–) and the product is called invert sugar.

2.9 Polysaccharides
(a) Starch: It is the main storage poly saccharide of plants having general formula (C8H16O5)n. The main source is
maize, wheat, barley, rice and potatoes. It is a polymer of α-glucose and consists of two components – Amylose
and Amylopectin. Amylose is made up of long, unbranched chain of α-D-(+)-glucose linkage. Amylopectin is
a branched chain polymer of α-D-glucose units, in which chain is formed by C1–C6 glycosidic linkage whereas
branching occurs by C1–C6 glycosidic linkage.
H +
(C6H10O5 )n + H2O → nC6H12O6
Starch Glucose

CH2OH CH2OH CH2OH


6 O 6 O 6 O
H H H H H H H H H
 4 1 4 1
H H H
HO OH O OH
H
O OH O

H OH H OH H OH

-link -link
Amylopectin
CH2OH CH2OH
O O
H H H H H5 H
4 1
OH H 1 4 H
O O OH
-link
H OH H OH
O
Amylose
CH2OH CH2 CH2OH
O O 6 O
H H H H H5 H H H H
4 1 4 1
OH H 1 4 H 1 4 H
O O OH O OH O
H
H OH H OH H OH
-link -link
2 6 . 1 4 | Biomolecules and Polymers

Table 26.2: Difference between Amylose and Amylopectin

S.No. Amylose Amylopectin


1. It is water soluble fraction for starch It is water insoluble fraction of starch.
2. It is 20% of starch It is 80% of starch.
3. It is straight chain polymer of D-glucose units. It is branched chain polymer of D glucose units.
4. Glucose units are joined by α-1, 4 glycosidic linkage In amylopectin, the glucose units are joined by α-1,
6 glycosidic linkage.
5. Its molecular mass lies in the range of 10,000–50,000 Its molecular mass is in the range of 50,000–1,00,000.

(b) It occurs exclusively in plants and is a main constituent of cell wall of plant cell. It is a linear polymer of β-D-
glucose in which glucose units are linked together by C1–C4 glycosidic linkage. It is a non-reducing sugar.

CH2OH CH2OH CH2OH


6 O 6 O 6 O
H H H H H H H H H
1 4 1 4 1
O OH H O OH H O OH H O
H

H OH H OH H OH

Table 26.3: Relative sweetness of sugar

Sugar Relative Sweetness Sugar Relative Sweetness


Sucrose 100 (standard) Glucose 74
Lactose 16 Invert sugar 130
Galactose 32 Fructose 173
Maltose 33

Chemical treatment of cellulose, the most important natural polymer, gives various useful derivatives
•• Rayon: Cellulose acetate and cellulose xanthate are used as a fibre.
•• Celluloid: Cellulose dinitrate or pyroxylin, combined with plasticizer and alcohol, is used for the manufacturing
of photographic film, spectacle frames, piano keys, etc. It is known as artificial ivory.
•• Explosive: Cellulose trinitrate is used extensively as a blasting and propellant explosive.
•• Lacquer: Collodion is used for manufacturing washable cellulose paints.
•• Water proofing: Solution of cellulose acetate is used to provide anti-shrink property to textile fabric.
•• Methyl cellulose: is used in fabric sizing, paste and cosmetics. Ethyl cellulose is used for manufacturing of rain
coats and plastic films.

PLANCESS CONCEPTS

•• Both glucose and gluconic acid on oxidation with nitric acid yield a dicarboxylic acid.
•• Glucose on heating with HI gives n-hexane.
•• D-glucose reacts with phenylhydrazone to give glucose phenylhydrazine and excess use gives
osazone. But three molecules per molecule of glucose are used for oxidation while the other two are
attached to the molecule.
Chem i str y | 26.15

PLANCESS CONCEPTS

•• Glucose when heated with concentrated NaOH, establishes an equilibrium of D-glucose, D mannose
and D-fructose.
•• Sucrose on hydrolysis changes from Dextrorotatory to Levorotatory and hence is called invert sugar.
•• Sucrose is a non-reducing sugar since the hemiacetal hydroxyl groups are linked by glycosidic
linkage.
•• Sucrose on hydrolysis gives glucose and fructose.
•• Maltose is a reducing sugar that on hydrolysis gives two molecules of D-glucose.
•• Starch is non-reducing polysaccharides that on hydrolysis give D-glucose.
•• Starch is mixture of two polysaccharides: amylose and amylopectin.
•• Amylase is straight-chain polysaccharide that is soluble in water and gives blue colour with iodine.
•• Amylopectin is a branched chain polysaccharide insoluble in water and doesn’t give blue colour with
iodine.
•• Cellulose is a straight chain polysaccharide comprising of only D-glucose units.
•• The difference between starch and cellulose is that starch has α-glycosidic linkage while cellulose
has β glycosidic linkage.
Nikhil Khandelwal (JEE 2009, AIR 94)

Illustration 1: Write the names of monomers of the following polymers: (JEE MAIN)

H H O H O H
(i) (ii) (iii) CF2 CF2
N (CH2)6 N C (CH2)4 C C (CH2)5 N n
n n

Sol: (i) Hexamethylenediamine, H2N—(CH2)6—NH2 and adipic acid, HOOC—(CH2)4—COOH


(ii) Caprolactam (iii) Tetrafluoroethene, F2C = CF2.

Illustration 2: Describe the term D-and L-configuration used for amino acids with examples. (JEE ADVANCED)

Sol: Consider the following configurations (I and II) of a-amino acids


COOH COOH
 
H2N C H H C NH2
R R
I II
L-Amino acid D-Amino acid

NH2 group on the α-carbon oriented toward left (as in structure I) is referred to as L-amino acid and NH2 group
oriented toward right (as in structure II) is referred to as D-amino acid.

Illustration 3: The letters ‘D’ or ‘L’ before the name of a stereoisomer of a compound indicates the correlation of
configuration of that particular stereoisomer. This refers to their relation with one of the isomers of glyceraldehyde.
Predict whether the following compound has ‘D’ or ‘L’ configuration.
2 6 . 1 6 | Biomolecules and Polymers

1
CHO
2
HO H
3
H OH
4
HO H
5
HO H
6
CH2OH

Sol: The orientation of the OH group at the penultimate chiral carbon (i.e. last but one or C5) toward left gives the
compound it’s L-configuration.

Illustration 4: How will you distinguish 1° and 2° hydroxyl groups present in glucose? Explain with reactions.
 (JEE ADVANCED)

Sol: The –OH group on the terminal carbon atom (i.e. C6) is called the 1° hydroxyl while the rest of the four
remaining OH group present on C2, C3, C4 and C5 are called 2° hydroxyl groups. While 1° Hydroxyl groups are
easily oxidized to give carboxylic acids but 2° hydroxyl groups undergo oxidation only under drastic conditions.
For example, glucose on oxidation with HNO3 gives a dicarboxylic acid, saccharic acid having the same number of
carbon atoms as glucose. This indicates that glucose contains one 1° hydroxyl group while the remaining four are
2° hydroxyl groups.
CHO COOH
2 (CHOH)4 HNO3
(CHOH)4
[O]
1 CH2OH COOH
Glucose Saccharic acid

3. AMINO ACIDS
α-Amino Acids: Carboxylic acids in which one α-hydrogen atoms of alkyl group is substituted by amino (–NH2)
group are called α-Amino acids. The general formula is

R CH COOH
NH2

Where R = H or alkyl group

Structure of α-amino acids: The amino acids containing one carboxylic group and one amino group behave
like a neutral molecule. This is because in aqueous solutions the acidic carboxylic group and basic amino group
neutralize each other intramolecularly to produce an internal salt structure known as zwitter ion or dipolar ions.
+
NH2 NH3
R C COOH R C COO-
Zwitter ions

However, the neutral zwitter ion (dipolar ions) changes to cation in acidic solution and exist as anion in alkaline
medium. Thus amino acids exhibit amphoteric character.

H2N CH COO Alkali H3N CH COO Acid H3N CH COOH


R R R
Anion Zwitter ion Cation

Therefore, amino acid exists as zwitter ion when the solution is neutral or pH-7. The pH at which the structure of
an amino acid has no net charge is called its isoelectric point.
Chem i str y | 26.17

Classification of Amino Acids: Based on the relative number of –NH2 and –COOH group, α-amino acids are
classified in three main groups
(a) Neutral Amino acids: Amino acids containing one –NH2 group and one –COOH group. For example, glycine,
valine, alanine etc.
(b) Basic amino acids: These contain one –COOH group and two –NH2 groups, such as lysine and arginine.
(c) Acidic amino acids: Amino acids that containing two –COOH groups and one –NH2 group are called acidic
amino acids; for example, aspartic acid and glutamic acid, etc.

Isoelectronic Point: In acidic solution, an amino acid being a positive ion moves toward the cathode in an electric
field. On the other hand in alkaline solution, it is available as a negative ion that migrates toward anode. At a
specific hydrogen ion concentration (pH), the dipolar ion exists as neutral ion and does not show migration toward
any electrode. This pH is termed as the isoelectric point of the amino acid.
The isoelectric point is dependent on other functional groups in the amino acids. Neutral amino acids have the
range between pH 5.5 and 6.3. At isoelectric points, the amino acids are least soluble in water. This property is
utilized in the separation of various amino acids formed by the hydrolysis of proteins.

Table 26.4: List of 20 naturally occurring α- amino acids

Nature α-Amino acid Abbreviation or code Structure


NEUTRAL
1. Glycine Gly H2N–CH2–COOH
2. Alanine Ala H2N CH COOH
CH3

3. Valine Val H2N CH COOH


CH(CH3)2

4. Proline Pro
COOH
N
H
ACIDIC
5. Serine Ser H2N CH COOH
CH2 OH

6. Aspartic acid Asp H2N CH COOH


CH2 COOH

7. Glutamic acid Glu H2 N CH COOH


CH2 CH2 COOH

BASIC

8. Lysine* Lys H2N CH COOH


CH2 CH2CH2 CH2NH2

9. Arginine Arg H2N CH COOH


CH2 (CH2)2NH C NH2
NH
2 6 . 1 8 | Biomolecules and Polymers

4. PROTEINS
Proteins are complex nitrogeneous organic substance that occurs in all animals and plants. Proteins are called the
most vital chemical substance and are necessary for the normal growth and maintenance of life. Protein serves
following functions in our body:
•• Proteins promote growth
•• Proteins supply essential amino acids to blood.
•• Proteins help maintain body tissues.
•• Proteins synthesize various enzymes.
•• Proteins protect body from infections.

Protein content of food stuffs


Food stuff % Protein Food stuff % Protein
Milk 5 Meat 24-26
Wheat 14 Egg yolk 16
Peas 21 Egg (white) 12.5
Maize 10 Cheese 33

Classification of proteins: Proteins are classified on the basis of two different methods. The first mode of
classification, proteins are of two types is based on their shape and functions:
(a) Fibrous proteins (b) Globular proteins

(a) Fibrous proteins: They are thread like molecules that lie side by side to form fibers. They are held together by
hydrogen bonds. These are insoluble in water but soluble in concentrated acids and alkalis. A few examples
are keratin (present in hair, nails, wood, feathers and horns). Muscles have myosin, silk is composed of fibroin,
bones and cartilages have collagen.

(b) Globular proteins: These proteins have molecules folded into compact units that often acquire spheroidal
shape. Such proteins are soluble in water, diluted acids and alkalis, such as insulin, haemoglobin, albumin, etc.

Structure of Proteins: The structure of proteins is quite complex. Study of its structure is carried out under the
following headings.
(a) Primary structure of protein: The primary structure of protein refers to its covalent structure, that is, the
sequence in which various α-amino acid are arranged in protein or in the polypeptide structure of protein.
The linkage (–CO–NH–) is known as peptide linkage.

H2N CH COOH+H2N CH COOH


R R’
H2N CH CO NH CH COOH+H2O
R Peptide bond R’
The dipeptide still has free amino and carboxyl groups that can react with other molecules of amino acid
resulting in polypeptide formation.
O H R’ O H R’’’
C N CH C N CH
CH C N CH C N
R O H R’’ O H
C-terminal N-terminal

In the polypeptide chain, the free amino end is termed as N–terminal and the free carboxyl end is called C–
terminal end.
Chem i str y | 26.19

(b) Secondary structure of protein: This refers to the arrangement of polypeptide chains into a definite three-
dimensional structure that protein assumes as a result of hydrogen bonding. Depending upon the size of the
R-group of the amino acids in polypeptides, two different types of secondary structure are possible:
(i) α-helix structure (ii) β-Pleated structure
(i) α-Helix structure: This type of secondary structure is possible when the alkyl groups present in amino
acids are large and involved in coiling of the polypeptide chains. The intramolecular hydrogen bond
between the >C = O group of one amino acid and –NH group of the fourth amino acid stabilizes the
helical pattern in right-handed coil and the shape.
(ii) β-Pleated structure: Such secondary structure is acquired when the alkyl groups of amino acids are
small. In this kind of structure, the linear polypeptide chains are arranged side by side and are held
 O 
 || 
together by intermolecular hydrogen bond between the  − C −  and –NH group.
 
 
 
R
C N O
H H C C
C C O H R C
N H N
R C R N H
O C
H H N O
C
C
H O C C R
N N H
C R
C H N
O O H R O C O
N H C
H C C N N R C
C H N H
R C R
O C
H H C N O C O R
H C
R C H N
C N H N O
C R R C
O C
H H H C O
C R C
C N R H N N H
R C C R
O C
O O O
O C C R
H H C N N H N
R C H C H C O
C N R
C R C O
O N R C
H
C R

-helix
-pleated sheet

(c) Tertiary structure of protein: The tertiary structure of protein is the most stable shape that a protein assumes
under the normal temperature and pH conditions. Attractive forces between the amino acid chains are
involved in acquiring tertiary structure,. These attractive forces, like hydrogen bond, disulphide bonds, ionic,
chemical and hydrophobic bonds, result in a complex and compact structure of protein. The two important
tertiary structures of proteins are fibrous structures and globular structure. Fibrous proteins have largely
helical structure and are rigid molecules of rod-like shape. Globular proteins show a polypeptide chain that
consists partly of helical sections and partly β-pleated structure and remaining in random coil form. These
different segments of secondary structure then fold up to give protein a spherical shape.

(d) Quaternary structure of proteins: The quaternary structure of proteins develops when the polypeptide
chains, which may or may not be identical, are held together by hydrogen bonds. It results in the increase of
molecular mass of protein greater than 50,000 amu. For example, hemoglobin contains four subunits, two
identical α-chains containing 141 amino acids each and the other two identical β-chains containing 161 amino
acids each.
2 6 . 2 0 | Biomolecules and Polymers

(e) Denaturation of proteins: Proteins when subjected to the action of heat, mineral acids or alkali, the water
soluble form of globular protein changes to water insoluble fibrous protein resulting in the precipitation or
coagulation of protein. This is called denaturation of proteins.

5. NUCLEIC ACIDS
Nucleic acids are vital biomolecules that present in the nuclei of all living cells as nucleoproteins. These are long-
chain polymers with a high molecular mass. Also called biopolymer, they have nucleotide as a repeating structural
unit (monomer). These play an important role in transmission of the heredity characteristics from one generation to
the next and also in the biosynthesis of proteins. Therefore, the genetic information coded in nucleic acid governs
the structure of protein during its biosynthesis and hence controls the metabolism in the living system.

Structure of nucleic acids: The nucleic acid is the prosthetic component of nucleoproteins. Nucleic acid on
stepwise hydrolysis gives following products as shown in the chart.
Nucleic acid
Hydrolysis

Nucleotides

Hydrolysis

Nucelosides Phosphoric acid

Hydrolysis

Pentose sugar Purine bases Pyrimidine bases


Ribose or (Adenine and guanine) (Thymine and cytosine) from DNA
Deoxyribose (Uracil and cytosine) from RNA
Flowchart 26.1: Components of nucleic acid

Difference between DNA and RNA


The main points of difference between the two types of nucleic acids are given in the table.

Table 26.5: Difference between DNA and RNA

DNA RNA

1 The pentose sugar present in it is 2- deoxy D(–) ribose. It has D(–) ribose sugar.

2. It contains cytosine and thymine as pyrimidine bases. It contains cytosine and uracil as pyrimidine bases.

3. DNA is double strand and pairing of bases is present It is a single strand molecule looped back on itself. The
throughout the molecule. pairing of bases in present only in helical part.

4. It occurs in the molecules of the cell. It mainly occurs in the cytoplasm of the cell.

5. It is a very large molecule and the molecular weight It is a much smaller molecule and its molecular weight
varies from 6 million to 16 million amu. ranges from 20 thousand to 40 thousand amu.

6. It has a characteristic property of replication. It does not replicate.

7. DNA controls the heredity character. RNA only governs the biosynthesis of proteins.
Chem i str y | 26.21

Ribonucleic acids
(a) The pentose sugar in RNA is ribose.
(b) Adenine and guanine represent the purine bases of RNA; the pyrimidine bases are uracil and cytosine.
(c) The thymine in DNA is replaced by uracil in RNA.
(d) RNA is single stranded, but double stranded RNA is present in Rheovirus and wound tumor virus.
(e) There are three major classes of RNA, each with specific functions in protein synthesis

(i) mRNA
•• Messenger RNA is produced by DNA; the process is called transcription.
•• Messenger RNA encodes the amino acid sequence of a protein in their nucleotide base sequence.
•• A triplet of nitrogenous bases specifying an amino acid in mRNA is called codon.

(ii) tRNA
•• tRNA is also known as soluble RNA (sRNA) as it is soluble in 1 molar solution of sodium chloride.
•• tRNA identifies amino acids in the cytoplasm and transports them to the ribosome.
•• Molecules of tRNA are single-stranded and relatively very small.
•• Anitcodon is a three-base sequence in a tRNA molecule that forms complementary base pairs with
a codon of mRNA.
•• All transfer RNA possess the sequence CCA at their three ends; the amino acid is attached to the
terminal as residue.

(iii) rRNA
•• Ribosomal RNA is found in ribosomes of cell and is also called insoluble RNA.
•• The main function of rRNA is to attract provide large surface for spreading of mRNA over ribosome’s
during translocation process of protein synthesis.

Table 26.6: Types of RNA

Sedimentation Number of nucleotide Percentage of total cell


Type Mol. Wt.
Coefficient residues RNA
mRNA 6 to 25 25,000-1000,000 75-3000 2
tRNA 4 23,000-23,000 75-90 16
rRNA 5 35,000 100

16 550,000 1500 82

23 1100,000 3100

•• The relationship between the sequence of amino acids in polypeptide with base sequence of DNA
or mRNA is genetic code.
•• Genetic code determines the sequence of amino acids in a protein.
•• A triplet would code for a given amino acid as long as three bases are present in a particular sequence.
•• Later in a cell-free system. Marshall Nirenberg and Philip (1964) were able to show that GUU codes
for the amino acid valine.
•• The spellings of further codons were discovered by R. Holley. H. Khorana and M. Nirenberg.
•• They have been awarded the Nobel Prize in 1968 for researches in genetic code.
2 6 . 2 2 | Biomolecules and Polymers

Lipids: The term lipids represent a group of biomolecules that are insoluble in water but soluble in organic solvents
of low polarity, such as chloroform, toluene, ether, carbon tetrachloride.
(a) They serve as the energy reserve for living cell.
(b) Lipids are classified in three groups:
(i) Triglyceride ester of higher fatty acids or oils and fats
(ii) Phospholipids
(iii) Waxes

PLANCESS CONCEPTS

•• Proteins: Complex biopolymers of amino acids. Proteins on hydrolysis give peptides that on hydrolysis
give amino acids. Almost all amino acids found in our body is α-amino carboxylic acid.
•• Amino acids are generally colorless, water soluble, have high melting point and are crystalline solids.
•• In aqueous solutions, carboxylic group loses a proton and the amino group accepts it and thus it exists in
form of dipolar ion called zwitter ion.
•• At a particular pH, the dipolar ion exists as a neutral ion and does not show migration toward any electrode,
and this pH is known as isoelectric point.
•• At isoelectric point, the amino acids are least soluble in water and this property is used for separation of
various amino acids formed by hydrolysis of proteins.
•• –CO–NH– bond in peptide are called the peptide linkage. There is restriction in rotation about the peptide
bonds and thus free rotation is not possible.
•• Test for protein:
(i) Biurets’ test: the blue color reagent turns violet in presence of protein.
(ii) Xanthoprotic test: protein + concentrated HNO3 gives orange color in alkaline solution.
(iii) Millons test: protein + Million’s reagent gives white precipitate that on heating gives red precipitate.
(iv) Ninhydrin test: proteins or amino acids in presence of Ninhydrin reagent give a colored product.

Saurabh Gupta (JEE 2010, AIR 443)

Illustration 5: Two samples of DNA, A and B have melting temperature, Tm 340 K and 350 K, respectively. Can you
draw any conclusion from these data regarding their base content?  (JEE MAIN)

Sol: We know that CG base pair has three H-bonds and AT base pair has two H-bonds; therefore, CG base pair is
more stable than AT base pair. Since sample B has higher melting temperature than sample A; therefore, sample B
has higher CG content as compared to sample A.

Illustration 6: Which of the following will reduce Tollens’ reagent? Explain.  (JEE MAIN)
6 6 6 6
CH2OH CH2OH CH2OH CH2OH
5 O 5 5 O
HO HO O HO HO O 5 OH
H H H H
4 H O 4 H 1 4 H O 1 OH 4
HO 1 HO HO 1 H
H 2 H 2 OH H 2 H H
3 3 3 2 3
HO H H OH H H OH
(P) (Q)
Chem i str y | 26.23

Sol: In disaccharide (Q), both the monsaccharides are linked through their reducing centres (C1); therefore, it is
not a reducing disaccharide. In disaccharide (P), the reducing end (C1) of one monosaccharide is linked to non-
reducing end (C4) of the other monosaccharide. In other words, reducing end of one monosaccharide is free.
Therefore, it is a reducing disaccharide.

Illustration 7: An optically active amino acid (A) can exist in three form depending upon the pH of the medium. If
the molecular formula of (A) is C3H7NO2 write.  (JEE ADVANCED)
(i) Structure of compound (A) in aqueous medium. What are such ions called?
(ii) In which medium will the cationic form of compound (A) exist?
(iii) In alkaline medium, towards which electrode will the compound (A) migrate in electric field?

Sol: An optically active amino acid having the M.F. C3H7NO2 is alanine, i.e. CH3 — CH — COOH
|
NH2

Depending upon the pH of the medium, alanine can exist in the following three forms:
CH3 CH3 CH3
+ OH- + OH-
H3N CH COOH H3N CH COO- H2N CH COO-
H+ H+
Cationic form(I) Zwitterion(II) Anionic(III)
(exists in acid medium) (exists in aqueous medium) (exists in basic medium)

(i) In aqueous medium, the compound (A) exists as a zwitterion (II).


(ii) In acidic medium, it will exist in the cationic form (I).
(iii) In alkaline medium, it will exist in the anionic form (III) and will migrate towards anode under the influence of
an electric field.

Illustration 8: Following two amino acids, lysine and glutamine form dipeptide linkage. What are the two possible
dipeptides?
NH2 NH2

H2N COOH HOOC CONH2

Sol:
NH2 H
NH2 H NH2
CONH2 N
N
(i) H2N (ii) H2NOC C
C COOH
COOH O
O Glnyllysine
Lyslglutamine
(Gln-Lys or Q-K)
(Lys-Gln or K-Q)

6. POLYMERS
A polymer is a compound of high molecular mass formed by the combining large number of small molecules and
the process is called polymerization. The small molecules that constitute the repeating units in a polymer are called
monomer units. These large molecules have relative molecular masses in the range 104 – 106 amu.

E.g. nCH
=2 CH2 → CH2 – CH2 –  .
n
ethene polyethene

Where n is as high as 105. The number of monomer units in a polymer is called the degree of polymerization.
2 6 . 2 4 | Biomolecules and Polymers

7. CLASSIFICATION OF POLYMERS
(a) On the Basis of Source
(i) Natural polymers: These polymers are found in plants and animals. Examples are proteins, cellulose,
starch, some resin and rubber.
(ii) Semi-synthetic polymers: Cellulose derivatives as cellulose acetate (rayon), cellulose nitrate, etc. are the
usual examples of this sub category.
(iii) Synthetic polymers: A variety of synthetic polymers such as plastic (polythene), synthetic fibers (nylon
6,6) and synthetic rubbers (Buna –S) are examples of manmade polymers extensively used in daily life
as well as in industry.

(b) On the Basis of Structure: There are three different types based on the structure of the polymers.
(i) Linear polymers: These polymers consist of long and straight chains. The examples are high density
polythene, polyvinyl chloride, etc. These are represented as:

(ii) Branched chain polymers: These polymers contain linear chains having some branches, For example,
low density polythene. These are depicted as follows:

(iii) Cross-linked or Network polymers: These are usually formed from bifunctional and trifunctional
monomers and contain strong covalent bonds between various linear polymer chains. For example,
bakelite, melamine, etc. These polymers are depicted as follows:

(c) On the Basis of Synthesis: These are of two types based on synthesis –
(i) Condensation polymerization: In this, the monomer (same or different) units link with each other by
the elimination of a small molecule (for example, water, methyl alcohol) as a byproduct. The polymer
formed is known as condensation polymer. Nylon and terylene are the most common examples.
Since the condensation polymerization proceeds by a stepwise intermolecular condensation, it is also
known as step polymerization and the polymer formed is known as step growth polymer.

n H2N(CH2)6NH2 + n HOOC (CH2)4 COOH → – NH(CH2 )6 NHCO(CH2 )6 CO  – + nH2O


n
Nylon− 6,6

(ii) Addition polymerization: This involves self-addition of several unsaturated molecules of one or two
monomers without loss of any small molecule to form a single giant molecule. The polymer formed is
known as addition polymer. Polythene is the most common example. However, the addition polymers
formed by the polymerization of a single monomeric species is known as homopolymers, for example,
polyethene.

nCH
=2 CH2 → –(CH2 – CH2 ) –Homopolymer
Ethene n
Polyethene

The polymers made by addition polymerization from two different monomers are termed as copolymers,
for example, Buna-S, Buna-N, etc.
Chem i str y | 26.25

(d) On the Basis of Molecular Forces: A large number of polymer applications in different fields depend on their
unique mechanical properties like tensile strength, elasticity, toughness, etc. These mechanical properties are
governed by intermolecular forces, for example, van der Waals forces and hydrogen bonds.
(i) Elastomer: In these polymers, polymer chains are held together by the weakest intermolecular force.
Weak binding forces permit the polymer to be stretched. A few “crosslinks” are introduced in between
the chains, which help the polymer to retract to its original position after the force is released as in
vulcanized rubber. The examples are Buna-N, neoprene, etc.
(ii) Fibers: In these polymers, polymer chain held together by strong intermolecular forces, like hydrogen
bonding. These strong forces also lead to close packing of chains and thus impart crystalline nature.
Fibers are the thread-forming solids that possess high tensile strength and high modulus. The examples
are polyamides (nylon -6, 6), polyesters (terylene), etc.
(iii) Thermoplastic polymers: These polymers possess intermolecular forces of attraction intermediate
between elastomers and fibers. These are the linear or slightly branched long-chain molecules capable
of repeatedly softening on heating and hardening on cooling. Some common thermoplastics are
polythene, polystyrene, polyvinyls, etc.
(iv) Thermosetting polymers: These polymers are cross linked or heavily branched molecules, which on
heating undergo extensive cross linking in molds and again become infusible. These cannot be reused.
Some common example are bakelite, urea-formaldelyde resins, etc.

(e) Classification on the Basis of Type of Polymerization: Synthetic polymers have been classified into definite
classes in various manners, for example,
(i) All the synthetic polymers may be classified into two groups based on of the type of the process involved
during their preparation, viz. condensation or addition polymers involving condensation or addition
polymerization processes, respectively, during their synthesis.
(ii) A more rational method of classification is based upon the sequence of synthesis of the polymer (i.e.,
mode of addition of the monomer units to the growing chain).
According to this method, polymers may be of two types, viz. chain growth polymers and step growth polymers.
•• Chain growth polymers (earlier called as addition polymers). These polymers are formed by the
successive addition of monomer units to the growing chain having a reactive intermediate (free
radical, carbocation or carbanion)
The most important free radical initiator is benzoyl peroxide.
• •
heat
E.g. RCO — O — O — OCR  → 2RCOO → 2R + 2CO2
Benzoylperoxide

• • •
heat CH =CH
R + H=
2C CH2  → R – CH2 CH2 
2 2 → R — CH — CH — CH →→ R  –CH – CH  —
2 2 2  2 2
n

Examples of chain growth polymers


Monomer(s) Polymer
Ethylene Polyethylene
Propylene Polypropylene
Tetrafluoroethylene Polytetrafluoroethylene (PTFE) or Teflon
Vinyl chloride Polyvinyl chloride (PVC)
Isoprene Polyisoprene (Natural rubber)
Butadiene Polybutadiene
Butadiene and styrene Buna-S
2 6 . 2 6 | Biomolecules and Polymers

•• Step growth polymers (earlier called as condensation polymers). These polymers are formed through
a series of independent steps (reactions). Each step involves the condensation (bond formation)
between two difunctional units (monomers) leading to the formation of small polymer (say dimer,
trimer, tetramer, etc.)
Step 1. A + B → A–B
Monomer Monomer Dimer
Step 2. A — B + A → A — B —A
Trimer
Step 3. A—B+A—B → A—B—A—B
Dimer Dimer Tetramer
Step 4. A — B — A — B + A — B → A — B — A — B — A — B or (A —B)3
Step 5. (A — B)3 → ( — A — B — )n
Polymer

Examples of step growth polymers


Monomers Polymers
Adipic acid hexamethylene diamine Nylon-66
Terepthalic acid and ethylene glycol Terylene
Phenol and formaldehyde Bakelite

(f) Based on the magnitude of molecular forces the polymers may be classified as:
(i) Elastomers
•• In which the intermolecular forces of attraction are the weakest. They are amorphous and
have high degree of elasticity, for example, Buna-S.
(ii) Fibers
•• In which the intermolecular forces of attraction are the strongest like H bonding or di-
pole-dipole attraction.
•• They have high tensile strength, high melting point and low solubility, for example, polyes-
ters, polyacrylonitriles
(iii) Thermoplastics
•• In which the intermolecular forces of attraction are in between those of elastomers and
fibers
•• They are hard at room temperature, soften on heating without any change in mechanical
properties of plastic, have little or no cross links, for example, polythene, polyacrylonitrile,
Teflon, etc.
•• Semi-fluid substances with low molecular weight that once set into particular mold cannot
be used again and again.
•• This is because of extensive crosslinking between two polymer chains to give a three-dimen-
sional network, for example, bakelite, urea-formaldehyde; melamine formaldehyde etc.
Chem i str y | 26.27

8. MOLECULAR MASS OF POLYMERS


(a) There are two types of average molecular mass of polymers.

( )
(b) Number average molecular mass Mn : It is determined by methods depending upon the number of molecules
present in the polymer sample, for example, colligative properties like ∆Tf; ∆Tb, Osmotic pressure

=(c) Mn
N1M1 + N2M2 + N3M3
=
∑ N1M1
N1 + N2 + N3 ∑ N1
Where N1,N2, N3 are the number of molecules. M1, M2, M3 are the molecular masses.

Weight Average Molecular Mass: It is determined by using the methods depending upon the masses for the
individual molecules like light scattering, ultracentrifugation, sedimentation, etc.

Mw
=

=
m1M1 ∑ N1M12
; M– is mass of macromolecule
∑ m1 ∑ N1m1
Example in a polymer sample, 30% molecules have molecular mass 20,000; 40% have molecular mass 30,000 and
the rest 30% have 60,000. Calculate the number average and mass average molecular masses.

30 × 20000 + 40 × 30,000 + 30 × 60,000 600000 + 1200000 + 1800000 3600000


Mn = = Mn = 36000
30 + 40 + 30 100 100

30(20000)2 + 40(30,000)2 + 30(60,000)2


Mw = 43,333
30 × 20000 + 40 × 30,000 + 30 × 60,000

9. VARIOUS TYPES OF POLYMERS


Table 26.7: Types of polymers

Name of the polymer Structure of Natural of polymer Properties Uses


along with abbreviation monomer
A. Addition Polymers
I. Polyolefins
1. Polymers or polythome CH2 = CH2 (a) Low-density Transparent, Packing material
polyethylene (LDPE) in an moderate tensile (plastic films, bags etc.)
CH2 CH2 addition or chain-growth strength, high insulation for electrical
homopolymer. It is a toughness. wires and cables.
highly branched polymer
Translucent, Manufacture buckets,
and is obtained by free-
chemical tubs, pipes, bottles and
radical polymerization
inert, growth toys.
(b) High-density toughness and
polyethylene (HDPE) is tensile strength.
obtained by coordination
polymerization. It is a
linear addition or chain-
growth homopolymer.
2. Polypropylene or CH3CH=CH2 Addition homopolymer Harder and Packing of textile and
Polypropene linear can be obtained by foods, for making liners
stronger than
free-radical or Ziegler- of bags, heat shrinkable
polythene
—CH–CH2— Natta polymerization. wraps, carpet fibers,
CH3 ropes, automotive
— molding, stronger pipes
and bottles.
2 6 . 2 8 | Biomolecules and Polymers

Name of the polymer Structure of Natural of polymer Properties Uses


along with abbreviation monomer
3. Polystyrene or Styron C6H5CH=CH2 Addition homopolymer, Transparent Plastic toys, household
linear chain wares, radio and
CH CH2 television bodies,
n refrigerator linings etc.
II Polydienes
1. Neoprene Cl Addition homopolymer Rubber like, Hoses, shoe heels,
CH2 CH C CH2 inferior to natural stoppers etc.
CH2 CH C CH2 rubber but
Cl Chloroprene or 2- superior in its
n
Chloro-1, 3-butadiene. stability to serial
oxidation and its
resistance to oils,
gasoline, etc.
III. Polyacrylates
1. Poly (methyl Cl Addition homopolymer Hard, transparent, Lenses, light covers,
methacrylate) Plexigas, CH2 C COOCH3 excellent light light shades, signboards,
Lucise, Acrylite Perspex transmission transparent domes,
(PMMA) Methyl methacrylate properties, skylights, aircraft
optical clarity windows, dentures and
better than glass. plastic jewelry.
2. Poly (ethyl acrylate) CH2=CHCOOC2H5 Addition homopolymer. Tough, rubber like —
product.
CH2 CH
COOH n

3. Polyacrylonitrile (PAN) CH2 = CH–CN Addition homopolymer. Hard and has Manufacture of fibers,
Acrylonitrile high melting orlon, acrilon used for
CH2 CH
material. making clothes, carpets,
CN n blankets.

IV. Polyhalolefines
1. Polyvinyl chloride (PVC) CH2=CH–Cl Homopolymer, chain Pliable (easily (i) When plasticized
Vinyl chloride growth molded) with high-boiling esters
used in rain coats, hand
CH2 CH
bags, shower curtains,
Cl n upholstery fabrics, shoe
soles, vinyl flooring

(ii) Good electrical


insulators for wires and
cables

(iii) Making hose pipes.


2.Polytetrafluoro ethylene F2C = CF2 Chain growth, Flexible and (i) For making nonstick
of Teflon (PTFE) Tetrafluoroethylene homopolymer inert to solvents, utensils coating
boiling acids,
F F (ii) Making baskets,
even aquaregia,
C CH pump packings, valves,
stable up to
seals, nonlubricated
F F 598 K.
n bearings.
Condensation Polymers
I. Polyesters
Chem i str y | 26.29

Name of the polymer Structure of Natural of polymer Properties Uses


along with abbreviation monomer
1. Terylene or Dacron HO–CH2–CH2–OH Copolymer, linear, step- For making paints
growth. and lacquers.
Ethylene glycol or
O Ethane-1, 2-diol and Thermoplastic, dissolves
O O O in suitable solvents
HO C C OH and its solutions, on
evaporation leaves a
Terephthalic acid tough but non-flexible
or Benzene-1, a film.
O 4-dicarboxylic acid
O

2. Glyptal or Alkydresin HO–CH2–CH2–OH Copolymer, linear, step- For making paints


growth. and lacquers.
Ethylene glycol and
O
Thermoplastic, dissolves
HOOC HOOC
CH2 in suitable solvents
O O and its solutions, on
Phthalic acid or evaporation leaves a
Benzene-1, tough but non-flexible
a film.
2-dicarboxylic acid
O O
CH2
O

II. Polyamides
Nylon-6,6 O O Copolymer, step, growth, High tensile (i) Textile fabrics, carpets,
NH2 HO C (CH2)4 C OH linear strength, abrasion bristles for brushes
resistant,
Adipic acid and (ii) Substitute for metals
somewhat elastic.
in bearings and gears.
H2N—(CH2)4—NH2
(iii) Crinked nylon is
Hexamethylene used for making elastic
O
diamine hosiery.
OH
2. Nylon-610 H2N(CH2)6NH2 Copolymer, step- growth, For making (i) Textile fabrics, carpets,
linear points and bristles for brushes
H H O O Hexamethylene
lacquers.
diamine (ii) Substitute for metals
[ N (CH2)6 N N (CH2)8 C [
in bearings and gears.
and
(iii) Crimped nylon is
HOOC(CH2)8
used for making elastic
COOH hosiery.
Sebacic acid
3. Nylon-6 or Parlon H Homopolymer, step Mountaineering ropes,
O N growth, linear tire cords and fabrics
(CH2)5 C O
N
H E-Caprolactam
III Formaldehyde resins
2 6 . 3 0 | Biomolecules and Polymers

Name of the polymer Structure of Natural of polymer Properties Uses


along with abbreviation monomer
1. Phenol-formaldehyde Phenol and Copolymer, step (i) Bakelite with low
resin or Bakelite formaldehyde growth, highly branched degree of polymerization
thermosetting polymer is used as binding glue
for wooden planks and in
varnishes and lacquers

(ii) Bakelite with high


degree of polymerization
is used for making combs
and micatable tops,
fountain pen barrels,
electrical goods (switches
and plugs), gramophone
records etc.

PLANCESS CONCEPTS

•• Polymers are product of large number of small molecules, called monomers, chemically bonded to
each other.
•• The individual large polymer molecules are known as macromolecules.
•• Polymers are characterized by the average molecular mass of the chains and the number of repeating
units in such polymers is known as the degree of polymerization.
•• The physical properties of a polymer are determined by such factors as the flexibility of macromolecules,
the sizes and types of group attached to the polymer chains and the magnitude of intermolecular
forces.
•• Polymers may be linear, branched or cross linked.
•• Copolymers are produced from two monomers combined randomly or in a specific manner
•• To participate in polymerization, a molecule must be able to react at both ends.
•• The principal types of polymerization reaction are chain reaction polymerization (initiation
propagation and termination), which is undergone by monomers such as vinyl bromide and step
reaction polymerization, which involves reactions between functional group on different monomer
molecules like adipic acid and hexamethylene diamine to give nylon 66.

Neeraj Toshniwal (JEE 2009, AIR 21)

Illustration 9: Write the structures of the monomers of the following polymers:  (JEE MAIN)

(a) ( CH2CH2—O—CH2CH2 )n (b) [ = CH(CH2)4CH = N(CH2)6N = ]n

Sol: (a) CH2—CH2 (b) HO — CH —(CH2)4—CH2 — OH and H2N—(CH2)6—NH2


Hexane-1, 6 diol Hexamethylenediamine
O
(Monomer) (Monomer)
Ethylene oxide
(Monomer)
Chem i str y | 26.31

Illustration 10: (a) Can a copolymer be formed in both addition and condensation polymerization? Explain.
(b) Can a homopolymer be formed in both addition and condensation polymerization? Explain. (JEE MAIN)

Sol: (a) Yes, copolymers can be formed both in addition and condensation, polymerization. For example, Buna-S
is an addition copolymer of styrene and 1, 3-butadiene while nylon-6,6, bakelite and polyester are condensation
copolymers.
(b) Yes, homopolymers can be formed both in addition and condensation polymerization. For example, polythene,
PVC, PMMA, PAN, neoprene, etc. are example of addition homopolymers while nylon-6 is an example of
condensation homopolymer.

Illustration 11: How does the presence of benzoquinone inhibit the free radical polymerization of a vinyl derivative?
 (JEE ADVANCED)
Sol: Benzoquinone react with radical of the growing O O OR
polymer chain (R᛫) to form a new radical (I) which is
extremely uncreative, since it is highly stabilized by R + etc.
resonance. Because of the lack of reactivity of this
new radical further growth of the polymer chain is O O O
interrupted and hence the reaction stops. p-Benzoquinone I

Illustration 12: Differentiate the following pairs of polymers based on the property mentioned against each. 
 (JEE ADVANCED)
(i) Novolac and bakelite (structure).
(ii) Buna-S and terylene (intermolecular forces of attraction).
- -
O O
Sol: (i) Novolac is a linear but
bakelite is a cross-linked polymer O CH2 CH2 O C
+
C
+
of phenol and formaldehyde. n
(ii) Terylene contains ester functional Dipole-dipole interaction Dipole-dipole interaction
groups that are polar in nature. - -
O O
Therefore, the intermolecular forces
of attraction involved in terylene are O CH2 CH2 O C
+
C
+
strong dipole–dipole interaction as n
shown below:
Terylene polymer chains
Buna-S, on the other hand, does
not have polar functional groups. It has only nonpolar hydrocarbon C6H5
chains and hence has only weak Van Der Waals forces of attraction as shown below:
Weak van der Waals
Forces of attraction

C6H5
Buna-S polymer Chains
2 6 . 3 2 | Biomolecules and Polymers

POINTS TO REMEMBER

Sugar
On the basis of physical
Non-sugar
characteristics
Monosaccharides
Carbohydrates On the basis of hydrolysis Oligosaccharides
Polysaccharides
Reducing
On the basis of test with reagents
Non-neducing
Maltose
Dissachharides Sucrose
Lactose

Open chain (Fischer Projection)


Representation
Structure Haworth
Cyclic structure
Chain form
Fibrous Proteins
Proteins
Globular Proteins

Primary structure
-helix
Structure Secondary structure
of -pleated
Protein Tertiary structure
Quaternary structure

Basic amino acids


Amino acids Neutral amino acids
Acidic amino acids

On the basis of source.


On the basis of structure.
Polymers On the basis of synthesis.

On the basis of molecular force.


On the basis of type of polymerization.
Chem i str y | 26.33

Solved Examples

JEE Main/Boards Example 5: Define the terms:


(A) Gene (B) Genetic code
Example 1: What type of bonding occurs in globular (C) Transcription (D) Translation
protein?
(E) Codons
Sol: Globular protein is spherical in nature and are
Sol: (A) Gene: A gene is a sequence of base triplets in a
water soluble. Globular protein may have the following
strand of DNA helix that factions to code a polypeptide
types of bonding: hydrogen bonding, disulphide
chain. The polypeptide chain ultimately becomes the
bridges, ionic or salt bridges, and hydrophobic
part of the protein synthesized. Every protein in a cell
interactions.
has the corresponding gene.
(B) Genetic code: The relation between the amino acids
Example 2: What will be the sequence of bases on
and the nucleotide triple is called is genetic code.
mRNA molecules synthesized on the following strand
of DNA? (C) Transcription: The way the code on DNA is copied
to give the complementary code on RNA is called
TATCTACCTGGA transcription.
Sol: The opposite bases bind with the strand i.e. A-T (D) Translation: The way the four-base code in nucleic
and G-C. acid is turned into a 20 unit code needed to specify
Sequence of bases on mRNA molecule: The amino acid sequence in proteins during synthesis
DNA strand TAT CTA CCT GGA is called translation.
m-RNA AUA GAU GGA CCU (E) Codons: The nucleotide bases in RNA function in
groups of three (triplets) in coding amino acids. These
base triplets are called codons.
Example 3: Name one reducing and one nonreducing
disaccharide.
Example 6: a. Write the structure of histidine when pH
Sol: Reducing sugar has a free aldehydic group which < 1.82 and pH > 1.82.
can be easily oxidized and can thus reduce other
substances. Sol: a. HN

-H
Maltose or lactose (reducing) COO H
(COOHCOO-)
Sucrose (non-reducing)
:


N NH3
HN
H
COO-
pH>1.82
Example 4: Explain the functions of nucleic acids.
:


N NH3
Sol: Nucleic acids are large biomolecules consisting of H
RNA and DNA. pH<1.82

Nucleic acids have two important functions:


i) Replication: Due to its unique property of self- Example 7: ‘The two strands of DNA are not identical,
replication (process by which a single DNA molecule but are complementary’. Explain this statement.
produces two identical species of itself), it is responsible
for maintaining the heredity traits from one generation Sol: DNA consists of two strands of polynucleotides
to another. coiled around each other in the form of a double helix. The
nucleotides making up each strand of DNA are connected
ii) Protein synthesis: RNA helps in the biosynthesis of
by phosphate ester bonds. This forms the backbone of
proteins and is, in one way, responsible for the process
each DNA strand, from which the bases extend. The
of learning and memory storage. Furthermore, it
bases of one strand of DNA are paired with the bases
sends information and instructions to the cell for the
on the other strand by means of hydrogen bonding. This
manufacture of specific proteins.
2 6 . 3 4 | Biomolecules and Polymers

hydrogen bonding is very specific as the structure of (i) HO CH2 6


bases permits only one mode of pairing. Adenine pairs
only thymine via two hydrogen bonds and guanine pairs HO CH 5 O
Me
with cytosine through three hydrogen bonds. The two 4
OH
1 +2 O
strands of DNA are said to be complementary to each OH
Me
2
other in the sense that the sequence of bases in one 3

strand automatically determines that of the other. These OH (i) PhCH2Cl/OH
(i) -D-Glocofuranose (B)
strands are not identical. (A)
Acetone/H2SO4
(A) (i) H
-D-Glucopyranose (C)
Example 8: Which purine and pyrimidine bases are (Two pair of OH are cis)
present in DNA and RNA? (One pair at C-1 and C-2)

(Another pair at C-5 and C-6) (i) PhCH2Cl/OH
Sol: Adenine and guanine (purine base) in DNA and (ii) -D-Glocofuranose
Acetone/H
(E)
(D)  (i) H
RNA. Cytosine, thymine, and uracil (pyrimidine base) O CH2
6
(i) PhCH2Cl/OH Me (F )
(ii) H
Example 9: What are the monomers constituting
Me CH 5 O
O 
proteins? (i) PhCH2Cl/OH
(iii)HOCH
-D-Glucopyranose
6 (H)
2
Acetone/H 4 OH 1
(i) H O
Sol: The amino acids such as glycine and alanine are 5
(G)
Me
the monomers that constitute proteins. HOCH 2
O 3 (F )
O
4 1 OH Me
OCH Ph
Example 10: What is the effect of pH on the action of (iv) -D-Glucopyranose
2
(K) (B)
2 OH 
enzyme? 3 (J) Acetone/H
(Cyclic diketal) (Diacetonide)
OH
Sol: The low or high pH values can cause denaturation
of the protein and hence make enzyme’s protein inactive.
(ii) HO CH2 6

JEE Advanced/Boards HO CH 5 O OH Me H
 4 1 + O
OH
Example 1: Complete the following reactions.
(i) -D-Glocofuranose (B)
(i) PhCH2Cl/OH Me
Acetone/H2SO4 (i) PhCH(i)Cl/OH 
 3 2
(A) H
(i) -D-Glocofuranose (B) 2 OH
Acetone/H2SO4 (C)
(A) (i) H
(C) (D)
 -D-Glucofuranose
(i) PhCH2Cl/OH (One pair of cis OH group)
(ii) -D-Glocofuranose (E) 
(D) Acetone/H (i) PhCH(i) H
2Cl/OH (at C-5 and C-6)
(ii) -D-Glocofuranose (E)
(D) Acetone/H (i) H (F )
(F ) Me
 (i) 3PhCH2/Cl2 OH- O-CH26
(i) PhCH2Cl/OH
(iii) -D-Glucopyranose (H) 
Acetone/H (i) PhCH(i)2Cl/OH
(ii) H
(iii) -D-Glucopyranose
(G) (H) H Me O-CH 5
Acetone/H O
(G) (i) H (F ) 6
HOCH2 4 1
(F ) OH
(iv) -D-Glucopyranose 
(K) HOCH 5 O OCH2Ph 3 2
(iv) -D-Glucopyranose
(J) Acetone/H (K)
 4 OH
(J) Acetone/H OCH2Ph 1 (E)
(Cyclic ketal) (acetonide)
Sol: Acetone forms a cyclic ketal called an acetonide 2
3
with two cis (OH) groups. D-Glucopyranose is in OCH2Ph
6
equilibrium
(ii) HOwith
CHsome
2 D-glucofuranose (having two
(ii) HO CH2 6 (F)
pais of cis OH groups), the formation of diketal shift the
HO CH55 O
equilibrium toward
HO CH O the reaction
OH
OH D-glucofuranose.
Me 
H
4
OH 1 + O Me H
4 1 + O Me
OH
Me
3 2
3 2
OH
OH

(D)
(D)
-D-Glucofuranose
-D-Glucofuranose
(One pair of cis OH group)
Chem i str y | 26.35

(iii) 1
CHO
1
COOH
6
CH2OH HO 2 [O] HO 2

3 OH HNO3 3 OH
5
O Me
H 4
CH2OH
4
COOH
4 1 + O
OH Me D-Threose D(-)Tartaric acid
HO 2 OH (Optically active enantiomer)
3
H2/N2
OH [H] or
(G)
NaBH4
-D-Glucopyranose
(One pair of cis OH group) 1
(at C-1 and C-2) CH2OH D-Buan-1, 2, 3, 4-tetraol
CH2O
OH HO 2 (Optically active enantiomer)
 5 3 OH
(i) PhCH2Cl/OH 1 4
4 OH CH2OH
(ii) H HO 2 Me
3
Me Example 3: a. Name the smallest aldose that forms
CH2OCH2Ph cyclic hemiacetal and the functional groups are involved
(H)
5
O in its formation.
4 OCH2Ph b. What is invert sugar?
PhHO2CO 3
OH c. Calculate the specific rotation of invert sugar.
OH Given, α|D of D-glucose = 52.7º
|α|D of D-fructose = –92.4º
(iv) 6
CH2OH
d. Give the mechanism of mutarotation of β-D-gluco-
5
O OH
Me pyranose in (i) aq. H⊕ and (ii) OHΘ.
4 OH 1 + O No reaction e. Why is the mutarotation faster in the presence of
Me
HO 2-pyridinol?
3 2

OH
Sol: a.
(J) 1
-D-Glucopyranose CH=O 1
O
H C OH
2 OH O 
(No pair of cis OH group) 2 OH
3 OH 3 OH OH
4
Example 2: Differentiate between D-erythrose and CH2OH 4
OH OH
D-threose by D-Erthrofuranose
1
(a) Mild oxidation and (b) Reduction. HO C OH O OH
2 OH O  4 1
Sol: 1
CHO
1
COOH
3 OH
4 3 2
2 OH [O] 2 OH OH OH
-D-Erthrofuranose
3 OH HNO3 3 OH 1
4 4 CH=O O
CH2OH COOH 2 H C OH
HO O 
D-Eryhrose Meso-tartaric acid HO 2
OH
Plane of symmetry 3 OH OH
H2/N2 3 OH
(Optically inactive) 4
CH2OH
[H] or 4
OH
NaBH4 -D-threofuranose
1 O
CH2OH HO C OH OH
2 O 
2 OH Meso-butan-1, 2, 3, 4-tetraol HO OH
(Optically inactive) 3 OH
3 OH
4
4 OH
CH2OH
-D-threofuranose
1 1
CHO COOH
HO 2 [O] HO 2
HNO
2 6 . 3 6 | Biomolecules and Polymers

b. Equimolar mixture of D-glucose and D-fructose Sol: (i)


obtained but the hydrolysis of sucrose is called invert 1 1 1
sugar. Since the specific rotation of sucrose is positive CHO CHO CHO
2 HO 2 HO HO 2
and after hydrolysis it changes to negative value and
Killiani
synthesis
this process is called inversion of cane sugar. HO 3
Refer to Section
3 HO 3 HO

c. The specific rotation is half the sum of specific 4 HO HO 4 HO 4

rotation of glucose and fructose. 5 HO 5 HO 5 HO


1
Specific rotation of invert sugar = [ +52.7º +(–92.4º )]
6
CH2OH 6 HO
= –19.99º 2 D-glucose 6 HO
7 7
(A) CH2OH CH2OH
d. i. In acidic medium: (C-2) Epimer of
(B) (B)
The smallest aldose is a tetraldose that has four C atoms HNO3 HNO3
and an O atom to form a five-membered ring. The CHO 1 1
group and 1º OH group of tetrose are involved in the COOH COOH
formation of ring. Both erythrose and threose form 2 HO HO 2

cyclic hemiacetals. The rate depends on the conversion Plane of symmetry 3 HO 3


of cyclic hemi- (ii)acetal to open-chain aldehyde (i). 4 4
HO H HO
e.2-Pyridinol is an acid-base catalyst [containing both
 5 HO 5 HO
basic (N) and acidic (OH) group] that gives H ion to
hemiacetalic O atom and simultaneously removes H⊕ 6 HO 6 HO
ions from the HO − group of the hemiacetal by the 7
COOH
7
COOH
formation a cyclic intermediate transition state. Meso-acid (C) (It is optically active)

(ii)
1 1 1
CHO CHO CHO
:

H O N
2 OH 2 OH HO 2
O H
Killiani
:

O O H 3 OH 3 OH 3 OH
synthesis
H OO
:

N
H (I) 4 OH 4 OH 4 OH
H
-Pyranonse
5 OH 5 OH 5 OH
O H 6
CH2OH 6 OH 6 OH
OH D-Allose
7 7
(A) CH2OH CH2OH
-Pyranonse
(C-2) Epimer of
(B) (B)
Example 4: Complete the following reactions: HNO3 HNO3

HNO 1 1
(i) Killiani HNO33
(i) Glucose Killiani Pair
(i) DD -- Glucose Pair COOH COOH
synthesis
synthesis
(A)
(A) (B)
(B) 2 OH HO 2
Meso-Heptaldaric
Meso-Heptaldaric acid
acid ((CC)) Plane of symmetry 3 OH 3 OH
4 4
(ii)
(ii)
(ii) DD -- Allose
Allose Killiani Pair HNO
Killiani HNO33 HO OH HO
Pair
synthesis
synthesis
(D)
(D) (B)
(B) 5 OH 5 OH
Meso-Heptaldaric
Meso-Heptaldaric acid
acid (F)
(F)
6 OH 6 OH
Explain whether the acids (C) and (F) are same or 7 7
COOH COOH
different. Which pair out of (B) and (E) gives meso-acids
Meso-acid (It is optically active)
(C) and (F)?
Although both acids (C) and (F) are heptaldric acid and
both are meso (optically inactive) but they are different.
Chem i str y | 26.37

Example 5: a. How is the mixture of aspartic acid Ph3C - C1 only


CH2OCPh3
(A) histidine (B) and threonine (C) separated by CH2OH is etherified O 
Me2SO4OH
electrophoresis method? protection of OH OMe Methylation
HO of C-2, C-3, C-4.
pI (pH at isoelectric point) are given. (CH2OH)
(OH) gp.
OH
(C)
pI of (A), (B), and (C) are 2.77, 7.59 and 5.60, respectively.
Tritylated methyl-D-glucopyranoside
b. How are they separated by solubility method? 6
CH2OCPh3 CH2OCPh3
O O
Sol: a. Choose the intermediate pH of 5.60. This is the OMe
5
OMe H
OMe OMe
4
pI of threonin, which has a zero net charge and does eMe
1
MeO
2
not migrate in the electric field or in the electrophoresis
3
OMe OMe
experiment.
(E) (D)
Aspartic acid (pI = 2.77) [Refer to solved example No. 8 (d) 2,3,4-Trimethyl glucopyranoside
above] donates an H⊕ and is converted to anion (III), and
migrates to the anode. Histidine (pI = 7.59) accepts an H⊕
and is converted to a cation, and migrates to the cathode. Example 7: Write the names and structures of the
monomers of the following polymers:
b. At isoelectrical point (pI) the amino acids have least
solubility in water and this property is exploited in the (i) Buna-S (ii) Buna-N (iii) Dacron (iv) Neoprene.
separation of different amino acids obtained from the
hydrolysis of protein. Sol: The names and structures of the monomers are:
To the mixture of three amino acids (A, B and C) set the (i) Buna-S:
pH of the solution by adding acid upto 2.77, at which
CH2 CH — CH CH2 and C6H5 — CH CH2
(A) will be least soluble in H2O and will precipitate out.= =
1,3−Butadiene
=
Styrene
It is followed by the separation of amino acid (A).
(ii) Buna-N:
Increase the pH of the remaining solution by adding
base upto 5.60, at which amino acid (c) will precipitate= CH2 CH — CH CH2 and CH
= =2 — CH CN
out. Similarly, (B) will precipitate out at the pH of 7.59. 1,3−Butadiene Acrylonitrile

(iii) Neoprene:
Example 6: Convert D-glucopyranose to 2, 3,
Cl
4-trimethyl glucopyranoside. CH2
CH2
Sol: First convert D-glucose to methyl glucopyranoside
Chloroprene or 2 − chloro − 1,3 − butadiene
with MeOH/HCl. Then CH2OH is protected by reacting
methyl glucopyranoside with Ph3C—Cl (trityl chloride), (iv) Dacron:

CH2OH CH2OH HO — CH2 — CH2 — OH and


Ethyleneglycol
O O
MeOH/HCl
OH OH OH OMe
HO HO HOOC HOOC
OH OH
(A) (B)
Terephthalic acid
(-or -D-glucopyrnose) Methyl-D-glucopyranoside

Only CH2OH is etherified giving CH2OCPh3 group. The


Example 8: Identify the monomer in the following
2º OH groups are sterically hindered and do not react
polymeric structures:
with the bulkyl (Ph3C—Cl) group.
(i) H H
The free OH groups of tritylated methyl glucosides are
now methylated with Me2SO4/NaOH followed by acidic
hydrolysis removes both trityl group and the glycosidic
Me group leaving OMe group at C2, C3, and C4 intact.
N
 6
N
 8
The hydrolysis of –OCPh3, groups proceeds by the O O
formation of stable Ph3 C⊕ (triphenyl methyl carbocation). n
2 6 . 3 8 | Biomolecules and Polymers

Example 9: How does the presence of double bonds


(ii) N
HN NH CH2 in rubber molecules influence their structure and
reactivity?
N N
Sol: Natural rubber is cis-polyisoprene and is obtained
NH n by 1, 4-polymerization of isoprene units. In this polymer,
double bonds are located between C2 and C3 of each
isoprene unit. These cis-double bonds do not allow the
Sol: Monomers are:
polymer chains to come closer for effective interactions
and hence intermolecular forces are quite weak. As
(i) HOOC — (CH2 )8 — COOH
Decanoic acid or Sebacic acid
a result, natural rubber, i.e., cis-polyisoprene has a
randomly coiled structure and hence shows elasticity.
and H2N — (CH2 )6 — NH2
Hexamthylenediamine 2 4
1 n
6 N 2 1 3
(ii) H2N NH2
Isoprene
3
N N
5 4
NH2
Melamine or 2 3 2 3
1
2.4.6.-Triamino-1,3,5-triazine CH2
4 1 CH2
and 4
H
C=O cis-Polyisoprene
(Natural rubber)
H
Formaldehyde

JEE Main/Boards

Exercise 1 Q.5 How do you explain the amphoteric behavior of


amino acids?
Q.1 What happens when D-glucose in treated with
the following reagents? Q.6 What is the effect of denaturation on the structure
(i) HI (ii) Bromine water (iii) HNO3 of proteins?

Q.2 Define the following terms in relation to proteins Q.7 Describe the following: (i) Glycosidic linkage

(i) Peptide linkage (ii) Denaturation


Q.8 Enumerate the reactions of D-glucose which cannot
be explained by its open chain structure.
Q.3 Define the following as related to proteins
(i) Peptide linkage (ii) Primary structure Q.9 What are essential and non-essential amino Acids?
(iii) Denaturation Give two examples of each type.

Q.4 What are the common types of secondary structure Q.10 List any four vitamins. Mention the chief sources
of proteins? and functions of two of them.
Chem i str y | 26.39

Q.11 (i) What are essential and non-essential amino Q.24 Distinguish between α-glucose and β-glucose.
acids? Give two example of each.
(ii) What is a denatured protein? Q.25 What happens when L-glucose is treated with the
following reagents?
Q.12 How are vitamins classified? Name the vitamin (i) HI (ii) Bromine water (iii) HNO3
responsible for the coagulation of blood.
Q.26 Write the important structural and functional
Q.13 Why are vitamins A and vitamin C essential to us? differences between DNA and RNA.
Give their important sources.
Q.27 What are the different types of RNA found in the cell?
Q.14 Draw open chain structure of aldopentose and
aldohexose. How many asymmetric carbons are present Q.28 Define the following and give one example of
in each? each
(a) Isoelectric point (b) Mutarotation
Q.15 (a) Describe the following giving one example: (c) Enzymes
Nucleotide.
(b) List four functions of carbohydrates in living Q.29 Answer the following queries about proteins?
organisms.
(i) How are proteins related to amino acid?
Q.16 What type of bonding helps in stabilizing the (ii) How are oligopeptides different from polypeptides?
α-helix structure of proteins? (iii) When is a protein said to be denatured?

Q.17 Differentiate between globular and fibrous


Q.30 (a) Name the three major classes of carbohydrates
proteins.
and give the distinctive characteristic of each class.
Q.18 (a) Give reasons for the following statements: (b) What are nucleotides? Name two classes of nitrogen
containing bases found amongst nucleotides.
(i) Amino acids have comparatively higher melting
points than the corresponding haloacids.
(b) What deficiency diseases are caused due to lack to Exercise 2
lack of vitamins A, B1, B6 and K in human diet?
Biomolecules
Q.19 The two strands in DNA are not identical but are
complementary. Explain. Single Correct Choice Type

Q.1 The chromophore in the dye


Q.20 State difference between the following pair
(i) α-helix and β-pleated structures. HO3S N=N N(CH3)2 is

(ii) Primary and secondary structures of a protein.


(A) –N(CH3)2 (B) –SO3H
Q.21 What are nucleic acids? Mention their two (C) –C6H5 (D) –N = N–
important functions.
Q.2 At the isoelectric point for an amino acid the
Q.22 What is the difference between a nucleoside and species present are
a nucleotide?
(A) R – CH – COOH (B) R – CH – COOH
| |
+
NH2 NH3
Q.23 What are reducing and non-reducing sugar?
What is the structural feature characterizing reducing –
(C) R – CH – COO – (D) R – CH – COO
sugars? | |
+
NH2 NH3
2 6 . 4 0 | Biomolecules and Polymers

Q.3 Secondary structure of a protein refers to Q.12 The reagent used in Ruff’s degradation is
(A) Mainly denatured proteins and structures of (A) Baeyer’s reagent (B) Tollen’s reagent
prosthetic groups
(C) Fentons’ reagent (D) Benedict’s reagent
(B) Regular folding patterns of contiguous portions of
the polypeptide chain Q.13 If Ka1 and Ka2 are the ionization constants of
(C) Linear sequence of amino acid residues in the H3N+CHICOOH and H3N+CHICOO–, respectively, the pH
polypeptide chain of the solution at the isoelectric point is

(D) None of these (A) pH = pKa1 + pKa2 (B) pH = (pKa1 + pKa2)1/2


(pK a1 + pK a2 )
(C) pH = (pKa1 + pKa2)1/2 (D) pH =
Q.4 The general formula of carbohydrates is: 2

(A) CnH2n+1O (B) CnH2nO Q.14 Coordination polymerization was developed by


(C) Cn(H2O)n or Cx(H2O)y (D) Cn(H2O)2n (A) Zeigler and Natta (B) Linus Pauling
(C) Beckamann (D) None of these
Q.5 Which of the following is a disaccharide?
(A) Sucrose (B) Glucose Q.15 Teflon, polystyrene and neoprene are all
(C) Fructose (D) Starch (A) Copolymers (B) Condensation polymers
(C) Homopolymers (D) Monomers
Q.6 The iron in hemoglobin is bound by
(A) Hydrogen bonds (B) Chelation Q.16 Carbohydrates which differ in configuration at the
glycoside carbon (i.e. C1 in aldose and C2 in ketoses) are
(C) Ionic bonds (D) Covalent bonds called
(A) Anomers (B) Epimers
Q.7 Anomers have different
(C) Diastereomers (D) Enantiomers
(A) Properties (B) Melting points
(C) Specific rotation (D) All of these Q.17 Choose the correct relationship for α-D-glucose
(1) and β-D-glucose (2)
Q.8 Peptide bond is (A) A and B are epimers
(A) – CO – NH – (B) NH2 – CO – NH – R (B) A and B are crystal modification
(C) A is a pyranose sugar and B is furanose sugar
(C) R – CO – NH – R (D) – CONH2
(D) A is an aldose and B is a ketose.

Q.9 Glucose and Fructose are


Q.18 Natural rubber is a polymer of
(A) Tautomers (B) Chain isomers
(A) Chloroprene (B) Isoprene
(C) Functional isomers (D) Geometrical isomers
(C) 1, 3-Butadiene (d) None

Q.10 Glucose is
Q.19 Hydrolysis of sucrose is called
(A) Aldopentose (B) Aldohexose
(A) Saponification (B) Inversion
(C) Ketopentose (D) Ketohexose
(C) Esterification (D) Hydration

Q.11 A substance which can act both as an antiseptic


and disinfectant is Q.20 In vulcanization of rubber

(A) Aspirin (B) Chloroxylenol (A) Sulphur reacts to form a new compound
(C) Bithinal (D) Phenol (B) Sulphur cross-links are introduced
(C) Sulphur forms a very thin protective layer over rubber
(D) All statements are correct
Chem i str y | 26.41

Q.21 The simplest amino acid is Q.30 Main structural unit of protein is
(A) Glycine (B) Alanine (A) Ester linkage (B) Ether linkage
(C) Guanine (D) All of the above (C) Peptide linkage (D) All the above

Q.22 Which of the following belong to the class of Q.31 Which of the following statements is true of
natural polymers? proteins?
(A) Proteins (B) Cellulose (A) They catalyse the biochemical reactions
(C) Rubber (D) All of the above (B) They act as antibodies
(C) They perform all these functions
Q.23 D-Glucose and β-D glucose differ from each other
due to difference in one of carbon with respect to its (D) They perform all these functions
(A) Size of hemiacetal ring
(B) Number of —OH groups Q.32Which of the following is a polysaccharide?

(C) Configuration (A) Glucose (B) Galactose


(D) Conformation (C) Sucrose (D) Pectines

Q.24 Glucose gives the silver mirror test with ammoniacal Q.33 Starch can be used as an indicator for the detection
solution of silver nitrate because it contains of traces of
(A) Aldehydes group (B) Ester group (A) Glucose in aqueous solution
(c) Ketone group (D) Amide group (B) Proteins in blood
(C) Iodine in aqueous solution
Q.25 Oligosaccharides contain ……………. Simple sugar
units (D) Urea in blood
(A) 2 to 10 (B) 4 to 8 (C) 6 to 12 (D) 6 to 10
Q.34 Which of the following statements about ribose
Q.26 A pair of diastereomers that differ only in the in incorrect?
configuration about a single carbon atom are called (A) It is polyhydroxy compound
(A) Anomers (B) Epimers (B) It is an aldehyde sugar
(C) Conformes (D) Enantiomers (C) It has six carbon atoms
(D) It exhibits optical activity
Q.27 Pick out the incorrect statement about ATP.
(A) It is a nucleotide
Q.35 Which of the following is the structure of D-xylose?
(B) It contains the purine, adenine
CHO CHO
(C) The enzyme-catalysed hydrolysis of ATP to ADP and
AMP is accompanied by absorption of energy HO H HO H
(D) Energy is stored in the cell in the form of ATP. (A) H OH (B) HO H
H OH H OH
Q.28 Cellulose is a linear polymer of
CH2OH CH2OH
(A) Glucose (B) Glucose
(C) Fructose (D) None of these
CHO CHO

Q.29 Glucose molecule reacts with X number of H H H OH


molecules of phenylhydrazine to yield osazone. The (C) OH H (D) H OH
value of X is H OH H OH
(A) Three (B) Two (C) One (D) Four
CH2OH CH2OH
2 6 . 4 2 | Biomolecules and Polymers

Q.36 Glucose gives the silver mirror test with Q.44 P.V.C. is formed by polymerization of-
ammoniacal solution of silver nitrate because it contains
(A) 1-Chloroethene (B) Ethene
the group
(C) Propene (D) 1-Chloropropene
(A) Aldehyde (B) Ester
(C) Ketone (D) Amide Q.45 Polyacrylonitrile, characterized by the repeating
unit, is made from which of the following monomer?
Q.37 A condensation polymer among the following is-
(A) CH3CH2CN (B) HOCH2CH2CH3
(A) Dacron (B) PVC
(C) CH3CH = CHCN (D) CH2 = CHCN.
(C) Polystyrene (D) Teflon
Q.46 On the basis of intermolecular forces, polymers
Q.38 Melamine polymer is copolymer of are classified as
(A) Melamine and acetaldehyde (A) Elastomers, Fibres, Thermoplastics and Thermosetting
(B) Melamine and formaldehyde (B) Elastomers, Fibres, Chain growth and Step growth
(C) Phenol and formaldehyde (C) Addition polymers and Condensation polymers

(D) None of the above (D) None of these

Q.47 Which of the following polymers do not involve


Q.39 Which one of the following pairs is not correctly
cross linkages-
matched?
(A) Melmac (B) Bakelite
(A) Terylene: Condensation polymer of terephthalic
acid and ethylene glycol (C) Polythene (D) Vulcanised rubber
(B) Perspex: A homopolymer of methymethacrylate
Q.48 Ziegler-Natta catalyst is-
(C) Teflon: Thermally stable cross-linked polymer of
phenol and formaldehyde (A) K[KPtCl3(C2H4)] (B) (Ph3P)3RhCl
(D) Synthetic rubber: A copolymer of butadiene and (C) Al(C2H5)3+TiCl4 (D) Fe(C5H5)2
styrene
Q.49 Which one of the following monomers give the
Q.40 Orlon is a polymer of- polymer neoprene on polymerization?
(A) Styrene (B) Tetrafluoroethylene (A) CH2 = CHCl (B) CH2 = CCl – CH = CH2
(C) Vinyl chloride (D) Acrylonitrile (C) CF2 = CF2 (D) CCl2 = CCl2

Q.41 Which one of the following is not an example of Q.50 Which of the following pairs is not correctly
chain growth polymer- matched?
(A) Neoprene (B) Buna-S (A) Terylene-condensation polymer of terephthalic acid
and ethylene glycol
(C) PMMA (D) Glyptal
(B) Teflon-thermally stable cross linked polymer of
phenol and formaldehyde
Q.42 Ebonite is-
(A) Natural rubber (C) Perspex-a homopolymer of methyl methacrylate

(B) Synthetic rubber (D) Synthetic rubber –a copolymer of butadiene and


styrene
(C) Highly vulcanized rubber
(D) Polypropene
Q.51 Which one of the following is used to make
‘non-stick’ cookware?
Q.43 F2C = CF2 is a monomer of-
(A) PVC
(A) Teflon (B) Glyptal (C) Nylon-6 (D) Buna-S
(B) Polystyrene
Chem i str y | 26.43

(C) Polyethylene terephthalate Q.5 The two functional groups present in a typical
carbohydrate are: (2009)
(D) Polytetrafluoroethylene
(A) –OH and –COOH (B) –CHO and –COOH
Q.52 Which compound/set of compounds is used in (C) > C = O and –OH (D) –OH and –CHO
the manufacture of nylon-66?
(A) HOOC(CH2)4COOH+H2N(CH2)6NH2 Q.6 Buna-N synthetic rubber is a copolymer of:(2009)
Cl
(B) CH2 = CH–CH(CH) = CH2 |
(A) H2C = CH − C = CH2 and H2C = CH − CH = CH2
(C) CH2 = CH2 HOOC COOH
(B) H2C = CH − CH = CH2 and H5C6 − CH = CH2
(D) HOOC COOH + HOCH2 – CH2OH
(C) H2C = CH − CN and H2C = CH − CH = CH2

+ HOCH2 – CH2OH (D) H2C =CH − CN and H2C =CH − C =CH2


Q.53 Teflon, styron and neoprene are all- |
CH3
(A) Copolymers (B) Condensation polymers
(C) Homopolymers (D) Monomers Q.7 Biuret test is not given by  (2010)
(A) Carbohydrates (B) Polypeptides
(C) Urea (D) Proteins
Previous Years’ Questions
Q.8 The polymer containing strong intermolecular
Q.1 Which of the following pairs give positive Tollen’s
forces e.g. hydrogen bonding, is (2010)
test?  (2004)
(A) Teflon (B) Nylon 6, 6
(A) Glucose, sucrose (B) Glucose, fructose
(C) Polystyrene (D) Natural rubber
(C) Hexanal, acetophenone (D) Fructose, sucrose

Q.9 The presence or absence of hydroxyl group on


Q.2 Two forms of D-glucopyranose, are called  (2005) which carbon atom of sugar differentiates RNA and
(A) Enantiomers (B) Anomers DNA? (2011)
(C) Epimers (D) Diastereomers (A) 2nd (B) 3rd (C) 4th (D) 1st

Read the following questions and answer as per the Q.10 Which one of the following statements is correct?
direction given below:  (2012)
(a) Statement-I is true; statement-II is true; statement-II (A) All amino acids except lysine are optically active
is not the correct explanation of statement-I.
(B) All amino acids are optically active
(b) Statement-I is true; statement-II is true; statement-II
is not the correct explanation of statement-I. (C) All amino acids except glycine are optically active
(c) Statement-I is true; statement-II is false. (D) All amino acids except glutamic acid are optically
active
(d) Statement-I is false; statement-II is true.

Q.11 Synthesis of each molecule of glucose in


Q.3 Statement-I: Glucose gives a reddish-brown
photosynthesis involves: (2013)
precipitate with Fehling’s solution.
(A) 18 molecules of ATP (B) 10 molecules of ATP
Statement-II: Reaction of glucose with Fehling’s
solution gives CuO and gluconic acid. (2007) (C) 8 molecules of ATP (D) 6 molecules of ATP

Q.4 α − D − ( + ) - glucose and β − D − ( + ) - glucose are  Q.12 Which one is classified as a condensation
 (2008) polymer?  (2014)
(A) Conformers (B) Epimers (A) Dacron (B) Neoprene
(C) Anomers (D) Enantiomers (C) Teflon (D) Acrylonitrile
2 6 . 4 4 | Biomolecules and Polymers

Q.13 Which one of the following bases is not present in (A) It is a poor conductor of electricity.
DNA? (2014)
(B) Its synthesis required dioxygen or aperoxide initiator
(A) Quinoline (B) Adenine as a catalyst.
(C) Cytosine (D) Thymine (C) It is used in the manufacture of buckets, dust-bins
etc.
Q.14 Which polymer is used in the manufacture of (D) Its synthesis requires high pressure.
paints and lacquers?  (2015)
(A) Bakelite (B) Glyptal Q.17 Which of the following is an anionic detergent?
 (2016)
(C) Polypropene (D) Poly vinyl chloride
(A) Sodium lauryl sulphate
Q.15 Which of the vitamins given below is water (B) Cetyltrimethyl ammonium bromide
soluble? (2015)
(C) Glyceryl oleate
(A) Vitamin C (B) Vitamin D
(D) Sodium stearate
(C) Vitamin E (D) Vitamin K
Q.18 Thiol group is present in:  (2016)
Q.16 Which of the following statements about low
(A) Cystine (B) Cysteine
density polythene is FALSE?  (2016)
(C) Methionine (D) Cytosine

JEE Advanced/Boards

Exercise 1 Q.7 What are essential and non-essential amino acids?


Give two examples of each.
Q.1 State a use for the enzyme streptokinase in
medicine. Q.8 (a) Define the following term:
(i) Co-enzymes
Q.2 Why is cellulose in our diet not nourishing?
Q.9 (a) Answer the following questions briefly
Q.3 Explain muta-rotation taking D-glucose as an
example. (i) What are any two good sources of vitamin A?
(ii) What are nucleotides?
Q.4 Enumerate the structural difference between DNA
and RNA. Write down the structure of sugar present in (b) How are carbohydrates classified?
DNA.
Q.10 Write two main functions of carbohydrates in
Q.5 Answer the following queries about proteins- plants.

(i) How are proteins related to amino acids? Q.11 What happens when D-glucose is treated with the
(ii) How are oligopeptides different from polypeptides? following reagents?

(iii) When is a protein said to be denatured? (i) alk.KMnO4 (ii) Br2 + CS2 (iii) H2SO4

Q.6 (a) Define and classify vitamins. Give at least two Q.12 Name the four bases present in DNA. Which one
example of each type. of these is not present in RNA?

(b) Define an enzyme and comment on the specificity Q.13 Name two fat soluble vitamins that sources and
in action of an enzyme. Illustrate with an example. diseases caused due to their deficiency in diet.
Chem i str y | 26.45

Q.14 State a use for the enzyme streptokinase in Q.2 During hydrogenation of oils, higher melting
medicine. ‘vegetable ghee’ is formed because
(A) Hydrogen is dissolved in the oil
Q.15 Write the major classes in which the carbohydrates
are divided? (B) Hydrogen combines with oxygen of the oil
(C) Ester of unsaturated fatty acids are reduced to those
Q.16 Aspartame, an artificial sweetener, is a peptide
of saturated acids
and has the following structures:
NH2 CH2C6H5 (D) Hydrogen drives off the impurities from the oil
| |
HOOC – CH2CH – CH – COOCH3
Q.3 Structurally a biodegradable detergent should
(a) Identify the four functional groups. contain a
(b) Write the zwitter ionic structure (A) Normal alkyl chain (B) Branched alkyl chain
(c) Write the structure of the amino acids obtained from (C) Phenyl side chain (D) Cyclohexyl side chain
the hydrolysis of aspartame.
(d) Which of the two amino acids is more hydrophobic? Q.4 Thrust imparted to the rocket is governed by the
(A) Third law of thermodynamics
Q.17 Give the chemical name of vitamin B12.
(B) Gravitational law
Q.18 What are the following substances?
(C) Newton’s third law
(i) Invert sugar (ii) Polypeptides
(D) None of these
Q.19 Which forces are responsible for the stability of
α-helix? Why is it named as 3.613 helix? Q.5 Which of the following represent a bi-liquid
propellant?
Q.20 What are complementary bases? Draw structure (A) N2O4 + unsymmetrical dimethylhydrazine
to show hydrogen bonding between adenine and
thymine and between guanine and cytosine. (B) N2O4 + acrylic rubber
(C) Nitroglycerine + nitrocellulose
Q.21 Give reasons for the following:
(D) Polybutadiene + ammonium perchlorate
(i) On electrolysis in acidic solution amino acids migrate
towards cathode, while in alkaline solution these Q.6 ‘Placedo’ is often given to patients. It is
migrate towards anode.
(A) An antidepressant
(ii) The monoamino monocarboxylic acids have two pK
values. (B) A broad spectrum antibiotic
(C) A sugar pill
Q.22 Glycine exists as a Zwitter ion but anthranilic acid
does not comment. (D) A tonic

Q.23 Write the difference between DNA and RNA? Q.7 An aldohexose (e.g., glucose) and 2-oxohexose
(e.g., fructose) can be distinguished with the help of
Q.24 Explain structure of protein.
(A) Tollen’s reagent (B) Fehling’s solution
(C) Benedict solution (D) Br2 / H2O
Exercise 2
Single Correct Choice Type Q.8 The open-chain glucose on oxidation with HIO4
gives
Q.1 If the sequence of bases in one strand of DNA (A) 5 HCOOH + H2C = O
is ATGACTGTC, then the sequence of bases in its
complementary strand is (B) 4 HCOOH + 2 H2C = O

(A) TACTGACAG (B) TUCTGUCUG (C) 3 HCOOH + 3 H2C = O

(C) GUACTUAUG (D) None of these (D) 2 HCOOH + 4 H2C = O


2 6 . 4 6 | Biomolecules and Polymers

Q.9 Glucose and fructose give the same osazone. One Q.14 The best way to prepare polyisobutylene is
may, therefore, conclude that
(A) Coordination polymerization
(A) Glucose and fructose have identical structures
(B) Free radial polymerization
(B) Glucose and fructose are anomers
(C) Cationic polymerization
(C) The structure of glucose and fructose have mirror-
(D) Anionic polymerization
image relationship
(D) The structure of glucose and fructose differ only Q.15 Which of the following is not correctly matched?
in those carbon atoms which take part in osazone
formation.
 
 —CH — C = CH — CH — 
(A) Neoprene;  2
| 2 
Q.10 For α-amino acid having the structure
 Cl 
R — CH — CO2H  n

|
NH2 O O
(B) Nylon-6; NH (CH2)6 NH CO (CH2)4 C O
Which of the following statements are true? n
(a) Water solubility is maximum at a pH when O O
concentration of anions and cations are equal.
(C) Terylene; OCH2 CH2 O C C
(b) They give ninhydrin test
(c) On reacting with nitrous acid give of N2 n
CH3
(A) All (B) b and c
(C) a and b (D) None of these (D) PMMA; CH2 C

COOCH3
n
Q.11 Bakelite is obtained from phenol and
formaldehyde. The initial reaction between the two
compounds is an example of Q.16 Acrilan is a hard, horny and a high melting
material. Which of the following represent its structure?
(A) Aromatic electrophilic substitution
(B) Aromatic nucleophilic Substitution  
 CH3 
   —CH2 — CH — 
(C) Free radical reaction |  | 
(A)  —CH2 —C—  (B)  COOC2H5 
(D) Aldol reaction  | 
  
COOCH3  n
 
 n
Q.12 If N1, N2, N3,………. are number of molecules with
molecular masses M1, M2, M3, …………. respectively, then
(C)  —CH2 — CH —  (D)  —CH2 — CH
average molecular mass is expressed as  |   | 
 Cl   CN 
2
ΣNM
i i ΣNM
i i



n



n
(A) (B)
NM
i i ΣNi

(C) Both (A) and (B) (D) None of these Q.17 Which of the following statement/s is (are) correct?
(A) Vinyon is a copolymer of vinyl chloride and vinyl
Q.13 The ratio of weight average molecular mass to acetate
number average molecular mass is called as (B) Saran is copolymer of vinyl chloride and vinylidene
(A) Planck’s disposal index chloride

(B) Polydiagonal index (C) Butyl rubber is a copolymer of isobutylene and


isoprene
(C) Polydispersity index
(D) All are correct
(D) None of these
Chem i str y | 26.47

Q.18 Plexiglass (perspex) is Q.22 Assertion: The enzyme amylase hydrolyses starch
to maltose.
(A) Polyacrylonitrile (B) Polyethylacrylate
Reason: Starch is polymer containing glycosidic
(C) Polystyrene (D) Polymethylmethacrylate
linkages.

Multiple Correct Choice Type Q.23 Assertion: A solution of sucrose in water is


dextrorotatory but on hydrolysis in the presence of
Q.19 Which of the following statement (s) is (are) true?
small amount of dil. HCl, it becomes laevorotatory.
(A) All amino acids contain one chiral centre
Reason: Sucrose on hydrolysis gives unequal amounts
(B) Some amino acids contain one, while some contain of glucose and fructose as a result of which change in
more chiral center or even no chiral center sign of rotation is observed.
(C) All amino acids found in proteins have L configuration
Q.24 Assertion: Each turn of the α-helix structure of
(D) All amino acids found in proteins have 1º amino protein forms a 13 membered ring the containing 3.6
group amino acids.
Reason: α-helix is a secondary of protein which gets
Q.20 Pick out correct statements. stabilized via hydrogen bonding and disulphide linkages.
(A) In an electrolysis experiment, amino acids migrate
at the isoelectric point towards electrodes Q.25 Assertion: Styrene is more reactive than propylene
towards cationic polymerization.
(B) p-aminobenzensulphonic acid is a dipolar ion: while
p-aminobenzoic acid is not Reason: The carbocation resulting from styrene is more
stable than that resulting from propylene.
(C) Sulphanilic acid is soluble in base, but not in acid

(D) H3 N CH2COOH(pka = 2.4) is more acidic than Q.26 Assertion: Natural rubber is all cis-polyisoprene.

RCH2COOH (pKa = 4–5) Reason: trans-Polyisoprene cannot be formed.

Q.27 Assertion: PMMA is used for making lenses and


Q.21 Which of the following statements is/ are correct? light covers.
(A) Polyethylene contains double bonds Reason: It has excellent light transmission properties.
(B) The monomer used to make Teflon is C2F4
(C) Condensation polymers are known as copolymers Q.28 Assertion: Polyvinyl alcohol is obtained by
polymerization of vinyl alcohol
(D) A denatured protein could have the same primary
structure as the active protein. Reason: Polyvinyl alcohol is prepared by hydrolysis of
polyvinyl acetate.

Assertion Reasoning Type


Q.29 Assertion: Nylon-6 is obtained by polymerization
Questions 22-29 of caprolactum
Each of the questions given below consists of two Reason: It is a polyamide.
statements, an assertion (A) and reason (R). Select the
number corresponding to the appropriate alternative Comprehension Type
as follows
Paragraph 1: The utility of the polymers in various
(A) If both assertion and reason are true and reason is
fields is due to their mechanical properties like tensile
the correct explanation of assertion.
strength, elasticity, toughness etc.
(B) If both assertion and reason are true but reason is
These properties mainly depend upon intermolecular
not the correct explanation of assertion.
forces like van der Waal’s forces and hydrogen bonding
(C) If assertion is true but reason is false. operating in polymer molecules. Polymers have been
(D) If both assertion and reason are false. classified on this basis, e.g.,
2 6 . 4 8 | Biomolecules and Polymers

(A) Elastomers (B) Fibers


List I List II
(C) Thermoplastics (D) Thermosetting II. Nucleic acid (q) Sex hormone
III. Ascorbic acid (r) Vitamin C
Q.30 The molecular forces of attraction are weakest in
IV. Testosterone (s) Digestive enzyme
(A) Elastomers (B) Fibers
(A) I → s, II → p, III → r, IV → q
(C) Thermoplastics (D) Thermosetting polymers
(B) I → s, II → p, III → q, IV → r
Q.31 Which of the following have usually a linear structure? (C) I → s, II → p, III → r, IV → q
(A) Thermoplastics (B) Thermosetting polymers (D) None of these

(C) Polyethylene (D) Nylon-66


Q.37 Match list I with list II and select the correct answer
using the codes given below the lists.
Q.32 Which of the following is hard?
(A) Elastomer (B) Fibre List I List II

(C) Thermoplastic (D) Thermosetting polymers (Polymer) (Polymerizing units)


I. Bakelite (p) Butadiene and styrene
Paragraph 2: A natural elastomer polymer when with
II. Dacron (q) Phenol and methanal
sulphur got stiff and resistant to action of common
solvents and wear & tear. III. Nylon-66 (r) 1, 2-dihydroxyethane and
dimethylterephthalate
Q.33 The natural polymer is:
IV. Buna-S (s) Urea and methanol
(A) Rubber (B) Cellulose (C) Silk (D) Starch
(t) 1, 6-hexanedioic acid and
1, 6-dimino hexane
Q.34 Heating of polymer with sulphur is called:
(A) I → s, II → r, III → t, IV → p
(A) Galvanisation (B) Saponification
(B) I → q, II → r, III → t, IV → p
(C) Vulcanization (D) None of these
(C) I → t, II → q, III → r, IV → p
Match the Columns (D) None of these

Q.35 Match list I with list II and select the correct answer Q.38 Match list I with list II and select the correct answer
using the codes given below the lists. using the codes given below the lists.
List I List II
List I List II
I. Nucleic acids (p) D.N.A.
I. Phenol + formaldehyde. (p) Synthec rubber
II. Uracil (q) Hormones
II. Terephthalic acid (q) Bakelite + ethylene
III. Thymine (r) Polynucleotides glycol

IV. Double-helix structure (s) R.N.A. III. Caprolactam (r) Nylon-6

(A) I → s, II → r, III → p, IV → p IV. Butadiene+styrene (s) terylene

(B) I → r, II → s, III → p, IV → p (A) I → q, II → r, III → s, IV → p


(C) I → r, II → p, III → s, IV → p (B) I → r, II → p, III → q, IV → s

(D) None of these (C) I → q, II → s, III → r, IV → p


(D) None of these
Q.36
Q.39 Match list-I (Monomer) with list-II (Polymer) and select
List I List II
the correct answer using the codes given below the lists:
I. Pepsin (p) Genetic material
Chem i str y | 26.49

(A) Nylon (B) Poly (vinyl chloride)


List I List II
I. Hexamethylenediamine (p) Bakelite
(C) Cellulose (D) Natural rubber

II. Phenol (q) Dacron


Q.3 The correct statement about the following
III. Phthalic acid (r) Glyptal disaccharide is  (2012)
IV. Terephtalic acid (s) Melamine CH2OH
H
(t) Nylon 6, 6 H O H2COH O H
H
(A) I → t, II → p, III → q, IV → r OH H HO
(B) I → t, II → p, III → r, IV → q HO CH2OH
OCH2CH2O
(C) I → s, II → r, III → p, IV → q H OH OH H
(D) I → s, II → r, III → p, IV → q (a) (b)

(A) Ring (a) is pyranose with α-glycosidic link


Previous Years’ Questions (B) Ring (a) is furanose with α-glycosidic link
Q.1 Cellulose upon acetylation with excess acetic (C) Ring (b) is furanose with α-glycosidic link
anhydride/H2SO4 (catalytic) givesAcO
cellulose triacetate
O (D) Ring (b) is pyranose with β-glycosidic link
whose structure is  H O (2008)
AcO H
AcO O AcH
O
H O O
H Q.4 The following carbohydrate is  (2011)
AcO H H O
AcO O AcH HH OAc
O O H O AcH
(A) H O H OH
H H O
H
H OAc H2C
AcO O AcH H OAc
O H O AcH O
O H
(A) H H OAc O HO
H H OAc
O AcH HO
O H AcO OH
H OAc O OH
H O
AcO
AcO
H
OH H (A) A ketohexose (B) An aldohexose
O O O H
AcO
H
H H O (C) An α-furanose (D) An α-pyranose
AcO OH H HH OH
O H OH H
H O O O H
(B) H
H
AcO OH H HH OH H OH Q.5 The correct statement (s) about the following
OH H
O
H
O H
O H sugars X and Y is (are):  (2011)
(B) H OH
H H OH
OH H
O H CH2OH
H OH H O
H HOH2C H
H
AcO AcO AcO
H H H OH H H HO
O O OO CH2OH
H OH
(C) H H H O
AcO O AcH AcO O AcH AcO O AcH H
O O
H O H
O O H OO H OH OH OH
H
(C) H
H OAc HH OAc HH OAc
O AcH O AcH O AcH H X
O O O
(A) X is a reducing sugar and Y is a non-reducing sugar
H OAc H OAc H OAc

AcO
(B) X is a non-reducing sugar and Y is a reducing sugar
AcO AcO
H H H
H
O O OO (C) The glucosidic linkage in X and Y are α and β
H
(D) AcO OAc H
H H AcO OAc H
H
AcO OAc
HH respectively
O O O H O O H OO
H
(D) H
H HH OAc HH OAc (D) The glucosidic linkage in X and Y are β and α,
OAc H
O O
OAc H
O
OAc H H
respectively
H H OAc H OAc
Q.2 Among cellulose, poly (vinyl chloride), nylon and
Q.6 For ‘invert sugar’, the correct statement(s) is (are)
natural rubber, the polymer in which the intermolecular
(Given: specific rotations of (+)-sucrose, (+)-maltose,
force of attraction is weakest is  (2012)
2 6 . 5 0 | Biomolecules and Polymers

L-(-)-glucose and L-(+)-fructose in aqueous solution Q.13 Aspartame, an artificial sweetener, is a peptide
are +66°, + 140°, −52° and +92° , respectively. )(2016) and has the following structure  (2001)
(A) ‘invert sugar’ is prepared by acid catalyzed hydrolysis
CH2C6H5
of maltose |
H2N — CH — CONH — CH — COOCH3
(B) ‘invert sugar’ is an equimolar mixture of D-(+)- |
glucose and D-(-)-fructose CH2 — COOH
(C) Specific rotation of ‘invert sugar’ is −20° (i) Identify the four functional groups.
(D) On reaction with Br2 water, ‘invert sugar’ forms (ii) Write the Zwitter ionic structure.
saccharic acid as one of the products
(iii) Write the structures of the amino acids obtained
from the hydrolysis of aspartame.
Q.7 Match the chemical substances in column I with
type of polymers/type of bonds in column II. Indicate (iv) Which of the two amino acids in more hydrophobic?
your answer by darkening the appropriate bubbles of
the 4 × 4 matrix given in the ORS. (2007) Q.14 Name the heterogenous catalyst used in the
polymerization of ethylene.  (2003)
Column I Column II
(A) Cellulose (p) Natural polymer
Q.15 Statement-I: Glucose gives a reddish-brown
(B) Nylon-6, 6 (q) Synthetic polymer precipitate with Fehling’s solution.  (2007)
(C) Protein (r) Amide linkage Statement-II: Reaction of glucose with Fehling’s
solution gives CuO and gluconic acid.
(D) Sucrose (s) Glycoside linkage

(A) Statement-I is True, statement-II is True;


Q.8 The total number of basic group in the following statement-II is a correct explanation for statement-I
form of lysine is  (2010)
(B) Statement-I is True, statement-II is True;
H3N

CH2 CH2 CH2 H2C O statement-II is NOT a correct explanation for statement-I
(C) Statement-I is True, statement-II is False
CH C

(D) Statement-I is False, statement-II is True
H2N O

Q.9 A decapeptide (Mol. Wt. 796) on complete Q.16 Cellulose upon acetylation with excess acetic
hydrolysis gives glycine (Mol. Wt. 75), alanine and anhydride / H2SO4 (catalytic) gives cellulose triacetate
phenylalanine. Glycine contributes 47.0 % to the total whose structure is  (2008)
weight of the hydrolysed products. The number of AcO
glycine units present in the decapeptide is  (2011) O
H O
AcO H
O AcH
Q.10 Write the structure of alanine at pH = 2 and H
O O H
H
pH = 10.  (2000) (A) AcO
O AcH
H OAc
O O H
H
H H OAc
Q.11 Give the structure of the products in the following O AcH
O H
reaction (2000)
+ H OAc
H
Sucrose →A +B
AcO
O
Q.12 Give the structure of the products in the following H
H
O
AcO
reaction (2000) OH H
O O H
NOH H
(B) AcO H H OH
OH H
O O H
H
H+ Polymerisation H H
C * D * OH H OH
n O H
H OH
Chem i str y | 26.51

O
AcO AcO AcO
H H
O H O H OO H H N
H O O H O
(C) H H H N N N N
O AcH O AcH O AcH H N N N
O O O
N CH2 O H H CH2 O
O
H OAc H OAc H OAc H
O
AcO AcO AcO
H H H
O O OO
H
H H H Q.22 The structure of D-(+)-glucose is  (2015)
(D) H H H H H H H
O O O CHOH
OAc OAc OAc OAc OAc OAc H OH
HO H
Q.17 The substituents R1 and R2 for nine peptides are
H OH
listed in the table given below. How many of these
peptides are positively charged at pH = 7.0?  (2012) H OH
⊕ Θ CH2OH
H3 N− CH − CO − NH − C H − CO − NH − C H − CO − NH − CH − CO O
| | | |
H R1 R2 H The structure of L-(–)-glucose is

Peptide R1 R2 CHO CHO


HO H H OH
I H H
H OH HO H
II H CH3 (A) (B)
HO H H OH
III CH2COOH H
HO H HO H
IV CH2CONH2 (CH2)4NH2 CH2OH CH2OH
V CH2CONH2 CH2CONH2
VI (CH2)4NH2 (CH2)4NH2 CHO CHO
VII CH2COOH CH2CONH2 HO H HO H

VIII CH2OH (CH2)4NH2 HO H HO H


(C) (D)
H OH HO H
IX (CH2)4NH2 CH3
HO H H OH
CH2OH CH2OH
Q.18 When the following aldohexose exists in its
D-configuration, the total number of stereoisomers in
Q.23 Under hydrolytic conditions, the compounds
its pyranose form is  (2012)
used for preparation of linear polymer and for chain
CHO − CH2 − CHOH − CHOH − CH2OH termination, respectively, are (2015)

Q.19 A tetrapeptide has –COOH group on alanine. (A) CH3SiCl3 and Si ( CH3 )
4
This produces glycine (Gly), valine (Val), phenyl alanine (B) ( CH3 ) SiCl2 and ( CH3 ) SiCl
(Phe) and alanine (Ala), on complete hydrolysis. For 2 3

this tetrapeptide, the number of possible sequences (C) ( CH3 ) SiCl2 and CH3SiCl3
2
(primary structures) with −NH2 group attached to a
chiral center is (2013) (D) SiCl4 and ( CH3 ) SiCl
3
Q.24 On complete hydrogenation, natural rubber
Q.20 The total number of lone-pairs of electrons in produces (2016)
melamine is  (2013) (A) Ethylene-propylene copolymer
(B) Vulcanised rubber
Q.21 The total number of distinct naturally occurring
amino acids obtained by complete acidic hydrolysis of (C) Polypropylene
the peptide shown below is  (2014) (D) Polybutylene
2 6 . 5 2 | Biomolecules and Polymers

Q.25 The correct functional group X and the reagent/ (A) X = COOCH3, Y = H2/Ni/heat
reaction conditions Y in the following schemes are
(B) X = CONH2, Y = H2/Ni/heat
 (2011)
(i) Y (C) X = CONH2, Y = Br2/NaOH
X (CH2)4 X Condensation polymer
O O (D) –X = CN, Y = H2/Ni/heat

(ii) C (CH2)4 C
heat
HO OH

PlancEssential Questions
JEE Main/Boards JEE Advanced/Boards
Exercise 1 Exercise 1
Q.5 Q.14 Q.18 (i) Q.1 Q.11 (a) Q.14
Q.23 Q.24 Q.22 Q.25

Exercise 2 Exercise 2
Q.1 Q.11 Q.20 Q.2 Q.8 Q.19
Q.27 Q.33 Q.39 Q.13 Q.15 Q.27
Q.43 Q.38 Q.40
Q.48 Q.51 Previous Years’ Questions
Q.1 Q.5 Q.12
Previous Years’ Questions
Q.13
Q.3

Answer Key

JEE Main/Boards
Exercise 2
Single Correct Choice Type

Q.1 D Q.2 D Q.3 C Q.4 C Q.5 A Q.6 B

Q.7 C Q.8 A Q.9 C Q.10 B Q.11 D Q.12 C

Q.13 D Q.14 A Q.15 C Q.16 A Q.17 A Q.18 B

Q.19B Q.20 B Q.21 A Q.22 D Q.23 C Q.24 A


Chem i str y | 26.53

Q.25 A Q.26 B Q.27 C Q.28 B Q.29 A Q.30 C

Q.31 D Q.32 D Q.33 C Q.34 C Q.35 C Q.36 A

Q.37 A Q.38 B Q.39 C Q.40 D Q.41 D Q.42 C

Q.43 A Q.44 A Q.45 D Q.46 A Q.47 C Q.48 C

Q.49 B Q.50 B Q.51 D Q.52 A Q.53 C

Previous Years’ Questions


Q.1 B Q.2 B Q.3 C Q.4 C Q.5 C Q.6 C

Q.7 A Q.8 B Q.9 A Q.10 C Q.11 A Q.12 A

Q.13 A Q.14 B Q.15 A Q.16 C Q.17 A Q.18 B

JEE Advanced/Boards
Exercise 2
Single Correct Choice Type

Q.1 A Q.2 C Q.3 D Q.4 C Q.5 A Q.6 C

Q.7 D Q.8 A Q.9 D Q.10 B Q.11 A Q.12 A

Q.13 C Q.14 C Q.15 B Q.16 D Q.17 D Q.18 D

Multiple Correct Choice Type

Q.19 B, C Q.20 B, C, D Q.21 B, C, D

Assertion Reasoning Type

Q.22 A Q.23 C Q.24 C Q.25 C Q.26 C Q.27 A

Q.28 D Q.29 B

Comprehension Type

Paragraph 1: Q.30 A Q.31 A Q.32 D

Paragraph 2: Q.33 A Q.34 C

Match the Columns

Q.35 B Q.36 C Q.37 B Q.38 C Q.39 B

Previous Years' Questions


Q.1 A Q.2 B Q.3 A Q.4 B Q.5 B, C Q.6 B,C

Q.7 A → p, s; B → q, r; C → p, r; D → s Q.8 2 Q.9 6 Q.15 C Q.16 A

Q.17 D Q.18 8 Q.19 4 Q.20 6 Q.21 1 Q.22 A

Q.23 B Q.24 A Q.25 C, D


2 6 . 5 4 | Biomolecules and Polymers

Solutions

JEE Main/Boards of protein.


(iii) Denaturation – when the proteins are subjected
Exercise 1 to the action of heat, mineral acids or alkali, the water
soluble form of globular protein changes to water
insoluble fibrous protein resulting in the precipitation or
Sol 1: (i) coagulation of protein, called denaturation of proteins.
CHO

H C OH Sol 4: (i) α ‒ Helix structure: This structure is acquired


when the alkyl groups in amino acids are large and are
OH C H
HI H 3C CH2 CH2 CH2 CH2 CH3 involved in the coiling of the polypeptide chain. This
H C OH is stabilized by the intermolecular hydrogen bond
H C OH n-hexane between the C = O group of one amino acid and
CH2OH
−NH group of the fourth amino acid.
(ii) (ii) β ‒ pleatedstructure: This structure is acquired
CHO CHO when the alkyl group are small in this structure. Linear
Br2H2O polypeptide chains are arranged side by side and
(CH OH)4 (CH OH)4 stabilised by intermolecular hydrogen bond between

CH2OH CH2OH C = O and −NH group.

D-Gluconic Acid Sol 5: The amino acids containing one carboxylic group
and one amino group behave like a neutral molecule.
(iii) This is due to the formation of a zurtter ion structure.
CHO COOH
+
HNO3 NH2 NH3
(CH OH)4 (CH OH)4
R-CH-COOH R-CH-COOH
CH2OH COOH
This zwitter ion changes to cation in acidic solution
Glucaric acid and anion in alkaline medium. making it amphoteric in
nature.

Sol 2: (i) Peptide Linkage: The linkage ( −CO − NH−) is H2N -CH-COO
Alkali
H3N
+
CH-COO-
known as peptide linkage. This linkage is found in the
primary structure of proteins. R R
(ii) Denaturation – when the proteins are sulgected to
the action of heat, mineral acids or alkali, the water Acid +
soluble form of globular protein changes to water H3N CH COOH
insoluble fibrous protein resulting in the precipitation or
R
coagulation of protein, called denaturation of proteins.
Sol 6: Due to denaturation, water soluble form of
Sol 3: (i) Peptide Linkage: The linkage ( −CO − NH−) is globular protein changes to water insoluble fibrous
known as peptide linkage. This linkage is found in the protein, resulting in the precipitation or coagulation of
primary structure of proteins. protein.

(ii) The primary structure of proteins refer to its covalent


Sol 7: (i) A glycosidic linkage is a type of covalent bond
structure, i.e sequence in which various α − amino acids
that joins a carbohydrate molecule to another group
are arranged in protein or in the polypeptide structure
which may or may not be another carbohydrate.
Chem i str y | 26.55

Sol 8: (i)Glucose does not undergo certain reactions (v) Antioxidant activity
of aldehydes, for ex reaction with NaHSO3 schiff’s test
Deficiency causes night blindness.
etc.
Source: Liver, orange, carrots, pumpkin
(ii) Reaction of glucose with NH2OH
Vitamin C
(iii) Mutarotation of α and β glucose
Function: Highly effective antioxidant
(iv) Formation of two isomeric methyl glycosides when
treated with methanol. Lessen oxidative stress
Natural antihistamine
Sol 9: Essential amino acids: Amino acids which cannot
Functioning of immune system
by synthesized by the body and therefore needs to
taken through external diet. For ex. Phenylalanine, Deficiency causes scurvy, bleeding gums
valine Sources: Virus fruits, amla, green leafy vegetable
Non-Essential amino acids: - These amino acids can by
synthesized by the body and therefore. need not be Sol 14: Aldopentose
supplied by an external diet. For ex. Alanine, aspartic
acid. CHO

H * OH
Sol 10: (i) Vitamin A: Chief source orange, ripe yellow
H * OH Asymmetric carbons are 3
fruits, leafy vegetables, carrots, pumpkin Function.
helps in vision, gene transcription, bone metabolism, H * OH
antioxidant, activity.
CH2OH
(ii) Vitamin B: Chief source pork, oatmeal, brown rise,
potatoes, eggs Aldohexose

Function. growth, regulation of apetite Functioning of CHO


heart, muscles and nervous system.
H * OH
(iii) Vitamin D
H * OH
(iv) Vitamin K Asymmetric carbons are 4
H * OH

Sol 11: (i) Refer solution 9. H * OH

(ii) Refer solution 2. CH2OH

Sol 12: Vitamin are classified into two groups. Sol 15: Nucleotides are organic molecules that serves
Depending upon their solubility in water or fat. as the monomers. Or subunits of nucleic acids like DNA
or RNA
(i) Fat soluble Vitamin: Vitamin which are soluble in fats
and oils, but insoluble in water. for ex. Vitamin A,P,E O
For E.g.
and K O
O- P O CH2 Base
(ii) Water soluble vitamins: Vitamins which are soluble
in water. For ex B group vitamins and vitamin C. O-
H H
H H
Vitamin K is responsible for coagulation of blood.

OH OH
Sol 13: Function of vitamin A
(i) Vision (b) (i) As a source of energy (more than 50–80% of
energy in the diet is supplied by carbohydrates)
(ii) Gene transcription
(ii) Protein sparing action: As carbohydrates are mainly
(iii) Immune function
used for energy need of body, proteins are spared for
(iv) Bone metabolism tissue building and repairing.
2 6 . 5 6 | Biomolecules and Polymers

(iii) Essential for fat oxidation


C = O and −NH group.
(iv) Gastro intestinal function.
(ii) (a) Primary Structure: This structure refer to its
Sol 16: α Helix structure is stabilized by the intra covalent structure which various sequence in which
molecular hydrogen bond between the C ==0O group various α − amino acids are arranged in protein or in
the polypeptide structure of protein.
of one amino acid and −NH group of the fourth amino
(b) Secondary Structure: This refers to the arrangement
acid.
of polypeptide chains into a defined three dimension
structure which protein assumes as a result of hydrogen
Sol 17: Fibrous proteins have largely helical structure bonding.
and are rigid molecules of rod like shape. Globular
proteins, on the other hand have a polypeptide chain
Sol 21: The particles in nucleus of the cell, responsible
which consist partly of helical section and partly β
for heredity, are called chromosomes, which are made
pleated structure and Remaining random coil form.
up of proteins, and another type of biomolecules called
nucleic acids.
Sol 18: (i) Amino acids have very strong intermolecular
forces due to highly effective hydrogen bonding Function:
between (i) Nucleic acid serves as chemical basis of heredity
C ==0O and −NH groups. and may be regards regarded as reserve of genetic
information.
O (ii) Protein synthesis in cells.
H O
R C N H
H C N Sol 22: Nucleoside:
H
R (a) Consist of a nitrogenous base covalently attached
to a sugar (ribose or deoxyribose) but without the
(ii) Vitamin A Night blindness Vitamin B1 Beri Beri phosphate group.
Vitamin B6 Convulsions
Nucleoside = sugar + base
Vitamin K increasing blood clotting time
(b) Used as antiviral or anticancer agents

Sol 19: In DNA, two nucleic acid chains are wound about (c) E.g. Cytidine, uridine, adenosine
each other held together by hydrogen bonds between Nucleotide:
pair of bases. The two strands are complementary to
(a) Consists of a nitrogenous base, a sugar (ribose or
each other because the hydrogen bonds are formed
deoxyribose ) and one to three phosphate groups.
between specific pair of bases. Adenine forms hydrogen
bonds with thymine whereas cytosine forms hydrogen (b) Nucleotide sugar + base + phosphate
bonds with guanine.
(c) Malfunctioning nucleotides are one of the main
causes of cancer.
Sol 20: (i) α − Helix
E.g. S-uridine monophosphate
(a) Alkyl groups in amino Acids are large.
(b)Polypeptide chains are coiled leading to right Sol 23: Reducing sugar: sugar that contain aldehyde
handed helical coil group that are oxidized to carboxylic acids are classified
as reducing sugar.
(c)Stablised by intermolecular H – bonding between
groups on one amino acid and –NH group of 4 amino For e.g. Sucrose act as reducing agents.
acid.
Non reducing sugar: they cannot act as a reducing
β − Helix group due to absence of an aldehydic group.
(a) Alkyl groups in amino acids are small Reducing sugars must either contain aldehyde group
or in is capable of forming one in solution through
(b) Polypeptide chains are arranged side by side
isomerism.
(c) Stabilized by intermolecular H-Bonding between the
Chem i str y | 26.57

Sol 24: α glucose Sol 29: Refer theory.


(a) CH2OH
O Sol 30: (a) Refer text Pg. 2
H H H
(b) A nucleotide is an organic molecule made up of
nucleotide base, a five carbon sugar and at least one
HO OH H OH
phosphate group.

H OH The two classes are

(b) −OH group lies below the ring on carbon (i) Purine: contains adenine and guanine

(c) α glucose folds up into a helix. (ii) Pyrimidine: contains cytosine, thymine and uracil.

β glucose
Exercise 2
(a) CH2OH
Single Correct Choice Type
O
H H OH Sol 1: (D) Chromophore is the colouring agent, which
is diazo group (–N = N -)
OH OH H H
Sol 2: (D) Zwitter ion is present
H OH
R CH COO-
(b) −OH group lies above the ring on carbon
(c) β glucose folds up into a pleated sheet. *NH3

Sol 25: Refer solution 1. Sol 3: (C) This is secondary structure of a protein
Same reaction as D-Glucose, only structural difference is
in the configuration of atoms around different carbons. Sol 4: (C) General formula of carbohydrates:

Cn (H2O)n or Cx (H2O)y
Sol 26: Refer text pg. 15
Sol 5: (A) Rest are all monosaccharide except sucrose.
Sol 27: Different types:
(i) mRVA Sol 6: (B) Iron is bonded by co-ordination ring
formation (chelation)
(ii) fRNA
(iii) rRNA
Sol 7: (C) Anomers have different specific relation.

Sol 28: (i) This pH at which the structure of amino acid


Sol 8: (A) Amide linkage ( −CO − NH−) is called peptide
has no net charge is called it’s isoelectronic point for ex.
bond
Aspartic acid hs isoelectric point of 2.77
(ii) Mutarotation is the change in the optical rotation Sol 9: (C) Glucose is a hydroxyl aldehyde whereas
that occurs by epimerization (change in equilibrium Fructose is a ketone.
between two epimers, when the corresponding
stereocentres interconvert) for ex. β − D glucose .
Sol 10: (B) Glucose is a hydroxy aldehyde with 6 carbon
With specific rotation of +18.7°, when dissolved in or aldehexose.
water undergo mutarotation and attains a final specific
rotation of 52.50
Sol 11: (D) Phenol acts both as an antiseptic and
(iii) An enzyme is a substance produced by a living disinfectant.
organism which acts as a catalyst to bring about a
specific biochemical reaction for ex. Streptokinase
2 6 . 5 8 | Biomolecules and Polymers

Sol 12: (C) Fenton’s reagent (H2O2 + Fe) converts


Sol 28: (B) Cellulose is a linear polymer of β glucose.
D -glucose (6 carbon) to D arabinose (5 carbon)
Sol 29: (A)
Sol 13: (D) At isoelectronic point.
H H
pk a1 + pk a2 C=O
pH = C = N NH C6H5
2 H C OH H C OH

Sol 14: (A) Zeigler and Natta developed zeigler Natta HO C H C6H5NH NH2 HO C H

Catalysts (Al(C2H5 )3 + TiCl3 ) which are used for H C OH H C OH

coordination polymerization. H C OH H C OH
CH2OH C
Sol 15: (C) Teflon is homopolymer of tetraflourethylene. CH2OH
D-Glucose
Polystyrene is a homopolymer of styrene. Neoprene is a
homopolymer of chloroprene. C6H5NH NH2
H
C = N NH C6H5
Sol 16: (A) Anomer have different configurations at H
Glycosidic carbon and hence different specific rotation. C = N NH C6H5 C = N NH C6H5
HO C H C=O
Sol 17: (A) They are called epimers. C6H5NH NH2
H C OH HO C OH

Sol 18: (B) Natural rubber is polymer of isoprene H C OH H C OH

CH2OH H C OH


H3C
C-C
CH2  3 molecules of phenylhydrazine is used.
CH2OH

H2C H

Sol 30: (C) Main structural unit is peptide linkage


Sol 19: (B) It is called inversion as sucrose, which is
( −CO − NH−)
dextrorotatory gives a laevorotatory mixture on dilution.
Sol 31: (D) All these are function of the proteins
Sol 20: (B) Sulphur cross links are present in vulcanised
rubber.
Sol 32: (D) Glucose and galactose are monosaccharide
while sucrose is disaccharide.
Sol 21: (A) Glycine (H 2 N − CH 2 − COOH) is the
simplest amino acid.
Sol 33: (C) Starch on reaction with iodine gives blue
colour which serves as a test for presence of I2
Sol 22: (D) All of these are natural polymers.
Sol 34: (C) Ribose has five carbon atoms, rest all
Sol 23: (C) They differ in configuration of Cl options are correct.
Ribose
Sol 24: (A) Aldehydic group gives silver mirror test with
tollen’s reagent. H C O
H C OH
Sol 25: (A) This is definition of oligosaccharides.
H C OH

Sol 26: (B) This is definition of epimer. H C OH

CH2OH
Sol 27: (C) Energy is evolved with ATP is hydrolysed to
ADP and AMP. Rest are correct.
Chem i str y | 26.59

Sol 35: (C) D – xylose is a diastereomer of ribose with Sol 43: (A) Teflon is a polymer of F2C = CF2
formula.
CHO F F
nCF2 = CF2 → ( C C (n
H OH
F F
OH H
H OH

CH2OH Cl Cl
Sol 44: (A) H3C CH2 ( H2C CH (n
Sol 36: (A) Glucose gives the silver mirror test with PVC
ammoniacal solution of silver nitrate because it contains
the group Aldehyde. Sol 45: (D) Polyacrylonitrile is a polymer of acrylonitrile
(CH2 = CHCN)
Sol 37: (A) Dacron is a copolymer of ethylene glycol
and terephthalic acid. Sol 46: (A) This is the classification based on inter
molecular forces.
PVC is a polymer of : Vinyl chloride
Polystyrene is a polymer of styrene Sol 47: (C) Polyethene involves linear linkage between
Teflon is a polymer of tetraflouroethylene chains (CH2 – CH2)n

Sol 38: (B) Melamine resin: Sol 48: (C) Al (C2H5)3 + TiCl4 is Zieglar – Nata catalyst

HN Sol 49: (B) Neoprene is a polymer of chloroprene

N N CH2 =C CH=CH2 CH2 CH=C CH2

Cl Cl
NH N NH n
Sol 50: (B)Teflon is a polymer of CH2 = CF2.
F F
NH2 Sol 51: (D) Polytetraflouroethylene or Teflon ( C C (n
F F
N N

+ 3HCHO is used because of its being a thermoplastic polymer.

N NH2
H2N Sol 52: (A)
nHOOC ( CH2 )4 COOH + nH2N ( CH2 ) 6 NH2 →
F F
Sol 39: (C) Teflon is a polymer of C C O O
(CH2)6
F F
(CH2)4 N N
Sol 40: (D) Acrylonitrile n H2C = C – C ≡ N →
H H n
(HC = C – C = N )n

Sol 41: (D) Glyptal is a condensation polymer. Sol 53: (C) They are made from a single compound i.e.
they contain a single repeating unit.
Sol 42: (C) Highly vulcanised rubber is called ebonite.
2 6 . 6 0 | Biomolecules and Polymers

Previous Years’ Questions Acrylonitrile, Neoprene and Teflon are addition


polymers of acrylonitrile, isoprene and tetrafluoro
Sol 1: (B) Both glucose and fructose are reducing ethylene respectively.
sugars, reduces Tollen’s reagent to metallic silver.
Sol 13: (A) DNA contains ATGC bases
Sol 2: (B) “ α ” and “ β ” cyclic hemiacetals of D-glucose A – Adenine
having difference in configuration at C-1 only are called
anomers. T – Thymine
G – Guanine
Sol 3: (C) Statement I is Correct: Presence of -CHO
C – Cytocine
group in glucose is tested by Fehling solution test
where a reddish-brown precipitate of Cu2O is formed. So quinoline is not present.
Hence, statement II is incorrect.
Sol 14: (B) Glyptal is used in the manufacture of paints
Sol 4: (C) α − D ( + ) glucose and β − D ( + ) glucose are and lacquers.
anomers.
Sol 15: (A) Vitamin B and C are water soluble and
Sol 5: (C) Carbohydrates are polyhydroxy carbonyl Vitamin A, D, E and K are water insoluble.
compounds.
Sol 16: (C) Low density polythene is not used in the
Sol 6: (C) Buna-N synthetic rubber is a copolymer of manufacturing of buckets, dust-bins etc. because buckets,
acrylonitrile (ACN) and butadiene. dustbins are manufactured by high density polythene.

Sol 7: (A) It is a test characteristic of amide linkage. Sol 17: (A)


Urea also has amide linkage like proteins.
Sodium lauryl sulphate = detergent, anionic
Cetyltrimethyl ammonium bromide = detergent,
Sol 8: (B) Nylon 6,6 is a polymer of adipic acid and
cationic
hexamethylene diamine
Glyceryl oleate = detergent, non-ionic
O O
Sodium stearate = soap, anionic
 C (CH2)4 C NH (CH2)6 NH 
n
Sol 18: (B) NH2
Sol 9: (A) In RNA, the sugar is β − D -Ribose, where as
in DNA the Sugar is β − D − 2 - deoxy Ribose. Cystine HO S COOH
C S
O NH2
NH2
Sol 10: (C) Glycine CH2 Cysteine COOH
COOH HS
NH2
Sol 11: (A) Fact
Thiol group (SH) is present in cysteine.
6CO2+12NADPH+18ATP → C6H12O6+12NADP+18ADP O
||
Methionine CH3 − S − CH2 − CH2 − C H − C − OH
Sol 12: (A) Dacron is polyester formed by condensation |
polymerisation of terephthalic acid and ethylene glycol NH
2
Cytosine NH2
HOOC COOH +HO CH2 CH2 OH
N

CO CH2 CH2 O O N
n H
Dacron
Chem i str y | 26.61

JEE Advanced/Boards Sol 5: (i) Proteins are polymer of α‒amino acids and
they are connected to each other by peptide bond or
peptide linkage.
Exercise 1
(ii) A polypeptide is a single linear chain of amino acids
Sol 1: Streptokinase is used as a medicine for blood whereas an oligopeptide is a polypeptide less then
clots. 30-50 amino acids long.
(iii) Refer Exercise I, Q.6
Sol 2: Cellulose in our diet is not nourishing as because,
it is a complex from of carbohydrate, and no mammal
makes the necessary enzyme to break down cellulose. Sol 6: A vitamin is an organic compound required by an
organism as a vital nutrients in limited amounts.
Sol 3: Mutarotation is the change in the optical (a) Two types Fat Soluble Vitamins: Vitamins which
rotation that occurs by epimerization (that is the are soluble in fat and oil, but insoluble in water for
change in he equilibrium between two epimers, when E.g. Vitamin A,D,E And K
the corresponding stereo centers interconvert epimers (b) Water Soluble Vitamin: Vitamins which are soluble
of D-glucose in water, for Ex. B group vitamins and vitamins C
CH2OH
(b) Refer theory part.
O
H H H
Sol 7: (a) (i) A coenzyme is a substance that works with
HO OH H OH an enzyme to initiate or aid the function of enzyme.
They can not function on their own and require the
H OH presence of an enzyme.
α ‒ D ‒ (+) ‒ Glycopyranose
Sol 8: (a) (i) Oranges, carrots, pumpkins
θ = +112°C
CH2OH Sol 9: (i) Cellular Respiration: Like other organisms,
O plants store corbohydrates and burn them for energy.
H H OH
(ii) Mechanical Strength: Certain carbohydrats, like
OH OH H
cellulose helps plants in enhancing mechanical strength.
H

H OH Sol 10: (i)

CHO COOH
β ‒ D ‒ (+) ‒ Glycopyranose alk.MnO4-
(i) (CHOH)4 (CHOH)4
θ = 19°C
CH2OH COOH
When either of these forms of D-glucose is dissolved Saccharic acid
in water and allowed to stand, a gradual change in
specific rotation occurs the specific rotation of the α
form falls and that of the β form rises until a constant CHO COOH
values of 53° is obtained. (CHOH)4
Br2+CS2
(CHOH)4
(ii)
CH2OH COOH
Sol 4: For differences, refer text. Gluconic acid
Sugar: 2 ‒ deoxy D(‒)ribose

O CHO
HO CH2 OH H2+SO4
(iii) (CHOH)4 6C+6H2O
H CH2OH
H H

OH H
2 6 . 6 2 | Biomolecules and Polymers

Sol 11: (i) Adenine (ii) Thymine (iii) Cystosine C=O group of one amino acid and ‒HN group of fourth
amino acid stabilises α‒helix structure.
(iv) Guanine
In 3.613‒Helix, 3.6 is the number of residues per turn
Thymine is not present in RNA.
and 13 is the number of atoms in the hydrogen bonded
loop.
Sol 12: Two fat soluble vitamin are vitamin A and D.
Vitamin A: Deficiency disease night blindness.
Sol 19: Complementary bases are specific pairs that
Sources: (i) Carrots (ii) Liver (iii) Pumpkin (iv) Orange join up the two strands of double stranded DNA Via
hydrogen bonds.
Vitamin D: Deficiency disease: Rickets sources. H
(i) Sunlight ( chief source ) N O H N

(ii) Some mushrooms


N N-H N
R
Sol 13: Refer theory. N N
N-H O
R
Sol 14: Refer Exercise I, two (ii) H
Guanine Cytosine

Sol 15: (a) Functional group H


N N-H
(i) Carboxylic acid (ii) Amine (GC)
O (A =T)
Amide Ester N N
NH3+ CH2C6C5 R
(b) N H-N N
-OOC-CH -CH-CONH-CH-COO CH R
2 3
O
Adenine Thymine
(c) (i) NH2

HOOC - CH 2CH - COOH Sol 20: (i) In acidic medium,

(ii) CH2 C6 H5 R-CH-NH3+ +


H R-CH-NH+
3

H2 N - CH - COOH COO - COOH

Since this is a positively charged ion (cation), they


(d) (ii) is more hydrophobic as (i) is more polar due to migrate towards the cathode.
presence of 2 COOH groups,
In basic medium,
Helping, (i) to make hydrogen bonds with water,
increasing its hydrophilicity R-CH-NH3+ OH
+
R-CH-NH2
COO COO -
Sol 16: α ‒ (5, 6 ‒dimenthylbenzimidazolyl)
Since this is an anion (negatively charged), they migrate
Cobamidcyanide more commonly called cobalmin. towards the anode.
(ii)Monoamino carboxylic acids exits as zwitter ions
Sol 17: (a) A mixture of equal parts of glucose and and exhibit two different k a values and hence two pk a
fructose resulting from the hydrolysis of sucrose is values.
called invert sugar.
(b) Polypeptide is a linear organic polymer consisting R-CH-COOH PKa1 R-CH-COO- Pka2 R-COO-
of a large number of amino acid residues bonded NH+
3 NH+
3 NH2
together in a chain, forming part of (or the whole of) a
protein molecule.
Sol 21: Glycine (H2N ‒ CH2 ‒ COOH) exists as zwitter
ion (H3N ‒ CH2 ‒ COO‒) as it contains both acid and
Sol 18: Intramolecular Hydrogen bonding is between
Chem i str y | 26.63

amino group on the same α ‒ Carbon (unsymmetrical dimethyl hydrazine )


COOH Is used as bi liquid propellant in rocket fuels.
NH2
Anthranilic acid
 
Cannot exist as zwitter ion, because
Sol 6: (C) In ‘placebo’, a group of patients is given an
actual medicine, while the other group is given an
ordinary sugar pill. The ordinary pill is called placebo.
COO-
+
NH3 Sol 7: (D)
The resulting zwitter ion
  CHO

(CHOH)2
Br2, H2O
COOH

(CHOH)2
is highly destabilised by the strong +I And no m-effect
of NH3+ group. CH2OH CH2OH
Glucose
Sol 22: Refer theory Part.
Fructose does not undergo any reaction.

Sol 23: Refer theory Part.


Sol 8: (A)
CHO
Sol 24: Refer theory Part.
HIO4
(CHOH)4 5HCOOH + H2C=O

Exercise 2 CH2OH

Biomolecules Sol 9: (D) This is correct.

Single Correct Choice Type Sol 10: (B) Amino acids give minhydrin test as they are
primary amines.
Sol 1: (A) In complementary strand, complementary NH2
With HNO2, |
bases will be present, that is according to pairing ( A =
HNO2
T ) and ( C = G ). Now R CHCOOH  → R CH2COOH+N2
DNA strand.
Sol 11: (A)
OH OH
A T G A C T G T C CH2OH
T A C T G A C A G (A) + HCHO

Complementary strand
This is an electrophilic substitution.
Sol 2: (C) Saturated acids have high melting point due
to low bond polarity ΣN i M :2
Sol 12: (A) Average molecular mass =
ΣN i M :
Sol 3: (D) Biodegradable detergent should contain a
cyclohexyl side chain. Sol 13: (C) Polydispersity index
ΣNi Mi2
Sol 4: (C) The gases ejected by the rocket exert an Weight Average Molecular Mass ΣNi Mi
= =
equal and opposite thrust on the rocket. Number Average Molecular Mass ΣNi Mi
ΣNi
ΣNi × ΣNi Mi2
Sol 5: (A) =
( ΣNi Mi )
2
H CH3
N2O4+ N=N
CH3 H
2 6 . 6 4 | Biomolecules and Polymers

Sol 14: (C) Isobutane CH3 − CH2 − CH3 Due to very high acidity of ‒SO3H group and stabilisation
| of resulting dipolar ion due to charge distribution. In
CH3 p – aminobenzoic acid
COOH
+
We see then its cation CH3 − CH − CH3 is very stable,
|
CH3

as it is tertiary, therefore the best way to polymer it is


  NH2
by cationic polymerization.
the resulting dipolar ion
O COO-
Sol 15: (B) Nylon-6 : N – (CH2 )6 – C
H n   NH2
Sol 16: (D) Acrilan CH2 – CH
Does not exist as relatively low acidity of ‒COOH group
CN n and less stablisation of dipolar ion, as negative charge
is distributed between only 20 atoms.
Sol 17: (D) They are all correct.
SO3H
Sol 18: (D) Commercial name for

polymethylmethacrylate, is
1
O O
(c) Sulphanilic acid
  NH2

plexiglass (orperpox)
  n Exists as dipolar ion, SO-3

NH+3
Multiple Correct Choice Type
Which is an insoluble salt.
Sol 19: (B, C) Self explanatory.
Now in base,
Sol 20: (B, C, D) SO-3 SO-3
(a) At isoelectric point, amino acid solution is neutral.
OH-
(b) φ-amino benzesulphonic acid
SO3H
NH+3 NH2

  NH2
Reacts with base as OH − is more basic than
group.

NH 2

exists as dipolar ion In acid,


SO-3
SO-3

  H-
No Reaction

NH+3
NH+3
Chem i str y | 26.65

No reaction, as H+ is less acidic than ‒SO3H group. Trans-Polyisoprene can be formed but is generally not
found naturally.
(d) Since pKa of H3N+CH2COOH is less than pKa of
RCH2COOH, it is more acidic
Sol 27: (A) PMMA (Poly (Methyl methacrylate)
Sol 21: (B, C, D) Polyethylene contains single bonds has excellent light transmission properties as it is
only. transparent and thus used for making lanses end light
covers.
Polyethylene : (CH2 – CH2)n
F F Sol 28: (D) Vinyl alcohol, H2C = CH – OH is unstable
Teflon : ( C C )n O
||
F F and converts to H3C − CH . Therefore, poly vinyl alcohol
(C) and (D) are correct is prepared by hydrolysis on polyvinyl acetate.

CH2OH CH2OH
O O
H H
Assertion Reasoning Type MeOH/HCl
H H H
D-Glucose +
OH H OH H
Sol 22: (A) Strach Maltose HO OMe HO
Amylase

H OH H O
Starch contains glycosidic linkage which are broken
down by amylase. Methyl--D (+) Methyl--D
glucopyranoside glucopyran

Sol 23: (C) On hydrolysis, sucrose (θ = 66.47°) gives


CH2OH CH2OH
equal amount of glucose (θ = 52.5°) and fructose O O
(‒92.4°). Thus total specific rotation of mixture is H H H H H OMe
D-Glucose MeOH/HCl
‒39.9° which makes it laevorotatory, whereas the initial +
OH H OH H
solution was dextrorotatory. HO OMe HO H
Sucrose → Glucose + Fructose OH OH
H H
θ = 66.470 θ = 52.50 θ = −92.40
Methyl--D (+) Methyl--D (+)
glucopyranoside glucopyranoside
Sol 24: (C) A is correct
Helix is stabilised via intramolecular
R is wrong as α − Helix Sol29: (B) It is a polyamide
hydrogen bonding between -CO of one amino acid
H
and -NH group of fourth amino acid O
N O

+ N (CH2)6 C
Sol 25: (C)
H
n
Carbonation form styrene Caprolactum
Carbocation form styrene Nylon-6

+
propylene CH3 -HC-CH3 (ii)
Carbocation from prokylene Comprehension Type

(i) is more than (ii) as it is resonance stabilised through Paragraph 1:


conjugation with benzene ring.
Sol 30: (A) Elastomers have weakest forces of attraction
e.g. rubber.
Sol 26: (C) Natural rubber is all cis-polyisoprene.

CH3 H2C Sol 31: (A) Thermoplastics usually have a linear


CH2= C HC = C structure which allows them to be repeatedly softened
H2C - H2O H2C (or hardened) by an increase (decrease) in temperature.
n
2 6 . 6 6 | Biomolecules and Polymers

Sol 32: (D) Thermosetting polymers are hard due to Previous Years’ Questions
three dimensional network of bonds.
Sol 1: (A) Cellulose is biopolymer of
Paragraph 2:
β -D –glucopyranose as:
Sol 33: (A) Rubber on reaction with sulphur forms
CH2OH CH2OH CH2OH
vulcanized rubber which is shift and resistant to action
O O O
of common solvents and wear and tear. H
H O H
H
O H
H
O

OH H H OH H OH H H
H
Sol 34: (C) This process is called vulcanisation. O
H OH H OH H OH
Match the Columns cellusose
(CH3CO)2O
+
Sol 35: (B) Nucleic acids are polynucheotides uracil is H

found only in RNA. CH2OAc CH2OAc CH2OAc


Thymine is found only in DNA, DNA has double helix H
O O
H O
O H O O
structure. H H H
OAc H H OAc H H OAc H H
O
Sol 36: (C) Pepsin is a digestive enzyme Nucleic acid
H OH H OAc H OAc
contains genetic material Ascorbic acid is chemical
name for vitamin C. Testosterone is a sex hormone. Tri-acetylated cellulose

Sol 37: (B) Bakelite is copolymer of phenol and Sol 2: (B) Cellulose and nylons have H-bonding type
methanol dacron is copolymer of 1, 2-dihydroxyethane of intermolecular attraction while poly (vinyl chloride)
and dimethylterephthalate. is polar. Natural rubber is hydrocarbon and has the
weakest intermolecular force of attraction, ie, van der
Nylon-66 is a copolymer of 1, 6-hexanedioic acid and 1, Waals’ force of attraction.
6-diaminol hexane.
(o) Buna-S is copolymer of butadiene and styrene. Sol 3: (A) The six-membered cyclic ether is known
as pyranose while the five membered cyclic ether is
Sol 38: (C) known as furanose. Hence, ring (a) is a pyranose and
it has ether linkage at α-Position that is known as
Polymer Monomer α- glycosidic linkage in carbohydrate chemistry.
Bakelite Phenol and formaldehyde
Terylene Terephthalic acid and ethylene glycol Sol 4: (B) Here, the –OH of hemiacetal group is equatorial
Nylon-6 Caprolactam therefore, it is a β -pyranose of an aldohexose.
Synthetic rubber Butadiene and styrene
Sol 5: (B, C) X is acetal, has no free hemiacetal, hence
Sol 39: (B) a non-reducing sugar while Y has a free hemiacetal
Polymer Monomer group, it is reducing sugar. Also, glucosidic linkage of X
is ‘ α ’ while that of Y is β -linkage.
Nylon-66 Hexamethylene diamine + adipic acid
Bakelite Phenol and formaldehyde Sol 6: (B, C)
Glyptal Phthalic acid and ethylene glycol (A) False (B) Factual
Dacron Terephthalic acid and ethylene glycol. H
(C) C12H22O11 + H2O →
+
C6H12O6 + C6H12O6
Invertase
Sucrose D-Glucose D-Fructose

Net specific Rotation of an equimolar mixture of


52 − 92 −40
Invert = = = −20
2 2
Chem i str y | 26.67

Sol 7: A → p, s; B → q, r; C → p, r; D → s = 796 + 18 × 9= 958


(A) Cellulose 958 × 47
⇒ Mass of glycine in hydrolysis product
= = 450
100
O O O ⇒ Number of glycine molecule in one molecule of
O O 450
decapeptide
= = 6
75

(Glycoside linkage) Sol 10:

pH = 10

CHO CH2OH H2N CH COO H2N CH COOH
H OH C O
CH3 CH3
HO H HO H alanine

H OH H OH
pH=2 +
H2N CH COOH H3N CH COOH
H OH H OH
CH2OH CH3 CH3
CH2OH
Alanine
(B) Nylon 6, 6
H+
Sol 11: Sucrose  → D-glucose + D –fructose
H O
HO 2
N C
C N
Sol 12:
O
(Amide linkage)

(C) Protein
H O
R H

C N C
N C N
Sol 13:
R’ H
H O H O CH2C6H5
(Amide linkage) H2N CH C NH CH COOCH3
(D) Sucrose CH2 COOH
CH2OH
aspartame
O
H H H
O CH2OH
(i) Aspartame has amine, acid, amide and ester groups.
HO OH H O HO H O CH2C6H5
H +
H OH H
(ii) H3N CH C NH CH COOCH3
OH

(Glycoside linkage)
CH2 COO-

Sol 8: –COO- and –NH2 are basic groups in lysine. (iii) CH2C6H5
+
H
Aspartame H2N CH COOH + H2N CH COOH + CH3OH
H2O
Sol 9: A decapeptide has nine peptide (amide) linkage as - II
CH2COOH
Therefore, on hydrolysis, it will absorb nine water I
molecules.
(iv) II is more hydrophobic due to presence of phenyl
Hence, total mass of hydrolysis product group.
2 6 . 6 8 | Biomolecules and Polymers

Sol 14: Zeigler-Natta catalyst, which is a mixture of Phenyl alanine (P)


triethylaluminium “(C2 H5) 3Al” and TiCl4, is used as
Alanine (A)
heterogeneous catalyst in polymerization of ethylene.
So the number of possible sequence are 4.
Sol 15: (C)
Sol 20: lone pairs
C6H12O6 + Fehling solution → ( C6H11O7 ) + Cu2O ↓

(Red ppt. )

:
:
NH2 N NH2

Sol 16: (A) As in cellulose β 1 − 4 glycosidic linkage is


N N
present.

:
Sol 17: (D) Peptides with isoelectric point (pI) > 7, NH2

:
would exist as cation in neutral solution (pH = 7). Melamine
IV, VI, VIII and IX Sol 21: This peptide on complete hydrolysis produced
4 distinct amino acids which are given below:
Sol 18: O O
(1) H2N CH2 C OH (2) HO C NH2
6
Fixed in D-configuration Glycine
1 CHO CH2 OH
(natural)
H
2
H
5* O CH2
3 H H H
H OH 4 * * 1
4
H OH HO H H OH
5 *
H OH 3 2 O
6 OH H O
OH O
H2C C OH
(3) HO C (4) HO C NH
Hence total number of stereoisomers in pyranose form NH2
3
= 2=
of D-configuration 8
CH2

Sol 19: Because –COOH group of tetrapeptide is


intact on alanine, its NH2 must be participating in
Only glycine is naturally occurring amino acid.
condensation.
∴ Alanine is at one terminus, – – –A.
Sol 22: (A)
To fill the 3 blanks, possible options are:
CHO CHO
(i) When NH2 group attached to non chiral carbon
H OH HO H
G V P
OH H H OH
G P V
H OH HO H
(ii) When NH2 group attached to chiral carbon
H OH HO H
V G P P V G
CH2OH CH2OH
V P G P G V
D(+)glucose L(-)glucose
where, Glycine (G)
Valine (V)
Chem i str y | 26.69

Sol 23: (B)

 
CH3 CH3 CH3 CH3
H2O
Cl Si Cl HO Si OH H O Si O Si O H
CH3 CH3 CH3 CH3 n

Me3SiCl, H2O

 
Me CH3 CH3
Me
Me Si O Si O Si O Si Me
Me Me
CH3 CH3 n

Sol 24: (A)


n CH2 = C CH = CH2 ( CH2 CH = CH2)n

CH3 CH3
Natural Rubber

Sol 25: (C, D)

(a) When X = COOCH3


H2 /Ni HOOC—(CH2)4 —COOH
CH3OOC—(CH2)4 —COOCH3 HOCH2 —(CH2)4—CH2OH+2CH3OH
heat heat

O O

— O—(CH2)6—O—C—(CH2)4—C —
n
Ester, condensation polymer

(b) When X = CONH2

H2 /Ni HOOC—(CH2)4 —COOH


H2NOC — (CH2)4 — CONH2 H2N — (CH2)6 — NH2
heat heat

O O

—HN—(CH2)6—NH—C—(CH2)4—C —
Nylon, condensation polymer n

Br2 HOOC—(CH2)4 —COOH


(c) H2NOC — (CH2)4 — CONH2 H2N —(CH2)4 — NH2
NaOH heat
Hofmann’s bromamide reaction

O O

—HN—(CH2)4—NH—C—(CH2)4—C —
Nylon, condensation polymer n

(d) When X = CONH2

H2 /Ni HOOC—(CH2)4 —COOH


NC—(CH2)4 —CN H2N —(CH2)6 — NH2
heat heat

O O

—HN—(CH2)6—NH—C—(CH2)4—C —
Nylon, condensation polymer n
2017-18 100 &
op kers
Class 12 T
By E ran culty
-JE Fa r
IIT enior emie .
S fP r es
o titut
Ins

CHEMISTRY
FOR JEE MAIN & ADVANCED
SECOND
EDITION

Exhaustive Theory
(Now Revised)

Formula Sheet
9000+ Problems
based on latest JEE pattern

2500 + 1000 (New) Problems


of previous 35 years of
AIEEE (JEE Main) and IIT-JEE (JEE Adv)

5000+Illustrations and Solved Examples


Detailed Solutions
of all problems available

Plancess Concepts
Topic Covered Tips & Tricks, Facts, Notes, Misconceptions,
Key Take Aways, Problem Solving Tactics
Prac cal Organic Chemistry
PlancEssential
Questions recommended for revision
27. PRACTICAL ORGANIC
CHEMISTRY

1. PURIFICATION

1.1 Introduction
Organic chemistry is based on carbon compounds either natural or synthetic. Such organic compounds are in a
crude form. In order to observe the properties of them, a pure compound is necessary. Purity of the compound
is required for their reactions. There are various methods of purification of organic compounds based on their
physical and chemical properties such as adsorption, boiling point, sublime nature, etc.
Few of them are as follows:
(a) Fractional Crystallization (b) Sublimation (c) Chromatography
(d) Distillation (e) Differential Extraction
Hot saturated
(a) Fractional crystallization: Fractional crystallization is based on the solution
solubility of a compound in a specific solvent. It is used when an
organic compound is sparingly soluble in some solvent at room
temperature but considerably soluble at higher temperature and Hot water
impurity is highly soluble. Its solution can be heated dissolving the jacket
compound along with impurities and cooling it to form crystals of
highly pure organic compound leaving impurities behind in the
solution called mother liquor.

Examples:
 1. p-Xylene
Figure 27.1: Hot Water funnel
 2. Benzoic acid- It is sparingly soluble in cold water but highly soluble
in hot water.
 3. Fructose

(b) Sublimation: It is the property of a substance to change its state from a solid state to a vapour state without
passing through a liquid state. Such compounds can be heated and separated from the impurity by separating
its vapour by using the inverted funnel method.
2 7 . 2 | Practical Organic Chemistr y

Cotton
Inverted
funnel
Sublimate Perforated
asbestos sheet

Mixture

Figure 27.2: Sublimation

Example: Camphor, benzoic acid, anthracene.

Table 27.1: Melting Points

Substance M.P. Substance M.P.


1,4-Dichlorobenzene 55 Benzoic acid 122
Naphthalene 82 Salicylic acid 159
1-Naphthol 96 Camphor 177
Acetanilide 114 Caffeine 235

(c) Chromatography: Chromatography is used for the separation of most of the compounds. There are two phases
used in this technique namely stationary phase and mobile phase. Stationary phase is immobile throughout
the process while mobile phase or eluent is a moving phase. Chromatography is based on its solubility in
mobile phase as well as adsorption of the compound in stationary phase.

Terms involved in chromatography:


(i) Rf factor: Retardation factor is used in identifying the product separated using paper or thin layer
chromatography. R f factor

Migration distance of substance


is given as: R f =
Migration distance of solvent front

Solvent front

Distance moved by solute


Rf =
Distance moved by solvent
x
x

Starting point
Figure 27.3: Chromatogram

In paper chromatography, if compounds are colourless, then ninhydrin spray is used to visualize the
compounds in the form of various colours.
Chem i str y | 27.3

(ii) Stationary Phase: The phase in chromatography which is stationary with respect to the components
of a mixture is called stationary phase. In case of column chromatography, stationary phase adsorbs
components at different heights.

(iii) Mobile Phase or Eluent: The phase which moves along with components of a mixture is called a mobile
phase or an eluent. Mobile phase determines R f factor which is dependent on the solubility of the
components in a given solvent or eluent.

The following are the different types of chromatography:


•• Column chromatography: Column chromatography is a separation technique in which the stationary bed
is within a tube. The particles of the solid stationary phase or the support coated with a liquid stationary
phase may fill the whole inside volume of the tube (packed column) or be concentrated on or along the
inside tube wall leaving an open, unrestricted path for the mobile phase in the middle part of the tube
(open tubular column). Differences in rates of movement through the medium are calculated to different
retention times of the sample. Silica gel is a commonly used stationary phase.
•• Paper chromatography: Paper chromatography is a technique that involves placing a small dot or
line of sample solution onto a strip of chromatography paper. The paper is placed in a jar containing
a shallow layer of solvent and sealed. As the solvent rises through the paper, it meets the sample
mixture, which starts to travel up the paper with the solvent.
•• Thin layer chromatography (TLC): Thin layer chromatography (TLC) is a widely employed laboratory
technique and is similar to paper chromatography. However, instead of using a stationary phase of paper,
it involves a stationary phase of a thin layer of adsorbent like silica gel, alumina, or cellulose on a flat, inert
substrate.
Example: Separation of vitamins.
•• Gas chromatography: In this type if the stationary phase is a solid it is called solid gas chromatography.
If the stationary phase is a high-boiling liquid deposit on a suitable support, it is called gas liquid
chromatography. The mobile phase used is generally N2, He or Ar, which do not react with the sample to
be separated out. Gas chromatography is based on solubility of different gases in their stationary phase.
The least soluble constituent comes out first while the most soluble comes out at the end. Thus various
components of a gaseous mixture are separated. Gas chromatogram is a graph of the detector response
when a gas comes out vs time.
Detector respones

Xylenes
Benzene
Meta
Toulene Ortho
Injection
point Mesitylene

Air
Time

Figure 27.4: Plot of Detector response vs time

Example: Separation of amino acids.

(d) Distillation: There are two types of distillations based on the difference between the boiling points of the
organic compound and its impurity.

(i) Simple distillation: This method is used if there is sufficient difference in the boiling points of volatile
liquids.
2 7 . 4 | Practical Organic Chemistr y

Example: a) Chloroform (CHCl3 ) and aniline (PhNH2 ) can be separated by simple distillation due to
large difference in their boiling points.
b) ether (b.p. 308K) and toluene (b.p. 384K)

(ii) Fractional distillation: This method is used to separate two volatile liquids when the difference in their
boiling points is small. In such a case when one liquid starts boiling the other is also converted to vapour
considerably. So, we get the vapours of both compounds. Hence, to avoid this fractionating column is
used. Fractionating column opposes the vapour of one with higher boiling point and its vapour again
condenses and goes to solution.

Thermometer

Water out

Fractionating
column
Condenser
Glass beads

Ethanol-water mixture
Water in
Antibumping
granules
Heat
Distillate
Figure 27.5: Distillation

Example: (a) Ethanol-water mixture, (b) Petroleum products are separated by fractional distillation, (c)
Cyclohexane-toluene mixture, (d) Acetone (b.p. 330K) and methyl alcohol (b.p. 338K)

(iii) Steam distillation: This method is used to separate the compounds which are steam volatile and are
insoluble in water. In this method, steam from a steam generator is passed through a heated flask
containing the liquid to be distilled. The mixture of steam and vapour is condensed and collected in a
receiver. Then the compound being insoluble in water can be separated easily.
Example: (a) Aniline-water mixture, (b) Isolation of citral, (c) Nitro benzene

(iv) Vacuum distillation: The lowering of pressure on the surface of a liquid lowers its boiling point. As
a result of this, a liquid can be boiled and distilled, without any decomposition, at temperature much
below its normal boiling point.
Example: Distillation of crude oil in oil refineries, Glycerol.

(v) Azeotropic distillation: Azeotropic mixture is a mixture having a constant boiling point. The constituents
of this mixture cannot be separated by fractional distillation. So, in this method a third component is
used in distillation. The process is based on the fact that dehydrating agents like C6H6,CCl4 , diethyl ether,
etc. depress the partial pressure of one of the original components. As a result, the boiling point of that
component is raised sufficiently and thus the other component distills over.
Example: Separation of ethanol and water.

(e) Differential extraction: This method is based on the different solubilities of an organic compound in different
solvents. If a compound is soluble in water but more soluble in an organic solvent, it is separated by shaking
it with an organic solvent. As a result, the organic compound gets dissolved into a organic solvent which can
be extracted from water.
Chem i str y | 27.5

Organic
compound
in solvent
Solvent layer
layer

Organic
compound Aqueous
in aqueous layer
layer
Before extraction After extraction

Figure 27.6: Separating Funnel

Example: (a) Iodine (I2) can be purified as it is sparingly soluble in water but highly soluble in CCl4 (b) Extraction
of caffeine from tea

Illustration 1: Two volatile compounds differ in their boiling points by 20K, how will they be separated?
(a) What types of compound are purified by sublimation?
(b) How will I2 be separated from KCl?
(c) How are o- and p-nitro phenols separated?
(d) How is aniline purified?
(e) How is a mixture of naphthalene and kerosene oil separated?

Sol: Fractional distillation.


(a) Substances whose vapour pressures become equal to the atmospheric pressure much below their b.p.
(b) Either by sublimation or by extraction with CCl4 followed by evaporation.
(c) o-nitrophenol is steam volatile while p-nitro phenol is not due to intramolecular H-bonding. Hence, they are
separated by steam distillation.
(d) Vacuum distillation or steam distillation.
(f) By simple distillation.

Illustration 2: p-nitrophenol and o-nitrophenol are separated by


(A) Crystallization
(B) Fractional crystallization
(C) Distillation
(D) Steam distillation

Sol: Steam distillation is used to separate the compounds which are steam volatile and are insoluble in water. This
process effectively allows for distillation at lower temperatures.
2 7 . 6 | Practical Organic Chemistr y

2. QUALITATIVE ESTIMATION

2.1 Introduction
Qualitative estimation is used in identifying elements in a compound. While judging the properties of a compound,
it is necessary to find elements constituting it. Organic compounds contain mainly C and H; in addition to these
they may contain O, N, S, halogens and P.

2.2 Detection of Elements

2.2.1 Detection of C and H


Many organic compounds burn with a sooty flame or char when strongly heated. C and H are detected by heating
the compound with cupric oxide (CuO) in a dry test tube. They are oxidized to CO2 and H2O respectively. CO2 turns
lime water milky and H2O turns anhydrous CuSO4 blue.

(a) C + 2CuO  → 2Cu + CO2

(b) 2H + CuO  → Cu + H2O
(c) CO2 + Ca(OH)2 → CaCO3 ↓ + H2O

(d) CuSO 4 + 5H2O → CuSO 4 .5H2O


White Blue

2.2.2 Preparing Sodium Fusion Extract


Most of the organic compounds are covalent. So, it is not possible to detect elements in it due to a strong covalent
bond. Therefore, it is necessary to convert them into ionic forms which can split easily in a polar medium. Sodium
(Na) being strongly electropositive is used in this process by preparing sodium fusion extract. While preparing
sodium fusion extract, the organic compound is strongly heated with clean sodium metal in a test tube fusing it
with the sample. The fused sample is plunged into water, and a few tests are performed on the solution to identify
elements in it. Following reactions occur:
∆ ∆ ∆
Na + C + N  → NaCN ; 2Na + S  → Na2S ; Na + X  → NaX (X = Cl, Br, Or I)
C, N, S and X come from organic compound.

2.2.3 Lassaigne’s Test


N, S, halogens and phosphorous present in organic compound is detected by Lassaigne’s test, after preparing
sodium fusion extract.
(a) Test for nitrogen: The sodium fusion extract is boiled with FeSO4 and then acidified with Concentrated H2SO4.
The appearance of Prussian blue colour confirms the presence of N. The following reaction occurs:
4−
 
(i) Fe2+ + 6 CN− → Fe(CN)6 
 
Hexa cyanoferrate (II)

Conc.
(ii) Fe2+ → Fe3+ + e
H2SO 4

4−
   
(iii) 3 Fe(CN)6  + 4Fe3+ → Fe4 Fe(CN)6  . H2O
   3
Ferriferro cyanide
(Pr ussian blue)
Chem i str y | 27.7

Exception:
1. This test is not given by compounds which do not contain C atoms but contain N atoms.

Example: NH2NH2 (hydrazine), NH2OH (hydroxyl amine).

Reason: Since, these compounds do not contain carbon, in a sodium fusion extract, CN‒ ion will not form and
Prussian blue colour is not observed.
2. This test is also not given by diazonium salts although they contain both C and N elements, because the nitrogen
in them is released due to strong heating while forming a sodium fusion extract.
3. If S is also present in organic compound along with C and N, this test gives a blood red colour.
Na + N + C + S → NaCNS (Sodium thiocyanate)

3CNS − + Fe+3 → Fe(CNS)3


Thiocyanate ion Ferric thiocyanate
(Blood−red colour)

4. If fusion is carried out with excess of sodium, the fusion gives cyanide and sulphide giving usual tests.
NaSCN + 2Na → NaCN + Na2S

(b) Test for sulphur:


(i) The sodium fusion extract is treated with sodium nitroprusside. Appearance of violet colour confirms the
presence of S.

2− −4
   
S2−
+ Fe(CN)5 NO  → Fe(CN)5 NOS 
Sulphide ion 
   
Violet colour

(ii) The extract is treated with acetic acid and lead acetate is added to it. The appearance of black ppt. of
lead sulphide confirms presence of S.
Pb2+ + S2− → PbS
Black ppt.

(c) Test for halogens:


(i) Extract is acidified with HNO3 and then treated with AgNO3. A white ppt. soluble in NH4OH indicates the
presence of Cl, a yellowish ppt. sparingly soluble in NH4OH indicates presence of Br, and a yellow ppt.
insoluble in NH4OH shows presence of I.
AgNO3 + NaCl → AgCl ↓ +NaNO3 ; AgNO3 + NaBr → AgBr ↓ +NaNO3
White ppt. Pale − yellow ppt.
soluble in (NH4 OH) partly soluble in (NH4 OH)


AgNO3 + NaI → AgI ↓ +NaNO3
Dark − yellow ppt.
insoluble in (NH4 OH)

If N and S are also present in the compound, this reaction forms NaCN or Na2S . These ions interfere with
AgNO3 test. If these ions are not removed they give white ppt. which will confuse it for AgCl.

(ii) Organic layer test: Extract is treated with CS2 and CCl4 and then Cl2 water is added to it. Then it is kept
for some time. Appearance of orange colour in organic layer indicates the presence of Br, while a violet
colour confirms iodine.

2Br − + 2Cl2 → Br2 + 2Cl− ; 2I − + 2Cl2 → I2 + 2Cl−


Orange colour Violet colour
2 7 . 8 | Practical Organic Chemistr y

(iii) Beilstein test: The organic compound is heated on a clean copper wire in Bunsen flame. A green coloured
flash indicates chlorine, blue-green of bromine and blue of iodine. Fluorine is not detected in this test.
Exception: Urea (NH2CONH2) and thiourea (NH2CSNH2) do not contain halogens but still give this test.

(d) Test for phosphorous: The organic compound is heated with an oxidizing agent (sodium peroxide).
Phosphorous is oxidized to phosphate. The solution is then boiled with conc. HNO3 and then treated with
ammonium molybdate. A canary yellow ppt. confirms the presence of phosphorous.
Na3PO 4 + 3HNO3 → H3PO 4 + 3NaNO3

H3PO 4 + 12(NH4 )2 MoO 4 + 21HNO3 → (NH4 )3 PO 4 .12MoO3 + 21NH4NO3 + 12H2O


Ammonium Ammonium phospho
molybdate molybdate

Exception: If the organic compound contains arsenic (As), then it also gives the same test forming ammonium
arsenomolybdate forming a canary yellow ppt.

2.3 Detection of Organic Groups


(a) Hydroxyl group (alcohol and phenol)

1. Lucas’s reagent (ZnCl2; conc. HCl): This reagent converts alcohols into the corresponding alkyl chlorides.
Zinc chloride (a Lewis acid) increases the reactivity of alcohols towards acid. The test depends on the rate of
reaction of primary, secondary, and tertiary alcohols with the reagent at room temperature.
(i) RCH2OH → No reaction at room temperature.
(ii) R 2CHOH → R 2CHCl + H2O (1 hour or maybe longer)
(iii) R3COH → R3CCl + H2O (immediately)
To 1 mL of the alcohol in a small test tube add 6 mL of Lucas’ reagent at room temperature. Close the
tube with a cork, shake and allow to stand.
•• Primary alcohols - The aqueous phase remains clear (except allyl alcohol - droplets after 7 minutes).
No turbidity or cloudiness is observed.
•• Secondary alcohols - Very slow reaction (~ 1 hour or maybe longer) when droplets of alkyl chloride
may be seen. Turbidity or cloudiness is observed.
•• Tertiary alcohols - Very fast reaction and droplets of the alkyl chloride formed almost immediately.
Turbidity or cloudiness is observed.

2. Victor meyer’s test: This test consists of the following steps:


(i) The given alcohol is first converted into its alkyl iodide by treating it with P & Iodine.
(ii) The alkyl iodide is then treated with silver nitrite (AgNO2) to convert it into a corresponding nitro alkane.
(iii) The nitroalkane is then treated with nitrous acid (HNO2), i.e., NaNO2 + HCl.
(iv) The resulting solution is finally made alkaline with aqueous NaOH or KOH. If blood red colouration
appears it is a primary alcohol. If blue colouration appears it is a secondary alcohol. If the solution
remains colourless, indicates a tertiary alcohol.
RCH2OH + P / I2 → RCH2 − I + AgNO2 → RCH2NO2 + HONO →

RC( = NOH) − NO2 + NaOH → Blood Red colour
R 2CH − OH + P / I2 → R 2CH − I + AgNO2 → R 2CHNO2 + HONO →

R 2C(N =O) − NO2 + NaOH → Blue colour
R C − OH + P / I2 → R3C − I + AgNO2 → R3C − NO2 + HONO →
3
No Reaction + NaOH → Colourless
Chem i str y | 27.9

 
 
3. Iodoform test: Iodoform test is given by alcohols having  CH3 − C − OH  group. In Iodoform test, the
 | 
 H 
compound reacts with I2 / NaOH reagent to give iodoform (CHl3) which is pale yellow in colour. It is insoluble
in water and has an antiseptic smell.

(b) Phenol detection


(i) Bromine water: Phenols are generally highly reactive towards electrophilic reagents and are readily
brominated by bromine water. e.g.
OH OH
Br Br
+ 3Br₂

Br
Dissolve or suspend about 0.05 g of the compound in 2 mL of dilute hydrochloric acid and add bromine
water dropwise until the bromine colour remains. A white precipitate of the bromophenol may form.
Solid bromophenol derivatives can be used for the confirmation of the structure of a phenol.

(ii) Ferric chloride test: Most phenols react with iron (III) chloride to form coloured complexes. The colours
vary - red, purple, blue or green - depending on various factors, e.g. the phenolic compound used, the
solvent, concentration. Since some phenols do not give colours, a negative test must not be taken as
significant without supporting information.

(c) Aldehyde detection


(i) Fehling’s solution: Aldehydes reduce Fehling’s solution to yellow or red copper (I) oxide.
Preparation of the reagent: Mix equal volumes of Fehling’s solution I (aqueous alkaline potassium
tartrate) and Fehling’s solution II (copper sulphate solution).
Exception: Only aliphatic aldehydes give this test. Aromatic aldehydes don’t give this test.

(ii) Tollen’s reagent (Ammoniacal silver nitrate solution): Aldehydes are readily oxidized to carboxylic
acids and will reduce Tollen’s reagent to produce a silver mirror on the inside of a clean test tube.
Exception: α -hydroxy ketones also give this test but other ketones do not. Hemiacetals and formic acid
also give this test. But acetals do not.

(d) Aldehyde or Ketone detection


(i) Brady’s reagent (2,4-Dinitrophenylhydrazine): A test for the carbonyl group (C=O) in aldehydes and
ketones. 2,4-Dinitrophenylhydrazine gives sparingly soluble yellow or red 2,4-dinitrophenylhydrazones
with aldehydes and ketones.
R R H
C O + H₂N N NO₂ C N N NO₂
R R
H
NO₂ NO₂
R,R’ = hydrogen or alkyl

(ii) Iodoform Test: Iodoform test is given by simple methyl ketones and acetaldehyde only i.e. compounds
having (CH3 − C = O−) group. In Iodoform test, the compound reacts with I2 / NaOH reagent to give
iodoform (CHI3 ) which is pale yellow in colour. It is insoluble in water and has an antiseptic smell.
2 7 . 1 0 | Practical Organic Chemistr y

(e) Carboxylic acid: Test with 5% aq. NaHCO3


R − CO2H + NaHCO3 → R − CO2 − Na+ + CO2 + H2O

Sodium hydrogen carbonate reacts with carboxylic acids to give the sodium salt of the acid and liberates carbon
dioxide. If the acid is insoluble in water and the reaction is sluggish, dissolve the acid in methanol and add
carefully to a saturated sodium hydrogen carbonate solution, when a vigorous effervescence will be observed.

(f) Distinction between alcohol, phenol and carboxylic acid


(i) All alcohols react with sodium metal (Na) to evolve H2 gas. But do not react with NaOH or NaHCO3.

2CH3CH2OH + 2Na → 2CH3CH2ONa + H2


(ii) All phenols react with sodium metal (Na) and NaOH. But do not react with NaHCO3.
_
OH O Na+

+ NaOH + H₂O

Sodium phenoxide

(iii) All carboxylic acids react with sodium metal (Na), NaOH and NaHCO3.

(g) Amines
(i) Reaction with nitrous acid:
AMINE REACTION
1° aliphatic → N2 evolved
RNH2 + HNO2 → ROH + N2 + H2O
1° aromatic → Diazonium salt is formed.
ArNH2 + HNO2 → ArN ≡ N+

Add the cold diazonium solution and with swirling to a cold solution of 2-naphthol (0.2 g) in 5% NaOH
solution (2mL). An orange-red azo dye is formed.
2° aliphatic → Yellow oily nitrosamines are generally formed.
2° aromatic → R NH + HNO → R N − NO
2 2 2

3° aliphatic → No visible reaction

3° aromatic → Dialkylanilines yield green solid p-nitroso compounds


(if p-position unsubstituted).
(ii) Hinsberg reaction: In this test amine is shaken well with Hinsberg reagent (benzene sulphonyl chloride)
in the presence of aqueous alkali (either NaOH or KOH). A primary amine will form a soluble sulfonamide
salt which precipitates after addition of diluted hydrochloric acid. A secondary amine in the same reaction
will directly form an insoluble sulfonamide. A tertiary amine will not react with the sulfonamide but is
insoluble. After adding a dilute acid this insoluble amine is converted to a soluble ammonium salt. In this
way the reaction can distinguish between the three types of amines.
Chem i str y | 27.11

O O H
H HCl
Ph S Cl + H N Ph S N R
R
O O
N-Alkylbenzene
N
sulphonamide (insoluble)

HCl NaOH

O H 
Ph S N R Cl
O H
(Soluble)

A precipitate which is an insoluble compound (which is insoluble in NaOH solution also) indicates 2°
amines.
O O R
R
Ph S Cl + H N Ph S N R + HCl
R
O O
N,N-Dialkyl benzene sulphonamide
(insoluble in NaOH)

3° amines do not react with the reagent.

(iii) Reaction with Carbon disulphide (CS2): Primary amine reacts with CS2 and further reacts with base due
to acidic hydrogen present on nitrogen.
R – NH2 + CS2 → R – NH – CSSH → reacts with base
Secondary amine reacts with CS2 but doesn’t react with a base due to the absence of acidic hydrogen.
R 2NH + CS2 → R 2`N − CSSH does not react with base.
Tertiary amine do not react with CS2 .

(iv) Carbylamine test: 1° aliphatic and aromatic amines on heating with CHCl3 and KOH give an offensive
smell due to the formation of carbylamine.

RNH2 + CHCl3 + 3KOH → R − NC +3KCl + 3H2O


Alkyl isocyanide
or
Alkyl carbyl amine

PhNH2 + CHCl3 + 3KOH → Ph − N ≡ C+ 3KCl + 3H2O


Aniline

2° and 3° aliphatic and aromatic amines do not give this test.

(v) Azo-dye test: 1° aromatic amines on heating with NaNO2 and HCl give stable diazonium salts which
couple with alkaline solution of β -naphthol to give orange-red azo dye.
NaNO₂ + HCl
PhNH₂ + HNO₂ Ph N N Cl
273 K
Benzene diazonium chloride
OH OH

Ph N N
pH = 9-10
-Naphthol

Orange-red dye
2 7 . 1 2 | Practical Organic Chemistr y

(h) Amides
(i) Simple primary amides can be decomposed by boiling with alkali and thereby evolving ammonia which
has pungent smell. E.g. CH3 − CO − NH2 + NaOH → CH3 − CO2 − Na+ + NH3

(ii) Hofmann rearrangement: Amides show Hofmann rearrangement to form a primary amine when
treated with Br2 / NaOH
O O
Br₂ C H₂O
R R NH₂
NaOH N CO₂
R NH₂

This test confirms presence of amide.

(i) Ester detection


(i) Hydroxamic acid test: R – CO – OR’ + H2N – OH → R – CO – NH – OH + R’ – OH
Esters react with hydroxylamine in the presence of sodium hydroxide to form the sodium salt of the
corresponding hydroxamic acid. On acidification and addition of ferric chloride the magenta-coloured
iron (III) complex of the hydroxamic acid is formed.
(ii) Hydrolysis test: Esters undergo saponification reaction i.e. hydrolysed with NaOH. The compound is
heated with NaOH with a small amount of phenolphthalein. The initial pink colour of the solution is
discharged due to the formation of acid, since phenolphthalein shows pink colour in basic medium and
is colourless in acidic medium.
O O

R – C – O – R' + NaOH R – C – ONa + R'OH

(j) Nitro group detection

(i) Mulliken’s test: The nitro compound is dissolved in alcohol and is reduced with Zn / NH4 Cl or CaCl2
solution and Zn dust, and boiled. It is then filtered, cooled, and heated with ammoniacal AgNO3 , white
to grey and the black precipitate confirms the presence of nitro group.
Zn
RNO2 + 4[H]  → RNHOH + H2O
NH4 Cl

(ii) Azo dye test: Aromatic nitro compounds are first reduced to −NH2 group with SnCl2+conc. HCl which
on reaction with nitrous acid forms diazonium salts. The stable diazonium salt forms orange red dye with
alkaline solution of β -naphthol.
[H]
PhNO₂ Sn + HCl PhNH₂
Nitro
benzene HNO₂

OH
Orange-red Ph N N Cl
azo dye -Naphthol

(k) Sugars, Carbohydrates detection


(i) Molisch’s test: This is a general test for carbohydrates. Dissolve 20 - 30 mg of the compound in 2 mL
water and add 0.5 mL of the reagent (a 20% solution of 2-naphthol in ethanol). Pour 2 mL of concentrated
sulphuric acid from a dropper carefully down the side of the tube so that the acid forms a layer beneath
the aqueous solution without mixing with it. A red colouration, changing to dark purple forms at the
interface.
(ii) Fehling’s test: This forms the reduction test of carbohydrates. Fehling’s solution contains blue alkaline
cupric hydroxide solution, heated with reducing sugars gets reduced to yellow or red cuprous oxide
and is precipitated. Hence, formation of the yellow or brownish-red coloured precipitate helps in the
detection of reducing sugars in the test solution.
Chem i str y | 27.13

(iii) Osazone Test: The ketoses and aldoses react with phenylhydrazine to produce a phenylhydrazone
which further reacts with another two molecules of phenylhydrazine to yield osazone. Needle-
shaped yellow osazone crystals are produced by glucose, fructose and mannose, whereas
lactosazone produces mushroom shaped crystals. Crystals of different shapes will be shown by
different osazones. Flower-shaped crystals are produced by maltose.

(l) Proteins
(i) Ninhydrin test: Amino acids also react with ninhydrin at pH = 4. The reduction product obtained from
ninhydrin then reacts with NH3 and excess ninhydrin to yield a blue colored substance. This test is given
by only amino acids and proteins which contain free −NH2 groups in their structure. Proline doesn’t give
this test due to absence of free −NH2 group.
(ii) Xanthoproteic test: Some amino acids contain aromatic groups that are derivatives of benzene. These
aromatic groups can undergo reactions that are characteristics of benzene and benzene derivatives.
One such reaction is the nitration of a benzene ring with nitric acid. The amino acids that have activated
benzene ring can readily undergo nitration. This nitration reaction, in the presence of activated benzene
ring, forms yellow product.
Example: tyrosine, tryptophan, phenylalanine and glutamic acid.
(iii) Millon’s Test: Millon’s test is specific to phenol containing structures (tyrosine is the only common
phenolic amino acid). Millon’s reagent is concentrated HNO3 , in which mercury is dissolved. As a result
of the reaction a red precipitate or a red solution is considered as positive test. A yellow precipitate of
HgO is NOT a positive reaction but usually indicates that the solution is too alkaline.
Example: tyrosine, phenylalanine, glycine and β -naphtol.
(iv) Biuret Test: The Biuret Test positively identifies the presence of proteins (not less than two Peptides). The
reaction in this test involves the complex formation of the proteins with Cu2+ ions in a strongly alkaline
solution.
Example: gelatin, casein and albumin.

(m) Alkene and alkyne


(i) Br2- water test: The decoloration of a solution of bromine in water with dichloromethylene as catalyst
is an analytical test for the presence of alkenes:
CH2 CH2 + Br2 → BrCH2 − CH2Br
=
It is also used as a quantitative test of unsaturation.
(ii) Bromine in CCl4: Bromine will add to the carbon-carbon double bond of alkenes to produce
dibromoalkanes and with alkynes to produce tetrabromoalkanes. When this reaction occurs, molecular
bromine is consumed, and its characteristic dark redbrown color disappears if bromine is not added in
excess. The rapid disappearance of the bromine color is a positive test for unsaturation.

C=C + Br₂ CCl₄ Br C C Br


Alkene Red-Brown
Colorless

Br Br

C ≡C + 2Br₂ CCl₄ C C
Alkyne Red-Brown
Br Br
Colorless
2 7 . 1 4 | Practical Organic Chemistr y

(iii) Baeyer’s reagent: (Alkene only): Baeyer’s reagent is an alkaline solution of cold potassium permanganate,
which is a powerful oxidant making this a redox reaction. Reaction with double or triple bonds (–C = C –
or – C ≡ C –) in an organic material causes the color to fade from purplish-pink to brown. It is a syn addition
reaction.
2KMnO₄ + H₂O 2KOH + 2MnO₂ + 3[O]
Purple Brownish
colour red ppt.

R³ R⁴
1.2KMnO₄
2.2H₂O
R³ R⁴OH
2
+ MnO₂+K₂MnO₄
R¹ R² R¹ OH

HC CH + H₂O + 4[O] HOOC COOH

(n) Detection of terminal alkyne


(i) AgNO3 in ammonia: The reactions of alkynes with certain metal cations, e.g. Ag+ gives acetylides. Thus,
few drops of diammine silver(I) hydroxide Ag(NH3)2OH reacts with terminal alkynes signaled by the
formation of a silver white precipitate of the silver acetylide.
RC ≡ C − H + Ag(NH3 )2+ (in NH4 OH) → RC ≡ C − Ag (insoluble) + NH3 + NH+4

(ii) Acidic nature of terminal alkyne: Terminal alkyne being acidic reacts with strong bases like NaNH2 to
form sodium acetylide.
H − C ≡ C − H + NaNH2 (in ammonia or ether) → H − C ≡ C − Na (sodium acetylide)+NH3

(iii) With Grignard reagent: Terminal alkynes react with Grignard reagent to form alkanes.
RC ≡ C − H + C2H5MgBr (in ether) → RC ≡ C − MgBr + C2H6

(o) Detection of alkoxy group


(i) Hot Hydroiodic Acid (HI) test: The relatively unreactive C – O bonds in an ether can be cleaved by
treatment with conc. HI to give two alkyl iodides and water. The alkyl iodides produced can react with
mercuric nitrate, Hg(NO3)2 , to give the orange-red coloured mercuric iodide (HgI2).

ROR '+ 2HI (Hot) → RI + R 'I + H2O


2RI + Hg(NO3 )2 → HgI2 + 2R − O − NO2

Vermillion coloured

(ii) Cold HI Test: When ether is treated with cold HI, the smaller alkyl group in ether forms iodide while
bigger one forms alcohol in case of asymmetric ether.
CH3 − CH2 − O − CH3 + cold HI → CH3 I + CH3 − CH2 − OH

PLANCESS CONCEPTS

Exceptions:
•• Lassaigne’s test is not given by compounds which do not contain C atoms but contain N atoms.
Example: NH2NH2 (hydrazine), NH2OH (hydroxyl amine).
Reason: Since, these compounds do not contain carbon, in sodium fusion extract, CN− ion will not form
and Prussian blue colour is not observed.
Chem i str y | 27.15

PLANCESS CONCEPTS

•• This test is also not given by diazonium salts although they contain both C and N elements, because the
nitrogen in them is released due to strong heating while forming sodium fusion extract.
•• If S is also present in organic compound along with C and N, this test gives a blood red colour.

Na + N + C + S → NaCNS (Sodium thiocyanate)


3CNS − + Fe+3 → Fe(CNS)3
Thiocyanate ion Ferric thiocyanate
(Blood−red colour)

•• If fusion is carried out with excess of sodium, the fusion gives cyanide and sulphide giving usual tests.

NaSCN + 2Na → NaCN + Na2S

•• In Beilstein’s test, Urea (NH2CONH2 ) and thiourea (NH2CSNH2 ) do not contain halogens but still give
the test.
•• In detection of phosphorous using ammonium molybdate, if the organic compound contains arsenic (As),
then it also gives this test forming ammonium arsenomolybdate forming canary yellow ppt.
•• In Fehling’s solution test, only aliphatic aldehydes give this test. Aromatic aldehydes don’t give this test.
•• In Tollen’s reagent test, α -hydroxy ketones give this test but other ketones do not.
•• All amino acids give ninhydrin test except Proline due to absence of free –NH2 group.
•• 2° and 3° amines do not give carbylamine test.
Nikhil Khandelwal (JEE 2009, AIR 94)

PLANCESS CONCEPTS

•• L ithium is not used in Lassaigne’s test even being more electropositive than sodium because it reacts
slowly and its compounds are covalent with less ionic nature. Potassium is not used because it reacts
vigorously and cannot be handled properly.
•• In the Lassaigne’s test for nitrogen, if Fe3+ ions (with yellow colour) are in excess, it makes the blue colour
of ferrocyanide to appear green.
•• Tollen’s test is also given by formic acid and hemiacetals. But acetals do not give this test.

Saurabh Gupta (JEE 2010, AIR 443)

Illustration 3: Lassaigne’s test is not shown by diazonium salts and hydrazine (NH2NH2) even though they contain
nitrogen. Why?

Sol: Lassaigne’s test is not given by compounds which do not contain C atoms but contain N atoms. In case of
diazonim salt the nitrogen is released due to strong heating while forming sodium fusion extract.
Diazonium salts usually lose N2 on heating much before they react with fused Na metal. That is why they do not
give the above test. Hydrazine does not have C atoms, so it does not form NaCN and hence does not give a positive
test for N. In order to test the presence of N in such compounds, during fusion with Na, some charcoal or starch is
added to supply C atoms. Under these conditions, NaCN is obtained and it will give a positive test.
2 7 . 1 6 | Practical Organic Chemistr y

Illustration 4: The Prussian blue colour obtained in the Lassaigne’s test for nitrogen is due to the formation of
(A) Iron (II) hexacyanoferrate (III) (B) Iron (III) hexacyanoferrate (II)
(C) Iron (III) hexacyanoferrate(III) (D) Iron (II) hexacyanoferrate (II)
(E) Sodium hexacyanoferrate (III)

Sol: (B) The Prussian blue colour obtained in the Lassaigne’s test for nitrogen is due to the formation of
4−
2+ −
 
Fe + 6 CN → Fe(CN)6 
 
Hexa cyanoferrate (II)

3. QUANTITATIVE ANALYSIS
Quantitative analysis of an organic compound includes the measurement of a quantity of an element in the
compound. While determining molecular formula of an organic compound, it is necessary to know the percent
quantity of every element present in it. There are various processes to determine this percentage based on that
element.

(a) Liebig’s combustion method: A known mass of compound is heated with CuO. The carbon present is
oxidized to CO2 and hydrogen to H2O. The CO2 is absorbed in KOH solution, while H2O vapours are absorbed
in anhydrous CaCl2. So, change in their weights will give percentage of C and H.

Sample in Coarse
platinum boat CuO CaCl₂
Combustion guard tube
tube CuO
gauge

Furnace
CuO
gauge
KOH solution
Anhydrous and asbestos
magnesium
Pure dry perchlorate
oxygen

Figure 27.7: Combustion Tube

12 Mass of CO2
Percentage of C = × × 100 ;
44 Mass of compound

2 Mass of H2O
Percentage of H = × × 100
18 Mass of compound

(b) Carius Method: The Carius halogen method in analytical chemistry is a method for the quantitative
determination of halogens in chemical substances. A known mass of an organic compound is heated with
fuming nitric acid (HNO3) in the presence of silver nitrate (AgNO3) contained in a hard glass tube known as
carius tube, in a furnace. Carbon and hydrogen present in the compound are oxidised to carbon dioxide
and water. The halogen present forms the corresponding silver halide (AgX). It is filtered, washed, dried
and weighed. This chemical test works equally well for the determination of sulfur but without the addition
of silver nitrate. The sulfuric acid intermediate formed after reaction of sulfur with fuming nitric acid forms
insoluble barium sulfate on the addition of barium chloride.
Chem i str y | 27.17

Iron tube
Carius tube

Organic HNO₃ + AgNO₃


substance

Furnace

Figure 27.8: Carius Tube

Atomic mass of X Mass of AgX × 100


(i) Percentage of X = ×
Molecular mass of AgX Mass of compound

35.5 Mass of AgCl × 100


(ii) Percentage of Cl = ×
143.5 Mass of compound

80 Mass of AgBr × 100


(iii) Percentage of Br = ×
188 Mass of compound

127 Mass of AgI × 100


Percentage of I = ×
235 Mass of compound

(c) Estimation of sulphur: A known mass of compound is heated with fuming HNO3 or sodium peroxide (Na2O2).
In the presence of BaCl2 solution in Carius tube. Sulphur is oxidized to H2SO4 and precipitated as BaSO4. It is
filtered, dried and weighed.

Atomic mass of S Mass of BaSO 4 × 100 32 Mass of BaSO 4 × 100


Percentage of S = × = ×
Molecular mass of BaSO 4 Mass of compound 233 Mass of compound

(d) Estimation of phosphorous:

First Method: A known mass of compound is heated with fuming HNO3 in Carius tube which converts phosphorous
to H3PO4 (phosphoric acid). It is precipitated as ammonium phosphomolybdate [(NH4)3PO4.12MoO3] by adding
NH3 and ammonium molybdate. It is filtered, dried, and weighed.

Atomic mass of P
Percentage of P = ×
Molecular mass of ammonium phospho molybdate

Mass of ammonium phospho molybdate × 100 31 Mass of (NH4 )3 .PO 4 .12MoO3 × 100
= ×
Mass of compound 1877 Mass of compound

Second Method (Carius method): A known mass of compound is heated with fuming HNO3 or sodium
peroxide in Carius tube which converts phosphorous to H3PO4. Magnesia mixture (MgCl2 + NH4 Cl) is then
added, which gives the precipitate of magnesium ammonium phosphate MgNH4.PO4 which on heating gives
magnesium pyrophosphate (Mg2P2O7 ) , which is weighed.

Atomic mass of P Mass of Mg2P2O7 × 100 62 Mass of Mg2P2O2 × 100


Percentage of P = × = ×
Molecular mass of Mg2P2O7 Mass of compound 222 Mass of compound
2 7 . 1 8 | Practical Organic Chemistr y

(e) Estimation of nitrogen:


(i) Dumas method: A known mass of the organic compound is heated with cupric oxide in an atmosphere
of carbon dioxide. The carbon and hydrogen in the compound are oxidized to carbon dioxide and
water respectively, while nitrogen is set free. Any oxide of nitrogen produced during this process, is
reduced back to free nitrogen by a heated copper gauze. The gaseous mixture consisting of CO2, H2O
and N2 is collected over an aqueous solution of potassium hydroxide. All the gases except nitrogen
are absorbed by the solution. The volume of gas (nitrogen) collected is measured. From the volume of
nitrogen obtained the percentage of nitrogen in the compound is calculated.

Reduced
CuO+ copper Nitrogen
organic Coarse gauze
compound CuO
CuO
gauze
KOH
CO2 solution
Furnace

Mercury seal Nitrometer

Figure 27.9: Dumas Apparatus

Calculations
Let, The mass of the organic compound taken be = Wg; Volume of nitrogen collected = V1g
Atmospheric pressure = P mm Hg; Temperature at which gas is collected = T1K
Therefore, Pressure of the N2 gas, P1 = (P - p) mm of Hg

P1 V2 × 273 P1 V1 P2 V2
Volume of nitrogen at STP = = V mL (Using = )
760 × T1 T1 T2

28V
Mass of V mL of nitrogen at STP = g (1 mol of N2 = 28 g = 22400 mL)
22400
28V 100
Therefore, Percentage of nitrogen in the compound = ×
22400 W

(ii) Kjeldahl’s method: Kjeldahl’s method is a faster method than Dumas’ method. However, this method
is used only for those organic compounds that are converted quantitatively to ammonium sulphate on
heating strongly with concentrated sulphuric acid.
Kjeldahl’s method cannot be used for the organic compounds,
1. Containing nitrogen in the ring, e.g., pyridine, quinoline etc.
2. Containing nitro (‒NO2) and diazo (‒N = N‒) groups.
3. Kjeldahl’s method involves two steps:

•• Digestion: A known mass (0.3 to 0.5 g) of the given organic compound is digested with concentrated
H2SO4, in the presence of a small quantity of potassium sulphate and copper sulphate in a Kjeldahl’s flask.
Potassium sulphate raises the boiling point of sulphuric acid and copper sulphate catalyzes the digestion.
In 3 to 4 hours, the organic compound is completely decomposed to form ammonium sulphate.
Cu 2+
Organic compound + H2SO 4 →(NH4 )2 SO 4
digestion
Chem i str y | 27.19

•• Distillation: The digested reaction mixture, on cooling, is transferred to a round bottomed distillation
flask, and distilled with a concentrated alkali solution (NaOH). Ammonia produced is absorbed in a
known volume of HCl solution of a known strength.

(NH4 )2 SO 4 + 2NaOH → Na2SO 4 + 2H2O + 2NH3

NH3 + HCl → NH4 Cl


The un-neutralised HCl is then back-titrated against a standard alkali. From the acid consumed, the
amount of ammonia produced and hence the mass of nitrogen is calculated.

Figure 27.10: Kjeldahl’s Method

Apparatus for the estimation of nitrogen by Kjeldahl’s method

Calculation
Let, Mass of the organic compound = W g
Volume of the standard acid required for complete neutralization of the evolved ammonia = V mL; Normality of
the standard solution of acid = N
From the law of equivalence (normality equation),
1000 mL of 1 N acid = 1000 mL of 1 N NH3 = 17g NH3 = 14g nitrogen
Then, V mL of N acid = V mL of NH3
NV milli equivalent of acid = NV milli equivalent of ammonia
14 × N × V
Therefore, Mass of nitrogen in the evolved ammonia = g
1000
14NV 100 1.4NV
Then, Percentage of nitrogen in the sample = × =
1000 W W
Percentage of nitrogen in the sample =

1.4 × Normality of HCl × Volume of HCl used for complete neutralization of NH3
Mass of the compound taken

(f) Estimation of oxygen:
First Method: It is usually the difference between total percentage composition (100%) and sum of the
percentage of all other elements.
Percentage of O = 100 ‒ (Percentage of C + Percentage of H + Percentage of N +…..)
2 7 . 2 0 | Practical Organic Chemistr y

Second Method (Aluise’s method): A known mass of compound is decomposed by heating it in the
presence of N2 gas. The mixture of gaseous products containing O2 is passed over red hot coke when all O2
is converted into CO. This mixture is heated with I2O5 in which CO is converted into CO2 liberating I2 .

Compound → Other gaseous product + O2
1373K
2C + O2 
→ 2CO

I2O5 + 5CO → 5CO2 + I2



Molecular mass of O2 Mass of CO2 × 100 32 Mass of CO2 × 100
Percentage of O = × = ×
Molecular mass of CO2 Mass of compound 44 Mass of compound

4. DETERMINATION OF EMPIRICAL FORMULAE AND MOLECULAR


FORMULAE
The empirical formula of a chemical compound is a representation of the simplest whole number ratio between
the elements comprising the compound. The molecular formula is the representation of the actual whole number
ratio between the elements of the compound. This step by step tutorial shows how to calculate the empirical and
molecular formulas for a compound.

Problem: A molecule with molecular weight of 180.18 g/mol is analysed and found to contain 40.00% carbon,
6.72% hydrogen and 53.28% oxygen. What are the empirical and molecular formulas of the molecule?

Solution: Finding the empirical and molecular formula is basically the reverse process used to calculate mass
percent.
Step 1: Find the number of moles of each element in a sample of the molecule.
Our molecule contains 40.00% carbon, 6.72% hydrogen and 53.28% oxygen. This means a 100 gram sample
contains:
40.00 grams of carbon (40.00% of 100 grams)
6.72 grams of hydrogen (6.72% of 100 grams)
53.28 grams of oxygen (53.28% of 100 grams)
Note: 100 grams is used for a sample size just to make the maths easier. Any sample size could be used, the ratios
between the elements will remain the same.
Using these numbers we can find the number of moles of each element in the 100 gram sample. Divide the number
of grams of each element in the sample by the atomic weight of the element (from the periodic table) to find the
number of moles.
moles C = 40.00 g x 1 mol C/12.01 g/mol C = 3.33 moles C
moles H = 6.72 g x 1 mol H/1.01 g/mol H = 6.65 moles H
moles O = 53.28 g x 1 mol O/16.00 g/mol O = 3.33 moles O

Step 2: Find the ratios between the number of moles of each element.
Select the element with the largest number of moles in the sample. In this case, the 6.65 moles of hydrogen is the
largest. Divide the number of moles of each element by the largest number.
Simplest mole ratio between C and H: 3.33 mol C/6.65 mol H = 1 mol C/2 mol H
The ratio is 1 mole C for every 2 moles H
Simplest ratio between O and H: 3.33 moles O/6.65 moles H = 1 mol C/2 mol H
The ratio between O and H is 1 mole O for every 2 moles of H
Chem i str y | 27.21

Step 3: Find the empirical formula.


We have all the information we need to write the empirical formula. For every 2 moles of hydrogen, there is one
mole of carbon and one mole of oxygen. The empirical formula is CH2O .

Step 4: Find the molecular weight of the empirical formula.


We can use the empirical formula to find the molecular formula using the molecular weight of the compound and
the molecular weight of the empirical formula.
The empirical formula is CH2O . The molecular weight is
molecular weight of CH2O = (1 x 12.01 g/mol) + (2 x 1.01 g/mol) + (1 x 16.00 g/mol)
molecular weight of CH2O = (12.01 + 2.02 + 16.00) g/mol
molecular weight of CH2O = 30.03 g/mol

Step 5: Find the number of empirical formula units in the molecular formula.
The molecular formula is a multiple of the empirical formula. We were given the molecular weight of the molecule,
180.18 g/mol. Divide this number by the molecular weight of the empirical formula to find the number of empirical
formula units that make up the compound. Number of empirical formula units in compound = 180.18 g/mol/30.03
g/mol
Number of empirical formula units in compound = 6

Step 6: Find the molecular formula.


It takes six empirical formula units to make the compound, so multiply each number in the empirical formula by 6.
molecular formula = 6 x CH2O
molecular formula = C(1×6)H(2×6)O(1×6)
molecular formula = C6H12O6

Sol: The empirical formula of the molecule is CH2O .


The molecular formula of the compound is C6H12O6 .

PLANCESS CONCEPTS

1. Leibig combustion method: (Estimation of C and H)


12 Mass of CO2
Percentage of C = × × 100
44 Mass of compound
2 Mass of HO2
Percentage of H = × × 100
18 Mass of compound

2. Carius’s method ( Estimation of halogens):

Atomic mass of X Mass of AgX × 100


(i) Percentage of X = ×
Molecular mass of AgX Mass of compound

35.5 Mass of AgCl × 100


(ii) Percentage of Cl = ×
143.5 Mass of compound

80 Mass of AgBr × 100


(iii) Percentage of Br = ×
188 Mass of compound
2 7 . 2 2 | Practical Organic Chemistr y

PLANCESS CONCEPTS

127 Mass of AgI × 100


(iv) Percentage of I = ×
235 Mass of compound

3. Estimation of sulphur:
Atomic mass of S Mass of BaSO 4 × 100 32 Mass of BaSO 4 × 100
Percentage of S = × = ×
Molecular mass of BaSO 4 Mass of compound 233 Mass of compound

4. Carius method (Estimation of phosphorous):


Atomic mass of P Mass of Mg2P2O7 × 100 62 Mass of Mg2P2O7 × 100
Percentage of P = × = ×
Molecular mass of Mg2P2O7 Mass of compound 222 Mass of compound

5. Dumas’s method (Estimation of nitrogen):


28V 100
Percentage of nitrogen in the compound = ×
22400 W

6. Kjeldahl’s method (Estimation of nitrogen):


Percentage of nitrogen in the sample
1.4 × Normality of HCl × Volume of HCl used for complete neutralization of NH3
=
Mass of the compound taken

7. Estimation of oxygen in general:


Percentage of O = 100 ‒ (Percentage of C + Percentage of H + Percentage of N +…..)
Molecular Formula= n × (Empirical formula)

T P Varun (JEE 2012, AIR 64)

Illustration 6: 0.50 gm of an organic compound was treated according to Kjeldahl’s method. The ammonia evolved
was absorbed in 50 ml of 0.5 M H2SO 4 . The residual acid requires 60 ml of M/2 NaOH solution. Find the percentage
of nitrogen in the compound.

Sol: Percentage of nitrogen present in the given compound can be determined using Kjeldahl’s method. We
can use the following expression, this method makes use of titration of given compound with acid, and thus it
involves neutralization reaction. From the volume of acid require for complete neutralization reaction percentage
of nitrogen can be determined.
Percentage of nitrogen in the sample
1.4 × Normality of HCl × Volume of HCl used for complete neutralization of NH3
=
Mass of the compound taken

mEq. of acid= 50 × 0.5 × 2 = 50: mEq. of base = 60 × 0.5 × 1=30


Excess of acid used to neutralise ; NH3 = 50 – 30 = 20 mEq

1.4 × mEq. of acid used 1.4 × 20


Percentage of N2 = = = 56.0%
Weight of compound 0.5

mEq. of acid= 50 × 0.5 × 2 = 50: mEq. of base = 60 × 0.5 × 1=30


Chem i str y | 27.23

Excess of acid used to neutralise ; NH3 = 50 – 30 = 20 mEq

1.4 × mEq. of acid used 1.4 × 20


Percentage of N2 = = = 56.0%
Weight of compound 0.5

Illustration 7:
(a) How is a mixture of two organic compounds separated, which have different solubilities in the same solvent?
(b) How is an organic liquid purified which decomposes below its boiling point?
(c) Why is sodium extract boiled with HNO3 before testing for halogens?
(d) What type of organic compounds cannot be estimated by Kjeldahl’s method?
(e) What does blood-red colouration during lassaigne’s test indicate?
(f) What happens when AgNO3 solution is added to bromo methane?

Sol: (a) By fractional crystallization.


(b) Distillation under reduced pressure i.e. vacuum distillation.
(c) To decompose NaCN or Na2S present in the Lassaigne’s extract, otherwise, these will produce white precipitate
with AgNO3 and thus interfere with the test of halogens.
(d) Compounds containing N in the ring and those compounds in which N atom is directly linked either to an
oxygen atom or another N atom such as in azo (–N = N–) and nitro ( −NO2 ) compounds.
(e) It indicates the presence of both N and S.
(f) It would not give pale-yellow precipitate of AgBr because CH3Br is a covalent compound and hence does not
contain Br- ion. To get the precipitate of AgBr, CH3Br is treated with aqueous KOH to get CH3OH and Br − and then
addition of AgNO3 . It would now give the precipitate of AgBr partly soluble in NH3 or NH4 OH .

Illustration 8: ’O’ 1693 gm of a volatile substance when vapourised displaced 58.9 ml of air measured at 270 C and
746 mm pressure. Calculate the molar mass of the substance. (Aqueous tension at 270 C =26.7 mm Hg)

Sol: First calculate the volume of air displaced at STP condition by using the ideal gas equation. Now using the
volume term calculate the molecular mass of the given compound.

Step I: To convert the volume at experimental condition to volume at STP.

P1 = 746 – 26.7 = 719.3 mm P2 = 760 mm


V1 = 58.9 ml V2 = ?
T1 = 273 + 27 = 300 K T2 = 273 K

Substituting these values in the gas equation.

P1 V1 P2 V2 719.3mm × 58.9ml 760mm × V2ml


= , we get =
T1 T2 300K 273K

719.3mm × 58.9ml × 273K


∴ V2 = 50.73 ml
300K × 760mm
Mass of subs tance × 22400 0.1693 × 22400
Molecular mass = = = 74.75 gm
Volume of displaced air at STP 50.73

Illustration 9: 0.15 gm of an organic compound gave 0.12 gm of silver bromide by the Carius method. Find the
percentage of bromide in the compound.
2 7 . 2 4 | Practical Organic Chemistr y

Sol: We are asked to find out the percentage of bromide in the compound. As we are provided with mass of given
compound and mass of silver bromide produced by that compound by the carius method. So we can use the
following expression Percentage of
80 Mass of AgBr
Br = ×
188 Mass of compound

80 Mass of AgBr 80 0.12


Percentage of Br = × = × × 100 = 34.0%
188 Mass of compound 188 0.15

Illustration 10: 0.12 gm of an organic compound containing phosphorous gave 0.22 gm of Mg2P2O7 by the usual
analysis. Calculate the percentage of phosphorous in the compound.

Sol: From the given data i.e mass of Mg2P2O7 produced and mass of given compound percentage of phosphorous
can be determined using following formula:

62 Mass of Mg2`P2O7
Percentage of phosphorus = × × 100
222 Mass of compound

62 Mass of Mg2`P2O7 62 0.22


Percentage of phosphorus = × × 100 = × × 100 = 51.20%
222 Mass of compound 222 0.12

5. DETERMINATION OF MOLECULAR MASS


(a) Silver Salt Method For Carboxylic Acids: Most of the organic acids form sparingly soluble silver salts, which
on ignition leave a residue of metallic silver. This forms the basis of the silver salt method for the determination
of molecular mass of acids. The organic acid solution is neutralized with NH2OH and heated to boil off excess
NH3. It is then treated with excess of AgNO3 solution and the precipitated silver salt is filtered, dried. A known
mass of the dry silver salt is ignited in a crucible to a constant mass.

RCOOAg  → Ag
Let the equivalent weight of acid (RCOOH) be E.
The equivalent weight of RCOOAg = E – 1 + 108 = (E +107)
Equivalent weight of Ag =108; Weight of RCOOAg = W gm
Weight of Ag = w gm; 1 equivalent of RCOOAg = 1 equivalent of Ag

W w  W × 108 
= ==
∴ E  − 107 
E + 107 108  w 
Molecular mass of an acid = E × n (basicity of an acid)

(b) Chloroplatinate or platinichloride method for organic bases: Organic bases form salts with HCl and
the chloride give double salts with platinum chloride. Organic bases react with dibasic chloroplatinic acid
(H2PtCl6 ) and form double salts and are called chloroplatinates or platinichlorides. These salts leave a residue
of metallic platinum on ignition.
Equivalent weight of organic base = B
Mass or weight of chloroplatinic salt = W gm
Mass or weight of platinum = w gm
Equivalent weight of platinum = 195

2B + H2PtCl6 → B2H2PtCl6  → Pt

Equivalent weight of B2H2PtCl6 = (2B + 2 + 195 + 35.5 × 6) = 2B + 410
Chem i str y | 27.25

W w
1 equivalent of B2H2PtCl6 = 1 equivalent of Pt = =
2B + 410 195
 W 195 
∴ B=  ×  − 205
w 2 
Molecular mass of the organic base = Equivalent mass × Acidity = B × n.

(c) Victor Meyer’s method: This method is used for volatile organic compounds. A known mass of compound
(W gm) is vapourised in an instrument called Victor Meyer tube. The vapours so obtained displace an equal
amount of air into a graduated tube. The volume of vapours is measured and reduced to STP.
Let the volume of vapours at STP be V ml.
22400 ml of vapours are obtained from 1 mol of the compound.
V ml
V ml of vapours are obtained from mol of compound.
22400 ml

Mass of the compound (W gm) W


Mole = =
Molecular mass of the compound (Mw) Mw

W V W × 22400 Mass of compound × 22400


∴ = ∴ Mw = =
Mw 22400 V Volume of vapours at STP

Illustration 10: 0.984 gm of the chloroplatinate of a diacid base gave 0.39 gm of platinum. Calculate the molecular
mass of the base.

Sol: Mass of platinichloride taken = 0.984 gm; Mass of platinum left = 0.39 gm
To calculate the equivalent mass of base.
Let the equivalent mass of the base be B.
∴ Molecular mass of the platininchloride (B2H2 P t Cl6 ) = 2B + 410.

Molecular mass of chloroplatinichloride Mass of platinichloride taken


Now, =
Atomic mass of platinum Mass of platinum left

2B + 410 0.984 1  0.984 


or = or B =  × 195 − 410  =
41
195 0.39 2  0.39 
Thus, the equivalent mass of the base be 41.
To calculate the molecular mass of the base,
Acidity of base = 2.
Molecular mass of base = Equivalent mass of base × acidity of base = 41 × 2 = 82
Thus, the molecular mass of base is 82.

Illustration 11: On analysis, 0.2 gm of a monobasic acid gave 0.505 gm of CO2 and 0.0864 gm of H2O . 0.305 gm of
this acid required 25 ml of N/10 NaOH for complete neutralization. Find the molecular formula of this acid.

Sol: (a) Calculation of percentage composition

12 Mass of CO2 produced 12 0.505


(i) Percentage of carbon = × × 100 = × × 100 =
68.86%
44 Mass of subs tance taken 44 0.2

2 Mass of H2O produced 2 0.0864


(ii) Percentage of Hydrogen = × × 100 = × × 100 =
4.8%
18 Mass of subs tance taken 18 0.2
2 7 . 2 6 | Practical Organic Chemistr y

(iii) Percentage of oxygen = 100 – (Percentage of C + Percentage of H) = 100 – (68.86 + 4.8) = 26.34%

(b) Calculation of empirical formula:

Element Percentage Atomic mass Relative number of Simplest atomic ratio Simplest whole number
atoms atomic ratio
Carbon 68.86 12 7
68.86 5.74
= 5.74 = 3.48
12 1.65
Hydrogen 4.80 1 6
4.80 4.80
= 4.80 = 2.91
1 1.65
Oxygen 26.34 16 2
26.34 1.65
= 1.65 = 1.00
16 1.65

Hence, the empirical formula of the compound is C7H6 O2 .

(c) Calculation of molecular mass:


1
mEq. Of NaOH = 25 × = 2.5 mEq. Of acid = 2.5
10
Equivalent of acid = 2.5 × 10−3
Weight 0.305
Equivalent of acid = Ew = = = 122 gm of acid
Equivalent of acid 2.5 × 10 −3

Molecular weight of acid = Ew × n = 122 × 1 = 122

(d) Calculation of molecular formula:


Empirical formula (C7H6 O2 ) mass of the compound = (7 × 12 + 6 × 1 + 16 × 2) = 122
Molecular mass 122
∴ n= = = 1
Empirical formula mass 122

Thus, molecular formula of the compound = n × (Empirical formula) = 1 × ( C7H6 O2 ) = C7H6 O2

POINTS TO REMEMBER

•• Distillation process is used for the purification of liquids which boil without decomposition and contain non-
volatile impurities.
•• Fractional distillation process is used to separate a mixture of two or more miscible liquids which have boiling
points close to each other.
•• Vacuum distillation is used for liquids which decompose at a temperature below their normal boiling points.
•• Steam distillation is used for separation and purification of liquid which is appreciably volatile in steam, from
non-volatile components of a mixture.
•• Differential extraction method is used to separate a given organic compound present in aqueous solution by
shaking with a suitable organic solvent in which the compound is more soluble than water.
•• Chromatography is based on the difference in rates at which the components of a mixture are absorbed as a
suitable adsorbent.
•• Lassaigne’s test is used for detection of nitrogen, halogen and sulphur.
Chem i str y | 27.27

Solved Examples

JEE Main/Boards test:


O OCH O 3 OCH3 O O O OH O OH O
Example 1: Ethylamine and diethylamine cannot be (A) (A) (A) (B) (B)
(B) (C) (C) (D) (D)
distinguished by OCH3 OCH3 O
O 3 OCH3 O O
O OCH O OHO OH O O
(A) Hinsberg test
(A)
(B)
(A)Carbylamine
(B) test
(B) (C)
(C) (C) (D)
(D) (D)
(C) Iodoform test (D) Both (a) and (b) OCH3 OCH3 OH OH

Sol: Only primary amine gives carbylamine test and Sol: Hemiacetal due to presence of hydrogen atom at
hinsberg reagent gives different product with primary chiral carbon gives positive test with Tollen’s reagent.
and secondary amine. A primary amine will form (C) Hemiacetals give positive Tollen’s test.
a soluble sulfonamide salt which precipitates after
addition of diluted hydrochloric acid. A secondary amine Example 5: Give test to differentiate between
in the same reaction will directly form an insoluble 1,1-dichloroethane and 1,2-dichloroethane:
sulfonamide. Thus these tests can be used. (C) 1° and
2° amines can be distinguished by Hinsberg test and (A) 2,4-DNP then aq. KOH
carbylamine test. (B) Aq. KOH then 2,4- DNP
(C) NaHSO3
Example 2: Which of the following give positive Tollen’s
reagent test. (D) Lucas reagent
CO2H
CO2H CO
CO2H
2H
CO
CO2H2H
Sol: (B)(D)
Gem dihalide forms aldehyde or ketone which
(A)
(A) (CH
(A) (CH)
2 22)2 (B)(B)CH
(B) CH (C(C
)) (D)HCO
HCO 2H
2H
gives test of 2,4-DNP
22

CO2H
CO2H CO
CO2H
2H CO
CO2H2H

CO2H CO2H CO2H CO2H Example 6:


) CH2 (C(C)
) (C) (D) H HCO2H
(D) HCO2(D) OH CO2H NH2
CO2H CO2H CO2H CO2H

NO2 NO2 NO2


Sol: Formic acid will get oxidized to carbon dioxide and (m-nitrophenol) (m-nitro benzoic acid) (m-nitro aniline) (m-d
water, this is due to the carbonyl-containing carbon in
OH CO2H NH2 NO2
aldehydes having an attached hydrogen. (D) Formic
acid is the only acid giving tollen’s reagent test.
NO NO NO NO
Example 3: Sodium nitroprusside when 2
added to an 2 2 2
(m-nitrophenol) (m-nitro benzoic acid) (m-nitro aniline) (m-dinitro benzene)
alkaline solution of sulphide ions produces a colouration
which is:
Above compounds can be differentiated by which of the
(A) Prussian blue (B) Purple following chemical test used in same order?
(C) Blood red (D) Turnbull Blue
(A) NaOH, NaHCO3 , HCl (B) HCl, NaOH, NaHCO3
Sol: (C) Na + N + C + S → NaCNS (C) NaHCO3 , NaOH, HCl (D) NaOH, HCl, NaHCO3
(Sodium thiocyanate)
Sol: NaHCO3 separates carboxylic acid, NaOH separates
3CNS − + Fe+3 → Fe(CNS)3 phenol. HCl further separates amine.
Thiocyanate ion Ferric thiocyanate
(Blood −red colour)
Example 7: Which of the following reagent is used for
Example 4: Which of the following give positive Tollen’s the separation of acetaldehyde from acetophenone?
2 7 . 2 8 | Practical Organic Chemistr y

(A) NH2OH (B) NaOI (iii) (A) Acid with NaHCO3 gives brisk effervescence due
to formation of Carbon dioxide
(C) Tollen’s reagent (D) C6H5NHNH2
(iv) (B) Ether on hydrolysis give alcohol.
Sol: ketone and aldehydes can be distinguished by
Tollen’s reagent as Ketones do not give Tollen’s reagent
test, while aldehydes do not give. Example 2:
(C) Ketones do not give Tollen’s reagent test, while
O
aldehydes do not give. 14
NaHCO3 Na
Ph – C – OH (A) gas ; Ph – OH (B) gas

JEE Advanced/Boards Sum of molecular mass of gas (A + B = ?)


Sol: Molecular mass of gas is 48; The gas formed in first
Example 1: Given are the isomers of C8H8O2. reaction is C14O2 and from other reaction is H2.

O Example 3: Statement-I: Aniline on reaction with
O
C O Et CHO CO₂H O H C CH₃2/HCl at CH₂
NaNO 0°C followed
CHO by coupling with β-naphthol
gives a dark blue coloured precipitate. and
Statement-II: The colour of the compound formed in
the reaction of aniline with NaNO2/HCl at 0°C followed
OCH₃ CH₃ O CH CH₂ by coupling
O H with
OH β-naphthol is due to the extended
(A) (B) (C) (D) conjugation. (F)
(E)
(A) Statement-I is true, statement-II is true; statement-II
O is correct explanation for statement-I

O H C CH₃ CH₂ CHO (B) Statement-I is true, statement-II is true; statement-II


is not a correct explanation for statement-I
(C) Statement-I is true, statement-II is false
(D) Statement-I is false, statement-II is true
O CH CH₂ O H OH
D) (E) (F) Sol: (D) C6H5N2Cl gives scarlet red coloured dye with
β-naphthol.
(i) Which isomer gives positive iodoform test?
(A) a (B) b (C) d (D) e Example 4: Propyne and propene can be distinguished
by
(ii) Which isomer gives +ve Tollen’s test, also reacts with
FeCl3? (A) Conc. H2SO 4 (B) Br2 in CCl4

(A) b (B) f (C) c (D) d (C) Dil. KMnO 4 (D) AgNO3 in ammonia

(iii) Which isomer reacts with NaHCO3? Sol: Propyne being terminal alkyne forms silver ppt.
(A) c (B) b (C) e (D) f with AgNO3 in ammonia.

(iv) 
Which isomer on hydrolysis gives 1, 4-di RC ≡ C − H + Ag(NH3 )2+ (in NH4 OH) →
hydroxybenzene?
(A) a (B) d (C) e (D) f RC ≡ C − Ag (insoluble) + NH3 + NH+4

Sol: (i) (D) Iodoform test is given by compounds having (D) Propyne being terminal alkyne forms silver ppt. with
free methyl group. AgNO3 in ammonia.
(ii) (B) Tollens test is given by aldehyde as Aldehydes are
readily oxidized to carboxylic acids Example 5: 2,4-pentadione is treated with excess of KCN
in acetic acid solution and finally hydrolysed with dilute
Chem i str y | 27.29

H2SO4 giving products only with molecular formula Example 6: A hydrocarbon (A) of molecular weight 54
C7H12O6 (dicarboylic acids). The correct statement reacts with an excess of Br2 in CCl4 to give a compound
regarding product(s) of this reaction is/are (B) whose molecular weight is 593% more than that of
(A). However, on catalytic hydrogenation with excess of
(A) Product mixture contains four stereoisomers which
hydrogen, (A) forms (C) whose molecular weight is only
on fractional crystallization gives two fractions.
7.4% more than that of (A). (A) reacts with CH3CH2Br in
(B) Product mixture contains three stereoisomers which the presence of NaNH2 to give another hydrocarbon
on fractional crystallization gives two fractions. (D) which on ozonolysis yields diketone (E). (E) on
(C) Product of either fraction obtained after oxidation gives propionic acid. Give the structures of (A)
crystallization, on strong heating lactonizes to give the to (E) with reason.
following compound: Sol: To determine the molecular weights of compounds
CH3 (B) and (C).
O (i)The molecular weight of a compound (A) is 54 while
that of compound (B), which gives on treatment with
O an excess of Br2 in CCl4 , is 593% more than that of (A).
O
(100 + 593)
∴ Molecular weight of (B) = × 54 = 374.22
O CH3 100
Thus, the increase in weight due to addition of Br atoms
(D) Product of only one fraction obtained after
is 374.22 – 54.0 = 320.22
crystallization, on strong heating lactonizes to give
following compound: Since, the atomic weight of Br is 80, the number of Br
atoms added = 320.22/80 = 4
CH3
As such the hydrocarbon (A) must be an alkyne.
O
(ii) Further, since the molecular weight of compound (C),
O which hydrocarbon (A) gives on catalytic hydrogenation,
O is only 7.4% more than that of (A), the molecular weight
of (C) is
O CH3
(100 + 7.4) × 54 = 57.994 = 58 (approx.)
Sol: (B,D) The reaction with KCN in acetic acid solution 100
and hydrolysis with dilute H2SO 4 , gives a product
Thus, the increase in weight due to addition of H atoms
with two chiral centers giving three stereoisomers.
= 58 – 54 = 4
Nucleophilic addition occur at sp2 carbons, both front
and back side are equally probable. Since, product is Since, the atomic weight H is 1, the number of H
symmetric, one meso product and a pair of enantiomers. atoms added during catalytic hydrogenation is 4/1 = 4.
Enantiomers having same solubility form same fraction Therefore, hydrocarbon (A) must be an alkyne.
after fractional crystallization. To determine the structures of the hydrocarbons (A), (B),
CH3 CH3 (C) and (D).
OH CN HO COOH
H+ Heat (i) The two possible structures of hydrocarbon (A) i.e. an
CH2 CH2
H2O
does not lactonise
alkyne with molecular weight 54 (C4H6) are: CH3CH2 ‒
OH CN HO COOH
C ≡ CH (But-1-yne) (I) ; CH3 ‒ C ≡ CH3 (But-2-yne) (II)
CH3 CH3
meso (ii) Since, (A) reacts with CH3CH2Br in the presence of
NaNH2 to give another hydrocarbon (D), (A) must be
CH3 a terminal alkyne i.e. but-1-yne and not but-2-yne.
CH3 CH3
OH CN HO COOH
O
(iii) If hydrocarbon (A) is but-1-yne, then the structure of
H+ Heat
CH2 CH2 O the compounds (B), (C) and (D) may be worked out
CN OH H2O HOOC OH
O
as follows:
CH3 CH3 O CH3
Pair enantiomers
(lactone)
2 7 . 3 0 | Practical Organic Chemistr y

Br Br
Br2/CCl4
C2H5 C C H(1,1,2,2-Tetrabrom-butane) (B)
CH3 CH2 C  CH
But-1-yne(A) Br Br
(Mol.wt. = 54)
(Mol.wt. 374.22, i.e. 593% more than that of A))
NaNH2
2H2/Catalyst
(-NH3) CH3 CH2CH2CH2CH3
 CH3 CH2Br Butane(C)
CH3 CH2 C  C Na
-NaBr
Sodium but-1-ynide
CH3 CH2C  CCH2CH3
Hex-3-yne (D)

To determine the structure of the diketone of (E).


Since the hydrocarbon (D) i.e. hex-3-yne on ozonolysis gives a diketone (E) which on further oxidation gives
propionic acid, the diketone (E) must be hexane-3,4-dione as explained below:
O3
CH3 CH2 C  C CH2 CH3
Ozonolysis
Hex-3-yne
(D)
CH3 CH2 C C CH2CH3

O O
(E) Hexane-3,4-dione

Oxidation
2 CH3 CH2 C OH
O
Propionic acid
Thus,
(A) = But-1-yne, (B) = 1,1,2,2 -Tetrabromobutane, (C) = Butane (D) = Hex-3-yne, and
(E) = Hexane-3, 4-dione

JEE Main/Boards

Exercise 1 Q.3 0.12 gm of an organic compound containing


phosphorous gave 0.22 gm of Mg2P2O7 by the usual
analysis. Calculate the percentage of phosphorous in
Q.1 Compounds (P) and (Q) can be differentiated by:
the compound.

NH3Cl- NO2
Q.4 An organic compound containing sulphur is
and estimated by Carius method in which fuming HNO3 is
used to convert S into?
(P) (Q)
Q.5 In organic layer test, CS2 or CCl4 is added to
Lassaigne’s extract and then Cl2 water or KMnO4 is
Q.2 0.002 gm of an organic compound was treated
added. This test is used to distinguish between?
according to Kjeldahl’s method.
0.2 × 10‒4 mol of H2SO4 was required to neutralize NH3.
Calculate the percentage of N2. Q.6 A Dumas bulb full of air weighs 22.567 gm at 20°C
and 755 mm pressure. Full of vapours of a substance
Chem i str y | 27.31

at 120°C and the same pressure, it weighs 22.8617 Q.18 A ample of a 0.50 g on organic compound was
gm. The capacity of the bulb is 200 ml. Find out the heated according to Kjerldahl’s method. The ammonia
molecular mass of the substance. evolved by absorbed in 50 mL of 0.5 M H2SO4.The
residual acid required 60 mL of 0.5 M solution of NaOH
[Density of air = 0.00129 gm/ml
for neutralisation. Find the percentage composition of
nitrogen in the compound.
Q.7 Percentage composition of an organic compound
is as follows:
= C 10.06,H
= 0.84,Cl
= 89.10
Q.19 0.15 g of an organic compound gave 0.12 g AgBr
What is its molecular formula if the vapour density is by carious method. Find the percentage of bromine in
60.0? the compound.

Q.8 The empirical formula of a compound is CH2. One Q.20 0.395 g of an organic compound by Carius method
mole of the compound has mass of 42 g, its molecular for the estimation of sulphur gave 0.582 g of BaSO4.
formula is? Calculate the percentage of sulphur in the compound.

Q.9 How will you separate a mixture of o-nitro-phenol


and p-nitrophenol ? Exercise 2
Single Correct Choice Type
Q.10 How will you separate benzoic acid from a mixture
of naphthalene and benzoic acid? Q.1 In Kjeldahl’s method, nitrogen present is estimated
as:
Q.11 Suggest methods for the separation of the (A) N2 (B) NH3
following mixtures:
(C) NO2 (D) None of these
(A) A mixture of liquid A (b.p. 365 K) and liquid B
(b.p.355 K)
Q.2 A compound which doesn’t give a positive test in
(B) A mixture of liquid C (b.p. 348 K) and liquid D Lassaigne’s test for nitrogen is:
(b.p. 478 K).
(A) Urea (B) Hydrazine

Q.12Will a precipitate of AgCl be formed on treating (C) Azobenzene (D) Phenyl hydrazine
CHCl3 with AgNO3? Explain :
Q.3 Compounds (X) and (Y) can be differentiated by:
Q.13 Name two compounds which are purified by O
sublimation. O O
and
O O
Q.14 What is the basic principle of chromatography? (X) (Y)

Q.15 What conclusions would you draw if the Lasagne’s (A) H3O + , Na (B) H3O + , then Fehling test
extract gives a blood red colouration with FeCl3?
(C) H3O + , then Na (D) Both (B) and (C)

Q.16 0.257 g of an organic substance was heated with


Q.4 In Kjeldahl’s method of estimation of nitrogen
conc. H2SO4 and then distilled with excess of strong
K 2SO 4 acts as:
alkali. The ammonia gas evolved was absorbed in
50 mL of M/10 which required 23.2 mL of M/10 NaHO (A) An oxidant (B) Catalytic reagent
for neutralisation at the nitrogen in the compound. (C) Hydrolyzing agent (D) Boiling point elevator

Q.17 0.85 g of an organic substance was Kjeldahlised Q.5 The Prussian blue colour obtained during the test
and the ammonia obtained was passed into 100 mL of of nitrogen by Lassaigne’s test is due to formation of :
M/10 H2SO 4 . The excess acid required 154 mL of M/10
NaOH for neutralisation. Calculate .the percentage of (A) Fe4 [Fe(CN)6 ]3 (B) Na3 [Fe(CN)6 ]
nitrogen in the compound. (C) Fe[Fe(CN)6 ] (D) Na4 [Fe(CN)5 NOS]
2 7 . 3 2 | Practical Organic Chemistr y

Q.6 A compound of carbon, hydrogen, and nitrogen S; the appearance of blood red colour takes place in
contains the elements in the ratio 18 : 2 : 7. Calculate Lassaigne-test due to formation of
its empirical formula and molecular formula if its
(A) NaCNS (B) Fe(CNS)3 (C) NaSCN (D) Fe(CN)2
molecular mass is 108.

(A) C3H4N, C6H8N2 (B) CHN, C5H6N3 Q. 13 Layer test is used to detect the presence of
(C) C5H6N3 , C5H6N3 (D) C2H4N, C 4H8N2 (A) Chlorine (B) Phosphorus
(C) Bromine (D) Fluorine
Q.7 In Dumas’s method for the estimation of nitrogen
in an organic compound, nitrogen is estimated in the Q.14 Carius method is used for the estimation of
form of:
(A) Halogens (B) Sulphur
(A) Gaseous nitrogen (B) Sodium cyanide
(C) Phosphorus (D) All of these
(C) Ammonium sulphate (D) Gaseous ammonia

Q.15 During estimation of Duma’s method 2.36 g of an


Q.8 In Leibig’s combustion method for estimation of organic compound gave 448 mL of N2 (g) at NTP. The %
C and H, if the compound contains N, which of the of N in the compound is
following is kept near the exit of the combustion tube?
(A) 23.7% (B) 40% (C) 47.4% (D) 12%
(A) Silver wire (B) PbCrO4
(C) Both A and B (D) Cu gauge Q.16 Estimation of C and H in an organic compound is
done by
Q.9 Which of the following statements are wrong? (A) Duma’s method (B) Leibig’s method
(multiple choice)
(C) Lassaigne method (D) Carius method
(A) The gas displaced in Victor Meyer’s method is air.
(B) The simplest formula that shows the ratio of the Q.17 The principle involved in paper chromatography is
constituent atoms present in the molecule is called
molecular formula. (A) Partition (B) Sublimation

(C) Estimation of oxygen in an organic compound is (C) Adsorption (D) Solubility


also done by Aluise’s method.
Q.18 Naphthalene and benzoic acid can be separated
(D) An organic monobasic base B on reaction with
H2PtCl6 forms an insoluble compound B2H2PtCl6 . from each other best by the method of
(A) Crystallisation (B) Sublimation
Q. 10 Which of the following statement(s) is/are correct? (C) Distillation (D) Chromatography
2−
(A) Nitroprusside ion is [Fe(CN)5 NO] .
2− Q.19 Which of the following compound will not give
(B) Nitroprusside ion is [Fe(CN)5 NOS]
Lassaigne’s test for nitrogen?
(C) Prussian blue and turn bull blue are respectively
Fe4 [Fe(CN)5 ]3 and Fe3 [Fe(CN)6 ]2 . (A) Azobenzene (B) Hydrazine
(C) Phenylhydrazine (D) Urea
(D) Prussian blue and turn bull blue are respectively
Fe3 [Fe(CN)6 ]2 and Fe4 [Fe(CN)6 ]3 .
Q.20 What happens when AgNO3 solution is added to
bromo methane?
Q.11. Sulphur cannot be detected by
(A) Gives yellow ppt
(A) Beilstein test (B) Lassaigne test
(B) Will gives white ppt
(C) Lead acetate test (D) Sodium nitroprusside test
(C) Gives yellow ppt only in presence of KOH
Q.12 If an organic compound contains both N and (D) Gives white ppt only in presence of KOH
Chem i str y | 27.33

Previous Years Questions (B) Isopropyl alcohol


(C) 3–Methyl – 2– butanone
Q.1 Which of the following compounds can be detected
(D) Isobutyl alcohol
by Molisch’stest? (2012)
(A) Nitro compounds (B) Sugars
Q.8 29.5 mg of an organic compound containing
(C) Amines (D) Primary alcohols nitrogen was digested according to Kjeldahl’s method
and the evolved ammonia was absorbed in 20 mL of
Q.2 Silver Mirror test is given by which (2011) 0.1 M HCl solution. The excess of the acid required 15
mL of 0.1 M NaOH solution for complete neutralization.
one of the following compounds ? The percentage of nitrogen in the compound is(2010)
(A) Acetone (B) Formaldehyde (A) 59.0 (B) 47.4
(C) Benzophenone (D) Acetaldehyde (C) 23.7 (D) 29.5

Q.3 Which of the following reagents may be used to Q.9 For the estimation of nitrogen, 1.4 g of an organic
distinguish between phenol and benzoic acid? (2011) compound was digested by Kjeldahl method and
(A) Tollen’s reagent (B) Molisch reagent M
the evolved ammonia was absorbed in 60 mL of
10
(C) Neutral FeCl3 (D) Aqueous NaOH sulphuric acid. The unreacted acid required 20 mL of
M
sodium hydroxide for complete neutralization. The
Q.4 Biuret test is not given by  (2010) 10
(A) Carbohydrates (B) Polypeptides percentage of nitrogen in the compound is  (2014)

(C) Urea (D) Proteins (A) 6% (B) 10% (C) 3% (D) 5%

Q.5 The ammonia evolved from the treatment of Q.10 In Carius method of estimation of halogens, 250
0.30 g of an organic compound for the estimation of mg of an organic compound gave 141 mg of AgBr. The
nitrogen was passed in 100 mL of 0.1 M sulphuric acid. percentage of bromine in the compound is:
The excess of acid required 20 mL of 0.5 M sodium (at. mass Ag = 108 ; Br = 80) (2015)
hydroxide solution for complete neutralization. The
organic compound is  (2004) (A) 24 (B) 36 (C) 48 (D) 60

(A) Acetamide (B) Thiourea


Q.11 The hottest region of Bunsen flame shown in the
(C) Urea (D) Benzamide figure below is:  (2016)
(A) Region 2 (B) Region 3
Q.6 Among the following the one that gives positive
iodoform test upon reaction with I2 and NaOH is: (2006) (C) Region 4 (D) Region 1

(A) CH3CH2CH(OH)CH2CH3 (B) C6H5CH2CH2OH


Q.12 The distillation technique most suited for separating
(C) CH3CH(CH3 ) − CH2OH (D) Ph − CH(OH)CH3 glycerol from spent-lye in the soap industry is: (2016)
(A) Fractional distillation
Q.7 Iodoform can be prepared from all except:
 (2012) (B) Steam distillation
(A) Ethyl methyl ketone (C) Distillation under reduced pressure
(D) Simple distillation
2 7 . 3 4 | Practical Organic Chemistr y

JEE Advanced/Boards

Exercise 1 Q.11 Why is it necessary to use acetic acid and not


sulphuric acid for acidification of sodium extract for
Q.1 0.92 gm of an organic compound containing testing sulphur by lead acetate test?
carbon, hydrogen, and oxygen was analysed by
Leibig’s combustion method. The increase in the Q.12 The R f value of A and B in a mixture determined by
mass of U-tube and the potash bulbs at the end of TLC in a solvent mixture are 0.65 and 0.42 respectively.
the operation was found to be 1.08 gm and 1.76 gm, If the mixture is separated by column chromatography
respectively. Determine the percentage composition of using the same solvent mixture on a mobile phase,
the compound. which of the two components A or B elute first?

Q.2 In an estimation of sulphur by the Carius method Q.13 A mixture contains 71 per cent of calcium sulphate
0.2175 g of the substance gave 0.5825 g of Ba2SO4. What and 29 per cent of camphor. Name a suitable technique
is the percentage of sulphur in the substance? of separation of the components of the mixture?

Q.3 An organic compound having molecular mass 60 is Q.14 Suggest a suitable technique of separating
found to contain C = 20% , H = 6.67% and N = 46.67% naphthalene from kerosene present in a mixture.
while rest is oxygen .On heating it given NH3 along with
a solid residue. The solid residue give violet colour with Q.15 Write the molecular formula of iron (III)
alkaline copper sulphate solution. The compound is: hexacyanoferrate (II).

Q.4 An hydrocarbon contains 10.5 g carbon and 1 g Q.16 0.2475 g of an organic compound gave on
hydrogen. Its 2.4 g has 1 L volume at 1 atm and 127 C. combustion 0.4950 g of carbon dioxide and 0.2025
Determine the molecular formula of the Hydrocarbon. g of water. Calculate the percentage of carbon and
hydrogen in it.
Q.5 0.1877 g of an organic compound when analysed
by the Duma’s method yields 32 mL of most nitrogen Q.17 An organic compound contains 69% carbon and
measured at 287 k and 758 mm mercury pressure. What 4.8% hydrogen, the remainder being oxygen. Calculate
is percentage of nitrogen in the organic compound? the masses of carbon dioxide and water produced
(aqueous tension at 287K = 12mm ) when 0.20 g of this substance is subjected to complete
combustion.
Q.6 If 0.228 g of silver salt of dibasic acid gave a
residue of 0.162 g of silver on ignition then what is the Q.18 On complete combustion, 0.246 g of an organic
molecular weight of the acid? compound gave 0.198 g of carbon dioxide and 0.1014
g of water calculate the percentage composition of
Q.7 0.5 g of hydrocarbon gave 0.9 water on combustion. carbon and hydrogen in the compound.
Calculate the percentage of carbon in hydrocarbon.
Q.19 In a Duma’s nitrogen estimation 0.3 g of an
Q.8 In a compound, C,H and N are present in 9 : 1 : 3.5 organic compound gave 50 mL of nitrogen collected at
by weight. If molecular weight of the compound is 300 K and 715 mm pressure. Calculate the percentage
108, then determine the molecular formula of the of nitrogen in the compound.
compound.
(vapour pressure of water at 300 K is 15 mm).
Q.9 Name a method suitable for separating the mixture
of iodine and sodium chloride. Q.20 During nitrogen estimation of an organic compound
by Kjeldahl’s method, the ammonia evolved by 0.5 g of
Q.10 Why is nitric acid added to sodium extract before the compound neutralised 10 mL of 1 MH2SO4. Calculate
adding silver nitrate solution for testing halogens? the percentage of nitrogen in the compound.
Chem i str y | 27.35

Exercise 2 Q.10 An organic compound containing C, H and N gave


the following results on analysis C = 40%, H = 13.33%,
Single Correct Choice Type N = 46.67%. Its empirical formula would be
(A) C2H7N2 (B) CH5N (C) CH4N (D) C2H7N
Q.1 Prussian is formed when
(A) Ferrous sulphate reacts with FeCl3 Q.11 A dibasic organic acid gave the following results:
(B) Ferric sulphate reacts with Na4[Fe(CN)6] C = 34.62%, H = 3.84%. Also, 0.1075 gm of this
acid consumers 20 ml of 0.1 N NaOH for complete
(C) Ferrous ammonium sulphate reacts with FeCl3
neutralisation the molecular formula of the acid.
(D) Ammonium sulphate reacts with FeCl3
(A) C 4H8 O2 (B) C 2H6 O (C) C 3H4 O 4 (D) C8H9O3
Q.2 Lassaigne’s test is used to detect
Match the Columns
(A) Nitrogen (B) Sulphur (C) Chlorine (D) All of these
Q. 12 Match the columns with compounds in column I
Q.3 Kjeldahl’s method is used in the estimation of and the reagent to separate them in column II
(A) Nitrogen (B) Halogens (C) Sulphur (D) Oxygen
Column I Column II
Q.4 In sodium fusion test of organic compounds, the Compounds Reagent for separation
nitrogen of the organic compound is converted into
(A) 1° and 2° amines (p) NaHSO3
(A) Sodamide (B) Sodium cyanide
(B) Ethanal and ethanol (q) H
 insberg reagent
(C) Sodium nitrite (D) Sodium nitrate (PhSO2Cl) or

Q.5 A is a lighter phenol and B is an aromatic carboxylic Me SO2Cl


acid. Separation of a mixture of A and B can be carried
out easily by using a solution of (C) (C2H5)2NH and butanol (r) Dil. NaOH and distillation
(A) Sodium hydroxide (B) Sodium sulphate (D) (C2H5)2C = 0 and (s) Dil. H2SO4 and steam
(C) Calcium chloride (D) Sodium bicarbonate CH3COOH distillation

Q.6 The most suitable method for separation of 1:1 Q.13 Match the columns with method of sepretion in
mixture of ortho and para nitrophenols is column I with the compounds in column II
(A) Sublimation (B) Chromatography
Column I Column II
(C) Crystallisation (D) Steam distillation
Methods of separation(I) Compound(II)

Q.7 Lassaigne’s test for the detection of nitrogen fails in: (A) S
 eparated by treatment with (p) Toluene and
dil. NaOH aniline
(A) NH2CONHNH2 .HCl (B) NH2NH2 .HCl
(B) E
 xtraction with dil. HCl, a (q) Toluene and
(C) NH2CONH2 (D) C6H5NHNH2 .HCl
compound passes into the phenol
aqueous layer in the form
Q.8 An organic compound contains C = 40% O = 53.34% of hydrochloride salt and
and H = 6.60%. The empirical formula of the compound is: recovered by neutralization.
(A) CH2O (B) CHO (C) CH4O2 (D) C2H2O  eparated by NaHCO3 solution,
(C) S (r) Diethyl ether and
a compound forms salt and is chlorobenzene
Q.9 Which of the following techniques is most suitable recovered after acidification.
for purification of cyclohexanone from a mixture
 eparated by conc. H2SO4,
(D) S (s) o-Cresol and
containing benzoic acid, isoamyl alcohol, cyclohexane
which dissolves a compound benzoic acid
and cyclohexanone?
and recovered from solution
(A) Crystallisation (B) IR spectroscopy by dilution with H2O
(C) Sublimation (D) Evaporation
2 7 . 3 6 | Practical Organic Chemistr y

Comprehension Type Q.20 Black precipitate in the detection of sulphur with


lead acetate and acetic acid is due to formation of:
Paragraph (14-23)
(A) Pb2S (B) PbS (C) PbS2 (D) PbSO 4
Qualitative analysis of organic compounds is performed
by Lassaigne’s test by fusion with sodium metal, by
which the covalent compounds are converted to ionic Q.21 Blood-red colour in the detection of both N and S
compounds. Extra elements like N, S, P and halogens in the Lassaigne’s extract is due to formation of :
are detected by their usual tests. (A) Fe(CNS)2 (B) Fe(CNS)3
(C) K 4 [Fe(CN)6 ] (D) K 3 [Fe(CN)6 ]
Q.14 Which of the following compounds will give
positive Lassaigne’s test for nitrogen?
Q.22 Yellow precipitate in the detection of phosphorous
(A) NH2OH (B) NH2NH2 when an organic compound is heated with Na2O2 and
then boiled with conc. HNO3 followed by the addition
(C) KCN (D) of ammonium molybdate is due to formation of:
N = N - Cl
(A) (NH4 )3 . PO 4 . 12MoO3 (B) (NH4 )3 . PO 4 . 6MoO3

Q.15 Which of the following will give blood red colour (C) (NH4 )3 . PO 4 . 12MoO2 (D) (NH4 )3 . PO 4 . 6MoO2
in Lassaigne’s test for nitrogen ?
(A) PhNH2 Q.23 Beilstein test is given by:

(B) PhNO2 (A) RX (X = Cl, Br, I) (B) Urea


(C) Thiourea (D) All of these
(C) O2N SO3H

(D) PhSO3H Assertion Reasoning Type


(A) If both assertion and reason are correct and reason
Q.16 Prussian blue colour in the detection of nitrogen is correct explanation of assertion.
in Lassaigne’s test is due to formation of: (B) If both assertion and reason are correct and reason
(A) Fe3 [Fe(CN)6 ] (B) Fe4 [Fe(CN)6 ]3 is not a correct explanation of assertion.

(C) Fe3 [Fe(CN)6 ]2 (D) Fe[Fe(CN)6 ] (C) If assertion is correct and reason is incorrect.
(D) If assertion is incorrect and reason is correct.
Q.17 Violet colour in the detection of sulphur with (E) If both assertion and reason are incorrect.
sodium nitroprusside is due to formation of
(A) Na3 [Fe(CN)5 NOS] (B) Na4 [Fe(CN)5 NOS] Q.24 Assertion: In organic layer test, Cl2 water is
(C) Na4 [Fe(CN)4 S] (D) Na2 [Fe(CN)5 S] added to sodium extract, which oxidises Br − and I −
ions to Br2 and I2 respectively.
Q.18 An organic compound containing N,S, and O as Reason: Reduction potential of Cl2 is greater than that
extra elements is fused with metallic sodium and then of Br2 and I2 .
extracted with distilled water. Which species is not
present in the Lassaigne’s extract? Q.25 Assertion: Hydroxylamine ( NH2OH ) contains N,
(A) NO3− (B) CN −
(C) CNS −
(D) S 2− and hence gives Lassaigne’s test.
Reason: Hydroxylamine does not contain C, so with Na
Q.19 The Lassaigne’s extract is boiled with dil.HNO3 metal, CN− ion is not formed.
before testing for halogens because
(A) AgCN is soluble in HNO3 Q.26 Assertion: Benzene (b.p. 353K) and methanol
(b.p. 338K) are separated by simple distillation.
(B) Silver halides are soluble in HNO3
Reason: Fractional distillation is used to separate two
(C) NaCN and Na2S are decomposed by HNO3 . liquids from their mixture when their boiling points
(D) Ag2S is soluble in HNO3 . differ by 20° or so.
Chem i str y | 27.37

Previous Years Questions Column I Column II


 (q) Gives positive FeCl3
Q.1 The compound that does NOT liberate CO2 , on (B) HO
NH3 I
test.
treatment with aqueous sodium bicarbonate solution, COOH
is  (2013)
 (r) Gives white
(A) Benzoic acid (B) Benzenesulphonic acid (C) HO NH3 Cl
precipitate with AgNO3
(C) Salicylic acid (D) Carbolic acid (Phenol)
 (s) Reacts with
(D) O N NHNH3 Br
Q.2 Identify the binary mixture(s) that can be separated 2
aldehydes to form
into individual compounds, by differential extraction, NO2 the corresponding
as shown in the given scheme.  (2012) hydrazone derivative.
NaOH (aq)
Binary mixture containing
compound 1 and
compound 2 NaHCO2 (aq) Q.5 A hydrocarbon contains 10.5 gm of carbon for each
1 gm of hydrogen. The mass of 1 litre of hydrocarbon
vapours at 127°C and 1 atm is 2.8 gm. Find out the
Compound 1 + Compound 2
molecular formula of the hydrocarbon. (1980)

Compound 1 + Compound 2 Q.6 Match the compounds/ions in Column I with


their properties / reactions in Column II. Indicate your
answer by darkening the appropriate bubbles of the
(A) C6H5OH and C6H5COOH
4 × 4 matrix given in the ORS. (2007)
(B) C6H5COOH and C6H5CH2OH
Column I Column II
(C) C6H5CH2OH and C6H5OH
(A) C6H5CHO (p) Gives precipitate with 2,3
(D) C6H5CH2OH and C6H5CH2COOH dinitrophenylhydrazine

Q.3 Amongst the compounds given, the one that would (B) CH3C ≡ CH (q) Gives precipitate with AgNO3
form a brilliant coloured dye on treatment with NaNO2
(C) CN‒ (r) Is a nucleophile
in dil. HCl followed by addition to an alkaline solution
of β -naphthol is (2011) (D) I‒ (s) Is involved in cyanohydrin formation

N(CH
N(CH333))222 NHCH333
NHCH
NHCH
N(CH3)2 NHCH3 Q.7 Amongst the following, the total number of
(A)(A)
(A) (B)(B)
(B) compounds (2010)
(A) (B) H3C CH3 soluble in aqueous NaOH is: 
N COOH OCH2CH3 OH
H3C CH3
NH22
NH CH
CH22NH
CH NH2
NH CH2OH
NH2 CH22NH222 N COOH OCH2CH3 OH
(C)
(C)
(C) (D)(D)
(D)
(D)
(C) H C CH2OH
H33C
H3C
Q.4 Match the compounds in column I with their
characteristic test(s) / reaction(s)
H3C given
CH 3 in column II. NO2 OH OCH2CH3 COOH
 N (2008)
COOH OCH2CH3
NO2
OH
OH OCH2CHCH 2OH COOH
3
CH2OH
Column I Column II CH2OH

⊕ − (p) Sodium fusion N


(A) H2N − NH3 Cl extract of the H3C N CH3
compound gives H3C CH3
NO2 OH OCH2CH3 COOH
Prussian blue colour
with FeSO4. CH2OH

N
H3C CH3
2 7 . 3 8 | Practical Organic Chemistr y

PlancEssential Questions
JEE Main/Boards JEE Advanced/Boards
Exercise 1 Exercise 2
Q.3 Q.5 Q.9 Q.2 Q.3 Q.5

Exercise 2 Previous Years’ Questions


Q.1 Q.6 Q.2 Q.4

Previous Years’ Questions


Q.4 Q.6 Q.7

Answer Key

JEE Main/Boards

Exercise 1
Q.1 Ammonical AgNO3 and NaOH Q.2 28% Q.3 51.20% Q.4 SO −4

Q.5 Br‒ and I‒ Q.6 86.69 Q.7 CHCl3 Q.8 C3H6

Q.9 Steam distillation Q.10 Hot water

Q.11 (a) Fractional distillation (b) Simple distillation Q.12 CHCl3 is a covalent

Q.13 Camphor, naphthalene Q.14 Adsorption Q.15 Presence of N and S

Q.16 14.6% Q.17 18.4% Q.18 56% Q.19 34.04% Q.20 20.24%

Exercise 2

Single Correct Choice Type

Q.1 B Q.2 B Q.3 D Q.4 D Q.5 A Q.6 A


Q.7 A Q.8 D Q.9 B Q.10 A Q.11 B Q.12 D
Q.13 C Q.14 A Q.15 A Q.16 D Q.17 A Q.18 A
Q.19 B Q.20 C
Chem i str y | 27.39

Previous Years' Questions


Q.1 B Q.2 B,D Q.3 C Q.4 A Q.5 C Q.6 D

Q.7 D Q.8 C Q.9 B Q.10 A Q.11 A Q.12 C

JEE Advanced/Boards

Exercise 1
Q.1 % of C = 60.03%, % of H = 13.30%, % of O = 26.67% Q.2 36.78%

Q.3 NH2CONH2 Q.4 m = 79 Q.5 19.90%

Q.6 90 Q.7 80% Q.8 C6H8N2

Q.9 Sublimation Q.12 A will be eluted first Q.14 By differential extraction.

Q.15 Fe4[Fe(CH)6]3 Q.16 9.09 Q.17 0.506g; 0.0864g

Q.18 4.58% Q.19 17.49% Q.20 56%

Exercise 2
Single Correct Choice Type

Q.1 B Q.2 D Q.3 A Q.4 B Q.5 D Q.6 D

Q.7 B Q.8 A Q.9 B Q.10 C Q.11 C

Match the Columns

Q.12 A → q; B → p; C → s; D → r Q.13 A → q; B → p; C → s; D → r

Comprehension Type

Q.14 C Q.15 C Q.16 B Q.17 B Q.18 A Q.19 C

Q.20 B Q.21 B Q.21 A Q.22 D

Assertion Reasoning Type

Q.23 A Q.24 D Q.25 C

Previous Years' Questions


Q.1 D Q.2 B,D Q.3 C Q.4 A → r, s; B → p, q; C → p, q, r; D → p

Q.5 C7H8 Q.6 A → p, q, s; B → q; C → q, r, s; D → q, r

Q.7 5
2 7 . 4 0 | Practical Organic Chemistr y

Solutions

JEE Main/Boards Sol 3:

At.mass of P Mass of Mg2P2O7


Exercise 1 %P
= ×
Molar mass of Mg2P2O7 mass of compound
× 100

Sol 1: Only phenols show colour change. = 51.20%


NH3Cl Sol 4: An organic compound containing sulphur is
estimated by Carius method in which fuming HNO3 is
NaOH Regeneration of used to convert S into SO −4
aniline (yellow liquid)
ammonical
Sol 5: Br‒ and I‒ forms a separate layer.
Regeneration of
AgNO3 NaOH
Sol 6: Wt. of bulb = 22.567 gm; P = 755 mm
NO2
ammonical Wt. of bulb with substance = 22.8617 gm
Cl (No reaction)
AgNO3 Capacity of bulb = 200 ml.

NaOH Density of air = 0.0012.9 gm/ml


(No reaction)
Wt. of substance = 22.8617 – 22.567 = 0.2947 gm.
m = 86.69

(Normality)H × (a− b) × 0.014


2SO 4
Sol 2: %N × 100 =28%
Mass of sample

Sol 7:

b
Atom Atomic mass (a) Percentage (b) =x Ration
a

10.06
C 12 10.06 = 0.83 1
12

0.84
H 1 0.84 = 0.84 1
1

89.10
CI 35.5 89.10 = 2.50 3
35.5

Empirical formula = CHCl3


Empirical formula mass = 12 + 1 + 106.5 = 119.5 ≈ 120
Molecular mass = 2 × VD = 2 × 60 = 120
molar mass 60
n= = = 1
empirical formula mass 60
=
Molecular formula (=
CHCl3 )
1
CHCl3
Chem i str y | 27.41

Sol 8: Mass of 1 mol. of compound = Mol. Wt. of


compound =42 mol 1000 mL of 1 NH3 solution contains nitrogen = 14 g

Mol. wt M
∴ Molecular formula = empirical formula × NH3 solution contains nitrogen
26.8 ml of
EF wt. 10
14 × 26.8
42 = 10 × 1000
= CH2 × = C3H6
14
14 × 26.8 × 100
Percentage of nitrogen = = 14.6%
Sol 9: A mixture of o-nitro phenol and p-nitro phenol 10 × 1000 × 0.257
can be separated by steam distillation. O-nitrophenol
being less volatile distils over along with water while Q.17: Vol. of M/10 H2SO 4 taken =100 mL Let us first
p-nitrophenol being non-volatile in the flask. calculate the volume of excess M/10 H2SO 4 which was
neutralised by 154 mL of M/10 NaOH
Sol 10: Benzoic acid can be separated from naphthalene M 154 M
by treating the mixture with hot water. Benzoic acid 154 mL of NaOH = mL of H SO
10 2 10 2 4
dissolves but naphthalene remains insoluble. On
M
cooling pure benzoic acid crystallises out. ∴ Volume of H SO left unused = 77mL
10 2 4
M
Sol 11: (a) Fractional distillation because the boiling of Volume of H SO used for neutralisation of NH3
the two liquids differ by only 10 . 10 2 4
100 − 77 =
23 mL
(b) Simple distillation because the boiling points of the
two liquids differ much Now,
M M
23 mL of H2SO 4 = 2 × 23mL of NH3
Sol 12: The precipitate of AgCl will not be formed 10 10
because CHCl3 is a covalent compounds and does M
not ionize to give Cl− ions. Therefore, Ag+ ions (from = 46 mL of NH3
10
AgNO3 ) to from ppt. of AgCl
Now 1000 mL of 1 M NH3 contain nitrogen =14 g

Sol 13: Camphor, naphthalene. M 14 46 × 1


46 mL of NH3 contain nitrogen
= ×
10 1000 10
Sol 14: The method of chromatography is based on the
14 × 46 × 100
difference in the which the compounds of mixture are ∴ Percentage of nitrogen = = 18.4%
1000 × 10 × 0.35
adsorbed on a suitable absorbent.

Sol 18: Mass of organic compound taken = 0.50 g


Sol 15: It indicates that the compound contains both
N and S. During fusion, sodium thiocyanate is formed Vol. of 0.5 M H2SO 4 taken = 50 mL
which given blood red colouration. The residual acid required 60 mL of 0.5 M solution of
3NaSCN + FeCl3 → Fe(SCN)3 + 3NaCl NaOH for neutralisation
Blood red
60 mL of 0.5 M NaOH solution
M M 60
Sol 16: Volume of NaOH = 23.2mL.of HCl = mL of 0.5 M H2SO 4 solution
10 10 2
M = 30mL of 0.5 M H2SO 4 solution
Volume of HCl unused =23.2 mL
10
Volume of 0.5 M H2SO 4 used = 50 − 30 = 20mL
M Now, 20 mL of 0.5 M H2SO 4 = 2 × 20mL of 0.5 M NH3
∴ Volume of HCl required for neutralisation of
10 solution
NH3 = 50 ‒ 23.2 = 26.8mL Or 1000 mL 0.5 M NH3 contain nitrogen = 14
14 × 40 × 0.5
M 40 mL of 0.5 M NH3 contain =
26.8 mL of HCl = 26.8mL of M / 10NH3 1000
10
2 7 . 4 2 | Practical Organic Chemistr y

14 × 40 × 0.5
% of N
= × 100 = 56% 18 3 2 7
1000 × 0.50 = : =2: =1
12 2 1 7

Sol 19: Mass of AgBr formed = 0.12g


∴ C3H4N3
AgBr = Br
188 80 molecular weight 108
=n = = 2
e.f.w. 54
188 g of AgBr contain bromine = 80 g
∴ Molecular formula =
C6H8N4
80 × 0.12
0.12 g of AgBr contain =
bromine = 0.051
188
Sol 7: (A) In Dumas’s method for the estimation of
0.051
Percentage of bromine
= × 100 = 34.04% nitrogen in an organic compound, nitrogen is estimated
0.15 in the form of gaseous nitrogen

Sol 20: Mass of BaSO 4 = 0.582 g


Sol 8: (D) Refer text-Liebig’s Combustion Method
We know BaSO2 = S
233 = 32 Sol 9: (B) Formula showing simplest ratio is the
empirical formula.
233 g BaSO2 contain sulphur 32 g
32
0.582 of BaSO2 contain sulphur
= × 0.582 Sol 10: (A) Self-explanatory
233

Wt. of sulphur Sol 11: (B) Belistein test is a method of determining


Percentage
= of sulphur × 100
Wt. of xompound halogens in a given compound.

32 × 0.582
= =× 100 20.24% Sol 12: (D) If both nitrogen and sulphur are present in
233 × 0.395
an organic compound, sodium thiocyanate is formed
which then react with Fe3+ to form blood colour complex
Exercise 2 [Fe(SCN)]2+
Na + C + N
   +  S  →N
  aSC       
Sol 1: (B) Refer kjeldahl’s method. 2+
Fe3+  + 2Na → Fe ( SCN) 

Sol 2: (B) H2 N− NH2 : Does not form complex.  


Sol 13: (C) In Lassaigne’s extract when sodium iodide
O is treated with chlorine water the bromide oxidise to
O O Fehling test the corresponding halogens. This halogen dissolves in
identifies carbonly
compounds. carbon di sulphide. Bromine impart orange colour in
H carbon disulphide layer.
O O
(x) (y)
Sol 14: (A) To determination of halogens.
Sol 3: (D) Fehling test identifies carbonyl compounds.
Sol 15 :(A) Mass of the organic compound taken = w g
Sol 4: (D) Self-explanatory.
Volume of nitrogen collected = v g
1 mol of nitrogen = 28 gm = 22400ml
Sol 5: (A) Self-explanatory.
28v/22400 x 100/w
Sol 6: (A) Percentage of nitrogen in the compound
= 28 × 448/22400 × 100/ 2.36
C: H: N:
18 g : 2g : 7g:
= 23.72 %
Chem i str y | 27.43

Sol 16: (D) Carbon and hydrogen present in an organic was used in reaction with organic compound.
compound are estimated by ‘Liebig’s method’.
Therefore

Sol 17: (A) This is useful for separating complex Moles of base × acidity of base = 2 × mol of H2SO 4
mixtures of compounds having similar polarity. The Solving we get molecular mass = 60 and acidity = 2
mobile phase is developing solutions that can travel solution. Therefore urea is the answer.
up to the stationary phase carrying the sample along
with it. In paper chromatography, the stationary phase
Sol 6: (D) Compound consists of (CH3 − CH(OH)−)
is a uniform absorbent paper. The mobile phase is a
group which is easily oxidisable to give methyl ketone
suitable liquid solvent.
which gives iodoform reaction.

Sol 18: (A) Crystallisation.


Sol 7: (D) Iodoform is given by (1) methyl ketones
R − CO − CH3
Sol 19: (B) Hydrazine does not have C atoms, so
(2) Alcohols of the type R − CH(OH)CH3 where R can be
it does not form NaCN and hence does not give a
hydrogen also
positive Lassaigne’s test for nitrogen. In order to test
the presence of N in such compounds, during fusion
with Na, some charcoal or starch is added to supply C Sol 8: (C) Moles of HCl reacting with
atoms. Under these conditions, NaCN is obtained and Ammonia = (moles of HCl absorbed) – (moles of NaOH
it will give a positive test. solution required)
( ) (
= 20 × 0.1 × 10−3 − 15 × 0.1 × 10−3 )
Sol 20: (C) It would not give pale-yellow precipitate
= moles of NH3 evolved.
of AgBr because CH3Br is a covalent compound and
Θ = moles of nitrogen in organic compound
hence does not contain Br ion. To get the precipitate
of AgBr, CH3Br is treated first with aqueous KOH to get ∴ wt. of nitrogen in org. comp = 0.5 × 10−3 × 14
Θ
CH3 OH and Br and then the addition of AgNO3 . It = 7 × 10−3 g
would now give the precipitate of AgBr partly soluble
7 × 10−3
in NH3 or NH4 OH . = % wt = 23.7 %
29.5 × 10−3

Sol 9: (B) As per question


Previous Years Questions
Sol 1: (B) Molisch’s Test : when a drop or two of Normality Volume
alcoholic solution of α –naphthalene added to sugar
solution and then conc. H2SO4 is added along the sides N 60 mL
H2SO 4
of test tube, formation of violet ring takes place at the 5
junction of two liquids.
N 20 mL
NaOH
Sol 2: (B, D) Silver mirror test is tollen’s reagent test 10
given by all aldehydes.

(n=
) (n )
geq H SO geq NaOH (
+ ngeq )
NH3
Sol 3: (C) Phenol gives violet coloured complex 2 4

compound with neutral FeCl3, benzoic acid gives pale


1 60 1 20
dull yellow ppt. with neutral FeCl3. × = ×
5 1000 10 1000
+ ngeq ( )
NH3

6 1
Sol 4: (A) It is a test characteristic of amide linkage.
Urea also has amide linkage like proteins.
=
500 500
(
+ ngeq )
NH3

5 1
−3
Sol 5: (C) On solving we get that 5 × 10 mol of H2SO 4
(n ) =
geq NH
3
=
500 100
2 7 . 4 4 | Practical Organic Chemistr y

(nmol )
= (=
nmol ) (=
ngeq )
1 JEE Advanced/Boards
N NH3 NH3 100
14 Exercise 1
(Mass=
)N = 0.14 g
100
Sol 1: Leibig’s Combustion Method
0.14
Percentage of “N” = × 100 = 10%
1.4 Potash tube: Wt. = 1.76 gm (increase)
44 gm of CO2 → 12g C
Sol 10: (A)
a
∴ 1.76 gm of CO2 → 'a'g C ∴ % C = × 100
Atomic mass of Br Wt. of AgBr 0.92
%X= × × 100
Molecular mass of AgBr Wt. of organic Bromide U-tube: Increase = 1.08 gm

80 141 If 18 gm H2O → 2gm of H2


Thus % Br = × × 100 = 24
188 250 ∴ 1.08 gm H2O → 'b'g H2
b
Sol 11: (A) It is fact. ∴ %H = × 100
0.92
∴ % O = 100 − (%H+ %C)
Sol 12: (C) Glycerol is high boiling liquid with B.P. 290°C .
It can be separated from spent-lye by distillation under
Sol 2: S in organic substance
( I ) HNO3 ∆ BaSO
reduced pressure. Liquid is made to boil at lower
temperature than normal temperature by lowering
32 g ( II ) BaCI2 233 g4
pressure on its surface, so external pressure is reduced 32 Weight of BaSO 4
% of S = × × 100
and B.P. of liquid is lowered hence glycerol is obtained 233 weight of subs tance
without decomposition at high temperature.
32 0.5825
= × × 100 = 36.78%
233 0.2175

Sol 3:

Atom Atomic Mass Percentage b Simple


=x
(a) (b) a ratio

C 12 20.0 20 1.66
= 1.66 =1
12 1.66

H 1 6.67 6.67 6.67


= 6.67 =4
1 1.66

N 14 46.67 46.67 3.33


= 3.33 =2
14 1.66

O 16 26.66
26.66 1.66
= 1.66 =1
16 1.66

Empirical formula = CH4N2O


Chem i str y | 27.45

Empirical formula mass = 12 + 4 + 28 + 16 = 60 E = 152


molecular mass 60 Equivalent mass of acid = equivalent mass od silver salt
n
= = = 1
empirical formula mass 60
‒ Equivalent mass of Ag + basicity
∴ Molecular
= formula (=
CH4N2O )
1
CH4N2O = 152 ‒ 108 + 1
 The compound gives biuret test hence, given = 45
compound is urea, i.e., NH2CONH2
∴ Mol mass of acid = equivalent mass of acid × basicity
10.5 = 45 × 2 = 90
4: C 10.5g
Sol = = =mol 0.87 mol
12
1 Sol 7:
H 1g
= = mol
= 1mol
2
2 weight of H2O
∴ ( C0.87H1 ) =
C6.09H7 % of H = × × 100
7 18 weight of organic compound
w weight of H2O
PV nRT
= = RT × 100
m weight of organic compound
≈ C6H7 2 0.9
= × × 100
18 0.5
2.4
1 × 1= × 0.082 × 400
m = 20%
m = 79 ∴ The percentage of carbon = 100 ‒ 20 =80%

Sol 5: Pressure = 785 mm, Sol 8:


Aqueous tension = 12 mm C H N
Ratio of weights 9 1 3.5
∴ Actual pressure of dry N2 at K = 758 ‒ 12 = 746 mm
Ration of atoms
= 9 / 12 0.75
= 1 / 1 1= 3.5 / 14 0.25
Volume of N2 gas collected = 32 mL 0.75 1 0.25
0.25 0.25 0.25
P1 V1 P2 V2
= = 3 = 4= 1
T1 T2

760 × V1 746 × 32 ∴ Empirical formula C3H4N


=
273 287 108
=n = 2
54
V1 = 29.88 mL ( at STP )
Hence, molecular formula
= ( C3H4N) × 2 = C6H8N2
28 × V × 100
% of N =
22400 × w Sol 9: Sublimation is used to separate the mixture of
28 × 29.88 × 100 iodine and sodium chloride iodine sublimes readily.
= = 19.90%
22400 × 0.1877
Sol 10: Sodium extract is boiled with nitric acid to
Sol 6: Mass of silver salt = 0.228 g decompose NaCN and NaS if present.

Mass of silver left = 0.162 g NaCN + HNO3 → NaNO3 + HCN ↑

Basicity of acid = 2 Na2S + HNO3 → 2NaNO3 + H2S ↑

If cyanide and sulphide ions are not decomposed, they


equivalent mass of silver salt mass of acid taken
= will interfere with the test by forming precipitate of
equivalent mass of silver mass of silver left AgCN and Ag2S

E 0.228
=
108 0.162
2 7 . 4 6 | Practical Organic Chemistr y


NaCN + AgNO3 → AgCN + NaNO3 44
White ppt
0.138g of carbon on combustion give CO=
2 × 0.138
12
Na2S + 2AgNO3 → Ag2S + 2NaNO3
= 0.506 g
Silver sulphide
(Black ppt)
2H = H2O
Sol 11: For testing sulphur, the sodium extract is 2 g of hydrogen on combustion give water = 18
acidified with acetic acid because lead acetate is soluble
0.0096 g of hydrogen on combustion give water
and does not interfere with the test. If H2SO 4 were, lead
acetate will react with it forming white precipitate of = 18 × 0.0096 = 0.0864 g
lead sulphate. 2

( CH3COO )2 + H2SO4 → PbSO4 + 2CH3COOH Sol 18: Wt. of organic compounds = 0.246 g
White ppt
Wt. of CO2 produced = 0.198 g
Sol 12: R f value of A is 0.65, therefore, it is less strongly Wt. of H2O produced = 0.1014 g
adsorbed as compared to compound B which has R f
value of 0.42. Therefore, A will be eluted first. 12 0.198 × 100
Percentage of carbon
= ×
44 0.246
Sol 13: The mixture can be separated by the process = 20.95%
of sublimation. Camphor will sublime whereas calcium 2 0.1014 × 100
sulphate will be left behind. Percentage of hydrogen
= × = 4.58%
18 0.246

Sol 14: By differential extraction. Sol 19: To calculate the volume of N2 at S.T.P

= V1 50mL
= V2 ?
Sol 15: Fe4 Fe ( CH) 
 6 3

= P1 700mm
= P2 760mm

Sol 16: Wt. of organic compound = 0.2475 g


= T1 300K
= T2 273 K

Wt. of CO2 produced = 0.4950 g p1 V1 p2 V2


Applying =
Wt. of H2O produced = 0.2025 g T1 T2

22 Wt. of CO2 p1 V1 T2
Percentage of carbon = × × 100 Or V2 =
44 Wt. of compound p2 T1

12 0.4950 700 × 50 × 273


= × × 100 =54.54 ∴ V2 = = 41.9 mL
44 0.2475 760 × 300
22400 mL of nitrogen at S.T.P. weigh = 28 g
2 Wt.of H2O
Percentage of hydrogen = × × 100 28 × 41.9
18 Wt. of compound 41.9 mL of nitrogen at S.T.P weigh = = 0.0524g
22400
2 0.2025 0.0524
= × × 100 =9.09 Percentage of nitrogen= × 100
= 17.49%
18 0.2475 0.3

Sol 17: Percentage of carbon = 69% Sol 20: 1 M of 10 mL H2SO 4 = 1 M of 20 mL of NH3


69 1000 mL of 1 M ammonia contain = 14 g nitrogen
= 0.2 ×
Amount of carbon in 0.20 g of compound
100
= 0.138 g 14 × 20
20 mL of 1 M ammonia contain = g nitrogen
Percentage of hydrogen = 4.8% 1000
0.2 × 4.8 14 × 20
Amount of hydrogen in 0.20 g of compound = ∴ Percentage of =
nitrogen × 100 = 56.0%
100 1000 × 0.5
Now, = 0.0096 g
12g of carbon on combustion give CO2 = 44 g
Chem i str y | 27.47

Exercise 2
Single Correct Choice Type

Sol 1: (B) Prussian is formed when Ferric sulphate reacts with Na4[Fe(CN)6]. Carbon and nitrogen of organic
compound on fusion with Na gives NaCN, water soluble. This converted in sodium ferrocyanide by addition of
FeSO4. Ferric ions generated react with Ferro cyanide to form prussion blue ppt of ferric Ferro cyanide.
Na + C + N → NaCN
6NaCN + FeSO4  Na4[Fe(CN)6] + Na2SO4
Na4[Fe(CN)6 + Fe3+  Fe4[Fe(CN)6]3

Sol 2: (D) Lassaigne’s test is used for detection of nitrogen, halogen and sulphur.

Sol 3: (A) Kjeldahl’s method is used in the estimation of Nitrogen

Sol 4: (B) Sodium cyanide

Sol 5: (D) Sodium bicarbonate (NaHCO3). Since carboxylic acids dissolve in NaHCO3 but phenol do not.

COOH + HCO3 COO + H2CO3

A carboxylic acid
soluble in organic
solvent

OH + HCO3 No reaction

Sol 6: (D) Due to intra molecular hydrogen bonding Ortho nitro phenol is more volatile than para nitro phenol
which have intermolecular hydrogen bonding. In steam distillation orthonitro phenol is distil over with water and
para nitro phenol remains in the flask.

Sol 7: (B) Hydrazine does not have C atoms, so it does not form NaCN and hence does not give a positive
Lassaigne’s test for nitrogen.

Sol 8: (A)

Element Percentage At. Mass Relative number of Simplest ratio


atoms
Carbon 40/12 = 3.33 12 3.33 1
Hydrogen 6.60/1 = 6.60 1 6.60 2

Oxygen 53.34/16 = 3.33 16 3.33 1

Empirical formula = CH2O

Sol 9: (B) As in IR spectroscopy each functional group appears at a certain peak. So it is used for the purification
of cyclohexane from a mixture of benzoic acid, iso amyl alcohol, cyclohexane and cyclohexanone.
2 7 . 4 8 | Practical Organic Chemistr y

Sol 10: (C)


Element Percentage At. Mass Relative number of Simplest ratio
atoms
Carbon 40/12 = 3.33 12 3.33 1
Hydrogen 13.33/1 = 13.33 1 13.33 4

Nitrogen 46.67/14 = 3.33 14 3.33 1

Empirical formula = CH4N

Sol 11: (C)

Sol: Calculation of empirical formula:

Element Percentage At. mass Relative number or atoms Simplest ratio of atoms
Carbon 34.62 12
34.62 2.88
= 2.88 = 1 ×3 = 3
12 2.88
Hydrogen 3.84 1
3.84 3.84
= 3.84 = 1.33 × 3= 4
1 2.88
Oxygen 61.54 16
16.54 3.84
(by difference) = 3.84 = 1.33 × 3= 4
16 2.88

Empirical formula of the acid = C 3H4 O 4 Match the Columns


Empirical formula =
Sol 12: A → q; B → p; C → s; D → r
( 3 × 12 ) + ( 4 × 1) + ( 4 × 16 ) =
104
1° amine + Hinsberg reagent → soluble sulfonamide
Calculation of molecular mass:
HCl
 → ppt
20 ml 0.1 N NaOH ≡ 0.1075 gm acid dil.
2° amine + Hinsberg reagent → ppt.
20 × 0.1 ml 1 N NaOH ≡ 0.1075 gm acid
(b) – p
0.1075
⇒ 1000 ml 1 N NaOH ≡ × 1000 gm acid NaHSO3 Addition bisulphite
20 × 0.1 product (white crystails)
≡ 53.75 gm acid C2H5OH
Equivalent mass of the acid = 53.75 No addition
product
Molecular mass of the acid = Equivalent mass × basicity
× 2 = 107.50
= 53.75 (c) – s Dil. H2SO 4 converts butanol to ether, which can
be separated by distillation.
Molecular mass 107.50
=n = ≈1 (d) – r CH3COOH + NaOH → CH3COONa
Empirical mass 104.0 soluble sodium salt

Molecular formula = C 3H4 O 4


Chem i str y | 27.49

Sol 13: A → q; B → p; C → s; D → r Sol 23: (D) Beilstein test is given by all of these.

(b) – p. Aniline with dil. HCl gives anilium chloride.


Assertion Reasoning Type

NH2 NH3 Cl NH2 Sol 24: (A) In organic layer test, Cl2 water is added
to sodium extract , which oxidises Br − and I − ions to
dil. base
Br2 and I2 respectively as reduction potential of Cl2 is
HCl neutralisation
greater than that of Br2 and I2 .
Regeneration

COONa Sol 25: (D) Hydroxylamine (NH2OH) does not gives


Lassaigne’s test.
COOH
+ NaHCO3 + H2O + CO2
Sol 26: (C) Benzene (b.p. 353K) and methanol (b.p.
338K) cannot be separated by fractional distillation as
(c) – s. fractional distillation is used to separate two liquids
H2O from their mixture when their boiling points differ by
O
H2SO4
OH + HO 20° or so.

(d) – r self explanatory


Previous Years’ Questions
Comprehension Type Sol 1: (D) pKa of PhOH (carbolic acid) is 9.98 and that
of carbonic acid (H2CO3 ) is 6.63 thus phenol does not
Sol 14: (C) KCN will give positive Lassaigne’s test for give effervescence with HCO3− ion.
nitrogen.
Sol 2: (B, D) (A) Both are soluble in NaOH, hence
Sol 15: (C) O2N SO2H will give blood red inseparable.

colour in Lassaigne’s test for nitrogen. (B) Only benzoic acid (C6H5COOH) is soluble in NaOH
and NaHCO3 , while benzyl alcohol (C6H5CH2OH) is not.
Sol 16: (B) Prussian blue colour in the detection of Hence, separable.
nitrogen in Lassaigne’s test is due to formation of (C) Although NaOH can enable separation between
Fe4 [Fe(CN)6 ]3 benzyl alcohol (C6H5CH2OH) and phenol (C6H5OH) as
only the later is soluble in NaOH. However, in NaHCO3 ,
Sol 17: (B) Violet colour in the detection of sulphur both are insoluble. Hence, inseparable.
with sodium nitroprusside is due to formation of
Na4 [Fe(CN)5 NOS] (D) α -phenyl acetic acid (C6H5CH2COOH) is soluble in
NaOH and NaHCO3 . While benzyl alcohol (C6H5CH2OH)
Sol 18: (A) NO3− is not. Hence, separable.

Sol 19: (C) NaCN and Na2S are decomposed by HNO3 Sol 3: (C) Only aromatic primary amines give this azo-
dye test. Amine in option (d) is not aromatic.
Sol 20: (B) Black precipitate in the detection of sulphur
with lead acetate and acetic acid is due to formation NH2 N2+Cl-
of PbS. NaNO2

H3C H3C
Sol 21: (B) Blood-red colour in the detection of both
N and S in the Lassaigne’s extract is due to formation
OH
of Fe(CNS)3 .

Sol 22: (A) Yellow precipitate in the detection of -naphthol


N=N CH3
phosphorous when an organic compound is heated
with Na2O2 and then boiled with conc. HNO3 followed OH
by the addition of ammonium molybdate is due to
formation of (NH4 )3 . PO 4 . 12MoO3
2 7 . 5 0 | Practical Organic Chemistr y

Sol 4: A → r, s; B → p, q; C → p, q, r; D → p
Sodium fusion extract gives Prussian blue colouration, nitrogen and carbon both are present in the compound.
Phenolic group and salt of carboxylic acid gives FeCl3 test. Chloride salt gives white precipitate of AgCl on treatment
with AgNO3 .
Hydrazone formation occur effectively at pH = 4.5. The reaction proceeds in that condition only
when H+ concentration is sufficient to activate the following enolization.

Sol 5: Carbon: Hydrogen: : 10.5 : 1


Calculation of empirical formula

Element Percentage At. Mass Relative number of atoms Simplest ratio


Carbon 12
10.5 91.3 7.6
× 100 =
91.3 = 7.6 =1×7 = 7
11.5 12 7.6

Hydrogen 1
1 8.7 8.7
× 100 =
8.7 = 8.7 = 1.14 × 7= 8
11.5 1 7.6

Empirical formula = C7H8 ; Empirical formula mass = (12 × 7) + (1 × 8) = 92


Calculation of molecular mass: Experimental conditions STP Conditions
V1 = 1 litre V2 = ? ; P1 = 1 atm P2 = 1 atm ; T1 = 127 + 273 = 400 K; T2 = 273 K
Applying the gas equation, we get
P1 V1 T2 1 × 1 × 273
V2 = × = = 0.6825 Litre
T1 P2 400 × 1

0.6825 litre of the gas weighs 2.8 gm.


2.8
∴ 22.4 litre of the gas weighs ×22.4 = 91.89 = 92
0.6825
Molecular mass` 92
N= = =1
Empirical mass 92
∴ Molecular formula = Empirical formula = C7H8
Chem i str y | 27.51

Sol 6 : A → p, q, s; B → q; C → q, r, s; D → q, r
(Note: Assuming AgNO3 in ammoniac)
NO2 O2N
NO2 O2N
(A) PhCHO+O2N NH NH2 PhHC N NH NO2
(A) PhCHO+O2N NH NH2 PhHC N(ppt.) NH NO2
NH3 (ppt.)
PhCHO+Ag2O PhCOO- + Ag


NH3
PhCHO+Ag2O PhCOO- + (White
Ag ppt.)


(White ppt.)
CN
CN
KCN
PhCHO KCN Ph C O-
PhCHO Ph C O-

H
H
ammonical AgNO3 -
CH3C  CH ammonical AgNO3 CH CC Ag +

CH3C  CH CH3C3C- Ag+

(White ppt.)
(White ppt.)

CN
CN

KCN
O-O-
KCN
(B)
(B) PhCHO
PhCHO Ph
Ph CC

HH

--


(C)) AgNO3+CN AgCN




(C AgNO 3+CN AgCN

-


(D) AgNO3+I AgI


(D) AgNO3+I- AgI

Sol 7 : (5)
Are soluble in aqueous NaOH.

COOH OCH2CH3 OH OH COOH


CH2OH
&

N
H3C CH3
2017-18 100 &
op kers
Class 12 T
By E ran culty
-JE Fa r
IIT enior emie .
S fP r es
o titut
Ins

CHEMISTRY
FOR JEE MAIN & ADVANCED
SECOND
EDITION

Exhaustive Theory
(Now Revised)

Formula Sheet
9000+ Problems
based on latest JEE pattern

2500 + 1000 (New) Problems


of previous 35 years of
AIEEE (JEE Main) and IIT-JEE (JEE Adv)

5000+Illustrations and Solved Examples


Detailed Solutions
of all problems available

Plancess Concepts
Topic Covered Tips & Tricks, Facts, Notes, Misconceptions,
Key Take Aways, Problem Solving Tactics
Co-ordina on Compounds
PlancEssential
Questions recommended for revision
28. C O - O R D I N AT I O N
COMPOUNDS

1. INTRODUCTION
The concept of coordination compounds originates from the tendency for complex formation of transition elements.

Molecular or Addition Compounds


When solutions of two or more simple stable compounds in molecular proportion are allowed to evaporate, crystals
of new substances, called molecular or addition compounds, are obtained. Some common examples are given below:
CuSO4 + 4NH3 → CuSO4·4NH3
AgCN + KCN → KCN·AgCN
4 KCN + Fe (CN)2 → Fe (CN)2.4KCN
K 2SO 4 + Al2 ( SO 4 ) + 24H2O → K 2SO 4 .Al2 ( SO 4 ) .24H2O
3 3
Alum

Simple stable Addition or


Compounds Molecular compounds

There are two types of molecular or addition compounds: (1) double salts or lattice compounds and (2) coordination
or complex compounds.
Double Salts or Lattice Compounds
The addition compounds which are stable only in solid state but are broken down into their individual constituents
when dissolved in water are called double salts or lattice compounds.

Nomenclature of Double Salts


(a) A
 hyphen (-) is used between the compounds while writing the names of double salts with the number of
molecules appearing inside brackets at the end.
Example: KCl·MgCl2·6H2O – potassium chloride-magnesium chloride-water (1/1/6)
K2SO4·Cr2 (SO4)3·24H2O – potassium sulphate-chromium sulphate-water (1/1/24)
(a) C
 ations and anions with the same oxidation number are represented with English alphabets (cation first). If
more than one type of cations are present, they are arranged in the increasing order (ascending order) of
oxidation numbers. Anions of different oxidation numbers are arranged in the following order:
Oxide (O2–), hydroxide (OH–), inorganic anion, organic anion, hydride (H–).
The above rules are further clarified by following examples:
(i) KNaCO3 – Potassium sodium carbonate (mixed salt)
(ii) KHCO3 – Potassium hydrogen carbonate (mixed salt)
2 8. 2 | Co-ordination Compounds

Note: Hydrogen is an exception and is written at the end.


(iii) NH4·MgPO4·6H2O — Ammonium magnesium phosphate-6-hydrate or water
(iv) NaCl·NaF·2Na2SO4 — (Hexa) sodium chloride fluoride (bis) sulphate

Coordination or Complex Compounds: Those molecular compounds which retain their identity in solid as well as
in solution, are known as coordination or complex compounds. A part (or whole compound) of these compounds
is not dissociated in solution and its behavior is different from its constituents.
4−
Example: K 4 Fe ( CN)  → 4K + + Fe ( CN) 
 6  6
Complex ion

Illustration 1: Aqueous solution of potassium ferrocyanide does not give the test of Fe (II) and it is not poisonous
like potassium cyanide. Why? (JEE MAIN)

Sol: Being a complex salt, it ionizes to K+ and [Fe (CN)6]4– ions. Due to absence of Fe (II) it does not give the test of
Fe (II). Absence of free CN– makes it nonpoisonous.

Note: An imperfect complex compound may be too unstable to exist and may be completely dissociated in
solution, and then becomes a double salt.

2. TYPES OF COORDINATION COMPOUNDS


(a) Based on the nature of cations and anions, coordination compounds are classified into four types:
(i) Simple cation and complex anion: K4 [Fe (CN)6], K2 [PtCl6], etc.
(ii) Complex cation and simple anion: [Cu (NH3)4] SO4, [Ni (NH3)6] Cl2, etc.
(iii) Complex cation and complex anion: [Pt (NH3)4] [PtCl4], etc.
(iv) Neutral complex compounds: Ni (CO)4, Fe (CO)5, etc.

(b) Based on stability, coordination compounds are of two types:


(i) P
 erfect or penetrating complexes: These are compounds in which the complex ion is feebly dissociated
in solution and is highly stable.
E.g. K4 [Fe (CN)6] → 4K+ + [Fe (CN)6]4–

[Fe (CN)6]4– Fe2+ + 6CN– (Feebly dissociated)


(ii) Imperfect or normal complexes: These are compounds in which the complex ion is appreciably dissociated
and is less stable.
E.g. K4 [Cd (CN)4] → 2K+ + [Cd (CN)4]2–
[Cd (CN)4]2– Cd2+ + 4CN– (Reversible dissociation)

3. TERMINOLOGIES IN COORDINATION COMPOUNDS

3.1 Central Metal Atom/Ion


Metal atom of complex ion which coordinates with atoms or group of atoms/ions by accepting NH3 NH3 +2
their lone pair of electrons. Here Cu is the central metal atom. Some complex ions may also Cu
have more than one metal atom. NH3 NH3
Chem i str y | 28.3

3.2 Ligands
The anions, cations or neutral molecules, which form coordinate bonds with the central metal atom by donating
an electron pair (lone pair) are ligands. These electron pair donors are also known as Lewis bases. Thus, a complex
ion is formed as: Mn+ + xL → [MLx] n+
In the above example, NH3 is the ligand.

Classification of Ligands

Ligands are classified as follows:

(a) On the basis of the charge of ligand:


(i) A
 nionic ligands: These are negatively charged and are the most common type of ligand, such as, F–, Cl–,
Br–, OH–, CN–, SO32– , S2–, SO42–, etc.
 OH,
(ii) Neutral ligands: These are uncharged and are the electron pair donor species such as H2 O,R  NH

3
    
RNH2 , R 2 NH, R3 N etc.

(iii) Cationic ligands: They are positively charged and are rare such as NO+, etc.

(b) On the basis of denticity: The number of donations accepted by a central atom from a particular ligand is
known as the denticity of the ligand. Based on this, ligands are classified as follows:
(i) Monodentate: when only one donation is accepted from the ligand. For example,
H2O, NO, CO, NH3, CO2–, Cl–, etc.

(ii) Bidentate: when two donations are accepted from the ligand. For example,
(1) En: Ethylenediamine (2) Pn: Propylenediamine

CH2 CH2 CH3 CH CH2


H2N NH2
NH2 NH2

(iii) Tridentate: when three donations are accepted from the ligand. For example,

(1) Dien: Diethylenetriamine (2) Imda: Iminodiacetate


CH2 CH2 NH
CH2 CH2
H2C NH2 CH2
O=C C=O
H2N NH2 O
-

O-

(iv) Tetradentate: when four donations are accepted from the ligand. For example,
(1) Trien: Trie thylenetetraamine (2) NTA: Nitrilotriacetate

CH2 CH2 CH2


O CH2
CH2 NH CH2 C N C=O
CH2
-
H2N NH O O

H2N CH2 C
O O
CH2
2 8. 4 | Co-ordination Compounds

(v) Pentadentate: when five donations are accepted from the ligand. For example, EDTA: ethylenediamine
triacetate
CH2 CH2 CH2 CH2
O=C N N C=O
CH2 H -
O
-
O
C
O O
-


(vi) H
 exadentate: when six donations are accepted from the ligand. For example, EDTA: Ethylenediamine
tetraacetate
CH2 CH2 CH2 CH2

O=C N N C=O
-
CH2 CH2 -
O O
C C
O O
-
O O
-

Some other types of ligands:

(a) Flexidentate: Some ligands exhibit variable denticity and are called flexidentate ligands. For example,

(i) O O O O
S S
and O
O O O
Bidentate ion Monodentate ion

(ii) [Co (NH3)4CO3] Br and [Co (NH3)4CO3] Br

CO32 − is bidentate CO32– is monodentate

(b) Chelating: A ligand which forms a ring structure with the central atom is called a chelating ligand. All
polydentate ligands are chelating ligands.
Chelated complex compounds are more stable than similar complexes with monodentate ligands because
dissociation of the complex involves breaking down two bonds rather than one. However, it must be noted
that NH2NH2 and H2N(−CH2−CH2−)NH2 cannot act as chelating ligands due to their three-member ring and
locked structure, respectively.

(c) Ambidentate ligand: A ligand with more than one kind of donor sites but only one site is utilized at a time
is called an ambidentate ligand. Ambidentate ligands are of two types:

(i) Monodentate and ambidentate:

C N O—C N S—C N
or or
or
C N S—C N

or O—C N

(ii) Bidentate and ambidentate:


S O- S
-
O
C C
or
C C
-
S O- S O

dithicoxalate
Chem i str y | 28.5

(d) Classification of ligands on the basis of bonding interaction between the central metal atom and ligand:

(i) Classical or simple donor ligand: These ligands donate their lone pair of electrons to the central atom.
For example, O2–, OH–, F–, NH2− , NH3, N3–, etc.

(ii) Nonclassical or π-acceptor ligand: These ligands donate the lone pair of electrons to the central atom
and accept the electron cloud from the central atom in their low-lying vacant orbital. This kind of back
donation is known as ‘synergic effect’ or ‘synergic bonding’. For example, CO, CN–, NO+, PF3, PR3 (R = H,
Et, Ph…), C2H4, C2H2, CO2, etc.
•• In the case of CO, the back donation to the π* orbital of the central atom can be depicted as:

*

M :C  O: M = C =O
*

Figure 28.1: Back bonding in metal carbonyl

As per valence bond or molecular orbital theory, it is implicit that the bond order of C–O bond
decreases but the C–O bond length increases due to synergic effect. Similarly, since CN– and NO+ are
isoelectronic with CO, back donation takes place in these species also in the π* orbitals and the same
conclusion can be drawn for the bond order and bond lengths.

•• In PR3, the back donation can be depicted as:


vacant 3d orbital
accepts the back
donation

M :P R3

Figure 28.2: Back bonding in case of phosphine ligand

•• In C2H4, the back donation is clearly depicted with the example of Zeise’s salt.

H
Cl Cl *
H
P1 C

Cl
C H
*

H
Figure 28.3: Back donation in case of ethylene ligand

Here, back donation is received in the p* orbital of the C–C bond. Hence, the bond order of C–C bond
decreases and the bond length increases as compared to the free C2H4 molecule. Due to backbonding,
the C2H4 molecule loses its planarity and likewise the C2H2 molecule loses its linearity.
2 8. 6 | Co-ordination Compounds

Illustration 2: What is meant by the denticity of a ligand? Give examples of a unidentate and a bidentate ligand.
(JEE MAIN)

Sol: Denticity indicates the number of donor sites in a ligand. It can be 1, 2, 3, 4 and 6 for unidentate, bidentate,
terdentate, tetradentate and hexadentate ligands respectively. Unidentate ligands: Cl, H2O, NH3, Bidentate ligands:
Ethylene diamine, Propylene, diamine.

Illustration 3: Although NH2·NH2 possesses two electron pairs for donation, it does not act as a chelating agent.
Why? (JEE MAIN)

Sol: The coordination by NH2·NH2 leads to a three-member strained ring which is highly unstable and hence it does
not act as chelating agent.

3.3 Coordination Number


(a) T
 he number of atoms in a ligand that directly bond to the central metal atom or ion by coordinate bonds is
called the coordination number of the metal atom or ion.
(b) In other words, it is the number of coordinate covalent bond which the ligands form with the central metal
atom or ion.
(c) S
 ome common coordination numbers exhibited by metal ions are 2, 4 and 6. The light transition metals
exhibit 4 and 6 coordination numbers while heavy transition metals exhibit coordination numbers above 6.
(d) F or example, the coordination number of Ni in the complex [Ni (NH3)4] Cl2 is 4 and that of Pt in the complex
K2 [PtCl6] is 6.

3.4 Coordination Sphere


Coordination Sphere
The central metal atom/ion and the ligands directly attached to it Central metal ion
Ligand
are collectively termed as the coordination sphere. Coordination
2+ [Ni (NH3)4 ] Cl2 - lonization Sphere
sphere is represented inside square brackets, e.g. Ni (NH3 ) 
 4 Coordination number

3.5 Oxidation Number


The actual charge that a metal atom experiences in a complex is known as its oxidation number. In other words,
oxidation number of a metal atom will be equal to the total charge on this atom if all the ligands are removed
without their electron pair.

Calculation of oxidation number: Algebraic sum of oxidation numbers of all the atoms of a molecule/ion is equal
to the charge on it.
For example, for Ma [M’b (L)x]
[a× (O.N. of M)] + [b × O.N. of M’] + [x × (O.N. of L)] = 0
For ion [Ma (L) x]y+ a × (O.N. of M) + x × (O.N. of L) = y
e.g:- Oxidation number of Co (let it be x) in ion [Co (CN) (H2O) (en)2]2+ can be calculated as:
x + (O.N. of CN) + (O.N. of H2O) + 2 (O.N. of en) = +2
x + (–1) + (0) + (2×0) = +2 ∴ x = +3
Chem i str y | 28.7

PLANCESS CONCEPTS

(a) Transition metals exhibit variable oxidation states.


(b) Oxidation number of different species:
(i) Alkali metals (Li, Na, K, Rb, Cs) = +1
(ii) Alkaline earth metals (Be, Mg, Ca, Sr, Ba, Ra) = +2
(iii) Oxidation number of ion = charge on ion
(iv) Oxidation number of neutral molecule = 0

Vaibhav Krishnan (JEE 2009, AIR 22)

Illustration 4: Specify the oxidation number of the central metals in the following coordination entities:
(a) [Co (CN) (H2O) (en)2]2+ (b) [PtCl4]2– (c) [CrCl3 (NH3)3] (d) [CoBr2 (en)2]+
(e) K2 [Fe (CN)6] (JEE MAIN)

Sol:
(a) +3 (b) +2 (c) +3 (d) +3 (e) +3

3.6 Effective Atomic Number


(a) Effective atomic number was first proposed by Sidgwick in order to explain the stability of the complex.
(b) It can be defined as the resultant number of electrons present in the metal atom or ion after accepting
electrons from the donor atoms of the ligands.
(c) In some cases, the effective atomic number coincides with the atomic number of the next noble gas.
(d) Effective atomic number is calculated as follows:
EAN = atomic number of the metal – number of electrons lost in ion formation + number of electrons gained
from the donor atoms of the ligands.

PLANCESS CONCEPTS

Calculation of EAN: Effective atomic number = Atomic number (Z) – Electron donated (Equal to O.N.) +
Electrons accepted from ligands (2 × No. of coordinate bonds formed)
Or EAN = Z–O.N. + 2 × (C.N.)

Note: EAN and stability: An ion with central metal atom possessing EAN equal to next inert gas will be
more stable.

Vaibhav Krishnan (JEE 2009, AIR 22)


2 8. 8 | Co-ordination Compounds

Illustration 5: Metal carbonyls having formula M (CO)x where the number of carbonyl units coordinated to metal
M are formed by Fe, Cr and Ni. If effective atomic number of each metal is 36, write the formulas of these metal
carbonyls. (JEE MAIN)

Sol: M (CO)x, In Fe (CO)x EAN = At. No. of Fe + 2 × No. of ligands.


( O.N. of Metals in Metal carbonyls is zero as these compounds are neutral species formed by neutral ligand CO).
i.e. CO ∴ 36 = 26+2x ; ∴ x=5
∴ Formula of iron carbonyl is Fe (CO)5, Similarly, we get Cr (CO)6 and Ni (CO)4.

4. FORMULA AND IUPAC NOMENCLATURE OF COORDINATION


COMPOUNDS

4.1 Formula of a Complex


(a) In formulas of both simple and complex salts, cation precedes the anion. Nonionic compounds are written as
single units.
(b) Complex ions are written inside square brackets without any space between the ions.
(c) Metal atom and ligands are written in the following order:
(i) In the complex part, the metal atom is written first followed by ligands in the order, anionic → neutral →
cationic.
(ii) If more than one ligand of one type (anionic, neutral or cationic) are present, then they are arranged
in English alphabetical order, e.g. between H2O and NH2, H2O should be written first. Similarly, order of
NO2− , SO32 − and OH– will be NO2−
(iii) When ligands of the same type have similar name for the first atom, then the ligand with less number of
such atoms is written first. Sometimes the second atom may be used to decide the order. When number
 , NH
of atoms are also same e.g., Out of NO − , NH− will be written first. In H and N  will be written first
2 2 3 2 3
as it contains only one N-atom.
(iv) Polyatomic ligands and abbreviations for ligands are always written in lower case letters. e.g. (en), (py),
etc.
(v) Charge of a complex ion is represented as over script or square bracket.
E.g.
•• K4 [Fe (CN)6] — First cation and then anion [Rule 1 and 2]
•• [CrCl2 (H2O)4] Br—Cl– (negative ligand) before H2O (neutral ligand [Rule 3-(i)]

4.2 Nomenclature of Coordination Compounds


Mononuclear coordination compounds are named by following these rules:
(a) In both the positively and negatively charged coordination compounds, the cation is named first followed by
the anion.
(b) T
 he ligands are named in alphabetical order before the name of the central atom/ion. (This procedure is
reversed in writing its formula).
Chem i str y | 28.9

(c) Names of the anionic ligands end in –o. E.g.

Symbol Name as ligand Symbol Name as ligand


N 3–
Azido OH –
Hydroxo

Cl– Chloro CO32– Carbonato

O– Peroxo Oxalato
C2 O −42

Br– Bromo Sulphato


SO −42

O2H– Perhydroxo Nitrato


NO3−

CN– Cyano SO32 − Sulphito

S2– Sulphido CH3COO– Acetato

O2– Oxo (Bonded through oxygen) nitrite


NO2−

NH2− Amido (Bonded through nitrogen) nitro

(d) N
 ames of neutral and cationic ligands are the same except for aqua for H2O, ammine for NH3, carbonyl for CO
and nitrosyl for NO. These are placed within parentheses ( ).

Symbol Name as ligand Symbol Name as ligand


H 2O Aqua NO Nitrosyl

NH3 Ammine CS Thiocarbonyl

CO Carbonyl

(e) Positive ligands are named as:

Symbol Name as ligand


NH+4

NO+ Nitrosylium

NH2NH3+ Hydrazinium

(f) P
 refixes mono, di, tri, etc. are used to indicate the number of the individual ligands in a coordination
compound. When the names of the ligands include a numerical prefix, then the terms, bis, tris, tetrakis are
used, and the ligand to which they refer is placed in parentheses. For example, [NiCl2 (PPh3)2] is named as
dichlorobis (triphenylphosphine) nickel (II).
(g) O
 xidation state of the metal in a cation, anion or a neutral coordination compound is indicated by a Roman
numeral in parenthesis.
(h) When the complex ion is a cation, the metal is named same as the element. For example, Co in a complex
cation is called cobalt and Pt is called platinum. In an anion, Co is called cobaltate. For some metals, their
Latin names are used in the complex anions, e.g. ferrate for Fe.
(i) Nomenclature of a neutral complex molecule is done in the similar way as that of a complex cation.
2 8. 10 | Co-ordination Compounds

The following examples illustrate the nomenclature for coordination compounds:


•• [Cr (NH3)3 (H2O)3] Cl3 is named as: Triamminetriaquachromium (III) chloride
•• [Co (H2NCH2CH2NH2)3]2 SO4 is named as: Tris (ethane-1, 2-diammine) cobalt (III) sulphate
•• [Ag (NH3)2] [Ag (CN)2] is named as: Diamminesilver (I) dicyanoargentate (I)

( j) Ligands which join two metals are known as ‘Bridge ligands’ and they are prefixed by ‘µ’ (mu).

NH2
E.g. (NH3)4Co (NO33))44, in this complex
Co (NH3)4 (NO
NO2

Here, NH2 and NO2 are bridge ligands and they are named µ-amido and µ-nitro, respectively.

Illustration 6: Write the formulas for the following coordination compounds: (JEE MAIN)
(A) Tetraammineaquachloridocobalt (III) chloride (B) Potassium tetrahydroxidozincate (II)
(C) Potassium trioxalatoaluminate (III) (D) Dichloridobis (ethane-1, 2-diamine) cobalt (III)

Sol: (A) [Co (NH3)4 (H2O) Cl] Cl2 (B) K2 [Zn (OH)4] (C) K3 [Al (C2O4)3] (D) [CoCl2 (en)2]+

Illustration 7: Write the IUPAC names of the following coordination compounds: (JEE MAIN)
(A) [Pt (NH3)2Cl (NO2)] (B) K3 [Cr (C2O4)3] (C) [CoCl2 (en)2] Cl
(D) [Co (NH3)5 (CO3)] Cl (E) Hg [Co (SCN)4] (F) [Ni (CO)4]

Sol: (A) Diamminechloridonitrito-N-platinum (II) (B) Potassium trioxalatochromate (III)


(C) Dichloridobis (ethane-1, 2-diamine) cobalt (III) chloride (D) Pentaamminecarbonatocobalt (III) chloride
(E) Mercury tetrathiocyanatocobaltate (III) (F) Tetracarbonylnickel (0)

5. ISOMERISM IN COORDINATION COMPOUNDS


Compounds having the same molecular formula but a different arrangement of atoms and different properties are
called isomers and the phenomenon is called isomerism. Types of isomerism exhibited by complex compounds are
summarized below:
Isomerism

Structural isomerism Stereoisomerism

Ionization Salt or Linkage Coordination Geometrical Optical


Isomerism Isomerism Isomerism Isomerism Isomerism

Hydrate Polymerization Coordination


Isomerism Isomerism Position
Isomerism
Chem i str y | 28.11

5.1 Structural Isomerism


Structural isomerism occurs due to the difference in chemical linkages and distribution of ligands within and
outside the coordination sphere. In Structural isomerism, isomers possess dissimilar bonding pattern. Different
types of isomers are discussed below:
(a) Ionization Isomerism: Ionization isomerism is the result of the exchange of groups or ions between the
coordinating sphere and the ionization sphere.
This isomerism occurs only in compounds where counter ions act as potential ligands. Ionization isomers
exhibit different physical as well as chemical properties.

Ionisation
[CoBr(NH3 )5 ]SO 4  →[CoBr(NH3 )5 ]2 + + SO2–
4
( A ) Red violet
Pentaamminebromocobalt (III) sulphate

Ionisation
[Co(SO 4 )(NH3 )5 ]Br  →[Co(SO 4 )(NH3 )5 ]+ + Br –
(B) Red
Pentaamminesulphatocobalt (III) bromide

Here (A) and (B) are ionization isomers. (A) forms white precipitate (BaSO4) with BaCl2 whereas (B) does not
react with BaCl2. Similarly (B) gives yellowish white precipitate (AgBr) with AgNO3 while (A) does not react with
AgNO3. Other examples of ionization isomers are:
(i) [PtCl2 (NH3)4] SO4 and [Pt (SO4) (NH3)4] Cl2
(ii) [CoCl2 (NH3)4] NO2 and [CoCl (NO2) (NH3)4] Cl
(iii) [Pt (OH)2· (NH3)4] SO4 and [Pt (SO4) (NH3)4] (OH)2

(b) Hydrate Isomerism (Solvate Isomerism): In a complex compound, water molecules behave in two ways:
(i) Water molecules which behave as ligands are coordinated with the metal atom and are part of the complex
ion, e.g. [M (H2O)x].
(ii) Water molecules act as water of crystallization and these appear outside the coordination sphere, e.g.
[MLx].nH2O.
Isomerism which occurs due to dissimilar number of water molecules as ligands (inside the sphere) and as water
of crystallization (outside the sphere), is known as hydrate isomerism. This isomerism is analogous to ionization
isomerism, in which water molecules inside and outside the sphere are exchanged.
For example,
Cr (H2O)6Cl3 has three possible structures:
•• [Cr (H2O)6] Cl3 (violet)
•• [Cr (H2O)5Cl] Cl2H2O (green)
•• [Cr (H2O)4Cl2] Cl.2H2O (dark green)
These complex compounds differ from one another with respect to the number of water molecules acting as
ligands.
Other hydrate isomers are:
•• [Co (NH3)4 (H2O)Cl]Cl2.
•• [Co (NH3)4Cl2] Cl H2O
2 8. 12 | Co-ordination Compounds

(c) Linkage or Salt Isomerism:


(i) Linkage isomerism occurs in complex compounds having ambidentate ligands like
—CN, —NC, —NO2, —ONO, —CNO, —NCO, —CNS, —NCS, —SCN, etc.
(ii) In this isomerism, an ambidentate ligand coordinates with different atoms.
(iii) These isomers can be differentiated by IR spectroscopy.

For example,
[Co (NO2) (NH3)5] Cl2 and [Co (ONO) (NH3)5] Cl2
(A) (B)
Pentaamminenitrocobalt (III) chloride Pentaammine nitritocobalt (II) chloride
(Yellow-red) (Red)

(A) is not decomposed by the action of acids whereas (B) liberates HNO3 by the action of acid. Other examples of
linkage isomers are:
(i) [Cr (SCN) (H2O)5]2+ and [Cr (NCS) (H2O)5]2+
(ii) [Co (NO2) (py)2 (NH3)2] NO3 and [Co (ONO) (py)2 (NH3)2] NO3

(d) Polymerization Isomerism: When two compounds possess stoichiometric composition but different
molecular formulas, they are known as polymerization isomers of each other. Molecular formula of one isomer
will be the integral multiple of the other one.
Example: [PtCl2 (NH3)2] and [Pt (NH3)4] [PtCl4]

(e) Coordination Isomerism:


(i) This isomerism occurs only in those complexes in which both cation and anion are complex.
(ii) It occurs as a result of the exchange of ligands between the cation and anion.
(iii) It may occur in those complexes also in which both cation and anion have the same metal atoms.

Example:
(i) [Cr (NH3)6] [Cr (SCN)6] and [Cr (SCN)2 (NH3)4] [Cr (SCN)4 (NH3)2]
(ii) [Co (NH3)6] [Cr (C2O4)3] and [Cr (NH3)6] [Co (C2O4)3]

(f) Coordination Position Isomerism: It occurs in complexes containing bridge ligands and is the result of
OH
dissimilar arrangement of metal atoms forming bridge, e.g. (NH3)4Co Co (NH3)2Cl2 SO4 and
Cl
OH
Cl (NH3)4Co Co (NH3)3Cl SO4
Cl

5.2 Stereoisomerism
Stereoisomerism occurs as a result of the different arrangements of ligands around the central metal atom. It may
be of two types: (1) Geometrical isomerism and (2) Optical isomerism.
Chem i str y | 28.13

5.2.1 Geometrical Isomerism

Isomerism which occurs due to different relative arrangements of ligands around the central metal atom is known
as geometrical isomerism. Geometrical isomers are of two types:
(a) Cis-isomer: In a disubstituted complex molecule/ion, when two similar ligands are at right angle (90º), the
geometrical isomer is known as Cis-isomer.

(b) T
 rans-isomer: When two ligands are positioned in opposite directions, i.e. at 180º to each other, the isomer
formed is trans-isomer.
Cis- and Trans- positions are indicated in figures:
5
4 1 4 1

M M

3 2 3 2
6
Square planar Octahedral

Cis- positions: (1, 2), (2, 3), (3, 4), (1, 4) (1, 2), (2, 3), (3, 4), (1, 4), (1, 5),
(4, 5), (3, 5), (2, 5), (1, 6), (2, 6),
(3, 6) and (4, 6)
Trans- positions: (1, 3) and (2, 4)

Geometrical Isomerism and Coordination Numbers


Geometrical Isomerism with Coordination Number 4:
Tetrahedral complexes do not show geometrical isomerism as all the four valences are identical.

Square–planar complexes:
(a) C
 omplexes of type MA4, MA3B and MAB3 do not show geometrical isomerism, where A and B are monodentate
ligands.
(b) C
 omplexes of formula MA2B2 and MA2BC types have two geometrical isomers, where A and B are monodentate
ligands.

Example:
(i) [PtCl2 (NH3)2] resembles MA2B2 in formula and exists in two isomeric forms:
Cl NH3 Cl NH3

Pt Pt

Cl NH3 NH3 Cl
Cis-isomer Trans-isomer
(light yellow) (Dark yellow)

(ii) [PtCl (NH3) (py)2] resembles MA2BC and exists in two isomeric forms:
Py NH3 NH3 Py

Pt Pt

Py Cl Py Cl
Cis Trans
2 8. 14 | Co-ordination Compounds

(c) Complexes of formula MABCD exist in three isomeric forms:


A B A C A C A B

M M M M

D C D B B D C D
(I) (II) (III) (IV)

(III) and (IV) are similar.


e.g. [Pt (NO2) (NH2OH) (NH3) (py)] + exists in 3 isomeric forms.
A = NO2, B = NH2OH, C = NH3, D = py

(d) If A is an unsymmetrical bidentate ligand, then compounds having formula MA2 tend to exhibit geometrical
isomerism, e.g.

[Pt (gly)2]  —CH —COO–)


gly = glycinate ( NH 2 2

CH2—NH2 NH2—CH2 OC O NH2—CH2

Pt Pt
and
OC O O CO CH2—NH2 O CO
Cis Trans

(e) Bridged dinuclear complexes of formula M2A2B4 also exhibit geometrical isomerism, e.g. PtCl2 P ( C6H5 )3 
 2 ( )
(C6H5)3P Cl Cl Cl Cl Cl
Pt Pt Pt Pt
Cl Cl P(C6H5)3 and (C6H5)3P Cl P(C6H5)3
Trans Cis

Geometrical Isomerism with Coordination Number 6:


(a) Complexes of type MA6 and MA5B type do not show geometrical isomerism.
(b) Complexes of type MA4B2 or MA4BC exist in two isomeric forms, e.g. [CoCl2· (NH3)4]+
Cl Cl
NH3 Cl NH3 NH3

+ +
Co Co

NH3 NH3 NH3 NH3


NH3 Cl
Cis Trans

(c) C
 omplexes of type MA3B3 exist in two geometrical forms which are named as facial (fac–) and meridonial
(mer–) isomers. When three ligands of the same type are arranged in one triangular face, then isomer is facial.
fac- and mer- isomers of complex MA3B3 are as follows:
A B
B A A A
M M
B A B A
B B
fac-isomer mer-isomer
Chem i str y | 28.15

E.g. [Co (NO2)3) (NH3)3] can be represented in fac- and mer- isomeric forms as follows:

NO2 NH3
NH3 NO2 NO2 NO2
Co Co
NH3 NO2 NH3 NO2
NH3 NH3
fac-isomer mer-isomer

Similarly, [RhCl3 (py)3] also exists in fac- and mer- forms.

(d) C
 omplex compound of formula MABCDEF may exist in 15 isomeric forms and only one compound of this type
is identified so far [Pt (Br) (Cl) (I) (NO2) (NH3) (py)].
(e) C
 omplexes of formula M(AA)2B2 and M(AA)2BC also exhibit geometrical isomerism, where A is the symmetrical
bidentate ligand, e.g. ethylenediamine (en), oxalate (ox), etc. [CoCl2 (en)2]+
Cl Cl
Cl

en CO+ en CO
+ en

en
Cl
Cis-isomer Trans-isomer

(f) Complex of type M(AA')3 also exists in Cis- and Trans- forms. Where AA’ is unsymmetrical bidentate ligand, e.g.
[Cr (gly)3], gly: glycinate (NH2CH2COO–)
O O
CH2 C O CH2 C O
NH2 NH2 NH2 NH2
CH2 CH2
+ +
CO CO
C= O C= O
O O NH2 O
O=C NH2 CH2 C O
CH2
=

O
Cis-isomer Trans-isomer

Illustration 8: Draw the structure of geometrical isomers of [Pt (gly)2] where gly is NH2CH2COO–. (JEE ADVANCED)

Sol: CH2 NH2 NH2 CH2 and CH2 CO


NH2 O

Pt Pt
OC O O CO OC O H2N CH2
Cis-isomer trans-isomer

5.2.2 Optical Isomerism

Optical activity: Compounds which rotate on the plane of polarized light are optically active. If the plane rotates
clockwise, then the isomer is said to be dextro rotator (d or +) and if the plane rotates anticlockwise then the isomer
is said to be laevo rotator (l or -). Equimolar mixture of d– and isomer is optically inactive and is called racemic
mixture. Optical isomers differ in optical properties.
2 8. 16 | Co-ordination Compounds

(a) Optical isomerism in complexes with coordination number 4:

(i) T
 etrahedral complexes: Like carbon compounds, complex MABCD must be optically active but due to
their labile nature, such complex cannot be resolved in d or l form. However, tetrahedral complexes with
unsymmetrical bidentate ligand are optically active. In optically active tetrahedral compounds, the ligand
must be unsymmetrical. It is not necessary whether it is chiral (asymmetric) or not, e.g. bis (benzoyl
acetonato) beryllium (II)

H5C6 C6H5 H5C6 C6H5


C= O O=C C= O O=C
CH Be CH CH Be CH
and
C O O C C O O C
CH3 CH3 CH3 CH3
Dextro
Leavo

Another example of this type is [Ni (CH2NH2COO)2]—bis (glycinato) nickel (II)


O O O O
Ni Ni
N N N N

NO2
Illustration 9: Draw all the optical isomers for [(en)2Co Co(en)2]4+ (JEE MAIN)
NO2

Sol: Complex compound shows optical isomerism and exists in d l and meso forms.

4+ en en 4+
en en en 4+
NO2 NO2 NO2

(1) CO CO 2) CO CO (3) en
CO CO
NO2 NO2 NO2
en en en en en en

I and II d and l form (mirror image of each other), III meso-form

(ii) S
 quare planar complexes: Generally square planar complexes are not optically active as they have all the
ligands and metal atoms in one plane. That is why there is a plane of symmetry.

Note: However some optically active square planar complexes are identified, e.g. isobutylenediaminemesostilben-
ediaminoplatinum (II) ion.
2+
C6H5 CH NH2 NH2 CH NH2
Pt
C6H5 CH NH2 NH2 CH C6H5

(a) Optical isomerism in compounds of coordination number 6 – Octahedral complexes:

(i) C
 omplexes of type MA4R2 exist in cis- and trans- forms and both forms are optically inactive due to plane
of symmetry.
(ii) Complexes of type MA3B2 exist in facial and meridonial forms but both are optically inactive.
(iii) C
 omplexes of type MA2B2C2 are optically active, e.g. five geometrical isomers of [PtCl2 (NH3)2 (py) 2]2+ are
possible. Out of these five possible isomers, three have been prepared. Their cis- form is optically active
while trans- forms are optically inactive due to symmetry.
Chem i str y | 28.17

py 2+ py 2+
Cl NH3 py Cl

Pt Pt

Cl NH3 NH3 Cl
Py NH3
Cis-isomer Trans-isomer

(iv) C
 omplex MABCDEF has 15 geometrical isomers and each isomer exists as pair of enantiomers and hence
total 30 optical isomers will be possible. Only one such compound has been identified so far – [Pt (Br)
(Cl) (I) (NO2) NH2) (py)].

PLANCESS CONCEPTS

Number of Possible Isomers for Specific Complexes

Formula Number of Stereoisomers Pairs of Enantiomers

MA6 1 0

MA5B 1 0

MA4B2 2 0

MA3B3 2 0

MA4BC 2 0

MA3BCD 5 1

MA2BCDE 15 6

MABCDEF 30 15

MA2B2C2 6 1

MA3B2CD 8 2

MA3B2C 3 0

M (AA) BCDE 10 5

M (AB)2 CD 11 5

Saurabh Gupta (JEE 2010, AIR 443)


2 8. 18 | Co-ordination Compounds

PLANCESS CONCEPTS

Number of Possible Isomers for Specific Complexes

Formula Number of Stereoisomers Pairs of Enantiomers

M (AB) (CD)EF 20 10

M (AB)3 4 2

M (ABA)CDE 9 3

M (ABC)2 11 5

M (ABBA)CD 7 3

M (ABCBA)D 7 3

Saurabh Gupta (JEE 2010, AIR 443)

6. PREPARATION AND IDENTIFICATION OF COMPLEX COMPOUNDS

Preparation of Complex Compounds

(a) By substitution reactions: Ion of a salt can be substituted by a ligand to form a complex compound, e.g.

CuSO 4 + 4NH3 → [Cu(NH ) ]SO


3 4 4
tetraamminecopper(II)sulphate

(b) By combination reaction: Various complexes can be formed by combination reactions:


 → [Ni(NH ) ]Cl
NiCl2 + 6NH
3 3 6 2

 → [Ag(NH ) ]Cl
AgCl + 2NH
3 3 2

(c) By redox reactions: Two important examples are:


 + 2NH NO + H O → 2[Co(NO )(NH ) ](NO ) + 14H O
2[Co(H2 O)6 ](NO3 )2 + 8NH
3 4 3 2 2 3 3 5 3 2 2

2CoCl2 + 2NH4 Cl + 10NH3 + H2 O2 → 2[Co(NO3 ) 6 ]Cl3 + 2H2 O + 9H2


Identification of Complex Compound Formation

(a) Change in solubility: Solubility of a complex compound changes abnormally when complex is formed, e.g.
AgCN+ KCN → K[Ag(CN)2 ] (Solubility increases)
Partialy soluble Soluble

AgCl+ 2NH3 → [Ag(NH3 )2 ]Cl (Solubility increases)


Insoluble Soluble

Ni2 + + 2dmg → [Ni(dmg)2 ]2 + (dmg = dimethylglyoxime)


Soluble Insoluble
Chem i str y | 28.19

(b) C
 hange in conductivity: As complex formation changes the solubility, the number of ions in solution also
changes and hence conductance changes suddenly.

(c) C
 hange in chemical properties: Change in chemical properties of metal ion also indicates the formation of
complex, e.g.
 , KCl does not precipitate Ag+ due to formation
Ag+ is precipitated by KCl solution but in the presence of NH 3
of [Ag (NH3)2] Cl.

(d) Change in color: Change in color indicates complex formation, eg.


Co2 + + 4Cl− → [CoCl4 ]2–
Pink
Blue

 → [Cu(NH ) ]2 +
Cu2 + + 4NH
Lightblue 3 3 4
Deepblue

[Ni(H2 O)6 ]2 + + 6NH3 → [Ni(NH3 )6 ]2 + + 6H2 O


Green Blue

(e) Change in pH, EMF, Magnetic properties or colligative properties also indicate the complex formation.

7. THEORIES OF BONDING IN COORDINATION COMPOUNDS

7.1 Werner’s Theory


In 1898, Werner propounded his theory of coordination compounds. Werner proposed the concept of a primary
valence and a secondary valence for a metal ion. Main postulates of his theory are:
(a) In coordination compounds metals show two types of linkages (valencies) – primary and secondary.
(b) The primary valencies are normally ionizable and are satisfied by negative ions.
(c) T
 he secondary valencies are nonionizable. These are satisfied by the neutral molecules or negative ions
(ligands). The secondary valency is equal to the coordination number and is constant for a metal.
(d) T
 he ion groups bounded by the secondary linkages to the metal have a characteristic spatial arrangement
corresponding to their different numbers.

Illustration 10: PtCl4 and NH3 may form five complexes, A (PtCl4·6NH3), B (PtCl4·5NH3), C (PtCl4·4NH3), D PtCl4·3NH3
and E (PtCl4·2NH3). One mole of each A, B, C, D and E reacts with excess of AgNO3 to yield 4, 3, 2 and 1 mole AgCl
respectively, while E gives no AgCl. The conductances of their solutions are in the order A > B > C > D > E. On the
basis of Werner’s theory, write their structure and give the total number of ions given by one complex.
(JEE ADVANCED)
Sol:

Formula Structural formula Ionization No. of ions

(A) PtCl4·6NH3 [Pt (NH3)6] Cl4 [Pt (NH3)6]4+ + 4Cl– 5

(B) PtCl4·5NH3 [PtCl (NH3)5] Cl3 [PtCl (NH3)5]3+ + 3Cl– 4

(C) PtCl4·4NH3 [PtCl2 (NH3)4] Cl2 [PtCl2 (NH3)4]2+ + 2Cl– 3

(D) PtCl4·3NH3 [PtCl3 (NH3)3] Cl [PtCl3 (NH3)3]+ + Cl– 2

(E) PtCl4·2NH3 [PtCl4 (NH3)2] No isonisation possible 0


2 8. 20 | Co-ordination Compounds

7.2 Valence Bond Theory


The salient features of the valence bond theory are summarized below:
(a) T
 he central metal ion has a number of empty orbitals for accommodating electrons donated by the ligands.
The number of empty orbitals is equal to the coordination number of the metal ion for the particular complex.
(b) T
 he atomic orbitals (s, p or d) of the metal ion hybridize to form hybrid with definite directional properties.
These hybrid orbitals tend to form strong chemical bonds with the ligand orbitals.
(c) T
 he d-orbitals involved in the hybridization may be either inner (n – 1) d orbitals or outer n d-orbitals. The
complexes formed in these two ways are referred to as low spin and high spin complexes, respectively.
(d) Each ligand possesses a lone pair of electrons.
(e) A
 covalent bond is formed when a vacant hybridized metal orbital and a filled orbital of the ligand overlap.
The bond is also known as a coordinate bond or dative bond.
(f)  hen a complex contains unpaired electrons, it is paramagnetic in nature, whereas if it does not contain
W
unpaired electrons, it is diamagnetic in nature.
(g) 
The number of unpaired electrons in the complex, determines the geometry of the complex as well as
hybridization of the central metal ion and vice-versa. In practice, the number of unpaired electrons in a complex
is found from magnetic moment measurements as illustrated below. µ = n(n + 2) where n = no. of lone pair.
(h) Thus the knowledge of the magnetic moment can be of great help in ascertaining the type of complex.
(i) U
 nder the influence of a strong ligand, the electrons can be forced to pair up against the Hund’s rule of
maximum multiplicity.

Coordination Number Type of Hybridization Distribution of hybrid orbital in space

4 sp3 Tetrahedral

4 dsp2 Square planar

5 sp3d Trigonal bipyramidal

6 sp3d2 Octahedral

6 d2sp3 Octahedral

Application of Valence Bond Theory on Coordination Complexes

(a) C
 omplex with Coordination Number 4: Tetra coordinated complexes have either tetrahedral or square
planar geometry depending on the nature of orbitals involved in hybridization. If one ns and three np orbitals
are involved in bonding, geometry will be tetrahedral and hybridization sp3. If (n – 1) d, ns and two np are
involved in bonding, geometry will be square planar and hybridization dsp2. Tetra coordinated complexes are
common with Ni (II), Cu (II), Pt (II), Pd (II), etc.
(n-1)d ns np
4dsp² hybrid orbital, squar e planar geometry

(n-1)d ns np
4dsp² hybrid orbital, tetrahedral geometr y

Some examples of tetra coordinated complexes are given below:


Chem i str y | 28.21

(i) Tetrahedral Complexes:


•• Ni (CO)4: In Ni (CO)4, Ni has zero oxidation state and exists as Ni (0). Four ligands (CO) are attached
to central metal atom Ni and require four orbitals. The electronic configuration in
Ni (CO)4 can be written as:
8 2 0
3d 4s 4p
28Ni
8
atom = 3d , 4s
2

When four CO ligands are present, it is a strong ligand and the electrons pair up against “Hund’s rule
for maximum multiplicity”.
10 0 0
3d 4S 4p
3
sp
hybridization

3 CO
4sp
hybridization
Ni
CO CO CO CO OC CO
Four 1CO pairs from four
CO
CO molecules

Explanation: Four sp3 hybrid orbitals are arranged tetrahedrally making it a tetrahedral complex.
Since all the electrons are paired, it is diamagnetic.

(ii) Square planar complexes:


•• [Ni (CN)4]2–: Here, Ni is in (II) oxidation state and the electronic arrangement is as follows:
8 2 0
3d 4s 4p

Ni-[Ar]: 3d8. 4s2


NC NC
2- 2-
CN CN

4dsp2 hybrid orbital, square planar geometry Ni Ni

Ni2+ ion [Ar] 3d8, 4s0 NC NC CN CN

4dsp2 hybrid orbital,


3d
8 tetrahedral geometry

3
4dsp hybrid orbital

Ni2+ ion in [Ni (CN) 4]2– 3d8, 4s0 CN CN CN CN

Explanation:

•• CN– is a strong ligand and so it pairs up 3d-electrons against Hund’s rule. The d-orbital thus
made vacant, takes part in hybridization.
•• Four dsp2 hybrid orbitals are arranged in this manner and hence the geometry is square planar.
•• Complex compounds are diamagnetic because all the electrons are paired.
•• The complex makes use of the inner d-orbital, and so it is known as inner orbital or low spin or
hyper ligand or spin paired complex.
2 8. 22 | Co-ordination Compounds

PLANCESS CONCEPTS

Exception: Structure of [Cu (NH3) 4]2+ ion: It is an exceptional case which involves sp2 d hybridization. Here,
Cu is tetra coordinated and may exist as square planar or tetrahedral complex. Physical measurement have
indicated that tetrahedral geometry for [Cu (NH3)4]2+ is not possible. If square planar geometry is supposed
to be correct, then the following electronic arrangement must be followed:
3d
10
4s
1
4p

Cu [Ar]: 3d10, 4s1


Cu2– ion: → [Ar]3d9, 4s0 3d
9
4s
1 0
4p

For dsp2 hyb.3d-electron must be excited to 4-p with the following configuration.
2+
NH3 NH3

Cu2+ ion in [Cu (NH3)4]2+: Cu

NH
2 3 NH3

dsp hybridisation

NH3 NH3 NH3 NH3

Now if the above configuration is correct, the unpaired electrons present in higher energy, 4-p orbital
should be expected to be easily lost and Cu2+ must be easily oxidized to Cu3+, but it never occurs, so the
configuration is not satisfactory. To explain it Huggin suggested sp2d hybridization.

Cu2+ ion in [Cu (NH3)4]2+ :

Note: Pt (II) and Au (III) always form square planar complexes irrespective of their ligands being strong or
weak.

Neeraj Toshniwal (JEE 2009, AIR 21)

(b) Complexes with Coordination Number 6: Hexacoordinated complexes are of two types, inner orbital
complexes and outer orbital complexes. They possess octahedral geometry.
(i) Inner orbital complexes: In this type of complexes the d-orbitals used are of lower quantum number, i.e.
(n – 1). Some examples are given below:
•• Complexes formed by using the inner orbitals are diamagnetic or have reduced paramagnetism.
•• These are also known as low spin or spin paired complexes,

Example 1: [Fe (CN)6]4–


e– configuration of Fe26 = [Ar] 3d64s2
6 0 0
3d 4s 4p

e– configuration of Fe+2 = [Ar] 3d6 =

e– configuration of Fe+2 after rearrangement =


2 3
d sp

The above rearrangement is due to presence of the cyanide ligand.


Chem i str y | 28.23

At this stage, Fe2+ undergoes d2sp3 hybridization to form six d2sp3 hybrid orbitals, each of which accepts an
electron pair donated by CN– ions. The complex is Diamagnetic as it has no unpaired electron.

Example 2: [CO(NH8)6]3+

Example 3: [Cr (NH3)6]3+


5
3d 4s 4p
Cr → Cr24
24

Cr3+ → Cr
3+

Cr3+ in d2sp3 hybridized state

2 3
d sp bybridized state

As this d2sp3 hybridization leads to octahedral geometry, the complex [Cr (NH3)6]3+ will be octahedral in shape.
Since the complex ion has 3 unpaired electrons, it is paramagnetic.
Other complexes of chromium with similar inner structure are [Cr (CN)6]3– and [Cr (H2O)6]3+.
(ii) Outer orbital complexes
•• In these complexes s, p and d orbitals which are involved in hybridization, belong to the highest
quantum number (n).
•• Complex compound formed by the use of outer n and d orbitals will be paramagnetic.
•• Outer orbital complexes are also known as high-spin or spin free complexes.
•• The outer orbital complexes have a high number of unpaired electrons, E.g. [CoF6]3–
3d 4s 4p 4d

Co 27

Co3+ ion →

Co3+ ion in sp3d2 hybridized state


3 2
sp d

Owing to the octahedral orientation of six sp3d2 hybridized orbitals, shape of [CoF6]3– complex ion is
octahedral.
As it possesses four unpaired electrons in the 3d orbital, [CoF6]3– ion is paramagnetic.
Some other examples are [FeF6]3–, [Fe (NH3)6]2+, [Ni (NH3)6]2+, [Cu (NH3)6]2+, [Cr (H2O)6]3+, etc.

Limitations of valence bond theory: Even though the valence bond theory explains the formation, structures
and magnetic behavior of coordination compounds to a larger extent, it suffers from the following short comings:
•• It includes a number of assumptions.
•• It fails to provide quantitative interpretation of magnetic data.
•• It lacks explanation to the color exhibited by coordination compounds.
•• It does not provide a quantitative interpretation of the thermodynamic or kinetic stabilities of coordination
compounds.
•• It is unable to predict the tetrahedral and planar structures of 4-coordinate complexes accurately.
•• This theory does not distinguish between weak and strong ligands in compounds.
2 8. 24 | Co-ordination Compounds

7.3 Crystal Field Splitting Theory


The important terms in Crystal Field Splitting theory are as follows:
(a) D
 egenerate orbitals: in free state, all the d-orbitals (viz., dxy, dyz, dxz, dx2 − y2 and dz2 ) will possess the same
energy and are said to be degenerate.
(b) t2g and eg set of orbitals: In a d-subshell, there are five d-orbitals and on the basis of orientation of lobes of
these five d-orbitals with respect to coordinates, they have been grouped into two sets.

(i) eg set of orbital: dx2 − y2 , and orbitals have their lobes arranged along the axes and they constitute eg
set. These orbitals are also called axial orbital. Term eg refers to ‘doubly Degenerate’, according to group
theory (e = doubly degenerate set)
(ii) t2g set of orbital: This set includes orbitals whose lobes lie between the axes and this set includes dxy,
dyz and dxz orbitals. These orbitals are also known nonaxial orbitals. Group theory called these orbital t2g
where ‘t’ refers to ‘triply degenerate’.

Crystal Field Theory: The crystal field splitting theory (CFT) is an electrostatic model which considers the metal–
ligand bond to be ionic occurring purely due to the electrostatic interaction between the metal ion and the ligand.
Ligands are treated as point charges in case of anions and dipoles in case of neutral molecules. The five d orbitals in
an isolated gaseous metal atom/ion have the same energy, i.e. they are degenerate. This degeneracy is maintained
if a spherically symmetrical field of negative charges surrounds the metal atom/ion. However, when this negative
field is resulted by the ligands (either anions or the negative ends of dipolar molecules like NH3 and H2O) in a
complex, it becomes asymmetrical and the degeneracy of the d orbital is lifted. It results in splitting of the d
orbitals. The pattern of splitting depends upon the nature of the crystal field. Let us discuss this splitting in different
crystal fields in detail.

(a) C
 rystal field splitting in octahedral field: The orientation of d-orbital in octahedral field is represented in
the diagram.
The lobes of t2g, set of orbital (dxy, dyz and dxz) point in between x, y and z axes while lobes of eg set ( d 2 and
z

dx2 − y2 ) point along the x, y and z axes. Thus, energy of the eg set increases higher than that of the t2g set. The
splitting of orbital can be represented by Fig. 4:

eg, (dz2, dx2-y2)

+6 Dq. CFSE (o) = 10Dq.


-4 Dq. Bari centre

Energy

Degenerate d-orbital
n+
In free ion (M )

Figure 28.4: Splitting of d-orbital is a octahedral crystal field

The difference in energy of t2g and eg set is known as crystal field splitting energy or crystal field stabilization energy
(CFSE), which is represented by ∆o (o stands for octahedral) or 10 Dq. The value of 10 Dq or ∆o can be measured
by UV-visible spectrum.
The crystal field splitting, ∆o, depends upon the field produced by the ligand and the charge on the metal ion.
Some ligands are able to produce strong fields, and correspondingly, the splitting will be large whereas others
produce weak fields and these consequently result in small splitting of d orbital. Ligands can be arranged according
to their order of increasing field strength as follows:
I– < Br– < SCN– < Cl– < S2– < F– < OH– < C2O42– < H2O < NCH– < edta4– < NH3 < en < CN– < CO
Chem i str y | 28.25

This series is known as the spectrochemical series. It is an experimentally determined series based on the absorption
of light by complex compounds with various ligands. Let us assign electrons in the d orbital of the metal ion in
octahedral coordination entities. Obviously, the single d electron occupies one of the lower energy t2g orbital. In d2
and d3 coordination compounds, the d electrons occupy the t2g orbital singly in accordance with the Hund’s rule.
For d4 ions, two possible patterns of electron distribution arise:
(a) The fourth electron could either enter the t2g level and pair with an existing electron, or
(b) It could avoid paying the price of the pairing energy by occupying the eg level.

Either of these two possibilities depends on the relative magnitude of the crystal field splitting, ∆o and the pairing
energy, P (presents the energy required for electron pairing in a single orbital). The two options are:
(i) If ∆o < P, the fourth electron enters one of the eg orbital exhibiting the configuration t32ge1g . Ligands for which
∆o < P are known as weak field ligands, form high spin complexes.
(ii) If ∆o > P, it becomes more energetically favorable for the fourth electron to occupy a t2g orbital with
4 0
configuration t2geg . Such ligands are known as strong field ligands and they form low spin complexes. It is
observed from calculations that d4 to d2 coordination entities are more stable for strong field cases compared
to their weak counterparts.

Energy
(dz2, dx2-y2)

Figure 28.5: Splitting of d-orbital is a octahedral crystal field

(b) Crystal field splitting in tetrahedral complexes: The orientation of ligands in a tetrahedral complex is
given in fig. 5. Although none of the d-orbitals point towards axes, the t2g set is close to the direction in which
ligands are approaching so their energy is higher.

The magnitude of ∆t is considerably less than that in the octahedral field, which is mainly due to two reasons:
(i) In tetrahedral complex, number of the ligands is only four instead of six.

(ii) In tetrahedral complexes, the direction of the orbitals does not coincide with the direction of the ligands, both
the factors reduce the CFSE by 2/3 and so ∆t is roughly 4/9 times to ∆o.

(c) Crystal Field Splitting in Square Planar Complexes: The square planar geometry can be considered to be
derived from the octahedral by removing negative charges from the z-axis. As these negative charges are
removed, dxy, dxz and dyz orbital, all of which have a Z-component become more stable as shown in Fig. below.
This type of splitting can be further explained as follows:
As the lobes of point towards the ligands, this orbital has the highest energy. The lobes of dxy orbital lie
between the ligands but are coplanar with them, hence this orbital has the second highest energy. The lobes
of dx2 orbital point out of the plane of the complex but the belt around the center of the orbital (which
contains about 1/3rd of the electron density) lies in the plane. Therefore, dz2 orbital is next highest in energy.
The lobes of dxz and dyz orbital point out of the plane of the complex, and so they are least affected by the
electrostatic field of the ligands, they degenerate and have the lowest in energy.
2 8. 26 | Co-ordination Compounds

Energy

Figure 28.6: Splitting of d-orbital in a square planar crystal


Planar

PLANCESS CONCEPTS

Weak ligands favor high spin complexes because they cannot pair up the electrons against Hund’s rule while
strong ligands favor low spin complexes.

Vaibhav Krishnan (JEE 2009, AIR 22)

Illustration 11: Mn2( aq.


+
) ion is light pink colored while [Mn (CN)6] is blue in color. Explain.
4–
(JEE ADVANCED)

Sol: In complexes, where Mn (II) is present, configuration of a metal ion is d5. There may be two types of spin
arrangements in the presence of different kinds of ligands.
(A) High spin complex (with weak field ligands) and (B) Low spin complex (with strong field ligands)
The arrangement of electrons in these complexes can be depicted as:

eg
d-d transition spin allowed
laportae selection rule
d-d transition spin forbidden
(l = l)forbidden
and laport selection rule
(l = l)forbidden


In high spin complex compounds, it is observed that d–d transition requires reversion of spin which is against the
spin selection rules and this makes them spin forbidden and the intensity of color is of only about 1/100 when
the transition is allowed.
In [Mn (CN)6]4– on the other hand, d–d transitions do not have any such restrictions and are spin allowed. Intense
color also is observed when transition takes place.
Chem i str y | 28.27

7.3.1 Factors Affecting CFSE

(a) N
 ature of ligand: The value of ∆ depends upon the nature of ligands. Ligands with a small degree of crystal
field splitting capacity are termed as weak field ligands and those ligands which cause large splitting are
called strong field ligands. In general, ligands can be arranged in the ascending order of CFSE caused by
them. This series remains practically constant for different metals and is known as spectrochemical series. It
is an experimentally determined series. The order is difficult to explain due to involvement of both σ and π
bonding. Some ligands in spectrochemical series are given below:
I– < Br– < S2– < Cl– < N3− , F– < Urea, OH– < Oxalate, O2– < H2O < NCS– < EDTA < py, NH3 < en = SO32- < bipy,
phen < NO2− < CH3− < C6H5− < CN– < CO. For strong field ligands, the order depends on the donor atom and
is in the following order:
C-donor > N-donor > O-donor > Halogen donor

(b) G
 eometry of the Complex: ∆t is approximately 4/9 times of ∆o. The lower value of ∆t is due to lesser number
of ligands in tetrahedral complex. Also, in tetrahedral complexes the orbital does not point toward the axes,
resulting in less interaction.

(c) O
 xidation state of metal ion: It is observed that the higher the charge on the central metal atom
(or oxidation state), the higher the CFSE.
E.g., ∆o for [Fe (H2O)6]3+ is greater than [Fe (H2O)6]2+, ∆o for [Co (H2O)6]3+ is greater than [Co (H2O)6]2+ and ∆o for
[V (H2O)6]2+ is greater than [Cr (H2O)6]3+.

(d) N
 ature of metal ion: The value of CFSE is also determined by the transition series to which the metal belongs
and the order for this is observed to be 3d < 4d < 5d. The value of ∆ increases by 30% to 50% for 3d to
4d series and from 4d to 5d series. Hence, metals of 4d and 5d series have more tendency to form low spin
complexes, e.g. CFSE for the given complexes follow the order:
[Co (NH3)6]3+ < [Rh (NH3)6]3+ < [Ir (NH3)6]3+]
When two metal ions possess the same charge but different number of d-electrons, the magnitude of ∆o
decreases with increase in the number of d-electrons in the central metal atom. E.g. ∆o for [Co (H2O)6]2+ is
greater than ∆o for [Ni (H2O)6]2+ because Co2+ possesses 3d7 configuration while Ni2+ has 3d8 configuration.

7.3.2 Applications of CFSE

(a) M
 agnetic character of complexes: Complexes containing unpaired
electrons tend to be attracted by magnetic fields and hence known as
paramagnetic. In contrast, when all the electrons are paired, the complex
is slightly repelled by a magnetic field and is said to be diamagnetic. The
Weak field Strong field
magnetic moment of a transition metal wholly depends on the number 2+ 6 2+ 6
Fe (3d ) ion in Fe (3d ) ion in
of unpaired electrons and is equal to n(n + 2) B.M., where n is number [Fe(H2O)6]
2+
[Fe(CN)6]
4-

(paramagnetic)
of unpaired electrons. For diamagnetic substance, the magnetic moment (diamagnetic)
will be zero.
Magnetic moments of coordination compounds can be experimentally determined and this data provide
information to examine the nature of coordination entities further. These measurements are termed as magnetic
susceptibility measurements. For example, [Fe (H2O)6]2+ is paramagnetic while [Fe (CN)6]4– is diamagnetic.
This observation can be explained on the basis of the electronic configurations of Fe2+ in [Fe (H2O)6]2+ and
[Fe (CN)6]4–. H2O is a weak field ligand while CN– is a strong field ligand. So [Fe (CN)6]4– is the inner orbital low
spin complex whereas [Fe (H2O)6]2+ is an outer-orbital high spin complex. The configurations of Fe2+ in both
the compounds are further explained in the diagram here.
2 8. 28 | Co-ordination Compounds

PLANCESS CONCEPTS

Complexes possessing d0 or d10 configuration of a metal ion are always diamagnetic.

Nikhil Khandelwal (JEE 2009, AIR 94)

(b) C
 olour of complexes: In many complexes the d-orbital split takes place in the two sets t2g and eg, which
possess different energies. The difference in energies of t2g and
e
and eg lies in the visible region of the spectrum and this helps
g
E = energy
Lying in
transition metal complexes to absorb color. This makes them E
+hv energy ( E)
E visible
colored complementary to the color absorbed. This transition reagion.
involves t2g and eg sets of d-orbital and is called as d–d
transition. Thus d–d transition is responsible for the color of Ground state
Excited state
transition metal complexes. d–d transition can be represented
diagrammatically as shown here.

PLANCESS CONCEPTS

2− 2−
Complexes like CrO 4 , Cr2 O7 , and MnO 4 , etc. have d0 configuration of the metal ion but still exhibit intense

color. Here the color is caused by the charge transfer spectra (CT) and not by the d–d transition.

Saurabh Gupta (JEE 2010, AIR 443)

7.3.3 Stability of Complexes

Complexes normally exhibit two kinds of stabilities: (i) Thermodynamic stability and (ii) Kinetic stability.
Thermodynamic stability deals with the metal–ligand bond energy, stability constants, redox potentials, etc.,
that affect the equilibrium. On the basis of thermodynamic stability, Blitz classified the complexes into stable or
penetration complexes and unstable or normal complexes.
Kinetic stability deals with the rates of reaction of complexes in a solution. On the basis of kinetic stability, Taube
classified the complexes into labile and inert complexes. Ligands of labile complexes are easily replaceable while
ligands of inert complexes cannot be replaced with easily.

Chelate effect: Complexes containing chelate rings are more stable, e.g. [Ni (NH3)6]2+ and so is less stable than
[Ni (en)3]2+.
Macrocylic effect: When a multidentate ligand is cyclic without any considerable steric effect, then the complex
formed is more stable than acyclic ligand. This phenomenon is called the macrocyclic effect.

Illustration 12: [Cu (CN)4]2- is a more stable complex than [Cu (NH3)4]2+. Why? (JEE MAIN)

Sol: The higher stability constant K = 2 × 1027 for Cu2+ + 4CN– → [Cu (CN)4]2– than for [Cu (NH3)4]2+ (which is 4.5 ×
1011) explains stability. Also CN– is stronger ligand than NH3.
Chem i str y | 28.29

8. ORGANOMETALLIC COMPOUNDS
Organometallic compounds are defined as compounds in which carbon forms a bond with an atom (metal/non-
metal) which is less electronegative than carbon.
These compounds are classified into two – covalently bonded compounds and ionic organometallic compounds.
Covalently bonded compounds: In covalently bonded compounds, the metal and carbon atoms are attached to
each other by a covalent bond. They can be further classified into: three groups:
(i) σ
 (sigma) bonded complexes: A σ-bonded complex consists of H3C CH3 CH3
a metal atom and a carbon atom of the ligand joined together
with a σ bond. In another words, the ligand contributes one Al Al
electron and is called one electron donor. Tetramethyltin, (CH3)4Sn H3C CH3 Ch3
and trimethyl aluminum, (CH3)3 Al are examples of σ-bonded Trimethyl aluminium
organometallic compounds. The latter exists as dimmer and has
a structure analogous to diborane. In this, two methyl groups bridge between two aluminum atoms.

(ii) π
 complexes: Organometallic compounds with
H H
π-bonds present in them are called π-complexes.
Zeise’s salt, ferrocene and dibenzene chromium C
are π-complexes. In these compounds, the π Cl C K+ 2+
Fe Cr
electrons interact with the metal ion and occupy
one of the coordination sites. For example, in Pt H H
ferrocene and dibenzene chromium, the iron and Cl Cl
chromium atoms are sandwiched between two
aromatic rings. Zeise’s salt Ferrocene Dibenzene chromium
K[PtCl3(2-C2H4)] Fe[2-C5H5)]2 6
Cr[ -C6H6)]2
The number of carbon atoms taking part in the
formation of π-complexes is indicated by the power of ηx (pronounced as eta). For example, ferrocene is
represented as [Feη5–C5H5)2] indicating that five carbon atoms or cyclopentadienyl anion are involved in the
π- complication with the metal. Similarly, one can write dibenzene chromium as [Cr (η6–C6H6)2] indicating that
all the six carbons of benzene are involved in π-complexation with chromium.

8.1 Bonding in Organometallic Compounds


Bonding in Metal Carbonyls: The metal–carbon bond in metal carbonyls exhibits σ as well as π characteristics.
(i) σ
 -overlap: The lone pair of electron is present on the bonding orbital of carbon monoxide in a σ bonded
complex and it interacts with the empty d-orbital of the metal to form a metal–carbon bond.
+ +
M + C=O: M C=O
Metal orbital Bonding Bonding in metal
orbiital carbon

(ii) π
 -overlap: Besides, the antibonding orbitals of CO also overlaps with the filled d-orbital of the metal resulting
in back bonding as previously explained. Thus metal carbonyls become much more stable due to this multiple
bonding.
It is important to note that the σ-bond is positioned in the nodal plane of the σ-electrons whereas π-overlap
is perpendicular to the nodal plane.

M C=O M C = O:

Metal orbital Anibonding orbital


Backbonding metal corbonyl
of carbon mono-oxide
2 8. 30 | Co-ordination Compounds

Bonding of Alkenes to a Transition Metal: There are two components in the bonding of alkenes to a transition
metal to form complexes. First, the σ-electron density of the alkene overlaps with a π-type vacant orbital of the
metal atom. Second is the backbonding resulting from the flow of electron density from a filled d-orbital on the
metal into the vacant σ-antibonding molecular orbital on the carbon atom as depicted in the following diagram:

C C

M + M
C C
p overlap

C C
M + M
C C

PLANCESS CONCEPTS

As the electron density on metal atom increases, strength of backbonding from the metal to carbon increases
and the metal–carbon bond length decreases. Likewise, when C–O bond order decreases, C–O bond length
increases and vice versa.
Neeraj Toshniwal (JEE 2009, AIR 21)

8.2 Synthesis of Organometallic Compounds


Some important methods to generate metal–carbon bond as follows:
By the direct reaction of metals:
(a) n-Butyl lithium is prepared by the reaction of n-butyl bromide with lithium in ether.
Ether
n − C 4H9Br + 2Li  → n − C 4H9Li + 2LiBr
n −Butyl bromide n −Buthyl lithium

(b) Likewise, tetra ethyl lead can be prepared as follows:

4C2H5 Cl + 4NaPb → (C2H5 )4 Pb + 4NaCl + 3Pb


Sodium −lead
Tetra − ethyl lead
alloy

(c) Grignard reagents are obtained by the reaction of alkyl halide (in ether) with magnesium:
R
Mg + RX Mg
X
Grignard reagent

By using an alkylating agent: Grignard reagent and alkyl lithium or reaction with most of the metal and non-metal
halides in the presence of ether as solvent yield other organometallic compounds.
Chem i str y | 28.31

Ether
PCl3 + 3C6H5MgCl  → P(C6H5 )3 + 3MgCl2
Triphenyl phosphine

SnCl4 + 4n − C 4H9Li → (n − C 4H9 )Sn + 4LiCl


Tetrabutyl tin

Preparation of Metal Carbonyls:

(a) Nickel carbonyl is obtained when finely divided nickel reacts with CO at room temperature.
Ni + 4CO → Ni(CO)4(g) 

→ Ni + 4CO

The nickel carbonyl so formed is in gaseous state and its subsequent thermal decomposition gives Ni and CO. This
principle is used in the purification of Ni via Mond’s process.
(b) Iron carbonyl is formed when iron reacts with CO at high pressure and temperature.
Fe+5CO [Fe (CO)5]

PLANCESS CONCEPTS

Carbonyls containing only metal and CO are known as homoleptic carbonyls.

Nikhil Khandelwal (JEE 2009, AIR 94)

POINTS TO REMBEMBER

Coordination compound A compound containing central metal atom or ion bonded to a fixed number of ions or
molecules (called ligands).
Ligands The ions or molecules which donate a pair of electrons to the central metal atom or ion and
form coordinate bonds.
Coordination sphere The central metals atom or ion and the ligands attached to it is collectively called coordination
sphere.
Coordination number The number of coordinate bonds formed by the ligands with the metal atom i.e. number of
unidentate ligands or double the number of bidentate ligands, etc.
Denticity The number of coordinating or ligating atoms preset in a ligand
Chelation When a bidentate or a polydentate ligand uses it two or more donor atoms to bind to the
central metal atom or ion forming ring structure, it is called chelation
Coordination polyhedron The spatial arrangement of the ligands which are directly attached to the central metal atom.
Homoleptic complexes The complexes which contain only one type of ligands.
Structural isomers The isomers having same molecular formula but different structural arrangement of atoms or
group of atoms around the central metal ion.
Stereiosomers The isomers which have the same position of atom or group of atoms but they differ in the
spatial arrangement around the central metal atom.
Crystal field splitting The conversion of five degenerate d-orbitals of the metal ion into different sets of orbitals
having different energies in the presence of electrical field of ligands is called crystal field
splitting.
Spectrochemical series The arrangement of ligands in the increasing order of crystal field splitting
2 8. 32 | Co-ordination Compounds

Solved Examples

JEE Main/Boards (E) Hexaaquamanganese (II) ion.

Example 1: Which of the following will give white Example 5: Write the correct formula for the following
precipitate with AgNO3 solution? coordination compounds:
(A) [Co (py)2 (H2O)2Cl2]Cl (A) CrCl3·6H2O (violet, with 3 chloride ions/unit formula)
(B) [Co (py)2H2OCl3]H2O (B) CrCl3·6H2O (light green, with 2 chloride ions/unit
formula)
Sol: (a) will give white precipitate (C) CrCl3·6H2O (dark green, with I chloride ion/unit
[Co (py)2 (H2O)2Cl2] Cl Co (py)2 (H2O)2Cl2] +Cl
+ – formula)

AgNO3 + Cl –
AgCl + NO–3 Sol: (A) [Cr (H2O)6] Cl3
(B) [CrCl (H2O)5] Cl2·H2O
Example 2: Specify the oxidation numbers of the metals
in the following coordination entities: (C) [CrCl2 (H2O)4] Cl·2H2O]

(A) [Co (CN) (H2O) (en)2]2+


(B) [PtCl4]2– Example 6: Draw the structure of:
(C) [CrCl3 (NH3)3] (A) Cis-dichlorotetracyanochromate (III) ion.
(D) [CoBr2 (en)2] + (d) K3 [Fe (CN)6] (B) Mer-triamminetrichlorocobalt (III).
(C) Fac –triaquatrinitrito-N-cobalt (III).
Sol: (A) +3, (B) +2, (C) +3, (D) +3, (E) +3]
Sol: 3-
Example 3: Using IUPAC rules, write the formula for the CN
following: (A)
NC Cl

(A) Tetrahydroxozincate (II) ion Cr
(B) Hexaamminecobalt (III) sulphate NC Cl
(C) Potassium tetrachloropalladate (II) CN
(D) Potassium tri (oxalato) chromate (III)
(E) Diamminedichloroplatinum (II) NH3
H3N Cl
Sol: (A) [Zn (OH)4]2–
(B)
(B) [Co (NH3)6]2 (SO4)3 (C) K2 [PdCl4] Co
(D) K3 [Cr (C2O4)3] (E) [PtCl2 (NH3)2] Cl Cl
NH3
Example 4: Using IUPAC norms write the systematic
names of the following:
(A) [Co (NH3)6]Cl3 (B) [CoCl(NO2) (NH3) 4]Cl H2O
(C) [Ni (NH3)6]Cl2 (D) [PtCl (NH2CH3) (NH3)2]Cl (C) O2N H2O
(E) [Mn (H2O) 6] 2+
Co
Sol: (A) Hexaamminecobalt (III) chloride. O2N H2O
(B) Tetraamminechloronitrito-N-cobalt (III) chloride. NH3
(C) Hexaamminenickel (II) chloride.
(D) Diamminechloro (methylamine) platinum (II) chloride. Example 7: Name the type of isomerism exhibited by
Chem i str y | 28.33

the following isomers: JEE Advanced/Boards


(A) [Cr(NH3)6] [Cr(CN)6] and [Cr(NH3)4(CN)2] [Cr(NH3)2(CN)4]
Example 1: What is the coordination number of the
(B) [Co(py)2 (H2O)3Cl]Cl2 and [Co(py)2 (H2O)2Cl2] Cl·H2O
central metal ions in the following complexes?
(C) [Pt (NH3)4Br2] Cl2 and [Pt (NH3)4Cl2] Br2
(A) [Cu (NH3)4]2+ (B) [Fe (C2O4)3]3–
(D) [Co (NH3)5NO2] Cl2 and [Co (NH3)5ONO] Cl2
(C) [Pt (en)2Cl2] (D) [Mo (CN)8]4–
Sol: (A) Coordination isomerism (B) Hydrate isomerism (E) [Fe (EDTA)]– (F) [Pd (H2O)2 (ONO)2I2]
(C) Ionization isomerism (D) Linkage isomerism
Sol: (A) NH3 is a monodentate ligand, Coordination
number of Cu2+ = 4 × no. of electron pairs accepted =
Example 8: Draw the structure of geometrical isomers 4 × 1 = 4.
of [Pt (gly)2] where gly is NH2CH2COO–. 2−
(B) C2 O 4 is a bidentate ligand, coordination number of
CH2 NH2 CH2
NH2 CH Fe3+ = 3 × 2 = 6.
Sol:CH2 NH2 NH2 2

PtPt (C) ‘en’ is a bidentate ligand and Cl– is a monodentate


OC ligand, coordination number of
OC OO OO OC
OC
cis-isomer
cis-isomer (D) Mo4+ = 8 × 1 = 8.

and and (E) EDTA is a hexadentate ligand, coordination number


and
of Fe3+ = 6 × 1 = 6.
CH2 2
CH NH
NH OO OC
OC
(F) H2O, ONO and I2 are monodentate ligands,
2 2

PtPt
coordination number of Pd4+ = 2 × 1 + 2 × 1 + 2 × 1 = 6.
OC
OC OO HH2N C C
2N

trans-isomer
trans-isomer
Example 2: A solution containing 2.665 g of CrCl3·6H2O
Example 9: [Cr(NH3)6] is paramagnetic while [Ni(CN)4]
3+ 2– is passed through a cation exchanger. The chloride
is diamagnetic. Why? ions obtained in solution were treated with excess of
AgNO3 to give 2.87 g of AgCl. Deduce the structure of
Sol: Electronic structure of the two complexes may be compound.
written as
Sol: Moles of Cl– ions ionized from moles of CrCl3·6H2O
[Ni (CN) 4]2–:
dsp
2
2.665
= = 0.01 Mol. Wt. of CrCl3·6H2O = 266.5
2.665
3d 4s 4p

CN CN CN CN
∴ Moles of AgCl obtained = Moles of Cl– ionized
In [Cr(NH3)6]3+, all the electrons are paired and hence it 2.87
is diamagnetic. = = 0.02
2
143.5
dsp

Thus, 0.01 mole of complex CrCl3·6H2O gives 0.02 moles


3d 4s 4p

of Cl– on ionization.
NH3 NH3 NH3 NH3 NH3 NH3
Now, since the coordination number of Cr is 6 and
Due to presence of three unpaired electrons in only one Cl ion is attached to Cr by coordination bond
[Ni(CN)4]2–, it is paramagnetic.] or secondary valency, the compound is [CrCl· (H2O)5]
Cl2·H2O.
Example 10: [Cu(CN)4]2– is more stable complex than [CrCl·(H2O)5]Cl2·H2O [CrCl·(H2O)5]2+ + 2Cl– + H2O
[Cu(NH3)4]2+. Why?
2Cl– + 2AgNO3 2AgCl + 2NO–3
Sol: The higher stability constant K = 2× 10 for Cu 27 2+

+ 4CN– → [Cu (CN)4]2– than for [Cu(NH3)4]2+ (which is Example 3: Two compounds have empirical formula
4.5 × 1011) explains stability. Also CN– is stronger ligand corresponding to Cr(NH3)3(NO2)3. In aqueous solution,
than NH3.] one of these is a non-electrolyte while the other conducts
electricity. What is the lowest possible formula weight of
2 8. 34 | Co-ordination Compounds

the conducting reagent? What is the highest possible Example 7: The magnetic moment of [MnCl4]2– is
formula weight for the non-conducting reagent? 5.92 B.M. On the basis of its magnetic moment, write
configuration of Mn2+ in this complex.
Sol: For complex Cr (NH3)3· (NO2)3 (H2O)2, the two
structures can be written as: Sol: For an atom/ion

Cr (NO2 ) · (NH3 ) · (H2 O ) ·NO2 ·H2 O Magnetic moment (µ) = n(n + 2)
 2 3  and (n = No. of unpaired electrons)
A
Given that µ = 5.92 B.M.
Cr (NO2 ) · (NH3 ) ·2 H2 O
 3 3 ∴ 5.92 = n(n + 2) n=5
B
Thus is this complex Mn contains 5 unpaired electrons
A will be a conducting reagent as it will give two ions on and so its possible configuration may be
dissolving in water. B will be a non-conducting reagent
Mn2+ in [MnCl4]2– = [Ar] 3d54s0
as it will not be ionized in water.
So hybridization of Mn2+ in the given complex must be sp3.
Example 4: Write the IUPAC name of the compound 3d
5
4s 4p
[Cr(NH3)5(NCS)][ZnCl4]. Is this compound colored? Mn2+:
sp3 hybridization
Sol: Pentammineisothicyanatochromium (III)
tetrachlorozincate (II). Compound will be colored
because Cr (III) has d3 configuration and so d–d Example 8: [Ni (CN)4]2– is diamagnetic while [NiCl4]2– is
transition is possible. paramagnetic, Why?

Sol: In [Ni (CN)4]2– all orbitals are doubly occupied,


Example 5: On the basis of valence bond theory explain hence it is diamagnetic whereas in [NiCl4)]2–, two orbitals
geometry, nature of hybridization, magnetic property are singly occupied, hence it is paramagnetic in nature.
and optical isomerism in:
(i) [Co (Ox)3]3– (ii) [CoF6]3– [NiCl4]2–:
Rearrangement
Sol: (i) [Co (Ox)3]3– i.e., trioxalato cobaltate (III) ion has 3
sp hybridization
sp3d2 hybridization, having octahedral geometry and
is paramagnetic in nature due to the presence of 4 Strong field ligands like CN–, CO, en, NO–2 have very
unpaired electrons. It has 2 optical isomers. strong electron donating tendency, hence electrons of
central metal ion pair up against Hund’s rule (low spin
(ii) [CoF6]3– i.e., Hexafluoro cobaltate (III) ion has sp3d2 complex). In [Ni (CO)4], Ni is sp3 hybridized with no
hybridization, having octahedral geometry and is unpaired electron, and hence it is diamagnetic.
paramagnetic in nature due to the presence of 4
unpaired electron. It does not show optical isomerism.
Example 9: What is the coordination entity formed
when excess of KCN is added to an aqueous solution
Example 6: The EAN of each Mn (Z = 25) in Mn2 (CO)10 of CuSO4? Why is that no precipitate of CuS is obtained
is 36. What is the structure of this complex? when H2S(g) is passed through this solution?

Sol: EAN = 25 (electrons from Mn atom) + 10 (electrons Sol: CuSO4 + 4KCN K2[Cu(CN)4] + K2SO4
from five CO ligand) + 1 (electron from Mn—Mn bond)
[Cu (CN)4]2– is stable complex having
= 36
Thus, structure will be, K = 2 × 1027 and so it does not provide sufficient Cu2+
CO CO CO CO
ion to give precipitate of CuS.
OC OC
Mn Mn Example 10: What do you understand by macrocyclic
CO CO CO CO
effect?

Sol: It is the increased thermodynamic stability of a


complex formed with a cyclic polydentate ligand when
compared to the complex formed with a noncyclic ligand,
Chem i str y | 28.35

e.g. Zn2+ complex with (I) is more stable than with (II).

NH NH NH NH
is
NH NH NH2 NH2

(II) Non-cyclic
(I) Cyclic

JEE Main/Boards

Exercise 1 Q.8 [Cr (NH3)6]3+ is paramagnetic while [Ni (CN)4]2– is


diamagnetic. Explain why?
Q.1 Explain the bonding in coordination compounds in Q.9 A solution of [Ni (H2O)6]2+ is green but a solution of
terms of Werner’s postulates. [Ni (CN)4]2– is colorless. Explain.

Q.2 FeSO4 solution mixed with (NH4) 2SO4 solution in 1:


Q.10 [Fe (CN)6]4– and [Fe (H2O)6]2+ are of different colors
1 molar ratio gives the test of Fe2+ ion but CuSO4 solution in dilute solutions. Why?
mixed with aqueous ammonia in 1: 4 molar ratio does
not give the test of Cu2+ ion. Explain why?
Q.11 Give the oxidation state, d orbital occupation and
coordination number of the central metal ion in the
Q.3 How many geometrical isomers are possible in the
following complexes:
following coordination entities?
(i) K3 [Co (C2O4)3] (ii) (NH4)2 [CoF4]
(i) [Cr (C2O4)3]3– (ii) [PtCl2 (en)2]2+
(iii) cis- [Cr (en)2Cl2] Cl (iv) [Mn (H2O)6] SO4
(iii) [Cr (NH3)2Cl2 (en)]

Q.12 Write down the IUPAC name for each of the


Q.4 Draw the structures of optical isomer of:
following complexes and indicate the oxidation state,
(i) [Cr (C2O4)3]3– (ii) [PtCl2 (en)2]2+ electronic configuration and coordination number. Also
(iii) [Cr (NH3)2Cl2 (en)] give stereochemistry and magnetic moment of the
complex:

Q.5 Draw all the isomers (geometrical and optical) of: (i) K [Cr (H2O)2 (C2O4)2].3H2O

(i) [CoCl2 (en)2]+ (ii) [Co (NH3) Cl (en)2]2+ (ii) [Co (NH3)4Cl] Cl2

(iii) [Co (NH3)2Cl2 (en)] (iii) CrCl3 (py)3


(iv) Cs [FeCl4]
Q.6 Aqueous copper sulphate solution (blue in color)
(v) K4 [Mn (CN)6]
gives:
(i) A green precipitate with aqueous potassium fluoride and
Q.13 What is meant by the chelate effect? Give an
(ii) A bright green solution with aqueous potassium example.
chloride. Explain these experimental results.
Q.7 What is the coordination entity formed when excess of Q.14 Amongst the following ions which one has the
aqueous KCN is added to an aqueous solution of copper highest magnetic moment value?
sulphate? Why is it that no precipitate of copper sulphide
(i) [Cr (H2O6)]3+ (ii) [Fe (H2O)6]2+
is obtained when H2S (g) is passed through this solution?
(iii) [Zn (H2O)6]2+
2 8. 36 | Co-ordination Compounds

Q.15 What will be the correct order for the wavelengths Q.7 Which type do d-electron configuration exhibit
of absorption in the visible region for the following? both low and high spin in octahedral complexes?
[Ni (NO2)6]4–, [Ni (NH3)6]2+, [Ni (H2O)6]2+? (A) d1 (B) d4 (C) d3 (D) d2

Q.16 Why NH4+ ion does not form complexes? Q.8 Transition elements have the maximum tendency to
form complexes because
Q.17 Write the formula of the following Complex. (A) They are metals and all metal form complexes
Pentaamminechlorocobalt (III) ion (B) They contain incompletely filled d-orbitals
(C) Their charge/size ratio is quite large
Exercise 2 (D) Both (B) and (C)
Q.9 [(H2 O)5 Co – O – O – Co(H2 O)5 ]4 +
Single Correct Choice Type ( A)


Q.1 Which of the following compound is not having
[(H2 O)5 Co – O – O – Co(H2 O)5 ]5 +
synergic bonding? (B)

(A) Fe (CO)5 (B) [Ni (CN)4] 2–

What will be the bond length of O–O in complex A and B?


(C) [Fe (π–C5H5)2] (D) [CoF6]3–
(A) A = B (B) A > B (C) B > A (D) None
Q.2 Which of the following is a low spin complex?
Q.10 In the complex of [Ma3b3]n±, if two ‘a’ are replaced
(A) Ni (CO)4 (B) [Ni (NH3)6]2+
by AA type of ligand, then isomer of it, only one isomer
(C) [Cu (NH3)4]2+ (D) All of these of the product will be formed.
(A) Only Fac-isomer
Q.3 Which of the following name is incorrect for the
(B) Only Mer-isomer
given complex. K [Fe(H2O)2(NCS)3(NO3)]
(C) Fac and Mer-isomer both
(A) Potassium diaquatrisothiocyanatonitrato ferrate (III)
(D) None of these
(B) Potassium diaquanitratotrithiocyanato-N- ferrate (III)
(C) Potassium diaquatrithiocyanato-N-nitrito ferrate (III)
Q.11 What is the hybridization of Fe in sodium
(D) Potassium diaquatrisothiocyanatonitrato ferrate (II) thionitroprusside?
(A) sp3d2 (B) d2sp3
Q.4 What is the hybridization of Fe in [Fe (CO) 4]?
(C) sp3d (D) No hybridization
(A) sp3 (B) dsp2 (C) sp (D) sp3d2

Q.12 What is correct name of linkage isomer of [Cr (H2O)5


Q.5 Which of the following complex/molecule acts as a (NO2)] Br2.
reducing agent?
(A) Pentaaquanitro-‘O’ chromium (III) bromide
(A) [Co(NH3)6]2+ (B) [Mn (CO)6]
(B) Pentaaquanitro chromium (III) bromide
(C) NO (D) All are acts as a Reducing
(C) Pentaaquonitro chromium (III) bromide
(D) Pentaaquanitrito-‘O’’ chromium (II) bromide
Q.6 Which of the following molecule is square planar.
(A) K4 [Ni (CN)4] (B) [Be (acac)2] Q.13 The effective atomic number of Co (CO)4 is 35. It
does not attain stability by
O
(C) B (D) Pd (NH3 )(H2 O )(Br ) Cl º (A) Oxidation of [Co (CO)4]
O 2
(B) Reduction of [Co (CO)4]
(C) Dimerization of [Co (CO)4]
(D) By both (B) and (C)
Chem i str y | 28.37

Q.14 Which one of the following ions is colored? Previous Years’ Questions
(A) Sc3+ (B) Ti4+ (C) Zn2+ (D) V2+
Q.1 The coordination number of a central metal atom in
a complex is determined by  (2004)
Q.15 Which of the following statement is correct.
(A) The number of ligands around a metal ion bonded
(A) The name of [Fe (H2O)5NO] SO4 is
by sigma and pi-bonds both.
pentaaquanitrosyliron (II) sulphate
(B) The number around a metal ion bonded by pi-bonds
(B) [Co(C2O4)3]3– does not have any unpaired electron
(C) The number of ligands around a metal ion bonded
(C) E.A.N. of [Fe (CO)2(NO)2] is 34 by sigma bonds
(D) [SnCl3]– acts as ligand where the Cl-atom as donor (D) The number of only anionic ligands bonded to the
atom. metal ion
Q.16 How many stereoisomer are possible for complex Q.2 Among the properties (i) reducing (ii) oxidizing (iii)
[Co(NH3)3Cl3] complexing, the set of properties shown by CN– ion
(A) 1 (B) 2 (C) 3 (D) 4 towards metal species is  (2004)
(A) iii, i (B) ii, iii (C) i, ii (D) i, ii, iii
Q.17 In the volumetric analysis, Mohr’s salts is preferred
to ferrous sulphate because Q.3 Ammonia forms the complex ion [Cu (NH3)4]2+
(A) It is more readily oxidizable than FeSO4 with copper ions in alkaline solutions but not in acidic
solution. What is the reason for it?  (2003)
(B) It is less easily oxidizable than FeSO4
(C) It can undergo both oxidation and reduction (A) In acidic solutions hydration protects copper ions
(D) It can be oxidized even in the absence of dilute H2SO4. (B) In acidic solutions protons coordinate with ammonia
molecules forming NH4+ ions and NH3 molecules are
not available
Q.18 [RhF6]5– complex ion is a
(C) In alkaline solutions insoluble Cu(OH)2 is precipitated
(A) Outer orbital complex which is soluble in excess of any alkali
(B) Inner orbital complex (D) Copper hydroxide is an amphoteric substance
(C) No outer or no inner orbital complex
(D) None of these Q.4 In the coordination compound, K4[Ni(CN)4] oxidation
state of nickel is  (2003)
Q.19 Which of the following complex are tetrahedral in (A) –1 (B) 0 (C) +1 (D) +2
shape.
(A) [FeCl4]– (B) [BrF4]– Q.5 The coordination number and the oxidation state of
(C) [Cu(CN)4]3– (D) [AuCl4]– the element ‘E’ in the complex [E(en)2(C2O4)]NO2 (where
(en) is ethylene diamine) are, respectively  (2008)
Q.20 How many ions are produced from the complex (A) 4 and 2 (B) 4 and 3
Co (NH3)6Cl2 in solution? (C) 6 and 3 (D) 6 and 2
(A) 6 (B) 4 (C) 3 (D) 2
Q.6 Which among the following will be named as
Q.21 The oxidation number of cobalt in K [Co (CO)4] is dibromidobis (ethylene diamine) chromium (III)
bromide  (2012)
(A) +1 (B) +3 (C) –1 (D) –3
(A) [Cr (en)3] Br3 (B) [Cr (en)2Br2] Br
Q.22 Amongst the following the most stable complex is (C) [Cr (en) Br4]– (D) [Cr (en)Br2] Br
(A) [Fe (H2O6)]3+ (B) [Fe (NH3)6]3+
(C) [Fe (C2O4)3]3– (D) [FeCl6]3–
2 8. 38 | Co-ordination Compounds

Q.7 The “spin-only” magnetic moment [in units of Bohr Q.15 In Fe (CO)5, the Fe-C bond possesses (2006)
magneton, (µB)] of Ni2+ in aqueous solution would be
(A) π-character only (B) Both σ and π characters
(At. No. Ni = 28)  (2006)
(C) Ionic character (D) σ-character only
(A) 2.83 (B) 4.90 (C) 0 (D) 1.73

Q.16 The coordination number and the oxidation state


Q.8 The pair in which both species have same magnetic
moment (spin only value) is  (2006) of the element ‘E’ in the complex E ( en) ( C2O 4 )  NO2
 2 
(A) [Cr (H2O)6]2+, [CoCl4]2– (B) [Cr (H2O)6]2+, [Fe (H2O)6]2+ (where (en) is ethylene diamine) are respectively, (2008)

(C) [Mn (H2O6)]2+, [Cr (H2O)6]2+ (D) [CoCl4]2–, [Fe (H2O)6]2+ (A) 6 and 2 (B) 4 and 2
(C) 4 and 3 (D) 6 and 3
Q.9 Which of the following has an optical isomer? (2009)
(A) [Co (NH3)3Cl]+ (B) [Co (en) (NH3)2]2+ Q.17 Being lesser energy difference between 5f and
6d than 4f and 5d orbitals. In which of the following
(C) [Co (H2O)4 (en)]3+ (D) [Co (en)2 (NH3)2]3+ octahedral complexes of Co (at no. 27), will the
magnitude of be the highest? (2008)
Q.10 Which of the following pairs represents linkage
3− 3−
isomers ?  (2009) (A) Co ( CN)  (B) Co ( C2O 4 ) 
 6  3
(A) [Cu (NH3)4] [PtCl4] and [Pt (NH3)4] [CuCl4]
3+ 3+
(B) [Pd (P Ph3)2 (NCS)2] and [Pd (P Ph3)2(SCN)2] (C) Co (H2O )  (D) Co (NH3 ) 
 6  6

(C) [Co(NH3)5NO3]SO4 and [Co (NH3)5SO4]NO3


(D) [PtCl2 (NH3)4] Br2 and [Pt Br2 (NH3)4] Cl2 Q.18 Which of the following has an optical isomer?
(2009)

Q.11 Which one of the following has a square planner + 2+


(A) Co (NH3 ) Cl (B) Co ( en)(NH3 ) Cl
geometry?  (2007)  3   2 

(A) [CoCl4]2– (B) [FeCl4]2– (C) [NiCl4]2– (D) [PtCl4]2– 3+ 3+


(C) Co (H3O ) ( en)  (D) Co ( en) (NH3 ) 
 4   2 2

Q.12 Which of the following facts about the complex


[Cr (NH3)6] Cl3 is wrong  (2011) Q.19 Which of the following pairs represents linkage
isomers? (2009)
(A) The complex involves d2sp3 hybridization and is
octahedral in shape
 ( 4  )
(A)  Cu (NH3 )  PtlCl4  and Pt (NH3 )  CuCl4 
(B) The complex is paramagnetic
(C) The complex is an outer orbital complex (B) Pd (PPh3 )(NCS )  and Pd (P Ph2 ) ( SCN2 ) 
 2  2 
(D) The complex gives white precipitate with silver nitrate
solution (C) Co (NH3 ) NO3  SO 4 and Co (NH3 ) SO 4  NO3
 5   5 

(D) PtCl2 (NH3 )  Be2 and PtBr2 (NH3 )  Cl2


Q.13 One mole of the complex compound Co (NH3)5Cl3,  4  4
gives 3 moles of ions on dissolution in water. One mole
of the same complex reacts with two moles of AgNO3 Q.20 A solution containing 2.675 g of CoCl3.6 NH3
solution to yield two moles of AgCl (s). The structure of (molar mass = 267.5 g mol-1) is passed through a cation
the complex is  (2003) exchanger. The chloride ions obtained in solution were
treated with excess of AgNO3 to give 4.78 g of AgCl
(A) [Co (NH3)5Cl] Cl2 (B) [Co (NH3)3Cl3].2NH3
(molar mass = 143.5 g mol‒1). The formula of the complex
(C) [Co (NH3)4Cl2] Cl.NH3 (D) [Co (NH3)4Cl] Cl2.NH3 is (At. Mass of Ag = 108 u) (2010)

(A) Co ( NH3 )6  Cl3 (B) CoCl2 ( NH3 )4  Cl


Q.14 How many EDTA (ethylenediaminetetraacetic acid)
molecules are required to make an octahedral complex
with a Ca2+ion  (2006) (C) CoCl3 ( NH3 )3  (D) CoCl ( NH3 )5  Cl2
(A) Six (B) Three (C) One (D) Two
Chem i str y | 28.39

Q.21 Which one of the following has an optical Q.26 The octahedral complex of a metal ion M3+ with
isomer? (2010) four monodentate ligands L1, L2, L3 and L4 absorb
2+ wavelengths in the region of red, green, yellow and blue,
(A)  Zn ( en ) ( NH3 )2  (B) [Co ( en )3 ]3+ respectively. The increasing order of ligand strength of
the four ligands is (2014)
3+
(C) Co ( H2O )4 ( en )  (D) [ Zn ( en )2 ]2+ (A) L4 < L3 < L2 < L1 (B) L1 < L3 < L2 < L4
(C) L3 < L2 < L4 < L1 (D) L1 < L2 < L3 < L4

Q.22 Which of the following facts about the complex Q.27 The number of geometric isomers that can exist
Cr (NH )  Cl wrong? (2011)
for square planar Pt ( Cl)(py )(NH3 )(NH2OH)  is
+
 3 6 3
 
(A) The complex is paramagnetic (py = pyridine): (2015)
(B) The complex is an outer orbital complex (A) 2 (B) 3 (C) 4 (D) 6
(C) The complex gives white precipitate with silver
nitrate solution Q.28 Which of the following compounds is not colored
(D) The complex involves d sp hybridization and is
2 3 yellow? (2015)
octahedral in shape.
(A) Zn2 Fe ( CN)  (B) K 3 Co (NO2 ) 
 6  6

(C) (NH4 )3  As (Mo3O10 ) 4 


2−
Q.23 The magnetic moment (spin only) of NiCl4  is (D) BaCrO 4
(2011)
(A) 5.46 BM (B) 2.83 BM (C) 1.41 BM (D) 1.82 BM
Q.29 The pair having the same magnetic moment is:
[At. No.: Cr = 24, Mn = 25, Fe = 26, Co = 27] (2016)
Q.24 Which among the following will be named
as dibromidobis (ethylene diamine) chromium (III) 2+ 2+
(A) Cr (H2O )  and Fe (H2O ) 
bromide? (2012)  6  6

2+
(B) Mn (H2O ) 
2+
(A) Cr ( en)  Br3 (B) Cr en Br2  Br
 3  2  ( )  6
and Cr (H2O ) 
 6

2+
(C) Cr ( en) Br4  ( )

and Fe (H2O ) 
2−
(D) Cr en Br2  Br (C) CoCl4 
   6
Q.25 Which of the following complex species in not 2+
(D) Cr (H2O ) 
2−
expected to exhibit option isomerism? (2013) and CoCl4 
 6

3+ +
(A) Co ( en)3  (B) Co ( en)2 Cl2  Q.30 Which one of the following complexes shows
optical isomerism? (2016)
+
(C) Co (NH3 )3 Cl3  (D) Co ( en)2 (NH3 )2 Cl2  (A) cis Co ( en) Cl2  Cl (B) trans Co ( en)2 Cl2  Cl
 2 
(C) Co (NH3 ) 4 Cl2  Cl (D) Co (NH3 )3 Cl3 
2 8. 40 | Co-ordination Compounds

JEE Advanced/Boards

Exercise 1 Q.11 Square planar complexes with co-ordination


number four exhibit geometrical isomerism whereas
Q.1 Explain with two examples each of the following: tetrahedral complexes does not. Why?
coordination entity, ligand, coordination number,
coordination polyhedron, homoleptic and heteroleptic. Q.12 Determine the oxidation state of metal in the
complex ion, [PtCl6]2–?
Q.2 What is meant by unidentate, bidentate and
ambidentate ligands? Give two examples for each. Q.13 The color of Hg2Cl2 changes from white to black
when treated with NH3. Why?
Q.3 (A) What is the basis of formation of spectrochemical
series? Q.14 [Fe (CN)6]3– is weakly paramagnetic while [Fe(CN)6]4–
(B) Draw the structure of geometrical isomers of the is diamagnetic, why?
following coordination complexes: [Co(NH3)3Cl3] and
[CoCl2 (en)2]+ Q.15 The magnetic moment of [MnCl4]2– is 5.92 B.M. On
the basic of its magnetic moment, write configuration of
Q.4 Write all the geometrical isomers of [Pt (NH3) (Br) Mn2+ in this complex.
(Cl) (py)] and how many of these will exhibit optical
isomers? Q.16 List various types of isomerism possible for
coordination compounds, giving an example of each.
Q.5 What is meant by stability of a coordination Q.17 Discuss the nature of bonding in the following
compound in solution? State the factors which govern coordination entities on the basis of valence bond
stability of complexes. theory.
(A) [Fe (CN)6]4– (B) [FeF6]3–
Q.6 Draw a sketch to show the splitting of d-orbitals
in an octahedral crystal filed. State for a d5 ion how (C) [Co (C2O4)3]3– (D) [CoF6]3–
the actual configuration of the split d-orbitals in an
octahedral crystal field is decided by the relative values Q.18 Draw figure to show the splitting of d orbital in an
of D and P. octahedral crystal field.

Q.7 Writ the IUPAC name of [Co (en)2Cl2] Cl and draw Q.19 What is spectrochemical series? Explain the
the structures of all the isomers with this formula of difference between a weak field ligand and a strong
complex. field ligand.

Q.8 Draw diagram to show crystal field splitting in a Q.20 What is crystal field splitting energy? How does
tetrahedral crystal field the magnitude of ∆o decide the actual configuration of
d orbital in a coordination entity?

Q.9 Discuss briefly giving an example in each case the


Q.21 Discuss the nature of bonding in metal carbonyls.
role of coordination compounds in:
Q.22 Specify the oxidation numbers of the metals in the
(i) Biological systems
following coordination entities and geometry of these
(ii) Medicinal chemistry compound?
(iii) Analytical chemistry and
(i) [Co (H2O) (CN) (en)2]2+ (ii) [CoBr2(en)2]+
(iv) Extraction/metallurgy of metals.
(iii) [PtCl4]2– (iv) K3 [Fe (CN)6]
(v) [Cr (NH3)3Cl3]
Q.10 NH2.NH2 although possesses two electron pair for
donation but not acts as chelating agent. Why?
Chem i str y | 28.41

Q.23 Using IUPAC norms write the formulas for the


following and explain all property according to VBT? Q.4 The effective atomic number of Co(CO)4 is 35. It
(1) Tetrahydroxozincate (II) does not attain stability by
(2) Potassium tetrachloridopalladate(II) (A) Oxidation of [Co(CO)4] (B) Reduction of [Co(CO)4]
(3) Diamminedichloridoplatinum (II) (C) Dimerization of [Co(CO)4] (D) By both (B) and (C)
(4) Potassium tetracyanonickelate (II)
(5) Pentaamminenitrito-O-cobalt (III) Q.5 In the volumetric analysis, Mohr’s salt is preferred to
(6) Hexaamminecobalt (III) sulphate ferrous sulphate because
(7) Potassium tri (oxalate) chromate (III) (A) It is more readily oxidizable than FeSO4
(8) Hexaammineplatinum (IV) (B) It is less easily oxidizable than FeSO4
(9) Tetrabromideocuprate (II) (C) It can undergo both oxidation and reduction
(10) Pentaamminenitrito-N-cobalt (III) (D) It can be oxidized even in the absence of dilute H2SO4

Q.24 Write the difference between VBT and CFT?


Multiple Correct Choice Type

Q.25 Write the formulas for the following coordination Q.6 Select the correct statement.
compounds:
(A) CO bond order in bridging carbonyl group is found
(i) Tetraaminequachloridocobalt (III) chloride to be lower than in that terminal carbonyl group in
(ii) Potassium tetrahydroxozincate (II) Fe2(CO)9.
(iii) Potassium trioxalatoaluminate (III) (B) Bond angle in NO2 < Bond angle in NO2–
(iv) Dichloridobis (ethane-1, 2-diamine) cobalt (III) (C) CO is a σ donor as well as π acceptor ligand.
(v) Tetracrabonylnickel (0). (D) NO is the only σ donor ligand.

Exercise 2 Q.7 Which of the following complex is/are square planar


as well as paramagnetic?
Single Correct Choice Type (A) [AuCl4]– (B) [CuCl4]–

Q.1 Select the correct order of C—O bond order in (C) [Cu (NH3)4]2+ (D) [Co (dmg)2]º
following complexes.
(A) [M (CO3(PCl3)2(PMe3)] > [M(CO)3(PCl3)3] > [M(CO)3(PCl3) Q.8 Which one of the following statements is incorrect
(PMe3)2] about transition elements?
(B) [M(CO)3(PCl3)2(PMe3)2] > [M(CO)3(PCl3)3] > [M(CO)3 (A) The last electron enters in the d-orbital
(PCl3)2(PMe3)]
(B) Their properties are in between those of s-block
(C) [M(CO)3(PCl3)(PMe3)2] = [M(CO)3(PCl3)3] < [M(CO)3 elements and p-block elements
(PCl3)2(PMe3)] (C) The transition element with smallest atomic number
(D) [M (CO)3(PCl3)3] > [M(CO3(PCl3)2(PMe3)] > [M(CO)3 is scandium
(PCl3)(PMe3)2] (D) Lanthanum is not a transition element but is a
lanthanide.

Q.2 What is the hybridization of Fe in [Fe (CO)4]?


Q.9 Which of the following type of complex (s) is/are
(A) sp3 (B) dsp2 (C) sp (D) sp3d2 having six number of stereoisomers.
(A) [MA2B2C2]n± (B) [M (AB) A2BC]n±
Q.3 Which of the following complex/molecule acts as a
reducing agent. (C) [M (AA) A2B2]n± (D) [M (AB) A2B2]n±

(A) [Co(NH3)6]2+ (B) [Mn(CO)6]


(C) NO (D) All act as a reducing agent
2 8. 42 | Co-ordination Compounds

Q.10 Which of the following complex (s) is/are low spin Q.17 Which of the following ligand can show linkage
isomerism?
(A) K4 [Fe (CN)6] (B) [PtCl4]2–
(C) [Co (C2O4)3]
3–
(D) [Ni (NH3)6]2+ (A) NMe3 (B) O CN (C) SO42– (D) None of these

Q.11 Which of the following molecules have same Q.18 Which of the following ligand can show linkages
number of unpaired electrons. isomerism?
(A) [Cu (NH3)6]2+ (B) [Ti (H2O)6]3+ (A) It increase the stability of complex.
(C) K4 [Fe (CN)6O2] (D) [Cr (NH3)6]3+ (B) It increase the strength of metal–ligand bond.
(C) More negative charge on the central atom causes
Q.12 Which of the following type of complex (s) do/ more extent of synergic bonding.
does not produce optically active isomer (s). (D) All of these
(A) MA2B2C2 (B) MA4B2
(C) MA3B2C (D) MA2B2CD Q.19 Choose the incorrect statement from the following
options.

Q.13 For which of the complex, the E.A.N. of the central (A) C2H2 can acts as π-acid ligand
atom of the complex obeys Sidgwick E.A.N. rule. (B) [MA3B2C]n±: All isomers are optically inactive.
(A) [Ti (σ–C5H5)5]2 (B) [Fe(NO)2(CO)2] (C) [MABCDE2]n± complex has total 12 optically active
isomer
(C) [Ag(CN)2]– (D)[Hg(SCN)4]2–
(D) All statements are incorrect.

Q.14 Identify the ligands can cause the linkage isomerism.


Paragraph 2: Metals from complexes with various lewis
(A) NO3– (B) NO2– (C) OCN– (D) S2O32– bases called ligands, more the basic character better is
the complex. Electron donating tendency decreases with
increase in electronegativity of donor atoms in a period.
Assertion Reasoning Type Higher the bond order of metal ligand bond more is
the stability of complex. Geometry and symmetry also
(A) Statement-I is true, statement-II is true and influence the stability of complex. More symmetricity of
statement-II is correct explanation for statement-I. the complex, more is the stability.
(B) Statement-I is true, statement-II is true and
statement-II is not the correct explanation for
Q.20 Little Mohr’s salt was dissolved in a water sample
statement-I.
containing CO32–, Cl–, CN– and OH– in minute amounts.
(C) Statement-I is true, statement-II is false. Fe2+ prefers formation of complex with
(D) Statement-I is false, statement-II is true.
(A) CO32– (B) Cl– (C) CN– (D) OH–

Q.15 Statement-I: [Cr(NH3)6]3+ has three unpaired


Q.21 Fe2+ forms diamagnetic complex with either of
electron in t2g orbital
CO32–, Cl–, CN– or OH–. Geometry of the complex formed
Statement-II: [Cr(NH3)6]3+ does not show geometrical is supposed to be
isomerism.
(A) Tetrahedral (B) Triangular bipyramidal
(C) Octahedral (D) Square planar
Q.16 Statement-I: The square planar complex, [Mabcd]n+
type shows geometrical isomerism.
Q.22 Least stable complex is
Statement-II: Restricted rotation around the single
(A) Mn (CO)5 (B) K [V (CO)5]
bond is present within the above complex.
(C) K [Fe (NC)6]
4–
(D) [Fe (NC)6]4–
Comprehension Type

Paragraph 1: Isomerism is a phenomenon where the


molecules are having same formula but they have
different structures.
Chem i str y | 28.43

Match the Columns Q.27

Q.23 Column I Column II


(A) [Pt (NH3)5Cl] Cl3 (p) 229
Column I Column II
(B) [Pt (NH3)4Cl2] Cl2 (q) 9
7
(A) Na [Co (CO)4] Pt (p) C
 omplex having only
monodentate ligand (C) [Pt (NH3)3Cl3] Cl (r) 404
(B) (Trien) Cl2 (q) C
 omplex not following Sidgwick
EAN rule. (D) [Pt (NH3)6] Cl3 (s) 523
(C) [Fe (H2O)5NO] SO4 (r) C
 omplex is having 3 or 5
member of ring within it.
(D) [Fe (EDTA)]– (s) Complex having positively Previous Years’ Questions
charged ligand
(t) Complex having chelating ligand Q.1 Amongst the following, the lowest degree of
paramagnetism per mole of the compound at 298 K will
be shown by (1988)
Q.24
(A) MnSO4·4H2O (B) CuSO4·5H2O
Column I Column II
(C) FeSO4·6H2O (D) NiSO4·6H2O
(A) [Co(CO)4]– (p) E.A.N ≠ 36

(B) [Zn(gly)2] (q) S


 ynergic bonding is involved Q.2 Which of the following is formed when excess of
KCN is added to aqueous solution of copper sulphate?
(C) [Ag(CN)2]– (r) T
 wo optically active isomer
(1996)
(s) The complex is diamagnetic (A) Cu(CN)2 (B) K2[Cu(CN)4]
(C) K[Cu(CN)2] (D) K3[Cu(CN)4]
Q.25
Q.3 The complex ion which has no ‘d’ –electrons in the
Column I Column II central metal atom is (2001)
(A) [M(AA)3] (p) T
 wo pair of enantiomer (A) [MnO4]– (B) [Co (NH3)6]3+
(B) [M(AB)B2C2] (q) F our geometrical isomer (C) [Fe (CN)6]3– (D) [Cr (H2O)6]3+
(C) [M(AB)3] (r) T
 wo optically inactive isomer
Q.4 The compound having tetrahedral geometry is
(D) [MA3BCD] (s) Two optically active isomer
(2004)
(t) A
 t least one stereoisomer is
planar
(A) [Ni(CN)4]2– (B) [Pd(CN)4]2– (C) [PdCl4]2– (D) [NiCl4]2–

Note: AA,AB, a, b, c, d are not having chiral center. Q.5 The IUPAC name of [Ni (NH3)4] [NiCl4] is (2008)
(A) Tetrachloronickel (II)-tetraamminenickel (II)
Q.26
(B) Tetraamminenickel (II)-tetrachloRonickel(II)
Column I Column II
(C) Tetraamminenickel (II)-tetrachloronickelate (II)
(A) Zn [Fe (CN)5NO] (p) Blue color due to charge transfer
(D) Tetrachloronickel (II)-tetraamminenickelate (0)
(B) [Fe (H2O)5NO] SO4 (q) d2sp3 hybridization

(C) Fe4 [Fe (CN)6]3 (r) P


 aramagnetic compound
Q.6 Geometrical shapes of the complexes formed by
the reaction of Ni2+ with Cl–, CN– and H2O, respectively,
(D) K4[Fe(CN)5O2] (s) NO acts as positive ligand are (2011)
(t) C
 harge transfer metal to metal in (A) Octahedral, tetrahedral and square planar
complex.
(B) Tetrahedral, square planar and octahedral
(C) Square planar, tetrahedral and octahedral
(D) Octahedral, square planar and octahedral
2 8. 44 | Co-ordination Compounds

Q. 7 Among the following metal carbonyls, the C—O Q.15 NiCl2 in the presence of dimethyl glyoxime (DMG)
bond order is lowest in (2007) gives a complex which precipitates in the presence of
NH4OH, giving a bright red color. (2004)
(A) [Mn(CO)6]+ (B) [Fe(CO)5] (C) [Cr(CO)6] (D) [V(CO)6]–
(A) Draw its structure and show H-bonding
Q.8 Statement-I: The geometrical isomers of the (B) Give oxidation state of Ni and its hybridization
complex [M(NH3)4Cl2] are optically inactive.
(C) Predict whether it is paramagnetic or diamagnetic
Statement-II: Both geometrical isomers of the complex
[M(NH3)4Cl2] possess axis of symmetry. (2008)
Q.16 Statement-I: Fe (H2O )5 NO  SO 4 is paramagnetic
(A) Statement-I is true, statement-II is true and
statement- II is correct explanation for statement-I. Statement-II: The Fe in Fe (H2O )5 NO  SO 4 has three
unpaired electrons. (2008)
(B) Statement-I is true, statement-II is true and
statement-II is not the correct explanation for (A) Statement-I is true, statement-II is true; statement-II
statement-I. is correct explanation for statement-I
(C) Statement-I is true, statement-II is false. (B) Statement-I is true, statement-II is true; statement-II
(D) Statement-I is false, statement-II is true is NOT a correct explanation for statement-I
(C) Statement-I is true, statement-II is false
Q.9 Match the Columns (2007) (D) Statement-I is false, statement-II is true

Match the complexes in column I with their properties


listed in column II. Q.17 As per IUPAC nomenclature, the name of the
complex is Co (H2O ) (NH3 ) Cl3  (2012)
Column-I Column-II
 4 2 

(A) [Co(NH3)4(H2O)2]Cl2 (p) G


 eometrical isomers (A) Tetraaquadiaminecobalt (III) chloride

(B) [Pt(NH3)2 Cl2] (q) Paramagnetic (B) Tetraaquadiamminecobalt (III) chloride


(C) Diaminetetraaquacobalt (III) chloride
(C) [Co(H2O)5Cl2] (r) Diamagnetic
(D) Diamminetetraaquacobalt (III) chloride
(D) [Ni(H2O)6]Cl2 (s) M
 etal ion with+2 oxidation state

Q.18 Consider the following complex ions, P, Q and R


Q.10 Total number of geometrical isomers for the
3− 2+ 2+
complex [RhCl (CO) (PPh3) (NH3)] is (2010)
= FeF6  ,Q  V (H2O ) =
P = and R Fe (H2O ) 
 6  6

The correct order of the complex ions, according to their


Q.11 The volume (in mL) of 0.1 M AgNO3 required for
spin–only magnetic moment values (in B.M.) is (2013)
complete precipitation of chloride ions present in 30 mL
of 0.01 M solution of [Cr (H2O)5Cl] Cl2, as silver chloride (A) R < Q < P (B) Q < R < P
is close to …… (C) R < P < Q (D) Q < P < R

Q.12 Identify the complexes which are expected to be


Q.19 The pair(s) of coordination complexes/ions
colored. Explain (1994)
exhibiting the same kind of isomerism is(are) (2013)
(i) [Ti (NO3)4] (ii) [Cu (NCCH3)] + BF4
(iii) [Cr (NH3)6] Cl3 (iv) K3 [VF6] (A) Cr (NH3 ) Cl Cl2 and Cr (NH3 ) Cl Cl
 5   4 
+ +
(B) Co (NH3 ) Cl2  and Pt (NH3 ) (H2O ) Cl
Q.13 Draw the structures of [Co (NH3)6]3+, [Ni (CN)4]2–  4   2 
and [Ni (CO)4]. Write the hybridization of atomic orbitals 2− 2−
of the transition metal in each case. (2000) (C) CoBr2Cl2  and PtBr2Cl2 

(D) Pt (NH3 ) (NO3 )  Cl and Pt (NH3 ) Cl Br


Q.14 Write the IUPAC name of the compound K2 [Cr (NO)  3   3 
(CN)4(NH3)]. Spin magnetic moment of the complex
µ = 1.73 BM. Give the structure of anion. (2003)
Chem i str y | 28.45

Q.20 Match the orbital overlap figures shown in list-I Q.22 Among the complex ions,
with the description given in list-II and select the correct + 3−
answer using the code given below the lists. (2014) Co (NH2 − CH2 − CH2 − NH2 ) Cl2  , Crl2 ( C2O 4 )  ,
   2
−.
( )
+
Fe (H O ) ( OH)  , Fe NH ( CN ) 
List-I List-II  2 4 2  3 4 
2+
(p) p − dπ anticoding Co (NH2 − CH2 − CH2 − NH2 ) Cl
 
(A) 2+
and Co (NH3 ) (H2O ) Cl2 
 4 
the number of complex ion(s) that show(s) cis-trans
(q) d − dσ  bonding isomerism is (2015)
(B)

Q.23 In the complex acetylbromidodicarbonylbis


(triethylphosphine) iron(II), the number of Fe–C bond(s)
(r) p − dπ bonding is (2015)
(C)
Q.24 Among [Ni(CO)4], [NiCl4]2‒, [Co(NH3)4 Cl2]Cl,
Na3[CoFe6], Na2O2 and CsO2 the total number of
(s) d − d σ paramagnetic is (2016)

(D) (A) 2 (B) 3 (C) 4 (D) 5

Q.25 The number of geometric isomers possible for the


Code: complex [CoL2Cl2]– (L = H2NCH2CH2O–) is (2016)

p Q r Ss
Q.26 The geometries of the ammonia complexes of
(A) 2 1 3 4
respectively, are (2016)
(B) 4 3 1 2
(C) 2 3 1 4 (A) Octahedral, square planar and tetrahedral
(D) 4 1 3 2 (B) Square planar, octahedral and tetrahedral

(C) Tetrahedral, square planar and octahedral


Q.21 For the octahedral complexes of Fe in SCN 3+ ‒
(D) Octahedral, tetrahedral and square planar
(thiocyanato-S) and in CN‒ ligand environments, the
difference between the spin-only magnetic moments
in Bohr magnetons (When approximated to the nearest
integer) is
[Atomic number of Fe = 26] (2015)
2 8. 46 | Co-ordination Compounds

PlancEssential Questions
JEE Main/Boards JEE Advanced/Boards

Exercise 1 Exercise 1

Q.2 Q.5 Q.12 Q.6 Q.13 Q.19


Q.23 (iii, v, vii, ix) Q.25

Exercise 2
Exercise 2
Q.6 Q.8 Q.13
Q.18 Q.22 Q.4 Q.11 Q.15
Q.18 Q.29

Previous Years' Questions


Previous Years' Questions
Q.2 Q.5 Q.8
Q.10 Q.11 Q.14 Q.1 Q.6 Q.12

Answer Key

JEE Main/Boards

Exercise 2

Single Correct Choice Type

Q.1 D Q.2 D Q.3 D Q.4 B Q.5 D Q.6 D Q.7 B

Q.8 D Q.9 B Q.10 C Q.11 B Q.12 A Q.13 B Q.14 D

Q.15 B Q.16 B Q.17 B Q.18 B Q.19 A Q.20 C Q.21 C

Q.22 C

Previous Years’ Questions

Q.1 C Q.2 A Q.3 B Q.4 B Q.5 C Q. 6 B Q.7 A


Q.8 B Q.9 D Q.10 B Q.11 D Q.12 C Q.13 A Q.14 C
Q.15 B Q.16 D Q.17 A Q.18 D Q.19 B Q.20 A Q.21 B
Q.22 B Q.23 B Q.24 B Q.25 C Q.26 B Q.27 B Q.28 A
Q.29 A Q.30 A
Chem i str y | 28.47

JEE Advanced/Boards
Exercise 2

Single Correct Choice Type

Q.1 D Q.2 B Q.3 D Q.4 A Q.5 B

Multiple Correct Choice Type

Q. 6 A, C Q.7 C, D Q.8 C, D Q.9 A, D Q.10 A, B, C Q.11 A, B, C


Q.12 A, B, C, D Q.13 A, C, D Q.14 B, C, D

Assertion Reasoning Type

Q.15 B Q.16 C

Comprehension Type

Paragraph 1: Q.17 B Q.18 D Q.19 C Paragraph 2: Q.20 C Q.21 C Q.22 B

Match the Colulmns

Q.23 A → p; B → q, r, t; C → p, q, s; D → q, r, t Q.24 A → q, s; B → r, s; C → p, q, s
Q.25 A → s; B → p, q, r; C → p; D → q, s Q.26 A → q, s; B → r, s; C → p, q, r, t; D → q, r
Q.27 A → r; B → p; C → q; D → s

Previous Years’ Questions

Q.1 B Q.2 D Q.3 A Q.4 D Q.5 C Q.6 B Q.7 B

Q.8 B Q.9 A → p, q; B → p, r; C → q; D → q, s Q.10 3 Q.11 6 mL Q.12 (iii) (iv)

Q.16 A → p, q, s; B → p, r, s; C → q, s; D → q, s Q.17 B Q.18 C Q.19 B

Q.20 A Q.21 D Q.22 5 Q.23 D Q.24 B Q.25 5 Q.26 A


2 8. 48 | Co-ordination Compounds

Solutions

JEE Main/Boards Cl Cl
Cl
Cl
Cl

Exercise 1 CO
CO
CO

Sol 1: According to Werner Metal show two types of Cl


linkages /valences namely primary and secondary.
Primary are ionisable and exhibit negative charge.
(ii)
Secondary are non-ionisable. Secondary valency is the NH3
NH3 NH3
coordination no. of central metal atom and is fixed.
Cl Cl

Sol 2: 1: 1 ratio, FeSO4.(NH2)2SO4 is a double salt. So, we CO CO CO

can detect all the ions.


In 2nd case, we get [Cu(NH3)4]2+, so we cannot get Cl
positive test for Cu2+
(iii)
Sol 3: (i) [Cr(C2O4)3] doesn't exhibit geometric isomerism.
3– NH3 NH3 NH3

Cl H3N NH3
(ii) [PtCl2(en)2]2+ has 2 geometrical isomers.
CO CO CO
(iii) [Cr(NH3)2 Cl2(en)] has 2 geometrical isomers. Cl Cl Cl

NH3 Cl Cl

Sol 6: (i) We get a precipitate of CuF2.


Sol 4: (i)
(ii) We get a complex K2[CuCl4].

Leavo Dextro Sol 7: K3[Cu(CN)4] is formed. No precipitate is obtained


because of the lack of Cu2+ ions in the solutions. All the
copper is in complex form.
Cl Cl

Cl Cl
Sol 8:
3d 4s
(ii) 3+
Cr 21e-

3 unpaired e− which do not pair up after hybridisation as


is a weak field ligand.
Cl Cl 3d 4s
2+
Cl Cl Ni 26e-
(iii) (Strong ligand)
H3N NH3
Present even after hybridisation as CN− is a strong field
Sol 5: (i) NH3 NH3 ligand.
Sol 9: Ni2+ is present in both cases but H2O is present
weak ligand
∴ Configuration in 1st Case
Chem i str y | 28.49

3d 4s
2 unpaired e-
3d 4s
3d 4s
Configuration in 2nd Case 2 unpaired
no e--
unpaired e
3d 4s
no unpaired e-

Due to presence of unpaired e– [Ni(H2O)6]2+ is coloured 

Sol 10: Both the complexes have Fe2+ as the central metal ion, but H2O is weak ligand, CN– is strong ligand.
3d 4s
3d 4s
[Fe(H2O)6]2+
3d 4s
3d 4s
[Fe(CN)6]4–

Hence, no. of unpaired eΘ are different in both cases which results in different colours in dilute solutions.

Sol 11: (i) K3[Co(C2O4)3]


3d 4s 4p

Co +3 Co.no=6

(ii) [CoF4]2–
3d 4s 4p
Co +2 Co.no=4

(iii) Cis [Cr(en)2Cl2]Cl


3d 4s 4p

Cr +3 Co.no=6

(iv) Mn 2+
3d 4s 4p 4d
Co.no=6

Sol 12: (i)K[Cr(H2O)2(C2O4)2]·3H2O


Potassium diaquadioxalateochromate (III) Trihydrate
3d 4s

Cr +3 Co. No. = 6
2 8. 50 | Co-ordination Compounds

It shows geometric, optical isomerism

MM = 3(5) = 15 BM [
∴ MM = n(n + 2) BM ] where n is no. of unpaired e–
(ii) [Co(NH3)4Cl]Cl2
Tetraammine chloro Cobalt (III) Chloride
3d 4s 4p

Co +3 co. = 5

No stereo isomerism

MM = 4(6) = 24 BM
(iii) CrCl3(py)3
Trichloro Tripyridinium Chromium (III)
3d 4s

Cr +3 Co. No. = 6

Shows geometric isomerism


MM = 3(5) = 15 BM

(iv) Cs(FeCl4) → Caesium Tetrachloro Ferrate (III)


3d 4s 4p

Fe +3 Co. No. = 4

No. isomerism, MM = 5(7) = 35 BM


(v) K4[Mn(CN)6]
Potassium Hexacyano Maganate (II)
3d 4s 4p
Mn +2, Co. No. = 6

No Isomerism, MM = (5) 7 BM = 35 BM

Sol 13: Chelation describes a particular way that ions and molecules bind metal ions. Chelation involves the
formation or presence of two or more separate coordinate bonds between a polydentate ligand and a metal atom.
Chelate effect describes the enhanced affinity of chelating ligands for a metal ion compared to the affinity of
collection of similar non-chelating ligands for same metal.

N CH2 N CH3
Cu is stable than Cu
N CH2 N CH3

Sol 14: (i) [Cr(H2O)6]3+, H2O is weak ligand.


3d 4s

3 unpaired e– i.e. n = 3
Chem i str y | 28.51

(ii) [Fe(H2O)6]2+, H2O is weak ligand Sol 6: (D) Ni → O (zero), dsp2 hybridisation is not
3d 4s
possible.

3 unpaired e– i.e. n = 4 Pd2+ → Square planar

(iii) [Zn(H2O)6]2+, H2O is weak ligand


3d 4s dsp
2

No unpaired e– i.e. n = 0
Sol 7: d4, d5, d6, d7
(ii) Has highest magnetic moment as it has highest no.
of unpaired e– Sol 8: (D) Reasons are vacant d-orbitals and that their

charge to size ratio is large but not that they are metals
Sol 15: Strength of ligands → NO2 > NH3 > H2O
Sol 9: (B) In (A) charge is +4, so there is more donation
∴ Wavelength of [Ni(H2O)6]2+ > [Ni(NH3)6]2+ > [Ni(NO2)6]4– in (A) from Co to O. So B.O. in A is low. So, bond length
is A is higher.
Sol 16: NH+4 cannot form complexes due to lack of lone
pair Sol 10: (C)
a b

a
Sol 17: [Co(NH3)5Cl]2+  A b A
Only 1 Isomer Only 1

A b A b
Exercise 2
b b
Facial Meridional
Single Correct Choice Type
a b
Sol 1: (D) [CoF6]3– doesn't have synergic bonding due to
lack of empty orbitalsA in F–. b A a
Only 1 Isomer Only 1 Isomer
Sol 2: (D) CO, NH3 are
A strong ligands.
b So all are low spin A b
complexes
b b
Facial Meridional
Sol 3: (D) K[Fe(H2O)2(NCS)3NO3]
Potassium di aqua-tri-isothiocynato-nitrato Ferrate (III)
Sol 11: (B) It has 5 CN– ligands (strong) Fe2+, pairing
takes place. So, d2sp3
Sol 4: (B) Fe → 26
3d 4s 4p Sol 12: (A) IUPAC of [Cr(H2O)5ONO]Br2 Penta-aquo-
nitrito-O chromium(III) Bromide

dsp2 (Tetrahedral)
Sol 13: (B) By reduction or dimension, it can attain EAN
CO is a strong ligand. Pairing takes place. of 36. So, oxidation doesn't lead to stability.

Sol 5: (D) NO → NO+, Co2+ → Co3+ (Stable) Sol 14: (D) V2+ because of presence of unpaired e–. Rest
all have no unpaired e–.
Mn+ can form stable complex with strong ligand CO.
Sol 15: (B) Co3+ → 24
So, all are reducing agents
2 8. 52 | Co-ordination Compounds

Previous Years' Questions


2
d sp
3
Sol 1: (C) The number of atoms of the ligand that are
directly bonded to the central metal atom or ion by
So, (b) is correct . In (a) Not exists as NO+ coordinate bond is known as the coordination number
In [Fe(CO)2(NO)2], EAN = 36 (Since not exists as NO+) of the metal atom or ion coordination number of metal
= number of σ bonds formed by metal with ligand.

Sol 16: (B)


Sol 2: (A) CN– ions acts both as reducing agent as well

NH3 NH3 as good complexion agent.

H3 N Cl Cl NH3
Sol 4: (B) + 1 × 4 + x – 1 × 4 = 0
4 + x – 4 = 0 ⇒ x = 0 for Ni.
H3 N Cl H3 N Cl

Cl Sol 5: (C) C.N. of E in complex is 6 and oxidation state is


Cl
Facial Meridional x+0–2–1=0;x=3

Both have plane of symmetries


Sol 6: (B) [Cr(en)2Br2]Br
So, no optical isomer.
Stereo -2 Dibromidobis (ethylenediamine) chromium (III) Bromide.

Sol 17: (B) Mohr's salt has very high mol wt but same Sol 7: (A) Ni2+ in aqueous solution means [Ni(H2O)6]2+.
reducing points. So, its less readily oxidizable than The electronic configuration of
FeSO4 and error in calculations decrease. Ni2+(aq) :
3d
3d
Sol 18: (B) [RhF6] 3–
→ Rh 3+

4s 4p
It's low spin inner orbital complex 4s 4p 4d
4d
3 2
5s 3 sp d hybridization
4d 2
sp d hybridization

Total number of unpaired electron = 2


∴ Spin only magnetic moment of Ni2+ = 2(2 + 2)
Sol 19: (A) Fe3+, weak Tetrahedral = 2 2 = 2.83 B.M.

[Cu(CN)4]3–Cu+ Sol 8: (B) both have 4 unpaired electron.

[AuCl4]–Au3+
Sol 9: (D) cis-form [Co(en)2(NH3)2]3+ has optical isomers.
2
dsp (Square Planar)
2
dsp (Square Planar)
dsp2 (Square Planar)
NH3 NH3 3+ 3+
NH3 NH3
Sol 20: (C) [Co(NH3)6]Cl2
en Co Co en
3 ions i.e. [Co(NH3)6] , 2Cl
2+ –

en en
Mirror
Sol 21: (C) K[Co(CO)4]
CO → Neutral Sol 10: (B) SCN– ion can coordinate through the sulphur
K → +1 ⇒ Co → –1 or nitrogen atom. Such possibilities give rise to linkage
isomerism in coordination compounds.
M ← SCN Thiocyanato or thiocyanato-S
Sol 22: (C) [Fe(C2O4)3]3– is most stable because of chelate
effect M ← NCS Isothiocyanato or thiocyanato-N
Chem i str y | 28.53

Sol 11: (D) 27Co2+ – 1s2 2s2 2p6 3s2 3p6 3d7 4s0 Sol 16: (D)
3d 4s 4p

en

As Cl– is weak field ligand so no pairing up.


Hence it is sp3 hybridized giving tetrahedral geometry.
E ox NO2
Fe2+ – 1s2 2s2 2p6 3s2 3p6 3d6 4s0
3d 4s 4p

en

Due to Cl–, back pairing is not observed so it will be sp3


hybridized giving tetrahedral geometry. Coordination no = 6 and Oxidation no = 3
Ni2+ – 1s2 2s2 2p6 3s2 3p6 3d6 4s0
3d 4s 4p Sol 17: (A) CN- is stronger ligand hence ∆o is highest.

Sol 18: (D) It is octahedral complex of the type [M(AA)2x2]


Because of weak ligand, back pairing is not observed so Where AA is bidentate ligand.
it will be sp3 i.e. tetrahedral geometry. All the complexes
of Pt2+ are square planner including those with weak
field ligand such as halide ions thus (d) is correct. Sol 19: (B) NSC‒ is ambidentate ligand and it can be
linked through N (or) S

Sol 12: (C) In case of d3 configuration, the number


of unpaired electrons remains 3 whether the ligand is Sol 20: (A) CoCl3 .6NH3 → xCl− 
AgNO3
→ x AgCl ↓
strong field or weak field. The hybridization scheme can
be shown as follow
n ( AgCl ) = x n ( CoCl3 .6NH3 )
3d 4s 4p
4.78 2.675
[Cr(NH3)3] = 3+ =x ∴x =3
2 3
143.5 267.5
d sp hybridisation

Hence, the complex is inner orbital complex as it involves ∴ The complex is Co ( NH3 )6  Cl3
(n – 1)d orbitals for hybridization, 3.93 = n(n + 2) ; so
n = 3 (here n is number of unpaired electron(s)). Sol 21: (B) Only option (B) is having non-super
imposable mirror image & hence one optical isomer.

Sol 13: (A) 2Cl– ions are ionizable

∴ [Co(NH3)5Cl]Cl2 [Co(NH3)5Cl]2++2Cl– en
3 ion +2
Zn
2Cl– + 2AgNO3 → 2AgCl + 2NO–3.
NH3

Sol 14: (C) As EDTA is an Hexadentate Ligand, only one NH3


molecule of EDTA will be required to form an octahedral No optical isomer. It is
complex with a Ca2+ ion. Tetrahedral with a plane of symmetry

Sol 15: (B) There is dπ-pπ metal to ligand back bonding


in Fe-C bond of the organometallic compound Fe(CO)5.
So it possesses both σ and π characters.
2 8. 54 | Co-ordination Compounds

en en Red < Yellow < Green < Blue


The complex absorbs lower energy light lower will be its
strength. So order of ligand strength is L1 < L3 < L2 < L4
en +3 +3 en
Co Co
Sol 27: (B) The complex is of the type [Mabcd]
M = metal

en a, b, c, d = Monodentate ligands.
en
H2O a c a c a b
M M M
H2O b d d b d c
+3
Co en 3 geometrical isomers
H2O

H2O Sol 28: (A) Zn2 Fe ( CN)  is white.


Horizontal plane is plane symmetry  6

2+
Sol 29: (A) Each Cr (H2O ) 
2+
and Fe (H2O ) 
en  6  6

+2 Contain 4 unpaired electron.


Zn

Sol 30: (A) With coordination number six, if two


bidentate ligands in cis-position are present, then it is
optically active.
No optical isomer it is
tetrahedral with a plane of symmetry

JEE Advanced/Boards
Sol 22: (B) Cr (NH3 )  Cl3 involves d2 sp3 hybridization
 6
and it is an inner orbital complex. Exercise 1

Sol 23: (B) In NiCl4  , n = 2


2−
Sol 1: [Co(NH)3Cl3] Coordination entity
NH3, Cl → ligand
=
µ n (n + 2 ) BM
Co. no = 6
Polyhedron octahedral, different ligands so
= 2 ( 2 + 2 )= 2.82 BM heteroleptic
[Cr(C2O4)3]3–

Sol 24: (B) Cr ( en) Br2  Br - dibromido bis (ethylene CO24− → ligand
 2 
Co. no = 6
diamine)chromium(III) bromide
Octahedral and homoleptic

Sol 25: (C) Co (NH3 )3 Cl3  show facial as well as


Sol 2: Unidentate → ligand with single e– donor, ex. H2O,
meridional isomerism. But both contain plane of NH3
symmetry. So, the answer is (C).
Bidentate → ligand with two e– donor atoms, ex. C2O24−, en
Ambidentate → ligand with more than 1e– donor atom
Sol 26: (B) The energy of red light is less than that of but both cannot donate pair at the same time.
violet light.
So energy order is
Chem i str y | 28.55

Sol 3: (A) The main basis of the spectrochemical series is If energy released while pairing is higher than D. We get.
the energy difference in the orbitals of the control metal
atom after coming close to the ligands. More is the ∆0, Case - I
stronger is ligand.
(B) else Case - II
NH3 Cl

H3N Cl H3N NH3 Sol 7: [Co(en)2Cl2]Cl


CO CO Dichloro Bisethylene Diamine Cobalt (III) Chloride
H3N Cl H3N Cl Cl
Cl Cl
Facial Meridional
Co

Sol 4:
Py NH3 Py NH3 Cl
Py Pt NH3 Py Pt NH3 Cl Cl

Cl Pt Br Br Pt Cl Cl Cl
Br NH3 Co Co
Cl Br Br Cl
Br Pt NH3

Py Pt Cl

Py Cl dl-pair

No one shows opitcal isomers as all of these are planare dx -y


2 2

and have plane of symmetry.

Sol 5: Stability of a complex in solution refers to the dz


degree of association between the species involved in Sol 8:
the state of equilibrium. d xy
Ex. M + 4L ⇔ ML4
[ML 4 ]
β= dyz, dzx
[M] [L]4
Sol 9: (i) Enzymes
Their stability depends on
•• Chelate effect (ii) Ex. Cisplatin H3N Cl
•• Macrocyclic effect Pt
H3N Cl
•• Geometric factors (like shape)
•• Charge of metal ions (iii) For determining the presence of certain ions using
•• Ionic radius complexing ligands
(iv) Au, Ag Complexes Ex. [Au(CN)2]–
dx -y , dz
2 2 2

Sol 10: If it acts as a chelating agent


Sol 6: 0 ·
: NH2
M
dxy, dyz, dzx : NH2
2 8. 56 | Co-ordination Compounds

There is a formation of 3-membered ring. It's unstable → Linkage Isomerism


Linkage Isomerism
because of high steric hindrance.

M ONO, M NO2
Sol 11: Square planar exhibit geometric isomerism
because of possibility of 2 different locations from a
single location.
→ Coordination Isomerism
AA AA
E.g.
[Co(NH3)6] [Cr(CN)6], [CO(CN)6] [Cr(NH3)6]
MM MM
AA BB Stereo Isomerism
→ Geometrical Isomerism
It's not possible in Tetrahedral. So, it can't show b a b a
geometric isomerism
M M

Sol 12: Let pt 0.5 be x a b b a

x + (–6) = –2 ⇒ x = 4 → Optical Isomerism


Pt → +4 X X

Sol 13: In Hg2Cl2, Hg2Cl2 HgCl2 + Hg X X

HgCl2 + 2NH3 [Hg(NH3)2Cl2]

[Hg(NH2)Cl] + NH4Cl

The liberated mercury renders this precipitate black Sol 17: All the bonds between and ligand are coordinate
covalent bonds.

Sol 14: [Fe(CN)6]3– Fe3+ (i) [Fe(CN)6]4– Fe2+, strong ligand


3d 4s 3d 4s 4p

1 unpaired e

[Fe(CN)6]4– Fe2+
(ii) [FeF6]4– Fe2+, weak ligand
3d 4s
3d 4s 4p 4d
No unpaired e–

Sol 15: n(n + 2) =


5.92 ⇒ n = 5
(iii) [Co(C2O4)3]3– Co3+, strong
i.e. There are 5 unpaired e–
3d 4s 4p
3d 4s

∴ Mn2+

Sol 16: Structural Isomerism (iv) [CoF6]3– Co3+, weak


3d 4s 4p 4d
→ Ionisation Isomerism
[Co(NH3)4Br2]SO4[Co(NH3)SO4]Br2
→ Hydrate Isomerism
(i), (iii) → d2sp3
[Cr(H2O)6]Cl3 [Cr(H2O)5Cl]Cl2 · H2O
(ii), (iv) → sp3d2
Chem i str y | 28.57

Sol 18: The new e– in CO enters into the anti bonding orbitals
of CO and reduces its bond order. The bond order
dx -y , dz
between M and C increase. The extent depends on no.
2 2 2

of lone pairs present on metal for donation. It's called


synergic effect
0
Sol 22: (i) [Co(H2O)CN(en)2]2+
Co +3, Octahedral
dx2-y2, dyz, dzx
(ii) [PtCl4]2–

Sol 19: Spectrochemical series is a series depicting the Pt +2, Square Planar
strength of the ligands during complex formation.
(iii) [Cr(NH3)3Cl3]
Weak field ligand create small ∆o (energy difference b/w
Cr +3, Octahedral
newly formed orbitals) compared to strong field ligands.
So, pairing is not preferred against weak ligands (iv) [CoBr2(en)2]+
Co +3, Octahedral
Sol 20: It's the energy difference between the new
(v) K3[Fe(CN)6]
formed orbitals in the field of a ligand.
If ∆o is high pairing is preferable Fe +3, Octahedral
eg
Sol 23: (i) [Zn(H2O)4]2+
0 3d 4s 4p

t2g
3
sp (Tetrahedral)
for ex. if we take d6 configuration.
(ii) K2[PdCl4]
if ∆o is high is more stable
4d 5s 5p

if ∆o is low is more stable


2
dsp (Square Planar)

Sol 21: In metal carbonyls, CO acts as a ligand and forms


(iii) [Pt(NH3)2Cl2]
5d 6s 6p

CO
2
dsp (Square Planar)
 bond
(iv) K2[Ni(CN)4]
After the formation of σ bonding. There is a possibility 3d 4s 4p
for back bonding from metals to CO.

2
dsp (Square Planar)
CO
(v) [Co(NH3)5ONO]2+
2 8. 58 | Co-ordination Compounds

3d 4s 4p Exercise 2
Single Correct Choice Type
2 3
d sp (Octahedral)
Sol 1: (D) Back bonding in PCl3 > PMe3
(vi) [Co(NH3)6]2(SO4)3
So complex having more PMe3 will save higher back
3d 4s 4p bonding to CO. So, it will have lower C–O bond order.
∴ BO of CO in.

2 3
[M(CO)3(PCl3)3] > [M(CO)3(PCl3)2PMe3] > [M(CO)3PCl3
d sp (Octahedral)
(PMe2)2]
(vii) K3[Cr(C2O4)3]
3d 4s 4p Sol 2: (B) Fe 26
3d 4s 4p

2 3
d sp (Octahedral)
2
dsp (Tetrahedral)
(viii) [Pt(NH3)6]4+
Co is a strong ligand. Pairing takes place.
5d 6s 6p

Sol 3: (D) NO NO+, Co2+ Co3+ (Stable) Mn+


can form stable complex with strong ligand Co.
2 3
d sp (Octahedral)
So, all are reducing agents
(ix) [CoBr4]2–
Sol 4: (A) By reduction or dimension, it can attain EAN
3d 4s 4p
of 36. So, oxidation doesn't lead to stability.

sp
3 Sol 5: (B) Mohr's salt has very high mol wt but same
(Tetrahedral)
reducing points. So, its less readily oxidizable than
FeSO4 and error in calculations decrease.
Sol 24: VBT doesn't account for the difference in
configuration with ligands of different strengths. It also
doesn't account for the colour of complexes. Multiple Correct Choice Type
Where as CFT clearly explains the configurations of Sol 6: (A, C) (A) is true because bridge CO will take e–
metals with different ligands using crystal field model pair from 2 Fe atoms.
so, that we can account for shape, colour and magnetic
properties. (B) is false.

Sol 25: (i) [Co(NH3)4H2OCl]Cl2 N


N N
N
(ii) K2[Zn(OH)4] O
O O
OOO O more repulsion by LP. So angle is less
O
(iii) K3[Al(C2O4)3] (C) is true.
(iv) [CoCl2(NH2 – CH2 – CH2 – NH2)2] + No can also act as acceptor ligand
(v) [Ni(CO)4]
Sol 7: (C, D) (A) Au3+
Diamagnetic

(B) Cu3+
(C) Cu2+ Tetrahedral
Chem i str y | 28.59

a a

A b b A
Paramagnetic, Square Planar

B a a B
(D) Co
b b

b b
Paramagnetic, Square Planar
A a a A

Sol 8: (C, D) Transition elements → last e– enters in B b b B


d-orbital and they are named as such because their a a
properties lie between s and p block.
6 Stereoisomers
Lanthanum is a d-block transition metal
(C) Will have less than (D) as (3), (5) and (4), (6) Coincide
Sol 9: (A, D) (A) (B) Will have more than 6, as we can get 2 for each of
a a (3), (4), (5), (6)

c b b b
Sol 10: (A, B, C) [Fe(CN)6]4– → low spin as CN– is
strong
b c c c
[PtCl4]2– → Pt2+ always forms low spin even with
a a weak ligands

[Co(C2O4)3]3 → low spin as C2O24− is strong ligand


b c
[Ni(NH3)6]2+ →
a a b b
3d 4s 4p 4d

c c a a
3 2
sp d
b c High spin as 6 orbital are needed for hybridisation.
a a Pairing need not occur.
a b b a
Sol 11: (A, B, C) Cu2+ → 1 unpaired e–
Ti3+ → 1
c b b c
Fe3+, strong ligand → 1
c c
[Cr(NH3)6]3+ → 3
(A) Has 6 stereoisomer
(D) [M(AB)A2B2] Sol 12: (A, B, C, D) MA4B2 can't produce due to presence
of plane of symmetry.
b a
MA3B2C will also have plane of symmetry.
A a A b a

a b
B a B b
m has a plane of symmetry
b a
a c

b
2 8. 60 | Co-ordination Compounds

Sol 13: (A, C, D) (A) 22 + 4(2) = 30 Sol 18: (D) Due to synergic bonding M-L bond strength
increases, so complex stability increases More negative
(B) 26 + 2 + 4 (2) = 36 Not exists as NO+
charge of M, increases synergic bonding as more e–
(C) 46 + 2(2) = 50 pairs can be donated.
(D) 78 + 4(2) = 86 Sol 19: (C) C2H2 can act as π-acid ligand All [MA3B2C]
Isomers are optically inactive due to presence of plane
of symmetry. Complex of type [MABCDE2] has 15 active
Sol 14: (B, C, D) NO3− can't show because of no LP on
isomers
N.
O
Paragraph 2:


N - -
O-CN : :S  S  O Sol 20: (C) CN–, because in CO32− , OH– the e– donar.



O O N, Cl have same EN but CN– is a strong ligand


O

Sol 21: (C) Fe 26


Fe2+ 24
Assertion Reasoning Type

Sol 15: (B) Cr+3 → 21e–


eg

t2g as there are 2 d orbitals octahedral

as all ligands are identical, it doesn't show geometrical


isomerism. Sol 22: (B) [V(CO)5]– Metals are mostly unstable under
negative charge
But S-2 doesn't imply S-1
S-1 reason is that t2g orbitals have less energy compared
to eg orbitals. Match the Columns

Sol 23: A → p; B → q, r, t; C → p, q, s; D → q, r, t
a b a b a b
(A) Na[Co(CO)4] monodentate ligand Follows EAN
Sol 16: (C) M M rule M
b a c b d d c (B) c[Pt(triene)Cl2] d no EAN (80) 5 membered ring
M chelating ligand (trien)

c c d (C) [Fe(H2O)5NO]SO4 No EAN (37) all monodentate


NO+ is +ve charged ligand
S-2 is false, as rotation is possible around a single bond (D) [Fe(EDTA)]– hexadentate ligand No EAN 5,
and [MABCD]n± can show geometrical isomerism  membered ring (Chelating ligand)

Comprehension Type Sol 24: A → q, s; B → r, s; C → p, q, s


Paragraph 1: (A) [Co(CO)4]– EAN = 36 Synergic bonding is
present Diamagnetic, no isomers
Sol 17: (B) NMe3 only N can donate e– pair. (B) [Zn(sly)2] EAN = 36 dimagnetic, dl pair
SO24− only O can donate e– pair. (C) [Ag(CN)2]– EAN ≠36 (50) There is synergic
Θ
OCN O and N can donate e– pair. bonding It's diamagnetic

So, OCNΘ can show linkage isomerism


Chem i str y | 28.61

Sol 25: A → s; B → p, q, r; C → p; D → q, s (D) [Ma3bcd]


C B B
(A)
A A A A A A
M M M
M M
A B A C A C

D D B
Two optically active isomers C B B
C B B
A A A A A A A A
(B) [M(AB)b
A 2c2] A A A A A
A M B A M B M
A M M A M
A b B A M B A M C A C
A B A C A A C
A C A C
D D B
B Da A D B B
D D
B B A A
A A
B B
A A A B A B
B B
A a A A
M M M M

A b C A C B A C A C
B B

A D D A D D
A A
4 geometric, 2 optically active
A A A B
and
Sol 26: A → q, s; B → r, s; C → p, q, r, t; D → q, r
B A B B
(A) Zn[Fe(CN)5NO] d2sp3, Fe +2, So
B A
diamagnetic, NO NO +

(C) [M(AB)3] (B) [Fe(H2O)5NO]SO4 sp3d2 paramagnetic



A A [ H2O is a weak ligand]NO NO+
B A A B (C) Fe4[Fe(CN)6]3
M M
[Fe(CN)6]4– i.e. F2+ d2sp3
B B B B
There is charge transfer between Fe ions.
A A
B B
Fe3+ Paramagnetic

A A A A (D) K4[Fe(CN)5O2]
M M
Fe3+ d2sp3 and paramagnetic
B B B B
(CN– is strong ligand)
A A

Sol 27: A → r; B → p; C → q; D → s
(A) No. of ions 4
(B) 3
(C) 2
(D) 5
∴C 97 (q)
2 8. 62 | Co-ordination Compounds

B 229 (p) N2+ + 4Cl– → [NiCl4]2–


A 404 (r) Here Ni2+ has d8-configuration with Cl– as weak ligand.
D 523 (s)  3d

3
sp
Previous Years' Questions d8- configuration in weak ligand field gives sp3
hybridisation, hence tetrahedral geometry.
Sol 1: (B) Salt with least no. of unpaired electrons in
Ni2+ with H2O forms [Ni(H2O)6]2+ complex and H2O is a
d-orbital of central metal will show lowest degree of
weak ligand.
paramagnetism.
Mn2+(3d5, 5 unpaired electrons)
Sol 7: (B) (A) Mn+ = 3d54s1 in presence of CO effective
Cu2+(3d9, 1 unpaired electron) configuration = 3d54s0 Three lone pair for back bonding
Fe2+(3d6, 4 unpaired electrons) with vacant orbital of C in CO.
(B) Fe0 = 3d64s2 in presence of CO effective configuration
Ni2+(3d8, 2 unpaired electrons)
= 3d8 four lone pair for back bonding with CO.
Hence, CuSO4 · 5H2O has lowest degree of paramagnetism. (C) Cr0 = 3d54s1 Effective configuration 3d6. Three lone
pair for back bonding with CO.
Sol 2: (D) Cu2+ + CN– CuCN↓ (D) V‒ = 3d44s2 effective configuration = 3d6 three lone
CuCN + 3CN– [Cu(CN)4]3– pair for bonding with CO.
Maximum back bonding is present in Fe(CO)5 there for
CO bond order is lowest here.
Sol 3: (A) In MnO–4, Mn+7 has 3d0 configuration.
Sol 8: (B) Both statements are true. However, axis of
Sol 4: (D) [NiCl4]2– : Ni2+ (3d8) symmetry is not a criteria of optical isomerism. Optical
inactivity of the two geometrical isomers of [M(NH3)4Cl2]
(tetrahedral)
3
is due to presence of lane of symmetry.
sp -hybridisation
Cl NH3
H3N NH3 H3N Cl
8
3d 4s
M M
Under influence of weak
Lignad field
H3N NH3 H3N Cl
In all other complexes, hybridization at central metal is Cl NH3
trans cis
dsp2 and complexes have square planar geometries. (has several planes of symmetry) (has several planes of symmetry)

Sol 5: (C) [Ni(NH3)4]2+ = tetraamminenickel (II)


[NiCl4]2– = tetrachloronickelate (II) Sol 9:
Cationic part is named first, hence : 2+ 2+
OH2 NH3
tetraamminenickel (II)-tetrachloronickelate(II) NH3 OH2
H3N H2O
(A) Co Co

Sol 6: (B) Ni2+ + 4CN– → [Ni(CN)4]2– H3N NH3 H3N NH3


OH2 OH2
Here Ni2+ has d8-configuration which CN– as strong
(trans) (cis)
ligand 2+ 7
Co =3d (Paramagnetic)
8
3d 4s 4p
[Pt(NH3)2Cl2] is square planar.

H3N Cl H3N Cl
2 2+ 2+
dsp (B) Pt Pt

d -configuration in strong ligand field gives dsp


8 2 H3N Cl Cl NH3

hybridisation, hence square planar geometry. 2+ 8


(cis)
0
(trans)
Pt =5d 4s (Diamagnetic)

+ (D) +
(C)
OH2 OH2
OH OH
2+ 2+
(B) Pt Pt

H3N Cl Cl NH3

2+ 8
(cis)
0
(trans)
Chem i str y | 28.63
Pt =5d 4s (Diamagnetic)

(C)
+ (D) + Sol 13:
OH2 OH2
OH2 OH2 NH3
H2O H2O
H3N NH3
Co Co
H2O OH2 H2O OH2
Cl OH2 Co
2+ 7 2+ 8
Co =3d (Paramagnetic) Ni =3d (weak field ligand, paramagnetic)
H 3N NH3 OC
NH3
d2sp3-octahedral dsp2-square planar

Sol 10:
NH3 CO
Cl PPh
H3N3 Cl NH3 CO

Rh Rh Ni
Co CO
H3N CO H3N PPh3
H 3N OC CO
NH3
NHPPh sp -tetrahedral
3
Cl 3
3

Rh
Sol 14: The spin-only magnetic moment (µ) of the
OC H 3N
3 isomers complex is 1.73 BM. It indicates that nucleus of complex,
chromium ion has one unpaired electron. So, the ligand
Sol 11: m mol of complex = 30 × 0.01 = 0.3 NO is unit positively charged.
Also, 1 mole of complex [Cr(H2O)5Cl]Cl2 gives only tow K2[Cr(NO) (CN)4 (NH3)]
moles of chloride ion when dissolved in solution Potassium ammine tetracyanonitrosonium-chromate (I)
[Cr(H2O)5Cl]Cl2 [Cr(H2O)5Cl] + 2Cl 2+ –

CrCr+1 :
+1

⇒ m mol of Cl ion produced from its 0.3 m mol = 0.6



3d
5
4s
0

Hence, 0.6 m mol of Ag+ would be required for Cr(I) “under influence of strong ligand field”.
precipitation.
⇒ 0.60 m mol of Ag+ = 0.1M × V(in mL) ⇒ V = 6 mL.
2 3
d sp
octahedral
Sol 12: (i) [Ti (NO3)4] : Ti4+ (3d0) No d-electron, no d-d
transition possible, colourless.
NO
(ii) [Cu(NCCH3)]BF4 : Cu+ (3d10) All d-orbitals are NC CN
completely filled, no d-d transition possible, colourless.
(iii) [Cr(NH3)6]Cl3 : Cr3+ (3d3) Complex has allowed d-d M
transitions from t2g to eg level, hence, coloured.
NC NH3
(iv) K3[VF6] : V3+ (3d2) Complex has allowed d-d transitions CN
from t2g to eg level, hence, coloured.
Sol 15: Oxidation state of Ni is +2 and hybridization is
dsp2. µ = 0
H H
O O
(nounpaired electron). H3C N N CH3
C C
Ni
C C
N N CH3
H3C
O O
H H
2 8. 64 | Co-ordination Compounds

Sol 16: (A) Fe (H2O )5 NO  SO 4


(s) d − d σ anticoding
Here Fe has +1 oxidation state.
Fe+ = 3d64s1 in presence of NO+ 4s1 electron are paired
in 3d sub shell.
Sol 21: (D) [Fe(SCN)6]3– and [Fe(CN)6]3–. In both the cases
So electronic configuration of Fe+ is
the electronic configuration of Fe3+ will be 1s2, 2s2, 2p6,
3p6, 3d5. Since SCN is a weak field ligand and CN is a
strong field ligand, the pairing will occur in case of
3d

5
(high spin)
Case-1 3d
Sol 17: (D) Co (H2O ) 4 (NH3 )2 Cl3 
(no pairing)

Diamminetetraaquacobalt (III) chloride (low spin)


Case-2 (pairing)

Sol 18: (B) P = Fe (no. of unpaired e = 5 )


+3

Q = V+2 (no. of unpaired e = 3 ) Case-1 µ= n ( n + 2 )= 5 (5 + 2 ) = 35= 5.91 BM


R = Fe+2 (no. of unpaired e = 4 ) 3
Case-2 µ = n (n + 2 ) = 1 (1 + 2 ) = =1.73 BM
As all ligands are weak field, hence the no. of unpaired
electrons remains same in the complex ion.
Sol 22: (5) [Co(en2Cl2)]+ → will show cis - trans isomerism
=µ n (n + 2 )B.M
[CrCl(C2O4)]3‒ → will show cis - trans isomerism
Hence (B) is correct.
[Fe(H2O)4(OH)2]+ → will show cis trans isomerism

+
[Fe(CN)4(NH)3]‒ → will show cis trans isomerism
Sol 19: (B, D) Co (NH3 ) Cl2  and Pt (NH3 )2 (H2O ) Cl
+
 4  [Co(en)2(NH3)Cl]2+ → will show cis trans isomerism
(a square planar complex) will show geometrical [Co(NH3)4(H3O)Cl]2+ → Will not show cis – trans isomerism
isomerism.
(Although it will show geometrical isomerism)
Pt (NH ) (NO )  Cl and Pt (NH3 ) Cl Br will show
 3 3 3   3 
ionization isomerism. Sol 23: (D)
PEt3 O
Sol 20: (C) Et3P CH3
C
Fe

(p) d − dσ  bonding OC Br
CO

Sol 24: (B) [Ni(CO)4] ‒ sp3 ‒ Diamagnetic


(q) p − dπ bonding [NiCl4]2‒ ‒ sp3 ‒ Paramagnetic
[Co(NH3)4 Cl2]Cl2 ‒ d2sp3 ‒ Diamagnetic
Na3 [CoF6] ‒ sp3d2 Paramagnetic
(r) p − dπ anticoding Na i.e O22− −‒ Diamagnetic
Na22OO22 i.e.
i.e O2− −‒ Paramagnetic
CsO2 i.e.
Chem i str y | 28.65

Sol 25: (5)


Cl Cl

NH2 NH2 NH2 O

CH2 +3 CH2 CH2 +3 CH2


Co Co

CH2 CH2 CH2 CH2


O O O NH2
CH2 CH2
Cl Cl
NH2 CH2 O CH2

NH2 O NH2 NH2

CH2 +3 CH2 +3
Co Co
CH2 CH2

O Cl O Cl
Cl Cl
CH2
O CH2

CH2 +3
Co
CH2

NH2 Cl
Cl

Sol 26: (A) [Ni(NH3)6]2+ = Octahedral


[Pt(NH3)4]+2 = Square planar
[Zn(NH3)4]+2 = Tetrahedral
2017-18 100 &
op kers
Class 12 T
By E ran culty
-JE Fa r
IIT enior emie .
S fP r es
o titut
Ins

CHEMISTRY
FOR JEE MAIN & ADVANCED
SECOND
EDITION

Exhaustive Theory
(Now Revised)

Formula Sheet
9000+ Problems
based on latest JEE pattern

2500 + 1000 (New) Problems


of previous 35 years of
AIEEE (JEE Main) and IIT-JEE (JEE Adv)

5000+Illustrations and Solved Examples


Detailed Solutions
of all problems available

Plancess Concepts
Topic Covered Tips & Tricks, Facts, Notes, Misconceptions,
Key Take Aways, Problem Solving Tactics
Extrac ve Metallurgy
PlancEssential
Questions recommended for revision
29. EXTRACTIVE
M E TA L L U R G Y

1. OCCURRENCE OF METALS
The earth’s crust is the largest source of metals, besides some soluble salts of metals found in sea water. The mode of
occurrence of a metal is largely dependent on its chemical nature. Those metals, which are relatively inert, occur in free
or native state (i.e. in uncombined state) but most of the metals are reactive and hence are found in a combined state.
Mineral: The compounds of a metal which are naturally available in the earth’s crust and can be obtained by mining
are called minerals. A mineral may consist of one or more metallic compounds, having almost a fixed composition.
Ore: The minerals from which a metal can be extracted economically and conveniently are called ores.
Some important terminologies used in the process are:
Gangue or Matrix: Gangue or matrix are the non-metallic impurities present in the ore. In fact, impurities associated
with an ore is known as gangue.
Flux: Flux are chemical substances which are generally added to an ore in order to remove the impurities or
gangue. E.g., CaO, SiO2 etc.

Flowchart 29.1: Classification of flux

Slag: Gangue + Flux → Slag


"Slag” is the fusible, light and floatable substance formed due to the reaction between gangue and the flux.

PLANCESS CONCEPTS

All ores are minerals but all minerals are not ores.
Vaibhav Krishnan (JEE 2009, AIR 22)
2 9 . 2 | Extractive Metallurgy

2. TYPES OF ORES
Ores may be classified mainly into following 4 classes:-
(a) Native ores: They contain the metal in a free state, and are found in the association of rock or alluvial
impurities like clay, sand etc. Silver, gold, platinum etc. occur as native ores. Sometimes, lumps of almost pure
metals are also found. These are called nuggets.
(b) Oxidized ores: These ores consist of oxides or oxysalts (e.g. carbonates, phosphates, sulphates and silicates)
of metals.
Table 29.1: Some important ores of metals

Metal Ores Composition

Aluminium Bauxite AlOx(OH)3–2x [where 0 < X < 1] Al2O3


Diaspore Al2O3.H2O
Corundum Al2O3
Kaolinite (a form of clay) [Al2 (OH)4 Si2O5]
Iron Hematite Fe2O3
Magnetite Fe3O4
Siderite FeCO3
Iron pyrite FeS2
Limonite Fe2O3.3H2O
Copper Copper pyrite CuFeS2
Copper glance Cu2S
Cuprite Cu2O
Malachite CuCO3.Cu(OH)2
Azurite 2CuCO3.Cu(OH)2
Zinc Zinc blende or Sphalerite ZnS
Calamine ZnCO3
Zincite ZnO
Lead Galena PbS
Anglesite PbSO4
Cerrusite PbCO3
Magnesium Carnallite KCl.MgCl2.6H2O [K2MgCl4.6H2O]
Magnesite MgCO3
Dolomite MgCO3 CaCO3
Epsom salt (Epsomite) MgSO4 7H2O
Langbeinite K2Mg2(SO4)3
Tin Cassiterite (Tin stone) SnO2
Silver Silver glance (Argentite) Ag2S
Pyrargyrite (Ruby Silver) Ag3SbS3
Chlorargyrite (Horn silver) AgCl.
Stefinite Ag2SbS4
Prousitite Ag2AsS3
Chem i str y | 29.3

Illustration 1: Which metals are supposed to occur in the native state in nature?  (JEE MAIN)

Sol: Elements below hydrogen in the electrochemical series like Cu, Ag. Au etc, exist as native ores.

Illustration 2: Match the ores listed in column I with their correct chemical formula listed in column ll.
 (JEE ADVANCED)
Column I Column II

(A) Cassiterite (p) FeCO3


(B) Siderrte (q) SnO2
(C) Cerussite (r) PbSO4
(D)Anglesite (s) PbCO3

Sol: SnO2 is called cassiterite or tin stone, FeCO3 is called siderite. PbCO3 is called cerussite and PbSO4 is called
anglesite. So correct match is A → q; B → p; C → s and D → r.

3. PRINCIPAL STEPS IN THE RECOVERY OF A METAL FROM ITS ORE


The isolation and extraction of metals from their ores involve the following major steps:
(a) Crushing of the ore
(b) Dressing or concentration of the ore
(c) Isolation of the crude metal from its ore
(d) Purification or refining of the metal

3.1 Crushing of the Ore


Ore is first crushed by jaw crushers and ground to a powder (pulverization of the ore) with the help of equipment
like ball mills and stamp mills.

3.2 Concentration of the Ore


The removal of unwanted, useless impurities from the ore is called dressing, concentration or benefaction of the
ore. It involves several steps, and, selection of these steps depends on the difference in the physical properties
of the compound of the metal and that of the gangue. Hence, this process can be carried out by two methods of
separation - Physical and Chemical.

Physical Methods of separation:


(a) Gravity separation or levigation
(b) Magnetic separation
(c) Froth floatation
Chemical methods of separation, i.e. leaching.
2 9 . 4 | Extractive Metallurgy

3.2.1 Physical Methods of Separation


Crushed ore
Hydraulic Washing/Gravity Separation/ Levigation:
Water
It is based on the difference in the densities of the gangue and Raffies
ore particles. In this, the powdered ore is agitated with water or
washed with an upward stream of running water. The lighter Ore particles Gangue mixed
particles of sand, clay etc. are washed away leaving behind with water
heavier ore particles. For this, either hydraulic classifier or wilfley
table is used. This method is generally used for the concentration
of oxide and native ores. Figure 29.1 : Representation of gravity separation
using wilfley table

Electromagnetic Separation: It is based on the differences in magnetic properties of the ore components. It is
used when either the ore or the impurities associated with it are magnetic in nature. A magnetic separator consists
of a belt (of leather or brass) moving over two rollers, one of which is magnetic. When the powdered ore is dropped
on the belt at the other end, the magnetic component of the ore is attracted by the magnetic roller and falls nearer
the roller while non-magnetic impurities fall away from it.

Powdered ore

Electromagnets

Moving belt Gangue

Magnetic ore

Figure 29. 2 : Electromagnetic separation using Electromagnets

Examples: Chromite ore (FeO.Cr2O3) is separated from non-magnetic silicious impurities and cassiterite ore (SnO2)
is separated from magnetic Wolframite (FeWO4 + MnWO4).

PLANCESS CONCEPTS

•• Gravity separation utilizes the difference in density between the ore and the impurity.
•• Magnetic separation is used to separate the magnetic impurities/ore from non-magnetic ores/impurities.

Nikhil Khandelwal (JEE 2009, AIR 94)

Froth Floatation Process: This process is used to separate the sulphide ore from the impurity in the ore. It is
based upon the differential wetting of the ore by oil and the impurity by water. In this process, the ore is ground
to fine powder and mixed with water to form a slurry mixture. Any one of the oily components, such as pine oil,
eucalyptus oil, crude coal tar cresols, etc. is added together with sodium ethyl xanthate as a collector. Air, which acts
as an agitator, creates bubbles through the mixture. Finally, the ore is floated to the froth and silicious impurities
are settled at the bottom of the tank. Then the froth is collected into a separate container and washed thoroughly
and dried. The oil added acts as a frothing agent. It reduces the surface tension of the water and helps to produce
Chem i str y | 29.5

a stable froth. Reagents such as aniline or cresol are used to stabilize the froth. The formation of bubble involves an
increase in air-water surface that means doing work against surface tension. The energy required to create a bubble
of radius r is 2 × 4 π r2 × E. It is the energy required to create a unit surface area and it is directly proportional to the
surface tension. The froth floatation process is usually applied for sulphide ores and the schematic representation
is shown in figure. Sodium ethyl xanthate acts as a collector of sulphide ore.
Air

Finely
Divided ore
+ Pine oil + sodium Sulphide ore
ethyl xanthate +
+ water Froth

Figure 29.3: Froth floatation process for sulphide ore

The hydrophobic end prefers to stay in the air and the hydrophilic end prefers to stay in the water. Since the
molecule has both hydrophobic and hydrophilic parts, the best place for it to stay is the air-water interface and
hence it exhibits floating characteristics. The hydrophilic end attracts the sulphide particles and forms surface
coated particles as shown in figure.

S
CH3 CH2 O C
- +
S Na

Hydrophobic Hydrophilic end


end

Figure 29.4-a: Bonding between dirt particle and sodium ethyl xanthate

Finally, the collected sulphide particles are floated to the froth and transferred to a separate container, washed with
huge amounts of water, dried and sent for the next step of extraction.

Hydrophobic part prefers


to stay in air

Hydrophilic end
prefers to stay in water

Figure 29.4-b: Sodium ethyl xanthate at the air water interface


2 9 . 6 | Extractive Metallurgy

(a) The other alkyl groups in xanthate may be amyl (C5H11 –), octyl C8H17

( )
, etc. instead of the ethyl group.
(b) The other collectors that may be used are;
(i) Sodium lauryl sulphate : C12H25–O–SO3 Na
(ii) Trimethyl cetly ammonium bromide: C16H12N+(Me)3)Br–
(iii) Dicresyl dithiophosphate: (CH3 –C6H4 -O)2

S
(also called acrofloat)
P
S H

(iv) Mecraptobenzthiazole:
N
C SH
S (also called floatogen)

The overall ore collecting process occurs through adsorption and depends upon the unbalanced forces on the
solid surface.
Activator is a substance that is added to help the collector stay attached to the particles to be floated and hence,
improves the floating characteristics of the ore. For example, the floating characteristic of ZnS is poor, compared
to that of CuS. Hence CuSO4 is used as an activator to form a coating of CuS on the ZnS surface and improve its
floating characteristics.

2+ 2-
ZnS + CuSO4 ZnS + Zn + SO4

Particle Coating of CuS

Figure 29.4-c: Action of activator

Accordingly, using suitable activators, the froth flotation process can also be applied for non sulphide ores. For
example, Na2S is a suitable activator for malachite (CuCO3. Cu (OH)2) and anglesite (PbSO4) in which the coatings of
CuS and PbS are formed by the activator, respectively.
A depressant is a substance that is added to suppress the floating characteristic of the ore particles. For example,
galena (PbS) is associated with another sulphide impurity ZnS, which also rises to the surface with the froth. The
reagent NaCN/KCN is used to suppress the floating characteristics of ZnS by forming soluble complex with KCN.
PbS(s)+ NaCN/KCN → No reaction ZnS(s) + 4[NaCN/KCN] → [Zn(CN)4]2– + 4 [Na+/ K+] + S2–

PLANCESS CONCEPTS

•• Froth floatation is used to separate the sulphide ores from its impurities.
•• Froth floatation is based on the principle of differential wetting of the ore by oil and impurities by water.
•• Activator is a substance that helps the collector to stay attached to the particles to be floated.
•• Depressant is a substance which suppresses the floating characteristic of the ore.

Saurabh Gupta (JEE 2010, AIR 443)


Chem i str y | 29.7

Illustration 3: How does NaCN act as a depressant in preventing ZnS from forming the froth?  (JEE MAIN)

Sol: NaCN reacts with ZnS and forms a layer of Na2[Zn(CN)4] complex on the surface of ZnS and prevents it from
the formation of froth.

Illustration 4: What is the role of a stabilizer in the froth floatation process?  (JEE MAIN)

Sol: Froth can last for a longer period in the presence of a stabilizer.

3.2.2 Chemical Methods/Leaching


Leaching: Leaching is often used if the ore is soluble in some suitable solvent, e.g, acids, bases and suitable
chemical reagents.
Leaching of alumina from bauxite: The principal ore of aluminium, bauxite, usually contains SiO2, iron oxide
and titanium oxide (TiO2) as impurities. The concentration is carried out by digesting the powdered ore with a
concentrated solution of NaOH at 473 - 523 K and 35 - 36 bar pressure. This way, Al2O3 is leached out as sodium
aluminate (and also SiO2 as sodium silicate) leaving behind the impurities, iron oxide and titanium oxide.
Al2O3(s) + 2NaOH (aq) + 3H2O (l) → 2Na [Al (OH)4](aq)
The aluminate in solution is neutralised by passing CO2 gas and hydrated Al2O3 is precipitated. At this stage, the
solution is seeded with freshly prepared samples of hydrated Al2O3 which induces the precipitation.
2Na [Al (OH)4](aq) +CO2(g) → Al2O3.xH2O(s) + 2NaHCO3(aq)

The sodium silicate remains in the solution and hydrated alumina is filtered, dried and heated to give back pure
1470K /caloination
Al2O3: Al2O3.xH2O(S) → Al2O3(s) + xH2O (g)
These steps comprise the Bayer’s process.
Other examples: In the metallurgy of silver and that of gold, the respective metal/ore is leached with a dilute
solution of NaCN or KCN in the presence of air (or O2) from which the metal is obtained later by displacement with
zinc scrap.
4M(s) + 8CN–(aq) + 2H2O(aq) + O2(g) → 4[M(CN)2]–(aq) + 4OH–(aq) (M = Ag or Au)
2[M(CN)2]–(aq) + Zn(s) → [Zn(CN)4]2–(aq) + 2M(s)

PLANCESS CONCEPTS

Leaching is based on the difference of reactivity of the ore and impurity towards acids and bases.

Neeraj Toshniwal JEE 2009, AIR 21

3.3 Extraction of Concentrated Ore from Crude Metal


The concentrated ore must be converted into a form which is suitable for reduction. Usually the sulphide ore is
converted to oxide before reduction. Oxides are easier to reduce. Thus isolation of metals from concentrated ore
involves two major steps as given below.
(i) Conversion to oxide (ii) Reduction of the oxide to metal

3.3.1 Conversion to Oxide


Conversion of ore into oxide is carried out in two ways depending upon the nature of ore.
2 9 . 8 | Extractive Metallurgy

Calcination: It is a process of heating the concentrated ore strongly in a limited supply of air or in the absence of
air. The process of calcination brings about the following changes:
(a) The carbonate ore gets decomposed to form the oxide of the metal, e.g.
∆ ∆
FeCO3 (siderite) → FeO + CO2; PbCO3 (cerrussite) → PbO + CO2

CaCO3 (calcite ore / lime stone) → CaO + CO2

ZnCO3 (calamine) → ZnO + CO2

CuCO3. Cu (OH)2 (malachite) → 2CuO + H2O + CO2

MgCO3.CaCO3 (dolomite) → MgO + CaO + 2CO2
(b) Water of crystallization present in the hydrated oxide ore is lost as moisture, e.g.

2Fe2O3.3H2O (limonite) → 2Fe2O3(s) + 3H2O (g) ↑
Al2O3.2H2O (bauxite) Al2O3(s) + 2H2O (g) ↑
(c) Organic matter, if present in the ore, gets expelled and the ore becomes porous. Volatile impurities are
removed.

Roasting: It is a process of heating the concentrated ore (generally sulphide ore) strongly in the excess of air or O2
below its melting point. Roasting is an exothermic process; once started, it does not require additional heating. The
process of roasting does the following things:
(a) Roasting at moderate temperature: Some portion of the sulphide ores like galena (PbS), Zinc blende (ZnS) is
converted into metallic oxide. If the temperature is fairly low (about 500°C) and the concentration of SO2 in
the gaseous environment is more, sulphate may be produced, that is stable, and high temperature is needed
to decompose them.
∆ ∆
2PbS + 3O2 → 2PbO + 2SO2; 2ZnS + 3O2 → 2ZnO + 2SO2
∆ ∆
PbS + 2O2 → PbSO4; ZnS + 2O2 → ZnSO4
Sometimes roasting may not bring about complete oxidation.
2CuFeS2 (copper pyrite) + 4O2 → Cu2S + 2FeO + 3SO2
(b) Roasting at high temperature: The sulphide ores of some of the metals like Cu, Pb, Hg, Sb etc., when heated
strongly in the free supply of air or O2 are reduced directly to the metal rather than to the metallic oxides, e.g.
Cu2S (copper glance) + O2 → 2Cu + SO2
PbS (galena) + O2 → Pb + SO2; HgS (cinnabar) + O2 → Hg + SO2
The reduction of the sulphide ore directly into metal by heating it in air or O2 is known by various names, like
self-reduction, auto-reduction, air-reduction etc. The SO2 produced is utilized for the manufacture of H2SO4.
(c) It removes easily oxidisable volatile impurities like arsenic (as As2O3)), sulphur (as SO2), phosphorus (as P4O10)
and antimony (as Sb2O3).
4M (M=As, Sb) + 3O2 → 2M2O3↑; S + O2 → SO2↑; P4 + 5O2 → P4O10↑
These oxides being volatile, escape as gases through the chimney.
(d) When the concentrated tin stone ore SnO2 (ore of Sn), is heated strongly in a free supply of air (roasting),
the impurities of CuS and FeS present in the ore are converted into CuSO4 and FeSO4 respectively
∆ ∆
CuS + 2O2 → CuSO4; FeS + 2O2 → FeSO4
Both calcination and roasting are generally carried out in a reverberatory furnace. In case of roasting, the air holes
are kept open, while they are partially or completely closed during calcination.

Slag Formation: In many extraction processes, an oxide is added deliberately to combine with other impurities to
form a stable molten phase immiscible with molten metal, called a slag. The process is termed as smelting.
Chem i str y | 29.9

The principle of slag formation is essentially the following:


Non metal oxide (acidic oxide) + Metal oxide (basic oxide) → Fusible (easily melted) slag
Removal of unwanted basic and acidic oxides: For example, FeO is the impurity in the extraction of Cu from copper
pyrite. 2CuFeS2 + 4O2 → Cu2S + 2FeO + 3SO2

Cu2S + FeO + SiO2 → FeSiO3 (Fusible slag)+ Cu2S (matte )


  
(Roasted phsite) (upper layer ) (lower layer )

Matte also contains a very small amount of iron (ll) sulphide.


To remove unwanted acidic impurities like sand and P4O10, smelting is done in the presence of limestone.
CaCO3 → CaO + CO2; CaO + SiO2 → CaSiO3 (fusible slag)
6CaO + P4O10 → 2Ca3 (PO4)2 (fusible slag - Thomas slag)

Properties of a Slag:
(a) Slag is a fusible mass.
(b) It has a low melting point.
(c) It is lighter and is immiscible with the molten metal. It is due to these properties that the slag floats as a
separate layer on the molten metal and so, can be easily separated from the metal. The layer of the slag on
the molten metal prevents the metal from being oxidised.

Types of Flux: Flux are of two types, viz, acidic flux and basic flux.
(a) Acidic flux: It is an acidic oxide (oxide of a non-metal) like SiO2, P2O5, B2O3 (from borax). It is used to remove
basic impurities like CaO, FeO, MgO etc. The acidic flux combines with a basic impurity to form a slag.
(b) Basic flux: It is a basic oxide (i.e. oxide of a metal) like CaO (obtained from lime stone, CaCO3), MgO (from
magnesite. MgCO3), haematite (Fe2O3) etc. It is used to remove acidic impurities like SiO2, P2O5 etc. The basic
flux combines with an acidic impurity to form a slag.
Thus, slag can be defined as a fusible mass, which is obtained when a flux reacts with an infusible, acidic or basic
impurity present in the oxide ore.

PLANCESS CONCEPTS

•• In Calcination, the concentrated ore is heated to a high temperature in the absence of air.
•• In Roasting, the concentrated ore is heated to a high temperature in the presence of air.

Aman Gour (JEE 2012, AIR 230)

3.3.2 Reduction of the Oxide to Metal


The free metal is obtained by reduction of a compound, using either a chemical reducing agent or by an electrolysis.
Reduction with Carbon: A large number of commercial processes come under this category. Carbon can be
used to reduce a number of oxides and other compounds, and because of the low cost and availability of coke,
this method is widely used. The disadvantages are that, high temperature is needed, which is expensive and
necessitates the use of the blast furnace.
PbO + C → Pb + CO (extraction of lead)
2Fe2O3 + 3C → 4Fe (spongy iron) + 3CO2
1200ºC
ZnO + C 
→ Zn + CO (extraction of zinc)
2 9 . 1 0 | Extractive Metallurgy

1800ºC
SnO2 + 2C (anthracite) 
→ Sn + 2CO (extraction of tin)
1200ºC
MgO + C 
electric furnace
→ Mg + CO

Reduction with CO: In some cases CO produced in the furnace itself, is used as a reducing agent.
Fe2O3 + 3CO → 2Fe + 3CO2; Fe3O4 + 4CO → 3Fe + 4CO2
PbO + CO → Pb + CO2; CuO + CO → Cu + CO2
The reduction process of carbon or carbon monoxide is usually carried out in a blast furnace. There are some
disadvantages of using carbon as a reducing agent e.g.:

(a) Some metallic oxides like CaO give metallic carbides instead of metals. CaO + 3C → CaC2 + CO
(b) During the cooling of the products, in many cases, reformation of the oxide and carbon may take place.


MgO + C ←→ Mg + CO
 cool

Reduction by other Metals: If the temperature needed for carbon to reduce an oxide is too high, for economical
or practical purposes, reduction by other metals is done. Also, certain metallic oxides cannot be reduced by carbon
because the affinity of oxygen for the metal is greater than its affinity for carbon. Such metallic oxides (Cr and Mn)
can be reduced by a highly electropositive metal such as aluminium that liberates a large amount of energy (1675
kJ/mol) on oxidation to Al2O3. The process is known as Goldschmidt or aluminothermic process and the reaction is
known as thermite reaction.
Cr2O3 is mixed with a requisite amount of Al-powder (this mixture is called thermite mixture) and is placed on a
large fire-clay crucible. An intimate mixture of Na2O2 or BaO2 and Mg powder (called ignition mixture or igniter)
is placed in a small depression made in the thermite mixture. The crucible is surrounded by sand, which prevents
the loss of heat by radiation. A piece of Mg ribbon is struck into the ignition mixture and the charge is covered by
a layer of fluorspar (CaF2) which acts as a heat insulator. Now the Mg-ribbon is ignited so that the ignition mixture
catches fire and flame is produced, leading to a violent reaction between Mg and BaO2 with the evolution of a large
amount of heat.
Mg + BaO2 → BaO + MgO + Heat
Heat produced in the above reaction makes Cr2O3 and Al powder to react together.
Cr2O3 + Al → 2Cr (l ) + Al2O3
Molten Cr-metal formed settles down at the bottom of the crucible.
Magnesium ribbon
Fluorspar
(CaF2)
Fire-day crucible

Cr2O3 + Al-powder

BaO2 + Mg-Powder
(Ignition mixture)

Figure 29.5: Reduction of Cr2O3 by Al-powder (Aluminothermic process)

In this process, thermite mixture consisting of Fe2O3 and Al-powder in 3: 1 ratio is placed in a funnel-shaped
crucible, lined internally with magnesite, with a plug hole at its bottom. The thermite mixture is covered with a
mixture of BaO2 and Mg-powder (ignition mixture) in which a piece of Mg ribbon is inserted. The ends of the iron
pieces to be welded are thoroughly cleaned and surrounded by a fire-clay mould. When the Mg ribbon is ignited,
the ignition mixture catches fire and Fe2O3 gets reduced to Fe by Al-powder.
Chem i str y | 29.11

2Al + Fe2O3 → Al2O3 + 2Fe (molten); ∆H = – 3230 kJ (The reaction is used for thermite welding)
3Mn3O4 + 8Al → 4Al2O3 + 9 Mn; B2O3 + 2Al → 2B + Al2O3 (extraction of boron)
As it is a strongly exothermic reaction, it proceeds with explosive violence and only initial heating is needed.

Magnesium Reduction Method: Magnesium is used in a similar way to reduce oxides. In certain cases where the
oxide is too stable to reduce, electropositive metals are used to reduce halides.
kroll process IMI process
TiCl4 + 2 Mg 
1000–1150ºC
→ Ti + 2 MgCl2,; TiCl4 + 4Na → Ti + 4NaCI
Advantages of using Na and Mg as reducing agents are, the higher reducing power of the metals and solubility of
their halides in water, so that the reduced metals can be washed free from impurities.

Self Reduction method: This method is also called auto-reduction method or air reduction method. If the sulphide
ore of some of the less electropositive metals like Hg, Cu, Pb, Sb, etc. are heated in air, a part of these is changed
into oxide or sulphate. Then that reacts with the remaining part of the sulphide ore to give its metal and SO2.

Examples: 2HgS + 3O2 → 2HgO + 2SO2; 2HgO + HgS → 3Hg + SO2


2Cu2S + 3O2 → 2Cu2O + 2SO2; 2Cu2O + Cu2S → 6Cu + SO2
2PbS + 3O2 → 2PbO + 2 SO2; 2PbO + PbS → 3Pb + SO2
The extraction of Pb by heating its sulphide ore (PbS) in air can also be represented as
PbS + 2O2 → PbSO4; PbSO4 + PbS → 2 Pb + 2SO2

Electrolytic Reduction Method: It presents the most powerful method of reduction and gives a very pure product.
As it is an expensive method compared to chemical methods, it is used either for very reactive metals, such as
magnesium or aluminum, or for the production of samples of high purity. Electrolytic reduction of copper has the
additional advantage of allowing the recovery of valuable minor contaminants such as silver.
(a) In aqueous solution: Electrolysis can be carried out conveniently and cheaply in an aqueous solution, so that
the products do not react with water. Copper and zinc are obtained by electrolysis of the aqueous solution of
their sulphates.
(b) In other solvents: Electrolysis can be carried out in solvents other than water. Fluorine reacts violently with
water and it is produced by electrolysis of KHF2 dissolved in anhydrous HF.
(c) In fused melts: Elements that react with water are often extracted from fused melts of their ionic salts.
Aluminum is obtained by the electrolysis of a fused mixture of Al2O3 and cryolite Na3 [AlF6]. Both sodium and
chlorine are obtained from the electrolysis of fused NaCl. In this case, upto two-thirds by weight of CaCl2 is
added as an impurity to lower the melting point from 803 to 505ºC.

Electrochemical Principles of Metallurgy: Electrolytic reduction can be regarded as a technique for driving a
reduction by coupling it through electrodes and external circuits to a reactive or a physical process with a more
negative ∆G. The free energy available from the external source can be assessed from the potential it produces,
across the electrodes, using the thermodynamic relation:
∆G = – nFE … (i) where, n is the number of electrons transferred, F is Faraday’s constant (F = 96.5 kJ/mol) and E° is
the electrode potential of the redox couple, formed in the system.
Hence, the total Gibb’s energy of the coupled internal and external process is ∆G + ∆G (external) = ∆G – nFEext
∆G
If the potential difference of the external source exceeds Eext = – then, the reduction is thermodynamically
nF
feasible. Thus, the overall process occurs with a decrease in free energy. More reactive metals have large negative
values of the electrode potential. So their reduction is difficult. If the difference of two Eº values corresponds to a
positive Eº and consequently negative ∆Gº in equation (i), then the less reactive metal will come out of the solution
and the more reactive metal will go to the solution, e.g., Cu2+ (aq) + Fe(s) → Cu(s) + Fe2+(aq)
2 9 . 1 2 | Extractive Metallurgy

Hydrometallurgy: The processing of extraction of ores and minerals as well as metals and their compounds
at relatively low, often ambient temperatures employing an aqueous solution is known as hydrometallurgy.
Occasionally, organic reagents are also used. This method of extraction is generally used for low grade ores.
Copper is extracted by hydrometallurgy from a low grade ore. It is leached out using acid and bacteria. The solution
containing Cu+2 is treated with scrap iron or H2.
CuSO4 + Fe → Cu(s) + FeSO4
A hydrometallurgical process for the extraction of metals from ores, concentrates, or secondary materials essentially
contains three basic steps—dissolution of the valuable metal in the aqueous solution (leaching), purification of the
leach solution and the subsequent recovery of metal from the purified solutions either by electrolysis or by adding
some electropositive metal to it. Some of the metals obtained by hydrometallurgy are as follows:

Extraction of Ag and Au: Metals like Au and Ag can be precipitated for their salt solution by electropositive metals
e.g. Zn. Metallic Ag is dissolved from its ore in a dilute NaCN solution, and the solute so obtained, is treated with
scrap Zn when Ag is precipitated. Air is blown into the solution to oxidize Na2S. Leaching the metals like silver, gold
with CN– is an oxidation reaction (Ag → Ag+ or Au → Au+)
Ag2S(s) + 4CN–(aq) → 2 [Ag (CN)2]– (aq) + S2–(aq)
2[Ag (CN)2]– (aq) + Zn(s) → [Zn(CN)]2– (aq) + 2Ag (s)
4Au (s) + 8 CN– (aq) + O2 (g) + 2H2O (I) → 4 [Au (CN)2]– (aq) + 4OH– (aq)
2[Au (CN)2]– (aq) + Zn (s) → [Zn(CN)4]–2(aq) + 2 Au (s)
Here Zn acts as a reducing agent.
The leaching pitch blends with H2SO4 or sodium carbonate to dissolve uranium:
1 1
U3O8 + 3Na2CO3 + O2 → 3Na2UO4 + 3CO2; U3O8 + 3 H2SO4 + O → 3UO2SO4 + 3H2O
2 2 2
Precipitation of Mg (OH)2 from sea water using lime solution:
MgCl2 + Ca (OH)2 → Mg(OH)2 + CaCl2

Oxidation Reduction: Besides reductions, some extractions are based on oxidation particularly for non-metals.
(a) A very common example of extraction based on oxidation is the extraction of chlorine from brine (chlorine is
found abundantly in sea water as common salt). 2Cl–(aq) +2H2O(I) → 2OH-(aq) + H2(g) + Cl2(g)
The ∆Gº for this reaction is + 422 kJ, when it is converted to Eº (using ∆Gº = -nE°F), we get E° = -2.2 V. Naturally, it
will require an external e.m.f. that is greater than 2.2 V. But the electrolysis requires an excess potential to overcome
some other hindering reactions. Thus, Cl2 is obtained by electrolysis giving out H2 and aqueous NaOH as by-
products. Electrolysis of molten NaCl is also carried out. But in that case, Na metal is produced and not NaOH.

Illustration 5: Common impurities present in Bauxite are:


(A) CuO (B) ZnO (C) Fe2O2 (D) SiO2  (JEE ADVANCED)

Sol: (C) Red Bauxite contains Fe2O3 as impurity.


(D) White Bauxite contains SiO2 as impurity

Illustration 6: Which metals are generally extracted by the electrolytic reduction and why?  (JEE MAIN)

Sol: More reactive and electropositive elements acts as strong reducing agents like. Sodium, aluminium, magnesium
etc. Hence they cannot be extracted by any of the chemical methods.
Chem i str y | 29.13

3.4 Purification or Refining of Metal


Metals obtained by any of the above mentioned reduction processes are not pure and require further refining.
There are two methods for the refining of crude metals -Thermal refining and Electro refining.

3.4.1 Thermal Refining


Oxidation by Air and Poling: This process is applicable for refining of Cu and Sn. In this process, the crude metal
is melted and air is blasted through the melt. After air is blown, the melt is stirred with a raw wooden pole and its
unburnt condition produces sufficient amount of carbon and CO to reduce the metallic oxide formed and yield
a refined metal. A coke powder layer is maintained at the top of the surface to prevent reoxidation of the metal
formed (figure). A small amount of metal to be refined, however, may get oxidized in this process.

Raw wooden
pole is used
as stirrer

Coke powder layer


at the top surface

Air

Figure 29.6: Diagrammatic representation of oxidation by air and poling

The more basic metallic impurities are preferably oxidized by oxygen or air, forming volatile or nonvolatile oxides
(i.e. scum). The less basic impurities (if any) are not removed by oxidation, because under this situation, metal to be
refined will be oxidized in preference.

Fractional Distillation: This refining process utilizes the boiling point difference between the metal and that of the
impurity. Using this process, crude zinc containing Cd, Fe and Pb as impurities can be refined as follows:
Zn (vap)
Cd : b.p = 767C T > 767C T > 920C
Pb
Crude Zn Fe : Zn
-Cd (vap) Fe
b.p > 1500C
Pb : Fe and Pb

b.p = 920C

At a temperature >767º C, Cd separates as vapour; and at temperature >920 °C, pure Zn separates as vapour,
leaving behind Fe and Pb impurities in the melt.

Liquation: This method is applicable for metals, such as Sn, Pb and Bi, which
have low melting points as compared to their impurities. In this method,
the block of crude metal is kept at the top of the sloping furnace and
heated just above the melting point of the metal to be refined. The pure
metal melts and flows down the sloping hearth and gets collected in a
receiver at the bottom of the slope (figure). The perforated block of
impurity is thrown later.
The impurity content has to be high enough in the crude metal, otherwise Fuel

the impurities also flow down with the molten metal.


+ air

Molten
metal

Figure 29.7: Diagrammatic representation for


liquation
2 9 . 1 4 | Extractive Metallurgy

Zone Refining: Metals like Si, Ge and Ga of high purity (which are used in semiconductors) are purified by this
method. This process is known as ultra-purification, because it results in the impurity level decreasing to ppm
level. Zone refining is based upon fractional crystallization, as the impurity prefers to stay in the melt and on
solidification only the pure metal solidifies on the top surface of the melt. In this process, a ring furnace is heated to
a suitable temperature for melting the metal rod (figure (a)), producing a thin zone throughout the cross-sectional
area as shown in Figure (b). It is desirable that the diameter of the rod, d, is small enough to give a uniform melt.

Ring furnace

Metallic rod
to be purified

Melted zone

Figure 29.8-a: Metal rod heated by ring furnace

When the melted zone in the metal rod is ready, the furnace is allowed to move downwards very slowly, together
with the melted zone (figure (c)). The furnace is then switched off, cooled down and taken to the top again for a
repetition of the process. Almost all the impurity sweeps out to the bottom after several repetitions of the process.

Correct Incorrect

Figure 29.8-b: Cross-sectional view of the metal rod

Metallic rod

Ultrapure metal

Ring furnace

Figure 29.8-c: Ring furnace moving down the metal rod

Vapour Phase Refining: The two essential criteria for the vapour refining process are listed below:

(a) The intermediate compound formed has to be volatile.


(b) The intermediate compound formed has to be relatively unstable, i.e. it should decompose on heating at a
practically achievable temperature.
Chem i str y | 29.15

This refining technique is used in the following purification processes:


(i) Mond’s process (for purification of nickel):
50–80ºC 150–180ºC
 Ni(s) + 4CO(g) → Ni(CO)4 (g)  → Ni(s) + 4CO(g)
Impure volatile Pure Recycled

If Ni (CO)4 is not volatile, it cannot be separated from impurities, so, its volatile nature helps to free it from
impurities. Also Ni (CO)4 needs to undergo thermal decomposition easily, otherwise it cannot produce any
pure metal.

(ii) Van-Arkel-de Boer process (for purification of zirconium, boron and titanium):

l ( vap)
Zr or Ti 
2 1400ºC
→ Zrl4 or Til4  → Zr(s)or Ti(s) + 2l2 (g)

  250ºC
 on gestem filsment  recycled
impure Volatile pure

3 ∆ 3
B + l2 (vap) → Bl3  → B + l2 (g)
impure 2 volatile Pure 2
recycled

3.4.2 Electro Refining


This method is applicable for the purification of Cu, Zn, Sn, Ag, Au, Ni, Pb and Al. The cathode is made of thin strips
of pure metal (same as that to be refined) and the anode is made of large slabs of impure metal (to be refined).
The electrolyte is the aqueous solution of a suitable salt of the metal (to be refined) or sometimes, the melt of an
oxide/salt.
+

Cathode of pure
Metal (m)
Anode of
impure Metal (M) Electrolyte (MX)

Anode mud

Figure 29.9: Diagrammatic representation of electro refining

The metal gets corroded from the anode. Pure metal gets deposited at the cathode and is purified in this way.

Reactions at the electrodes: At cathode: Mn++ne- → M


1
At anode : (i) M → Mn++ne- (wanted) (ii) Xn– → X + ne- (unwanted)
2 2
There are two reactions competing at the anode. So the anionic part of the electrolyte is to be chosen in such a way
that the reaction (ii) does not take place at the anode.
Hence at a particular moment, the number of moles of metal dissolved in the electrolyte is equal to the number of
moles of metal ions deposited at the cathode. The concentration of metal ion in the electrolyte remains the same
at a particular time.
The metallic impurities having lower oxidation potential than that of the metal to be refined are separated in the
form of anode mud at the bottom. The suitable electrolytes used in general for respective metals are:
Cu: CuSO4 solution Sn: SnSO4 solution Ni : N2(NO3)2 solution
Zn: ZnSO4 solution Ag: AgNO3 solution Pb: PbSiF6 solution
Au: AuCl3 solution Al: Al2O3 + (Na2AIF6) melt + BaF2
2 9 . 1 6 | Extractive Metallurgy

PLANCESS CONCEPTS

•• In the Poling process, a wooden pole is used and the hydrocarbons reduce the metal oxide impurities
to refined metal.
•• Fractional distillation utilizes the difference in the boiling points between the metal and its
impurities.
•• Liquation is applicable to metals having much lower melting points, as compared to its impurities.
•• Zone refining is based upon fractional crystallization, as the impurity prefers to stay in the melt and
the pure metal solidifies on cooling.
•• Mond’s process is used for nickel, as Ni(CO)4 is volatile and on high temperature decomposes to give
back pure nickel metal.
•• Van arkel de boer’s process is used for purification of Zr, B and Ti, as their iodides are volatile and on
higher temperature gives back the pure metal. Electro refining is used for purification of Cu, Zn, Sn ,
Ag, Au, Ni, Pb and Al. Anode consists of impure metal and cathode consists of pure metal.

B Rajiv Reddy (JEE 2012, AIR 11)

4. THERMODYNAMICS OF EXTRACTION: ELLINGHAM DIAGRAM OF


A METAL
The standard electrode reduction potential of metal provides a very good indicator, or, the ease or difficulty of
extracting the metal from its compounds. However, since most metals of industrial importance are obtained by
the chemical reduction of their oxide, the free energy changes occurring during these processes are of more
fundamental importance. Despite the fact that redox reactions do not always reach equilibrium, thermodynamics
can at least be used to identify which reactions are feasible. For a spontaneous reaction the change in free energy
∆G must be negative,
∆G = ∆H - T∆S.
It is sufficient to consider ∆G because it is related to the equilibrium constant through ∆G=-RTlnK.
Here, a negative value of ∆G corresponds to K > 1 and, therefore, a favorable reaction. Reaction rates are also
relevant, but at a high temperature, reactions are often fast and we can normally assume that any thermodynamically
permissible process can occur. The problem of extracting a metal from its ore is essentially concerned with
decomposing the oxide of the metal (apart from simple binary compounds such as metal sulfides and chlorides
that occur in nature). Most metal ores consist essentially of a metal oxide in association with one or more nonmetal
oxides. Ores like carbonates, sulphides etc. are also converted to oxides, prior to reduction.
The free energy of formation, ∆G is the standard free energy of the reaction.
y
xM+ O → MxOy  … (i)
2 2
2x 2
or M + O2 → MO … (ii)
y y x y

∆G is the free energy of formation per mole of O2 consumed.


If the standard free energy of formation, ∆G has a negative sign at a given temperature, then the oxide can be
expected to form spontaneously from the metal plus oxygen. If ∆G has a positive sign, the oxide will be expected
to decompose spontaneously into its elements.
The free energy of formation of an oxide can now be determined, provided we know the entropy of the formation.
∆G = ∆H – T∆S  ... (iii)
Chem i str y | 29.17

In reaction (ii) oxygen is used up in the course of the reaction. Gases have a more random structure (less ordered) than
liquid or solids. In this reaction, the entropy or randomness decreases, and hence ∆S is negative (provided that neither
the metal nor its oxide MxOy are vaporized). Thus, if the temperature is raised then T∆S becomes more negative. Since,
T∆S is subtracted in equation (ii), ∆G becomes less negative. Thus, the change in free energy decreases with increase
in temperature. The free energy change that occurs when 1 mol of common reactant (in this case O2) is used, may be
plotted graphically against the temperature for a number of reaction of metals to their oxides. This graph is shown in
the following figure and is called an Ellingham diagram for oxides. Similar diagrams can be produced for 1 mol of S,
giving Ellingham diagram for sulphides and similarly for halides using 1 mol of halogen.

This figure shows a number of oxide plots with slopes defined Ag O 2

by ∆G / T = – ∆S. lt is noted that the entropy change in reaction HgO CuO


(ii) is roughly the same for all metal oxides, provided that the +250
boiling point of neither the metal nor oxide is exceeded. Thus,
below the boiling point of metal, the slope of all the graphs are

Free energy change (kJ/mole)


roughly the same, since T∆S factor is the same whatever the 0
COCO
metal may be. When the temperature is raised, a point will be
2

ZnO
reached where the graph crosses the ∆G = 0 line. Below this
temperature the free energy of formation of oxide is negative,
FeO
CCO
so the oxide is stable. Above this temperature the free energy
2

of formation of the oxide is positive, and the oxide becomes -500

unstable and should decompose into metal and oxygen. This


explains why HgO3 for instance, decomposes spontaneously
SiO 2
CCO
into its elements when heated. Theoretically, all oxides can be
decomposed to give metal and oxygen, if a sufficiently high Al O MgO
temperature can be attained. In practice, that is easily attainable
2 3
-1000 TiO 2
CaO
and these metals can be extracted by thermal decomposition
of their oxides. The diagram predicts that MgO and ZnO ought
to decompose if heated strongly enough, but it does not hold 500 1000 1500 2000 2500
out much hope for obtaining, say, pure Mg by straight forward
Figure 29.10: Free energy vs temperature ˚C
heating of the oxide to a high temperature where the boiling
point of the metal is exceeded. However the slope increases, since the reaction now involves a larger entropy change
as the randomness increases in reactants. For example, 2Mg (g) + O2(g) → 2MgO(s)

Here, three moles of gas phases are converted into solid phases in the reaction. This takes place above 1120°C,
which is the boiling point of Mg. Similarly Hg—HgO line changes slopes at 365°C. Several of the plots show abrupt
changes in the slopes. These breaks occur at temperatures at which the metal undergoes a phase transition. A
smaller effect is seen at the melting point. If, however the oxide undergoes a phase change, there will be an increase
in the entropy of the oxide, and at such a point the curve becomes less steep. For example in the case of Pb, the
oxide (PbO) boils, while lead is liquid. In these instances the entropy change becomes positive for the reaction and
hence the slope ∆G/T changes sign, and, the situation reverts to normal once the boiling point of Pb is reached.

In principle, when the plot of one metal lies below that of another, the first metal is capable of reducing the oxide
of the second. A vertical line drawn on the Ellingham plot of the metal oxides at any T gives the sequence of the
stabilities of metal oxides. A metal forming a more stable oxide (higher - ∆G) will be the potential reducing agent
for a less stable oxide. If the two lines intersect, the free energy change for the reduction will be zero at that
temperature and equilibrium results, but a change of temperature will make the reaction to proceed, provided no
kinetic barriers (activation energy) exist. Thus, Mg metal will reduce CuO and FeO, but not CaO. Also, it is seen that
at room temperature (27°C) the order of the reducing ability approximates that of standard electrode potential.

Although the SiO2 line is above the MgO line, Si can successfully reduce MgO to free metal. Upto 1100°C, the
normal boiling point of Mg, the ∆G plot for formation of SiO2 and MgO are parallel. However, above 1100°C the
plot for MgO changes slope, owing to the increased entropy effect, and above 1700°C the reaction between Si and
MgO proceeds with decrease in free energy. In practice, the reaction is further enhanced by the distillation of Mg
metal from the reaction mixture.
2 MgO + Si → 2 Mg + SiO2
2 9 . 1 8 | Extractive Metallurgy

Carbon/Carbon Monoxide as reducing agent: In figure, the plot corresponding to the change
C(s) + O2(g) → CO2(g) is shown by a horizontal line. For this reaction ∆S is relatively small because in this case one
mole of gaseous product is formed while one mole of gaseous reactant is used up. ∆G for this reaction is almost
independent of temperature. The plot for CO2 is relatively high in the figure, and at a lower temperature, C will
reduce only a few of the metal oxides as shown. However, the slopes of the plots for several of the metals are such
that they cross the CO2 plot; hence, theoretically these metals can be reduced by C at an elevated temperature. An
alternative reaction involving carbon and oxygen is the formation of CO.
2C(s) + O2 (g) → 2CO(g)
Since two gaseous products are formed from one mole of a gaseous reactant, this process is accompanied by an
increase in entropy. Hence, the slope of the corresponding line is negative as shown, by the downward sloping
line in the figure. If the temperature is high enough, C should reduce all the metal oxides, being converted into
CO. The plot for the reaction of CO with oxygen is also shown. There are three curves for carbon, corresponding
to complete oxidation of C to CO2, partial oxidation to carbon monoxide, and oxidation of CO to carbon dioxide.
The three curves pass through a common point at 710°C. Thus, the free energies of formation of CO2 from carbon
monoxide and carbon dioxide from carbon are identical.
2CO (g) + O2 (g) → 2 CO2 (g) ∆G = x kJ/mol ; C(s) + O2(s) → CO2(g) ∆G = x kJ/mol
Subtracting one equation from the other and rearranging, the following is obtained:
CO2 (g) + C(s) → 2CO (g) ∆G = 0
i.e. an equilibrium is set. It is clear, that, below a temperature of 710°C, CO is a more effective reducing agent than
carbon, but above this temperature, the reverse is true. All three oxidation curves for the carbon system lie above
that for the oxidation of zinc, until a temperature of approximately 1000°C is reached. At this point, carbon is
thermodynamically capable of reducing ZnO to Zn. Since this temperature is greater than the boiling point of Zn
(907°C), it will be formed as a vapour. The overall equation for reduction is ZnO(s) + C (s) → Zn(g) + CO(g)

It is interesting to note that the value of carbon as a reducing agent is due to a marked increase in disorder that
takes place when carbon (an ordered solid) reacts with one mole of oxygen to give two moles of CO. The net effect
is an extra mole of gas and hence an increase in disorder (an increase in entropy). It is a fact, that, in the region
of 2000°C, carbon is thermodynamically capable of reducing most metal oxides to metal. Thus, for most metal
oxides, a reducing agent is required and we should consider the overall reaction obtained by subtracting the metal
oxidation from one carbon oxidation as

0 Zinc melts
Zinc boils

2Zn + O22ZnO
100
2CO+ O22CO2
Free energy change (kJ/mole)

200
2Fe+O2  2FeO

C + O2  CO2
400

2C+ O2 2CO

600

800
500 750 1000 1500 2000
Figure 29.11: Free energy vs temperature ˚C

∆Goverall = ∆G(O) – ∆G(M)


Chem i str y | 29.19

Metals as reducing agents: Metal oxide reduction is thermodynamically favorable for temperatures at which the
line for the metal oxide is above any one of the lines for carbon oxidation. The ∆G for metal oxide reduction by
carbon is negative.
Note: The Gibb’s energies of formation of most sulphides are greater than that for CS2. In fact, CS2 is an
endothermic compound. Therefore, the ∆FG° of MxS is not compensated. So reduction of MxS is difficult. Hence, it
is common practice to roast sulphide ores to corresponding oxides prior to reduction. Similar principles apply to
other types of reduction. For instance, if the plot of ∆G(M) lies above ∆G(M’), M’ is now taking the place of C. When
∆G = ∆G(M’) – ∆G(M) is negative, the reaction is, M’O + M → M + M’O
Hydrogen as a reducing agent: Hydrogen is not a very effective reducing agent for obtaining metals from their oxides.
The reason is that ∆ S is negative for the reaction: 2H2 (g) + O2 (g) →  2H2O(g) as the products are less
3 moles of gas 2 moles of gas

disordered. The plot of ∆G against T therefore rises with temperature, meaning that not many metal oxide plots
are intersected. H2 will therefore reduce oxides such as Cu(I) oxide and Cu(ll) oxide, but not the oxides of Al, Mg,
and Ca. Oxides of iron are reduced only with difficulty. In the case of magnetic iron oxide, Fe3O4, an equilibrium
composition is readily established.
In the case of W, Mo, and Co, ∆G is above that of H2O so that H2 can reduce these oxides.
MoO3 + 3H2 → Mo + 3H2O ; GeO2 + 2H2 → Ge + 2H2O
Co3O4 + 4H2 → 3Co + 4H2O ; WO3 + 3H2 → W + 3H2O
This method is not widely used because many metals react with H2 at elevated temperatures.

PLANCESS CONCEPTS

•• If in Ellinghams diagram, a metal A is placed above metal B then metal A can be reduced by metal B.
•• The free energy change increases with temperature because the slope is ∆S which is negative.
Rohit Kumar (JEE 2012, AIR 79)

Illustration 7: The reaction Cr2 O3 + 2Al → Al2O3 + 2 Cr (∆Gº = – 421 kJ) is thermodynamically feasible as is
apparent from the Gibb’s energy value. Why does it not take place at room temperature?  (JEE MAIN)

Sol: Certain amount of activation energy is essential even for such reactions which are thermodynamically feasible,
therefore heating is required.

Illustration 8: Is it true, that under certain conditions, Mg can reduce Al2O3 and Al can reduce MgO? What are
those conditions?  (JEE MAIN)

Sol: Below 1350ºC, Mg can reduce Al2O3 and above 1350°C, Al can reduce MgO, as evident from the Ellingham
diagram.

5. METALLURGY OF SOME IMPORTANT METALS

5.1 Extraction of Silver


Important ores of silver are
(a) Argentite or silver glance: Ag2S
(b) Ruby silver: Ag3SbS3
(c) Horn silver: AgCl
2 9 . 2 0 | Extractive Metallurgy

There are three processes commonly used for the extraction of Ag; The cyanide process is described below. The
other two processes, i.e. Parke’s process and Pattinson’s process are beyond the scope of discussion. Cyanide
process or Mc-Arthur Forest Process: This process is depicted in the flow diagram given in figure.

Flowchart 29.2: For Mc-arthur forest for extraction of silver

Reactions taking place at the different stages in the cyanide process are as follows:

(i) Ag2S + 4Na CN(excess) ←→ 2[Ag(CN) | – + Na S + 2Na–
2 2

4Na2S + 5O2(air) + 2H2O → 2Na2SO4 + 4NaOH + 2S


Since the above reaction is reversible, the conversion ratio is not good. Hence the process is carried out in the
presence of air which converts the Na2S produced into Na2SO4 and S, and the overall reaction becomes unidirectional.
(ii) In the step involving the precipitation of Ag, a little excess of Zn powder is added by which Na[Ag(CN)2]
becomes the limiting reagent, otherwise the loss of Ag will be more. Here Zn is chosen because
it is more electropositive as compared to Ag and the replacement reaction occurs very easily.
2Na[Ag(CN)2] + Zn → Na2[Zn(CN)4] + 2Ag ↓

Refining of Ag: Refining of silver is carried out by electrolytic process.


electrorefining
Crude Ag  
(mobiusprocces )
→ Pure Ag
( Zn, Cu, Au as impurity ) (upto 99.6 – 99.9% pure Ag)
Electrolyte : AgNO3 solution + 10% HNO3
Cathode : Pure Ag strip
Anode : Impure Ag slab
Reactions at the electrodes:
At cathode : Ag+ + e- → Ag
At anode : Ag → Ag+ + e-

5.2 Extraction of Gold


The flow diagram of different stages involved in the cyanide process for extraction of gold is shown in figure.
Residue: Solid impurity Residue: Grade Au
Finely
divided Digestion Zn powder
auriferous in excess and
rock filtered
Filtrate Filtrate:
Na[Au(CN)2] Na2[Zn(CN)4]
Flowchart 29.3: Cyanide process for extraction of gold
Chem i str y | 29.21

Auriferous rock is rock of quartz contaminated with gold linings.


Reactions taking place in different steps are:
In step 1: 4Au + 8NaCN + 2H2O + O2 → 4Na[Au(CN)2] + 4NaOH
Here, oxidation of Au is not possible without the presence of air and NaCN acts as a complexing agent.
In step 2: 2Na[Au(CN)2]+ Zn → Na2[Zn(CN)4] + 2Au ↓

Refining of Au: The steps involved in the refining of Au are given in figure. In the third step, on heating with borax,
the soluble metaborate of Cu, i.e. Cu(BO2)2 is formed and washed out with water. Similarly, in the fourth step also,
Ag dissolves out as Ag2SO4 leaving behind pure Au.
Note:
•• The removal of Ag can also be carried out using chlorine or by electrolysis.
•• Cupellation is a process, where crude gold is taken in a small bowl called cupell and melted in the presence
of air. Due to the high oxygen affinity of Pb, it is converted into PbO (volatile) which escapes from the system.

Crude Au
(Zn, Pb, Cu Add dil. H2SO4 Cupellation
Au [Pb, Cu, Ag] Au [Cu, Ag]
and Ag are Removes Zn as Removes Pb as
impurities) ZnSO4 + H2 volatile PbO Heated with
borax and washed
with hot water
(To remove Cu)
Pure Parting Au [Ag]
Au Heated with conc.
H2SO4 and washed
with hot water

Flowchart 29.4: Refining of gold

PLANCESS CONCEPTS

•• Extraction of gold from auriferous rock is done in 2 steps; first it is oxidised by oxygen and treated with
NaCN and then it is reduced by zinc.
•• During refining, firstly, dilute H2SO4 removes Zn, then cupellation removes Pb, Cu is removed after heating
with borax and washing with water and finally parting’s process removes Ag.
Krishan Mittal (JEE 2012, AIR 199)

5.3 Extraction of Tin


An important ore of tin is cassiterite or tin stone (1-5% SnO2 present in it). The main impurities present in the ore
are sand (SiO2), pyrite of Cu and Fe, and wolframite [FeWO4 + MnWO4]. The for extraction of Sn is depicted below.
2 9 . 2 2 | Extractive Metallurgy

SnO2
Gravity
Finely seperation Concentrated Roasting CuSO4, CuO
divided ore containing
It is performed in a FeSO4, FeO
ore SiO2 controlled way so that (FeWO4 +....) SiO2 Washed with
formation of soluble Small amount hot water,
sulphate is favored. removes CuSO4
Removes SO2 and AS2O3 and FeSO4

Metallic
Sn Black SnO2
Carbon Magnetic
Tin separation CuO + FeO
reduction
Slag Anthracite SnO2 Removes (FeWO4 +....)
Coke + 75% (FeWO4+ ...) SiO2
Smelted with 10-20%
limestone
Coke + CaO Sn
Temp. -
FeSiO4 1200C
to 1300C
+ SnSiO3

Metallic
Sn

Flowchart 29.5: Extraction of tin

The reactions taking place in the extraction of Sn are:


1. In step 1: SnO2 + 2C → Sn + 2CO ↓
2. In step 2: SnSiO3 + CaO + C → Sn + CaSiO3 + CO ↓
Refining of Sn: The steps involved in the refining of Sn are shown below.

Crude Sn Liquation Purer Poling 99.5% Electrorefining 99.98%


Fe, Cu oxides Sn Remove scum pure Sn Electrolyte: SnSO4 pure Sn
cathode: Pure Sn
and tungsten are
Anode: Impure Sn
impurities

Flowchart 29.6: Refining of tin

PLANCESS CONCEPTS

•• Extraction of tin involves gravity separation, followed by roasting, followed by magnetic separation and
finally carbon reduction. The slag is also smelted.
•• In refining, liquation is followed by poling’s process and finally by electro refining.
T P Varun (JEE 2012, AIR 64)

5.4 Extraction of Magnesium


The important ores of magnesium are:
1. Magnesite : MgCO3
2. Dolomite : MgCO3.CaCO3
3. Carnallite : KCl.MgCl2.6H2O
Chem i str y | 29.23

4. Kainite : K2SO4.MgSO4.MgCl2.6H2O
5. Kieserite : MgSO4.H2O
6. Asbestos : CaMg3(SiO3)4
7. Spinel : MgO.A2O3
Two processes recommended for extraction of Mg are electrolytic reduction and carbon reduction. The latter is a
very costly process due to very high temperature requirement.

(a) Electrolytic reduction: The process for extraction of magnesium consists of the following three steps.
Step 1: Preparation of hydrated MgCl2
(i) From Carnallite (KCl.MgCl2.6H2O): The steps involved in the preparation of hydrated MgCl2 from Carnallite
are depicted in figure. The principle behind this process is that KCl is less soluble as compared to MgCl2
and crystallizes first.
Residue
KCl crystals
KCl.MgCl2.6H2O Residue:
Concentrated by MgCl2.6H2O
Dissolved in water evaporation. Cooled and Concentrated
Filtrate: Crystals
allowed to crystallize and Cooled and
filtered MgCl2 solution allowed to
crystallize, Filtrate:
filtered MgCl2
Saturated
Recycled solution

Flowchart 29.7: Preparation of hydrated MgCl2 from carnallite

(ii) From sea water: Sea water consists of a lot of MgCl2. The process for extraction of MgCl2, from sea water
is known as Dow sea water process. The steps involved in the process are depicted in figure. The principle
behind this process is that Ca(OH)2 is soluble in water while Mg(OH)2 is sparingly soluble in water.

Filtrate: Thrown

Lime water
Concentrated
Sea water
Add dil.
Residue: MgCl2
Mg(OH)2 HCl solution Solution
Concentrated
by evaporation
Cooled and
allowed to
crystallize

Filtrate: Residue
Saturated MgCl 2.6H2O

Solution of crystals
MgCl2 (recycled)

Flowchart 29.8: Preparation of hydrated MgCl2 from sea water


2 9 . 2 4 | Extractive Metallurgy

Step 2: Conversion of hydrated MgCl2 to anhydrous MgCl2: On direct heating of MgCl2.6H2O2, anhydrous MgCl2

cannot be obtained due to its hydrolysis: MgCl2 . 6H2O → Mg(OH)Cl + HCl↑ + 5H2O↑
Mg(OH)Cl → MgO + HCl↑
Also, MgO formed is an unwanted substance in the electrolytic reduction step due to its very high melting point (i.e.
2850ºC). Hence hydrated MgCl2 is heated to 175ºC in vacuum, in a current of dry HCl gas, and MgCl2 (anhydrous)
is formed.

MgCl2.6H2O → MgCl2 + 6H2O; MgCl2 + H2O →Mg(OH)Cl + HCl
When dry HCl is present in the system, the hydrolysis equilibrium shifts towards left but due to increase in thermal
energy of the system, the decomposition reaction continues and results in anhydrous MgCl2. An alternative method
for preparation of anhydrous MgCl2 involves heating calcined magnesite. (MgO) to 1000ºC in a current of dry Cl2
gas.
–1000ºC
MgCO3 

→ MgO + CO2; MgO + C + Cl2 → MgCl2 + CO↓

Step 3: Electrolytic reduction of anhydrous MgCl2: The schematic representation of electrolytic reduction of
anhydrous MgCl2 is shown in figure:
MgCl2 → Mg2+ + 2Cl– : NaCl → Na+ + Cl–
At cathode : Mg2+ + 2e– → Mg ;
At anode : 2Cl– – 2e– → Cl2
The container for electrolytic cell has to be covered and the air present inside is removed by passing the coal gas
through an opening (not shown in the diagram) to prevent the oxidation of Mg formed, which is floating on the
top surface of the molten electrolyte.

Cl2 gas

Graphite rods
act as anode

Steel container itself


acts as cathode

Electrolyte: Four parts


Anhyd, MgCl2 +
one part anhyd.
NaCl (in molten state)

Figure 29.12: Electrolytic reduction of anhydrous MgCl2

The electrolyte in the molten state consists of four parts of anhydrous MgCl2 and one part anhydrous NaCl. One
part of NaCl is added to reduce the melting temperature of the electrolyte from 1200 °C (m.p. of pure MgCl2) to
700°C and at the same time electrical conductivity of the melt is also increased.
Fused carnallite may also be used as an electrolyte because both Na and K are more electropositive as compared
to Mg and hence Mg2+ is preferably discharged at the cathode.
Chem i str y | 29.25

(b) Carbon reduction process: In this process for extraction of magnesium, initially MgO is produced from
calcination of MgCO3, which is then directly heated with coke powder at around 2000 °C in a closed electric
furnace. MgCO3 → MgO + CO2; MgO + C → Mg ↑ + CO ↑
Mg thus obtained comes out in the vapour form and cooled suddenly to about 200 °C by dilution with a large
volume of H2 gas to prevent reoxidation of Mg.

Note: Boiling point of Mg is –1100 °C.

(c) Other processes: Some other processes used for extraction of magnesium are:
(i) Thermal reduction of MgCl2: The mixture of anhydrous MgCl2 and CaC2 is heated at 1500ºC and the
vapours of Mg are collected in the same way as in the carbon reduction process.
MgCl2 + CaC2 → CaCl2 + Mg+2C
(ii) Pidgeon process: Here the calcined mixture of dolomite and powder of ferrosilicon (80% Si) is heated
to 1200°C in a closed container. Mg is vaporized and collected in a condenser.
CaCO3 +MgCO3 → CaO +MgO + 2CO2; 2MgO + 2CaO + Si → 2Mg↑ +2CaSiO3

PLANCESS CONCEPTS

Extraction of magnesium involves the following steps - preparation of hydrated MgCl2 and conversion of
hydrated MgCl2 to anhydrous MgCl2
Aishwarya Karnawat (JEE 2012, AIR 839)

5.5 Extraction of Aluminium


The important minerals of Al are:

1. Hydrated oxides: Bauxite (Al2O3.3H2O) Gibbsite (Al2O3.3H2O)

china clay or Kaolin (Al2O3.2H2O) Diaspore (Al2O3.H2O)

2. Oxide : Corundum (Al2O3)

3. Sulphate : Alunite [K2SO4.Al2(SO4)3.4Al(OH)3]

4. Fluoride : Cryolite (3NaF.AlF3)

5. Aluminate : Spinel (MgO.Al2O3)

6. Silicate : Feldspar (K2O.Al2O3.6SiO2)

Bauxite is the ore most commonly used for extraction of aluminium. Bauxite is of two types:

1. Red bauxite: Al2O3.2H2O + Fe2O3 (major impurity) + SiO2+TiO2

2. White bauxite: Al2O3.2H2O+SiO2 (major impurity) + Fe2O3+TiO2

The main steps involved in the extraction of Al from bauxite are:

1. Beneficiation of bauxite and preparation of pure alumina.


2. Electrolytic reduction of pure alumina.
3. Electrorefining of aluminium.
2 9 . 2 6 | Extractive Metallurgy

Beneficiation of bauxite: Different processes adopted for beneficiation of red and white bauxite are described
below.
Bayer’s process: This is used for beneficiation of red bauxite. Various stages of the process are depicted in the
figure.

Finely powdered bauxite


Al2O3.2H2O + Fe2O3 Calcination
Calcined ore
SiO2+TiO2 FeO [if any]Fe2O3
Removes organic matter
Digested with 45% NaOH
Solution at 150C and
80 Ib pressure for 8 hrs
in an autoclave
Filtrate:
Na2SiO3 + NaOH Filter

(i) Diluted with water


Filtrate Residue
Together with
Freshly prepared
NaAlO2 + Na2SiO3 Fe2O3.TiO2
All(OH)3 as seeding + other insoluble
Residue agent impurities
Al(OH)3 or
ignited (ii) CO2 is passed
at  100C

Pure Al2O3

Flowchart 29.9: Bayer's process for beneficiation of bauxite

The reaction involved in the above process are:

In step 1: Al2O3 + 2OH– + 3H2O → 2[Al(OH)4]– ; SiO2 + 2NaOH → NaSiO3 + H2O

− 
→ Al(OH) ↓ ←

→ Al3+ + 3H O Since Al(OH) is amphoteric in nature, it gets dissolved
In step 2: [Al(OH)4 ] ← 3 2 3
OH
white
in alkali as well as in acid as shown in the reactions above. Hence to get Al(OH)3 from [Al(OH)4]-, the pH of the
medium is to be reduced, but care has to be taken, that the pH is not low enough to dissolve it further in the form
of Al3+. The decrease in pH can be done by:-
•• Adding large amounts of water which increases the volume and decreases the pH. But without the seeding
agent (which provides the nucleus of the precipitate) the precipitation is delayed.
•• Acidification which reduces the pH of the medium. But here a strong acid is not recommended to avoid any
further dissolution of Al(OH)3 in the form of Al3+. Hence weak acids like CO2 gas are passed, which reacts as
follows: CO2+2OH– → CO32–+ H2O
CO2 + H2O → H2CO3 [weak enough to dissolve Al(OH)3] ; [Al(OH)4]–  Al(OH)3 ↓ +OH–
OH– ions are consumed in the above reaction and the formation of Al(OH)3 is favoured.
Note: The Bayer’s process cannot be adopted for white bauxite because the major impurity SiO2 is also separated
out along with Al2O3 and finally Al2O3, of inferior quality is formed.

Hall’s process: This process is adopted for beneficiation of low grade red bauxite. Various stages of the process are
depicted in the flow diagram in Figure.
Chem i str y | 29.27

Residue:
SiO2 + Fe2O3. etc
Powdered Fused at 1100C
Bauxite ore Together with
Na2CO3 -Little CaCO3
-
And finally extracted Filtrate: [Al(OH)4]
with water and filtered
CO2 passed
and filtered

Al2O3 Ignited Residue: Filtrate:


(pure alumina) Al(OH)3 Na2CO3 solution
at 1100C
Flowchart 29.10: Hall's process for beneficiation of low grade red bauxite

The reactions involved in the above process are:


In step 1: Al2O3 + Na2CO3 → 2NaAlO3 + CO2 ↑ ; SiO2 + Na2CO3 → Na2SiO3 + CO2 ↑
Fe2O3 + Na2CO3 → 2NaFeO2 + CO2 ↑ ; CaO + SiO2 → CaSiO3

In step 2: 2NaAlO2 + CO2 + 3H2O → 2Al (OH)3 ↓ + Na2CO3



In step 3: 2Al (OH)3 
–3H O
→ Al2O3 + 3H2O
2

Serpeck’s process: This process is used for beneficiation of white bauxite. Various steps of this process are depicted
in the flow diagram in figure.

Heated to 1800C in CO + Si


Finely divided presence of N2 gas
Bauxite + Coke
AlN(s) residue + impurity
(Fe2O3 + TiO2)

Hot and dilute NaOH


solution is added

Filtrate:
Filter
NaOH/Na2CO3
solution
Huge water
Filtrate: Residue
or CO2 passed Na[Al(OH)4] Solid impurity
And Filtered
(Fe2O3 + TiO2)
Al2O3 Ignited Residue
(Pure At 1100C Al(OH)3
alumina)

Flowchart 29.11: Serpeck's process for beneficiation of white bauxite

The reactions involved in the above process are:


2 9 . 2 8 | Extractive Metallurgy

In step 1: Al2O3 + 3C + N2 → 2AlN(s) + 3CO; SiO2 + 2C → Si ↑ + 2CO↑

In step 2: AlN + NaOH + 3H2O → Na [Al(OH)4] + NH3 ↑


HO

2
In step 3: Na [Al(OH)4] ← → Al(OH) ¯ + NaOH
 3


or [Al(OH)4]– ←→ Al(OH) ↓ + OH– ; CO + 2OH– → CO 2– + H O
3 2 3 2

Electrolytic reduction of pure Al2O3: In the electrolytic reduction of Al2O3 i.e. molten alumina (20 %) mixed with
cryolite (60%) and fluorspar (20 %) is taken in an iron tank with carbon lining that acts as the cathode. A graphite
rod hanging from the top acts as the anode. A powdered coke layer is maintained at the top figure.
Powered
Coke layer
Graphite rods
act as anode

Iron tank
Lining of carbon
act as cathode

Molten Al

Eletrolyte: Molten alumina (20%)


Tap
+ cryolite (60%) and
hole Fluorspar (20%)
Figure 29.13: Electrolytic reduction of alumina.

On electrolysis, molten Al is deposited at the cathode and since Al is heavier compared to the electrolyte, it gets
deposited at the bottom, while oxygen gas is liberated at the anode.
(a) Reactions Al2O3 → Al3++3O2–
At the cathode: Al3–+3e– → Al
Since Na and Ca are more electropositive than Al, only Al3+ gets deposited at the cathode.
At the anode: 2O2– → O2 +4e–

(b) Functions of fluorspar (CaF2): (i) It reduces the melting temperature of the mixture to 900°C from 2050°C
(Melting point of pure Al2O3) and saves on the fuel cost.
(ii) It also improves the electrical conductivity of the melt, compared to that of molten Al2O3

(c) Functions of cryolite: (i) It acts as a solvent and helps dissolve Al2O3.
(ii) It also helps to reduce the melting temperature of the mixture.
Alternative theory for electrolytic reaction at the anode can explain the dissolution of Al2O3.
AlF3 3NaF → Al3+ + 3Na– + 6F–, Al2O3 → 2Al3+ + 3O2- and CaF2 → Ca2+ + 2F-
Since the [F–] >>> [O2–], F– ions get discharged at the anode first which in turn react with Al2O3 and liberate O2
3
at the anode. 2F– → F2 + 2e- ; 3F2 + Al2O3 → 2AlF3 + O
2 2
(d) Function of the coke powder layer at the top: (i) The oxygen liberated at the anode corrodes the anode
surface, reacting with graphite to produce CO and CO2 and finally the anode cuts down at the bottom and
electrical connectivity is lost.
Chem i str y | 29.29

At the junction of liquid-solid-air interface, the energy available is maximum (this can be proved
thermodynamically) and corrosion is maximum at this point. To prevent this corrosion, the coke powder
(having a large surface area for reaction) layer is kept at the top, which reacts with liberated oxygen.

b c

Figure 29.14: Corrosion of graphite anode

(ii) The surface becomes rough unlike the shiny mirror like molten electrolyte; and the radiation loss of heat
is also prevented.

Electro Refining of Aluminium: Impure aluminium mixed with copper melt is taken in an iron tank with graphite
lining. The layer of pure Al acts as the cathode. The graphite rods at the top are essential for electrical connection.

Graphite rod

Layer of pure Al
Impure Al (act a cathode)

Electrolyte

Graphite

Al (impure)
+ Cu melt

Figure 29.15: Electrorefining of aluminium

Here, the electrolyte is the molten mixture of cryolite and BaF2 saturated with Al2O3. BaF2 is added instead of CaF2
to adjust the density in such a way that it exists as a separate middle layer. Similarly in impure Al, the Cu melt is
deliberately added to increase the density in such a way that it exists as a separate bottom layer.
The reactions involved in the process are: At the anode: Al → Al3++3e-
Here, the top surface of bottom layer acts as the anode and Al only enters into the electrolyte as Al3+ because
EοAl3 /Al3 + > ECu/Cu
ο
2+
2 9 . 3 0 | Extractive Metallurgy

At the cathode: Al3+ + 3e- + → Al


Here, the bottom surface of the top layer (the pure Al melt) acts as the cathode and Al3+ enters as Al from the
electrolyte.
Note:
(a) Impurities like Fe, Si and Cu remain intact in the bottom layer.
(b) When the thickness of the top layer is increased to a certain limit, it is drained out into a separate container.
(i) Pure Al2O3 is not used as the electrolyte because:
•• The melting point of Al2O3 is 2050°C, hence the power consumption is very high.
•• Al obtained at this temperature gets volatilized and the loss is very high.
••  luminium is lighter than Al2O3 and floats at the top surface and volatilizes easily, as well as attacked
A
by the oxygen, liberated at the anode.
(ii) The aqueous solution of Al salt cannot be used as an electrolyte because hydrogen is discharged at the
cathode in preference to Al, as the discharge potential of H+ is smaller compared to Al3+. For example, if
Al (NO3)3 is used, then the reactions involved are

AI (NO3)3 → Al3++ 3NO3− ; H2O → H+ + OH–

-
+2e
At the cathode: 2H+ → H2

At the anode: 4OH– → 2H2O + O2 ↑ + 4e–


(b) Molten AlCl3 is not chosen as electrolyte because, being covalent in nature, it is a poor conductor of electricity.
It also sublimes easily.

PLANCESS CONCEPTS

(a) Main steps involved in the extraction of Al from Bauxite are:


(i) Beneficiation of bauxite and preparation of pure alumina.
(ii) Electrolytic reduction of pure alumina.
(iii) Electro refining of aluminium.

(b) Beneficiation of bauxite includes 3 different processes namely Bayer’s process, Hall’s process and Serpeck’s
process, depending on the nature of ore.
Saurabh Chaterjee (JEE Advanced 2013, AIR)

5.6 Extraction of Lead


The important ores of lead are:
1. Galena : PbS
2. Cerrusite : PbCO3
3. Anglesite : PbSO4
4. Crocoisite : PbCrO4
5. Lanarkite : PbO.PbSO4
Chem i str y | 29.31

Finely PbO
Froth Floatation Concentrated Galena Roasting
powdered + PbSO4 and
(50-60% Pb content)
Galena SiO2

Coke Smelting in
Added the blast
furnace
+
C2O + Fe2O3

flux
Slag

Molten
lead

Separated
through two
Molten separate tap Slag
lead holes

Flowchart 29.12: Carbon reduction process for extraction of lead

The ore used commercially for extraction of lead is galena. Depending upon the impurity content, lead can be
extracted from galena by one of the following two processes:
(a) Carbon reduction (when the impurity content is high enough)
(b) Self-reduction (when the impurity content is low)
The reactions involved at various stages of the process are:

1. In the froth floatation step: PbS does not react with NaCN solution while ZnS gets dissolved in NaCN solution
and its floating characteristics are completely lost.

ZnS + NaCN → Na2 [Zn(CN)4 ] + Na2S ; PbS + NaCN → No reaction



Both soluble

3
2. In the roasting step: PbS + O → PbO + SO2 ↑
2 2
PbS + 2O2 → PBSO4 (minor) ; PbO + SiO2 → PbSiO3 (minor)
Function of lime: Since lime is more basic compared to PbO, CaO reacts preferably with SiO2 and its presence acts
as a negative catalyst towards the formation of PbSiO3 and PbSO4, It also helps to keep the mass porous and helps
to complete reaction.

3. In the smelting step: Initially coke is burnt in a blast of air to produce CO and CO2
C + O2 → CO2; C + CO2 → 2CO
Pb formation reaction PbO + C → Pb(l) + CO ↑
PbO + CO → Pb(l) + CO2 ↑ ; PbSO4 + 4C → PbS + 4CO
Pbs (unreacted) + 2PbO → 3Pb (l) + SO2 ↑ ; PbS + PbSO4 (if any) → 2Pb(l) + 2SO2 ↑

Slag formation reaction: CaO + SiO2 → CaSiO3


Fe2O3 + CO → 2FeO + CO2; FeO + SiO2 → FeSiO3
2 9 . 3 2 | Extractive Metallurgy

Self-reduction process: The steps involved in the self-reduction process are shown in the flow diagram. The
reactions involved in the process are:

Finely divided
concentrate ore Partial roasting Crude Pb
after froth Followed by molten
floatation Self-reduction

Flowchart 29.13: Self-reduction process for extraction of lead

3
 1. In the roasting step: PbS + O → PbO + SO2; PbS + 2O2 → PbSO4
2 2
 2. In the self-reduction step: PbS + 2PbO → 3Pb + SO2 ↑ ; PbS + PbSO4 → 2Pb + 2SO2 ↑

Note: After partial roasting, air blasting is stopped and heating is continued for self-reduction reactions.

Refining of Lead: The steps for refining of lead are given in the in figure.

Softening of lead Desilverization Desilverized


Crude Pb (molten) Soft lead
Removes scum of By Parke’s process lead
different oxides. or Pattinson’s process
Cu, Ag, Au, Sn,
Only Ag + Au left Removes Ag
As, Sb, Bi Bett’s
are the impurities Electrolytic
Refining
process

* Electrolyte: PbSiF6 + H2SiF6 + gelatine


Gelatine is added to electrolyte to adjust the Pure Pb
viscosity of electrolyte
Anode: Impure Pb.
Cathode: pure Pb strip
Flowchart 29.14: Refining of lead

PLANCESS CONCEPTS
(a) Depending on the impurity content, one of 2 following processes is used:
(i) Carbon reduction. (ii) Self-reduction.
(b) Carbon reduction involves froth floatation, roasting and smelting.
(c) Self reduction involves roasting followed by self-reduction steps.
Mredul Sharda (JEE Advanced 2013, AIR)

5.7 Extraction of Copper


The important ores of Cu are:
1. Chalcocite or copper glance : Cu2S
2. Copper pyrite or chalcopyrite : CuFeS2
3. Cuprite or ruby copper : Cu2O
4. Malachite : Cu(OH)2.CuCO3
5. Azurite : Cu(OH)2.2CuCO3
Chem i str y | 29.33

Less important ores of copper are chrysocolla (CuSiO3.2H2O) and malonite (CuO).
The chief ore used for extraction of copper is copper pyrite (Cu2S.FeS.FeS2). The flow diagram for extraction of
copper from copper pyrite is depicted in figure.

Copper Froth floatation Roasted ore


Pyrite ore process Concentrated Roasting Cu2S (unaffected)
Finely ore + SiO2 In reverberatory furnace FeO.SiO2
divided Cu2O

Smelting
Roasted ore
+coke + silica

Matte:
Blister Self Slag Cu2S + FeS Slag
copper reduction formation (small amount) [FeSiO ]
3

Flowchart 29.15: For extraction of copper from copper pyrite

The Changes and reactions involved in the above process are:


(a) In the roasting step: Roasting is done with excess of air and at temperatures below the fusion temperature
of the ore, Cu2S.Fe2S + O2 → Cu2S + 2FeS + SO2 ↑ ; Cu2S.Fe2S3 + 4O2 → Cu2S + 2FeO + 3SO2 ↑
Since iron is more electropositive as compared to copper, its sulphide is preferentially oxidized and Cu2S
remains unaffected. If any Cu2O (little amount) is formed, it also reacts with unreacted FeS to give back
3
Cu2S. Cu2S + O2 → Cu2O + SO2 ↑ ; Cu2O + FeS → Cu2S + FeO
2
(b) In the smelting step: Coke is used here as fuel to maintain such temperature that keeps the mixture in the
3
molten state. FeS + O2 → FeO + SO2 ↑
2
Cu2O + FeS → Cu2O + FeO ; FeO + SiO2 → FeSiO3 (slag)
Slag being lighter, floats as an immiscible layer on the top surface of Cu2S (matte) and is removed through a
separate hole.

(c) In the Bessemer converter: The raw material for the Bessemer converter is matte, i.e. Cu2S + FeS (little). Here
air blasting is initially done for slag formation and SiO2 is added from an external source.
3
FeS + O2 → FeO + SO2 ↑ ; SiO2 + FeO → FeSiO3 (slag)
2
During slag formation, the characteristic green flame is observed at the mouth of the Bessemer converter
which indicates the presence of iron in the form of FeO. Disappearance of this green flame indicates that the
slag formation is complete. Then air blasting is stopped and slag is removed.
Again air blasting is restarted from partial roasting before self-reduction, until two-thirds of Cu2S is converted
into Cu2O. After this, only heating is continued for the self-reduction process.
3
Cu2S + O → Cu2O + SO2 ↑ ; Cu2S + 2Cu2O → 6Cu (l) + SO2 ↑
2 2
And Cu2S + 2O2 → Cu2SO4; Cu2S + Cu2SO4 → 4Cu + 2SO2 ↑

Thus the molten Cu obtained is poured into a large container and allowed to cool and during cooling, the
dissolved SO2 comes up to the surface and forms blisters. It is known as blister copper.
2 9 . 3 4 | Extractive Metallurgy

Refining of blister copper: Blister Cu contains 2-3% impurity (mainly Fe, S and As). The steps involved in its
refining are depicted in the flow diagram in figure.

Purer
Blister Cu Poling Purer Copper Electrorefining
Copper
Fe, S, As Removes SO2 (gas). (99.5%pure) Electrolyte: 15%CuSO4+
As2O5 (volatile) 5%H2SO4 (99.99%)
and FeSiO3 (slag) Cathode:Pure Cu strip
Anode: Thick sheet
of Cu(impure)

Flowchart 29.16: Refining of blister copper

In the poling step: The little Cu2O formed is reduced to metallic Cu by the reducing gases produced from charring
of green wooden pole. The powdered anthracite (coke) spread on the top surface of the molten mass also helps to
produce a reducing environment.
In the electrorefining step, the impurities like Fe, Ni, Zn get dissolved in the solution, while Au, Ag and Pt are
deposited as anode mud below the anode.

PLANCESS CONCEPTS

•• Extraction of copper includes froth floatation, followed by roasting, then smelting and finally by
bessemerization.
•• Refining of copper includes poling’s process followed by electrorefining.

Vaibhav Krishnan (JEE 2009, AIR 22)

5.8 Extraction of Zinc


The various ores of zinc are:
1. Zinc blende : ZnS
2. Zincite : ZnO
3. Franklinite : ZnO Fe2O3
4. Calamine : ZnCO3
5. Willemite : ZnSiO3
6. Electric calamine : ZnSiO3.ZnO.H2O
The chief ore used for extraction of zinc is zinc blende, which is also known as Black Jack due to the invariable
association of galena (PbS) that is black in colour. Sometimes, calamine is also used to extract Zn by carbon
reduction process.
The flow diagram for extraction and refining of zinc from zinc blende is depicted in figure. The changes and
reactions involved at various stages of zinc extraction are:
(a) In the froth floatation step: This is done in two steps to separate out PbS and ZnS depending upon their
different floating characteristics. On addition of pine oil, PbS floats first and is removed. Then more pine oil is
added and ZnS floats on the top.
Chem i str y | 29.35

Finely divided Froth floatation Concentrated


Zinc blende Zinc blende

Roasting
(i)Controlled current of air
(ii)Temp = 850C to 900C

Roasted ore
ZnO + other
Impurities

Thermal Thermal Carbon reduction


refining Spelter Zn
Pure Zn refining Distillate + Impurity
(99.9%) At 767C Zn & Cd At 920C Temp = 1300C
(Pb, Cd, Fe)
Removes Cd (distilled) to 1400C
(vap) byproduct
Electrorefining
Electrolyte ZnSO4 + H2SO4
Cathode : Al sheet acts as cathode
Anode : Impure Zn

Pure Zn

Flowchart 29.17: For extraction and refining of zinc from zinc blende

(b) In the roasting step: During roasting, the temperature has to be above 850°C and the air current must be
controlled. ZnS is converted into ZnSO4, which converts back to ZnS during carbon reduction of roasted ore.
3 above
ZnS + O2 
850ºC
→ ZnO + SO2 ↑
2
below during
ZnS + 2O2 
850ºC
→ ZnSO4; ZnSO4 + 4C 
carbon reduction
→ ZnS + 4CO ↑

(c) In the smelting step: During smelting, excess coke is used, to stop the production of CO2 .Otherwise, Zn
reduced will be converted back to ZnO. Hence if any CO2 is produced, it is allowed to convert into CO by the
reaction with coke.
ZnO + C → Zn + CO

2ZnO + C ←→ 2Zn + CO (this reaction is reversible) ; CO + C → 2CO
2 2

The temperature during smelting is kept above 1300°C, though the b.p. of Zn is 920°G. The temperature is
kept much higher, compared to that required for vapourizing of zinc in the furnace. This is done because the
reaction of carbon dioxide with coke is highly endothermic and brings down the temperature to below 920°C
and the evaporation of Zn is affected. Hence, the temperature is maintained at 1300°C-1400°C.

(d) In the electrorefining step: For electrorefining of Zn (crude), Al sheet is used as cathode instead of pure Zn
strip. This is because the electrolyte used is ZnSO4 + H2SO4 (dil.), and in dil. H2SO4. Zn gets dissolved, while Al
does not. Zn + H2SO4 (dil) → ZnSO4 + H2 ↑
Al + H2SO4 (dil) → No reaction; 2Al + 6H2SO4 (cocn.) → Al2(SO4)3 + 3SO2 + H2O
Reactions at the electrode: ZnSO4 → Zn2+ + SO42–
At the cathode: Zn2+ + 2e– → Zn
At the anode: OH- → OH + e– ; 4OH → 2H2O + O2 ↑
2 9 . 3 6 | Extractive Metallurgy

Note: The H2SO4 is added in the electrolyte together with ZnSO4 to increase the over voltage of H+. This helps in
the deposition of only Zn2+ at the cathode. Otherwise, H2 will be evolved at the cathode.

PLANCESS CONCEPTS

Extraction and refining of zinc includes froth floatation, followed by roasting, then smelting and finally
refined by either thermal refining or electrorefining.
Nikhil Khandelwal (JEE 2009, AIR 94)

5.9 Extraction of Iron


The various ores of iron are:
1. Hematite : Fe2O3
2. Magnetite : Fe3O4
3. Brown hematite or limonite : Fe2O3.3H2O
4. Siderite or spathic iron are : FeCO3
5. Iron pyrite : FeS2

The chief ore used for extraction of iron is hematite, while FeS2 is never used because iron obtained from this ore
contains a lot of Sulphur, which makes it brittle and useless. The flow diagram for extraction of iron from different
ores is depicted in figure.

Magnetite Fe3O4 Concentrated Roasted


Hematite Fe2O3 Gravity separation ore Roasting process ore
Limonite Fe2O3.3H2O + Removes CO2,SO2, Fe2O3 +
Siderite FeCO3 SiO2 As2O4 and H2O SiO2

Smelting
Roasted ore: 8 parts
Coke powder: 4 parts
Lime stone: 1 parts

Molten metal Slag


(Cast iron or pig iron) (CaSiO3)

Flowchart 29.18: For extraction of iron from hematite

The reaction at various steps of the iron extraction are:


(a)
In the roasting step:
Fe3O4 → FeO + Fe2O3
FeCO3 → FeO + CO2 ↑
1
2FeO + O2 → Fe2O3
2
Fe2O3.3H2O → Fe2O3 + 3H2O ↑
Chem i str y | 29.37

Charge
Hence the final product of roasting is Fe2O3.Though there is no
sulphideore yet, roasting is adopted here to convert all FeO Cup and cone
present, into Fe2O3. As Fe2O3 does not form slag, this prevents the arrangement
loss of FeO as slag (FeSiO3). 400C
660C
(b) In the smelting step: The various changes taking place during
650C
smelting in the blast furnace are shown in figure. 700C

The reactions involved are: 900-1000C


Bosch
At 600-900ºC: Fe2O3 + 3CO → 2Fe + 3CO2 ↑ 1200-1500C
 Hot air
(Partially reduced) Hot air
Tuyeres (nozzeles)
At 900-1000ºC; CaCO3 → CaO + CO2 ↑ ; CO2 + C → 2CO ↑ 1500
Hearth
At 1000-1300ºC ; Fe2P3 + 3C → 2Fe + 3CO ↑ ; Molten cast Slag
iron
CaO + SiO2 → CaSiO3 (stag)
At 1500ºC (i.e. at the hearth): The coke powder crossing the line Figure 29.16: Smelting in the blast
of tuyeres does not have the scope of burning anymore and reacts furnace
with MnO2, SiO2 to produce impurities like Mn and Si as follows:
MnO + 2C → Mn + 2CO ; SiO2 + 2C → Si + 2CO
Ca3(PO4)2 present in the limestone reacts with SiO2 to produce slag and P2O5 is reduced by coke to produce P4
as impurity: Ca3(PO4)2 + 3SiO2 → 3CaSiO3 + 2P2O3 ; 2P4O5 + 10C → P4 + 10CO ↑
Finally the cast iron produced consists of impurities like Mn, Si, P, C and S.

Purification of iron or preparation of wrought iron


Wrought iron is the purest form of iron which contains the total Cast iron
impurity less than 0.5%. The carbon content is 0.1 - 0.15% and Mn, P, Si, C, S
other impurities (Mn, P, S, Si) are less than 0.3%. The steps involved
Puddling process Wrought
Taken in puddling
in preparation of wrought iron are shown in the flow diagram. furnace which is Hot air is blasted iron
lined with haematite and stirred well
The hematite lining has a special significance as it removes removes CO2, SO2
the impurity as well as produces iron at that place. The various and slag
reactions taking place in the process are:
Flowchart 29.19: For preparation of wrought
S + O2 → SO2 ↑ C + O2 → CO2 ↑ iron from cast iron
3S + 2Fe2O3 → 4Fe + 3SO2 ↑
3Si + 2Fe2O3 → 4Fe + 3SiO2
3Mn + Fe2O3 → 2Fe + 3MnO; MnO + SiO3 → MnSiO3 (slag)
3C + 2Fe2O3 → 2Fe + 3CO ↑ ; 4P + 5O2 → P2O5 ; Fe2O3 + P2O5 → 2FePO4 (slag)

Byproducts of iron extraction

(a) Slag: It consists of huge amounts of CaSiO3 and little Al2(SiO3)3 .These days it is used for making cement and
is now known as slag cement.
(b) Blast furnace gas: The composition of the blast furnace gas is 58% N2, 25% CO, 10.5% CO2, 6.5% H2 and the
rest are hydrocarbons. It contains a very large quantity of CO and the H2, which constitutes a good fuel. It is
used for preheating the air used and for cooking purposes also.
Steel Making: Steel is made by removing most of the carbon and other impurities from pig iron. Composition of
various steels depending upon percentage of carbon is given as follows:
%C Type of steel
0.15-0.3 Mild steel
0.3-0.6 Medium steel
2 9 . 3 8 | Extractive Metallurgy

0.6-0.8 High carbon steel


0.8-1.4 Tool steel
The steel making process involves melting and oxidizing C, Si, Mn, S and P present in the pig iron so that these
impurities are removed as gases or converted into slag. This is followed by addition of required additives (i.e.
different elements) to iron to impart desired properties to steel.
Different elements present in steel provide different properties as described below:
Element Properties imparted
P above 0.05% Imparts low tensile strength and cold brittleness.
Mn Imparts high hardness and increases tensile strength, e.g. rail road contains 13% Mn.
Cr and Ni Imparts stainless characteristics by producing impervious coating of their oxides on the surface.
N (above 0.01%) Makes steel brittle as well as difficult to weld.
C improves hardness and strength.
The addition of mixture of C and Mn into produced steel is known as spiegeleisen. C and Mn act as deoxidizers,
remove any dissolved O2 and reduce FeO (if any). The excess carbon (if any) supplies the desired quality. Mn makes
the steel hard and improves its tensile strength also. Various processes used for the preparation of good quality
steel are: FeO + Mn → MnO + Fe; FeO + C → Fe + CO ↑
(a) Puddling process: This process involves stirring of molten iron in a reverberatory furnace by rods, which are
consumed in the process
(b) Bessemer and Thomas process
(c) Siemens open hearth process
(d) Basic oxygen process (also called Linz Donawitz (L.D) process)
All these processes are developed to economize the production of steel from iron. In the Bessemer and
Thomas processes, or the Siemens open hearth process, impurities are oxidized by the air, while in the L.D.
process, pure O2 is used for the oxidation of impurities. This is because in the first two processes the molten
metal takes up small amounts of nitrogen from the air. In concentrations above 0.01% nitrogen makes steel
brittle and nitrating the surface makes the metal more difficult to weld. The use of O2 not only helps in
overcoming these problems but also has the following advantages:
•• There is faster conversion, so a given plant can produce more in a day i.e. larger quantities can be handled
in lesser time. For example, a 300 tonnes charge can be converted in 40 minutes compared to 6 tonnes in 20
minutes by the Bessemer process.
•• It gives a purer product and the surface is free from nitrides.
The lining of the furnace is designed, based on the impurities present in the cast iron:
•• If the cast iron contains Mn, but not P, S, Si, then the lining used is silica brick and the process is known as the
acid Bessemer process.
•• If the cast iron contains acidic impurities such as Si, S, P, a lining of calcined dolomite (CaO.MgO) or magnesia
(MgO) is used and the process is called basic Bessemer process. In this process, the P2O3 formed from P
combines with lime and forms basic slag [Ca3 (PO4)2.CaO], which is known as Thomas slag. It is a valuable
byproduct and is sold as phosphate fertilizer.

Some heat treatment processes related to steel


(a) Annealing: The hard steel is heated to bright redness (700-800°C) and then allowed to cool to the room
temperature very slowly by which the hard steel becomes soft. This process is known as annealing.
(b) Quenching or hardening: The soft steel is heated to bright redness and then cooled suddenly to room
temperature immersing it into water or oil, by which the steel becomes hard and brittle. This process is known
as quenching.
Chem i str y | 29.39

(c) Tempering: The hard and brittle steel is healed to a 200-300°C temperature range and cooled very slowly
to room temperature by which the brittleness of the steel disappears while the hardness remains same. This
process is called as tempering.
(d) Case-hardening: For this process the mild steel or wrought iron is heated to bright redness in the presence
of hydrocarbons or K4[Fe(CN)6] or heated to bright redness followed by dipping into NaCN/KCN solution. This
makes the surface layer hard due to the formation of carbide of Fe (called cementite). This kind of steel is used
for making armor plates, cutting tools and machinery parts which are in constant wear and tear.
(e) Nitriding: The mild steel containing 1% Al is heated to 550°C in the presence of NH3. This makes the surface
layer hard, due to the formation of the nitrides of Fe and Al. This kind of steel is used for making bore well
drilling equipment.

PLANCESS CONCEPTS

•• Extraction of iron includes gravity separation followed by roasting and then smelting to give pig or
cast iron.
•• Puddling’s process is used for purification of iron and converts pig iron to wrought iron.
Saurabh Gupta (JEE 2010, AIR 443)

Illustration 9: Auto reduction process issued in extraction of:


(A) Cu (B) Hg (C) Al (D) Fe (JEE ADVANCED)

Sol: (A) and (B) Cu2S + 2Cu2O → 6Cu+SO2 ; HgS + 2HgO → 3Hg + SO2

Illustration 10: Why is sulphide ore roasted to convert it into the oxide before reduction?  (JEE MAIN)

Sol: ∆tG° of most sulphide ores are greater than those of CS2 and H2S. Hence neither carbon nor hydrogen is
a suitable reducing agent for the metal sulphides. Moreover the roasting of a sulphide to the oxide is quite
advantageous thermodynamically because ∆tG0 of oxides are much lower than those of SO2.

POINTS TO REMEMBER
Metallurgy A collection of processes used for the extraction of metals from their ores which
includes its purification and alloy formation
Ore A mineral from which one or more metals can be extracted easily and profitably.

Flux A substance used to reduce the m.p. of ore or react with gangue to convert it to slag.

Gangue or matrix Earthly impurities present with minerals

Pyrometallurgy Method of thermal reduction (using reducing agent and heat) of ore to metal.

Hydrometallurgy Method of extraction of metals using leaching and displacement employing cheaper
and reactive metal.
Leaching Method of reacting an ore with some reagent to collect the required metal as water
soluble salt.
Smelting Heating purified oxide form of ore with coke. It may give metal or matte. It is generally
known as carbon reduction method.
Aluminothermy Method of reducing oxide of a metal by heating with powdered aluminium.
2 9 . 4 0 | Extractive Metallurgy

JEE Main/Boards

Exercise 1 Q.20 Outline the principles of refining of metals by the


following methods:
Q.1 Copper can be extracted by hydrometallurgy but (i) Zone refining (ii) Electrolytic refining
not zinc. Explain.
(iii) Vapour phase refining
Q.2 Out of C and CO, which is a better reducing agent
Q.21 What is the role of depressant in froth floatation
at 673 K?
process?
Q.3 How is leaching carried out in case of low grade
Q.22 Why is the extraction of copper from pyrites more
copper ores?
difficult than that from its oxide ore through reduction?
Q.4 Why is zinc not extracted from zinc oxide, through
Q.23 Explain:
reduction using CO?
(i) Zone refining (ii) Column chromatography
Q.5 Name the common elements present in the anode
mud in electrolytic refining of copper. Why are they so Q.24 Write down the reactions taking place in different
present? zones in the blast furnace during the extraction of iron.

Q.6 State the role of silica in the metallurgy of copper. Q.25 Write chemical reactions taking place in the
extraction of zinc from zinc blende.
Q.7 What is meant by the term “chromatography”?
Q.26 What criterion is followed for the selection of the
Q.8 How is ‘cast iron’ different from ‘pig iron”? stationary phase in chromatography’?

Q.9 Differentiate between “minerals” and “ores”. Q.27 How can you separate alumina from silica in a
bauxite ore associated with silica? Give equations, if any?
Q.10 Out of C and CO. which is a better reducing agent
for ZnO? Q.28 The value of ∆ rG0 for formation of Cr2 O3 is -540 kJ
mol–1and that of Al2O3 is - 827 kJ mol–1. Is the reduction
Q.11 What is the role of graphite rod in the of Cr2O3 possible with Al?
electrometallurgy of aluminium?
Q.29 The choice of a reducing agent in a particular case
Q.12 Describe a method for refining nickel. depends on thermodynamic factor. How far do you
agree with this statement? Support your opinion with
Q.13 Giving examples, differentiate between ‘roasting’ two examples.
and ‘calcination”.
Q.30 Name the processes from which chlorine is
Q.14 Why copper matte is put in silica lined converter? obtained as a by-product. What will happen if an
aqueous solution of NaCl is subjected to electrolysis?
Q.15 What is the role of cryolite in the metallurgy of
aluminium? Q.31 Predict conditions under which Al might be
expected to reduce MgO.
Q.16 What types of ores are roasted?

Q.17 Name one each of (a) acidic flux (b) basic flux.

Q.18 What is gangue?

Q.19 What name is given to carbon reduction process


for extracting the metal?
Chem i str y | 29.41

Exercise 2 Q.11 Purification of silicon element used in semi-


conductors is done by
Single Correct Choice Type (A) Zone refining (B) Heating
Q.1 In presence of oxygen, removal of sulphur from an (C) Froth floatation (D) Heating in vacuum
ore is included in
(A) Calcination (B) Roasting Q.12 Aluminothermic process is used for metallurgy of

(C) Smelting (D) Fluxing (A) Pb (B) Ag


(C) Al (D) None of these
Q.2 In the extraction of copper from its sulphide ore, the
metal is formed by reduction of Cu2O with Q.13 In electrorefining of copper some gold is deposited
(A) FeS (B) CO (C) Cu2S (D) SO2 as
(A) Anode mud (B) Cathode mud
Q.3 Pyrolusite is a/an (C) Cathode (D) Electrolyte
(A) Oxide ore (B) Sulphide ore
(C) Carbide ore (D) Not an ore Q.14 Poling process is used
(A) For the removal of Cu2O from Cu
Q.4 The metal always found in the free state is (B) For the removal of Al2O3 from Al
(A) Au (B) Zn (C) Cu (D) Na (C) For the removal Fe2O3 from Fe
(D) In all the above
Q.5 In the extraction of iron, the slag produced is?
(A) CO (B) FeSiO3 (C) MgSiO3 (D) CaSiO3 Q.15 Which of the following is not a basic flux?
(A) CaCO3 (B) CaO (C) SiO2 (D) MgO
Q.6 Which alloy of aluminium is used in aircraft industry
(A) Duralumin (B) Magnalium Q.16 Zone refining is a method to obtain
(C) Nickeloy (D) alunico (A) Very high temperature
(B) Ultra pure Al
Q.7 The common impurities present in the bauxite ore
are (C) Ultra pure metal
(A) Fe2O3 and CuO (B) Fe2O3 and PbO (D) Ultra pure oxides
(C) Fe2O3and SiO2 (D) SiO2 and CuO
Q.17 Mac Arthur process is use for
Q.8 The material used in solar cell contains (A) Ag (B) Fe (C) Cl (D) O2
(A) Si (B) Sn (C) Ti (D) Cs
Q. 18 Use of electrolysis is
Q.9 Cyanide process is used for the extraction of: (A) Electroplating (B) Electrorefining
(A) Ag (B) Hg (C) Cu (D) Zn (C) Both (A) and (B) (D) None of these

Q.10 During smelting, an additional substance is added Q.19 Calcination is used in metallurgy for removal of
which combines with impurities to form a fusible (A) Water and sulphide (B) Water and CO2
product. It is known as
(C) CO2 and H2S (D) H2O and H2S
(A) Slag (B) Mud (C) Gangue (D) Flux
2 9 . 4 2 | Extractive Metallurgy

Q.20 In blast furnace, maximum temperature is in Previous Year’s Questions


(A) Zone of fusion
Q.1 Which of the following mineral does not contain Al
(B) Zone of combustion
 (1992)
(C) Zone of slag combustion (A) Cryolite (B) Mica
(D) Zone of reduction (C) Feldspar (D) Fluorspar

Q.21 Of the following which cannot be obtained by Q.2 Which of the following is not an ore  (1982)
electrolysis of the aqueous solution of their salts?
(A) Bauxite (B) Malachite
(A) Ag (B) Mg and Al (C) Cu (D) Cr
(C) Zinc blende (D) Pig iron

Q.22 Van Arkel method of purification of metals involves


converting the metal into a Q.3 Copper can be extracted from  (1978)

(A) Volatile stable compound (A) Kupfernickel (B) Dolomite

(B) Volatile unstable compound (C) Galena (D) Malachite

(C) Non-volatile stable compound


Q.4 Among the following statements, the incorrect one
(D) None of the above is (1997)
(A) Calamine and siderite are carbonates
Q.23 In the froth floatation process for the purification
(B) Argentite and cuprite are oxides
of ores, the ore particles float because
(C) Zinc blende and pyrites are sulphides
(A) They are light
(D) Malachite and azurite are ores of copper
(B) Their surface is hydrophobic i.e., not easily wetted
by water
Q.5 Which ore contains both iron and copper (2005)
(C) They bear electrostatic charge
(A) Cuprite (B) Chalcocite
(D) They are insoluble
(C) Chalcopyrite (D) Malachite
Q.24 Which method of purification is represented by
(A) If both assertion and reason are true and reason is
the following equations?
the correct explanation of assertion, then mark (A)
773K 1675K
Ti + 2I2  → TiI 4  → Ti + 2I2 (B) If both assertion and reason are true but reason is
not the correct explanation of assertion, then mark (B)
(A) Cupellation (B) Poling
(C) If assertion is true but reason is false, then mark (C)
(C) Van Arkel (D) Zone refining
(D) If both assertion and reason are false, then mark (D)

Q.25 Calcination is the process of heating the ore


Q.6 Assertion: Gold is recovered from its solution containing
(A) In a blast furnace (B) In absence of air aurocyanide complex by adding zinc dust. (1982)
(C) In presence of air (D) None of these Reason: Zinc is more electropositive than gold.

Q.26 Which one of the following metals cannot be Q.7 Assertion: Zinc is used and copper is not used in
extracted by carbon reduction process? the recovery of Ag from the complex [Ag(CN)2]–. (1995)
(A) Pb (B) Al (C) Hg (D) Zn Reason: Zinc is a powerful reducing agent than copper.

Q.27 Cyanide process is used for obtaining Q.8 Assertion: Coke and flux are used in smelting. (1998)

(A) Cr (B) Ag (C) Cu (D) Zn Reason: The phenomenon in which ore is mixed with
suitable flux and coke is heated to fusion is known as
smelting.
Chem i str y | 29.43

Q.9 Assertion: Leaching is a process of reduction. (1999) Q.12 Which method of purification is represented by
the following equation: Ti+2I2 → TiI4 (2012)
Reason: Leaching involves treatment of the ore with
a suitable reagent so as to make it soluble while (A) Zone refining (B) Cupellation
impurities remains insoluble.
(C) Poling (D) Van Arkel

Q.10 Assertion: Ethyl xanthate is used as a collector in


Q.13 In the context of the Hall-Heroult process for the
froth floatation process.  (1999)
extraction of Al, which of the following statements is
Reason: Collectors depress the floatation property of false?  (2015)
one of the components of the ore and thus help in the
(A) CO and CO2 are produced in this process
separation of different minerals present in the same ore.
(B) Al2O3 is mixed with CaF2 which lowers the melting
point of the mixture and brings conductivity
Q.11 Which of the following factors is of no significance
for roasting sulphide ores to the oxides and not (C) Al3+ is reduced at the cathode to form Al
subjecting the sulphide ores to carbon reduction
(D) Na3AlF6 serves as the electrolyte
directly?  (2008)

(A) Metal sulphides are thermodynamically more stable Q.14 Which one of the following ores is best
than CS2 concentrated by froth floatation method?  (2016)
(B) CO2 is thermodynamically more stable than CS2 (A) Siderite (B) Galena
(C) 
Metal sulphides are less stable than the (C) Malachite (D) Magnetite
corresponding oxides
(D) CO2 is more volatile than CS2 Q.15 Galvanization is applying a coating of  (2016)
(A) Cr (B) Cu (C) Zn (D) Rb

JEE Advanced/Boards

Exercise 1 Q.5 Write chemical equations for metallurgical processes


to represent:
Q.1 What is Goldschmidt Thermite Process? (a) Roasting of galena (PbS).
(b) Reduction of Cu2O using charcoal as a reducing
Q.2 Al2O3 can’t be reduced by carbon to get Al metal. agent.
Explain.
(c) deposition of pure silver from an aqueous solution
of Ag+.
Q.3 Cu can reduce Ag+ to metallic Ag but Ag can’t
reduce Cu2+ to metallic Cu. Explain.
Q.6 Why zinc and not copper is used for the

Q.4 A metal is in combined state as sulphide. Identify recovery of metallic silver from the complex [Ag(CN)2]-?
the steps (A), (B), (C)
Q.7 Sodium is prepared by the electrolysis of molten
Sulphide A Oxide NaCl but not by the electrolysis of its aqueous solution.

Oxide B Impure metal Q.8 Elements of alkali metal group are strong reducing
agent. Explain.

Pure metal C Q.9 In the metallurgy of iron, why limestone is added


to the ore.
2 9 . 4 4 | Extractive Metallurgy

Q.10 What is the actual reducing agent of haematite in Q.18 Name two metals which are used for reduction in
blast furnace? metallurgical process. Give one chemical equation for
each
Q.11 Write chemical equations to represent the most
probable outcome in each of the following. If no Q.19 Aluminium metal is frequently used as a reducing
reaction is likely to occur, state so: agent for the extraction of metals such as chromium or
∆ manganese from their respective oxides. Why?
(i) CdCO3 →

(ii) MgO →
Q.20 What is the function of basic furnace linings in

(iii) SnO2 + CO → steels manufacture?
electrolysis
(iv) CdSO4 (aq) →
∆ Q.21 Write down reactions involved in the extraction
(v) 2HgO →
of Pb. What is the oxidation number of lead in litharge?

(vi) MgO + Zn →
Q.22 A1 and A2 are two ores of metal M. A1 on calcination
Q.12 Why graphite is used as anode but not diamond? gives black precipitate, CO2 and water?
Black solid + CO2 + H2O
Calcination
Q.13 The following reactions take place during the
extraction of copper from copper ore:
A1
(i) 2Cu2S() + 3O2(g) → 2Cu2O() + +2SO2(g) dil. HCl
KI
(ii) 2Cu2O() + Cu2S() → 6Cu() + SO2(g) I2 + ppt
Identify the oxidizing and reducing agents.
Roasting
A2 Metal + gas
Q.14 Dolomite (MgCO3.CaCO3) can also be treated to
get MgCl2 which in turn is electrolyzed to get Mg. (Dow K2Cr2O7 + H2SO4
natural brine process). Give reactions of this process.
Green water

Q.15 [CaCO3.MgCO3] → CaO.MgO
Q.23 Gold is also extracted by cyanide process as in
Fe/Si
CaO. MgO 
1150ºC
→ Mg + Fe + Ca2SiO4 case of silver. Outline the reactions.
Name this process.
Q.24 How is Ag extracted from silver coin?
Q.16 Convert Mg into MgO.
Q.25 In the purification of bauxite ore as preliminary
Q.17 (i) A black mineral (A) on treatment with dilute step in the production of Al, [Al(OH)4]– can be converted
sodium cyanide solution in presence of air gives a clear to Al(OH)3 by passing CO2 through it. Write an equation
solution of (B) and (C). for the reaction that occurs.

(ii) The solution of (B) on reaction with zinc gives


precipitate of a metal (D). Q.26 When the ore hematite is burnt in air with coke
around 2000°C along with lime, the process not only
(iii) (D) is dissolved in dilute HNO3 and the resulting produces steel, but also produces a silicate slag that
solution gives a white precipitate (E) with dilute HCl. is useful in making building material such as cement.
(iv) (E) on fusion with sodium carbonate gives (D). Discuss the same and show through balanced chemical
equation.
(v) (E) dissolves in aqueous solution of ammonia giving
a colourless solution of (F). Identify compounds (A) to
(F) and give chemical equations for reactions in steps Q.27 Explain the following
(i) to (iv). (i) In the metallurgy of iron, lime-stone is added to the
ore.
(ii) Although Au is soluble in aqua-regia, Ag is not.
Chem i str y | 29.45

Exercise 2 Q.8 The function of flux during smelting of ores is to:


(A) Make the ore porous
Single Correct Choice Type
(B) Remove gangues
Q.1 Sulphide ores are generally concentrated by (C) Facilitate reduction
(A) Froth floatation (B) Roasting (D) Precipitate slag
(C) Gravity (D) Reduction by carbon
Q.9 The chemical composition of slag formed during
Q.2 Which method of purification is represented by the the smelting process in the extraction of copper is:
following equations? (A) Cu2O + FeS (B) FeSiO3
523K 1700K
Ti + 2l2  → TiI 4 
→ Ti + 2I2 (C) CuFeS2 (D) Cu2S + FeO
(A) Cupellation (B) Poling
(C) Van Arkel (D) Zone refining Q.10 The chemical process in the production of steel
from haemotite ore involves:
Q.3 Cryolite is (A) Reduction
(A) Na3AlF6 and is used in the electrolysis of alumina for (B) Oxidation
decreasing electrical conductivity (C) Reduction followed by oxidation
(B) Na3AlF6 and is used in the electrolysis of alumina for (D) Oxidation followed by reduction
lowering the melting point of alumina
(C) Na3AlF6 and is used in the electrolytic purification Q.11 In the commercial electrochemical process for
of alumina alumniuim extraction, the electrolyte used is
(D) Na3AlF6 and is used in the electrolysis of alumina (A) Al (OH)3 in NaOH solution
(B) An aqueous solution of Al2(SO4)3
Q.4 Cassiterite is an ore of
(C) A molten mixture of Al2O3 and Na3AlF6
(A) Mn (B) Ni (C) Sb (D) Sn
(D) A molten mixture of AlO(OH) and Al(OH)3
Q.5 When an aqueous solution of sodium chloride
is electrolysed using platinum electrodes, the ions Q.12 Calcination is used in metallurgy to remove
discharged at the electrodes are (A) H2O and H2S (B) H2O and CO2
(A) Sodium and hydrogen (C) CO2 and H2S (D) H2O and H2S.
(B) Sodium and chloride
(C) Hydrogen and chloride Q.13 The metallic luster exhibited by sodium is

(D) Hydroxyl and chloride explained by the presence of


(A) Na+ ions
Q.6 The luster of a metal is due to (B) Conducting electrons
(A) Its high density (C) Free protons
(B) Its high polishing (D) A body-centred cubiclattice.
(C) Its chemical inertness
(D) Presence of free electrons Q.14 Stainless steel contains
(A) Fe, Cr, Cu (B) Fe, Cr, Ni
Q.7 Which metal can’t be obtained from electrolysis? (C) Fe, Ni, C (D) Fe, Ni, Cu
(A) Ca (B) Mg
(C) Cr (D) All of the above
2 9 . 4 6 | Extractive Metallurgy

Q.15 Which one of the following beneficiation processes Comprehension Type


is used for the mineral, Al2O3.2H2O?
A black coloured compound (A) on reaction with dil
(A) Froth floatation (B) Leaching H2SO4 form a gas ‘B’ and a solution of compound (C).
When gas B is passed through solution of compound
(C) Liquation (D) Magnetic separation
(C), a black coloured compound A is obtained which is
soluble in 50% HNO3 and forms blue coloured complex
Q.16 The extraction of which of the following metals ‘D’ with excess of NH4OH and chocolate brown ppt. ‘E’
involves bessemerisation ? with K4[Fe(CN)6]
(A) Fe (B) Ag (C) Al (D) Cu
Q.23 ‘A’ is
Q.17 One of the characteristic properties of non-metals (A) CuS (B) FeS (C) PbS (D) HgS
is that they
(A) Are reducing agents Q.24 ‘B’ is
(B) Form basic oxides (A) H2S (B) SO2 (C) NH3 (D) SO3
(C) Form cations by electron gain
Q.25 ‘C’ is
(D) Are electronegative
(A) CuS (B) CuSO4 (C) Cu(NO3)2 (D) HgSO4
Q.18 In the froth floatation process for the purification
of ores, the ore particles float because Q.26 ‘D’ is
(A) They are light (A) Cu(OH)2 (B) [Cu(NH3)2]SO4
(B) Their surface is hydrophobic i.e., not easily wetted (C) [Cu(NH3)4] (NO3)2 (D) [Cu(NH3)6]SO4
by water
(C) They bear electrostatic charge Q.27 ‘E’ is
(D) They are insoluble (A) Cu2[Fe(CN)6] (B) Cu4[Fe(CN)6]
(C) Cu3[Fe(CN)6]2 (D) None of these
Q.19 Which is not a basic flux?
(A) CaCO3 (B) Lime (C) SiO2 (D) CaO
Assertion Reasoning Type
(A) If both assertion and reason are true and reason is
Q.20 The substance which is added to remove impurities
the correct explanation of assertion, then mark (a)
is known as
(B) If both assertion and reason are true but reason is
(A) Slag (B) Flux (C) Gangue (D) Catalyst
not the correct explanation of assertion, then mark (b)

Q.21 Electrolytic reduction method is used in the (C) If assertion is true but reason is false, then mark (c)
extraction of (D) If both assertion and reason are false, then mark (d)
(A) Highly electronegative elements
Q.28 Assertion: Calamine is a ore of boron.
(B) Highly electropositive elements
(C) Transition metals Reason: Boric acid is a tribasic acid.

(D) Noble metals


Q.29 Assertion: FeS2 is also known as fools’ gold.
Reason: FeS2 has yellow metallic appearance.
Q.22 The role of calcination in metallurgical operations is
(A) To remove moisture
Q.30 Assertion: Pb4+ can be reduced easily to Pb2+.
(B) To decompose carbonate
Reason: Pb2+ is paramagnetic.
(C) To drive off organic matter
(D) To achieve all the above
Chem i str y | 29.47

Q.31 Assertion: Diamond is harder than graphite. Paragraph


Reason: Graphite is more stable than diamond. Copper is the most noble of the first row transition
metals and occurs in small deposits in several countries.
Q.32 Assertion: Carbon dioxide and nickel react to Ores of copper include chalcanthite (CuSO4 .5H2O),
form tetracaronyl nickel (0). atacamite (Cu2Cl(OH)3, cuprite (Cu2O), copper glance
(Cu2S) and malachite (Cu2(OH)2CO3). However, 80%
Reason: Ni(CO)4 decomposes on heating to form Ni
of the world copper production comes from the ore
and CO.
chalcopyrite (CuFeS2). The extraction of copper from
chalcopyrite involves partial roasting, removal of iron
and self-reduction. (2010)
Previous Years Questions
Q.1 Which ore contains both iron and copper (2005) Q.7 Partial roasting of chalcopyrite produces

(A) Cuprite (B) Chalcocite (A) Cu2S and FeO (B) Cu2O and FeO
(C) Chalcopyrite (D) Malachite (C) CuS and Fe2O3 (D) Cu2O and Fe2O3

Q.2 Extraction of zinc from zinc blende is achieved by Q.8 Iron is removed from chalcopyrite as
 (2007)
(A) FeO (B) FeS (C) Fe2O3 (D) FeSiO3
(A) Electrolytic reduction
(B) Roasting followed by reduction with carbon
Q.9 In self-reduction, the reducing species is
(C) Roasting followed by reduction with another metal
(A) S (B) O2– (C) S2– (D) SO2
(D) Roasting followed by self-reduction

Q.3 Native silver metal forms a water soluble complex Q.10 Match the following Metals listed in column I with
with a dilute aqueous solution of NaCN in the presence extraction processes listed in column II. (1979)
of  (2008)
Column I Column II
(A) Nitrogen (B) Oxygen
(A) Silver (p) Fused salt Electrolysis
(C) Carbon dioxide (D) Argon
(B) Calcium (q) Carbon Reduction
Q.4 Oxidation states of the metal in the minerals (C) Zinc (r) Carbon monoxide reduction
haematite and magnetite, respectively, are  (2011)
(D) Iron (s) Amalgamation
(A) II, III in haematite and III in magnetite (E) Copper (t) Self reduction
(B) II, III in haematite and II in magnetite
(C) II in haematite and II, III in magnetite
Q.11 Write the chemical reactions involved in the
(D) III in haematite and II, III in magnetite extraction of silver from argentite. (2000)

Q.5 Addition of high proportions of manganese makes


Q.12 Write the balanced chemical equation for
steel useful in making rails  (1998)
developing photographic films.
(A) Gives hardness to steel
(B) Helps the formation of oxides of iron Q.13 Write balanced chemical equation for developing
(C) Can remove oxygen and sulphur a black and white photographic film. Also give reason,
(D) Can show highest oxidation state of +7 why the solution of sodium thiosulphate on acidification
turns milky white and give balance equation of this
reaction. (2005)
Q.6 Extraction of metal from the ore cassiterite
involves  (2011)
Q.14 Given the number of water molecule (s) directly
(A) Carbon reduction of an oxide ore
bonded to the metal center in CuSO4. 5H2O.  (2009)
(B) Self-reduction of a sulphide ore
(C) Removal of copper impurity Q.15 Given the coordination number of Al in the
(D) Removal or iron impurity crystalline state of AlCl3.  (2009)
2 9 . 4 8 | Extractive Metallurgy

Q.16 Extraction of zinc from zinc blende is achieved by, Q.19 The carbon-based reduction method in NOT used
 (2007) for the extraction of  (2013)
(A) Electrolytic reduction
(A) Tin from SnO2
(B) Roasting followed reduction by reduction with (B) Iron from Fe2O3
carbon
(C) Aluminium from Al2O3
(C) Roasting followed by reduction with another metal
(D) Magnesium from MgCO3 .CaCO3
(D) Roasting following by self-reduction
Q.20 Match the anionic species given in column I that
Q.17 Match the conversions in column I with the type are present in the ore (s) given in column II  (2015)
(s) of reaction (S) give in column II. (2008)
Column I Column II
Column I Column II
(A) Carbonate (p) Siderite
(A) PbS → PbO (p) Roasting
(B) Sulphide (q) Malachite
(B) CaCO3 → CaO (q) Calcination
(C) Hydroxide (r) Bauxite
(C) ZnS → Zn (r) Carbon reduction (D) Oxide (s) Calamine
(D) Cu2S → Cu (s) Self reduction (s) Argentite

Q.18 Sulfide ores are common for the metals  (2013) Q.21 Extraction of copper from copper pyrite ( CuFeS2 )
(A) Ag, Cu and Pb (B) Ag, Cu and Su involves  (2016)
(C) Ag, Mg and Pb (D) Al, Cu and Pb (A) Crushing following by concentration of the ore by
froth-flotation
(B) Removal of iron as slag
(C) Self-reduction step to produce ‘blister copper’
following evolution of SO2
(D) Refining of ‘blister copper’ by carbon reduction

PlancEssential Questions
JEE Main/Boards JEE Advanced/Boards

Exersice 1 Exersice 1

Q.2 Q.6 Q.15 Q.4 Q.12 Q.17

Q.21 Q.27 Q.30 Q.20 Q.27

Exersice 2 Exersice 2

Q.8 Q.24 Q.2 Q.13 Q.32

Previous Years’ Questions Previous Years’ Questions

Q.9 Q.3 Q.10 Q.15


Chem i str y | 29.49

Answer Key

JEE Mains/Boards
Exercise 2
Single Correct Choice Type

Q.1 B Q.2 C Q.3 A Q.4 A Q.5 D Q.6 A Q.7 C


Q.8 A Q.9 A Q.10 D Q.11 A Q.12 D Q.13 A Q.14 A
Q.15 C Q.16 C Q.17 A Q.18 C Q.19 B Q.20 B Q.21 B
Q.22 A Q.23 A Q.24 C Q.25 B Q.26 B Q.27 A

Previous Years’ Questions


Q.1 D Q.2 D Q.3 D Q.4 B Q.5 C Q.6 A Q.7 B
Q.8 B Q.9 D Q.10 C Q.11 A Q.12 D Q.13 D Q.14 B
Q.15 C

JEE Advanced/Boards
Exercise 2
Single Correct Choice Type

Q.1 A Q.2 C Q.3 B Q.4 D Q.5 C Q.6 D Q.7 C


Q.8 D Q.9 B Q.10 D Q.11 C Q.12 C Q.13 C Q.14 B
Q.15 B Q.16 D Q.17 D Q.18 B Q.19 C Q.20 B Q.21 B
Q.22 D

Comprehension Type

Q.23 A Q.24 A Q.25 B Q.26 C Q.27 A

Assertion Reasoning Type

Q.28 D Q.29 C Q.30 C Q.31 C Q.32 D

Previous Years’ Question


Q.1 C Q.2 B Q.3 B Q.4 D Q.5 A, C Q.6 A, D Q.7 B

Q.8 D Q.9 C Q.10 A → s; B → p; C → q; D → q, r; E → t Q.16 B

Q.17 A → p; B → q; C → p, r; D → p, s Q.18 A Q.19 C, D

Q.20: A → p, q, s; B → t; C → q, r; D → r Q.21 A, B, C
2 9 . 5 0 | Extractive Metallurgy

Solutions

JEE Main/Boards 
2C
O2
G
+O
2

O
Exercise 1 2C
O

Sol 1: The reduction potential of zinc and iron is lower ZnO - Zn

than that of copper. In hydrometallurgy, Zn/Fe can be


used to displace Cu from the solution
Fe(s) + Cu(aq.)
2+
→ Fe(aq.)
2+
+ Cu(s) ↓ T
r

M.P B.P T0
But to displace Zn, more reactive metals i.e. Na, K are of Zn of Zn
required but they violently react with water and gives
hydrogen gas. T0 > TB.P of Zn So, Zn will be in vapour form and its very
tough to maintain such high temperature in furnace
1 for long time and at this temperature, carbon forms
K + H2O → KOH + H2 ↑
2 carbides.
∴ By hydrometallurgy, Cu can be extracted but not Zn.

Sol 5: Element present in the anode mud:- Se, Te, Ag,


Sol 2: Ellingham diagrams. Au, Pt and Sb
In this range of temperature, diagrams show that These elements are very less reactive and are not
carbon monoxide act as stronger reduction agent than affected during the purification process. Hence, they
carbon. settle down below anode as anode mud.

g
O
Sol 6: In the process of Bessemerisation, air is blown
into molten matte with silica. So, by self reduction CuS
gets reduced to Cu but at same time, FeS also gets
converted to FeO but to avoid further reaction, silica
1 CO
2 is added.
FeO + SiO2 → FeSiO3 (Slag)
CO Thomas slag
By this, molten copper gets separated out.
2CO + O2 → 2CO2
has more negative free energy than
Sol 7: Refer Theory - Chromatography
2C + O2 → 2CO
∴ In the upper part of furnace, haematite is reduced by Sol 8: The composition difference between cast and
CO even in the presence of C. Pig iron is, Pig iron has 4% carbon content whereas cast
iron has at least 2% carbon content.
Sol 3: In case of low grade copper ores, such as
chalcopyrites, Ag is used to reduced the remaining Cast iron
copper.
Cu2+ + 2Ag → Cu + 2Ag+
Impurities
and by formation of shiny Ag2S layer, it is confirmed
that Cu got reduced.
After extracting from furnace, we get pig iron and we
put them in a case from which impurities can be cut out
Sol 4: Ellingham diagram clearly indicates that the CO so there is difference of amount in impurity.
graph cut at much higher temperature to Zinc oxide’s
graph.
Chem i str y | 29.51

Sol 9: Minerals are the substances in which metals are Sol 17: (a) Acidic flux:- Used to remove basic gangues
present either in native or combined forms but ores
E.g. SiO2, Na2B4O7.10H2O
are that minerals from which metal can be extracted
economically and conveniently. (b) Basic flux:- Used to remove acidic gangues
E.g. CaO, MgCO3, CaCO3.
Sol 10: From Ellingham diagram, graph of C cuts the Zn
graph at lower temperature than CO. Sol 18: Gangue:- (Matrix)
These are the non-metallic impurities present in the
ore. In fact, impurities associated with an ore is known
as gangue.
CO CO2
Zn ZnO

Sol 19: Refer theory


C CO2

Sol 20: (1) Zone Refining: Fractional crystallisation


:-Based on difference in solublities of impurities in
molten and solid state.
∴ ZnO + C → Zn + CO, ∆G1 E.g. Ge, Ga, Si etc (To extrapurify the metal)
ZnO + CO → Zn + CO2, ∆G2 (2) Electrolytic Refining: Faraday’s law of electrolysis
|∆G1| > |∆G2| so, first reaction is favourable Metals which are not reduced by C/CO, are reduced by
∴ C is better reducing agent than CO. this method. E.g. Na, Ca, K, Al etc.
(3) Vapour phase refining:
Sol 11: Graphite rod is used as an anode and graphite
Ni+4CO→ [Ni(CO)4]  ∆
→ Ni↓ +4CO Purification by
line iron as cathode during the electrolysis of alumina.
forming volatile carbonyl complex.

Sol 12: Refer theory


Sol 21: Froath floatation:- Based on the different
wetting property of the ore and gangue with oil and
Sol 13: Refer theory water. Depressants are used to depress the frothing
tendency of the frothing agent Eg. NaCN, KCN
Sol 14: Copper matte contains sulfides of copper and Example: In the selective froth floatation of PbS, ZnS
iron. While reducing, it is Bessemer converter by self- can be depressed using depressant.
reduction, CuS gets converted to Cu but to avoid Fe in
the final product, silica is used which forms slag with Zn + 4CN– → [Zn(CN)4 ]2−
FeO
FeO + SiO2 → FeSiO3 Sol 22: Copper pyrites → CuFeS2 composes sulfides of
CuS and FeS both.
(slag)
While extracting copper from pyrites, when we oxidise
Sol 15: Generally, aluminium is extracted by using the ore 2FeS + 3O2 → 2FeO+2SO2, ferrous oxide also
electrometallurgy, but for that purpose Al should be in gets formed which has to be removed by slag formation
molten state. But, melting point of aluminium is very
high. So, to increase the solubility and to decrease the CuS + O2 → CuO + SO2
Self
melting point, cryolite is added.
2CuO + CuS → 3Cu + SO2 reduction
Sol 16: Roasting is a process in which ore is heated
in presence of oxygen to convert it into corresponding But in simple oxide ‘CuO’, it can be easily reduced by
oxides. So the ores whose oxides can be easily reduced carbon reduction
by C/CO or by self reduction are roasted. E.g. CuS, FeS, CuO + C → Cu + CO2 ↑
but ores of Al, Na, K cannot be roasted.
2 9 . 5 2 | Extractive Metallurgy

Sol 23: (i) Zone Refining: Method to obtain metals of CO2


very high purity metals like Ge, Ga, Si etc. This method Filtrate (Sod. aluminate + Sod. Silicate)


filtered
→
is based on difference in solubilities of impurities in Al(OH)3 → Pure Al2O3
molten and solid state. This refining is as shown below: (ii) Hall’s process:
Finely Bauxite:
Inert
Finely Bauxite:-
atmosphere
Na2CO3 Solution+
Molten
Fused, extracted (Residue SiO2 +Fe2O3)
metal
with water

Circular CO2
ppt. Al(OH)3
furnace 50 - 60 & filtered
Filtrate (Na2CO3) 
Heater melts that part of metal and impurities flows in
the molten part and by moving this, eventually get at
Pure Al2O3
the end and cut out, this process is repeated several
times. (iii) Serpeck’s Process:-
Coke +N2
(ii) Column Chromatography:- This technique is based Finely powdered bauxite ore  →
∆ to 1800°C
on different adsorbing power of different metal ions on silica reduced + AlN
a surface. This technique is used for that metals whose to Si which
volatises H2O
separation is difficult. Eg. Lanthanoids, actinoids.
Pure Al2O3 ←

 Al(OH)3
Sol 24: At 600-900ºC: Fe2O3+3CO → 2Fe+3CO2
At 900-1000ºC ; CaCO3 → CaO + CO2 ; CO2 + C → 2CO
Sol 28: Refer Elingham diagram
Sol 25: Extraction of any ore is done in 3 parts.
Sol 29: Yes, Reducing agent’s selection widely depends
(i) Concentration:- froth floatation process.
on thermodynamic factor because free energy is the
(ii) Roasting:- factor which decides the reaction feasibility.
2ZnS + 3O2 2ZnO + 2SO2 E.g. (1) In the upper part of blast furnace, carbon
monoxide is better reducing agent than carbon
because in that temperature range, ∆G of CO’s reaction
is less than for C.
Carbon Reduction (2) Reduction of ZnO is not feasible by carbon because
ZnO + C Zn + CO it can be only possible at high temperature at which
(iii) Purification:- fractional distillation & electrolysis. both compounds are in vapour phase.

Sol 30: In the electrolysis of NaCl, Cl2 is obtained as a


Sol 26: Refer theory
by-product

Sol 27: Silica in alumina is a natural impurity and before


Sol 31: Ellingham Diagram:-
electrolysis, this impurity should be removed. So any of
the following chemical process can be used to purify
alumina from bauxite. G
O3
Al 2
(i) Bayer’s process:- Finely powdered 2/3
Roasted O2
Bauxite  → Roasted ore 4/3
Al+
FeO →Fe2 O3
gO
2M
+O
2
Caustic soda solution
Roasted ore → 2M
g
High pressure, filtered
Fe2O3 as residue
1750C Temp.
Chem i str y | 29.53

Reduction of MgO by Al is feasible at very high Sol 14: (A) Poling:- Molten metal is stirred with green
temperature around 1750°C anode poles which releases hydrocarbon (gas) which
reduces the oxide e.g. Cu
2Al+3MgO → 3Mg↓ + Al2O3
Cu2O + CO → 2Cu + CO2 ; Cu2O + H2 → 2Cu + H2O
Because ∆G is negative after this temperature. So,
aluminothermite is not useful to extract Mg from MgO.
Thus, electrolytic reduction is used. Sol 15: (C) SiO2 is an acidic flux SiO2 + CaO → CaSiO3
Acidic Basic

Sol 16: (C) Zone refining is a method to obtain ultra


Exercise 2
pure metal.
Sol 1: (B) During roasting, air is blown to convert metal
sulphides into metal oxides. Sol 17: (A) Mac Arthur process is basically cyanide
process.
3
FeS + O → FeO + SO2
2 2
Sol 18: (C) Electroplating: To deposit a specific metal
Sol 2: (C) Cu can be reduced by self-reduction. So, on some other to protect the inner ore is done by
electrolysis.
3
Cu2S + O2 → Cu2O + SO2 ; 2Cu2O + Cu2S → 6Cu + SO2
2 Electrorefining: Electrochemical series decide which
metal to be deposited first on anode and using it, we
Sol 3: (A) Pyrolusite → MnO2 can remove the impurities by electrolysis.

Sol 19: (B) Calcination: Ore is heated at high


Sol 4: (A) Ag, Au, Hg, Pt are chemically inert metals.
temperature in the absence of air.
E.g. Applied for carbonate and hydrated ores.
Sol 5: (D) CaO+ SiO2 → CaSiO3
lime silica slag CuCO3.Cu(OH)2 → CuO + CO2 + H2O

Sol 6: (A) Duralumin an alloy of aluminium is used in Sol 20: (B)


aircraft industry
873 K Reduction zone
Sol 7: (C) Bauxite ore impurities:- SiO2 + Fe2O3 1273 K Slag formation zone
Acidic impurities

Sol 8: (A) Si is a semi-conductor.


1573 K Fusion zone

2173 K Combustion zone


Sol 9: (A) Ag + 2CN → Ag(CN)
– −
2

Zn, 2CN-
Sol 21: (B) Mg and Al’s aqueous solution on
electrolysing gives hydrogen at anode.
Ag + [Zn(CN)4]2–
∴ Molten mixture is used.
Sol 10: (D) Flux + Gangue → slag.
Sol 22: (A) Van-Arkel method:- Metal is converted into
a volatile stable compound (e.g., Iodine) and impurities
Sol 11: (A) Zone refining:- Purification is done at are not affected during compound formation
microscopic level because we need ultra-purified
E.g. Zr, Ti
element to use in semi-conductors. Zone refining is
based on fractional crystallisation. Ti + 2I2 → TiI4 (g)
1700K
Ti I4(g) → Ti(S) + 2I2(g)
Sol 12: (D) None of this elements can be reduced by Al
but other example are Cr, Mn, Ti, etc.
Sol 23: (A) In the froth floatation process for the
purification of ores, the ore particles float because they
Sol 13: (A) Anode mud are the elements which get are light.
deposited in the bottom of anode Eg. Ag, Au, Pt.
2 9 . 5 4 | Extractive Metallurgy

Sol 24: (C) Van arkle method is used. Sol 11: (A) Metal sulphides are thermodynamically
more stable than CS2
Sol 25: (B) Calcination is the process of heating the ore
in absence of air.
Sol 12: (D) Van Arkel method
Sol 26: (B) Highly electropositive metal can not be Ti ( s ) + 2I2 ( g) 
523K
→ Til4 ( g)
reduced because it requires high temperature at which
carbide formation occurs. Til4 ( g) 
1700K
→ Ti ( s ) + 2I2 ( g)

Sol 27: (A) Cyanide process is used for obtaining Cr.


Sol 13: (D) 13 In the context of the Hall-Heroult process
for the extraction of Al, Na3AlF6 serves as the electrolyte
Previous Years’ Questions
Sol 14: (B) Galena = Pbs
Sol 1: (D) Fluorspar (CaF2), Cryolite (Na3AlF6), Feldspar
(KAlSi3O8), Mica (K2O.3Al2O3.6SiO2.2H2O). For sulphur ores froth floatation is carried out.

Sol 2: (D) Pig iron → It is the most impure form of iron Sol 15: (C) Galvanization is applying a coating of Zn.
and contains highest proportion of carbon (2.5 – 4%)
Malachite → Cu(OH)2.CuCO3
Zinc blende → ZnS
JEE Advanced/Boards
Bauxite → Al2O3.2H2O
Exercise 1
Sol 3: (D) Malachite (Cu(OH)2.CuCO3). Sol 1: Goldschmidt thermite process:- Reduction by
powdered aluminium. This process is employed in
Sol 4: (B) Cuprite (Cu2O) and Argentite (Ag2S). cases where metals have very high m.pt and are to be
extracted from their oxides.
Sol 5: (C) Among cuprite [Cu2O], Chalcocite [Cu2S], Process:-
Chalcopyrite [CuFeS2] & Malachite [Cu(OH)2.CuCO3].
Mg + BaO2 → BaO + MgO + Heat → (in large amount)
Ignition mixture
Sol 6: (A) Au is recovered from the solution by the
addition of electropositive metal. Cr2O3+Al 

→ 2Cr()+Al2O3 + Heat
2NaAu(Cu)2 + Zn → Na2Zn(CN)4 + 2Au↑
E.g. Reduction of Cr, Ti, W, Mn etc.
Soluble complex
Mg ribbon

Sol 7: (B) Both assertion and reason are true and


reason is the correct explanation of assertion.

Sol 8: (B) Both assertion and reason are true but reason
is not the correct explanation of assertion. Non-fusible
mass present in ore in mixing with suitable flux are fused
which are then reduced by coke to give free metal.

Sol 9: (D) Leaching is a process of concentration. Sand

Ignition
Sol 10: (C) Assertion is true but reason is false. mixture
Collectors absorb themselves on polar groups to grains Al power
of ores and thus derive them on the surface to pass on + metal oxide
into the froth.
Chem i str y | 29.55

Sol 2: Al2O3 + C → 2Al + CO2 Sol 7: Instead of molten NaCl, if we use aqueous
solution of NaCl, then, at cathode, Na+ ions are not
∆G of this reaction is negative at very high temperature,
discharged at cathode on the contrary, H+ ions get
so there are 2 reasons that it is not useful:-
discharged to form H2.
(i) Such high temperature is difficult to achieve in 1
H+ + e– → H2 ↑
furnace for very long time and at such temperature, 2
Al2O3 will also be in vapour phase.
At anode, Cl2↑ will form irrespective of solvent.
(ii) At high temperature, C instead of reducing forms
carbide Al4C3 with aluminium. In molten NaCl, there is no competition for Na+ to get
discharged at cathode.
Sol 3: Redox reaction depends upon standard reduction Na(aq.)
+
+ e− → Na(s)
potential of that metals.
EoAg+ / Ag − ECu
o
2+ > 0 and to fease the reaction, E° should Sol 8: The members of group 1 i.e. alkali metals have
/ Cu
be positive of that whole reaction. low ionization potential value and thus possess high
electropositive character as well as high oxidation
∴ Cu + 2Ag+ → 2Ag + Cu2+
potential which enables then to get easily oxidised so
Oxidation half:- Cu → Cu2+ + 2e- ; E1 = – EoCu2+ / Cu that’s why they are strong reducing agent but they are
not widely used because they react violently with water
Reduction half:- Ag+ + e– → Ag ;E2 = EoAg+ / Ag
Na + H2O → NaOH + H2↑
EReaction
o
= 2EoAg+ / Ag − ECu
o
2+
/ Cu
>0

Sol 9: In blast furnace, lime stone, CaCO3 is useful in


Sol 4: Sulphide Roasting Oxide fusion zone, where at 800°C – 1200°C, CaCO3 gets
Oxide Impure metal dissociated in CaO + CO2 and it acts as flux, which
Carbon Reduction
reacts with acidic gangue SiO2 to form slag.
CaCO3 → CaO + CO2
Electrolysis
(Purification) CaO + SiO2 → CaSiO3 (slag)

Pure metal Sol 10: In blast furnace, temperature differs widely


from top to bottom.
O2
E.g. FeS  → FeO 
 C + CO
→ Fe → Fe So, in different temperature range different compound
∆ (inpure) (pure)
behaves as a reducing agent.
3 At 300 – 800°C,
Sol 5: (a) PbS +  O2 PbO + SO2
2
CO is better reducing agent than C
Pb3O4 + SO2 3Fe2O3 + C → 2Fe3O4 + CO
Fe3O4 + CO → 3FeO + CO2
(b) Cu2O + C → 2Cu + CO
FeO + CO → Fe + CO2
(c) Ag +
+ e → Ag(s)

(aq.) For this, in zone of combustion.
C + O2 → CO2
Sol 6: E(OP)
o
of Zn is more positive than EoOP of Cu and
this Zn is more powerful reductant than Cu and thus, Zn CO2 + C → 2CO
can easily replace Ag from [Ag(CN)2− ]
Sol 11: (i) CdCO3 

→ CdO + CO2
Zn + [Ag(CN)2− ] → Ag + [Zn(CN)4 ]2− E1, K1
Thermal decomposition
Cu + [Ag(CN)2 ]− → Ag + [Cu(CN)4 ]2− E2, K2
(ii) MgO 

→ No reaction
K1 >> K2 because |E1| > |E2|
(iii) SnO2 + CO 

→ Sn + 2CO2
Furthermore, Zn being cheaper in comparison to Cu. Reduction
(iv) CdSO4(aq)  → H2SO 4 + O2 + Cd

electrolysis  at cathode
at anode
2 9 . 5 6 | Extractive Metallurgy

Sol 15: Calicnation


(v) 2HgO 

→ Hg + O2
Thermal decomposition
Sol 16: Mg
HNO3
Mg(NO3)2 MgO
(vi) MgO + Zn → NO reaction

Cl3
Sol 12: Graphite and diamond, both are isomers but -H2O

their physical properties differ widely. KHO


MgCl2 Mg(OH)2

Sol 17:
Dil.
(A) (B) + (C)
NaCN+O2
Black
mineral
Zn

(D) Dil. Dil.


(E)
HNO3 HCl
Metal White ppt

fusion with
aq. NH3
Graphite conducts electricity because there exist free Na 2CO3

electrons between two sheets of graphite. On the


other hand, diamond is a bad conductor because it (F)
does not have free electrons and during electrolysis, Colourless
it is important for anode to conduct electricity and (A) → (D) leaching process
complete the circuit.
∴ these are the compounds of Ag

Sol 13: (i) 2Cu2S-2+ 3O2(g) 2Cu2O(l) + 2S O2(g)


+4 ∴ (A) Ag2S
(B) [Ag(CN)2]– Na+
(C) Na2S
(D) Ag
Reducing Oxidising
(E) AgCl
agent agent
(F) [Ag(NH3)2]Cl
(ii)
+1 +4
2Cu2 O(l) + Cu2S(g)-2 6Cu(l) +SO2(g)
Sol 18: (i) Aluminium:- Gold Schimdt process:-
Al + Mn3O4 → Mn + Al2O3
(ii) Fe is also used
Oxidising Reducing
Fe + HgS → Hg + FeS
agent agent

Sol 19: Aluminium metal is frequently used as a


Sol 14: Calcination:- reducing agent for the extraction of metal such as Cr
and Mn from their respective oxides because aluminium
MgCO3.CaCO3 
Calcination
→ MgO + CaO + 2CO2↑ is more electropositive than Cr/Mn. The process of

Dolomite reduction is known as Aluminothermite process (Gold
shmidt process)
MgCl2+CaCO3↓ 
CO2
→ MgCl2+CaCl2
Cr2O3 + 2Al → Al2O3 + 2Cr; ∆H = –ve
Now, electrolysis of MgCl2 gives Mg
3Mn3O4 + 8Al → 4Al2O3 + 9Mn; ∆H = –ve
Mg2+ + 2e– → Mg Anode reaction
2Cl– → Cl2↑ + 2e– Cathode reaction
Chem i str y | 29.57

Sol 20: During the extraction of iron (Fe) from haematite Sol 23: Cyanide process is a type of leaching process
(Fe2O3) it contains some impurities such as P4O10 or SO2 , which a chemical reaction is used to extract metal from
which are acidic in nature. So, to remove this gangue ore. In this process, crushed rock containing traces
as slag basic lining of lime (CaO) or magnesia (MgO) of gold is treated with 0.1 – 0.2% solution of NaCN
is used and aerated O2 (air) oxidized free metal to Au+ which
complexes with CN–
6CaO + P4O10 → 2Ca3(PO4)2
4Au + 8CN– + 2H2O + O2 → 4[Au(CN)2]– + 4OH–
MgO + SO2 → MgSO3
The pure metal is displaced from solution by an active
Sol 21: Lead is extracted from its mineral galena (PbS). metal
(i) It is firstly concentrated by froth floatation process. 2[Au(CN)2]–+Zn→ [Zn(CN)4]2– + 2Au↓
(ii) Roasting in reverbatory furnace
Sol 24: Silver coin contains Ag, Au and Cu also. So, to
2PbS + 3O2 → 2PbO + 2SO2↓
extract silver, it should undergo various processes.
PbS + 2O2 → PbSO4
So, individually every metal impurities can be removed.
The idea of Cu ores come from calcined black solid and
Silver coin
ppt. after reaction with KI.
Conc. 2+ + 3+
(Cu+Ag+Au) (Cu +Ag +Au )NO3-
HNO3
Sol 22: A1 will be malachite i.e., CuCO3.Cu(OH)2
A2 will be copper glance i.e., Cu2S The confirmatory
NaCl

reaction are
AgCl (white ppt.)
(a) For malachite AuCl3
(i) CuCO3.Cu(OH)2 
Calcinations
→ 2CuO + CO2 ↑ H2O
Black solid Now, since Ag, Au, both are noble metals, they possess
similar chemical properties.
(ii) CuCO3.Cu(OH)2 
HCl
→ CuCl2 + CO2↑ + H2O But due to difference in physical properties, Au can
be removed by fractional distillation.
CuCl2 
KI
→ Cu2 I2 ↓ + KCl + I2
ppt(D)
After oxidising minerals,
(b) For copper glance
The roasted mineral is smelted into lead
Cu2S 
Roas ting
→ Cu2O + SO2
O2 2PbO + PbS → 3Pb + SO2 ↑
Cu2S + Cu2O → Cu + SO2 → Self reduction
PbSO4 + PbS → 2Pb + 2SO2 ↑
SO2 is the gas which gives green colour with acidified
PbO (litharge) oxidation state → +2
K2Cr2O7 as
Purification is done by cupellation, polling and most
3SO2 + K 2 Cr2 O7 + H2SO4 → K2SO4 + Cr2 (SO 4 )3 + 4H2O importantly, electrolysis.
orange green

Now reduction of AgCl, Sol 25: In the purification of bauxite, due to impurities
such as silica (SiO2) and other silicates, Bauxite is
(i) 2AgCl + Na2CO3 → Ag2CO3 + 2NaCl converted into alumina by Bayer’s process. During this
process, CO2 is passed through it. Aqueous solution of
1
Ag2CO3 

→ 2Ag↓+ CO2↑+ O↑ CO2 is acidic. Hence, CO2 when passed into [Al(OH)4]–
2 2 solution makes it acidic. Hence, Al(OH)3 is precipitated.
(ii) Cyanide process with AgCl. CO2 + H2O → H3O+ + HCO3−
[Al(OH)4]–+H3O+ → Al(OH)3↓ + 2H2O
2 9 . 5 8 | Extractive Metallurgy

Sol 26: Haematite ore majorily contain [Fe2O3/FeO)] but Sol 6: (D) Metal makes lattices in which nucleus is
also contains silica (SiO2). So, during roasting in blast arranged in specific array of position but valence
furnance, carbon reduction is done along with lime. So, electrons are free to move into whole lattice this
silica can be removed as slag. property gives lustre to a metal.
CaO + SiO2 → CaSiO3
(Slag) Sol 7: (C) For Cr, instead of electrolysis, we can use more
Simultaneously, Fe also gets reduced cheaper process, aluminothermite, because electrolysis
gives high yield but it’s a very costly process.
C + O2 → CO2 ; CO2 + C → 2CO
3CO + Fe2O3 → 2Fe + 3CO2
(Steel) Sol 8: (D) Flux’s basic use is to combine with gangues/
So difference in densities make them easily separable at matrix or other impurities and then remove them by
the bottom of furnace and this calcium silicate because slag formation.
of its hard nature is widely used in cement industry. E.g. Formation of CaSiO3 in extraction of iron (Fe).

Sol 27: (ii) Au and Ag both are generally chemically Sol 9: (B) In extraction of Cu, FeO is a gangue to remove
inert but they differ during their reaction with HCl + it, SiO2 (silica) is added and it forms
HNO3
Thomas slag
Au + 4HCl + 3HNO3 → HAuCl4 + 3NO2 + 3H2O
soluble
FeO + SiO2 → FeSiO3
Gangue Flux Thomas slag
Ag + HCl + HNO3 → AgCl + NO2 + H2O
insoluble

So, in Aquaregia, Au forms a soluble complex Sol 10 (D) In blast furnace


compound, HAuCl4 but Ag forms a precipitate which
FeO/Fe2O3 Fe3O4 Fe
do not dissolve.

Roasting
Exercise 2
Fe3O4
Single Correct Choice Type

Sol 1: (A) Sulphides have fundamental property based Sol 11: (C) Al2O3 is used in molten state because
on the different wetting property of the ore and gangue aqueous solution will give H2 at cathode and Na3AlF6
with oil and water. is used to increase the conductivity of Al2O3 and to
decrease the melting point of mixture.
Sol 2: (C) Van Arkel
This is vapour phase refining process by forming volatile Sol 12: (C) Calcination is used in metallurgy to remove
complex compound. CO2 and H2S

Sol 3: (B) Cryolite-Na3AlF6 is used during the electrolysis


Sol 13: (C) The metallic luster exhibited by sodium is
of alumina to increase the electrical conductivity and
explained by the presence of Free protons
decrease the melting point of alumina (Al2O3).

Sol 14: (B) Stainless steel:- Fe + Cr + Ni


Sol 4: (D) Cassiterite is an ore of Sn.

Sol 15: (B) Leaching:-


Sol 5: (C) Discharging of element at cathode and anode
depends upon various factors mainly on Al2O3 + 2NaOH → 2NaAlO2 + H2O
(i) standard reduction potential 2NaAlO2+4H2O → 2Al(OH)3+2NaOH
(ii) Nature of electrode 2Al(OH)3 

→ Al2O3 + 3H2O
1200

Using platinum and graphite electrode, we get different


products so it is entirely experimental.
Chem i str y | 29.59

Sol 16: (D) Bessemerisation of Cu Sol 27: (A)


(A) Dil. H SO
2 4
Black (B) + (C)
Gas
Imp.+Si (XS) NH4OH
Matte
(D)
Bessemer converter Complex
2FeS + 3O2 → 2FeO + 2SO2
2CuS + SiO2 → FeSiO3 (slag) K4[Fe(CN)6]
Chocolate brown ppt.
2CuS + 3O2 → 2Cu2O + 2SO2 (E)
2Cu2O + Cu2S → 6Cu + SO2 Chocolate brown ppt. form with Cu metal
(E) is Cu2 [Fe(CN)6]
Sol 17: (D) Generally they are more electronegative
than metals.
Assertion Reasoning Type

Sol 18: (B) In the froth floatation process for the Sol 28: (D) Calamine (ZnCO3)
purification of ores, the ore particles float because their
surface is hydrophobic i.e. not easily wetted by water (R) Boric acid is monobasic acid
B(OH)3 + H2O → [B(OH)4]– + H+
Sol 19: (C) SiO2 Acidic flux.
Sol 29: (D) Based on experiments.
Sol 20: (B) Flux + Gangue → slag
Sol 30: (C) Pb4+ is easily reduced to Pb2+ because of
Sol 21: (B) E.g. Na, K, Ca, Al, etc. inert pair effect.

Group I, II metals. (R) Pb2+ is diamagnetic.

Sol 22: (D) The role of calcination in metallurgical Sol 31: (C) Diamond is hardest natural occurring
operations is to remove moisture, to decompose substance because of its strong 4 valence lattice.
carbonate,to drive off organic matter. (R) Graphite is more stable because of electrical
conductivity and availability of free electrons.
Comprehension type
Sol 32: (C) Mond’s Process:-
Sol 23: (A) CuS
Ni+4CO 

→ Ni(CO)4

 → Ni↓ + 4CO
volatile stable
complex compound
Sol 24: (A) H2S

Sol 25: (B) CuSO4


Previous Years’ Questions
Sol 26: (C) [Cu(NH3)4](NO3)2
Sol 1: (C) Chalcopyrite contain both iron and copper.

Sol 2: (B) Zinc blende contain ZnS which is first roasted


partially and then subjected to reduction with carbon
ZnS + O2 → ZnO + SO2 Roasting

ZnO + C → Zn + CO↑ Carbon reduction
2 9 . 6 0 | Extractive Metallurgy

Sol 3: (B) A water soluble complex with silver and (E) Copper is extracted by self-reduction methods
dilute aqueous solution of NaCN is Na[Ag(CN)2]. In the
Cu2S + O2 → Cu2O + SO2
cyanide process, the native silver is crushed and treated
with aqueous NaCN solution and aerated. Cu2O + Cu2S → Cu + SO2
4Ag + 8NaCN + 2H2O + O2 → 4Na[Ag(CN)2] + 4NaOH
Sol 11: 4NaCN +Ag2S → 2Na[Ag(CN)2] + Na2S
Sol 4: (D) Haematite is Fe2O3, in which oxidation 2Na[Ag(CN)2] + Zn → Na2[Zn(CN)4] + 2Ag
number of iron is III. Magnetite is Fe3O4 which is infact a
mixed oxide (FeO.Fe2O3), hence, iron is present in both Sol 12: The common photographic film is coated with
II and III oxidation state. AgBr and during developing of photographic film, the
unreacted AgBr is removed by Na2S2O3 as
Sol 5: (A, C) Addition of manganese to iron improve AgBr + 2Na2S2O3 → Na3[Ag(S2O3)2] + NaBr
hardness of steel as well as remove oxygen and sulphur.

Sol 13: 2AgBr + C6H4(OH)2(Hydroquinone) →


Sol 6: (A, D) The important ore of tin is cassiterite
(SnO2). Tin is extracted from cassiterite ore by carbon 2Ag + 2HBr + C6H4O2
reduction method in a blast furnace. (developer) quinone
SnO2 + 2C → Sn + 2CO AgBr + 2Na2S2O3 → Na3[Ag(S2O3)2] + NaBr
The product often contain traces of iron which is (b) Na2S2O3 + 2H+ → 2Na+ + H2SO3 + S↓
remov-ed by blowing air through the melt to oxidize to
FeO which then floats to the surface. Colloidal

2Fe + O2 → 2FeO Sulphur

Sol 7: (B) 2CuFeS2 + O2 → Cu2S + 2FeS + SO2↑ Sol 14: Four, the complex has formula [Cu(H2O)4] SO4.H2O

2Cu2S + 3O2 → 2Cu2O + 2SO2↑


Sol 15: In crystalline state, AlCl3 has rock-salt like
2FeS + 3O2 → 2FeO + 2SO2↑ structure with coordination number of Al = 6.

Sol 8: (D) FeO + SiO2 → FeSiO3 (slag) Sol 16: (B) Extraction of zinc from zinc blende is
achieved by
Sol 9: (C) Cu2S + 2Cu2O → 6Cu + SO2
In Cu2S, sulphur is S2– and in SO2, sulphur is in +4 state. Sol 17: A → p; B → q; C → p, r; D → p, s
Hence, S2– is acting as a reducing agent. PbS → PbO - Roasting
CaCO3 → CaO - Calcination
Sol 10: (A) A →s; B → p; C→ q; D → q, r; E→ t
ZnS → Zn - Roasting, Carbon reduction

Silver is extracted by amalgamation process Cu2S → Cu - Roasting, Self reduction


Distillation
→ Ag(s) + Hg(g)↑
Ag + Hg → Ag(Hg) 
Sol 18: (A)
(B) Calcium is extracted by electrolysis of fused CaCl2.
Sulfinde ore of Ag → Argentite ( Ag2S )
(C) Zinc is extracted by carbon reduction method
Pb → Galena (PbS )
ZnO + C → Zn + CO
Cu → Copper glance ( Cu2S )
(D) Iron is extracted by both, carbon reduction method
and CO reduction methods Hence (A) is correct.

Fe2O3 + 3C → 2Fe + 3CO


Sol 19 : (C, D) Fe2O3 and SnO2 undergoes C reduction.
Fe2O3 + 3CO → 2Fe + 3CO2 Hence (C) and (D) are correct.
Chem i str y | 29.61

Sol 20: A → p, q, s; B → t; C → q, r; D → r
Siderite FeCO3
Malachite CuCO3 .Cu ( OH)
2
Bauxite AlO x ( OH) ; 0< x <1
3−2x
Calamine ZnCO3
Argentite Ag2S

Sol 21: (A, B, C) Extraction of copper from copper pyrite


(CuFeS2) involve crushing following by concentration of
the ore by froth-flotation, removal of iron as slag, self-
reduction step to produce ‘blister copper’ following
evolution of SO2
2017-18 100 &
op kers
Class 12 T
By E ran culty
-JE Fa r
IIT enior emie .
S fP r es
o titut
Ins

CHEMISTRY
FOR JEE MAIN & ADVANCED
SECOND
EDITION

Exhaustive Theory
(Now Revised)

Formula Sheet
9000+ Problems
based on latest JEE pattern

2500 + 1000 (New) Problems


of previous 35 years of
AIEEE (JEE Main) and IIT-JEE (JEE Adv)

5000+Illustrations and Solved Examples


Detailed Solutions
of all problems available

Plancess Concepts
Topic Covered Tips & Tricks, Facts, Notes, Misconceptions,
Key Take Aways, Problem Solving Tactics
Qualita ve Analysis
PlancEssential
Questions recommended for revision
30. Q U A L I TAT I V E
A N A LY S I S

1. INTRODUCTION

Salt → Cation (C+ ) + Anion (A − )


 
from base from acid
(basic radical) (acidic radical)

Determination of quality of any salt mixture is called Qualitative analysis or Salt analysis.

Flowchart 30.1: Procedure of salt analysis


3 0 . 2 | Qualitative Analysis

2. PHYSICAL APPEARANCE
Table 30.1: Physical properties

Experiment Observation Inference


Color (due to d-d transition of e- in Blue or bluish green Cu2+ or Ni2+
d-block elements and f-f transition Green Ni+2
of e- in f-block elements)
Reddish brown Fe+2
Pink violet
Fe+3
Light pink
Co+2
White
Mn+2
Absence of above transition metals ions
Ba+2 ,Sn+2 ,Ca+2 ,Mg+2 , Al+3 , Zn+2 etc 
 
Smell Rotten egg S2-
Vinegar CH3COO-
Ammonical NH4+
Density Heavy
Light powder Pb+2 ,Ba+2 can beCO32−

Deliquescence: Colorless → Mg+2 , Al+3 , Zn+2


Colored → Cu+2 ,Fe+3
Deliquescence → Substance absorbs moisture from the atmosphere until it dissolves 
 
in the absorbed water and forms a solution 
Hygroscopic → Substance absorbs and hold water molecules from the surroundings 
 
(Water of crystallisation)

3. DRY HEATING TESTS


Table 30.2: Dry tests

Observation Inference
1. Gas evolved:
Colorless + odorless CO32− ,HCO3−
Colorless
(
Rotten egg H2S ↑ ) S2 −
Vinegar like CH3COOH CH3COO −
Burning sulphur SO32− , S2O32− , S2−
Ammonical
NH+4

Colored (Pungent Smell) Cl−


NO3− , NO2−
(
Reddish brown NO2 ↑ )
Yellowish green Cl−

(
Reddish brown Br2 ↑ ) Br −
(
Violet black I2 ↑ ) I−
Chem i str y | 30.3

Observation Inference
2. Sublimate: Decomposition of solid substance and deposition White⇾NH4Cl,Hg(I),Hg(II) chlorides As(III),Sb(III) or Se(IV)
in the upper portion of test-tube. Yellow⇾ S , As2 S3 ,Hg(I) / Hg(II)iodides
 
free exception
sulphur to above

3. Decripitation: Decomposition with crackling sound on


heating. Salts not having water of crystallization. {halides of alkali metals , Pb (NO ) } 3 2

4. Fusion: Salts with water of crystallization will fuse generally. CuSO 4 .5H2O,FeSO 4 .7H2 O

5. Swelling:
PO 43− , BO33−
6. Residue:
Yellow when hot and white when cold ZnO
Brown when hot and yellow when cold PbO
White residue which glows on heating Ba+2 , Sr +2 , Ca+2 , Mg+2
Original salt is blue in color, becomes white on heating Cu2+ or CuSO4
Colored salts become brown or black on heating Co+2 ,Fe+3 ,Fe+2 ,Cr +3 ,Cu+2 ,Ni+2 ,Mn+2 ,etc

4. OTHER DRY TESTS


Figure 30.1: Different regions of flame Figure 30.2: Diagram showing non luminous and luminous flame

5. CHARCOAL CAVITY TEST


Charcoal piece (Carbon)
ZnSO 4 + Na2CO3 → ZnCO3 + Na2SO 4
 Bunsen burner
charcoal will
absorbNa2SO 4

ZnCO3 → ZnO + CO2

CuSO 4 → CuO → Cu

reddish residue
of metallic Cu

Metal carbonate decompose on heating to form metal oxides and Salt + Na2CO3
may further get reduced to metal especially if metals are less reactive
Figure 30.3: Set up for charcoal cavity test
(e.g Ag, Cu, gold).
Charcoal absorbs the anion formed by Na.
3 0 . 4 | Qualitative Analysis

6. COBALT NITRATE TEST


Residue of charcoal cavity test + cobalt nitrate.
If residue in charcoal cavity test is white, then this test is performed.

Table 30.3: Charcoal cavity test

Test Inference
Distinct colour residue is obtained for different metal cations.
(
Residue+ Co NO3 )2 + ∆
They are mixed cobalt-metal oxides.
(
E.g. 2Co NO
3 )2 → 2CoO + 4NO2 + O2
ZnO + CoO → ZnO.CoO

 
 
 CoZnO2 
   
Rinman' s green
ZnO.CoO Green

Al2O3.CoO Blue
MgO.CoO Pink
SnO.CoO Bluish

7. FLAME TEST
Generally, alkali and alkaline earth metal salts impart characteristic color to the flame. For this metals only, the
electronic de-excitation will be in visible range. To perform this test, the metal salts are converted into their
corresponding chlorides [generally, these metal chlorides are volatile] (volatile ⇾ vaporizes easily)

Figure 30.4: Steps to preform flame test


Chem i str y | 30.5

Table 30.4: Flame test for transition metals

Sr. No. Metals Color of flame


1 Sr , Li Crimson Red
2 Ca Brick Red
3 Ba Apple Green
4 K Violet (lilac)
5 Cu Blue Green
6 Na Golden Yellow
7 Ni Brown

8. BORAX BEAD TEST


Borax → salt of boric acid [Na2B4O7.10H2O]. The
free end of Pt wire is coiled into a small loop. This
loop is heated in Bunsen flame until it is red hot
and then is quickly dipped into powdered form of
solid. The adhering solid is held in the hottest
part of the flame. At first, salt loses water of Pt. Wire
Red Hot
crystallization and shrinks on the loop forming a
colorless, transparent glass like bead consisting of Figure 30.5: Heating a Pt wire
mixture of sodium meta borate and boric anhydride.
∆ ∆
Na2B 4 O7 .10H2O  → Na2B 4 O7  → NaBO2 + B2O3

colorless,tranparent glasslikebead.

The bead is moistened and dipped into the finely powdered subs. (salt) so that a minute of it sticks to the bead and
this bead is heated in lower reducing flame, is allowed to cool and the color is observed. This is then heated in an
oxidizing flame, allowed to cooled and color is observed again.
Coloured beads are due to the formation of various coloured transition metal borates. The Bead “R” and Bead “O”
is due to the variable oxidation states of the metal ions.

CuCO3 ∆
 CuO + CO2

CuO + B2O3 → Cu (BO2 )


2
blue colored bead is oxdising flame

2Cu(BO2 )3 + 2NaBO2 + C → CuBO2 + Na2B 4 O7 + CO

Copper (I) metaborate → Colorless

2Cu(BO2 )3 + 4NaBO2 + 2C → 2CuBO2 + 2Na2B 4 O7 + 2CO


Red colored
bead in
reducing flame

Table 30.5: Colour of flame of different ions during borax bead test

Color in oxidizing flame Color in reducing flame


Metal When Hot When Cold When Hot When Cold
Copper Green Blue Colorless Brown red
3 0 . 6 | Qualitative Analysis

Color in oxidizing flame Color in reducing flame


Metal When Hot When Cold When Hot When Cold
Iron Brown yellow Pale yellow Bottle green Bottle green
Chromium Green Green Green Green
Cobalt Blue Blue Blue Blue
Manganese Violet Red Grey Grey
Nickel Violet Brown Grey Grey

Non luminous flame = Oxidizing flame.


Luminous flame = Reducing flame.

9. MICROCOSMIC SALT BEAD TEST


Na(NH4 ).HPO 4 .4H2O  → NaPO3 + H2O ↑ NH3 ↑
  
micro cosmic
salt

(Ammonium sodium phosphate)

Na(NH4).HPO4 when heated first, colorless transparent bead of sodium metaphosphate is obtained.
This combines with metallic oxide to form orthophosphate which are of characteristic colors.
It will have color similar to borax bead test.
NaPO3 + CoO → NaCoPO 4 orthophosphate

(blue color)

10. CONFIRMATORY TESTS (WET TESTS)

Table 30.6: Wet tests

Sr. No. Acid radical Characteristics


1
CO32− All carbonates are ppt. except Na+ ,K + ,Rb + ,Cs+ ,NH+4

2 All bicarbonates are soluble (i.e. are found in soln. phase only).
HCO3−
Only alkali metals NH+4 bicarbonate can be obtained in solid form or can be isolated.
3
SO32− All sulphides are insoluble except group I- NH+4

4
S2 − The acid , bisulphide, polysulphides of alkali metal & NH+4 are soluble, rest are ppt.

Generally S2- ppt are common.

Sn2+ 
 are normally soluble
HS − 

5 NO2− ,NO3− ,CH3 COO − All are soluble except AgNO2,CH3COOAg, CH3COOHg
Chem i str y | 30.7

Sr. No. Acid radical Characteristics


6
All soluble except Ag+ ,Hg22+ ,Cu+ ,BiO + ,SbO + , Pb +2
Cl− 
soluble in
hot water

7 + +2 + +2
Br − All except Ag , Pb
 ,Cu ,Hg2
soluble in
hot water

8
I− All soluble except Ag+ ,Hg22+ ,Hg+2 ,Cu+ ,Bi+3 ,Pb +2
+2 +2
9 All soluble except Pb , Ba
SO24−
Sr +2 ,Ca+2 or Hg(II)
 
slightly soluble

Basic sulphates of Hg, Bi and Cr are insoluble but these dissolves in dil.HCl or dil. HNO3

E.g. HgO . HgSO 4 → insoluble


 


(basic) (neutral salt)

10 All phosphates are ppt except


PO34−
Grp I + NH+4
 
slightly
soluble

Primary phosphates of alkaline earth metals are also soluble (dihydrogen phosphate)
(
E.g. Mg H2PO 4 )2

11. ANALYSIS OF ANIONIC OR ACIDIC RADICAL

Flowchart 30.2: Analysis of anionic or acidic radical


3 0 . 8 | Qualitative Analysis

Table 30.7: Analysis of group A and group B radicals

Group A (dil H2SO4)


Sr. No. Observation Gases Inference
1 Effervescence of a colorless & odorless gas , which turns lime water milky CO2 2−
CO32-, HCO3

2 Evolution of colorless, suffocating gas with burning sulphur smell which SO2 SO32-
turns K2Cr2O7 paper green

Cr2O72– + SO2 → Cr3+(green)


3 Evolution of colorless gas with rotten egg smell which turns lead acetate H 2S
paper black(PbS) S2 −

4 Evolution of pungent smelling reddish brown gas which turns starch iodide NO2 NO2−
paper blue

I-+NO2 → I2 (I2+starch-blue complex)


5 Evolution of colorless gas having vinegar like smell CH3COOH CH3COO-
Group A(conc. H2SO4)
1 Colorless, pungent smelling gas which gives white fumes with rod dipped in HCl Cl-
NH3 (conc. H2SO4 cant oxidise Cl− to Cl2 )
2 Reddish brown gas with pungent smell and intensity of these fumes/ Br2 Br-
vapours increase on addition of a pinch of MnO2 & these also turn starch
iodide paper orange red.
3 Evolution of violet vapours which turn starch paper blue I2 I–
4 Evolution of brown fumes, intensity of which increase on addition of Cu NO2 NO3-
pieces/turnings and turns starch iodide paper blue.
Group B Ppt.
1 Water extract (WE)+sodium carbonate extract+soda extract(SE)+BaCl2(aq) White insoluble SO42-
ppt.
2 WE+SE+conc.HNO3(1N or 2N)+ammonium molybdate Canary yellow PO43-
ppt.

12. SODIUM CARBONATE EXTRACT

Filter paper

2g salt +
Funnel
3 times Na2CO3 Wire gauze
+ 20 ml distilled
Tripod stand
water
Burner Beaker
Sodium carbonate
extract

Figure 30.6: Set up for preparation of sodium carbonate extract

Sodium carbonate reacts with other inorganic salts to form water soluble salt of acid radical and the cation of salt
will be ppt. in the form of CO32-. SE is used when given salt is partially soluble or insoluble in water. Cation of the
salt interfere with the test of acid radical.
Chem i str y | 30.9

Dil.H2SO4 grp (CO32-,CO2)


Table 30.8: Sodium carbonate extract test

Sr. No. Reagent Observation Remarks


1 Dil.H2SO4 Colorless, Some carbonates like FeCO3,MgCO3
and CaCO3 should be powdered for
CO32− + H2SO 4 → SO24− + H2O + CO2 Odourless, brisk
 appreciable reaction.
H2CO3 Effervescence gas

2 Lime water Ca(OH)2 White turbidity White turbidity dissolves in dil. acid
liberating CO2 gas.
Ca(OH)2 + CO2 → CaCO3 ↓ (For CaCO3, prolonged
passage of CO2 CaCO3 ↓ + HCl
 → CaCl2 + CO2 ↑ +H2O
Ba(OH)2 + CO2 → BaCO3 ↓
dissolves the turbidity) dil.

CaCO3+CO2 →

Ca(HCO3)2

3 MgSO4/BaCl2 soln (colorless) White

Mg2++CO32- → MgCO3

Mg2++2HCO3 → Mg(HCO3)2

Mg(HCO3 )2  → MgCO3 ↓ +CO2 + H2O

4 AgNO3 soln. White ppt(dissolves in Carbonates are more easily polarized


NH3 or HNO3)
2Ag +CO3 → AgCO3
+ 2-
Metal carbonates are generally constant
∆ or Yellow/brown ppt. and not stable to heat.
AgCO3 →
s tanding
Ag2O + CO2

∆ 1
 → 2Ag + O2
2
5 HgCl2/Hg(NO3)2 Reddish brown ppt

Hg2++HCO3 →-No reaction

Hg2++3CO32-+3H2O→3HgO.Hg(CO3)2.Hg(CO3)2
(basic mercuric carbonate)
6 Phenolphthalein (HPh) CO32-+HPh = Pink

HCO3+HPh = Colo rless

ACTION OF HEAT

Bicarbonates  → Carbonates

2NaHCO3  → Na2CO3 + CO2 ↑ +H2O

Carbonates  → metal oxide + CO2

 exception Na+ , K + , Rb+ , Cs+ 
 
 

Li2CO3 → Li2O + CO2 ; PbCO3 → PbO + CO2

Carbonates of less reactive metals



 Ag, Au,Hg,Cu  → metal + CO2 ↑ +O2 ↑
∆ 1
Ag2CO3  → 2Ag + CO2 ↑ + O2 ↑
2
3 0 . 1 0 | Qualitative Analysis

Table 30.9: Confirmatory test for sulphite radical

S. No. Reagent Observation Remarks

SO32−

1 Dil.H2SO4 Colorless, suffocating


burning sulphur smelling
SO32− + H2SO 4 ( aq) → SO2 ↑ + H2O + SO24−
gas.
Above gas+lime/baryta water Milkiness If both SO32-,CO32- are
present,then SO32- will
Ca(OH)2 + SO2 → CaSO3 ↓ +H2O
be oxidized to SO42- by
K2Cr2O7

On prolonged passage, milkiness disappears No milkiness (a) and (b) not given by
CO32-
CaSO3+H2O+SO2 → Ca(HSO3)2

3+ 2− Green
a) acidifiedK 2Cr2O7 + SO2 → Cr
 + SO 4
   green
moist filter paper

Blue
b) SO2 + IO3− → I2 + SO24−
 
starchiodate (blue)
filter paper

2 BaCl2 Soln. White ppt. BaSO3 dissolves in-

Ba2+ + SO32− → BaSO3 ↓ (i) Dil.HCl BaSO3 + HCl →


On exposure to air, changes to BaSO4. SO2+H2O + BaCl2
Oxidising agents show such changes. (ii) Dil.H2SO4

H2O2 → H2O;Br2 → Br BaSO3 + H2SO4 → SO32-
HNO3 → NO; MnO −4 → Mn+2
Sodium nitroprusside Rosy-red coloration If both SO32- and S2- are
present, this test is not
Na[Fe(CN)5NO]+ SO32- → Na4[Fe(CN)5NO]SO3
used.
3 AgNO3 Soln. No Visible change

Ag+ + SO32− → [Ag(SO3 )]−



soluble
sulphito argentate

On adding more AgNO3 to the above complex White ppt.

[Ag(SO3)]-+Ag+ → Ag2SO3 Dissolves in NH3 and dil.


HNO3
Dissolves in excess of SO32-

Ag2SO3+SO32- → 2[Ag(SO3)]-(aq)

2  Ag ( SO3 )  

→ 2Ag ↓ +SO24− + SO2
Chem i str y | 30.11

S. No. Reagent Observation Remarks


4 Pb(NO3)2/ Pb(CH3COO)2 White
+2 O2
Pb + SO32− → PbSO3  → PbSO 4

5 Zn + H2SO 4 (aq)
Zn + SO32− + H+ → Zn+2 + H2S ↑ +H2O

Illustration 1: M2X.7H2O(A) has water and M2X (M is monovalent alkaline cation and X is divalent anion) in 1:1 ratio
by wt. (A). On reaction with dil. H2SO4 gives a gas that turns K2Cr2O7/H+ soln green. Identify (A) and explain.

Sol: Anion → SO32− M2SO3


2x+32+48 = 18x7 = 126
2x = 46 ⇒ x = 23
A ⇾ Na2SO3

Table 30.10: Confirmatory tests for sulphide and nitrite ion

S2- Reagent Observation Reaction


1 Dil H2SO4 Colorless, rotten egg smell.

S2-+H2SO4 → H2S+SO42-

Black
(i) Above gas + Pb NO3 ( )2 / Pb ( CH3COO)2 →PbS ↓

moist filter paper

Yellow
(ii) Cd+2 + H2S → CdS ↓

All sulphide
2OH−
(iii) Pb +2 
→ Pb OH ( )2 ↓ 2OH− One ppt except Al2S3 (aq.)
2−
PbS + 2H2O + 2OH − H2S
 → Pb
 ((OH)4  H2S+Al(OH)3

2 Oxidising agent Yellow/white ppt.


− H2S +2
(i) MnO 4  →S ↓ +Mn
2− H2S
(ii) Cr2O7  →S ↓ +Cr 3+
− H2S
(iii) I3  → 3I + S +

3 Sodium Nitroprusside Purple colour

S2− + Na2 Fe ( CN) NO  → Na4 Fe ( CN) NO 


 5   5 

4 Methylene blue test Methylene blue


3 0 . 1 2 | Qualitative Analysis

H3C CH3
N
(CH3)2N + N(CH3)2
S
3+ 2+ + +
+ Fe (soln.) + H2S + Fe +NH4 + H
N

NH2
N, N-Dimethyl phenyl diamine

NO2- Reagent Observation Reaction


1 Dil.H2SO4 Pale blue liquid (contains
HNO2+N2O3) gives reddish
Dil.H2SO4 + solid nitrites
brown NO2 fumes-Pungent
smell)
2 BaCl2 soln.

Ba2+ + NO3− → No Reaction

3 Conc.AgNO3 White ppt


+
Ag + NO2− → AgNO2
4 KI Soln (excess) Yellow brown vapours

2NO2− + 3I− + 4H+ ⇒ I3− + 2NO ↑ +H2O

5 Decolourised by a soln. of
MnO 4 − (acidic)
nitrite
NO2− + MnO 4 − → NO3− + Mn+2 (No gas)

6 Solid urea
NO2− + urea → N2 ↑ +CO2 ↑

acidified

7 Solid thiourea SCN– ions can be


confirmed by reaction
(NH2 )2 CS + NO
 2

→ H+ + N2 ↑ +SCN− + H2O
with FeCl3 + HCl
acidified

Blood red colour


Fe3+ + 3SCN− → ↓ Fe ( SCN)
3
(aq.)

13. BROWN RING TEST


When nitrite solution is added carefully along the sides of test tube to a saturated solution of FeSO4, acidified
with acetic acid or dil. H2SO4, a brown ring is formed due to formation of complex of variable composition best
represented as
Chem i str y | 30.13

Brown
Solution of Ring
+2
nitrite [Fe(H2O)5NO]
(NO-2)
Saturated
FeSO4 Solution
+
dil H2SO4

Reaction : NO-2 + H
+
HNO2
3HNO2 HNO3+ 2NO + H2O
2+ 2-
NO + Fe + + SO4 + 5H2O [Fe(H2O)5NO]SO4

Figure 30.7: Brown ring test for estimation of unsaturation

NO3− will also give similar test if NO − and NO3− both are present, then NO − can be selectively decomposed by
2 2
using
Sulphamic acid (NH2 − SO3H)
O

HO − S − NH2 + 2HNO2 → N2 ↑ + SO 42− + 2H+ + H2O

O
Sodium oxide (NaN3)
NO2− + N3− + H+ → N2 ↑ +SO 42− + 2H+ + H2O
Boil the mixture with NH4Cl
NO2− is decomposed to N2
NaNO2 + NH4 Cl → N2 ↑ +SO 42− + 2H+ + H2O

Table 30.11: Confirmatory test for acetate ion

CH3COO‒ Test Observation Remarks

1 Vinegar like smell All acetate are soluble


Dil.H2SO4
except.
CH3COO − + H+ → CH3COOH
CH3COOAg and
CH3COOHg
2 AgNO3 Soln: CH3COO − + Ag+ → CH3COOAg White ppt.

3 BaCl2 soln: BaCl2 + CH3COO– → No reaction


4 Fruity smell of an
C2H5OH + conc.H2SO 4
ester
( 2 − 3ml) (1ml)
+1gm acetate salt+heat

5 Neutral FeCl3 Red blood soln



6CH3COO + 3Fe +3
+ 2H2O → Fe3 (H2O )  ( CH3COO )
 2 6

Above redblood soln. ∆


4H2O Reddish brown
3 0 . 1 4 | Qualitative Analysis

Illustration 2: An aqueous solution of salt containing an anion Xn− gives the following reactions: It gives the purple
or violet coloration with sodium nitroprusside solution. It liberates a colorless, unpleasant smelling gas with dilute
H2SO4 which turns lead acetate paper black. Identify the anion ( Xn− ) and write the chemical reactions involved.

Sol: Xn− is S2- because


2− 4−
Fe ( CN) NO  + S2− → Fe ( CN) NOS  (purple or violet coloration)
 5   5 
S2− + H2SO 4 → H2S ↑ (colorless, unpleasant smelling) SO 4 + H2S + Pb ( CH3COO )2 → PbS ↓ (black) + 2CH3COOH
2−

Illustration 3: Sulphite on treatment with dil H2SO4 liberates a gas which:


Turns lead acetate paper black
Turns with blue flame
Smells like vinegar
Turns acidified K solution green

Sol: SO32− + H2SO 4 → SO2 + SO 42− + H2O


SO2 turns acidified K 2Cr2O7 solution green
K 2Cr2O7 + H2SO 4 + 3SO2 → Cr2 ( SO 4 ) (Green) + K 2SO 4 + H2O
3
Therefore , (iv) option is correct.

Illustration 4: A colorless pungent smelling gas (x) is obtained when a salt is reacted with dil. H2SO4. The gas (X)
responds to the following properties.
It turns lime water milky
It turns acidified potassium dichromate solution green
It gives white turbidity when H2S gas is passed through it aqueous solution.
Its aqueous solution in NaOH gives a white ppt. with barium chloride which dissolves in dil HCl liberating (X).
Identify (X) and write the chemical equations involved.

Sol: As gas X turns lime water milky it may be CO2 or SO2. But CO2 is colorless and odorless , so ‘X’ may be SO2 .
This is further , confirmed by the following reactions:

SO32− + H2SO 4 → SO 42− + SO2 + H2O

Ca(OH)2 + SO2 → CaSO3 (Milky) + H2O

K 2Cr2O7 + H2SO 4 + 3SO2 → K 2SO 4 + Cr2 ( SO 4 ) (Green) + H2O


3

SO2 + 2H2S → 3S ↓ (White) + 2H2O

SO2 + 2NaOH → Na2SO3 + H2O

Na2SO3 + BaCl2 → BaSO3 ↓ (White) + 2NaCl

BaSO3 + 2HCl → BaCl2 (Soluble) + SO2 + H2O


Chem i str y | 30.15

Conc: H2SO4 group


Table 30.12: Confirmatory test for chloride ion

[Cl‒] Reagent Observation Remarks

1 − H SO Dipped rod of Being a stronger oxidizing agent, Cl2 is


Cl 2 4 → HCl ↑ +HSO −
  4 NH3+HCl=white fumes not produced.
of NH4Cl
+ H3PO 4

HCl ↑ +H2PO 4 −

2 Yellowish green; turns Permanently bleaches dyes by oxidation


MnO ( OH) + 2H2SO 4 + 2Cl− →
2 iodide paper blue
Mn+2 + Cl2 ↑ +2SO24− + 3H2O

3 White ppt soluble in


Ag+ + Cl− → AgCl ↓
NH3
( AgNO3 )
Yellow (distinct from
AgCl + AsO33− arsenite → Ag3 ASO3 ↓
AgBr/AgI)

4 2+ White ppt-soluble in hot


Pb
 + 2Cl− → PbCl2
water) conc.HCl/KCl
Pb( CH3COO )
2

5 Chromyl chloride test Red vapours Cl2 & K2Cr2O7.have equal oxidizing
power. Br- & I-don’t give similar test since
K2Cr2O7+conc.H2SO4 + 4Cl- → CrO2Cl2
Cr2O72- oxidises Br-, KI- to Br2 & I2
Bright yellow ppt.
CrO24− + Pb+2 → PbCrO 4 ↓

Illustration 5: Comp X imparts imparts a golden yellow flame and shows following reaction
(1) Zn (powder) when boiled with conc. Ag. Soln of X dissolves and H2 gas is evolved .
(2) When Ag. Soln of X is added to Ag Soln of SnCl2, a white ppt is obtain which dissolves in excess of X
When dissolved in Zn which is amphoteric, H2O gas is evolved which signified Basic nature

NaOH + Zn → Na2 ZnO2


NaOH + ZnCl2 → Sn(OH)2 → Na2SnO2

Illustration 6: A gaseous mixture of X, Y, Z when passed into acidified K2Cr2O7, gas X was absorbed and soln turned
green. The remainder mixture was passed through lime which turned milky by absorbing gas Y. The residue gas
when passed through alc. pyrogallol solution turned black.

Sol: K2Cr2O7 and lime water to differentiate between SO2 and CO2 .pyrogallol is used to absorb O2 gas Q.

Illustration 7: Colourless salt (A) gives apple green flame with conc. HCl,(A) or reaction with dil. H2SO4 gives brown
fumes (D) turing KI starch paper blue
(A) + CH3COOH + K 2CrO 4 →Yellow
yellow ppt
ppt (A) + H2SO 4 → White ppt (C) +(D)

(D) + CH3NH2 → CH3OH + H2O + gas (E) (E) + Mg → (F)
3 0 . 1 6 | Qualitative Analysis

(F)+ H2O → NH2


Identify A to F

Sol: Apple green → Ba2+


Brown fumes with dil H2SO 4 → NO2−

Ba(NO2 )2 + CH3COOH + K 2CrO 4 → BaCrO 4 (yellow ppt)

Ba(NO2 )2 + H2SO 4 → BaSO 4 + HNO2



whiteppt

HNO2 + CH3NH2 → CH3OH + H2O + N2 ↑


N2 + Mg → Mg3N2
Mg3N2 + H2O → NH3

Table 30.13: Confirmatory test for bromide ion


Br Reagent Observation Remarks
1 Reddish brown Br2 vapours and pungent
i) 2Br − + SO24−+ + 4H+ → Br2 + SO2 + 2H2O
smell
ii) Br − + H3PO 4 → HBr + H2PO −4

2 Pinch of MnO2 added to above test-tube. Intensity of coloured fumes increases. Fluorescence Cl2,
− 2+ Irritating pungent smell. Br2, I2,
Br + MnO2 → Br2 + Mn + H2O
Bleaches litmus paper. Colourless red violet
Filter paper dipped in fluorescein
dye(yellow) turns red.
Turns starch iodide paper blue.
3 Ag+ + Br − → AgBr ↓ Pale yellow ppt.
(AgNO3 ) Dissolves in conc. NH3, KCN hypo soln.

4 Pb+2+2Br-→PbBr2 (Pb-acetate) White ppt.Soluble in conc.HBr or conc, KBr

5 8HNO3+6Br- → 3Br2↑+NO↑+6NO3-+4H2O
6 K 2Cr2O7 + H2SO 4 + Br − → Br2 + Cr +3

7 Layer test Br2 vapours colours the soln orange red.

With Cl2 water.

i)Cl2 water + Br − → Br2 ↑


 
Dropwise

ii) Above soln. +CHCl3+shake Upper layer: colourless


Lower layer: Red brown
Greater solubility of Br2 in organic solvent
iii) Excess of Cl2water Yellow soln.
− −
Br + Cl2 → Br2 + 2Cl
Br2 + Cl2 → 2BrCl
Chem i str y | 30.17

Table 30.14: Confirmatory test for Iodide ion

I‒ Reagent Observation Remarks


1 Violet coloured vapours All are soluble except
HI + HSO −4 I2 ↑ +SO2 ↑ Ag+,Hg+2, Cu+, Pb+2, Pb+2 =
↑ ↑ soluble in hot water

I + H2SO 4
↓ ↓
I2 ↑ +S ↓ I2 ↑ +H2S ↑

– –
Feasible only with I , not with Br

HI is a good reducing agent

2 Intensity of violet vapours


MnO2 + H2SO 4 + I− → I2 + Mn+2
increase
3 Yellow.
Ag+ + I − → AgI ↓
 Soluble in KCN,
AgNO3
Partially soluble in conc. NH3.

4 +2 − Yellow
Pb
 + 2I → PbI2 ↓
Pb −acetate

5
I− + K 2Cr2O7 + H2SO 4 → I2 + Cr 3+

6
KNO2 + I− + H2SO 4 → I3− + NO ↑ +H2O

7 Scarlet Red
HgCl2 + 2I − → HgI2 ↓ +2Cl− K 2 HgI 4  + KOH
  
2−
HgI2 + 2I − → HgI 4  Nessler ' sreagent

Used for detection of NH4+


and NH3
8 Iodometric Titrations

i) 2Cu+2 + 2I − → 2Cu+ + I2

ii) I2 + 2S2O32− → 2I − + S 4 O62−

9 2FeCl3 + 3I− → 2Fe+2 + I3−

10 Layer Test Brown soln.



Cl2 water +I → I2

Above solution. + CHCl3 Upper layer: Colourless lower


layer: violet layer

Excess of Cl2 water Colourless iodic acid formed


− −
i) Cl2 + I → Cl + I2

I 2 + I − → I3 −
ii) I − + 8Cl + 9H O →
3 2 2

3IO3− + 16Cl− + 18H+


3 0 . 1 8 | Qualitative Analysis

Remark for HgCl2


K2[HgI4] + KOH as alkaline solution of K2HgI4 . Is called Nessler’s reagent used for detection of NH4+or NH3

Table 30.15: Confirmatory test for nitrate ion


NO3 Reagent Observation Remarks
1 All nitrate are soluble
i) NO3− + H2SO 4 → HNO3

+HSO 4 −

Reddish Brown
1
ii) 2HNO → 2NO ↑ + O ↑ H O
3 2 2 2
2
2 Cu turnings+Above soln.→ Intensity of above colour increases
+2 2−
Cu + 2NO ↑ +4SO 4 + 4H2O

3 Diphenyl /blue ring test Blue complex

NO3 + ( C6H5 ) NH ↓

2
Ph2 − N − N − Ph2

4 Brown Ring Test

Brown ring Test


3 – 4 ml conc. H2SO4 + 2ml of NO-3 solution +mix+cool the mixture under a stream of cold water+ saturated
solution of FeSO4 from the side of the test tube to form a layer on the top of the liquid. A brown ring will formed
at the junction of two liquids. This brown ring complex is unstable and decomposes on shaking or on heating into
NO and a yellow colour solution of Fe+3 ion.
Reduction of NO3− or NO2− is alkaline medium into NH+4 or NH3 can be done by Zn, Al , Devarda’s Alloy(50% Cu,
45% Al, 5% Zn)
NO3− + 4Zn + 7OH− + 6H2O → NH3 ↑ +4[Zn(OH)4 ]−

3NO3− + 8Al + 5OH− + 18H2O → 3NH3 ↑ +8[Zn(OH)4 ]−

Action of heat on nitrate



Alkali metal nitrate  → Metal nitrate + O2
∆ 1
NaNO3  → NaNO2 + O2
2
(E xcept Li)

Metal nitrate  → Metal oxide + NO2 + O2
(E xcept Na+ ,K + ,Rb+ , Cs+ )

∆ 1
3LiNO3  → Li2O + 2NO2 + O2
2

2Cu(NO3 )2  → 2CuO + 4NO2 + O2

2Pb(NO3 )2  → 2PbO + 4NO2 + O2

2Hg(NO3 )2  → 2Hg + 4NO2 + O2

2AgNO3  → 2Ag + 4NO2 + O2
Chem i str y | 30.19


NH4NO3  → N2O + 2H2O

NH4NO2  → N2 + 2H2O

N2H5NO2  → N2H + 2H2O

Nitrogen oxides are supporter of combustion as these fumes in to N2 and O2.

N2O → Supporter of combustion (better than air)


2N2O → 2N2 + O2
 
66.67% 33.33%

Air → O2

20%

Table 30.16: Confirmatory test for sulphate ion


SO42 Reagent Observation Remarks

1 +2 2− White ppt. All sulphates are


Ba
 + SO 4 → BaSO 4 ↓ soluble except
BaCl2 Insoluble in dil.HCl
Ba+2,Pb+2,Sn+2,Ag+,Ca+2
Soluble in hot, conc. H2SO4 which are partially soluble.
2 2Ag+ 2− White ppt.
 + SO 4 → Ag2SO 4 ↓
AgNO3

3 Pb +2 2− White ppt
 + SO 4 → PbSO 4 ↓
Pb(NO3 )
2

4 Yellow ppt.
3Hg+2 + SO 42− + 2H2O ↓ 2HgO.HgSO 4

Hg(NO3 )
2

5 Match Stick Test White ppt


+2 2−
Ba + SO 4 → BaSO 4 ↓

Above ppt + Na2CO3 Purple coloration


(s)


paste

Apply paste on wooden piece of match stick+Burn in


reducing flame+Dip in sodium nitroprusside soln.

i) Paste → BaCO3 + Na2SO 4

ii) Na2SO 4 + 4C → Na2S + 4CO ↑

 ( )5
iii) Na2S + Na2 Fe CN NO 
 
S 2 −
  
Na4 Fe( CN) NOS 
 5 
3 0 . 2 0 | Qualitative Analysis

Table 30.17: Confirmatory test for phosphate ion


PO43 Reagent Observation Remarks
1 White ppt
 + PO 4 → Ba3 (PO 4 )2 ↓
+2 3−
Ba
BaCl2

2 + 3− White ppt
3Ag
 + PO 4 → Ag3PO 4 ↓
AgNO3 Soluble in NH3, KCN, hypo, etc.

3 Magnesia

i) PO 43− + Mg+2
 + NH4
+
White ppt.
3 2 Mg(NO )
 
( ) +
Mg NH PO ↓ +Mg →Mg PO ↓
4 4 3 4

White ppt
ii) AsO 43− + Mg+2 + NH4 +
 
Brownish red ppt
( 4 ) 4
+
Mg NH AsO ↓ +Mg →Mg AsO ↓
3 4

4 Canary yellow ppt.


PO 43− + conc.HNO3+ammonium molybdate (excess)
(NH4 )2 MoO4 ↓ (NH4 )3 PO4 .12MoO3 ↓
+H2O + NH4NO3 + NaNO3

Table 30.18: Confirmatory test for borate ion


BO33 Reagent Observation Remarks

0.2g salt+Conc. H2SO4(1ml)+C2H5OH(4- Green edged flame


5ml)+.Ignite on a Bunsen flame.
3H SO
2 4 → 2H BO
2Na3BO3 
−3Na SO 3 3
2 4
3C2H5OH
2H3BO3 
−3H2O
→(C2H5 )3 BO3

Group O

1 - Ammoniacal smell
NH4 + + dil.NaOH
- White fumes (NH4Cl) with HCl

- HgNO3 on filter paper becomes black (Hg)
NH3 ↑ +H2O + Na+
- Red litmus turns blue
- (MnCl2+H2O2)on filter paper becomes brown
black (MnO2[MnO(OH)2])
2
NH4 + + K 2 HgI 4 

OH
+ 
Nessler 'sreagent

HgO.Hg NH I
2 ( )
Chem i str y | 30.21

3 Yellow ppt
3NH4 + + Na3 Co (NO2 ) 
 6

(NH4 )3 Co (NO2 )6  ↓ +3Na+
4 Yellow ppt
2NH4 + + Na2 PtCl6  NH4 + salts
are insoluble

(NH4 )2 PtCl6  ↓ +2Na+

Illustration 8: A compound (A) of S, Cl and O has vapour density of 67.5 (approx). It reacts with water to form two
acids and reacts with KOH to form two salts (B) and (C) while (B) gives white ppt. with AgNO3 solution and (C) gives
white ppt. with BaCl2 solution. Identify (A), (B) & (C).

Sol: As mixture gives white ppt. with BaCl2 and AgNO3, it should be SO42- and Cl- ions. As SO2Cl2 when dissolves in
water gives, a mixture of H2SO4 & HCl which then reacts with KOH to form KCl and K2SO4. Therefore (A) is SO2Cl2
and (B) & (C) are K2SO4 and KCl respectively.
Vapour density of SO2Cl2 = molecular weight/2.
Vapour density of SO2Cl2 = 135/2 = 67.2.

Illustration 9: Na2S2O3 + I2 → NaI + ………[X], [X] is:


(A) Na2S4O6 (B) Na2SO4 (C) Na2S (D) Na3ISO4

Sol: 2Na2S2O3 + I2 → 2NaI + Na2S4O6


Therefore A is correct option.

Illustration 10: Column I and II contains four entries each. Entries of column I are to be matched with some entries
of column II. Each entry of column I may have the matching with one or more than one entries of column II.

Column I Column II
(A) Colourless gas evolved on addition of dil. H2SO4 (p) Cl-

(B) White ppt. on addition of AgNO3 (q) S2-


(C) Ppt. with solution containing Pb+2 ions (r) NO2−
(D) Its acidified salt solution decolourises pink KMnO4 solution. 2−
(s) SO3

Sol: A → p, q, s; B → p, r, s; C → p, q, s; D → p, q, r, s

− − 2− +
(A) Cl + H2SO 4 → HCl ↑ (C olourless) + HSO 4 ; S + 2H → H2S ↑ (C olourless)

NO2− + 2H+ → NO2 ↑ (Redish brown) + H2O ; SO32− + 2H+ → SO2 ↑ (C olourless) + H2O

(B) Ag+ + Cl− → AgCl ↓ (White) ; Ag+ + S2− → Ag S ↓ (Black)


2

Ag+ + NO2− → Ag NO2 ↓ (White) ; 2Ag+ + SO32− → Ag2SO3 ↓ (White)

(C) Pb2+ + 2Cl− → PbCl ↓ (White) ; Pb2+ + S2− → PbS ↓ (Black)


2

Pb2+ + NO2− → PbNO2 (Soluble) ; Pb2+ + SO32− → PbSO3 ↓ (White)



3 0 . 2 2 | Qualitative Analysis

(D) 2MnO − + 16HCl → 5Cl + 2Mn2+ + 6Cl− + 8H O


4 2 2

2MnO −4 + 5H2S + 6H+ → Mn2+ + 5S ↓ +8H2O

2MnO −4 + 5NO2− + 6H+ → Mn2+ + 5NO3− + 3H2O

2MnO −4 + 5SO2− + 2H2O → 2Mn2+ + 5SO24− + 4H+

Classification of Cation Radicals: For the purpose of systematic qualitative analysis, the cations are classified into
various groups and the classification is based on whether the cation with the given group reagent and forms a ppt or not

Table 30.19: Classification of cation radicals

Group Cation Reagent Observation


Zero NH4 +
NaOH or Ca(OH)2, heat if Ammonium gas is evolved.
required AgCl 

I. Ag+ Hg2+2 Pb+2 dil. HCl Hg2Cl2  White
PbCl2 

II (A) Cu+2, Hg+2, Pb+2, Bi+3, Cd+2 H2S gas in dil. HCl insoluble
Cu+2  CuS
in YAS (Yellow Ammonium 
Sulphide) (NH4)2Sn Hg+2  HgS black

Pb+2  PbS

Bi+3  Bi2S3

CdS → yellow CdS


II (B) SnS → Brown ppt.
Sn+2 ,Sn+4 , As+3 , As+5 ,Sb+3 ,Sb+5
SnS2, As2S3 → Yellow
Sb2S3 → Orange
As2S5 → Yellow solution
Sb2S5 → Orange solution
III Fe , Al , Cr
+3 +3 +3
NH4OH in presence of NH4Cl Fe(OH)3 → Reddish brown
Al(OH)3 → Gelatinous white
Cr(OH)3 → Dirty/ grey green
IV Mn+2, Co+2, Zn+2, Ni+2 H2S in presence of NH4OH and NiS, CoS → Black
NH4Cl ZnS → Dirty black
MnS → Buff colored (light pink)
(V) Ba , Sr , Ca
+2 +2 +2
(NH4)2CO3 in presence of NH4OH
BaCO3 
and NH4Cl Na2HPO4 in presence 
of NH4OH and NH4Cl SrCO3  White ppt
CaCO3 

Mg(NH4)PO4 ↓ (White)

Preparation of Original Solution (O.S): Original solution is used for the analysis if basic radicals except NH4+. It is
prepared by dissolving given salt or mixture in a suitable solvent as follows:
 1. H2O  2. Dil. HCl 3. Conc. HCl

Salt or Mixt.+ H2O → Soluble (then H2O is suitable solvent)
If given salt or mixture is insoluble in H2O,then it is dissolved in dil. HCl.

Salt or Mixt.+dil. HCl  → Soluble (then dil HCl is taken as solvent)
If given salt or mixture is insoluble in dilute HCl, then it is dissolved in conc. HCl.

Salt or Mixt.+conc. HCl  → Soluble
Chem i str y | 30.23

In this way after selecting suitable solvent, given salt or mixture is dissolved in small quantity in the solvent and
filtered. Obtained filtered is called as original solution (O.S.) and that is used for the detection of basic radicals
except NH4+.

Remarks:
 1. Pb+2 is placed in both group I and group II because PbCl2 is soluble is not water and all of Pb+2 is not separated
by addition of HCl.
 2. In group II, only those sulphides are ppt. which have very low values of Ksp for this. H2S gas is added in acidic
medium. Dil. HNO3 cant be added to prevent oxidation of H2S to sulphur. (It is yellow and may be confused
with CdS). Dil. H2SO4 cant be added to prevent the formation of ppt. of sulphates.
 3. Before proceeding with test of group III, the solution is boiled to remove dissolved H2S gas to prevent ppt.
of sulphides in group III. Then dil. HNO3 is added as we don’t have to perform tests with Fe+2.HNO3 oxidises
Fe+2 to Fe+3
 4. NH4OH is added in presence of NH4Cl to decrease the degree of dissociation of NH4OH by common ion effect.
So, only those salts are ppt. which have low values of Ksp. (NH4)2SO4 and (NH4)2CO3 cannot be used in place
of NH4Cl to prevent the ppt of carbonates and sulphates.
 5. For the test of group IV, H2S is added in presence of NH4OH to increase the degree of dissociation of H2S
resulting in increase of S2- conc. So that sulphides of higher values of Ksp can be separated.
 6. We cant use NaCO3 in test of group V as Na2CO3 is highly soluble resulting in high conc. of CO32− which may
lead to ppt of group VI, Mg+2 ions.

Table 30.20: Classification of cation radicals

Group 0:
Reagent Observation Remarks
1. Dil.NaOH NH+4 +dil. NaOH → NH3 ↑ +H2O+Na Ammoniacal smell forms white fumes
in presence of HCl. NH3(g)+HCl
→ NH4Cl(g) (dense White fumes)
Filter paper dipped in HgNO3 (aq.)
becomes black
3→ NH
HgNO3 
Hg + Hg(NH2 )NO3 ↓
Black
white

Filter paper dipped in red litmus


become blue
Filter paper dipped in MnCl2+H2O2
(aq.) Becomes brown black due to
formation of MnO2 [MnO(OH)2]
2. Test with K2HgI4 Yellow / brown
NH+4 + K 2 [HgI 4 ] + OH− → Hg(NH2 )I + 2K ⊕
  
Nesslare' s reagent basic mercuric
amido iodide

3. Test with Yellow ppt.


3NH+4 + Na3 [Co(NO2 )6 ] → (NH4 )3 [Co(NO2 )6 ]+ 3Na+
Na3[Co(NO2)6] Yellow

4. Na2[PtCl6] or H2 Yellow ppt


[PtCl6] {sodiumchloro 2NH+4 +Na2[PtCl6] → (NH4)2[PtCl6] ↓ +2Na+
Generally, NH+4 salts are insoluble.
palatinate} or
{chloropalatinate acid}
3 0 . 2 4 | Qualitative Analysis

Group I

Flowchart 30.3: Systematic path for the analysis of group I radicals


Chem i str y | 30.25

Table 30.21: Analysis of group I and group II radical

Pb2+ Test/Reagents Observation

Dilute HCl solution: White ppt is formed in cold solution.

Pb2+ +HCI → PbCI2 ↓ (white) + 2H+ White ppt is soluble in hot water. White ppt is also
− 2− soluble in concentrated HCl or concentrated KCI.
PbCl2 ↓ +2Cl → PbCl4 

Sodium hydroxide solution White ppt is formed which is soluble in excess of


Pb 2+
+ 2OH → Pb ( OH) ↓;
− the reagent.
2
2− Black/brownish black
Pb ( OH) ↓ +2OH− → Pb ( OH) 
2  4
2−
Pb ( OH)  + H2O2 → PbO2 ↓ +2H2O + 2OH−
 4
2−
Pb ( OH)  + S2O82− → PbO2 ↓ +2H2O + 2SO24−
 4

Potassium iodide solution A yellow ppt is formed which is soluble in excess


more conc.(6M) soln of the reagent.
PbCl2 + 2Kl → Pbl2 ↓ +2KCl;
Yellow ppt of Pbl2 is moderately soluble in boiling
Pbl2 + Kl → K 2 Pbl4  ↓
water to give a colourless solution.

Yellow ppt reappears on dilution with water. Yellow


ppt of Pbl2 does not dissolve in excess of dilute
solution of Kl.

Potassium chromate solution (in neutral, acetic or A yellow ppt is formed.


ammonia solution)
Yellow ppt is soluble in sodium hydroxide and
PbCl2 + K 2CrO 4 → PbCrO 4 ↓ 2KCl HNO3 (nitric acid).Both reversible reactions on
++  22 −−22 buffering the solution with ammonia or acetic acid
2PbCrO
2PbCrO
2PbCrO4444 +
2PbCrO + 2H
++ 2H  2Pb
2H++ 
2H 
 2Pb
2Pb22 +
2Pb + Cr
++ Cr2222O
Cr
Cr O
O77−77−22 +
O +H
++ H2222O
H
H O
O
O respectively, PbCrO4 reprecipitates.
22−−
Pb( OH
OH)  +
22−−
 Pb
22−−
 (( ))4444
PbCrO
PbCrO
PbCrO4444 +
PbCrO + 4OH
++ 4OH
4OH 
4OH 
 Pb
Pb OH
OH + CrO
++ CrO
CrO244244−−
CrO

Ammonia solution With ammonia solution, Pb2+ gives a white ppt of


lead hydroxide
Pb2+ + 2NH4 OH → Pb ( OH) ↓ +2NH+4
2

Dilute H2SO4: White ppt is formed which is soluble in more conc.


ammonium acetate (6M) solution or ammonium
PbCl2 + H2SO 4 → PbSO 4 ↓ +2HCl tartrate in the presence of ammonia.
2−
PbSO 4 ↓ +4CH3COO − → Pb ( CH3COO )  + SO24− Hot,conc.H2SO4 dissolves the ppt due to the
 4
2− formation of PbHSO4.
PbSO 4 ↓ +2C 4H4 O62− → Pb ( C 4H4 O6 )  + SO24−
 2

PbSO 4 ↓ +H2SO 4 → Pb2 + 2HSO −4


3 0 . 2 6 | Qualitative Analysis

Hg22+ Test/Reagents Observation

Dilute HCl solution White ppt is formed in cold solution.

Hg22+ + 2HCl → Hg2Cl2 ↓ ( white ) + 2H+

Ammonia solution A mixture of mercury metal (black ppt) and basic


2Hg2Cl2 + 4NH4 OH → HgO.Hg (NH2 ) Cl ↓ +Hg ↓ mercury (II) amido chloride (white ppt) is formed.

+3NH4 Cl + 3H2O

Dissolution of white ppt. Hg2Cl2 in aquaregia (a) Stannous chloride test: White ppt is formed
which finally turns to black.
3Hg2Cl2 + 2HNO3 + 6HCl → 6HgCl + 2NO + 4H2O
(b) Potassium iodide test: Scarlet/red ppt is
2HgCl2 + SnCl2 → Hg2Cl2 ↓ +SnCl4 ; formed which is soluble in excess of the reagent.
Hg2Cl2 + SnCl2 → 2Hg ↓ (black ) + 2SnCl4
(c) Copper chips test: Shining grey deposition of
HgCl2 + KI → HgI2 ↓ +2KCl; mercury on copper chips is formed.
HgI2 + KI(excess) → K 2 HgI 4  ( soluble )
HgCl2 + Cu → Hg ↓ +CuCl2

Potassium iodide solution: A green ppt is formed. Green ppt in excess of reagent undergoes
2+ − disproportionation reaction and a soluble
Hg2 + 2l → Hg2l2 ↓
2−
Hgl4  ions and black mercury are formed.
Hg2l2 + 2l− → Hgl4  + Hg ↓ ( finely divided)
2−

Boiling the mercury (l) iodide ppt with water,


Hg2l2 → Hgl2 ↓ +Hg ↓
disproportionation takes place and a mixture of red
mercury (ll) iodide ppt and black mercury is formed.

Potassium chromate solution A red crystalline ppt is formed which turns black
when solution of sodium hydroxide is added.
Hg22+ + CrO 42− → Hg2CrO 4 ↓;
Hg2CrO 4 + 2OH− → Hg2O ↓ +CrO 42− + H2O

Potassium cyanide solution A black ppt of mercury is obtained

Hg2 2+
+ 2CN → Hg ↓ +Hg ( CN) ( soluble ) .

2

(Ag+) Dilute hydrochloric acid/soluble chlorides White ppt

Ag+ + HCl → AgCl ↓ +H+ Soluble in conc.HCl


− On dilution with water, the equilibrium shifts back
AgCl + Cl− →  AgCl2  to the left and the ppt reappears. Dil.ammonia
+ solution dissolves the ppt forming a soluble
AgCl + 2NH3 →  Ag (NH3 )  + Cl−
 2 complex.
 Ag (NH )  Cl + 2HNO → AgCl ↓ +2NH NO Dil.HNO3 or HCl neutralizes the excess ammonia
 3 2 3 4 3
and the ppt reappears because the equilibrium is
shifted backwards.

Potassium iodide solution A bright yellow ppt. is formed which is insoluble in


+ − dilute ammonia but partially soluble in concentrated
Ag + l → Agl ↓
ammonia. The yellow ppt. is soluble in KCN and in

Agl + 2CN− →  Ag ( CN)  + l− ; Na2S2O3.
 2
3−
Agl + 2S2O32 →  Ag ( S2O3 )  + l−
 2
Chem i str y | 30.27

Potassium chromate solution Red ppt. is formed which is soluble in dilute HNO3
++ 2− and in ammonia solution.
2Ag
2Ag +
CrO44422−−
++CrO → Ag
→ Ag222CrO
CrO444 ↓↓
2−
2Ag222CrO
2Ag CrO444 ++2H + 2H+++
4Ag++ ++Cr
 4Ag
2H 
 Cr222O
+
O77722−− ++H
H222O
O
++
Ag (NH
NH )  ++CrO
 ( 333)222
+
2Ag222CrO
2Ag CrO444 ++ 4NH
4NH333 →
→ 22Ag CrO444222−−−

Disodium hydrogen phosphate solution In neutral solution a yellow ppt. is formed with the
reagent. The yellow ppt. is soluble in nitric acid and
3Ag+ + HPO 42− → Ag3PO 4 ↓ +H+
ammonia solution.

Hydrazine sulphate (saturated) With diammineargentate (l) reagent forms finely


+ divided silver which adheres to the cleaned glass
4  Ag (NH3 )  + H2N − NH2 .H2SO 4 → walls of the test tube forming an attractive mirror.
 2

4Ag ↓ +N2 ↑ +6NH4 + + 2NH3 + SO 42−

Ammonia solution

2Ag+ + 2NH3 + H2O → Ag2O ↓ +2NH4 + Brown ppt. is formed


+
Ag2O ↓ +4NH3 + H2O → 2  Ag (NH3 )  + 2OH− Ppt. dissolves in ammonia.
 2

Hg2+ Test/Reagents Observation

Precipitation with H2S in acidic medium Black ppt. is formed.


+
H
Hg2+ + H2S → HgS ↓ +2H+ Insoluble in water, hot dilHNO3, alkali hydroxides, or
2− colourless ammonium sulphide.
HgS + S2− → HgS2 
Na2S(2M) dissolves the ppt. forming soluble
3HgS + 6HCl + 2HNO3 → complex.
3HgCl2 + 3S ↓ +2NO ↑ +4H2O Aqua regia dissolves the ppt.
2− +
2HNO3 + S → SO 4 + 2H + 2NO ↑ HgCl2 is undissociated. On heating, white ppt. of
sulphur dissolves forming H2SO4.

Stannous chloride solution When added in moderate amounts silky white ppt.
is formed.
2HgCl2 + SnCl2 → SnCl4 + Hg2Cl2 ↓
If more reagent is added, Hg (I) chloride is reduced
HgCl2 + SnCl2 → SnCl4 + 2Hg2 ↓
to black ppt. of mercury.

Potassium iodide solution On slow addition red ppt. is formed. Ppt. dissolves
2+ − in excess of Kl forming colourless soluble complex.
Hg + 2I → HgI2 ↓
KCN does not have any effect.
2−
HgI2 + 2I − → HgI 4 

Copper chips, sheet or coin A black ppt. of mercury is formed.


2+ 2+
Hg + Cu → Hg ↓ +Cu

Sodium hydroxide solution When added in small quantity brownish-red ppt. of


varying composition is formed and in stoichiometric
Hg2+ + 2OH− → HgO ↓ +H2O
amounts ppt. turns to yellow when Hg (II) oxide
is formed. Ppt. is insoluble in excess reagent but
dissolves readily in acids and this can be used to
differentiate Hg (I) from Hg (II).
3 0 . 2 8 | Qualitative Analysis

Ammonia solution White ppt. of mixed composition (Mercury (II)


2Hg 2+ −
+ NO3 + 4NH3 + H2O → oxide+Mercury (II) amido nitrate) is formed with
metal nitrate.
HgO.Hg (NH2 ) NO3 ↓ +3NH4 +

Cobalt (II) thiocyanate test When reagent is added to an aqueous solution of


2 Hg2+ ions and the walls of the test tube is stirred with
 (
Hg 2+
+ CO 2+
+ 4SCN → CO − +2 Hg SCN)  ↓
4 a glass rod, deep-blue crystalline ppt. is formed. In
place of Cobalt (II) thiocyanate, Co(CH3COO) and
NH4SCN can be added to the aqueous solution of
Hg2+ ions.

Cu2+ Test/Reagents Observation

Precipitation with H2S in acidic medium Black ppt. is formed.


H+
Cu2+ + H2S 
→ CuS ↓ +2H+ Ppt. is insoluble in boiling dilute (M) H2SO4
(distinction from cadmium), in NaOH,
3CuS + 8HNO3 →
Na2S and (NH4 ) S. Ppt. dissolves in hot conc.
3Cu (NO3 ) (blue ) + 2NO + 4H2O + 3S 2
2 HNO3
3−
2CuS ↓ +8CN− → 2 Cu ( CN)  + S 22 − When boiled for longer, S is oxidized to H2SO4 and
 4
a clear solution of Cu(NO3)2 is obtained.

KCN dissolves the ppt. forming a clear solution.


(disulphide ion)

Ammonia Solution When added sparingly a blue ppt. of basic salt


2
2Cu + SO 4 2−
+ 2NH3 + 2H2O → (basic copper sulphate) is formed with CuSO4.

Cu ( OH) .CuSO 4 ↓ +2NH4 + It is soluble in excess of reagent forming a deep


2
blue colouration.
Cu ( OH) .CuSO 4 + 8NH3 →
2
2+
2 Cu (NH3 )  + SO 42− + 2OH−
 4

Sodium hydroxide in cold solution A blue ppt. is formed.

Cu2 + 2 OH− → Cu ( OH) ↓


2

Cu ( OH) → CuO ↓ (black ) + H2O


Heat
2

Potassium iodide It gives a white ppt. of Cu (l) iodide but the solution
2+ − − is intensely brown because of the formation of tri-
2Cu + 5l → Cu2l2 ↓ +l3
iodide ions (or iodine). The soln becomes colourless
l3− + 2S2O32− → 3l− + S 4 O62− and a white ppt. is visible when excess of sodium
thiosulphate solution is added. These reactions are
used in quantitative analysis for the iodometric
determination of copper.

Potassium ferrocyanide (Potassium hexacyanidoferrate


(II) solution Cu2+ ions gives brown/chocolate brown ppt.

2Cu2+ + K 4Fe ( CN) → Cu2 Fe ( CN)  ↓ +4K +


6  6

3−
2 Fe ( CN)  + 3Cu2+ → Cu3 Fe ( CN)  ↓ Green
 6  6 2
Chem i str y | 30.29

Potassium cyanide When added sparingly forms first a yellow ppt.


which decomposes into CuCN and cyanogen.
Cu 2+
+ 2CN → Cu ( CN) ↓

2
Excess reagent dissolves the ppt. forming a
2Cu ( CN) → 2CuCN ↓ ( white ) +
2 colourless soluble complex.
( CN)2 ↑ (highlypoisonous ) Complex is so stable that H2S cannot ppt. Cu(l)
3−
CuCN + 3CN− → Cu ( CN)  sulphide (distinction from cadmium).
 4

Potassium thiocyanate solution


+2
The Cu ions solution initially gives a black ppt.
Cu+2 + 2SCN− → Cu ( SCN) ↓ which then slowly decomposes to give white ppt. of
2
2Cu ( SCN) → 2CuSCN ↓ + ( SCN) ↑ Cu (l) thiocyanate.
2 2
Cu (ll) thiocyanate can be immediately converted
2Cu ( SCN) + SO2 + 2H2O →
2 into Cu (l) thiocyanate by adding a suitable reducing
2CuSCN + 2SCN− + SO 42− + 4H+ agent like saturated solution of SO2.

Bi3+ Test/Reagents Observation

Precipitation with H2S in acidic medium Black ppt. is formed which is insoluble in cold dilute
HNO3 and yellow ammonium sulphide.
H+
→ Bi2S3 ↓ (black ) + 6H+
2Bi3+ + 3H2S 
Bi2S3 + 8HNO3 → 2Bi (NO3 ) + 2NO + 3S + 4H2O
3

Bi2S3 + 6HCl(boiling,conc.) → 2Bi3 + 6Cl− + 3H2S

Sodium hydroxide White ppt. is formed with the reagent,slightly


soluble in excess reagent in cold solution but
Bi3+ + 3OH− → Bi ( OH) ↓
3 soluble in acids. Ppt. on boiling loses water and
Bi ( OH) + 3H+ → Bi3+ + H2O turns yellowish white which is oxidised to BiO3– by
3
H2O2.
Bi ( OH) → BiO.OH ↓ +H2O
3

BiO.OH + H2O2 → BiO3− + H+ + H2O

Ammonia solution White basic salt of variable composition is formed.

Bi3+ + NO3− + 2NH3 + 2H2O → Bi ( OH) NO3 ↓ +2NH4 +


2

Alkaline sodium stannite (Sodium A black ppt. of metallic bismuth is obtained.


tetrahydroxidostannate (ll))
The reagent must be freshly prepared and test must
Bi3+
+ 3OH → Bi ( OH) ↓
− be carried out in cold solution.
3
2− 2−
2Bi ( OH) + Sn ( OH)  → 2Bi ↓ +3 Sn ( OH) 
3  4  6

Dilution with water Solution of bismuth salts gives white ppt. when
water is added in larger quantity. Soluble in
Bi3+ + NO3− + H2O → BiO (NO3 ) ↓ +2H+ mineral acids (dilute) but insoluble in tartaric acid
Bi3+ + Cl− + H2O → BiOCl
 ↓ + 2H+ (distinction from antimony) and in alkali hydroxide
bismuthoxychloride (distinction from tin).
or bismuthylchloriode
3 0 . 3 0 | Qualitative Analysis

Potassium iodide When the reagent is added dropwise to a solution


3+
Bi3+ −
+ 3I → BiI3 ↓ containing Bi ions, a black ppt. is formed. The
− ppt. dissolves in excess Kl forming orange coloured
BiI3 + I − → BiI 4  soluble complex. On dilution the reaction is
BiI3 ↓ +H2O → BiOl ↓ +2H+ + 2l− reversed and black BiI3 turns orange.

(Cd2+) Test/Reagents Observation

Precipitation with H2S in acidic medium Yellow ppt. is formed which dissolves in hot dil.
HNO3.
H+
Cd2+ + H2S 
→ CdS ↓ +2H+
Ppt. does not dissolve in KCN.
CdS + 8HNO3 → 3Cd (NO3 ) + 4H2O + 2NO + 3S
2

Ammonia solution (Dropwise addition) Ammonium hydroxide first gives white ppt. of
Cd(OH)2 which gets dissolve in excess of reagent
Cd2+ + 2NH3 + 2H2O → Cd ( OH) ↓ +2NH4 +
2 forming a soluble complex.
2+
Cd ( OH) + 4NH3 → Cd (NH3 )  + 2OH−
2  4

Potassium cyanide White ppt. of Cd(CN)2 is formed which in excess of


reagent dissolves forming a soluble complex.
Cd2+ + 2CN− → Cd ( CN) ↓
2
2− The colourless soluble complex is unstable,therefore,
Cd ( CN) + 2CN → Cd ( CN) 

reacts with H2S gas forming a yellow ppt. of CdS.
2  4
2−
Cd ( CN)  + H2S → CdS ↓ +2H+ + 4CN− Kl forms no ppt. (distinction from Copper)
 4

Sodium hydroxide White ppt. is obtained which insoluble in excess of


Cd 2+
( aq) + 2NaOH( aq) → Cd ( OH)2 ↓ +2Na + NaOH

Illustration 11: When NaOH solution is mixed with aqueous solution of a salt ‘A’, and warmed, a black ppt. is formed.
Black ppt. is filtered and dissolved in concentrated HNO3 by boiling. The resulting solution gives a chocolate brown
coloured ppt. with potassium ferrocyanide solution. The filtrate obtained after filtering off the black ppt., upon
warming with Zn and NaOH evolves an alkaline pungent smelling gas. The resulting solution also responds to the
brown ring test. The filtrate does not evolve N2 gas when it is boiled with urea in the presence of H2SO4. Identify
the cation and anion present in the salt ‘A’.

Sol: Cu(NO3)2 + NaOH → Cu(OH)2 ↓ (Blue) + 2NaNO3



Cu(OH)2 ↓ CuO ↓ (Black) + H2O
4Zn + NaNO3 + 7NaOH → 4Na2ZnO2 + 2H2O + NH3 ↑ (Pungent smelling alkaline gas)

CuO + 2HNO3 Cu(NO3)2 + H2O
2Cu(NO3)2 + K4 [Fe(CN)6] → Cu2 [Fe(CN)6] ↓ (Chocolate brown) + 4KNO3

Illustration 12: A compound on heating with an excess of caustic soda solution liberates a gas (B) which gives
while fumes on exposure of HCl. The resultant alkaline solution thus obtained after heating again liberates the
same gas (B) when heated with zinc powder. Compound (A) on heating alone gives a neutral oxide of nitrogen not
nitrogen gas. Identify (A) and (B) and give the relevant chemical reactions.

Sol: As NH3 gives white fumes with HCl, therefore, (B) should be NH3 and (A) should be the salt of ammonium.
Further we know that nitrite of ammonium gives a NH3 with Zn and alkali and when heated alone gives neutral
oxide (N2O) not N2. Hence the salt should be ammonium nitrate not ammonium nitrite.
Chem i str y | 30.31

NH4NO3 (A) + NaOH → NaNO3 + H2O + NH3 ↑ (B ) ;NH3 + HCl → NH4 Cl ( White fumes )
Zn/NaOH
NaNO3 + 8 H  → NaOH + 2H2O + NH3 ;NH4NO3 → N2O (Neutral) + 2H2O

Illustration 13: A certain metal (A) is boiled with dilute HNO3 to give a salt (B) and an neutral oxide of nitrogen (C).
An aqueous solution of (B) gives a white ppt. (D) with brine which is soluble in ammonium hydroxide. An aqueous
solution of (B) also gives red/brick red ppt., (E) with potassium chromate solution. Identify (A) to (E) and write the
chemical reactions involved.

Sol: As solution of (B) gives white ppt. with NaCl (aq) ppt. is soluble in ammonium hydroxide, it may be of silver
salt. Further it gives brick red ppt. with K2CrO4, therefore, metal (A) may be silver.
3Ag(A) + 4HNO3 → 3AgNO3 (B) + NO ( C ) + 2H2O;
AgNO3 + NaCl → AgCl ↓ ( White ) (D) + NaNO3
AgCl + 2NH4 OH →  Ag (NH3 )  Cl ( Soluble ) + 2H2O
 2
2AgNO3 + K 2CrO 4 → Ag2CrO 4 ↓ (Red / Brick red) (E) + 2KNO3

Illustration 14: Which of the following salt will give white ppt. with the solution containing Pb2+ ions?
(A) Na2CO3 (B) NaCl (C) Na2SO3 (D) All of these

Sol: Pb + CO3 → PbCO3 ↓ ( White )


2+ 2−

Pb2+ + 2Cl− → PbCl2 ↓ ( White )


Pb2+ + SO32− → PbSO3 ↓ ( White )

Therefore, (D) option is correct.

Table 30.22: Some important reactions of group III radicals

Fe+3 Test/Reagents Observation


Potassium ferrocyanide (Potassium hexacyanidoferrate (ll)) Intense blue ppt. (Prussian blue) of
4−
iron (lll) hexacyanidoferrate (ll) is
4Fe+3 + 3 Fe ( CN)  → Fe4 Fe ( CN)  ↓ formed.
 6  8 3

4− This is insoluble in dilute acids but


Fe4 Fe ( CN)  + 12OH− → Fe ( OH) ↓ +3 Fe ( CN)  decomposes in concentrated HCl. A
 8 3 3  6
large excess of the reagent dissolves
it partly or entirely, when an intense
blue solution is obtained. Sodium
hydroxide turns the ppt. red.

Oxalic acid also dissolves Prussian


blue forming a blue-solution.
Important:

If iron (lll) chloride is added to an excess of potassium hexacyanidoferrate (ll), a product with the composition of
( )6 
KFe Fe CN  is formed. This tends to form colloidal solutions (‘soluble Prussian blue’) and can not be filtered.

3 0 . 3 2 | Qualitative Analysis

Potassium ferricyanide (Potassium hexacyanidoferrate (lll)) A brown colouration is formed.


3− Upon adding hydrogen peroxide or
Fe3+ + Fe ( CN)  → Fe Fe ( CN)  some tin (ll) chloride solution, the
 6  6
3− 4− hexacyanidoferrate (lll) part of the
Fe2+ + Fe ( CN)  → Fe + Fe ( CN)  compound is reduced and Prussian
 6  6
blue is ppt..
4−
4Fe3+ + 3 Fe ( CN)  → Fe4 + Fe ( CN)  Fe2+ gives dark blue ppt. with
 6  6 3
potassium ferricyanide. First
hexacyanidoferrate (lll) ions
oxidise iron (ll) to iron (lll), when
hexacyanidoferrate (ll) is formed.

And these ions combine to form a


ppt. called Turnbull’s blue.
Note: Composition of this ppt. is identical to that of Prussian blue. Earlier the composition suggested was

Fe3 Fe ( CN)  , hence different name.


 6 2

3Fe2+ + 2K 3 Fe ( CN)  → Fe3 Fe ( CN)  (Ferrous ferric cyanide)+ 6K +


 6  6 2


Turnbull’s blue

Fe (ll) in ammonical solution gives red solution with DMG-colouration fades on standing due to the oxidation of iron (ll)
complex. Fe (lll) does not give such complex.

In complete absence of air, Fe (ll) ions produces white ppt. with potassium hexacyanidoferrate (ll).
4−
Fe2 + 2K + + Fe ( CN)  → K 2Fe Fe ( CN)  ↓
 6  6

Under ordinary atmospheric conditions a pale-blue ppt. is formed.


Cr+3 Test/Reagents Observation
Acidified H2O2 test

Na2CrO 4 + H2SO 4 → Na2SO 4 + H2CrO 4 ; Blue colouration


Amylalchohol
H2CrO 4 + 2H2O2 
→ CrO5 + H2O On acidifying the yellow solution
with dil.H2SO4+few drops of
4CrO5 + 6H2SO 4 → 2Cr2 ( SO 4 ) + 7O2 + 6H2O ether/amyl alcohol +H2O2= Blue
3
colouration ,can be extracted into
the organic layer by gently shaking.

Blue colouration fades slowly due


to the decomposition of perchromic
acid (or chromium peroxide) with
the liberation of oxygen.
Tests for: In excess of NaOH. Sodium
meta-aluminate (soluble). White
Al ( OH) + NaOH → NaAlO2 + 2H2O gelatinous ppt.
3

NaAlO2 + H2O + NH4 Cl → Al ( OH) + NaCl + NH3


3
Chem i str y | 30.33

Fe3+ Test/Reagents Observation


Tests for

Fe ( OH) + 3HCl → FeCl3 + 3H2O ( )


Red brown Fe OH dissolves in dil.
3
3 HCl as
4FeCl3 + 3K 4 Fe ( CN)  → Fe4 Fe ( CN)  + 12KCl ( )6 
Reaction with K 4 Fe CN  :
 6  6 3 
FeCl3 + KSCN → Fe ( SCN) Cl2 + KCl Ferric ferrocyanide (Dark Blue)

Reaction with KCNS:

Ferrithiocyanate (dark red colour)


Cr3+
Test/Reagents Observation
Tests for On boiling with

( CH3COO )2 Pb / CH3COOH NaOH / Br2 ,Cr ( OH) gives


3

2NaOH + Br2 → NaOBr + NaBr + H2O sodium chromate which gives


yellow ppt. of PbCrO4 with
NaOBr → NaBr + O 
Yellow solution
2Cr ( OH) + 4NaOH + 3 O  → 2Na2CrO 4 + 5H2O
3
Yellow solution
Yellow ppt.
Na2CrO 4 + ( CH3COO ) Pb → PbCrO 4 ↓ +2CH3COONa
2
Yellowppt. Yellow solution
2Cr ( OH) + 2Na2CO3 + 3KNO3 → 2Na2CrO 4 + 3KNO2 + 3H2O + 2CO2
3
Yellow solution Yellow ppt.

Na2CrO 4 + ( CH3COO ) Pb → PbCrO 4 ↓ +2CH3COONa


2
Yellowppt.

Illustration 15: A black coloured compound (A) on reaction with dil. H2SO4 gives a gas (B) and a green colour
solution. The gas (B) on passing in a solution of an acid (C) gives a white/yellow turbidity (D). Gas (B) when passed
in acidified solution of (E) gives a ppt. (F) soluble in dil HNO3. After boiling this solution when excess of NH4OH is
added, a blue coloured compound (G) is formed. To this solution on addition of acetic acid and aqueous potassium
ferrocyanide a chocolate coloured ppt. (H) is obtained. On addition of an aqueous solution of BaCl2 to an aqueous
solution of (E), a white ppt. insoluble in HNO3 is obtained. Green colour solution on reaction with ammonium
hydroxide in presence of air gives reddish brown ppt.. Identify (A) to (H).

Sol: FeS ( A ) + H2SO 4 → FeSO 4 + H2S (B )


HNO3 ( C ) → NO2 + H2O + O;H2S + O → H2O + S ↓ (D )
CuSO 4 (E ) + H2S → CuS ↓ (F ) + H2SO 4
3CuS + 8HNO3 → 3Cu (NO3 ) + 2NO ↑ +4H2O + 3S ↓
2
2+
Cu2+ + 4NH3 → Cu (NH3 )  ( G)
 4

2Cu + K 4Fe ( CN) → Cu2 Fe ( CN)  ↓ (H) + 4K +


2+
6  6

Ba + SO 4 → BaSO 4 ↓ ( white )
2 2−

Fe2+ + 2H+ + O → Fe3+ + H2O


Fe3+ + 3NH3 + 3H2O → Fe ( OH) ↓ (reddish brown) + 3NH4 +
3
Hence,(A)
= FeS;(B)
= H2S;(C)
= HNO3 =
;(D) S;(E)
= CuSO 4=
;(F) CuS;

 ( 3 )4  (
(G) Cu NH  NO3 ) ;(H) Cu2 Fe ( CN) 
=
2  6
3CuS + 8HNO3 → 3Cu (NO3 ) + 2NO ↑ +4H2O + 3S ↓
2
2+
Cu2+ + 4NH → Cu (NH )4  ( G)
3 0 . 3 4 | Qualitative
3 Analysis
3

2Cu2+ + K 4Fe ( CN) → Cu2 Fe ( CN)  ↓ (H) + 4K +


6  6
Ba + SO 4 → BaSO 4 ↓ ( white )
2 2−

Fe2+ + 2H+ + O → Fe3+ + H2O


Fe3+ + 3NH3 + 3H2O → Fe ( OH) ↓ (reddish brown) + 3NH4 +
3
Hence,(A)
= FeS;(B)
= H2S;(C)
= HNO3 =
;(D) S;(E)
= CuSO 4=
;(F) CuS;

 ( 3 )4  (
(G) Cu NH  NO3 ) ;(H) Cu2 Fe ( CN) 
=
2  6

Flowchart 30.4: Systematic path for the analysis of group IV radicals


Chem i str y | 30.35

14. REACTIONS OF Ni2+ (NICKEL) AND Co2+ (COBALT)


The black ppt. is dissolved in aqua-regia.

3NiS + 6HCl + 2HNO3 → 3NiCl2 + 2NO + 3S + 4H2O


3CoS + 6HCl + 2HNO3 → 3CoCl2 + 2NO + 3S + 4H2O

The solution is evaporated to dryness and residue extracted with dilute HCl. It is divided into three parts.

Table 30.23: Some important reaction of group IV

Part I Part II Part III


Excess CH3COOH (excess)+KNO2=yellow ppt. confirms Solution containing either nickel or cobalt is treated
NH4OH+ the presence of cobalt. with NaHCO3 and bromine water. Appearance of apple
dimethyl green colour confirms the presence of cobalt. If no
KNO2 + CH3COOH → CH3. COOK + HNO2
glyoxime = apple green colour is observed, the solution is heated
rosy red ppt. CoCl2 + 2KNO2 → Co (NO2 ) + 2KCl when black ppt. is formed, which shows the presence
2
if nickel is Co (NO2 ) + 2HNO2 → Co (NO2 ) + NO + H2O of nickel.
2 3
present CoCl2 + 2NaHCO3 → Co (HCO3 ) + 2NaCl
Co (NO2 ) + 3KNO2 → K 3 Co (NO2 )  2
 6
Co (HCO3 ) + 4NaHCO3 → Na4 Co ( CO3 ) 3H2O
3
2 3
+3CO2 Br2 + H2O → 2HBr + O
2Na4 Co ( CO3 ) + H2O + O → 2Na3Co ( CO3 ) + NaOH
3 3
Sod. cobalt carbonate
(Green colouration)
NiCl2 + 2NaHCO3 → NiCO3 + 2NaCl + H2O + CO2
2NiCO3 + O → Ni2O3 + 2CO2
Black

Zn2+ (zinc)
The sulphide dissolves in HCl.
ZnS + 2HCl → ZnCl2 + H2S
When the solution is treated with NaOH, first a white ppt. appears which dissolves in excess of NaOH

ZnCl2 + 2NaOH → Zn ( OH) ↓ +2NaCl


2
Whiteppt.
Zn ( OH) + 2NaOH → Na2 ZnO2 + 2H2O
2
Soluble

On passing H2S , white ppt. of zinc sulphide is formed,

Na2 ZnO2 + H2S → ZnS + 2NaOH


Whiteppt

Mn2+ (manganese)
Manganese sulphide dissolves in HCl.
MnS + 2HCl → MnCl2 + H2S
On heating the solution with NaOH and Br2 -water, manganese dioxide gets ppt..
3 0 . 3 6 | Qualitative Analysis

MnCl2 + 2NaOH → Mn ( OH) + 2NaCl


2
Mn ( OH) + O → MnO2 + H2O
2

The ppt. is treated with excess of HNO3 and PbO2 or Pb3O4 (red lead). The contents are heated. The formation of
permanganic acid imparts pink colour to the supernatant liquid.

2MnO2 + 4HNO3 → 2Mn (NO3 ) + 2H2O + O2


2
2Mn (NO3 ) + 5Pb3O 4 + 26HNO3 → 2HMnO 4 + 15Pb (NO3 ) + 12H2O
2 2
Permanganic acid(pink)

BaCO3 + 2CH3COOH → ( CH3COO ) Ba + CO2 + H2O


2
SrCO3 + 2CH3COOH → ( CH3COO ) Sr + CO2 + H2O
2
CaCO3 + 2CH3COOH → ( CH3COO ) Ca + CO2 + H2O
2

Flowchart 5: For the analysis of group V radical


Chem i str y | 30.37

GROUP V (Ba (II), Sr (II), Ca (II))


Ammonium carbonate ppt. V group radicals in the form of carbonates.These carbonates are soluble in acetic acid.
Table 30.24: Confirmatory test for group V radicals

Ba2+ Test/Reagents Observation


White ppt.
Ba ( CH3COO ) + K 2CrO 4 → BaCrO 4 ↓ +2CH3COOK
2
Ba ( CH3COO ) + (NH4 ) SO 4 → BaSO 4 ↓ +2CH3COONH4
2 2
Ba ( CH3COO ) + (NH4 ) C2O 4 → BaC2O 4 ↓ +2CH3COONH4
2 2
White ppt.
Sr ( CH3COO ) + (NH4 ) SO 4 → SrSO 4 ↓ +2CH3COONH4
2 2
Whiteppt.
White ppt.
White ppt.
Sr ( CH3COO ) (NH4 ) C2O 4 → SrC2O 4 ↓ +2CH3COONH4
2 2
Calcium chlorate and calcium
Ca ( CH3COO ) + (NH4 ) C2O 4 → CaC2O 4 ↓ +2CH3COONH4 sulphate are soluble.
2 2
Whiteppt.
White ppt.

VITH GROUP
Table 30.25: Confirmatory test for group VI radical

Mg2+ Test/Reagents Observation


Disodium hydrogen phosphate solution White crystalline ppt. is formed in
presence of NH4Cl prevent precipitation
2
Mg + NH3 + HPO 4 2−
→ Mg (NH4 ) PO 4 ↓
of Mg(OH)2 and NH3 soln
Mg2 + HPO 42− → MgHPO 4 ↓
white flocculent ppt.
Ammonia solution White gelatinous ppt is sparingly
soluble in water but readily soluble in
Mg2 + 2NH4 OH → Mg ( OH) ↓ +2NH4 +
2 ammonium salts.
Mg ( OH) → Mg +2
+ 2OH−
2 NH4 + ions ‘remove’ OH− causing the
NH4 Cl → NH4 + + Cl− ; hydroxide to dissolve more. Not possible
with NaCl.
NH4 + + OH− → NH4 OH(weak base)

Ammonium carbonate solution +


Absence of NH4 salts.
5Mg 2+
+ 6CO32− + 7H2O → 2MgCO3 .Mg ( OH) .5H2O ↓ +2HCO3 −
2 In the presence of NH4 + salts no
NH4 + + CO32− → NH3 + HCO3− precipitation occurs, because the
equilibrium is shifted towards the
formation of HCO3− ions. K sp of the
ppt. being high (Ksp of pure MgCO3
−5
is 1 × 10 ), the concentration of
carbonate ions necessary to produce a
ppt. is not attained.
4-(4-Nitrophenyl azo resorcinol) or Magneson I Ppt. is dissolved in dilute HCl (min.)
+NaOH +0.5ml magneson-I reagent =
MgCl2 + 2NaOH → Mg ( OH) + 2NaCl
2 Blue lake
Magneson reagent = p-nitrobenzene-azo resorcinol, a dye stuff,
absorbed over Mg(OH)2 to give a blue coloured lake.
3 0 . 3 8 | Qualitative Analysis

Titan yellow (a water soluble yellow dyestuff) Deep red colour or ppt.

It is adsorbed by Mg(OH)2
Dissolve ppt. in dil.HCl(min.)+1 drop of NaOH soln (2M) + 1 drop titan Deep red colour solution or ppt. is
yellow soln obtained.

Ba2+ andCa2+ do not react but intensify


the colour.

Table 30.26: Action of heat on different compounds

(a) Some oxides liberate O2: (e) Some sulphates liberate SO 2 :


Heat Heat
2HgO → 2Hg + O2 ↑ 2MgSO 4  → 2MgO + 2SO2 + O2
(Red) (Silvery deposit) Heat (hightemp)
2ZnSO 4 
→ 2ZnO + 2SO2 + O2
Heat
2Pb3 O 4 → 6PbO + O2 ↑ Heat
2BeSO 4  → 2BeO + 2SO2 + O2
(Red) (Yellow)
(f) Some sulphates lose water of crystallization:
Heat
2PbO2 → 2PbO + O2 ↑
Heat 2 ( CaSO 4 .2H2O ) 
Heat
→ 2 ( CaSO 4 .H2O ) + 2H2O
2Ag2O  → 4Ag + O2 ↑
70°C 100°C
ZnSO 4 .7H2O → ZnSO 4 .6H2O  →
(b) Some carbonates liberate CO: −H O
2 −5H O 2
450°C
Heat
CuCO3  → CuO + CO2 ↑ ZnSO 4 .H2O 
−H2O
→ ZnSO 4

(Green) (black) (g) Some nitrates liberate NO2 and O2:


Heat
ZnCO3 → ZnO + CO2 ↑
2Zn (NO3 ) 
Heat
→ 2ZnO + 4NO2 + O2
(white) Yellow (hot) 2
white Brown
white(cold)
Heat 2Cu (NO3 ) 
Heat
→ 2CuO + 4NO2 + O2
2Ag2CO3  → 4Ag + 2CO2 ↑ +O2 ↑ 2
Heat
CaCO3 → CaO + CO2 ↑ 2Pb (NO3 ) 
Heat
→ 2PbO + 4NO2 + O2
2
Heat
MgCO3 → MgO + CO2 ↑ 2Mg (NO3 ) 
Heat
→ 2MgO + 4NO2 + O2
2

2Ca (NO3 ) 


Heat Heat
Li2CO3 → Li2O + CO2 ↑ → 2CaO + 4NO2 + O2
2
Heat
(c) Some bicarbonates liberate CO2 : 2LiNO3 → Li2O + 2NO2 + 1 / 2O2

2NaHCO3 → Na2CO3 + CO2 + H2O Hg (NO3 ) 


Heat
→ Hg + 2NO2 + O2
2
NH4HCO3 → NH3 + CO2 + H2O Heat
2AgNO3  → 2Ag + 2NO2 + O2
2Co (NO3 ) 
Heat
→ 2CoO + 4NO2 + O2
(d) Some sulphates liberate SO3 : 2

Heat
CuSO 4 .5H2O  → CuSO 4 ; (h) Some nitrates liberate O2 :
−5H O 2
Heat
Heat
 → CuO + SO3 2NaNO3  → 2NaNO2 + O2
Heat
Heat
2FeSO 4  → .Fe2O3 + SO 2 + SO3 2AgNO3  → 2AgNO2 + O2

Al2 ( SO 4 ) 
Re dHot
→ Al2O3 + 3SO3
3
Chem i str y | 30.39

(i) Some nitrates liberate N2O: (k) Some chlorides decompose as:
Heat Heat
NH4NO3  → N2O + 2H2O 2FeCl3  → 2FeCl2 + Cl2
Heat
2CuCl2  → Cu2Cl2 + Cl2
(j) Hydrated chlorides liberate HCl: Heat
NH4 Cl  → NH3 + HCl
Heat
2  AlCl3 .6H2O   → Al2O3 + 6HCl + 9H2O Heat
Hg2Cl2  → HgCl2 + Hg
Heat
MgCl2 .6H2O → MgO + 2HCl + 5H2O
(l) Some other salts decompose as:
Heat
ZnCl2 .2H2O  → Zn ( OH) Cl + HCl + H2O
2 ( ZnCl2 .H2O ) 
Heat
→ Zn2OCl2 + 2HCl + H2O (NH4 )2 Cr2O7 
Heat
→ N2 + Cr2O3 + 4H2O
Heat orange Green
2 FeCl3 .6H2O   → Fe2O3 + 6HCl + 9H2O
Heat
4K 2Cr2O7 → 4K 2CrO 4 + 2Cr2O3 + 3O2
Heat
SnCl2 .2H2O  → Sn ( OH) Cl + HCl + H2O Heat
NH4NO2  → N2 + 2H2O
Heat
NH4NO3  → N2 + 2H2O
2Mg (NH4 ) PO 4 
Heat
→ Mg2P2O7 + H2O + 2NH3
2Zn (NH4 ) PO 4 
Heat
→ Zn2P2O7 + H2O + 2NH3

( CH3COO )2 Pb 
Heat
→ PbCO3 + CH3COCH3
Heat
FeC2O 4  → FeO + CO2 + CO
Heat
SnC2O 4  → SnO + CO2 + CO
Heat
CaC2O 4  → CaCO3 + CO
K 4 Fe ( CN)  
Heat
→ 4KCN + Fe + 2C + N2
 6
100°C 160°C
H3BO3  → HBO2  → H2B 4 O7 ;
Red Hot
→ B2O3
Heat
2KCiO3  → 2KCl + 3O2
Heat
2KMnO 4  → K 2MnO 4 + MnO2 + O2
Heat
Na2B 4 O7 .10H2O 
−10H O
→ Na2B 4 O7 ;
2
Heat
 → 2NaBO2 + B2O3

(Glassy bead)

Na (NH44) )HPO  ∆
Na(NH HPO44  NH33 ++NaPO
→ NH NaPO33++HH
2O
2
O

Table 30.27: Different colored inorganic compounds

Black Coloured Blue Coloured Compounds Green Coloured Compounds Yellow Coloured
Compounds Compounds
Light Blue

1. PbS 1. Cu(OH)2 1. Ni(OH)2 (Green ppt.) 1. As2S3

2. Ag2S 2. Cu(NO3)2 2. Hg2I2 (Green ppt.) 2. As2S5

3. CuS 3. CuCl2 3. Cr2O3 (Green solid) 3. CdS

4. Cu2S 4. CuSO4.5H2O (Blue vitrol) 4. Cr2(SO4)3 4. SnS2 (Artificial gold)


3 0 . 4 0 | Qualitative Analysis

Black Coloured Blue Coloured Compounds Green Coloured Compounds Yellow Coloured
Compounds Compounds
5. NiS 5. Zn2[Fe(CN)6] (Bluwish white 5. CrCl3 5. FeS2 (Fool’s gold)
6. CoS ppt) 6. FeSO4. 7H2O 6. (NH4)2Sx (where X=2 to 5)
7. HgS 6. Co(OH)2 7. FeCl2 7. PbCrO4
8. FeS 8. FeSO4.(NH4)2SO4.6H2O 8. BaCrO4
9. NiO Deep Blue (Mohr’s salt) 9. SrCrO4
10. MnO 1. [Cu(NH3)4]SO4 (Swizzer’s 9. Na2MnO4 10. AgBr (light yellow)
reagent) 10. K2MnO4
11. FeO 11 AgI (Dark yellow)
2. [Cu(NH3)4](NO3)2 11. B(OC2H5)3 (Burns with green
12. CuO 12. PbI2
3. Fe4[Fe(CN)6]3 (Prussian’s blue) edge flame)
13. PbO2 13. PbO (in Cold)
4. Fe3[Fe(CN)6]2 (Turnbull’s blue) 12. CoO.ZnO (Riemann’s green)
14. MnO2 14. ZnO (in Hot)
5. Na4[Fe(CN)5(NOS)] (Violet)
15. Mn3O4 15. HgO(Yellow ppt.)
16. Fe3O4 16. Na2O2(Pale yellow)
17. Co3O4 17. Ag3PO4
18. Ni(OH)3 18. Ag2CO3
19. Cu3P2 19. Ag3AsO4
20. BiI3 20. Cu(CN)2
21. Hg+Hg(NH2)Cl 21. K3[Co(NO2)6]
22. (NH4)3PO4.12MoO3
23. (NH4)3AsO4.12MoO3
24. Na2CrO4
25. CrO42-(Yellow in solution)

Red Coloured Brown Coloured Compounds Orange Coloured Pink Coloured


Compounds Compounds Compounds
1. Ag2CrO4 (Brick red) 1. SnS 1. Sb2S3 Mn(OH)2

2. Hg2CrO4 (Brick red) 2. Bi2S3 2. Sb2S5 MnS

3. HgI3 (Scarlet red) 3. CdO 3. KO3 MnO4-

4. Pb3O4 (2PbO + PbO2) 4. PbO2 4. CsO2 (pink or purple in aq.soln)

5. CrO2Cl2 (Reddish Brown) 5. Fe(OH)3 (Reddish Brown) 5. Cr2O72- (orange in aq. soln) Co(CN)2

6. Fe(CH3COO)3 6. Fe2O3 (Reddish Brown solid) (NH4)2SnCl6


(Blood red)
CoCl2.6H2O
7. Fe(SCN)3 (Blood Red) 7. Fe2(CO3)3

8. AsI3 8. Cu2O (Reddish Brown)

9. SbI3 9. Ag3AsO4 (Reddish Brown)

10. SnI2 10. Cu2I2 + I3− (Brown ppt.)

11 CuBr2 11 Cu2[Fe(CN)6] (Chocolate Brown)

12. [Ni(DMG)2] (Rosy red) 12. NO2 (Brown gas)

13. [Fe(H2O)5(NO)]SO4 (Brown ring)


Chem i str y | 30.41

POINTS TO REMEMBER

An aqueous solution containing :


+ +2 +2 +2 +3 +2 +2 +2
Ag , Hg2 , Pb , Hg , Bi , Cu , Ni , Mn ,
+3 +3 +2 +2 +2 + + +2
Fe , Al , Ba , Sr , Ca , K , Na , Mg

Add dilute HCl


Precipitate forms Ions remain in soln.
Group II-V:
Group I:
+2 +2 +3 +2 +2 +2 +3 +3
AgCl(s) Pb ,Hg ,Bi ,Cu , Ni ,Mn ,Fe ,Al
Hg2Cl2(s) Ba
+2
,Sr
+2
,Ca
+2
,NH4+, K ,Na
+ +
,Mg
+2

PbCl2(s)
Add H2S in 0.3 M HCl

Precipitate forms Ions remain in soln.


Group II: Group III-V:
PbS(s) +2 +3 +3 +3 +2 +2
Ni ,Mn ,Fe ,Al , Ba ,Sr ,
HgS(s) +2 + + +2
Ca ,NH4+ , K ,Na ,Mg
Bi2S2(s)
CuS(s) Add H2S in basic soln.
(NH3/NH4Cl buffer).

Precipitate forms Ions remain in soln.


Group III: Group IV-V:
NiS(s) Ba
+2 +2
, Sr , Ca
+2
MnS(s) + + +2
NH4+ , K ,Na , Mg
FeS(s)
Al(OH)3(s) Add (NH4)2CO3 in basic
soln. (NH3/NH4Cl buffer).

Precipitate forms Ions remain in soln.


Group IV: Group V:
+
BaCO3(s) NH4 , K+ ,
+2
Na+ , Mg
SrCO3(s)
CaCO3(s)
3 0 . 4 2 | Qualitative Analysis

Solved Examples

JEE Main/Boards Sol: Gas X is soluble in water forming basic solution


because it turns red litmus blue. Hence, the gas may
Example 1: A colourless solid (A) liberates a brown be NH3.
gas (B) on acidification, a colourless alkaline gas (C) on NH3 + H2O → NH4 OH
treatment with NaOH and a colourless non-reactive gas
CuSO 4 + 4NH4POH →[Cu(NH3 )4 ]SO 4 + 4H2O
(D) on heating. If heating of (A) is continued, it completely
disappears. Identify the compounds (A) to (D). (T etra− amine cupric
sulphate) D eep blue
Sol: (A) From the given data it appears to be NH4NO2.
Ferric chloride gives brown ppt. of Fe(OH)3
This can be explained by the following reaction
FeCl3 + 3NH4 OH → Fe(OH)3 + 3NH4 Cl
With mineral acid NH4NO2 yields Nitrous acid which
eventually gets converted to Nitric acid liberating NO. (Brown precipitate)

NO with oxygen forms brown coloured NO2 gas. Brown ppt. is soluble in HNO3

(1) NH4NO2 + HCl → NH4 Cl+ HNO2 Fe(OH)3 + 3HNO3 → Fe(NO3 )3 + 3H2O
(A) Nitrous acid (Soluble complex)
3HNO2 → HNO3 + H2O + 2NO
(Nitric acid)
*** NaCl on heating with conc. H2SO4 gives HCl
whereas NaBr and NaI gives Br2 and I2 respectively.
2NO + O2 → 2NO2 Why?
(Brown gas) (B)
[Hint: NaCl + H2SO 4 → NaHSO 4 + HCl
NH4NO2 with NaOH gives out Sodium nitrite and NaBr + H2SO 4 → NaHSO 4 + HBr
ammonia gas.
2HBr + H2SO 4 → Br2 + SO2 + 2H2O
(2) NH4NO2 + NaOH → NaNO2 + NH3 ↑ +H O
(C) Ammonia gas
2 HBr and HI are reducing agent whereas H2SO4 is
oxidising agent and thus Br2 and I2 are formed.]
On heating it gives off colourless Nitrogen gas and
water.
Example 3: A compound (A) on heating with an excess
(3) NH4NO2 → ∆
N2 ↑ + 2H2O of caustic soda solution liberates a gas (B) which gives
Non−reactive (D) white fumes on exposure of HCl. Heating it continued to
expel the gas completely. The resultant alkaline solution
*** Both NO2 and Br2 are brown gases. How can they again liberates the same gas (B) when heated with zinc
be identified if placed separately in two containers? powder. However the compound (A), when heated alone
does not give nitrogen. Identify (A) and (B).
[Hint: Water is added in both the container. Br2
forms yellow solution whereas NO2 forms colourless
Sol: We all know that NH3 with HCl gives white fumes of
solution.]
NH4Cl with popping noise.
Hence it should be NH3. Thus, compound (A) must be
Example 2: An aqueous solution of gas (X) shows the
an ammonium salt.
following reactions
Also we know that nitrates and nitrites on heating with
(a) It turns red litmus blue.
Zn and alkali liberate NH3 gas. Hence the compound (A)
(b) When added in excess to a copper sulphate solution, should be ammonium nitrate or nitrite
a deep blue colour is obtained.
But compound (A) does not give N2 on heating hence it
(c) On addition of FeCl3 solution, a brown ppt., soluble in may not be ammonium nitrite.
dilute HNO3 is obtained. Identify (X) and give equations
for the reactions at step (b) and (c).
Chem i str y | 30.43

Reactions involved: Example 5: (i) Solution salt of an acid (A) is formed on


boiling white phosphorus with NaOH solution.
NH4NO3 + NaOH → NaNO3 + H2O + NH3 ↑
(ii) On passing chlorine through phosphorus kept fused
(A) under water, another acid (B) is formed which on strong
heating gives metaphosphoorus acid.
NH3 + HCl → NH4 Cl
(White fumes) (iii) Phosphorus on treatment with conc. HNO3 gives an
acid (C) which ais also formed by the action of dilute
Zn/NaOH
NaNO3 + 8[H]  → NaNO3 + 2H2O + NH3 ↑ H2SO4 on powdered phosphorite rock.
(iv) (A) on treatment with a solution of HgCl2 first gives a
*** Why original solution is not prepared in conc.
white ppt. of compound (D) and then grey ppt. (E).
HNO3 ?
Identify (A) to (E) and write balanced chemical equations
[Hint: HNO3 is an oxidising agent which on
for the reactions at step (i) to (iv)
decomposition gives oxygen. A yellow ppt. of
sulphur is obtained in presence of HNO3 when H2S
is passed for detecting the radicals of group II and Sol: ( i)  P4 + 3NaOH + 3H2O → 3NaH2PO2 + PH3
group IV. H2S + 2HNO2 → 2NO2 + 2H2O + S ] (Sodium hypophosphate)

Example 4: An inorganic compound (A) shows the Thus, acid (A) is H3PO2, i.e. , hypophosphorus acid
following reactions.
(ii) 2P + 3Cl2 + 6H2O → 2H3PO3 + 6HCl
(i) It is white solid and exists as dimmer; gives fumes of
(Phosphorus acid)
(B) with wet air.
(ii) It sublimes on 180° C and forms monomer if heated Thus, acid (B) is H3PO3
to 400° C.
(iii) P4 + 20HNO3 → 4H3PO 4 + 20NO2 + 4H2O
(iii) Its aqueous solution turns blue litmus to red.
(C)
(iv) Addition of NH4OH and NaOH separately to a
solution of (A) gives white ppt. which is however soluble P4 + 10HNO3 → 4H3PO 4 + 10NO2 + 4H2O
in excess of NaOH. Phoshoric acid(C)
Thus acid (C) is H3PO4
Sol: (i) (A) is a characteristic dimerized compound which
sublimes on 180° C and forms monomer if heated to (iv) H3PO2 + 2H2O → H3PO 4 + 4H
400° C and thus, (A) is (AlCl3)2 or Al2Cl6
180°C 400°C
Al2Cl6(s)  → Al2Cl6(v)  → 2AlCl3 HgCl2 + 2H → Hg2Cl2 + 2HCl
(white)(D)
(ii) It fumes with wet air

Al2Cl6 + 6H2O → 2Al(OH)3 + 6HCl ↑ Hg2Cl2 + 2H → 2Hg + 2HCl


Fumes(B) (Grey)(E)

*** During testing of Cl- and Br- by chloroform


(iii) Its solution in water is acidic due to hydrolysis
test, CHCl3 layer first turns violet and then brown,
2AlCl3 + 6H2O → 2Al(OH)3 + 6HCl(aq) confirming the presence of I- and Br- respectively.
(iv) (A) gives white ppt. with NH4OH, soluble in excess When layer turns brown then violet colour
of NaOH. disappears or not and why?
[Hint: Violet and brown colours are due to oxidation
Al2Cl6 + 6NH4 OH → 2Al(OH)3 + 6NH4 Cl
of I- to I2 and Br- to Br2 respectively. The violet
Al2Cl6 + 6NaOH → 2AlOH+ 6NaCl colour of I2 disappears when layer is brown due to
formation of ( IO3− ) (iodate ion) which is colourless.]
3 0 . 4 4 | Qualitative Analysis

Example 6: (i) A black coloured compound (B) is formed Note: *** Subquestions placed after the answers
on passing hydrogen sulphide through the solution of a
compound (A) in NH4OH.
JEE Advanced/Boards
(ii) (B) on treatment with hydrochloric acid and potassium Example 1: (i) An ore (A) on roasting with sodium
chlorate gives (A). carbonate and lime in the presence of air gives two
(iii) (A) on treatment with potassium cyanide gives a buff compounds, (B) and (C).
coloured ppt. which dissolves in excess of this reagent (ii) The solution of (B) in conc. HCl on treatment with
forming a compound (C). potassium ferricyanide gives a blue colour or ppt. of
(iv) The compound (C) is changes into a compound (D) compound (D).
when its aqueous solution is boiled. (iii) The aqueous solution of (C) on treatment with conc.
(v) The solution of (A) was treated with excess of sodium H2SO4 gives a yellow coloured compound (E).
bicarbonate and then with bromine water. On cooling (iv) Compound (E) when treated with KCl gives an
and shaking for some time, a green colour compound orange-red compound (F) which is used an oxidizing
(E) is formed. No change is observed on heating. Identify reagent.
(A) to (E) and gives chemical equations for the reactions
at steps (i) to (v). (v) The solution of (F) on treatment with oxalic acid and
then with an excess of potassium oxalate gives blue
Sol: Here it is given that a black coloured compound (B) crystals of compound (G).
is formed on passing hydrogen sulphide through the Identify (A) to (G) and give balanced chemical equations
solution of a compound (A) in NH4OH. for reactions at steps (i) to (v).
This indicates that (A) is salt of the IV group radicals
( )
Co2+ , Ni2+ or Zn2+ . Sol: (a) Reaction (i) suggests that the ore contains Fe.

Further it is given that on treatment with potassium (b) Reaction (ii) and (iv) suggests that the ore also
cyanide (A) gives a buff coloured ppt. which dissolves in contains chromium.
excess of this reagent forming a compound (C). Reaction (i), (iii) and (IV) are involved in the preparation
Which appears to be a cobalt salt (CoCl2) of K2Cr2O7 from chromite ore, FeOCr2O3.
Hence the given reactions can be written as below
(i) CoCl2 + 2NH4 OH+ H2S → CoS + 2NH4 Cl + 2H2O
lime
(A) (B) (i) 4FeO.Cr2O3 + 8Na2CO3 + 7O2 →
(Rosting)
2Fe2O3 ↓ +8Na2CrO 4 + 8C

(B)
Chromite ore (A) (B) (C)
(ii) CoS + 2HCl + O  → CoCl2 + H2O + S
lime
(from KClO ) (i) 4FeO.Cr2O3 + 8Na2CO3 + 7O2 →
(Rosting)
2Fe2O3 ↓ +8Na2CrO 4 + 8CO2 ↑
3
Chromite ore (A)
2KClO3 → 2KCl + 3O2 (B) (C)
(ii) Fe2O3 + 6HCl → 2FeCl3 + 3H2O
(iii) CoCl2 + 2KCN → Co(CN)2 ↓ +2KCl
buff coloured 4FeCl3 + 3K 4 [Fe(CN)6 ] → Fe4 [Fe(CN)6 ]3 + 12KCl
Blue(D)
CO(CN)2 + 4KCN → K 4 [Co(CN)6 ]
(iii) 2Na2CrO 4 + H2SO 4 → Na2 Cr6 O3 + Na2SO 4 + H2O
(iv) 2K 4 [CO(CN)6 ] + O + H2O → 2K 3 [Co(CN)6 ] + 2KOH soluble (C) Yellow colour (E)
(D)
(iv) Na2Cr2O7 + 2KCl → K 2 Cr2 O7 + 2NaCl
(v) CoCl2 + 6NaHCO3 → Na4 [Co(CO3 )3 ] + 2NaCl+ 3CO2 + 3H2O(E) (F)
oCl2 + 6NaHCO3 
→ Na4 [Co(CO3 )3 ] + 2NaCl+ 3CO2 + 3H2O
K 2 Cr2 O7 (F) is well known oxidizing agent.

2Na4 [Co(CO3 )3 ] + 2NaHCO3 + O → 2Na3 [Co(CO3 )3 ] + 2Na


(v)2CO + 2HO
Cr23(C 2O4 )3 + 3K 2 C2 O 4 → 2K 3 [Cr(C2 O 4 )3 ] + Cr2 (C2O 4 )3
(E)(v) Cr (C O ) + 3K C O Potassium oxalate
→ 2K 3 [Cr(C Potassium trioxalatochromium
CO3 + O → 2Na3 [Co(CO3 )3 ] + 2Na2CO3 + H2O 2 2 4 3 2 2 4  2 O 4 )3 ] + Cr2 (C2 O 4 )3
Blue crystals (G)
Potassium oxalate Potassium trioxalatochromium
(E)
Blue crystals (G)
Chem i str y | 30.45

*** Chromyl chloride test for chlorides is not (iv) FeSO 4 + K 3 [Fe(CN)6 ] → KFe[Fe(CN)6 ] + K 2SO 4
performed if mixture contain chlorates, why? (D) (E)
[Hint: Because ions react with H2SO4 to
ClO3−
form chlorine dioxide (yellow greenish gas) which Example 3: A white substance A reacts with dilute
dissolves in H2SO4 forming orange-yellow solution. H2SO4 to produce a colourless gas B and a colourless
This solution is highly explosive. solution C. The reaction between B and acidified K2Cr2O7
solution produces a green solution and a slightly
3KClO3 + 3H2SO 4 → 2ClO2 + ClO −4 + 3SO24− + 4H+ + 3K + + H2O
coloured participate D. The substance D burns in air to
O −4 + 3SO24− + 4H+ + 3K + + H2O ] produce a gas E which reacts with B to yield D and a
colourless liquid. Anhydrous copper sulphate is turned
Example 2: (i) A black mineral (A) on heating in presence blue on addition of this colourless liquid. Addition of
of air gives a gas (B). aqueous NH3 or NaOH to C produces first a participate
which dissolves in the excess of the respective reagent
(ii) The mineral (A) on reaction with dilute H2SO4 gives a to produce a clear solution in each case. Identify A, B, C,
gas (C) and solution of a compound (D). D and E. Write the equation of the reactions involved.
(iii) On passing gas (C) into an aqueous solution of (B) a
white turbidity is obtained. Sol:
dil.H2SO4
(iv) The aqueous solution of compound (D) on reaction


A B +C
with potassium ferricyanide gives a blue compound (E). (White) (Colourless) (Colourless solution)

Identify (A) to (E) and give chemical equations for +


reactions at steps (i) to (iv). K2Cr2O7/H

Sol: Summary of the given statement can be written as:


Green solution + D
dilH SO
2 4 → Black mineral A  O
2 → Gas B i)H O
2 →White turbidity (Burns in air to form E)
Gas C + Solution of D ← ii)gas C (Coloured)
O i)H O CuSO4
2 → Gas B
Black mineral A  2 →White turbidity


E +B D + Colourless liquid Blue


ii)gas C
Aq.NH3 Excess of
D C
or NaOH
Precipitate
reagent
Clear solution

K3[Fe(CN)6] The above set leads to following conclusions.


(i) Because Gas (B) is colourless and turns acidified
Blue compound E K2Cr2O7 solution green, it appers to be H2S.
(ii) H2S gas is obtained by the reaction of dil. H2SO4 on
Compound D foms blue coloured solution of E by
A, thus A must be a sulphide.
ton treatment of potassium ferricyanide.This indicates
that compound D contains Fe2+ ion which is also the (iii) Zns sulphide is white thus it indicates that A is ZnS
constituent ion of the compound A. Further it is given
that A, is a black mineral of Fe2+ ion, so it has to be ZnS +H2SO4(dil) ZnSO4 +H2S
ferrous sulphide (FeS) which is confirmed by following (A) (C) (B)
reaction: 3H2S +K2Cr2O7 K2SO4 + Cr2(SO4)3 + 7H2O + 3S
(B) (green) (D)
(i) 4FeS + 7 O2 → 2Fe2O3 + 4SO2 
2H2S(B)
(A) (B) S + O2 SO2  2H2S + 3S
(D) (E) (Colourless liq) (D)

(ii) FeS + H2S O 4 → FeSO 4 + H2S + S


CuSO4(white)
(A) (D) (C)

(iii) a) SO2 + H2 O → H2SO3


CuSO4. 5H2O (blue)
 2NaOH
b) 2H2 S + H2 SO3 → 3S + 3H2O ZnSO4 + 2NaOH Zn(OH)2 Na2ZnO2 + 2H2O
(C) (Soluble)
(C) Turbidity
3 0 . 4 6 | Qualitative Analysis

*** Before testing II group, iodide ions are separated. acid should be CuS; hence (F) must be CuS and thus (B)
Why? is H2S.
[Hint: Because I- ions react with air to form I2 which (iv) According to first point compound (E) contains
reacts with H2S and give white or light yellow ppt. of SO24− hence (E) must be CuSO4.
sulphur i.e., why I- are removed by boiling original
(v) Gas (B) (identified as H2S) is obtained by the
solution with NaNO2.
decomposition of black coloured compound (A) with
4HI + O2 → 2H2O + 2I2 ; I2 + H2S → 2HI + S ] dil. H2SO4. Hence (A) must be sulphide of Cu, Pb, Hg,
and Fe. Co, Ni, etc.
Example 4: A black coloured compound (A) on Thus the various compounds from (A) to (H) and their
reaction with dilute sulphuric acid gives a gas (B) which reactions can be written as below.
on passing in a solution of an acid (C) gives a white
turbidity (D). Gas (B) when passed in acidified solution FeS + H2SO 4 → FeSO 4 + H2S ↑
of a compound (E) gives a ppt. (F) soluble in dilute Ferrous sulphide (A) Hydrogen sulphide (B)

nitric acid. After boiling this solution when an excess


of ammonium hydroxide is added, a blue coloured H2S + HNO3 → 2H2O + 2NO2 + S ↓
Hydrogen sulphide(B) Nitric acid(C) Sulphur (turbidity, D)
compound (G) is formed. To this solution on addition
of acetic acid and aqueous potassium ferrocyanide
H2S + HNO3 → 2H2O + 2NO2 + S↓
a chocolate ppt.sulphide(B)
Hydrogen (H) is obtained. On addition of an
Nitric acid(C) Sulphur (turbidity, D)
aqueous solution of barium chloride to an aqueous
solution (E), a white ppt. insoluble in HNO3 is obtained. in presence
Identify from (A) to (H). CuSO 4 + H2S 
of dil.HCl
→ CuS ↓ + H2SO 4
Copper sulphide(E) (B) Copper sulphide(F)

Sol: Summary we can draw from the given Data:


3CuS + 8HNO3 → 3Cu(NO3 )2 + 2NO+ 3S + 4H2 O
Dil.H2SO4 Acid (F)
Black compound Gas White turbidity
(A) (B) (D) Cu(NO3 )2 + H2SO 4 → CuSO 4 + 2HNO3

(E) + Acid CuS O 4 + 4NH4 OH → [Cu(NH3 )4 ]SO 4 + 4H2O


Cupric ammonium sulphate
(blue compound (G))
Dil
Solution Precipitate(Soluble in dil. HNO3)
HNO3
(F) [Cu(NH3 )4 ]SO 4 + 4CH3COOH → CuSO 4 + 4CH3COONH4
i) Boil (G)
ii) NH4OH
2CuS O 4 + K 4 [Fe(CN)6 ] → Cu2 [Fe(C)6 ] ↓ + 2K 2SO 4
Ptassium ferrocyanide Cupric ferrocyanide
CH3COOH (Chocolate precipitate, H)
Blue compound Chocolate ppt.
Aq.K4[Fe(CN6]
(G) (H) CuSO 4 + BaCl2 → BaSO 4 ↓ +CuCl2
BaCl2 (E) Barium sulphate
E(aq. solution) White precipitate(insoluble in HNO3) (insoluble in water)

(i) Aqueous solution of compound E with BaCl2 to give Thus the compounds (A) to (H) are
white ppt. which is insoluble in nitric acid indicates that
A = Ferrous sulphide, FeS,
the salt (E) contains SO24− ions.
B = Hydrogen sulphide, H2S,
(ii) Compound (G) with potassium ferricyanide in
presence of acidic acid to give chocolate ppt. (H) this C = Nitric acid, HNO3
indicates that (G) must contain Cu2+ and hence (H) has D = sulphur, S,
to be cupric ferricyanide, Cu2 [Fe (CN) 6].
E = Copper sulphate, CuSO4,
(iii) As compound (G) is derived from (F), compound F= Copper sulphide, CuS,
(F) also contains Cu2+. Further since (F) is derived from
the reaction of the gas (B) and compound (E), (E) must G = Cupric ammonium sulphate, [Cu(NH3 )4 ]SO 4
contain Cu2+ ion. Ppt. of Cu2+ ion soluble in dilute nitric H = Cupric ferricyanide, Cu2 [Fe(CN)6 ]
Chem i str y | 30.47

*** An original solution is prepared in conc. HCl. Example 6: A substance X dissolves in hot conc. HCl
When diluted a while ppt. is formed. What does it to give solution which when treated with caustic soda
indicate? solution gives a white ppt. which however dissolves
in excess of caustic soda solution giving a strongly
[Hint: Formation of White ppt indicates the
solution. On heating X with sulphur, a brown powder
Presence of Sb+3 or Bi+3. Their chlorides hydrolyse
Y is formed which dissolved on warming with yellow
to oxychlorides in presence of excess of water.
ammonium sulphide solution. The solution gives a grey
BiCl3 + H2O → BiOCl + 2HCl ; ppt. with HCl. When X is heated in air, a white powder
Z is obtained which can be dissolved in conc. H2SO4.
SbCl3 + H2O → SbOCl + 2HCl ]
When Z is fused with NaOH, extracted with hot water,
Example 5: (i) The yellow coloured ppt. of compound then treated with mineral acid, white gelatinous ppt.
(A) is formed on passing H2S through a neutral solution is obtained. Identify X, Y, Z and give the reactions
of salt (B). involved.
(ii) (A) is soluble in hot dilute HNO3 but insoluble in
yellow ammonium sulphide. Sol:

(iii) The solution of (B) on treatment with small quantity conc.HCl NaOH
X → Solution  NaOH
→ White ppt.  → Solution
of NH3 gives a white ppt. which become soluble in (Strongly
excess of its forming a compound (C). reducing)

(iv) The solution of (B) gives white ppt. with small


concentration of KCN which becomes soluble in excess
S (NH ) S HCl
of this reagent forming a compound (D). X  4 2
→ Brown powder → Solution 
→ Grey ppt.
Y
(v) The solution of (D) on treatment with H2S gives (A).
air i)NaOH HCl
X  → White powder  → Solution 
→ Gelatinous ↓
(vi) The solution of (B) in dil. HCl on treatment with a heat ii)hot water

Z(soluble only white


solution of BaCl2 gives a white ppt. of compound (E)
in conc .H2SO 4 )
which is insoluble in conc. HNO3.
Identify (A) to (E) and give chemical equations for the
reactions at steps (a) to (c) to (f) (i) Solution of X (with HCl) reacts with NaOH and forms
white ppt.This ppt dissolves in excess of NaOH to
Sol: Yellow ppt. of CdS is (A) give solution which has strongly reducing nature. This
reducing properties of this solution points out that the
(i) CdSO 4 + H2S → CdS + H2SO 4
(a)
solution might be containing sodium stannite and here
(B) (A) X must be tin.

(ii) CdS + 2HNO3 → Cd(NO3 )2 + H2S


(b) (ii) The nature of X as tin is confirmed by its reaction with
S forming SnS2 which dissolves in yellow ammonium
soluble
sulphide but regenerates in presence of HCl.
CdS → Insoluble in yellow ammonium sulphide. Thus the various reactions and nature of X, Y, and Z can
be written as below.
CdSO 4 + 2NH4 OH → Cd(OH)2 + (NH4 )2 SO 4
(i) Sn+ 2HCl → SnCl2 + H2
(iii)
(c) Cd(OH)2 + 4NH4 OH →[Cd(NH3 )4 ](OH)2 + 4H2O (X)

(C) SnCl2 + 2NaOH → Sn(OH)2 + 2NaCl

(iv)
(d) CdS O2 + 2KCN → Cd(CN)2 + K 2 SO 4 Sn(OH)2 + 2NaOH → Na2SnO2 + 2H2O
white Sodium stannite
(Strongly reducing)

Cd(CN)2 + 2KCN → K 2 [Cd(CN)4 ]


(ii) Sn + 2S → SnS2
(D)soluble
SnS2 + (NH4 )2 S →(NH4 )2 SnS3
(v) K 2 [Cd(CN)4 ] + H2S → CdS + 2KCN + 2HCN
(e)
(vi)
(f) CdSO 4 + BaCl2 → BaSO 4 + CdCl2 (NH4 )2 SnS3 + 2HCl → SnS2 + 2NH4 Cl + H2S
(E)
3 0 . 4 8 | Qualitative Analysis

(iii) Sn+ O2 → SnO2 (Soluble only in conc. H2SO4) (Remember: NO3− ions are not attacked by dil. Acids.).
(Z)
Consequently, A must be compound of NO3− which can
SnO2 + 2NaOH → Na2SnO3 + H2O give NO3− (B) on heating.

Na2SnO3 + 2HCl → H2SnO3 ↓ +2NaCl Thus compound A is NaNO3.


Stannic acid Reactions are as follows:

Example 7: A colourless solid A on heating gives (i) 2NaNO3 → 2NaNO2 + N2 ↑


(A) (B) (C)
a white solid B and a colourless gas, C; B gives off
(ii)
(ii)2NaNO2 + H2SO 4 (dil.) → NaSO 4 + 2HNO2
reddish brown fumes on treatment with dilute acids. (A)
On heating with NH4Cl, B gives a colourless gas D and
3HNO2 → HNO3 + H2O + 2NO
a residue E. The compound A also gives a colourless
gas F on heating with ammonium sulphide and white
residue G. Both E and G impart bright yellow colour 3NO + O2 → 2NO2 ↑
Re ddish brown fumes
to Bunsen flames. The gas C forms white powder with
strongly heated magnesium metal. The white powder (iv) 2NaNO3 + (NH4 )2 SO 4 → Na2SO 4 + 2NH3 ↑+ 2HNO3
forms magnesium hydroxide with water. The gas D, on (A) (G) (F)
the other hand, is absorbed by heated calcium which
gives off ammonia on hydrolysis. Identify the substance heat
(iii) NaNO2 + NH4 Cl  → NaCl + N2 ↑+ 2H2O
A to G and gives reactions for the changes involved. (A) (E) (D)

Sol: It is advisable to summarize the given facts in the (iv) 2NaNO2 + (NH4 )2 SO 4 → Na2SO 4 + 2NH3 ↑+ 2HNO3
form of a chart. (A) (G) (F)

(NH4 )2 SO 4
A → G + F
Heat Colourless residue Colourless gas (v) 2Mg + O2 → 2MgO
(C) White powder
Heat
A  → B + C
Colourless solid Colourless solid Colourless gas
MgO + H2O → 2Mg(OH)2
NH Cl
4 →
B  D + E
Heat Colourless residue Residue (vi) 3Ca + N2 → Ca3 N2
Dil. HCl (D)
B 
→ Reddish brown fumes
Ca3 N2 + 6H2O → 2Ca(OH)2 + 2NH3 ↑
2 → Mg(OH) H O
C + Mg → White powder  2
Colourless residue
Thus substance (A) to (G) can be represented as
H2O
D + Ca (Heated)  → NH3 (A) NaNO3 (B) NaNO2 (C) O2
Colourless residue

(D) N2 (E) NaCl (F) NH3 (G) Na2SO4


The above reactions lead to the following conclusions.
(A) And (D) on reaction with calcium forms a compound Note: *** Subquestions placed after the answers
which on hydrolysis gives ammonia, this indicates that
D must be nitrogen.
Compound (B), Residues E and G burns with yellow
flame this indicates that these are sodium salts. Hence
compound B (which give E) and A (which give G) must
be sodium salts.
(C) The colourless solid B with dilute acid gives reddish
brown fumes, the reddish brown fumes are probably
of NO2.
Hence compound B must be nitrite
Chem i str y | 30.49

JEE Main/Boards

Exercise 1 Q.7 Explain, while performing qualitative analysis of


basic radicals of third group, why ammonium chloride is
added in excess before adding ammonium hydroxide?
Q.1 Sometimes, a white ppt. is obtained even in the
absence of members of Ist group on the addition of
HCl. Explain it. Q.8 What happens when
(i) Copper sulphate is treated with excess of NH4OH
Q.2 Give examples and explain with equations:
(ii) Bismuth chloride is treated with sodium stannite in
(i) Two colourless solution give a black ppt. on mixing. presence of NaOH
(ii) Two colourless solution give a red ppt. on mixing, (iii) Stannous chloride is treated with mercuric chloride
soluble in excess of one of them.
(iv) Excess of water is added to concentrated solution of
(iii) Two colourless solutions give a white ppt. on mixing, antimony chloride
soluble in ammonium hydroxide.
(iv) Two colourless solution give a yellow ppt. on mixing. Q.9 (i) What is the function of concentrated HNO3 in
third group?
Q.3 What is yellow ammonium sulphide? Why is yellow (ii) Will you add HNO3 in third group even if iron is
ammonium sulphide and not ordinary ammonium given in ferric state in the mixture?
sulphide used for the separation of II A and II B sub-
(iii) Can you use NaCl and NaOH instead of NH4Cl and
groups?
NH4OH in third group?

Q.4 A certain inorganic compound (X) shows the


Q.10 Identify compounds A to G from the following
following reactions:
reactions
(i) On passing H2S through an acidified solution of (X) a heat KI
A (white crystals)  → B + C ; C  → I2 A
brown ppt. is obtained. solidsolution
gas

(ii) The ppt. obtained at step (i) dissolve in excess of HNO NH


NaOH
3 → Solution D  3→
yellow ammonium sulphide. B  → E ↓ 
brown
(iii) On adding an aqueous solution of NaOH to a SolutionF 
H CO OH
→ Black precipitate
solution of (X), first a white ppt. is obtained which
dissolves in excess of NaOH. Q.11 Complete the following
(iv) The aqueous solution of (X) reduce ferric chloride.
Acid
(i) PbS+ Acid → Gas → Yellow ppt.
Identify the cation of (X) and give chemical equations (A) C (B) (D)
for reactions at steps (i), (iii) and (iv) NH4 OH
(ii) A + H2 S  → White ppt.+ 2HCl
(B)
Q.5 A mixture of the three gases A, B and C is passes NaOH
first into an acidified dichromate solution when A is A + NaOH 
(C)
→ ppt.  → solution
(D)
absorbed turning the solution green. The remainder
of the gas is passed through an excess of lime water heat in air
(iii) Pbs  B
→ A + PbS  → Pb + SO2
which turns milky, resulting in the absorption of B. The
residual gas C is absorbed by an alkaline pyrogallol
solution. However, the original gaseous mixture does Q.12 Explain the following:
not turn lead acetate paper black. Identify A, B and C. (i) Lead (Pb2+) is placed in the first as well as second
group of qualitative analysis.
Q.6 You are given unlabelled four packets of white
(ii) The colour of mercurous chloride, Hg2Cl2 , change
substance of zinc, namely ZnO, Zn(OH)2, ZnCO3 and
from white to black when treated with ammonia.
ZnS. How will you proceed to identify each of them?
3 0 . 5 0 | Qualitative Analysis

(iii) During the qualitative analysis of a mixture Q.16 A certain metal (A) is boiled in dilute nitric acid to
containing Cu2+ and Zn2+ ions, H2S gas is passes give a salt (B) and an oxide of nitrogen (C). An aqueous
through an acidified solution containing these ions in solution of (B) with brine gives a ppt. (D) which is
order to test Cu2+ alone. Explain briefly. solution in ammonium hydroxide. On adding aqueous
solution of (B) to hypo solution, a white ppt. (E) is
Q.13 Identify the unknown species and complete the obtained. (E) on standing turns to a black compound
following: (F). Identify (A) to (F).

(i) (A)+BaCl2 → White ppt.


Q.17 A yellow solid (A) is unaffected by acids and bases.
(ii) NaOH+(B) → NH3 gas It is not soluble in water. It dissolves slowly in hot conc.
HNO3 and a brown gas (B) is released. The solid (A)
(iii) (C)+MnO2+H2SO4 → Violet vapours dissolves only in a boiling solution of sodium sulphite
giving a clear solution (C). Acidification (C) causes a
(iv) (D)+K2Cr2O7+H2SO4 → Green coloured solution colourless gas (D) to be liberated, accompanied by an
appearance of a milky ppt. (E) in the solution. Identify
(v) (E) Heat
 → Yellow compound (A) to (E).
Colourless
solid
Q.18 State, whether the following statements are true
or False:
Q.14 Fill in the blanks
(a) AgCl dissolve in NH4OH.
(a) Lime water is used for the test of…………
(b) Sb2S3 is yellow in colour.
(b) Acetates are………… in water.
(c) Copper suplhate forms a violet colour with potassium
(c) Nitrates when treated with conc. H2SO4 evolve………….
ferrocyanide solution.
gas.
(d) Both phosphate and arsenic ions give yellow
(d) Chromyl chloride test is performed for……….
ppt. when heated with nitric acid and ammonium
(e) Chlorides of …………… are insoluble in dilute HCl. molybdate.
(f) H2S is used as a group reagent in……… group in………. (e) Addition of ammonium chloride to a sodium
medium while in……………. group in……………medium. containing ferric and magnesium ions is essential for
(g) HgS is……………… in dilute HNO3 selective precipitation of ferric hydroxide by aqueous
ammonia.
(h) Yellow ammonium sulphide dissolves sulphides
of……………. (f) Ammonium sulphate can be used in place of
ammonium chloride in third group.
(i) Group reagent for third group radicals is…………
(g) Iodine is liberated when an iodine in heated with
( j) If metal ions of group III are ppt. by NH4Cl medium
conc. H2SO4.
and NH4OH without prior oxidation by concentrated
HNO3 …………….is not completely ppt.. (h) It is not necessary to use HNO3 in the third group if
ferric compound is given in a mixture.
(k) colour of zinc sulphide is …..
(l) In V group, K2CrO4 is used for the test of…………. (i) In the test of acetate radical, neutral ferric chloride
is used.
(m) ……………… ppt. is formed when Na2HPO4 is added
to magnesium salt in presence of NH4OH. ( j) Nessler’s reagent is the alkaline solution of K2HgI4.

(n) Ammonium thiocyanate is used in the detection (k) The solubility product of cadmium sulphide is
of………….. highest amongst the sulphides of second group.
(l) Cobalt can be tested with dimethyl glyoxime.
Q.15 A compound on heating with an excess of caustic
(m) Cobalt salt with KNO2 and acetic acid gives yellow
soda solution liberates a gas (B), which gives white
ppt..
fumes on exposure to HCl. Heating is continued to
expel the gas completely. The resultant alkaline solution (n) The carbonates of barium, strontium and calcium
again liberates the same gas (B), when heated with zinc are soluble in acid.
powder. However, the compound (A), when heated
alone, does not give nitrogen. Identify (A) and (B).
Chem i str y | 30.51

Q.19 Explain with proper reasoning. Exercise 2


(a) The aqueous solution of ferric chloride can not be
stored. It is always acidified with hydrochloric acid. Single Correct Choice Type

(b) The aqueous solution of FeCl3, possesses yellow Q.1 Which of the following gives a suffocating gas
colour. The colour becomes green on passing H2S gas. when treated with dilute HCl?
(c) The aqueous solution of K2Cr2O7, is orange. On (A) Carbonate (B) Sulphite
adding an alkali, it turns yellow.
(C) Sulphate (D) Borate
(d) In the test of oxalate, the evolved gas burns with
blue flame inly initially.
Q.2 The acidic solution of a salt produces blue colour
(e) Why yellow ammonium sulphide is used in group (II) with KI starch solution. The salt may be
sulphides separation?
(A) Sulphite (B) Bromide
(f) Why zinc sulphide is not ppt. when H2S is passed
(C) Nitrite (D) Chloride
through ZnCl2 solution.
(g) CaSO4 is insoluble but it is not ppt. when excess of Q.3 Sulphite on treatment with dil, H2SO4 liberates a
(NH4)2SO4 is added to CaCl2. gas which
(h) Why (NH4Cl+NH4OH) and not [(NH4)2SO4+NH4OH] (A) Turns lead acetate paper black
is used in group (III) analysis?
(B) Burns with blue flame
(i) Why is it necessary to added few drops of conc.
HNO3 to the filtrate of group (II) before the use of (C) Smells like vinegar
NH4Cl+NH4OH. (D) Turns acidified K2Cr2O7 paper green
( j) Why NaOH cannot be used to separate Al(OH)3 and
Zn(OH)2? Q.4 A gas is evolved which burns with blue flame when
the mixture is heated with conc. H2SO4. The mixture
Q.20 A certain salt (X) gives the following tests: contains.

(i) Its aqueous solution is alkaline to litmus. (A) Carbonate (B) Oxalate

(ii) On strongly heating it swells to give glassy material. (C) Nitrate (D) Nitrite

(iii) When concentrated H2SO4 is added to a hot


concentrated solution. Q.5 Mercurous chloride turns black on treatment with
ammonium hydroxide. This is due to the formation of
Identify the salt (X) and give the equations for the
reactions. (A) Hg(NH2)Cl (B) Hg2Cl2.NH4OH
(C) Hg and HgNH2Cl (D) HgCl2.NH4OH
Q.21 An aqueous solution of a gas (X) shows the
following reactions. Q.6 Bromine vapours turn starch iodide paper
(i) It turns red litmus blue. (A) Violet (B) Blue
(ii) When added in excess to copper sulphate solution (C) Yellow (D) Red
deep blue colour is obtained.
(iii) On addition to a ferric chloride solution a brown ppt. Q.7 A mixture when heated with dil. H2SO4 does not
soluble in dilute nitric acid is obtained. Identify (X) and evolve brown vapours but when heated with conc.
give equations for the reactions at steps (ii) and (iii). H2SO4, brown vapours are obtained. The vapours when
brought in contact with silver nitrate solution do not
give any ppt.. The mixture contains.
(A) NO2− (B) NO3−
(C) Cr (D) Br −
3 0 . 5 2 | Qualitative Analysis

Q.8 Ammonium dichromate is used in some fireworks. Q.16 When dimethyl glyoxime solution is added to
The green coloured powder blown in air is due to an aqueous solution of nickel (II) chloride followed by
ammonium hydroxide:
(A) CrO3 (B) Cr2O3 (C) Cr (D) CrO(O2)
(A) No ppt. is obtained
Q.9 A mixture, on heating with conc. H2SO4 and MnO2, (B) A blue coloured ppt. is obtained
liberates brown vapours of
(C) A red coloured ppt. is obtained
(A) Br2 (B) NO2 (C) HBr (D) I2
(D) A black coloured ppt. is obtained

Q.10 A white solid is first heated with dil. H2SO4 and


Q.17 An organic salt when heated evolves a coloured
then with conc. H2SO4. No action was observed in
gas which bleaches moist litmus paper. The evolved gas
wither case. The solid salt contains.
is
(A) Sulphide (B) Sulphite
(A) NO2 (B) Cl2 (C) Br2 (D) I2
(C) Thiosulphate (D) Sulphate
Q.18 Which of the following metal oxides is white in
Q.11 A light yellow ppt. is formed in the second group colour but becomes yellow in heating?
of the qualitative analysis on passing H2S even when
(A) AgO (B) Ag2O (C) FeO (D) ZnO
no radical of second group is present. This is due to
presence of …………… in the mixture:
Q.19 A white ppt. obtained in the analysis of a mixture
(A) Phosphate (B) Acetate
becomes black on treatment with NH4OH. It may be.
(C) Oxalate (D) Nitrate
(A) PbCl2 (B) AgCl (C) HgCl2 (D) Hg2Cl2

Q.12 On adding water to BiCl3 solution in HCl, the


Q.20 Which one among the following soluble in excess
compound formed is
of NaOH?
(A) Bi2O3 (B) Bi(OH)3
(A) Fe(OH)3 (B) Al(OH)3
(C) BiOCl (D) BiOCl2
(C) Cr(OH)3 (D) Mn(OH)2

Q.13 The sulphide which is insoluble in 30% HNO3 is


Q.21 Which compound does not dissolve in hot dil.
(A) HgS (B) CuS (C) PbS (D) CdS HNO3
(A) HgS (B) PbS (C) CuS (D) CdS
Q.14 NiS is separated from ZnS by treating with
(A) NaOH Q.22 An aqueous solution of FeSO4Al2(AO4)3 and
chrome alum is heated with excess of Na2O2 and
(B) Conc.HCl
filtered. The materials obtained are
(C) Yellow ammonium sulphide
(A) A colourless filtrate and a green residue
(D) Aqua-regia
(B) A yellow filtrate and green residue

Q15.Soda extract is prepared by (C) A yellow filtrate and a brown residue

(A) Fusing soda and mixture and then extracting with (D) A green filtrate and a brown residue
water
Q.23 All ammonium salts liberate ammonia when
(B) Dissolving NaHCO3 and misture in dil. HCl
(A) Heated
(C) Boiling Na2CO3 and mixture in dil. HCl
(B) Heated with caustic soda
(D) Boiling Na2CO3 and mixture in distilled water
(C) Heated with H2SO4
(D) Heated with HaNO2
Chem i str y | 30.53

Q.24 One of the following compounds gives a white Q.31 Concentrated aqueous sodium hydroxide can
ppt. with aqueous AgNO3 and a green flame test. separate a mixture of
(A) NaCl (B) KCl (C) BaCl2 (D) CaCl2 (A) Al3+ and Sn2+ (B) Al3+ and Fe3+
(C) Al3+ and Zn2+ (D) Zn2+ and Pb2+
Q.25 Which one of the following pairs of ions cannot
be separated by H2S in dilute hydrochloric acid?
Q.32 Potassium ferricyanide [potassium hexacyano
(A) Bi3+, Sn4+ (B) Al3+, Hg2+ ferrate (III)] has
(C) Zn2+Cu2+ (D) Ni2+, Cu2+ (A) Fe(II) (B) Fe(III) (C) Cu(II) (D) Cd(II)

Q.26 When H2S is passes through an ammonium salt Q.33 Which of the following sulphate is insoluble in
solution X, a white ppt is obtained. The X can be water?
(A) Cobalt salt (B) Nickel salt (A) CuSO4 (B) CdSO4 (C) PbSO4 (D) Bi2(SO4)3
(C) Manganese salt (D) Zinc salt
Q.34 Calcium burns in nitrogen to produce a white
powder which dissolve in sufficient water to produce
Q.27 The best explanation for the solubility of MnS in
a gas (A) and an alkaline solution. The solution on
dil. HCl in salt
exposure to air produces a thin solid layer of (B) on the
(A) Solubility product of MnCl2 is less than that of MnS surface. Identify the compounds A and B.
(B) Concentration of Mn2+ is lowered by the formation (A) C2H2, CaCO3 (B) NH3, CaCO3
of complex ions with chloride ions.
(C) NH3, Ca(OH)2 (D) CH4, CaCO3
(C) Concentration of sulphide ions is lowered by
oxidation to free sulphur.
Q.35 A gas ‘X’ is passed through water to form a
(D) Concentration of sulphide ions is lowered by saturated solution. The aqueous solution on treatment
formation of the weak acid H2S with silver nitrate gives a white ppt.. The saturated
aqueous solution also dissolve magnesium ribbon with
Q.28 A white solid is first heated with dilute H2SO 4 the evolution of a colourless gas ‘Y’. Identify ‘X’ and ‘Y’.
and when with concentrated. No action is observed in (A) X=CO2, Y=Cl2 (B) X=Cl2, Y=CO2
either case. The solid contains.
(C) X=Cl2, Y=H2 (D) X2=H2, Y=Cl2
(A) Sulphide (B) Sulphide
(C) Sulphate (D) Thiosulphate
Previous Years' Questions
Q.29 The salt used for performing ‘bead test’ in
Q.1 A gas ‘X’ is passed through water to form a
qualitative inorganic analysis is
saturated solution. The aqueous solution on treatment
(A) K2SO4.Al2(SO4)3.24H2O with silver nitrate gives a white ppt.. The saturated
aqueous solution also dissolves magnesium ribbon
(B) FeSO4.(NH4)2SO4.6H2O
with evolution of a colourless gas ‘Y’. Identify ‘X’ and ‘Y’.
(C) Na(NH4)HPO4.4H2O  (2002)
(D) CaSO4.2H2O
Q.2 [X]+H2SO4 → [Y] a colourless gas with irritating
Q.30 The only cation present in a slightly acidic solution smell [Y]+K2Cr2O7+H2SO4 → green solution [X] and
are Fe3+, Zn2+ and Cu2+. The reagent which when added [Y] are (2003)
in excess to this solution would identify and separate (A) SO32− ,SO2 (B) C l− , HCl
Fe3+ in one step is
(C) S2-, H2S (D) CO32− ,CO2
(A) 2MHCl (B) 6 M NH3
(C) 6 M NaOH (D) H2S gas
3 0 . 5 4 | Qualitative Analysis

Q.3 A sodium salt of an unknown anion when treated with Statement-II: Solubility product of CdS is more than
MgCl2 gives white ppt. only on boiling. The anion is (2004) that of NiS.  (1989)
(A) SO24− (B) HCO3− (C) CO32− (D) NO3−
Q.8 Statement-I: Sulphate is estimated as BaSO4, not
as MgSO4.
Q.4 (NH4)2Cr2O7 on heating gives a gas is also given by
(2004) Statement-II: Ionic radius of Mg2+ is smaller than that
of Ba2+ (1998)
(A) Heating NH4NO2 (B) Heating NH4NO3
(C) Mg3N2+H2O (D) Na(comp.)+ H2O2 Q.9 An aqueous solution FeSO4. Al2(SO4)3 and chrome
alum is heated with excess of Na2O2 and filtered. The
Q.5 A metal nitrate reacts with KI to give a black ppt. materials obtaining are (1996)
which on addition of excess of KI convert into orange (A) A colourless filtrate and a green residue
colour solution. The cation of metal nitrate  (2005) (B) A yellow filtrate and a green residue
(A) Hg 2+
(B) Bi
3+
(C) Sm 2+
(D) Pb 2+
(C) A yellow filtrate and a brown residue
(D) A green filtrate and brown residue
Q.6 A solution when diluted with H2O and boiled, it
gives a white ppt. On addition of excess NH4Cl/NH4OH, Q.10 In nitroprusside ion the iron and NO exist as Fe (II)
the volume of ppt. decreases leaving behind a white and NO+ rather than Fe(III) and NO. These forms can be
gelatinous ppt. Identify the ppt which dissolves in differentiated by (1998)
NH4OH/NH4Cl. (2002)
(A) Estimating the concentration of iron
(A) Zn(OH)2 (B) Al(OH)3
(B) Measuring the concentration of CN
(C) Mg(OH)2 (D) Ca(OH)2 (C) Measuring the solid state magnetic moment
(A) If both assertion and reason are true and reason is (D) Thermally decomposing the compound
the correct explanation of assertion, then mark (a)
Q.11 An aqueous solution if a substance gives a white
(B) If both assertion and reason are true but R is not the
ppt. on treatment with dilute hydrochloride acid, which
correct explanation of assertion, then mark (b)
dissolves on heating. When hydrogen sulphide is
(C) If assertion is true but reason is false, then mark (c) passed through the hot acid solution, a black ppt. is
(D) If both assertion and reason are false, then mark (d) obtained. The substance is a  (2000)
(A) Hg32+ salt (B) Cr 2+ salt
Q.7 Statement-I: A very dilute acidic solution of Cd2+
(C) Ag+ salt (D) Pb2+ salt
and Ni2+ gives yellow ppt. of CdS on passing H2S.

JEE Advanced/Boards

Exercise 1 (iv) An acidic solution of (X) does not give a ppt. with
H2S. Identify (X) and give chemical equations for
reactions at steps (i) to (iii).
Q.1 An inorganic Lewis acid(X) shows the following
reactions:
Q.2 An aqueous solution of salt (A) gives white ppt.
(i) It fumes in moist air.
(B) with NaCl solution. The filtrate gives black ppt., (C)
(ii) The intensity of fumes increases when a rod dipped when H2S is passed into it. Compound (B) dissolves
in NH4OH is brought near to it. in hot water and the solution gives a yellow ppt., (D)
on treatment with sodium iodide and cooling. The
(iii) An acidic solution of (X) on addition of NH4Cl and
compound (A) does not give any gas with dilute HCl
NH4OH gives a ppt. which dissolves in NaOH solution.
but liberates a reddish-brown gas on heating. Identify
the compounds (A) to (D).
Chem i str y | 30.55

Q.3 A mixture of two salts was treated as follows: (iii) (A) on treatment with potassium cyanide gives a
buff colored ppt. which dissolves in excess of this
(a) The mixture was heated with manganese dioxide
reagent forming a compound (C).
and concentrated H2SO4, when yellowish green gas was
liberated. (iv) The compound (C) is changed into a compound (D)
when its aqueous solution boiled.
(b) The mixture on heating with NaOH solution gave a
gas which turn red litmus blue. (v) The solution of (A) was treated with excess of sodium
bicarbonate and then with bromine water. On cooling
(c) Its solution in water gave blue ppt. with potassium
and shaking for same time, a green colour of compound
ferricyanide and red colouration with NH4CNS.
(E) is formed. No change is observed on heating.

Q.4 An unknown inorganic compound (X) loses its Identify (A) to (E) and give chemical equations for the
water of crystallization on heating and its aqueous reactions at steps (a) to (e).
solution gives the following reactions.
Q.8 A colourless solid (A) on hydrolysis produces a
(i) It gives a white turbidity with dilute hydrochloric acid
heavy white ppt. (B). Solid (A) gives a clear solution
solution.
in conc. HCl; however, when added to large amount
(ii) It decolourises a solution of iodine in potassium of water, it again gives ppt. (B). When H2S is passed
iodide. through a suspensions of (A) or (B), a brown black ppt.
(iii) It gives white ppt. with silver nitrate solution which of (C) is obtained. Compound (A) liberates a gas (D) on
turns black on standing. treating with H2SO4. The gas (D) is water soluble and
gives white ppt. (E) with solution of mecurous salt but
not with mercuric salt. Identify (A) to (E). Also report (A),
Q.5 Identify the compound (X) and give chemical (B), (C) if (C) is orange ppt..
equations for the reactions at steps (i), (ii) and (iii). A
certain compound (X) shows the following reactions.
Q.9 Identify the unknown species and complete the
(i) When KI is added to an aqueous suspension of (X) following
containing acetic acid, iodine is liberated. Heat
(i) (A) + NaOH  → NaCl + NH3 + H2O .
(ii) When CO2 is passed through an aqueous suspension
of (X) the turbidity transforms to a ppt.. (ii) NH3 + CO2 + H2O → (B).

(iii) When a paste of (X) in water is heated with ethyl (iii) (B) + NaCl →(C) + NH4Cl.
alcohol a product of anesthetic use is obtained. Identify Heat
(iv) (C)  → Na2CO3 + H2O + (D) .
(X) and write down chemical equation for reactions at
steps (i), (ii) and (iii).
Q.10 Element A burns in nitrogen to give an ionic
compound B. Compound B reacts with water to give
Q.6 Identify the unknown species and complete the C and D. A solution of C becomes ‘milky’ on bubbling
following carbon dioxide. Identify A, B, C and D.
(i) (A) + dil.H2SO4 + K2Cr2O7 → (B) green coloured
Q.11 An inorganic compound (A) in its aqueous
(A) + dil.H2SO4 + (C) → (B) MnSO4
solution produced a white ppt. With NaOH, which gets
H O
2 → (D)
(A) + O2  dissolved in excess of NaOH. The aqueous solution of

(A) also produced white ppt. With NH4OH which also
(D) + BaCl2 → White ppt.
dissolved in excess of NH4OH. Also its aqueous solution
Heat
(ii) (A)aq. + Zn  → (B)gas produced light yellow ppt. with AgNO3 solution, soluble
Heat in dil. HNO3, identify (A).
(A)aq. + (C)  → PH3

Heat
(A)aq. + NH4 Cl  → (D)gas
Q.12 (i) An aqueous solution of a compound (A) is acidic
towards litmus and (A) is sublimed at about 300° C .
Q.7 (i) A black coloured compound (B) is formed on
(ii) (A) on treatment with an excess of NH4SNC gives a red
passing hydrogen sulphide through the solution of a
coloured compound (B) and on treatment with a solution
compound (A) in NH4OH.
of K4[Fe(CN)6] gives a blue coloured compound (C).
(ii) (B) on treatment with hydrochloric acid and KCIO3 (iii) (A) on heating with excess of K2Cr2O7 in presence of
gives (A). concentrated H2SO4 evolves deep red vapours of (D).
3 0 . 5 6 | Qualitative Analysis

(iv) On passing the vapours of (D) into a solution of (iv) On adding a solution of (A) into the solution of
NaOH and then adding the solutions of acetic acid and cupric chloride, a white ppt. is first formed which
lead acetate, a yellow ppt. of compound (E) is obtained. dissolves on adding excess of (A) forming a compound
(E). Identify (A) to (E) and give chemical equations for
Identify (A) to (E) and give chemical equations for the
the reactions at steps (i) to (iv).
reactions at steps (ii) to (iv).

Q.17 A metal chloride (X) shows the following reactions:


Q.13 An aqueous solution of gas (X) gives the following
reactions. (i) When H2S is passed in an acidified aqueous solution
of (X), a black ppt. is obtained.
(i) It decolourizes on acidified K2Cr2O7 solution.
(ii) The ppt. obtained at step (i) is not soluble in yellow
(ii) On boiling it with H2O2, cooling it and then adding
ammonium sulphide.
an aqueous solution of BaCl2 a ppt. insoluble in dilute
hydrochloric acid is obtained. (iii) When a solution of stannous chloride is added to an
aqueous solution of (X) a white ppt. is obtained which
(iii) On passing H2S in the solution, a white turbidity is
turns grey on addition of more of stannous chloride.
obtained.
(iv) When an aqueous solution of KI is added to an
Identify (X) and gives equations for the reactions at
aqueous solution of (X) a red ppt. is obtained which
steps (i),(ii) and (iii).
dissolves on addition of excess of KI.

Q.14 A colourless solid A, when placed into water, Identify (X) and write down the equations for the
produces a heavy white ppt. B. Solid A gives a clear reactions at steps (i), (iii) and (iv).
solution in conc. HCl; however when added to large
amount of water, it again gives ppt. of B which dissolves Q.18 A well known orange crystalline compound (A)
in dilute HCl. When H2S is passed through the suspension when burnt imparts violet colour of fame. (A) on treating
of A or B, a brown black ppt. (C) is obtained. Compound with (B) and conc. H2SO4 gives red gas (C) which gives
A liberates a gas D with conc. H2SO4. The gas D is water red yellow solution (D) with alkaline H2SO4 gives red
soluble and gives white ppt. E with solution of mercurous gas (C) which gives red yellow solution (D) with alkaline
salts but not with mercuric salts. Identify A to E. water. (D) on treating with acetic acid and lead acetate
gives yellow ppt. (E). (B) sublimes on heating. Also on
Q.15 (i) A blue coloured compound (A) on heating heating (B) NaOH, gas (F) is formed which gives white
gives two products, (B) and (C). fumes with HCl. What are (A) to (F)?

(ii) A metal (D) is deposited on passing hydrogen


Q.19 Complete and balance the following chemical
through heated (B).
equations:
(iii) The solution of (B) in HCl on treatment with
(i) Au + HCl + HNO3 → ....... + ....... + H2O
K4Fe(CN)6 gives a chocolate brown coloured ppt. of
compound (E). (ii) C + HNO3 (conc.) → CO2 + ....... + H2O
(iv) (C) turns lime water milky which disappears on (iii) Sn + KOH(hot) + H2O → ....... + .......
continuous passage of (C) forming a compound (F).
(iv) Cu(OH)2 + NH4NO3 + NH4 OH(aq) → ....... + H2O
Identify (A) to (F) and give chemical equations for the
reactions at steps (i) to (iv).
Q.20 A gaseous mixture containing (X), (Y) and (Z)
gases, when passed into acidified K2Cr2O7 solution, gas
Q.16 (i) An inorganic compound (A) is formed on
(X) was absorbed and the solution was turned green.
passing a gas (B) through a concentrated liquor
The remainder gas mixture was then pass through
containing sodium sulphide and sodium sulphite.
lime water, which turns milky by absorbing gas (Y). The
(ii) On adding (A) in to a dilute solution of silver nitrate, residual gas when passed through alkaline pyrogallol
a white ppt. appears which quickly changes into black solution, it turned black. Identify gas (X), (Y) and (Z) and
coloured compound (C). explain the reaction involved.
(iii) On adding two or three drops of ferric chloride into
the excess of solution of (A), a violet coloured compound
(D) is formed. This colour disappears quickly.
Chem i str y | 30.57

Exercise 2 Q.7 Assertion: In the analysis of group III-radicals


NaOH can also be used as group reagent.
Single Correct Choice Type
Reason: NaOH can be used only in the presence of
Q.1 Which compound does not dissolve in hot dil. NaCl as ppt. agent for group III.
HNO3 ?
Q.8 Assertion: In charcoal cavity test in intimate mixture
(A) HgS (B) PbS (C) CuS (D) CdS of salt and Na2CO3 is heated on a charcoal block.
Reason: Charcoal cavity test is meant only for coloured
Q.2 An aqueous solution of FeSO4Al2(SO4)3 and chrome
salts
alum is heated with excess of Na2SO3 and filtered. The
materials obtained are
Q.9 Assertion: Match-stick test is meant for all sulphur
(A) A colorless filtrate and a green residue containing radicals.
(B) A yellow filtrate and a green residue Reason: Match-stick test is not given by Na2S.
(C) A yellow filtrate and a brown residue
(D) A green filtrate and a brown residue Q.10 Assertion: CdS and As2S3 both have yellow colour.
Reason: CdS and As2S3 can be separated by yellow
Q.3 Magnesium carbonate does not ppt. with the ammonium sulphide.
carbonates of group V radicals in presence of NH4OH
and NH4Cl because Comprehension Type
(A) MgCO3 is soluble in water. The following observation were made on Na2CrO4 and
(B) MgCO3 is soluble in NH4OH Na2Cr2O7.

(C) MgCO3 is soluble in NH4Cl (A) When CO2 was passed over Na2CrO4, then Na2Cr2O7
was formed.
(D) MgCO3 is soluble in (NH4)2CO3
(B) When Zn is added to acidic solution of Na2Cr2O7, the
colour changes from orange to green, then to blue and
Q.4 The extent of splitting in d-orbitals is more when then back to green.
the chromium in the solution is in
(C) Na2CrO4 when added to a nitrate salt solution gave a
(A) +1 oxidation state (B) +2 oxidation state yellow coloured ppt. which after separation and drying
(C) +3 oxidation state (D) +6 oxidation state followed by flame test gave a green coloured flame.

Assertion Reasoning Type Q.11 What is the function of CO2 in the first observation?
(A) If both assertion and reason are true and reason is (A) Acts as an oxidising agent
the correct explanation of assertion, then mark (A) (B) Acts as a reducing agent
(B) If both assertion and reason are true but reason is (C) Produces chromium and oxygen
not the correct explanation of assertion, then mark (B) (C) Makes the solution acidic
(C) If assertion is true but reason is false, then mark (C)
(D) If both assertion and reason are false, then mark (D) Q.12 The reason for the colour Na2Cr2O7 solution to first
change from orange to green on adding Zn is because
Q.5 Assertion: Ammonium phosphomolybdate is a
yellow coloured ppt. (A) Zn is reducing agent and changes Cr+4 to Cr+3
(B) Zn is a reducing agent and changes Cr+6 to Cr+3
Reason: Yellow colour of compound is due to
ammonium ions. (C) Zn is a reducing agent and it reduces Cr+6 to Cr+2
(D) None of these
Q.6 Assertion: Ring test for nitrates is performed from
the water extract of the salt. Q.13 The second change in colour in the solution that
Reason: All nitrates are generally soluble in water. is from green to blue is due to the conversation of
(A) Cr+3 to Cr+1 (B) Cr+3 to Cr
(C) Cr+3 to Cr+2 (D) Cr+3 to Cr+4
3 0 . 5 8 | Qualitative Analysis

Q.14 The nitrate salt which gives a yellow ppt. with Paragraph 1: p- amino –N, N- dimethylaniline is added
Na2CrO4 and yellow ppt. gives green flame with bunsen to a strongly acidic solution of X. The resulting solution
burner is treated with a few drops of aqueous solution of Y to
yield blue colouration due to the formation of methylene
(A) Pb2+ (B) Ca2+ (C) Mg2+ (D) Ba2+
blue. Treatment of the aqueous solution of Y with the
reagent potassium hexacyanoferrate (II) leads to the
formation of an intense blue ppt.. The ppt. dissolves
Previous Years' Questions on excess addition of the reagent. Similarly, treatment
Q.1 MgSO4 on reaction with NH4OH and Na2HPO4 forms of the solution of Y with the solution of potassium
a white crystalline ppt. What is its formula?  (2006) hexacyanoferrate (III) leads to a brown colouration due
to formation of Z.
(A) Mg(NH4)PO4 (B) Mg3(PO4)2
(C) MgCl2 . MgSO4 (D) MgSO4 Q.7 The compound X, is (2009)
(A) NaNO3 (B) NaCl (C) Na2SO4 (D) Na2S
Q.2 A solution of metal ion when treated with KI gives
a red ppt. which dissolves in excess KI in excess KI to
give a colourless solution. Moreover, the solution of Q.8 The compound Y, is  (2009)
metal ion on treatment with a solution of cobalt (II) (A) MgCl2 (B) FeCl2 (C) FeCl3 (D) ZnCl2
thiocyanate gives rise to a deep blue crystalline ppt.
The metal ion is  (2007)
Q.9 The compound Z, is  (2009)
(A) Pb2+ (B) Hg2+ (C) Cu2+ (D) Co2+
(A) Mg2[Fe(CN)6] (B) Fe[Fe(CN)6]

Q.3 Passing H2S gas into a mixture of Mn2+, Ni2+, Cu2+, (C) Fe4[Fe(CN)6]3 (D) K2Zn3[Fe(CN)6]2
and Hg2+ ions in an acidified aqueous solution ppt. −
SCN (excess)
 (2011) Q.10 Fe3+  → Blood red (A)
F (excess) −
(A) CuS and HgS (B) MnS and CuS  → Colourless (B)
(C) MnS and NiS (D) NiS and HgS Identify A and B. (a) Write IUPAC name of A and B.
(b) Find out spin only magnetic moment of B.  (2005)
Q.4 The reagents, NH4Cl and aqueous NH3 will ppt.
 (1991)
Q.11
(A) Ca2+ (B) Al3+ (C) Bi3+ (D) Mg2+
Moist air Zn
B ← MCl4  → A
(E) Zn2+ (W hite fumes M = (T ransition (Purple Colour)
having smell) element colourless)

Q.5 Which of the following statement (s) is (are) correct


Identify the metal M and hence, MCl4. Explain the
with reference to the ferrous and ferric ions?  (1998)
difference in colours of MCl4 and A.  (2005)
(A) Fe3+ gives brown colour with potassium ferricyanide
(B) Fe2+ gives blue ppt. with potassium ferricyanide Paragraph 2: An aqueous solution of a mixture of two
inorganic salts, when treated with dilute HCl, gave a
(C) Fe gives red colour with potassium ferricyanide
3+
precipitate (P) and a filtrate (Q). The precipitate P was
(D) Fe2+ gives brown colour with ammonium thiocyanate found to dissolve in hot water. The filtrate (Q) remained
unchanged, when treated with H2S in a dilute mineral
Q.6 A solution of colourless salt H on boiling with acid medium. However, it gave a precipitate (R) with
excess NaOH produces a non-flammable gas. The gas H2S in anammoniacal medium. The precipitate R gave
evolution ceases after sometime. Upon addition of Zn a coloured solution (S), when treated with H2O2 in an
dust to the same solution, the gas evolution restarts. aqueous NaOH medium.
The colourless salt (s) H is (are)  (2008)
(A) NH4NO3 (B) NH4NO2 Q.12 The precipitate P contains  (2013)

(C) NH4Cl (D) (NH4)2SO4 (A) Pb2+ (B) Hg2+ (C) Ag+ (D) Hg2+
Chem i str y | 30.59

Q.13 The coloured solution S contains  (2013) Ag


S2O2−3  →
+
X
Ag
 →
+
Y
(2016)
(A) Fe2 ( SO 4 ) (B) CuSO 4
Clear solution White precipitate
3 With time

→ Z
(C) ZnSO 4 (D) Na2CrO 4 Black precipitate

3−
Q.14 The reagent(s) that can selectively precipitate S2− (A)  Ag ( S2O3 )  , Ag2S2O3 , Ag2S
 2
from a mixture of S2− and SO 2−4 in aqueous solution is
5−
(are)  (2016) (B)  Ag ( S2O3 )  , Ag2S2O3 , Ag2S
 2
(A) CuCl2 (B) BaCI2
3−
(C)  Ag ( SO3 )  , Ag2S2O3 , Ag
(C) Pb ( OOCCH3 ) (D) Na2 Fe ( CN) NO   2
2  5 
3−
(D)  Ag ( SO3 )  , Ag2S2O 4 , Ag
Q.15 In the following reaction sequence in aqueous  3

solution, the species X, Y and Z, respectively, are

PlancEssential Questions
JEE Main/Boards JEE Advanced/Boards

Exercise 1 Exercise 1
Q.2 Q.3 Q.10 Q.1 Q.4 Q.6
Q.19 (f,i) Q.11 Q.15 Q.18

Exercise 2 Exercise 2
Q.2 Q.7 Q.15 Q.5
Q.24 Q.30
Previous Years Questions
Previous Years' Questions Q.5 Q.10 Q.11
Q.2 Q.6
3 0 . 6 0 | Qualitative Analysis

Answer Key

JEE Main/Boards

Exercise 2
Single Correct Choice Type

Q.1 B Q.2 C Q.3 D Q.4 B Q.5 C Q.6 B

Q.7 B Q.8 B Q.9 A Q.10 D Q.11 A Q.12 C

Q.13 A Q.14 B Q.15 D Q.16 C Q.17 A Q.18 D

Q.19 D Q.20 B Q.21 A Q.22 C Q.23 B Q.24 C

Q.25 A Q.26 D Q.27 D Q.28 C Q.29 C Q.30 B

Q.31 B Q.32 B Q.33 C Q.34 B Q.35 C

Previous Years Questions


Q.1 X-Cl2, Y-H2 Q.2 A Q.3 B Q.4 A Q.5 B Q.6 A

Q.7 C Q.8 B Q.9 C Q.10 C Q.11 D

JEE Advanced/Boards

Exercise 2
Single Correct Choice Type

Q.1 A Q.2 C Q.3 C Q.4 D

Assertion Reasoning Type

Q.5 C Q.6 A Q.7 D Q.8 C Q.9 C Q.10 B

Comprehension Type

Q.11 D Q.12 B Q.13 C Q.14 D

Previous Years Questions


Q.1 A Q.2 B Q.3 A Q.4 B, C Q.5 B, C Q.6 A, B

Q.7 D Q.8 C Q.9 B Q.12 A Q.13 D Q.14 A, C


Q.15 A
Chem i str y | 30.61

Solutions

JEE Main/Boards B + Ca(OH)2 → White ppt


∴ B = CO2
Exercise 1 C + Alkaline pyrogallol → Absorbed

Single Correct Choice Type ∴ C = O2


A + B + C + Lead acetate → Does not turn black
Sol 1: It due to the formation of PbCl2.
∴ A = SO2 ; B = CO2 ; C = O2
Sol.2 (i) Lead salt + sulphide → PbS (black)
Sol 6: ZnO, Zn (OH)2, ZnCO3 and ZnS
E.g Pb(CH3COO)2 + Na2S → PbS + 2CH3COONa

(ii) HgCl2 + 2KI → Hg I2 + KCl Zn(OH)2  → H2O + ZnO
red
(Yellow when hot
Hg I2 + KCl → K2HgI4 (soluble)
white when cold)
(iii) AgNO3 + NaCl → AgCl + NaNO3
white ZnCO3 → CO2 + ZnO
AgCl + 2NH4OH → Ag(NH3)2Cl + 2H2O

(iv) Pb(NO3)2 + 2KI → Pb I2 + 2KNO3
yellow Ca(OH)2 → CaCO3

Sol 3: Yellow ammonia sulphide. (turns lime water milky)


YAS = yellow ammonium sulphide (NH4)2 SX ∆
ZnO  → no gas
The group II A and II B elements are differentiated on

the basis of their solubility in YAS. ZnS  → with dil H2SO4 gives H2S
The group 2A elements are insoluble in YAS and 2B
elements soluble in YAS, whereas they are all insoluble Sol 7: To decrease the conc. of (OH-) ion in solution as
in sulphides. if ( NH+4 ) ion, conc. is high. Due to common ion effect,
the conc. of OH– ions is maintained low.
Sol 4: X + H2S + acid gas → Brown ppt.
Sol 8: (i) CuSO4 + 2NH4OH → Cu(OH)2 + (NH4)2SO4
Brown ppt → soluble in YAS
Cu(OH)2 + (NH4)2 SO4 + 2NH4 OH → Cu(NH3)4 SO4 +
excess
→ soluble
X + NaOH → white ppt  2H2O excess
NaOH
X(aq) + FeCl3 → FeCl2 + ……. . (ii) BiCl3 + 2Na2SnO2 + 6NaOH → 2Bi + 3Na2SnO3 +
6NaCl + 3H2O
X is a group 2B salt which gives a brown sulphide ∴ X
is Sn+2 (iii) 2HgCl2 + SnCl2 →Hg2Cl2 + SnCl4
SnCl + NaOH → Sn(OH)2 + 2NaCl HgCl2 + SnCl2 → 2Hg + SnCl4
Sn(OH)2 + 2NaOH → Na2SnO2 (iv) SbCl3 + H2O → SbOCl + 2HCl
(Strongly reducing)
Sol 9: Conc. HNO3 is added for the
Sn+2 + 2FeO3 → Sn4+ + 2FeCl2
(i) Oxidation of metal to its highest oxidation state
(ii) No
Sol 5: A + K2Cl2O7 → Green soluble
(iii) No. NaOH is a strong base
A = SO2/H2S
3 0 . 6 2 | Qualitative Analysis

∴ It has high conc of OH– ions. ∴ So, when H2S gas is passed through acidified solution
contain Cu2+ and Zn2+ only. Cu2+ ions will precipitate out

Sol 10: A(White crystals)  → B + C due to low conc, of S2– ion.
solid gas
KI
C → I2 Sol 13: (a) A + BaCl2 → White ppt
Sol

B 
HNO NaOH
3 → solution D  → A = H2SO4 or some sulphate
(b) NaOH + B → NH3(g)
E ↑ Sol F
brown B = NH+4 salt
HCOOH
(c) C + MnO-4 + H2SO4 → Violet vapours
C = iodide
Black ppt
(d) D + K2Cr2O7 + H2SO4 → Green soluble
∆ 1 D = Some reducing agent
A = AgNO3  → Ag + NO2 + O2
2

Ag 
HNO
3 → D = AgNO (e) E  → Yellow component
3

KI E = ZnO
O2  → I2

AgNO3 
NaOH
→ E ↓ Sol 14: (a) CO32−
low
E = AgO (b) Soluble
NH (c) NO2
AgO 
3 → F = [Ag (NH ) OH]
3 2
(d) Chloride ions
Acid (e) Ist group (Pb2+ Ag+ Hg+)
Sol 11: (i)PbS + Acid → Gas  → Yellow ppt.
B D
(f) II group → Acidic medium
Acid (A) : dil HCl of H2SO4
IV group → Alkaline medium
B = H 2S
(g) Insoluble
C = conc. HNO3
(h) Group IIB
D = S (yellow ppt)
(i) NH4OH
NH OH
(ii) A + H2S 
4 → White ppt. + 2HCl ( j) Fe (iron)

A = ZnCl2+H2S 
4 NH OH
→ ZnS + 2HCl (k) White
(l) Ba2+↑
ZnCl2 + NaOH → Zn(OH)2 
NaOH
→ Na2ZnO2 (m) White
(iii) PbS  ∆ in
→ A + PbS → Pb + SO2 (n) Co2+ + Fe3+ ion
air

A = PbO ∆
Sol 15: A  → B↑+C
NaOH

HCl
Sol 12: (i) PbCl2 is formed after Ist group partly soluble B  → white green ∴ B = NH3
in water and hence Pb2+ ions pass to the first group Zn
C  → B
filtrate, i. e. to the II group and is detected in the test

for IInd group as well. A No N2 ;A= NH4NO2

→ Hg (NH2 ) Cl + Hg
NH OH
(ii) Hg2Cl2 4 Sol 16: A = AgNO3
white Black

Ag + DilHNO3 → AgNO3 + NO
(iii) Ksp (CuS) < Ksp (ZnS)
AgNO3 + Brine → AgCl
Ionisation of H2S is further suppressed in presence of
NH OH
acid (common ion effect). AgCl 
4 → Ag (NH)3Cl
Chem i str y | 30.63

AgNO3 + H2S2O3 → Ag2S2O3 (b) FeCl3 + H2S → 2FeCl2 +2HCl + S


yellow green
Ag2S2O3 → Ag2S
Due to the reduction of FeCl3, the colour changes.
∴ A = Ag B =Ag NO3
(c) K2Cr2O7 + alkali → CrO24− + H2O
C = NO D = Ag Cl
the dichromate changes to yellow colored chromate.
E = Ag2S2O3 F = Ag2S
(d) CO is evolved along with CO2. CO2 burns with blue
flame while CO2 prevents burning
Sol 17:
H2C2O4 

→ H2O + CO + CO2
A = sulphur
hot conc HNO CO diffuses faster than CO2. Once CO burns with blue
S = →
3 NO2 + H2SO4
(brown) flame, it is put off by CO2 which diffuse later.
Na SO
S 
2 3 → Na S O (C)
2 2 3 (e) YAS possess excess free sulphur. It combines with
group 2B sulphide which convert it and forms Sulphates
Na2S2O3 (acidified) → Na2S 4 O6 + NaOH from soluble complex will (NH4)2S
white
E.g: SnS + S → SnS2
NH4 OH
Sol 18: (a) AgCl  → Ag(NH3 )2 Cl SnS2 + (NH4)2S → (NH4)2SnS3
(soluble)
true (f) Reaction of ZnCl2 with H2S produces HCl which
dissolve ZnS.
(b) Sb2S3 (is orange in colour) false
dissolves
(c) CuSO4 + K4FeCN6 → CuFeCN6 + K2SO4 (g) CaSO4 → forming a stable complex
(NH4 )2
choclote ppt
False CaSO4 + (NH4)2 → (NH4)2 Ca(SO4)2
(d) True
(h) Presence of SO24− brings preicipitation of group V
(e) True
ion such as Ca2+, Ba2+ etc.
(f) False, sulphates of V group radicals will be precipitated
(i) Adding of conc. HNO3 serves 2 purposes
conc. H2SO 4
(g) I 

→ I2 true (1) It precipitates the dissolved H2S
(h) False IInd group ferric salts are reduced as H2S. Hence (2) It convert Fe2+ → Fe3+ ions
it is always necessary to use HNO3 in IIIrd grp.
( j) Al(OH)3 and Zn(OH)2 dissolve in NaOH
(i) True 3(CH3COO )2Ca + FeCl3 → 2Fe (CH3COO)3 +

3CaCl2 Al(OH)3+NaOH → NaAlO2 +2H2O


soluble
( j) K2HgI4 alkaline true Zn(OH)2+2NaOH → Na2 ZnO2 +2H2O
(k) True soluble

(l) False Ni can be tested with dimethyl glyoxime as it ∆


Sol 20: X (aq)  → Alkaline
forms a colored complex.

(m) Co2+ + KNO3+ acetic acid → Yellow ppt. A  → Swells to give glassy material

CoCl2 + KNO3 → K3[Co(NO2)6] Conc, H SO


2 4 → Crystals
X 
Yellow X is an alkali metal salt and as it swells up to give a
(n) BaCO3, SnCO3, CaCO3 are soluble in acid to give glassy mass, it may be borax.
corresponding sulphates or chloride. True ∴ X is borax
Na2B4O7 + 7H2O → 2NaOH + 4H3BO3
Sol 19: (a) FeCl3 is a salt of weak base and a strong acid.
It readily hydrolyses to form Fe(OH)3 Na2B4O7. 10H2O → Na2B4O7+10H2O

FeCl3 + 3H2O Fe(OH)3 + 3HCl Na2B4O7 2NaBO2 + B2O3
addition of HCl prevents hydrolysis glassy mass
3 0 . 6 4 | Qualitative Analysis

Na2B4O7 + H2SO4 + 5H2O → Na2SO4 + 4H3BO3 X+conc. H2SO4



brown vapours
weak acid

AgNO3
Sol 21: X(aq) → Red litmus blue
∴ X is basic no ppt.
excess
X  → Deep blue colour Brown vapours = NO2
CuSO 4

X + FeCl3 Brown ppt (Soluble in dil. HNO3) X → Nitrate NO3−

∴ X must be NH3 as X + CuSO4 → Deep blue solution


Sol 8: (B) (NH4)2Cr2O7 → Cr2O3
CuSO4+4NH3 → Cu(OH)3 + 3NH4Cl
3NH3 + 3H2O + FeCl3 → Fe(OH)3 + 3NH4Cl Sol 9: (A) Only brown gas was Br2

Fe(OH)3+3HNO3 → Fe(NO3)3+H2O
Sol 10: (D)

White solid + dil H2SO4 →


Exercise 2 No action
Conc. H2SO4 observed
Single Correct Choice Type

Sol 1: (B) Sulphite + HCl → S (Suffocating)
Salt contains sulphate
Sol 2: (C) CuNO3 + KI(Starch) → blue
Sol 11: (A) IInd group MS (is yellow)
Sol 3: (D) SO3− + dil H2SO4 → SO2 When no radical of group II is present
Phosphate is present in mix.

K2Cr2O7 Sol 12: (C) BiCl3 + H2O → BiOCl


Green
Sol 13: (A) HgS is insoluble in dil. HNO3

Sol 4: (B) Oxalate  → CO + CO2
(C2O24− )
Sol 14: (B) NiS and ZnS are seperated by Conc. HCl
ZnS dissolved in dil HCl
burns with blue flame
Where ZnS + 2HCl → ZnCl2 + H2S
NiS is insoluble in dil HCl.
Sol 5: (C) Hg2Cl2 +NH4OH → Hg + HgNH2CI 

Sol 15: (D) Heating Na2CO3 mix in distilled water.

Black
Sol 16: (C) Dimethyl glyoxime + Ni → Red coloured
complex ppt.
Sol 6: (B) Br2 + KI → Blue
Sol 17: (A) NO2 bleaches moist litmus paper.

Sol 7: (B) X + dil. H2SO4  → do not evolve brown
vapour ∆
Sol 18: (D) ZnO  → ZnO
white yellow
Chem i str y | 30.65

Sol 19: (D) White ppt + NH4OH → black Sol 34: (B) Ca + N2 → Ca3N
Compound is Hg2Cl2 Ca3N + 6H2O → 3Ca(OH)2 + NH3
Hg2Cl2 + NH4OH → Hg + HgNH2Cl Ca(OH)2 + CO2 → CaCO3
∴ gas = NH3 and solid = CaCO3
Sol 20: (B) Al(OH)3 + NaOH (excess) → Na[Al(OH)4]
Soluble complex
Sol 35: (C) X + H2O → Saturated solution

Sol 21: (A) HgS does not dissolve in hot dil. HNO3. X(ag) + AgNO3 → White ppt. AgCl
X(ag) + Mg → Colourless gas MgCl2
Sol 22: (C) An aqueous solution of FeSO4Al2(AO4)3 X = Cl2 ; Y = H2
and chrome alum is heated with excess of Na2O2 and
filtered. The materials obtained are a yellow filtrate and
a brown residue Previous Years’ Questions

Sol 23: (B) NH+4 (salt) + NaOH  → NH3 Sol 1: Cl2 + H2O → HCl + HOCl
‘X’
Sol 24: (C) Green frame ∴ Cation is Ba2+
HCl + AgNO3 → AgCl ↓ (White) + HNO3
BaCl2 + 2Ag+ → 2AgCl + Ba2+
White 2HCl + Mg → MgCl2 + H2(g) ↑ Y
y
Sol 25: (A) Bi and Sn both belong to IInd group. Both
give precipitate. Sol 2: (A) SO32− + H2SO 4 → SO2 ↑ + H2O + SO24−
X Y

Sol 26: (D) H2S + NH4OH + X → White ppt. SO2 is a colourless gas with irritating odour.
ZnCl2 SO2 + K2Cr2O7 + H2SO4 → K2SO4 + Cr2 (SO 4 )3 + H2O
∴ X = Zinc salt Green
solution

Sol 27: (D) Conc. of S2– ion is covered by formation of Sol 3: (B) A sodium salt of an unknown anion when
weak acid H2S. treated with MgCl gives white precipitate (MgCO3) only
on boiling. Hence, the action must be HCO3− ion.

Sol 28: (C) A white solid is first heated with dilute and MgCl2 + 2NaHCO3  → MgCO3 + 2NaCl + H2O +
when with concentrated. No action is observed in either CO2
case. The solid contains sulphate.
Sol 4: (A) Both (NH4)2Cr2O7 and NH4NO2 on heating
Sol 29: (C) Bead’s test’s salt is Na(NH4)HPO4. 4H2O gives nitrogen gas.

I −

Sol 30: (B) To separate Fe from Zn and Cu, use excess Sol 5: (B) Bi3+ + 3I– → BiI3 ↓ → [BiI4 ]
Excess
NH3 Black Orange
solution

Sol 31:(B) Al and Fe can be separated by NaOH excess Sol.6: (A) Zn2+ + 2H2O → Zn(OH)2 ↓ + 2H +
Al + NaOH → Al(OH)3 White

Al(OH)3 + NaOH → NaAlO2 Sol 7: (C) Cation Cd2+ belongs to group II white Ni2+
belongs to group I of analytical group. Group II radicals
Sol 32: (B) K4[Fe(CN)6], Fe+3 ferricyanide are precipitated by passing H2S(g) through acidic
solution of salt but radicals of group III are precipitated
by passing H2S(g) in NH3/NH4Cl buffer solution of salt
Sol 33: (C) PbSO4 is water insoluble.
due to greater solubility products of later salts.
3 0 . 6 6 | Qualitative Analysis

Sol 8: (B) As MgSO4 is soluble in water, so not used for 2 K HgI


Mix 
4→
brown ppt.
estimation of SO24− ion.
The mixture contains Fe+2 and Fe+3 (from NH+4 ) (from
test b and d) and Cl– from test A.
Sol 9: (C) Yellow filtrate contain CrO5 and brown residue
Conc, H SO
contain Fe2O3. ∴ The reactions are Cl– + MgO 
2 4 → Cl
2


NH+4 + NaOH  → NH3
Sol 10: (C) Fe(II) and Fe(III) will have different values of
magnetic moment due to different number of unpaired Fe+2 → K4FeCN3
electrons in their d-orbitals. Fe+3 → Blue ppt will be K4FeCN6
K HgI
Sol 11: (D) PbCl2 is soluble in hot water and PbS (black) NH3 
2 4 → Brown ppt.

is formed on passing H2S(g) through acidic solution.



Sol 4: X . nH2O  → X + nH2O

JEE Advanced/Boards with dil HCl


X(aq) → White turns brown
KI + X → Decolourises
Exercise 1
Ag NO3 + X → White ppt
Sol 1: X (Lewis acid → fumes in moist air)
X + NH4OH → fumes intensity increases black or strong
X (acidic) + NH4Cl + NH4OH → ppt → soluble in NaOH
X + H2S → No ppt. X containing S2O32− as it decolourises I– which also
coincides with the other 2 statement I and (II)
X is some chloride as its fumes increases in presence
of NH4OH hence the compound is Na2S2O3. 5H2O

X + NH4Cl + NH4OH → ppt → soluble in NaOH ∆


Na2S2O3. 5H2O  → Na2S2O3+5H2O↑
∴ X = AlCl3 or ZnCl2
(i) Na2S2O3 + 2HCl → 2NaCl + H2O
But X is a Lewis acid ∴ X = AlCl3
+ SO2 + S (Turbidity)
Sol 2: A(aq) → B (ppt) + O solution
NaCl
(ii) 2Na2S2O3+I2 → Na2S4O6 + 2NaI
H2S
C  → Black ppt
(iii) Na2S2O3 + 2AgNO3 → Ag2S2O3↓ + 2NaNO3
hot water +Na I2
B  → Yellow ppt. White ppt.
dil. HCl
→ X no gas reddish brown gas
A  Ag2S2O3 + H2O → Ag2S↓ + H2SO4


Black
A = Pb(NO3)2
B = PbCl2 Sol 5: X + CH3COO– + KI → I2

Pb 
H S
2 → PbS (black) CO2 + aq(X) → ppt.
X (H2O) + ethyl alcohol →
Pb2+ + KI → PbI2 yellow
Product = Ester

Sol 3: Salt + MgO2  X = CaOCl2 bleaching powder


→ Yellow greenish gas
Conc. H2SO 4

∆ (i) CaOCl2 + CH3COOH → (CH3COO)2 Ca + Cl2 + H2O


Mix + NaOH  → Red litmus blue
2KI + Cl2 → 2KCl + I2
X (aq) → Blue ppt K4(Fe3CN3)
(ii) CaOCl2 + CO2 → CaCO3 + Cl2
and let NH4CNS
(iii) CaOCl2 + H2O → Ca(OH)2 + Cl2
Chem i str y | 30.67

C2H5OH + Cl2 → CH3CHO + 2HCl CoCl2 + NaHCO3 → Na4Co(CO3)3 + H2O + 3CO2


CH3CHO + 3Cl2 → CCl3CHO + 3HCl Br2 + H2O → HBr + O
CCl3CHO + 2Ca(OH)2 → CHCl3 + (HCOO)2Ca 2Na4Co(CO3)3 + H2O + O → 2Na3Co(CO3)3 + NaOH
sod. cobalt carbonate
Sol 6: A = SO2
(green)
SO2 + dil. H2SO4 + K2Cr2O7 → K2SO4 + Cr2(SO4)3

SO2 + dil. H2SO4 + KMnO4 → MnSO4 Sol 8: A + H2O  → White ppt. (B)
SO2 + O2 → H2SO4 A + conc. HCl → Clear solution
H2SO4 + BaCl2 → BaSO4
Excess H2O
A = SO2
B = Cr2(SO4)3 B precipitate
H2S
C = KMnO4 A/B  → Brown / Black ppt. (C)
D = H2SO4 A + H2SO4 → D
D + Hg+(Salt) → White E
heat
Sol 7: (i)A(aq) + Zn  → B
∴ D = Cl2, E = Hg2Cl2
heat
A(aq) + C → PH3 A = BiCl3, B = BiOCl, C = BiS
A(aq) + NH4Cl → (D) gas D = HCl, E = Hg2Cl2
(ii) A = NaOH/KOH C = P4 BiCl3 + H2O → Bi(OH)3 + 2HCl
B = H2 D = NH3 Bi(OH)3 + HCl → Clear BiCl3
CO : CoS BiCl3 + H2O (Excess) → BiOCl
↓ ↓ BiCl3 + H2S → BiS (Brownish black)
NH4 OH
A + H2S  → B (black) D = HCl and E = Hg2Cl2

B CoS + HCl + KClO3 → A(CoCl2) Sol 9: A = NH4Cl


A + KCN buff coloured ppt. B = NH4HCO3
(COCl2 )

A C = NaHCO3
Excess NH4CO3+NaCl → NaHCO3 + NH4Cl
reagent ∆
NaHCO3 → Na2CO3 + H2O + CO2

C D = CO2.
Co(CN)6


A (aq)  → B Sol 10: A + N2 → B
COCl2
B = H2O → C + D
Br
A + NaHCO3 (excess) → 
2→
C + CO2 → Milky soln
H2O
A = CoCl2 ∴ C = Ca(OH)2
B = CoS A = Ca
C = K4[Co(CN)6] B = Ca3N2

E = 2Na3Co(CO3)3 Green C = Ca(OH)2


D = NH3
CoCl2+H2S+NH4OH → CoS (black)
CoS + HCl + KClO3 → CoCl2
CoCl2 + KCN → K3 [Co(CN)6]
3 0 . 6 8 | Qualitative Analysis

Sol 11: A(aq) + NaOH → White ppt Sol 13: R + K2Cr2O7 → Decolourises
X + H2O2 → + Aq. BaCl2 → A ppt
Excess NaOH
Soluble
in dil
Dissolves
HCl

A(aq) + NH4OH → White ppt
H2 + X → Turbidity
Excess NH4OH According to the reaction X = SO2
K2Cr2O7 + 4H2SO4 → K2SO4 + Cr2(SO4)3 + 4H2O + 3O
Dissolves
H2O + SO2 +O → H2SO4
AgNO3 + A → Yellow ppt. H2O2 + SO2 → H2SO4
dil HNO
3 → Solution
 H2SO4 + BaCl2 → BaSO4 + 2HCl

∴ A = AlBr3. H2S + SO2 → 3S↓ + 2H2O

Sol 12: A(acidic) → Sublimes at 300°C Sol 14: Refer theory.

A + NH4 SNC → B ∆
Excess Red compound Sol 15: A  → B+C
B + H2 → D (Metal)
A + K4FeCN6 → Blue compound
(C) = Fe4 (Fe(CN)6)3 [B + HCl + K4Fe(CN)6] → Chocolate ppt.
(E)
∴ A = Fe
A + K2Cr2O7 + conc. H2SO4 → D C + Ca(OH)2 → Milkiness 
excess C
→ Clear solution (F)
(Deep red vapour) 2CrO2Cl2 E = Cu2[Fe(CN)6]
∴ Cl
∴ A is a copper compound
D + NaOH + CH3COOH +
C = CO2/SO2
Pb(CH3COO)2 F = CaHCO3/Ca
E A = CuSO4

Yellow B = SO2 + CuO


CuSO4 → 2CuO + SO2
∴ A = FeCl3 B = Fe(SCN)3
CuO + H2 → Cu
C = Fe4[Fe(CN)6]3 D = CrO2Cl
CuO + K4Fe(CN)6 = Cu2[Fe(CN)6]
E = PbCrO4
SO2+Ca(OH)2 → CaSO4 → CaS2O3
∴ Reactions are
FeCl3 + NH4SNC → Fe(SCN)3
Sol 16: B + conc. liquor+NaS + NaSO2 → A
FeCl3 + K4 [Fe(CN)]6 →Fe4[Fe(CN)6]3
A + Dil AgNO3 → White ppt. → C (Black)
FeCl3 + K2Cr2O7 + H2SO4 → 2CrO2Cl2 + Fe2(SO4)3
FeCl3 + A → Violet coloured D (Disappear)
CrO2Cl2 + NaOH + CH3COOH + Pb(CH3COO)2 → PbCrO4
+ CH3COONa A + CuCl2 → White ppt.
Excess

E
Chem i str y | 30.69

A contains S2O32− ion from (ii) and A = Na2S2O3 E = PbCrO4


SO2 + conc. EtOH + NaS + NaSO2 → Na2S2O3 A = K2Cr2O7

Na2S2O3 + dil AgNO3 → Ag2S2O3


Sol 19: (i) Au + HCl + HNO3 → HAuCl4 + 3NO2 + 3H2O
White ppt.
(ii) C + 4HNO3 (conc) → CO + 4NO2 + 2H2O
(iii) Sn + 2KOH (hot) + H2O → K2SnO3 + 2H2
(iv) Cu(OH)2 + 2NH4NO3 + 2NH4OH → Cu(NH3)4(NO3)2
AgS (Black)
+ 4H2O
FeCl3 + 3Na2S2O3 → Fe2(S2O3)3 +3NaCl
Sol 20: K2Cr2O7 + (Mix) → X absorbed solution
Violet
green solution
CuCl2+ Na2S2O3 → NH4[Cu6(S2O3)5]
(Y + Z) + Ca(OH) → Milky
Sol 17: MCl (Z) Alkali pyragallol → Black
MCl + H2S (acidified) → black ppt ∴ Z = O2, Y = CO2 X = SO2
(not soluble in YAS) (i) 3SO2 + K2Cr2O7 + H2SO4 → H2SO4 + Cr2(SO4)3 + H2O
∴ MS is group IIA (ii) Ca(OH)2+CO2 → CaCO3↓ + H2O
SnCl2
SnCl2+X → White ppt. → Grey (iii) O2 + Pyragallol → Absorbed
Excess

Excess
KI + X → Red ppt.  → Dissolve
KI
Exercise 2
Acc. To these facts, X = HgCl2
+
Single Correct Choice Type
H
HgCl2 + H2S → HgS + 2HCl
Black Sol 1: (A) Theoretical : HgS does not dissolve in hot
dil. HNO3.
2HgCl2 + SnCl2 → Hg2Cl2 + SnCl4
White
Sol 2: (C) FeSO4. Al2(SO4) and chrome alum + Na2SO3
HgCl2 + SnCl2 → Hg ↓ + SnCl4 ∆
Grey
 → (C) a yellow filtrate and brown residue.

HgCl2 + 2KI → HgI2 + 2KCl


Red
Sol 3: (C) MgCO3 is soluble in NH4Cl forming MgCl2.

Hg I2 + 2KI → K2HgI4
Sol 4: (D) Excess of splitting ∝ oxidation state.

Sol 18: A  → Violet colour flame
Assertion Reasoning Type
∴ A contains Cr
Sol 5: (C) Yellow colour of compound is not due to
A + B + conc. H2SO4 → C (Red gas)
NH+4 ion (as NH+4 ion does not impart colour to its
C + alk H2SO4 → Red yellow solution D molecules).
D + CH3COOH → Yellow ppt.
Sol 6: (A) (A) Both statements are true.
B + NaOH → F (fumes)
∴ F = NH3 Sol 7: (D) both statements are false as NaOH will react
B = NH4Cl with salt and not form ether hydroxide.
C = CrO2Cl2
Sol 8: (C) Charcoal cavity test can only be used for non
D = Na2CrO4 coloured salt,.
3 0 . 7 0 | Qualitative Analysis

Sol 9: (C) All S containing molecules give match state Fe2+ + K3[Fe(CN)6] → KFe[Fe(CN)6] + 2K+
test.
The red colour of Fe3+ion with potassium thiocyanate is
due to formation of [Fe(SCN)3].
Sol 10: (B) Both Statement true but reason does not
explain assertion. Fe3+ + 3KSCN → [ Fe(SCN)3 ] + 3K+
red coloured

Comprehension Type
Sol 6: (A, B) NH4NO3 + NaOH → NaNO3 + NH3 +
Sol 11-14: (D, B, C, D) H 2O

(a) Na2CrO4 + CO2 → Na2Cr2O7 NH4NO2 + NaOH → NaNO2 + NH3 + H2O

(b) Zn + acidic Na2Cr2O7 → Green Cr2O3 Sol 7: (D) Sol 8: (C) Sol 9: (B)
Orange The comprehension describing methylene-blue test.
NMe2
NMe2
Burn
2-
S +2 + Fe (III)
Green S
2-
+2 + Fe (III)

CO2 acts to make the solution acidic NH2


NH2
H2O + CO2 → H2CO3
Zn is the reducing agent Me2N
+
+
S NMe2
Me2N NMe2
Cr+6 to Cr+3 S

Cr+3 to Cr+2 (blue in colour) N


N
Blue solution
Previous Years’ Questions
Blue solution
Therefore,
Sol 1: (A) MgSO4 + NH4OH + Na2HPO4 → 1. X is Na2S
Mg(NH4 )PO 4 ↓ + Na2SO4 + H2O 2. Y is FeCl3
white
3. Compound Z is Fe[Fe(CN)6]

Sol 2: (B) Hg2+ + 2I– → HgI2 (red)


Sol 10: (a) Fe3+ + 3SCN– → [ Fe(SCN)3 ] (blood red
HgI2 + 2KI → K 2 [ HgI 4 ] colouration) A

soluble
3−
Hg + Co(SCN)2 → Co[ Hg(SCN)4 ]
2+ Fe(SCN)3 + F– (excess) → [ FeF6 ] + 3SCN–
blue B

(b) Magnetic moment ( µs ) = n(n + 2) BM


Sol 3: (A) In acidic medium, H2S is very feebly ionized
giving very small concentration of sulphide ion for
= 3S BM
precipitation. Therefore, the most insoluble salts CuS
and HgS are precipitated only. = 5.92 BM

Sol 4: (B, C) Both Al3+ and Bi3+ are precipitated as their Zn


Sol 11: MCl4  → Purple coloured compound (A)
hydroxides. colourless
moist
M → B (white fumes)
Sol 5: (B, C) The blue precipitate of Fe ion with
2+
tr. metal air

potassium ferricyanide is due to formation of Turnbull’s ⇒ M = Ti, A=[Ti(H2O)6]3+; B = TiO2


blue KFe[Fe(CN)6].
Chem i str y | 30.71

Ti (IV) contains no d-electron, while d-d transition of


single electron of Ti (III) will cause colour change.

Hot
Sol 12: (A) Pb+2 + 2HCl → PbCl2 
Water
→ Soluble

Sol 13: (D)


Hot
Pb+2 + 2HCI → PbCI2 
Water
→ Soluble

→ Cr ( OH) ↓
H S
Cr +3 
2
Ammxmiacal 3

Cr ( OH) 
NaOH
H O
→ Na2CrO 4
3 2 3 Yellow solution

Sol 14: (A, C) (A) CuCI2 + S −2 → CuS ↓ +2CI −


Black ppt
2−
CuCI2 + SO 4 → No. ppt

(B) BaCI2 + S −2 → BaS + 2CI −


No.ppt

BaCI2 + SO 4 −2 → BaSO 4 ↓ 2CI−


White ppt

(C) Pb ( OAc ) SO −2 → PbS ↓ + 2CH3 − COO −


2 (Black ppt )

Pb ( OAc ) + S −2 → PbSO 4 ↓+ 2CH3 − COO −


2
Black ppt

PbS can be selective ppt out first as Ksp s much less than
K sp of PbSO 4

K sp of PBS= 3 × 10−28
K sp of PbSO=
4 25 × 10−8

(D) Na2 Fe ( CN) NO  + S −2 → Fe ( CN) NOS 


 5   5 
Purple Colour

Na2 Fe ( CN) NO  + SO 42− → No. ppt


 5 

Sol 15: (A)


+ 3−
S2O2−3 →  Ag ( S2O3 ) 
Ag
 2
(X)
Ag+
→ Ag2S2O3 → Ag2S(Black ppt)
(Y) (Z)
2017-18 100 &
op kers
Class 12 T
By E ran culty
-JE Fa r
IIT enior emie .
S fP r es
o titut
Ins

CHEMISTRY
FOR JEE MAIN & ADVANCED
SECOND
EDITION

Exhaustive Theory
(Now Revised)

Formula Sheet
9000+ Problems
based on latest JEE pattern

2500 + 1000 (New) Problems


of previous 35 years of
AIEEE (JEE Main) and IIT-JEE (JEE Adv)

5000+Illustrations and Solved Examples


Detailed Solutions
of all problems available

Plancess Concepts
Topic Covered Tips & Tricks, Facts, Notes, Misconceptions,
Key Take Aways, Problem Solving Tactics
d and f-block Elements
PlancEssential
Questions recommended for revision
31. d A N D f- B L O C K
ELEMENTS

d-BLOCK ELEMENTS

1. INTRODUCTION
The elements in which the last electron enters (n – 1)d orbitals of the atom are called d-block elements. Also, these
elements lie in between s and p block elements in the long form of the periodic table. So, they are also called
transition elements.

2. ELECTRONIC CONFIGURATION AND IRREGULARITIES


The valence shell configurations of these elements can be represented by (n – 1)d1–10ns0,1,2. All the
d-block elements are classified into four series viz 3d, 4d, 5d and 6d orbitals of (n – 1)th main shell. Each series
has 10 elements. Cr(3d5, 4s1), Cu(3d10, 4s1), Mo(4d5, 5s1), Pd(4d10, 5s0), Ag(4d10, 5s1) and Au(5d10, 6s1) clearly show
irregularities in the configurations. These are explained on the basis of the concept that half-filled and completely
filled d-orbitals are relatively more stable than other d-orbitals.

PLANCESS CONCEPTS

It should be noted here that when atoms of these elements form cations, electrons are removed from
the outermost s-subshell instead of the penultimate d-subshell, although the former was filled earlier.
25
Mn : [Ar] 3d5, 4s2 Mn2+ : [Ar]3d5
26
Fe : [Ar]3d6, 4s2 Fe2+ : [Ar] 3d6
Vaibhav Krishnan (JEE 2009 AIR 22)

Illustration 1: To what extent do the electronic configurations decide the stability of oxidation states in the first
series of the transition elements? Illustrate your answer with example.  (JEE MAIN)

Sol: Empty, Half-filled and completely filled orbitals have extra stability
E.g. Mn2+ = [Ar]3d5 , Sc3+ = [Ar]3d0 , Zn2+ = [Ar] 3d10
3 1 . 2 | d and f-block Elements

Illustration 2: What may be the stable oxidation state of the transition element with the following d-electron
configurations in the ground state of their atoms: 3d3, 3d5, 3d8 and 3d4?  (JEE ADVANCED)

Sol: Ground state configuration Stable oxidation state


3d3 +5
3d +2, + 7
5

3d8 +2
3d4 3d4 does not exist

3. GENERAL PROPERTIES OF THE TRANSITION METALS


3.1 Atomic and Ionic Radii
(a) The atomic and ionic radii for transition elements are smaller than their corresponding s-block elements and
are greater than their corresponding p-block elements.
(b) The atomic and ionic radii for transition elements for a given series show a decreasing trend for the first five
elements and then becomes almost constant for next five elements of the series. For example, in 3d-series
atomic radius decreases from 21Sc to 25Mn and then becomes constant for next five i.e. 26Fe to 30Zn
Explanation: This is due to the combined effect of the increasing 17
effective nuclear charge, (ENC) and increasing screening effect along
16
the period. An increase in ENC favors a decrease in atomic radii, whereas
15
increase in number of d-elements increases the screening effect and
thus increases the atomic radii. Thus both ENC and screening effect act 14
opposite to each other and therefore the atomic size is governed by the 13
net influence of these two. 12
11
(c) The atomic and ionic radii of the elements of 4d-series are higher than Sc Ti V Cr Mn Fe Co Ni Cu Zn
3d-series as the number of shells increases down the group. However, Figure 31.1: Atomic radii of element
the elements of 4d-series and 5d-series on moving down the group of 3d-series
reveal almost constant value. For example, Zirconium and Hafnium, the
members of 4d and 5d-series, respectively have the almost same size i.e. 145 pm. Similarly, Zr4+ and Hf4+ have
their atomic radii as 80 pm and 81 pm respectively. This is due to the Lanthanoid contraction.
(d) The ionic radii decreases as charge on the cation increases (i.e., higher oxidation state). e.g.
Ti2+ > Ti3+ > Ti4+
Cr2+ > Cr3+ > Cr5+ > Cr6+
Fe2+ > Fe3+
(e) For ions having same oxidation states, the ionic radii decreases with increase in atomic number e.g.
For 3d-series
Sc2+ > Ti2+ > V2+ > Cr2– > Mn2+ > Fe2+ > Co2+ > Ni2+ > Cu2+
Ionic radii (in Å) 0.95 0.90 0.88 0.84 0.80 0.76 0.74 0.72 0.69

Illustration 3: In a transition series, with an increase in atomic number, the atomic radius does not change very
much. Why is it so?  (JEE MAIN)

Sol: With increase in atomic number along a transition series, the nuclear charge increases which tends to
decrease the size of the atom. But, the addition of electrons in the d-subshell increases the screening effect which
counterbalances the increased nuclear charge. Hence, along a transition series the atomic radius does not change
very much.
Chem i str y | 31.3

3.2 Atomic Volume and Density


(a) The size decreases along the period and, therefore, atomic volume also decreases along the period.
(b) Atomic volumes are smaller than group 1 and 2 members i.e. s-block elements.
(c) The density, however, increases along the period.

3.3 Melting and Boiling Points


(a) All the transition elements have a higher melting point as compared to s-block elements due to strong
metallic bonding as well as unpaired d-electrons leading to covalence.
(b) It is evident from that the melting point of transition metals or a given series increases on moving left to right
in a period and attains a maximum value and after that the m.p. goes on decreasing towards the end of period.
This is due to the fact that the strength of inter particle bonds in transition elements is also directly related to the
number of half-filled d-orbitals. In the beginning, the number of unpaired electrons in d-orbitals increases till
the middle of the period (d1 to d5). After this, the pairing of electrons occurs in d-orbitals (d6 to d10). An increase
in inter atomic bonds due to the increase in number of unpaired electron results in higher m.p.

W
3500
Re
Ta
3000 Os
Mo
Ru
lr
2500 Nb
Tc
Melting point (K)

Cr Rh
Ti Zr Pt
2000
Y Fe
Hf Pd
(4d-series) V
1500 Co
Sc Ni
(3d-series) Mn Cu
Au
1000 Ag
La
(5d-series) Zn
500 Cd

Hg

1 2 3 4 6 5
7 8 9 10 11 12
Groups
Figure 31.2: Trends in melting points of different group

3.4 Metallic Character


(a) All the transition metals possess one or two electrons in their outermost shell and thus exhibit metallic nature.
(b) All are hard, ductile and malleable solids with strong metallic bonding (except mercury which is liquid) and
possess hcp, bcc or ccp crystal lattices.
(c) Transition metals show a gradual decrease in electropositive character on moving along the period.
(d) Strong metallic bonding in transition metals is due to greater effective nuclear charge and a large number of
valence electrons (inner d-subshell and outermost s-subshell).
(e) Due to strong metallic bonding, transition metals are hard, possess high densities and high enthalpies for
atomization.
3 1 . 4 | d and f-block Elements

(f) Due to metallic bonding, these are good conductors of heat and electricity.
(g) Transition metals form numerous useful alloys with other metals.

3.5 Enthalpies of Atomisation


Transition elements have a high enthalpy of atomization due to strong interatomic attraction. Greater the number
of valency electrons, stronger is the resultant bonding and higher is the enthalpy of atomization. The members
of 4d and 5d-series have greater enthalpy of atomization than those of 3d-series. Thus, they form metal-metal
bonding frequently in their compounds.

3.6 Ionization Energy


(a) The ionization energy (IE) of d-block elements lies in between s-and p-block elements showing less
electropositive character than s-block
(b) Smaller atomic size and a fairly high IE is noticed for transition metals.
(c) IE values first increases up to 25Mn and then becomes irregular or constant values due to irregular trend of atomic
size after 25Mn in 3d-series. Similar trend is noticed in 5d-and 6d-series.
(d) The magnitude of ionization energies give an idea about the relative stabilities of various oxidation states of
transition elements.

lr
Pt Au(5d)
9 Os
IE1 (x 10-2 kj mol-1)

Pd
8 W Re Fe Co
Hf Ta
Cu(3d)
Mn
Ni
7 Mo Rh Ag(4d)
Zr Nb Tc Ru
Ti
V Cr
6 Sc
Transition elements
Figure 31.3: Ionization energies of transition elements

PLANCESS CONCEPTS

The ionization energy of 5-d series are higher than that of 3-d and 4-d series due to the poor shielding
effect of 4-f electrons present in 3-d series
Ionization energy of Zn, Cd and Hg are abnormally higher on account of greater stability of s-subshell

Nikhil Khandelwal (JEE 2010 AIR 443)

Illustration 4: In the series Sc(Z = 21) to Zn(Z = 30), the enthalpy of atomization of zinc is the lowest, i.e. 125 kJ mol–1.
Why?  (JEE ADVANCED)

Sol: In the formation of metallic bonds, no electrons from 3d-orbitals are involved in case of zinc, while in all other
metals of the 3d series, electrons from the d-orbitals are always involved in the formation of metallic bond. This is
why the enthalpy of atomization of zinc is the lowest in the series.
Chem i str y | 31.5

3.7 Reactivity
(a) Transition metals are less reactive than s-block elements. Their low reactivity is due to:
(i) High ionization energy.
(ii) Existence of metallic bonding among atoms which gives rise to higher heat of sublimation.
M(s) → M(g) ∆H = Heat of sublimation (∆Hs)

M(g) → M(g)


+
+ e- ∆H = Heat of ionization or ionization energy (IE)
+
+
M(g) → M(aq) ∆H = – Heat of hydration (– ∆Hh)

+
M(s) + H2O → M(aq) + e- ∆H = ∆Hs + IE – ∆Hh

+
 ore negative the value of ∆H for the change, lesser is the energy level for M(aq)
M and greater will be the stability
of that oxidation state in aqueous solution.

3.8 Variable Valency and Oxidation State


(a) Most of the transition elements show variable valencies or different oxidation states because of incomplete
d-subshell. The variable oxidation states of transition elements are due to the participation of ns and (n – 1)
d electrons in bonding.
(b) It is thus evident that for the first five transition elements, the minimum oxidation state is given by the
electrons in outermost s-subshell and the maximum oxidation state by the total number of ns and (n – 1)
d-subshell electrons.
(i) 21
Sc shows +2 and +3 (due to 4s2 and 3d1 electrons)
(ii) 22Ti shows + 2, +3 and +4 (due to 4s2 and 3d2 electrons)
(iii) 23V shows +2, +3, +4, +5 (due to 4s2 and 3d3 electrons)
(iv) 24Cr shows +2, +3, +4, +5, +6 (due to 4s1 and 3d5 electrons)
(v) 25
Mn shows +2 to +7 (due to 4s2 and 3d5 electrons)

PLANCESS CONCEPTS

•• The transition elements in their lower oxidation states (+2 and +3) usually form ionic compounds. In
higher oxidation state, compounds are normally covalent. For example, Mn in MnCl2 has Mn2+ ion whereas
in KMnO4, Mn exists in +7 state of covalent nature.
•• Some transition metals also exhibit zero oxidation state in their compounds such as carbonyls. Ni and Fe
in Ni(CO)4 and Fe(CO)5 have zero oxidation state.
•• The highest oxidation states of transition metals are found in their compounds with fluorine and oxygen.
This is due to higher electronegativity and small atomic size of fluorine and oxygen. Eg.MnO4-
Neeraj Toshniwal (JEE 2009 AIR 21)

Illustration 5: Which metal in the first series of transition metal exhibits +1 oxidation state most frequently and
why?  (JEE ADVANCED)

Sol: Copper, because it will achieve a completely filled d-orbital and a stable configuration on losing an electron.
3 1 . 6 | d and f-block Elements

3.9 Colour
Substances appear coloured when they absorb light of a particular wavelength in the visible region of the spectrum
and transmit light of other wavelengths. The colour which we see is the colour of the transmitted wavelengths. In
other words, the colour of the compound observed by us is the complementary colour of the colour absorbed by
the compound.
In the s-and p-block elements, there cannot be any d-d transistions and the energy needed to promote the s or p
electrons to a higher level is much greater and may correspond to ultraviolet region, in which case the compound
will not appear coloured to the eye.
Table 31.1: Color spectrum

Wavelength absorbed in nm Colour absorbed Colour observed


< 400 UV region White/colourless
400–435 Violet Yellow-green
435–480 Indigo Yellow
480–490 Green-blue Orange
490–500 Blue-green Red
500–560 Green Purple
560–580 Yellow-green Violet
580–595 Yellow Indigo
595–605 Orange Green-blue
605–750 Red Blue-green
> 750 Infra-red White/colourless

Illustration 6: Transition metal ions like Cu+, Ag+, Zn2+, Hg2+ and Cd2+ are colourless. Explain.  (JEE ADVANCED)

Sol: Due to forbidden transition some metal ions are colourless. All the ions reported above have no unpaired
electrons in them and the d-orbital [(n – 1)d10] is also completely filled. Thus, due to d-d transition above mentioned
metal ions are colourless.

3.10 Magnetic Properties


Magnetic Properties: When a substance is placed in a magnetic field of strength H, the intensity of the magnetic
field on the substance may be greater than or less than H.
Diamagnetic: The substances which are weakly repelled by a magnetic field; absence of unpaired electrons.
Paramagnetic: The substances which are weakly attracted by the magnetic field and lose their magnetism when
removed from the field; presence of unpaired electrons.
Paramagnetism is expressed by magnetic moment,
µ= n(n + 2) B.M.
n = Number of unpaired electrons
B.M. = Bohr Magneton, unit of magnetic moment
Chem i str y | 31.7

Illustration 7: The paramagnetic character in 3d transition series increases up to Cr and then decreases. Explain
 (JEE MAIN)
Sol: As number of unpaired electron increases paramagnetic nature increases.The number of unpaired electrons
increases from 21Sc: [Ar]3d1,4s2 to 24Cr : [Ar] 3d5, 4s1 and after chromium, the pairing of electrons takes place and
thus number of unpaired electrons goes on decreasing continuously to 30Zn: [Ar] 3d10, 4s2.

PLANCESS CONCEPTS

The magnetic properties of d-block elements are due to the only spin value of the unpaired electrons
present in d-orbital while in the case of f-block elements, it is due to both the orbital motion as well as
spin contribution.
Magnetic moment for d-block elements: µ = n(n + 2) B.M. (where n is the number of unpaired electrons)

Magnetic moment of f-block elements:


= µ 4s(s + 1) + ( + 1) . (where s is sum of spin quantum number
and  , the angular momentum quantum number)

Aman Gour (JEE 2012 AIR 230)

4. SOME IMPORTANT COMPOUNDS OF d-BLOCK ELEMENTS


4.1 Chromate and Dichromate
(a) Preparation: 4FeO.Cr2O3 + 8Na2CO3 + 7O2 
Roasting
→ 8Na2CrO4 + 2Fe2O3 + 8CO2
inair
The roasted mass is extracted with water when Na2CrO4 goes into the solution leaving behind insoluble Fe2O3.
The solution is treated with calculated amount of H2SO4.
2Na2CrO4 + H2SO4 → Na2Cr2O7 + Na2SO4 + H2O
The solution is concentrated when less soluble Na2SO4 crystallizes out. The solution is further concentrated
when crystals of Na2Cr2O7 are obtained. Then a hot saturated solution of Na2Cr2O7 is treated with KCl, then
reddish orange crystals of K2Cr2O7 are obtained on crystallization
(b) K2Cr2O7 is preferred to Na2Cr2O7 because Na2Cr2O7 is hygroscopic but K2Cr2O7 is not.
(c) Similarities between hexavalent Cr & S-compounds:
(i) SO3 & CrO3 → Both acidic
(ii) CrO −42 & SO24− Isomorphous
OH −
(iii) SO2Cl2 & CrO2Cl2  → SO24− & CrO24− respectively
OH −
(iv) SO3Cl– + CrO3Cl–  → SO24− & CrO24−
O O O
(v) CrO3 & B(SO3) has same structure Cr O Cr O Cr

O O O
(vi) Potassium dichromate reacts with hydrochloric acid and evolves chlorine.
K2Cr2O7 + 14HCl → 2KCl + 2CrCl3 + 7H2O + 3Cl2

(vii) It acts as a powerful oxidizing agent in an acidic medium (dilute H2SO4)


Cr2O27− + 14H+ + 6e– → 2Cr3+ + 7H2O (Eº = 1.33V)
The oxidation state of Cr changes from +6 to +3
3 1 . 8 | d and f-block Elements

(d) Uses: As a volumetric reagent in the estimation of reducing agents such as oxalic acid, ferrous ions, iodide
ions, etc.
(i) For the preparation of several chromium compounds such as chrome alum, chrome yellow, chrome red,
zinc yellow, etc.
(ii) In dyeing chrome tanning, calico printing, photography etc.
(iii) Chromic acid as a cleansing agent for glass ware

4.2 Manganate and Permanganate


Preparation: This is the most important and well known salt of permanganic acid. It is prepared from the pyrolusite
ore. It is prepared by fusing pyrolusite ore either with KOH or K2CO3 in the presence of atmospheric oxygen or any
other oxidizing agent such as KNO3. The mass turns green with the formation of potassium magnate, K2MnO4
2MnO2 + 4KOH + O2 → 2K2MnO4 + 2H2O
2MnO2 + 2K2CO3 + O3 → 2K2MnO4 + 2CO2
The fused mass is extracted with water. The solution is now treated with a current of chlorine or ozone or carbon
dioxide to convert magnate into permanganate.
2K2MnO4 + Cl2 → 2KMnO4 + 2KCl
2K2MnO4 + H2O + O3 → 2KMnO4 + 2KOH + O2
3K2MnO4 + 2CO2 → 2KMnO4 + MnO2 + 2K2CO3

Another Method of Preparation:


3K2MnO4 + 2H2SO4 → 2KMnO4 + MnO2 ↓ + 2K2SO4 + 2H2O
or 3K2MnO4 + 2H2O + 4CO2 → 2KMnO4 + MnO2 + 4KHCO3
1
But, in the above method
of Mn is lost as MnO2 but when oxidized either by Cl2 or by O3,
3
2K2MnO4 + Cl2 → 2KMnO4 + 2KCl [Unwanted MnO2 does not form]
OR
2K2MnO4 + O3 + H2O → 2KMnO4 + 2KOH + O2

Heating effect: 2KMnO 4 → K 2MnO 4 + MnO2 + O2
green 200ºC Black
at red
2K2MnO4 
→ 2K2MnO3 + O2
hot

Oxidising Property of KMnO4: (In acidic medium)


(i) MnO −4 + Fe+2 + H+ → Mn+2 + O2 + H2O +Fe3+

(ii) MnO −4 + H2O2 + H+ → Mn+2 + O2 + H2O

(iii) MnO −4 + H2S → Mn2+ + S ↓ + H2O


In alkaline solution: KMnO4 is first reduced to manganate and then to insoluble manganese dioxide. Colour
changes first from purple to green and finally becomes colourless. However, a brownish precipitate is formed
2KMnO4 + 2KOH → 2K2MnO4 + H2O + O

2K2MnO4 + 2H2O → 2MnO2 + 4KOH + 2O


alkaline
2KMnO4 + H2O  → 2MnO2 + 2KOH + 3[O]

or 2MnO −4 + H2O → 2MnO2 + 2OH– + 3[O]


Chem i str y | 31.9

In neutral or weakly acidic solution:


in presence Zn+2or ZnO
(i) 2KMnO4 + 3MnSO4 + 2H2O 
→ 5MnO2 + K2SO4 + 2H2SO4

Conversion of Mn+2 to MnO −4 :

(i) PbO2 (ii) Pb3O4 + HNO3 (iii) Pb2O3 + HNO3

(iv) NaBiO3/H+ (v) (NH4)2S2O8/H+ (vi) KIO4/H+

PLANCESS CONCEPTS

In the oxidation reactions of KMnO4 in acidic medium, only H2SO4 is used to produce an acidic medium
and not HCl or HNO3 because HCl reacts with KMnO4 and produce Cl2 while HNO3, itself acts as an
oxidising agent.
B Rajiv Reddy (JEE 2012 AIR 11)

Illustration 8: Complete the equation of following chemical reactions:  (JEE MAIN)


(i) MnO −4 (aq) + S2O32− (aq) + H2O(  ) →
(ii) CrO7− (aq) + H2S(g) + H+(aq) →

Sol:
(i) In neutral or faintly alkaline solutions
MnO −4 + 2H2O + 3e– → MnO2 + 4OH– × 8

S2O32− + 10OH– → 2SO24− + 5H2O + 8e– × 3



8MnO32− + 10OH– + H2O → 8MnO2 + 6SO24− + 2OH–

(ii) In acidic solutions


Cr2O27− + 14H+ + 6e– → 2Cr3+ + 7H2O

H2S → S + 2H+ + 2e– × 3



CrO27− + 3H2S + 8H+ → 2Cr3+ + 3S + 7H2O

Illustration 9: Write steps involved in the preparation of  (JEE ADVANCED)


(i) Mn2CrO4 from chromite ore and
(ii) K2MnO4 from pyrolusite ore.

Sol: (i) Chromite ore is fused with sodium carbonate in excess of air.
4FeCr2O4 + 8Na2CO + 7O2 → 8Na2CrO4 + 2Fe2O3 + 8CO2
Chromite ion Sod. Chromites
(ii) K2MnO4 from pyrolusite ore
2MnO2 + 4KOH + O2 → 2K2MnO4 + 2H2O

2MnO2 + 2K2CO3 + O3 → 2K2MnO4 + 2CO2


3 1 . 1 0 | d and f-block Elements

4.3 Silver Nitrate


Properties:
(i) It is called lunar caustic because in contact with skin it produces a burning sensation that of caustic soda with
the formation of finely divided silver (black colour).
(ii) Thermal decomposition
1
AgNO3 → AgNO2 + O2
2
(iii) 6AgNO3 + 3I2 + 3H2O → 5AgI + AgIO3 + 6HNO3 (excess)

(iv) Ag2SO4 → 2Ag + SO2 + O2
B
(v) A - (AgNO3) → white ppt appears quickly
added

A
B - (Na2S2O3) → It takes time to give white ppt.
added

(vi) Ag2S2O3 + H2O  → Ag2S + H2SO4

AgCl, AgBr, AgI (but not Ag2S) are soluble in Na2S2O3 forming [Ag(S2O3)2]–3 complexes

(vii) AgBr+AgNO3 


KBr
→ AgBr ↓ + KNO3
Pale yellow ppt.

212ºC
Heating effect: 2AgNO3  → 2AgNO2 + O2

2AgNO3 
300ºC
→ 2Ag + 2NO + O2
(viii)
Aqua regia
Insoluble
Zn/HCl
Ag NO3 Dil.HCl AgCl [H] Ag + HCl
Na Na2CO3
OH NaOH Ag [4AgCl + 2Na2CO3 4Ag + 4NaCl + 2CO2 + O2]
(conc.)
K2S2O8
Ag2O[2AgCl + 2NaOH Ag2O + 2NaCl + H2O]
2
O

AgO Black Glucose


2
H

Ag [Ag2O + C6H12O6 2Ag + C5H11CO2H]


Ag
Gluconic acid

Ag2O + H2O2 → 2Ag + H2O + O2

K2S2O8 + 2AgNO3 + 2H2O → 2AgO + 2KHSO4 + 2HNO3


Chem i str y | 31.11

PLANCESS CONCEPTS

AgO is supposed to be paramagnetic due to d9 configuration. But actually it is diamagnetic and exists
as AgI[AgIIIO2]
Silvering of mirror: The process of depositing a thin and uniform layer of silver on a clean glass surface is
known as silvering of mirrors. It is employed for making looking glasses, concave mirrors and reflecting
surfaces. The process is based on the reduction of ammoniacal silver nitrate solution by some reducing
agent like formaldehyde, glucose, etc. The silver film deposited on the glass is first coated with a varnish
and finally painted with red lead to prevent its loss due to scrap.
Rohit Kumar (JEE 2012 AIR 79)

4.4 Zinc Compounds


 1. Zinc oxide, ZnO (Chinese white or philosopher’s wool)
It found in nature as zincite or red zinc ore.
(a) Preparation:
(i) 2Zn + O2 → 2ZnO

(ii) ZnCO3  → ZnO + CO2

(iii) 2Zn(NO3)2  → 2ZnO + 4NO2 + O2
(iv) Zn(OH)2 

→ ZnO + H2O

(b) Physical Properties: It is a white powder, which becomes yellow on heating and again turns white on cooling,
is insoluble in water, and sublimes at 400ºC.
(c) Chemical Properties:
(i) ZnO + H2SO4 → ZnSO4 + H2O
(ii) ZnO + 2NaOH → Na2ZnO2 + H2O

(iii) ZnO + H2 → Zn + H2O
400ºC
(iv) ZnO + C → Zn + CO

2. ZnCl2 (Zinc Chloride)


(a) Preparation:

ZnO + 2HCl → ZnCl2 + H2O 



ZnCO3 + 2HCl → ZnCl2 + H2O + CO2  It crystallises as ZnCl2 . 2H2O

Zn(OH)2 + 2HCl → ZnCl2 + 2H2O 

Anhydrous ZnCl2 cannot be made by heating ZnCl2.2H2O because

ZnCl2. 2H2O  → Zn(OH)Cl + HCl + H2O

Zn(OH)Cl  → ZnO + HCl
To get anh. ZnCl2 :

Zn + Cl2 → ZnCl2


Zn + 2HCl(dry) → ZnCl2 + H2
Or Zn + HgCl2 → ZnCl2 + Hg
3 1 . 1 2 | d and f-block Elements

(b) Properties:
(i) It is deliquescent white solid (when anhydrous)
(ii) ZnCl2 + H2S → ZnS
excess
(iii) ZnCl2 + NaOH → Zn(OH)2  → Na2[Zn(OH)4]
excess
(iv) ZnCl2+ NH4OH → Zn(OH)2  → [Zn(NH3)4]2+
(c) Uses:
(i) Used for impregnating timber to prevent destruction by insects
(ii) As a dehydrating agent when anhydrous
(iii) ZnO. ZnCl2 used in dental filling

3. ZnSO4 (Zinc Sulphate)


(a) Preparation:
Zn + dil H2SO4 → ZnSO4 + H2

ZnO + dil H2SO4 → ZnSO4 + H2O

ZnCO3 + dil H2SO4 → ZnSO4 + H2O + CO2

ZnS + 2O2 → ZnSO 4 



3  Parallel reaction
ZnS + O2 → ZnO + SO2 
2 

ZnS + 4O3 → ZnSO4 + 4O2

(b) Properties

ZnSO4 .7H2O 39-70°C


ZnSO4 .6H2O >70°C
ZnSO4 .H2O >280°C
ZnSO4

>800°C
1
O + SO2 + ZnO
2 2
(c) Uses:
(i) In eye lotion
(ii) Lithopone making (ZnS + BaSO4) as white pigment

Illustration 10: The addition of NH4OH to ZnSO4 solution produces white precipitate but no precipitate is formed
if it contains NH4Cl. Why?  (JEE ADVANCED)

Sol: NH4OH is a weak hydroxide. It ionizes slightly, furnishing OH– ions. However, the OH– ions are sufficient to
cause the precipitation of Zn(OH)2 as its ionic product exceeds the Ksp.
ZnSO4 + 2NH4OH → Zn(OH)2 + (NH4)2SO4]

White ppt
In the presence of NH4Cl, the ionization of NH4OH is further suppressed and sufficient OH– ions are not available
to cause precipitation as the ionic product does not exceed the Ksp.
Chem i str y | 31.13

PLANCESS CONCEPTS

Zinc oxide is white at room temperature but turns yellow on heating


Krishan Mittal (JEE 2012, AIR 199)

4.5 Copper Compounds


 1. CuO:
(a) Preparation:

(i) CuCO3.Cu(OH)2  → 2CuO + H2O + CO2 (Commercial process)
Malachite green
(Native Cu-carbonate)
(ii) 2Cu + O2 → 2CuO & Cu2O + 1 O2 → 2CuO
2

(iii) Cu(OH)2  → CuO + H2O
250ºC
(iv) 2Cu(NO3)2  → 2CuO + 4NO2 + O2

(b) Properties:
(i) CuO is insoluble in water
(ii) Readily dissolves in dil. acids
CuO + H2SO4 → CuSO4 + H2O

CuO + HCl → CuCl2

CuO + HNO3 → Cu(NO3)2


(iii) It decomposes when, heated above 1100ºC
4CuO → 2Cu2O + O2
(iv) CuO is reduced to Cu by H2 or C under hot condition
CuO + C → Cu + CO ↑

CuO + H2 → Cu + H2O↑

2. CuCl2:
(a) Preparation:
CuO + 2HCl(conc.) → CuCl2 + H2O
Cu(OH)2.CuCO3 + 4HCl → 2CuCl2 + 3H2O + CO2

(b) Properties:
(i) It is crystallized as CuCl2.2H2O of Emerald green colour
(ii) Dilute solution in water is blue in colour due to the formation of [Cu(H2O)4]2+ complex.
(iii) conc. HCl or KCl added to dil. solution of CuCl2, the colour changes into yellow, owing to the formation
of [CuCl4]2–
3 1 . 1 4 | d and f-block Elements

(iv) The conc. aq. solution is green in colour having the two complex ions in equilibrium
2[Cu(H2O)4]Cl2 → [Cu(H2O)4]2+ + [CuCl4]2– + 4H2O
(v) CuCl2 → CuCl

•• CuCl2 + Cu-turning  → 2CuCl
•• 2CuCl2 + H2SO3 + H2O → 2CuCl + 2HCl + 2H2SO4
•• 2CuCl2 + Zn/HCl → 2CuCl + ZnCl2
•• CuCl2 + SnCl2 → CuCl + SnCl4

CuF2 .2H2O → light blue Anh. CuCl is dark brown mass obtained
2

CuCl2 .2H2O → green  by heating CuCl2 .2H2O at150º C in presence

CuBr → almost black  of HCl vap.
2

CuI2 does not exist


150ºC
CuCl2.2H2O  → CuCl2 + 2H2O
HCl gas

3. CuSO4:
(a) Preparation:
CuO + H2SO4(dil) → CuSO4 + H2O.
Cu(OH)2 +H2SO4 (dil) → CuSO4 + 2H2O.Cu(OH)2.
CuCO3 +H2SO4(dil) → CuSO4 + 3H2O + CO2
1
Cu + H2SO4 + O2 → CuSO4 + H2O [Commercial scale]
2
(Scrap)
Cu + dil. H2SO4 → no reaction {Cu is below H in electrochemical series}

(b) Properties:
(i) It is crystallized as CuSO4.5H2O
On exposure 100°C
(ii) CuSO4.5H2O CuSO4.3H2O CuSO4.H2O
Effloroscence Pale blue Bluish white
Take place
230°C

CuSO4(anh.)
800°C white
750°C
1
CuO + SO2 + O2
2 CuO + SO3

PLANCESS CONCEPTS

Anhydrous copper sulphate (white) regains its blue colour when moistened with a drop of water (it is a
test of water).
T P Varun (JEE 2012, AIR 640)
Chem i str y | 31.15

Illustration 11: Blue copper sulphate turns white on heating. Why ?  (JEE MAIN)
Sol: Hydrated copper sulphate (CuSO4.5H2O) is blue which on heating loses its water of crystallization to form
Heat
anhydrous CuSO4 a white compound. CuSO4.5H2O  → CuSO4 + 5H2O

Illustration 12: Cu+ ion is not stable in an aqueous solution. Why ?  (JEE ADVANCED)
Sol: Cu (aq) is much more stable than Cu (aq). This is because although the second ionization enthalpy of copper
2+ +

is large but Dhyd H for Cu2+(aq) is much more negative than that for Cu+(aq) and therefore it more than compensates
for the second ionization enthalpy of copper. Thus many copper (I) compounds are unstable in aqueous solution
and undergoes disproportion as follows 2Cu+(aq) → Cu2+(aq) + Cu(s)

4.6 Iron Compounds


1. FeSO4.7H2O:

(a) Preparation:
(i) Scrap Fe + H2SO4 → FeSO4 + H2↑
(dil.)
(ii) From Kipp’s waste
FeS + H2SO4(dil) → FeSO4 + H2S↑
7
(iii) FeS2 + 2H2O + O → FeSO4 + H2SO4
2 2
(b) Properties:
(i) It undergoes aerial oxidation forming basic ferric sulphate

4FeSO4 + H2O + O2 → 4Fe(OH)SO4

300ºC high
(ii) FeSO4.7H2O  → FeSO 4  → Fe2O3 + SO2 + SO3
anh. white temp.

(iii) Aq. Solution is acidic due to hydrolysis

FeSO4 + 2H2O Fe(OH)2 + H2SO4


Weak base
(iv) It is a reducing agent
•• Fe2+ + MnO −4 + H+ → Fe3+ + Mn2+ + H2O
•• Fe2+ + Cr O2− + H+ → Fe3+ + Cr3+ + H O
2 7 2

•• Au + Fe
3+ 2+
→ Au + Fe 3+

•• Fe2+ + HgCl2 → Hg2Cl2 ↓ + Fe3+


White ppt.
(v) It forms double salt. Example (NH4)2SO4.FeSO4.6H2O

2. FeO (Black):

(a) Preparation: FeC2O4 →



FeO + CO + CO2
in absence of air

(b) Properties: It is stable at high temperature and on cooling slowly disintegrates into Fe3O4 and iron
4FeO → Fe3O4 + Fe
3 1 . 1 6 | d and f-block Elements

3. FeCl2:
heating in current of HCl
(a) Preparation: Fe + 2HCl  → FeCl2 + H2
OR

2FeCl3 + H2  → 2FeCl2 + 2HCl

(b) Properties:
(i) It is deliquescent in air like FeCl3
(ii) It is soluble in water, alcohol and ether also because it is sufficiently covalent in nature
(iii) It volatilizes at about 1000ºC and vapour density indicates the presence of Fe2Cl4. Above 1300ºC density
becomes normal
(iv) It oxidizes on heating in air
12FeCl2 + 3O2 → 2Fe2O3 + 8FeCl3
(v) H2 evolves on heating in steam 3FeCl2 + 4H2O → Fe3O4 + 6HCl + H2
(vi) It can exist as a different hydrated form
FeCl2.2H2O → colorless
FeCl2.4H2O → pale green
FeCl2.6H2O → green
4. FeCl3:
(a) Preparation: 2 Fe(s) + 3 Cl2(g) → 2 FeCl3(s)
(b) Properties: Anhydrous ferric chloride is prepared by heating metallic iron in a stream of dry chlorine gas.
(i) FeCl3 solid is almost black. It sublimes at about 300ºC, giving a dimeric gas.
(ii) FeCl3 dissolves in both ether and water, giving solvated monomeric species.
(iii) Iron (III) chloride is usually obtained as yellow-brown lumps of the hydrate FeCl3.6H2O.
(iv) This is very soluble in water and is used both as an oxidizing agent, and as a mordant in dyeing.

PLANCESS CONCEPTS

Anhydrous ferric chloride is soluble in non-polar solvents like ether, alcohol, etc as it possesses covalent
bonds and has a chlorine bridge structure. Cl Cl Cl
Fe Fe
Cl Cl Cl

Saurabh Chaterjee (JEE Advanced 2013, AIR)

Illustration 13: FeCl3(aq) gives CO2 with NaHCO3(aq) Explain.  (JEE ADVANCED)

Sol: Fe3+ ions hydrolyse to form alkaline solution which reacts with NaHCO3 to liberate CO2.
Fe3+ + 3H2O → Fe(OH)3 + 3OH–
OH– + 2NaHCO3 → Na2CO3 + CO2 + H2O
Chem i str y | 31.17

f-BLOCK ELEMENTS

1. INTRODUCTION

1.1 Inner transition elements/f-block elements


The elements in which the additional electrons enters (n – 2)f orbitals are called inner transition elements. The
valence shell electronic configuration of these elements can be represented as (n-2) f0.2….14(n –1)d0,1,2ns2. These are
also called as f-block elements because the extra electrons go to f-orbitals which belongs to (n–2) th main shell.
4f-block elements are also called Lanthanides or rare earths. Similarly, 5f-block elements are called actinides or
actinones. The name Lanthanides and Actinide have been given due to close resemblance with Lanthanum and
Actinium respectively. Lanthanides constitutes the first inner transition series while actinides constitutes second
inner transition series.

General Characteristics:
 1. Electronic Configuration : [Xe] 4fn+15d06s2 or [Xe] 4fn5d16s2
 2. Oxidation state: They readily form M+3 ions. Some of them also exhibit oxidation state of +2 and +4.
 3. Colouration: Ions of Lanthanides and Actinides are coloured in the solid state as well as in aqueous solution
because of absorption of light due to f-f-transition, since they have partly filled f-orbitals.

Magnetic properties: La3+(4f0) and Lu3+(4f14) having no unpaired electrons do not show paramagnetism while all
other tri-positive ions of lanthanides are paramagnetic

Illustration 14: What is the basic difference between the electronic configuration of transition and inner transition
elements.  (JEE MAIN)

Sol: General electronic configuration of transition elements = [Noble gas] (n – 1) d1–10ns1–2 and for inner transition
elements = (n – 2) f1–14(n – 1)d0–1ns0–2. Thus, in transition elements, last electron enters d-orbitals of the penultimate
shell while in inner transition elements, it enters f-orbital of the penultimate shell.

Illustration 15: What are inner transition elements ? Decide which of the following atomic numbers are the atomic
numbers of the inner transition elements: 29, 59, 74, 95, 102, 104.  (JEE MAIN)

Sol: Inner transition elements are those which have incomplete 4f of 5f orbitals. Thus 59, 95 and 102 are inner
transition elements.

2. LANTHANIDES AND THEIR PROPERTIES


The lanthanide series of chemical elements comprises the fifteen metallic chemical elements with atomic numbers
57 through 71, from lanthanum through lutetium. These fifteen lanthanide elements, along with the chemically
similar elements, scandium and yttrium, are often collectively known as the rare earth elements.

2.1 Lanthanide Contraction


In lanthanides, the additional electron enters the 4f-sub shell but not in the valence-shell i.e. sixth shell. The
shielding effect of one electron in 4f-sub-shell by another in the same sub-shell is very little, being even smaller
than that of d-electrons, because the shape of f-sub shell is very much diffused, while there is no comparable
increase in the mutual shielding effect of 4f-electrons. This results in outermost shell electrons in the experience
increasing nuclear attraction from the growing nucleus. Consequently, the atomic and ionic radii go on decreasing
as we move from La57 to Lu71.
3 1 . 1 8 | d and f-block Elements

Lanthanides in the periodic table

Figure 31.4: Position of lanthanides in the periodic table

Consequence of Lanthanide contraction


 1. Atomic and ionic radii of post-Lanthanide elements: The atomic radii of second row transition elements
are almost similar to those of the third row transition elements because the increase in size on moving down
the group from second to third transition elements is cancelled by the decrease in size due to lanthanide
contraction.
 2. High density of post Lanthanide elements: It is because of a very small size due to lanthanide contraction
and increase in molar mass.
 3. Basic strength of oxides and hydroxides: Due to lanthanide contraction, the decrease in size of lanthanides
ions, from La3+ to Lu3+ increases the covalent character (i.e. decreases the ionic character) between Ln+3 and
OH– ions in Ln(III) hydroxides (Fajan’s rules). Thus La(OH)3 is the most basic while Lu(OH)3 is the least basic.
Similarly, there is a decrease in the basic strength of the oxides.
 4. Separation of Lanthanides: Due to the similar size (lanthanide contraction) of the lanthanides, it is difficult
to separate them. But a slight variation in their properties is utilized to separate.

PLANCESS CONCEPTS

The existence of lanthanoids in oxidation state of +2 and +4 is due to the fact that empty, half-filled or
completely filled f-subshells provide lower energy levels and the ions get stabilized. For example, Ce and
Tb show +4 oxidation state by attaining stable f0 and f7 configuration respectively whereas Eu and Yb
show +2 oxidation state by attaining stable f7 and f14 configuration, respectively.

Mredul Sharda (JEE Advanced 2013)

Illustration 16: Why is the separation of lanthanoids difficult? Explain. (JEE MAIN)

Sol: All the Lanthanoid ions are of almost the same size, so they have almost similar chemical and physical properties
and thus their separation becomes difficult.

Illustration 17: Name the members of the lanthanoid series which exhibit +4 oxidation states and those which
exhibit +2 oxidation states. Try to correlate this type of behavior with the electronic configuration of these
elements.  (JEE MAIN)

Sol: +4 = Ce, Pr, Nd, Tb, Dy. +2 = Eu, Yb. These states are accounted by the extra stability of half-filled and
completely filled f-orbitals.

2.2 Chemical Reactivity


These are very reactive metals like alkaline earth metals, however, they show very little difference in their chemical
reactivity. On strong heating with H2 and carbon, these form salt like non-stoichiometic hydrides and carbides. They
burn in oxygen to give sesquioxides M2O3 (except Ce which gives CeO3). Their ionic oxides react with water to form
Chem i str y | 31.19

insoluble hydroxides. The oxides and hydroxides being strong base react with CO2 to form carbonates (M2CO3). On
burning in sulphur these form sulphides (M2S3).

With sulphate
From sulphide, M2S3
With acid
Liberate hydrogen
With halogens
Produce halides, MX3
With water
Lanthanoid metal From hydroxides, M(OH)3+H2
With C, 2770 K From carbides, MC2
With N2
From nitrides, MN
With O2
From oxides, M2O3

Figure 31.5: Chemical reactivity of lanthanoid metal

2.3 Uses of Lanthanoids


(i) Pyrophoric alloys, known as Misch metals, contain lanthanoids about 90-95% (Ce 40.5%, lanthanum and
neodymium 44%), iron 4.5%, calcium, carbon and silicon about 10.5% are used in cigarette and gas lighters,
flame throwing tanks, toys, tank and tracer bullets as well as in shells.
(ii) Any alloy containing 30% Misch metals and 1% Zr are used for making parts of jet engines.
(iii) Cerium salts are commonly used as catalyst in petroleum cracking (cerium phosphate), volumetric analysis
and as oxidizing agent (ceric sulphate), in dying cotton, in lead accumulators etc,
(iv) Oxides of praseodymium(Pr2O3) and neodymium (Nd2O4) are used in the preparation of coloured glasses and
standard filters.
(v) Oxides of cerium and thorium are used in the preparation of gas lamp mantles.
(vi) Cerium oxide is used to prepare sunglasses as cerium cuts off heat and ultraviolet light.

3. ACTINIDES AND THEIR PROPERTIES


(i) The differentiating electron occupies 5f-subshell and thus these elements also have three outermost shells
not filled to their capacity . These are called actinoids or actinones.

89
Ac: …………..5f0, 6s2 6p6 6d1, 7s2
90Th: …………..5f1, 6s2 6p6 6d1, 7s2
103Lw: …………..5f0, 6s2 6p6 6d1, 7s2
(ii) The electronic configuration of actinoids is [Rn] 5f0–14, 6d0–2, 7s2 where [Rn] stands for radon core. Like
lanthanoids, they are placed together because of similar chemical nature.
(iii) Like lanthanoids contraction, these too show actinoid contraction due to poor shielding effect of 5f-subshells.
Thus, atomic size of actinoids too decreases gradually from Ac to Lw.
(iv) Actinoids show a range of oxidation states, which is due to comparable energies of 5f, 6d and 7s-orbitals. Tl
general oxidation state of actinoids is +3; the elements in the first half of the series show higher oxidation
states.
(v) All these elements are strong reducing agents and are very reactive metals. Actinoids are radioactive and,
therefore, it is difficult to study their chemical nature. However, relatively more stable isotopes of these
elements beyond uranium have been discovered and the chemistry of these elements has been studied to an
extent by using radiotracer techniques.
Like lanthanoids, they react with oxygen, halogens, hydrogens, sulphur and acids.
3 1 . 2 0 | d and f-block Elements

Uses of Actinoids: Only U, Th have found applications in nuclear reactions undergoing nuclear fission to produce
nuclear power and nuclear bombs.

PLANCESS CONCEPTS

The 5f orbitals extend in space beyond 6s and 6p orbitals and participate in bonding.

Vaibhav Krishnan (JEE 2009, AIR 22)

Illustration 18: The chemistry of the actinoid elements is not so smooth as that of the lanthanoids.  (JEE MAIN)

Sol: This difference is due to occurrence of a wide range of oxidation states in actinoids. Also, their radioactivity
causes a hindrance in their study.

POINTS TO REMEMBER

•• General electronic configuration of d-block elements is (n – 1)d1–10ns0,1,2 and that of f-block element is
(n-2) f0.2….14(n –1)d0,1,2ns2

•• Their melting and boiling points are high which are attributed to the involvement of (n–1)d electrons resulting
in strong metallic bonds.

•• Successive ionisation enthalpies do not increase as steeply as in the main group elements with increasing
atomic number. Hence, the loss of variable number of electrons from (n –1) d orbitals is not energetically
unfavourable.

•• Ionisation energies where the electron is removed from half-filled or completely filled orbiatls are especially
large. Hence, Zn3+ is not formed.

•• The metals, in addition to variable oxidation states, they exhibit paramagnetic behaviour, catalytic properties
and tendency for the formation of coloured ions, interstitial compounds and complexes.

•• Reactivity of these elements is calculated as a sum of heat of sublimation, ionization enthalpy as well as heat
of hydration.

•• The transition elements are sufficiently electropositive to dissolve in mineral acids. Of the first series, with the
exception of copper, all the metals are relatively reactive.

•• The transition metals react with a number of non-metals like oxygen, nitrogen, sulphur and halogens to form
binary compounds. The first series transition metal oxides are generally formed from the reaction of metals
with oxygen at high temperatures.

•• These oxides dissolve in acids and bases to form oxometallic salts.

•• The two series of inner transition elements, lanthanoids and actinoids constitute the f-block of the periodic
table. With the successive filling of the inner orbitals, 4f, there is a gradual decrease in the atomic and ionic
sizes of these metals along the series (lanthanoid contraction). This has far reaching consequences in the
chemistry of the elements succeeding them.

•• Lanthanum and all the lanthanoids are rather soft white metals. They react easily with water to give solutions
giving +3 ions. The principal oxidation state is +3, although +4 and +2 oxidation states are also exhibited by
some occasionally.
Chem i str y | 31.21

Solved Examples

JEE Main/Boards AgNO3 decomposes on exposure to light and is thus


stored in brown bottles to prevent the action of light.
Example 1: On what basis can you say that scandium 2AgNO3 → 2Ag + 2NO2 + O2
(Z = 21) is a transition elements since Zinc (Z = 30) is not?
Example 7: The species [CuCl4]2– exist but [Cul4]2– does
Sol: On the basis of incompletely filled 3d-orbitals. In
not.
case of scandium atom in its ground state (3d1), it is
regarded as a transition element. On the other hand, Sol: This can be explain by considering the reducing
zinc atom has completely filled d-orbitals (3d10) in its properties of the two ion. I– ion is a stronger reducing
ground state as well as in its oxidized state, hence it is agent than Cl– ion. It reduces Cu2+ ion into Cu+ ion.
not regarded as transition element. Hence, cupric iodide is converted into cuprous iodide.
Thus, the species [Cul4]2– does not exist.
Example 2:Explain briefly how +2 becomes more and
more stable in the first half of the first row transition Example 8: Which of the two compounds Lu(OH)3 and
elements with increasing atomic number? La(OH)3 is more basic and why?
Sol: Elements of 1st half 3d-series are: Sc, Ti, V, Cr, Mn. Sol: Large is cation more is covalent character–Fajan’s
Their electronic configuration are Sc = [Ar]4s2,3d1; Ti = rule. More the ionic character more is the basicity. Due
[Ar]4s2, 3d2; V = [Ar]4s2, 3d2; Cr = [Ar] 4s1, 3d5 and Mn = to lanthanoid contraction, the size of Lu3+ is increased
[Ar]4s2, 3d5. As we move from Sc to Mn, the number of and therefore Lu(OH)3 shows more covalent character.
empty d-orbitals reduced in +2 ion thereby imparting Thus, La(OH)3 is more ionic and thus more basic.
more stability.
Example 9: Why is europium (II) more stable than
Example 3: Give reasons for the following. Transition cerium (II)?
metals have high enthalpies of atomization.
Sol: In Eu2+, f-subshell is half-filled and d-subshell is
Sol: Due to strong interatomic interaction between completely filled and thus more stable.
valence electrons.
Example 10: In d-block metal ions, the colour of the
Example 4: What are interstitial compounds? Why are complex changes with the ligand used while in f-block
such compounds well known for transition metals? metal ion complexes, the colour remains almost constant
and does not depend upon the nature of ligand, why?
Sol: Interstitial compounds are formed when small
non-metallic atoms like H and C are trapped inside the Sol: Colour of a metal ion depends upon energy
crystal lattice of metals. difference of two energy levels involved in transition.
Splitting of degenerate d-orbitals into t2g and eg
Example 5: Why is Eº value for the M3+/M2+ couple orbitals is greatly affected by nature of ligand. i.e., why
much more positive than that of Cr3+/Cr2+ or Fe3+/Fe2+? [Ni(NH3)6]2+ is blue, [Ni(H2O)6]2+ is green and [Ni(NO2)6]4–
Explain. is brown. Color of f block metal is due to f-f transition.
4f-orbitals in lanthanoids are deeply embedded in the
Sol: Much larger third ionization energy of Mn (where atom and are deeply shielded bu 5s and 5p orbitals. Thus
the required change is d5 to d4) is mainly responsible for they remain almost unaffected by complex formation.
this. This also explains why the +3 state of Mn is of little Because of this color does not depend upon nature of
importance. ligand.

Example 6: Why is AgNO3 kept in brown coloured


bottles ?

Sol: Due to decomposition of AgNO3 in presence of light.


3 1 . 2 2 | d and f-block Elements

JEE Advanced/Boards as indicated by E° value. The EoOP for H/H+, Mn2+/Mn3+,


Cr2+/Cr3+ and Fe2+/Fe3+ are 0, – 1.5, + 0.4 and – 0.8 V.
Example 1: What is the effect of increasing pH of Thus, stability of Fe3+ ion in acid medium is less than Cr3+
K2Cr2O7 solution? but more than Mn3+.

Sol: In aqueous solution, we have Example 5: The yellow colour aqueous solution of
Cr2O27− + H2O 2CrO24− + 2H+ Na2CrO4 changes to orange red on passing CO2 gas.
Explain.
When pH < 4 (acidic medium), it exists as Cr2O27− and
the colour is orange. When pH > 7 (basic medium), it Sol: CO2 on dissolution in water produces acidic
exists as CrO −4 and the colour is yellow. medium.
CO2 + H2O → H2CO3 2H+ + CO32−
Example 2: Why does Mn(II) ion show the maximum
paramagnetic behavior among bivalent ions of the first Na2CrO4 changes to Na2Cr2O7 (orange-red) in acidic
transition series ? medium.
2CrO24− + 2H+ → Cr2O27− + H2O
Sol: The electronic configurations of Mn and Mn (II) ion
are
Example 6: An aqueous solution of inorganic compound
25
Mn : 1s22s22p63s23p63d54s2 (X) gives following reactions:
Mn+2: 1s2,2s22p63s23p63d54s0 (i) With an aqueous solution of barium chloride a
The Mn+2 ion has five unpaired electrons in its 3d subshell precipitate insoluble in dil. HCl is obtained.
which is the maximum value for a transition metal (ii) Addition of excess of KI gives a brown ppt. which
ion. Hence, Mn(II) shows the maximum paramagnetic turns white on addition of excess of hypo.
behavior (due to unpaired electrons) among bivalent
ions of the first transition series. (iii) With an aqueous solution of K4Fe(CN)6 a chocolate
coloured precipitate is obtained. Identify (X) and give
equations for the reaction for (i), (ii) and (iii) observations.
Example 3: How can AgNO3 be determined
volumetrically? Sol: (i) Step (i) suggests that compound (X) contains
SO24− radical.
Sol: AgNO3 is titrated using NH4SCN as an intermediate
solution with ferric alum as indicator. When precipitation (ii) Step (iii) suggests that the compound (X) contains
is completed, an additional drop of NH4SCN produces Cu2+ radical.
red colour with the indicator of ferric alum. (iii) Hence, the salt is CuSO4
AgNO3 + NH4SCN → AgSCN + NH4 NO3 Reactions:
  White ppt (i) CuSO 4 + B   aCl2 → BaSO 4 + CuCl2
Fe3+ + 3SCN– → Fe(SCN) 3      ( X )                     White ppt.
  Red colour                         (Insoluble in HCl)

(ii) 2CuSO 4 + 4KI → 2Cu I2 + 2K 2SO 4


Example 4: For M2+/M and M3+/M2+ systems the E°
values for some metals are as follows: ( X )          
       
2CuI2 → Cu2 I2 + I2
Cr2+/Cr – 0.9V/Cr3+/Cr2+ –0.4V
Unstable
Mn2+/Mn – 1.2V/Mn3+/Mn2+ +1.5V
I2 + 2Na2S2O3 → Na2S 4 O6 + 2NaI
Fe2+/Fe – 0.4V/Fe3+/Fe2+ +0.8V                          Colourless
Use this data to comment upon:
(iii)
The stability of Fe3+ in acid solution as compared to that
of Cr3+ or Mn3+. 2CuSO 4 + K 4 Fe ( CN)  → Cu2 Fe ( CN)  + 2K 2SO 4   
 6  6
      ( X )             Chocolate coloured ppt.
Sol: Fe3+ converts more easily to Fe2+ and Mn3+ do not
Chem i str y | 31.23

Example 7: Give complete and balanced chemical AgCl + 2NH4OH → Ag(NH3)2Cl + 2H2O
equations for the following:
(D) Soluble
(i) Mercurous nitrate reacts with excess of KI solution.
Ag2NO3 + Na2S2O3 → Ag2S2O3 + 2NaNO3
(ii) Sodium chromite solution reacts with H2O2 in
presence of NaOH. (B) (E) White

(iii) Nickel sulphate reacts with dimethyl glyoxime Ag2S2O3 → Ag2S + SO3
reagent in ammoniacal solution. (E)   Black

Sol: (i) Hg2(NO3)2 + 2KI(excess) → Hg2I2 + 2KNO3


Example 10: Pyrolusite on heating with KOH in the
Hg2I2 + 2KI → K2HgI4 + Hg presence of air gives a dark green compound (A). The
(ii) Na2CrO3 + H2O2 → Na2CrO4 + H2O solution of (A) on treatment with H2SO4 gives a purple
coloured compound (B), which gives following reactions:
CH3–C=NOH (i) KI on reaction with alkaline solution of (B) changes
(iii) NiSO4+2 +2NH4OH →
CH3–C=NOH into a compound (C)
(ii) The colour of the compound (B) disappears on
OH O
treatment with the acidic solution of FeSO4.
CH3–C=N N=C–CH3
Ni +(NH4)2SO4 +2H2O (iii) With conc. H2SO4, compound (B) gives (D) which
CH3–C=N N=C–CH3 can decompose to yield (E) and oxygen. Identify (A)
to (E) and write balanced chemical equations for the
O OH formation of (A) and (B) and for the steps (i) to (iii).

Example 8: Out of cobalt and zinc salts, which is [O]


Sol: MnO2 + 2KOH  → K2MnO4 + H2O
attracted in a magnetic field?
Pyrolusite Dark green (A)
Sol: Out of cobalt and zinc salts, the cobalt salts are 3K2MnO4 + 2H2O 
Dil. H2SO 4
→ 2KMnO4 + MnO2 + 4KOH
attracted in a magnetic field, because the cobalt ion
containing unpaired electron is characterized by a Purple (B)
permanent magnetic moment. Zn2+ ion contains 3d10 (i) 2KMnO4 + KI + H2O → KIO3 + 2MnO2 + 2KOH
configuration, i.e, no unpaired electrons, so zinc salts
are not attracted in magnetic field. (C)
(ii) 2KMnO4 + 8H2SO4 + 10FeSO4
Example 9: A certain metal (A) is boiled in dilute nitric → 2MnSO4 + K2SO4 + 5Fe2(SO4)3 + 8H2O
acid to give a salt (B) and an oxide of nitrogen (C). An
aqueous solution of (B) with brine gives a precipitate (iii) 2KMnO4 + H2SO4(conc) → K2SO4 + Mn2O7 + H2O
(D) which is soluble in NH4OH. On adding aqueous (D)
solution of (B) to hypo solution a white precipitate (E) is
2Mn2O7 → 4MnO2 + 3O2
obtained. (E) turns black on standing. Identify (A) to (E).
(D) (E)
Sol: (i) The compound (B) reacts with NaCl (brine) to
give white precipitate (D) soluble in NH4OH, so (D) is
AgCl.
(ii) Thus, (B) must contain Ag+ ion.
(iii) (B) is obtained from (A) and dil. HNO3, so (B) is
AgNO3 and (A) is Ag.

Reactions:
3Ag + 4HNO3 → 3AgNO3 + NO + 2H2O
(A)  
(B)  
(C)
AgNO3 + NaCl → AgCl + NaNO3
(B)  
(D)
3 1 . 2 4 | d and f-block Elements

JEE Main/Boards

Exercise 1 Q.11 Explain the following observations:


(a) The elements of the d-series exhibit a larger number
Q.1 Why are Mn2+ compound more stable than Fe2+ of oxidation states than the elements of f-series.
towards oxidation in their + 3 state?
(b) The Cu+ salts are colorless while Cu2+ salts are
coloured.
Q.2 Write complete equation for oxidation of Fe2+ by
Cr2O27− in acidic medium.
Q.12 Mention the direct consequence of the following
factors on the chemical behavior of the transition
Q.3 Answer the following equations: elements:
(i) Which element in the first series of transition elements (i) They have incomplete d-orbitals in the ground state
does not exhibit variable oxidation state and why ? or in one of the oxidized states of their atoms.
(ii) Why do actinides, in general exhibit a greater range (ii) They contribute more valence electrons per atom in
of oxidation states than the lanthanides ? the formation of metallic bonds.

Q.4 Explain briefly how +2 state becomes more and Q.13 What are the characteristics of the transition
more stable in the first half of the first row transition elements and why are they called transition elements?
elements with increasing atomic number ? Which of the d-block elements may not be regarded as
the transition elements?
Q.5 Write chemical equations for the reactions involved
in the manufacture of potassium permanganate from Q.14 How would you account for the following
pyrolusite ore. situations ?
(i) The transition metals generally form coloured
Q.6 What is misch metal ? Mention its two important uses. compounds.
(ii) With 3d4 configuration, Cu2+ acts as a reducing agent
Q.7 To what extent do the electronic configurations but Mn3+ acts as oxidizing agent
decide the stability of oxidation states in the first series
(iii) The actinides exhibit a larger number of oxidation
of the transition elements? Illustrate your answer with
states than the corresponding lanthanides.
examples.

Q.15 How would you account for the following


Q.8 (a) Name two properties of the central metal ion
which enable it to form stable complex entities. (i) The transition elements have high enthalpies of
atomization.
(b) Account for the following: Zinc salts are white, Cu2+
salts are coloured. (ii) The transition metals and their compounds are
found to be good catalysts in many processes.
Q.9 How do you account for the following ?
Q.16 Explain giving reasons:
(a) All scandium salts are white ?
(i) Transition metals and many of their compounds
(b) The first ionization energies of the 5d transition show paramagnetic behavior.
elements are higher than those of the 3d and 4d
(ii) The enthalpies of atomization of the transition
transition elements in respective groups ?
metals are high.

Q.10 What may be the stable oxidation state of the (iii) The transition metals generally form coloured
transition element with the following d electron compounds.
configurations in the ground state of their atoms: 3d3, (iv) Transition metals and their many compounds act as
3d5, 3d8 and 3d4? good catalyst
Chem i str y | 31.25

Q.17 Give reason for each of the following: Q.25 Describe the preparation of potassium dichromate
from iron chromite ore. What is the effect of increasing
(i) Size of trivalent lanthanide cations decreases with
pH on a solution of potassium dichromate ?
increase in the atomic number
(ii) Transition metal fluorides are ionic in nature, whereas
Q.26 Assign reason for the following:
bromides and chlorides are usually covalent in nature.
(i) From element to element, the actinides contraction
(iii) Chemistry of all the lanthanides is quite similar
is greater than lanthanide contraction.
(ii) The Eº value for Mn3+/Mn2+ couple is much more
Q.18 Discuss the general characteristics of the 3d series
positive than that for Cr3+/Cr2+.
of the transition elements with special reference to their.
(iii) Scandium (Z = 21) does not exhibit variable oxidation
(i) Atomic sizes
states and yet it is regarded as transition element.
(ii) Enthalpies of atomization
(iii) Tendency for complex formation Q.27 Describe the oxidizing action of potassium
dichromate and write the ionic equations for its reaction
Q.19 Predict which of the following will be coloured with:
in aqueous solution. Ti3+, V3+, Cu+, Sc3+, Mn3+, Fe3+ and (i) Iodide (ii) iron (iii) H2S
Co3+. Give reasons for each.
Q.28 (a) Describe the general trends in the following
Q.20 Write down electronic configuration of the properties of the first series of the transition elements:
following:
(i) Stability of +2 oxidation state
(i) La3+ (ii) Gd3+ (iii) Eu2+
(ii) Formation of oxometal ions.
(iv) Zn4+ (v) Ru2+ (vi) Ce4+
(b) Assign reason for each of the following:

Q.21 What are alloys ? Name an important alloy which (i) Transition elements exhibit variable oxidation states.
contains some of the lanthanide metals. Mention its (ii) Transition metal ions are usually coloured
uses.
Q.29 (a) Write the steps involved in the preparation of:
Q.22 (a) Give one example each of amphoteric and
(i) K2Cr2O7 from Na2CrO4
acidic oxides of transition metals
(ii) KMnO4 from K2MnO4
(b) Describe the trends in the following cases:
(b) What is meant by lanthanide contraction?
(i) Melting points of elements in the 3d transition series.
What effect does it have on the chemistry of the
(ii) Atomic sizes of elements in the 4f transition series.
elements which follow lanthanides?

Q.23 How would you account for the following ?


(i) Sc, the first member of first transition series does not Exercise 2
exhibit variable oxidation state.
Single Correct Choice Type
(ii) K2PtCl6 is well-known compound whereas
corresponding compound of nickel is not known. Q.1 The number of moles of acidified KMnO4 required
(iii) Only transition metals form complex compounds to convert one mole of sulphite ion into sulphate ion is
with ligands such as CO. (A) 2/5 (B) 3/5 (C) 4/5 (D) 1

Q.24 Use Hund’s rule to derive the electronic Q.2 N2(g) + 3H2(g)
Fe + Mo
2NH3(g); Haber’s process,
configuration of Ce3+ ion, and calculate its magnetic
Mo is used as
moment on the basis of ‘spin-only’ formula.
(A) A catalyst (B) A catalytic promoter
(C) An oxidizing agent (D) None of these
3 1 . 2 6 | d and f-block Elements

Q.3 An ornament of gold is made up of 75% of gold, it Q.10 Coinage metals show the properties of
is of……….carat.
(A) Typical elements
(A) 18 (B) 16 (C) 24 (D) 20
(B) Normal elements
(C) Inner-transition elements
Q.4 Solution of MnO −4 is purple-coloured due to
(D) None of these
(A) d-d transition
(B) Charge transfer from O to Mn
Q.11 Bayer’s reagent used to detect olefinic double
(C) Due to both d-d-transition and charge transfer bond is
(D) None of these (A) Acidified KMnO4
(B) Aqueous KMnO4
Q.5 Transition elements having more tendency to form
(C) 1% alkaline KMnO4 solution
complex than representative elements (s and p-block
elements) due to (D) KMnO4 in benzene
(A) Availability of d-orbitals for bonding
Q.12 The transition metal used in X-rays tube is
(B) Variable oxidation states are not shown by transiton
elements (A) Mo (B) Ta (C) Pb (D) Tc
(C) All electrons are paired in d-orbitals
Q.13 The higher oxidation states of transition elements
(D) f-orbitals are available for bonding are found to be in the combination with A and B, which
are
Q.6 During estimation of oxalic acid Vs KMnO4 self-
(A) F, O (B) O, N (C) Cl,O (D) S, F
indicator is
(A) KMnO4 (B) Oxalic acid Q.14 1 mole of Fe2+ ions are oxidised to Fe3+ ions with
(C) K2SO4 (D) MnSO4 help of (in acidic medium)
(A) 1/5 moles of KMnO4 (B) 5/3 moles of KMnO4
Q.7 A compound of mercury used in cosmetics, in
(C) 2/5 moles of KMnO4 (D) 5/2 moles of KMnO4
ayurvedic and yunani medicines and known as Vermilon
is
Q.15 The metals present in insulin and haemoglobin
(A) HgCl2 (B) HgS (C) Hg2Cl2 (D) HgI are respectively
(A) Zn Hg (B) Zn Fe (C) Cu, Hg (D) Cu, Fe
Q.8 Acidified chromic acid + H2O2
Org. solvent
→ X + Y, X and Y are Q.16 Solid CuSO4.5H2O having covalent, ionic as well
(A) CrO5 and H2O (B) Cr2O3 and H2O as co-ordinate bonds. Copper atom/ion forms……….co-
ordinate bonds with water.
(C) CrO2 and H2O (D) CrO and H2O
(A) 1 (B) 2 (C) 3 (D) 5

Q.9 Transition element are usually characterised by


Q.17 Purple of cassius is:
variable oxidation states by Zn does not show this
property because of (A) Pure gold (B) Colloidal solution of gold
(A) Completion of np-orbitals (C) Gold (I) hydroxide (D) Gold (III) chloride
(B) Completion of (n – 1)d orbitals
Q.18 Number of moles of SnCl2 required for the
(C) Completion of nd orbitals reduction of 1 mole of K2Cr2O7 into Cr2O3 is (in acidic
(D) None of these medium)
(A) 3 (B) 2 (C) 1 (D) 1/3
Chem i str y | 31.27

Q.19 The aqueous solution of CuCrO4 is green because Q.3 Potassium manganate (K2MnO4) is formed when
it contains.  (1988)
(A) Green Cu2+ ions (A) Chlorine is passed into aqueous KMnO4 solution
(B) Manganese dioxide is fused with potassium
(B) Green CrO24− ions
hydroxide in air
(C) Blue Cu2+ ions and green CrO24− ions (C) Formaldehyde reacts with potassium permanganate
(D) Blue Cu2+ ions and yellow CrO24− ions in presence of a strong alkali
(D) Potassium permanganate reacts with conc. sulphuric
Q.20 Manganese steel is used for making railway tracks acid
because
Q.4 The aqueous solution of the following salts will be
(A) It is hard with high percentage of Mn
coloured in the case of  (1990)
(B) It is soft with high percentage of Mn
(A) Zn(NO3)2 (B) LiNO3
(C) It is hard with small concentration of manganese (C) Co(NO3)2 (D) CrCl3 (E) Potash alum
with impurities
(D) It is soft with small concentration of manganese Q.5Which of the following alloys contains Cu and Sn
with impurities  (1993)
(A) Bronze (B) Brass (C) Gun metal (D) Type metal
Q.21 Transition elements in lower oxidation states act
as Lewis acid because Q.6 Which of the following statement(s) is (are) correct.
When a mixture of NaCl and K2Cr2O7 is gently warmed
(A) They form complexes
with conc. H2SO4  (1998)
(B) They are oxidizing agents
(A) A deep red vapours is evolved
(C) They donate electrons (B) The vapours when passed into NaOH solution gives
(D) They do not show catalytic properties yellow solution of Na2CrO4
(C) Chlorine gas is evolved
Q.22 The Ziegler-Natta catalyst used for polymerization (D) Chromyl chloride is formed
of ethane and styrene is TiCl4 + (C2H5)3Al, the catalyzing
species (active species) involved in the polymerization is Q.7 Addition of high proportions of manganese
(A) TiCl4 (B) TiCl3 (C) TiCl2 (D) TiCl makes steel useful in making rails or railroads, because
manganese  (1998)
(A) Gives hardness to steel
Previous Years’ Questions (B) Helps the formation of oxide of iron

Q.1 In nitroprusside ion, Iron and NO exist as FeII and (C) Can remove oxygen and sulphur
NO+ rather than FeIII and NO. These forms can be (D) Can show highest oxidation state of +7
differentiated by  (1998)
(A) Estimating the concentration of iron Q.8 Assertion: Rusting of an iron is an example of
(B) Measuring the concentration of CN– corrosion. (2008)
(C) Measuring the solid state magnetic moment Reason: Rusting of iron is decreased by acids and
(D) Thermally decomposing the compound electrolytes

Q.2 Among the following the compound that is both Q.9 Assertion: AgBr is used in photography (1996)
paramagnetic and coloured is  (1997)
Reason : AgBr undergoes photochemical reaction.
(A) K2Cr2O7 (B) (NH4)2(TiCl6)
(C) VOSO4 (D) K3[Cu(CN4)] Q.10 Assertion: Tungsten filament is used in electric
bulbs. (1994)
Reason : Tungsten is a metal of high melting point.
3 1 . 2 8 | d and f-block Elements

Q.11 Assertion: In transition elements ns orbital is Q.17 In context of the lanthanoids, which of the
filled up first and (n – 1)d afterwards, during ionization following statements is not correct? (2012)
ns electrons are lost prior to (n – 1)d electrons. (1995)
(A) All the members exhibit +3 oxidation state
Reason: The effective nuclear charge felt by (n – 1)d
(B) Because of similar properties the separation of
electrons is higher as compared to that of ns electrons.
lanthanoids is not easy.
(C) Availability of 4f electrons results in the formation
Q.12 Assertion: The degree of complex
of compounds in +4 state for all the members of the
formation in actinides decreases in the order
series.
M4 + > MO22+ > M3+ > MO2+ (1997)

(D) There is a gradual decrease in the radii of the
Reason: Acitnides form complex with π-bonding members with increasing atomic number in the series.
ligands such as alkyl phosphines and thioethers.
Q.18 Which of the following arrangements does not
Q.13 Larger number of oxidation states are exhibited represent the correct order of the property stated
by the actinoids than those by the lanthanoids, the against it? (2013)
main reason being  (2008)
(A) V 2+ < Cr 2+ < Mn2+ < Fe2+ : Paramagnetic behaviour
(A) 4f orbitals more diffused than the 5f orbitals
(B) Ni2+ < Co2+ < Fe2+ < Mn2+ : Ionic size
(B) Lesser energy difference between 5f and 6d than
(C) Co3+ < Fe3+ < Cr3+ < Sc3+ : Stability in aqueous
between 4f and 5d orbitals
solution
(C) More energy difference between 5f and 6d than
(D) Sc < Ti < Cr < Mn: Number of oxidation states
between 4f and 5d orbitals
(D) More reactive nature of the actinoids than the
Q.19 The pair having the same magnetic moment is:
lanthanoids
 (2016)

Q.14 Knowing that the Chemistry of lanthanoids (Ln) [At. No.: Cr = 24, Mn = 25, Fe = 26, Co = 27]
is dominated by its +3 oxidation state, which of the 2+ 2+
following statements in incorrect?  (2009) (A) Cr (H2O )  and Fe (H2O ) 
 6  6
2+ 2+
(B) Mn (H2O )  and Cr (H2O ) 
(A) Because of the large size of the Ln (III) ions the  6  6
2+
and Fe (H2O ) 
2−
bonding in its compounds is predominantly ionic in (C) CoCl4 
 6
character. 2+
(D) Cr (H2O ) 
2−
and CoCl4 
(B) The ionic sizes of Ln (III) decrease in general with  6
increasing atomic number.
(C) Ln (III) compounds are generally colourless.
(D) Ln (III) hydroxides are mainly basic in character.

Q.15 The correct order of E 2+ values with negative


M /M
sign for the four successive elements Cr, Mn, Fe and Co
is (2010)
(A) Mn > Cr > Fe > Co (B) Cr > Fe > Mn > Co
(C) Fe > Mn > Cr > Co (D) Cr > Mn > Fe > Co

2−
Q.16 The magnetic moment (spin only) of NiCl4  is
 (2011)
(A) 5.46 BM (B) 2.83 BM
(C) 1.41 BM (D) 1.82 BM
Chem i str y | 31.29

JEE Advanced/Boards

Exercise 1 Q.10 (i) K2MnO4 in acidic medium changes to MnO2


and KMnO4. What would be the equivalent weight of
Q.1 Explain why the greenish solution of potassium K2MnO4.
manganate turns purple when CO2 is bubbled in the (ii) Draw the structures of MnO −4 and CrO24−
solution.
Q.11 A complex has the formula PtCl4.2KCl. The
Q.2 Explain why mercurous compounds are formed electrical conductance of the compound shows that
when mercury is oxidized in a limited amount of an each formula unit had 3 ions. AgNO3 on treatment with
oxidizing agent whereas with an excess of oxidizing the complex does not give a precipitate of AgCl. What
agent mercuric compounds are formed. should be the correct formula of the complex?

Q.3 Explain why [Co(NH3)6]3+ is diamagnetic and [CoF6]3– Q.12 [NiCl4]2– and [Ni(CO)4] both are tetrahedral in
is strongly paramagnetic. shape but [NiCl4]2– is paramagnetic whereas [Ni(CO)4]
is diamagnetic. Explain in the difference in magnetic
Q.4 What happens when NaOH or NH4OH are added in behavior both the complexes.
excess to AlCl3 and ZnCl2 ?
Q.13 FeSO4 solution is mixed with (NH4)2SO4 in the molar
Q.5 Why is zinc oxide used in paints instead of lead ratio 1 : 1. It gives test of Fe2+. When CuSO4 is mixed
salts? with liquid ammonia (in the ratio 1 : 4) the mixture does
not give test of Cu2+. Explain the difference.

Q.6 Identify from [A] to [E].


Q.14 (A), (B) and (C) are three complexes of Cr(III).
Its formula is H12O6Cl3Cr. All the three complexes
Colorless salt [A] NaOH [B] (White precipitate)
have water and chloride ions as ligands. (A) does
+AgNO3 Dissolves in NaOH not react with conc. H2SO4 whereas (B) and (C)
loss 6.75% and 13.5% of their original weight
[C]
[E] White respectively on treatment with conc. H2SO4. Find
precipitate Pass H2S [A], [B] and [C]
[D] (White precipitate)
Q.15 A metal complex having composition Cr(NH3)4Cl2.
Q.7 Why are melting and boiling points of zinc,
Br has been isolated in two forms (A) and (B). (A) reacts
cadmium, and mercury is lower than those of other
with AgNO3 to give a white precipitate soluble in dilute
transition metals?
ammonia while (B) gives a pale yellow precipitate
soluble in concentrated ammonia. Write the formulae
Q.8 Why is HCl not used to acidify KMnO4 for volumetric of (A) and (B) and hybridization state of Cr in the
estimations? compounds.

?
Q.9 Colourless salt (A)  → (B) + (C) gas. Q.16 A monomeric compound of cobalt gives the
(B) Dissolves both in acid and alkali solution. following data on quantitative analysis: Co3+: 21.24%;
NH3: 24.77%; Cl–: 12.81%; SO34− : 34.65%; H2O: 6.53%
(C) Turns lime water milky and acidified K2Cr2O7 solution Deduce the empirical formula of the complex and the
green. possible isomers.
(A) Gives white precipitate (D) with H2S when the
solution is alkaline. Identify [A] to [D].
3 1 . 3 0 | d and f-block Elements

Exercise 2 Q.9 Pick out the incorrect statements:


(A) MnO2 dissolves in conc. HCl, but does not form
Single Correct Choice Type Mn4+ ions
Q.1 Cr2O27− 2CrO24− , X and Y are respectively (B) MnO2 oxidizes hot concentrated H2SO4 liberating
oxygen
(A) X = OH–, Y = H+ (B) X = H+, Y = OH– (C) K2MnO4 is formed when MnO2 in fused KOH is
(C) X = OH , Y = H2O2

(D) X = H2O2, Y = OH – oxidized by air, KNO3, PbO2 or NaBiO3
(D) Decomposition of acidic KMnO4 is not catalysed by
Q.2 CrO3 dissolves in aqueous NaOH to give sunlight
(A) Cr2O27− (B) CrO24−
Q.10 The rusting of iron is formulated as Fe2O3.xH2O
(C) Cr(OH)3 (D) Cr(OH)2 which involves the formation of
(A) Fe2O3 (B) Fe(OH)3
Q.3 (NH4)2Cr2O7 (Ammonium dichromate) is used in fire
works. The green coloured powder blown in air is (C) Fe(OH)2 (D) Fe2O3 + Fe(OH)3

(A) Cr2O3 (B) CrO2 (C) Cr2O4 (D) CrO3


Q.11 Metre scales are made-up of alloy

Q.4 The d-block elements which is liquid at room (A) Invar (B) Stainless steel
temperature, having high specific heat, less reactivity (C) Electron (D) Magnalium
than hydrogen and its chloride (MX2) is volatile on
heating is Q.12 A metal M which is not affected by strong acids
(A) Cu (B) Hg (C) Ce (D) Pm like conc. HNO3, conc. H2SO4 and conc. solution of
alkalies like NaOH, KOH forms MCl3 which finds use for
toning in photography. The metal M is-
Q.5 Iron becomes passive by………………due to formation
of…………… (A) Ag (B) Hg (C) Au (D) Cu
(A) Dil. HCl, Fe2O3
Q.13 KMnO4 + HCl → H2O + X(g), X is a (acidified)
(B) 80% conc. HNO3, Fe3O4
(A) Red liquid (B) Violet gas
(C) Conc. H2SO4, Fe3O4
(C) Greenish yellow (D) Yellow-brown gas
(D) Conc. HCl, Fe3O4
Q.14 In nitroprusside ion, the iron exists as Fe2+ and
Q.6 Cu + conc. HNO3 → Cu(NO3)2 + X (oxide of NO as NO+ rather than Fe3+ and NO respectively. These
nitrogen); then X is- forms of ions are established with the help of

(A) N2O (B) NO2 (C) NO (D) N2O3 (A) Magnetic moment in solid state
(B) Thermal decomposition method
Q.7 When KMnO4 solution is added to hot oxalic acid (C) By reaction with KCN
solution, the decolourisation is slow in the beginning
but becomes instantaneous after some time. This is (D) By action with K2SO4
because
(A) Mn2+ acts as auto catalyst Multiple Correct Choice Type

(B) CO2 is formed Q.15 The metal(s) which does/do not form amalgam
(C) Reaction is exothermic is/are

(D) MnO −4 catalyses the reaction (A) Fe (B) Pt (C) Zn (D) Ag

Q.8 CuSO4 solution reacts with excess KCN to give Q.16 The highest oxidation state among transition
element is
(A) Cu(CN)2 (B) CuCN
(A) + 7 by Mn (B) + 8 by Os
(C) K2[Cu(CN)2] (D) K3[Cu(CN)2]
(C) +8 by Ru (D) + 7 by Fe
Chem i str y | 31.31

Q.17 Amphoteric oxide(s) is/are Q.25 Addition of non-metals like B and C to the
interstitial sites of a transition metal results the metal-
(A) Al2O3 (B) SnO (C) ZnO (D) Fe2O3
(A) Of more ductability (B) Of less ductability
Q.18 Interstitial compounds are formed by- (C) Less malleable (D) Of more hardness
(A) Co (B) Ni (C) Fe (D) Ca
Q.26 Mercury is a liquid at 0ºC because of-
Q.19 To an acidified dichromate solution, a pinch of (A) Very high ionization energy
Na2O2 is added as shaken. What is observed:
(B) Weak metallic bonds
(A) Blue colour
(C) High heat of hydration
(B) Orange colour changing to green
(D) High heat of sublimation
(C) Copious evolution of oxygen
(D) Bluish-green precipitate Q.27 The ionization energies of transition elements are-
(A) Less than p-block elements
Q.20 Amongst CuF2, CuCl2 and CuBr2
(B) More than s-block elements
(A) Only CuF2 is ionic
(C) Less than s-block elements
(B) Both CuCl2 and CuBr2 are covalent
(D) More than p-block elements
(C) CuF2 and CuCl2 are ionic but CuBr2 is covalent
(D) CuF2, CuCl2as well as CuBr2 are ionic Q.28 The catalytic activity of transition elements is
related to their-
Q.21 CuSO4(aq) + 4NH3 → X, then X (A) Variable oxidation states
(A) [Cu(NH3)4] 2+
(B) Surface area
(B) Paramagnetic (C) Complex formation ability
(C) Coloured (D) Magnetic moment
(D) Has magnetic moment 1.73 BM
Q.29 In the equation: M + 8CN– + 2H2O + O2 →
Q.22 Amphoteric oxide(s) of Mn is/are 4[M(CN)2]– + 4OH–, metal M is-

(A) MnO2 (B) Mn3O4 (A) Ag (B) Au (C) Cu (D) Hg

(C) Mn2O7 (D) MnO


Q.30 An elements of 3d-transtion series shows two
oxidation states x and y, differ by two units then-
Q.23 The lanthanide contraction is responsible for the
(A) Compounds in oxidation state x and ionic if x > y
fact that
(B) Compounds in oxidation state x are ionic if x < y
(A) Zr and Hf have same atomic sizes
(C) Compound in oxidation state y are covalent if x < y
(B) Zr and Hf have same properties
(D) Compounds in oxidation state y are covalent if x > y
(C) Zr and Hf have different atomic sizes
(D) Zr and Hf have different properties
Q.31 Ion(s) having non zero magnetic moment (spin
only) is/are
Q.24 Potash alum is a double salt, its aqueous solution
(A) Sc3+ (B) Ti3+ (C) Cu2+ (D) Zn2+
shows the characteristic of-
(A) Al3+ ions (B) K+ ions
(C) SO24− ions (D) Al3+, ions but not K+ ions
3 1 . 3 2 | d and f-block Elements

Assertion Reasoning Type Q.3 Which of the following pair is expected to exhibit
same colour in solution?  (2005)
Questions given below consist of two statements each
printed as Assertion (A) and Reason (R); while answering (A) VOCl2; FeCl2 (B) CuCl2; VOCl2
these questions you are required to choose any one of (C) MnCl2; FeCl2 (D) FeCl2; CuCl2
the following four responses:
(A) if both assertion and reason are true and reason is Q.4 Which of the following will not be oxidized by O3?
the correct explanation of assertion  (2005)
(B) if both assertion and reason are true but reason is (A) KI (B) FeSO4 (C) KMnO4 (D) K2MnO4
not correct explanation of assertion
(C) if assertion is true but reason is false Q.5 Which of the following alloys contains Cu and Zn ?
 (1993)
(D) if assertion is false and reason is true
(A) Bronze (B) Brass (C) Gun metal (D) Type metal
Q.32 Statement-I: Equivalent mass of KMnO4 is equal
to one-third of its molecular mass when it acts as a Q.6 Which of the following statement(s) is/are correct
oxidizing agent in an alkaline medium. when a mixture of NaCl and K2Cr2O7 is gently warmed
with conc. H2SO4? (1998)
Statement-II: Oxidation number of Mn is + 7 in KMnO4.
(A) A deep red vapours is formed
Q.33 Statement-I: K2CrO4 has yellow colour due to (B) Vapours when passed into NaOH solution gives a
charge transfer. yellow solution of Na2CrO4
Statement-II: CrO24− ion is tetrahedral in shape. (C) Chlorine gas is evolved
(D) Chromyl chloride is formed
Q.34 Statement-I: The highest oxidation state of
chromium in its compounds is +6. Q.7 Which of the following statement(s) is/are correct?
Statement-II: Chromium atom has only six electrons in  (1998)
ns and (n – 1) d orbitals. (A)The electronic configuration of Cr is [Ar] 3d54s1
(Atomic number of Cr = 24)
Q.35 Statement-I: Tungsten has a very high melting (B) The magnetic quantum number may have a negative
point. value
Statement-II: Tungsten is a covalent (C) In silver atom 23 electrons have a spin of one type and
24 of the opposite type (Atomic number of Ag = 47)
Q.36 Statement-I: Cu+ ion is colourless. (D) The oxidation state of nitrogen in HN3 is – 3
Statement-II: Four water molecules are coordinated to
Cu+ ion. Q.8 Reduction of the metal centre in aqueous
permanganate ion involves  (2011)
(A) Three electrons in neutral medium
Previous Years’ Questions (B) Five electrons in neutral medium
(C) Three electrons in alkaline medium
Q.1 The pair of compounds having metals in their
(D) Five electrons in acidic medium
highest oxidation state is  (2004)
(A) MnO2, FeCl3 (B) [MnO4]–,CrO2Cl2
Q.9 A compound of vanadium has a magnetic moment
(C) [Fe(CN)6]3–, [Co(CN)3] (D) [NiCl4]2–, [CoCl4]– of 1.73 BM. Work out the electronic configuration of
the vanadium ion of the compound.  (1997)
Q.2 When I– is oxidized by MnO −4 in alkaline medium,
I– converts into- (2004) Q.10 Give reasons: CrO3 is an acid anhydride.  (1999)
(A) IO3− (B) I2 (C) IO −4 (D) IO–
Chem i str y | 31.33

Q.11
Moist air Zn Q.17 Among the following complexes (K − P ) (2011)
B (MCl4) A 
K 3 Fe ( CN)  (K ) , Co (KNH3 )  CI3 , (L ) ,

{
White fumes Purple
with pungent (M=Transition color  6  6
element-colorless)
smell
Na3 Co ( oxalate )  (M) , Ni (H2O )  CI2 (N) ,
 3  6
Identify the metal M and hence MCl4. Explain the
difference in colours of MCl4 and M. (2005) K 2 Pt ( CN)  ( O ) and  Zn (H2O )  (NO3 ) (P )
 4  6 2

Q.12 Among the following metal carbonyls, the C − O (A) K, L, M N (B) K, M, O, P


bond order is lowest in  (2007)
(C) L, M, O, P (D) L, M, N, O
+
(A) Mn ( CO )  (B) Fe ( CO ) 
 6
 5
Q.18 The colour of light absorbed by an aqueous
(C) Cr ( CO )  (D)  V ( CO )  solution of CuSO 4 is  (2012)
 6
 6
(A) Orange-red (B) Blue-green
Q.13 Among the following the coloured compound is (C) Yellow (D) Violet
 (2008)
(A) CuCI (B) K 3 Cu ( CN) 
 4 Q.19 Consider the following complex ions, P, Q and R
(D) Cu ( CH3CN)  BF4
2+ 3+
(C) CuF2 = FeF6  , Q  V (H2O ) =
P =
3−
 and R Fe (H O ) 
 4  6  2 6

The correct order of the complex ions, according to


Q.14 The spin only magnetic moment value (in Bohr
their spin–only magnetic moment values (in B.M.) is 
magnetron units) of Cr(CO)6 s (2009)
 (2013)
(A) 0 (B) 2.84 (C) 4.90 (D) 5.92 (A) R < Q < P (B) Q < P < Q
(C) R < P < Q (D) Q < P < R
Q.15 Match each of the reactions given in column I
with the corresponding product(s) given in column II.
Q.20 For the octahedral complexes Fe3+ in SCN‒
 (2009)
(thiocyanato-S) and in CN‒ ligand environments, the
Column I Column II difference between the spin-only magnetic moments in
Bohr magnetrons (When approximated to the nearest
(A) Cu + dilHNO3 (p) NO integer) is [Atomic number of Fe = 26]  (2015)
(B) Cu + concHNO3 (q) NO2
(C) Zn + dil HNO3 (r) N2O Q.21 Among [Ni(CO)4], [NiCl4]2‒ [Co(NH3)4 Cl2], Cl,
Na3 [CoFe6], Na2O2 the total number of paramagnetic
(D) Zn + conc HNO3 (s) Cu(NO3)2
compounds is (2016)
(t) Zn(NO3)2
(A) 2 (B) 3 (C) 4 (D) 5

Q.16 The complex showing a spin-only magnetic


Q.22 The geometries of the ammonia complexes
moment of 2.82 B.M. is  (2010)
Ni2+ ,Pt2+ and Zn2+ respectively, are  (2016)
(A) Ni ( CO )
2−
(B) NiCI 4  (A) Octahedral, square planar and tetrahedral
4
2− (B) Square planar, octahedral and tetrahedral
(C) Ni (PPh2 ) (D) Ni ( CN) 
4  4
(C) Tetrahedral, square planar and octahedral
(D) Octahedral, tetrahedral and square planar
3 1 . 3 4 | d and f-block Elements

PlancEssential Questions
JEE Main/Boards JEE Advanced/Boards

Exercise 1 Exercise 1
Q.3 Q.5 Q.6 Q.5 Q.7 Q.9

Q.9 Q.12 Q.14 (ii) Q.11 Q.13

Q.17 Q.19 Q.22 (a)


Exercise 2
Q.23 (ii) Q.29
Q.4 Q.8 Q.11
Exercise 2 Q.14 Q.19 Q.20

Q.3 Q.4 Q.8 Q.24 Q.26 Q.29

Q.11 Q.19 Q.21


Previous Years’ Questions
Previous Years’ Questions Q.8 Q.11

Q.1 Q.2 Q.6


Q.9

Answer Key

JEE Main/Boards
Exercise 2
Single Correct Choice Type

Q.1 A Q.2 B Q.3.A Q.4 B Q.5 A Q.6 A

Q.7 B Q.8 A Q.9 B Q.10 D Q.11 C Q.12 A

Q.13 A Q.14 A Q.15 B Q.16 D Q.17 B Q.18 A

Q.19 D Q.20 A Q.21 A Q.22 B

Previous Years’ Questions


Q.1 C Q.2 C Q.3 B, C Q.4 C, D Q.5 A, B Q.6 A, B, D Q.7 A, C

Q.8 C Q.9 B Q.10 A Q.11 A Q.12 B Q.13 B Q.14 C

Q.15 A Q.16 B Q.17 C Q.18 A Q.19 A


Chem i str y | 31.35

JEE Advanced/Boards Q.8 See text for the reaction of HCl and KMnO4.

Q.9 [(A) = ZnSO3; (B) = ZnO; (C) = SO2; (D) = ZnS]


Exercise 1
Q.10 3/2 M Q.11 K2[PtCl6]
Q.1 Formation of MnO −4
Q.12 Cl– is a weak ligand, so no pairing of electrons. CO
Q.2 Due to disproportionation of Hg22+ is a strong field ligand and, therefore, causes pairing of
electrons.
Q.3 NH3-strong ligand causes pairing of electrons. So
complex is diamagnetic. F– is a weak ligand and pairing Q.13 In the first case, a double salt is formed. In the
is not possible. second case, a complex is formed. So, no test.
Q.4 [Al(OH)3] and NaAlO2; Zn(OH)2 and Na2ZnO2 also Q.14 (A) = [Cr(H2O)6]Cl3; (B) = [Cr(H2O)5]Cl2H2O
[Zn(NH3)4]2+
(C) = [CrCl2(H2O)4]Cl2.2H2O
Q.5 [ZnS is white but PbS is black]
Q.15 (A) = [Cr(NH3)4ClBr]+Cl– and (B) = [Cr(NH3)4Cl2]+Br–.
Q.6 [A] = ZnCl2; [B] = Zn(OH)2; [C] = Na2ZnO2; [D] = ZnS;
[E] = AgCl Q.16 [Co(NH3)4Cl(H2O)]SO4; [Co(NH3)4(SO4)(H2O)]
Cl[Co(NH3)4(SO4)(Cl)]H2O
Q.7 All the three elements have d10

Exercise 2

Single Correct Choice Type

Q.1 A Q.2 B Q.3 A Q.4 B Q.5 B Q.6 B


Q.7 A Q.8 D Q.9 D Q.10 D Q.11 A Q.12 C
Q.13 C Q.14 A

Multiple Correct Choice Type

Q.15 A, B Q.16 B, C Q.17 A, B, C Q.18 A, B, C Q.19 A, C Q.20 A, B

Q.21 A, B, C, D Q.22 A, B Q.23 A, B Q.24 A, B, C Q.25 B, C, D Q.26 A, B

Q.27 A, B Q.28 A, B, C Q.29 A, B Q.30 B, C Q.31 B, C

Assertion Reasoning Type

Q.32 A Q.33 B Q.34 A Q.35 C Q.36 C

Previous Years’ Questions


Q.1 B Q.2 A Q.3 B Q.4 C Q.5 A, B, C Q.6 A, B, C, D

Q.7 A, B, C Q.8 A Q.12 B Q.13 C Q.14 A

Q.15 A → p, s; B → q, s; C → r, t; D → q, t Q.16 B Q.17 C Q 18 A

Q.19 B Q.20 4 Q.21 B Q.22 A


3 1 . 3 6 | d and f-block Elements

Solutions

JEE Main/Boards Sol 7: Electronic configurations play an important role


in deciding the stability of an ion. Generally half-filled &
full filled states are more stable. It also depends on no.
Exercise 1 of unpaired e–, symmetry, exchange pairs, etc.

Sol 1: 2+
Mn
4s 3d stability
2+
Mn 2+
Cr
4s 3d stable
2+ → Not so stable.
Fe

Mn2+ has half-filled 3d orbitals. Sol 8: (a) To form stable complexes, the metal most
have
So, its stable.
(i) Electronic configuration (ii) Hybridisation

Sol 2: Fe+2 → Fe+3 + e– 4s


4s 3d
3d
(b) Zn+2+2
Cr2O27− + 14H+ + 6e– → 2Cr+3 + 7H2O Zn
unpaired ee--
no unpaired
→ nonounpaired e–.
Sol 3: (i) Zn doesn’t show variable O. S. because of 4s 3d
stable pseudo inert gas configuration of Zn+2 4s 3d

+2
Cu
Cu
+2

Zn2+ → [Ar] 3d10


unpaired ee--
11 unpaired
(ii) Actinides show greater range of O. S. because of → 1 unpaired e–.
comparable energies of 5f, 6d, 7s-orbitals. So, Cu+2 shows colour but not Zn2+.

Sol 4: In the first half of 3d elements, Mn2+ implies Sol 9: (a) All scandium salts have Sc+3
that there are unpaired e– in 3d shell as we go from Sc
to Mn, the exchange pairs increases as Mn2+ is nearly Sc+3 → [Be] 3s2 3p6
symmetrical. There are no unpaired e– to show colour.
(b) Due to lanthanide contraction, the sizes of 5d
Sol 5: Pyrolusite → MnO2 (impure) elements are smaller compared to expected size. But
fusion nuclear charge will be high.
2MnO + 4KOH +MnO
Pyrolusite
2
O2  → 2K2MnO4 + 2H2O
2 (impure)
2MnO2 + 4KOH + O2 fusion fusion
3MnO2 + 6KOH + KClO3  → 3K2MnO4 + KCl + Sol 10: 3d3 → V → +3
2K2MnO4 + 2H2O
3H2O 3s5 → Mn → +7, +2
3MnO2 + 6KOH + KClO2 fusion
K2MnO4 
electrolysis 3K KMnO
→ 2MnO4 4+ KCl + 3H2O 3d8 → Ni ; → +2
electrolysis
K2MnO4 KMnO4 3d4 → Cr → +6, +3
+ -
H2O H + OH
Sol 11: (a) Most of the f-block elements show +3
2- - -
MnO4 MnO4 + e
+
2H + 2e
-
H2 oxidation state (few have +2, +4). It is because of the
fact that its outer shell has 2s electron & penultimate
Sol 6: Misch metal is an alloy of rare-earth elements shell has 1d e–. So, stability of there O. S has less
in various naturally occurring proportion. Generally it dependence on f e–.
contains 50% cerium, 25% lanthanum & small amounts (b) Cu+ → 3d10 → No. unpaired e– → White
of other. It is used preparation of most of the rare earth Cu2+ → 3d9 → 1 unpaired e– → Colour
elements. When alloyed with iron, its was is lighters.
Chem i str y | 31.37

Sol 12: (i) They can exhibit multiple oxidation sates. They Sol 19: Ti+3, V+3, Mn3+, Fe3+, Co3+ have unpaired e–. So,
can accept e– pairs from ligands to form complexes. they are colored. Cu+, Sc3+ have no unpaired e–. So, they
(ii) Metallic bond strength is very high. So, they are are not colored.
strong metals.
Sol 20:
Sol 13: They are called transition elements because (a) La3+ → [Xe]
their position in the periodic table is between s & p
blocks their properties are transitional between highly (b) Cd3+ → [Xe] 4f7
reactive s-metals & constant bond forming p elements. (c) Eu2+ → [Xe] 4f7
Zn, Cd, Hg has ns2 (n – 1)d10 configuration, so, they are
not included in transition elements. (d) Zn4+ → [Ar] 3d10
(e) Ru2+ → [Kr] 5s2 4d4
Sol 14: (i) Most of the transition metals form coloured (f) Ce4+ → [Xe]
compounds due to the presence of unpaired e– or
complex formation.
Sol 21: Alloy is a mixture a solid solution compared
(ii) Mn+2 has half-filled orbitals, So Mn+3 acts as oxidizing of a metal and another element. Misch metal contains
agent. Cr+ has full-filled d-orbitals, So Cr2+ acts as Lanthanoids. It is used in preparation of f-block
reducing agent. elements. When mixed with iron, its used in lighters.
(iii) The energy difference between 7s, 7d, 5f is very less.
So, they can show multiple oxidation states. Sol 22: (a) ZnO → Amphoteric.
Chromium oxide → Acidic
Sol 15: (i) They have high enthalpies of atomisation
because of the strong metallic bonds. Each element in (b) (i) M. P’s first increases to the right till chromium
d-block provides more electrons for metallic bonding group, then decreases.
compared to s-block metals. (ii) The size decreases as we go right in the series. It
(ii) They are good catalysts, because of the presence decreases due to poor-shielding effect of e–.
of more than 1 stable O. S So, they can be used for
oxidation & reductions. Sol 23: (i) Configuration of Sc → 4s2 3d1
It has only 1 stable O. S. i. e. +3 because Sc+, Sc+2 have
Sol 16: (i) Paramagnetic is due to the presence of
no stabilising factors & Sc+3 is inert gas configuration.
unpaired e– many transition metals have unpaired e– in
d-orbitals. (ii) K2NiCl6 doesn’t exist because of the small size of Ni
compared to Pt. It cannot hold 6 Cl– ions around it.
Sol 17: (i) d-orbitals have poor shielding effect. As (iii) Co is a strong ligand and metals need vacant
atomic number increases, there is increase in nuclear d-orbitals for forming bonds.
charge also. So, the size decreases.
(ii) The electronegativity difference between transition Sol 24: Ce → [Xe] 4f1 5d1 4f1
metals & F is high but not so for Cl, Br.
Ce+3 → [Xe] 4f1
(iii) Most of them have similar sizes, they show a
common stable oxidation state (+3). Magnetic moment = n(n + 2) BM
and n = 1 i. e. no of unpaired e–
Sol 18: (i) The size of atoms decrease as we go from
∴ MM = 3 = 1. 73 BM
left to right in the series because poor screening effect
of d-electrons.
(ii) There is not a good trend for enthalpies of
atomisation for 3d-elements. As we go from left to
right, if first increases and then decreases with a sudden
dip at the centre.
(iii) Tendency of complex formation increases as the
size decreases.
3 1 . 3 8 | d and f-block Elements

red hot Sol 29: (a) (i) 2Na2CrO4 + H2SO4 → Na2SO4 + Na2Cr2O7
Sol 25: 4FeCr O +8Na2CO + 7O2 
red hot →
2 4O + 8Na
4FeCr 2
3 + 7O
CO 4 2 3 2 air
air + H2O
double
8 Na2CrO4 + 2Fe2O3 + 8CO2 Na2Cr2O7 + KCl  → K2Cr2O7 + 2NaCl
decomposition

(ii) 3K2MnO4 + 2H2O  2KMnO 4 + MnO2 ↓ + 4KOH


Dissolve Purple dark brown
in water
(b) Lanthanide contraction is used to describe the
greater than expected decrease in ionic radii of the
elements in the lanthanide series. It is due to the poor
Fillerate Residue shielding-effect of 4f electrons. The atomic radius of Hf
Na2CrO4 Fe2O3 (6th period) is less than that of Zr(5th period).

2Na2CrO
2Na+2CrO
H2SO 2SONa
4 + H→ SO
Na2+SONa
4 + Cr O + H O

Exercise 2
4
4 4 2 4 2 2 7 2
Na2Cr2O7 + H2O
H2- +
Cr2O27−Cr → CrO 2− +
H 2-
2O7 3Cr2O3
Single Correct Choice Type

Sol 26: (i) The actinoid contraction is high because of Sol 1: (A) KMnO4 → Mn+2, Z = 5
poor shielding effect of earlier 4f e– and nearly coming SO32− → SO24− , Z = 2
5f e–.
2
∴ n=
(ii) Mn
2+
Mn
2+ 5

→ Stability
Sol 2: (B) Fe is catalyst
2+
CrCr
2+ Mo is promoter

Mg 3
→ Not so stable Sol 3: (A) Carats = 24 × = 24 × = 18
Mm 4
So E for Mn3+/Mn2+ is much positive than Cr+3/Cr+2
(iii) It is regarded as transition metal because if has Sol 4: (B) MnO −4 , Cr2O27− show colour due to charge
incomplete d-orbitals and its last electron enters into transfer from O to metal.
3d orbitals.
Sol 5: (A) It is due to availability of vacant d-orbitals
Sol 27: In all the reaction for bonding.
K2Cr2O7 → Cr+3
Sol 6: (A) KMnO4 is the self indicator as it’s colour
(i) I– → I2
changes from purple (+7) during it’s reduction.
(ii) Fe2+ → Fe3+
(iii) H2S → S Sol 7: (B) Vermilon is from the ore cinnabar (HgS).

H +
Sol 28: (a) (i) Mn, Zn have stable +2 O. S. because of Sol 8: (A) Chromic acid + H2O2 → CrO5
Org Solvent
presence of half or full filled orbitals. Fe, Ni, Co also Chromium
peroxide (Blue )
have stable +2 O. S. whereas others don’t show +2 in
most of the compounds. + H2O

(ii) Most of the transition metals don’t form oxometal


ions (Cr, Mn are exceptions. ) They form MO. Acidity Sol 9: (B) Zn has completed (n – 1) d orbitals. So, it’s
increases, ionic nature decreases. not included in transition elements.
(b) (i) They exhibits variable O. S. because of the
presence of unpaired e– in d-orbitals. (ii) Same is the Sol 10: (D) Coinage metals → Cu, Au, Ag i. e. transition
reason for paramagnetism & colour of compounds. metal.
Chem i str y | 31.39

Sol 11: (C) Bayer’s reagent is alkaline KMnO4 Previous Years’ Questions

Sol 12: (A) Molybdenum (MO) is used in X-rays tube. Sol 1: (C) The existence of Fe2+ and NO+ in nitroprusside
ion [Fe(CN)5NO]2– can be established by measuring
the magnetic momentum of the solid compound
Sol 13: (A) F, O because of high electro-negativity. which should correspond to (Fe2+ = 3d6) four unpaired
electrons.
Sol 14: (A) Fe+2 → Fe+3, Z = 1
Sol 2: (C) V+4 → 3d14s0 1 unpaired electrons. Hence it is
KMnO4 → Mn2+, Z = 5
paramagnetic and coloured compound.
∴ 1/5 moles of KMnO4
Sol 3: (B, C) 2KOH + MnO2 + O → K 2MnO 4
Manganese Potassium manganate
Sol 15: (B) Haemoglobin → Fe dioxide
Insulin → Zn +H2O

HCHO + 2KMnO4 + 2KOH → K2MnO4 + H2O + HCOOH
Sol 16: (D) Cu → 5H2O as ligands
H2O will form H-bonding with SO24− ion. Sol 4: (C, D) Co(NO3)2 and CrCl3 has unpaired electron,
hence they are coloured; while Zn(NO3)2, LiNO3 and
Sol 17: (B) It’s colloidal solution of gold. potash alum have no unpaired electron hence they are
colourless.
It’s formed by the reaction of gold salts with Tin(II)
chloride.
Sol 5: (A, B) Gun metal contain Cu and Sn while type
metal contains Pb, Sn and Sb.
Sol 18: (A) SnCl2 → Sn4+, Z = 2
K2Cr2O7 → Cr2O3, Z = 6 Sol 6: (A, B, D) 4NaCl + K2Cr2O7 + 6H2SO4 → 2CrO2Cl2
6 + 4NaHSO4 + 2KHSO4 + 3H2O
n= = 3.
2 CrO2Cl2 + 4NaOH → Na2CrO4 + 2NaCl + 2H2O
Chromyl chloride Yellow solution
Sol 19: (D) Cu2+ → blue
CrO24− → yellow. Sol 7: (A, C) Manganese (Mn) imparts hardness to steel
as well as removes oxygen and sulphur from steel by
Sol 20: (A) It is very hard. It has high proportions of forming slag as MnSiO3
manganese. Fe2O3 + 3Mn → 3MnO + 2Fe
MnO + SiO2 → MnSiO3
Sol 21: (A) They can form complexes by taking e– pairs
from ligands. (Slag)

Sol 8: (C) Rusting involves reduction of absorbed oxygen


Sol 22: (B)TiCl3 is an intermediate during catalysis.
to OH– ions and oxidation of iron to Fe2+ ions. The two
TiCl4 → TiCl3 + Cl. ions combine to yield Fe(OH)2 which gets oxidized to give
Fe2O3.nH2O (rust). The presence of acid helps dissolution
of pure iron to ferrous ions while electrolytes increases
the conductivity and assist the cell action.

Sol 9: (B) AgBr is the most sensitive silver halide to


photo reduction. Hence it is used as the light sensitive
material in photographic films. The unchanged AgBr
is dissolved in hypo solution to cast an image on
photographic plate.
hv
2AgBr → 2Ag + Br2
3 1 . 4 0 | d and f-block Elements

Sol 10: (A) Tungsten is a metal of high melting point Sc3+ is highly stable (it does not show + 2)
and its filament gives brilliant light on passing electric
(D) The oxidation states increases as we go from group
current.
3 to group 7 in same period.

Sol 11: (A) In transition elements ns orbital is filled 2+ 2+


Sol 19 : (A) Each Cr (H2O )  and Fe (H2O ) 
up first and (n – 1)d afterwards, during ionization ns  6  6
electrons are lost prior to (n – 1)d electrons because Contain 4 unpaired electron.
the effective nuclear charge felt by (n – 1)d electrons is
higher as compared to that of ns electrons.
JEE Advanced/Boards
Sol 12: (B) The higher the charge on metal ion, smaller
is the ionic size and more is the complex forming Exercise 1
decreases in the order M4 + > MO22+ > M3+ > MO2+ . The
higher tendency of complex formation of MO22+ of Sol 1:
charge on metal atom M in MO22+
K 2MnO 4 + CO2 + H2O → KMnO 4 + MnO2 + KHCO3
Purple
Sol 13: (B): Being lesser energy difference between 5f
and 6d than 4f and 5d orbitals. Sol 2: Hg22+ → Hg + Hg2+

Sol 14: (C) Ln+3 compounds are mostly coloured. Sol 3: No. of e– in Co3+ → 24
NH3 is a strong ligand
Sol 15: (A) The correct order of E° 2+ values with
M /M
4s 3d
negative sign for the four successive elements Cr, Mn,
Fe and Co is Mn > Cr > Fe > Co
-
F is a weak ligand
2− 4s 3d
Sol 16: (B) In NiCl4  ,n=2

=
µ n (n + 2 ) BM there are unpaired e-

= 2 ( 2 + 2 )= 2.82 BM Sol 4: For NaOH we get MaAlO2, Na2ZnO2 .


For NH4OH, we get [Zn(NH3)4]2+.
Sol 17: (C) The general O.S of lanthanides is +3, only
few elements exhibit +4 O.S. But it doesn’t form complexes.

Sol 18: (A) Sol 5: PbS is black solid where as ZnS is white.
(A) V 2+ = 3 unpaired electrons
Sol 6: A → ZnCl2 because Zn(OH)2 & Al(OH)3 dissolve
Cr 2+ = 4 unpaired electrons in excess NaOH but Al2S3 is gray solid.
Mn2+ = 5 unpaired electrons
NaOH
2+ ZnCl2 Zn(OH2) (White ppt)
Fe = 4 unpaired electrons
Hence the order of paramagnetic behaviour should be AgNO3 Excess NaOH
2+ 2+ 2+ 2+
V < Cr = Fe < Mn
AgCl Na2ZnO2
(B) Ionic size decrease from left to right in same period
White ppt
(C) As per data from NCERT. H2S
Co3+ / Co2+ = 1.97 ; Fe3+ / Fe2+ = 0.77 ; Cr3+ / Cr 2+ = −0.41
ZnS (White ppt)
Chem i str y | 31.41

Sol 7: They are not transition metals. They have (x – 1) In [NiCl4]2–, there is magnetic moment but not in
d10 configuration. [Ni(CO)4].

Sol 8: KMnO4 will oxidize Cl– to Cl2 Sol 13: Fe+2 doesn’t form complex with NH3 so, there
are Fe2+, NH+4 , SO24− ions in the solution.
?
Sol 9: Colourless salt (A) → (B)+(C) gas. In 2nd case, [Cu(NH3)4]2+ will be formed & there will no
B must be Zn or Al salt. Cu2+ Ions.

ZnS is white
Sol 14: In A, all H2O are ligands to Cr.
⇒ B = ZnO
In B, some H2O is evaporated So, that is water of
C is a reducing agent & turns lime water milky crystalisation.
⇒ C = SO2 6.75
WH O = Mcomplex = 18 g.
2 100
⇒ A = ZnSO4 & D = ZnS
So, there is 1 water of crystallisation.

Sol 10: MnO24− → MnO −4 + 1e−  ……(i) In (C)


13.5
MnO24− → 4H+ + 2e− → MnO2 + 2H2O  ……(ii) WH = M = 36 gm.
2O 100
2 × (i) A → [Cr(H2O)6] Cl3
3MnO24− + 4H+ → 2MnO −4 + MnO2 + 2H2O B → [Cr(H2O)5Cl]Cl2 H2O
So 3 moles of MnO24− need C → [Cr(H2O)4Cl2]Cl. 2H2O
2
2e– ⇒ Z =
3 Sol 15: AgBr is not soluble in dilute NH3
M 3M
∴E= = So, there is Cl– in A Cr3+ & strong
2 2
AgBr is pale yellow
(b) MnO-4 CrO -4
2

So, there is Br– in B


O O
A → [Cr(NH3)4Cl Br]Cl → d2sp3
Mn Cs
O O O- O O- O- B → [Cr(NH3)4Cl2]Br → d2sp3

Sol 16: Ratio of moles of Co3+ :


Sol 11: K2[PtCl6] → It has 3 ions.
NH3 : Cl– : SO24− : H2O
We don’t get : AgCl precipitate because there are no.
21.24 24.77 12.81 34.65 6.53
Cl– ions in the solution. There are only K+ and [PtCl6]2– : : : :
ions. 57 17 35 96 18
i. e. 1 : 3 : 1 : 1 : 1
Sol 12: Ni2+ with weak ligands is paramagnetic So, complex can be
4s 3d 4p → [Co(NH3)3 ClSO4] . H2O

[Co(NH3)3 SO4(H2O)]Cl

There are unpaired e- [Co(NH3)3 (H2O)Cl] SO4

Ni with strong ligands diamagnetic


4s 3d 4p
3 1 . 4 2 | d and f-block Elements

Exercise 2 Sol 12: (C) Ag, Pt, etc, are inert to conc. acids & alkalis.
AuCl3 is used for toning in photography.
Single Correct Choice Type
X Sol 13: (C) Cl– will be oxidised to Cl2 (greenish-yellow
Sol 1: (A) Cr2O27− 2CrO27−
Y gas).
y is H ,
+
Cr2O27− is stable in acidic medium.
Sol 14: (A) NO has unpaired e–. NO+ is stable taking
x is OH–
magnetic moment into consideration.
NaOH
→ CrO24−
Sol 2: (B) CrO3 
Multiple Correct Choice Type
Sol 3: (A) (NH4)2 Cr2O7 

→ N2+Cr2O3+4H2O Sol 15: (A,B) Pt doesn’t form any alloys. Fe doesn’t
  ↓ form amalgam.
Green
Sol 16: (B, C) Os, Ru show +8 O. S.
Sol 4: (B) Mercury is ligand in room temperature (Hg). There are in same group as Iron (Fe)

Sol 5: (B) 80% conc HNO3 is an oxidising agent Sol 17: (A, B, C) Al2O3, SnO, ZnO are amphoteric
So Fe → Fe3O4 Fe2O3 → Basic.

Sol 6: (B) Cu + Conc HNO3 → Cu(NO3)2 + NO2 + H2O Sol 18: (A, B, C) Transition metals from interstitial
compounds. So, Co, Ni, Fe.
Sol 7: (A) Mn+7 
reduction
→ Mn+2

Acts as catalyst and increases further rate of reaction. Sol 19: (A, C)
Cr2O27− + 3H2O2 + 8H+ → 2Cr3+ + 3O2 ↑ + 2H2O
Blue colour
Sol 8: (D) CuSO4 + KCN → Cu(CN)2
(Excess)

Sol 20: (A, B) CuF2 is ionic CuCl2, CuBr2 are covalent


CuSO4 + KCN Cu(CN)2
(Excess) (exist as polymers).

Sol 21: (A, B, C, D) CuSO4(aq)+4NH3→ [Cu(NH3)4]2+ SO24−

Cu (CN) +
1
(CN)2 Cu2+
Cu
2+
& NH
& NH3 isis aa strong
strongligand
ligand
3
2
3s 3d
KCN

K3[Cu(CN)4] It’s paramagnetic, so coloured


MM
= n(n + 2) BM & n = 1 = 3 BM.
Sol 9: (D) MnO4 + 4HCl → Cl2 + MnCl2 + xH2O
Acidic KMnO4 decomposition can be catalysed by Sol 22: (A, B) MnO2, Mn3O4 → Amphoteric
sunlight. MnO → Basic
Mn2O7 → Acidic
Sol 10: (D) Fe2O3.xH2O & FeO(OH).Fe(OH)3 constitute
the rust.
Sol 23: (A,B) Zr, Hf have similar sizes & properties due
to lanthanide contraction.
Sol 11: (A) Invar is used to make meter scales. Invar
consists of Nickel, Iron & small quantities of other
metals.
Chem i str y | 31.43

Sol 24: (A, B, C) Potash alum → KAl (SO4)2 Sol 33: (B) The yellow colour of CrO24− is due to charge
transfer from O to metal.
SO, we get K+, Al3+, SO24− .
O
Sol 25: (B, C, D) They will get harder due to the filling of
interval spaces. So, they will be less ductile & malleable.
2-
CrO4 Cr i.e. tetrahedral

O O-
Sol 26: (A, B) Metallic bonding is very weak in Hg. O-

Sol 34: (A) Maximum O. S. of Cr is +6. Because it has


Sol 27: (A, B) Transition elements have high IP only 6e– in 4s & 3d orbitals.
than s-block & lower IP than p-block. They are like
connecting blocks between s & p-blocks. So, they are
Sol 35: (C) Tungsten has very high MP due to strong
called transition elements.
metals bondings. It’s a metal, not a covalent compound.

Sol 28: (A, B, C) Catalytic activity depends on


Sol 36: (C) Cu+ is colourless because of no unpaired
Variable OS e–. All of Cu+ salts need not have 4 H2O molecules
Surface area statement-2 is false.
Reactivity (Complex forming ability)
It doesn’t depend on magnetic moment. Previous Years’ Questions

Sol 29: (A, B) Ag, Au form such complexes Sol 1: (B) In MnO −4 , Mn7+ is in highest oxidation state
possible for Mn. In CrO2Cl2, Cr6+ is in highest oxidation
Cu generally forms [Cu(CN)4]3–.
state possible for Cr.

Sol 30: (B, C) If x > y then x will be covalent Sol 2: (A) MnO −4 + I– + OH– → MnO24− + IO3−
Y will be Ionic
Sol 3: (B) In CuCl2, Cu2+ had d9 configuration, exhibit
Ex. KMnO4 → MnO −4 is covalent
d-d- transition and show colour. Similarly in VOCl2, V4+
MnCl2 → Ionic. had d1 configuration, can exhibit d-d transition and
show colour.
Sol 31: (B, C) Sc3+ → no unpaired e–
Sc
3+
no unpaired e
-
Sol 4: (C) KMnO4 is itself a very strong oxidising agent,
Ti
3+
1 unpaired e- O3 cannot oxidise it.

Sol 5: (A,B,C) Brass = Cu and Zn


Gun metal = Cu, Sn, Zn
2+
Cu 1 unpaired e-

Bronze = Cu, Zn and Sn


Zn
2+
0 unpaired e- Type metal = Pb, Sn, Sb

Sol 6: (A,B,C,D) 4NaCl + K2Cr2O7 + 6H2SO4 → 2CrO2Cl2


3+ 2+
So, Ti , Cu
Chromyl chloride
(Red vapour)
Assertion Reasoning Type + 4NaHSO3 + 2KHSO4 + 3H2O

Sol 32: (A) In alkaline medium


mol. wt mol. wt
Mn+7 → Mn+4, Z = 3 = =
2 3
Z is different for different conditions
3 1 . 4 4 | d and f-block Elements

Sol 7: (A,B,C) Cr : [Ar]3d54s1 Since (CO) is strong ligand, in Cr ( CO ) no unpaired


6
electron is present. So ‘spin only’ magnetic moment is
Magnetic quantum number: –  ………0…….+ 
zero.
Ag(4d105s1): All paired electrons have opposite spin.
The last one has unpaired spin.
Sol 15: A → p, s; B → q, s; C → r, t; D → q, t

Sol 8: (A) In neutral medium: 3Cu + dil 8HNO3 → 3Cu (NO3 ) + 4H2O + 2NO
2

MnO −4 → MnO2(Mn7+ + 3e– → Mn4+) Cu + conc. 4HNO3 → 4Zn (NO3 ) + 5H2O + N2O
2

In alkaline medium: 4Zn + dil. 10HNO3 → 4Zn (NO3 ) + 5H2O + N2O


2

MnO −4 → MnO2 (Mn7+ + 3e– → Mn4+) Zn + conc.4HNO3 → Zn (NO3 ) + 2H2O + 2NO2


2

In acidic medium: 2−
Sol 16: (B) Ni ( CN)  ,O.S. of Ni = +2
MnO −4 → Mn (Mn + 5e → Mn )
2+ 7+ – 2+
 4

Ni ( 28 ) = 3d8 4s2

Sol 9: µ = n(n + 2) BM where ‘n’ is number of unpaired
Ni+2 = 3d8
electrons.
No. of unpaired electrons = 2
1.73 = n(n + 2)
Magnetic moment µ =2.82 BM
n = 1; V4+ 3d1

Sol 17: (C) Following compounds are diamagnetic.


Sol 10: CrO3 is anhydride of chromic acid:
L : Co (NH3 )  CI3
CrO3 + H2O → H2CrO4  6

Chromic acid M : Na3 Co ( Ox ) 



 3

O : K 2 Pt ( CN) 
 4
Sol 11: A = [Ti(H2O)6]3+ and M = Ti, B = TiO2, Ti(IV) has
no electron in 3d orbital, no d-d transition is possible, P :  Zn (H2O )  (NO3 )
 6 2
therefore MCl4 is colourless. In A, there is one electron
in 3d orbital and its d-d transition is responsible for
Sol 18: (A) Aqueous solution of copper sulphate absorbs
colour.
orange red light and appears blue (complementary
colour).
Sol 12: (B) B
V G
(A) Mn+ = 3d5 4s1 in presence of CO effective configu- R Y
ration = 3d6 4s0. Three lone pair for back bonding with O
vacant orbital of C in CO Sol 19: (B)
(B) Fe0 = 3d6 4s2 in presence of CO effective configura-
P = Fe+3 (no. of unpaired e– = 5)
tion = 3d8 four lone pair for back bonding with CO.
(C) Cr0 = 3d5 45s1 Effective configuration = 3d6 three Q = V+2 (no. of unpaired e– = 3)
lone pair for back bonding with CO. R = Fe+2 (no. of unpaired e– = 4)
(D) V– = 3d4 4s2 Effective configuration = 3d6 three pair
As all ligands are weak field, hence the no. of unpaired
for back bonding with co. Maximum back bonding is
electrons remains same in the complex ion.
present in Fe(CO)s there for CO bond order is lowest
here. =µ n (n + 2 )B.M
Hence (B) is correct.
Sol 13: (C) In the crystalline form CuF2 is blue coloured.
3− 3−
Sol 20: Fe ( SCN)  and Fe ( CN) 
Sol 14: (A) Cr ( CO )   6  6
 6
In both the cases the electronic configuration of Fe3+
Cr ( 24 ) =  Ar  3d5 4s1
will be 1s2 ,2s2 ,2p6 ,3s2 ,3p6 3d5
Chem i str y | 31.45

Since SCN‒ is a weak field ligand and CN– is a strong


field ligand, the pairing will occur only in case of
3−
Fe ( CN) 
 6

5
(high spin)
Case -1 3d (no paining)

(low spin)
Case - 2 (pairing)

Case-1 µ= n ( n + 2 )= 5 (5 + 2 ) = 35= 5.91 BM

Case-2 µ = n (n + 2 ) = 1 (1 + 2 ) = 3 = 1.73 BM

Difference in spin only magnetic moment


= 5.91 − 1.73 = 4.18
≈4

Sol 21: (B) Ni ( CO )  − sp3− - Diamagnetic


 4
−2
NiCI 4  sp3 − Paramagnetic

Co (NH ) CI  CI − d2sp3 − Diamagnetic


 3 4 2

Na3 CoF6  − sp3d2 − Paramagnetic

Na2O2i.e. O12− − Diamagnetic

CsO2 i.e O −2 − Paramagnetic

2+
Sol 22: (A) Ni (NH3 )  = Octahedral
 6
+2
Pt (NH )  = Square planar
 3 4

+2
 Zn (NH )  = Tetrahedral
 3 4
2017-18 100 &
op kers
Class 12 T
By E ran culty
-JE Fa r
IIT enior emie .
S fP r es
o titut
Ins

CHEMISTRY
FOR JEE MAIN & ADVANCED
SECOND
EDITION

Exhaustive Theory
(Now Revised)

Formula Sheet
9000+ Problems
based on latest JEE pattern

2500 + 1000 (New) Problems


of previous 35 years of
AIEEE (JEE Main) and IIT-JEE (JEE Adv)

5000+Illustrations and Solved Examples


Detailed Solutions
of all problems available

Plancess Concepts
Topic Covered Tips & Tricks, Facts, Notes, Misconceptions,
Key Take Aways, Problem Solving Tactics
Chemistry in Everyday Life
PlancEssential
Questions recommended for revision
32. CHEMISTRY IN
E V E R Y D AY L I F E

1. DRUGS
1.1 Introduction
Chemistry in everyday life deals with the overall body mechanism, when in the normal and under abnormal
conditions faced by it. It also talks about the various drugs to be consumed, their effects, their side-effects and
their effects after overconsumption.
The classification of drugs is done on various grounds to highlight their functional areas, throwing light on their
mechanism.
Drugs: Drugs are chemicals of low molecular masses (~100 – 500u), interacting with macromolecular targets to
produce a biological response.
Medicines: Medicines are chemicals, useful in diagnosis, prevention and treatment of diseases. A dose consumed
higher than the recommended can cause harm.
Therapeutic Effect: It is a desirable or beneficial drug effect like treatment of symptoms and cure of a disease on
a living body. Such use of chemicals for therapeutic effect is called chemotherapy.

1.2 Classification of Drugs


Table 32.1: Drug classification

On the basis of Explanation

Pharmacological Effect This effect is used by the specialists to prescribe a drug for a particular disorder
from a whole category of drugs.
Greek origin: Pharmakon -poison/drug
logia- study of It is concerned with the E.g. Analgesics kill pain while antiseptics kill or arrest a microorganism’s growth.
study of drug action.
Drug action Release of a specific compound in a body or a biochemical process under
consideration can be treated by various ways by a single category of drug.
E.g., Antihistamine inhibits the action of the compound, histamine, which
causes inflammation in the body. Thus, various ways exist to block the action of
histamines.
Chemical structure This typically depends on the chemical structure of the drug. Similar functional
groups show common pharmacological activity.
O
H2N S NHR
O
E.g., Structure of sulphonamide show the common functional group
3 2 . 2 | Chemistr y in Ever yday Life

On the basis of Explanation

Molecular targets Drugs are very specific in action and have their targets decided. They interact
with biomolecules such as carbohydrates, lipids, proteins and nucleic acids.
Common structural features show same mechanism of action.

1.3 Drug Target Interaction


The interaction between the drug and the target i.e. a part of the body is the major part of the action of a
drug. Enzymes-the biological catalyst action serves as the best example for this interaction. These proteins named
enzymes hold a major position in the communication system of the body. They are called as receptors which
carry polar molecules across the cell membrane. Similarly, nucleic acids have codes for passing on the genetic
information across various cells.

1.3.1 Enzymes as Drug Targets


Enzymes, as known to us, are the biological catalysts which have two major functions as seen in the Fig. 32.1.

Active
site

Substrate
Enzyme Enzyme
Active site holds
the substrate molecule The enzyme provides
in a firm position, to be functional groups which
attacked by the reagent enable the attack on the
effectively. substrate

Figure 32.1: Enzyme substrate interaction

Drug Enzyme interaction


When drugs are taken, it either increases or decreases the enzyme catalyzed reactions. Enzyme inhibition is the role of
a drug done in two ways-Competitive or Non-competitive. The drug is an enzyme inhibitor which inhibits the catalytic
activity of enzymes or blocks the binding site which eventually prevents the binding of substrate with enzyme.

(a) Competitive Inhibition: Competitive Inhibitors are the drugs which attach themselves (as shown in the Fig. 32.2)
onto the active site of an enzyme by competing with the substrate for the space.

Drug

Active Drug
site

Substrate
Enzyme Enzyme Substrate

Drug and substrate Drug blocks the active


competing for active site of enzyme
site of enzyme

Figure 32.2: Steps involved in competitive inhibition


Chem i str y | 32.3

(b) Non-Competitive Inhibition: Non-competitive drugs change the shape of the active site by binding
themselves to an allosteric site, due to which the substrate is unable to identify the active site and is thus
disabled to attach itself. The presence of a strong covalent bond between an enzyme and an inhibitor blocks
the enzyme. Degradation of this complex gives a new enzyme. Non-competitive inhibitor changes the active
site of enzyme after binding at allosteric site.
Active site with
Active changed shape
site

Enzyme Allosteric site Inhibitor occupying


Inhibitor allosteric site
Figure 32.3: Steps involved in non-competitive inhibition

1.3.2 Receptors as Drug Targets


Receptors are selective in nature. Every binding site has a different shape, structure and an amino acid which suits
a specific chemical messenger.

Communicators
sit ing
Bin tes

made up of
d
din

es
si

Bin

Protein
g

Receives
Chemical Massengers Shapes change
so as to receive
Posting of messages messages
between
neurons & muscles

Chemical Binding site


Binding messenger
site
Outer
surface Induced fit
of cell
membrane

Cell
Interior membrane
of cell Messenger
(a) (b) (c)
Receptor receiving Shape of the receptor Receptor regains
chemical messenger changed after structure after
attachment removal of chemical
of messenger messenger

Figure 32.4: Receptors as drug targets

Antagonists are the drugs which bind with the receptor site and inhibits its natural function. These are useful when
blocking of message is required. Drugs that mimic the natural messenger by switching on the receptor are called
agonists. These are useful when there is lack of natural chemical messenger.
3 2 . 4 | Chemistr y in Ever yday Life

1.4 Therapeutic Actions of Different Classes of Drugs

Antacid N.A.D
(NeutralizeAntacid
excess acid [Neurologically active drugs]
N.A.D
in the gastric
(Neutralize excess juices.
acid (The message
[Neurologically transfer mecha-
active drugs]
Therapeutic actions nism from
(The message nerve
transfer to receptor)
mecha-
Eg.gastric
in the Eno,gelusil)
juices.
Therapeutic actions
Eg. Eno,gelusil) of different classes nism from nerve to receptor)
Anti histamines
of different
of classes
drugs Tranquilizers
Anti histamines of drugs Tranquilizers
(Treatment of stress and mild
(Diminish or abolish
(Diminish (Treatment of stress
or severe and mild
mental diseases)
the effects or
of abolish
histamine; or severe mental diseases)
the effects of histamine;
prevent allergic reactions;
prevent allergic reactions;
Eg: Terfenadine)
Eg: Terfenadine)

Anti
Antifertility
fertility AntiAnti
microbials
microbials Analgesics
Analgesics Non narcotic
Non narcotic
(Conception of fertility (Destroy/prevent
(Destroy/prevent (Relieve Pain without
(Relieve Pain without(Non Addictive)
(Conception of fertility (Non Addictive)
Eg: Norethindrone development or inhibit
development causing
or inhibit any disturbances
Eg: Norethindrone causing any disturbances
the the
pathogenetic
pathogeneticaction) in theinnervous
action) system)system)
the nervous

Narcotic
Narcotic
(Addictive: Induce sleep)
(Addictive: Induce sleep)

Anti septics Dis infectants


Antigrowth
(Prevent septics Anti biotics Dis infectants
(Kill microorganisms;
of microorganisms
(Prevent growth(Kill & prevent the
Anti biotics cannot be
(Kill applied on
microorganisms;
ofEg: dettol)
microorganismsgrowth of other
(Kill & prevent the living
cannot tissues
be applied on
Eg: dettol) microbes) Eg: iodoform)
living tissues
growth of other
microbes) Eg: iodoform)

On basis of mode of control of microbial diseases: On basis of its spectrum of action:


Bactericidal Bacteriostatic Brand spectrum antibiotics Narrow spectrum Limited spectrum
On basis
Drugs of kills
that mode of control
organisms in of microbial
Drugs diseases:
that inhibits growth of On basis of its spectrumantibiotics
of action: antibiotics
Bactericidal
body Bacteriostatic
organisms Brandwhich
Antibiotics spectrum
kill or antibiotics Narrow
Antibiotics whichspectrum
are AntibioticsLimited
effectivespectrum
Drugs that kills organisms inExamples:
Examples: Drugs that inhibits growth of inhibit a wide range of effectiveantibiotics
mainly against against a single
antibiotics
Penicillin,
body Aminoglycosides, Erythromycin,
organisms Tetracycline, Gram-positive andwhich
Antibiotics Gram-positive
Gram-kill or or Gram-
Antibiotics organism or
which are disease effective
Antibiotics
Ofloxacin Chloramphenicol negative bacteria negative bacteria
Examples: Examples: inhibit a wide range of effective mainly against against a single
Penicillin, Aminoglycosides, Erythromycin, Tetracycline, Examples:
Gram-positive and Gram-Examples: Gram-positive or Gram- organism or disease
Ofloxacin Chloramphenicol Ampicillin and Amoxycillin
negative bacteria Penicillinnegative
G bacteria
Examples: Examples:
Ampicillin and Amoxycillin Penicillin G

Flowchart 32.1: Classification of drugs and therapeutic action


Chem i str y | 32.5

Antihistamines
Brompheniramine Terfenadine (Seldane) (Dimetapp, Dimetane)

N N
Ph OH
HO Ph
Br

Non-Narcotic Analgesics:
Ibuprofen
CH3
COOH
CH3
COOH OCOCH3

H3C Aspirin

Narcotic Analgesics:

NCH3 NCH3

HO O AcO O
OH OAc
Morphine Heroin

Antiseptics:

Chloroxylenol
+
terpineol
Antiseptic

Antifertility Drugs

H3C OHCCH H3C OHCCH

H H H

H H H H
HO O
Ethynylestradiol (novestrol) Norethindrone

2. CHEMICALS IN FOOD / FOOD ADDITIVES


Food additives are the substances added to food to preserve its flavour or improve its taste and appearance

Table 32.2: Different types of food additives

No. Name of food additive Examples


1 Artificial Sweetening Agents: These chemical compounds give a Aspartame, Sucralose and Alitame
sweetening and flavouring effect to the food.
2 Food preservatives: These chemical substances on adding to food Sugar, Salts, Sodium benzoate
material prevents its spoilage due to microbial growth.
3 2 . 6 | Chemistr y in Ever yday Life

No. Name of food additive Examples


3 Food colours: These substances adds to the attractiveness and Allura Red AC, Tartrazine
acceptability in the market.
4 Nutritional supplements: These substances improves the Vitamins, minerals etc.
nutritional value of the food.
5 Fat emulsifiers and stabilizing agents: These agents give stability Egg yolk (where the main emulsifying chemical
and a good consistent nature to the food. is Lecithin)
6 Antioxidants: These agents prevent the oxidation of food materials. Butylated Hydroxy Toluene (BHT), Butylated
Hydroxy Anisole (BHA)

3. SOAPS AND DETERGENTS

SOAP

 
Na or K salts of long chain fatty acids

O
CH2-O -C- C17H35
O CH2-OH
CH-O-C- C17H35+3NaOH 3C17H35COONa+CH-OH
O (Soap) CH2-OH
CH2-O-C- C17H35

Glyceryl ester Sodium Sodium Glycerol


of stearic acid(Fat) hydroxide stearate

SAPONIFICATION

Hydrolysis of fats or oils with alkalis

Figure 32.5: Saponification

Shaving soap
Potassium sodium stearates
contains glycerol to
prevent rapid drying,
gum,rosin forms sodium
rosinate(lathers)

Soap chips
Medicated soaps Running a thin sheet of melted
soap onto a cool cylinder,
Soft Soaps, medicinal, scrapping off the soaps in
Neem Soap small broken pieces.
AP
SO

Types
Transparent soaps
Soap in ethanol by Granules soap
Evaporation of excess solvent Dried miniature soap bubbles

Soap powder &


scouring soaps
Floating Soaps Some soap, a scouring agent
Float in water, beating tiny (abrasive), powdered pumice,
air bubbles, before it hardens cleaning action

Figure 32.6: Different types of soap


Chem i str y | 32.7

Table 32.3: Advantages and disadvantages of soap

Advantages of using soaps: Disadvantages of using soaps:


(a) Being a good cleanser, it is 100% biodegradable (a) Use in hard water gives the formation of white precipitate with
i.e. oxidation of the soap by the micro-organisms in the Ca2+ and Mg2+ ions present in the water.
the sewage water.
2C17H35COONa+ CaCl2 → 2NaCl + (C17H35COO)2 Ca
(b) Does not cause pollution. Soap Insoluble calcium
Stearate(Soap)

2C17H35COONa+ MgCl2 → 2NaCl + (C17H35COO)2 Mg


Soap Insoluble
Magnesiumstearate(Soap)

Due to the formation of precipitates, the soap is unable to perform


its function of removing the oil from the clothes. The gum-like
precipitates stick to the fibres of the cloth.

(c) The insoluble free fatty acids get precipitated in acidic medium
and stick to the fabric blocking the oil-removing ability of the soaps.

3.1 Detergents


CH3(CH2)11 SO 3Na

RG
ETE EN
D

Anionic Detergents:
1. Sodium salts of sulphonated long
Cationic Detergents
chain alcohols or hydrocarbons by
1. Quaternary ammonium salts of
treating long chain alcohols with
amine with acetates, chlorides
concentrated sulphuric acid are
or bromides as anions.
neutralized with alkali Non-lonic Detergents 2.Germicides
2. In household cleaning like dishwasher 1. Do not contain any ion in their 3.Expensive.
liquids, laundry liquid detergents, constitution.
3. Effective in slightly acidic solutions. 2. Esters of high molecular mass, formed
by condensation reaction between
stearic acid and polyethylene glycol
3. Liquid washing detergents
4. Effective H-bonding groups at one
end of the alkyl chain makes them
freely water soluble

Figure 32.7: Classification of detergents

Anionic detergents:

H SO – +
NaOH(aq)
2 4 → CH (CH ) CH OSO H 
CH3 (CH2 )10 CH2OH  3 2 10 2 3 → CH3 (CH2 )10 CH2OS O3 Na
Laurylalcohol Laurylhydrogensulphate Sodiumlaurylsulphate
(Anionic detergent)

H2SO4 NaOH
CH3(CH2)11 CH3(CH2)11 SO3H CH3(CH2)11 SO3Na
(aq)

Dodecylbenzene Dodecylbenzensulphonic acid Sodium Dodecylbenzenesluphonate


3 2 . 8 | Chemistr y in Ever yday Life

+
Cationic detergents:
CH3
-
CH3(CH2)15 N CH3 Br
CH3

Cetyltrimethylammonium bromide
Non: ionic detergents:
Non-ionic detergents: CH3(CH2)16COOH + HO(CH2CH2O)nCH2CH2OH
Stearic acid Polyethyleneglycol
-H2O

CH3(CH2)16COO(CH2CH2O)nCH2CH2OH

Table 32.4: Difference between biodegradable and non-biodegradable detergents

Biodegradable detergents Non-Biodegradable detergents


1. Have straight hydrocarbon chains. 1. Have branched hydrocarbon chains

2. They can be easily decomposed by microorganisms. 2. They cannot be easily decomposed by microorganisms.

Example: Sodium lauryl sulphate

Soaps & Detergents

Soaps Detergents

Sodium and Potassium salts of higher fatty Are sodium and potassium salts of sulphuric
acids(carbon atoms 12 or highter) acids of alkanes

Saponification
Process in which oil or fat (glyceride) is
hydrolised with NaOH to get soap and glycerol Advantages of Detergents Disadvantages of Detergents

 Works well with hard water  Not full biodegradable


 Can be used in saline or  Expensive than soaps
acidic water
 More easily soluble in water
than soaps
Advantages of Soap Disadvantages of Soaps  Can be used to clean woolen
 Dose not work well with clothes
 Cheaper and readily available
 Works well with soft Water hard water
to clean clothes  Not suitable for washing
 100% bio degradable woolen clothes
 Less Effective in saline and
acidic water

Flowchart 32.2: Difference between soaps and detergents


Chem i str y | 32.9

POINTS TO REMEMBER
Sr. No Terms Description

1 Drugs Drugs are chemicals of low molecular masses (~100 – 500u). These interact with
macromolecular targets and produce a biological response.

2 Medicines: Medicines are chemicals that are useful in diagnosis, prevention and treatment of
diseases

3 Therapeutic Effect Desirable or beneficial effect of a drug like treatment of symptoms and cure of a
disease on a living body is known as therapeutic effect.

4 Enzymes Proteins which perform the role of biological catalysts in the body are called
enzymes.

5 Receptor Those which are crucial to communication system in the body are called receptors

6 Competitive Competitive Inhibitors are the drugs that compete with the natural substrate for
Inhibition their attachment on the active sites of enzymes

7 Non-Competitive Some drugs do not bind to the enzyme’s active site, instead bind to a different site
Inhibition of enzyme called allosteric site. This binding of inhibitor at allosteric site changes
the shape of the active site in such a way that substrate cannot recognize it.

9 Cationic Detergents Quaternary ammonium salt of amines with halide or acetate as anions.

10 Non-ionic detergents Do not contain any ion, they are esters of high molecular mass.

Solved Examples

JEE Main/Boards CH3-(CH2)11



SO3Na

Example 1: In order to wash clothes with water containing


Sodium-4-(-dodecyl) benzene
dissolved calcium hydrogen carbonate, which cleaning
sulphonate (synthetic detergent)
agent will you prefer and why: soaps or synthetic detergents? +
Give one advantage of soaps over synthetic detergents? Ca(HCO3)(Hard water)

Sol: Calcium salts of soaps are soluble in water thus lot


of soap is wasted.
•• Hard water contains salt of calcium and magnesium.
CH3-(CH2)11 SO3 Ca

•• Thus water containing calcium hydrogen carbonate


2

is hard water.
Calcium 4-(1-dodecyl) benzene
sulphonate
•• In order to wash cloths with hard water detergents
+
are preferred over soaps
2NaHCO3

•• This is because calcium salts of detergents are As a result, lot of soap is wasted.
soluble in water while calcium salts of soaps are
insoluble.
3 2 . 1 0 | Chemistr y in Ever yday Life

Example 3: What are antihistamines? Give two


2C17H35COONa + Ca(HCO3 )2 examples. Explain how do they act on the human body?
Sodiumstearate(soap) Cal.hydrogencarbonate
(Soluble) (Fromhard water)
Sol:
→ (C17H35COO)2 Ca ↓ + 2NaHCO3
•• The hypersensitivity towards certain drugs
(penicillin, sulpha drugs), dust, pollen grains, cat fur,
Cal. stearate
(insoluble )
a particular food or fabric, etc. is called as allergy.
Advantage of soaps : Soaps are biodegradable they •• Allergy is caused by release of a substance called
do not cause pollution i.e microorganism present in sea histamine is the body.
water completely oxidizes soap while detergents having
•• Antihistamines are drugs which either reduce or
branched hydrocarbon chains are not biodegradable
inhibit the action of histamine in the body there by
and hence cause water pollution in rivers and waterways.
preventing allergy.
•• Two important antihistamine or anti-allergic drugs
Example 2: Account for the following: are Brompheniramine and Terfenadine.
(i) Aspirin drug helps in the prevention of heart attack.
Mode of action:
(ii) Diabetic patients are advised to take artificial
sweeteners instead of natural sweeteners. •• Histamines interact with the binding sites of
receptor in the body to produce allergy.
(iii) Detergents are non-biodegradable while soaps are
•• Antihistamines compete with histamines for these
biodegradable.
binding sites of receptor and thus do not allow
Sol: (i) histamine to produce allergy.
•• Heart attack is caused due to blood clotting in the •• Thus, antihistamines are drugs which interfere with
coronary arteries. the natural action of histamine by competing for
histamine binding sites of receptor.
•• In order to prevent heart attack aspirin is used.
•• Aspirin (Acetyl salicylic acid) helps to make the
blood thinner and thus prevents the formation Example 4: Sleeping pills are recommended by doctors
of blood clots in the coronary arteries there by to the patients suffering from sleeplessness but it is not
preventing heart attacks. advisable to take doses without consultation with the
doctor. Why?
(ii)
•• This is because Diabetic patients do not produce Sol: Slipping pills are habit forming. Most of drugs
enough insulin to metabolize the natural sugar. taken in doses higher than recommended may produce
•• Because of this the sugar remains in the blood and harmful effects and, act as a poison and even cause
affects liver, heart and kidneys. death. Therefore, a doctor must always be consulted
before taking the drug.
•• Therefore, diabetic patients are advised to take
artificial sweeteners.
Example 5: Write the chemical equation for preparing
•• Artificial sweeteners contains saccharin.
sodium soap from glyceryl oleate and glyceryl palmitate.
•• It is not metabolized in the body and is excreted as Structures of these compounds are given below.
such through urine without affecting the heart, liver
and kidneys. (i) (C17H33COO)3C3H5— Glyceryl oleate.

(iii) (ii) (C15H31)COO)3C3H5— Glyceryl palmitate


•• Soaps have a structure of straight hydrocarbon
Sol: (i)
chains which are easily degraded by microorganisms
or bacteria present in the sewage water and hence O
do not cause water pollution.
CH2 O C _ C17H33
_ _ CH2OH
•• On the other hand, detergents have branched O
=

hydrocarbon chains which are either not attacked CH O C _ C17H33 + 3NaOH


_ _  CHOH + 3 C17H33COONa
or attacked only slowly by bacteria. As a result,
O
Sodium oleate
detergents remain undegraded in rivers and CH _ O _ C _ C H
2 17 33
CH2OH

waterways and thus cause water pollution. (C17H33COO)3C3H5) (Glycerol)


(Glyceryl oleate)
Chem i str y | 32.11

(ii) Example 6: If a soap has high alkali content it irritates


O
skin. How can the amount of excess alkali be determined?
CH2 O C C15H31 What can be the source of excess alkali?
O
Sol: Amount of alkali can be determined by acid-base
+3 NaOH
CH O C C15H31 titration i.e., titrating a solution of soap with standard
hydrochloric acid using phenolphthalein as indicator.
O
The source of this excess alkali is the alkali left unused
CH2 O C C15H31 when the soap is prepared by hydrolysis of oils.
(C15H31COO)3C3H5 Heat
Example 7: What is the scientific explanation for the
(Glyceryl palmitate)

feeling of depression?

CH2OH Sol:
•• Noradrenaline is a neurotransmitter which plays an
important role in mood changes.
CHOH + 3 C15H31COONa
Sodium palmitate
CH2OH •• If a person has low level of noradrenaline, then the
Glycerol message transfer process becomes slow and the
person suffers from depression.

JEE Main/Boards

Exercise 1 Q.7 Explain the following terms with suitable examples.


(i) Cationic detergents (ii) Anionic detergents
Q.1 Alkaloids are poisonous and habit forming yet many (iii) Neutral detergents
of them are used as potent medicines. Can you name
the alkaloid used for the treatment of (i) hypertension, Q.8 Give one important use of each of the following in
(ii) malaria fever and (iii) severe pain? pharmacy?
(i) Equanil (ii) Morphine
Q.2 Sulpha drugs work like antibiotics but they are not
antibiotics. Is this a valid statement and why?
Q.9 Give one example of an artificial sweetener used by
the diabetic patients
Q.3 Pick out the odd ones from among the following
compounds on the basis of their medicinal properties. Q.10 Why certain drugs are called enzyme inhibitors?
(i) Luminal, seconal, phenacetin, equanil
(ii) Chlorxylenol, phenol, chloramphenicol, bithional. Q.11 What are biodegradable and non-biodegrable
detergents. Give one example of each.
Q.4 Explain the term, target molecules or drug targets
Q.12 Why do soaps not work in hard water?
as used in medicinal chemistry.

Q.13 Name a substance which can be used as an


Q.5 While antacids and anti-allergic drugs interfere antiseptic as well as disinfectant.
with the function of histamines, why do not these not
interfere with the function of each other?
Q.14 What are main constituents of dettol?

Q.6 Why are cimetidine and ranitidine better antacids Q.15 What is tincture of iodine? What is its use?
than sodium hydrogen carbonate or magnesium or
aluminium hydroxide? Q.16 How do antiseptics differ from disinfectants ? Give
one examples of each.
3 2 . 1 2 | Chemistr y in Ever yday Life

Exercise 2 Q.5 Cetyltrimetyl ammonium bromide is a popular


(A) Anionic detergent (B) Cationic detergent
Single Correct Choice Type
(C) Non-ionic detergent (D) Sweetener
Q.1. Which of the following can possibly be used as an (E) Antioxidant
analgesic without causing addiction and modification?
(A) Acetylsalicylic acid Q.6 The role of phosphate in detergent powder is to

(B) N-Acetyl-para-aminophenol (A) Control pH level of the detergent water mixture

(C) Phenyl salicylate (B) Form solid detergents as phosphate less detergents
are liquid in nature
(D) Methyl salicylate
(C) Provide whiteness to the fabrics
(D) None of the above
Q.2 Which among the following is not an antibiotic?
(A) Penicillin (B) Oxytocin
Q.7 Which of the following is not a neurotransmitters?
(C) Erythromycin (D) Tetracycline
(A) Acetylcholine (B) Adrenaline
(E) Ofloxacin (C) Noradrenalin (D) Serotonin

Q.3 Which of the following can disperse benzene is OCOCH3


water?
COOH
Q.8 The compound is used as
(A) O

- + (A) Antiseptic (B) Antibiotic


O Na
(C) Analgesic (D) Pesticide
(B)
O
+- Q.9 2-acetoxybenzoic acid is used as
Na O
(A) Antimalarial (B) Antidepressant
O-Na+
O
(C) Antiseptic (D) Antipyretic

(C) Q.10 Aspirin is an acetylation product of


(A) p-dihydroxybenzene
CH3 (B) o-hydroxybenzoic acid
(C) o-dihydroxybenzene
(D)
(D) m-hydroxybenzoic acid

Cl Q.11 An antibiotic contains nitro group attached to


aromatic nucleus its structure. It is
(A) Penicillin (B) Streptomycin
Q.4 The organic detergents that is used in hair
conditioners is (C) Tetracyclin (D) Chloramphenicol

(A) Sodium dodecylbenzene sulphonate


Q.12 Which of the following is not a surfactant?
(B) Sodium lauryl sulphate
(A) CH3-(CH2)15-N+(CH3)3Br−
(C) Tetramethylammonium chloride
(B) CH3-(CH2)14-CH2NH2
(D) Sodium stearly sulphate
(E) Cetyltrimethylammonium bromide (C) CH3 − ( CH2 ) − CH2OSO2− Na+   
16

(D) OHC-(CH2)14 –CH2-COO-Na+


Chem i str y | 32.13

Q.13 The detergent which is used as a germicide is III. NaOH RCH SO-3 Na+
(A) Sodium Lauryl sulphate
(A)
(B) Cetyltrimethylammonium chloride CH3

(C) Lauryl alcohol ethoxylate


These steps should be in sequence
(D) Sodium-2-dodecylbenzenesulphonate
(A) I, II, III (B) II, I, III (C) II, III, I (D) I, III, II
Q.14 Which of the following sets of reactants is used
for preparation of paracetamol from phenol? Q.19 Aspartame is a non-nutritive sweetener. Assuming
that both, amide and ester bonds are hydrolysed in the
(A) HNO3, H2/Pd, (CH3CO)2O stomach, the amino acids obtained is
(B) H2SO4, H2/Pd, (CH3CO)2O
(A) CH2CHCOOH
(C) C6H5N2Cl, SnCl2/HCI, (CH3CO)2O
(D) Br2/H2O, Zn/HCl, (CH3CO)2O NH2
(B) H2N − CHCH2COOH
Q.15 In the following sets of compounds, the one which |
contains only medicinal compounds is COOH

(A) Alizarin, phenacetin, morphine (C) Both (A) and (B)


(B) Aspirin, gention violet, phenolphthalein (D) None of these
(C) Boric acid, chloramphenicol, aspirin
(D) 9-oxodecanoic acid, boric acid, morphine Q.20 Which detergent can cause maximum pollution?

(A) CH3CH(C4H9) – H2CH2CH(C4H9)


Q.16 OH
CH3
KMnO4
(CH2)3CH(C4H9)CH2CH2SO3Na
CH3COCl/Py
X Y
(B) CH3(CH2)11 SO3Na
The final product ‘Y’ is medicine. Which of the following
(C) SO3Na
is incorrect regarding ‘Y’?
(A) It has analgesic as well as antipyretic properties (D) Detergents are always pollution free
(B) It helps to prevent heart attack
Q.21 Which of the following is known as invert soap?
(C) It has anti-blood clotting action
(D) It suppresses the gastric anomalies (A) Pentaerythritol monostearate
(B) Sodium stearyl sulphate
Q.17 Aspartame is one of the good artificial sweeteners (C) Trimethyl stearyl ammonium bromide
whose use is limited to cold foods and soft drinks because
(D) Ethoxylated non-phenol
(A) It has very low boiling point
(B) It gets dissociated at cooking temperature Q.22 Antiseptics are different from disinfectants as .
(C) It is sweetener at low temperature only (A) Antiseptic merely inhibit, the growth and disinfectant
(D) It is insoluble at higher temperatures kill the microorganisms
(B) Antiseptic are used against ‘microorganisms while
Q.18 For the preparation of a detergent ‘A’ (given disinfectants are used against insects
below) from benzene, the following steps are involved (C) Antiseptic are used only over skin disinfectants
can be taken orally also
I. 
RCH=CH2

HF, Friedel−Crafts (D) Antiseptic are used over living tissues while
disinfectants cannot be used over living tissues
II. →
H2SO 4 /SO3
3 2 . 1 4 | Chemistr y in Ever yday Life

Q.23 Which of the following is an example of non- Q.28 Baby talcum powders contain
biodegradable detergent?
(A) Benzoin, glyceryl diacetate
(A) CH3(CH2)11 SO3Na (B) Zinc acetate, glyceryl diacetate
(C) Zinc stearate, boric acid
(B) CH3 (CH2)9CH SO3Na (D) Zinc stearate, cinnamic ester

CH3
Previous Years’ Questions

( (
CH3 CH3

(C) CH3 CHCH2 CH SO3Na Q.1 The compound is used as  (2002)

COOH
(D) CH3(CH2)10CH2OSO3Na O
O C CH3
Q.24 Morphine on alkylation and acetylation gives, Aspirin
respectively
(A) Heroin, codeine (B) Heroin, meperidine Acetyl salicylic acid
(C) Codeine, heroin (D) Meperidine, heroin
(A) Pesticide (B) Antibiotic

Q.25 Which of the following statement is not true (C) Analgesic (D) Antiseptic
about the drug barbital?
(A) It causes addiction Q.2 Which one of the following types of drugs reduces
 (2005)
(B) It is a non-hypnotic drug
(A) Analgesic (B) Antipyretic
(C) It is a tranquilizer
(C) Antibiotic (D) Tranquilizer
(D) It is used in sleeping pills
Q.3 Insulin production and its action in human body are
Q.26 Select the incorrect statement. responsible for the level of diabetes. This compound
(A) Equanil is used to control depression and belongs to which of the following categories?  (2004)
hypertension. (A) A co-enzyme (B) An antibiotic
(B) Mifepristone is a synthetic steroid used as “morning (C) An enzyme (D) A hormone
after pill”
(C) 0.2 percent solution of phenol is all antiseptic while Q.4 Which of the following compounds is not an
its 1.0 percent solution is a disinfectant antacid?  (2005)
(D) A drug which kills the organism in the body is called (A) Aluminium hydroxide
bacteriostatic.
(B) Cimetidine

Q.27 Which set has different class of compounds? (C) Phenelzine

(A) Tranquilizers: Equanil, heroin, valium (D) Ranitidine

(B) Antiseptics: Bithional, dettol, boric acid


Q.5 Aspirin is known as:  (2012)
(C) Analgesics: Naproxen, morphine, aspirin
(A) Acetyl salicylic acid
(D) Bactericidal: Penicillin, aminoglycosides, ofloxacin
(B) Phenyl salicylate
(C) Acetyl salicylate
(D) Methyl salicylic acid
Chem i str y | 32.15

PlancEssential Questions
JEE Main/Boards

Exercise 1
Q.1 Q.3 Q.4 Q.6 Q.8 Q.10

Exercise 2
Q.3 Q.6 Q.10 Q.13 Q.19

Previous Years’ Questions


Q.1 Q.5

Answer Key

JEE Main/Boards

Exercise 2

Single Correct Choice Type

Q.1 B Q.2 B Q.3 A Q.4 B Q.5 B Q.6. A

Q.7 B Q.8 C Q.9 D Q.10 B Q.11 D Q.12 B

Q.13 B Q.14 A Q.15 C Q.16 D Q.17 B Q.18 A

Q.19 C Q.20 A Q.21 C Q.22 D Q.23 C Q.24 C

Q.25 B Q.26 D Q.27 A Q.28 C

Previous Years' Questions


Q.1 C Q.2 B Q.3 D Q.4 C Q.5 A
3 2 . 1 6 | Chemistr y in Ever yday Life

Solutions

better antacids because they prevent the interaction


JEE Main/Boards
of histamine with the receptors present in the stomach
wall and thus release lesser amount of HCl.
Exercise 1
Sol 7: Anionic detergents. These are so called because
Sol 1: (i) Hypertension: Reserpine ; (ii) Malaria fever :
a large part of their molecules are anions. These are of
Quinine and (iii) Severe pain : Morphine.
two types:

Sol 2: This is a valid statement since sulpha drugs like (i) Sodium alkyl sulphates. For example, sodium lauryl
antibiotics kill microorganisms. However, sulpha drugs sulphate, C11H23CH2OSO3Na.
are purely synthetic but antibiotics may be either wholly (ii) Sodium alkylbenzenesulphonates. The most widely
or partially obtained from microorganisms. For example, used domestic detergent is sodium 4-(1-dodecyl)
penicillin is wholly obtained from microorganisms while benzenesulphonate (SDS).
amoxcillin and ampicillin are semi-synthetic.
CH3 (CH2)11 SO3Na
Sol 3: (i) Phenacetin is an antipyretic while the remaining
three are tranquillisers. Cationic detergents. These are quaternary ammonium
(ii) Chloramphenicol is an antibiotic while the remaining salts. For example, cetyltrimethylammonium chloride.
three are antiseptics.  +  –
CH3 (CH2 )15 N(CH3 )3  Cl
 
Sol 4: A knowledge of the physiological function of Cetyltrimethylammonium chloride
the drug in the body helps us to choose a compound
called lead compounds and drugs are designed from Neutral or Non-ionic detergents:These are esters of
these compounds. Drugs interact with macromolecules high molecular mass alcohols with fatty acids. For
like proteins, carbohydrates, lipids and nucleic acid example, polyethylene glycol stearate.
and hence these are called target molecules or drug
targets. Proteins perform several roles in the body. CH3 (CH2 )16 COO(CH2CH2O)n CH2CH2OH
Proteins which act as biological catalysts are called Polyethylenle glycolstrearate

enzymes, those which are involved in communication


system are called receptors. Carrier proteins carry polar Sol 8: (i) Equine is a tranquillizer. It is used for removing
molecules across the cell membrane. Nucleic acids have depression and hypertension.
coded genetic information in the cell and, lipids and (ii) Morphine is an alkaloid. It is used as an analgesic.
carbohydrates form structural part of cell membranes.
Sol 9: Saccharin (in form of its sodium salt) is taken as
Sol 5: Drugs designed to cure some ailment in one artificial sweetener by diabetic patients.
organ in the body do not affect the other because they
work on different receptors. For example, secretion of CO
histamine cause allergy. It also causes acidity. Due to _

release of hydrochloric acid in the stomach, since anti N Na+


allergic and antacids drugs work on different receptors,
SO2
therefore, antihistamines remove allergy while antacids
Sodium salt of saccharin
remove acidity.

Sol 10: Enzymes have active sites which bind the


Sol 6: Sodium hydrogen carbonate or magnesium or
substrate for carrying out chemical reactions quickly
aluminum hydroxide neutralize the excess HCl and raise
and effectively. The functional groups present at the
the pH to an appropriate level in stomach. Therefore,
active site of enzyme interact with functional groups
these antacids control only the symptoms and not
of substrate through ionic bonding, hydrogen bonding,
the cause. In contrast, cimetidine and ranitidine are
vander Waals interactions, etc. Some drugs interfere
Chem i str y | 32.17

with this interaction by blocking the binding site of Exercise 2


enzyme thereby preventing the binding of actual
substrate with the enzyme. As a result, catalytic activity Sol 1: (B) N-Acetyl-Para-aminophenol is also called
of enzyme is inhibited and the drugs which inhibit the paracetamol.
catalytic activity of enzymes are called inhibitors.

Sol 2: (B) Oxytocin is a hormone (Nano peptide) which


Sol 11: Detergents having straight hydrocarbon chains contracts uterus after the child birth and produces
are easily degraded (or decomposed) by microorganism lactation in the mammary glands.
and hence are called Biodegradable detergents while
detergents containing branched hydrocarbon chains are
not easily degraded by the microorganisms and hence Sol 3: (A) A surfactant has a polar head and non-polar
are called non-biodegradable detergents. Consequently, tail, and hence can disperse benzene in water, i.e.,
non-biodegradable detergents accumulate in rivers and option (a) is correct.
other waterways there by causing severe water pollution.
Examples of biodegradable detergents are: sodium lauryl Sol 4: (B) The organic detergents that is used in hair
sulphate, sodium 4-(1-dodecyl) benzene- sulphonate conditioners is Sodium lauryl sulphate
and sodium 4-(2-dodecyl) benzenesulphonate. An
example of non-biodegradable detergent is sodium
Sol 5: (B) Cetyltrimetyl ammonium bromide is a
4-(1, 3, 5, 7-tetramethyloctyl) benzene sulphonate.
popular cationic detergent

Sol 12: Hard water contains calcium and magnesium


Sol 6: (A) The role of phosphate in detergent powder is
salts. Therefore, in hard water, soaps get precipitated as
to control pH level of the detergent water mixture
calcium and magnesium soaps which being insoluble
stick to the clothes as gummy mass.
Sol 7: (B) Adrenaline is a hormone but not a
neurotransmitter.
Sol 13: 0.2 % solution of phenol acts an antiseptic while
its 1% solution acts as a disinfectant.
Sol 8: (C) The name of given compound is acetyl
salicylic acid which is also known as aspirin, a well
Sol 14: 2-3% solution of iodine in alcohol and water is
known analgesic.
called tincture of iodine. It is a powerful antiseptic. It is
applied on wounds.
Sol 9: (D) The structure of 2-acetoxy benzoic acid is as
follows
Sol 15: The main constituents of Dettol are chloroxylenol OCOCH3
and α terpineol.
COOH

Sol 16: Antiseptics are chemical substances which


prevent the growth of microorganisms and may
even kill them but are not harmful to human or
animal tissues. For example, dettol and savlon. They It is commonly called aspirin, which is used as analgesic
are generally applied on wounds, cuts, ulcers and as well as antipyretic.
diseased skin surfaces. Furacin and soframycin are well
known antiseptic creams. Disinfectants are chemical Sol 10: (B)
substances which kill microorganisms but are not OCOCH3
OH
safe to be applied to the living tissues. These are
COOH COOH
generally used to kill microorganisms present in the Anhy.AlCl3
drains, toilets, floors, etc. Some common example of + CH3COCl
(acetylation)
disinfectants are phenol (≥ 1% solution) and chlorine
Salicylic acid Acetyl salicylic acid
(0.2 to 0.4 ppm). or (Aspirin)
-hydroxybenzoic acid
3 2 . 1 8 | Chemistr y in Ever yday Life

Sol 11: (D) Among the given antibiotics, only Sol 17: (B) Aspartame gets dissociated at cooking
chloramphenicol contains a nitro group attached to temperature.
aromatic ring.
NHCOCHCl2
Sol 18: (A)
O2N CH CH CH2 OH
R
OH
Chloramphenicol CH CH3

Sol 12: (B) Every surfactant contains two parts viz a RCH CH2 H2SO4/SO3
hydrophobic part and a hydrophilic part. The compound HF, Friedel (II)
CH3 ( CH2 ) CH2NH2 contains only hydrophobic part -Crafts’ reaction
14 (I)
but no hydrophilic part, hence it is not a surfactant. R

Sol 13: (B) Cationic detergents are used as germicides. CH CH3 R CH CH3
Cationic detergents are usually quaternary ammonium
salts. Thus, cetyltrimethyl ammonium chloride, a NaOH
quaternary ammonium salt is a cationic detergent and (III)
used as a germicide
SO-3 Na
+
SO3H
Sol 14: (A) Sol 19: (C)
OH
OH OH
OH OH
OH
HO H OH
HNO3 H NH2 O H COO CH3
HNO 3 H22/Pd
/Pd
(Nitration)
(Nitration) (Reduction)
(Reduction) HOOC CH2 CH C NH CH CH2 C6H5
Phenol
Phenol aspartame
NO2
NO NH2
NH NH2
2 2
p-Amino COOH
p-Nitrophenol
p-Nitrophenol p-Amino
phenol
phenol
HOOCCH2 CHCOOH + C6H5CH2 CHNH2 +CH3OH
OH
OH Hydrolysis

(CH
(CH33CO)
CO)22O
O
(Acetylation)
(Acetylation) Sol 20: (A) Greater the branching, greater is the non-
biodegradability of a detergent. A non-biodegradable
NHCOCH33
NHCOCH detergent cause maximum pollution.
Paracetamol
Paracetamol
Sol 21: (C) Cationic detergents are also called invert
Sol 15: (C) Boric acid is a mild antiseptic, soaps. Tri methyl steady ammonium bromide, being a
chloramphenicol is an broad spectrum antibiotic and quaternary ammonium salt is a cationic detergent.
aspirin is an analgesic as well as antipyretic.
Sol 22: (D) Antiseptics and disinfectants both inhibit
Sol 16: (D) and kill the microorganisms but disinfectants are not
safe to apply over living tissues.
OH OCOCH3
CH3 CH3
CH3COCl Sol 23: (C) Branched alkyl groups are not dissociated
easily and that’s why such compounds are non-
biodegradable.
OCOCH3
COOH
KMnO4

o-Acetyl salicylic acid


(Aspirin)
Chem i str y | 32.19

Sol 24: (C) Previous Years’ Questions


NCH3 NCH3
Sol 1: (C) The given compound is aspirin and it is used
CH3Cl’ as analgesic.
anhy. AlCl3
(alkylatioin)
Sol 2: (B) Antipyretic are the class of drugs which
HO O
morphine
OH H3CO O
codeine
OH reduces the body temperature during fever.

NCH3
Sol 3: (D) A hormone is responsible for Insulin
production and its action in human body.

CH3COCl, anhy. AlCl3 Sol 4: (C) Phenelzine is tranquilizer. It is not an antacid.


CH3OCO O OCOCH3
(acetylation) heroin

Sol 5: (A)
Sol 25: (B) Barbital or vernal is a sleep producing , ie COOH
hypnotic tranquilizer. It is used in sleeping pills and O
causes addiction.
O C CH3
Aspirin
Sol 26: (D) Bacteriostatic drugs inhibit the growth
of organism while bactericidal drugs kill the Acetyl salicylic acid
microorganisms.

Sol 27: (A) Heroin is not a tranquilizer, it is a narcotic


analgesic.

Sol 28: (C) In baby talcum powders, boric acid is used


as antiseptic.

Anda mungkin juga menyukai